Sunteți pe pagina 1din 1602

AMC CLINICAL COLLECTION

2017-2018
Table of contents
1. Rusty nail
2. Linear rash
3. Infective endocarditis
4. Henoch schonlein purpura
5. Mirena counselling
6. Compartment syndrome
7. Nephrotic syndrome
8. Hoarseness of voice
9. TIA Amaurosis fugax
10. Endometriosis
11. Rectal bleeding
12. Pericarditis
13. PPROM
14. Vulvovaginitis suspected abuse
15. Diabetic ketoacidosis
16. Transverse lie
17. Orbital cellulitis
18. Facial nerve palsy
19. Prolonged QT syndrome
20. Upper arm DVT
21. CS request counselling
22. Marijuana counselling
23. Mesenteric ischemia
24. Infectious mononucleosis
25. Asthma child
26. Cerebellar ataxia examination
27. PSVT
28. Hydrocele counselling
29. Thalassemia in pregnancy
30. Common peroneal nerve injury
31. MMSE
32. Hypothyroidism
33. Lymphoma after mastectomy
34. Weight loss
35. Duodenal atresia
36. Subfertility
37. Mitral stenosis in pregnancy
38. Recurrent candidiasis
39. HIV in pregnancy
40. SLE pre pregnancy counseling
41. Respiratory distress syndrome
42. Bulimia nervosa
43. Pubertal menorrhagia
44. Post op pulmonary atelectasis
45. Osteoporosis
46. Recurrent miscarriage
47. Gastroenteritis unimmunized child
48. CVS risk assessment
49. Acute sinusitis
50. Preterm labour
51. Cervical spondylosis
52. Preeclampsia and eclampsia
53. Quinsy
54. Umbilical hernia, inguinal hernia and undescended testis
55. Pneumonia risk assessment
56. Aortic stenosis
57. Brief resolved unexplained event
58. Antenatal checkup
59. Para pneumonic effusion
60. Meningitis
61. Slipped upper femoral epiphysis
62. Perthes disease
63. Acute pancreatitis
64. HRT counselling
65. Breath holding attack
66. Lichen sclerosis
67. Transient synovitis
68. Hypoglycemia
69. Assessment of comatose patient
70. GERD
71. OCP request
72. Recurrent fall diabetes
73. Cerebral palsy
74. Spontaneous pneumothorax
75. Threatened miscarriage
76. Bacterial vaginosis
77. Psychogenic cough
78. Pertussis
79. Indigestion and weight loss
80. Sore eyes
81. Breast reduction request
82. Irritable baby (irreducible hernia)
83. Infantile colic
84. Drug induced hepatitis
85. Splenic trauma
86. Stroke CT interpretation counselling
87. Hypoglycemia counselling
88. Lymphoma
89. Peripheral vascular disease
90. Cellulitis and PVD
91. Lithium in pregnancy
92. Toddlers diarrhea
93. Post cholecystectomy bleeding angry partner
94. Primary amenorrhea
95. Permanent Sterilisation
96. Advanced age pregnancy
97. Epiglottitis
98. DM Prepregnancy counselling
99. Chronic venous insufficiency
100. Abdominal aortic aneurysm counselling
101. Head collision
102. HSIL
103. Recurrent genital ulcer
104. Smoking counselling
105. Herpes zoster
106. Urticaria
107. Secondary post-partum hemorrhage
108. Post CS fever
109. Endometritis fever
110. Post partum mastitis
111. Breast cancer-breaking bad news
112. Hyperkalemia
113. Spiral fracture child
114. Back pain
115. Heroin addiction
116. Patellofemoral syndrome
117. Testicular torsion
118. Child abdominal trauma
119. Haemopnemothorax CT
120. Pulmonary embolism
121. Post-partum check up
122. Cyclical mastalgia
123. Perimenopause
124. Myocardial infarction
125. Melanoma breaking bad news
126. Croup
127. Angina
128. Cardiovascular examination physiological murmur
129. Headache examination
130. Respiratory examination peak flow meter
131. Prostate cancer counselling
132. Down syndrome baby counselling
133. Morton neuroma examination
134. Plantar fasciitis examination
135. Post MVA blurring of vision
136. Pleural effusion
137. Occupational lung disease
138. Back examination
139. Hip examination
140. Lung cancer
141. Facial trauma
142. Gynecomastia examination
143. Polycystic ovary syndrome
144. Tiredness anemia
145. Peripheral vascular disease examination
146. Exercise induced amenorrhea
147. Bulimia nervosa irregular period
148. Seizure child
149. Diabetic neuropathy
150. Spider bite
151. Lymphadenitis
152. Clavicle fracture examination
153. Tremor examination
154. Anorexia induced amenorrhea
155. Anniversary grief
156. Thyroid examination
157. Anemia in pregnancy
158. Delayed PTSD post colonoscopy
159. Ear examination
160. TIA lower limb
161. Osteomyelitis, septic arthritis
162. Knee examination
163. Atrophic vaginitis, endometrial hyperplasia
164. Neonatal examination
165. Assessment of dehydration
166. Breast examination
167. Bee sting
168. Pancoast tumor examination
169. Achilles tendonitis
170. Chronic liver disease examination
171. Acute abdomen examination
172. Vasovagal syncope
173. Funny turn alcohol
174. Giardiasis
175. Scrotal examination
176. Hematology examination
177. Irritable baby (reducible hernia)
178. Breastfeeding jaundice
179. Conjugated hyperbilirubinemia
180. UTI in children
181. Alcoholic neuropathy examination
182. Hypertension examination
183. Frontal lobe dementia
184. Incomplete miscarriage
185. Large for date uterus
186. Tension headache
187. Asthma severity assessment
188. Pregnancy induced hypertension
189. Gestational hypertension headache
190. Concealed placental abruption
191. Preeclampsia tummy pain
192. Rash in children
193. Hyperthyroidism nervous lady
194. Pelvic examination
195. Post-operative? UTI
196. Post-operative heart failure
197. Postural hypotension
198. Atypical pneumonia
199. IDA cow milk
200. Oral thrush
201. MSE VET , DV
202. MSE mania
203. MSE delirium
204. ENT examination tonsillitis
205. Obsessive compulsive disorder
206. Post traumatic stress disorder
207. Normal pregnancy examination
208. Urinary incontinence
209. Vaginal vault prolapse, uterovaginal prolapse with candidiasis
210. Headache Somatisation
211. MSE psychosis
212. Borderline personality
213. Carpal tunnel syndrome
214. Cubital tunnel syndrome
215. Claw hand
216. Hypocalcemia hand examination
217. Hand examination (OA, RA, DQT, scaphoid Fx)
218. Primary PPH telephone case
219. Encopresis
220. Obstructive sleep apnea
221. Septicemia
222. Medication non compliance (sertraline)
223. Depression on paroxetine
224. Schizophrenia relapse risk assessment
225. Major depression fraud no suicide
226. Domestic violence
227. Pyloric stenosis
228. Ureteric calculi
229. Papillary thyroid cancer BBN
230. DVT leg
231. IUGR
232. Hyperemesis gravidarum
233. Placenta previa
234. Bleeding in pregnancy
235. Solitary thyroid nodule
236. Temporal arteritis, Polymyalgia
237. Migraine
238. Atrial flutter
239. Urinary tract diseases
240. Nonspecific abdomen pain child
241. Tummy pain in children and IBS adult
242. Cholangitis (post cholecystectomy syndrome)
243. Trauma during pregnancy
244. Serotonin syndrome
245. Delirium hyponatremia
246. DRE BPH examination
247. Benzodiazepine dependency
248. Liver metastasis
249. Diverticulitis, ruptured diverticula
250. Rheumatoid arthritis counselling
251. Collection of pilot and uncommon but important cases
252. Buckle fracture
253. Acute cholecystitis
254. Ovarian cyst torsion
255. Failure to thrive+ fussy eater
256. Fussy eater
257. Adenocarcinoma rectal
258. COPD
259. Pneumonia
260. Fibroadenoma counselling
261. Sore throat requesting antibiotics
262. Sore throat requesting painkillers
263. GERD adult
264. Post viral runny nose/ cough
265. Recurrent fall stroke
266. Pancreas cancer
267. Gout
268. STI screening counselling
269. Down syndrome psych case
270. Hypertension non-compliance
1-Rusty Nail
60 year old male patient with injury to his foot with rusty nails while working in his farm. He has
just moved to this area. He hasn’t received any health care before.
Task
-further relevant history
-management plan
-general health review for his patient

History
1-Injury questions
-I can see from the notes that you stepped on a nail while walking in the farm so can you tell me
more about how have you injured your foot?
-did it penetrate the skin? Were you wearing shoes at that time?
-did you take it out by yourself?
-have you taken any medications?
-any other injuries?
-has this happened before?
2-Local symptoms questions (all normal)
-have you had any pain in the site of the wound?
-any redness? Any swelling?
-any bleeding?
-any tingling or numbness?
3-systemic symptoms questions? (All normal)
-Any fever
-any nausea, vomiting?
-any headaches?
4-Tetanus symptoms (normal)
-any spasm or stiffness in your muscles?
-any difficulty breasting or swallowing?
5-Tetanus vaccine
-have you been immunized against tetanus? (Since childhood)
-when was the last tetanus shot? (No boosters)
6-General health questions
-do you take any medications? Any allergies? (No)
-have you had any illnesses like high BP, high blood sugar or cholesterol levels? any heart or lung
diseases? (No)
-do you smoke? If yes ask how many cigarettes and for how long? (Chronic smoker but quitted)
so any SOB or cough? (Yes, I got intermittent cough, there was also sputum)
-do you drink alcohol? If yes ask how much and for how long?
-can you tell me briefly about your diet?
-how is your activities?
-any illnesses in your family?
-do you follow sun protection measures while you are working in the farm?
Counselling
Explain condition+ tetanus
-from the history it seems like your wound now is clean and dressed well.

-Such type of puncture wounds carry a risk of tetanus infection, which is an infection with a bug
called clostridium tetani. This bug usually enters the skin when there is a cut or penetrating
wounds then it can multiply and produce toxin that can cause spasm and stiffness of the muscles
and serious consequences later on.

-we usually protect ourselves from this infection by being vaccinated against tetanus along with
boosters every 5 years. In you case you haven’t had any booster since childhood so that I would
like to give you tetanus vaccine 3 shots now then 6 weeks then after 6 months.

-I will prescribe you painkillers and antibiotics prophylaxis as well.

COPD
-as you have been smoking for a long time in the past and I appreciate that you quit it now but
just in case because you have intermittent cough. I would like to arrange for chest X-ray and
pulmonary function test to assess your lungs.

Regular checkup
-as you will be my regular patient, I need to do FBE, BSL, lipid levels, I also need to check your
vision and BP.

SNAP
-it is important to eat healthy balanced diet more in fruit and vegetables, less in fatty and junk
foods. Increase your water intake. try to do regular exercises 30 minutes a day for 5 days a week.
Important to keep your alcohol drinking level to safe limit.
Note/ Advice about smoking if was smoker and arrange another consultation.

Work and sun protection


-whenever you work in the field you need to wear protective gear like boot, shoes to avoid any
further accidental injuries. Since you are working under the sun and to prevent skin cancer it is
important to follow sun protection measures: try to avoid sun exposure by slipping on protective
clothing, slop on sunscreen, slap on a hat, rap on sunglasses and avoid going out in the sun
during peak hours.

Other screening
-have you ever tested for FOBT?

I will follow u up regularly to check your injury and general conditions.


Case (21/4/2017)
A man presents to you who is a new patient and stepped on a rusty nail yesterday.
He is going to move to your area and wants to become your regular patient. ( this case was
similar to karen notes but the patient was not a case of copd in my opinion because he denied
any physical symptoms and was quite healthy and took no medication, had stopped smoking and
no alcohol intake
Task:
1. Take history
2. Advise management
3. Discuss long term and regular( periodic) health advice that you will give him

Case (27/4/2017) Pass


Old age stepped on rusty nail, Wound was already dressed and clear. He wanted to be ur regular
pt, recently move to the farm.
Task
H/o,
explain ur immediate mx,
further mx.
I entered the room and introduced myself. An old man was sitiing there. I started with ‘how are
you feeling ‘ right now.
- Any pain. Any numbness, colour change. He said I am fine now.
- I asked him about the injury history that how did he get the injury.
- Then, I asked about his immunization history, he said he don’t have any shot after childhood
immunization. No booster shot of any.
- General health like DM, HTN, Ht disease, he said no.
- I told him that he was chronic smoker and appreciate him that he quit it. Ask him about
problem with lungs like copd, asthma, SOB, he said no. when asked about cough, he said yes, I
got intermittent cough, there was also sputum.
-No soB, no other problems including poo and pee.no alcohol.
I explained him about immediate mx like tetanus vaccination and antibiotic and pain killer.
I give him advices about 5p and need to see me immediately.
For long term mx,
- since u got some cough with sputum and also smoke a lot in the past but I really appreciate it u
already quit it. But, I am a bit concern about problem with lungs. I will do necessary test lkie
CXR, lung function test to assess ur lungs. If needed, I will give u medication to control ur lungs
problem including the vaccination to prevent getting infection. Since you are going to be my
regular pt, I will also check your vision for you. if needed, refer u to specialist. Also, I would like
to check your heart and do ECG to make sure they are ok. I also would like to arrange blood fat
level and sugar level for you. You know, it is a good idea to wear boot or shoes when you go out
to prevent nail injury. The roleplayer laugh a lot and said that’s good idea. I will also liase with
occupation therapist to make sure your home enviroment is safe for you and to make sure there
is adequate lightining. Check his understanding. And said I will follow u up regularly to check
your injury and general conditions.
FB-PASSED, APPROACH-5, HISTORY-5, MX-5, PT COUNCELLING AND EDUCATION-6
Feedback 27-10-2018 pass FOOT INJURY
60 year old male patient with injury to his foot with rusty nails while working in his farm. He has
just moved to this area. He hasn’t received any health care before.
Task
-further relevant history
-management plan
-general health review for his patient
History
built some rapport with patient
asked about his foot injury
cleaned with water at home
received tetanus immunization before, but that was many years ago.
told him that as he is new to this area and never come to this surgery before, i’d like to ask you a
few more question regarding your general health.
chest pain (-), shortness of breath (-)
cough (+) → dry cough and doesn’t bother him much
chronic smoker, but stopped recently (appreciated him)
drink occasionally
waterworks and bowel habits → normal
asked him whether he has heard of screening programs and he said no.
asked him his home situation, occupation, lifestyle and dietary habits.
asked his past medical and surgical history (unremarkable)

Management
told him i will ask the nurse to clean your wound with soap and water, then antiseptic solution.
I will give you vaccination.
then examiner asked me to mention the name of the vaccine.
then I told the role player that i will give you tetanus toxoid vaccine and i will also make sure that
you are up to date with the vaccination.
i will also check your wound and prescribe antibiotics if it is necessary.
regarding his general health, i’d like to perform whole body examination and will manage
accordingly.
told him i will give him reading materials on screening programs available and arrange him to
join them.
encouraged him to eat healthy balanced diet and do physical activities.
appreciated him again on giving up smoking and told him standard limit on alcohol drinking.

Grade Global score


Key steps - pass 5
1,2,3,4,5 - No, Yes, Yes Yes, No
Approach - 4
History - 5
Management - 6
Patient counseling - 4
2-Linear rash
Case/ Young patient presented with linear rash picture provided
Tasks
-History
-perform Physical examination
-Diagnosis and differential diagnosis

History
1-open ended question: I can see from the notes that you have this rash along your arm. Can
you tell me more about it? (He has painful rash)

2-pain questions (as you need to address pain first) just severity
-how severe is your pain from 1-10? 1 is the least 10 is the most. Ask about allergy then give him
painkillers

3-rash questions
-how long have you had this rash? When did it actually start?
-has it suddenly appeared?
-is it getting worse? (No)
-any change in size or shape since it started? (no change)
-what were you doing before that? (I worked in the garden two days back)
-apart from pain. Is it itchy? (No)
-is it hot to touch? (No)
-is there rash anywhere else? (No)
-any swelling or lumps and bumps around your body? Weight loss?

4-Differential diagnosis questions


-any chance you had an insect bite?
-any trauma?
-do you use any drugs or medication intravenous? Any needles?
-do you have any fever?
-any recent air travel? (No)
-have you been in contact with someone with similar symptoms?
-do you have allergies to anything?

5-General questions (all negative)


-do you take any medications like steroid or any OTC?
-any history of Diabetes, infections, clot in the vein of your legs, blood disorder?
-any recent surgical operations?
Perform examination
1-WIPE + Consent
I need to examine you now to look for the cause this would involve me having a look, feeling
down your arms and feeling glands around your neck and armpits would that be all right?

2-General appearance and Vital signs


-pulse (ask examiner for a watch and measure the pulse)
-blood pressure (examiner will give you the finding normal)
-Temperature (in the exam there will be ear thermometer so put speculum and when you just
start examiner will give the finding 36.5)

3-Rash examination
Inspection
-site and extension of the rash (middle of the arm)
-shape (linear)
-colour
-entry point (close to the cubital fossa)
-trauma, bruising, skin changes
-compare with the other arm and look for rash anywhere else

Palpation
-tenderness (tender)
-temperature, if hot to touch (not hot)
-elevated or not (not elevated)
-blanchable or not (blanchable)
-is it cord like

Lymph nodes examination


-Epitrochlear LN
-axillary LN (not enlarged but tender)
-Cervical LN (no enlargement)

Diagnosis and differential diagnoses


from history and examination you most likely have a condition called lymphangitis, which is an
inflammation or infection of the lymphatic channels, which is part of our immune system. This
condition occurs due to infection by bug bacteria at a site distal to the channel.

Other possibilities are:


Could be due to cellulitis which is infection of the skin and area under the skin may be due to an
insect bite while working in the garden causing bug entry into the skin.

Could also be due to thrombophlebitis is an inflammatory process that causes a blood clot to
form and block one or more veins but its unlikely.
Feedback 14-3-2018 pass
Picture of a linear rash was given. take HX,PE,DX & DDX
2 min thinking.. cellulitis (so ask about h/o i/v drug, DM, temp, pain and pulse with signs of
toxicity) Dvt (ask about air travel, h/o bleeding or clotting problem, FH) Lymphangitis, insect bite
etc.
There was a thin guy with an extended arm in the room who looked very irritated. I started by
asking if he felt alright because I can’t see him to be fine. He told about the rash and pain. After
offering pain killers I started with history about the rash. How it started, he had no idea. Is it
increasing? No. Any change since it started? No. Apart from pain is it itchy? No. Does he feel hot
to touch? No. Is there any rash anywhere else? No. Any chances he had an insect bite? No. Any
I/V drug abuse? No. I told him I can see some entry point around the rash. So if he can recall
going out in the garden or anything like that? He said yes I worked in the garden two days back. I
asked if he can related the rash to it. He wasn’t sure about it. No H/O of DM or any surgery. He
had no air travel recently, no past history of clot in legs and was never diagnosed with any blood
disorder. There was no FH of and bleeding problem or clots. No h/o radiation to the arm. ( I
didn’t do well in the history as I forgot about asking about allergies or contact history)

I told him I needed to examine him to look for the cause. For which I would measure his vitals
most importantly pulse and temperature to look for signs of toxicity. I would look at, feel for his
arm and some glands in his neck and arm pit. If that was okay with him. He said yes! (I washed
hands) I asked the examiner for a watch to measure pulse. He gave it to me. The pulse was
almost 60/min. (I measured for 15 sec) Examiner gave me BP that was normal. I asked for a
thermometer. The examiner pointed behind me on a table. I held it up and inserted the
speculum it was then that the examiner told me it’s 36.5 degrees. I did inspection and
commented on the morphology of the rash, it’s extent, colour and location in the middle of arm
with an entry point closer to the cubital fossa (here I apologised the role player for using medical
jargons and requested to bare with me because these were for the examiner). There was no sign
of trauma or skin changes. The rash wasn’t elevated, was blanch-able. It was tender on
palpation. Temperature was not raised. I palpated his cervical LNs which weren’t enlarged.
Before palpating his arm pits, I wore the gloves. Though there was no enlargement of axillary LNs
but they were tender. I said sorry for causing pain.
Then I explained what might be the cause. It was most likely due to a bug gaining entry into the
skin through some accidental insect bite (because he wasn’t aware) during working in the
garden. The bug caused infection of the skin which is called cellulitis in medicine and also
infection of vessels. It could be lymphangitis, lymphedema or thrombophlebitis. It could be DVT
which although looked unlikely but I still needed to R/O by doing a test called duplex USG. I
referred him to the specialist for a second opinion and further testing and treatment because he
needed painkillers, antibiotics and elevation of his arm etc.
Bell rang.
Scenario … Arm rash
Grade… Pass
Global score… 4 Key steps 1,2,4… yes 3.. no
Approach to patient…. 4 History… 4 Diag/ D/D… 4
Another feedback
linear rash(real pt with this rash above the elbow) take
HX(no trauma,no insect bite,no cannula,no recent surgery,no rash elsewhere,nothing positive in
the hx)
focused PE(Did GA,asked vitals not available then asked temp specifically the examiner said here
is the thermo. But as I start ed to pick up the thermo he said it is normal (it was ear thermo)

The rash was blanchable not palpable and not cord like ,checked axillary lymph nodes but
couldn't feel anything

DX & DDX to the pt

My dx was Lymphangitis

Other ddx Thrombophlebitis,cellulitis

Feedback 22-6-2018
Linear rash PE. Task Hx, PE, DDs with reason
Linear rash in elbow flexure.. hx only positive for gardening, no injury, no insect bite, no
allergy. But in the picture it looked like a insect bite. It was a dd case so I asked questions in
hx and only mentioned the dd’s with reason.

PE- Consent, Wash hands,GA, Rash inspection, Checked temp, Then put on gloves for
palpation, Asked for glass slab for blanching was not available.
Checked epitrochlear and Axillary lymphnodes

DD
Trauma
Insect Bite
Cellulitis
Contact dermatitis
Thrombophlebitis
DVT
Lymphangitis ( mentioned as provisional dx)
Cutaneous larva migrans ( Some parasite can penetrate the superficial layer of your skin and
move within causing this rash)

I had a lot of time so I asked if pt understands everything. Then I asked about


3-Infective endocarditis

First Scenario
60yo female, since 2 weeks tired, no appetite.
Tasks
-History (also has SOB, saw cardiologist for murmur in past)
-explain diagnosis to patient and DDX

Second Scenario
1-lady presented with tiredness and fever.
Tasks
-History,
-pefe
-explain diagnosis to patient and ddx
2-Lady with history of tiredness, fever. She had a heart murmur long ago.
Tasks:
Take history
PE from examiner,
Dx/DDx to patient.

Note/ two scenarios of infective endocarditis one with the patient presented with tiredness only
for which there is no PEFE and the other when the patient presented with tiredness and fever
and there is PEFE.

History (first scenario)


1-Tiredness questions
-since when have you been feeling like this? (2 weeks)
-do you feel tired the whole day or at a specific time? (All the time)
-do you feel tired the whole body or a specific part? (The whole body)
-has this happened before?

2-Differential diagnosis questions (HEMIFAD)


hemochromatosis and hepatitis (all negative)
-any pigmentation on your body?
-any yellowish discoloration of skin or eyes?
Endocrine (hypothyroidism and diabetes) (all negative)
-any weather preference? Any change in your bowels habit?
-do you feel thirsty? Are you passing large amounts of urine?
Malignancy
-any loss of weight?
-Loss of appetite? (Yes)
-any lumps and bumps around your body?
Infections and infective endocarditis
-have you had any fever? (No)
-how high is it? -is it constant or come and go? -is it associated with chills or shivers or night
sweating? (Only if fever was positive ask more)
-any chest pain, shortness of breath or headache? (No, some feedback say there is SOB)
-have you seen any blood in your urine?
-have you had any dental procedure recently? (she had root canal treatment 3 weeks ago)
Atypical pneumonia and anemia (all negative)
-any coughs? If yes ask do you cough up anything?
-do you feel dizzy? Any funny racing of the heart?
Depression and drugs (all negative)
-how do you feel today?
-do you take any medications

3-General
-past medical history (heart disease) (yes Gp months ago told her she has heart problem)
-SAD (negative)
-family history (negative)

History (2nd scenario)


1-Tiredness questions
-since when have you been feeling like this? (2-3 weeks)
-do you feel tired the whole day or at a specific time?
-do you feel tired the whole body or a specific part?
-has this happened before?

2-Fever questions
-how high is it?
-is it constant or come and go? (On and off)
-is it associated with chills or shivers or night sweating? (Night sweating present)

3-Differential diagnosis questions (HEMIFAD)


hemochromatosis and hepatitis
-any pigmentation on your body?
-any yellowish discoloration of skin or eyes?
Endocrine (hypothyroidism and diabetes)
-any weather preference? Any change in your bowels habit?
-do you feel thirsty? Are you passing large amounts of urine?
Malignancy
-any loss of weight?
-Loss of appetite?
-any lumps and bumps around your body?
Infective endocarditis
-any chest pain, shortness of breath or headache?
-have you seen any blood in your urine?
-have you had any dental procedure recently? (Tooth taken out recently)
Atypical pneumonia and anemia
-any coughs? If yes ask do you cough up anything?
-do you feel dizzy? Any funny racing of the heart?
Depression
-how do you feel today?

3-General
-past medical history (heart disease)
-for how long have you had this murmur?
-SAD
-do you take any medications?
-family history

Physical examination from examiner


1-General appearance
dyspnea/ LAP
2-Vital signs and O2 saturations
3-hands (examiner will give a picture of splinter hemorrhage)
-clubbing
-splinter hemorrhage.
-osler nodes and janeway lesions
4-Face
-eyes (pallor/ jaundice/ subconjunctival hemorrhage)
-Mouth (cyanosis/ dental hygiene).
-fundoscopy (Roth spots)
5-Heart
-thrills, murmurs, displaced apex beat, 3 rd heart sound
6-chest and lungs
-air entry, dullness, crackles
7-abdomen
-hepatomegaly, splenomegaly.
8-office tests
-urine dipstick (blood, protein)
-BSL
-ECG
Explain
-From history and examination I am suspecting that you have a condition called infective
endocarditis. It is an inflammation and infection of the inner lining of the heart, heart valves and
blood vessels.
-it is usually caused by a bug; a bacteria called staph or strept.
-they entered the blood stream during a procedure like dental surgery then reach to abnormal
heart valves or other damaged heart tissues and settle there causing more damage.
-they can grow and form clots that may break off and travel to other organs.

-other possibilities why you feels tired could be due to diabetes, hypothyroidism, hepatitis, other
infections, anaemia, nasty growth but the most likely one is infective endocarditis.

Treatment
-I need to admit you to the hospital to be seen by a specialist.
if GP (call ambulance and tell that they will be reviewed by cardiologist
if HMO admit to hospital

-I am gonna order some tests to confirm the diagnosis.


1-blood tests:
a basic blood counts
inflammatory markers
kidney function and mineral body salts
2-chest x-ray
3-blood cultures (3 samples)
4-Echo (any valves abnormalities and vegetation)

Note/ No need to go further in the exam unless you have management in the task for example if
you got time you can say admission and tests quickly but not further.

-we will start you on antibiotics to kill the bacteria and once the cultures results back antibiotics
will be tailored.
-we will continue these antibiotics for 2 weeks and then you can have outpatient IV medications
for further 4-6 weeks
-you will be taken cared off by a team of specialists and once you are stable and discharged I will
give you a letter to your GP for regular follow up.
-In the future before any procedure you need to take a prophylactic antibiotic to prevent it.
Feedback 8-2-2018 (Scenario 1)
Case (11/2/2017) Scenario 2
case1/ Lady with history of tiredness, fever. She had a heart murmur long ago. (Can’t remember
the stem much but this was the essential info)

Tasks: Take history, PE from examiner, Dx/DD to patient.

Introduced myself, saw the patient, she didn’t look sad or anything, she seemed well.
I jotted down HEMIFADO (mnemonic for conditions causing tiredness) on the paper and told her
I’m sorry, i might be jotting notes on the paper as we speak, i hope u don’t mind, i am listening
to you.
She was ok and said sure dr, no prob. Since the task had a few components I didn’t ask all of
HEMIFADO. I asked how long did she have this tiredness, I think she said 2 or 3 weeks. Fever on
and off, night sweats present. No cough, no lumps or bumps noted, no problem with passing
urine or bowels, don’t remember if there was weight loss. No known medical/surgical history
besides the murmur. I don’t remember if she had chest pain or palpitations. Then I asked what
did her dr tell her about the murmur? Were you advised to take antibiotics before any dental
procedures or surgery? She said “nothing like that dr”.
So suddenly a lightbulb turned on in my head and I said that I was thinking of chronic fatigue
syndrome at first, but as I got more of your history and you had a murmur before, I suspect your
tiredness is related to that. I think you might have bacterial endocarditis. As I said bacterial
endocarditis, the bell rang. :S

Feedback: Tiredness, Pass, Global score 4

Case 2/ Infective endocarditis Lady with H/O tiredness and fever with chills. Was told long time
back she had heart murmur.
Take history. (Had tooth taken out recently only if you ask on history )
Ask Examiner PE (photo of splinter hges only if you ask Discuss prov diag and reasons.

Case (8/6/2017)
case 1/ endocarditis, feeling dizzy, Osler nodes and hx of dental procedure.

Case 2/ Infective Endocarditis – Patient was having fever and night sweats since 6 weeks . Had a
dental procedure as well ( he gave this information after sometime when I asked him if he is on
any medication . He took panadol for pain relief ) had positive findings for infective endocarditis.

Now when I think , patient had fever and night sweats ( Long history ) – Could be hodgkins +
Infective endocarditis.
Feedback 20-7-2018 (Important, very good feedback)
Infective endocarditis
Tiredness pass , global score 5
key step 1,2,3,4 yes
key step 5 No
approach to patient 4
history 4
choice and technique of examiantion, organisation and sequence 6
Dx /DDx 4
47 yr old , tiredness, LOA , but no LOW , fever and sweats , body aches + , history of heart
mumur 5 years ago , generallly healthy.
History
PEFE
Dx with reasons
introduce
Jenny , i do utd you are very tired and having fever and night sweat as well , would you
mind if i ask you more
tiredness... duration 3-4 wks
it is persistent or off and on , persistent
any particular time of day the whole day
fever ... same duration
didnt measure the temp
no chills
Night sweat
body aches +
LOA + but no LOW
no lumps and bumps in body
no anemia symptoms ( no pallor , no palpitation , no dizziness
no skin colour changes
no thyroid symp
no DM symp
Source of infection -no headache, cough and cold , chest pain , SOB , vomiting , abdominal
pain, pee and poo prolem
when i asked about period ... she said she does not have period as she is on mirena
no discharge from down below ,
partner ... safe sex, no STI before
pap smear .. was normal
SADMA negative
Past history heart murmur
have you done any invx before no
did the previous doctor tell you exact dx no
did the doctor tell you to take antibiotics when you do any surgery or
procedure ... no
have you ever recently done any dental procedure ... yes , root cannal 4 wks
back
no antibiotics taken at that time
no family history of heart disease

PEFE
GA , no pallor , no jaundice , fever 38 , PR regular , RR , BP
BMI , no lymphadenopathy
no thyoid gland enlargement
CVS 1st and 2nd sound normal
PSM that radiates to axilla ,
no raise JVP , no BBC , no oedema
REsp normal

said i will check periphery for sign of infective endocarditis


spinter haemorrhage ... examiner gave me the nail pic
he asked me to point out where
i asked osler node and janesway lesion roths spot , negative
Abdominal examination i check splenomegaly 2 cm below left costal margin
no other mass , no tenderness
no pedal oedema

urine dipstick , BSL and ECG not available

Explanation
Jenny , according to your story and PE findings , most likely cause of your symptoms is a
condition of infective endocarditis . let me explain , Drawing , in our heart , there are 4
chambers , 2 upper chambers and 2 lower chambers , which are separated by valves ( which
are door like structures ) , most likely you have got problem in your valves that produces
abnormal heart sound called murmur and when you have this , it is prone to get infection
easily . in your story , you recently had the root canal treatment , most possibly , the bugs in
your oral cavity got a chance to go into the blood stream and reach to the heart valve and
can cause the infection
There are other possible causes that can lead to your symptoms , for example
infection anywhereelse in your body ( lung , urine ) but less likely
nasty growth in body like lymphoma but i didnt find any glands enlargement on examination
thyroid and Dm, anemia problem can also cause tiredness but your symps does not fit into
those
i will liase with heart specialist to do echo to your heart and blood test to confirm this
diagnosis
I think i should have done some pelvic examiantion as well , as she has IUCD to check for any
sign of infection of it , if present, it can be the source of infective endocarditis as well

Feedback 5-7-2018
Young man presented with tiredness

Task: 1- HX 2- Dx

I follow this DDx

I infection ( all routs of infection like UTI, URTI, cough, fever qs, chills)

I iron def or overload (bleeding Hx, jaundice)

I inflammation (SLE, polymyalgia rhumatica)( rash, joint pain or stiffness)

I infective endocarditis (heart disease in childhood, murmers)

E endocrine (DM, thyroid)(wt change, bowel motion and water work)

T tumor (lumps or bumps, night sweat)

S OSA ( sleep disturbance)

S stress ( asking about home and work)

The pt HX of murmer and tooth extraction or root canal (I don’t remember exactly). I explained
with diagram a Dx of IE

Passed: global score 6


Feedback 5-7-2018
34 yr old male, presenting with tiredness

Tasks

Take relevant history

Explain the provisional diagnosis to the patient

I extended to shake his hands, he stared at me and didn’t shake. So I knew I wouldn’t be doing
that in next stations.

As I entered the room and sat down , I jotted down the acronym for tiredness as the d/d are
numerous. HEMIFADO

I asked about tiredness, he said it started 2-3 weeks ago, what happened 2-3 weeks ago?
Nothing.

Affecting your life? Yes. Cant do much

Sleep? Yes affecting sleep. Asked about sleep apnea (wake during night –yes) .. do you feel
sleepy during day – no . cough at night yes ( excluded cardiac by asking pillows used – he said
1) .. asked more about cough it was dry ..

Asked about atypical pneumonia (chest pain? no / is cough dry or sputum? Dry / headache? No)

Then I asked 2-3 questions for each d.d.

In asked about surgery . he said no .. any DENTAL surgery? – Yes ! root canal.

Any medical condition? No . any murmurs? –YES. (nice :@)

patient had a tooth extraction done 2 weeks ago. He also admitted that he was told that about a
murmur when he was teen. So I explained my diagnosis of IE with a diagram
4-Henoch schonlein Purpura
5 year old, had rash over buttocks and legs..Has some pain in legs and abdomen. Active, Afebrile.
Had URTI 10 days ago. Father is here now. (Detailed history and examination was already given in
stem)
Task-
-explain diagnosis,
-explain investigations you wanna order and examiner will give you specific ones.
-Counselling and education

Note/ the stem will be a long one and very detailed so that you can have all history and
examination you want to diagnose HSP

Explain diagnosis
What is your child (call by their name) is most likely having is a condition called HSP. Have you
heard about it?

Condition
this is an inflammation of small blood vessels of the body for example the skin, causing rash. It
can also involve small blood vessels of the kidneys, intestine or very rarely lungs and the brain.

Cause
the cause is usually unknown. However, it is usually an autoimmune disease usually trigerred by
the viral infection he had 2 weeks ago. So normally, our body produces certain substances called
antibodies to fight infection. In HSP these antibodies are attacking own body tissues and the
small blood vessels.

Clinical features
it can cause rash, limping, joint pain, and tummy pain

Complications
this is a common condition in children aged 2-10 years. Its outcome is good with treatment.
As this condition can also affect the kidneys it can cause hematuria or blood in urine, proteinuria
or leakage of protein in the urine and high blood pressure. Also it can affect the gut causing
abnormal folding of the bowels leading to obstruction that is why we need to keep an eye on
him.
Investigation
-So that I need to admit to hospital to be seen by a child specialist, and I am gonna run some Ix
to rule out other possibilities like meningitis, leukemia and ITP.
-I need to order:
urinalysis looking for protein and blood.
FBC, UCE, Coagulation profile
stool analysis looking for blood
imaging like US
-I just need to talk to the examiner for the investigation results. Is that all right? (Examiner will
give you card Blood shows normal platelets, urine showed rbcs )
-Explain report with the mother or father

Counselling and education


is it meningitis?
-it is unlikely, because from history and examination he has no fever, no headaches, no neck
stiffness or confusion.

Is it leukemia as his elder brother has leukemia?


-the blood picture is not suggestive of leukemia because in leukemia we will have decrease in
RBC, WBC and platelets causing pallor, bleeding or bruising and fever which your child is not
having.

-Reassure again and tell others less likely.


-we will give him analgesic for the pain (paracetamol or ibuprofen)
-the specialist might give him steroids only for a few days so that try not to worry about any side
effects.
-we will monitor him regularly measuring his blood pressure and doing urinalysis.
-this disease is of good outcome, usually resolve within 4 weeks.
Tummy pain within 2 days
joint pain within 3 days
rash can last for 4 weeks

-he will need regular follow up by his GP for the first 6 months.
-Reading materials

15-3-2018 AND 28-6-2018


ED, seeing a patient with rash, has urti 10 days ago, started limping 3 days ago and now joint
pain and tummy pain.
Tasks: explain the dx and ddx,arrange invx, possible mnx. (HSP. I forgot to mention abt possible
dialysis. Pt is worried abt meningo and leukemia, only get fbe from examiner, no renal function
or electrolytes)
Case (28/11/2017)
Station 20: rash - paedi - pass
5 year old, had rash over buttocks and legs..Has some pain in legs and abdomen.
Active, Afebrile. Had URTI 10 days ago. Father is here now. (detailed history and
examination was already given in stem)
Task-
-explain diagnosis,
- explain investigations you wanna order and examiner will give you specific
ones.
-Counselling and education
( Some candidates said upon mentioning full blood count, examiner gives u a
card detailing FBC. Everything normal incuding platelet count, tho I mentioned all
the investigations including full blood but examiner didn’t give me anything.
Either I should’ve turned to the examiners and ask, even tho the task wasn’t to
ask investigations from the examiner or probably I wasn’t loud enough, not sure
whats the reason for it., patient asks why not leukemia, why not meningitis as he
is worried)
Score key steps 4/4
Approach. 5
Choice of investigations 6
Diagnosis and differentials 5
Counselling 5
Global 6
It was a clear case of HSP. We will understand it from the stem given outside
along with the picture.
I went inside…greeted the examiner and role player
Built rapport as the father looked bit anxious
Explained to him about HSP(certain substances that are produced in our body to
fight infections sometimes acts against our own body like some tissues or blood
vessels leading to the condition) told him the son needs to be admitted
Told him about the need to keep him in hospital and prognosis etc
Told him about the investigations to be done like blood and urine
I turned to the examiner and asked for blood and urine investigations and he
gave me cards for it. Blood shows decreased platelets, urine showed rbcs
I explained to him the reports comparing with the normal values
The father was concerned about meningitis because of the rashes….I explained
to him why it was not meningitis
He was worried if the child had leukaemia as his elder kid had leukaemia….I
explained again with the blood result why it was not leukaemia and how a blood
picture of leukaemia would look like(which was not in this case with the child).
I reassured him about how we can follow up the child and watch out for
leukaemia because of the concern and also because of the family history.
(This case needed a lot of reassurance to the dad addressing all his concerns)
Counselled him about HSP and differentials like meningitis, ITP etc(why both of
them are least likely in this case).
Told about admission, specialist referral, investigations, possible treatments
decided by specialists like steroids depending upon how the child is progressing,
and going to higher modalities of treatment depending upon the response of the
child and specialists advise(I don’t think management was a task but said these
few points as a part of explaining the patient more about HSP)
5-Mirena counselling
you are at your GP when a 30 year old jane come to you requesting mirena. She had 2 previous
pregnancies. She was on OCP before but now she would like to be put on mirena.
Tasks
further history
counsel the patient

History
1-can you tell me why you want to be put on mirena? (bothered about the pill)
2-5Ps questions
periods
-when was your LMP? Are they regular?
-how many days of bleeding and how many days apart?
-any pain or heavy bleeding during menstruation?
-any bleeding or pain in between the periods? (Intermenstrual bleeding imp)

Partner or sexual
-are you in a stable relationship?
-have you or your partner ever been diagnosed with STI
-history of multiple partners and safe sex?

Pregnancy
-how many pregnancy have you had?
-any previous miscarriages? Have you ever had an ectopic pregnancy?

Pill
-how long have you been using the pills?
-are you still taking them?
-what type do you use?
-Have you had any side effects?

Pap and HPV

3-contraindications questions (LCP: liver, cancer, PID)


-have you had a history of active liver disease?
-any history of cancer breast or uterine?
-any abnormal vaginal bleeding, fever, tummy or back pain?

4-General questions
-Past medical and surgical history?
-medications and allergy?
Counselling
1-action and contents
-show the specimen
-mirena is a temporary method of contraception and I think you are a good candidate for mirena
as you do not have any contraindication like genital cancer or breast cancer, genital infection,
liver disease and also you are in a stable relationship.
-but because you are sexually active it is always better to do a urine pregnancy test before we
put mirena in.

-mirena contains hormone called progesterone.


-This prevents pregnancy by:
*thickening the cervical mucosa so that the sperms find it difficult to get through.
*Also it changes the lining of the womb make it thin a unfavorable for implantation.
*And because it is within the uterus so act as a barrier and inhibits normal sperm moving inside
the uterus or the tubes

2-Adminstration.
-it is a small T shaped device made of soft plastic and has a cylinder containing hormones and to
its base is attached to fine threads.
-Usually fitted by trained doctors so after a gynecological examination, a speculum will be put in
the vagina to see the neck of the womb or cervix which is then cleaned with antiseptic solution.
-Mirena is then fitted into uterus using a thin flexible application passed through cervix.
-once it is in its place the applicator is removed and the thread at its base will hang down into
the vagina, which is then cut short.
-it can cause a bit discomfort during insertion but there is no excessive pain.
-it is better to be fitted during your period or within 7 days of getting period because it provide
immediate protection against pregnancy if fitted within the first 7 days.
-otherwise, you have to wait for intercourse up to 24 hours to minimise the risk of conception.
-it provides contraception for up to 5 years.

3-Side effects
-it can affect your periods even though initially there could be an increase in the number of days
of bleeding but eventually you will have reduction in number of days and sometimes will stop all
together.
-could be spotting in between periods for about 3-6 months
-irregular periods sometimes
-cramping pain in lower part of the tummy or back usually settle in a few weeks.
-headache, genital discharge as hormonal effect
-can get extruded out of the body or go somewhere else and cause perforation but these are
very rare to happen.

4-advantages
-high efficacy (99%), also does not interact with oral medications
-forget about pregnancy for the next 5 years unless bothered by STI when you need to use
condoms.
-bring down heavy periods

5-disadvantage
-does not protect against STI

6-follow up
-I will teach you self palpation of the strings and it is better to get checked once a month after
each period.
-follow up every 4-12 weeks then annual follow up

Feedback (5/10/2017) Pass


Contraception request
37 yrs old lady comes to your GP to discuss about Mirena as she was told by her friend that it is
good. She is allergic to sulphur.
Task:
Relevant focused history
Explain her about Mirena
Answer her questions

Introduced myself and assure confidentiality.


Asked how was she doing and acknowledged her concern.
And ask permission to ask history before explaining about Mirena.
1- Asked about contraindications for Mirena ( DVT, undiagnosed vaginal bleeding, breast CA,
Stroke, migraine, PID, ectopic pregnancy ). she asked me what is ectopic and i explained to her
2- 5 P ( Pregnancy, pill, period, partner, pap smear).
*She has 3 children and happy family and good support from partner.
*Used COC (can't remember the duration she used, but no complications of COC) but stopped 6
months ago and only using condom sometimes.
*Normal period, LMP was 2 or 3 wks ago
* pap smear was normal last time..
3- She has no symptoms of UTI. No fever, bleeding per vagina, no abd pain at the moment.
4-She heard Mirena from her friend and she wants to know about it.
- Explained her about what is Mirena, content, mode of action and how long it lasts.
- Then i told her i will arrange pregnancy test and then refer her to specialist.
- I explained her the procedure ( measure the depth of uterus with probe, about the string)
- ( reassured her that it wont interfere her sexual activity).
- And then the side effect of Mirena (breakthrough bleeding, dysmenorrhea and reassured her
that they can b manageable and mostly goes away within few weeks.

I kept on asking her what more she wants to know and she kept on asking me what more she
should know.
I told her about the red flags of Mirena and then told her i will review her after she is seen by
specialist.
Covered 3 key steps out of 5
Approach 4 History 4 Choice and technique of exam 3 Patient Counseling 5 Global score 4

6-Compartment syndrome
HMO at ED, young man in his 20s was here due to a motor vehicle accident. Primary and
secondary surveys were done, he swollen left leg and a series of X-rays were done. The X-ray
with positive findings is given and a picture showing open wound on the left leg.

Tasks:
-Perform relevant physical examination on the lower leg of the patient. Explain to the patient
what you are doing. Not to remove the bandage on the patient’s leg.
-Explain the X ray to the patient
-Explain to the patient about further immediate management

Note/ on the screen there will be a picture of the leg and also an x-ray showing fracture.

Task 1
1-Introduce yourself and ask patient about pain then offer painkiller after asking about allergy
(you can give morphine)
-I can see from the notes that you’ve had a motor vehicle accident sorry to hear that. Let me
assure you that primary and secondary survey have been done and you are stable.
do you have any pain at the moment? (Yes) how sever is it? I would like to arrange painkiller for
you so do you have allergy to any medications? (No) offer painkillers

2-take consent and wash hands


-To assess the severity of the injury, I just need to examine your legs and feet. This would involve
me having a look, examining your vessels, nerves, tendons and bones. Would that be all right for
you?
Remember VNTB (vascular, nerve, tendon, bone)
3-Inspection
-look at the dressing (it would be slightly soaked with blood)
-ask the examiner to remove the dressing (when asking this the examiner will give you a picture
the same on the screen to comment on).
-comment on the photo looking for:
open or closed wound
foreign body
skin colour (pallor, black, bluish)
swelling.

4-Vascular (TCP) (look for pulse and pallor)


-Temperature
-Capillary refill time
-pulses (dorsalis pedis and posterior tibial artery)
note (in the exam vascular most likely to be normal with pulses present)

5-Nerve (cotton wool for light touch only for paraesthesia)


-First web space (deep peroneal nerve)
-lateral malleolus (superficial peroneal)
-sole (posterior tibial)
you can test for points on both dorsum and sole of feet. Always compare and ask if he feels the
same on both sides.

6-Tendons (look for paralysis)


-Active movements only (dorsiflexion, plantarflexion, eversion and inversion)
in the exam the patient will have pain in all movements so active movements restricted all
tell the examiner that you will skip passive movements due to pain.

-ask patient to wiggle his toes (can do that)

Note/ if painful request to skip proceeding with passive movements.


if not painful and in order to confirm the diagnosis of compartment syndrome do passive
dorsiflexion of the big toe and this will excruciate pain.

7-Bone (look for tenderness) press on these points:


-Lower tibia then medial malleolus then navicular bone
-lower fibulae then lateral malleolus then the base of fifth metatarsal bone.

8-thank the patient and rewash your hands


Task 2 (explain X-ray)
-look ---, this is the x-ray of your left leg showing two bones, this is tibia (shin bone) and this is
fibula. From the x-ray it appears that, there is a break in both left leg bones into multiple
fragments we call this comminuted fracture of both tibia and fibula.

-So ---, from the history, examination and x-ray findings you most likely have a condition called
compartment syndrome.
-This is a collection of blood or swelling into the soft tissue of the left leg due to fractures of the
long bones. This cause excessive pressure within the tissue of this compartment preventing
adequate blood supply to the compartment leading to death of the soft tissues and subsequent
scarring, loss of limb function or loss of limb itself. But let me assure you that you are in safe
hands and it is good that we have picked it up early, from examination both pulse and nerve still
intact and.

Task 3 (immediate management)


-I am gonna call orthopaedic registrar for early intervention, they might do fasciotomy which is a
surgical procedure to take the pressure of these tissues. along with this they will repair and fix
the fracture.
-other (may be not necessary) is to put IV line and send for blood investigations. Ask about
tetanus and give booster (patient will tell you long ago) and start antibiotics.
--check understanding, reassurance and offer contact with family.

Notes (not in the exam but just in case)


if not comminuted fracture:
-if open fracture
wound cleaning
tetanus and antibiotics
may need debridement
may need external fixation

-if closed fracture


*non displaced-cast from groin to the neck of metatarsals 12-16 weeks
*displaced-closed reduction under anaesthesia+ cast or ORIF

-in both cases repeat x-ray post reduction, review next day, and red flags
Feedback 20-2-2018
HMO at ED, young man in his 20s was here due to a motor vehicle accident. Primary and
secondary surveys were done, he swollen left leg and a series of X-rays were done. The X-ray
with positive findings is given and a picture showing open wound on the left leg.

Tasks:
-Perform relevant physical examination on the lower leg of the patient. Explain to the patient
what you are doing. Not to remove the bandage on the patient’s leg.
-Explain the X ray to the patient
-Explain to the patient about further immediate management

2 min thinking:
I will check the sensation on the peripheral nerves and do passive flexion/extension of the big
toes to rule out compartment syndrome

Introduced myself. Check whether the patient in orientated to time place and person and offer
analgesia after asking allergic history to medication.Explain to patient I am going to examine his
legs and gain consent.

Perform as per Tally O’Connor – look, feel, move (only able to wiggle his toes), sensation
(initially I did light touch according to dermatome on both lower limbs, then check the 1 st toe
web, lateral of 5th metatarsal, middle and lateral malleolus) (examiner came over to see how I
examine the peripheral nerves). Passive extension/flexion of toes (did not aggravate pain)

Explain to patient the XR findings and the picture of his leg using lay man terms. I said it was
compound fracture but I think the correct term is comminuted fracture)

Explain to him that he will be seen by my senior and we will call in orthopaedic specialist. First
we need to immobilise his left leg, put IVL and send some blood investigation. Ask for his
tetanus immunisation status (long time ago) and said will need to give booster. Will give
antibiotics as well. Most likely he will undergo surgery to fix the broken bones. Assess his
understanding, reassurance and offer to contact his family members.
AMC Feedback – Leg pain: PASS

Feedback 20-4-2018
Feedback 27-10-2018
30+ year old man was involved in an accident and injured his left leg. He is in severe pain. Picture
of right and left leg (with bandage applied to the left leg) was given. Do not disturb the bandage.
Task
-Perform examination on the lower limb
-explain the X ray to the patient
-explain the condition
-Management

30+ year old male patient was lying on the couch with bandage on his left leg with blood stained
in the midway.
He looked anxious, in pain and angry.
Introduced
offered him pain killer after checking the allergy status.
Washed hands
put on the glove
explained what i’m going to do and get the consent.

Inspection
Mr.—- is lying on the couch.
I’d like to check the patient’s vitals (examiner told me it was stabilized)
Described what I saw. (bandage applied)
then, examiner handed me a picture and asked me to describe according to picture
in the picture, there was obvious swelling in the left leg.
the size of the left leg is about twice the size of that of the right.
there was some abrasion and bleeding from the shin of the left leg
normal skin color
no other injury or no deformity

Palpation
-Role player told me to be gentle as it is very painful when i told him i’m going to feel your leg.
-temperature (normal)
-tenderness (++++++)
-pulse (present)
-capillary refill test (normal)
-cotton wool and toothpick are given on the table. Sensation
-fine touch with cotton wool
-pain sensation with toothpick.

Then disposed them of properly into the bin. Movement


ankle movement (very limited on the left leg)
toe movement (slightly limited on the left leg)
then examiner told me time is up for the first task and moved on to next task

examiner handed me the X ray (tibia and fibular fracture seen)


-explained the finding to the patient
Explanation of Dx/DDx
-Based on the examination findings, your leg is at risk of having a condition what we call
compartment syndrome.
-Drew and explained the compartment syndrome.

Management
-I’m going to call my registrar and they will come and assess your leg again.
-then they will perform a procedure called fasciotomy.
-it is the procedure to relieve the pressure inside your leg.
-then, you will be shifted to the operation room to put your bone back into its original place and
stabilize them.

Showed some empathy


-do you want me to call someone to be with you?
-you are in safe hands now.

Grade - pass
Global score - 4
Key steps 1,2,3,4,5 - Yes, Yes, Yes, No, Yes

choice and technique of examination - 5


organization and sequence
accuracy of examination - 5
Dx/DDx - 4
Management plan - 4
7-Nephrotic syndrome
You are working in GP and now seeing 4-year-old boy with puffiness in his eyes and face. He had
viral URTI 2 weeks back. He is generally healthy boy.
Tasks
-Ask relevant HO
-PE findings from examiner
-Explain diagnosis/ differential diagnoses with reasons

History
1-swelling questions
-since when has he had swelling In his eyes and face? Can you tell more about it? (He has got
swelling of his face which I noted since last week)
-has it appeared suddenly or gradually? Is it getting worse? (Gradually)
-is there any time when it becomes more prominent like in the morning or evening? (In morning)
-does he have swelling anywhere else like in his tummy or legs?
-is this the first time? (Yes)

2-has he had any infection recently? (if not mentioned in the stem)

3-Differential diagnosis questions


Kidney
-how is his urine?
-any decrease in urine output?
-any change in the color of urine?
Liver
-have you noticed any yellowish Discolouration of skin or eyes?
-any liver problems?
Heart
-does he have any shortness of breath, chest pain, funny racing of heart, or any heart problems?
Nutrition
-can you tell me briefly about his diet, and what he is eating? (Eats family diet)
Medication and allergy
-does he take any medications?
-any problems with allergies? Any watery eyes?
Cellulitis and insect bite
-any fever, rash, skin infections? Any insect bite?
GIT
-any nausea, vomiting? How is his bowel motions?

4-BINDS
-is he immunized up to date?
-is he thriving and growing well?
-any hsitory of similar symptoms in his family or siblings?

Physical examination from the examiner


1-General appearance
-rash, pallor, jaundice, LAP
-oedema (site, pitting or not, redness of overlying skin, warmth skin) (leg swelling and pitting)
2-Vital signs
-especially BP and temperature
3-growth chart
4-CVS
-heart sounds, murmur.
5-chest
-air entry, breathing sounds
-any crackles, basal crepitation, any signs of pleural effusion like stony dullness (bilateral basal
crepitation and stony dullness)
6-abdomen
-inspection: distension, mass (distension)
-palpation: tenderness, organomegaly
-shifting dullness (present)
-bowel sounds.
7-sacral oedema + genitalia
8-office tests
-urine dipstick (examiner give paper contains UDS result Protein - 4+, RBC, Leucocytes –ve)

Explanation
Condition
-from history and physical examination, your child most likely have a condition called Nephrotic
syndrome.
-it is a condition where the filtering mechanism of the kidney is affected leading to protein loss in
the urine.
-it is common in this age group and treatable.
Cause and clinical features
-the cause is unknown but could be related to his recent URTI.
-so what happened in this condition is that the proteins are essential to maintain fluid inside the
blood vessels and since these protein are losing from his urine, fluid comes out of the vessels
and accumulating around his eyes, tummy, legs leading to swelling.
Complications
-it can cause infection, clots in the blood vessels but it has good outcome if treated properly.

Other possibilities
-Acute post strept glomerulonephritis which is an immune reaction to viral infection
-heart problems
-liver problems
-diet problems
-insect bite
-medication allergies
but I did not find anything in the history or examination to suggest those so they are unlikely.

Management (not a task but just in case)


1-admit to hospital to be seen by a specialist
2-Investigation
FBC, ESR/CRP, lipid profile, LFT, UCE, ASO titer
urine MCS, 24 hours urine collection for protein

3-monitor weight, BP, input and output charts. Diet restriction

4-+/- antibiotics……………. +/- steroids

5-once discharge as it recurs again parents need to monitor UDT and GP will monitor RFT

Feedback 23-6-2018:
Feedback: Health review: child , eye swelling, viral infection 2 weeks ago. Take Hx, PEFE, DDx.
Hx:
How long? What set it off? Progressive? What make it worse, better?
Infection: fever, pain, discharge, visual impairment?
Allergy? Ate anything strange before that?
Systemic: fever, rash, headache, runny nose, sore throat (2 wks ago, resolved with some cold
meds),
vomitting, cough, SOB, chest pain, tummy pain, water work, bowel motion, legs swelling (Yes)?
BINDA
Vision and Hearing check
Family, Friend, School.
Past Hx?
Urine test: (Protein +++, blood -)

Dx: Nephrotic Syndrome  explain to the pt. Draw the basement membrane and antigen-
antibody
complex,…
Grade: PASS, Gs: 5
Key steps 1,2,3: Y (all covered)
Hx: 4, PEFE: 5, DDx: 5

Case (22/2/2017)
You are working in GP and now seeing 4 year old boy with puffiness in his eyes and face. He had
viral URTI 2 weeks back. He is generally healthy boy.
Tasks
- Ask relevant HO
-PE findings from examiner
-Explain diagnosis/ differential diagnoses with reasons
2 mins thinking
-This is child with facial oedema
-DDx -
1-Renal cause (APSGN or nephrotic)
2-Liver
3-Heart
4-Nutrition
5-insect bite
6-medication allergy.
-Not to forget to ask BP, urine dipstick in PE
History taking
-I entered the room and greeted both examiner and role player
-Good morning Margaret, how are you? How can I address ur son? (He is David).
-Is he around here? (Yep, Dr he is playing around) (I wrote down pt’s son name not to forget later
on)
-How can I help u and ur son today? (RP : He has got swelling of his face which I noted since last
week)
* Any eye swelling? (Yep)
*Apart from this, does he have any leg swelling, tummy swelling? (I cant remember RP answer
now)
* Is it first time for him? (Yep).
* Did it happen suddenly or gradually? (gradually).
* Is there any particular time of day when that swelling is prominent? (In morning).
*Alright, I know that he had cough and cold a few weeks back. At the moment, is he having any
fever, running nose? (No, it is settled) Any skin infection? (no)
DDx questions
Renal - Any changes in urine colour? (No), Does he cry when he pass urine? (No), decreased
amount?
Liver - any past history of liver problem like yellow color of skin and urine (No)
Heart - Does he tell u that he has SOB, racing of heart, chest pain (He cant play well during these
days, and he is tired I think)
Nutrition - What type of diet is he eating? (He eats family diet)
Cellulitis - Is there any chance that he is bitten by any insect? (No)
Allergy - Did he take any medication recently (No)
General health, bowel work, BINDSMA - All normal, no concern

PE from examiner
GA - Any pallor, jaundice, lymphadenopathy, oedema (Examiner - Facial and eyes oedema
present)
Any leg swelling (present), is it pitting oedema? (Yes)
VS - All normal
Growth chart - normal
Focus CVS examination - No murmur
Resp exam - (Examiner - what do u want to know) Bilateral basal crepitations? (Present) Any
signs of pleural effusion like stony dullness on percussion (Yes)
Abdominal exam
Distension (Yes)
Palpation - Hepatosplenomegaly (No), Are kidneys ballotabble? (No)
Percussion - Shifting dullness (+)
BS - present
When I asked for urine dipstick, examiner handed me a paper which contains UDS result
Protein - 4+, RBC, Leucocytes – negative
Thank you examiner.

Explanation to mom
Jenny, according to David’s HO and PE findings, most like condition he is having now is called
nephrotic $. It is basically problem in his kidneys. Let me draw a picture to make it more clear for
u.
(I drew diagram of tummy, two kidneys and ureters) This is our tummy and these are kidneys.
Basic function of kidney is to filter blood to form urine. In David’s case, that filter is not working
well. (I draw diagram of glomerular basement membrane)
Normally, proteins are not filtering through this membrane but in nephrotic syndrome, it
happens.
Exact cause is unknown but it could be related to his recent viral URTI.

So far, are u with me Jenny? (yes Dr)


Ok, good. So, what happened is this protein are essential for maintenance of fluid inside the
blood vessels and since these proteins are losing from his urine, fluid come out of vessels and
accumulating around eyes. That’s why he has swelling of eyes and face, leg swelling and tummy
swelling as well.
(I talked in detail coz the task is explain with reasons)
Other causes can be APSGN which is immune reaction to viral infection, heart problem, liver
problem, diet problems, insite bite and medication allergy but I didn’t find anything for those.
So, they are unlikely.
Do u understand my explanation? (Yes Dr)
(I have some time left, and asked RP : is there any one u want me to repeat? No.)
The bell ranged
AMC Feedback - Health review : Pass (Global Score - 6)
History - 4
Examination - 6
Dx/ DDx - 6

Case (11/11/2017)
Health review ( Paediatrics )
Father brings his 4 year old daughter with puffiness around the eyes
Task:
1.History from father
2.Ask examination findings from examiner.
3.Give differentials to father
Introduction
Role player seemed very disinterested in answering questions. I asked duration he said he
doesn’t remember. So I inferred- is it because the facial puffiness is not so prominent then he
said it has been 10 days. I asked for swelling elsewhere. He said he doesn’t know. Then I
specifically asked legs- no. Tummy? He said yeah she looks a bit fat around there and her clothes
don’t fit well. Further questions No SOB, chest pain. No fever, no decrease in wee or frothy urine
or yellow urine. No allergies, no insect bite. I asked past history. BINDS. Social situation.
Examination: Vitals stable normal BP, Growth charts normal.
ENT normal, no nodes, no pallor or rash
No JVP
B/l Pleural effusion
CVS: normal
Abdomen: distended, fluid thrill not done ?
Legs: normal
Urine dipstick ( given only if asked ) : 4+ Protein, no blood, no pus cells

Case (7/6/2017) 4 yr old boy, developed puffiness around eye. Tasks - hx from mother, PEFE, dx
and dds to mother ( child has an attack of sore throat few days back. Urine output is normal, no
fever/ breathlessness, no tummy pain/jaundice/allergy/bee sting. PE - dont quite remember the
vitals, but pallor+, some crackles and diminished air entry in one lung, urine dip stick protein ++
+, no hematuria)

Explained to father- That she has nephrotic syndrome- its relatively common in this age group
where the kidneys have become leaky to protein. Because of loss of protein there is
accumulation of water which starts off around the eye and then rest of the body. She has a bit of
water in her lungs which may make it difficult to breath. It can cause low BP, infection and clots
in her blood vessels. However, the condition has very good prognosis when promptly treated.
Other causes insect bite, allergies, heart failure, liver disease. Briefly explained why I dint think
those were likely.
Mentioned about reading materials.
Key steps 1,2,3,4 – Yes
Global score: 5 ( Pass)
Assessment domain
History: 5
Choice and technique of examination, organisation and sequence: 5
Diagnosis/ differential diagnosis: 5

8-Hoarsness of voice
GP, middle aged female came in for hoarseness of voice. Unremarkable pass medical history
Tasks:
-Relevant history
-Physical examination findings from the examiner
-Explain to the patient probable diagnosis and differential diagnoses
-Explain what is the investigation you would order for the patient during this consultation

History
1-hoarsness questions
-when did it start? Can you tell me more about it? (Had a cold and cough a couple of days back,
hoarseness started after that.)
-has it started suddenly or gradually? Is it constant or come and go? Is it getting worse?
-Does anything make it better or worse?
-is it worse at the end of the day?
-has this happened before?

3-Differential diagnosis questions (VOICE)


Viral
-any recent viral infections? (no need if answered above)
-have you had any fever? Runny nose? Sore throat? (No)
Overuse
-what do you do for living? (a primary school teacher)
-does your job or hobby need you to use more of your voice? (Yes)
Irritation
-do you smoke? How many and for how long? (Smokes 15 cigarettes for 20 years)
-do you think it is related to air pollution or allergy?
Cancer
-any loss of weight? Loss of appetite? Lumps or bumps?
-any SOB, chest pain? (Lung cancer)
-any difficulty swallowing or pain on swallowing? (Esophageal cancer)
Endocrine
-any weather preference? Any Change in bowels motion? Neck lump? (Thyroid)

3-General questions
-have you had any illnesses?
-any surgical operations?
-do you take any medications? Alcohol?
-have you had any illnesses in your family?

Physical examination from the examiner


1-General appearance
dehydration, rash, pallor, jaundice, LAP
BMI

2-Vital signs

3-ENT
allergy signs

4-Neck
thyroid
lymph nodes

5-Chest
inspection---chest movement
palpation----trachea and chest expansion, vocal fremitus
percussion---dullness
auscultation----breath sounds, added sounds, vocal resonance
note/ in the exam there will be dullness on left lower zone and decrease breath sounds on the
base of the left lung and crackles.

6-cranial nerves and horner’s signs (proptosis, myosis, anhydrosis)

7-quick abdomen and CVS

8-office tests
urine dipstick
blood sugar level
indirect laryngoscopy (unremarkable)

Explain the diagnosis


-there are various reasons why you have hoarseness of your voice.
-from examination I have found some changes in the right chest like decreased air entry and
dullness this could be either fluid or solid. Also from the history, you have hoarseness of voice for
more than 2 weeks with long history of smoking. So it could be due to a nasty growth or cancer
of the lungs.
-it can also be due to overuse of your voice related to your job as a teacher or inflammation of
the voice box due to a recent viral infection or smoking.
-it can be a benign growth of the vocal cords or thyroid problems.
-nasty growth of food pipe or voicebox.
-in order to know more about the diagnosis, I would like to arrange investigations like FBC, TFT,
LFT, UCE, Chest X-ray). I would also like to refer you to ENT specialist for direct laryngoscopy.

Recall of 20-2-2018/2-3-2018/ 6-8-2018/ 5-9-2018

Feedback 7-7-2017
45 year old man is complaining of hoarseness of voice
TASKS
1. History
2. physical examination findings from examiner
3. tell patient what investigations are required and why

WHAT I DID
Hello examiner, hello John
Me: since when
John: a couple of days ago, had a cold and cough a couple of days back, hoarseness started after
that. no fever, no sob, no chest pain
Me: whats your occupation
John: im a primary school teacher
Me: you must be talking all the day then
John: yes
Me: asked about weight loss, ,u ps or bumps, smoking
John: smokes 15 cigarettes for 20 years
I will speak to my examiner now
GA..as is
VITALS all normal
LN no
Ent..all normal
Respiratory system
dullness right base, decreased air entry
Thank you examiner
All right John, there can be alot of reasons for your hoarseness which needs further investigation
there could be some problem with the vocal cords, as you are a teacher and speak alot..so an
ent specialist will do an indirect laryngoscopy...thats a camera to visualize your voice box
secondly your chest examination isnt quite normal..WHAT DO YOU MEAN DOC...there seems to
be fluid in your lungs which is suspicious...it can be infection or a nasty growth..WHAT DO YOU
MEAN NASTY...it could be cancerous, maybe due to your smoking...but as I said we need to
investigate...SO WHAT ARE YOU GOING TO DO? some baselines like fbe, crp, cxr..and once results
are back will refer to specialist
Bell rang...thanked John and examiner
DID NOT SAY CT SCAN, AND PLEURAL TAP...FORGOT
PASS
keystep 1 2 3 4 Yes approach 4
hiztory 4
choice of examination 5
diagnosis 4
choice of investigation 4

Case (3/3/2017) not sure about the diagnosis


55 year old lady with hoarseness of voice.
Task:
H/o 4mins.
Ask PE.
explain Pdx.
Investigation to patient.
Positive findings
Second time, first time after viral infection, now no fever, school teacher need to use more voice
recently.
smoking history positive, no Surgery, no swelling, no features of cancer,Physical examination all
normal except reduce air entry on left side with, dullness, I check for bone tenderness, and
features of cancer but negative.
exclude ddx:thyroid, Surgery, tonsil, infection, indirect laryngoscope she said not available.

Mention about base line test, laryngoscope, CXR, followed by Broncos copy and biopsy
depending on imaging results. Patient is calm afterwards.

Feedback 20-2-2018
GP, middle aged female came in for hoarseness of voice. Unremarkable pass medical history

Tasks:
Relevant history
Physical examination findings from the examiner
Explain to the patient probable diagnosis and differential diagnoses
Explain what is the investigation you would order for the patient during this consultation

2 min thinking: DDx: over usage, infection, vocal polyps, external obstruction, motor neuron
weakness, stroke, thyroid/neck surgery, GORD, malignancy

History:
Ask about chief complaint (how long, getting worse/better, aggravating/relieving factors,
associated symptoms – pain, sore throat, fever)
Rule out DDx: occupation (teacher, need to talk a lot), lumps in front of neck (no), weakness
elsewhere (no), neck surgery (no), reflux/heart burn (no), loss of weight (yes), lumps and bumps,
tiredness.
PMHx: unremarkable
SADMA – chronic smoker, social drinker, no recreational drugs, no known drug allergy, no allergy

Physical examination:
Positive findings is dullness on left lower zone of lungs, crackles.
Indirect laryngoscopy: unremarkable as well

Explanation:
Dear Terry, there are lots of causes for hoarseness of voice. It could be due to overuse as you are
a teacher, or any URTI, thyroid gland pressing on it, regurgitation due to heart burn, benign or
cancerous growth on the vocal cord. But don’t worry too much, we need further test to find out
the cause as I could not pin point towards any.
(Bell rang!) I quickly said I need to refer her to specialist for direct layngoscopy and one more
test, but I forgot to mention CXR.
Don’t think I did well for the last task.
AMC Feedback – Speech Complaint: FAIL (choice of investigations 2)
Feedback 16-8-2018
Station 16. Hoarseness of voice: Lung cancer
GP, 60 yrs old lady come to see you for hoarseness of voice.
Task: take history
Ask PEFE
Dx
DDx
Investigations to patient

Not give information in open-ended question. She told me what do you want to know.
Not 1st time. Previous attack recovered spontaneously. Now for 3 wks. Not resolved. Affect on
her work. It is persistent.
No fever, no sorethroat, No neck gland enlargement. No lumps & bumps, no loss of wt and loss
of appetite, no cough, no muscle weakness and abnormal sensation.
Works as school teacher. Has to use voice a lot.
Smoke 30 cig/day since teenager. No alcohol. No recreational drugs.
No chronic medical problem. No Sx in throat. Not taking medications
PEFE: GA & vitals: good, BMI: 24, no signs of dehydration, no anaemia, no jaundice, ENT: normal.
No thyroid gld enlargement, no lymph node enlargement. Lungs: dullness on left lower lung,
reduced vocal fromitus & resonance on left lower lung. No secondary features: no liver
enlargement, no bony tenderness, CNS normal. Examiner said That's all I can give you.
I said your hoarseness of voice is related with a condition we called lung cancer. Pt seem to be
worried. Reassure. She said Ok. I found some abn finding in your lungs. Some factors can
predispose to lung cancer. Smoking is the main factor and others: occupation and repeated
chronic lung infection.
Other factors can cause hoarseness of voice: use voice a lot, infection to throat, thyroid
enlargement, stroke, Sx in throat.
Invx: bld test
CXR
ECG
CT (chest)
Scopy to breathing system & take biopsy

Grade: Pass
  Global score: 5
Key Steps: 1- Yes
2- Yes
3- Yes
  4- Yes
Approach to patient: 5
History: 5
Choice & Technique of examination, organization and sequence: 5
Diagnosis/ Differential diagnosis: 4
Choice of investigations: 5
9-TIA Amaurosis fugax
Case 1
a 50-year-old man who has history of hypertension for more than 10 years on enalapril.
Complaining of blurred for a few seconds. Now it is completely resolved.
Tasks:
-physical examination
-dx and ddx to patient

Case 2
you are HMO at ED, Middle aged man had an episode of loss of vision yesterday evening at
home, and recovered spontaneously. He is having hypertension and T2DM on a list of
medication (Can’t recall the names). He is here for a check-up. ECG was done and was sinus
rhythm (if I remember correctly)
Tasks:
-Perform visual examination to the patient. Explain your findings to the examiner
-Perform other relevant physical examination
-Tell the patient your probable diagnosis and DDx

Note/ on the table, you can find: torch, ophthalmoscope, stethoscope, and Snellen chart on the
wall.

1-WIPE:
the same approach of introduce yourself, explain what you are going to do and why then take
consent and wash your hands.

2-inspection
Eyes inspection
-ptosis
-swelling
-redness
-watery eyes
-size, shape and symmetry of pupils
face inspection
-facial symmetry
-prominent or dilated temporal artery

3-palpation
-feel for temporal artery tenderness

Note/ in the exam inspection and palpation will be normal


4-Visual acuity
notes/
-say ideally I’d like to keep it 6 m away from the patient. If Snellen was on wall or through a
mirror.
-for the exam keep it 3 m on one arm away from patient if the Snellen chart was hand-held.
-ask about glasses because patient should wear glasses if so.
-patient should cover one eye at a time.
-only let the patient to read one line (for example the next last line)

*I’d like to test your visual acuity. Do you usually wear glasses?
*if you could cover one eye with the palm of your hand and read this line?
*now if you could cover the other eye?
*good thank you.

Note/ in the exam visual acuity will be normal

5-Visual field
notes/
-patient should take off glasses if present.
-doctor should sit on the same level as the patient.
-red pin half way between the dr and the pt.
-cover one eye at a time (both doctor and patient).
-ask the patient to look at your nose and let you know when he sees the pin comes into view.

*Now I’m gonna test your visual field if you could cover the right eye for me with the palm of
your hand. I’m gonna cover my opposite eye.
*keep looking into my nose and tell me when you see this red pin come into view? (if nor red pin
you can use your fingers and ask when you first see my fingers moving)
*now if you cover the o

Note/ in the exam visual field will be normal

6-Light reflex
-now I’d like to test your pupils by shining a light with this torch is that ok for you?
-just keep looking at my nose? Thank you
-test both direct and indirect reflexes
note/ in the exam light reflex will be normal

7-accomodation
-now I’d like to test the ability of your eyes to focus. If you could look at a point on the wall
behind me and now look at the pen.
-the pupils becomes convergent and constricted
note/in the exam accommodation will be normal
8-eye movements
-test both eyes
-H pattern with red pin
-ask for double vision
-look for 3 things (restriction of eye movements, diplopia, nystagmus)

*I’m gonna assess the movements of your eyes. First of all do you have any double vision at the
moment?
*if you could keep your head still and I’d like you to follow my finger and let me know if you have
any double vision throughout the exam.

in the exam there will be normal eyes movements, no double vision or nystagmus.

9-ophthalmoscope/fundoscopy
-assemble it
-adjust the light size and color
-adjust the lens you are looking to
*choose the smallest light size, yellow color. And choose zero lens.
-hold it like a pen, perpendicularly
-Take permission: I would like to test the back of your eyes with this fundoscope this might need
me to be at times close to you.
-remove your glasses and patient’s glasses
-make sure you are on the same level with the patient
-ask the patient to focus on a distant object
-test the right eye of the patient with your right eye and vice versa.

Note/ when you start doing it the examiner will give a picture of fundus the picture shows silver
wiring, AV nipping +/- papilledema

10-other relevant examination

-complete other cranial nerves: examiner will say normal

-vital signs especially pulse and blood pressure.

-feel and listen for carotid: examiner will say normal

-CVS exam with stethoscope: examiner will say normal

-I’d like to complete examination with both upper and lower limb neuro. Examiner will say
normal
Dx and DDx
1-Amaurosis fugax: this is a symptom of a condition called transient ischemic attack or mini
stroke causing transient loss or blurring of vision. It happens when the blood flow from one of
the arteries supplying the eye is blocked mostly by a piece of plaque or small clot then it resolves
spontaneously by itself. There are certain risk factors like high blood pressure, diabetes etc.. and
fundoscope shows some changes of the back of the eye related to high blood pressure.

2-hypertensive retinopathy: vascular damage of the back of the eyes due to high blood pressure

3-diabtic retinopathy

4-temporal arteritis: no prominent TA or tenderness

5-Carotid stenosis

6-migraine

7-space occupying lesion

Feedback 20-2-2018
HMO at ED, Middle aged man had an episode of loss of vision yesterday evening at home, and
recovered spontaneously. He is having hypertension and T2DM on a list of medication (Can’t
recall the names). He is here for a check-up. ECG was done and was sinus rhythm (if I remember
correctly)
Tasks:
-Perform visual examination to the patient. Explain your findings to the examiner
-Perform other relevant physical examination
-Tell the patient your probable diagnosis and DDx

2 min thinking: after visual examination I will perform cardiovascular examination

Perform as per Tally and O’Connor physical examination book. After every test I told the findings
to the examiner. After I did ophthalmoscopic examination to the patient, the examiner gave me a
picture which shows silver wiring and some soft exudate, blurring of optic disc.

I told the patient I will check his heart and blood vessels of neck. When I palpate the carotid
pulse and listen for carotid bruit, the examiner said CVS examination is normal.

Explanation:

Most likely he is having temporary loss of vision due to his prolong high blood pressure, but it
could be due to other conditions like DM (bell rang) ( I did not have the chance to tell amaurosis
fugax)

AMC Feedback – Visual problem: PASS

(approach to patient 4, choice & technique of examination, organisation and sequence 4,


familiarity with test equipment 4, accuracy of examination 4, diagnosis/differential diagnoses
3)
Feedback 20-4-2018
Feedback 9-5-2018
3. Rt eye blurring, task hx, pefe, ex ( carotid briuts + sliver wiring on fundoscopy )
2 Minutes outside- stroke, cataract, hypoglyceria,vasospasm, glaucoma. This case , the history is
about 5 minutes and the other task ia about 2.30 second. I think i should take shorter history
and spend more time for other task, otherwise i could not finish in time.

Inside the room_ ask VS, carotid bruit. the patient is hypertension and carotid bruit is positive. I
said about remove the patient to resusitation table but the examiner said that it is not necessary
so i give waning signs for patients.

I take history of eye symptoms, then neurological symptoms, BP treatment and follow up. I
finished history earlier a lot and jumb into the next task straight away. After take PE from
examiner as checking hypertension patient, i come back to patient and explained my diagnosis
and DD as i mentioned above.This is mini stroke most likely as well as hypertension retinopathy.
However, i concern about stroke then i told him this is severe situation, i need to admit him now
and run some investiagtions on him. Bell rang. ( Follow my experience, when the questions ask
about DD, you have to talk about investigations as it is the way to help you disinguide the
disease. Also, the most severe one need to come first as it can kill patient in short time).
Feedback 13-12-2018 Limb Weakness: Pass
2 Min outside: Typical history of TIA. Pt had weakness in right side of body for 10 mins (don’t remember
exact time). Which is gone now. No other positive symptoms (as long as I remember). Vitals were given.
Only positive was BP 160/90. Sensory examination was done already.

Tasks: perform lower limb and other relevant examination.

Dx and DDs to patient

Thinking outside: This is a PE of TIA. Sensory was done. Only motor needed plus ophthalmology
examination and fundoscopy, carotid artery and CVS examination.

Inside: The middle age lady aged probably 55-65yrs lying down on bed wearing cloths of her own
including short and hospital gown.

Me: I am so sorry to hear about your condition but to come to conclusion I need to do some examination
of your legs, eyes, neck vessels and heart examination, is it ok. She said ok.

Me: I will give u instructions as we go and you need to follow them, pl let me know if u don’t understand,
can’t do or in pain. Pt: Ok

Me: I kept giving running commentary (Pl give running commentary whether its mentioned in tasks or not
as it will save some time and examiner knows what u r doing)

Look: I don’t see any signs of trauma, redness, scars, deformities or any obvious muscle wasting.

Feel: Me: I am going to touch your both legs, pl let me know if u have any pain.

TTP: Then I touched thighs, calves and feet for temp (I said temperature symmetrical on both side). Pulses
were palpable on both sides. Then I started pressing her thighs, calves and feet, no tenderness noted.

Move:

I asked examiner I would like to check pt’s gait. Examiner: no need, normal.

I asked for heel toe walking: Examiner: No need, normal.

I am not sure I have asked for Romberg test. (I think it is not needed for this case)

Active movements for power (not for joints). I did not do passive movement as they are for joints:

Pl raise your both legs from hips (she did). (I thought I will ask for extention of hips at the end but
ultimately forgot and has not much time).

Me: pl bend your knees and straighten them (She did).

Me: pl move your feet up and down and u drive (she did).

I forgot to ask bout toes separately (probably would not needed, but good to ask).

Me: now You need to do these movements against my hand, is it okay? She: Ok
Me: Same movements against resistance (for power) all normal. Again, extension of hips was not done at
this stage.

Me: (for tone) now I am going to check tone of the muscles, so please let your legs go floppy. Pt: Pk.

Me: did tone for knee and ankle both sides. All normal.

Me: (For reflex) I am going to strike your knees and ankles with this hammer. Don’t worry it won’t be
painful. Pl let your legs go floppy.

Me: Did knee and ankle jerks. All normal. Forgot about planter reflexes (Examiner would have said normal
or not needed, but still good to mention).

I asked for vibration fork. Examiner: sensory done, but if u want to do then go ahead. Me: I will skip this
and proprioception.

Now I kept thinking about ITPRC for 15 seconds as I thought I missed something. But I did not.

Then I asked for fundoscopy examination directly without eye examination as I thought I was running out
of time. Examiner: Normal

I said I would like to do carotid artery examination. I thought examine will say normal but he said pl do.

I checked carotid arteries bilaterally with both diaphragm and bell. I said no bruit noted bilaterally.

I said I would like to do CVS examination. I was hoping again examiner would say normal. But he said pl do.

I asked for 45-degree angle. Examiner said that’s ok, go ahead. I quickly and only did auscultation on four
areas with diaphragm and bell (not sure about bell). I said no murmur heard. I was aware of dynamic
manoeuvres but did not do due to time problem. Then I checked the Pulse. Pulse were normal and regular,
mentioned to examiner. BP was given outside.

Dx: I think you have a condition called transient Ischaemic attack, do u know about it? Pt: NO

It is a called a mini stroke where pt have symptoms like u had and complete recovery occur within 24
hours. It generally occur when small fat part detached from neck arteries or heart and goes to brain or it
can occur when very small blood vessels break in brain generally occur when pt have high blood pressure
as you have.

Other less likely conditions can be possible such as Muscle problems ( less likely in you), joint problem
( less likely in you), back problem where nerves get pinched ( less likely because no symptoms now) or can
occur in some cases where problems in the brain. I did not have much time to give more DDS

Global score: 6 Key steps: 1,2,4,5 Yes. 3 no Approach to Pt: 6,

Choice & technique of Examination, organisation and sequence: 6 Accuracy of examination: 5 Dx/DDS: 6
10-Endometriosis
Your next patient at your GP is 28-year-old Samantha, complaining of severe abdominal pain
during menstruation since the last 6 months.
TASKS
- Focused and relevant history
- Examination findings from examiner
- Diagnosis
- Relevant investigations and management

Differential Diagnosis of Secondary Dysmenorrhea: (PEF)


1-Endometriosis
2-Fibroids
3-Pelvic Inflammatory Disease
4-Intrauterine Contraceptive Device

APPROACH
History
1-Pain questions
if in pain now
-Hi Samantha, I'm Dr. ---- your GP for today. I can see from the notes that you've been having
abdominal pains during your periods.
Do you have your period now? are you currently in pain?
-it must be very distressing for you. Let me assure that I’m her to help you. I just want to ask you
a few questions in order to reveal the nature of the problem.
Onset and Duration
- How long have you been experiencing this pain? (I've been having this pain during my
periods for the last 6 months)
-When does the pain start in relation to the periods? (It usually start 2 days before I get
my period, then it gets worse as my period starts)
-does the pain get relieved when the period flow started? (No)
-is there any pain in between the periods? (key issue)
Site and radiation
-Can you point exactly where you feel the pain? (It's around here in my lower tummy, and it's a
sort of a crampy pain )
-Does the pain go anywhere else? (Sometimes it goes right through my back.)
Character and severity (because she is not in pain now you can ask severity later)
-Can you describe it for me (sharp, throbbing pain)
-How severe was the pain from 1-10, 1 is the least 10 is the most? (It's around 8-9 on bad days
doctor)
Relieving and aggravating
- did you take anything to relieve the pain?
2-5P’s questions
Period history
-When was your last menstrual period? Around 3 weeks ago
-are they regular? Yes.
-How many days of bleeding and how many days apart?
- do you have heavy bleeding during menstruation? (fibroid)
-any pain in between the periods? (if you forget to ask this before)

Partner or Sexual history


-Are you currently sexually active?
-are you in a stable relationship? Yes, I have a boyfriend of 4 years.
-Do you experience pain during coitus? (dyspareunia for endometriosis) Yes oh my god doctor, it
hurts so bad during coitus
-have you or your partner ever been diagnosed with STI? (PID)

Pregnancy
- Any previous pregnancies or miscarriages in the past? None (infertility)

Pill
Do you use any contraception? (to rule out IUCD) Yes, I'm currently on the pill.

Pap
is your pap or HPV up to date? (It was just last year, everything's normal.)

3-Differential diagnosis questions (PEF)


PID
Any fever?
Any abnormal or offensive vaginal discharge?
Endometriosis
-Any pain during opening of bowels? Yes
-any pain on passing urine? Yes
Fibroids
-Do you feel any mass in your tummy? (fibroids) No.

Okay, thank you for those information Samantha, I'd just would speak to my examiner for a
while, I'd get back to your shortly.

4-General
Past medical or surgical history
SAD
Physical Exam finding from examiner
1-General appearance
there pallor, lymph node enlargement? BMI?

2-Vital signs (especially BP and Temp)

3-quick CVS and respiratory examination

4-focus on abdomen
inspection any visible distension, mass?
Palpation any mass (fibroid) any tenderness in the iliac fossae (PID)

5-Pelvic examination (consent and chaperone)


inspection
are there any abnormal/offensive discharge or bleeding from the vulva and vagina? Vesicles or
rash?
Speculum exam
any discharge or bleeding from the cervix?
Per vaginal exam,
is there cervical motion tenderness?
What is the position and mobility of the uterus? Any tenderness?
Any mass or tenderness in the adnexa?

6- Per rectal exam


any nodularity or tenderness in the pouch of Douglas and in the uterosacral ligament?

7-Office tests
I would like to do urine dipstick test and blood sugar level

Okay thank you for the information examiner, I would like to speak to my patient.

Diagnosis and Management


-Okay Samantha, based on your history, physical exam and office tests, your most likely diagnosis
is endometriosis. Do you know anything about this?

-Endometrium is the normal inner lining of the womb. During each period and under hormonal
influences, this endometrium is shed and it comes out with the bleed through the vagina. When
this endometrium is present in sites or places outside the uterus like in the pouches or the
ligaments surrounding the uterus or the ovaries this is called endometriosis.

-So during each period the endometrium at these sites also starts bleeding but there is no outlet
for the blood so the blood clots, form adhesions and scars causing pain during periods and in
between the periods.

-The exact cause is unclear, but one theory is the backward flow of blood through the tubes or
retrograde menstruation. Another theory is it can also spread through blood and lymphatics.

-If untreated, it can lead to complications such as dysmenorrhea or severe pain during pain ,
menorrhagia or heavy bleeding, and infertility.
Investigations (IMP)
- I would order blood exams for you such as a FBC, UEC, LFT, BSL, blood group.
- imaging tests such as pelvic Ultrasound and transvaginal ultrasound.
- Laparoscopy (key issue) refer you to specialist, where a flexible tube with a camera is passed
into your tummy to look for endometrium deposits. This is the investigation of choice and the
benefit is that it is therapeutic as well.

Management
-I would give you painkillers that you could use at the time of your periods, and refer you to an
OB-G specialist. (Key issue)
There are two types of treatment, medical and surgical treatment.

For medical treatment:


-you could take a combined oral contraceptive pill continuously for 6 months, skipping sugar
pills. You will not have your periods and the deposits will not bleed as well and start shrinking.

-Another option will be progestogens like Depo-provera or minipill and Mirena. Can cause
endometrial regression and can act as a method of contraception as well

-GnRH analogues can also be given which would induce a medical menopause. However it
should not be used for more than 6 months because you might develop severe menopausal
symptoms like hot flushes, bone pain and osteoporosis.

-lastly Danazol can be given which would also induce a medical menopause, but should not be
used for more than 6 months because it could cause you to develop male characteristics like
voice changes, hair growth because it is an androgen.

If medical management fails or if you present with infertility or severe symptoms, our next
option would be surgical management via laparoscopic excision which is cutting away or burning
away of endometrial deposits using laser or electrocautery.

I will give you reading materials and arrange for a review with you once your blood test results
are in.
Do you have any questions at this point?
Feedback 28-11-2017 Menstrual Dysfunction - Pass
Lady having pain during periods.
Task:
-Take history.
-Ask PEFE .
-Explain diagnosis to patient and arrange investigations.
( s/s for 12 months. Pain started before the period and stays the whole
time and goes away after the period. Dyschezia, Dysperunia present. No
Dysuria, no abdo pain or mass)
Examination – Left adnexal tenderness./this finding was not given to
some candidate. )
Key steps: ¾
Approach to pt: 4
History: 4
Choice of examination, organization and sequence: 6
Dx and DDx: 5
Management plan: 4
Global score: 5
History:
Is it first time?
PAIN: SORTSARA:
d/d endometrosis: pain during passing stool, passing urine and during
intercourse., previous pregnancies (infertility)
D/d Fibroid: Normal period question(increase bleeding)? lump in tummy?
D/D: PID: Previous history of PID? Fever? Vaginal discharge? Rash? Lower tummy
pain.
D/d IUCD: Any methods of contraception (pill/iucd)
Previous surgery? (adhesions)
Pap smear?
HOW IS THE PAIN AFFECTINGYOUR LIFE? SADMA? FHx? Past mhx?
(If you ask 5Ps all the dds will be covered)
Exm:
GA: PICKLED (asked whichever relevant- Pallor, Dehydration)
Vitals: Pulse, BP with postural drop
Adbomen: Inspection/palpation: marks, distention, tenderness
Pelvic examination: Inspection, Speculum examination, bimannual examination:
Left adnexal tenderness./this finding was not given to some candidate.
Complete the exam with other system
Explain the diagnosis:
Most likely a condition called endometriosis which is a painful condition in which
the tissues that line the inner lining of the womb is deposited at sites other than
the womb (draw picture). Can also be because of other conditions like any
inflammation of womb or associated organs (PID), any device in the uterus (iucd),
any mass/lump in womb (fibroid), previous procedures or operations done.
(while mentioning the differential diagnosis explain why it can be and why it
cannot be for the particular patient)
Arrange investigations:
- Basic blood tests, USG(scanning), specialist will do futher investigations like
looking inside the womb through a camera which we call laparoscopy.

Feedback 8-11-2018
11-Rectal bleeding Hemorrhoids
A 52 or 62 man with bleeding per rectum
Tasks:
-History
-specific findings from the examiner
-diagnosis and differentials
-further management

Note/ sometime the tasks without PEFE and others just Ix and no management, others all so like
preeclampsia case the task can be modified so I put everything in this lecture.
You need to know how to define every Ddx in one sentence in lay term from the website

History
1-is my patient hemodynamically stable?

2-Bleeding questions
-when did it start? (2-3 months)
-how much blood have you passed? (Moderate)
-was it bright red or dark red in colour? (Bright red)
-is it on the tissue paper only? (Anal fissure) or is it like streaks of blood on stool surface?
(Hemorrhoids) or is it mixed with stools? (IBD/ Colon cancer) (Separate from stools/ at timed on
paper)
-has your stool been black tarry? (Upper GIT bleeding)
-is there bleeding anywhere else? (Bleeding disorder) (No)
-has this happened before?

3-associated symptoms questions


-do you have pain on passing motions? (Anal fissure)
-do you feel a lump while straining to pass motions? Does it go away on its own or you need to
push it back? (Hemorrhoids) (No mass/ some said can feel lump down)
-have you had any constipation, diarrhea or alternating constipation and diarrhoea?
(Diverticulosis, hemorrhoids, IBD, colon cancer)
-have you had any tummy pain? Nausea and vomiting? Fever? (Diverticulitis)
-any loss of weight and appetite? Lumps or bumps? (Colon cancer) (no)

4-risk factors
-do you take any medications? (Blood disorder)
-Past medical history/ travel history.
-can you describe your typical daily diet. Do you eat fruits and vegetables and drink plenty of
water?
-what do you do for living? Do you need to sit for long periods of time?
-SAD
-family history: has anyone in your family had a history of colon cancer? (Yes father)
Physical examination from examiner
1-General appearance
pallor/ jaundice/dehydration/ LAP/ BMI

2-vital signs + postural drop

3-Focus abdomen
inspection: distension, mass
palpation: tenderness, mass, organomegaly
auscultation: bowel sounds
hernia orifices

4-Digital rectal examination (with consent of patient and lubricated gloves)


-inspection: fissure, scars, skin tags, hemorrhoids
-blood on gloved finger
-ask patient to strain and then feel for mass

5- office test (proctoscopy) (+ve hemorrhoids)


grade 1: no prolapse just prominent blood vessels
grade 2: prolapse upon bearing down but spontaneously reduced
grade 3: prolapse upon bearing down and require manual reduction
grade 4: prolapsed and cannot manually reduced

Counselling
-from history and examination you most likely hemorrhoids. Have you heard about it?
-all right, I will explain it to you with a diagram.
-in the lower end of the bowels, there are blood vessels within the supporting tissues of the
bowels wall. Sometimes because of increased pressure in the abdomen or long-term
constipation, these vessels are subjected to increased amount of stress, thereby becoming
enlarged and start to bleed giving blood in the stool.
-this is common benign condition in your age group.
-Other possibilities are
Differential diagnoses
1-internal hemorrhoids
2-anal fissure: a small tear in the lining of anus
3-colonic polyps’: small growths of tissue on the wall of the colon.
4-colon cancer
5-Inflamatory bowel disease
6-diverticulosis: formation of abnormal pouches in the bowel wall
7-mesenteric ischemia: inadequate blood flow to the small bowels.
If patient, ask do I have cancer or what is nasty growth
-look it does not mean that you definitively have it, but it is one of the things that I have to think
about. We still need to do Ix to rule it out and it is part of routine workup.

Investigations
-now I would like to take some blood samples for FBE.
-I would also like to refer you to a gastroenterologist specialist for colonoscopy as you are over
50 years and have a positive family history cancer to rule out any nasty growth.
(Colonoscopy is a tube attached to a camera pass through the back passage to check the inside
of the bowels if they saw anything suspicious they will take a sample from it to be examined
under microscope)
-I would like to refer you to general surgeon for treatment of your condition:
there are two options depending on the grade
1-non operative (grade 1 or 2)
-rubber band ligation
-injection of sclerosant
-cryotherapy
2-operative (grade 3 or 4, or if not improving)
-hemorrhoidectomy

-the best preventive is to follow life style modifications:


1-make some dietary changes, increase intake of fibers in form of fruits and vegetables and
whole grain food
2-improve your water intake
3-maintain weight within normal limits
these dietary changes can help by softening of the stools.
4-make sure not to sit in the toilet for prolonged periods

21-2-2018
CSAE 8: BLEEDING PR IN ELDERLY-PASS
62 year male with bleeding pr.
Tasks:
history,
Dd and management

vitals stable, bleeding few days,relation to stool-after pasing stool sometimes slightly mixed few
streaks, bright red, no mucus/loose stool/fever. Sometimes bit difficulty in passing stool . No
marked change in the stool pattern. No loss of weight/appetite/lumps and bumps.not on blood
thinners. Not undergone FOBT, no family history
Dds: fissure, haemorrhoid, bleeding problems, nasty growth. Patient asked what is nasty
growth , I mentioned for the safer side considering your age we do it to pick up early for good
prognosis.

Drew a picture and explained the dds.

Case (29/4/2017)
A man with bleeding per rectum
Tasks:
h/o
specific findings from the examiner,
diag and dds with Ix
2 mins: haemodynamic stability, anal fissure, haemorrhoids, polyps, ca, ibd, bleeding disorders,
blood thinners, giardiasis, ibs, pseudomembranous enterocolitis.
Asked the examiner for vital signs- stable
Reassured the patient and asked him to tell me if he is uncomfortable during the examination.
History
- Sob/ dizzy/ tired- no
- Bleeding
whether spots on the toilet paper- at times
Separate/ mixed with the stools- separate
Bright red, moderate qty, no clots, no foul smelling, for 2 to 3 months
- Any mass protruding from bottom- no
- Diarrhoea/ constipation- no,
- but when asked about prev episode- yes when he returned from Thailand for a month.
Antibiotics for that- yes. Did the bleeding start after the use of antibiotics- no.
- Loss of appetite/ weight- no.
- prev h/o of ca- no. family h/o of ca- yes, father had bowel ca
- Rash/ fever/ sore mouth/ red eyes/ joint pains- no
- Stool consistency (frothy/ greasy, difficult to flush), frequency, passage (explosive urge), foul
smelling- no; black stools- no
- Bleeding from else where- no
- Any blood thinners- no
- Any stress- no
- SADMA- no; prev hospitalisation/ surgeries- no

PE: gen appearance


- PICKLE, vitals again esp bp with postural drop, pr
- Cvs and rs- normal; rash/ red eyes/ mouth ulcers- no
- Abd:
inspection: no mass/ distension/ visible peristalsis
Palpation: no tenderness, mass
Auscultation- BS normal; hernia orifices- normal
- Pr with the consent of the patient
inspection- anal fissures- no.
Proctoscopy (ask specifically or the examiner wont tell the findings)- haemorrhoids positive
Palpation of the rectum- normal
- Office tests – unavailable

Most probable diag


- haemorrhoids- dilation of the blood vessels lining the anal canal
Causes- constipation, lack of dietary fibres (ask here for his diet)

Dds- , anal fissure, haemorrhoids, polyps, ca, ibd, bleeding disorders, blood thinners, giardiasis,
ibs, pseudomembranous enterocolitis

Ix:
fbe esp hb, serum fe studies
stool microscopy and culture (giardiasis),
colonoscopy (due to family h/o)
Global score- 5

Case (22/4/2017)
be careful may be diverticulosis so ask proctoscopy/ sigmoidoscopy/colonoscopy
Abdominal pain on left side abdomen..he is a farmer ..
in hx he said he have constipation from long time ..take Laxatives.. no other medications..and
asked about bowel and water works.. fine he said .. any blood in bowels .. only on tissue paper
he said .. any pain he said no.. then I thought about haemorrhoids ..
Pefe: she said only finding is appendicectomy scar and I thought it's a diversion so I didn't ask
anything about it..
I told I am suspecting haemorrhoids as u have constipation and straining and started drawing
diagram telling them I am not gud at drawing he said it's ok dr.. said about diverticulitis and
diverticulosis.. told will give painkillers.. have more fiber diet more water as ur farmer.. don't use
more laxatives as u can addict.. I will tell all my patients natural ways than medications. If it get
bad they will do rubber band ligation I said.He asked me u want to tell me anything said will give
u reading material for all the conditions I explained..he said it will be gud.. will see u in 3 days
meanwhile u have severe abdominal pain or more bleeding come back..thanked and came back

Case (4/10/2017)
Bleeding – Fail
52 year old man with rectal bleeding, a topic I’ve read very well. Tasks were history, explain to
patient examination that you will do and management.

When reading outside I remember thinking- How do I do this? Should it be like “I will check your
general appearance, vital signs etc.“ I went in and took a lengthy history/ gave DDs …Then forgot
about examination and went into management. I think you only have to tell about PR/
proctoscopy as it was a case of haemorrhoids. I think I have misunderstood a task here. Always
keep the paper given in front of you and complete all the tasks. ¾ keysteps covered.
Feedback 7-9-2018
Station 10 – bleeding per rectum –PASS

rectal bleeding after passing stool, 67 years old (external haemorrhoid)


Taking aspirin , nil family history of bowel problems)
Task
1. history ( 5 minutes )
2. explain the patient about what examination and investigations you want to do with reasons.
3. Explain possible causes to the patient

Task -1 –
Asking the examiner that – my patient is haemodynamically stable or not – the examiner said
that – as you see and you proceed your test – I appreciate to the examiner and – thank you for
your information , examiner -
open-ended question about bleeding - mention bleeding after opening bowel. Lumps comes out
and initially reducible but not now. , ask about SOB, tiredness – NO
asking about fever, N&V , weight loss, loss of appetite – NO
Then asked about risk for haemorrhoid.
Constipation – nil , jobs – I forget it , asked him , but not lifting heavy objects in his life
Chronic medical condition – like chest infection , recurrent coughing.
Medication – asked any regular medications- nil apart from aspirin , asked specifically – any
blood thinner like warfarin ? – he said – no
Family – nil bleeding disorder , nil bowel cancer
Sexual – asking about sexual ly active – he said - YES , ask about sexual preference – he said –
what do you mean – gentlemen or ladies for sexual practice – he smiled – doctor I am straight – I
appreciate him and said “ Thank you for your co-operation “ . He kept smiling – I tried not to
smile myself 
SADMA – I forget , he said possibly – NO

Task -2

- told the patient about


- examination to you including vital signs to see infection , fever , dehydration , pale or not –
Examiner didn’t tell me anything when I told the patient about running examination procedures.
Then, I will check your tummy to see any big organ or any pain ,
Then check your back passage with gloved fingers, ( I forget to tell chaperone ) , to see bleeding
or not , any cracking of your back passage what we called anal fissure or not , any lump like piles
or not , Then I will do proctoscopy into your back passage – little discomfort, if you cannot
tolerate , I will stop it . And – routine blood tests in details –

FBE – where we can see infection , bleeding problem and low blood count ,

Kidney , liver function tests – I explained to the patient , sometimes liver problem can cause –
bleeding from the body .
- bleeding profiles –

Task 3 –
Possible causes –
most likely – to be piles what we called haemorrhoid ( draw a picture to the patient ) – and
explain the grading with C-Section rectum , ask the patient , are you still with me ? – YES
other possibilities – inflammation – such as colitis and inflammatory bowel disease ( I didn’t
explain in details this one , he seems to be confused when I told IBD )
sometimes – cracking of anal mucosa and sphincter – anal fissure , anal tear
nasty conditions as least likely – such as bowel cancer

Feedback – Bleeding per rectum – PASS


Global score – 4
Key step 1 –yes
Key step 2 – no
Key step 3 – yes
Key step 4 – yes
Histoy - 4
Mgmt arrangement and examination – 5
Dx and DDx – 5
Feedback 7-9-2018 (haemorrhoids)
Bleeding- PASS)
67 years age old male, c/o rectal bleeding for 2 weeks.? Taking aspirin. Vitals were
mentioned I guess, all stable.
-HX
-DDX
-INVX
FEEDBACK-PASS
Global score: 4
Key steps: 4/4
HX: 4
DDX: 5
MX : 5
2 mint thinking: I was happy to get this case!! Finally, one case just hx, ddx!! No PEFE, I
will get enough time then, for explaining my ddx!!
* have to rule out malignancy.
DDX:
1. Anal fissure
2. Haemorrhoids.
3. Rectal growth/malignancy
4. Rectal polyp
5. Colorectal cancer
6. Diverticulitis.
7. Bleeding disorders.
8. Any blood thinning medication.
There was a thin and old guy, sitting inside comfortably. I started to ask about bleeding
details- how much, when, how long, colour of the bleeding, mixing with stool, increasing
with time or not.
Bleeding follow up questions- are you feeling tired? racing of heart? blood group?
Haemorrhoid questions- something coming down below? going back by itself? painful?
Stool hx- hard? colour? constipation?
constipation positive- asked about diet, water intake, any medication.
Then, malignancy questions- asked weight loss, lums n bumps, family hx, have you done
FOBT before?
Talked about aspirin(stop it right now) any bleeding problem.
I didn't ask SADMA in details. Just alcohol.(asked about medication already) I didn't ask any
other past or medical hx.
Then, I scribbled in the paper, draw a pipe and an opening, just like gut, rectum and anal
area. I explained all the ddx to him one by one along with explanation of haemorrhoid.
INVX:
Full blood invx, Hb level, Iron studies, LFT, blood group, clotting factors.
Colonoscopy and upper GI endoscopy.
If there's no fissure, I'll do proctoscopy right now, with presence of a chaperon, to check the
haemorrhoids.
I just mentioned, if anything comes out positive, we will treat you accordingly. Haemorrhoid
is a manageable condition. Then, talked about alcohol consumption, take 2 days off, try to
cut down as much as possible. ( he was drinking alcohol everyday 2/3 standard).

12-Pericarditis
Case 1 (14-3-2018 and 15-8-2018)
A woman complained of Chest pain. An ECG was provided that showed ST elevation in almost all
of the leads.
Tasks
-History
-explain ECG to examiner.
-diagnosis and differential diagnoses

Case 2 (1-6-2018)
52-year-old man presented with chest pain
tasks
-history
-Physical findings from examiner (No ECG)
-Dx and Ddx to patient (no Ix or Mx)

History (case 1+2)


1-Is my patient hemodynamically stable? (Stable)

2-Pain questions
-how severe is your pain from 1 to 10 1 is the least 10 is the most? (5) Offer painkillers if they
want
-when did it start? (6 hours)
-has it started suddenly? Is it constant or come and go? Is it getting worse?
-where exactly the pain is? Does it go anywhere else? (Centre of chest radiated to jaw, left arm
and back)
-can you describe it for me? (Like a stab)
-does anything make it better like rest, leaning forward? (Relieved by leaning forwards)
-does anything make it worse like chest movement, deep breathing or lying flat? (Worse by lying
flat, deep breathing)
-is this the first time?

3-associated symptoms questions (all No)


-any fever? Dry cough?
-any sweating, palpitations, SOB?
-any swelling of tummy or feet?
-any fatigue? Nausea or vomiting? Heartburn?
-bowel motions?

4-pericarditis questions (when you see the pain is most likely pericarditis)
-have you had a recent upper respiratory infection? (URTI 1 week ago)
-any trauma to the chest?
-any loss of weight or appetite?

5-Risk factors questions (all good)


-have you had a past history of high blood pressure, diabetes, or high cholesterol level?
-any history of heart, kidney or liver disease?
-any exposure to TB? Travel?
-do you smoke? Drink alcohol?
-do you take any medications?
-has anyone in your family has a history of heart problems, stroke or any illnesses?

Physical examination from examiner (Case 2)


1-general appearance
pallor, cyanosis, dyspnea, oedema (pedal and sacral oedema), dehydration, LAP

2-vital signs (all)

3-JVP

4-Heart
-apex beat? Thrill and heave?
- Heart sounds and murmur? Any pericardial rub? (Pericardial rub +ve/ muffled heart sounds)

3-chest
-air entry/ breathing sounds/ any basal crackles

4-Abdomen + DRE
-distension, mass
-tenderness, organomegaly

5-fundoscopy

6-office tests
-urine dipstick/ BSL
-ECG (pending)

Explain ECG to examiner (Case 1)


It had normal rate, regular rhythm and normal P wave. I pointed to all ST elevations in the leads
with concavity consistent with Widespread ST elevations in Pericarditis.
Explanation (Case 1+ 2)
-based on history, examination and ECG findings, it does not look like that you have a heart
attack.
-you most likely have a condition called pericarditis, which is inflammation of the outer covering
of the heart. It usually follows viral infection, bacterial or trauma. It could be associated with
heart or kidney diseases or medications. The commonest is viral infection. It usually presents
with …. .
-other possibilities could be infection of the lung called pneumonia, air trapped in the
membranes around the lung called pneumothorax, clot in one of the arteries supplying the lungs
called embolism. Could be a stomach ulcer, spasm of food pipe, trauma or chest muscle spasm.
All are unlikely from history and examination (you can mention why unlikely from history if you
have time)
-In order to rule out these I need to refer you to the hospital, where they will do some Ix like
(FBC, ESR/CRP, UCE. TFT, LFT, Lipid, troponin) and they might do imaging like (chest x-ray, echo)
-Pericarditis usually improved with some rest and painkiller NSAIDS.
-once you discharged, I will review you+ reading materials.

Differential diagnosis
1-Myocardial infarction, angina
2-pericarditis
3-pnumonia, pneumothorax, pulmonary embolism
4-stomach ulcer, esophageal spasm, GERD
5-musculoskeletal spasm

Causes of pericarditis
1-idiopathic
2-viral (commonest; coxackie)
3-TB
4-post MI (dressler)
5-post traumatic
6-uremia
7-malignancy
8-medications (INH, cyclosporine, hydralazine)
9-RA, SLE, radiotherapy

Management plan
-if viral or idiopathic rest + NSAIDS
-if bacterial- antibiotic+ colchicine+ steroids
-if recurrentcolchicine+ steroids
Case (10/2/2017) (1/3/2017)
-Pericarditis. Chest pain+ shoulder pain+ back pain. Task: Hx, ask PE findings (will tell them if only
specifically asked), DDs with reason. Positive: pain in the chest, radiating to the back, pain
increasing with breathing, improving when leaning forward. Common cold 1 week ago. S1-S2
presents + ( only if u ask about added sounds ?noisy inspiratory and expiratory sounds. Ask for
pericardial rub(!) DDs: Pericarditis, MI, pleural edema and Pulmonary embolism.
Most likely pericarditis and draw in diagram . I passed this case .

Case (1/6/2017)
Chest pain radiate to left arm
No underlying casue for MI, heart sound gave noisy heart sound
Dx pericarditis

Case (9/11/2017)
Pericarditis. Task- Hx, PEFE ( pericardial rub + ), Tell Dx and DDx, Mx of PDx.

Feedback 14-3-2018
Young woman with chest pain – ECG shown outside room. Hx-no PE – Counsel
Young woman – hx of flu like illiness 2 weeks ago – chest pain – related to movement – better
when leans forward – radiates to left shoulder – ECG – T inversion in some leads only – told her
this is mostly Pericarditis – DDx ACS – MI – Trauma – Pneumonia. We need a CXR – FBC/CRP and
referral to ED for admission. Reading Material handed to patient – She thanked me.

Feedback 14-3-2018
Chest pain with ECG interpretation to examinebr and diagnosis to ptient (pericarditis)
Symptoms of pericarditis but ECG was anterolateral MI
Hx
Dx and ddx to the pt

Feedback 15-8-2018 Failed because he diagnosed the case as MI


Next p,tient in ED is 60 yr old m,le compl,int of p,in in the chest which r,di,ted to shoulder ,nd ,rm
this morning. Vit,l signs provided ,nd st,ble.ECG is ,lso provided.
T,sk:history
Interpret ,bnorm,l ecg
findings to ex,miner
Expl,in dx ,nd ddx to p,tient
-Offered p,inkillers,,n,lyzed chest
p,in ,nd noted fe,tures of both AMI ,nd peric,rditis,explored the risk f,ctors ,nd found sever,l for
both condition
Interpreted ECG ,s l,ter,l.
AMI.
Expl,ined dx ,nd ddx but r,n out of time so they f,iled me with ddx
Feedback 14-3-2018
A woman complained of Chest pain. An ECG was provided that showed ST elevation in almost all
of the leads.
Tasks were Hx, diagnosis and other possible causes to examiner and explain ECG to examiner.
2 min thinking time.. MI, Angina, arrhythmias, pericarditis, pulmonary embolis, pnemothorax,
pnemonia, peptic ulcer, GERD, herpes zoster, costochondritis.
As I entered the room, the role player was sitting comfortably. I really got confused about
ensuring vital stability or not as the role player looked so well and healthy. I skipped on that part.
After introduction, I asked if she could describe the pain for me. She asked me what I wanted to
know. I asked since how long did she have it? It was there since 6 hours. On further questioning
she told it felt like a stab. Was in center of the chest and radiating towards jaw and left arm and
to the back. It was the first time she had such a pain. She graded it to be of 5 on the pain scale. It
was made worse when she tried to lie down. I asked the affect of coughing and deep breath
when she told it worsened with the latter.
She had no cough, sputum or fever. The pain was’t associated with exercise or food. Resting had
no relieving effect. It wasn’t associated with any sweating, dizziness, increased racing of the
heart beat. There was no trauma, no rash on her chest. The only other positive finding was an
URTI a week back for which she took medication. She hadn’t noticed any change in the colour of
her bowel motion. She hadn’t travelled anywhere recently.
No past or family history of clots in legs. No recent surgeries.
Cardiovascular risks were all negative i.e. no HTN, no elevated lipid or previous heart disease.
She had normal BMI and was in good health otherwise. No use of NSAIDs. I don’t remember her
response for Smoking and Alcohol.
I turned to face examiner to explain ECG. It had normal rate, regular rhythm and normal P wave.
I pointed to all ST elevations in the leads with concavity consistent with Widespread ST
elevations in Pericarditis.
I got back to the role player and drew and explained that it was most likely due to her URTI that
she had a week back that had caused inflammation of the covering of her heart, known as
Pericarditis in the medical term. It was supported by points in her history like pain made worse
when lying down because that position caused the inflamed covering to come in contact with
her rib cage so it caused pain.
So would do a couple of further tests like blood tests for antibodies, chest x-ray and usg of the
heart known as echocardiograph. I gave her some medication and advised about maintaining
posture to avoid pain.
Other causes could be MI which was a serious one and was an emergency but she didn’t have
ECG findings nor the history supported it. It could be Angina but it was not related to exercise. It
could be PE but her history wasn’t suggestive of that. And told all other and how I had ruled
them out.
Scenario … Chest pain
Grade… Pass Global score…. 4 Key steps 2,3,4,5…. Yes 1… No
Approach to patient… 3 History…. 4
Interpretation of investigation… 4 Diag/ D/D…. 4

Feedback 15-8-2018
Scenario : Chest Pain
Stem:A man( forgot the age) with chest pain radiating t back and shoulders. ECG given
Tasks:
~History
~explain ecg to examiner
~dx and ddx to patient
A middle aged man was sitting on chair leaning forward on table and looked in pain.
I started with introducing myself and shown empathy as he was in pain. Asked pain scale, allergy
and offered pain killers. Excused him and asked vitals from examiner. He said normal. Thanked
examiner and turned back to patient, said I can see you are in pain im sorry about that but I
need to ask few questions in order to find what is causing this. To this examiner said this is the
thing I have been looking for( to my surprise). Took pain history and ruled out ddx( keeping in
mind MI, dissection, anemia, pericarditis, pulmonary emb, effusion , pneumothorax, pneumonia,
gord, ulcer, MS related, herpes zoster, psychogenic etc). Positive was pain relieving on leaning
forward and he had a recent urti/fever.( as far as I remember)
Then explained ecg to examiner
Checked name and identity (examiner said yes it belongs to him)
Roughly commented on rate, rhythm and told about st elevations( upward concavity) He asked
can you show me where? I pointed out. And said this finding is consistent with a condition called
pericarditis.
Gave dx of pericarditis and explained it.. and ddx same as mentioned in history.
Grade: Pass

Feedback 1-6-2018
c/o chest pain, Hx, PEFE, DDx
aggravated on deep inspiration, better on leaning forward, radiating towards shoulder.
PE pericardial friction rub +
DDx. Acute pericarditis, MI, myocarditis, Aortic dissection, esophageal rupture, Pneumothorax,
pneumonia, costochondritis, herpes zoster, GORD, trauma
Feedback 1-6-2018 (different pattern of similar case)
Chest pain left side radiate to back and left shoulder.
Hx, pefe, dx and did to pt( recent flu and ex - pericardial friction rub. No need for Ix, mx)
HX:
Asked hemodynamic stability as k nee jerk: examiner said its normal, then shifted my pt to
treatment bay , hooked him to cardiac monitor and asked nurse to draw investigations,
maintain iv line and do ecg.
Went to pt asked if he is in pain, how bad is it he said 5-6 , I asked if he wants a pain killer he
said no , asked to locate the pain he pointed at epigastrium, character: dull radiating to back,.
Aggrevation on lying down and cough or deep breath and reliefed on leaning forward associated
flu a week back, no association with food, alcohol, no racing of heart, cough was dry, no fever
settled now, no hx of gall stones, no hx of abd sx, no hx of abd trauma, no acid taste in mouth,
no joint pain issues, no psychiatric / functional issues.
PEFE:
Vitals: normal
GPE: no anemia, jaundice, peripheral or central cyanosis, clubbing, LAD, rashes, dehrdration
PULSES: regular good volume
CVS:
Inspection: apex beat not visible
Palpation: no thrill or heave
Auscultation: muffled heart sounds, s1&s2 normal examiner will only give muffled sounds and
rub upon asking) pedal edema: negative. Jvp: normal
Chest: normal no basal crepts
ABD: soft non tender, bowl sounds normal (examiner expedited me here)
Urine dipstick: normal, ECG: pending, rbs: normal.
DD’s
There could be loads of reasons behind u having tummy pain, took paper and pen drem the
chest then heart and then a covering arounf heart told him that most likely he is suffering from
pericarditis which is inflammation of covering of heart secondary to viral infection . he was
struggling with dx so I broke the terminology for him : PERI : around, card : heart, it is:
inflammation (he was satisfied )
Then quickly gave him DD’s
MI,ACS
Arrhythmia (mesenteric ischemia , af, svt)
Pneumonia
Cholecystitis, cholangitis, cholelitiasis, pancreatitis
Pud, gerd
Trauma
Panic attack.
Feed back passed Global scor:6
Key step : all covered Approach to pt : 5
Hx:6 Choice and technique of exam,: 6
Dx & DDx: 6
Feedback 1-6-2018

c/o chest pain, Hx, PEFE, DDx


Assessed hemodynamic stability, offered pain meds. Then proceeded.
aggravated on deep inspiration, better on leaning forward, radiating towards shoulder.
PE pericardial friction rub +. ECG not done.
DDx. Acute pericarditis, MI, myocarditis, Aortic dissection, esophageal rupture, Pneumothorax,
pneumonia, costochondritis, herpes zoster, GORD, trauma
Told that pericarditis is most likely, drew a pic and told probably d/t episode of viral infxn. Gave
ddx and why others are unlikely.

Feedback 20-7-2018
Pericarditis ( unscored )
52 yr old , chest pain that radiate to back , shoulder , and neck
History
PEFE
Investigation from examiner and interpret to examiner
Dx and DDx to patient

There was a man sitting on the bed , i introduce and offer pain killer , ask whether
hemodynamically stable or not ..... examiner stick to your task
i start history ,
pain question ... pain in center of chest , radiating to back , neck and left arm , started this
morning , severity , worsened by breathing and coughing , reliving factor ( do you feel
relieved when you lean forward ... said not sure ) , How about lying flat , does it get worse ?
said not sure
patient offered information by himself ... he got the cold 2 wks back ,
no fever , cough and cold currently
no SOB , no wheezing
no sweating , no nausea , no vomiting , no dizziness ,
no trauma to chest
no rash on the chest
no recent travel , no recent surgery , no prolong immobilization, no leg pain or swelling
PMH - nil
PSH - nil
Family history - Nil
occupation- executive officer
SADMA not positive
I returened to examiner for PEFE ,
GA , no pallor , no Jaundice , Vital sign HR increase( round about 100) , temp
37.4 , RR and BP normal , BMI i dont remember ,
REsp normal
CVS normal first and second sound
any added sound .... nosiy sound present
is it pericardial rub.... yes rubbing in nature
any murmur ..... no
no features of heart failure ( BBC JVP, Oedema),
ECG interpretation
this is ECG of mr who presented with chest pain , done on today ,
Rate is /min
regular rhythm
P wave and QRS complex are normal
but there is wide spread concave ST elevation and slight PR depression in all leads apart from
aVR and V1 which has ST depresstion
( pointing out the findings to the examiner in ECG paper )
Examiner asked me to explain Dx to patient
John , based on the PE findings and your history , your chest pain is most likely due to a
condition called pericarditis , have you ever heard of it before . it is the inflammation of
covering of your heart , normally our heart is covered by a covering called pericardium . as
you recently had the flu , sometimes viral infection can affect on the heart and and causes
pericarditis..... the covering of heart can rub against the heart at the time when you cough or
deep breath , thats why you have more pain at those times.
other important thing we need to rule out is , heart attack , but based on findings and ECG ,
its less likely but we need to repeat the Ecg and blood test to check for heart enzymes to be
sure
when you have chest pain ,
we need to think of heart problem , which i have mentioned
lungs problem, like infection ( pneumonia ) , blood clot blocking in blood vessel of lungs (PE) ,
pneumothorax ( air leakage from the lungs )
musculoskeleton problem , like inflammation of ribs called costochodritis
skin problem like herpes rash
food pipe problem like acid reflux
but they are very less likely.
13-PPROM
25 year old Mary, who is 32 weeks pregnant, presents to you at your GP clinic with complaints of
passing fluid from vagina since the past 1 hour.
TASKS
-Relevant history
-Examination findings from examiner
-Management

Differential diagnosis
1-PROM
2-Urinary incontinence
3-abnormal vaginal discharge

APPROACH
History
1-Fluid questions (duration-action-trauma-amount or severity-colour-odour)
-how long have you been passing fluid?
-what were you doing the time you passed fluid?
-did you hurt yourself or have any trauma to your tummy?
-How many pads have you used? are they fully soaked?
-What is the color of the fluid? Is it like greenish or is it just like water? Is it blood stained? (Key
issue)
- Is it smelly?

2-Late pregnancy complications questions


-Any pain in your tummy (preterm labour) (Key issue)
- Any fever , nausea and vomiting or abnormal vaginal discharge before this happened?
(Infection) (Key issue)
- How’s your urine? Any leakage of urine, burning or stinging during urination (urine problems)
-how’s your bowels habit? (Bowels problems)
- Any bleeding from your vagina? (Placenta Previa, abruption etc..)
-any headaches, bluring of vision or leg swelling? (Preeclampsia)
-do you feel your baby kicking well or not? (Baby problem)

3-Regular Antenatal visit questions


-Have you had regular antenatal visits? When was the last one?
-have you done Down syndrome screening in the first trimester?
-have you done ultrasound at 18-20 weeks ? Repeat US at 32 weeks?
-have you had sugar test at 28 weeks? Was it normal?
-are you aware of your Blood group? (Key issue)
-did you take Folic acid in the first 3 months of pregnancy?
4-General questions (Imp but If I had no more time I would leave them)
-Smoking, alcohol, recreational drugs, medications, allergy
-Past medical and surgical history
-Support from partner

Physical Exam from Examiner:


1-General appearance (PODL)
Pallor, Oedema, dehydration, LAP

2-Vital signs (especially temperature and blood pressure).

3-quick chest and heart examination

4-focusing on abdominal examination: (key issue)


- Any Tenderness on plapation?
- Any uterine contractions per abdomen?
- Fundal height? (32 weeks)
- FHR?
- Fetal Lie? Presentation? (Longitudinal, cephalic)

5-Pelvic exam:
Consent
Inspection of the vulva and vagina:
-Any fluid leaking out of the vagina? (yes)
-Trickling of the fluid or gush of fluid that I can see?
-Color of the fluid? Smelly or not?
-Any other abnormal bleed or discharge that you can see?
-Any rash or vesicles?

Sterile speculum exam


-any fluid leakage from the cervix? (if there is discharge from cervix, it invariably tells you that it
is coming from the uterus)
-Is the OS open or closed? (Key issue) closed
-Can you see fluid collecting in the posterior fornix? (If yes, it is amniotic fluid) (Key issue) Yes,
you can see fluid in the posterior fornix

**I'd like to take


1-a sample of this fluid and send it for nitrazine test (to confirm if amniotic fluid)
2- vaginal swab and a cervical swab and give it for microscopic culture and sensitivity.
3- Low vaginal swab and an anorectal swab for GBS.

Do not do a pervaginal exam.

6-Office tests:
I'd like to do a UDT and BSL.
Explanation
-From history and examination you most likely have a condition called premature rupture of
membrane.
Normally when labor sets in, around 40 weeks, it is labour pain that happens first and it is then
followed by the rupture of the membrane or bag of water. But if the membrane rupture, before
the onset of labour pain, it is called premature rupture of membranes or PROM, and if PROM
happens before 37 weeks, it is called PPROM or preterm premature rupture of membranes.

-There could be several causes of PROM. But anything that overdistends the uterus can lead to
PROM like: (you don’t have to list all the causes) and sometime the cause is unknown.
1-polyhydramnios (excessive fluid in the bag of water).
2-multiple pregnancy (twins).
3-cervical incompetence.
4-maternal infections.
5-gestational diabetes causing big baby
-preeclampsia (sudden sever rise of BP during pregnancy)

-One of the complications that happens in PROM is infection. WE need to be careful about this.

Management
1-GP role
-You need to be referred to a tertiary hospital with a neonatal intensive care unit. (Critical
error) I'll arrange for an ambulance and call the hospital and liaise with the ED.

-I need to start you on an IV line with slow IV drip, and take blood for certain investigations like
FBE, ESR, CRP, UEC, Blood group, blood sugar level, coagulation profile. I will also send urine for
microscopic culture and sensitivity.

-I will give you your 1st dose of steroid, to bring about the lung maturity of the baby just in case
you progress into labor. (Key)

-I will also give you your first dose of antibiotic, erythromycin, and you need to be continued on
it for 10 days. (Key)

2-Hospital role
-Once you reach the hospital, you will be:
Admitted, seen by the specialist, who will do an ultrasound and CTG.

If Severe leakage (Do not say mild or severe just talk in general)
-need frequent monitoring, further steroid dose and continued antibiotic use.
-Just in case the labour doesn’t start the labour might need to be induced at 34 weeks gestation
-If she has any signs of infection like fever, blood counts going high, or baby becomes unwell,
delivery will be planned immediately either by induction or C-section.

-Do you have support? Anybody to phone in to be with you?


-I'll arrange a review with you once you are out of the hospital.
Notes (NOT THAT IMPORTANT)

**If mild
-advise bed rest till leakage stop and once stop and if CTG and US normal then she can be
discharged home and also if swab showed no infection.
-Rest of pregnancy should be managed in high risk pregnancy clinic
-warning signs (running a fever, tummy pain, further leakage, baby not kicking well she should
inform immediately the ED.
-more frequent ANC and scan

**Cervical suture
36 weeks pregnant, cervical suture in place, presenting with PROM.
*usually remove cervical suture at 37-38 weeks

But in this case, in the GP do all bloods and swabs, don't give antibiotics. In the hospital consider
removal of cervical suture (risk for uterine rupture if cervix remains tight), and sent for
microscopic culture and sensitivity. If labor does not set in, induce labor by 34 week if no
infection. Once the suture is taken out and sent for microscopic C&S, give her antibiotics.

Feedback 21-7-2018
Preterm PROM - PASS (G.S – 5)
Stem: 30yr old lady, 30wk primi, in rural hospital 200km from tertiary centre. Presenting with
vaginal fluid loss
Tasks: -History

-PEFE

-DDx

-Mx

-Primi with spontaneous vaginal fluid loss since the past hour or so, had a significant loss,
ongoing, appreciating fetal movement, no fever, not in labour. ANCs all ok, singleton pregnancy,
no fibroids etc.

- Asked all PE findings in a proper sequence, including vulval inspection, speculum exam (os
closed, clear fluid noted to be coming from cervix) Specifically said I don’t want to do bimanual
exam. Should have asked for fetal fibronectin, vaginal swab at that point (I mentioned vaginal
swab in investigations later)- maybe the reason for score of 4in choice of investigation

-Preterm PROM….could be coz of various reasons…many times no reason can be found. But
what we’ll do now is right now do a CTG, USG. Will send you to tertiary hosp by ambulance.
Specialist will see you. Will do blood tests, vaginal swab, give Abx, Steroid, monitor you
continuously. Sometimes ppl go into preterm labour, tocolytics will be given if that happens.
Asked for Blood gp, family support (as for ALL Obs cases)
14-Vulvovaginitis
GP, 3 yr old girl brought by concerned mother due to rash in the vulva. She is divorced and thinks
that her ex-husband abuses the child.
Task
-history
-ask PE / PEFE card
-Diagnosis and management to mom.

Differential diagnosis
1-vulvovaginitis
2-UTI
3-child abuse
4-foregin body
5-thread worms

History
1-Approach + Confidentiality + open-ended questions
-as you enter she start saying (could u believe he has been doing this to my child etc..)
-I can see that you are concerned about sexual abuse of your daughter. Show empathy, and
Ensure confidentiality then ask can you tell more about your concern?
(mother is very much concerned ,about ,abuse by her ex-husband ,and girl’s father because she
noticed rash only after the girl went to visit her father, and in the following days, the girl refuse
to go to her father’s place and tried to stay by mothers side most of the time).
-I can see, I would just like to ask you further questions is that ok?

2-has she had this episode previously?

3-Rash questions
-since when has she had the rash?
-has it appeared suddenly or gradually. Is it getting worse?
-is there rash anywhere else?
-is it itchy, is it painful?

4-associated symptoms (tummy pain/ vaginal discharge/ UTI)


-any tummy pain?

-have you noticed any discharge?


How much?
What is the colour? Is it blood stained?
Is it smelly?

-how is her urine output? Is she passing urine more frequently? Any change in the colour of
urine? Is it smelly? Any pain or discomfort on passing urine?
5-Risk factors
Trauma/ foreign body/ worms
-has she had any trauma?
-could there be any foreign body in there?
-have you noticed any thread worms from back passage?
Moisture
-is she wearing tight clothes, tight jeans?
-does she go for swimming?
Irritants
-any use of soaps, bubble bath, antiseptics?
-any change in her cosmetics?

Abuse
-how is the situation at home? (Parents separated from each other 6 months ago)
-Anytime you left her unsupervised?
-How is the relationship between you and your partner? And how is her relationship with her
father?
-Any change in her behaviour recently? Any nightmares? (Child was not exhibiting any unusual
behavior but being more clingy and saying that she doesn’t want to go)
-Does she go to childcare? (She goes to kindy so must be playing in sand)

Physical examination from examiner


1-General appearance
dehydration, rash or bruising, signs of trauma, pallor, jaundice, LAP

2-Vital signs especially temperature

3-quick systemic examination

4-abdomen exam (inspection and palpation)

5-Pelvic inspection only (consent of mom)


-redness, rash, scratch marks
-signs of trauma, bruises, finger marks
-signs of penetration
-visible discharge

6-office tests
-urine dipstick
-blood sugar level
Explain and management
-from history and examination your child most likely have a condition called vulvovaginitis. Have
you heard about it?
- it is an inflammation or infection affecting vulva and vagina
-it is common condition presents with redness and itchiness in this area.
-basically, the skin over there is thin because of lack of hormone estrogen in young children. This
makes it more prone to infection by organism from back passage or inside vagina.
-there are several triggers for this condition. Wearing tight clothes, bubble bath, soaps,
swimming.. these can cause the normal amount of yeast and bacteria inside vagina to get out of
balance.
-it can be UTI or foreign body or worms but less likely.
-because you are concerned, our first priority is to protect your baby and you. That is why I want
to involve child protection authority, they will come and interview your child, and they have their
own way to do that. They might examine your child and take swabs.
Your child will be admitted for protection, further Ix and treatment of the rash.
-do not worry our aim is to address your and your child’s concern. You will be having our support
all the way through.

**why don’t you report the police


-if they found anything suspicious the police might also be involved but at this stage CPA is the
best department responsible for child protection

**If case only vulvovaginitis


1-wear cotton clothes, not to leave moisture to avoid organism growing that area.
2-you can use cream (zinc oxide, caster oil) to relieve the pain. you can also use vinegar in warm
water for the child to sit in.
3-if by any chance she develops fever, lethargy, bloody smelly discharge you nned to bring her to
the hospital.
4-i will follow her up in few days to see the progress.

Case (3/3/2017)
Vulvovaginitis -3 yr old girl brought by concerned mother due to rash in the vulva. same in Karin
She is divorced and thinks that her ex husband abuses the child.
Task:
History, PEFE card, Dx and mx to mom
No discharges no fever no dysuria no behavior changes like nightmares etc just said she's very clingy
to the mother
Rash in vulva and vagina no bleeding or oozing.

Case (8/6/2017)
4 old girl rash on the vulva parents separated from each other 6 months ago. The mother is very
suspicious of sexual abuse and ask you to do something so that her partner won't be able to see her
daughter again.
Case (7/3/2017)
GP, 3 yr old girl, mom noticed that rash around her vulva, she thinks that the baby was sexually
abused by her ex-husband. Now,she refuse to go to her father.
Task–history, ask PE, Dx and Mx.
In history, mom said there is rash in down below, no discharge, no urinary symptom, no change in
behavior, active as usual but refuse to see her father and think he abused her.
On PE, all normal apart from rash, no bruises, no sign of penetration, all bedside test not done.
I said vulvovaginitis and its treatment.
Reassure mom that finding are not suggested of sexual abuse. [ I don’t inform CPA and failed this
case

Case (7/9/2017)
Vulvovagitis child - separated parents - Mom says child developed when stays with dad and child
does not want to go there anymore.
Ask PE form examiner. general, playing, No bruising, VAgina inspection - mild rash, no finger marks .
Told mom consider other things vulvovaginitis.
UTI - check then and FB insertion.
Refer to child protection services.

Feedback 15-8-2018
Scenario: Health Concerns
Stem: Mother of a 4 year old girl is here to see you with child having rash in genital area and mum is
suspecting abuse ( famou recall again)
Tasks
~History
~pefe
~dx and ddx.
Inside was a mum sitting she was already enraged and as soon as I sat she started telling me” could u
believe he has been doing this to my child etc..i calmed her down and reassured her. Ensured
confidentiality, introduced mtself and asked her whats going on. She gave the typical history that child has
a rash down there and she is sure that her ex husband is abusing her. Took history keeping in mind( vv, uti,
dermatitis, forign body, pinworms) child was not exhibiting any unusual behavior but being more clingy
and saying that she doesn’t want to go. Asked about swimming, sand playing and tight cloths, she said
yeah she goes to kindy so must be playing in sand etc. Binds ( good social hisoty) found nothing alarming.
Pefe
Positive was a rash in the vaginal area extending to the anus.
Found no signs of abuse
I started with keeping her concern first then told child with abuse exhibit some behavior which I am
unable to fine here. Told about vv and explained, told briefly about other ddx. But she was not convinced (
I was hoping that when I talk about cps she might step back ) but she didn’t. I said since u r concenrned
that makes my concerned too and in that case I m obliged to involve cps and explained their job. She
said yes, I want it I want it.. I need to teach him a lesson etc. so I calmed her reassured her. Said you have
all the support and you are a great mother etc.. she leaned back on the chair relaxed( very good actor) :P
Grade :Pass

Feedback 7-3-2018
Vulvovaginitis -3 yr old girl brought by concerned mother due to rash in the vulva. She is divorced and
thinks that her ex husband abuses the child.
Task:
History
PEFE card
Dx and mx to mom

No discharges no fever no dysuria no behavior changes like nightmares etc just said she's very clingy to the
mother
Rash in vulva and vagina no bleeding or oozing.

Another 15-8-2018 Feedback pass


Next p,tient in GP is 4 yr old girl brought in by mother bec,use she noticed r,sh in girlʼs priv,te p,rt.
T,sk:history
Ask PE finding
Expl,in dx ,nd ddx
Mx pl,n

-mother is very much concerned ,bout ,buse by her exhusb,nd(girlʼs f,ther) bec,use she noticed r,sh only
,fter the girl went to visit her f,ther ,nd in the following d,ys,the girl refuse to go to her f,therʼs pl,ce ,nd
tried to st,y by motherʼs side most of the time

Acknowledged her concern ,nd ,n,lyze r,sh symptom,ruled out sexu,l ,buse,injury,pinworm,UTI,n, ppy
r,sh,etc

PE-ruled out sexu,l ,buse

Expl,ined vulvov,ginitis ,s ,ll points f,vour it ,ccording to history ,nd PE

Re,ssured th,t it is not sexu,l ,buse ,nd expl,ined other ddx

Mother is still not convinced ,fter expl,ining two times ,nd ,sk for my opinion,so I s,id iʼll help her inform
Child protection ,uthority,she s,id wh,t will they do ,nd I expl,ined wh,t CPA might do

15-Diabetic Ketoacidosis
17 year old boy complaining of feeling very unwell and cannot do farm work. You are in rural
hospital ED.
Tasks
-take history
-physical examination findings examiner
-Investigation to examiner
-Tell your Provisional dx and Ddx with reasons.

History
1-approach
-Check hemodynamic stability.
-I can see that you are feeling unwell can you tell me more about it? (Feeling very tired for few
weeks)
-do you feel tired the whole day or specific time? (He feel tired the whole day)
-Do you feel tired the whole body or specific part? (No specific part)
- Constant or intermittent (constant)
2-Tiredness differentials ½ (HEMI)
Thyroid
-Any weather preference, Any change in bowel habits (no)
Diabetes (polyuria, polydipsia, nocturia, weight loss)
-do you feel thirsty? (Yes)
-are you passing large amount of urine? (Yes)
-do you need to go to toilet more frequently? Do you need to wake up at night more frequently
to pass urine? (Yes)
-have you lost weight recently? (Yes few kg over some period of time).
Malignancy/ infection
-any loss of appetite, any Lumps and bumps? (No)
-any fever? Rash? Any recent viral infection? (No)
3-Complication of DM
-any nausea or vomiting, tummy pain?
-any chest pain, funny racing of the heart?
-any confusion, drowsiness?
-any tingling, numbness or weakness?
-any blurring of vision? (Yes a bit)
4-General questions
-Any past history of DM or any medical illness (no)
-family history of DM?
-are you taking any medication or OTC? (No)
-Past surgical history
-SADMA, diet
-stress at home, work?

PEFE
-GA (DR PJL) + BMI, any rapid deep breathing (no dehydration, pallor, jaundice, LAP, rash)
-VS (normal)
-ENT (normal)
-chest and heart
-abdomen (clear)
-office test: urine dipstick (ketone +ve, glucose +ve, no blood or leukocyte)
BSL (high)
ECG not available

Investigation to examiner
-FBE, ESR/CRP
-RFT and electrolyte, LFT
-venous blood gases
-urine microscopy and culture

Diagnosis and Treatment


-from history and examination, you most likely have a condition we call diabetic ketoacidosis. It
usually occurs in people with type 1 DM. This means that there is a high sugar level in your
blood, but there is no enough insulin hormone. Insulin usually lets sugar get into the body’s cells
so if there is no enough insulin, the body cannot use glucose for energy and starts to use fat
instead. And the glucose level becomes high.

-this usually happens when the body’s demand for insulin suddenly increase may be due to
stress or recent infection or illnesses.

-so when the body uses fat for energy, chemicals called ketones are released into the blood
causing DKA.

-as the level of ketone increases, it can lead to dehydration, confusion and if not treated the
patient can become unconscious.

-this is medical emergency and require immediate treatment. As we are in rural hospital, I would
like to refer you to a tertiary hospital for further assessment and treatment. In the meantime I
will start you on IV fluid and insulin to reduce sugar level and I will call the registrar as well. Let
me assure that you that you are in safe hands and you will get the best possible care.

Case (8/3/2017)
young teen tired.. Glucose 34
-Ques 4 was a 17 years old girl... unwell with her parents at a farm. Parents brought her.
Tired and drinking a lot and Hungry as well. Wee also a lot, Simple – DKA, -BSL 34, -Ketones in
urine
-Refer to tertiary centre

Case (1/9/2017) (6/9/2017)


DKA
17 years old boy brought in by one of the parents beacuse he has been feeling unwell. He
become so ill that he cannot do farm work.
Task:
history 4 min
Ask physical examination specifically to the examiner
Tell your Provisional dx with reasons

Recall 2-3-2018
Dm 17yo complains of unwell. not able to work in farm. Thirsty, freq urinating. Hx pefe, ddx

16-Transverse Lie
35 year old Jenny is your next patient at your GP clinic. She is at 37 weeks pregnant and has
come to you for antenatal checks. This is her fourth pregnancy.
TASKS
Take a further history
PE from examiner,
explain diagnosis, Discuss further management with the patient

History
outside the room you might suspect that this case is anemia due to spacing but you should
suspect any late pregnancy complications and all the history will be normal
1-Late pregnancy complications questions
-How’s your pregnancy so far?
-any tummy pain, vaginal bleeding or discharge? (Preterm labour. Placenta Previa or abruption,
PROM)
-any headache, blurring of vision or leg swelling? (Preeclampsia)
-any fever, nausea or vomiting? (Infection)
-any burning or stinging on passing urine? (Urine problem)
-Nay problem with your bowels? (bowels problem)
-any dizziness, tiredness or funny racing of the heart? (Anaemia)
-is the baby kicking well? (Baby problem)

2-recurrent visits questions


-Have you had regular antenatal checks?
-How were the blood tests? Are you aware of your blood group?
-have you done down syndrome screening?
-US at 18-20 weeks gestation? Is it single baby? Any birth defects? What is the position of the
placenta?
-Sweet drink test at 28 weeks?
-Repeat ultrasound at 32/34 weeks?
-Bug test at 36 weeks?
-did you take your folic acid?
- Have you done your pap smear? What were the results?

3-past obstetric history questions


-How were your previous pregnancies and deliveries? Any complications in during pregnancy,
delivery or after delivery?
-When was your last pregnancy? (can predispose to anemia if no proper spacing)

4-General questions:
-Are you eating a healthy balanced diet? Do you drink a lot of fluids?
-Do you have a regular exercise routine?
-Are your influenza and pertussis vaccinations up to date?
-do you smoke? Do you drink alcohol? Do you take any medications or any recreational drugs?
-Do you have good support?
- have you had any medical illnesses? (clotting, DM, thyroid, hypertension)
Physical Exam from examiner
1-General appearance (PODL)
pallor, oedema, dehydration, LAP

2-Vital signs (all)

3-quick CVS and chest

4- Abdomen:
-any tenderness
-any uterine contractions
-what is the fundal height (36 cm)
- FHR,
- lie and presentation (hard ballotable mass on one iliac fossa and soft mass towards the other)
-is the lower pole of the uterus empty or not?

5-Pelvic:
-Inspection of the vulva and vagina: any bleed, discharge, rash, vesicles
-Speculum: cervical os open or closed, discharge or bleed from the os

6-Office test:
urine dipstick, blood sugar level

Explanation (4C)
Condition
-Most likely your baby is in a transverse lie. Normally, the baby lies in a longitudinal position
which means that the baby is parallel to your spine. But in transverse lie, the baby lies
perpendicular to your spine.
*Draw a picture of a longitudinal and transverse lie.

Clinical features
I can say this because on tummy examination I can feel the head on one side and the butt on the
other side

Cause
-The cause of this that I suspect in you is multiparity. As this is your fourth pregnancy, the uterus
could be lax and this allows the baby to adopt abnormal position and lies.
-The other causes can be polyhydramnios, or excessive fluid in the sac surrounding the baby, but
that is not a probability here as the uterine size is less.
-Another causes are:
placenta Previa; when the placenta attaches itself to the lower pole of the uterus,
Fibroids or benign overgrowth in the uterus
birth defects in the baby,
but these could have been detected in the ultrasound that have been done earlier.
Complication
The problem with a transverse lie position of the baby is, when you go into labor, the labor might
now progress, and you will end up having an obstructed labor, and there is a chance of cord
prolapse as well.
The cord usually comes out after the delivery of the baby, but in cord prolapse, the cord comes
out first and the cord can get compressed between the baby's head and the birth canal leading
to decreased oxygen and nutrient delivery to the baby. The baby then becomes distressed and
unwell.

Management
-I need to refer you to a tertiary hospital, where you will be admitted and seen by the
specialist. -An ultrasound will be done to rule out the other causes of transverse lie, and a CTG
will also be done to look for the baby's wellbeing.
-There are two options for you as far as delivery goes: the first option is an elective C-section by
37/38 weeks, and second option is to do an external cephalic version.
ECV will be done by the specialist after ruling out contraindications like fetal distress or unwell
baby, placenta previa, a cephalopelvic disproportion, or a short cord or oligohydramnios. In ECV,
the baby will be manipulated over the tummy by using the hands and finally the head of the
baby will be brought down so that the baby will have a longitudinal lie and a cephalic
presentation or head down.
Labor will then be induced by artificial rupture of the membranes.
There are complications associated with ECV such as fetal distress, abruptio placentae or
placenta separates from the wall of the uterus, rupture of membranes, and the most important
complication is the cord can get twisted around the neck of the baby. It is usually done around
37/38 weeks. After 38 weeks, it is not done anymore.

Do you have somebody to take you to the hospital now?

I will give you reading materials regarding transverse lie.


Feedback 10-5-2018
please tell us in case one what was the presenting complaint? how did you suspect it was a
transverse lie? in pe did examiner ask you how will u tell it is a transverse lie?
So the question for transverse lie case was she was coming for regular antenatal check for her
fourth pregnancy. She has been regular with her ANC and her blood test were normal. 18wks
scan showed single foetus with normal liquor and placenta in the posterior.
Task: take history
PE from examiner
Explain ur finding to the patient and the reasons for it.

She had no complains at all. All history was normal. When I asked PE from examiner, on asking
Lie and Presentation the examiner asked me how would you check for lie and presentation so i
explained it to the examiner and he replied there is no presenting part. Then once again i asked
and he smiled and said the same.
Then I went on to explain it to the patient. But i did not mention transverse lie coz it just dint
come to my mind. I went abt explaining things abt mobile head at term, so i explained all the
possible causes for it and i told her not to worry as everything in history taking and PE was
otherwise normal. So all the causes for mobile head and transverse lie are same so i guess i
passed this case
I showed him practically how I would check for it by doing the grips ( in air). Fixing one side and
checking the other side and then vice versa. Looking for spine on one side and looking for baby
limbs on the other...
And I kept giving assurance to patient that everything seems fine on history and PE so I don’t
want u to worry. Let us just get a scan and we will know if there is any cause present.
there was no management.
Feedback 5-12-2018
GP, Lady, 36 weeks, 4 th pregnancy long stem all antenatal test USG, Sugar test 18
normal, all previous checkup normal, BP normal, all details given.
Task:
1. Ask History for 4 min(was thinking what else to ask as everything in the stem)

2. Ask PEFE from examiner and he will give you specific findings you want

3. Tell mom possible causes of your findings

Greetings, Introduction. Appreciated that she was regular with her ANC and most of her tests
were normal. Just to be confirmed I would ask few more q , is that Ok X?Do you have any specific
concern before I start?
Started HX : ( kept in mind risk factors of transverse lie: poly hydrammions, structural defect,
Routine Q:
PETQ? Bleeding, discharge baby kick. Any infection or fever?
ANC HX: (most of them given in stem, just asked as gross)
I know all the Blood tests and sugar tests were normal, how about Down screening?, 18 wk USG
– single / multiple, position of placenta? Repeat usg at 34-(can’t remember whether she said—
not done or not sure about the result), Bug test- I didn’t ask (probably it’s the 3 rd key point that I
missed).
OBS HX: Any complications, mode of delivery (all were NVD)
MED/SX HX: I asked specifically about any tumor / fibroid or any previous sx in the womb
Asked about folic acid?
(Did not ask about all SADMA)
PEFE:
General appearance; ( I did not ask
Head to toe General PE: Pallor, icterus, oedema
Vitals: started asking BP.. Examiner said vitals normal
Focused abdominal examination: FH : 34, when I ask about lie and presentation: how will you
examine: I said rt and left lateral grip. Examiner asked what you looking for. I said: babys back
abd limbs.
Presentation: I will do pelvic grip placing my both hand in lower pole and I will look for head or
breech.
Then examiner gave me finding: you are feeling: hard ballot able structure in left. Lower pole is
empty.
FHR: normal
Pelvic examination: Examiner said all normal.
Pt counselling:
Hi X, I have examined you good thing is you baby is doing fine but what I found is your baby is
lying transversely in your womb which is not normal. But don’t worry it quite manageable
condition. Let me draw and explain it to you.
Then I explained it and all the possible causes..
Poly hyd: where there is excess fluid in your womb.
Placenta prev: where structure attaching you baby to your womb lying in lowrer part.
Structural abnormality or tumor like fibroid in your womb, or any structural defect in your baby.
But all these are less likely as all the usg and investigations are normal
in your case most likely its lax uterus due to multiparity. Baby gets enough rooms to freely move
and take abnormal position.
I am gonna refer you to Specialist and MDT in TLH. Sp will decide on further mx . There is 2
options they may try to reposition which we call ECV or they may suggest for C/s.
Key step 1234: yes, yes no yes
Approach to pt : 6
History: 5
Accuracy of exam: 5
Pt counselling: 6
17-Orbital cellulitis
You are a GP in rural area. A 6 year old boy is brought in by his mum with a swollen left eye for
24 hours. She deduced it was a mosquito bite similar to the one his sister had last year, so she
tried cetirizine. However, the swelling was not relieved, so she comes to seek you help.
Tasks
-History up to 3 min.
-PE from examiner.
-explain diagnosis
-Management to patient.

History
1-Eye swelling questions
-Since when?
-has it appeared suddenly or gradually?
- Is it getting worse?
-is it on one or both eyes?
-Is there a swelling anywhere else?
-is this the first time?

2-Differential diagnosis questions


Conjunctivitis
-any redness, itchiness, or sticky discharge?
- Is it watery eye?
Cellulitis (in general)
-any fever?
- Is it painful? is it hot or pain to touch?
Orbital cellulitis
-any double vision or blurring of vision?
-any pain on eye movements?
-Any restriction of eye movements?
Trauma/ allergy/ insect bite
-any trauma to the eye by any chance?
-any allergies?
-any insect bite?
3-Complications questions (rule out brain abscess and meningitis)
-any nausea or vomiting?
-any headaches?

4-Associated symptoms (ENT)


-any recent viral infection
-any sore throat, ear pain or runny nose

5-BINDS
-is he immunized up to date?
-how is his feeding? Is he eating well?
-is he thriving and growing well?

6-General
-medication
-PMH, PSH
-family history

Physical examination findings from the examiner


1-general appearance (DR PJL)
dehydration, rash, pallor, jaundice, LAP + oedema

2-Vital signs especially fever

3-growth chart

4-neck stiffness

5-ENT

6-eye examination
-ptosis, proptosis, size and shape of the pupils
-visual acuity, visual field, eye movements, light reflex and fundoscopy
-any signs of trauma

7-other systemic examination


Explain the diagnosis
Condition
-There are several causes why your child has eye swelling and … tell positive symptoms form
history. It can be due to insect bite, allergies or trauma but unlikely from the history and
examination.
-I am suspecting a condition we call orbital cellulitis; our eyes is present in a closed bony socket
called the orbit. In your child’s condition, there is an infection going around and behind the eye.
It most commonly refers to an acute spread of infection into the eye socket from either the
adjacent sinuses or through the blood. It may also occur after trauma

Note/ periorbital cellulitis does not cause painful eye movements or restriction of eye
movements or affect the vision.

Cause
it is usually caused by a bug a bacterial one most likely staph.

Complications
this is a surgical emergency condition and it is good that you brought him here so that we can
manage it appropriately. If left untreated the infection can spread to the brain causing puss
collection or abscess or meningitis can spread to some air filled cavities in the bone called
sinuses, it can also affect the vision lead to blindness.

Management
-refer to tertiary hospital (if rural GP) or admit if in emergency.
-need to be seen by eye, ENT, pediatric specialist’s team.
-in the meantime I am gonna put IV line, take blood tests for Ix like (FBC, ESR/CRP/ UCE, blood
culture, urine MCS)
-specialist might decide to do CT scan head in the hospital
-I will arrange painkillers and I will start him on broad spectrum antibiotics IV (flucloxacillin+
ceftriaxone)

Recall of 16-3-2018
Recall of 5-9-2018 orbital cellulitis pic provided outside take Hx PEFE explain
the condition Mx
18-Facial nerve palsy
Case/ young man woke up in the morning with the asymmetrical face. He is worried about
having stroke.
Tasks
-physical examination of facial nerve
-other relevant examinations to rule out the causes of his symptoms
-tell the patient the most likely Dx and DDx with reasons.

The examiner will give you pictures to comment on them

1-Inspection
-face: asymmetry, nasolabial fold, drooping of the corner of mouth)
-eyes: corneal ulcer, conjunctivitis
-ears: rash or vesicles
-parotid gland: visible lump or welling

Note/ in the exam, there might be widening of palpebral fissures, smoothing of nasolabial fold,
drooping of left or right sided corner of mouth.

2-facial nerve examination


-taste and hyperacusis
do you have any change in your taste or hearing?
-wrinkle forehead
can you look up as to wrinkle your forehead and do not let me push them down?
-close eyes tightly
can you close your eyes tightly and do not let me open them?
-puff out cheek
can you puff out your cheeks and do not let me push them together?
-show me your teeth
can you show me your teeth please?
Note/ in the exam, the examiner will give you the findings on pictures and all lower and upper
side of the face will be affected to know there is lower motor neuron palsy. For example: loss of
wrinkles, failure of eye closure and rolling up of eye under the upper eyelid, failure to blow out
cheeks, and deviated mouth to one side.

3-Corneal reflex

4-assess hearing with otoscopy and tuning fork. Examiner will say normal

5-feel the parotid gland: lump and tenderness. Examiner will say normal

6-feel for lymph nodes.

7-complete exam with other cranial nerve examination. Examiner will say normal
Diagnosis
from history and examination you most likely have facial nerve palsy due to bell’s palsy which is
weakness of one of the nerves supplying your face and head. I can see that it must be so scary
for you but Let me assure you that it is a common condition and not serious. The cause is usually
unknown but may be due to inflammatory compression on the nerve due to a virus infection. In
Most cases, this condition will usually resolve spontaneously without any complications.

Differential diagnosis
1-stroke/ cerebral or brain tumor (unlikely because the upper side of the face is also weak and
involved)
2-cerebellopontine angle tumor (unlikely because no hearing problems on examination)
3-Ramsy hunt syndrome (unlikely because of no rash or vesicles )
4-parotid tumor (no lump, swelling or LN on parotid gland and LN exam)

Treatment (not a task but just in case) from handbook


1-steroid (prednisolone) for 3 days then taper off over the next 7 days.
2-antiviral drug
3-patch over eye at night
4-artificial tear
5-if concerned CT scan to provide reassurance
6-referral to neurologist for confirmation of diagnosis and confirmation of management
7-review in few days for support and monitoring
8-follow up + red flag (conjunctivitis, corneal injury)

Feedback Case (13/10/2017)


The young man woke up in the morning with the asymmetrical face. He is worried about having
stroke.
Tasks:
Px of facial nerve and relevant examination to rule out the cause of his symptoms.
Tell the patient the most likely dx and ddx with reasons.
I started with introduction, Washing hand and the consent.
Inspection:
I said(pointing at the picture) I can see naso-labial fold is smoothed and palpebral fissure is
narrowed and asymmetrical no ulcer Keratitis no conjunctivitis.
Then I went for facial nerve. And the examiner showed the pictures then asked what is this? I
said in this way the patient could not keep blowing from pursed lips instead so it is deviated to
the other side.
I did inspection of ear no rash no vesicle no erythema then otocsope the examiner said it is NL.
And I said there is no tuning fork and… examiner said weber and rhinne is NL.
Then I said I want to check for the 9th nerve open mouth say ah examiner said it is NL as well.
Thence I said I want to check 10th nerve and 11 said NL.I said OK I want to check the parotid
gland. There is no lump no tenderness in palpation.
Later I wanna check inside mouth with gloves examiner said NL.
I said I wanna check LAP later.
Look John what you are having is called Bell's palsy. What's that??? I can't imagine how it is
concerning for you but let me tell you it is not serious although it looks scary. It is due facial
nerve damage. Due to many theories, one of them is a viral infection but mostly the cause is
unknown and 70% of pt will be OK after 2 months so please do not worry.
Other conditions can be brain tumor which is unlikely due to my examination.
It can be due to parotid gland that you have but again less likely and stroke is another one. Bell
rang( I forgot to talk about Ramsy-haunt syndrome)
Feedback: Altered appearance, PASS(G.S:6) Key steps:1,2,3 and 4 Yes
Approach to patient:6 Familiarity with test equipment:6
Choice and technique of examination and organization and sequence:6,
Accuracy of examination:6 Dx/DDx:6

Feedback Case (25/10/2017)


Bell's palsy-patient noticed change in his face when having breakfast. No change when he went
to bed last night. Able to swallow.
Don't take history. Examination of relevant cranial nerves and dd( no mgmt).
You vl be given pictures when u give instructions for examination.
There is a mannequin for examination when needed

Feedback Case (2/3/2017)


Over night swelling of face in young male. Noted while washing face.
1) Perform Ex - RS LMN facial n palsy(photos given), I looked for cause and patient was short of
hearing on same side ear and there were 2 tuning forks (both were 128hz ).
when asked for 512 or 256 Hz I was asked to proceed next examination. I told I need to do
otoscopy ex, and was shown a manikin to examine ear (in the manikin only left ear was present-
half face manikin - erythematous angry looking tympanic membrane suggestive of Otitis media.
But facial N lesion in the right side.

Then I told I need to examine the other ear to confirm whether this is BL Otitis media as facial N
palsy is on the right side. Examiner said yes. Give DD to patient with reasons (otitis media
complicated with LMN Facial N palsy.)
Reading notes
Facial nerve examination talley o connor
History
1- noticed the onset of difficulty with speaking and
2-difficulty keeping liquids in the mouth
3- may have noticed facial asymmetry in the mirror.
4-He or she may be aware of dryness of the eyes (decreased lacrimation) or the mouth
(decreased salivary production).
5-Paralysis of the stapedius muscle can cause hyperacusis or intolerance of loud or highpitched
sounds. Normal contraction of the stapedius muscle occurs in response to loud rebarbative
noises such as popular music and dampens movement of the ossicles.

Examination
Inspect for
1- facial asymmetry
2- unilateral drooping of the corner of the mouth.
3- Smoothing of the wrinkled forehead.
4- Smoothing of the nasolabial fold.

Test the muscle power.


1- Ask the patient to look up so as to wrinkle the forehead, Look for loss of wrinkling and feel
the muscle strength by pushing down against the corrugation on each side. This movement is
relatively preserved on the side of an upper motor neurone lesion (a lesion that occurs above
the level of the brainstem nucleus) because of bilateral cortical representation of these muscles.
The remaining muscles of facial expression are usually affected on the side of an upper motor
neurone lesion, although occasionally the orbicularis oculi muscles are preserved.

2- Ask the patient to puff out the cheeks, Look for asymmetry. In a lower motor neurone lesion
(at the level of the nucleus or nerve root), all muscles of facial expression are affected on the
side of the lesion.

3- Next, ask the patient to shut the eyes tightly. Compare how deeply the eyelashes are buried
on the two sides and then try to force open each eye. Check whether Bell’st phenomenon is
evident. Bell’s phenomenon is present in everyone, although not usually visible unless a person
has a VII nerve palsy. In this case, when the patient attempts to shut the eye on the side of a
lower motor neurone VII nerve palsy, there is upward movement of the eyeball and incomplete
closure of the eyelid.

4- Next ask the patient to grin. And compare the nasolabial grooves, which are smooth on the
weak side.

5- If a lower motor neurone lesion is detected, check quickly for the ear and palatal vesicles of
herpes

A facial paralysis due to a cortical lesion may spare facial movements due to emotion such as
crying or smiling, and indeed these movements may be exaggerated.
The opposite abnormality (preservation of voluntary but loss of emotional movements) can also
occur as a result of lesions in a number of areas, including the frontal lobes.

6-Examining for taste on the anterior two-thirds of the tongue is not usually required. If
necessary, it can be tested by asking the patient to protrude the tongue to one side: sugar,
vinegar, salt and quinine (sweet, sour, saline and bitter) are placed one at a time on each side of
the tongue. The patient indicates the taste by pointing to a card with the various tastes listed on
it. The mouth is rinsed with water between each sample.

Causes of a seventh (facial) nerve palsy

1-Vascular lesions or tumours are the common causes of upper motor neurone lesions
(supranuclear).
Note that lesions of the frontal lobes may cause weakness of the emotional movements of the
face alone; voluntary movements are preserved.

2- In lower motor neurone lesions


-pontine causes (often associated with V and VI lesions) include vascular lesions, tumours,
syringobulbia and multiple sclerosis.
-Posterior fossa lesions include an acoustic neuroma, a meningioma or chronic meningitis.
-At the level of the petrous temporal bone (Bell’s palsy [an idiopathic acute paralysis of the
nerve; a fracture, the Ramsay Hunt syndrome or otitis media may occur,
while the parotid gland may be affected by a tumour or sarcoidosis. Remember, Bell’s palsy is the
most common cause (up to 80%) of a facial nerve palsy.

Regrowth of the nerve fibres that occurs as the patient recovers from Bell’s palsy can lead to
aberrant connections. The most striking is the regrowth of fibres meant for the salivary gland to
the lacrimal gland in up to 5% of patients. This leads to tear formation when a patient eats :
crocodile tears.

Bilateral facial weakness may be due to the Guillain-Barré syndrome, sarcoidosis, bilateral
parotid disease, Lyme disease or, rarely, mononeuritis multiplex.
Myopathy and myasthenia gravis can also cause bilateral facial weakness, but in these cases it is
not due to facial nerve involvement.

Unilateral loss of taste, without other abnormalities, can occur with middle-ear lesions involving
the chorda tympani or lingual nerve, but these are very rare.

Irritative changes Tonic and clonic movements of the facial muscles can occur in seizures. Various
abnormal movements of the facial muscles can occur as a result of basal ganglia or
extrapyramidal abnormalities. These include athetoid and dystonic movements. Irritative lesions
in the brainstem can cause increased secretion of saliva (sialorrhoea). This can also occur in
Parkinson’s disease or accompany attacks of nausea.
Handbook 94
Acute idiopathic facial nerve palsy ('Bell Palsy') in a 40-year-old man

You are consulting in a general practice. You have just completed taking a history from and
examining a 40-year-old man who is very upset because of the sudden onset of paralysis of his
face.

He felt discomfort behind the left ear last night and on waking today found that the left side of
his face would not move. He arranged an urgent appointment with you.
Examination confirms a near complete left 7th cranial nerve facial palsy of lower motor neurone
type.
The accompanying illustrations show the findings. There are no other abnormal neurological or
other signs including normal ear canals and tympanic membranes. The patients parotid salivary
glands show no abnormality. You have made a confident clinical diagnosis of acute idiopathic
facial nerve palsy (Bell Palsy')

YOUR TASKS ARE TO:

• Explain the problem to the patient.

• Advise the patient of the management you would advise.

• Respond to any questions asked by the patient.


The patient is very upset and concerned that this may be a stroke, and wishes to know the
cause, whether recovery will occur, what treatment and tests he should have and how long it will
take to recover.

Time should not be wasted taking further history or asking for any other physical findings. The
main issues to be addressed are patient counselling and management.

AIMS OF STATION

To assess the candidate's knowledge of Bell Palsy, its prognosis and its management, and skills in
counselling an upset and anxious patient.

EXAMINER INSTRUCTIONS

The patient has the condition as illustrated. He is very concerned, but if informed and reassured
appropriately, will accept the diagnosis and management plans.

EXPECTATIONS OF CANDIDATE PERFORMANCE


You would expect the candidate to:
(Task 1)

• Acknowledge the patient's distress about his appearance and to provide support and guarded
reassurance, particularly reassurance that the patient has not had a stroke.

• Explain the diagnosis and natural history of the condition. The cause is unknown but is
consistent with inflammatory compression of the facial nerve in the temporal bone (probably
viral).
Expected course — about 70% of patients completely recover within two months. First signs of
recovery appear within two weeks. About 20-25% take up to six months for full recovery; and 5-
10% do not recover by the end of one year. An older patient age is associated with slower
recovery.

(Task 2)
• Advise about immediate management:

1- Steroids are usually prescribed empirically: prednisolone 40-80 mg daily for three days then
taper off and cease over the next seven days.
2- Antiviral drugs may also be given because of its presumed viral aetiology.
3- Wear patch over left eye at night.
4- May prescribe artificial tears.

5- Investigations are not essential but CT head would be appropriate for reassurance in view of
patient anxiety about a stroke.

6- Referral to a neurologist should be offered for confirmation of diagnosis and possibly for
nerve conduction studies. Referral is also appropriate for confirmation of management,
because of possibility of incomplete or nonrecovery.

7- Consider early referral to a physiotherapist as an aid to self-management strategies. There is


no evidence that exercises or nerve stimulation aid recovery, but they may support patient
confidence in recovery.
8- Review within a few days for support and monitoring.
9- Arrange continuing followup to monitor progress, watch for symptoms of conjunctivitis and
corneal injury.

KEY ISSUES

• Approach to patient — acknowledging distress about his appearance, providing support and
guarded reassurance about recovery.

• Initial management plan — protection of eye and possible use of steroids and antiviral agents.
Offer referral to neurologist and physiotherapist.

• Patient counselling about prognosis and natural history, stressing that complete recovery is
usual (although not invariable).

CRITICAL ERRORS

• Telling patient that complete recovery always occurs.

• Very unsatisfactory counselling skills displaying insensitivity in dealing with an anxious patient.

COMMENTARY
The most common cause of unilateral facial nerve palsy without a clear history of local injury is
the condition of idiopathic acute facial nerve palsy ('Bell Palsy).

Bell Palsy is of unknown aetiology and affects all ages and both sexes. Patients present with an
acute or subacute onset over a few hours. Pain around the ear is followed by unilateral facial
paralysis of lower motor neurone type, with complete or partial paralysis of muscles supplied
by the facial nerve. The clinical features are consistent with a lesion due to inflammatory
oedema and compression of the nerve within the bony canal of the petrous temporal bone.

Clinical features of the lesion, if complete and all muscles equally affected are as follows (see
illustrative figures):

• Facial asymmetry is accompanied by loss of voluntary, emotional and associated movements.

• The affected side of the face is immobile, the eyebrow drops, the lines on the forehead and
nasolabial fold are smoothed out.

• The palpebral fissure is wider due to the unopposed action of levator palpebrae.

• Tears fail to enter the lacrimal puncta medially because they are no longer held against the
conjunctivae and the eye weeps.

• The direct corneal reflex is absent, but the patient appreciates the discomfort from testing and
the indirect corneal reflex is present (the other eye blinks). Corneal abrasion and ulceration are
significant risks.

• Efforts to close the eyes cause the affected globe to roll up under the upper lid (Bell reflex).

• The a/a nasi does not flare or dilate with vigorous breathing.

• The lips stay in contact but cannot be pursed for whistling. When smiling, the angle of the
mouth on the affected side does not move; and in repose 'wry-mouth' can be identified.

• Hyperacusis in the affected ear can be troublesome when the patient is subjected to local
noise.

• During mastication food accumulates in the cheek and dribbling of saliva can occur from
between paralysed lips.

• The articulations of labial consonants (m, b, p) may be affected.


• Loss of taste sensation may be noted in the anterior tongue on the affected side.

• The cosmetic and psychological effects of the disfigurement can be profound.

• Patients with Bell Palsy are frequently concerned that they may have suffered a paralytic
stroke.

Differential Diagnoses — other causes of facial nerve palsy

• Patients with Bell Palsy are frequently concerned that they have had a stroke', or have a
cerebral tumour. Facial weakness due to 'stroke' is usually upper neurone in type and part of a
hemi-paresis on the same side as the facial paralysis. Movements of the upper muscles to
forehead and eyes (which are bilaterally innervated from the upper motor neurone) are spared,
as may be emotional movements. However, an infarct in the pons may produce a nuclear (lower
motor neurone) lesion of the facial nerve. While In Bell Palsy the motor lesion is confined to the
facial nerve alone and is lower motor neurone in type.
• If hearing loss or other cranial nerve lesions are associated with facial nerve palsy the
diagnosis is more likely to be a cerebellopontine angle tumour (for example, auditory
neurofibroma) or a vascular event from Vertebrobasilar insufficiency.
• If vesicles within the external ear or on the palate accompany the 7th nerve palsy the
condition is viral herpes zoster infection affecting the geniculate ganglion, not Bell Palsy. This is
the Ramsay Hunt syndrome, and occasionally other cranial nerves are also affected. Such
patients are often elderly. Pain may precede the facial palsy and the associated herpetic eruption
in the ear and sometimes on tongue or palate. Recovery of facial nerve function is rare. Prompt
treatment with aciclovir may improve prognosis and diminish post-herpetic neuralgia.

• If the nerve is affected within the parotid gland, this is usually due to a parotid malignancy
giving partial or total lower motor palsy. Benign parotid tumours do not cause facial palsy.

• Basal skull fractures of the petrous bone are another important cause of facial nerve palsy.

Pictures from handbook


Face in repose: note widening of palpebral fissure due to unopposed action of the levator of the
upper lid, smoothing of facial lines and failure of eversion of mucosa of patient's left lower lip
('wry-mouth') due to paralysis of depressor anguli oris.

Attempted eye closure: note failure of left eye closure with rolling up of the eye under the upper
eyelid, and accentuation of the 'wry-mouth' triangular deformity.

Obvious deformity when smiling: note immobile left eye and mouth musculature and absence of
nasolabial fold.

Attempting to blow out cheeks: note failure of left buccinator muscle with flaccid paralysis of
patient's left cheek. The patient is unable to prevent air from escaping from the mouth when
he tries to build up intraoral pressure.
19-Prolonged QT syndrome
A 15 y/o girl who has fainted in school is in ED. She has multiple faints in last 3months.
Task:
1- take further history from the patient
2- ask about the physical findings and investigations that you want from the examiner.
3- Explain for the patient about the possible diagnosis.

History
1-collapse questions
-Did you lose your consciousness completely? How long for? (1 minute)
-has this happened before? How often? (This is the 3rd / 4th attack)

2-Before
-what were you doing before it happened? (sitting/ singing at school)
CNS, Vasovagal
-did you get any warnings like feeling dizzy, nauseous or strange smell? (Migraine/ epilepsy/
vasovagal)
-did you get any headache, blurring of vision, weakness? (TIA/ Stroke)
-Any fever or recent infections (Meningitis)
Cardiac
-have you had funny racing of the heart? (Arrhythmia) if yes ask: how long did it last? Regular or
irregular? Do you have it now?
-any chest pain, SOB, dizziness?
-did these symptoms occur during heavy exercise? (Aortic stenosis)
Endocrine
-did you skip your meals? When was your last meal? (Hypoglycemia)
any head injury

3-During (epilepsy)
-anyone witnessed and noticed any jerky movements?
-any change in colour of lips or skin?
-have you passed urine or soiled yourself?
-have you bitten your tongue?
-have you injured yourself?

4-After
-how long does it last until your recovery?
-after recovery do you feel normal or drowsy? Any N&V or headache?

5-General
-PMH, PSH, SAD
-medications (imp)
-family history of sudden death? (imp)

PEFE (like any CVS exam)


1-General appearance
-pallor, cyanosis, dyspnea
2-V/S + postural hypotension
3-JVP, feel carotid bruit
4-Heart
-thrill, heave, apex beat
-heart sounds (s1, s2) + added sounds
-heart murmurs (if any ask site and radiation)
-listen carotid bruit
5-chest
-chest movement, air entry, breathing sounds, dullness, crackle or wheeze.
6-abdomen
7-CNS
8-sacral and pedal oedema
9-office tests
-UDT
-BSL
-ECG
Then ECG will be given; showing prolonged QT with slow heart rate.
Note/ in any ECG interpretation: check the labels comment on rate, rhythm, P wave, axis
deviation, then the specific abnormality.

Explanation
-From history and examination and ECG findings, most likely you have prolonged QT syndrome.
-Draw a heart: Just like our house is wired by an electrical circuit, the heart has one too. In your
case, there is a problem in the conduction of current through the circuit. And that is why your
heart is not functioning properly and that is why you are having syncope or you fainted.

-There are many causes for it, but it looks like you're not on any medications, you have no eating
issues, no CNS problems, but I would like to order some investigations like FBE, ESR/CRP, LFT,
RFT, UEC. If the investigations come back normal, since you are not on any medications then we
can call it a Congenital Prolong QT Syndrome. Congenital= disease or physical abnormality that
present from birth.

Treatment.
-Unfortunately we have to treat this condition accordingly, as it would have serious
consequences. I’d like to admit you into the hospital, call for cardiologist registrar.
- They might put you on some medication like Beta-blockers. If it fails or if they think, it’s
required they might put on a pacemaker.

-Once you are discharged from the hospital, I will give a letter to your GP and he will follow you
up regularly.

-I will also arrange a family screening for this condition (only say this if congenital)

Case (27/4/2017)
multiple faints due to prolonged QT ( score 6 Dx )
HMO in ED, 20 yrs old woman c/o fainting in school. 4th attack, mom is coming with taxi.
H/o, ask pe, ask initial ivx from examiner, explain it to pt. Explain the possible causes to pt.

A little bit language difficulty for me in this case, I couldn’t properly understand what she was
doing just before the fainting attack. Kinds of like school gathering and singing. Just ask some
routine question like how are u feeling, get hurt, first time. B4, during, after….
*pf (attack happened during singing in school, previously quite similar, no seeing of blood or
emotional change,

before -no skip meal, no palpitation, no vision problem,


during -loc. 30 sec, no jerky m/m, no bite or wet herself,
recovery -no weakness, spontaneous, no FH of epilepsy.

O/E; BP 100/60 standing and 106/65 on lying, HR is 56/min.


No other injuries.
ECG also show bradycardia but no other significance (mistake should recognise long QT).
IVx - BSL 5.7.
I explained the ECG first, this is your ECG, we can say your heart rate is very slow which is around
56, normally, it should be within 60 and 90. So, when the heart rate is slow, the heart cannot
pump adequate blood especially to the brain and cause you fainting attack. I also found your
blood pressure is a little bit low. These may be the reasons that u have fainting attack.

Sometimes, it can be due to a condition we called it vasovagal syncope (mistake should add
unlikely), we have a nerve called vagus nerve which control our heart rate and bowel function. It
can be over stimulated by certain condition like emotion events, seeing blood, being in crowded
place, hearing bad new, things like that.

Sometimes, it can be due to low blood sugar level, but in your case, it is less likely, your BSL is
good.

Sometimes, can be due to injury to head but you have no head injury.

Check her understanding. I still have time, asked her about exercise a lot, she said yes. I just
reassure and time up (mistake should say admission and serious).

FB-FAILED, OVERALL-3, HX-6, ASK PE-4, IVX INTERPRETATION-2, DX-1.

P.S-multiple faints due to prolonged QT ( score 6 Dx )

ECG rate looked slow bcoz paper speed 50, QTc interval ? 514, v prolonged.

Case (4/10/2017) Collapse – Pass


Scenario
15year old girl presented with sudden loss of consciousness. She has an ECG done soon after.
This will be provided after completing other tasks.

Cannot remember tasks that well. I think history and explain the diagnosis to patient.
I was paralysed by previous case where I did not complete tasks.

Started with asking for vital signs from examiner. Vitals stable, pulse regular, no postural
hypotension.
(History)
- Collapsed at school. Asked details of the incident but cannot remember in detail. I think she
collapsed while sitting, no warning signs.
- History of exertion + but some time before the incident????
- Loss of consciousness for one min. Witnessed.
- Nothing to suggest seizures.
- Nothing to suggest cardiac cause in history.
- No indication of vasovagal.
- No trauma/ fall.
- No recent illness /fever. Now feeling completely fine.
- This was the 3rd attack. First similar attack occurred one month before.
- LMP 10 days before, normal menstrual bleeding. Never been sexually active. Never used SAD.
- Social history unremarkable. Asked for permission and offered confidentiality before asking
sensitive questions as she is an adolescent.
- Family history of similar symptoms or sudden unexplained death is very important here. I can’t
remember any positive responses.
- Medication history nil of note.

PEFE All normal.

Asked for ECG : Shows Prolonged QT interval.

Explained:
Probably a congenital abnormality with electric impulse conduction in heart.
Explained how electrical impulses control heart activity and how we record it in this ECG strip.
This condition makes her prone to arrhythmias resulting in collapse. Used simpler words in
explanation.
Mentioned it is potentially dangerous and need further management. (Did this all the while
wondering why do they ask us to explain this to a teenager in one or two minutes when it took
me such a long time to learn all these.

4/4 key steps covered

Feedback 9-5-2018
15 yr old fainting. 3rd time today, she is in hospital, ecg is done and will be provided to you after
pefe. Task hx, pefe, interpret ecg, explain dx to pt ( all 3 episodes doing different things, nil sob,
chest pains, nauseous n vomiting. Nil palpitations, pefe, nil murmurs, PR 54/min regular. Dx sinus
bradycardia? Unknown cause)

2 minutes outside: this is the case of patient loss of concious ness so i have around 17 dd on this
such as drug, infection, hypogycemia, thyroid, heart issue, stroke, trauma, epilepsy....

Inside- I think this is easy case but the thing is you have to ask all the dd questions and read the
ECG properly. If you could not recognize the ECG, this can fail you. This is the QT prolong cases.

Feedback9-5-2018
Case: Prolonged QT
ED case, young girl with history of syncope last one today at school or uni or with friend I can´t
remember where she was sorry. She has now recovered and doesn´t have any symptoms
currently.
Task
Take history
And ECG has been taken ask the examiner for
Interpret ECG
Diagnosis and differentials to the patient with reasons
She has a history of syncope today the third episode, she felt a bit dizzy just before fainting, but
recovered completely after each episode, no abnormal movements during fainting, no passing
urine or stool, no fever, no other symptoms. No relevant family or personal medical history.
Although I asked sadma I forgot to ask about street drugs used. So I asked for the ECG to the
examiner and it was prolonged QT for me. I explained to the patient but not to the examiner
(mistake of course, outside just said interpret the ECG so I wasn´t sure to who to interpret the
ECG) I asked for a blood glucose which was normal. And draw and explained prolonged QT to the
patient. Then I said differentials to patient like low glucose, low blood pressure and the reasons. I
think I didn´t mention nothing else.
Passed. Global score 4

Feedback 25-10-2018
Scenario: collapse
Stem : 15 yr old girl with 3rd episode of fainting.
Task :
hx
pefe
Interpret inx which the pt has
Dx ddx
Old recall prolong Qt case. Did it as a syncope cluster
GS: 6
4/4 key steps covered Hx:5 Choice n technique of exam: 6 Interpretation of inx :6
(to the pt not examiner) but examiner cqme close to me on table n checked how m i explaining.
I explained to pt that our heart has some electric wiring n from this point(drew it) current
generates n spreads out n we record the current in this graph. And then told her rate looks a bit
decreased than what we expected n ur age but ur heart is generating regular beats. And as u can
see this qrs complex n this is the t wave, we want them to be a bit closer to each other n drew
how it should be but in ur case they are way more apart. We call this qt prolongation n this is the
reason of ur frequent fainting attacks.
Dx/ddx :4

Feedback 25-10-2018
15 yr old girl comes due to repeated faints. Her mom is on the way.
Task:
History
Interpret investigation
Diag/dd
I took all history for faints as much as I could remember: vasovagal,palpitations, hypoglycaemia,
anemia, head injury, dehydration, stroke, epilepsy , family history, fever.
Was given ecg, it had bradycardia and what looked like prolonged qt. had to explain to pt.
Explained layman terms, said it looks like you hve prol qt syndrome but other possibilities are-
vasovagal, epilepsy, nasty growths, arrhythmias. Will explain everything to mom. Dw etc etc.
GS 6

20-Upper arm DVT


Middle-aged lady has a swollen right arm.
Task-
-Take history.
-Ask Physical examination findings from the examiner
-What is the diagnosis and what investigations would you arrange?

History
1-swelling questions
-how long? (4 days ago)
-sudden or gradual? (Gradual)
-is it constant or come and go?
-is it getting worse or increasing in size?
-is there swelling anywhere else?
-anything makes it better or worse?
-has this happened before?

2-associated symptoms questions


-any pain in your arm? (Yes)
since when? Suddenly or gradually? Getting worse? Pain anywhere else? Radiation? Severity?
Type? Aggravating and relieving?
-any redness of overlying skin?
-any weakness, numbness or tingling?
-any fever? Loss of weight? Loss of appetite?
-have you had any pain in your neck?
-have you had any chest pain or shortness of breath?
-have you had any trauma or fracture? any insect bite?
-have you had any repetitive movements of arm or shoulder? (key) (Yes she is basketball player)

3-Risk factors questions


-past medical history (heart disease, liver, cancers, varicose, clotting problems)
-past surgical history (recent surgery)
-Past obstetric and gynecological history (OCP use, pregnancy)
-travel history (how long? Did you move around during travel?)
-SADMA
-occupation (if forget the key ask here: does your work or your hobby require you to do any
repetitive movements of arm?)
-family history of clotting problems, thrombophilia

Physical examination findings from the examiner


1-general appearance
-pallor, cyanosis, dyspnea + BMI
2-V/S
3-arm examination + compare
inspection
-extent of swelling
-redness
-trauma, injury or insect bite
palpation
-tenderness
-temperature
-CRT, Pulse
-oedema (pitting or non pitting)
-neuro (tone, power, reflex, sensation).
4-Neck
-look, feel, move (deformity, tenderness, movements)
5-Leg examination (DVT + varicose as upper steps)
6-chest and heart

Explanation
Condition
-there are various reasons why you have swelling in the upper arm. It could be cervical rib or an
extra rib in the neck area that can cause compression of blood vessels of that area.
Other possibility is surgery of armpit or breast but unlikely, as you have no history of surgery.
Cellulitis unlikely.
However, you most likely have a condition called deep venous thrombosis, which is a formation
of clot in the veins draining the arm, causing blockage of blood flow.
Clinical features.
-leading to swelling of the upper arm, pain and redness.
Cause
it can be caused by many factors, could be an acute effort thrombosis which is not uncommon
condition in young active people who is doing effort or repetitive movements like basketball
players.
Other risk factors are……
Complications
-if left untreated, the clot can travel to another part of the bod especially the heat or the lung
causing serious consequences.

Investigations
-our main concern is to rule out DVT so that I need to arrange for Doppler US of the upper limb,
along with basic blood Ix; FBC, LFT, UCE, coagulation profile, thrombophilia or clotting screen. X-
ray to rule out cervical rib.

Case (2/3/2017)
arm pain since 2 days.
1)Hx - plays tennis .
2)PEFE- redness and swelling on arm, rest normal.
3)DD to pt likely D and Investigations

Case (6/5/2017) DVT upper arm (Hx & ddx)

Case (2/6/2017)
Rt arm pain and swelling ( 26 or 27 years old girl ) She was painting the wall few days ago History
2-3 minute, PEFE

Case (28/11/2017)
DVT in arm
Middle-aged woman has a swollen right arm.
Task-
-Take history.
-Ask Physical examination findings from the examiner
-What is the diagnosis and what investigations would you arrange?
(Started 4 days ago. Gradually. Never happened before. No IVDU/Trauma/Infection/Insect bite.
No swelling anywhere else.
There is pain in arms too. Just gave a history of playing competitive/excess of basketball and did
painting work at home recently. )

Recall 9-5-2018
24 yr old, nil ivdu, insect, trauma, Dm, Rt whole arm swollen, task hx, dx/ddx investigations
with reasons ? Subclavian thrombosis? Lymphangitis
Outside- trauma, insect bite, DVT, nasty growth, surgery block the lympho...investigation are
need US, thrombophila, X ray of chest.BSL, FBC, ESR< CRP, celulitis

Inside- This is easy case but investigations need to be careful.Also, need to ask about COSTVMPF,
lung symptoms and leg symptoms.

Case (28/11/2017) Station 6: Painful swollen right arm – pass


Middle aged lady has a swollen right arm.
Task-
-Take history.
-Ask Physical examination findings from the examiner
-What is the diagnosis and what investigations would you arrange?
(Started 4 days ago. Gradually. Never happened before. No
IVDU/Trauma/Infection/Insect bite. No swelling anywhere else. There is pain in
arms too. Just gave a history of playing competitive/excess of basketball and did
painting work at home recently. )
Key steps: 4/4
Approach to the patient: 6
History: 6
Choice of examination, organization and sequence: 6
Dx and DDx: 6
Choice of investigations: 7
Global score : 7
History:
Pain questions: Sortsara
Duration of swelling
Swelling in any other parts.
First time happening?
Any recent trauma
Any hx of insect bite
Hx of surgery/procedures in arm or shoulders
Dvt q: pain/swelling / change in colous of the legs
Difficulty in breathing/SOB
Recent histpry of travel
On any contraceptive
Any repeatative movement of arm/shoulder (pt said she is a basketball player)
Past hx : varicose vein, cacer, clotting
Past sx hx.
Sadma
Fhx of blood disorder/clotting problem / similar complain
Exam:
GA/ Vitals/ BMI
Exam of upper limb:
- Inspection: site and extend of oedema, any s.s of truma, s/s of
injury/inflammation, inspection of opposite arm ,
redness/pallor/cyanosis.engorged vein
- Palpation: puls, CRT, pitting or non pitting oedema, tenderness, sensation ,
movement
Examination of lower limb for dvt
Quick examination of body fr swelling of any other parts. Chest examination
Diagnosis: Most likely it’s a blockage of any veins draining your arm. Can also
be due to
D/D: Cervical rib(an extra rib can be present on the neck area can causes
compression of the blood vessl which can lea d to the same problem.), surgery of
your armpit or breast causeing swelling (but least likely as no surgery ) , any clot
travelling from any other parts of the body causeing blockage.
But our main concern is to rule out dvt of the upper limb for which we will do
Doppler usg of the upper limb and along with the basic investigations like full
blood examination, Clotting profile, rft, x ray to rule out cervical rib. Later on
depending upon the cause we will do the thrombophilia screen.

Feedback 9-5-2018
Case: Effort thrombosis
I think it was at the ED department. Middle age male, with a swollen arm for 2 days. No other
symptoms.
Task:
Take history
PE from examiner.
Give diagnosis and differentials with reasons and management.
When taken history he denied trauma, insect bites, fever, pain, changes in color of skin, no
reduce of sensation. When asked he said he was painting a week ago or 3 days ago. No any
relevant medical history I think family history of High BP. PE from examiner vital signs were
normal, presence of oedema, I think it was right arm, pulses presents (radial and brachial) no any
changes in color of skin, neurological normal. All normal apart from the oedema.
Diagnosis: acute effort thrombosis/subclavian vein thrombosis, cellulitis, lymphangitis. But my
main diagnosis was Lymphangitis and I explained to the patient about it. Then I mention
Thrombosis and cellulitis as differentials. Karen notes case
I knew the case but I didn´t practice this case I just read it. I was so nervous I got very confused
my mind was a chaos at that time I am pretty sure I fail this case. However in the management I
really emphasized Echo Doppler to rule out any cloth.
I passed this case I don´t know how.
Global score 5

Feedback 5-10-2018 DVT Upper arm effort thrombosis

Station 6 (Subclavian vein thrombosis)

Question: A young man came to you because of painful right arm swelling started few days ago.

Tasks: Take a history for 4 min. Ask PE findings from examiner. Tell the Dx and DDx to the patient
and explain about investigations you would like to order to the patient.

Score: I passed this case with global score 4.

Key steps 1,2,3,4 : All Yes

Approach to patient: 4

History: 4

Choice and technique of examination, organization and sequence: 4

Dx/DDx: 4

Choice of Investigations: 4

My performance

Hello…. David…. I’m Dr. MM, one of the doctors in this clinic. Nice to meet you. I knew that you
have some painful swelling in your right arm for which I have to ask questions to you. Are you
comfortable right now? (Yes) If you have pain you can’t bear during our discussion, let me know
at any time. OK?

Is this swelling first time for you? (Yes) Since when did you notice it? (like few days ago) Did it
happen suddenly or gradually? (kind of sudden) Can you please tell me up to which part of your
arm did the swelling extend? (the patient pointed from shoulder joint to around the elbow and
then to the wrist) Is there any pain in the swelling? (Yes) Do you notice any redness over the
swelling? (No) Did you have injured to right arm recently? (No) May I know your occupation?
(I’m a postman) By the way, did you use your right hand excessively during these days, especially
by raising it up? (Yes, doctor, I have to paint a lot in my house these days) All right, David. Did you
have any blood clotting problem before? (No) Any similar problem in your family members? (No)
Any underlying medical problem? (No) Any operation done to your armpit or around your right
shoulder? (No) Do you smoke? (No) Any regularly taking medication? (No) Any history of
diagnosed with cancer before? (No) Thank you for the history, David. I have to examine you right
now with my examiner. Could you please wait for a while?

Dear examiner, I want to start my examination. Firstly I will measure temperature, blood
pressure and pulse rate. (Examiner gave me each finding and all are normal) Then I will examine
my patient’s right arm comparing with normal left arm. On inspection, any signs of injury? (No)
Any redness? (mild redness present) I want to know extent of swelling. (whole right upper limb)
On palpation, any local rising temperature? (No) Any tenderness in the swelling? (slight
tenderness present) I’ll palpate to look for any axillary lymph node enlargement. (No) Thank
you ,examiner. That’s the end of my examination. I’ll get back to my patient.

Ok, David, according to your story and my PE findings, you have a condition we called deep vein
thrombosis. Let me explain more about it. There is a large vessel called subclavian vein carrying
blood from whole of your right arm. I suspect there might be some blood clot occluding in that
vein. So the blood from your right arm can’t flow back to your body and trap in the tissues of
your right arm giving this painful swelling. Are you with me so far now? There are other possible
causes, too like inflammation of skin and muscles in your right arm because of some infection we
called cellulitis but it’s unlikely since you don’t have fever. Sometimes having injured to your right
arm can present like this but you didn’t have any such history and so it’s unlikely. In some cases,
clear fluid we called lymph can be trapped in tissues of your right arm we called lymphedema
which doesn’t fit with your detailed history.

So, to confirm my working diagnosis, I have to order special ultrasound imaging to your right arm
we called Doppler ultrasound. Then I’ll have to do some basic blood tests and inflammatory
markers to rule out other causes like cellulitis. Don’t worry, if it’s clotting problem, we can give
you some drugs to dissolve that clot and the problem will go away. I’ll give you some reading
material to read about this condition too. Then the bell rang.

21-C-section request
You are at your GP when jenny, 25 year old, at her 20th week of her first gestation presents to
you with a request of C-section to be conducted as she does not want a vaginal delivery.
TASKS
Take a further history
Counsel the patient

History
1-WHY
-hi jenny I can see from the notes that you are pregnant at 20 th week, is it a planned pregnancy?
Congratulation!
-So I can see that you come to ask for CS instead of vaginal delivery, can you tell me why?
(Because of pain one of my friend had CS and had lots of pain)

2-recurrent visit questions


-How is your pregnancy going so far?
-have you had regular antenatal checkup?
-have you done down syndrome screening at 11-13 weeks?
-US at 18 weeks (no) ok I’ll give you a referral because it is important to have one
-have you done blood tests, blood group? Did you take folic acid?

3-late pregnancy complications questions


-any headache, blurring of vision, leg swelling?
-tummy pain?
-any vaginal bleeding or discharge?
-any fever, nausea and vomiting?
-burning or stinging on passing urine? How’s your bowel habits?
- Have you started to feel the baby kicks?

4-General questions
-SAD
- exercise
-support
-past medical and surgical history

Counselling
1-explain the mode of delivery (draw a diagram)
-Vaginal birth is a natural way of giving birth to your baby. Whereas C-section is a surgery where
we a cut is made in the lower part of your tummy along the bikini line and another similar cut
along the lower part of your womb through which the baby and placenta will be delivered under
anesthesia.

2-Indications
-We usually go for a C-sections if there are definite indications.
-C section can be done in a planned or an elective way or as an emergency procedure.
*So Indications for elective CS are:
- placenta previa (low lying placenta covering the birth canal)
- cephalopelvic disproportion (due to narrow birth canal or big baby)
-abnormal presentation or lie (transverse rather than longutidinal)
- previous2 CS or a vertical CS.
*An emergency CS can be done in the following situations:
- prolonged labour
-labour obstructed at any stage
-some cases of abruptio placenta
-uncontrolled preeclampsia
-cord prolapse (cord presents earlier than the baby)
- Fetal distress (if the baby becomes unwell at any point in time)

So far you have no indication for CS

3-Complications of CS
C-sections carries certain complications to you and to your baby.
Immediate complications can be:
-risk of anesthesia
-bleeding problems
-infections
-injury to the surrounding structures like the bowel or the bladder.
-breathing difficulty in baby
Let me assure that all of these can be minimized because it is usually done by a trained
specialist.

Long term complications include:


-clots formation in the vessels of your legs
-when you will be pregnant next time and as you will have a scar in your womb so there can be a
high risk of rupture if you choose normal vaginal delivery.
-pain in case of normal vaginal birth will be gone by the time the delivery is complete but for CS
even though you do not have pain at time of delivery but the pain continues till the wounds
heal.

4-advantages of the vaginal birth over a C section:


-recovery is quick
-less hospital stay (in a day or two then you can go home while in CS you need to stay for 3-5
days)
-you can walk around soon after birth but in CS you require rest and more support to look after
yourself and the baby.
-there are no complications like bleeding or risk of anesthesia, or infection from the womb
because it is not a surgery.
- You can have any number of future vaginal deliveries. Whereas in CS it is always limited to 3

5-Risks of vaginal delivery


However, there are some risks with vaginal birth as well. These include
-a failure which can lead to a C-section.
- there could also be some damage to the pelvic floor muscles that can lead to future
complications like incontinence. But it can very well be managed if you start doing pelvic floor
muscle strengthening exercises after delivery.
-tears and lacerations which will be repaired at time of birth.
6-pain relief options
As I see that you are concerned with the pain associated with the vaginal birth, there are
excellent pain relief options that are available for you at the time of labor. It could be
pharmacological or non-pharmacological.

*Pharmacological options:
-epidural, where anesthetic drugs will be introduced into the outer covering of your spine and it
can be topped up anytime.
- By giving nitrous oxide and oxygen inhalation via mask

*Non-pharmacological methods
- Certain positions during labor can also reduce the pain
- Deep breathing techniques can also be used as well.
- Hydrotherapy can also be done, which is giving birth while on water
- Using TENS (transcutaneous electrical nerve stimulation) where 2 electrodes are placed on
either side of the spine and a small electric current will be passed which can inhibit the pain
fibers.

Ending
-by the end of the day, it is your choice.

-But if you go through CS without indications you need to cover the expense by yourself.

-You are only at 20 weeks, and there is still a long way to go. If any complications will occur, we
may go ahead with the C-section if indicated.

-When the specialist meet you next time make sure to discuss this with them (no need to refer
her)

-reading materials regarding vaginal birth and c section, and also pain relief options that are
available during your labor and delivery.

OTHER CASE:
when you ask why the reason they give you: incontinence following vaginal birth

(all the same but you can say these instead of pain relief options)
Incontinence sometimes happen after vaginal birth, which is due to weakness of pelvic floor
muscles that support the womb, birth canal and vagina.

Vaginal birth is not always the cause of incontinence. There are other causes like:
- weight gain after delivery
- chronic constipation,
- chronic cough
- injury or tears of the birth canal

To deal with these


1-make sure that your weight is in the normal range after delivery
2-healthy diets with fibers and fluids to prevent constipation
3-just in case you developed a chronic cough report immediately
4-tears also will be repaired at the time of delivery

Key things you need to do is pelvic floor exercises, you can go to prepartum and postpartum
classes where you will be taught correct way of doing such exercises which you can continue
lifelong.

Feedback 26-10-2018 Obstetric counselling Passed (GS – 4)


28 year old women with 20 week GA comes to your clinic with request of C-section to be
conducted as she doesn’t want Vg delivery.
Task: take Hx, counsel her regarding her delivery plan
Hx:

 Any symptoms now? (BPV, discharge, pain)

 ANC Hx ( blood group and blood tests, STI screening, USG at 18 week)
 SADMA, Past medical and surgical history, Obstetric Hx (G1 P0)

 Her concern & reason why you want to do C-section – she is worried about labour pain)

Counsel

 Explain procedure of NVD and CS

C-section

 Indication of CS (CPD, big baby, abnormal lie and presentation, if mom is unwell due to
PE, other medical comorbidities such as epilepsy, or if baby is unwell)

 Risk of CS over NVD

i. risk of anaesthesia

ii. risk of surgery (bleeding, injury to surrounding structure, infection such as


pneumonia, UTI, wound infection

iii. Although there is no pain during operation due to LA, pain will continue after CS
until your wound is healed. However, in NVD, pain will go away shortly after
delivery. Therefore, compared with NVD, CS will need longer hospital stay.

iv. Other post-partum risk (risk of DVT in C-section are higher than women with
NVG)

 Limitation of CS: can do maximum 3 times.

NVG

 As you are concerned about labour pain during NVD, we can give effective medication
which will be injected into spine. There are also some non-pharmacological methods
such as hydrotherapy.

I will give reading materials regarding delivery options. You can read at home and discuss with
your partner. Also you are now in 20 weeks GA. There is long way to go. We will follow you up
throughout the pregnancy. If there is no complication, decision ic up to you. Obstetrician will be
the best person to discuss delivery plan in late pregnancy.
Are you with me so far?

22-Marijuana Counselling
Case 1
24 years old boy coming to you GP clinic and he is known marijuana addict. His father knows
about this and he wants the boy to quit, the boy had also fights with his brother. So the boy had
cutting marijuana down a little bit.
TASKS
-Assess his dependency on marijuana
-Counsel the patient accordingly

Case 2 (AMC exam Case)


GP, 22-year-old boy came to see u because his father wants him to see you. He had a fight with
his elder brother because of his marijuana use a few days back. He didn’t want to see any doctor.
But he showed up today.
Tasks
- take history for 3 minutes
- assess his willingness to address his marijuana use
- counsel further management plan

History
CMT DM
1-Appreciate him and make Confidentiality
- Hi ----I appreciate your coming to see me today. I can see from the notes that you’ve been
taking marijuana and it’s good that you are trying to cut it down.
-can I ask you a few questions first to assess your current condition and let me assure that
everything we gonna discuss here will be confidential, unless there is a harm to yourself or to
others.

2-Marijuana questions
-Since how long have you been using marijuana? (since 13 years old when his family separated
because of financial issues)
-how often do you take it? (daily)
-can you tell why you started using it? (fun!!)

3-Tolerance question
-since you started using marijuana, have you increased the number of time or dosage to get the
same effect? (reduced recently)

4-Dependence question
-how long can you go without taking it?
-have you noticed any withdrawal symptoms if you didn’t take it?
-do you think you are dependent on marijuana?

5-Motivation question
-On a scale of 1-10 how motivated are you to stop marijuana? (don’t know)

6- Psychosocial history and HEADS


Mood questions (normal but stressed because of financial issues)
-how is your mood?
-have you ever thought of harming yourself or others?
-how is your sleep?
-how is your appetite? Have you lost or put on weight recently?
Delusion and hallucination
-just a routine questions I ask to my patients. Do you see, hear or feel things that others do not?
(Sometimes get hallucination while on marijuana)
-do you think someone is trying to harm you or spying on you?
Insight and judgement
-do you think you need a professional help?
-if we supposed there is a fire in this room what would you do?

HEADS.
Home
-Who do you live with? (parents separated)
-any stressor at home? (got fights with brothers)
-How’s your relationship with your brother? Could you please tell me what exactly happened
during the fight? Were you on marijuana when the fight happened? Do you have fights quite
often?
-do you have good support?

Education/ work
-do you work? (part time job after he dropped out of school)
-any stresses at work?

Anhedonia (you can ask this here or with mood questions as it is one of the 2 major criteria of
depression)
-have you lost interest in things you used to enjoy?

SAD
-do you smoke, drink alcohol, take recreational drugs aside from marijuana? (use speed and
alcohol but not smoking)

Social activities
how is your social life? Do you go with your friend quite often?

7-past mental history


-Any past history or family history of mental illness?
- Any problems with law so far?

Counselling
Talk about Marijuana
Condition and clinical feature
-Marijuana is an illicit drug at the time of its use, you could feel happy and excited, relaxed and
confident and more talkative but these are just the immediate effects of marijuana. And it is the
long term harmful effects that becomes quite predominant.
Complications
so smoking marijuana could lead to:
-damage of the lungs resulting in asthma, bronchitis and even cancer.
-It can lead to loss of memory, decreased attention span, learning and performance difficulties.
-It can also result in mental health problems like mood swings and schizophrenia.
-It also decrease the sex drive, can cause subfertility.
-Can also have problems at work, social and financial problems
-and you can end up having trouble with law.
So more than the physical addiction that marijuana causes it is the psychological addiction that
predominates which makes you feel that you need to take marijuana to act normally.

Management
-you require a referral to the drug addiction centre and the usual preferred mode is complete
abstinence from Marijuana.
-once you cut it down completely you can get withdrawal symptoms like feeling a bit anxious,
irritable, angry and aggressive, you can have cravings, sweating, chills and rigors but all of these
can be dealt with antidepressants and other medications.
-you will be given CBT by psychologist to help you stop drug use and enhance self-control as
well.
-other is motivational enhancement therapy, which will motivate you to change your habit and
engage better in treatment.
-with your consent, family based therapy can also be arranged.
-support groups are available like marijuana anonymous.
-Life style advice
-many people can get over it successfully with all these behaviour therapy
-reading material and refer to drug rehab centre.
-any specific concern I’ll be here to help you.

Feedback 21-6-2018 FAIL


A young man had a fight with his brother today. No one was hurt during the argument. The
father blamed his son for using Marijuana. Assess his motivation for quitting and counsel him.
Behavioural Complaint: Fail: Global Score 3

Before I enter the room I was almost sure that I was going to fail this station. However, tried to
do my best. However, I finally did not quantify his motivation. I did not tell him what symptoms
he may develop if he quits, I did not exactly tell him what kind od help is available for him. And
yet I failed with a global score of 3! Which probably means that with just a non-judgemental
approach you may do half of the station.

Case (29/4/2017)
GP, 22-year-old boy came to see u because his father wants him to see you. He had a fight with
his elder brother because of his marijuana use a few days back. He didn’t want to see any doctor.
But he showed up today.
Tasks
- take history for 3 minutes
- assess his willingness to address his marijuana use
- counsel further management plan

the patient mentioned mari as DOPE. He has been using it since 13 years of age when his family
was separated coz of financial issues. So he dropped out of school and has been doing part
time jobs. When ask about dose and level of addiction, he said the amount and timing of use
was reduced lately. Didn’t exactly tell the amount but he used it daily. Mood was normal but
stressed out because of financial problems and jobs. So he cant quit. Use of Speed and alcohol
as well occasionally but no smoking. When I assessed his motivation to quit mari, he said I
don’t know.

Feedback 5-12-2018 Marihuana Counselling


You are working as GP, A father has sent his son who is very much concerned about his son had a
fight last night with his brother, none injured and no police involved. Hi is taking Marihuana from
teenage years and come to see you now for help.
Task:
1 assess motivation level
2. Counsel him accordingly

+ve - having dope since teenage, he is not increasing dose as he wants to quit to concentrate on
work and avoid fights, no withdrawal symptom, nice role player, motivation level 7/10, no legal
issues just got fine during driving, Ice sometimes and smokes 20 Cigs.
Approach: I introduced myself. Confidentiality statement. How are you feeling at this moment?
I’m really sorry to hear about the incident, anyone injured? Is it the1st time (he said no
happened quite a few times before). I asked do you think this is happening because of Dope. He
said yes, I think so.
Let me assure that mark I will help you to go through this and I really appreciate that you have
come to see me today. Is it okay if I get some more details? he said fine.
How long? How many times/day? (he said he used to take a lot now with Cigs so 20 times),
Tolerance Q- Have you increased the amount/frequency recently to get the same effect? (No,
actually I have reduced), I really appreciate that Mark.
Dependence Q: Have you ever tried to quit Mark? (No, but this time I need to as I’m looking for a
job)
Why do you take this? (to calm me down, I feel good). How long you can go without it? (not a
single day)
How much motivated Mark are u at this moment in scale of 1-10? (7)- that’s great Mark.

Then Psychosocial Hx- HEADSSS


Mood, sleep, psychosis Q, Anxiety Q
Any Legal issue?( Got driving fine for speeding and lost couple of points)
Any cough, chest pain, leg pain? (no)
Any psychiatric problem? (no)
Any medication? Any other drugs? Alcohol?

Counselling- Look Mark from your story I have found that you are quite dependent on dope but
you are quite motivated as well which is good. Do you know the harmful effects of dope? (No)
Ok 1st of all as it’s a illicit drug though you feel calm and relaxed when u use it but long term
effect is it can cause you much dependent upon it and at the same time it may damage small air
channels of your lungs as you smoke we call it COPD, may cause Cancer as well. It may also cause
legal issue thank got you got only driving fine but things could be worse than that. He nodded.
Are you with me? Yes Doc. I am here to help you out. There are some good therapy to help you.
Like when you stop it we call it going to Cod Turkey, you might feel anxious, restless, agitated,
sweaty, but to keep it under control we might prescribe a special medication called Fluxitine, but
I will refer you to Drug Rehab Centre for this they will do supervised withdrawal. At this point he
said ‘Will they get my all details?’ I assured him that they are highly skilled and they will keep ur
details confidential. He smiled. I said I can see that you are not quite happy about what happed
last night and distressed so I am planning to send u for a visit to Counsellor, Ok Doc. There are
support group called Marihuana Anonymous will be able to help you. Please avoid those people
who takes Marihuana, healthy life style, family support with your consent, Exercise, fruits,
healthy balanced diet, here is pamphlet for you for you better understanding and Im making
arrangement for your referral. He said thanks Doc, Bell Rang.
Key step-1,2,3- all covered

Global score- 4

23-Mesenteric ischaemia
You are HMO in ED. 50 year old man come with abdominal pain for few days.
Tasks
-Take relevant history
-Ask PE from examiner
-Explain possible causes to patient

History
1-is my patient hemodynamically stable (yes) then reassure the patient that he is stable.

2-Pain questions
-how severe is the pain from 1-10 1 is the least 10 is the most? (8-9) Ask for allergy then offer
painkillers.
-can you tell me more?
-since when? Sudden or gradual? Constant or come and go? Is it getting worse?
-show me exactly where the pain is? (all over the tummy)
-does it go anywhere else? (No)
-can you describe it for me?
-does anything make it better or worse? (Lying down and movements)
-has this happened before? (No)

3-associated symptoms
Nausea and vomiting
-do you feel nauseous?
-have you had any vomiting? (Yes twice) what does it contain? Any blood? (just food)
Bowels and bladder
-how is you bowel motions? Any constipation or diarrhea or alternating constipation with
diarrhea? Any change in the colour of your motions? Any blood? (Dark motions)
-how is your urine? Any change in colour or urine stream?
Fever, rash, recent infections?
LOW, LOA, lumps and bumps?
Chest pain, SOB, yellowish skin?

4-because of black stool


-any dizziness, funny racing of the heart? (yes) for how long? Are you having it now? Can you tap
it on the table?

5-General questions
-medications?
-PMH (peptic ulcer, heart disease)
-PSH (+ve)
-SAD
-family history

Note/ in some candidate palpitations will not be given in history and they should ask specifically
if the pulse rate is regular or not in the PEFE or the examiner will comment only about the rate.

Physical examination findings from the examiner


1-General appearance
pallor, jaundice, anxious, sweaty, in pain
2-Vital signs especially pulse (ask rate, rhythm)
3-Focus abdomen
inspection: movement with respiration, distension, visible masses, scars, dilated veins
palpation: tenderness (+ve). site?
auscultation: bowels sounds
-complete with: hernia orifice, genitalia, LN, per rectal (+ dark stool)
4-CVS
apex beat, thrill, heave, heart sounds and murmur
5-Chest
-dullness, air entry, breathing sounds, crepitation or wheeze,
6-Offiec tests
-urine dipstick -BSL -ECG

Diagnosis and differentials


1-mesenteric ischemia (most likely)
2-others
-peptic ulcer perforation
-rupture diverticula
-pancreatitis
-trauma
-bowels obstruction

Explanation
-most likely you have acute abdomen mostly due to a condition called mesenteric ischaemia. The
blood supply to your bowels has been cut off due to blockage of one of the arteries and this
might have resulted from a clot, which has travelled from the heart mostly due to arrhythmia
may be AF as you have history of palpitation and irregular pulse on exam.

-other possibilities are perforation of ulcer in the stomach or 1 st part of the gut, rupture
diverticula or an out pouches in the wall of the large bowels, could be pancreatitis or an
inflammation of an organ called pancreas, bowels obstruction, or trauma.

Treatment
-this is a medical emergency
I would like to: admit you, secure 2 IV lines, start IV fluids, please do not eat or drink anything by
mouth, take blood for FBE, blood group and cross matching, U&E, coagulation profile, lactate.
Call registrar to come and have a look, arrange for abdominal x-ray.
-the registrar might decide to do CT angiography of the artery if they think they have enough
time. But if they believe it is urgent they might do surgery to open up the block ASAP to restore
the blood supply to the bowels. During the surgery if they find that the part of the gut has been
affected and is not viable they might remove that part.
-they will start you on Iv antibiotics.

Case (22/2/2017)
You are HMO in ED. 50 year old man come with abdominal pain for few days.
Tasks - Take relevant history
- Ask PE from examiner
- Explain possible causes to patient
2 mins thinking
- DDx depends on site of pain but it doesn’t mention where the pain is
- Not to miss HD stability, pain killer, Pain questions and special tests on PE if applicable
History taking
When I stepped into the room, role player is lying on couch. I approached to him and greeted.
Good morning John, I am Dr Khine. I am one of the Drs in this ED. Let me talk to my examiner first.
Examiner, I d like to know whether my patient is HD stable or not (Yes, U can proceed)
John, I ve checked ur BP and important signs, u are stable at the moment but if u feel that ur
tummy pain is getting worse or severe discomfort, just let me know, I ll take the action
immediately.
John, I understand that u have tummy pain.
Can u grade it from a scale of 1 to 10? (It’s 8-9 Dr)
Would u like to have pain killer first? (Yep)
Are u allergic to any meds (No)
Alright, I will ask my nurse to give pain killer for u.
Could u pls tell me more about ur tummy pain?
Is it first time? (Yep)
Can u point the site of pain with one finger? (It is all over tummy)
Ok, Did it start from any specific site and spread all over tummy? (Not sure Dr, when I noticed it, it
s all over tummy)
Can u describe the nature of pain? (I dun know, what I know is its very severe)
Does it go to anywhere else? (No)
Anything that can make it better? (Lying down)
Anything that can make it worse? (Movement)
Ok, apart from that, any fever (No)
Nausea, vomiting? (I vomited twice) What did it contain? (Just the food I took)
Any diarrhea? (No, but the stool was black color)
What is ur waterwork? (It’s fine)
Do u have any past problem like food bag ulcer? (No)
Any significant past history like high BP, heart disease, DM? (I had my heart beating very fast)
For how long? (since few months back).
Did u see any Dr for that? (No) Are u having it now? (yes)
(I got a hint that it might be mesenteric ischaemia, I forgot to ask him to tap with fingers)
General health, SADMA - all clear
Thank you John for ur information. I d like to ask my examiner for some findings and I ll get back to
u soon.
PE from examiner
GA - patient is in pain (I’d like to add more pain killer)
VS - T : normal, BP -120/80, PR - 90 and irregular
Focus abdominal exam - guarding, rigidity, tenderness (+), BS absent PR - black stool
CVS - No murmur, Resp - no basal creps
Bed side test esp ECG - not available
Explain to pt
John, according to ur HO and PE, most likely ur tummy pain is caused by a condition called
mesenteric ischaemia. Let me explain it to you. (I drew a diagram of tummy with gut inside)
This is our tummy, this is gut and its blood vessel. In ur case, the blood vessel is blocked by some
clot and ur bowel doesn’t get O2 and nutrition. It causes damage to ur bowel and it bleeds. That’s
why u have got tummy pain and pass black stool with blood in it
So far are u with me?
There are other conditons as well, like perforation of ur gut from underlying bowel ulcer or injury
to ur tummy, pancreatitis which is inflammation of a gland behind ur food bag. (I scribbled to add
pancreas in diagram)
I will inform my surgeon who will come and have a look at u. We will do invx like USG and Xray of
ur tummy to find out the exact cause.
AMC Feedback - Abdominal pain : Pass (Global Score - 4)
History - 3
Examination - 4
Dx/ DDx - 4

Case (30/5/2017)
acute abdomen ( irregular pulse, high temp. Decrease bowel sound, bloody diarrhea)
I said mesenteric cause irregular, perforation yes could be due to diverticulitis, she had 2scars of
previous op
so i said could obstruction as will , and unlikely due to colangitis or heart like MI and others.

Case (1/9/2017)
ischaemic colitis
57 years old woman complaint of abdominal pain
Task: history 4 min
Ask PE the examiner will tell u specific findings that u ask, Tell diagnosis and ddx with reasons

Case (10/11/2017)
Severe abd pain, old lady, mesenteric ischaemia, hx of heart disorder AF+, Pdx, DDx.

Recall 10-5-2018
Acute abdomen was mesentric ishcemia. Patient 60 yrs or so old, presenting with abdominal
pain.
Task:
History
PE from examiner
Diagnosis and ddx with reasons
And i don’t remember exact but i think. There was a task to talk abt complications also.
But i failed this case so cannot really give proper input abt it.

recall 11-7-2018
Acute abdomen PE with history taking,DD(mesenteric ischemia? Diverticulitis

24-Infectious mononucleosis
A child who had sore throat 4 days ago and was given amoxicillin by another doctor is brought
now with worsening symptoms and rash all over his body. His mother is here and concerned....
Task:
1. Take history
2. Ask examination from examiner
3. Tel the most likely diagnosis and differential diagnosis of the rash to mother

History
1-Sore throat questions
-I can see that --- has sore throat for 4 days. How severe is it at the moment?
-does he have any difficulty swallowing?
-is he able to eat and drink well?
-pain anywhere else? Joint, legs?
-has this happened before?

2-Rash questions.
-what about the rash? Can you tell me nore?
-when did it start? Suddenly or gradually? Is it increasing in size or number?
-where is the rash?
-is it itchy?
-I see it comes after amoxicillin. Has he had any allergy to this medication?

3-Fever
-has he had any fever? For how long?
-is it constant or does it come and go?
-how high is it? Did you take the chance to measure it?
-is it associated with shivers or chills?
-any night sweats?

4-Associated symptoms
-any yellowish Discolouration of skin or eyes? Any tummy pain? change in colour of urine?
Change in colour of stool?
-any nausea or vomiting?
-any headache or vision problems?
-LOW, LOA, lumps or bumps?

5-General
-did he come in contact with person with similar symptoms?
-have you travel recently?
-is his immunisation up to date?
-PMH?
-is he thriving and growing normally?

Physical examination from the examiner


1-General appearance (DR PJL)
-dehydration
-Rash
Inspection (site and extension of the rash, shape, colour, trauma, bruising, skin changes)
Palpation (tenderness, temperature, if hot to touch, elevated or not, blanchable or not)
-pallor, jaundice
-LAP (cervical, axillary, inguinal)

2-Ear, nose, throat


-ear and nose discharge
-throat (inflamed or not, any pus on tonsils, if tonsils enlarged, any petechial, gingival
hypertrophy)
-neck stiffness

3-Growth chart

4-vital signs especially temperature

5-Focus abdomen especially hepatosplenomegaly and masses

6-chest (air entry, breathing sounds, crackles or wheeze)

7-heart

8-CNS (tone, power, reflex)

Differential diagnosis
1-infectious mononucleosis
2-streptococus tonsillitis
3-lymphoma/ leukemia
4-hepatitis
5-CMV, toxoplasmosis

Explanation
-from history and physical examination, your child most likely has a condition called glandular
fever or it might be a reaction to the antibiotic. It is caused by a virus called EBV, which is usually
spread by direct contact like sharing drinking vessels, or kissing.

-This viral infection most of the time can initially mimic tonsillitis caused by a bacteria bug that is
why the previous GP might have started him on antibiotic.

-in this viral infection, 90% of cases develop rash after start of antibiotic, so we need to stop
antibiotic and the treatment is just supportive.
Treatment
-give adequate rest and fluid. Panadol for fever.
-will give him a school certificate and if you want a medical certificate for you as a carer.
-I need to run some Ix to confirm the diagnosis like:
FBC, ESR/CRP, UCE. LFT
throat swab.
blood film
viral serology (hetrophil antibody tests like paul bunnel or monospot)
-this disease is spread by direct contact so please maintain hygiene
-avoid contact sport till full recovery
-it might take time to recover approximately 6-8 weeks.

Red flags
-high fever (not responding to Panadol)
-severe vomiting (not able to tolerate food)
-lethargic, drowsy

Recall 22-2-2018
Sore throat ,prescribed AbX and got a rash with splenomegaly o/e

Recall 23-2-2018
Child developing rash following treatment with Antibiotics for a strep throat.

Recall 6-9-2018
Tonsillitis in a young girl. Treated with antibiotics. Developed rash. Task is take further history
and pefe. Differential and reasons

Recall 30-5-2018
‘Rash’:
EBV
Acute tonsillitis Dx 4d prior –amoxi. Now getting worse + rash.
Task: Hx, PEFE, Ddx, Dx
Feedback: Pass
HOPC
-Tonsil Qs; Fever Qs; Rash Qs
-Dehydration, Feed, Toilet, Sleep
-Sick contacts
O/E
-GA, Vitals, GC, ENT LA, nuchal, CVS, RS, ABDo (+ organomegaly), consent>Hernial orifices
-Rash in detail
Ax & P
-Rash Ddx with and w/o fever
-Top Dd EBV>w clinical correlates
Case (21/4/2017)
Child with fever for 3 days not responding to AB
hx
ex
mx.
Rash, maculopapular, no peeling of skin, doesn't feel rough
Dx IMN. Reassure. I didn't admit the pat
Red falgs, rest
Fb rash: pass

Case (21/4/2017)
A child who had sore throat 3 days ago and was given amoxicillin by another doctor is brought
now with worsening symptoms and rash all over his body. His mother is here and concerned....
Task:
1. Take history
2. Ask examination from examiner
3. Tel the most likely cause of the rash to mother
* Sore throat in 5 year after penicillins rash n generalised lymphadenopathy n spleen
enlargement. Task hx , pefe, dx,Dex
25-Asthma child
Cough in a 3-year-old child since last night.
Tasks
-History
-PEFE
-Diagnosis and other possible causes.

History
1-Cough questions (OCSTAR-1)
onset
-when did it start? Sudden or gradual? Is it constant or come and go? (Since last night and cough
all the night)
Character
-can you describe it for me?
-Is it barking or whooping? Doe he turn blue and vomit?
-dry or wet? (Dry)
Severity.
-does it affect his sleep or wake him up at night? (Disturbed sleep)
Timing
-does it occur at day or night mostly or both? (Usually at night)
Aggravating/relieving
-does anything make it better or worse?
1st episode
-is this the first time? (Had previous episode of coughing and runny nose when he was very
young)
-has he been diagnosed with asthma before?
-has he been hospitalized before due to any lung problem?

2-Associated symptoms questions


-any chest tightness or shortness of breath? (Had trouble playing too as he got SOB.)
-any fever or rash?
-any noisy sound in his breathing? (some wheeze)
-any runny nose? (yes)

3-Trigger or risk factors questions


-any recent flu infection? (yes)
-anyone in family or school has similar symptoms?
-any pets, carpets, or soft toys? (had carpets at home that they clean often, had cat and dog)
-does anyone smoke at home? (father smoke but not at home)
-does she take any medications?
-any family history of asthma, hay fever, eczema? (father had asthma in childhood, mother has
skin allergy)
4-BINDS
-Birth (The baby was born preterm )
-Immunisation (His immunization was up-to-date.)
-Feeding
-development (but his development had been uneventful.)
-social (He didn’t go to childcare and didn’t come in contact with anybody with URTI.) (There
were no stresses at home and they were a happy family).

Physical examination from examiner


1-General appearance
-alert, irritable, restless, drowsy
-pallor, cyanosis
-respiratory distress signs
*nasal flaring
*accessory muscle use
*subcostal or intercostal recessions
-DR PJL.
2-Vital signs especially RR + O2 saturation (temperature 37.8/38)
3-Growth chart (50% percentile)
4-ENT (sore throat, runny nose, ear discharge)
5-chest examination
inspection: deformity, chest movement
palpation: chest expansion, tracheal position
percussion: dullness.
Auscultation: air entry, breathing sounds, wheeze or crackles (bilateral expiratory wheezes.)
6-CVS, abdomen

Explanation
-from history and examination he most likely has asthma, which is a disease of the lung that
occurs because the airways are hypersensitive or overreacting to a stimuli causing airways
inflammation, swelling and narrowing. Leading to cough, SOB and noisy breathing.
-asthma can be triggered by several stimuli could be a viral infection like cold or flu which is the
most common trigger, other triggers are smoking, dust, animal fur or feather, pollen, medication
and stress, and usually patients with asthma has a history of asthma, hay fever or skin allergy in
their families.

-other possibilities could be a post viral cough, could be lung infection by a bug called
pneumonia, could be infection of the voice box, windpipe called croup but they are less likely
from history and examination findings.
Feedback 14-3-2018
Cough in a 3-year-old child since last night.
Tasks
-History
-PEFE
-Diagnosis and other possible causes.
2 min thinking
Asthma, pneumonia, URTI, cystic fibrosis causes of asthma especially smoking and allergy,
Medication, Family History Asthma Action plan Use of MDI/spacer.
After introduction, I asked about how was the baby at the moment.
His dad told he had been coughing all night. Not checked the fever. It was dry cough, with some
wheeze. There was no vomiting or blueness of his colour. Along with it he had a runny nose. It
happened usually at night and disturbed the sleep of the child.
On further inquiry he told his son had trouble playing too as he got SOB.
I asked if he had noticed any swelling in his body or wether he was diagnosed with any
congenital heart problem. The child had no skin allergy or rash and no food allergy. He wasn’t
troubled with any diarrhoea or repeated URTI.
They had carpets at home that they clean often. They had a cat and a dog. And his dad smoked.
On further questioning he told that he smoked out of the home and I told I’d arrange another
consultation regarding his smoking as it acted as an allergen and could aggravate his baby’s
condition. The mother had some skin allergy and the dad himself had asthma in his childhood.
The child had previous episode of coughing and runny nose when he was very young.
The child was active, eating well with no reduction in urine.
He didn’t go to childcare and didn’t come in contact with anybody with URTI.
The baby was born preterm but his development had been uneventful.
His immunisation was uptodate. There were no stresses at home and they were a happy family.

On Physical examination, I asked the examiner about general appearance which was normal. No
anaemia, dehydration or oedema. Growth chart was 5Oth percentile.
Vitals showed increased temperature of 37.8 degrees. No respiratory distress was seen..
There were bilateral expiratory wheezes. Percussion was fine.
Nothing significant on CVS examination.

I then got back to the role player and told about him Asthma. How airways constrict in response
to allergic stimuli.
Explained the mechanism and causes like allergy, infection, smoking, medicine, cold and
emotional stimuli.
Told about Asthma Action Plan and use of spacer though there wasn’t any. I concluded with
warning signs that the parents may recognise as an emergency. Told about other causes.
Emphasised on smoking cessation again and avoidance of allergens like carpets and pets.
Referred to the allergist and specialist.
Scenario …. Cough
Grade…. Pass Global score… 4
Key steps 1,2,3,4 Yes 5… No History…. 4 Choice of exam…. 4 Diag/ D/D…. 4
Feedback 14-3-2018
3.5yrs old child with cough all the night HX I think 5min(I asked everything about the cough and
then allergy hx it was positive in the family as well .
Pefe (fever 38, long expiratory phase with ronchi)
DX and DDX to the mother (dx was asthma triggered by viral infection)

Case (29/3/2017)
3 year old girl with cough for 2 days, but worse yesterday night (she could not sleep)
Tasks:
h/o,
pefe,
dds

2 mins: haemodynamic stability

Dds: asthma, croup, pertussis, heart failure, post viral, psychogenic

Vital signs- stable

Cough
- 2 days,
- first episode,
- dry,
- no barking or whooping nature (ask specifically),
- progressive,
- worse at night, wakes the child at night,
wheeze
- noisy breathing- wheeze positive
Previous urti 2 days back but that resolved with only the cough remaining,
No fever,
Sick child questions negative. Gaining weight normally, no cyanosis, no frequent urti
Contact h/o: positive,
BINDS: normal,
previous hospitalization when she was 6 months old for bronchioloitis.
Family h/o: positive asthma h/o in elder sibling, hay fever +ve
Risk factors: parental smoking –positive, carpets- positive, pets- positive, soft toys
No stress at home

PEFE: generalized wheeze with no signs of obvious respiratory distress

Diagnosis: asthma, talked about the risk factors, and gave other dds

Global score- 7
Feedback 7-9-2018
Station 12 – coughing – pass
3 and half year old - Running nose and cold for 2 weeks. and the mother concerns that her child
catched up cold .
TASK:
1. Focus history ( 4 minutes ) –
Asked about haemodynamically stable – examiner said – stable and proceed your tasks
Check – infection – fever, N&V,
Dry or wet cough - ,any characterized sound like whooping sound after excessive coughing , any
barking – NO .
Associated with ear pain , nasal discharge – yes – nasal clear discharge , but nil ear problem
Associated with SOB , complaint about chest pain – NO , any skin rashes and neck pain – NO
Eating and drinking – NORMAL , but slightly reduced these days
Pee and poo – no problem
Similar condition – nil
Travel history –nil
BIND - BIND – uneventful pregnancy , . but at 6 month of age – bronchiolitis , immunization –
UTD , not concern about growth and development .
Family - father has childhood asthma , other 2 elder siblings have eczema and asthma - the child
doesn’t have the habit of small toys into the nose ( when asked, mother was laughing , I don’t
know :P )
Social – nil smoker at home.

2. PEFE from the examiner –


GA – nil respiratory distress , alert and interested In surrounding
I ask drooling of saliva from at the corner of the mouth – NIL
Vitals – nil fever , RR – 28 , SPO2 –not available,
ENT – normal
Neck lymphadenopathy – NAD
Focus Respiratory examination :
bilateral wheezing on both lung fields,
Bronchial asthma (strong family history of asthma , wheezing on both lungs)

3. Explain Dx and DDx to the mother with reasons.


- main dx – Bronchial asthma ( could be acute exacerbation of bronchial asthma )
Other DDx – pneumonia ( may be from viral , but bacterial infection is least likely ) , allergic
rhinitis , foreign body ingestion ( least likely ) , the last but not least is – septicemia
Feedback – Cough – PASS
Global score – 4
Key step 1 to 4 – YES.
Key step 5 – NO
History – 5
Examination – 6
Dx and Dx - 4
26-Cerebellar ataxia
GP, 60 years old lady comes to see you for difficulty in walking. She has no history of injury to her
leg. She does not have diabetes mellitus and is not taking any medication. She drinks 5-6 bottles
of beer per day for many years.
Task:
-Perform relevant Neurological examination
-Explain the diagnosis
-Differential diagnosis

Approach
1-Gait
-ask patient to walk towards the wall or anything and asses walking (wide based gait +ve)
-tandem gait (+ve)
I am gonna start by asking you to walk for me. If you could start by just walking towards that
wall. And just turn, and if you can walk one foot in front of the other. Great thank you.
-Romberg test
ask patient to stand still and put feet together and hands by their side then assess ( patient will
be unsteady with open eyes so no need to ask for closing eyes)
Now I am gonna test your balance ok. If you can just put your arms down by your sides and put
your feet together. I am just gonna put my arms out if you feel unsteady.

2-Face
-speech
I am just gonna ask you to repeat a few words for me to assess your speech
british constitution, baby hypopotimus (look for slurred speech)
-Nystagmus
I am just gonna ask you to follow my finger with your eyes. Please keep your head as still as you
can.

3-upper limb
-Tone
I am just gonna move your arms around now, please let me take the weight and go floppy.
Any pain in your arms?
-coordination (+ve)
*Finger nose test
so now if you can touch my finger with your index finger then touch your nose and touch my
finger again and keep going back and forth. Good….good. And the same with the other hand.
*Dysdiodokinesia
can you place one of your hands on top of the other like this. Just turn the top hand over then
turn the back over again and keep doing like this as quick as you can. Same on the other hand.

4-Lower limbs
-sensation (light and deep touch, vibration and proprioception) examiner will say normal
-reflex
-coordination (heel shin test/ tapping test ) (+ve)
Diagnosis and differential diagnosis
-from history and examination most likely you have what we call cerebellar ataxia. This is a
condition that occurs when the cerebellum or the back of your brain get damaged or inflamed
causing difficulty in walking and lack of muscle coordination. In your case most likely because of
long-term heavy alcohol consumption that can affect the cerebellum causing some damage.

-other possibilities but less likely are


Vitamin B12 deficiency
trauma
stroke
space occupying lesion

Feedback 17-8-2018
UNSTEADINESS
A patient has presented to you with unsteady gait, she was consuming 5 to 6 drinks per day for a
long time.
Your task is to perform relevant neurological exam.
Tell the dd’s to patient
I started with greeting the pat and examiner and wipe
The started with walk there was typical broad based gate,and it looked like she is going to fall .
so I continued to support her throughout the time. Then heel walk which was not possible. Toe
walk was also abnormal, she was falling, tandom gait was also not normal.
Then romberg’s test was positive with open eyes . so I mentioned the examiner that I am not
doing with close eyes.
Then requested her to sit on chair and started with eye movement by making H. checked for
nystegmus. That was not present.
Finger nose test was positive. There was pastpointing.
Dysdiadochokinesia was present
I just checked the tone of upper limb
I forgot to do rebound phenomenon. Although I remembered at that time but just forgot while
asking patient to sit on chair.
Then I moved her to the couch and started doing heel shin knee test that was also obviously
abnormal
Toe tap teast was also positive. Pastpointing.
Then asked pat to cross her arms around chest and try to sit up from lying down she was unable.
Knee jerk she gave me pendular.
Thanked the patient
Then I explained the most probable diag as cerebellar ataxia and explained it in layman terms.
Most probably because you were using alcohol more than your body can tolerate so it can cause
some bad effects on your body like in your case it’s effecting the part at the back of ur brain, it
could be due to vit B12deficiency, SOL, injury to your backbone (spinal cord), stroke. But
explained why unlikely.
4 out of 5 key steps covered . I think maybe rebound phenomenon. Approach to pat 6
Choice and technique of exam ,organization and sequence 5 Accuracy of examination 6
Dds 5 Global 6
PD Cerebellar
GA
Gait Heel-toe + Romberg (if
ataxic)
Tremor – rest/fine/coarse/flap
Face - Glab Mask
Eye – Lid Lag & Exopth
Voice Hypotonia/monotone
UL – ITPRCS Cogwheel/hyperreflexia
Spesh – do all Piano/micro/brady Finger-nose/dysdiadoko
Thy Neck/swallow
CLD Jaundice/spider/Tits/abdo/nuts
Feedback 17-8-2018
Unsteadiness Task-PE and dd
All cerebellar sign positive with alcohol hx,followed Tally.pt was looking continuously to examiner
which was annoying for me
dd-cerebellar degeneration(told in lay term)
alcohol induced brain injury,traum, tumor,parkinsonism bt less likely

Feedback 30-5-2018
Station 16
‘Unsteadiness’:
~50yof c unsteady gait, some CVS risk fx.
Task: PE, Ddx
Feedback: Pass
O/E
I used tremor approach, bit of a confusing presentation at the door
GITPRCS
All the cerebellar findings were +ve
So summarized possibilities
And arranged stroke unit admission
Feedback 16-8-2018
Station 12.Cerebellum ataxia
GP, 60 years old lady comes to see you for difficulty in walking. She has no history of injury to her
leg. She does not have diabetes mellitus and is not taking any medication. She drinks 5-6 bottles
of beer per day for many years.
Task: 1. Perform the Neurological examination
2. Explain the diagnosis
3. Differential diagnosis

General appearance
Vital signs: normal
Walking: wide based gait, can't heel toe walking
Romberg test: swinging when stand feet together. Close eyes- swinging also.
Wearing trousers: pull up her trousers above knees.
Inspections: Examiner said skin is normal. I imaging muscle, joint, hair and nail: normal
Palpation: Temperature, pulsation, capillary refill: normal
When I assess Sensory, examiner said Normal.
I let patient sit on the bed.
I check muscle tone: normal. Power: knee & ankle- normal. Reflex: Examiner said tell me what is
your finding: Knees- normal, Ankle- not in left ankle joint but right (+). I rechecked left ankle
again and see a bit muscle contraction but I am not sure.
I checked heel shin test: Positive
I checked upper limbs: Finger nose test: past pointing & adiadochokinesia.  
I explained cerebellum problem and said differential diagnosis.
Grade: Pass
   Global score: 4
Key Steps: 1- Yes
2- No
3- Yes
  4- Yes
5- Yes
Approach to patient: 4
Choice & Technique of examination, organization and sequence: 4
Accuracy of examination: 3
Diagnosis/ Differential diagnosis: 4
27-PSVT
Case 1
GP, 58 years old lady came to see you for dizziness and palpitation.
Task:
-take history
-PEFE card will be given
-Ask ECG from examiner
-Give diagnose and differential diagnosis

Case 2
GP, patient coming with palpitations and dizziness.
Tasks
-history
-PE from examiner for which he gives a card
-Ask for investigation
-Interpret investigation to examiner
-Explain diagnosis and ddx to patient with reasons and also tell risk factors.

History
1-Palpitation questions
Onset and timing
-can you tell me more?
-do you have it now? (Yes)
-for how long have you had it?
-is this the first time? (No another attack happened 3 days ago)
-how long do each episode last?
-has it started suddenly or stopped suddenly?
-what were you doing when it started? (Driving)
Site and character
-can you tap it on the table for me ? (She refused)
-does the heart seem fast or slow? Is it faster than ever goes at any time like on exercise?
-does it seem regular or irregular?
Aggravating and relieving.
-does anything bring on the palpitations? Like coffee or sleep deprivation.
-anything makes it better? Like holding your breath or deep breathing.

2-Associated symptoms
-have you ever lost you consciousness? How long for?
-any sweating, nausea or vomiting, chest pain, SOB?
-any headache, Blurring of vision, light-headedness?
-any weakness or numbness?
3-Differential diagnosis (TIHA)
-thyroid (weather preference and bowels habit)
-infections (fever)
-hypoglycemia (skipping meal)
-anaemia (bleeding)

4-General (imp to know risk)


-medications
-PMH (heart failure or attack, hypertension, DM)
-smoking, alcohol, coffee (Smoking x many years 3 cig/day. No alcohol. Coffee drinking 5 cups a
day for many years)
-family history and stress at work or home (stress work +ve)

Risk factors or cause


1-thyroids
2-infection
3-anaemia
4-medications
5-hypertension, heart disease
6-electrolyte imbalance
7-alcohol, coffee, stress
8-family history

Physical examination findings from examiner


1-general appearance
alert, anxious, sweaty, dyspnea, pallor, cyanosis
2-Vital signs
pulse and regularity, BP and postural drop, RR, Temp
3-neck
thyroid, JVP, carotid bruit
4-CVS
apex beat, heart sounds and murmurs
5-Chest
dullness, air entry, breathing sounds, crackles
6-abdomen
7-office tests
UDT, BSL , ECG

In exam will be card but you can ask office tests


GA: restless, irritable BMI: 31
T: 37C HR: 150/min, regular BP:140/80
No signs of heart failure
Lungs: clear
When I asked ECG, examiner gave very similar this ECG and interpret to examiner- I see Heart
rate is 150/mins, P wave (+), RR interval is regular, loss of R wave in II, III, aVF, V1 to V6 is no
abnormality (V1 is quite normal unlike this sample ECG).

Explanation
-most likely you have a condition called paroxysmal supra ventricular tachycardia. Which means
that your heart is beating too fast.
-normally we have the main connection or generator in the heart called SA node, which
generates the impulse that can cause the heart to beat.
-in your case your heart beat is not generated from the normal area that is why it is very fast.
The pathway in the pulse conduction is not normal that why you have these symptoms.
-in your case could be triggered by excessive coffee drinking and stress.

-other possibilities are a problem in the thyroid or the gland in front of the neck. Reduce sugar
level in the blood, anaemia or reduce in iron rich protein, mineral salts or electrolyte imbalance.
any medications but less likely.
Feedback 16-8-2018
PSVT or atrial flutter
GP, 58 years old lady came to see you for dizziness and palpitation. Sorry, Forget other given
facts.
Task:
-take history
-PEFE card will be given
-Ask ECG from examiner
-Give diagnose and differential diagnosis

I checked vital signs first (examiner said stick to your task).


I ask to tap on table according to her heart beat but she refused to do it. Not first time, last 3
days happen again. She has attack during driving. Happen suddenly. Associated with mild
sweating. No aggrevating & reliving factors. It comes and goes. No loss of conscious, no nausea,
no vomiting, no fever, no change in weather preference, no lumps & bumps, no loss of wt & loss
of appetite, no cough with sputum, no noisy breathing, no one comment pale. No history of
travel. No family history of ht problem & sudden death due to ht attack.
Menopause for 8-10 years. No menopausal symptoms.
Mild stress (+) at work (?accountant), No financial problem and is a happy family. Smoking x
many years 3 cig/day. No alcohol. Coffee drinking 5 cups a day for many years. No recreational
drugs. Not easily anxious person. No chronic medical & surgical problem.
PEFE card:
GA: restless, irritable BMI: 31
T: 37C HR: 150/min, regular BP:140/80
No signs of heart failure Lungs: clear  

When I asked ECG, examiner gave very similar this ECG and interpret to examiner- I see Heart
rate is 150/mins, P wave (+), RR interval is regular, loss of R wave in II, III, aVF, V1 to V6 is no
abnormality (V1 is quite normal unlike this sample ECG).

Explain the patient: Her condition is mostly due to excessive coffee drinking & stress. Other
possibilities are IHD, Thyroid, Anaemia, infection, lung p/b, anxiety, menopause (patient refused
it that she menopause for 8-10 years, so I said it is less likely)  
Grade: Pass
Global score: 4 Key Steps: 1- Yes 2- Yes 3- No 4- Yes
Approach to patient: 4 History: 4
Interpretation of investigation: 4
Diagnosis/ Differential diagnosis: 4
Case (11/2/2017)
You are a junior HMO in ED, a man came in with complains of thumping/racing heart and
giddiness, started today.
Task:
Hx from patient
PE from examiner
explain dx to ptn (can’t remember if management was there)

I went in and saw a man sitting comfortably on the bed, semireclined.


Introduced myself and ask him how is he feeling. asked examiner if ptn hemodynamically stable,
and he said something like ‘he is how you see him to be’.
He looked comfy so I proceeded with history taking, he had it twice before, asked him to tap the
beat on his thigh, he does have sweating when the dizzy & heart thumping spell happens,
currently having it. no other symptoms i think. he didn’t associate onset of symptoms with
anything he did. No significant past history.
Vital signs: all i can remember is HR 150, regular. No other significant PE findings.
ECG- pic of SVT
Explained to ptn, ‘ can u see this, this is called an ECG. This was taken when the nurses put
those leads on ur chest earlier.
These squiggly lines are the result of the electrical impulses conducted in your heart. you may
know, the heart has 4 chambers and somewhere there there are nodes or points that generate
these impulses and cause the heart to beat. Something seems to be wrong in those points and
that is why the impulses are not beating correctly and causing your heart to thump funnily and
causing your symptoms.
Does what I say make sense? Do you have any questions?’
The bell hadn't rang so I just said that ‘I’m sorry if my explanation is so basic, I’m so junior. But
you are in good hands, you will be on a cardiac monitor and we will keep an eye on you while
the cardiology registrar gets here. They will be able to explain to you better regarding your issue
and manage you accordingly. If you feel worse or need anything, call out to anyone here, dont
worry..
This was my first station and I did stutter a little bit because I kept looking back at the question
on the screen to double check my tasks. didnt want to miss a task.
Feedback: Palpitations, Pass, Global score 4
Case (5/10/2017)
Palpitations and dizziness

Pass

27 years old man comes in with Palpitation and Dizziness. When he came in, his pulse rate was
150 and did an ECG.

Task: History from patient

Physical examination card from examiner

Ask ECG from examiner and explain ECG to examiner

Dx and DDx to patient with reasons

Introduced myself and told the patient who was acting breathing heavily on the chair that i
would like to check his vitals as he was having SOB. Then i asked examiner about the vitals ( need
to ask specifically). Vitals were normal (T,P,BP,Spo2,RR). Pulse was 90 when i asked. He said he
had this racing of heart often and feels SOB and dizzy whenever he has racing heart. No
orthopnea and no aggravating and relieving factors for SOB and Palpitation. No chest pain, no leg
swelling. No fever, rash, no symptoms of thyroid. 5 cups of coffee a day and 4 or 5 beers a day.
No past medical and positive family history. PEFE card showed everything normal, no murmur,
no bruit. ECG showed SVT with 150/ min. Explained to examiner about ECG ( 12 lead ecg of
name of pt, was done because of palpitaion and dizziness, showed rate, rthym, any signs of MI
and hypertrophy, axis). Told the examiner that this is SVT. And explained the pt.

Dx- SVT , ( could be due to excessive coffee intake, alcohol intake, could be in condition like pre
existing heart disease (valvular ht, heart failure), thyroid and electrolyte imbalance. Forgot to
mention paroxysmal.
Covered 2 key steps out of 4
Approach 4
History 4
Interpretation of investigation 4
Dx and DDx 5
Global score 4
Feedback 26-10-2018 Passed (GS - 5)
You are working in hospital, a Man present with dizziness, palpitation.
TASK: HX, ask investigation which may be required, Dx & other possible causes and risk factors
HX:

 Symptoms details

 DDx Qs (arrhythmia, underlying heart diseases, DM, thyroid, hypertension, medication


side effect),

 SADMA & stress at work or home.

Ask Invx if required


To examiner: ask ECG (gave me a ECG combination of pieces of strips, not a full 12 lead ECG. I
couldn’t find same one but ECG was very clearly seen. It’s such an example just to get an idea.)

Interpret ECG to examiner


Rate is about 150bpm. It’s regular rhythm. I can see p wave. QRS complex is narrowed. There is
no feature of MI (ST depression or elevation), no feature of BBB. To me, this ECG is consistent
with SVT.

Dx & DDx:

explain SVT as the most possible cause.

There could be other types of heart problems (A flutter or AF making your heart beat fast and
irregular), or underlying heart diseases such as MI, metabolic causes such as DM, Thyroid, or
also could be due to medication side effect or stress. Those are unlikely as we confirmed your Dx
with ECG. It’s emergency but its manageable, so I will talk with my senior and treat you. (There is
no Mx in the task
28-Hydrocele Counselling
Young male, left sided scrotal swelling, physical examination was done . size given, can get above
the swelling, trans illumination test positive , left testis is not palpable , right testis and scrotum
normal.
Task:
-explain the significances of examination results to the patient(no further history and physical
examination)
-Diagnosis and ddx
-investigations
-Further management

Counselling
1-Approach
-Greet the patient
-How do you feel at the moment? Any pain? do you have any specific concerns?

2-Explain possibilities of scrotal swelling


-all right so I finished examining you let us discuss the findings. I will draw a diagram to make it
more clearly and if you have, any confusion or question please let me know?

-so there are several causes why you have scrotal swelling in the left side:
*lipoma which is accumulation of fat.
*varicocele which is prominent dilated veins
*epidydmal cyst which is a fluid filled sac that might contain dead sperms.
*Hematocele or accumulation of blood.
*hydrocele or accumulation of clear fluid.
*hernia or weakness in the tummy wall causing tummy content to get out f position causing
swelling.
*epididmoorchitis or infection of the testis.

3-explain the most likely diagnosis


-from examination I could get above the swelling and this means that the swelling is not coming
from above that is why hernia is unlikely.

-Trans illumination test which is when I put the torch behind the scrotum and the swelling
glowed up. And this means that the content could be a clear fluid. That is why Hematocele,
varicocele or lipoma are less likely in your case.

-so from history and examination you most likely have a condition called hydrocele
which is a swelling in the scrotum caused by build up of fluid around the testis. It can happen at
any age.
The cause is could be unknown, which we call primary hydrocele or it can follow trauma,
infection or testicular cancer, which we call secondary hydrocele.
That is why I need to arrange for some Investigations to rule out any underlying testicular
problems.
4-Investigation
-with your consent we need to take 1st pass urine sample for PCR to rule out chlamydia and
gonorrhea the STI.
Also, need to arrange for ultrasound of the scrotum to estimate the amount of fluid and address
the state of the testis.
These along with basic blood work, urine M&CS and tumor markers.

5-Management
-management will depend on the cause.
-if infection we will treat you accordingly.
-if cancer then we will refer you to specialist for possible surgery.
-if all results come back normal then it means that you most likely have primary hydrocele.
So if it is troublesome then treatment is usually by simple operation or aspiration of the fluid
along with injection a sclerosant agent to reduce recurrence.
if it is not troublesome then treatment is usually a conservative one by observation and some
advice like wearing comfortable underwear, scrotal support, avoidance of trauma or contact
sport.

6-reading materials, red flag, review, refer

*In this case try to check understanding and every time and again ask if he is with you so far
Feedback 6-6-2018
Station 1 scrotal swelling hydrocoele– pass
Young male, left sided scrotal swelling , physical examination was done . size given, can get above
the swelling, trans illumination test positive , left testis is not palpable , right testis and scrotum
normal.
Task: explain the significances of examination results to the patient( no further
history and physical examination )

Dx and ddx

investigations

Further management
Greet patient. Intro, Any pain and discomfort currently? Any specific concern at this moment?
Drew diagram and explained possible causes of scrotal swelling in layman term. starting from
skin - lipoma, then varicocele, epididymal cyst, hernia, hydrocoele, chylocoele, haematocoele.
Epididymo ochitis.
can get above the swelling means the swelling is not coming down from above, can exclude
hernia.
Transillumination positive so, the content could be clear fluid. Not chylocoele, not
haematocoele, unlikely varicocele. So, I suspect it is hydrocele.
Causes of hydrocele : infection, congenital, testicular ca. Testis is not palpable on examination ,
could be because of the large amt of fluid around it., but we need to rule out underlying
testicular problem. Will do investigation to check it.
Inv to confirm dx and to rule out other causes: Ultrasound, STI screening with patient’s consent,
tu maker, urine cx.
Mx: will refer to specialist. Tx options : conservative if the swelling is small and no symptoms,
wear comfortable underwear, scrotal support, avoid trauma, contact sport. If large or
symptomatic, aspiration or operation.
reading material and red flags.

Feedback 5-4-2018
Scrotal swelling You are a GP and a 32-year-old man comes to your clinic complaining of right
scrotal swelling. the PE was given. Hydrocele
Task:
1. Exapling the +ve finding to the pte
2. Dx and Dds
3. Inmediet and long term management

Recall 5-4-2018
Hydrocele
GP, 25 year old male c/o a lump in right scrotum & saw one of ur colleagues last week. He did
examination & it showed can get above the swelling, soft in consistency, testis
Cannot be palpable on rt side, transillumination test +ve.
Task: Explain the exam findings or Dx to pt
Ddx with reasons
Further mgt.
Case (27/4/2017)
Hydrocele counselling
20 yr old c/o painless swelling on testits, no urinary symptoms or other problems. O/E, swelling
on rt testis 6 cm, Non tender, testis can't be palpated, transallumination test + , no other
findings.
Task
Explain the condition and significance,
Expalin the differential diagnosis and possible causes of the condition.
Explain ur initial investigation with pt.
Further mx.

I introduced myself and asked him that how r u feeling today, he said fine.
Any pain, no.
I exlpain the conditon to pt that I have found some features of fluid collection around the rt
testis, we called it hydrocele. I explained the significance of this condition like, it can be
discomfort when the size is getting bigger and risk of trauma. It could be due to several
conditions like inflammation of testis, infection like uti, sti, And could be due to nasty conditon. I
forget to say about hernia in my ddx. i expalined the possible causes again like uti, sti or nasty
condition.

Firstly, I would like to do USG exmanation to check the condition. I am a bit concerned of nasty.
And further mx will depend on the findings.
If it is nasty condition, u need to be mx with team approach, like surgeon, radiologist and
oncologist. U may need more invx to support the diagnosis like ct scan, tumor markers. If it is
nasty condition, specialist will do operation to remove the testis via ur groin area. However, if it
is just benign swelling, we can remove the fluid out for u. pt asked me what can be the most
likely condition.

PF (examiner didn't allow to ask a single question. Stop me even when i asked any LOW, LOA.)

I just explained the pt that it is hard to tell at the moment. But, I am a bit concerned about the
nasty condition. Reassure the pt and check understanding. 6R.

FB-PASSED, OVERALL-4, APPROACH-4, DDX-4, CHOICE OF IVX-4, MX-4

Case (27/4/2017)
24 y/o man came to GP.has had a mass in his RT testis .your colleague has examined and found
that there is a 3cm mass, tense, you can reach the neck of the scrutom but testis and epididymis
are not palpable. The mass has been translucent on transillumination. The left testis is normal.

Task: talk about diagnosis with the patient.


Feedback 10-11-2018
Testicular Lump
I don’t know how to start this as no need hx just explained so I bit confused how to start as
examiner said no history when I try to give px some question but then what I did draw penis and
testicle and explained that swollen that you have due to there is some fluid in your scrotal bag
that’s why its hard for me to palpate you right testicle but I can feel easily left testicle and it was
normal, there is some causes why you having some accumulation fluid there could be due to
some infection like sexual transmitted infection , could be asscoited with hernia as well , could
be there is mass that grow in your testicle , or might be we just don’t know the cause .
So what I would do now , with your consent I will arrange screening , for STI , check your blood
wether any infection or not as well , I will chek your urine , and do some tumor marker and last
once send you for USG to make sure this is fluid or there is any lump we as well.

Once I got your result I will send you to specialist , most of them specialist will drain this fluid or
with conservative it will gone by it self
Will see 1st what your Ix result
You have any concern/question for me ? what els should I know ( yeah again this answer in 16 px
)
Feedback testicular lump / passed
Approach/dd/ix/mx pln : 2/4/4/2
29-Thalassemia in pregnancy
You are in Gp when Jane an 8 weeks pregnant comes to you to get the results of her blood tests
that you had ordered for her during the last visit.
FBC shows low Hg and low MCV
iron studies are normal
rest blood tests normal
blood group id AB +ve
tasks
take further history
tell diagnosis to patient
management

Differential diagnosis
1-Iron deficiency anemia
2-thallassemia
3-chronic disease

History
it nice to see you again I’ve got your blood tests results to discuss with you but before that can I
just ask you a few questions in order to get a better view for the diagnosis.
1-anaemia questions
-do you feel dizzy, tired?
-do you have shortness of breath, funny racing of the heart or chest pain?
-have you ever had any of these symptoms before?

2-current pregnancy questions


-how is your pregnancy so far
-do you have tummy pain, vaginal bleeding or discharge?

3-General questions
-can you tell me briefly about your diet?
-do you exercise regularly?
-do you smoke? Drink alcohol? Take any medications?
-do you have enough support from your partner?
-any past medical or surgical illnesses?

4-thallassemia questions (all key issues)


-have you had history of miscarriage?
-can I know please which country are you from? (Italy)
-and where your partner is originally from? (Italy)
-does your partner have any symptom?
-any family history of anaemia? (father looks pale but no reason)
-do you take folic acid?
-blood group? (if not with the results)
Interpreting the results and explain the diagnosis and ddx
-look Jane I’ve got the results of your blood tests with me let’s see it together.
The blood picture showing a low hemoglobin level and the size of the red blood cells is also
small. Normally your blood has 3 types of cells the RBC which contain an iron rich protein called
hemoglobin which enables these cells to carry and deliver oxygen to different parts of the your
body. Low hg means that you are anemic.
You also have White blood cells which helps fighting infections and also platelets that helps
blood clotting. Your WCC and platelets are normal.

-Usually this sort of blood picture is seen when someone is deficient in iron but we have already
ordered iron studies, which revealed to be normal. So it is unlikely that low hg id caused by Iron
deficiency.
Another reason could be due to any chronic illnesses but they are unlikely because your general
health is good.

-So what I am suspecting is a condition called thalassemia have heard about it?
Condition and cause
-This is a group of inherited blood disorder where there is a defect in production of hemoglobin
so it happens when your gene becomes mutated or permanently altered affecting the body’s
ability to make health hemoglobin.
-This is a lifelong condition and most common in Mediterranean, Middle East, Asian and African
people.

Clinical feature and complication


-Let me assure that you have a minor form of the disease in which you have no symptoms except
sometime mild anemia that could become more prominent during the latter half of the 2 nd
trimester and early 3rd trimester, this is because pregnancy demands higher rate of RBC
production.
-Normally baby won’t have any birth defects because of this but depends on the genetic
constitution the baby might get thalassemia.

Management
-first I would like to confirm thalassemia by another test we call hg electrophoresis for which I
would like to refer you to the specialist hematologist.
-Once it is confirmed, your partner also needs to be tested for thalassemia and DNA gene
testing also needs to be done.

-if you alone has got thalassemia the baby has a 50% chance of having the minor form and 50%
not having thalassemia.
if both you and your partner have thalassemia then there is 25% chance that you baby doesn’t
have thalassemia, 50% a minor form and 25% got the disease or major form.
- I will refer you for genetic counselling as well (CVS, amniocentesis, fetal blood sampling)

-the rest of your pregnancy will be monitored in high risk pregnancy clinic.

- I’m gonna start you on folic acid 5mg/day and it will be given throughout the pregnancy.

-you will be followed up with frequent blood check, US etc…

-I will give you reading materials regarding Thalassemia, and I will arrange a review with you
when the results of the hemoglobin electrophoresis come in.

Feedback 19-7-2018
STATION 15 PASS(all Key steps Yes score 6,6,6,7)
Thalassemia minor in pregnancy

24 years old primary gravida found to have Hb 9 on first antenatal visit and u ordered iron profile
which came out to be normal ,now she is here for her reports,

Task , further focused Hx , Most likely Dx, Further Investigation, council

Young girl sitting I asked about planned pregnancy and congratulated her, then asked abt how
pregnancy going, all anemia questions ( intake/loss/origin/ch disease) Fhx and sadma.

Then I explained Hb , Anemia , Iron ,thalassemia minor and major .

Advised Hb electrophoresis , partner testing and if positive chances in baby and diagnosis and
all.
Case (5/10/2017)
First trimester complication
Fail
9 weeks pregnant lady who is 24 yrs old comes back to be reviewed by you after the antenatal
blood tests. All the tests are normal except Hb 90, MCV 65. You ordered Iron studies which came
back normal.
Task:
Brief history
Explain what investigations you want to do to patient
Explain Dx and DDx to patient and Management

Introduced myself and reassured confidentiality. Asked if it is a planned pregnancy and she said
yes and congratulated her. She said this is her first pregnancy. Coping well with the pregnancy
and good support from her partner. No dizziness, SOB, Chest pain. Normal period before
pregnant and no history of heavy period, as well as no bleeding disorder. Diet is balanced and
she is not on any special diet and no family history of coeliac disease and no family history of low
blood count either. Italy descent and her partner is from same country too. Then, i explained her
about blood test. ( In our blood, there's 3 main blood cells, RBC , WBC and platelet. For now, i
will focus on RBC to explain to u. It helps carrying oxygen to various parts of the body and it has
protein called Haemoglobin which helps carrying oxygen. In you, there is low Hb count and it
also shows cells are small. We call it Microcytic Anaemia and usually most likely due to iron
deficiency but in ur case, iron studies is normal. So, I am suspecting u have the condition called
Thalassemia, which is a genetic condition in which u have small cells with low blood count. I will
arrange a blood test called Hb electrophoresis to confirm the diagnosis, and Patient asked me
what is that and i explained to her. Then, i asked her that i would like to see her partner as well
and would like to arrange some blood tests on him as well to rule out this condition. I explained
to her possible other differential dx with reasons ( iron def, coeliac, bleeding disorder and
multiple very close pregnancies). I told her that i will refer her to high risk clinic where she will b
followed up very closely by specialists. I offered her reading materials and told her red flags. I
told her i will start her on folic acid and she said she is already on folic acid.

I totally forgot to mention about detail antenatal care ( 8 week dating scan, Sweet drink test at
26 wks, USG at 28 and 32 wks, Bug test at 36 wks ).

Covered 3 key steps out of 5


History 3
Dx and DDx 4
Choice of InVx 4
Global score 3
I didnt ask details about 5 P as the task mentions brief history. I forgot to ask about LMP and
blood group.
Feedback 25-10-2018
Scenario: 1st trimester complication
Stem.

Young primi, returned for the blood reports. U ordered iron profile as her hb was low. Iron profile
comes out to be normal.

Tasks

Further relevant hx

explain results

further inx after greeting took a short hx that’s why got (3 in hx) thought relevant hx should be
short. Well then explained her the results. Explained her whats hb whats mcv n what is the
significance of them being low -> anemia i.e hypo micro anemia there are 2 most common
causes of this type of anemia, 1 is iron but we did ur iron profile and its normal which means
theres some thing else going wrong, as u r of Greek ethnicity also ur dad had some form of
anemia(she told me this when i asked anyone in family having anemia or blood transfusions) it
shows most likely this is something we call thalaseemia. Have you ever heard of it before ( NO)

explained thalaseemia . and that she could be minor type need to do hb electrophoresis. And
later if it comes out to be positive will test parter as well. Explained risk of having a baby with
thalassemia, n that we can do some tests to check baby n if major dx its incompatible with life
and she can decide to abort but don’t need to worry at this point as u don’t have any symptoms I
will see u again after hb electrophoresis and then will go from there, if needed willl refer u to
specialist, bell rang.

Grade: pass

GS:4

3/5 key steps covered

Hx: 3

Dx/ddx:6

Choice of inx: 6
30-Common peroneal nerve injury
Young man who is cherry picker (strawberry picker) presented to you complaining of walking
difficulty and tingling and numbness of the right leg
Tasks
1-perform examination telling the examiner what you are doing during the examination
2-dx and ddx to patient

-Remember GIT PR CS

-I entered the room greeted the examiner, checked my ID informing me about the presence of
another paper and screen of same stem outside and tell me the patient name.
there was a male patient inside lying on the bed.
On the table at the end of the bed there was these instruments: tendon hammer, toothpick and
cotton wool

-Hi --- my name is dr--- I’m one of the doctors here. I understand that you are complaining of
tingling and numbness of your right leg.
any pain at the moment---no
I would like to examine your lower limbs in order to uncover the nature of the problem this
would involve me having a look, testing the tone, power and sensation and watching you walking
would that be alright

-Washed my hands

1-Gait
-assess walking
-turning
-tandem gait (one foot in front of the other)
-walking on toes
-walking on heels

-now if you could come off the bed and walk in front of me?
Patient asked where should I walk? toward that wall please
during walking he was lifting his right leg up and down (high stepping gait)
-and now turn
-now walk one foot in front of the other like this.
-Now if you could walk on your toes (no problem)
-and now on your heels
the patient was unable to walk using the right leg and was about to fall. I was standing away in
front of the patient so I apologized and moved next to him.
-said to Examiner: there is high stepping gait of the right leg and right foot drop
2-Inspection (SWIFT)
-scars
-wasting
involuntary movement
-fasciculation
-tremor

-Now if you could lie on the bed for me


-can you lift your shorts up (examiner helped him exposed half of his thighs)
-I am just gonna start by having a look (no sacrs, wasting, involuntary movements, fasciculation
or tremor).
No findings on inspection

3-Tone
-leg roll
-knee lift
-knee clonus
-ankle clonus

-I’m just gonna assess the tone (including both rolling legs + knee and ankle clonus) when I just
start checking the tone Examiner said tone is normal on both lower limbs

4-Power
-Hip (4 movements)
-knee (2 movements)
-ankle (2 movements)
-subtalar (2 movements)

-now I’m gonna assess the power.


if you push your thighs up against my hand and don’t let me pushing them down--- examiner
said normal
if you could push down against my hands --- examiner normal
if you could push outwards—examiner normal
and inwards ---examiner normal.
-Now if you could bend your legs and kick against my hands –examiner normal
if you could pull them towards your bottom and don’t let me straighten your legs---examiner
normal.
-Push your feet up against my hands ( unable to push with right foot)
and now push down—it was normal examiner did not say anything
-push outwards (unable to push with right foot)
push inwards--- normal (examiner did not say anything.)

-Push your big toe up--- normal examiner did not say anything.

Said to Examiner: there is weak dorsiflexion and eversion of the right foot
5-reflexes
-knee
-ankle
-plantar

-I am just gonna tap on your legs using this hammer would that be alright?
-both ankle reflexes are present
-forget to do plantar reflex
in the exam all reflexes should be present

6-Coordination
-heel-shin test
-plantar tap

-now if you could lift your left heel and put it on your right knee and run it down your right leg
and keep going like this.
-if you could do the same with the right foot
examiner said coordination is normal

7-sensation
-cotton wool and pinprick
-with this cotton wool I am gonna assess the sensation of your lower limbs so if you could close
your eyes and let me know when you feel it
-repeated the same with tooth pick.
I tested all the dermatomes
L2= upper thig
L3=medial side of knee
L4= medial side of leg
L5= medial side of big toe
S1= sole or little toe
said to Examiner there is loss of sensation on the dorsum of the right foot; the level of L4-L5
dermatome

-I did not do vibration or proprioception (I do not know if there is tuning fork on the table)

8-Wash hands

Task 2
Thank you --- I finished my examination
from the history and examination you most likely have an injury to a nerve supplying the right
leg and foot that is why you have difficulty walking, weakness of movement of your foot upward
and outward and loss of sensation on the back of your right foot.
Most likely, this is a common peroneal nerve injury

Other DDx from the following feedback (I did not mention any ddx in my exam)
-sciatica
-radiculopathy
-nerve entrapment
Another Feedback 14-3-2018
Leg weakness in a cherry picker.
Perform PE and give diagnosis and D/Ds.
2 min thinking; D/D, sciatica, Peroneal nerve injury, cerebellar disease, DM or alcoholic
neuropathy.
Inspection, Palpation, tone, power, reflexes, Sensations ( In this station I ran out of time towards
the end but I kept giving running commentary during examination to the examiner and I think
that’s what saved me)

After introduction I took the role player’s consent for examination. She was a nice lady and
cooperated well. I asked her if she could walk. There was anatalgic gate/ foot drop. She could
walk on toes but couldn’t walk on Heel. So I added that loss of heel walk may be because of L4,
L5 nerve impingement or injury.
On inspection she had no SWIFT no muscle loss. There was some skin thickening in popliteal
fossa and some moles on legs which I commented about.

I did rest of the exam with her in the lying position and inspected feet for any fungal inspection,
trauma or skin changes and palpated for change in temperature and pulses in Dorsalis Pedis and
Posterior Tibial artery.
I told her I’ll feel for her leg next, she can stop me if she felt uncomfortable.
No tenderness or muscular atrophy was there.
Tone was normal. Power was normal till I reached the foot. There was loss of dorsiflexion and
eversion of right foot. I commented about that this further supports the loss of L4 n L5 nerve
function.
Knee reflex was normal but I couldn’t elicit ankle reflex. So I still commented on that although all
other was normal but this correlates with S1 loss (the thing is to be honest and tell what you
exactly feel or do, like a real doctor patient scenario).
Sensation was lost on the dorsum of the foot. The examiner told me my time was running out for
the other tasks. So I hurried to do Tinel test for Common peroneal nerve that was negative. I
couldn’t do SLR.
In last few seconds I just said it could be due to injury of peroneal nerve, or sciatica or
radiculopathy or nerve entrapment.

Scenario… Walking difficulty


Grade… Pass
Global score… 4
Key steps 1,2,3,4,5… yes
Approach to patient…. 4
Choice of exam…. 4
Accuracy of exam…. 4
Diag/ D/D.. 4
31-Mini mental state examination
Case 1
a lady in 30s who is alcoholic etc and encountering problem remembering things
Tasks
~ Explain the test and why you are doing it
~ perform MMSE
~ DX and ddx

Case 2
65 year old lady Taking excrssive alcohol consumption over 20 years, but reduced recently 2
weeks ago. Impaired Orientation
TASK
1. Explain what u are going to do - Perform MMSE (not need to calculate scores) –
2. explain possible reasons to the patient with reasons .

Case 3
middle age female, diabetic, hypertension, coming with poor glyecemic
control, blood sugar is 31 now, on questioning he says he forgets to take
his medications, heavy alcoholic
Tasks
Perform MMSE (MMSE chart given, a pen and a watch is there on the table)
You don’t have to calculate the score.
Explain the results to the patient

Case 4
40 yrs old lady with known DM on Metformin, comes to your GP as she forget to take pills very
often. She had history of alcohol abuse and now she is cutting down alcohol consumption. You
decide to do MMSE on her.
Task:
Explain to her about MMSE and why you are doing it on her.
Perform MMSE.
Explain abnormal findings to patient
Explain what is happening to her

Explain MMSE
-Good afternoon Mr Smith. My name is dr--- I am one of the doctor here. how are you today.
-I appreciate your coming. I can see that you have become a bit forgetful over the last few
months is that right?
-Today I need to perform a simple test called mini metal state examination. This is usually a
screening test not diagnostic in order to assess your memory function.
-During the test, I am going to ask you a few questions and I will ask you to follow some simple
commands. Do not worry I will guide you throughout the test.
-the whole test will take approximately 5 minutes.
-if you feel uncomfortable or you have any questions please let me know?

MMSE (chart will be given so you need only to familiarize no to memorize)


1-Orientation (in chronological order)
-what is the year?
-what is the season?
-what is the month?
-what is the day of the week?
-what is the date of today?
-where are you right now? In which state? Which city? Which suburb? Which street? (can ask
the street and suburb he live)

2-Registration (in the exam this will be affected)


-now I am gonna say three things and I want you to repeat after me? All right?
Table coin apple

(repeat only two times)

-I will say them once more, please keep them in mind because I will ask you about them later.

3-attention/ concentration (in case 3 and 4 sometime attention is affected as well)


-can you start at 100 and count backward by 7
-can you spell world backwards

In the exam choose only one if not affected no need to tell the other.

4-Recall (in the exam recall is affected)


-if you could tell me the 3 words that I have said earlier.

5-Language
-naming objects
(show 2 objects and ask to name)
-repetition
(can you say this sentence for me; NO IFS ANDS OR BUTS)
-3 Stage command
if you could take this paper with your right hand
fold it in half
place it in your lap.
-reading and obeying
if you could read this sentence (CLOSE YOUR EYES) and do it.
-sentence writing
can you write a sentence for me?

6-Construction
-can you look at this the card and draw this shape on the paper.
Diagnosis and differentials
-Thank you
-from examination, I found that you are able to perform correctly the tests for orientation,
calculation, language and construction. However, it seems that you have problem with
registration or learning new things and recall or remembering things. This indicate that you most
likely have a short-term memory loss.
-causes could be due to chronic alcohol use causing acquired brain injury. Could also be due to
vitamin B12 deficiency, hypothyroidism; when gland in front of the neck cannot produce enough
hormones or head trauma.
-in case of DM you can add could be due to mini stroke as the blood sugar not well controlled.

Feedback 21-2-2108
CASE 14: MMSE IN ALCHOL INDUCED BRAIN DAMAGE- PASS
Middle aged woman who often forget things, takes around 8-10 standard drinks per day for
years.
Tasks: Do MMSE and explain the findings to patient
Explained this is a test with set of questionnaries takes 5-minutes , I will be guiding you once
started. This helps to assess various brain functions. The mmse chart was in table , followed the
same pattern, patient had difficulty in registration and recall and she seemed to be sleepy , when
doing the task close your eyes- she started to sleep once closing eyes. After completing the task ,
explained each question what I have asseed like you are well oriented, language and
communication is good, only you have difficulty in registering and recalling. mentioned can be
due to alcohol, family history, ageing etc..this is a screening test and to find the exact cause I will
refer for further tests. Arrange other investigations for alchol damage-lft,etc. Assess the patients
level of understanding quite often about what you are telling- as she is drowsy . Make sure she is
feeling good throughout.

Feedback 5-7-2018
6 middle age female, diabetic, hypertension, coming with poor glyecemic
control, blood sugar is 31 now, on questioning he says he forgets to take
his medications, heavy alcoholic
Tasks
Perform MMSE (MMSE chart given, a pen and a watch is there on the table)
You don’t have to calculate the score.
Explain the results to the patient
Patient was sitting on the chair. Chart is there. I explained the test then
she asked why this is done. I said it is to test your mental capacity at this
moment. It also tests your memory and your ability to follow certain tasks.
She just didn’t know today’s date but she knew where she is and that it is
morning. I told her that’s fine even I forget the date sometimes (she
mentioned the month and year correctly, just the date she said 6 rather
than 5th of July).
There was dificits in registration and recall and rest was normal.
When I told her to repeat (pen, pencil and paper) she didn’t say them
correctly first time. I told her it’s ok don’t worry, let me repeat it for you… I
repeated until she said them correctly.
When she felt bad about not recalling later, I told her it’s fine, don’t stress
 . I kept reassuring and encouraging her and smiling… she liked that.
I explained that you have issue with short term memory and my d/d were
alcoholic dementia, vascular dementia and Alzheimer though unlikely for
his age..
Feedback 15-8-2018
Scenario: Cognitive impairement
Stem. Very long stem of a lady in 30s who is alcoholic etc and encountering problem
remembering things… (Quite similar to bookcase)
Tasks
~ Explain the test and why you are doing it
~ perform MMSE
~ DX and ddx
Inside was a lady in 30s sitting on chair and few papers lying on table with mmse questionnaire
etc. I greeted and explained her about the test.
I will be examining ur cognition by performing a test called mmse. It will help me asses ur
memory and thinking. Its in the form of a questionnaire so will be asking u questions and ask u
to do some problem solving. Each task has a score so try to come up with ur best answer (she
nodded head continuously) I will be doing this test in English so u have to read and write are u
okay with it .. She laughed and said yes off course.
Explained it is a screening test, bedside test, not diagnostic and hardly takes 5 mins. Throughout
the test I will be guiding u etc etc dnt hesitate to stop. Are you ready to start. She said yes dr.
Then performed mmse using those papers on table and as expected found problem with
registration and recall. Thanked her.
Explained that I have completed mmse and found problem with registration and recall (learning
new things and remembering) tt signifies short term memory loss( where old memories are
intact but new are not) in layman term.. told could be due to alcohol , b12, hypo thyroid, trauma
( long list of stml) and lastly said about dementia ( but needs to be confirmed by specialist). She
said oh okay . Grade : Pass

Feedback 7-9-2018 PASS


MMSE ( for this case – special thanks to my sis – HYL )
MMSE (Alcoholic neuropathy) Forgetfulness to take medication , I think over 65 year old lady
Taking excrssive alcohol consumption over 20 years, but reduced recently 2 weeks ago. Impaired
Orientation
TASK
1. Explain what u are going to do - Perform MMSE (not need to calculate scores) –
2. explain possible reasons to the patient with reasons .
For task 1 : ORARL -23 RWD
Orientation :
impaired at Date ( said 24th , rather than 07/SEP/2018 ) . Other orientations are normal – know
year , month , season according to the MMSE card
Registration : ( New Learning )
Cannot get 3 objects until 3rd time
ATTENTION : ( impaired )
cannot spell backwards “ WORLD “
RECALL:
Cannot recall the names of 3 objects ( only 1 name , she remember )
Language :
can name the object which is pointed out. Can say “ or if buts “
3-steps command:
Can understand 3 steps straight away and followed in order without difficulties , initially the
patient tried to follow a step one by one . I mentioned her that , please wait for all 3 steps and
do straight-away for 3 steps instead of doing one step after I told her .
Reading and obey Can read and obey the sentence ( Close the eyes )
Writing -nil difficulties
Drawing : Can copy 2 diagrams with overlapping.
- DDx –
-Alcohol induced brain injury
- early dementia ( Scores are not calculated out of 30 )
- Vitamin B12 deficiency
- Electrolyte imbalance.
- other metabolic such as – thyroid dysfunction , diabetes neuropathy
- infection such as Meningitis ( least likely )
- nasty growth in the brain such as Brain Tumor
Feedback – Cognitive impairment – Pass Global score -4
Key step 1 – no Step 2 – yes Step 3 – yes Step 4 – yes
Approach to patient – 3 Exaplantion procedure – 4
Performance – 4 Interpretation of invx – 4

Case (5/10/2017)
Cognitive impairment Pass
40 yrs old lady with known DM on Metformin, comes to your GP as she forget to take pills very
often. She had history of alcohol abuse and now she is cutting down alcohol consumption. You
decide to do MMSE on her.
Task:
Explain to her about MMSE and why you are doing it on her.
Perform MMSE.
Explain abnormal findings to patient
Explain what is happening to her
Introduced myself and reassured her confidentiality. Then try to build rapport with her by asking
questions like how is she doing and how is she feeling now. Then explained her about MMSE. I
told her i want to do examination called Mini Mental State Examination. It is one of the
examination to check your brain function about ur memory and orientation.
I am doing this because u r forgetting to take ur DM pill quite often. Then performed MMSE
( card was given). Then i explained to her abnormal findings that i got from her. She told me
wrong date ( not sure it was intentionally or accidentally), she said it is very hard to extract 7
backwards and she gave me only one answer, I asked her to spell WORLD backwards and she
spelled incorrectly, and of course she has impairment in Registration and Recall.
I explained to her that she has memory problem most probably due to damage to the brain due
to alcohol abuse. ( she told me she is cutting down alcohol consumption and i appreciated her),
and could be due to vitamin deficiency due to alcohol, and mini strokes as her sugar is not well
controlled because she is forgetting to take pills.
Covered 3 key steps out of 4 Approach 4 Explanation of procedure 4
Performance 4 Interpretation 4 Global score 4

Case (27/4/2017)
1- MMSE alcoholic
45 yr man DM and drink alcohol intermittently, last time drinking was 2 weeks ago. Today RBS is
27. He is quite forgot to take medication.
Explain pt why u want to do the test.
Perform MNSE with given papers and chart.
Explain what are normal and abnormal to pt.
Explain the likely causes to pt.
It was like routine MNSE, explain the pt why I need to do it as usual.
He asked me; how long will it take. I said; it will be just a few mins. He said ok.
MNSE papers were provided.
*pf (can't do correct today date on orientation, impaired registration , attention and recall
,others -nl)
So, I explained the patient with good things that he can do. I explained him that you got problem
with registration that you cannot remember that objects that I mentioned and also got problem
with recalling the memories. There was some problem with today date but it was not a big
problem.
So, I suspect you got some memory problem which might possibly related with your DM. DM if
not well controlled, it can affect the memory. Another thing I am considering is alcohol related
memory problem, we called it alcohol induced brain injury which can be found in chronic drinker
but I noticed you didn’t drink every day. I just said that. There can be other possible causes like
hypothyroid which is a low level of hormone in our body called thyroid hormone. It can affect
the memory. Sometimes, mood problem like depression can cause memory problem like
forgetfulness. Bell ring………..
FB-PASSED, OVERALL-4, APROACH-4, EXPLAIN THE PROCEDURE-4, PERFORMANCE OF
PROCEDURE-4, INTERPRET OF PROCEDURE-4.
Feedback 7-9-2018
MMSE - PASS
35/55 (?) age years old lady, used to drink a lot(6/7 sd per day), cut down to 2 sd for last 2
weeks. She has high blood sugar as well, known diabetic patient. Recently she is having some
issues with her memory??
- Perform MMSE. Chart will be given inside. Explain to pt what you are going to do.
- Explain your findings to your pt.
- Give possible reasons for your findings.
FEEDBACK: PASS
Global score: 4
Key steps: 2/4
Approach to patient: 5
Explanation of procedure: 3
Performance of procedure: 4
Interpretation of INVX: 4
2mints thinking- I was thinking the steps of MMSE . Somehow, I was feeling that I might get
MMSE chart before my exam. So, I gave a quick look from handbook in the waiting room.
DDX or reasons for her memory problem-
1. alcohol
2. diabetes
3. vit B12 deficiency
4. tumor/trauma
(If you get enough time, you can say all the reasons of Dementia. Check karen about the
mnemonic 'DEMENTIA' )
After introduction & confidentiality statement, I explained her; today I'm gonna perform a
bed side test called Mini Mental State Examination, which will assess your mental function.
Would it be alright with you? (yes)
However, it's just a screening test, it will not confirm any diagnosis but it will shed some
light on any changes of your mental function. This will involve a bunch of questions and
writing over a paper. Please be relaxed, I'll guide you through it and explain any confusion as
many times as you want . I'll also jot down all my findings as you are having difficulty to
recall. It'll take just a few minutes of yours. Shall we get started now?(yes)
The MMSE chart's writing style was different than handbook. I found it difficult to read the
chart and asked the pt at the same time. As I mentioned, I had already memorised the points
beforehand, I started to ask ORARLC. After each part, I quickly looked through the paper to
check if I had missed anything. Problems were : registration (I asked 3 times), attention
(asked 2 times- at that time I started to hurry up a little bit and accidently I jumped to next
question. Role player gave me a different expression that time and I realised my mistake),
recall (asked 3 times). Others were all normal.
After I finished, I thanked her for finishing all the tasks and started to explain:
You have answered correctly date, time and place. So, you are clearly oriented. You have
followed the orders I asked you to do , drew the diagram perfectly, no problem with writing
a sentence; you are good in language and construction part. Unfortunately, you had some
difficulties to register few words in your memory, difficulty to recall them as well. You had
some issues with concentration as well. Right? Does anything make sense to you? (yes)
This could be associated with high amount of alcohol consumption, high blood sugar can
also damage nerve and mental function, could be associated with any medication or use of
any street drug, any trauma, injury.
Bell rang!! I just said I'm going to refer you to specialist.

Handbook 146 (Just reading material)


Cognitive state assessment of a 50-year-old barman

CANDIDATE INFORMATION AND TASKS

This 50-year-old man works as a barman and is attending an appointment at a general practice,
which you arranged after a conversation with his wife. He has a history of consumption of up to
10 standard alcoholic drinks on most days, over the last 10 years.

In attempting to contact the patient at home, you spoke with his wife who said that the patient
has been quite forgetful and unreliable for some months, causing family and work colleagues
to be irritated and concerned.

The patient is in your consulting room. You have completed your history and you are now
proceeding to test his cognitive function.

YOUR TASKS ARE TO:

• Examine the cognitive state using the modified Folstein Mini-Mental State Examination
(MMSE) provided.

• Explain to the patient what you are going to do and why.

• Summarise for the examiner, the normal and abnormal MMSE findings.

• Tell the examiner your interpretation of the results, including what condition these results
signify.
AIMS OF STATION

To assess the candidate's ability to administer a Folstein Mini-Mental State Examination (MMSE),
summarise the results (impaired registration and recall, performance on other tests normal),
identify the deficient cognitive function (isolated impaired new learning/short-term memory),
and interpret its significance.

EXAMINER INSTRUCTIONS

The examiner will have instructed the patient as follows:


Be cooperative, but uninterested, follow the candidate's instructions literally (even if given the
wrong instructions according to the test), do not be helpful. These are the responses you must
give to the test the candidate gives you.

• Give correct information on your name, year, day and date, season and location of

consultation.

• On being asked to repeat three words, take three attempts to learn them all.

• On being asked to subtract 7s from 100 say '93, 86, 79, 72, 65...' then pause, ask 'what was I

taking off?' If instructed to continue with the task, do so to the best of your ability.

• On being asked to spell WORLD backwards, answer correctly DLROW.

• On being asked to recall the three words, say only one of the three, no matter how many

prompts. Say you can't remember any of the others.

• Name the watch and pen/pencil (or other objects the candidate indicates) correctly.

• Repeat the sentence 'no ifs, ands or buts'correctly, as you hear it said by the candidate.

• Follow the three-stage command (paper test) correctly.

• Read and follow the written instruction 'close your eyes' correctly.

• Write the following sentence ‘I like my garden'.

• Copy the design accurately.


Any other questions, (e.g. date of birth) give your own, or say you

don't know. The examiner will interrupt if the candidate seeks to obtain further history.
EXPECTATIONS OF CANDIDATE PERFORMANCE

The candidate is expected to administer and interpret the Folstein MMSE in a clinical setting,
where the patient has chronic excessive alcohol consumption, and a collateral history of
impaired memory and behaviour.

The station tests the candidate's skill in administering a screening tool for cognitive impairment,
and interpreting the results, given the relevant clinical information of chronic excessive alcohol
use, and poor memory interfering with everyday activities.

The candidate is given the Folstein MMSE form (Table 1) and asked to proceed with testing the
patient's cognitive state.

The candidate is required to explain the test and its purpose clearly to the patient, and proceed
through it in an orderly fashion, paying due regard to the patient's comfort and level of
understanding.
Candidates are not required to calculate the score, and for that reason, the score marks have
been deleted from the sheet they are given. If they attempt to calculate a score, tell them this is
not required.
If candidates attempt to obtain a further history or do other examinations, either physical or
general psychiatric (e.g. mood, psychotic symptoms), stop them.

Examiner questions to candidate:

At five minutes ask the candidate to summarise the findings and any abnormalities, and what
they signify.

• The candidate is expected to identify both normal and abnormal results on the MMSE, in
order to interpret the problem as one of impaired new learning/short-term memory function.

• Responses that identify abnormalities in other than registration and recall are incorrect and

unsatisfactory.

• A response that the patient is confused/delirious and has dementia or a bad longterm memory

is incorrect and unsatisfactory.

At six minutes:

• If the candidate has not already told you that this screen shows other cognitive functions are
normal, ask if other functions are normal, and what is the significance of this (i.e. not an acute
brain syndrome as in a withdrawn state, no definite diagnosis of dementia).

• Knowledgeable candidates may know that the focal impairment may be an amnestic
syndrome, and the association with Wernicke-Korsakoff syndrome (but this is not obligatory to
pass).

• If the candidate has not told you the likely cause, ask for this (i.e. alcohol-induced brain
injury).

KEY ISSUES

• Skill in using a screening tool for cognitive impairment.

• Ability to interpret the results — and to identify there is a focal problem of short-term memory
impairment.

• Knowledge that this is likely to be an acquired brain injury as a complication of hazardous

alcohol use.

CRITICAL ERROR

• Failing to identify the specific short-term memory deficit.

COMMENTARY

The Folstein MMSE is a screening tool assessment of cognitive function impairment — broadly
speaking, delirium/acute brain syndrome/acute confusional state and chronic brain
syndrome/dementia. Abnormal results indicate the need for further neuropsychiatry/
neuropsychological and medical evaluation to make an exact diagnosis.

The cognitive performance deficits as scripted are in (2) Registration and (4) Recall —these are
short-term memory functions; the deficits are almost certainly due to acquired brain injury
from chronic alcohol abuse. The cognitive deficits are confined to short-term memory functions.
It is very relevant that all other responses are correct, and that there are no indications with this
screening tool of a diffuse chronic brain syndrome/dementia of whatever kind, or an acute brain
syndrome/delirium/acute confusional state.
In particular this patient is fully oriented and alert, and hence he does not have an acute brain
syndrome/concentration deficit in answering the doctor's questions; the candidate is expected
to know this.

This short-term memory deficit picture, with preservation of longterm memory and other
intellectual function, in the absence of an acute brain syndrome, is a focal cognitive impairment
of an amnestic (Greek — without memory) type, and is one type of the cognitive decline found
with chronic alcohol abuse. Further neuropsychological testing is required to exclude more
diffuse impairment (e.g. dementia). It is possible the patient has or has had an episode of
Wernicke encephalopathy, and the knowledgeable candidates will appreciate this.

32-Hypothyroidism
You are in GP, middle age lady with complaint of weight gain and tiredness.
Tasks
- Take relevant history
- Ask PE from examiner
- Explain Diagnosis/ differentials with reasons

History
1-Tiredness questions
-how long have you been feeling like this? (a couple of weeks)
-do you feel tired the whole body or specific part? (The whole body)
-any particular time during the day when you feel more tired? (The whole day)
-has this happened before? (No)
-how does it affect your life? (I cant go to work and have to take off day)

2-weight gain questions


-how many Kg did you gain? For how many months? (7 Kg in 1 month)
-how is your appetite? Any increase in your diet intake? (Good appetite, no increase in food)

3-HEMIFADO
Hemochromatosis, hepatitis
-any pigmentation around your body?
-any yellowish Discolouration of your skin or eyes?
Endocrine (thyroid and DM)
-any weather preference? (Prefer hot weather)
-any change in your bowel habits? (Constipation)
-any change in your hair, nails? Any hoarseness of voice? Do you think your skin become thick
and dry? (brittle nail, thin hair)
-have you noticed any lump in your neck? (Not sure)
-how is your periods (heavier)

-do you feel thirsty?


-are you passing large amounts of urine?
Malignancy
-any lumps or bumps around your body?
Infections
-any fever
Anemia
-dizziness, funny racing of the heart
Depression/ dementia
-How is your mood? -anhedonia
-sleep -Memory
Obstructive sleep apnea
-do you snore loudly during sleep?
-do you feel drowsy during the day?
4-General questions
-have you had any past illnesses?
-past surgical operations
-do you take any medications?
-smoking/ alcohol
-Family history (DM)

Physical examination from the examiner


1-General appearance
look tired, dull, coarse features
2-BMI (27)
3-Vital signs (PR : 70/min, others : normal)
4-Hands
dry, brittle nails, cold hands.
5-neck
lump and LN (soft lump in front of neck, no cervical lymphadenopathy)
6-Head
pallor, jaundice, eye signs, dry skin, thin hair
7-CNS
power, tone, reflex (delayed reflex)
8-CVS
9-respiratory
10-abdomen

Explanation
-based on history and examination there could be several possibilities why you have tiredness.
-most likely, you have a condition we call hypothyroidism. Thyroid is a butterfly shaped gland
located in the lower part of your neck, it is responsible for secretion of thyroid hormones
essential for basic metabolic functions of the body. In your case the gland is not functioning well
so secreting low amount of thyroid hormones causing tiredness, weather preference,
constipation and … .

-Other possibilities are:


-Diabetes , infection, nasty growth, anaemia, depression, OSA.

Case (22/2/2017) (7/6/2017)


You are in GP, middle age lady with complaint of weight gain and tiredness.
Tasks
- Take relevant history
- Ask PE from examiner
- Explain Diagnosis/ differentials with reasons

2mins thinking
- This is book case
- DDx – HEMIFADS (Haemochromatosis, Endocrine – DM, hypothyroid, Malignancy which is
not case here, infection, CFS, Anaemia, Depression, sleep apnoea)

History taking
I am Dr Khine. I am one of the Drs in this GP. How can I help u today?
(RP : I ve gained wt and feeling tired)

Is it first time for you? (Yes)

Since when? (a couple of weeks ago)

Wt gain - How many kg did u gain wt? (7kg in 1 month) How’s ur appetite (Good). Did u
increased ur food intake? (No)
Tiredness - Do u have any particular time of day u feel more tiredness? (No, the whole day),
How does it affect ur daily life? (I cant go to work and have to take off day)

DDx
Hypothyroid
- How often do u open ur bowels? (I have constipation)
-Any weather preference like u feel cold to normal weather? (Yep, I prefer to stay in hot weather)
- Do u think ur skin become thick and dry? (yes)
- Nails and hairs (become brittle and thin)
- Any lumps in ur neck? (not sure)
- What about ur period? (They become heavier)

Sleep apnoea
- With whom are u living? (my husband) Does he tell u that u snore when u sleep? (No)
- Are u feeling drowsy during day time? (No)
Diabetes
Do u need to pass urine more frequently? Feeling like more thirsty? Drink a lot of water? (No)
Depression
How’s ur mood? (Ok Dr)
Infection/ malignancy
Fever, any lumps and bumps around ur body? (no)
General health - good, any significant past history (No)
SADMA - all clear

PE from examiner
GA - As you see (I said I am happy with GA)
VS - PR : 70/min, others : normal
BMI – 27
Neck exam - U can feel soft lump in front of neck, no cervical lymphadenopathy
CVS, Resp, abdomen – Normal
CNS - Delayed reflexes

Explanation
Jenny, according to ur HO and PE findings, most likely u are having problems with ur thyroid
gland.
I drew a diagram of neck and thyroid gland. This gland secretes hormones which is essential for
basic metabolic functions of our body. In ur case, thyroid gland is not functioning well, a
condition we call hypothyroidism.
Other causes can be obstructive sleep apnoea which is ur airway is blocked with soft tissues of ur
thorat when u sleep. But u dun have any daytime sleepiness and snoring, it is unlikely

U dun have any DM symptoms, No features of depression as well.

AMC Feedback - Lethargy : Pass (Global Score - 5)

Approach to patient- 4
History - 5
Examination- 3
Dx/ DDx - 5

Recall 8-5-2018
Hypothyroid tiredness tries to talk slow warm weather preference constipation flat low energy
history 5 min PeFe diagnosis dd family history of DM

Handbook 35
Lethargy in a 50-year-old woman
You are consulting in a general practice setting. Your next patient is a middle-aged widow (see
Figures 1 and 2 below) who is presenting for a 'check-up'. She looks apathetic and lethargic on
first impression.

YOUR TASKS ARE TO:

• Take a focused history.

• Ask the examiner for the results you would wish to elicit on a focused physical examination.

• Give your diagnosis and differential diagnosis to the examiner, and indicate what further
investigations you would require.
AIMS OF STATION

To assess the candidate's history-taking skills and diagnostic acumen in a patient with the
symptoms and signs of hypothyroidism. The case is deliberately presented as an undifferentiated
problem but the patient's initial unprompted statements should lead to the correct diagnostic
pathway with confirmation of suspected hypothyroidism by a focused selective physical
examination.
Examiner Instructions

This case requires the patient to reveal the symptoms of hypothyroidism in a slow and hesitant,
but nevertheless positive manner, in response to appropriate history-taking.

The examiner will have instructed the patient as follows: You are a 50-year-old widow.

Opening statement

'My daughter wants me to have a check-up because she says I am always tired. '

• Respond to the doctor's enquiries as follows — without prompting.

• You have not felt well lately (be vague about the duration).

• You feel weak especially your arms and legs.

• You also feel lethargic.

• Your daughter says you are not interested in anything, go to sleep during the day and can't be
bothered talking to people.

• Your voice has become 'croaky' — people say it has changed over the past year.

• You are always constipated and this seems to be getting worse.

• Your periods stopped last year and were scanty and irregular for a year before that.

• You have put on weight.

• Your responses are apathetic, lacking expression.

• You are slow to react with a croaky, husky, thick voice and poor memory.

• You respond slowly to doctor's questions, but appropriate questions evoke correct responses.

• Do not be evasive. Show paucity of body movements.

In response to other questions:

• No history of thyroid surgery.

• You feel cold all the time.

• Your hair has become thinner.

• You find it hard to concentrate.

• Your memory is not good.

• You are able to manage your own personal care but everything is an effort and takes longer.

• You get constipated if you don't take Coloxyl® regularly.

Your joints feel stiff and the muscles are sore.


• You wake up early in the morning and can't get back to sleep.

• Answer in the negative to any other questions about your health except to indicate that you
feel you are gradually going downhill.

• If asked, state that you have just come to stay with your daughter because you were unable to
carry on living alone.

• Past medical history and family history — nothing of note. Parents died from old age.

Current medication

• 'Tonic' obtained from Pharmacist by daughter.

• Aspirin irregularly for the rheumatism.

• Dioctyl sodium 120mg (Coloxyl®) 'for my bowel' 1 or 2 tablets daily, taken for about a year.

• Physical examination - the examiner should give the findings for selective and specifically
requested components of the physical examination.

• Appearance —~ Looks tired and dull. Expressionless face, coarse features and skin. ~
Overweight — BMI 29 kg/m2

~ Vital signs- Pulse rate 56/min regular, blood pressure 130/70 mmHg

~ neck- Thyroid not palpable

~ Skin and hair- Skin dry sparse axillary hair

~ hands- Cold hands and feet

~ Power and tone reduced in arms and legs

~Reflexes sluggish with delayed ankle jerks

At six minutes the examiner will ask the candidate for the diagnosis/differential diagnosis and
proposed investigation.

EXPECTATIONS OF CANDIDATE PERFORMANCE

• Diagnosis/differential diagnosis

Candidate's response to examiner's request for the diagnosis: candidate should strongly suspect
hypothyroidism as indicated by pattern recognition and from the patient's symptoms and signs.
Other possibilities such as depression, anaemia, early dementia may be mentioned but should
be considered unlikely.

• Investigations

Must request thyroid function tests and full blood examination.

Key Issues

• History

• Choice and sequence of examination

• Diagnosis/differential diagnosis

• Appropriate investigations

CRITICAL ERROR

• Failure to consider hypothyroidism in differential diagnosis.

COMMENTARY

Spontaneous atrophic hypothyroidism often gives gradually progressive symptoms as ir this case.
The pathology is destructive lymphoid infiltration of the thyroid gland leading tc atrophy with
no visible enlargement, or in some patients, associated with goitre.

Diagnosis is suspected by the constellation of symptoms and signs as exhibited in this patient
and would be confirmed by elevation of serum TSH with lowered T4 levels. The condition is an
organ-specific immune disorder and responds well to thyroxine treatment, beginning with a low
dose (50 ug daily) and increasing slowly to the dose required to restore TSH to normal.

Reading materials
Karin case
Case: You are a GP and Mrs. Jane Austin, a 55-year-old female, whom you organized blood test
for is in your clinic to discuss the results as follows:

- TSH 25 (0.5-5.0)
- Total cholesterol 8 (<5.5)
- FBE Hgb 110 with macrocytosis

Task
a. Explain results and answer her questions
b. Explain management
- The thyroid gland is a butterfly shaped gland in the lower part of the neck which is composed
of two lobes and a connection between them. This secretes T4 and T3 which impacts every cell
of the body. In your case, the gland is less active and we call this hypothyroidism. The TSH is high
which reflects that the thyroid hormone is low. This is a serious condition and the cause is most
likely autoimmune.
- We have in our body the autoimmune system which helps fight infections. However, for
unknown reasons, the immune system is attacking some of the organs or tissues of the body. In
your case, most likely it is the thyroid gland. This is why our first step is to do thyroid antibodies,
ultrasound, and radioisotope scanning to detect the activity and presence of any lesion in the
thyroid gland.
- To manage the condition, we have to refer you to an endocrinologist who will evaluate your
case.
- What if I don’t get treated? This condition is a serious condition. If not treated ASAP, you may
experience worsening of symptoms which ranges from severe depression followed by severe
psychosis, confusion, delusions, and sometimes, myxedema madness, coma, and maybe death.

- What’s wrong with the blood count? This is a common association with autoimmune and is
known as pernicious anemia. It has autoimmune effect on gastric parietal cells which leads to
destruction and decreased absorption of vitamin B12 leading to anemia with macrocytosis.

- We need to do ECG to rule out any ischemic condition then start 25 mcg for aged or patients
with IHD or 50 mcg otherwise. The test would be repeated in 4-6 weeks and increase the dose
by 25-50 mcg until it reaches the euthyroid stage. Maximum dose of replacement is 100-150
mcg. (start you on low dose then increase it slowly to restore the TSH to normal)
- Regarding your cholesterol, hypothyroidism is one of the risk factors for developing the
condition. You immediately need to start lifestyle modification with low-fat diet and brisk
walking 30 minutes a day for most of the week. We also need to assess your cardiovascular risk
to determine whether there is a need to prescribe cholesterol-lowering drugs.
- Control of symptoms? Improvement will start in 4-6 weeks but symptoms will not settle before
6 months.
- What are the side effects of medications? The medication may cause hyperthyroidism where
from being depressed you may become anxious, palpitations, nervousness, and feel hot.

33-Lymphedema after mastectomy


you are at your GP, when 58-year-old Lisa presents to you with a swelling of her right arm. She
had breast cancer in her right breast diagnosed to be stage 2 cancer. Following which, she had a
total mastectomy with axillary lymph node dissection; she also had radiotherapy 2 months ago.
TASKS
Take a further history
Discuss your management with the patient

Differential diagnosis
1-lymphoedema
2-recurrence of breast cancer
3-trauma
4-cellulitis
5-insect bites
6-axillary DVT

History
1-swelling questions (duration/onset/site)
- for how long have you had this swelling in your arm?
- has it started suddenly or gradually? Is it constant or does it come and go? Is it getting worse?
(gradual and progressive)
-Is there swelling in any other part of your body?

3-Differential diagnosis questions


- Is there pain in your arm? (Painless) (DVT)
- Have you noticed any redness or bluish Discolouration of the arm? (cellulitis)
- Any breakage in the skin? (cellulitis)
- Any fever? (cellulitis)
- Any trauma or injury?
- Any insect bite?

2-Complication questions
-Any tingling or numbness in your arm?
-Are you having trouble bending your finger, wrist, elbow or shoulder? (+ve)
-Do you think that it is affecting your daily activities?
4-general questions and recurrent breast cancer question
-have you taken any treatment for the swelling?
-have you had any past medical illnesses like hypertension, DM, heart or kidney disease
- I am sorry to hear that you had breast cancer. I read from the notes that you had surgery before
for that. When you finished the radiotherapy? Any immediate problem after the surgery? Did
you notice any other lumps on that side of the chest? Any loss of weight?
-do you have good support
- SAD

Explanation
Condition
-Most likely what you are having now is a lymphedema of your right arm. There are two systems
in our body, one is the circulatory system through which the blood flows and other one is the
lymphatic system that carrying lymph which is a clear colourless fluid having a few blood cells.
-lymphatic system has got a network of tiny vessels and small bean shaped structures called the
lymph nodes.
-this system maintain fluids in our body by filtering and draining the waste products from each
region and also helps fighting infections.
Cause
-often during breast cancer treatment some of the LN, which drain from the upper arm and
majority of the breast and under arm area, needs to be removed, so this disrupted the flow of
lymph from the arm causing swelling or lymphedema.
-another cause could be the radiation you have received.
Outcome
this condition cannot be cured but can be well managed

Management
-I will refer you to a lymphedema clinic, which has a specialist doctor and a physiotherapist
specifically trained in treating lymphedema.
The physio will teach you:
1-Exercise of the arm, which can improve the lymph drainage.
2-they will give you a prescription of Compression sleeve that you can put on the arm to prevent
pooling or build-up of fluid, and also helps to move the fluid out of the arm.
3- Massage therapy that helps to move the fluid out of the swollen area.

GP advice
-keep the arm raised above the level of the heart whenever possible, which lets the gravity drain
the fluid.
-I will advise you to adopt lifestyle modifications as well like a healthy diet, regular exercise, as
controlling the body weight is really important.
-There are certain precautions that you need to take as the arm is at more risk of infection:
1-Avoid taking injections or blood tests/ avoid blood pressure measurements on the affected
arm
2-Avoid carrying weight.
3- make sure to avoid cuts and wounds
4-Practice good care of the skin and fingernails
5-Use gloves when you are doing housework or other activities like gardening
6-Do not apply heating pads or ice packs
7-Avoid getting sunburns on the arm as well

Warning signs
any time that you are having any redness, warmth, pain, fever report immediately.

reading materials regarding lymphedema and precautions


review after lymphedema clinic consult.

Feedback 17-8-2018
Station-2(pass)
Arm swelling,Task –hx,dx,dd,mx
Picture of one breast given with arm swelling to opposite,after 2yrs of mastectomy arm
swelling(pt was sleeping while I was talking to her,I had to tell her repeatedly r u ok?)

-full Karen I followed no change…sent to specialist for check up to see any recurrence.i had lots
of time in every cases except cvs pe,so I asked do u have any questions?she asked will it get
worse?i said with proper treatment we can hope for the best bt still depend upon ur body
response to treatment,

Station-3(pass).
Feedback 17-8-2018
ARM SWELLING
OUTSIDE
In this all the typical history (just like Karen )of lymphedema was given outside. With a pic if one
sided breast removed and arm was swollen on that side .your task was to
Take further history to rule out other conditions which can present similar to this by taking short
history
Tell the patient all possible causes with reasons
Tell the patient management.

INSIDE
I took history to rule out DVT, CELLULITIS, INSECT BITE, THROMBOPHELEBITIS, TRAUMA
,ALLERGY. Similar to Karen .
Then I explained about all dd’s to the patient and told with reasons why they were unlikely.
Then explained all management 100 percent as Karen but the important thing is that I forgot to
mention about dopplar ultrasound to rule out DVT by investigations, most probably they haven’t
asked for investigation in the task specifically. I was worried and thought I failed the case
because of that but surprisingly I passed. Maybe if I have mentioned investigations I could get
better score.

All key steps covered


Approach to pat 4
History 5
Dds 5
Pat counselling/education 5
Management 5
Global 5

Feedback 17-8-2018
Left arm lymphedema after mastectomy. History and diagnosis differentials and management
for edema:
It was my last case in the exam. Lady. old age. Gp setting. There was a picture of left hand
lymphedema and total mastectomy. She was worried if it is recurrence of her breast cancer.
Had a surgery 4 or 5 years ago. Was on tamoxifen after surgery.
Hx: I asked about the graduate or sudden. Fever. Any trauma. Any insect bite. Working in
garden or ulcer in fingers. Does it get better during night? is it first time? any sob (r/0 dvt)(r/o
metastasis) . any bone pain. Any headache. Wt lost. Physical activity. Injection? History of
checking BP in this arm? 5p. and of course smoking. Medication,fh
Ddx: lymphedema. Cellulitis. Upper limb DVT. Thrombophlebitis
I explained why im more thinking about lymphedema. And why the cancer is less likely. She
asked why it is happening now after 4 years?. I told her there could be number of reason like
ph activity. Like the amount of time that ur hand is in lower height then ur heart and..
I mentioned lymphedema clinic, elastic band, getting rest, using pillow under her arm at night,
massage the hand, using glove in the garden , mentioned alarm sign and tell her we need to R?O DVT.
And we can also arrange for further investigation to make sure it is not reoccurrence of the cancer./
role payer was happy. Showed me tomb up when I was closing the door!

Feedback 7-12-2018
60 yrs female got mastectomy done 5 yrs back,radiation also, with regular follow up, now presented
with arm swelling , big history, along with picture given outside of mastectomy scar along with scar
on axillary area and swollen arm with typical peau d pattern.
TASK- history for 2 min,dd/dx, counselling patient. 2 min thinking - history
only 2 min so only exclude important dd, need to counsel with diagram .(as this is main task)
entered and greeted, told her i have seen her reports and know bit about her case, just need to ask
few more questions to know her better .(try to act familiar with case )
history - since when -5 months , increasing gradually , not painful, no injury , no bites , no aggravating
and relieving factors
coping well after surgery and radiation , enough support ,
past history -nil
family history - nil
SADMA

explained her there could be many reasons but lymphedema which is lymph extra fluid outside
vessels with diagram (as nodes from axillary area removed ), other dd like sub clavian venous
thrombosis, axillary artery occlusion, cellulitis, injury .explained bit more about lymphedema, how to
protect arm ,moisturiser regurlarly ,(skin lost its barrier function so need to protect very carefully )
dish washing and cleaning with gloves , no blood pressure measuring on that side, and all . (she
laughed and asked anything else i need to know -then i realised i forgot -colour doppler - i said ill
refer to hospital for doppler to confirm and exclude other , then she said fine) and refer to
lymphedeme clinic and special physio exercise will be taught to her . i said last that it takes times to
get better so need to be cooperative .

scenario- arm swelling

Global 5 (approach 5 , history 4 , d/ddx 5 , patient counselling 6 management plan 6)

Feedback 7-12-2018
Arm Swelling - PASS
50 something woman who had breast surgery in the past for breast cancer is now complaining of the
swollen right arm. She also had chemotherapy and tamoxifan as adjuvant therapies for breast cancer.
Task: History (3 mins)

Explain possible conditions

Management plan (forgot another task)

History – As soon as I have greeted the patient, she started to show worries about having the relapse of
breast cancer. “Don’t worry, I will surely address you concern, but first, I need to know more about ur
concern. Would like to spend a few mins asking a few questions. Alright?”
Asked about the swelling in details – Onset? Duration? Worsened suddenly these days? Extent? Skin
colour? Severity (effect on daily activities)?

DDx – fever? any cloting problems? Any injury? Insect bite? Any activities you enjoy doing in ur free time
like sewing or gardening? (she said NO, I think)
Breast cancer – regular visit? Last visit? Have u felt any lumps recently, esp around your armpit?
Breathlessness? Doing SBE on the other side? (No – “alright, I will show you how to do it later”)
Dx – Lymphedema; maybe cellulitis; DVT; insect bite; heart problems; injury on the respective arm
Mx-
After that, patient started to shoot questions..
“Is the cancer?”
-very unlikely because in cases of relapse of cancer, lumps can be felt and maybe tingling sensation in the
hands. So, do you feel anything unusual in ur fingers and hands? “No” – so according your story, its very
unlikely, and you have been having regular follow up with your specialist, and everything was normal until
the last visit, so I doubt this is cancer but to make things clear, I’d like to refer you to specialist again to
have a full assessment.
“Ok, but what about the swelling? Will it swelling ever subside, doc?”
-I cannot ensure you that, but definitely we can try many methods to reduce the swelling so that it wont
be of any issues cosmetically
“What can you do for me?”
-Investigations – FBE; Doppler; (I dunno if I offered other investigations); Treatment – after excluding DVT,
will refer to lymphedema clinic for exercises to reduce the swelling. If no response, there is role of
medications (diuretics) but these are not sth we consider first for this condition.
“Anything that I should avoid? Or aware of?”
-avoid having scratch injuries on ur arm; avoid insects; avoid BP being measured; don’t wrap ur arm or
shoulder tight with anything (I don’t know what made me say this)
“Any risks with this condition?”
-I don’t know the answer, tbh, so I just said a long “Well…..” then the bell rang to my relief.
GS: 5
KEY STEPS 1, 2, 3 – all covered Approach – 5
History – 4 Dx and DDx – 5 Counselling – 6 Mx plan – 6

34-Weight loss
47-year lady concerned about weight loss
Task
- History
- Pefe on card
- Possible reasons

History
1-Confidentiality (you can change the confidentiality not necessarily In the beginning)
I’d just like to ask you a few questions to find out what is happening. Some of these questions
might be sensitive and personnel as well but let me assure you that everything we gonna discuss
will be confidential unless there is harm to yourself or others. Is that ok?

2-Weight loss questions (how much/ diet/ exercise)


-exactly how much weight have you lost and over how long? (5 Kg over 3 months)
-have you changed your diet in any way? (Yes)
-do you think you eat healthy diet? Can you tell me briefly about your diets? (I eat less portions
and proteins over the last 3 months)
-do you do any sort of regular exercise? (Doing workout daily 1 hour a day for 3 months)
-have your clothes become looser?
-is this the first time you lost such weight?

3-psych questions (depression/ anxiety/ eating disorder)


-how do you feel these days? Do you feel uptight, worried or anxious? Do you get very irritable?
Do you feel depressed?
-have you lost interest in things you used to enjoy?
-has your appetite changed?
-how is your sleep?
-do you have trouble concentrationg?
-how do you feel about the future?
-how do you think about your appearance? do you ever force yourself to vomit?

4-Red flags (heart failure/ malignancy/ infections)


-are you short of breath? Any chest pain? any funny racing of the heart?
-any lumps or bumps around your body?
-any fever? Any cough? Any night sweating?
-have you travelled recently?

5-Masquerades (DM/ thyroid/ anaemia/ UTI)


-do you feel thirsty? Are you passing large amount of urine?
-any weather preference? Change in bowels habit?
-do you feel dizzy? Tired?
-any pain on passing urine? Change in color of urine?

6-Pitfalls (GIT/ renal failure)


-any tummy pain?
-is your urine stream as good as it used to be?

7-General health
-do you smoke?
-do you drink alcohol?
-do you take any medications?
-have you used any recreational drugs? OTC?
-are your periods normal?
-do you think you ever engaged in risky sexual activity?
-PMH, PSH, family Hx
-any stress at home or work?

PEFE CARD (all normal, weight 66, BMI 24)

Explain
-let me assure you that from history and examination, I could not elicit any serious condition. All
normal and your BMI, which is a measure of your weight is also normal.

-Most likely could be because you recently started exercising and dieting.

-Other possibilities are:


-infection (no fever)
-problem with thyroid (no weather preference, neck lump or bowels problem)
-diabetes (not thirsty, or abnormal urination)
-nasty growth (no loss of appetite, lumps or bumps)
-gut (no problem with motions or tummy pain)
-psych (mood ok, no stresses)

-red flag, review, reading material

**one feedback said she was heavy smoker 15 cig a day and a candidate who passed mentioned
doing Ix:
To rule out all of these I need to run some Ix
-FBC (RCC, WCC)
-ESR/CRP
-TFT
-Random blood sugar
-UCE, LFT
-chest x-ray
-urinalysis

If management then Ix+ 4R and refer to dietician!!

4-4-2018
47 yr lady concerened about weight loss
Task Hx
Pefe on card
Possible reasons
On Hx- she was on diet with less portions and proteins for 3 months. Doing workout for 1 hr daily
for 3 months. Trying to reduce weight, lost 5 kg.
On card weight 66, bmi 24, rest normal.
Feedback pass 4-4-2018
In weight loss she was trying to lose weight with small portions of protein diet and exercise 1 hr
a day. Nothing else positive not even smoking. I told all dds and reasons why they are not
present in her case and said most likely ur weight loss is due to dieting and exercise and its
intentional. And told all red flags that if present then come back.
Passed with GS 5

13-7-2018
40s woman, intentional weight loss, told by friends she is too thin, BMI 24, card with PE given, no
abnormality

Case (23/2/2017)
47 yo lady to GP, Wt loss Hx, PEFE ( on card ), likely Dx with reasons.

Case (9/11/2017) Wt loss Task-Hx, PEFE, ( BMI 24, ), tell her reasins of Wt loss.

comment
a young lady(age less than 30),wants to talk to gp about recent weight loss.
task:
take history,
pefe,
explain diagnosis...
she has no psychosocial issue, happy family, no lymhadenopathy, no disease, no medication,
smokes 20 sticks a day, social drinker, appetite good, wants to look good, no stress at job, have
healthy diet, doesnt skip meal, no diarrhea, no allergy, physical exam all normal, bmi 24.

comment pass
Stem said concerned about weight loss .Thats it..
task was history/PEFE/D/D......nothing positive in history apart from smoking and recently
started gym 3 times a week for around 2-3 months.PE : BMI 24 everything else was normal

I said its normal coz you have recently started exercising and your BMI is in normal range.but
chronic smoking can also loose weight.however there can be many reasons for loss of wait like
bla bla bla. I will review in 3 months from now and we will keep an eye on other symptoms as
well.thats what i said but haven’t got the feedback yet

35-Duodenal atresia
Case 1/
A midwife trainee comes and is concerned about the condition of a newborn (born yesterday)
who vomits frequently.
Task:
- ask further history from the midwife trainee
- ask the relevant physical findings and investigations from examiner
-Ask investigation from examiner and explain it to midwife
-explain the most probable diagnosis for the midwife

Case 2/
you are in rural hospital when a mother of newly born baby C/O frequent vomiting. X-ray given
showing double-bubble sign.
Tasks
-explain x-ray to mother
-explain probable diagnosis and management

For case 1
History
1-Vomiting questions
-how many times? (after every feed)
-what is the amount?
-what is the colour? Is it clear, yellow or greenish?
-what does it contain?
-is it forceful or not? (projectile)

2-tummy pain and bowel motions


-is he drawing his legs while crying?
-did he pass flatus or stools?

3-any fever or rash

4-dehydration questions
-did he pass urine? Is his nappy wet or dry?
-does he look drowsy, lethargic? Is he crying a lot?
-how is his feeding? Is he feeding well? Is he breast feeding or bottle feeding?

5-BINDS
-Birth: any complications or infections during pregnancy? Was it a normal vaginal or CS? Assisted
or spontaneous? Term or preterm? Did he need any resuscitation? Did the mother have down
syndrome screening?
-Immunisation
-development
-sibling’s similar symptoms? Who live with? Any support?

PEFE
1-General appearance (DR PJL)
-dehydration (fontanelle, skin turgor, dry mouth, sunken eyes, peripheral perfusion)
(frontanellae+, sunken eyes + ,dry mucous membrane + , skin turgur +.)
-dysmorphic feature ( any down features like slanted eyes, low set ears, single palmar crease)
(upward slanting of eyes +, any low set ears +, single palmar crease +)
-rash
-pallor, jaundice, LAP
2-Vital signs
3-Growth chart
4-Abdominal examination
5-rapid system examination
6-genitalia

Investigation from examiner


-Ask for FBC, UCE, LFT, TFT (not available)
-Ask for x-ray (available)

Explain X-ray
-this is the x-ray of the child’s lower chest and upper tummy and this is the pelvic area.
-the x-ray is showing the classic appearance of what we call double bubble appearance; there
are 2 bubbles the 1st one is in the stomach area where there are huge air in the stomach. The
other one is in 1st part of small bowel. But there is no gas beyond that which means milk not
going through and indicates absolute small bowel obstruction. That is why he has vomiting.

Explain Diagnosis and differential diagnosis


-the most probable diagnosis is duodenal atresia which is congenital absence or complete
closure of a portion of the lumen of the duodenum; the 1 st part of small bowel. From the
examination, I could find features of Down syndrome so this condition is commonly found in
babies with Down syndrome.

-other possibilities are:


*obstruction of small bowel down the tract like in the 2 nd part called jejunum; Jejunal atresia.
*Malrotation
*Volvulus when a loop of intestine twists around itself and the mesentery that supports it,
resulting in a bowel obstruction
*meconium ileus; which is a bowel obstruction that occurs when the meconium in your child's
intestine is even thicker and stickier than normal meconium, creating a blockage in a part of the
small intestine called the ileum.

However, the history along with X–ray double bubble appearance indicates that the most likely
one is duodenal atresia. So I need to admit him to hospital to be seen by specialist for further
evaluation (you can talk briefly about management if you have time)

…………….END CASE 1……………

For Case 2
Ask the mother for her concern (she is not concerned as her previous young child had vomiting
which turn out to be nothing significant)

Explain X-ray (show the mother the x-ray as well and point to it)
-this is the x-ray of the child’s lower chest and upper tummy and this is the pelvic area.

-the x-ray is showing the classic appearance of what we call double bubble appearance; there
are 2 bubbles the 1st one is in the stomach area where there are huge air in the stomach. The
other one is in 1st part of small bowel. But there is no gas beyond that which means milk not
going through and indicates absolute small bowel obstruction. That is why he has vomiting.

-the most probable diagnosis is duodenal atresia which is congenital absence or abnormal
narrowing of a portion of the lumen of the duodenum; the 1 st part of small bowel.

-other possibilities are:


*obstruction of small bowel down the tract like in the 2 nd part called jejunum.
*Esophageal atresia: closure of the tube that connect mouth to the stomach.
*Malrotation
*Volvulus when a loop of intestine twists around itself and the mesentery that supports it,
resulting in a bowel obstruction
* meconium ileus; which is a bowel obstruction that occurs when the meconium in your child's
intestine is even thicker and stickier than normal meconium, creating a blockage in a part of the
small intestine called the ileum.

However, the history along with X-ray double bubble appearance indicates that the most likely
one is duodenal atresia.

Management
-this is a medical emergency and it can go serious if not taken cared of now.

-Because we are in rural hospital we do not have enough facilities here so that I need to refer the
child ASAP to the nearest tertiary hospital. I will contact the neonatal emergency transfer
services to Organise air ambulance for the baby.

-He will be seen by a specialist; paediatric surgeon.

-In the meantime, I will keep him nil per mouth, I will pass a small tube through the nose to stop
secretion from the stomach and decompress it. I will put IV line, we will take blood for UCE
(mineral body salts) and glucose (sugar), and give fluid to secure his feeding.

-he will undergo an urgent surgery by an expert staff. This surgery has a very good outcome and
it is a safe procedure. You can come in the morning and leave in the evening. You can breastfeed
your baby immediately after surgery.

-would you like me to call somebody for you? (Husband is overseas)


-do you have any kids at home? (Yes 5 years old child). I can organise social worker to look after
him/her.
- will arrange for the lactation nurse to help you on how to express your breast milk.

Q1/ what if they do not do the surgery: his condition can worsen leading to minerals
disturbances, reduction in blood sugar level, choking and dehydration.

Q2/ can it be serious


there is a chance of recurrence of his condition after the surgery so at any time if he gets painful
or distended tummy you need to go back to ED immediately.

Case (27/4/2017) (CASE 1)


a midwife trainee comes and is concerned about the condition of a newborn (born yesterday)
who vomits frequently.
Task:
1- ask further history from the midwife trainee
2- ask the relevant physical findings and investigations from examiner
3-explain the most probable diagnosis for the midwife
In this station, I think the role player is from medical field. She knew medical terms.
When I introduced myself and asked her about today’s concern.
She said that a baby was recently born and got vomiting and she is here for the condition. I
heard she said that you did the x ray and how is the result.
I reassure her and asked more history like vomiting, onset, duration, frequency, vomitus.
When I said about vomitus like ‘does the vomitus go away from the body’, she replied , it was
projectile. It was funny. Then, asked routine question like BINDS and mom’s condition.
* pf
- vomit every time feeding, projectile,
- no fever, - have passed poo, - no changes in pee. - Delivered at home ,
- no doctor checked, Birth wt 3kg,

OE
- need to asked detail about dehydration status -depressed ant: frontanellae+, sunken eyes +
,dry mucous membrane + , skin turgur +.
- when asked about any congenital abnl , examiner asked detail , i said i would like to check any
features of down syndrome like upward slanting of eyes +, any low set ears +, single palmar
crease +.
- no problems in genital exam ,
when asked about X ray - big black shawdow in stomach area, no other tests are avaliable)
I ordered some investigation from examiner like blood tests for FBE and electrolytes level-not
done. I asked USG-not done. I asked X ray-he gave me the film.
I drew a pic to nurse and explained that I found the condition why baby is vomiting. I also found
some features of congenital abnl like Down’s syndrome. So, the baby got some features of bowel
problem, could be a condition of oesophageal atresia, or it could be a condition or duodenal
atresia which is narrowing of bowel called duodenum. That’s why baby got vomiting. This kind
of condition can be commonly found in baby with congenital problem like Down’s syndrome.
Reassure the nurse that the baby is in safe hand. I will admit the baby to hospital as baby need
further assessment by specialist.

He will be safe. Reassure…

FB-PASSED, OVERALL-4, HX-4, ASK PE-4, IVX-4, DX\sDDX-5.

Feedback Case 2
Case (31/3/2017)
Sbo in infant
You are a HMO and a nurse comes to you to discuss about a 3 day old infant vomiting.
Tasks
A) HX
B) PEFE
C) Xray from examiner and explain to nurse D) DDS
The baby has bilous vomiting now , pee is fine no poos yet BINDS ok. PEFE dehydrated no jaundice anus
patent, abdomen distended
XRAY was SBO
36- Subfertility
27 year old Janet is your next patient at your GP. She tells you that she had been trying to fall
pregnant since more than 1 year now but she could not and she is quite worried about this.

TASKS
1.Appropriate history
2.Examination findings from examiner
3.Relevant investigations
4.Management

Subfertile - if less than 35 y/o, unprotected sexual intercourse x 12 months; if >35 y/o
unprotected sexual intercourse x 6 months or more

Differential diagnosis:
-Unknown
-Infrequent sexual intercourse and ignorance about fertile period
-PCOS
-PID
-Endometriosis
-Fibroids
-Thyroid disorders - hyper/hypothyroid
-Hyperprolactinemia
-Eating disorders
-Stress, exercise
-Medical- DM, SLE, renal disorders
-Surgical- Asherman’s syndrome
-Medications- antipsychotics, spironolactone, chemotherapeutic agents
-SAD – marijuana
-Family history of subfertility

APPROACH

History
1-approaching+ confidentiality
-Hi Janet, I'm Dr. ___ your GP today. I can see from the notes that you have been trying to
become pregnant for 1 year now but you couldn’t. I just need to ask a few questions and some
of them would be sensitive and personal but let me assure you that everything we gonna talk
about will be private and confidential. Would that be alright with you?

2-5 P’s questions


Period
-when was your last menstrual period?
-Are they regular?
-How many days of bleeding and how many days apart?
-Have you had any heavy bleeding or pain during menstruation?
(Yes, regular. No problems with periods)
Partner (sexual)
-How long have you been in the stable relationship? (We've been together for 2 years now)
- Has your partner done evaluation for subfertility so far? (No he hasn't)
-Does your partner have any medical or surgical conditions?
-Any medications that he is taking? (None)
-Sorry to ask you this, but does your partner have any children from previous relationships? (No,
he has none)
-have you or your partner ever been diagnosed with sexually transmitted illnesses? (No)

Pregnancy
-have you ever been pregnant before?
-any previous miscarriages? (None)

Pill
-what type of contraception you were on before planning for pregnancy?

Pap or HPV screen


-when was your last pap or hpv screening?

3-Differential diagnosis questions:


infrequent sexual intercourse
-how frequent do you undergo intercourse? Do you live with your partner always? (About once
or twice a month or so cause we're not living together always cause he's working in something
and comedown once a month)

Unawareness about fertile period


-Do you know about your fertile period? (What is that doctor?)
-Fertile period is the time during which you undergo intercourse, there's a high chance of

PEF (PID, Endometriosis, fibroids)


-do you have any abnormal, offensive vaginal discharge? Any fever, back pain? (PID) (None)
-Any pain in between periods or during intercourse? (Endometriosis) (None)
-Any mass, lumps or heaviness that you can feel in your tummy? (Fibroids) (None)

PCOS
-Any recent weight gain, acne, excessive hair growth? (None)

Thyroid disorders
-Do you have any weather preference? How are your bowel habits? (None)

Hyperprolactinemia
-Any headache, blurring of vision, milky discharge from nipples? (None)

Eating disorders/ stress/ Excessive exercise:


-Do you think that you are excessively overweight? Do you try to reduce your weight through
crash dieting or excessive exercise? (Eating disorders) (no)
-What is your occupation? Do you have any stress at home or at work? (Stress) (I do office work.
None.)
-How much exercise do you do? (Excessive exercise)
4-General questions
past surgical and medical history
-Have you had any surgical procedures especially those done down below? (Asherman's) (No)
-Any medical illnesses like diabetes or renal disorders? (No)
Smoking/ alcohol/ Drugs or Medications
-Do you smoke, drink alcohol, or take any recreational drugs?
-Do you take any Medications or over the counter medications?
Family History
Do you have a family history of subfertility?

Thank you for those information, Janet. I'd just talk to my examiner and will get back to you
shortly.

Physical Exam findings from the examiner


1-General appearance
-what is the BMI? 23
-Any Acne, excessive hair growth?
-Any Pallor, lymph node enlargements, edema? None

2-Vital signs
-What is the BP and temperature? 110/70, 36.8

3-Quick respiratory and CVS examination

4-Thyroid examination

5- Abdominal examination:
Inspection
-Any visible distention? Any mass?
Palpation
- Palpable mass or tenderness? (No visible distention, no mass or tenderness)

6-Pelvic examination:
-Consent
with her consent and in the presence of a chaperon I need to do pelvic examination.
-Inspection of vulva and vagina
any bleed, discharge, vesicle, rash? (No bleeding or discharge)
-Speculum exam
is cervix healthy? Discharge and bleed? (No discharge or bleed from the cervix)
-Per vaginal exam (CMT+ Bimanual)
is there CMT? (None )
What is the size, position and mobility of uterus? Any Tenderness? (Uterus size is normal,
anteverted, mobile, no tenderness)
-Any Adnexal mass or tenderness? (No adnexal mass or tenderness)
7-Office tests
Urine dipstick
blood sugar level

Thank you for those information, examiner, I would like to go back to my patient.

Investigation
- I would also like to arrange for some investigations such as FBE, U&E, LFT , TFTs, serum
prolactin
-urine MCS (microscopy culture and sensitivity)
- Mid luteal hormone assessment (21st day serum progesterone; if >3nanograms/mL it means
lady is ovulating) (Key issue)
- FSH, LH, estrogen, progesterone
- transvaginal ultrasound (Key issue).
- If everything turns out to be normal, we need to arrange for a hysterosalphingogram. It is done
to check the patency of the tubes. A dye will be injected through the cervix, and we will look if
the dye is passing through the tubes or not. (Note say this only if everything normal)

Diagnosis and Management


-As far as the details you have given me and after examining you, I could not find any organic
reasons, the most likely cause is infrequent sexual intercourse and your ignorance regarding the
fertile period. So I'll advise you further on this.

-If you are planning for a pregnancy, it would be better to undergo intercourse 2-3 times/week.

-To find out about your fertile period or whether you're ovulating or not:
1-you can get ovulation kits from the pharmacy which are urine kits and just like a pregnancy
kit. You can start testing from the eleventh day of your periods.
2-Other methods by which you can detect your ovulation is by basal body temperature method.
For this you need to get a basal body thermometer from the pharmacy, you will also get a chart
along with this, and you need to record the temperature from your armpit, every morning
before you get out of bed, and record it on the chart. A rise of 0.2C for 3 days over the previous 6
days temperature will tell you that you're ovulating.
3-Next is cervical mucus method, You need to observe the cervical mucus everyday so at time of
ovulation the secretion will become thin, more in amount and lubricative. After ovulation it
changes to thick mucus .

-It is always better to treat both partners so when you come for your next appointment I would
like to see your partner as well.
-I will also give you reading materials about subfertility.
-if none of these methods work then I can refer you to infertility clinic.
-review when results of Ix also come.
Only Notes
If it is due to PCOS, you need to do lifestyle modifications. I will refer you to a dietician and you
need a structured exercise program. You need to continue this for 6 months. If it is not working, I
will refer you to specialist who will start you on ovulation-inducing agents like clomiphene
citrate, and give you metformin along with that. If that still doesn't work, you can undergo a
surgery we call laparoscopic ovarian drilling. We put multiple holes in the ovary through a
keyhole surgery and that itself can induce ovulation as it drastically brings down the testosterone
or the male sex hormone production.

If it is due to fibroids,
we need to do surgical management such as myomectomy or uterine artery embolization.

If it is due to endometriosis,
we need to do a laparoscopic excision of endometrial deposits or we can burn away the deposits
with electrocautery or laser.

If it is due to thyroid disorders, treat the disorder with medications.

If it is due to hyperprolactinemia,
-look for cause, either a micro or macroadenoma (<1cm micro, >1cm macro), use of dopamine
antagonists like antipsychotics, or hypothyroidism.
-Refer to specialist,
-Do a serum prolactin, TFTs, high resolution MRI.

-Pituitary microadenoma - treat by giving cabergoline or bromocriptine and review by measuring


serum prolactin and MRI imaging.
-Pituitary macroadenoma - do a transsphenoidal approach to remove the macroadenoma

-If due to Asherman's (unexplained uterine adhesions usually following a surgical procedure
within the uterus)
treatment is 3 steps
1-hysteroscopic removal of adhesions under antibotic cover,
2-then prevent adhesions by inserting a IUCD or Foley's catheter to keep the uterine cavity apart
until healing takes place.
3-To regrow the endometrium, we need to give you estrogen.

-If both tubes completely blocked, your option is to do an in-vitro fertilization. IVF is when
fertilization occurs outside the female body. It is done by a specialist. You will be put in
ovulation-inducing drugs such as clomiphene, and once the eggs are mature, it is taken out and
combined with the sperm from your partner in the lab. Once embryo happens, a healthy embryo
is chosen, and is implanted into the uterus. If only 1 tube is completely blocked, 1 tube patent,
we could do a GIFT procedure. It is a gamete intrafallopian tube transfer. Fertilization takes place
inside the female body. Introduce a healthy egg and a sperm into the patent tube and allow
fertilization take place in the tube. Once embryo is formed, it will be implanted in the uterus. We
could also do a ZIFT procedure. IT is a zygote intrafallopian transfer. Here fertilization happens
outside the female body. And once the embryo is formed, it is introduced into the patent tube.
Then it will travel down the tube and get implanted in the uterus.
Feedback 23-6-2018 FAIL
Feedback: Conception Difficulty: female pt, been trying to get pregnant for 1 year. Take Hx,
Investigations, Counselling.
Taking Hx:
- How are you doing? Could you tell your concern? To find out the possible cause which may
cause your infertility I need to ask you several questions which may be private, is that OK?
- I started ruling out possible causes of female infertility: eating disorder, excessive exercise, DM,
thyroid issue, POF, PCOS, Pituitary adenoma, PID, Endometriosis,
- Male issue: is your husband alright? Does he has any diseases related to this issue that you
know of? Do you or your husband any family or children before you guys?
- Sexual activity: how often? (ONCE a MONTH)? Why? (COAL MINER)? Any problems happened
during sexual performance? (NO)
- 6Ps., especially any miscarriages? (NO)
- SADMA.
- Medical, surgical Hx Family Hx.
I said: the most likely cause your infertility so far is decreased frequency of sexual intercourse.
Normally, we need 3 times per week to have good chance of getting pregnant. However, I need
to carry out many investigations to rule out other causes: USD to check if any uterine
abnormalities, hormone tests, GSL, TFT, maybe Ctscan abdo/pel to find out any hidden causes.
And I may do some test on your husband’s sperms, USD,..
Counselling:
- While waiting the test results to come back, I recommend you to increase the frequency of
sexual activities.
- If any tests turn out positively abnormal, I refer you to the specialist to treat you accordingly.
- Otherwise, I would like to refer you to infertility clinic where MDT will help you. Don’t worry
There are a lot of hope.
The examiner and the pt were very happy and gave me a big smile.
Grade: FAIL GS: 2
All key steps: no,no,no,no
Hx: 2, Investigations:2, Counselling:2
SHOCKED, I thought I definitely passed this case. What did I do wrong?
Feedback 27-10-2018 FAIL
25 year old female patient came to your GP. She has been trying to fall pregnant for more than a
year. height and weight was given in the stem.
Task
- further relevant history
- investigation that you would arrange
- management history
- never been pregnant before
- period history (normal)
- sexual history
sexual intercourse → 1-2 times / month STI symptoms (-), past history of STI (-)
- doesn’t remember what type of contraception she was on.
- having unprotected sex for 1 year as they are trying to get pregnant
- however, unaware of her fertile period
- she is generally healthy
- not taking any medications
- no symptoms of PCOS, thyroid, prolactinoma, no procedure done down below
- sticking to normal healthy lifestyles
- up to date with HPV screening.
- husband is also healthy and doesn’t have any children from previous marriage.

Investigation
- told her that I will run some test to check some hormone level in your blood.
- then, i changed my mind and told her that i will arrange those test in the next visit
Management
- told her to increase her frequency of intercourse (3-4 times/week)
- told her about ovulation kits and basal body temperature methods
- told her to try this for 6 months and will review her again
- if she still couldn’t get pregnant in 6 months time, i will arrange some tests (does not
mention name of the tests in details)

Grade - fail
Global score - 3
Key steps
1,2,3, 4 - Yes, No, No, No
History - 3
Choice of investigation - 2
patient counseling - 4
37-Mitral stenosis in pregnancy
You are a Gp when a 25 year old woman who is 10 weeks pregnant comes to see you. She has
done all the first antenatal visits tests with another GP, which are all normal. So she has just
moved to a place nearby your practice from another town the previous GP had picked up a
murmur during her first antenatal check and told her that she needs to be assessed further.
TASKS
Take a further history (5 minutes)
PE from examiner (card)
Discuss your diagnosis and management with the patient

Differential Diagnosis:
Physiological murmur
Anemia
structural heart defects
Infections (infective endocarditis)

History
1-current early pregnancy questions
-How is your pregnancy so far
-any tummy pain, vaginal bleeding or discharge

2-Murmur Differential diagnosis questions


Anaemia
-any dizziness, tiredness or funny racing of the heart?
-how’s your diet? Does it contain meat and green leafy vegetables?
-have you had a history of heavy periods?
-any bleeding?
-is this your first pregnancy? How many pregnancies have you had? (to rule out spacing)
Infections
-any fever?
-any surgical procedure recently? Especially tooth extraction?
Cardiac causes
-any chest pain,
-any Fainting episodes?
-any shortness of breath? (Mild SOB since pregnancy started)
are you SOB now?
Are you Short of breath on exertion or rest or both? (Walks or work)
anything make it better or worse? (Rest)
3-risk factors questions
-any previous heart disease?
-any previous medical or surgical illnesses? (Yes I had joint pain , sore throat , and fever when 8
years old admitted to hospital and given medication)
-what treatment have you had at that time? (antibiotics)
-was it a long-term treatment or not? (Took for 3 years then stopped)

4-regular visit questions


-early symptoms of pregnancy?
-how was your blood tests? Blood group?
-have you been advised of down syndrome screening? (no)
-any previous ultrasound?
-do you take folic acid?
-SAD?
-support?

Physical Exam from examiner


1-General appearance:
pallor, Dyspnea, cyanosis, clubbing, lymphadenopathy, rash

2-Vital signs: HR, RR with O2 saturations, BP with postural drop

3-Focus CVS system examination


-JVP
-visible pulsation
-apex beat/ thrill and heave
-S1 and S2 (loud S1)
- added sound S3 , S4
-murmurs (low pitched rumbling diastolic murmur best at apex with little radiation in left
lateral position)

4-Respiratory:
- Excess work of breathing like nasal flaring, tracheal tug, overuse of SCM or pectoralis.
Intercostal or subcostal recession)
-Air entry, breathing sounds and adventitious sounds
-basal crepitation

5-Abdomen:
-hepatosplenomegaly, soft or tender, mass

6-pelvic examination
-inspection (vulva and vagina) bleed, discharge, vesicles, rash
-speculum (cervix) healthy or not
-bimanual examination: cervical motion tenderness, uterine size and tenderness, adnexal mass
and tenderness

7- Office test: UDT, BSL, ECG (ECG not available)


Explanation
-From history and examination I think you have a structural defect in your heart called mitral
stenosis
-Draw a diagram
The heart has 4 chambers, two upper and two lower chambers. Blood normally flow from upper
to lower chambers and this is controlled by valves. The valve on the left side called mitral valve
and in mitral stenosis the opening of this valve become narrow so blood does not flow freely
from the upper to lower chamber of the left side and this causes a back pressure on your lungs
causing shortness of breath.

-the infection that you had during childhood could be a rheumatic fever and this can affect the
valves of your heart if you do not take a long-term treatment with antibiotics.

-And why you are SOB now is due to pregnancy which is a hyper dynamic state with increased
volume of blood so the heart need to pump more blood.

-This is considered a high-risk pregnancy as you can develop complications like heart failure and
the baby can also go for growth restriction. But you do not need to worry about complications at
this stage because you will be working by a MDT trying to make your pregnancy and baby as safe
as possible.

Management
-I will refer you to a cardiologist, and he will arrange further tests like echocardiography which is
a scan of the heart and its blood flow, for the confirmation of the diagnosis and assessing the
severity, and the functioning of the heart.
Further blood tests also would be arranged if necessary.
just in case you need any medications the cardiologist will put you through.

-I will also refer you to the high-risk pregnancy clinic where you will be looked after by MDT
consisting of obstetrician, cardiologist, pediatrician and anesthetist.
you need to go for more frequent antenatal visits. Like US at 18 weeks , sugar test 28 weeks,
repeated US at 32 weeks, bug test at 36 weeks.
I will give you referral for down syndrome screening now.

-as far as delivery goes unless severe mitral stenosis or heart failure happens, you can have a
normal vaginal delivery, but it should be in a tertiary hospital under the guidance of the
specialist. certain precautions will be taken like:
1-continuous monitoring of your vitals and continuous CTG for the baby as well.
2-You will be given oxygen during labor and after delivery.
3-You will be offered a stress free labour with adequate pain relief in consultation with the
anesthetist.
4-Usually the second stage of labor is cut short by using instruments such as forceps or vacuum.

-Try not to stress at the moment take some rest, limit your activity and have a healthy diet.
-reading materials
-review after she sees the heart specialist
38-Recurrent Candidiasis
You are at your GP when 32 year old Lisa presents to you with complaints of severe itching and
burning sensation in her vagina, with discharge. She tells you that this is the 4th time that this
has been happening for her during the past 3 months. The last time, her GP had done a culture
and it came out to be severe Moniliasis, and she was treated with Nystatin cream 100 mg
intravaginally for 5 days during each episodes.
TASKS
Further history
PE from examiner
Discuss further Investigations and Management with the patient

Note/ Recurrent candidiasis: 4 episodes or more in 1 year

History
1- discharge questions
-how long have you had this discharge form vagina? (Past 3 days)
-how many pads have you used?
-what is the colour? (White)
-is it smelly? (No)
-what is the consistency? (curdy, cheesy)
-any blood stained?

2- Associated symptoms.
-any burning or stinging on passing urine?
-how is your bowel habits?
-any ulcers, rash?

3-Causes questions
Medication non compliance
-I can see that you have given nystatin cream. Have you applied it at the correct dose and
number of days? (yes)
diabetes
-do you feel thirst? are you passing large amount of urine? Do you need to go to toilet more
frequently? Any history of diabetes? (ask them together use any..)
Immunosuppressive disease
-any loss of weight? Loss of appetite? Lumps or bumps around the body?
Antibiotics or steroids
-do you take any antibiotics or steroids medications
vaginal douches/ pessaries/ shower gels change
- Any vaginal pessaries or douches that you have used?
-have you changed your shower gels?
Tight clothes
Do you use tight clothing or tight jeans, panty hose
any recent weight gain? Any evidence of infections elsewhere?
4-5Ps questions
periods
-when was your LMP? are they regular?
Partner or sexual
-are you sexually active? Are you in a stable relationship?
-have you or your partner ever been diagnosed with STI?
-any pain during intercourse?
-any similar symptoms in the partner?
Pill
-what type of contraception do you use? (OCP micrpgynon 30)
-for how long? (5 years)
pregnancy
-any previous pregnancies? Any miscarriages?
Pap or HPV

5-general questions
-past medical and surgical history
-SAD

Physical Exam from examiner


1-General appearance:
BMI, pallor, lymph nodes, rash, evidence of infection anywhere in the body

2-Vital signs: temperature, BP

3-CVS/Respi

4-Abdomen:
inspection: visible distention, mass,
palpation: palpable mass and tenderness

5-focus Pelvic examination


Inspection of the vulva and vagina:
-discharge, color of the discharge, is it smelly.
-any swelling or erythema of the vulva or vagina (brick red vagina)
Speculum:
cervix healthy or not, discharge or bleeding, erythema of the cervix

Per vaginal: DON'T to avoid spreading the infection

6-take a high vaginal swab for MCS just to confirm tht this time she is having candidiasis and to
look for any resistant form

7-Office test: urine dipstick and blood sugar level (both imp), UPT
Investigations
Basic bloods: FBE, UEC, LFT (you will give an antifungal, you want to know if the liver is okay),

Explanation
-What you are most likely having is again Candidiasis. It is a common yeast infection that affects
many women and could be irritating and painful.
-It is caused by an overgrowth of a fungus called Candida albicans, which is normally present in
your vagina. The other healthy bacteria in the vagina, usually prevents the overgrowth of
Candida. However, changes in your lifestyle and other health conditions causes the yeast to
multiply rapidly leading to recurrent candidiasis.
-what I think the trigger factor in you is the combined pill. The estrogen and to lesser extent the
progesterone in the pill can favor the growth of the candida.
-so it is better to stop the OCP taking at the moment and during this time it would be advisable
to wear condom.

Treatment
-I will prescribe you oral antifungal to induce remission (fluconazole or itraconazole daily for 2
weeks or till the symptoms go off)
-next is to maintain remission using the same medication once weekly for next 6 months.
-have a good genital hygiene
-try not to wear tight jeans or pantyhose
-whatever you go for swimming remove wet clothes ASAP.
-better to avoid sexual intercourse till symptoms go off.
-reading materials
-review 2 weeks
Feedback (23/2/2017)
Gp, young lady with vaginal discharge.
Tasks:
1. Hx,
2. PEFE,
3. Probable diagnosis and DDx.

2 minutes: Ddx: Candidiasis, Trichomonas, Cervicitis, Bacterial vaginosis, other STI

Entered room. Greeted by examiner.


Greeted and introduced myself, patient was a bit embarrassed as this was not her first time
having such discharge.
Reassured her she was doing the right thing by coming to see me.
Gave confidentiality statement during my hx taking as well.
Discharge questions: duration, type, color, smell, pain/sore at private part, itchiness, ulcers/rash
at private part.
Any lower part tummy pain/dysuria/lumps and bumps/fever.
5Ps, PMHx, SADMA.

PEFE:

GA, V/S
P/A: mass, tenderness,
pelvic examination:
-inspection – rash, scratch marks, ulcers,
speculum exam – vagina and cervix (white chessy discharge),
bimanual exam – uterus position and size, adnexal and CMT.

Office tests: urine dipstick, BSL

Dx: Dear Cindy, based on the information you have told me and the physical examination, most
likely you are having a condition called candidiasis albicans. Have you heard about it?
Don’t worry I will explain to you, it is basically a kind of fungal infection, and it is common in
women.

I would like to reassure you that this is not a sexually transmitted infection and it can be curable.
(tried to limit myself from telling too much as Mx was not my task). Do you understand me so
far?

It could be other types of condition as well by other bugs, or overgrowth of the usual bug in the
private part. I will give you reading materials about the condition.

Thanked examiner and patient


Feedback (27/4/2017)

a 25 y/o lady comes on combined OCP,with 4 times vaginal candidiasis in the last 3 months . The
last time has been proved by culture as severe monoliasis infection and has treated by nystatin
cream 100 mg for 5 days but immediately aftef stopping the medication it has recurred.

Task:
1- take detailed history from the patient
2- ask the requested physical findings and investigations from the examiner
3- talk about the measures that you want to do with the patient

woman around 30 c/o vaginal d/c, itchines, no urnariy symptoms, she is taking ocp for
contraception. This is 4 th time. 4 wks ago, u did test and showing nisillina (not sure), so u give
her nastytin 10,000 ng sth to her. She got the symptoms again soon after the treatment.

H/o, ask pe, explain the condtion, mx.


After reading the question, I was thinking about recurrent candida infection and need to stop
OCP and ask other risk factors.
I entered the room and start introduction and as usual, greeting. How are you feeling today .
She said that she still got the problem and a little bit upset.

I reassure her and said that it will be very distressing for you. So, I would like to ask you a few
more questions to clarify the condition.
I asked routine discharge questions and previous time, and the treatment she had received.
Ask 5ps. No other risk factors like medications, steroid, tampon use, DM, wearing of tight jeans.

*pf (she is married and use ocp , no multiple partners, no use of steriod , itchiness ++, no fever
or nausea or vomiting..

O/E; systemic are normal.


valva-redness + , no rash,
SSE- vaginal wall -healthy but whitish d/c +, BME -nl)
No BST available.

I explained her that it could be recurrent infection called candida infection. Reassure her it has
been four times. I also explained her that in case of recurrent infection, sometimes, it can be
associated with prolonged use of medications like OCP, steroid, and medical condition like DM
and certain risk factors like wearing of tight jeans. So, I want you to stop using OCP right now and
use other form of contraception like condom. I will prescribe you oral medication again for your
condition. I will also give you medication to relieve your itchiness like antihistamine cream. I also
explained about general vulva hygiene and avoid wearing of tight jeans. Give advices about red
flags and reading tips. I told her that I will follow you up regularly to check your condition. You
need to be patient, alright. If we work out together, we can overcome the problem. Reassure…

FB-PASSED, OVERALL-4, HISTOR-3, ASK PE-5, MX-4


39-HIV in pregnancy
You are GP when 30 year-old woman presents to you. She is 12 weeks pregnant and has come
for blood tests results. She has done her blood tests with another GP and as her usual GP is not
available today, she has come to see you.
Tasks
-short history
-explain results to patient
-immediate and long term
Blood tests
FBC, UCE, BSL (all normal) blood group (O+ve)
rubella and varicella antibodies (+ve) hepatitis A and B (-ve)
VDRL (-ve) HIV (+ve)

History
1-current pregnancy and visits questions
-how is your pregnancy so far?
-is it planned (DO NOT CONGRATULATE HER)
-any tummy pain, vaginal bleeding or discharge?
-have you done a dating scan?
-have you been offered down screen?
-do you take folic acid?
-blood group (no need to ask as it is already in the result but ask if it is not In the stem)
-is your Pap or HPV up to date?
2-Sexual history
-are you in a stable relationship?
-how long have you been in this relationship? (3 months)
-how many partners have you had in the last 6 months? (multiple)
-do you practise safe sex?
-what type of contraceptive do you use? (condom) do you use it regularly? (yes)
-have you or your partners ever been diagnosed with STI?
3-SAD
-do you smoke or drink alcohol?
-have you used any recreational drugs? (yes)
-any chance you taken it through your veins? (yes)
-any needle sharing? (yes)
-are you still taking recreational drugs? (stopped 3 months ago)
4-past history
-Past medical
-past surgical
-past obstetrics (previous pregnancies or miscarriages)
-other medications
5-support
Explain results (good news then bad news)
Good news
-I got the results of your blood tests.
-It showed normal blood count, normal kidney function.
-Normal blood sugar so no DM at the moment.
-You have O+ve blood group and that’s good because –ve one can carry risk to the baby.
-your body has got the power to fight against infections like german measles and chicken pox.
-no hepatitis or syphilis infections.
Bad news
-the next news is a bit concerning so have you been told why HIV screening has been done for
you and what will happen if it was positive. (Yes)
-unfortunately, HIV screening test has turned to be positive.
-Start crying so offer tissue box and glass of water.
-do you need to call anybody to be with you?
-are you happy to continue the consultation today.
Explain what is HIV
-HIV is a virus that can get transmitted through methods like unprotected sexual intercourse or
IV drug use especially if there is needle sharing.
-this is a transmissible disease but at this point you don’t have AIDS, you are just HIV+ve and
AIDS may not develop for 10 years or longer. (use may not rather than may)
-what I’m concerning is about risk of transmission to the baby which can happen during
pregnancy, time of delivery or breastfeeding.

GP role management
1-Confirm HIV
-the first thing I need to do is to confirm the diagnosis by doing another test called western blot
test. However once the screening test turned to be +ve there is a high chance that the diagnostic
one can be +ve.

2-Report
-Hiv is a reportable disease so my obligation as a Gp legally is to report this to the health
services. But your confidentiality will be maintained throughout.

3-Contact tracing + STI screen


-partner need to be tested for HIV and other STI so contact tracing need to be done.
you also require screening for other STI. And monitor also for hepatitis C.

4-Advice safe sex


-follow safe sex practise (condom during intercourse)

5-Refer to high-risk pregnancy clinic


-I would like to refer you to a high risk pregnancy clinic when you will be looking after by MDT
(HIV specialist, obstetrician, pediatrician, psychologist, social worker)

6-review and reading materials (tell this at the end of the consultation)
-I will arrange a review after specialist counselling and I’ll give you reading materials.
MDT management role
-HIV specialist will order further test looking for CD4 cell count, viral load, HIV resistance test.
-will start you on antiretroviral agent which you need to take at the correct dose all throughout
the pregnancy, delivery and after delivery.
-the risk of transmission to baby is reduced by 2-5% if you take medication. If not the risk can be
25-30%.

-you need to go through more frequent ANC, blood check, sugar test and ultrasound.
-the specialist might opt for CS to minimise the transmission of HIV to the baby at time of labour.
-after delivery, the baby will be monitored by pediatrician and HIV specialist and put on
antiretroviral agent at least 1st 6 weeks.
-you will be advised not to breastfeed your baby as breast milk might contain the virus.

-both you and your partner will be offered counselling services


Case (4/10/2017)
Antenatal care
Scenario:
You are a GP. Seeing this patient for the first time. 30year old female 12 weeks POA, your
colleague had done antenatal bloods. Patient had come back for results. Usual GP not available
today. You will be given a card inside the room with investigation findings.
Tasks:
• Explain results to mum.
• Explain management. Immediate and long term.
I was expecting something like rubella or varicella positive. So, it was shocking to see HIV positive
(Antigen and antibody). Even before you finish reading Ix card patient talks to you asking how are
the tests. I decided to handle it as a breaking bad news case (I myself was still in shock!) and
asked if she knew what tests were done. (no)
I summarised that they’ve done blood group (…+ve) and some checks for certain infections that
are important in pregnancy. Rubella and varicella is negative but there is one concerning
result…… waited for a while... Did you know HIV test was done? I don’t remember the specific
words I used but said that she is HIV positive, and waited for that to sink in … She acted very
worried but did not cry (there was a tissue box, but I did not give it to her.)
…. Then I said something to show empathy… and gave hope saying that HIV is not what it used to
be, as nowadays there are very effective antiretroviral drugs…
Then I asked her I can explain things in more detail regarding her further care now or does she
wish to wait or want someone else to be present. She said its ok to continue.
Then I assessed her knowledge on this stated this does not mean she has AIDS… Asked a bit of
history- gathered that she has had casual partners before current long-term partner but always
used condoms in those encounters. Baby was unplanned but now they are happy to have the
baby.
IV drug abuse – when she was a teenager shared needles!
Current smoker and drinks alcohol (not much) – Addressed then and there but postponed more
discussion � Felt overwhelmed as so many issues kept coming up. ☹ I think she was already on
folic acid…

Management:

Will refer her to high risk pregnancy clinic where infectious disease specialist and obstetrician
will look after her with frequent clinic visits. They will do further testing to confirm HIV and see
the viral load as well. More Ix to check for other STI and blood borne viruses. Possibility of
vertical transmission explained but it’s not a must and means of minimising the risk will be
discussed with her and starting of antiretroviral drugs will be considered by ID specialist.

I totally forgot her partner or contact tracing…so I thought I lost the case, but with a worried
patient being led by her reactions is more important than covering all the points I think. I had to
talk slowly and show my empathy on and off, so this takes time…�

All 4 key steps covered


40-SLE Pre pregnancy counselling
Case
You are a GP and your next patient is 25 year old Mary, who was diagnosed with SLE 5 years
back. She wants to start a family and is now seeking your advice regarding her future
pregnancies.
TASKS
-Take history
-Counsel the patient

AMC exam Case


27 years old Lady who has SLE for 5 years. Now wants to get pregnant. Letter to GP from
Rhematologist 6 months ago said ANA, anticardiolipin, antiphospholipid are negative and
disease is well controlled on low dose of Prednisolone.
Tasks:
-Take history
-Physical findings from examiner
-Arrange investigations
-Management or Counsel the patient

History
1-I can see from the note that you have SLE and you would like to become pregnant. Is that
right? I appreciate your coming to see me. I’d just like to ask you a few questions would that be
all right?

2-SLE questions
-you have SLE for 5 years. Can you tell me what symptoms did you have when you had your SLE?
Like skin rash, joint pain? (yes, skin rash and joint pain)
(As it has already mentioned 5 years in the stem but if not ask for how long have you been
diagnosed with SLE?)
-do you have any of these symptoms now? (No)
-How long have you been symptom-free?
-when was the last episode of flare up? (2-3 years ago)
-Do you experience any symptoms of kidney problem? (No)

3-Medication questions
-I can see that your SLE is well controlled with prednisolone. So how long have you been taking
it?
-Any other medications you take? (No)
-have you ever had any Side effects from medication?
(as it has already mentioned she is on prednisolone. If not you can ask What medications were
you put on? How long have you been off the medication?

4-Well control questions


-do you have regular checkups with the specialist? (Yes)
-any hospitalisation due to SLE? (No)
5-5Ps questions
Periods
- when was your LMP? Is it regular? Any problems with your periods? (Normal periods)
Partner/sexual
-do you have good support? (good support)
-Any history of STIs?
Pregnancy
-When are you planning for your pregnancy?
-is this your first pregnancy?
-any previous miscarriages? (Never become pregnant)
Pill
-what type of contraception do you use?
Pap and HPV (up to date)

6-General
-Do you smoke, drink alcohol or take recreational drugs?
-How is your diet? Exercise?
-Any other medical illness?

Physical findings from examiner


1-General appearance
pallor, jaundice, rash, LAP, joint swelling.

2-Vital signs

3-Chest (air entry, dullness, abnormal breathing sounds, adventitious sounds)

4-CVS (JVP, heart sounds and murmurs, apex beat site, pericardial rub)

5-abdomen (distention, tenderness or mass)

6-Pelvic exam (examiner will tell you not available)

7-joint
-tenderness, joint effusion, warmth
-restriction of movements

8-office tests
BSl and UDP
Arrange investigation and counselling
?? Explain SLE
-Do you know what SLE is? (yes)
As you are already aware, SLE is an autoimmune disorder in which certain factors called
antibodies develop against your own cells leading to inflammation of the body tissues especially
the skin, the joints, and others.

Reassure
-I got a letter from your specialist with all the investigations normal so let me assure about it and
as you haven’t had any flare ups for the last 6 months so you can get pregnant.

SLE on pregnancy and pregnancy on SLE


-In majority of cases there will be no flare-ups of SLE during pregnancy especially if you have no
symptoms for the last 6 months so flare up in your case is unlikely. However, if flare up happened
it would be just mild to moderate.

- let me assure you that many women with SLE can go for a normal pregnancy and delivery. But
sometimes, SLE can affect the mom and the baby:
To mom:
*20% miscarriages
*20% hypertension in pregnancy.
*can go for preterm labour.

To baby:
*Intrauterine growth retardation: this happens because blood clots can form in the placenta and
that can interfere with the nutrition of the baby
*Birth defects: especially if the mom is on medications other than prednisolone.
*Prematurity: if the mom goes in for premature labor
*Neonatal Lupus syndrome: after birth; it is not SLE in the baby. The baby can present with skin
rash and some unusual blood counts and it usually settles in 3-6 months time.

Arrange investigations

-I need to do blood tests like FBC, blood grouping and RH, UCE, LFT, RFT including GFR, vitamin
D, German measles and chicken pox antibodies, STI check with consent.

- along with that I need to arrange for thrombophilia screening like protein C and S, antithrombin
3, factor V Leiden, antiphospholipid and anticardiolipin antibodies , SLE antibodies like anti Ro
and Anti La, lupus anticoagulant.

-We can also do a urine microscopy culture and sensitivity, urine protein/ creatinine ratio.
Management
-Start you on folic acid 5 mg 3 months before you consider pregnancy, and for 3 months after
you become pregnant.

-refer you to the specialist rheumatologist to check if your condition is under control

-Once your pregnancy is confirmed, you will be referred to a high-risk pregnancy clinic. MDT
which includes the obstetrician and rheumatologist will be looking after you.

*-You need to go for More frequent antenatal checkups , regular ultrasound to monitor baby
growth, blood sugar at 28, bug test at 36 weeks, heart scan to detect fetal heart block!, more
screening if needed, and you will be carefully monitored for any complications.

*-During the first and second trimester they will do monthly FBC especially platelets.

*-At the end of each trimester will repeat renal function test including GFR. Antibodies like
anticardiolipin and complements, urine protein/ creatinine ratio.

(regarding * *I’m not sure if we need to repeat these Ix now and again in this counselling we can
give a summary like the red star above them. However if you see you have time mention them)

-just in case flare up occurred in pregnancy. The specialist will decide to increase the dose of
prednisolone and you will be monitored for high BP and sugar levels.

-You need to follow life style modification, sow salt diet, Ca and Vit D supplementation, regular
exercise

-If everything is normal you can go for normal vaginal delivery .

-After delivery, there is a chance of flare up but you will be monitored for that. You can
breastfeed your baby.

-Don’t worry I gave you a lot of information, I gonna give you reading material regarding all of
these.

-I want to see you again when the results appear.

-Are you a member of the SLE association of Australia? If not, refer you to that.

Recall of 10-5-2018 and 6-7-2018


Case (5/10/2017)
Station 2 Pre-pregnancy counselling Pass
27 yrs old lady with SLE on prednisone wants to discuss with about future pregnancy. Her recent
blood tests are normal including Cardiolipin antibodies, C3 C4 level, dsDNA, ESR CRP, renal
function test.
Task: History, PEFE
Counsel the patients
Explain her what investigations you want to do

Introduced myself , acknowledged her concern and appreciated her effort to come to discuss,
then reassure confidentiality. Told her before i explained to her about pregnancy, would like to
ask few questions to get to know more about her. Then History of SLE - when diagnosed, follow
up, last appointment, how well controlled, last episode of flare up was 2 or 3 yrs ago, now she is
doing ok, no fever, rash, jt pain. Then 5 P ( never pregnant before, normal period, dont
remember what contraception she is using, partner is supportive, pap smear is normal). PEFE
from examiner was everything normal ( i asked for pelvic examination as i just wanted to do
inspection but examiner said not available, oops ).Then tell her about the blood tests which was
done and reassured her that they were normal. Told her that since her last flare up was more
than 6 months ago, she can get pregnant now.
Then explained to her about SLE on pregnancy and pregnancy on SLE effect ( that she could get
flare up during pregnancy, but it is very less likely and it could be mild flare up cus she hasnt had
flare up for over 6 months, she could develop complications like miscarriage, preterm labour,
Gestational HTN, preeclampsia, and baby could develop complications like IUGR, preterm
delivery).
I reassured her that most of SLE patents can go through normal pregnancy and normal delivery
and you wont be going thru this alone and we r here to support and help u. Then told her about
baby could develop Lupus like syndrome ( rash, heart beat abnormality) but it will go away
within few weeks but if it develped, there is high chance that baby could develop SLE in later life.
I am gonna start u on folic acid right now and will arrange to do blood test like ur basic blood
count FBE, inflammatory markers like ESR CRP, renal function test, LFT and all the infection tests
and SLE tests that mention on the stem. And once u r pregnant, i will refer u to High risk clinic
where they have Multidisciplinary team and u will be followed up very closely with blood tests
( RFT and SLE tests). Dont worry, i am giving you too much information right now, i will give u
reading materials about all of this. I dont remember if i mentioned i would refer her back to her
Rheumatologist or not.

Covered 3 key steps out of 4


History 5
Choice and technique of exam 4
Choice of investigations 4
Patient counselling 6
Global score 5
Case (28/11/2017)
Station 5 : Pre-pregnancy counselling - pass
Lady has SLE for 5 years. Now wants to get pregnant. Letter to GP from
Rhematologist 6 months ago said ANA, anticardiolipin, antiphospholipid are
negative and disease well controlled on low dose of Prednisolone.
-Tasks:
-Take history
-Arrange investigations
-Management
Key steps: 3 /4
History: 5
Choice of exm, organization and sequence : 5
Choice of investigation : 5
Pt counselling: 5
Global score: 5 – Pass
History:
rapport
When was dx?
How many flair ups
Tretment you are on
Last time you had relapse
Regular with specialist? Last time saw specialist?
atm any rash/joint pain?
Present drug ? what is the dose?
5ps in short (hx of any miscarriage )
fhx of sle
Arrange investigations:
-FBE, Blood grouping typing, Rubella antibody screening (I mentioned that I will
be doing all the inv that we normally do in the first visit of pregnancy earlier for
this patient before conceiving)
-Apart from inv mentioned in the stem I arranged other ones. (thrombophilia
screening: protein c, protein s, antithrombin 3, factor v leiden, anticardiolopin
antibody, antiphospholipid )
-Refer to rheumatologist and obstretritian before getting pregnant
-Complications:
Sle on pregnancy
Pregnancy on sle
-Reassuring the pt that she will be managed by a multideciplinary team (gp, obs,
rheumatologist), under high risk clinic)
-More frequent visits, more scanning if needed , carefully monitoring for
complications that may occur
-Delivery will be in a controlled manner under the obstetrin preferably in a
tertiary center.
-4Rs: SLe association aus.
Feedback 27-10-2018 PRE-PREGANCY COUNSELLING FAIL
Patient with known case of SLE is planning to get pregnant. Certain investigation has been done.
Task
- history
- PEFE
- investigation
- management

History
Details of SLE
- diagnosed a few years back.
- taking steroids
- last attack was 6 - 7 months ago
- no complications such as kidney Period (normal)
no past history of miscarriage

PEFE
- totally unorganized in this case.
office test
- urine dipsticks (protein) was arranged, but the result was not available

Investigation
- FBE
- blood group
- kidney function test
- Urea, electrolyte and creatinine

Management
- told her that I will refer you to your specialist to assess your condition again.

- they will review your medication and adjust the dose.

- patient with SLE should be symptoms free for 6 months to avoid complications during
pregnancy.

- if the symptoms are not well controlled during the pregnancy, it can lead to pre-
eclampsia.

- after consultation with the specialist and your condition is well controlled, I will refer you
to the high risk pregnancy clinic to prepare for the pregnancy.

- for the time being, you can start taking folic acid.

- reassure her that there are support group available.


• In this station, I was totally unorganized.

• didn’t explain how SLE would affect her and her baby in details if the condition was not
controlled.

• didn’t arrange albumin: creatinine ratio and GFR which are key steps in the investigation.
Grade - fail

Global score - 2
Key steps
1,2,3,4 - Yes, No, Yes, No
History - 3
choice and technique of examination - 2
organization and sequence
choice of investigation - 3
Patient counseling - 2
41-Respiratory distress syndrome counselling
AMC Case 20-4-2018
a lady who recently gives birth to a baby at 32 weeks wants to know about her baby’s condition.
(rural hospital). Baby has respiratory distress but O2 saturation improved with head box oxygen
Tasks
-explain baby’s condition to the mother
-explain Management
Explanation:
-I understand that you are so concerned about your child but I would like to assure you she is
stable now since we have given her oxygen by mask. Would you like some water and tissue?
-I will explain to you her condition. Your child has some problem with breathing. It can be due to
many causes but one of the main cause and most likely cause in your child's case is immaturity
of the lungs leading to problem with the expansion of the lungs. Normally, babies delivered after
34 weeks of gestation, their lungs are mature enough to take the burden of outside
environment. But in your case, since you delivered your baby at 32 weeks, the lungs of your baby
is not yet well developed and there is a problem with the expansion of the lungs. This condition
we call respiratory distress syndrome or hyaline membrane disease. But there can be other
causes like infection, stress of the delivery, among others.
-But since we are in a rural hospital, we have done everything to stabilize your child with the
facilities available here, but babies with respiratory problems needs to be transferred to a
tertiary hospital where there is an intensive care unit available and specially trained nurses and
doctors that could provide adequate care for your baby. I will liaise with the tertiary care hospital
and inform the specialist about your child's condition so that everything can be made ready once
you are transferred there. For your transfer, we have the PIPER team who is composed of
specially trained nurses and doctors who can take care of your child while in transit.
-In the tertiary hospital, the baby might be given artificially a substance called surfactant most
probably through the nose to keep the lung expanded, but it would be decided by the specialist.
At this stage, do not feed the child, we will maintain the nutrition through IV fluids. We will also
take blood for investigations such as FBE, UEC, LFT, TFT, CRP, ABG and blood culture. We will also
give the first dose of antibiotic after the discussion with the specialist in the tertiary center. We
might also do an x-ray of the chest, to rule out other causes and to confirm the diagnosis. Please
do not stress yourself, exposure to radiation is not very high and will be appropriated with the
child's weight.
-In the meantime, if you want me to call someone, I am happy to call your partner. Do you have
enough support?
-Can I go with the child doctor? It depends on your condition, and the obstetrician advice. If the
obstetrician is happy to let you go, then we can make all the arrangements for you to go with the
child.
-I would like to reassure you that many of premature babies are doing good after treatment has
been institute. We will do our very best to manage your child's condition.
20-4-2018
RDS- counselling the mother - this was her first new born and was in the
rural hospital. The mother was crying throughout but i was able to calm
her down and answer her questions.
she asked if she can see the baby. Is her baby going to die and what
exactly happened to the baby.

20-4-2017
A woman was visiting her husband in rural region far from tertiary hospital when she went into
labour at 32 weeks, the baby was delivered with no complications at all during delivery. The
baby developed breathing difficulty soon after birth and was placed in oxygen head box with
40% oxygen and is fine and stable now.

Mother is very anxious and wants to know what's going on with her baby. She had a miscarriage
2 months before this pregnancy. She is very concerned and distressed about her baby. Your task
is to talk to her and explain what's going with the baby and what your plan is.

For the last 2 questions you can ask like the material or this is also a comment from someone in
the group
-Ideally main points would be: you should tell her that as she has delivered the baby recently
you would like to examine her, and consult with your senior. Then when she is fit to be
discharged she can meet the baby.
-IN the main time you can arrange to call her relatives in her behalf, lactational nurse as she
won't be able to breastfeed, social worker if any other kid.

Feedback 20-2-2018
HMO rural hospital, tertiary hospital 150km away. A young woman had a preterm delivery at
32weeks few hours ago. She visited her family at the farm and had PPROM and followed by
preterm delivery by midwife. Soon after delivery the newborn is having reduced SPO2 and
currently being supported by oxygen head box. This is her second baby and she had a
miscarriage for her first pregnancy. Her husband is in the hospital but at the moment he is not
around her.
Tasks:
-Explain the newborn’s condition to the mother
-Explain what is the immediate management for the newborn

2 min thinking: I had mind block. Told myself to calm down as the mother will be very anxious
and teary. Just go along with her.
Introduction. Patient is having a tissue and very worried that she will lose her baby. Keep
reassuring her until she is a bit better. Then explain to her briefly about preterm baby and the
risks along with it (respiratory distress due to immature lung, other possibility like heart defect,
diaphragmatic hernia etc). What we will do right now is my senior is contacting the newborn
specialist in tertiary hospital. As you know we are in a rural hospital where advance facility like
NICU is unavailable here. We will need to transfer your baby via air ambulance. (Can I go with my
baby?) - yes you can and we will escort her as well. There at tertiary hospital your baby will be
seen by specialist who is experienced in handling preterm baby and we will make sure that we
do all we can to keep your baby safe and alive.
(I was nervous and had a mind block in terms of what else to tell her, so I kept reassuring her and
repeat what I said)
AMC Feedback – Health review: PASS

(Management plan 3 – please refer to https://www2.health.vic.gov.au/hospitals-and-health-


services/patient-care/perinatal-reproductive/neonatal-ehandbook/conditions/respiratory-
distress-syndrome
Or other reliable source for management plan)
(Most likely this is a scored station)
Here is another better approach by Dr Ahmed Eltaib (AMC clinical exclusive):
I understand you are so concerned about your child but I would like to assure you she is stable
now we give her oxygen by mask. Would you like some water and tissue? .. As you know we are
in a rural hospital and we did everything we can in this facility and because your daughter born
premature she needs special care this why we will arrange her transfer to the tertiary hospital
where the NUCU (neonatal intensive care unit) available with highly trained doctors and nurses
who can deal with small babies like your child. At the NICU they will give her substance called
(surfactant) which will help her lungs to expand more so she can breath easier . At the moment
my senior is speaking to the specialist on the phone arranging air ambulance for her. We will try
to secure IV line for her and start her on some fluids to give her the essential nutrients for her
body and specialist may ask us to start her on broad spectrum antibiotic. You can go with her
and you will know everything about her condition in details by the specialist. I would like to
assure you that many premature babies even more premature than your baby are doing so good
and they have normal life.
Anything you would like to ask about?
Recall 12-4-2018
In a small city you are working as a HMO in hospital a 32 year old lady in 32 w of her first pregnancy
delivered a baby.
your colleague saw the pt and put him in ancubator with 40% o2 is stable now.
mom is so concern about child.
Task:
* further Hx
* counsel mom
* mx
in the room mom was so concerned lying on bed and irritable and some of the time closed her eyes
and I thought she is pt and twice checked my tasks
her husband was 10 km away. has done all ANC and no any problem.

Feedback 5-12-2018
Sandy (Mom) gave birth to a baby girl at 32 weeks in rural hospital by ur colleague. Baby having
respiratory difficulties and 40% O2, Now on head box and stable (mom cried all throughout
consultation) and was seen by Registrar/senior. Tertiary care is 150 km away
Task:
1. Tell condition/ dx to mom
2. Advice further management plan.

When I entered the room I saw a young lady in hospital gown, she was crying.

I introduced myself, asked baby’s name (She said not yet) and said I can see that you are quite
anxious at this moment which is quite normal in this situation but let me assure you that we are
here to help you and your baby. I offered some tissues and water. She said ok but still crying
(Excellent role-player).

She asked why this happened, I told sometimes water in which baby floats break early before
onset of labor and sometimes labor sets in early before the due date I am not sure why they
delivered the baby but for sure there is some indication which I will check. Any shots given? Like
steroid? She said not sure. Ok, she was crying, offered tissue gave her some time to settle.

Then I asked are happy to carry on this consultation? She said sorry doc yes carry on.

I said let me assure you that your baby is already taken care of by senior doctors and consultant
and his condition is stable now.

Let me explain the current situation of your baby, So I said as your baby is delivered earlier than
dates all his organ are still not matured enough to function properly( she started crying more) I
said but let me assure you that there is something we can do to help him. So the condition he is
having we call it Acute Respiratory Distress Syndrome (ARDS). Do you know more about it? Ahe
said no. then I draw a picture of whole body with lungs and pointed out that its his lung and
there is some fluid usually surrounds the lung called Surfactant that helps it to expand properly
but due to immaturity this fluid is not sufficient enough.

I said Are you with me? She said continue, what will happen now?
I said as he is too small his condition can get worse anytime which we don’t want. He need
continuous monitoring as his temperature may not be stable, salt level should be checked,
glucose level as well and most importantly might need ICU support which is not possible in this
hospital unfortunately and also your baby will be taken care of Specialist and Experienced Nurse.
What do you think of that? She said yes I will do whatever you tell.

Then I told that I want refer him to Royal Children Hospital by air ambulance with special Nurse’s
company. Then she asked can I go with baby doc? I said I’m not sure about your condition so I
will ask my Registrar and check if can go with baby and it depends on the transfer facility as well.

So once your baby is in the hospital they will do some investigations give O2, and put him to
special box sometimes they put some air under pressure to make the small airway open we call
it CPAP and correct

salt balance if needed, specialist might pass some tiny tubes through the nose to lung and put
special substance called surfactant but let me assure u that they are expert in this.

Is there anyone with you at this moment waiting outside , she said my Husband , I said ok I will
talk with him as well if you want and make all the necessary arrangements and once you and
your baby come back I will arrange regular follow up with you GP for vaccination and growth and
development. She was still crying but thanked me….:) Bell rang.

Key Steps- 1,2,3- all covered

Global Score- 5
42-Bulimia nervosa (Dentist case)
Your next patient at your GP is 24 year old Alice refereed to you by her dentist because of
recurrent dental caries. She is otherwise in a stable condition. Her BMI is 22
Tasks
-take further history
-diagnosis to patient.
-counseling

History
1-Approach and confidentiality
-I can see that you are referred by a dentist because of teeth problems is that right?
(Yes, I do not know why I did not want to come to you at all. Only because my dentist told me to
come.)
-all right can I ask you a few questions in order to figure out what the problem is. it would be just
a short consultation. (Ok), and let me assure you that everything we gonna discuss will be
confidential between you and I unless there is a harm to yourself or others is that ok for you?
(Yes)
-So what sort of dental problem are you having (I asked this although mentioned in the stem) (I
have dental carries)
-is it under control (also mentioned in the stem) (yes)
-How long have you been suffering from this? (6-8 months)

2-Bulimia nervosa questions


-can you tell me briefly about your diet? (I think I am eating healthy diet)
-do you have any craving to sweet things? (No)
-have you ever lost control over how much you eat? (Yes, I take a lot of food at times and lose
control)
-do you feel guilty after that (yes)
-do you try to vomit or use any water pills in order to lose weight (yes vomiting )
-do you use any water pills or laxatives?
-how often do you exercise?
-what do you think about your appearance (I think I am overweight). for how long?
-anybody told you that you are overweight? (Yes my partner and he is in another state for some
business)

3-Check periods
-how is your periods? When was you LMP? Are they regular? Any problems with your periods?
(Period ok and regular).
4-Psychosocial history and HEADS
mood questions: (all good)
-how is your mood
-have you ever thought of harming yourself or others
-how’s your sleep
-do you find it hard to concentrate on things.
Delusion and hallucination
-I am gonna ask you certain questions that could be a bit sensitive but theses are just routine
questions I ask to some of my patients. (-ve)
-do you feel, hear or see things that others don’t?
-do you think someone is spying on you or trying to harm you?
Insight and judgment
-do you think you need any professional help? (No)
-if there is fire in this room what would you do? (runaway).
HEADS
-any support at home, any stress?
- What do you do for living, any stress at work?
- have you lost interest in things you used to enjoy?
- SAD
- how is your social life? Do you catch up with friends quite often? (Do not like to get out as she
thinks they will bitch behind her because she is overweight)

5-General questions
-past and family history of mental problems
-past medical and surgical history
-medications and allergies

Explain the diagnosis


-all right thank you. So from the history you most likely have a condition called bulimia nervosa.
It is one of the eating disorders when you think that you are overweight but in reality it is not as
your BMI is falling within the normal range.
-And because you think you are overweight even though you go for a control diet, at times you
lose control and eat a lot of food then feel guilty about this, so try compensatory behaviors like
vomiting or others.
-Keep going like this can cause some problems to your body. You already have dental caries,
which could be because of vomiting that you induce as the acid in the vomitus can damage the
enamel of the teeth. It can also cause irregularity of your periods, disturbance of hormones and
can cause loss of water and salts from your body, which can adversely affect various systems
including the heart.
But let me assure you that you are in safe hands we will manage you properly to ensure you will
be in good health.
Management
-management is usually a Multidisciplinary team.
-First, I would like to refer you to a dietician who can assess your nutritional status and give you
a proper dietary chart to help you attain the weight that is ideal for your age.

-also, I would like to refer you to a psychologist who will give you CBT which is a type of talk
therapy which will help to modify your thought which in turn will control your actions. It will also
induce a sense of positive thinking in you.

-and I will follow you up regularly as your GP

-also I would like to arrange family meeting with you consent and meeting with your husband
after return from travel to make him aware about you condition as you need support at this
time.

-give numbers of bulimia support groups.

-if all of these do not work then I can refer you to a psychiatrist who can start you on a
medication SSRI.

-reading material

-red flag if you develop chest pain, funny racing of the heart, fainting attacks please report
immediately
( what doctor? do you mean if I developed these I will come back to you? )
No it is better to go to the ED
Case (28/11/2017)
Station 7: Dental complaint – pass
Young lady has been referred to you by dentist for bad dental condition and for
concerns of self induced vomiting and calluses on hand. BMI 24.
Tasks:
-History
-Explain diagnosis to the patient and management
Key steps: 6/6
Approach: 5
History: 4
Dx and DDx: 5
Pt counselling and edu: 4
Global score: 5
History:
Asked why are you here? – she said she never wanted to be there, and it was the
dentist who insisted on seeing me. I built the rapport by reassuring this
consultation will be short and in attempt to help her in anyway.
Started with confidentiality
Started with bulimia questions.=> Can you describe your eating habbits? How is
your appetite? Can you describe your daily diet.? Could you tell me about your
perception about wt and body image? “Binge eating” at times? Loose control
over how much you eat and feel guility about eating habits? Habit of forced
induced vomiting.? Habit of taking laxa†ivies?(not positive) Any medicine to loose
wt?(-) Excessive exercise?(-) Since how long doing all these things?
Periods queation.
Headsss
Along with suicide, guilty, harming yourself or others, life is worth living (rule out
depression)
ASEPTIC (sometimes people under stress, can see/hear/feel things that other
people don’t)?
Past history (similar problem, in past have you ever diagnosed with any
psychiatric complain, child abuse)
Family history of similar complaint.
Diagnosis: eating disorder like bulimia nervosa, anorexia nervosa,
your case most likely bulimia nervosa as you eat and vomit a lot. We do see
people with similar complaints in practice. Please do not worry, I will
tell you in detail what to do. It natural to feel that we need to loose wt
and its always good to remain fit. But it should be in the proper way,
otherwise it can do more harm to your body then helping you out.
This can also be associated with any stress. Depression or any other psych
problem, but from your history it doesn’t seem you have any other psych illness.
For this I would like to refer you for a counselling session to a psychologist
for a talk therapy. There are people who can help you to overcome these
problems. I can also refer you to a dietitian who can chart out a good diet for
you without doing for binge eating and vomiting out and any such activities. I
want you to realise that what you are doing is not a permanent solution to your
problem, infact it is brining so much complications like the dental carries you
have now and also sometime because eof this vomiting there can be some
imbalance of the electrolyites that is the vbasic salts present in your body
leading to further complications.

43-Pubertal Menorrhagia (Anovulatory DUB)


Your next patient at you GP is 14 year old Maria, brought in by her mom Jenny. Jenny tells you
that Maria had just had her first period and she is having a heavy bleed .
TASKS
-Take a further history from the mom

-PE from the examiner

-Diagnosis and differential diagnoses

Differential Diagnosis:
 Pubertal menorrhagia
 Pregnancy complications
 Infection
 Thyroid disorders
 Bleeding disorders
 Blood-thinning medications

History
1-rapport+ immediate action

-Introduce yourself and say I can see from the notes that you are complaining of heavy
menstrual bleeding is that right?
-Are you still bleeding?
-I just need to ask you further questions to know about the problem but before that I first need
to check your vitals; your pulse and blood pressure I’ll ask the examiner about it.
-Examiner Is my patient hemodynamically stable? I would like to take her vitals and measure the
pulse and blood pressure
- Note/ in the exam the examiner will say she is hemodynamically stable

Note/Just in case she was not hemodynamically stable say this:

-for example, the examiner said: She is pale, anxious and sweating. BP is 80/60, PR 90 bpm
regular, RR 24/min, Temp 36.7, O2 93%

-I would like to shift the patient to the treatment room and proceed with the DRABC protocol:

Call for help. Put in 2 large bore IV cannulas, take blood for investigations such as FBE, ESR/CRP,
UEC, B-Hcg, thyroid function test, blood group crossmatching and hold, coagulation profile

Start IV bolus, then as IV infusion

Start O2 by mask
-Once the patient is stabilized, I would like to proceed with the rest of the history from the mom

2-reassurance

-I have checked your vitals and they are all ok so let me assure you that you are stable now. I
would just like to ask you some questions in order to unravel the nature of the problem.

Note/ if she was in shock and you stabilized her


-Since you were a little unstable a while ago, we have done measures to stabilize you. you are
stable now. I would like to ask further questions about your condition.

3-bleeding questions

-Is this your first period? (Yes)


-when was your menarche or first menstrual period? (10 days ago)
-How many days have you been bleeding? (10 days)
-What is the severity of the bleed? How many pads has she used per day? Is it fully soaked?
(More than 4 pads a day)
-What is the color of the bleed?
-is it smelly? (No)
-Any clots? (one feedback say +ve!!!)

4-associated symptoms (pain+ anaemia)

-are you complaining of any pain in your tummy? (No)


-do you feel tired and dizzy? Are you short of breath? Any chest pain or funny racing of the
heart? (No)

5-Differential questions:

-Any weather preferences? Any changes in weight recently? How about your bowel habits?
(Thyroid disorder)

-Any history of bleeding disorders? Do you bruise easily? (Bleeding disorders)

-Any medications that you are taking? (Blood thinner medications)


-any fever? (Infection)
-Any trauma? (Trauma)
- Any elder sisters that you have? Any history of heavy bleeds in your sister? (Pubertal
menorrhagia) (Her sister had her first menstrual period at 14 years old)

6-5Ps
-period (already asked)
-partner: are you sexually active? (no)
-Pills
-pregnancy (no need)
-have you had your Gardasil vaccine (no answer from feedback)

Physical Exam (in the exam all normal, may be mild anaemia!!)

-General appearance: dehydration, rash, bruising or petechial, pallor, LAP, acne, hirsutism

-I hope that the vitals of the patient is now stable, so I would like to re-assess all the systems

-Respiratory - equal air entry, abnormal breath sounds

-CVS - normal S1 and S2, abnormal heart sounds

-CNS

-Abdomen - visible mass, distention, palpate for mass and tenderness

-Thyroid

-Pelvic: inspection of the vulva and the vagina any bleed? What is the color? Any clots? Rash and
vesicles? DON’T GO FOR SPECULUM AND PER VAGINAL EXAM

-Office test:
UDT, BSL
Urine pregnancy test (-ve)

Explain the diagnosis+ DDX

1-Pubertal menorrhagia
condition
-What you are having is most likely a condition called pubertal menorrhagia. It is a condition
where you get heavy bleeding during the first periods.
Cause.
-Usually there are many hormones that regulate the menstrual cycle, and a disruption in the
hormonal balance can lead to a heavy bleed.
Course.
-It will take quite some time for the hormones to come back to normal.
2-DDX
other possibilities could be thyroid problem, Bleeding disorder, blood thinner medications,
Infection, trauma, miscarriage. Say all unlikely and give reasons from history and examination.

Management (not a task)


Investigations: (from RCH)
FBE & Ferritin
Coagulation screen
serum ßhCG
Measurement of serum TSH to exclude thyroid abnormalities
Consider pelvic ultrasonography if accompanied by pain or palpable mass (to exclude structural
causes, such as fibroids, polyps, and/or ovarian tumors)

Active bleeding management


-because you are bleeding heavily and you have gone into shock, you require a referral to the
hospital and need to be admitted and be seen by the specialist.
Bleeding has to be stopped immediately by giving you IV Premarin 25mg (high dose of
conjugated estrogen). Even with a single dose, the bleeding can be stopped. But if not, you can
be given up to 4 doses. This will inhibit the access in the brain that controls the menstrual cycle.
But you have to be consequently given oral Progestogen for the next 14 days to compensate for
the high levels of estrogen (otherwise she will go in for a withdrawal bleed).

if no active bleeding or when active bleeding stops


-Once the active bleeding stops, we need to consider medications or treatment for the next 2 -3
cycles, as it will take around 2-3 months or even up to 6 months for the hormone levels to
stabilize. For this, we can put you on non-hormonal conservative management with Tranexamic
acid or Mefenamic acid, or hormonal management with combined oral contraceptives, which
regularizes her periods and makes it lighter and shorter as well.
These are the options that I can put her on for the next 3 months.

-provide her with adequate psychological support during this time and give her adequate
nutrition especially iron rich foods. (if you are talking to mom)
If iron is low, start her on iron pills.

Feedback 20-2-2018
GP, 17 years old lady came complaining of heavy menstrual bleeding.
Tasks:
-Take history
-Physical examination from the examiner
-Explain the probable diagnosis and differential diagnosis to the patient

2 min thinking: assess haemodynamic stability, 5Ps, menorrhagia ?cause (hormonal, bleeding
diastasis, medication), rule out ectopic pregnancy

History:
Introduce myself, ask the patient how does she feel? How long has she been bleeding and how
many pads? (this is the 10th day of period and ? pads for today). Then explain to her I need to
make sure she is stable by measuring her blood pressure and other vital signs and turn to the
examiner. (Examiner said she is hemodynamically stable).

More questions about her period: clots, foul smelling, pain in lower part of belly, associated
symptoms eg: light-headedness, tiredness, fever. BO/PU unremarkable.

Then asked about when did she had menarche? (10 days ago) I asked again – when did you had
your first period? (10 days ago). I was confused at that time, but decided to let it go and asked
other Ps – partner? (no), sexual intercourse (never), so I did not ask about pills, pregnancy and
PAP smear.

PMHx: hypothyroidism, bleeding disorder, medication usage – all unremarkable

I decided to ask her when she attained her menarche again. (10 days ago). Suddenly I realised
this is her FIRST menstrual period! I explain now I understood what she said and apologised to
her for causing her feeling stressed by asking the same questions repeatedly.

SADMA unremarkable, family history: mother and elder sister attain menarche at 14 years old.

Physical examinations:
All unremarkable
Asked about Tanner staging for breast and pubic hair growth – (Examiner said there are breast
and pubic hair growth)
Only did inspection on pelvic examination as patient is a virgin

Explanation:
Dear Judy, you are having heavy bleeding in your first menstrual period. It is quite normal for
some people to experience that. (I forgot the term puberty menorrhagia). It could be other
possibility like hormone imbalance i.e hypothyroidism, bleeding disorder however you do not
have any family history of this condition. Infection but it is less likely. Don’t worry too much, it is
manageable and first I will give you some medication to reduce the bleeding . (I did not proceed
to much in management as it was not my task). Assess understanding, reading materials,
review.

(Most likely this is the non-scored station)


Feedback 20-4-2018
RCH reading
https://www.rch.org.au/clinicalguide/guideline_index/Adolescent_Gynaecology_Menorrhagia/

Definition
1. excessive menstrual flow in its duration (>7 days) or its volume (equates to needing to
change a super pad/tampon more frequently than every two hours)

2. Bleeding causing symptomatic anaemia or lifestyle disturbance

Background

Menstrual cycles are often irregular in the first years after menarche.
Most cases of cases of heavy menstrual bleeding in adolescents can be caused by anovulatory
cycles, which is related to immaturity of the hypothalamic-pituitary-ovarian axis. Other causes
include pregnancy, infection, the use of hormonal contraceptives, stress (psychogenic or exercise
induced), under- and over-weight or weight changes, and bleeding disorders.
Less common causes of heavy menstrual bleeding in adolescents include systemic illness and
endocrine disorders. Structural lesions that cause heavy menstrual bleeding in adolescents are
incredibely rare (cervical polyps and uterine leiomyomas such as fibroids).

Assessment

History:

1. Menstrual history (menarche, last menstrual period, frequency, duration, flow, pain)

2. Bruising

3. Galactorrhea

4. Lethargy, headache

Examination:

1. Pallor

2. Evaluation for signs of androgen excess: hirsutism; acne;

3. Examination of the skin for acanthosis nigricans or signs of abnormal bleeding (eg,
petechiae and/or bruising)

4. Palpation of the abdomen for uterine or ovarian mass

Investigations:

1. FBE & Ferritin

2. Coagulation screen

3. ßhCG

4. Measurement of serum TSH to exclude thyroid abnormalities

5. Consider pelvic ultrasonography if accompanied by pain or palpable mass (to exclude


structural causes, such as fibroids, polyps, and/or ovarian tumors)
Management

 Single or combination of non-hormonal and hormonal treatment (for example NSAIDS &
Tranexamic acid & Progesterone) can be used depending on severity

Non-hormonal forms of treatment

 If anaemic or recurrent/severe bleeding: Iron supplements

 First line to decrease flow:

o NSAIDS (Naproxen, Mefenamin acid, Ibuprofen) - unless contraindicated. Can


decrease flow up to 30% if taken regularly during the first 48 hours of
menstruation

o Tranexamic acid (1 gram, every 6 hours) can decrease flow 50%, does not
reduce the duration of menses or regulate the menstrual cycle, needs to be taken
for 3-5 days following cessation of bleeding

Hormonal forms of treatment

 Progesterone (e.g. Norethisterone 5mg, Medroxy-progesterone acetate 10mg). Good


with anovulation (infrequent periods) due to the lack of progesterone

o Acute treatment: 5-10 mg x 21 days (N.B. Will bleed when ceased!)

o Prophylactic treatment: 7-10 days/month

 Combined oral contraceptive pill : can decrease flow by 50%. Good with
anovulation/irregular menses: often commence with Ethinylestradiol
30mcg/Levonorgestrel 150mcg, and transition to continuous use after the first
month’s withdrawal bleed.

Discharge criteria & follow up

Organize follow up with GP within a month. If concerned or persistent symptoms refer to


Paediatrician or local Gynaecologist.

When to admit/consult local paediatric team

Admission for hemodynamically unstable adolescents with a low hemoglobin


concentration, or who have symptomatic anemia.
When to consider transfer to tertiary centre

If unable to control bleeding

44-Post operative atelectasis


Recall 8-2-2018
45 years old had a cholecystectomy a day before. Patient’s chart given showing raised
temperature and heart rate, history of smoking for many years, given clexane.
Tasks
-Hx (feels good)
-ask examiner for finding (basal crackles)
-diagnosis and differentials

Recall 7-3-2018 post chole atelectasis

Recall 21-4-2018
post op pyrexia. Patient had recent laparoscopic cholecystectomy and is having fever on pod 1.
Task history. Pefe. Diagnosis. Differentials and management.
There are hospital charts outside the room showing pulse ( with a spike) RR mildly increased
spo2 95% temperature( increased to 38.5) and blood pressure.
Patient is completely asymptomatic on history and had reduced air entry and mild left lung
basal crackles on examination.

Recall 13-7-2018
Postcholecystectomy day one, on observation chart temperature 38.8, increased RR, PEFE

History
1-approach
I can see that you have had operation of gall bladder and you want to be discharged is that right?
All right so how do you feel at the moment? Are you feverish or in pain? (No)
ok, can I just ask you a few questions first to assess your current condition?

2-post operative fever differentials questions


chest (pneumonia, heart problem, pulmonary embolisim)
-any chest pain
-funny racing of the heart
-shortness of breath
-cough
abdomen (wound infection, peritonitis, drains)
-any pain in your tummy
-have you passed your motions? How is it?
-have you noticed any discharge in the wound site? Is it smelly? Colour etc..
-have you noticed any blood coming through the drain?
UTI
-any pain or burning sensation on passing urine?
-do you need to go to toilet more frequently?

DVT
-any pain or swelling in your legs?
-have you travelled recently? immobilization?
Cannulae site
-any pain or redness?

3-General questions (Imp)


-PMH (lung problems or infections, heart attack, diabetes, Gout, clotting problems) (history of
recurrent bronchitis)
-PSH
-SADMA
do you smoke? (History of chronic smoking)
alcohol?
Drugs and medications and allergies.
-family history

Physical findings from the examiner


1-General appearance
-in pain, look well, lying comfortable, pallor, jaundice, cyanosis, LAP
2-chart of vital signs from the stem explain it showing (high temperature and tachycardia and
tachypnea).
3-chest examination
Lung
-inspection: chest movement
-palpation: trachea position, chest expansion
-percussion: dullness
auscultation: air entry, vocal resonance, basal crackles or wheeze (reduced air entry and crackles
on the base of the right lung)
Heart
-apex beat
-heave or thrills
-heart sounds S1+2 and added sounds
-murmurs.
4-Abdomen
-wound site (dressing, redness, pus, hematoma)
-tenderness
-bowels sounds
-drain (blood or bile leak)
5-leg examination
-redness, oedema, hot, and calf tenderness
6-cannulae site
-tenderness and redness
7-office tests
-UDT, BSL

Explanation
-from history and examination, it appears that your temperature looks a bit high and also you
heart rate is a bit fast and there are some signs on chest examination. I think you most likely
have a condition called pulmonary atelectasis, which is a common condition after surgery when
a part of your lung is collapsed and not functioning well.
-it is more likely occur in patients with preexisting lung problems or with major chest or tummy
surgeries and if there is poor pain control. The history of smoking is a major risk factors for this.

-other possibilities are:


pneumonia or infection
pulmonary embolism
wound infection
DVT (unlikely as given clexane)
UTI
Cannulae thrombophlebitis
bile leak
drug or anesthesia recations

Management
-I can understand that you want to go home but think it is better not to be discharged until you
will be assessed properly.
-I would like to keep you in hospital and hook you to a monitor to check you O2 sat.
-need to arrange Chest x-ray, FBC, ESR/CRP, urine MCS, blood culture
-would like to call the respiratory physician to have a look at you and also respiratory
physiotherapist to help treating your condition with some breathing exercises and rehab to let
the sputum go out of the airways.
-if you become better, you may be able to go home in 2 days. Full recovery of the lungs might
take up to 2 weeks. You will be followed up regularly.
-address smoking counselling with GP once resolved
Feedback 8-2-2018
Feedback 21-4-2018
Patient had recent laparoscopic cholecystectomy , Post op Day 1. Fever upto 38c. Asking to go
home. There are hospital charts outside the room showing pulse ( with a spike) RR mildly
increased spo2 - 95% temperature( increased to 38.5) and blood pressure normal. Inside room
patient lying on the bed with cannula and a drip attached.
Task
History – no positive findings except fever.
Pefe(from the examiner ) - had reduced air entry and mild left lung basal crackles on
examination. Diagnosis and Differentials - Lung Atelectasis / Other DDs – Pneumoniae , UTI,
Thrombophlebitis, DVT, Wound infections Management – Do bloods including blood cultures,
UA – to rule out other causes, Chest physiotherapy with breathing exercises , if needed Abx ,
Tedd stocking to prevent DVT , Change the Canula site in 24hours, Regular vitals
Global score - 4 ( Approach 4, Hx 4 , Examination 4 , DDs – 4 , Management – 4).
Feedback 21-4-2018
post op pyrexia. Patient had recent laparoscopic cholecystectomy and is having fever on pod 1.
Task history. Pefe. Diagnosis. Differentials and management.
There are hospital charts outside the room showing pulse ( with a spike) RR mildly increased
spo2 95% temperature( increased to 38.5) and blood pressure.
Patient is completely asymptomatic on history and had reduced air entry and mild left lung basal
crackles on examination.
Post op complication: D1 post Cholescystectomy, a obs chart given, Temp 38.8
Pt felt fine. I asked qx re SOB, chest pain, rigors, infections(cough, sputum, vomiting, nausea,
diarrhoea, dysuria, frequency, wound pain/discharge); all negative; long term heavy smoker
PE: vitals all in obs chart, right basal creps, other all normal Dx: Post op atelectasis
Mx: I mentioned CXR both pt seemed suprised, Chest physio, deep breathing, adequet pain
control, monitor, no abx.
Approach to pt: 4 Hx: 5 Choice and technique of exam, organisation and sequence: 4
Dx/DDx:4 Interpretation of Ix: 4 Global Score: 4 pass
Read also Handbook condition 150

Feedback 5-12-2018
Atelectasis
You are a intern in hospital in post op ward. Pt with 1st/2nd POD of lap chole, A post-op chart
given in the stem where SpO2 is reducing with increased temperature. In the stem it was given
that he was smoker Hx , PEFE , explain the condition to pt And management

Inside: Post-op patient middle aged pt was lying on the bed with iv line

Introduction, how is he feeling rt now? I took permission to talk to examiner. And asked vitals to
him, examiner its in the chart, you can have a look if you want. I thanked him and turned to pt.

I I had a look in the chart showing you got some fever and oxygen in your blood is also
decreasing.. Now there may be several reason for that would mind if I ask you few q?

Usual Hx taking of PO fever. All other causes of PO fever was negative. No pain except for slight
in operative site. Not in other site (I asked specifically for dds). No discharge from opn site (he
said don’t know, there is bandage there He denied any SOB as well. No cough or phlegm
production, No chest pain. Sx was elective. No complication, any blood transfusion?( pt does not
know),No problem with bowel bladder. Only smoking positive.

PEFE: - pt has fever, air entry findings and crepitation on respiratory examination. All other
causes on post-op fever negative on examination.

Pt counselling: X there may be several reasons for fever after operations. Most common is
infection in different parts of body. Like opn site, iv cannula site, or urine, sometimes blood
clogging the leg veins can also cause fever we call it DVT which is unlikely. Infection in the lung
can also be a reason but less likely as you don’t have any high fever, cough/ chest pain. What I
am suspecting most is a condition called post-operative atelectasis is very much common after
operation especially with a back ground of chronic smoking as you have. Let me draw and
explain it to you. Then I drew and explained it as Karen note. To be more sure we can do CXR. In
management I told will refer him to Chest physiotherapist who will teach him some breathing
exercise which will help to expand collapsed air sacs. Will give some medication for fever, am not
giving any o2 as lts not too low, but can give if reduced more.( bell ranged) I will also call lung
specialist who will review you condition. Thanked him and examiner.

Key step: 1234: yes no no yes

Approach to pt:4

Hx 4

Choice and tech of exam:4

Dx/ddx:4

MX

45-Osteoporosis
Case 1 (not a recall but good start material for osteoporosis approach)
Angela a 64 year-old come to your GP saying that she thinks her height is becoming less. She is
known hypertensive on perindopril. She lives alone.
Tasks
-history
-PEFE
-Management

History
1-Chief complaint questions
-how long have you noticed that your height is becoming less? (1 year)
-apart from you, has anybody noticed that? (her children)

2-Differential diagnosis questions


-any history of trauma or injury to your back?
-any loss of weight or appetite? Any lumps or bumps around your body?
-any pain or limitation of movements of your spine?
-any weakness or numbness?
-how is your bowel motions?
-how is your waterworks?

3-Osteoporosis causes questions


Diet
-can you tell me briefly about your diet? Do you take dairy products?
Exercise
-do you do regular exercise?
SAD
-do you smoke?
-Do you drink alcohol? How much?
-How much coffee do you drink?
Sunlight
do you expose yourself to sun quite often?
Past medical history
any history of diabetes, thyroid, kidney D., celiac?
Past surgical history
medications (steroids and anticonvulsant)
menopause
-when was your last menstrual periods?
-Any hot flushes, sweating or mood changes?
-Do you take HRT?
Family history
-history of osteoporosis in family or siblings

Differential diagnosis
1-trauma
2-secondary deposits
3-multiple myeloma
4-osteoporsis

Physical examination findings from examiner


1-GA
pallor, jaundice, LAP
BMI and height (imp)

2-Vital signs

3-CVS/ Chest/ abdomen

4-CNS (tone, power, reflex, sensation)

5-Muscloskeltal
look (deformity) feel (tenderness move (Range of movements)

Explanation
-you most likely have a condition called osteoporosis; this is a condition that affects the bones; in
another word bones with holes. It occurs when bones lose minerals such as calcium more
quickly than the body can replace them, causing the bones to become weak, fragile, and more
likely to break.

-the main factor that cause osteoporosis is menopause, at time of menopause the estrogen
levels go low and this estrogen is necessary to maintain the strength of bones.
Other contributing factors could be a lack of calcium in your diet, lack of exercise and lack of
vitamin D.

-could be trauma, nasty growth of bone marrow or somewhere else, I couldn’t elicit from history
or examination features suggestive of these but we still have to think about.

Investigation
-first what we need to do is to confirm osteoporosis and rule out other causes by doing some Ix:
-FBE, BSL, UCE, LFT, TFT
-25 hydroxy vitamin D
-X-ray spine
-DEXA/ bone density scan

Treatment
1-Lifestyle modification
-health balanced diet, include more of low fat dairy products, small fish with bones.
-regular exercise 30 minutes/5 days, moderate exercise like jogging or brisk walking.
-stop smoking, drink alcohol in safe limits, coffee less than 4 cups/ day
-expose o sun light 5-15 minutes on warm days and 25-50 minutes on cold days. Avoid danger
time (10 am-3 pm)

2-attention to fall prevention (because she lives alone)


-avoid fall as much as possible, support

3-Medications
-calcium and vitamin D supplements
-definitive is bisphosphonate
-if with menopause symptoms HRT
-others are
calcitriol (in steroid induced osteoporosis) but don’t give calcium along with it.
Raloxifen

4-Reading materials

5-review once results of DEXA appear.

Summary of first scenario, which is not a recall


so in this case the patient was complaining of height becoming less. You do not know the cause
is osteoporosis so you asked all DDX like trauma, malignancy, infection or multiple myeloma,
which all appear to be negative In history. Then asked osteoporosis questions so she had many
risk factors from the history.
You did examination and all normal.
You know osteoporosis is diagnosis of exclusion so you tell her most likely osteoporosis and all
the causes that are positive in the history then tell other Ddx could be so we need Ix especially
Vit D and DEXA no need for now to refer her to a specialist before the results of her
investigations appear.
so you just advised her of some lifestyle modifications and gave her Ca supplements and vitamin
D. no need to give bisphosphonate now without the results again especially DEXA.
gave her reading materials and review when results appear.

AMC Case 1 (Recall)


42 years old woman present at your GP with complaint of not having her periods for 3 years she
has hot flushes and sleep disturbances. She has crohn’s D. she is on prednisolone at first with a
high dose and now on maintenance does of 5 mg. she had a recent DEXA with T score -3. Vit D
level is low. BMI 19 and she is exercising like walking 3 minute 3 days a week.
Tasks
-Explain results
-discuss risk factors for her condition
-discuss with her further management.

Explain results
-Introduce yourself than ask her knowledge about Ix: do you know why DEXA scan has been
done for you?
-I have got the results with me let us see it together.
-DEXA scan is showing T score of -3. This is a bone density scan that usually gives you 2 scores of
which T score is the most important one. If T score is less than 2.5 it points to a condition called
osteoporosis. Yours is -3 so most likely you have osteoporosis.
- Osteoporosis is a condition that affects the bones; in another word bones with holes. It occurs
when bones lose minerals such as calcium more quickly than the body can replace them, causing
the bones to become weak, fragile, and more likely to break.
-blood results also showed low vitamin D level which again can predispose to osteoporosis.

Discuss risk factors


-From history, there are certain risk factors why you have this condition.
-First, you are taking medication steroids than can cause OP.
-Second factor is Menopause. You are only 42 years old but you have not had periods for about 3
years and you have menopause symptoms like hot flush and sleep disturbances. What happens
in menopause is the ovaries starts shutting down and estrogen production becomes low and
estrogen is necessary to maintain stability and integrity of the bones.
-Third could be crohn’s D itself as it increases the risk of Vitamin D deficiency. As intestinal
inflammation prevents proper absorption of nutrients.
-next could be all lifestyle factors like low BMI which only 19 which is the gowing toward the
lowest normal range. Lack of proper exercise as It is better to exercise 30 minutes 5 days a
weeks.
-other risk factors are alcohol, smoking, diet low in Calcium and no enough exposure to sunlight
all these can predispose to OP.

Management
1-you require referral to the specialist for further assessment. Might do further Ix.
2-Lifestyle modification
-health balanced diet, include more of low fat dairy products, small fish with bones. Rich in
calcium.
-regular exercise 30 minutes/5 days, moderate exercise like jogging or brisk walking.
-stop smoking, drink alcohol in safe limits, coffee less than 4 cups/ day
-expose to sun light frequently but avoiding the danger period (10 am-3 pm)

3-Medications
-calcium and vitamin D supplements
-put on HRT (??or refer to specialist for HRT) because you have menopausal symptoms and
osteoporosis (rule out contraindication, explain HRT)
4-Reading materials

Summary scenario 2
in this case you do not need to take history or examination or do basic Ix as all mentioned in the
stem. DEXA confirmed osteoporosis so no need to talk about other DDX. All you need is to
explain results and risk factors then refer to specialist here and give some advise.

Feedback 28-11-2017
Station 1: Health review - FAIL
42 years old Lady comes in after not having periods for 3 years. She has hot flushes and sleep
disturbance. She has Crohn’s disease and is on Prednisolone/Methotexate/folic acid. High dose of
prednisolone first and now on maintenance dose of 5 mg. walks her dogs 30 mins 3 days a week.Her
BMI is 19. Had a bone densitometry. Vit D was low.
Explain the results.
Explain her the risk factors she has for the disease and how would you manage?
(Bone densitometry showed Osteoporosis)
Key steps: 1/3
Approach to the pt: 4
-Built the rapoort by asking about present health condition (short history)
-Reviewed whatever was given before explaining the results.
Inv interpretation: 5
-(Results were consistent with osteoporosis. )
-Explained about Bone densitometry: The test we have done to see how thick is your bone and that
has come a bit low. This is consistent with a condition called osteoporosis. And in this condition
bones become fragile and even with minor injury it can break.
-Pt counselling: 3
-Management plan: 3
Explaination of risk factor:
-One of the risk factors is less female hormone
-Long term medications she is on (steroid, metho )
-Low vit D= decrease calcium absorption= osteoporosis
Management:
-Refer to specialist
-Start her on bisphosphonate which locks the calcium in bone
-S/E of bisphosphonates: heart burn, jaw problem, if any of these happens come asap
-SNAP
-Life style advice: taking care of not to trip over and fall down, carefull with lifting heavy objects.
Global score : 3 - fail

AMC Case 2 (Recall)


1- Material case: You are a GP, your next patient is 67 year old Mary, who has come to you for
back pain that she got 2 days ago while she was picking up a heavy shopping bag. The pain is at
the lower end of her back and she took Panadol for 2 days and the pain has not settled yet. Her
son has advised her to see a doctor as the pain is getting worse. She had menopause 18 years
back and at that time she suffered hot flashes, mood swings, and dry vagina but she did not
receive HRT because she was afraid of the side effects. Apart from that, her overall general
health is fine and she does not have any significant past medical or surgical history. You had
ordered an x-ray of the spine.

X-ray shows wedge compression fracture


TASKS

-Explain the x-ray to the patient


-Tell her how that happened
-Explain the implications of this condition on her life, with reasons
-Tell her what you are going to do for further management

2-I think AMC case is the same but without that much details
GP setting..50 year old woman, lifted something heavy and came with backpain and xray
given
Task
-explain Xray to the patient
-Tell her the immediare management
-explain to her the long term management

Brief History
1-Introduce yourself, show empathy and ask for severity of pain then offer pain killers.
2-Back pain questions
-when did it start? what were you doing when the pain started? Constant or intermittent?
2-Differential diagnosis questions
-any history of trauma?
-any loss of weight or appetite? Any lumps or bumps around your body?
-any weakness or numbness?
-how is your bowel motions?
-how is your waterworks?
3-Osteoporosis causes questions
-can you tell me briefly about your diet. Do you take dairy products?
-do you do regular exercise?
-do you smoke?
-Do you drink alcohol? How much?
-How much coffee do you drink?
-any past illnesses?
-Do you take any medications
-when was your last menstrual periods? Any hot flushes, sweating or mood changes?
-history of osteoporosis in family or siblings

Explain the x-ray


-This is an x-ray of your lower back. This is a side view that you can see. The spine is made of lots
of bones called vertebrae, placed one on top of the other, with a space in between. And
normally, all these vertebrae should be of equal height but as you can see here, one of the
vertebrae has a reduced height and this vertebrae, has undergone a wedge compression. The
bones also are having a glassy appearance, which means that the bone density has become low.

Tell her how that happened


-There are several possibilities for a wedge compression fracture of the vertebrae.
-Most likely is osteoporosis.
Condition
It is a condition that affects the bones; in another word bones with holes. It occurs when bones
lose minerals such as calcium more quickly than the body can replace them, causing the bones
to become weak, fragile, and more likely to break.
Cause/ risk factors
-there are some contributing factors to osteoporosis then explain. (Menopause, lack of CA in
diet, vitamin D low, lack of sunlight exposure, Smoking/ alcohol/ coffee/ lack of exercise, PMH,
Medications)
Complications
-Once osteoporosis occurs, a major complication that you can get, is fractures with trivial
trauma. If you sustain a fracture at this age, it might take a long time to heal.

-other possibilities could be a nasty growth to plasma cells of the bone marrow called multiple
myeloma or nasty metastasis of the bone secondary to cancer somewhere. However the most
likely due to osteoporosis.

Immediate management
1-Confirm that she is having osteoporosis
-Order blood tests (FBE, BSL, UCE, LFT, TFT)
-Vitamin D level
-DEXA scan

2-First priority is to treat the wedge compression fracture


-Refer to the orthopaedic surgeon to have an assessment of the fracture (if you are GP) admit
her to be seen by orthopaedic registrar (if you are at ED)
-Put her on adequate painkillers like Panadeine or NSAIDs
-A specialist might decide to give you a bracing for 6-12 weeks followed by physical therapy.
-If your nerves are getting affected, then surgery would be the option.

Long-term management
1-Lifestyle modifications
2-Supplementation of calcium and vitamin D
3-Put on bisphosphonates or denosumab
4-Attention to fall prevention - refer her to a fall prevention clinic.

Summary scenario 3
in this case the patient was having back pain and x-ray had done showed wedge fracture. after
explaining the x-ray you should not think of osteoporosis only as a cause because DEXA has not
been done yet so you need to give all DDX then your immediate management is to perform
other Ix and refer to orthopaedic specialist then talk about long term management as usual.

Feedback 1-6-2018
Thoracic wedge fracture.
-Explain X-ray,
-immediate mx,
-long term mx.( pt was in pain, Told no need for painkiller now and explain me about diagnosis.
On and off she told she is in pain and want to go home)
-When I entered showed her loads of empathy, ask her if she needs any pain killers she said no
but despite that she was in loads of pain so I told her although I m not comfortable as she is in
pain but we have an xray with us and I would like to discuss what this xray suggests if she is ok
with that , she immediately answered yes.
-So I took the xray explained as usual that this is ur side view and things which are less dense
looks black on xray and they keep becoming grey>>>> whitish and stony white as the body
organs start becoming denser, asked her if she is with me she said yes then I told her that if you
have a look at this xray its showing ur back bones we call them lumber spine and then tried to
copare the normal ones with the abnormal ones told her that if you look at L3 its smaller then
the rest of bones and more denser which is making me think that u have suffered a
compression fracture.
-there are some complications which I want to ensure that you are not suffering from, then I
asked her about bowl, bladder, tingling and numbness in the extremeties and she replied as
no. so I said that s good it means apparently you didn’t have any immediate complications but I
would like to examine you at the end.
Short term MX:
Let me admit u , call orthopedic registerar, give you a strong pain killer continuously to make u
feel better, then rum some investigations like, CBC, electrolytes: esp vit D , calcium, UCE, urine
bens johs protein, DEXA once she will be better.
Causes: there could be a lot of reasons for compression fracture but as I couldn’t take hx so just
uttered the DD’s I told her it could be cause of trauma, osteoporosis(thin bone), or some thing
growing nasty in the bones or some issue with Bone marrow (multiple myeloma).
Then told her that from the look of it , this seems like osteoporosis for which we need to run
dexa once you will be better, the resons for osteoporosis (old age, family hx, early menopause,
gi issues, endo issues, smoking, diet factors, less sun exposure, excessive coffee intake,
malignancy(bones, mets or multiple myeloma).
Long term MX:
Don’t worry let us evaluate you once the orthopedic surgeon will see you we will discuss it in
detail with you if there are no abnormal PE findings u will be discharged with bed rest for intial
few days followed by assisted active movement , will take help from physio therapist as well. Will
work as MDT: (ortho, physio, internal medicine specialist and your gp). In case if its osteoporosis
then ur physician will start you oon calcium rich diet, vit D suppliments and alendronate(which
has loads of side effect but I don’t want to ovr burden u right now), good sun exposure, fix the
modifiable reasons for osteoporosis like coffee, sun exposure etc, regular follow up with physio,
and gp and family hx screening as she had early onset osteoporosis in 50’s (not sure thought)
loads of reassurance , red flags and reading material.
FEED BACK: pass Score:4 Key step 1, 2, 4: yes, step 2: no
Approach to pt : 4 Interpretation of inv: 5 Mx plan: 3 Pt education : 5

Feedback 1-6-2018

X-ray given, wedge compression fracture vertebrae, Explain xray and management
Patient was in pain, offered pain killers before I started and checked stability too. Lot of empathy

Took brief hx. Back pain on picking up some object. Tried to Rule out cancer. Assessed Hx for
risk factors for osteoporosis ( Diet, Menopause and HRT, Sunlight, Steroids etc…..)
Explain Xray in detail in layman
Dx Osteoporosis most likely, DDx cancer primary, cancer secondary to mets
Arranged further inv to rule out possible causes, DEXA,
gave possible causes of fracture, short term and long term management.
Feedback 1-6-2018
-X-ray given, wedge compression fracture vertebrae, Explain xray and management
-Took brief hx. Back pain on picking up some object. Rule out cancer. Hx for risk factors for
osteoporosis.
-Arranged further inv, gave possible causes of fracture, short term and long term management.

Feedback 7-7-2017
GP setting..50 year old woman, lifted something heavy and came with backpain and xray
given
Task
-explain Xray to the patient
-Tell her the immediare management
-explain to her the long term management
WHAT I DID..hello examiner, hi Therease, Im your doctor for today. patient is sitting
holding her back as if in pain. Therease I will arrange pain relief for you.
Let me explain to you the xray ( xray on the table was a poor copy so I got up and
explained the one on the illuminator)
these are your spine bones, and this one doesnt look normal as compared to the rest. its
sort of compressed, that means there is probably a break in it..its called as a wedge
fracture
it could be due to a no. of reasons 1. osreoporosis if you are menopausal, hormones
become deficient 2. due to improper posture as you bent the wrong way 3. due to
cancer..obviously we need to do more investigations
If you are found to be menopausal, we will give you replacement hormones plus calcium
vit d. plus a healthy lifestyle. If its due to cancer, as I said we would investigate further to
look for chemicals in blood. If its just due to improper bending technique, will teach you
physiotherapy for that. (and according to me the case was over but the role player was a
nice lady and said what should I do now doctor, I had forgotten the second task)I will
send you to hospital straight away where the ortho registrar will assess you further..bell
rang

Thankyou role player thank you examiner

PASS
keystep 1 yes 2no 3 no 4 yes
approach 4
interpretation of invx 6
management 2
patient counselling 4

Feedback 18-9-2018 Osteoporosis


12. back pain - vertebral fracture
65 year old lady picked up a towel and had back pain ever since. X ray given, shows vertebral
fracture.
Task: explain the x ray to the lady
Immediate management and long term management
Enter the room, offered the pain killer. Explain the x ray showed her where the fracture is.
Immediate management: pain management (increase dose if needed, patient actually say she is
in pain again so give another dose of pain killer)
Refer to specialist to assess the need for surgery, I said most likely
this condition is managed conservatively.
Rest, ice or hot pad.
Refer to physiotherapist for rehabitation, slowly start movement
I forgot to stabilise the back spine (hence Mx 3?)
Long term management: possibly osteoporosis. Do a bone scan, basic blood test: FBC, LFT, RFT,
electrolyte and calcium and vitamin D.
Give calcium and vitamin D supplements. May consider bisphosphate. Talked about how to take
it briefly.
SNAP: weight bearing exercise.
Prevent fall.
Score: 4
Key step: yes/yes/yes/yes
Approach: 6, interpretation of Ix: 5, Mx: 3, patient counselling: 6
Feedback 18-9-2018 osteoporosis
Your next pt is a 65 years old lady who came to your GP center a week ago with a
complaint of back pain. The pain started suddenly when she was doing some
household work. She went to pick up a towel and felt a sharp pain in her back, since
then she is having continuous pain. You ordered an Xray in the previous visit, an
illustration of the Xray is given below. She didn't have any history of trauma to the
back and all the other test is normal.
Your task is to
1. Explain the xray to the patient
2. Explain the possible cause of her symptom
3.Explain the immediate and long term management of her condition.
( You are not allowed to take any further history from this pt)
My Approach:
I failed this case coz I didn't study about vertebral fracture case ( i don't know why). Anyway
thats why I was not sure about the Xray here. It looked like fracture but I chose the safe way and
told her it might be fracture due Osteoporosis and explained about that. I offered pain killer but
she refused, the I explained the xray and then management as PRICE and for long term Mx Vit D,
calcium and bisphosphonate. Sorry guys I can't help much with this case as I failed.
AMC Feedback: Scenario: Back pain Grade: Fail Global Score: 3
Key step: 1,2,3,4 no, no, yes, yes Assessment domain: Approach to pt/relativ: 3
Interpretation of investigation: 3 Mx plan: 3 Pt counselling/education 4

46-Recurrent miscarriage
Case 1
30 or 37 year old Lisa presents to your GP clinic. She thinks that she's pregnant now as her home
pregnancy test has turned out to be positive. She gives a history of having 3 miscarriages before.
TASKS
-take relevant history
-Counsel the patient

Case 2 (AMC exam 2018)


37 year old lady wanting to get pregnant for the past 6 years but has no health related issues.
General, abdominal and gynecological examinations are unremarkable.
Tasks
-take history for 6 mins.
-Investigations with reasoning.

Note/Recurrent miscarriage: greater than or equal to 3 miscarriages which is consecutive.

Causes of recurrent miscarriage: (CII SEE U)


1-Chromosomal abnormalities
in the mother/father or baby (parental or embryonic) - most common cause of first-trimester
miscarriages
2-Immune-mediated
-APAS
-SLE
-thrombophilia
3-Infections
-TORCH
-STIs
-Hepatitis B and C.
4-SAD
5-Endocrine causes
-DM
-Thyroid disorders.
6-Epidemiological factors
-advanced maternal age,
-number of previous miscarriages - after 3 consecutive miscarriages, the chance of a miscarriage
is 40%
7-Uterine abnormalities
-cervical incompetence
-gynecological surgeries
-septate or bicornuate uterus.
7-Unknown

Case 2 (AMC exam 2018)


History (6 minutes)
1-5 Ps Questions
Period
-When was your last menstrual period?
-Are they regular?
-do you have any pain or heavy bleeding during menstruation?
-any pain or bleeding between menstruation?
Partner or sexual
-are you in a stable relationship?
-do you have any pain during intercourse?
-do you have good support?
-have you or your partner ever been diagnosed with STI?
Pregnancy
-have you ever become pregnant?
-any previous miscarriages? (3 previous miscarriages)
Sorry to hear that.
When do you have these miscarriages at what age and which weeks of pregnancy? (all first
trimester)
When was the last miscarriage?
Any successful pregnancy for you so far?
Any surgical intervention done at the time of miscarriages? (None)
Any analysis been done on the fetal parts at that time?
Have you received counselling after the miscarriages?
-are you planning to become pregnant? Do you think you are pregnant?
-Early pregnancy questions:
any tummy pain, vaginal bleeding or discharge?
Any nausea or vomiting, breast tenderness or mood changes?
Pill (OCP)
Pap or HPV -is your pap or hpv up to date

2-General questions (to rule out other DDx)


-Past medical history
*any immune related or blood diseases like SLE or blood clotting problems?
*Any history of thyroid problems? any weather preference ? how is your bowels habit?
*Any history of diabetes? Do you feel thirsty? Are you passing large amounts of urine?
*Have you ever been tested for immunity against Rubella? Any pets at home? (Toxoplasma -
mainly the cat litter that contains the Toxoplasma) Any raw meat in your diet? (Toxoplasma)
-Past surgical history
-do you take any medications?
-SAD: Do you smoke, take alcohol or take recreational drugs?
-Family history of miscarriages? Are you aware of your blood group? (RH –ve can cause
miscarriage)

Case 2 (Task 2 investigation with reasoning)


I finish history and all examinations are unremarkable. I need to order some investigations to
know more about the possible causes of these recurrent miscarriages.
1-Urine pregnancy test to rule out current pregnancy.
2-Baseline and routine (FBE/ ESR&CRP/ UCE/ LFT) + blood group and RH
3-CII SEE U
-Chromosomal abnormalities (as all miscarriages happened during the first trimester so it is likely
that it could be due to a chromosomal abnormality in the baby so I need to refer you and your
partner for karyotyping)- Karyotyping.
-Immune and blood clotting problems SLE antibodies, thrombophilia screen for clotting
problem, antiphospholipid antibody
-Infections-- chicken pox and German measles antibodies, hepatitis serology and STI screen
with consent.
-SAD- no need just history
-Endocrine-- thyroid function test to look for thyroid problems and BSL to look for diabetes
-environmental-- no need just form history
-Uterine abnormalities-- pelvic ultrasound scan

Recall of 6-7-2018 and 21-4-2018


Feedback 21-4-2018
37 year old lady wanting to get pregnant for the past 6 years but has no health related
issues. General, abdominal and gynecological examinations are unremarkable.
Take history for 6 mins – 3 previous episodes of first trimester miscarriages , no
investigations done, all other negative in the hx ( frequency of intercourse normal )
Investigations with reasoning – Rule out Pregnancy now ( urine test ) , Karyotyping ,
FBC , EUC , LFT , TFT , FSH /LH and Prolactin , USS pelvis , Partners SFA

Global score - 5 ( Approach 4, Hx 5 , Choice of Ix – 4)

My Comment - Out side thought subfertility but when it comes to hx its Recurrent miscarriages( go
inside in an open mind)

Case 1
History
Period
-LMP and regularity (6 weeks ago)
-any concern about periods
-tummy pain, vaginal bleeding or discharge

Partner or sexual
-stable relationship?
-good support?
-STI?

Pregnancy
-Early signs of pregnancy?
-when did you do your pregnancy test?
-have you seen a doctor?
-did you start taking folic acid? For how long?
-blood group? (RH –ve can cause miscarriage)

Others
Any immune disease or blood disorder like SLE or clotting problems
Any history of thyroid problems? any weather preference ? how is your bowels habit?
Any history of diabetes? Do you feel thirsty? Are you passing large amounts of urine?
Any history of multiple cysts in the ovaries?
have you done pelvic scan before?
Life style modification (diet, exercise, SAD)

specific miscarriage questions


-sorry to hear you have had 3 miscarriages
-when did you have them? (all between 8-10 weeks)
-when was the last one?
Any successful pregnancy for you so far?
Any surgical intervention done at the time of miscarriages?
Any analysis being done on the fetal parts at that time?
Have you received counselling after the miscarriages?

Case 1 Counselling
-Confirm the pregnancy by doing the UPT
-Explain the causes one by on
-as all miscarriages happened during the first trimester so it is likely it could be a chromosomal
abnormality in the baby.

-Although we have to look at other causes as well, that is why we need to arrange some
investigations such as:
1-Routine: FBE, UEC, LFT, BSL, vitamin D level
2-Antenatal: blood group and Rh, antibodies to rubella, varicella.
3-Causes: TORCH screen, TFT, thrombophilia screen, antiphospholipid antibody screen, STI
screen including hepatitis B and C.
4-I will refer you and your partner for karyotyping, and we will also do a pelvic ultrasound.

Case 1 Management
-from now on your pregnancy will be managed at a high risk pregnancy clinic.
-if not start her on folic acid for the 1 st 3 weeks of pregnancy.
-depends on Ix you will be managed further
chromosomal abnormalities: you will be referred to a clinical geneticist
any blood disorders: she will be followed up by a hematologist
-You need to go for regular antenatal checks, and it is always advisable to do a Down syndrome
screening as well (important if 37 years old) Ultrasound scans and frequent monitoring.
-Lifestyle modifications - healthy diet with no unpasteurized diary products and no raw meat, if
you have a pet at home do not handle the litter, regular exercise, no SAD

-Reading materials regarding recurrent miscarriages. Arrange a review once the blood test
results are out.

Another feedback FAIL


7. 37 year old lady wanting to get pregnant for the past 6 years but has no health related issues.
General, abdominal and gynecological examinations remarkable.
Task take history for 6 mins. Investigations with reasoning.
Thought it was infertility outside but was a case of recurrent miscarriage when asked the history.
Had 3 miscarriages.
Station 10 First trimester complications
I am upset by this case.
When I read the stem, it appeared to be an infertility case, then DDx in my mind:
Female factor: PCOS, Pills, POF, Prolactinaemia, PID, STI, Fibroid; male factor: STI, sperm
quality issue, mumps in childhood etc.
When I sat down in front of pt, I said I understood it is frustrating, let me help, could you please
tell me more about your condition, AND pt replied what do you want to know, doctor. (after exam,
other candidates told me when they asked, pt just told them everything about her multiple times
of miscarriage).
Anyway, I carried on then, I mentioned confidentiality (I am not sure if I mentioned this earlier)
then started to ask relevant questions all about infertility but not in an well organized way.
Mx: blood tests to check hormone, check partner bala bala, pt seemed to be surprised when I
mentioned some tests, anyway, it was not important at all
Another mistake that made me miss this case is that in 5 Ps questions, I did not ask miscarriage!
I failed this case which was not unexpected.
Approach to pt: 4
Hx: 2
Choice of Ix: 2
Global Score: 2 fail
I feel it was just a bit unfair. But this is AMC. If one did not ask the specific question or make the
specific statement (confidentiality), he/she would not get the answer. One of my tutor and friend
told me in our class that he always made confidentiality before he started to take hx.

Feedback 5-7-2018
Young lady married for 6 years unable to have a baby.

Task: 1- Hx 2- Dx and DDX with investigation

My first Qs was any pregnancy before (to differentiate between infertility and recurrent
abortion) .. multiple miscarriages between 8-10 weeks of pregnancy. I asked details of
miscarriage. Any Ix done (none). Asked 5ps, her LMP was 2 weeks ago .. then I start to ask about
my DDx

1- SLE and antiphospholipids (rash, blood clot)

2- endocrine: PCOS qs, thyroid (bowel motion, wt gain, neck lump), DM

3- PID and previous surgeries

4- Uterine abnormalities

5- TORCH (contact with pets, fever)

6- asked about diet, exercise, headache, blurred vision, fertility period.

In explanation, I draw a diagram mention my DDx and what Ix to exclude each like thrombophilia
scree, hormonal study, TFT, FBS, screening for TORCH antibodies and I even mention US and HSG
and explained it. Reassure the pt she still has hope.

Passed: global score 6


Feedback 5-7-2018
37 yr old female , married for 6 years, unable to have children
Tasks
Take relevant history
Explain d/d to the patient and relevant investigations
Asked all 5 P’s… no contraception, stopped 1 year ago, period regular,
sexual history and if she knows her fertile (ovulation) she said all good.
She had 3 miscarriages all between (8-10 weeks gestation), no
investigation done, nil other positive findings (no hirsutism, acne, regular
periods, stable partner, no STI),. No known medical condition
I forgot to ask about thrombophilia screening.
I explained the most likely its due to her age as chromosomal
abnormalities are higher in this age group. Or could be antiphospholipid
syndrome.
Investigation: I FORGOT PREGNANCY TEST! Her LMP was 2 weeks ago
Thrombophilia screening
Hormonal investigations (prolactin, estrogen, progesterone, testosterone)
Antiphospholipid antibodies, SLE antibodies

47-Gastroenteritis unimmunized child


A mother brings her 6-month-old baby to your GP clinic. he had diarrhea and vomiting

History
1-Chief complaint + duration
-I can see that --- has diarrhea and vomiting so for how long? (2 days)

2-Diarrhea questions (ACC CSF)


-how often? (5-6 times)
-what is the Amount?
-What is the stool consistency is it watery, loose
-what is the content? Doe it contain any blood, mucous, pus or undigested food?
-what is the colour?
-is it smelly?
-is it hard to flush?

3-Vomiting questions
-How often? (2-3 times)
-Amount
-content?
-colour? Is it greenish?
-forceful or not?

4-Tummy pain
-is he drawing his legs while crying?
-does he turn pale while crying?

5-Fever and rash

6-Dehydration questions
-is he alert, drowsy, sleepy?
-are the number of wet nappies as usual or decreased? Any change in the colour of urine? Is it
smelly? Is he crying when passing urine?
-is he feeding well? Does he breast feed or formula feed? (Breast feed)

7-BINDS
-Birth
-immunization (No) KEY
-is he growing and thriving normally?
-anyone in the family has similar symptoms? Have you travelled recently? (Brother has diarrhea)

Physical examination from the examiner


1-General appearance (DR PJL)
-dehydration KEY
any depressed fontanelle, sunken eyes, dry mucous membranes, skin turgor, peripheral
perfusion.
-rash
-pallor, jaundice, LAP

2- Vital signs
especially temperature and pulse

3-Growth chart

4-ENT and neck stiffness

5-full systems focus on abdomen


-Inspection: distension, mass
-palpation: tenderness, organomegaly
-auscultation: bowel sounds
-genitalia and hernia orifices
6-office tests
urine dipstick

Note in exam all signs will be normal except the baby crying and might give you ketone in urine.
So you will know this is Mild dehydration

Differential diagnosis
1-viral gastro
2-UTI
3-Appendicitis
4-other infections
5-other abdominal surgical conditions

Explain
-from history and examination he most likely has gastro which is a bowel infection leading to
runny poo and sometimes vomiting. Mainly caused by bug usually virus as there is no evidence
of bacterial infections like high fever or bloody diarrhoea. So your child does not need any
antibiotics.
-let me assure that gastro is a safe limiting condition. The vomiting may settle quickly, but the
diarrhoea can last up to 10 days.
-his symptoms could be due to urinary tract infection, appendicitis or inflammation of a worm
shaped structure attached to bowel, could be any other infections but unlikely from history and
examination.

Now in term of management of gastro:


-it is important to keep drinking him fluid to replace the fluid and body salts he lost due to
vomiting and diarrhea.

- He is mildly dehydrated now. You have 2 options if you are confident and have enough support
you can take him home. If not we can keep him here so are you confident about managing him
home? (No)

-All right we gonna keep him here and we will give him ORS (a mixture of water, salt and sugar)
to replace the fluid loss and we will assess him regularly, so do not worry we will keep looking
after him well. However if dehydration doesn’t get better we might have to pass a tube or put Iv
line to fix it.

-breast-feeding is important so please, continue breast-feeding him but feed more often.

-it is important for children to be immunized to prevent them from serious infections, I strongly
recommend to keep the vaccination up to date. I can arrange another consultation to talk about
in details. (She will agree easily)
-as child with gastro is infectious so wash your hands thoroughly after contact him especially
before feeding and when changing nappies.
-keep him away from other children as much as possible until diarrhea stop.

-if you have any other concerns please let me know.

Note / If she accepted to take him home:


-advise her to give few mouthful of ORS every 15 minutes until vomiting stops.
-breast-feeding is important so please, continue breast-feeding him but feed more often.

- it is important for children to be immunized to prevent them from serious infections, I strongly
recommend to keep the vaccination up to date. I can arrange another consultation to talk about
in details

-as child with gastro is infectious so wash your hands thoroughly after contact him especially
before feeding and when changing nappies.
-keep him away from other children as much as possible until diarrhea stop.

-need to see him again after 6 hours

Follow up (for both)


-I gonna give you reading materials for better insight of your child’s condition
-red flags
(if you noticed any green vomit, blood in poo, bad stomach pain, can’t keep fluid down or
diarrhea continue more than 10 days you need to go to ED immediately)

Notes from RCH


 red flags
-severe abdominal pain or abdominal signs
-vomiting without diarrhea, greenish vomiting
-bloody poo, persistent diarrhea for > 10 days
-very unwell
- less than 6 months in age , any chronic diarrhea

Any blood in poo or abdomen pain take fecal sample

 if severe dehydration do BSL, UCE

 If in ED
-if mild: ORS 10-20 ML/KG over 1 hour
-if moderate: NG tube to replace deficit 6 hours then maintenance 1 hour
-if severe: IV normal saline 20 ml/kg
Monitoring
-weight/ dehydration signs
-urine output/ ongoing losses
-signs of fluid overload

MILD MODERATE SEVERE

-Thirsty -delayed CRT > 2 seconds -very delayed CRT > 3 seconds

-Mild decrease in skin turgor -decrease tissue turgor

-Increase RR -shock signs (tachy/ hypotens)

-decrease conscious level

-deep acidotic breathing

*Sunken eyes and dry mucous membranes are not reliable signs

Note/ if she is formula feeding- stop the formula and give only ORS for 1 st 12 hours then give
formula in small frequent amount.

Recall of 2-3-2018 and 6-8-2018


6 month old fully breastfed baby acute diarrhea and vomiting last 24hrs. Brother had diarhea 1
week ago. Ketones in urine. Gastroenteritis, no immunization. History, pe, dx, management

Recall of 28-6-2018
child 6/12 breastfed diarrhoea and vomiting for 4 h. Sibling 3yo, day care similar symptoms. not
immunised. Only breastfed, no travel. I explained most probably rotavirus as no immunisation
against it. Task I think differentials. One other candidate said it was management as well, i don;t
remember Also we had PEFE and child was crying, but settled, rest was normal I believe

Common questions our doctors are asked (RCH)


I’m concerned the fluids I am giving my child is making the diarrhoea worse. Should I give her
less to drink?
It is very important for fluids to be given, even if the diarrhoea seems to get worse. It is
important to replace the fluids that are lost due to diarrhoea or vomiting to prevent dehydration.

Should I be worried that my child doesn’t want to eat when he has gastro?
When your child first gets gasto they may refuse food to start with. This is not a problem as long
as they are drinking fluids. When your child becomes hungry again, give them the food they feel
like eating.

Can my child have dairy after a bout of gastro?


Many children become sensitive to dairy following a bout of gastro. You can usually manage this
by reducing their dairy intake for a period of three weeks following gastro. If the symptoms
persist beyond this, take your child back to your GP
Feedback 16-8-2018
Station 5. Acute gastro-enteritis
GP, 6 mths old baby brought in by father because of vomiting & diarrhoea for 24 hours. He was
delivered at the hospital and no problem at all.
Task: history taking from father
Ask PEFE
Diagnose
Advise the father

Vomiting: not go away, Yellow color fluid, x 2 times
Diarrhoea: watery, 6 times, no bld & mucus, not difficult to flush away.
1st time, no fever, no drowsy, exclusive breast feeding, feeding well, active but mild reduced
urine output.
BIND: No immunization. Normal development.
No travel. Brother has loose motion last week. He is ok right now.
PEFE: GA: active, Vitals: PR: 140/min. Growth chart: normal. Signs of dehydration: what would
you like to know- anterior frontalle: normal, skin turgor: normal, sunken eyes: normal. Capillary
refill < 2sec. Abd: no abnormal. Anus: excoriation (-). (Some did urine dipstick. Examiner gave
ketone (+) but I forgot).
I said acute gastro-enteritis: inflammation of bowels and food bag. It is most likely due to virus.
Others: food, bact, allergy, coeliac. Contact h/o (+) as well. Give more fluid & br feed. I advised
father giving immunization (He said OK). Personal hygiene: wash hands b/f preparing food & a/f
toileting.
Red flags:
I review in 2 days time. If drowsy, reduced UO, can't eat well, Admit.

Grade: Pass
    Global score: 4
Key Steps: 1- Yes
2- Yes
3- Yes
  4- Yes
History: 4
Choice & Technique of examination, organization and sequence: 4
Diagnosis/ Differential diagnosis: 4
Management plan: 4

48-Cardiovascular risk assessment


Case 1
GP, middle-aged man came to have a consultation after his brother went for coronary bypass 2
weeks ago. He is obese, no other pass medical history.
Tasks:
-Focused history
-Physical examination findings from examiner
-Explain what will be the initial management

History
1-Acknowledge
I can see that you are here for the checkup because your brother had a CABG recently.
It is great that you have come to search for the check from your own. You did a good step.
I am here to help you with this.
2-brief history of brother’s cardiac cause
-can you tell me more about your brother’s heart problems?
-what type? has it started suddenly?
3-Assess CVS risk factors
-would you mind if I asked you some questions about your general health
*SNAP
Diet, weight and exercise
-can you tell me briefly about your diet? (likes ? fatty food, lack of green vege)
-how do you consider your weight? Have you had BMI been checked?
-how is your daily activities? How much do you exercise a week? (? Only walk during weekends,
no aerobic exe)
Smoking and alcohol
-do you smoke? How many cigarettes a day and for how long? (Chronic smoker)
-do you drink alcohol? How much and for how long? (few glasses, when asked further he said 3-
4 beers? a day?)
*Occupation and stress
-what do you do for living?
-any stress at work or at home? (Stressful job)
*PMH and family history
-have you had a history of high cholesterol level? High blood pressure? high blood sugar level?
Previous heart problems? (if any present ask for how long and what medication) (BP was normal
the last consultation about a year ago) (BSL was normal as per last consultation)
-family history of HPT, DM, lipid, IHD? (Brother heart attack, father stroke)

4-current symptoms
-chest pain
-SOB
-Palpitations
-headache
-blurring of vision
Physical examination findings from examiner
1-GA
2-BMI (BMI 29 or 30)
3-Vital signs (BP 150/100)
4-chest and heart
5-quick other systems
6-office tests
-urine dipstick
-BSL
-ECG
-fundoscopy
7-ask for CVS chart (none)

Counselling
-be not too reassuring and not too scaring (critical)..
-from history you most likely have a Moderate to high risk of getting cardiovascular diseases
including HPT, heart attack, etc. However, there are several ways we can reduce the risk.
-I need to arrange for several investigations for you (FBE, BSL, Lipids, UCE, LFT, TFT, ECG and 24
hours urine protein)
-after getting the results I will plot them on a CVS risk assessment chart and discuss it with you in
another appointment.
-I would like to give you some advice about lifestyle modification measures so that we can
minimise the risk.
*Diet, weight and exercise
-it is imp to eat healthy balanced diet contain more fruits and vegetables. Try to avoid junk food
and any fatty food. Drink lots of water and less salt intake.
-it is also imp to consider losing some weight, to stay healthy for which I can refer you to a
dietician, who will help you attain the weight ideal for your age.
-stay physically active by exercising at least 30 minutes a day for 5 days a week.
*Smoking and alcohol
-keep your alcohol drinking within safe limit which is less than 2 SD a day
-also important to consider stopping smoking.
-arrange another consultation to discuss both.
*Occupation and stress
-it is understandable that your work is stressful but it is advisable to make a routine in your life to
get rid of the stress. You can do some relaxation techniques like yoga or pick up a hobby to
reduce stress.
*Family history
-I also need to see your siblings to assess for their risk factors so that we can deal with it earlier.

Feedback 20-2-2018 (Case 1)


GP, middle aged man came to have a consultation after his brother went for coronary bypass 2
weeks ago. He is obese, no other pass medical history.
Tasks:
-Focused history
-Physical examination findings from examiner
-Explain what will be the initial management

2 min thinking: need to ask all the CVS risk factors (ABCDEFS)

History:
Introduced myself, condolences to the brother’s surgery, asked about his feeling right now.
Praise him for coming for a check up. Get permission for ask more questions.

A – alcohol intake (few glasses, when asked further he said 3-4 beers? a day?) offer alcohol
cessation counselling for the next consultation

B – BP (was normal the last consultation about a year ago)

C – cholesterol level (don’t know)


D – DM (BSL was normal as per last consultation)

- Diet (likes ? fatty food, lack of green vege)

E – exercise (? Only walk during weekends, no aerobic exe)

F – Family history of DM, HPT, CVS diseases (sorry I can’t recall much)

S – smoking (chronic smoker), offer smoking cessation counselling for the next consultation

- Stress from work/home/financial

Past medical/surgical history


SADMA

Physical examination:
I think is unremarkable apart from BMI 29 or 30
Asked for CVS risk assessment chart - none

Explanation:
Moderate to high risk of getting cardiovascular diseases including HPT, heart attack, etc. however
there are ways to reduce the modifiable risks. Wrote ABCDFS on the paper and explain how we
can deal with each risks briefly. Explain that we can take some blood test and will see him again
during next consultation. Reading material.

AMC Feedback – Periodic health review: PASS (Most likely this is a scored station)
Feedback 20-4-2018 (case 1)
Feedback 20-2-2018 (Case 2)
GP, middle aged (about 57years old) man came for follow up. He came previously after his brother passed
away from heart attack 2 weeks ago. He is a smoker, unremarkable pass medical history. There is some
blood investigation results of the patient: 160/100
FBC – unremarkable, lipid profile: total cholesterol 8, HDL 1, LDL ?, total cholesterol:HDL ratio 8, BSL: ?
Estimation of cardiovascular risk chart is given.

Tasks:
-Take a focused history
-Explain the investigation results to the patient
-Explain about initial management
2 min thinking: check which category the patient likely falls into (I look at the chart for man, and note
down the one for smoker and non-smoker). Need to ask whether he smokes or not.
History: similar to the above case
Explanation:
-By using the chart, show the patient he is under the high risk category (red colour box). I marked an “X”
at the box (he is 57 years old, male, smoker, systolic BP 160 and total chol:HDL ratio of 8).
-Then point to the red box below the chart and explain to him that he has a more than 30 % chances of
having heart related diseases in the next 5 years.
-Reassure the patient that there are some modifications and changes that we can make in order to bring
the risk down.
-Gave him example if he quits smoking, his risks will drop to 20-24% by pointing to the appropriate box.
And risks can be further reduced if we tackle the cholesterol level and the blood pressure.
- Explain we can start from lifestyle modification – exercise, diet (give some example and refer dietician),
and medication control if LSM fails.
-Check his understanding and give reading materials, will see him again during next consultation.
(Most likely this is the non-scored station)
Note/ the second case is similar to the first but the Investigations have been given in the stem so that you
can plot in the chart during the 2 minutes and know which category the patient belongs to. In the
feedback it is high risk> 30% in next 5 years.

-Same history with little modification in the approach like say I am sorry to hear your dad has passed away.
Appreciate his coming. And so on.
-explain the category and why by showing the patient the chart.
-Explain Ix results
-then reassure that we can lower it down
-give same advice

Feedback 25-10-2018
Scenario : periodic health r/v
Stem : man coming bcz brother had bypass surgery
Tasks
Hx
Pefe on card
Cvs risk assement
Counsel
Old overweight man was sitting inside the room, told him im sorry to hear about his bro n
appreciate him for coming up to me so we can discuss his health condition. Asked a focused hx
as I thought I will have to do cvs risk assessment on chart.
Moved to pefe on card n asked for cvs chart which wsnt available.
Pefe bp was high n bmi overweight. He had sedentary lifestyle and strong family hx.
I explained with the pefe saying everything looks perfectly fine, he in a sarcastic tone asked are u
sure doc n looked at the examiner i again checked pefe card n bp was raised 2 reading 10 min
apart were taken n raised!!! I told him yes everything in ur examination is fine except the
pressures, they r on the higher side n along with other risk factor i would say ur risk of having
any cvs dx is moderate to severe n we need to work on it asap he said ok doc thanks n bell rang
Gs: 5 3/5 key steps Approach: 6 Hx: 4 Counselling: 5

Feedback 25-10-2018
Stat 20- fail (my first station)
50 yr old came for check up as his brother has coronary bypass surgery.
Task:
History
Counselling
I said sorry about your brother. Asked according to “ABCDE” as much as I could remember.
Had sedentary job, smoking, alcohol, no exercise, no healthy diet measures.
Examiner gave pe findings on card- bmi high, bp 150/96 Rest normal.
I said about the risk factors, diet modification, exercise regularly, regular review, blood pressure,
is a little high, will need to check again.
Gs 3
Approach- 7
History 4
Counselling- 3

49-Acute sinusitis
Case/23-2-2017
Young man with nasal congestion
tasks
-Hx,
-Perform PE
-likely Dx

Case/ 9-2-2018
30+ male, 5/7 history of flu like symptoms and feeling tired. Vitals were given: Temp was up
Tasks:
1. History
2. Examine
3. DDx with reason (no management)

Case/ 12-7-2018
Nasal Congestion for 10 days, presented with fatigue
+ve findings were : poor sleep, headache , tender Rt maxillary sinus, pain increased when leaning forward,
recent URTI, no cough, greenish- yellowish.
Perform the examination and DDx

Note/ this question can come with history or without. I’m sure some of this information is provided in the
stem so you can select the specific questions you need to make the diagnosis of sinusitis and rule out
others

History
1-nasal congestion questions
-how long has your nose been blocked?
-are both sides of your nose affected or only one?
-Does it affect your sleep?
-have you tried anything to unblock it?
-has this happened before?

2-Sinusitis questions
-have you had recent flu? Fever? Recurrent infections?
-any coughs or sneezing?
-any runny nose? What is the colour of the discharge clear, yellowish?
-any discharge running down your throat (postnasal drip)?
-do you have any headache or pain in your face?
Where exactly?
Can you describe it?
Anything make it better (painkillers) or worse (bending forwards)?
-any tooth pain or bad breath?
3-Differential diagnosis questions?
Allergies
-have you had any problem with allergies?
-is the problem worse at different times of the year?
Polyp
-have you had polyps in your nose in the past?
-any loss of sense of smell?
Foreign body
-could there be anything in your nose?

4-complications
-any blurred vision, swelling around eyes or confusion?

Perform examination
1-WIPE
now I ‘m gonna examine you this will involve having a look and feeling various parts of your face
and using some instruments to look inside your nose and ears. Would that e all right?
I will try to be as gentle as possible so if you feel uncomfortable at any time please let me know.

2-General appearance
-sitting comfortably, no distress , alert
-can breath easily while closing his mouth

3-Vital signs (one feedback say it is already given in the stem)

4-Inspection
Face
-symmetry
-injury
nose
-nasal discharge
-nasal deviation (best seen from behind the patient and look down)
sinuses
-redness and swelling (in the exam this could be +ve)
eyes
-periorbital swelling
-red eyelids
-red watery eyes
ear
-ear discharge, redness
mouth
-tooth infection, tonsils, pharynx
5-palpation
-tenderness over sinuses (frontal, ethmoid, maxilla) (in the exam there will be tenderness over
maxillary sinuses) (feel and percuss)
-elevate tip of nose and look for septal deviation and color of mucosa
-press over EAM for otitis externa
-tap over teeth looking for tenderness using tongue depressor

6-Nasal speculum
-discharge (watery/ purulent or bloody)
-colour of mucosa
-size and colour of turbinates
-deviation or perforation of septum
-polyps or foreign body

7-otoscopy

8-neck Lymph nodes

9-sinus transillumination +/-

10-others (eye movements for vision)

Diagnosis and differential diagnosis


1-most likely acute sinusitis after viral infection
most likely you have sinusitis, which is an inflammation of the sinuses, which are air filled
cavities adjacent to the nose lined by mucous membrane. Most likely to be affected are the
sinuses in the cheek bone called maxillary sinuses.
it is usually caused by a viral URTI involving the nose then spread to the sinuses followed by
Secondary infection by bacteria lives normally in the nose.
2-Other possibilities
allergic rhinitis
polyps
foreign body
common cold
tooth infection.

Management of acute sinusitis (not a task)


1-rest
2-steam inhalation
3-pain killers (aspirin or paracetamol)
4-nasal decongestant
5-blow your nose as often as necessary into tissues
6-antibiotics if suspected bacteria

Sinusitis usually clear up by itself assisted by antibiotics. If it doesn’t work we can do CT scan
then can be surgically drained.
Note If the presenting complaint was headache then do examination like headache
examination and the DDX WILL BE
-tooth infection
-pharyngitis
-trigeminal neuralgia
-TMJ dysfunction

Notes
In the exam there could be a thudicum speculum

There could also be another nasal speculum.


50-Preterm Labor
A 28-year-old primigravida presents to you at 30wks of gestation with sudden onset of
abdominal pain. You are a GP in a rural practice about 300km away from a hospital with O&G
facility.
TASKS
Relevant and focused history
Examination findings from examiner
Discuss management with patient

Differential diagnosis
1-preterm labour
2-placenta abruption
3-preeclampsia
4-acute abdomen (appendicitis, pyelonephritis, cholecystitis)
5-UTI
6-ovarian cyst rupture

History
1-Check for hemodynamic stability.
- Is my patient hemodynamically stable? What is the BP, pulse, RR, temp and sats of my patient?

2-Abdominal pain questions


severity
- Hi --- I'm Dr----your GP today. I understand that you suddenly experienced abdominal pain. How
severe is your pain from 1 to 10 1 is the least 10 is the most? (8-9)
-I would like to give a painkiller if you don't have allergies.
painkiller can take some time to work in the meantime can I ask you a few questions?
Duration and onset
-for How long have you had the pain?
-what were you doing when the pain start? Did you hurt your tummy by any chance?
-has it started suddenly?
-is it constant or does it come and go? (It is an on/off pain) (Abruption is continuous pain while
preterm labour come and go)
-Is the pain coming at regular intervals? Have you noticed that the intervals are coming shorter
and Pain duration coming longer? How frequent and how longer does it last? (Coming regularly,
shorter intervals, pain coming longer)
Site and radiation
-Where is the site of pain? (All over the tummy)
-Does it go anywhere else? (It goes to my legs, thighs, back)
Character
-can you describe it for me?
Aggravating or relieving
-does anything making it better or worse? Nothing.
3-Late Pregnancy complications question
- do you have any bleeding from your vagina. (placenta Previa, abruption)
- Any vaginal discharge? (PROM/ infection)
- Any fever, nausea or vomiting? (Acute abdomen/ Infection)
-any headache, blurred of vision or leg swelling? (Preeclampsia)
-Any burning or stinging while passing urine? (Urine problem)
-have you opened your bowels? (Bowels problems)
-Is the baby kicking well? (Baby problem)

4-regular visit questions?


- How’s your pregnancy so far? have you had Regular antenatal checks?
-have you had down syndrome screening at 11-13 weeks?
-US at 18 weeks? Is it a single baby? What was the Position of the placenta?
- Sweet drink test at 28 weeks?
- Folic acid in the 1st trimester?
- blood group?

5-General questions
-When was your last pap smear?
-Do you smoke, drink alcohol, or take recreational drugs?
-Do you have good support?

Physical Examination from examiner


1-general appearance (PODL)
- Is there pallor, edema, dehydration, lymph node enlargements?

2-Vital signs
especially BP, temperature.

3-abdomen examination
-Is there uterine tenderness? None (tenderness in abruption)
-is there any uterine contractions per abdomen? How frequent are the contractions coming?
How long do the contractions last? (Contractions happening 3-5minutes, lasting 30seconds to 1
minute.)
-What is the fundal height? 30cm (increase in abruption while decreased in preterm labour)
-FHR? 150bpm
-Lie, presentation? Longitudinal, cephalic
-Is the baby's head engaged? Not yet

4-pelvic examination
-Inspection of the vulva and vagina, any discharge or bleed, rash or vesicles?
-Sterile speculum exam, any bleed or discharge from the cervix? Cervical os open or closed?
Dilated 3cm
**Don't do a per vaginal exam.
-I would like to take a vaginal swab and give it for fibronectin test. (Fibronectin is the test to look
for preterm labor. If it is positive, she might deliver in the next 7-10 days. If it is negative, she will
not deliver in the next 7-10 days)
-low vaginal and anorectal swab for GBS

5-office tests
urine dipstick and BSL

Explanation
-From history and examination, you most likely have a preterm labor. Normally labor happens in
and around 40 weeks, but if it happens before 37 weeks, that is called preterm labor.
-there are several cause, but anything that over distends the uterus such as:
*excessive fluid in the bag of water we call polyhydramnios.
*big baby
*Multiple pregnancy.
*Cervical incompetence (could become earlier)
*Maternal infections (no fever or discharge)
*other maternal conditions such as diabetes and preeclampsia, which is a sharp rise in blood
pressure with leakage of proteins to urine.
*trauma
however we could not identify any of these so sometimes it can happen without a cause.

Management
-you need a referral to a tertiary hospital with a neonatal intensive care unit.
- I'll arrange an ambulance for you, I'll ring up the hospital and make them aware of your
condition so that all arrangements will be made once you reach the hospital.
- I will start you on an IV line, take blood for investigations like FBE, ESR, CRP, UCE, blood group
and Rh factor.
- I will give you your 1st dose of steroid (betamethasone), to bring about lung maturity in the
baby, (if less than 34 weeks) (2 doses at 12-24 hours interval), and 1st dose of tocolytic.
Tocolytic is the medication given to prevent further uterine contractions. (Nifedipine,
salbutamol)

-Once you reach the hospital, you will be admitted, seen by the specialist, ultrasound and a CTG
will be done to look for the wellbeing of you and the baby.
- Let me assure you that you will be in safe hand and they will do everything possible to continue
your pregnancy.
-They will give you further doses of tocolytics, and further dose of steroid.
-Just in case if your labor progresses and you deliver, the baby will be taken cared of by the team
at the neonatal intensive care unit.

Do you need me to call your partner to be with you during this time?
Feedback 20-7-2018
New case ( unscored)
27 years old Generalized abdominal pain
History , PEFE , diagnosis with reason
I was thinking surgery or medicine or OG ( like PID , ectopic)
But on entering pt is sitting on the chair , look tummy distension I thought that may be she
is fat or intestinal obstrution or bowel distension :D , I just took most of history time to
ask about surgery and medicine cause , when I asked about LMP is 6 months ago ,
immediately turned to obstetric question quickly… regular follow up , blood test normal ,
blood gp o pos , Folic acid + , USG normal , feeling baby kicking , no headache , no vision
disturbance , no bleeding , no water leakage, no leg swelling
PEFE need to ask all before giving me card ( long findings card)
vital sign is normal , BP 120/70
SFH is 26 cm
no uterine tenderness ( sure)
as I remember there is no abdominal sign ( I don’t remember contraction is given in
the card )
cervix is 3 cm dilated ….. ( as I remember ,not sure , no discharge or bleeding )
Tell about preterm labour , so that i need to transfer you immediately to hospital
Pt started to cry , no time for D/Dx ( was not asked though ) , I am also shocked cos
nothing about pregnant was given in stem
51-Cervical spondylosis
A middle-aged woman with Neck pain and shoulder pain.
Tasks
-history
-PEFE
-Diagnosis and differentials
-Investigation

History
1-pain questions
-how severe is your pain from 1 to 10 1 is the least 10 is the most? Offer painkiller after allergy?
(patient fine)
-when did it start? Sudden or gradual? Constant or come and go? Is it getting worse?
-can you show me exactly where the pain is? Does it go anywhere else? (back of neck, travel to
right shoulder and arm)
-can you describe it for me?
-anything make it it better or worse? (nothing even after Panadol)
-is it worse in the morning or night? (both!!!)

2-associated symptoms questions


-any tingling, numbness or weakness in your limbs? (Yes, tingling and numbness in hands also
weakness)
-any stiffness in your neck? Any joint swelling? (No)
-any trauma or injury? (No)
-do you have shoulder pain when combing or clipping bra? Any hip joint pain or stiffness? (No)
-any pain while chewing or headache? (No)
-any fever, any rashes around your body? (No)
-any problem with your urine or bowel motions? (No)

3-General questions
-past medical history
-past surgical history
-family history of joint problems
-smoking, alcohol, medications and allergies
-occupation (typist)
-any stress at home or work

4-How does it affect your life and daily activities? (Affect job)
Physical examination from examiner (Card)
1-General appearance
-pallor, jaundice, rashes, LAP
2-Vital signs
3-neck and shoulder examination
-look (scars, skin color changes, swelling, deformity, wasting)
-feel (spine tenderness, paraspinal muscle spasm) (tenderness neck)
-move (any restriction in range of movements) (restriction of neck movements)
5-upper neurological examination.
-power, tone, reflex, sensation (weakness of wrist and elbow extension, sensation loss C5, C6)
6-lower limb neuro examination
7-office tests

Explanation
-there are several possibilities why you are having neck and shoulder pain with some sensation
loss and weakness. Most likely have a condition called cervical spondylosis with radiculopathy.

-Draw a diagram (these are the bones in the back of the neck between them there are discs or
Cushing and there are nerves passing on each side)
-cervical spondylosis is a wear and tear of the bones and cartilage (discs) of the neck spine.
Sometimes, it can result in a narrowing of the space needed by the spinal cord and the nerve
roots that pass through the spine to the rest of your body. If the nerve roots become pinched
you can experience numbness, tingling and weakness.
Other changes include
*bony spur or overgrowth of bone that can press on a nerve causing pain.
*Herniated disc when the disc can develop cracks allow leakage of internal Cushing material that
can press on spinal cord or nerves causing numbness and pain travels to the arm.

-other possibilities are:


*Injury or trauma to the neck
*polymyalgia rheumatica less likely as no hip pain or stiffness and usually there is no nerve
involvement.
*Rheumatoid arthritis: when body produce antibodies against own cells but unlikely.

Investigation
-basic blood tests: FBC, ESR/CRP, RA factor
-X-Ray neck
-Refer to specialist for MRI neck
Feedback 14-3-2018
A middle aged woman with Neck pain and shoulder pain.
HX, PEFE,DX and investigations.
2 min thinking time;
D/Ds,.. cervical spondylosis, polymyalgia rheumatica, dermatomyositis, RA, ankylosing
spondolytis, Fibromyalgia
After entering the room and introduction, Started by asking if she was in pain at that moment
but she said she was fine. I asked all pain questions. She had pain on the back of neck, travelling
to the shoulder and the right arm. No pain on left side. Worst at night and morning. Nothing
made it better although she was taking Panadol but still it was getting worst. On inquiring if she
ever had any odd feeling in her hand she told about numbness and tingling in her index and
thumb and also complained about weakness. There was so history of trauma or stiffness. No
problem while turning head. No shoulder pain while combing or clipping her bra. No pain or
stiffness of hip joint or pain while chewing or headache (temporal artritis in PR). No rashes
(dermatomyositis). I also asked about life stresses to R/O fibromyalgia or CFS.
No Family history of joint problem. Or any medical condition. Non smoker and non drinker.
She was a typist so it was affecting her life and job.
In PE, Vitals all stable.
For neck, There was nothing significant on inspection. Palpation was painful and there was
tenderness. There restriction in the movements of neck although shoulder joint was fine.
There was sensory loss in the area supplies by C6 C7 nerves. Weakness in elbow extension and
wrist extension
There were no rashes or LNs or anything significant on systemic review.
I started with drawing and explained that this might be due to a medical condition called Cervical
spondylosis, and asked if she had heard about it? Explained as follows; It’s a condition where a
nerve impinges on a bony part of the bones in neck. It may be due to wear and tear of the bones
or due to a trauma. So due to damage of the nerve involved you are getting symptoms in the
areas supplies by those nerves. I’ll organise some basic tests to look for some changes in
proteins of blood and x-ray of neck. I’ll refer you to the specialist who will do special and most
sensitive investigations like MRI of neck or if he thinks he may do nerve testing to R/O other
causes, which isn’t very relevant otherwise.
Other possible causes could be PR but that causes hip pain and stiffness as well and no nerve
involvement so that’s not possible in your case, however, we can do ESR to R/O. It can be RA in
which body produces Abs against it’s own cells. We can do Ab tests to R/O, which will most likely
be negative. It could be dermatomyositis, sorry for the medical jargon, but it causes rashes which
you don’t have, there is a blood test for that when muscle release a protein called CPK so we
may do that to rule it out completely. There are no stresses so any psychogenic pain or
unexplained pains like fibromyalgia and CFS is also ruled out.
Scenario Neck pain Grade… pass Global score… 5 Key steps 1,2,3,4…
yes Approach to patient… 5 History… 5 Diag/ D/D…. 5 Choice of investgations… 5
Feedback 15-8-2018
Scenario: Neck pain
Stem : Lady in 50s comes with neck and shoulder pain.
Tasks
~ History
~pefe

~dx and ddx


~investigations

Inside was a lady sitting on chair. I greeted her. Asked pain scale. She said not much now and she
doesn’t need pain killers.
Took pain history and ruled out dd keeping in mind( trauma, prolapse, radiculopathy,spondylosis,
spondylitis, arthritis, muscle pain, shoulder( frozen, rotator cuff, pmr). i have forgotten the
details she gave but she complained of numbess and tingling in thumb and index finger ( made
me think radiculopathy) and on asking how is it affecting ur life she gave a long long answer that
I m unable to hangout, play golf etc etc..
Pefe
Examiner handed over a card ( positive was LOS in thumb and index finger) as far as I remember.
Gave dx of radiculopathy and explained. And ddx same as above in history.
Investigation: I told MRI
Grade: Pass

Feedback 26-10-2018 Neck pain Passed (GS – 4)


Man present to your GP with neck pain.
Hx:
• Pain Qs: neck pain for months, pin and needle sensation in left hand esp in thumb and
index finger (patient pointed out where the pain was)
• DDX (neck pain, dizziness, P&N sensation, problem with bladder and bowel, difficulty
with speaking and swallowing, occupation, any injury or fall, any sport)
• SADMA

PEFE card given:


No limitation of neck movement. Tone, Power, reflexes are normal, sensation loss in C5 areas. (It
was a short note with several lines. I don’t remember exactly. Those are findings what I can recall
so far. They gave a couple of lines regarding MSK system which is fine.)

Dx:
Cervical radiculopathy with compression of C5 nerve, other less likely DDx: muscle or ligament
injury, trauma, stroke or SOL.

Invx:
I will order neck(cervical) Xray. If required, will refer you to specialist and do more further
imaging like MRI
52-Pre-eclamptic/ Eclampsia
32 year old Maria, who is 32 weeks pregnant, presents to your GP, with headache since the last 2
days. She had regularly done her antenatal checks with you and a week before, when you saw
her, she had mild swelling of her legs. At that time all relevant investigations were done and they
were all normal.
TASKS
Relevant history.
Examination findings from examiner.
Explain diagnosis to patient.
Management.
Differential diagnosis
1-preeclampsia
2-Migraine
3-Tension headache
4-URTI
5-ear or tooth infections
6-meningitis/ encephalitis
7-head trauma
APPROACH
History:
Hi Maria, it is nice to see you again, I can see that you are complaining of headache is that right?
Just let me ask you few questions to unravel the nature of the problem
1-Headache questions (rule out tension and migraine)
-Can you tell me more about it?
-Severity
-First of all how severe is your headache from 1 to 10 1 is the least 10 is the most?
-I can offer painkillers for you so do you have allergy to any medications?
Onset and duration
-How long have you been having the headache?
-Sudden or gradual? Constant or come and go? Is it getting worse
Site and Radiation
can you show me exactly where you have the pain?
Does it go anywhere else?
Character
can you describe it for me?
Aggravating and relieving
does anything make it better or worse
Timing
any specific time when the pain is worse?
2-Differential diagnosis questions
-any recent infection? (URTI)
-any pain in your ear or teeth? (ear or teeth infections)
-Any fever, rash or neck stiffness. (Meningitis/ encephalitis)
-have you had any trauma or injury to your head? (head trauma)
3-Preeclampsia questions (Headache, BOV, leg swelling, tummy pain, vomiting, bladder and
bowels, complications: confusion, bleeding, discharge, baby kicking)
-do you have any blurring of vision? (no)
-has the swelling extended? (extended to the knee)
-have you had any vomiting? (vomited twice)
-any pain in your tummy?
-how’s your urine output? How often do you go to toilet? (not quite often)
-how’s your bowels motion?
-any confusion or dizziness? (Imminent eclampsia)
-Any discharge or bleeding from down below? (PROM, abruption)
-do you feel your baby is kicking? (Key) (fetal distress)
4-General questions
-Any past history of migraine?
-Any high blood pressure before your pregnancy?
-Any family history of migraine or high blood pressure?
Physical Exam from examiner
1-General appearance: PODL + J
-pallor
-oedema (oedema up to the level of the knees with facial puffiness)
-Any dehydration?
-LAP
-jaundice (HELLP)
2-Vital signs + O2 sat
- BP 180/100
- PR 100/min
-RR 20/min
-Temp 38.7
-O2 saturation 99% in room air
As her blood pressure is quite high I'd like to shift her to the resuscitation room, put in a large
bore IV cannula just in case to gain IV access, take blood for investigations ( FBE, LFT, UCE, BSL
and coagulation profile)
I would like to give her the first dose of antihypertensive medication IV labetalol, methyldopa or
Nifedipine whichever available
Then proceed to the rest of my examination.
3-CVS: S1, S2, murmurs
4-Respiratory system: air entry, adventitious sounds
5-CNS:
tone, reflexes there is hyperreflexia and clonus
Fundoscopy to look for papilledema mild papilledema
6-Abdomen:
-Uterine tenderness (abruptio placentae)
-Hepatic tenderness?
-Fundal height 34cm
-FHR? 150bpm
-Lie, presentation? Longitudinal, cephalic
7-Pelvic exam (consent and chaperone)
Inspection of vulva and vagina, any discharge or bleed, rash or vesicles
Speculum exam: any bleed or discharge? Is the OS closed or open?
No pervaginal or Bimanual ex
8-Office tests:
-UDT look for urinary proteins proteins 3+
- BSL (already taken)
-ECG
Explanation
-Most likely you are having a severe preeclampsia. It is a condition where there is a sharp rise in
your blood pressure and leakage of proteins in the urine.
-it is common in first pregnancy and The exact cause is unknown but anything that decreases the
blood supply to the placenta, can cause the placenta to secrete certain chemicals which could
damage the lining of the blood vessels of all major organs.
because of this, the pressure within your brain goes high causing headache and vomiting.-Pre-
eclampsia is an emergent condition and if it is not controlled, can result to fits and if runs for a
long time it can lead to growth restriction of the baby.
Management
-You need an immediate referral to the tertiary hospital with neonatal intensive care unit. I
would arrange an ambulance for you, and I will ring up the hospital and make them aware of
your condition so they can setup everything for when your arrive.

Once you reach the hospital:


- you will be admitted and seen by the specialist.
-Ultrasound and CTG will be done to monitor you and the baby. (key point)
- Antihypertensive medications: IV hydralazine will be given
- IV magnesium sulfate to prevent fits from happening
- If everything is well controlled with you and the baby is also doing well, they will try to prolong
your pregnancy till completed 36 weeks but you need to be in hospital during this time.
- Labour will be induced by 37 weeks.
- Just in case the baby becomes unwell or your condition become uncontrolled, an immediate
delivery will be planned either by C-section or induction.
- You have to take an absolute bed rest. Continuous monitoring of the vitals, BP recording every
2 hour, urine protein twice daily, fluid input output.

Notes/ 37 weeks pregnant: same management; but plan her delivery immediately. Because
delivery is the treatment of choice for pre-eclampsia and eclampsia
Another scenario (all steps very important)
While taking examination findings, the patient will start fitting. Patient has started fitting.
1-I would like to shift to the DR ABCDE protocol now.
2-Call for help.
3-Put her in a left lateral position.
4-Start her on oxygen by mask, 6-8 liters per minute.
5-to stop fit
If in GP Give her IV diazepam.
If in hospital ED give her IV MgSo4 (4g initially over 10-15 minutes, and then 1g/hour as
continuous infusion).
6- Anti hypertension medication
-If in GP give IV labetalol
-if in a hospital IV hydralazine
7-If in GP:
-What you have is fit or seizure during pregnancy due to preeclampsia or sudden sharp rise of
BP. Now I need to re-assess all the systems, and I need to refer you immediately to a tertiary
hospital, seen by the specialist, Ultrasound and CTG done, all the blood investigations, IV MgSO4
and IV hydralazine to control your condition.
And once eclampsia has happened, baby has to be delivered no matter the gestational age is.
So once your condition settles then the specialist will decide to deliver you immediately either
induction or CS.

MILD MODERATE SEVERE

Diastolic BP 90-99 100-109 >= 110

Systolic BP 140-149 150-159 >=160

Proteinuria 1+ 2+ 3+/more

Edema minimal up to the level of the calf Massive( up to knee and


above)

Management Can be managed at -Requires admission to the DRSABCDE.


home hospital. Secure airway, Oxygen
Bed rest -She needs to be seen by by mask, I/V line and
Salt and protein the specialist. blood for investigations.
restricted diet. -Bed rest with toilet MgSO4- 4 gm bolus,
Review by GP every privileges then 1-2gm
2nd day. -Blood pressure recorded infusion/hour at least for
Red flags. 4th hourly, urine protein 24 hours after last
If not controlled, twice daily, fluid input and seizure. If seizure recurs,
referral to hospital. output chart give 2 gm bolus.
-UTZ, CTG If MgSo4 not available,
-Oral antihypertensives: I/V diazepam 2mg/mL,
first line is labetalol: max 10mg.
Methyldopa, nifedipine Hydralazine I/V.
-Once her condition is Catheterize, Fluid intake
under control, and the baby and output chart.
is doing well, you can Immediate referral to
discharge her but refer her tertiary hospital.
to a high risk pregnancy CTG, U/S.
clinic
-If already at 37 weeks,
plan delivery by induction.
Feedback (23/2/2017)

ED, Primigravida, 36 wks, severe headache. All AN check up N.

Tasks:
1.History (2min),
2. PEFE,
3. DX & Mx

2min thinking:
DDx: preeclampsia, tension headache, migraine, other causes of headache.
Patient might fit, be prepared with initial management.

-Entered room. Greeted by examiner.


-Patient was holding her head, in pain.
-Introduced and greeted the patient,
-offered pain killer after assessing severity of pain.
-Asked pain Q (SORTSARA), any BOV/nausea/vomiting/epigastric pain/ankle oedema.
-Ruled out migraine, stress from home/work, recent URTI.
-How is baby’s movement/any contraction/ bleeding/discharge from private part.
-PMHx of HPT and DM.
-SADMA.
-Did not asked about previous antenatal check up and USG as was normal given in the stem.

PEFE:
GA, V/S – BP 180/110, HR, RR, T, O2,
P/A: symphysial fundal ht, presentation and lie of fetus, FHS.
Hypereflexia and clonus present. Ankle oedema.

Office test: urine protein +++

Dx: Dear Cindy, as you are having headache and nausea, and the urine test shown presence of
protein, you are having a condition called preeclampsia. Have you heard about it? Don’t worry I
will explain to you…
(Examiner interrupted me and said the patient suddenly has a fit)

I stood up, faced my patient who was “sitting comfortably” on the chair, and mentioned I would
manage my patient according to DRSABCD protocol, try to access IVL for administration of IV
diazepam, if unsuccessful will give PR diazepam, arrange ambulance immediately, liase with ED
doctor. In the hospital the doctors will do necessary Ix and she will be seen by obstetrician as
well. Aim is to prevent fit, control BP and aim to deliver baby but all will be dicided by
obstetrician.

Bell rang, thanked the examiner and patient.

Passed. Global score 5

Feedback (13/10/2017)

You are in GP, a Lady with 35 weeks old pregnancy presented to you with headache.

Task:Hx, PEFE, Dx to the patient with reasons.

I entered room after introducing myself I asked haemodynamic stability? Examiner said what you
are looking for? Bp: 180/100, PR: 80 regular, RR:NL, Temp:37 I said I want to secure two IV line
and start dose of labetalol IV line and transfer the patient to the treatment room while I am
taking history from my patient.

Hello Jilly? I am… one of the doctor in this GP. Where is your HA exactly (all around the head)
When did it start? It was there for couple of days but today it is more severe. How sever it is?(7-
8) Do you want pain killer?(yes) no allergies?(no) I will arrange a painkiller for you.

Just quick questions: BOV?tummy pain? trauma?discharge? BLD? (no) swelling on legs? Baby
kicking?(yes) no hx of HTN DM… this is first pregnancy and no miscarriage before. All the
antenatal tests were positive( sweet drink test, U/S 18 wks was NL).

If you are OK I want to ask some questions from my colleague?


Dear examiner, Is there any pallor, jaundice, dehydration in my pt's general appearance?(no)
Any change in V.S? Bp is now 150/90 all others the same.

I want to systemically do examination mainly focusing on abdomen. Neurological examination of


upper and lower limb? Tone(inc) reflex(brisk) power(nl) sensation(nl) ophtalmoscopy? (Blurring
of disc margins) Cardiovascular and respiratory (NL). As for the abdominal examination, in
inspection any scars?bruises? Dilated veins?(no) In palpation, tenderness?(no) Lie(longitudinal)
Presentation(cephalic) FH(34cm) FHR(150).
I want to do per-vaginal examination with the consent of the pt and presence of the chaperon.
Just inspection and speculum (OS closed) thanks I won't go further.
Urine dipstick? ++Protein, Nitrate
-,BSL AND ECG?No ECG no CTG no BSL available(he got cranky I think:D) OK thank you examiner I
go back to my patient.
Jilly during hx and px most likely you have condition called pre-eclampsia have you heard of it
before? This condition more common in first pregnancies and runs in family can be due to
smoking recreation drugs(not sure that was only my performance).
So some particles produced in placenta attaching the baby to the womb( drew picture) are going
to other places in body causing vessel damage and kidney damage as well specifically brain
vessel that is why you have high blood pressure that is serious condition please do not worry
you are in safe hand that is why I want to send you to hospital to be checked as you may develop
seizure and…Bell rang

Feedback: Third trimester Complication, PASS(G.S:5)


Key steps:2,3,4 &5: Yes, 1 :No
Hx:4

Feedback 14-3-2018
37 weeks lady with headache.
Task.. History
Physical examination from examiner
Diagnosis to the patient.

2 min thinking.. my key points for this case.. ( In History; vision problem, epigastric pain, cloudy
urine, oedema, past history of HTN., kidney disease or FH of HTN)

In PE; BP, pulse, reflexes and tone,, fundoscopy, oedema, urine dipstick.

Mg; usg, ctg, delivery and ensuring mother’s and baby’s well being!!

After entering the room and introduction I started by asking if she was okay for me to ask few
questions or needed any painkillers. She said she was fine. I started with typical headache
questions, SORTSARA. It was for the first time she had such a headache. She had it since last
night, constant and everywhere in the head. Almost 5 in intensity. No aggravating or relieving
factors. There was no vomiting or visual aura.
She had no visual problem, no epigastric pain, no change in colour of urine but had oedema
since a week. Baby was still kicking, no reduction in kicks. She hadn’t noticed any swelling on
tummy. There was no bleeding or discharge from her private area. Antenatal history was
uneventful. It was her first pregnancy (so no need to ask complications in previous pregnancies),
no previous miscarriages. No history of HTN or DM kidney disease. or any Family history of HTN.

On PE examiner gave me specific findings that I asked for. BP was 170/120mmHg (at this point I
told the examiner I’ll shift patient to the resuscitation cubicle and start with hypotensive agents)
Pulse 90
Temp 36.5
Reflexes and tone brisk and exaggerated
Oedema +ve
Fundoscopy showed hypertensive changes.
Urine dipstick +ve for proteins
FHR 140 bpm
Fundal height was consistent with age of gestation
Fetal lie was cephalic
I got back to the role player and explained that I’m a little concerned because the cause of your
headache is one of the complication near the end of pregnancy. We call it Pre-eclampsia. In this
the after birth or what we call placenta in medical term( I started drawing here). It starts sending
certain particles in blood that cause changes in blood vessels in brain, eyes, kidney, tummy and
legs etc so you have all that clinical picture. It is an emergency so I’ll refer you to ED to be seen
by specialist. They’ll do CTG, USG and certain blood tests to see the affect of your condition. Our
main concern is your and your child’s well being so you are in safe hands. Do you have any
questions to ask? She said no. The Bell rang
Feedback Scenario Third trimester complication. Grade Pass Global score 4
Key steps 1,3,4 yes 2, No History 4 Examination… 4 Diag/ D/D.. 4 Patient counselling 4

Feedback 19-7-2018
STATION 10 PASS (all key steps yes, score 6,4,6,5,6)

Its Pre eclampsia turning to Eclampsia)

Again long scenario of36 weeks primary gravida came with headache since morning, all previous
finding and test normal.

Task ,Hx ,PEFE , Dx and Mx.

I offered the patient painkiller, asked examiner abt vitals he said 180/90 I asked for nefedipine
spray and rectal diazepam on bedside, than asked very focused history( which he didn’t like
that’s y got 4) than asked focused PEFE and while explaining diagnosis she seized, I asked for
help and DRABCD protocol, I said I will consult my registrar and Obs specialist and start her on IV
anti HTN acc to them, MgSO4 to prevent further seizures, wanna go Blood test ,US and CTG.
Although she is at 36 weeks but obs will decide abt steroid for fetal lung if in case required.

Feedback9-5-2018
19 Case: Headache
Preeclampsia and Eclampsia.
ED department. Your next patient 36 weeks gestations is coming because severe headache. She
has done antenatal care and she has been well. She notices some days ago swelling of her feet.
Task
Take history
PE from examiner
Diagnosis to the patient.
Not sure if they asked about management but examiner asked me at the end.
Patient was lying on the bed. I forgot to ask vital signs before starting consultation. She had
severe headache for the last 3 hours, seeing lights and pain on epigastrium, she noticed swelling
on her legs for the last few days, baby was moving well no any bleeding or fluid per vagina, no
other symptoms. No any relevant personal or family medical history. Patient has done all the
antenatal care and everything has been well until now that she came with severe headache. PE
vital signs 180/110 positive findings: fundoscopy was normal no papilledema, no abdominal
tenderness, increased reflexes, and pitting oedema in lower limbs present. Fetal heart was
present, no uterine activity. Urine dipstick: +++ protein. And patient started to convulse. I moved
patient to resuscitation area gave diazepam and called the registrar and then the examiner stop
me and asked me: which is your management now doctor? I explained all the management for
eclampsia.
Passed. Global score 4

Feedback 19-7-2018
Station 10 : Headache (pre eclampsia turned into eclampsia) Pass
Total 5 key steps 4 yes 1 no Score : 5 5 6 5 overall 5
Case : 35/36 wks pregnant lady having headache since morning. All antenatals were normal so
far .B.P was given outside which was 180 / something (quite high )
Tasks : hx ( for not more then 2 min )
Pefe, dx to pt , mx
When i entered lady was lying on a couch. I asked abt is she is stable enough to continue
with the tasks (yes) Started with calling for help , i/v cannula, basic
blds, nasal nefidpine spray and all emergency equipments in my hands .
Then i asked her abt if she is having this kind of headache fir the first time (migrain ?) Right now
she is having high bld pressure so is it for the first time (already a k/c of Hypertension? ) asked
abt trauma , any previous pregnancy.. any problems during this pregnancy so far.is the baby
kicking? Etc Then asked pefe ... all the abdominal then pelvic examination n then hyper reflexia n
clonus ,
opthalmoscopy. I forgot urine dipstix while asking pefe n remembered it while Explaining dx to pt
... so asked later n then explained dx . While Explaining the lady had a fit and became
unconscious. So then told all the mx to examiner . Call for help , left lateral recovery position etc .
Admit her and will call seniors, do U/S + CTG , blds, start her on i/v mgso4 or hydralazine.then
seniors will decide whether to observe her or proceed with the delivery.

Feedback 9-5-2018
2. Ed setting, 35 wks preg c/o headache. Lying on bed.
Task hx, pefe, explain pt, then mx > pre eclampsia turning to eclampsia , on pefe once BP was
180/100 advised will send nurse to call obs register n another to put iv lines n bloods... Then
continued with rest of the pefe. While explaining the pt collapsed then moved to DRSABC.. then
the examiner asked what about seizure and what anti htn midazolam n hydralzine
2 minutes outside- this maybe pre eclampsia, eclampsia, pregnancy induce hypertension,
trauma on the head. The matter of this question is the task. There is 2 minutes for history only,
PE ask from examiner, tell diagnosis and dd and management. This is really long cases so i need
to be care full. Reflex, urine for protein, 12 cranio nerve, Investigations are needed even they did
not ask.
Inside the room: the patient laid on the bed, she is pregnacy lady. I came in, ask Vital signs, her
BP is high, i try to stable her by medications and call the senior.( It took me 30 s for this) then i
asked about pain questions, some questions about baby kicking and complications of HELLP.
Then I jumbed into PE. I checked her from head to toe including all important point. Then i
thanks to examiner and turn to patient to explain my diagnosis. In the middle of this, patient had
some abnormal movement, I did DRABCD, give her diazepam and call for obstetric for
Magiesium sulphate. I said about all investigations including- blood goup, liver FT, plaplate, KFT,
US, CTG, crossmatching.. Even i thown all of that out, the patient still have abnormal movement,
i said i will observer her and wait for my senior as the medication need time to acting. i will
follow her VS closely .I will call her family and explain the situation as well in case we need to do
CS for patient as this is the ultimte way to solve the eclampsia. Bell reng.

Feedback 25-10-2018 3rd trimester complication


Stem : 32?34 week pregnant lady , primi with headache. Previous all labs normal. Regular with
ante natal visits no complication till now.
Tasks:
Hx
Pefe
Explain probable dx
I entered the room after greetings asked pain scale (5 she said)offered p.k, she refused to take ir.
I told her no worries but if during this consultation u feel its becoming unbearable let me know
please. I excused her and asked my examiner for vitals (he said I will let you know once you
complete hx) I moved to my pt. asked her all SOCRATES of pain then ruled out all other headache
ddx like trauma, stroke, temporal arteritis, flu, sinusitis, meningitis, migraine.
Asked associated symptoms of tummy pain (she said yes for last few days n pointed on
epigastrium) no dizzy no blurry vision leg swelling for 1 week. No other issues. Asked fam hx it
was negative.
Pefe. Asked everything as per KARENs.
Positive findings BP was high, papilloedma was there, tone inc, clonus positive, udr 3+ proteins.
Explained pre eclampsia, she made a bad face while I was explaining so I stopped and asked her
if she couldn’t understand anything , she said yeah I do understand its affecting my kidneys and
eyes now but what is it. I was confused what to say more but bell rang n I came out of room.
Grade: pass
Global score: 6
2/4 key steps covered
Hx: 6
Choice and technique of ex: 6
Dx/ddx : 6
Feedback 25-10-2018
35 weeks of preg came with headache- pass( got 2 in history)
Task: history
Pefe
Diag/dd
I asked hd stability then took history- very short: like sortsara for headache, baby movement,
vision problems, migraine history.
Pefe as in handbook, got 5 for that
Dd said: protein in urine so preeclampsia, could b migraine. Cant remember other dd that I may
have said.

Feedback 5-12-2018
You are working as intern, 36 weeks old lady with right upper tummy pain, Leg swelling when you saw her
last week, All previous Checkup were normal and Investigation also.
Task
1 Take Hx
2 ask PEFE, exam will give u specific findings u want
3 Give Diagnosis and management
+ve- Headache, BOV, ankle edema, BP-180/110, Clonus and hyper-reflexia and UDS-4+ protein, Right
upper quadrant pain, no bleeding/ discharge paravaginal.
Pt was sitting with her Rt hand on tummy. I introduced myself, nice to meet you. I understand you have got
some pain in tummy can you pls show exactly where it is?( she indicated rt hypo chon. And epi. Gastrium)
asked to rate pain. It was 6 or 7 out of 10. (I did not offer pain medication as was confused about it). I took
permission to ask q to examiner. Asked examiner the vitals: BP 180/110,Resp normal, no fever. Told would
like to move to / rx room, give oral nifidipine, and per rectal Diaz, open iv line.
Then proceeded to hx taking: I have arranged initial mx for you. Will you be able to answer if I ask few q?
HOPI Q: pain Q (SORTSARA- continuous dull aching for ½ an hr or 1 hr can’t remember).
Ass symp q: Fever , rash , nausea , vomiting, BOV+ headache+, Leg swelling up to knee, No discharge or
bleeding down below, can feel baby kick, No other medical or sx condition. (did not ask about ANC hx as in
stem it was mentioned normal)
PEFE: general appearance: as you can see
Vitals: as before ( not changed)
GPE: Pickled (only edema upto knee, no jaundice or rash)
Then I said want to do focused abdominal exam: FH, Lie, presentation: normal. FHS: normal, Rt hypc
tenderness+, No tenderness or rigidity in other site, no Ut construction.
I want to do Full neurological exam: Examiner asked what you looking for? I asked about tone, reflex
clonus (hypertonia and clonus present), Fundoscopy- not available
Asked office test urine dipstick for protein +++
(did not ask about resp and cvs)
Turned to pt: Now X from hx and physical examination I think you are having a condition called
Preeclampsia where there is rise of BP with associated leakage of protein in urine. let me explain to you (I
draw the pic. ). Its womb and baby is attached to womb with placenta. Now what happen is some
chemicals are released from here ( I indicated placenta) which may damage the lining of our blood
channels in different parts of our body. That’s why you are having pain in your tummy, headache and BOV.
R you with me? Now what I am concerned is it may turn to a complication called eclamsia means fits while
pregnancy which if occurs can be harmful to you and baby inside. Don’t worry, its good that you came
early. I am gonna immediately send you to tertiary hospital where you will be managed my Sp obs. And
MDT. They will They will give BP lowering meds through your vein called hydralazine and fit preventing
med called MG SO4.They will frequently monitor your BP and urine and baby . if you or baby become
unwell Sp may do C/S. Bell rang. Don’t worry they will take good care of you. Thanked her and examiner
Key step 12345: yes yes yes yes no,
HX 4
Choice and tech of exam: 5
DX/DDX: 4
MX: 3

53-Quinsy
Your next patient in GP practice is a 14-year-old girl complaining of sore throat.
TASKS
-History
-PE from examiner
-Diagnosis and Management

Differential Diagnosis
-Tonsillitis
-Pharyngitis
-Viral URTI
-EBV
-laryngitis

History
1-Sorethroat questions (SORAR-1)
-severity- offer painkiller and allergy
-onset/ constant or come and go/ is it getting worse
-radiation
-aggravating
-relieving
-is this the 1st time? Any past history of tonsillitis?

2-Associated symptoms
Ear and nose
-any ear pain or discharge?
-any cough or runny nose?
Neck and voice
-do you feel any pain in your neck?
-have you noticed any change in your voice?
Dysphagia and odynophagia
-any difficulty or pain during swallowing?
Nausea and vomiting
-do you feel nauseous?
-do you have any vomiting?
Fever and rash
-any fever? Any rash?
Bowels and bladder
-any diarrhea or frequent bowel motions?
-how is your urine output?
LOW, LOA, L&B
Any loss of weight or appetite? Lumps or bumps around your body?

3-Complications questions
-any difficulty opening your mouth? (trismus)
-any shortness of breath?

4-General health questions


-medication and allergies (imp)
-SAD (imp)
-PMH/ PSH
-how is your periods
-Sexual history (if age appropriate) are you sexually active? What are your sexual preferences?
-Contact history (imp)

Physical Exam from examiner


1-General appearance: (DR PJL)
signs of dehydration, rash, pallor, jaundice LAP
2-Vital signs
3-growth chart
4-ENT examination (the examiner will give you photo with deviation of uvula and swelling)
5-Neck stiffness
6-Systemic examination: chest and abdomen (imp)
7-Office test: UDT, BSL, NO SWAB

Diagnosis and Management


-From history and examination, you have a condition called peritonsillar abscess or quinsy.
-The tonsil is a gland present in the throat and fight against bugs and part of our immune or
defence system of the body.
-There is infection behind this gland and there is collection of infectious fluid or pus behind the
tonsil. I can tell it because on examination, one part of the gland is swollen and inflamed, and is
pushing other structures to the other side of the mouth. This is the reason why he is having sore
throat and painful swallowing.

Treatment
-You need to be admitted in the hospital and you need to get the fluid removed or drained
because if not, the infection can go to the blood and spread infection to the other parts of your
body. In the hospital, you will be seen by the ENT specialist who will do the procedure after
numbing the area and after making you comfortable.
-Because you also appear to be dehydrated, I will start you with IV fluids and start you on the 1st
dose of antibiotics, and I will also take blood for investigations such as FBE, UEC, ESR/CRP, blood
culture.
-I will arrange an ambulance for you and I will liaise with the pediatric registrar and ENT registrar
to make them aware of your condition.
-Do you want me to call someone to be with you?

54-Umbilical hernias, inguinal hernia and undescended testis


Case
Umbilical inguinal hernia with cryptorchidism, baby was 7 months. Already diagnosed.
Tasks
A) Explain the findings to the mother
B) Explain further management and complications

Recall of 14-3-2018
7 month old infant with inguinal, umblical hernia and undescended testis counsel
Recall of 6-9-2018
Undescended Testis with hernia . Explain the condition, its implications and management

From a material
A 7-month old baby was brought to your GP clinic by his mother because she noticed a lump in
his groin area and another at the belly button. You diagnosed it as umbilical and inguinal hernia
and you also noticed that the left sided testicle is undescended.

Management:
I have examined your child. I will be explaining the conditions one by one, if you have any
confusion, please let me know.

Umbilical Hernia:
-This is the tummy wall. If there is any weakness in the tummy wall, the tummy contents can
protrude out. In your child's case, the weakness is there in the belly button. Usually in cases of
umbilical hernia, it gets resolved by itself by 1year of age, and larger ones at 4 years of age. The
only treatment is to wait and watch.
-In the meantime, you need to watch out if it becomes red, it doesn’t go back by itself, severe
pain, vomiting. If any of these happens, you need to seek help immediately.
-Don't put anything on it like a coin over that place because it can increase the risk for
strangulation. Strangulation is constriction of blood vessels that can arrest the flow of blood to a
tissue. In strangulation the child can have severe pain with change of the skin to blue or blackish.

Inguinal Hernia:
-It's the same as the umbilical hernia, but in the groin area.
-this has high chance of not going inside the tummy and usually the contents which is the gut
can get blocked and lead to vomiting, distension of tummy, etc.
-So at this stage, we need to refer to the specialist for surgery. Good that you have come early
and we can intervene early.

**6x2 rule: if hernia diagnosed at birth to 6 weeks, operate within 2 days; if at 6 weeks to 6
months, within 2 weeks; if 6 months onwards, within 2 months

Undescended Testis:
-This condition usually is found with indirect inguinal hernia (90%). It is also common in preterm
patient. (20%).
-Normally in males, testis develops in the baby's tummy while baby is still in your womb, and
descends to the scrotum or the sac at the time of birth. But if it is not found in the scrotum at
the time of birth, we usually observe and wait for 3 months. If after 3 months, it is still not
descended, then the child needs surgery since if the testis remains in the tummy, then it will lose
its function when the child reaches puberty.
-But for your child, it is a good thing is you have come early, and since he is already 7 months old,
I will refer you to a specialist now since the optimal time of surgery is 6 to 12 months (but can
operate until 2 years old so as not to affect sperm production)

Problem of non-descent
-Testicular dysplasia
-Trauma
-Risk of malignant change (5-10x)

Advantages of early orchidopexy


-Provides optimal chance of fertility
-Corrects indirect inguinal hernias (coexists in 80%)
-Reduces risk of trauma
-Reduces risk of torsion
-Reduces psychological consequences
-Lessens risk of malignancy

Reading materials and follow up.

Summary
umbilical hernia
-explain
-watch and wait and why (time 1 year till 4years)
-precautions

Inguinal hernia
-explain
-surgery refer (time within 2 months)
-if left untreated

Undescended testis
-explain
-refer and surgery (time 6-12 months up to 2years)
-if left untreated
Feedback 14-3-2018
7 months old with Umbilical Hernia and Inguinal Hernia, cryptorchidism. Task was to counsel the
mother.
2 min thinking time: R/O prematurity or hypothyroidism
Umbilical hernia.. leave it till 1 yr or max 4 yr
Inguinal hernia.. (6x2 rule) explain incarceration and strangulation and warn about sign n
symptoms
Cryptorchidism.. Complications..1) Testicular tumour, 2) trauma 3) infertility
Orcidopexy and it’s advantages

After introduction I asked her that I needed to ask a couple of questions before I tell her
anything. She was fine with that. Her son was a term baby with good weight and no
hypothyroidism.

I told her during my consultation I might tell you some complications so please don’t get
scared. It didn’t mean that her child would have them all.. I just wanted herto be aware of
them. She gave a reassuring nod.

I started by drawing the abdomen and umbilicus and explained how there are weak points in
tummy from where internal gut can come out and how we call them hernia. Explained about
non threatening Umblical hernia and then Inguinal hernia, it’s complications like strangulation,
Incarcertion and told her to be careful if the lump didn’t go back or changed it’s colour or if child
didn’t stop crying. Explained 6x2 rule and referred to specialist.

At this point I asked her if she is understanding and made sure I wasn’t going over board in
giving her the info or overfeeding her. She was happy and said to continue.

Then told and drew about cryptorchidism that how testes descend from tummy and get halted
midway. Explained complications and orchidopexy of both testes. Told her advantages of
orchidopexy that it is protective for Testicular torsion and psychological confidence and there are
good chances of fertility. Told about the safe time to do so which is 12-18 months. And ended
with usual do you have any questions? Etc.

Scenario … Swelling
Grade… Pass
Global score… 4
Key steps 1,2,3… Yes
Approach to patient…. 4
Diag/ D/D… 4
Patient counselling…. 4
Management plan…. 4
Feedback 14-3-2018
Mother asking about her 7months old with Inguinal Hernia – Umbilical Hernia – and
undescended testis – she said she was really worried - Counsel.

My first question was when did you know he had these issues – she said since birth – then I
said where have you been for 7 months !!?
why did you come only now ? she did not know what to say.
Then explained to her about Umbilical hernia no need to worry but we need to follow up as
most of the get closed – then told her about inguinal hernia that needs to be fixed then arrived
at the most important think – the undescended testis and told her this is the most important
thing to be managed as it might lead to sinister tumours if left – then explained to her with a
drawing about how testes descend from the abdomen and that how sometimes the do not
descend completely and cause the hernia then told her that I was to refer her to a surgeon for
management then asked her if she had any questions – she said no Dr – Thank you then I told
her that I would give her reading material - then bell rang

Feedback 15-8-2018
St,tion 11 P,ssed

Next p,tient in GP is 7 months old b,by brought in by f,ther bec,use of lump in the ,bdomen.(Long
stem ,bout umbilic,l herni,,inguin,l herni, ,nd undescended testis is provided)

T,sk:expl,in the conditions ,nd implic,tions


Expl,in ,nd counsel ,bout
furher Mx

-Greeted f,ther ,nd expl,ined e,ch condition ,nd its own implic,tion by dr,wing Expl,ined ,bout
when to do surgery for e,ch condition Referred to surgeon ,nd expl,ined ,bout red fl,gs of herni,
Feedback 15-8-2018
7 months old boy came with swellings in abdomen and inguinal region and undescended testi.
You performed examination and diagnosed as umbilical hernia, inguinal hernia and undescended
testis.
Tasks:
~Explain to the parent about the conditions
~discuss management

A young man was sitting inside room who was the father. I greeted, asked his name and child’s
name.

Told him that I have done the examination. Any concerns? He said just worried about these
swellings etc. I Appreciated him for being concerned and reassured at the same time.

Asked few questions. About lumps, prematurity, hypothyroidism, cough, constipation fhx of
hernias etc.

Started with umbilical hernia. Explained what is hernia and umbilical hernia can be watched till 1
year and sometimes can wait till 4 years. If not gone by then , I will refer to specialist. Gave red
flags.

Inguinal hernia. It’s the one I am concerned about. It can go into obstruction and
strangulation( trapped and can get blood supply cut off) .so surgery is the treatment. Reassured
that ur child will be put to sleep and he is not going to feel anything, done by experts etc
etc,explained 6*2( birth to 6 weeks-2 days, 6 weeks to 6 months-2 weeks, above 6 months-2
months rule and as his child is 7 months old we can go for surgery in 2 months time so I will refer
to specialist. Meanwhile watch for red flags.

Last, undescended testis. In baby boys, testis are inside tummy before birth and they come in
scrotum at or after birth. Sometimes they fail to come. We usually watch for 1 st 3 months of life
and if by that time it doesn’t come then they are to be seen by specialist. Since ur child is already
7 months it’s a good time to refer to specialist. Treatment is surgery ( same again) they will bring
testis down and fix it to thescrotum.

Asked if everythin is clear so far? Does he want me to repeat etc etc.He said no and acted
relaxed.

Grade: Pass
55-Pneumonia risk assessment
67-year-old lady planning a trip interstate in 2 weeks, just discharged from hospital 3 weeks ago.
Tasks
- explain X-ray on admission and x ray today to her.
- Take a brief focused history to asses her lungs
- explain your assessment of future pneumonia risk to her

Approach

Appreciate and acknowledgement

X-ray explanation
-I have got the two x-rays with me. Let us see them together. If you have, any confusion please
let me know.
-this one has been done 3 weeks ago and that one is you recent chest X-ray.
-as you can see this is the lung field, the heart….point on her pace maker shown on x-ray.
-when we compare the two images , you cans see white shadowing over here we call
consolidation which is when the lung tissue has filled with fluid instead of air and this is
suggestive of lung infection or pneumonia.
-the recent x-ray shows a completely clear chest with no shadowing, normal lung field suggestive
of full recovery so let me assure you about it.

History
1-Travel details
-where do you want to go?
-for how long the travel will last?

2-Current condition questions (all normal)


-how do you feel today?
-any SOB, confusion?
-any chest pain?
-any fever, rash? Cough or wheeze?
-funny racing of the heart?
-ankle or leg swelling?

3-General
medication
-do you still take the antibiotics? (No)
-do you take any other medications? (Yes PPI)
- Since when you are taking PPI?
- What would be used for? (GERD history)
-any other medications? (No)
Past medical history
-apart from pneumonia and GERD, have you had any other illnesses? (Yes, Obstructive sleep
apnea)
-since when? Is it under control? (Yes)
-what are you taking for it? (CPAP)
-any other illnesses like heart or lung problems? (I think she has heart disease for which
pacemaker has been done)
-is there any pain, swelling or redness at site of pacemaker? (No)

Have you had flu shot? (Not vaccinated for flu)


SAD (especially smoking).
Family history

Explanation
-there are some factors from the history that can lead to relapse of pneumonia.

-you have a history of GERD, which means reflux of acidic fluid from the stomach into the mouth.
The risk is when this reflux can get down to lungs get aspirated then infected causing relapse of
pneumonia. To reduce the risk try to keep your reflux under control by regular use of your PPI,
less food intake at night, less alcohol or coffee.

-the other one is the history of sleep apnea, this condition prevent air sacs from getting inflated
properly which cause them to collapse resulting in lung atelectasis or diminish in lung size makes
it prone to infection. Also, CPAP if not used properly can cause pneumonia. So, try to use CPAP
properly and make sure you are following the instruction to avoid collapse of air sacs.

-you have pacemaker that can be infected sometimes and if left untreated can cause pneumonia.
So if you noticed any redness, swelling or pain at PM site seek medical advice.

- Good hygiene, stop smoking, take vaccine.

-all of these in addition to increase risk of clotting development in lungs from travel.

-Ace to Australian protocol there are certain vaccines you are eligible for I will look them and
print them for you and don’t worry we will send you a reminder.

Further advice
-so you can travel to Sydney as you are now fully recovered but I need to give you some further
advice regarding your travel.

-make sure to stay close enough to medical centre or hospitals

-try to avoid staying in places with poor air circulation or air conditioning, with lots of smoke or
chemicals.

-try to avoid going to densely populated areas where there is high likely of to have people with
respiratory infections

- during the flight to decrease risk of clotting, try to increase water intake, walk around at least
every one hour or so, move your feet during flight.
Note/ Ask when planning to travel ?
-If not with anyone advice him to have someone.
-It’s advisable to see your cardiologist before travel and I can refer you to specialist for
vaccine( this if his travel is not near).
-If near travel
I will supply you with referral letter containing all your information if by any chance you needed
to go to medical center.

Recall of 9-2-2018
Pneumonia risk effect , I believe it's not fully resolved (on contrast to what is mentioned on the
fb group) patient age 67 years had pneumonia 3 months ago and history of sleep apnea with
cpap and pacemaker for bradycaria..xray chest given for her pneumonia 3 months ago and with
another xray today..she wants to travel to interstate and need advice about pneumonia
take HX and explain CXR to patient.explain pneumonia risk assessent after 7 min.

Recall of 4-7-2018
lady with post pneumonia 3rd wk –plan to travel 2 CXR given with PM
Task-explained Xray to pt, hx, assess future risk of pneumonia
CXR-explained in layman term-clear improvement-pointout that particularly & her PM
Hx-asked about current sym-no fever cough better, no phlegm/SOB/ Cardiac sym-no
CP/palpitation
PMHx-had GORD-on somac
Not vaccinated for flu /pneumonia
SADMA.
Explained-need vaccination
Cardiologist review, GORD aspiration risk /avoid overcrowding
To take medical history & letter
If needed liaise with local Dr
( I forgot to mention DVT preventive measures & not inquired about mode of travel)

Recall of 7-9-2018
Pneumonia risk assessment two X-ray provided she's known case of copd on cpap ppi she want
to travel there was pacemaker on x ray explain x ray, ask hx, counsel about pneumonia risk, mx
plan.

Case (1/3/2017)
Post pneumonia Xray

The patient had pneumonia admitted to hospital. she had IV antibiotic for 3 days and discharged
on oral antibiotic. I asked if any investigation done she said she can't remember what is the
name of antibiotics? she said hard doctor to remember.

Task hx and risk assessment post pneumonia 3 X-ray, the old one and 2 new after recovery it
looks better one AP venues and other lateral
Case (26/7/2017)
67 year old lady planning a trip interstate in 2 weeks, just discharged from hospital 3 weeks
ago.On CPAP for Obstructive Sleep Apnoea.
(pneumonia risk assessment)

comment
Please help if anybody passed this case
An elderly lady come in ED with 2pre and post pneumonia xray.has previous h/o of sleep apnoea
and usingCPAP
NOW HAS NO symptoms.want to travel sydney from Melbourne
Taskd were to take h/0for 3mins
Explain 2xrays to pt
Do the risk assesment for future pneumonia.

- I had this case and I passed it. She had heart burn, pacemaker and aspirations risk. I had to
advise her on all issues.
- Arash Azar I took a history first and realised that she has had sleep apnea. She was on cpap and
she had a history of reflux and heartburn. She had pacemaker and she was using a proton pump
inhibitor.
She also had a history of pneumonia.

and What I did was:


I told her that using cpap is important not to let air sacs collapse. It also avoids hypoxia and
prevents any chanc of having her pneumonia relapsed.
I mentioned that continuing ppi us also important since it avoids risk of aspiration and further
relapse of neumonia.
I advised her about the air travel and the fact that risk of dvt should be considered and i
suggested her to move legs, walk, rest feet on a stool and use ted stockings in order to prevent
dvt and pulmonary embolism. I suggested that she can fly to Sydney but she should stay close to
medical centre or hospital and make sure she has a good insurance policy.

I advised that she should use cpap properly and she should follow the instructions for useing it
properly.
I recommended her to take her medical history notes with her or ask her doctor in sydney to
contact me if required. I was initially very unsure about my performance but in the end it seems
to be enough for them to give a pass score.

- Arash Azar
What I was given at the station was 2 sets of ap and lateral views of a cxray one was taken a few
weeks ago and the other was recently taken. There were consolidations on the old image while
the new ones were completely clear. The patient didn't have any signs or symptoms of
pneumonia and looked quite healthy. It was a while since her medication for her pneumonia
were ceased and she was only taking ppi, I understood that the recent pneumonia is completely
off the list but it is very likely that she develope a similar problem again because of her risk
factors:
1- gord can end in aspirations.
2- cpap can cause pneumonia if misused.
3- sleep apnea can cause athelectasy
4- dvt as a result of travel can cause pulmonary embolism.
5- I'd say there must be issues regarding her pacemaker and increased risk of lung infection
probably as a foreign body, but I was not sure about it so i didn't mention it.

comment
Valentina Wilson CXR pneumonia risk assessment---- I took a history first and realised that she
has had sleep apnea. She was on cpap and she had a history of reflux and heartburn. She had
pacemaker and she was using a proton pump inhibitor.She also had a history of pneumonia. and
What I did was:
I told her that using cpap is important not to let air sacs collapse. It also avoids hypoxia and
prevents any chanc of having her pneumonia relapsed.I mentioned that continuing ppi us also
important since it avoids risk of aspiration and further relapse of neumonia.
I advised her about the air travel and the fact that risk of dvt should be considered and i
suggested her to move legs, walk, rest feet on a stool and use ted stockings in order to prevent
dvt and pulmonary embolism.
I suggested that she can fly to Sydney but she should stay close to medical centre or hospital and
make sure she has a good insurance policy.I advised that she should use cpap properly and she
should follow the instructions for useing it properly.I recommended her to take her medical
history notes with her or ask her doctor in sydney to contact me if required. I was initially very
unsure about my performance but in the end it seems to be enough for them to give a pass
score….

Pneumonia risk assessment: What I was given at the station was 2 sets of ap and lateral views of
a cxray one was taken a few weeks ago and the other was recently taken. There were
consolidations on the old image while the new ones were completely clear. The patient didn't
have any signs or symptoms of pneumonia and looked quite healthy. It was a while since her
medication for her pneumonia were ceased and she was only taking ppi, I understood that the
recent pneumonia is completely off the list but it is very likely that she develope a similar
problem again because of her risk factors:

1- gord can end in aspirations

2- cpap can cause pneumonia if misused


3- sleep apnea can cause athelectasy
4- dvt as a result of travel can cause pulmonary embolism.
5- I'd say there must be issues regarding her pacemaker and increased risk of lung infection
probably as a foreign body, but I was not sure about it so i didn't mention it.

She had a community acquired pneumonia,


As for management part I mentioned these:
1- you can fly to Sydney
2- you should move your feet during the flight or walk around at least once in an hour
3- you should use your cpap properly and you should make sure you are following the
instractions.
4- You should keep your gord under control by measures such as medications, avoiding certain
food or drinks, less food at night time, raising your head higher during the night, less fluids,
alcohol or cofee and....
5-You should take flu vaccine
6-You should make sure you stay close enough to a medical centre or hospital.
7- you should make sure you take deep breath regularly.
8- You should avoid staying in places with poor air circulation, poor air conditioning, smoke or
chemicals.
9- You should avoid going to densely populated places where it is high likely to have people
whith viral or bacterial respiratory infections.

Finally, it is quite clear that sleep apnea prevent air sacs from getting inflated properly and cause
them collapse down which can result in pulmonary infections.

In lay terms, I told the patient that it is important to have your sleep apnea treated in order to
prevent your air sacs collapse and get infected consequently

Copied from previous post reply :


You have to comment on her pre and post xray....post will show resolution of pneumonia...take
history ask about her current s/s any cough/fever/phlegm/sob now.chest pain.if she had any
complications for pneumonia.what treatment she had etc...smoking/alcohol/obesity/ocupation
then ask about any comorbidities..
1.heart problem-pacemaker etc may cause infective endocarditis ,
2.sleep apnea.if she is using cpap machine correctly otherwise it can cause worsening of s/s n
lung collapse.
3.gerd-reflux etc may cause aspiration pneumonia advise to take antacids.prop up position at
night...diet changes.
4.copd/asthma-taking meds regularly knows how to use puffers.deep breathing. exercises.
vaccination fr pneumococall n hinfluenzae.
also do lung function test or atleast pefr to see if she is on correct puffers etc next give her travel
advce:about dvt leg exercises hydration etc taking all her meds...extra as well...briefly abt
mosquito /food n waterborne diseases...if overseas,talk abt vaccination hep a/b/typhoid...then
general advice:quit smoking/safe levels of alcohol.red flags ...reading material
56-Aortic stenosis
A 70-year-old woman comes with complaining of unconsciousness 3 times in the past 1 month
Tasks
- History
- PEFE
- Diagnosis and Differential diagnosis

History
1-collapse questions
-Did you lose your consciousness completely? How long for? (1 minute)
-has this happened before? How often? (This is the 3rd attack)

2-Before
-what were you doing before it happened? (Gardening)
CNS, Vasovagal
-did you get any warnings like feeling dizzy, nauseous or strange smell? (Migraine/ epilepsy/
vasovagal)
-did you get any headache, blurring of vision, weakness? (TIA/ Stroke)
-Any fever or recent infections (Meningitis)
Cardiac
-have you had funny racing of the heart? (Arrhythmia) if yes ask: how long did it last? Regular or
irregular? Do you have it now?
-any chest pain, SOB, dizziness? (May be SOB +ve so ask if he is feeling SOB at rest or exertion or
both? Mostly on exertion. Is he feel SOB on lying flat? If he wake up in the morning short of
breath? Any leg swelling?)
-did these symptoms occur during heavy exercise? (Aortic stenosis)
Endocrine
-did you skip your meals? When was your last meal? (Hypoglycemia)
any head injury

3-During (epilepsy)
-anyone witnessed and noticed any jerky movements?
-any change in colour of lips or skin?
-have you passed urine or soiled yourself?
-have you bitten your tongue?
-have you injured yourself?

4-After
-how long does it last until your recovery?
-after recovery do you feel normal or drowsy? Any N&V or headache?

5-General
-PMH (diabetes, hypertension, hypercholesterolemia, heart D.), PSH, SAD (IMP)
-medications (imp)
-family history of sudden death?
PEFE (like any CVS exam)
1-General appearance
-pallor, cyanosis, dyspnea
2-V/S + postural hypotension
3-JVP, feel carotid bruit
4-Heart
-thrill, heave, apex beat
-heart sounds (s1, s2) + added sounds.
-Heart murmurs (examiner will give you ejection systolic murmur so ask site and radiation)
-listen carotid bruit
5-chest
-chest movement, air entry, breathing sounds, dullness, crackle or wheeze.
6-abdomen
7-CNS
8-sacral and pedal oedema
9-office tests
-UDT
-BSL
-ECG

Diagnosis and differential diagnosis


-Based on the history and PE findings, most likely you are having a condition called as aortic
stenosis. Do you know what it is? I will explain it to you.
-The heart has 4 chambers, 2 upper receiving chambers, and 2 lower pumping chambers.
Aortic stenosis is the abnormal narrowing of the aortic valve, which restricts the flow of blood
from the left lower chamber of the heart into the aorta, which is a big vessel that supply blood
to the rest of your body including the brain and heart itself. If this valve is narrowed, it means
that the heart can no longer pump blood efficiently.
-That is why the blood supply to the brain is decreased, and thus you are having these symptoms
such LOC.
-During exercise, because there is increased demand of oxygen and blood from the heart, you
can get symptoms like shortness of breath or chest pain.

- Other causes could be postural drop, Hypoglycaemia, CNS causes….. . but unlikely in your case

Treatment
-I would like to refer you to the hospital to confirm my diagnosis and to rule out other diagnosis.
-They will do some investigations such as FBE, UEC, ESR/CRP, ECG, and chest x-ray, LFT, RFT, TFT,
and an echocardiogram. And further treatment will be decided by the specialist.

-Once you dischargeded from the hospital, I will be following you up on a regular basis.

-Please avoid strenuous activities and adopt healthy lifestyle modifications.


Case (31/3/2017)
Aortic stenosis
A 70 year old woman comes with complaining of unconciousness 3 times in tge past 1 month
Task
1) History
2) PEFE
3) Dx DDs'
I asked all systemic symptoms, pre and post syncope , nothing positive. SADMA , no IHD , DM
nothing .
PEFE Examiber didnt give me any findings but gave other ESM

comment

Prasanna Buddhika Thanks and the fall case was middle age man 3 falls over one month and all
under similar circumstances where he was working in the garden for half an hour . Excluded the
whole list of DD including the musculoskeletal causes but nothing is positive .
O/E , the only positive finding was an ejection systolic murmur over aortic area with no radiation
to the neck .
I said Likely it's cardiac in origin I would like to thin about some valvular causes and drew the
heart and said could be stenosis of some heart valves or muscular thickening.
classically I would expect a radiation to the neck but still its a possibility and other causes for
recurring falls includes postural drop, hypoglycaemia, hypoxia, CNS causes and etc. but unlikely
in your case

Recall of 28-2-2018
Recurrent falls in elderly male. no history of HTN or DM, everything normal. O/E: ejection
systolic murmur grade 2/3 in aortic area. Dx: Aortic Stenosis. Explain to patient and further
management.
57-Brief resolved unexplained event
A mother of a 10-week-old child has come to ED as the child looked pale, and became floppy. On
arrival, the child is fine and alert.
TASKS
-Take history from the mother
-Tell the mother about the condition and management

Differential diagnosis (Just reading)


1-normal physiological response (gagging, laryngospasm)
2-inflicted injury
-shaken baby.
-drug overdose
-factitious illness by proxy
-intentional suffocation
3-infections
-pertussis.
-pneumonia
-septicemia
-meningitis
4-airway obstruction
-congenital abnormality
-infection
-hypotonia
5-abdominal causes
-intusseption
-testicular torsion
-strangulated hernia
6-metabolic causes
-hypoglycemia
-hypocalcemia or hypokalemia
-inborn error of metabolism
7-cardiac causes
-congenital
-arrhythmia
-prolonged QT
8-respiratory causes
-inhaled foreign body
9-toxin/ drugs
accidental or non-accidental
10-neurological
-head injury
-seizure
-infections
History
1-During the event
-could you describe exactly what happened during the event?
-for how long did it last?
-has this happened before?
-any change in her color; did she turn pale or blue?
-how was her breathing? Any choking or gagging?
-any change in tone; did he appear stiff, floppy or normal?
-any change in conscious state; was she responsive to voice, touch, or visual stimulus?
-have you noticed any jerky movements?
-any abnormal eye movements?

2-Prior to event
-what is her usual sleeping position? Prone, supine or side?
-what is her usual sleeping environment?
-was she awake or asleep when it happened?
-did this episode have any relation to feeding? Has she had any vomiting after feeding?
-any recent illnesses in the child or the family?
-are you aware of any items that could be swallowed?

3-End of events
-is it resolved by itself?
-any intervention done so far?
-was the recovery rapid or gradual?
-any residual symptoms?

4-Others
-Home situation? Any stresses?
-BINDS especially if term or preterm? Any resuscitation after birth?

Physical examination from the examiner


1-GA (DR PJL)
2-Vital signs
3-Growth chart
4-ENT and neck stiffness
5-CVS, chest and abdomen
6-signs of abuse
7-fundoscopy (no accidental injury)
Risk stratification (Just reading)
Low risk
1-age > 60 days (10 weeks)
2-event lasted < 1 minutes (30 seconds)
3-first event (1st)
4-born >= 32 weeks
5-No CPR by trained proff

Low risk = unlikely to have underlying disease and unlikely to recur

If not lower risk - FBE, UCE, BSL, Nasopharyngeal swab (pertussis), ECG (prolonged QT)

Diagnosis and management


-from history and examination your child most likely has a condition called BRUE which means a
marked change in breathing, tone , colour or altered level of consciousness that is followed by a
complete return to normal baseline state.
-the cause of which cannot be explained by any medical cause. However, it is thought to be
exaggerated airway reflexes in setting of feeding, reflux, or increase airway secretions.
-it may look scary but let me assure you that we couldn’t find any serious causes. It can happen
to children less than 1 year of age.

Note/if there is and DDX then you can tell briefly about them.

Management
-you did the right thing to bring her here. I would like to keep her in hospital for observation and
I will call the pediatric registrar to have a look and do further assessment and might consider
some basic Ix(FBC, UCE, BSL, ECG). Once he discharged, he will be reviewed by the GP.

Red flags (Imp)


fits, difficulty breathing, lethargy, not eating, drinking or peeing well.

Recall of 11-4-2018
2 mother with 10 wks girl found in baby cot pale & unresposive for 30 mins
Task
hx , pe from examiner , explain dgx/ddgx
A candidat said it was BRUE ( brief response unexplained event ) !!
58-Antinatal Checkup counselling
Sample case you are GP when 20 years old laura comes to you for advice regarding planning for
pregnancy. She also wants to know the care she is provided with once she becomes pregnant.
Tasks
-further history
-counsel accordingly

History
1-current condition
-when are you planning for pregnancy?
-Any specific concerns
2-5Ps questions
Period
-when was you LMP?
-are they regular?
-how many days of bleeding and how many days apart?
-any pain or heavy bleeding during menstruation?
-any bleeding in between menstruation?
Sexual history
-are you in a stable relationship?
-do you have good support?
-are you or your partner ever been diagnosed with STI?
Pregnancy
-Is this your first pregnancy?
-any previous miscarriages?
Pill
-what contraceptive methods do you use?
-do you still use pill?
Pap
-is your HPV or pap up to date?
3-Lifestyle
-can you tell me briefly about your diet?
-do you do regular exercise?
-SAD ( if alcohol say it is better to quit alcohol once planning for pregnancy as it can cause birth
defects)
-any PETS
4-Occupation
5-vaccination status
-are you immunized against chicken pox and German measles?
6-General
-PMH (DM, hypertension, thyroid, epilepsy, chicken pox, German measles)
-PSH
-family history of miscarriages or birth defects
-are you aware of your blood group?
Prepregnancy
1-SNAP
-make sure you stick on healthy diet with no raw meat or unpasteurized dairy products, no soft
cheese.
-limit tea or coffee intake up to 2 cups a day. Avoid smoking and alcohol.
-Do regular exercise (30 minute/ day for 5 days/ week)
-maintain BMI within the normal range.
-all these life style measure need to continue even during pregnancy.

2-Examination
-need to do complete physical examination.

3-Investigations
I would like to do some tests like:
-FBC, UCE, LFT, BSL, blood group and RH, Vitamin D level
-chicken pox and German measles antibodies to look if your body has the power to fight against
these infections.
-Urine sample
-if HPV not up to date do one now.

*if you are not immunised against rubella and varicella then you need to be given vaccination
but make sure not to become pregnant within 4 weeks of these vaccination.

4-Folic acid
-I will start you on folic acid 0.5 mg to be take 3 months before and the 1 st 3 months od
pregnancy

Pregnancy
1-Confirm the pregnancy
-stop the contraceptive when you are ready for pregnancy and inform me once you miss your
period.
-we will confirm your pregnancy by doing office test called urine pregnancy test amd also blood
tests.
-we will establish the date expected for delivery.
-if you were unsure about LMP we can arrange dating scan at 8 weeks.

2-Further Investigations
-FBC (Hg)/ BSL
-STI screen with consent
-urine MCS
3-ANC visits
-book you onto hospital which is a shared care that we usually give with GP, obstetrician and
Midwife.
-need to come for regular antenatal visits (once during 1 st trimester, every 4 weeks till 28 weeks
then every 2 weeks till 36 weeks then weekly till delivery.

-during each AN visit your weight and BP will be recorded and the baby will also be looked for
with Fundal height, fetal heart rate, lie and presentation.

-we will offer you Down syndrome screen in the first trimester, which is combined Ultrasounds
and blood tests.

-Routine Ultrasound at 18-20 weeks to give an idea regarding any birth defects, position of
placenta and fluid surrounding the baby. This Ultrasound will be repeated at 32-34 weeks.

-you start feeling baby kicks around 20 weeks when you can monitor your baby’s kick.

-sugar test will be done at 28 weeks to test for DM or high blood sugar during pregnancy. Along
with this FBC also will be done.

-Bug test at 36 weeks by taking a vaginal swab.

-just in case if your bay develop any complications you will be referred to high risk pregnancy
clinic with MDT

4-Advice
-it is advisable to take flu shot anytime during pregnancy and also whooping cough vaccine for
you and other family member after 28 weeks.

-it is normal to get a bit of leg swelling. Back pain and also as the pregnancy goes you can feel a
bit breathless.

-you can continue going to work even up to 1 week bedore date of delivery.

5-Rs
-review once blood tests results appear.
-reading materials (normal pregnancy and what to do once becoming pregnant)
-folic acid prescription
AMC exam case 5-4-2018
GP, 27 year old lady comes to see you coz home pregnancy test shows positive & the first
appointment in hospital will be in next 2 months. Her period is irregular
4 to 8 weeks cycle. ( it was quite a long stem )
Task:
-Explain the inv u’re going to do b4 the appointment.
-Management between now & hospital appointment.

Feedback 10-11-2017
ANC
-Long stem , all were normal and confirmed she is pregnant by UPTyour task explain what you
should do for your px before she get her 1st ANC in hospital that has been arranged .
-Again I don’t know what to do, as task was confusing for me but anyway I have to do something.
-Arrange blood test , STI,vit d B 12 level , varicella/rubella , FBC , LFT , UEC , BSL ,urine
microscopy and USG to make sure the embrio grow in right place and is viable , check hormone
pregnancy in blood and explained bit more about those Ix
Script folic acid, Iodine, do SNAP.
-As I don’t know what to say again I said about next ANC routine in trimester 2 and 3 but
examiner cut me off again stick to your task make me more nervous
-Then I talk about SNAP , 4R
-Again messy and disorganised.
Feedback passed approcach /ix/mx 4/3/4

Notes/ no more feedback is there for this case


1-the difference between exam case and the sample case is that the woman is confirmed to be
pregnant and (her cycle is irregular may be or may be not)

2-regarding history will be


current condition
-is it a planning pregnancy? Congratulation!
-then ask 5Ps then lifestyle, occupation, vaccination then general.

3-now as she is pregnant now start from


-doing office pregnancy test to confirm and all blood tests mentioned in Prepregnancy and
pregnancy.
-do not talk about vaccination now as she is pregnant already.
-talk about all lifestyle advice same.
-folic acid is important here.
-then talk about examination and HPV if not up to date.
-talk briefly about ANC if the examiner did not tell stick to task
-talk about advice similar to previous approach
-then Rs add her the red flags very important (vaginal bleeding, tummy pain, fluid leakage, baby
not kicking well)
Feedback 4-7-2018
22 yr lady with irregular periods 4-8wk interval

LMP 9wks ago, home preg+ve

PAP-10/12 ago-normal

Clinic visit in 2/12

Task Ix you order

How do you manage her until clinic visit

No hx/Ex

Ix-

Confirm POA-bhcg

Blood-FBC/Fe studies/bld gp & Ab/TFT/vitD/B12

Serology-Rubella/VZ/hepatits/HIV/syphilis

Urine-m/c/s, Chlamydia/gonorrhoea PCR

u/sscan-dating scan at 7/52 dp on bhcg & confirm intrauterine pregnancy

screening T1 combined screening( blood & scan), non invasive prenatal screening, T2 screening-
explained in brief

Mx-diet-what to eat & what not to eat-eg soft cheese/ exercise/smoking/alcohol- just asked 1-2
Q

Avoid pets/litter

Supplement-folic acid/multivitamin

Dps on Ix-act accordingly

Managing Morning sickness-

In case bleeding/abd pain- seek medical advise

Recall 12-7-2018
Long stem for 27 y old, period every 7 -8 weeks LMP 9 weeks ago, no significant finding in the stem,
recently +ve home pregnancy test, the next anenatal in the antenatal clinic 2 months from now
counsel the pt. about Invx Management until the coming antenatal visit!
59-Parapneumonic effusion
3 years old girl presented to you with fever 39 degree
Tasks
-history
-PEFE
-explain X-ray
-management

History
1-check hemodynamic stability

2-Fever questions
-Duration? Constant or on and off?
-how high is it?
-any chills, rigors or night sweating?
-any rash

3-chest symptoms
-recent viral infection? Runny nose? Sore throat?
-cough? Dry or wet? Describe? (+ve)
-rapid breathing? (+ve)
-any SOB?
-chest pain or tightness?

4-GIT symptoms
-tummy pain?
-vomiting?
-bowel motions?

5-Dehydaryion questions
-confusion, activity, sleep?
-waterworks?
-eating and feeding?

6-BINDS
-immunisation?
-is he thriving and growing well?
-contact? Travel?
-any one smoke at home?
-medications and allergies?
-family history and PMH?
Physical examination from examiner
1-General appearance
-alert, irritable, restless, drowsy, his posture
-pallor, cyanosis
-respiratory distress (*nasal flaring *accessory muscle use *subcostal/intercostal recessions)
-DR PJL. (Cervical LAP)
2-Vital signs especially RR + O2 saturation (temperature 39)
3-Growth chart
4-ENT (sore throat, runny nose, ear discharge) + Neck stiffness
5-chest examination
inspection: deformity, chest movement
palpation: chest expansion, tracheal position
percussion: dullness. (Right sided dullness)
Auscultation: air entry, breathing sounds, wheeze or crackles (decreased breathing sound+
bronchial breathing.)
6-CVS, abdomen

Explain X-ray
-this is the chest x-ray of your child.
-lung field looks clear, heart border size looks normal.
-when comparing both lungs there is white shadowing on the bottom of the right lung field, also
the windpipe is pushed out toward the left side. There is also loss of costophrenic angle in the
right side.
-the X-ray findings showing what we call a Para pneumonic effusion which is accumulation of
fluid between two membranes surrounding the lung in this case the right lung that occurs
secondary to pneumonia or infection of the lung usually by a bug; a bacterial one as he has high
fever.
-it could be tuberculosis which is infection by another bug but unlikely as no travel history or
contact. Could be viral infection but his fever is high. Could be heart, kidney, liver problems or
trauma but unlikely from history and examination.

Management
1-admission to be seen by a chest registrar or specialist, reassure in safe hands and appreciate
they brought him.
2-in the meantime maintain IV line , take blood tests( FBE, UCE, LFT, blood culture, UMCS)
3-start him on antibiotics
4-specialist might decide to do fluid aspiration (take fluid sample to be examined under
microscope)

Feedback 19-7-2018
-Parapneumonic effusion with CXR in a 3year old child. Unscored
- Ped , can’t remember the age of the baby , come with high fever and rapid breathing. Hx taking
, Pefe, explain the pt the x-ray and ur dx.
Feedback
3 yrs old baby girl had high fever ,rapid breathing etc ..hx,pefe,diagnosis n mx .
X ray was given outside which showed right sided pleural effusion n mediastinum shifted to lft
side

I just would like to elaborate on the case of pneumonia in a 3-year old child:
She has 4 days history of cough and fever ( 39C)... taken to a doctor who prescribed amoxylin
syrup.
Brought in to ED with ( ask for PEFE): percussion dullness on Rt side, bronchial breathing,
decreased breath sound ( Rt), SpaO2 93% room air. Cervical LAP. Another task is to explain the X
ray to the father. Look! Do not panic when seeing it. It is really messy CXR. However, you can
recognize the following: obliteration of Rt costophrenic angle, band of effusionextending up,
consolidation/ collapsed lung patch. The heart size and the Lt lung are radiologically clear.
60-Meningitis
Unistudent came to your GP clinic for a sickness certificate due to headache.
Tasks
-History
-PEFE
-Diagnosis with initial management

History
1-Headache questions
-can you tell me more?
-severity+ painkillers?
-onset? Sudden or gradual, constant or come and go? Getting worse?
-site, radiation?
-type?
-timing?
-aggravating and relieving?
-1st episode?

2-Differentia diagnosis questions


Migraine
-did you get any warnings signs?
-any nausea or vomiting?
-is it associated with sensitivity to light or noise? (photo or phonophobia)
Cluster headache
any watery eyes, runny nose
temporal arteritis
-any blurring of vision
-any jow pain? pain on chewing?
Sinusitis
-nasal congestions?
-pain in face?
-recent infections?
Cervical spondylosis
-neck pain
-weakness or numbness?
Meningitis.
-fever?
-neck stiffness?
-rash?
Cancer
-LOW, LOA, lumps or bumps?
Trauma
3-General questions
-PMH
-PSH
-medications, allergies
-SAD, contact, travel
-Family history
-stress at uni or home (tension headache)

Physical examination from the examiner


1-General appearance
DRPJL especially rash, LAP , pallor
2-Vital signs (fever)
3-Neck stiffness
4-ENT
5-CNS
-cranial nerves
-fundoscopy (papilledema)
-tone, power, reflex, sensation
6-CVS

Explain diagnosis + immediate treatment


-from history and examination, there are several possibilities why you have headache.
-it could be tension headache, migraine, cluster headache, problem with backbone of the neck
but very unlikely from history.
-could be infection of air filled sac around the face or viral infection or problem with vessel of
face but again less likely.
-you most likely have a condition called meningitis, which is infection and inflammation of the
coverings of the brain. Usually associated with headache, fever, neck stiffness and sensitivity to
light. Most likely caused by bug; a bacterial one.
-it is an emergency condition and need immediate and prompt treatment.

Treatment
-would like to refer you to the hospital for admission and I will call the ambulance.
-in the meantime, I am gonna secure IV line, take blood for tests FBE, ESR/CRP, LFT, UCE, blood
culture, PCR.
-I will give you the first dose of antibiotic ceftriaxone.
-in the hospital you will be taken cared by a team might need septic workup (chest x-ray+ urine
MCS, CT scan +/- LP.)

Recall 6-7-2018 Headache - w/ fever, papilloedema


61-Slipped upper femoral epiphysis
Your at your GP when 12 years old boy brought in by mom complain of left knee pain for the past
2 months, which makes it hard to play sports at school.
Tasks:
-take focused history
-ask relevant PE from the examiner
-give your diagnosis and management to mom.

Differential diagnosis for any limping child according to age


1-birth-2 years DDH / Abuse

2-2years-10 years Perthes/ transient synovitis

3- More than 10 years SUFE, Osgood schlatter

4-18 and more patellofemoral syndrome, meniscal tears, collateral ligament injuries

5-All ages HOST M  H= Henoch schonlein purpura


O= osteomyelitis
S= septic arthritis
T= trauma and transient synovitis
M= malignancy

History
1-Pain questions
-how severe is his pain from 1 to 10, 1 is the least 10 is the most? Offer painkiller after allergy
question
-when did it start? Sudden or gradual? Constant or come and go? Is it getting worse?
-where exactly the pain is? Any pain in the right knee or anywhere else? Does it go anywhere
else?
-how does he describe the pain?
-anything make it better or worse? Have you tried anything to relive it?
-is he able to walk or bear weight?
-has this happened before?

2-Differential diagnosis questions (HOST M)


-any tummy pain or rash? (HSP)
-any fever? (if yes ask fever questions and dehydration but here will be no) (OM)
-any joint pain or swelling? (SA)
-any recent flu or viral infections? (TS)
-has he had any trauma? (T)
-any loss of weight, appetite or lumps and bumps? (Malignancy)
3-key questions (only for SUFE)
-how is his weight compared to his classmates?

4-past history
-any history of thyroid disease, asthma or any illnesse?
-is he taking any medications (steroid) or chemoradiotherapy?
-any family history of joint or bone problems?

5-BINDS
especially growth and development

Physical examination from the examiner


1-General appearance (DR PJL)
dehydration, rash, pallor, jaundice , LAP

2-vital signs especially fever

3-Growth chart (can ask even if 12 y.o)

4-Quick chest and heart examination

5-leg focus exam


-with adequate exposure and in the prescence of mom I would like to examine both legs
including hip, knee and ankle joints
-Gait
-look (SSSDW) scar, skin color change + bruises, swelling, deformity, wasting
-feel: tenderness, temperature, joint effusion
-move: any restriction in rang of movement
-measure: limb length
-roll test

6-office tests
-UDT
-BSL
Explanation
-from history and examination he most likely have a condition called SUFE have you heard about
it?
It is a disease of hip joint so actually the problem is in his hip not the knee. Draw a diagram just a
ball and socket; this is the head of thighbone and this is the growth plate. Any weakness of
growth plate causes the head to slip out of its position. Leading to limping, pain and restriction
of movements.
-the cause is usually unknown but may be due to increase weight or hormones.
-it is important to treat to avoid complications like decrease in blood supply to the thighbone so
avoiding any bone damage or deformity.

-other possibilities could be infection of bone; osteomyelitis or infection of joint; septic arthritis
but unlikely as he has no fever or joint swelling. Could be inflammation of lining of hip joint;
transient synovitis but he has no recent viral infection. Osgood schlatter or inflammation of bony
prominent of shin bone but unlikely, Could be trauma or nasty growth but unlikely.

Management
-I need to do some blood tests and refer for an x-ray of both limbs 3 views.
-need to rest the limb and do not allow him to bear or carry weight on both legs.
-Give him painkillers

-If it is SUFE:
*send to hospital for admission to be seen by specialist
*treatment usually surgery by reducing the bone and fix it into its place.
*might need treatment of other leg as well.

-reassure
-reading materials
62-Perthes disease
23-2-2018 another child with limp. Xray showed Perthes disease

You are GP when a 6 year old child was brought in by mom because of limping on the left side.
Tasks
-History.
-PEFE
-Ix
-Diagnosis and management

Note/ this case is Karin it might not be the same exam case because the only information given
in in 23/2/ 2018 which showed that the chief complaint was limping and x-ray will be given in
the exam to know it is perthes.

History
1-Chief complaint questions
-for how long has he had limping?
-is he limping on one or both sides?
-does he have any pain?

2-Pain questions
-severity
-sudden or gradual/ constant or come and go/ if getting worse
-site and radiation
-character
-anything make it worse like movement or better like rest
-has this happened before

3-Differential diagnosis (HOST M)


-tummy pain, rash
-fever
-joint swelling
-recent infections, trauma
-LOW, LOA, lumps and bumps

4-Past history
-PMH and medications
-family history of joint or bone problems

5-BINDS + abuse
-birth, immunization , growth and development,
-social (home situation, school performance, who is taking care of the child most of the time?)
Physical findings from examiner
the same like other lectures of limping

Investigation
-ask about blood tests and X-ray both limbs 3 views ( will be given showing Perthes)

Explain
-from history and examination and x-ray findings, your child most likely has a condition called
Perthes disease or avascular necrosis of the head of femur.
-because of unknown cause the blood supply to the head of thigh bone is reduced, could be
related to viral infections or trauma.
-usually present as pain, limping and restriction of movements.
-it is good that we have caught it at early stage t prevent complications like deformity or
permanent damage or abnormal positioning of the head of femur.

Treatment
-stop weight bearing and rest the affected leg
-prescribe painkiller
-send to hospital for admission and to be seen by bone specialist
-specialist will do further assessment to find how much damage has occurred.
-leg will usually be fitted in a plaster cast or brace to contain the bone into the socket.
-Give crutches to prevent weight bearing
-it might take weeks, months or years to heal but mostly self-limiting.
63-Acute pancreatitis
Case 1
HMO at ED, 25 years old man presented with upper abdomen pain. Vitals: T: 37C, PR: 92, BP:
100/70, RR: 20, O2: 94%.
Tasks:
Take history
Dx
Investigations to patient

Case 2
52 year old female came with the complaint of RUQ pain,
Task:
Hx,
PE,
dg and dds to the pt with reasons

Case 3
Your next patient is a young lady in ED. She has pain in upper abdomen
Tasks
-Take history for no more than 3 minutes.
-Explain the results to the patients
-talk to her about management
* You are not supposed to perform any examination or ask for any examination findings.
The results of the provided investigations consisted of following:
-FBE: WCC 10,000.
-otherwise normal electrolytes in normal range with a borderline K.
-Urea Cr were slightly more than upper normal ( especially Urea)
-lipase:1200 ( highly increased)
-LFT: slightly increased aminotransferases( maybe double of Nl limit) I do not remember the
exact measures for alkaline phosphatase, but it was increased. GGT was also increased (more
than the rest of the LFT)
-ECG: NL (there was just written that ECG is normal. There was no ECG for interpretation)
History

1-is my patient hemodynamically stable (VS and O2 Sat)

2-Pain questions
-severity (painkillers + allergies) (sever and offer)
-onset? Sudden or gradual? Constant or come and go? Getting worse? (2-3 hour ago, sudden,
constant, getting worse)
-exact site and radiation? (epigastrium, radiate to back)
-character?
-does anything make it better like sitting or leaning forward? (relieved by leaning forwards)
-does anything make it worse like breathing, coughing, movement or fatty meal? (worse on lying
down)
-if it is the first time?

3-associated symptoms
-nausea and vomiting? How often and what color and content? (Yes vomiting)
-Bowels and bladder (esp colour)
-fever, rash and recent infections
-LOW, LOA, lumps and bumps
-yellowish Discolouration of skin, itching
-chest pain, SOB

4-General questions
-Past medical history (gall stones, stomach ulcer, hyperlipidemia, DM, HTN, mumps)
-past surgical history (ERCP, cholecystectomy)
-medications (NSAIDS, steroids)
-Smoking, ALCHOL if yes ask how much for how long, drugs

Causes of pancreatitis (GET SMASHED)


-gall stones
-ethanol
-trauma
-steroids
-mumps, malignancies
-autoimmune
-scorpion or spider bite-hyperlipidemia, hypercalcemia
-ERCP
-drugs
Physical examination findings from the examiner (may be card)
1-General appearance
anxious, sweaty, in pain
pallor, jaundice, dehydration, LAP.
2-Vital signs
3-Focus abdomen
inspection: movement with respiration, distension, visible masses, scars, dilated veins
palpation: tenderness, rebound tenderness, guarding, rigidity, murphy sign.
auscultation: bowels sounds
-complete with: hernia orifice, genitalia, LN, per rectal (+ dark stool)
4-CVS
apex beat, thrill, heave, heart sounds and murmur
5-Chest
-dullness, air entry, breathing sounds, crepitation or wheeze,
6-Offiec tests
-urine dipstick -BSL -ECG

Explanation (draw diagram)


-there are various reasons why you have upper abdominal pain. the most likely is acute
pancreatitis which is an inflammation of the pancreas which is a gland that secretes both
digestive enzymes and important hormones.
-it could be caused by gall stones or chronic heavy alcohol intake, rarely trauma, surgery or
medications. In your case is most likely because of heavy and chronic alcohol drinking we call
alcoholic pancreatitis.
-other possibilities are:
stomach ulcer or ulcer in the first part of the gut, inflammation of the gall bladder or bile duct,
liver infection, lung infection, heart problems.

Investigations
-FBC, LFT, UC&E, BSL, lipid, viral serology
-Lipase and amylase
-USD
-CT
*if interpretation so you need to explain one by one like the case in page 1.

Treatment
1-admit to hospital
2-analgesia to control the pain.
3-NPO+ IV fluid
4-Input/ output monitoring
5-switch either to oral if tolerate feeding or NG tube after 48 hours of IV
6-ERCP if stone only
7-surgery to remove gall bladder if stone is the cause only
NO ANTIBIOTICS.
8-need to address alcohol later as it can cause damage to many organs including liver and
pancreas.
Feedback 1-3-2018
abdominal pain PASS
ed woman with epigastric pain,.lipase increased, ggt and ast increased
murphy neg
hx, explain diagnosis,mx
vitally stable??
took complete hx for epigastric pain using socrates and ruling out other ddx, sadma. hx of
chronic alcohol abuse .
explained diagnosis as pancreatitis ,drew it, explained to her .
mx short term to treat her pain and inflammation so admit ,told her we will admit you and
correct your electrolytes ie suportive treatment and long term would be alcohol abstinence so
explained that to her that we will help you quit alcohol ,and make a plan for you ,might require
rehabilitation but it will be MDT approach and involve family etc

Feedback 4-4-2018
Abdominal pain. Vitals stable given.
Task
hx
Diag and dd with reasons
Investigations With reasons.
On hx pain was epigastric radiating to back, relieved by leaning forward, alcohol 5-6 glasses per
day, smoking 1 pack per day.
All remaining hx negative. Most likely- acute pancreatitis
Took whole history, told all DDs with reasons and then all investigations with reasons..

Case (30/3/2017)
Epigastric pain.
Task.
History.6 minutes.
Alcoholic. Off and on pain from last year. No pain at this time.
Most likely diagnosis and other D/D. Pancreatities and told other D/D of epigastric pain.

Case (31/3/2017) Acute pancreatitis


A long stem was given with a mid 40's lady complaining of severe abdominal pain radiating to
the back for the past few hours. Murphy's negative. A list of all Lab investigatons was given.
LFT's deranged and serum lipase extremely high
tasks
A) Take further hx
B) Dx and DD's
She was Alcoholic on asking

Case (26/8/2017)
A man came with epigastric pain , chronic drinker Pancreatitis ??
Task : hx , Ddgx with .......??
Feedback 16-8-2018
Acute pancreatitis
HMO at ED, 25 years old man presented with upper abdomen pain. Vitals: T: 37C, PR: 92, BP:
100/70, RR: 20, O2: 94%.
Task:
Take history
Dx
Investigations to patient

Patient is sitting on bed. Examiner said no need to give O2. I offer painkiller. He seems to be
very painful. I ask want to lie down- he said more painful when he lies down. The pain started 2-
3 hours ago, suddenly, 1st time, radiate to back, Relieved by leaning forwards, worse by lying
down, associated with nausea & vomiting x ?1 times. Yellow color fluid. No chest pain, no SOB,
no cough, normal poo & pee. no lumps & bumps on body, no wt loss & loss of appetite. no h/o
of injury to tummy.
No DM, no high lipid, no stone history.
He drinks alcohols for many years and recently has history of drinks a lot (?party). No smoking,
no recreational drugs. No taking medication.
I explained Acute pancreatitis with drawing picture. Other possibilities: liver, gall bladder,
stomach, lungs, heart.
I checked blood test (FBE, enzyme fr pancreas & ht, bld sugar, fat level, RFT, LFT, EL, .....), X ray to
tummy, USG, CT if needed. ECG, CXR, Urine test.
 Grade: Pass Global score: 4
Key Steps: 1- Yes
2- Yes
3- Yes
  4- Yes
Approach to patient: 4
History: 4
Diagnosis/ Differential diagnosis: 4
Choice of investigations: 4
Feedback 22-6-2018
52 year old female came with the complaint of RUQ pain,
Task:
Hx,
PE,
dg and dds to the pt with reasons

Approach
Greeted patient
Asked about the pain scale and if she needs any pain killer ( said already given).
So I told her please excuse me I need to check your vitals then I will come backto you
Asked examiner—examiner said as given in the stem ( I think I missed it in the stem because I
wasthinking about my performance in previous station and could not concentrate).
So I checked in the question inside and asked is it same as before. He said yes.
So I started asking SIQORRA and relation to food, alcohol (positive for pancreatitis)

Then asked specific questions related to dds


Organic
GORD, Peptic ulcer disease
Pancreatitis, Hepatitis
UTI, Constipation, Diarrhoea
MI, Pericarditis, Pneumonia, Tumor.
As she was female asked about periods she said menopause 3 years ago
Said I am going to ask you some sensitive questions
Then asked about discharge from down below, sexually active or not, Any history of STI,any post
coital bleeding.
The small bell rang I quickly asked about stress in work, family
PEFE
Asked examiner GA
He handed me a PE card (a time saver!!) But surprise everything was normal per abdomen
examination
As much I can remember I think in office test only Urine dipstick was done so I asked for ECG
and Blood sugar level ( examiner said not available).
Then I went to patient drew the whole abdomen and mentioned every dd with reasons while
showing the organ and mentioned most likely as pancreatitis
I had plenty of time in this station so I asked if sheunderstands everything.. if she needs to ask
me anything. She said no.
So I mentioned we are going to transfer you to hospital, keep you nill per mouth, start IV line and
do some blood and imagng investigations to confirm the diagnosis and rule out other cause.
The bell rang I said thank you
Feedback 21-6-2018 Abdominal Pain: Pass: Global score 6
Your next patient is a young lady in ED. She has pain in upper abdomen that has worsened
gradually during the last 2 hours. But it clearly guided toward a pancreatitis) the results of basic
investigations is ready. Take history for no more than 3 minutes. Explain the results to the
patients and talk to her about management. I am not sure whether it was mentioned immediate
management or not. I just explained everything. You are not supposed to perform any
examination or ask for any examination findings. The results of the provided investigations
consisted of following: FBE: WCC 10,000. otherwise normal electrolytes in normal range with a
borderline K. Urea Cr were slightly more than upper normal ( especially Urea) ( I inferred it as
dehydration)lipase:1200 ( highly increased) LFT: slightly increased aminotransferases( maybe
double of Nl limit) I do not remember the exact measures for alkaline phosphatase, but it was
increased. GGT was also increased (more than the rest of the LFT) (overall in suggested an
alcohol related problem. ECG: NL (there was just written that ECG is normal. There was no ECG
for interpretation) I entered the room and asked the role player about the severity of pain, and
offered pain killer after checking her medication allergy. As the stem had guided towards
Pancreatitis I took my history in a way to confirm pancreatitis and look for underlying reasons.
Such as the quality, location, duration and radiation of the pain, again, as the LFT results in the
stems guided towards an alcoholic liver disease, asked her about alcohol consumption, (before
that checked her past medical history, previous liver disease, any medication, smoking and
alcohol and recreational drug. Also asked about any previous episode like that and also about
any familial Hyperlipidaemia in the family or herself. She confirmed alcohol drinking, I forgot to
ask about consequences of alcohol on her social life, such as driving offences or relationship
problems, but explained the results to her that there could be many reasons for her pain. I
briefly explained about the anatomical location of the lungs, heart, stomach, liver, gall bladder,
pancreas, intestine, and explained that the irritation or inflammation in any of these organs can
cause a quite similar pain. however, the characteristics of her pain and also the results of her
blood investigation strongly suggests an inflammation of the pancreas as the reason for her pain.
and also the history that she has binge drinking and her LFT can justify this diagnosis even better,
so most likely she has alcoholic pancreatitis. then explained to her that in short term, we need to
control her pain, we need to hydrate her by IV infusion, and we need to provide some rest for
her GI system , by inserting an NG tube (used layman terms), after controlling her pain. she may
need some imaging and also may need antibiotic later in the course of treatment, which will be
decided later. After controlling her pain, we needed to address her binge drinking as her liver is
showing increase in Gamma Glutamyl Transferase, which is the most sensitive test for the
damage of the liver. (it shows that her liver is not damaged yet, but if she keeps drinking with the
same pattern she may develop serious health problems and also liver problems. As well, I
explained that I would like to do more investigation to exclude other possible reasons that could
cause pancreatitis, such as hyperlipidaemia. She looked happy with my explanation, the
examiner asked what I meant by “Imaging”, I explained that we need to perform an ultrasound
examination to make sure that all the other organs other than pancreas are normal and there is
no free fluid in the abdominal cavity, but to visualize the pancreas we need to do a CT scan, as
ultrasound cannot visualize pancreas well.
Recall 1-3-2018
Pancreatitis: Ed setting, 32 yr old women presented with epigastric pain, blood results showed raised
lipase, pancreatitis and raised ggt abd ast. H/o intake of 5-6 sd daily, murphy sign negative. Taskhx,
explain diagnosis and dd, treatment .

Recall 4-4-2018
Abdominal pain. Vitals stable given.
Task- hx
Diag and dd with reasons
Investigations With reasons.
On hx pain was epigastric radiating to back, relieved by leaning forward, alcohol 5-6 glasses per day,
smoking 1 pack per day. All remaining hx negative. Most likely- acute pancreatitis

Recall 21-6-2018
your next patient is a young lady in ED. She has pain in upper abdomen that has worsened gradually
during the last 2 hours. ( maybe some other details of the pain was mentioned in the stem. As history
taking was a part of the task I cannot remember which part of information was provided before
entering the room. But it clearly guided toward a pancreatitis) the results of basic investigations is
ready. Take history for no more than 3 minutes.
Explain the results to the patients and talk to her about management. I am not sure whether it was
mentioned immediate management or not. I just explained everything. You are not supposed to
perform any examination or ask for any examination findings. The results of the provided
investigations consisted of following:
FBE: WCC 10,000. otherwise normal electrolytes in normal range with a borderline K. Urea Cr were
slightly more than upper normal ( especially Urea) ( I inferred it as dehydration) lipase:1200 ( highly
increased)
LFT: slightly increased aminotransferases( maybe double of Nl limit) I do not remember the exact
measures for alkaline phosphatase, but it was increased. GGT was also increased ( more than the rest
of the LFT) ( overall in suggested an alcohol related problem. ECG: NL ( there was just written that
ECG is normal. There was no ECG for interpretation)

Recall 6-8-2018
Acute abdominal pain in young female
##Acute Pancreatitis##�

Recall 12-4-2018
in ED a 21 year old lady comes with upper abdominal pain started
recently.tasks:
* Hx
* no p/e
*DDX and Dx and Mx.
in the room , pt was pretending severe pain,bending forward, has pain
10/10, started 2 hours ago in the morning, first time, pain killers not
helpful, no trauma, LMP was last week, practice safe sex, no any previous
hx, FH of abdominal problem, no smoke, drank 6 cup of alcohol last night,
has vomited several times not bloody, pain get worse when lying down. no
fe er, no jaundice, no waterworks or bowel problem.
Radiates to back. no special diet , no farty food last night.
Case (23/2/2017)
Acute abdomen
middle age man, sudden severe pain in upper abdomen, alcoholic.
Task
Perform PE,
likely DDx with reason.

Sorry, I can’t really remember how I started my physical examination. This is what I guess I did:

Greeted by examiner. Patient was lying on the couch. Washed my hands.

Greeted and introduced myself.


Asked my patient about his pain and severity, any allergy on medication, offer pain killer after
asking patient V/S from examiner.

Explain to the patient about the physical examination and gained consent.

Asked patient to exposed himself till abdomen.

G/A: pallor, jaundice, diaphoresis.


Hand: CRT, pulses.
Face: conjunctival pallor and scleral jaundice, any coated tongue

P/A:
inspection – skin changes, bruise, pulsatile mass, distention.
Superficial palpation – started from furthest part away from the pain and there was voluntary
guarding over the epigastric region.
Did not do deep palpation as I would not want to induce more pain to him.
Tried to do Murphy sign – patient unable to do so due to pain.
Forgot to auscultate for bowel sound.
Told the examiner I would like to examine the inguinal area for hernia and LN, and would like to
perform a DRE (examiner said unnessary).
Thanked the examiner anyway.

Completed my examination and washed my hands.

Probable Dx: acute pancreatitis - explained in layman terms with diagram, reasons for this:
sudden onset, location, pain radiate to the back, no other sx like jaundice etc. (forgot to mention
chronic alcoholism).
Other causes (with diagram): gastritis/ulcers, hepatitis, cholecystitis, lower lobe pneumonia, MI,
oesophagitis. (DDx score 6)

Passed, global score 5


Case (31/3/2017) Acute pancreatitis
A long stem was given with a mid 40's lady complaining of severe abdominal pain radiating to
the back for the past few hours. Murphy's negative. A list of all Lab investigatons was given.
LFT's deranged and serum lipase extremely high
tasks
A) Take further hx
B) Dx and DD's
She was Alcoholic on asking

Case (29/4/2017) (diagnosis not confirmed could be peritonitis due to ruptured bowel,
diverticulitis)
ED, A middle age woman with tummy pain since last night. Pain in the epigastrium and chronic
alcoholic and she smoked one to two cigarettes for years. Otherwise normal.

Tasks- Perform relevant examination with running commentary to examiner


Explain Dx and DDx with reasons to patient

Positive findings- Patient was irritable with pain, and not cooperative at all. Tenderness the
whole abdomen and more prominent on epigastrium, No rigidity and guarding. I turned to the
examiner and told that I would like to skip to test organomegaly and percussion to test free fluid
coz my patient is in deep pain. The examiner said it was OK. But it did percussion to access liver
dullness obliteration present or not. Bowel sound was absent on Auscultation. Murphy sign was
positive but I didn’t do other special tests. PR and VE were normal. I forgot to mention ECG and
other bedside tests.

Dx – Acute abdomen probably acute pancreatitis.


64-HRT Counselling
Case 1
You are at your GP when 53 year old Mary presents to you. She has come to you to discuss about
HRT.
TASKS
-Focused history
-Counsel regarding HRT

History
1-Approach
-I can see from the notes that you are here requesting for HRT. Can you tell me why?

2-Indications Questions
Vasomotor symptoms:
-Are you experiencing hot flushes, heavy sweating?
Psychological:
-Are you having mood changes, sleep disturbances, depression?
Somatic symptoms:
-any muscle aches and pains? Any bone pain? Any history of fractures?
Reproductive symptoms
-Any vaginal dryness, itchiness, discharge?
Urinary symptoms
-Any burning or stinging while passing urine? Do you have to go to toilet more frequently? Do
you fell any lump downbelow?
-bowel motions along with it
history of osteoporosis

3-5Ps questions for old women


menopause (as period)
-At what age did you have your menopause?
-Any bleeding after menopause?
Partner: do you have good support
HRT (as pill): no need here as she is requesting)
pregnancy : How many pregnancies have you had?
Pap or HPV and mammogram: if up to date

4-Contraindications for HRT questions


- Any undiagnosed vaginal bleed?
-Any history of clotting in your leg veins?
-any history of cancers (breast, womb or ovaries? Lumps or bumps?
-Any history of stroke
-Any recent heart problems like angina or heart attacks?
-Any history of active liver disease?
-Diabetes, uncontrolled blood pressure, high cholesterol
-Any medical or surgical conditions in the past?

5-Lifestyle
-healthy balanced diet
-regular exercise
-SAD
-How does these symptoms affect your life?

Counselling
-Look Mary why you have menopausal symptoms is due to lack of hormone estrogen. As after
menopause, the ovaries shut down producing very low levels of estrogen and no progesterone
at all.
-HRT has both estrogen and progesterone in it and this will replace the lost estrogen and
progesterone added to estrogen to prevent the thickening of the endometrium or the inner
lining of your womb.

-HRT is usually a combination of both estrogen and progesterone so it can carry some risks: can
lead to DVT or increased clotting tendency in your veins, stroke, breast cancer (especially if you
use it for more than 3 years), womb and ovarian cancer, and gallbladder disease.
-The benefits of HRT is that it can relieve all menopausal symptoms, and can make your bone
strong and protects against fractures. Also, give some protection against bowel cancer it is found
that there is a possible decrease in the incidence of Alzheimer's and other forms of dementia.

-Before starting HRT, I need to do full examination looking for your general health and
reproductive health. Need to record you BMI and BP. Examine your lungs and heart and do pelvic
examination as well.
-I need to do certain blood tests like FBE, UEC, LFT, BSL, TFT, lipid profile.
-Mammogram and HPV should be up to date

-you need to take a continuous combined HRT, which is estrogen and progesterone to be taken
in continuous way. You can take it orally preferably the same time every day or patches.
- Side effects of HRT:
-Nausea, headache, breast tenderness, blotting
-breakthrough bleeding: Usually settles in 3-6 months' time

-Review every 6 months


-will try to stop it in 2 years and no more than 5 years
-lifestyle modifications: exposure to sunlight avoiding peaking hours, diet rich in calcium, regular
exercise

Note/There are different methods for giving HRT. HRT can be given in 3 separate ways:
-Estrogen only HRT: this is given if the patient had hysterectomy already
-Cyclic/Sequential HRT: if the patient is menopausal for up to 1 year, or perimenopausal
Give estrogen continuously for 28 days, add progestogen from 14 th day to 28th day of cycle.
Once you stop this, the patient will have withdrawal bleeding
-Continuous combined HRT: if the patient is menopausal for more than 1 year
Estrogen and progesterone for 28 days
She will not get a bleed at all

Case 2 (AMC Case)


You are GP, 52 year old lady come for requesting HRT. Her LMP was 18 mths ago.
Task
-Take relevant HO
-The examiner will give u PE findings on card
-Explain patient initial management and investigations that u want to do

History
1-Approach
-I can see from the notes that you are here requesting for HRT. Can you tell me why? (My friends
recommended it to me)

2-Indications Questions
Vasomotor symptoms:
-Are you experiencing hot flushes, heavy sweating? (No)
Psychological:
-Are you having mood changes, sleep disturbances, depression? (No)
Somatic symptoms:
-any muscle aches and pains? Any bone pain? Any history of fractures? (No)
Reproductive symptoms
-Any vaginal dryness, itchiness, discharge? (Yes dryness and itchiness)
Urinary symptoms
-Any burning or stinging while passing urine? Do you have to go to toilet more frequently? Do
you fell any lump downbelow? (No)
-bowel motions along with it
history of osteoporosis (No)

3-5Ps questions for old women


menopause (as period)
-At what age did you have your menopause? (18 months ago)
-Any bleeding after menopause? (No)
Partner:
-Gonna ask you sensitive questions
-Are you sexually active? (Yes)
-Any problems or pain during intercourse? (Yep Dr. I need to use a lot of lubricants for sex)
-do you have good support
HRT (as pill):
-no need here as she is requesting)
pregnancy :
-How many pregnancies have you had? (I have two children with normal delivery)
Pap or HPV and mammogram:
- if Pap or HPV up to date (pap smear it was done 4 years ago and normal)
- What about ur mammogram? (I haven’t done it before)
4-Contraindications for HRT questions (all normal)
- Any undiagnosed vaginal bleed?
-Any history of clotting in your leg veins?
-any history of cancers (breast, womb or ovaries? Lumps or bumps?
-Any history of stroke
-Any recent heart problems like angina or heart attacks?
-Any history of active liver disease?
-Diabetes, uncontrolled blood pressure, high cholesterol
-Any medical or surgical conditions in the past?

5-Lifestyle (all normal)


-healthy balanced diet
-regular exercise
-SAD

Physical examination from examiner


Thank you for ur information. I like to go to examiner for some findings and I ll get back to u
soon. (Findings will be given on card)

PE findings (a long list full of one page)


GA - well. BMI - 30, Vital signs - All stable.
CVS, Resp, Abdomen, Breast - all normal
VE - Dry atrophic vagina, all clear on BME.

Counselling
-I finished examination and I could not find anything serious. Your BP is OK; breast exam is fine,
Heart and lungs normal

-look Mary, HRT is a hormone replacement therapy, which contains two hormones estrogen and
progesterone. This is usually given if you have clear indications of some of menopausal
symptoms like hot flushes, sweating, and mood changes.
-HRT never prescribed without clear indications as it can carry many risks like increased clotting
tendency in your veins, stroke, breast cancer, womb and ovarian cancer, and gallbladder disease.
-at this time, it is better not to start you on HRT because of its risks.
-Just in case you develop any indications, we will be happy to start you on HRT if you do not have
any contraindications for it. And after complete examination and investigation.

-from the history you most likely have atrophic vagina means the lining of you vagina is dry and
thin most likely due to lack of hormone estrogen. As after menopause, the ovaries shut down
producing very low levels of hormones, which are essential to maintain the lining of the vagina
healthy. That is most likely why u have dry vagina and pain during sex and need to use lubricants.

For that, I can give u Oestrogen creams.


I need to run some Investigation as well to check your general health
-I need to do certain blood tests like FBE, UEC, LFT, BSL, TFT, lipid profile.
-Mammogram and HPV should be up to date

-From examination, it looks that your body weight, is higher than it should be. I would like you to
follow some life style measures
S - Smoking avoided
N - Nutrition : take balance diet. I can get involve dietician who can give u proper diet plan to
help u lose wt.
A - Avoid alcohol if possible
P - Physical exercise: do regular exercise at least 30 min per day, 5 days per week.
I can refer you to dietician.

Reading materials and review after Ix.

Feedback (22/2/2017)
You are GP, 52 year old lady come for requesting HRT. Her LMP was 18 mths ago.
Tasks
Take relevant HO
The examiner will give u PE findings on card
Explain patient initial management and investigations that u want to do
2 mins thinking
This is typical recall of HRT
Have to ask symptoms of menopause, contraindications for HRT, 5P including mammogram,
general health in HO
Breast exam, BMI, BP in PE
Mx depends on case

History taking
When I stepped into the room, examiner checked my ID and introduced the name of role player.
Greeting - Good morning, I am Dr Khine. I am one of the Drs in this GP. Nice to meet u. How can I
help you today? (RP : Dr, I want to know if I can take HRT)
Ok, what is ur concern to take HRT? (My friends recommended it to me)
All right, let me ask u a couple of questions in order to know if u are suitable candidate for HRT.
(Sure Dr)

Menopausal symptoms
Physical : At the moment, do u have any night sweats, tiredness and hot flushes? (No)
Psycho : Do u think ur mood is irritable and easily get angry? (No)
Urinary : Any burning pain when u pass urine? Do u need to pass it more frequently? (No)
Genital : Any itchiness and dryness in ur down below? (Yes), I wanna ask u sensitive and
private Q, is that ok for u? (Yep). Are u sexually active (yep). Are u in stable relationship (Yep).
Do u have pain during sex intercourse (yep Dr. I need to use a lot of lubricants for sex)
Alright, I see. We ll fix that problem surely.
Musculoskeletal : Any aches and pain? (No)
CI for HRT
- Have u ever been Dx with CA breast? (No)
- I understand that ur LMP was 18 mths ago. Do u have any bleeding from down below
after that? (No). Did u notice any mass in ur tummy? (RP : what’s that Dr? seems like she doesn’t
know the term mass) Ok, any lumps or bumps in ur tummy? (No)
- Any liver problem before like yellow coloration of skin and urine? (No)
- Any previous heart ds or heart attack? (No)
- Any migraine HA? (No)
- Any blood clotting problems before? (No)

5P - Any previous preg and miscarriages? (Yes, Dr I have two children with normal delivery).
What about pap smear (it was done 4 years ago and normal) Didn’t u go for it 2 years back? (No)
Why? (Coz I am healthy and I didn’t see any Dr) Ok

What about ur mammogram? (I haven’t done it before) Any bulging from u down below? (No)

General health, SADMA - all clear

Thank you Samantha for ur information. I like to go to examiner for some findings and I ll get
back to u soon. Then, the role player flipped over the paper which is already on the table. It
includes all PE findings.

PE findings (a long list full of one page, I cant remember all)


GA - well. BMI - 30, Vital signs - All stable.
CVS, Resp, Abdomen, Breast - all normal
VE - Dry atrophic vagina, all clear on BME.

Management with pt

Well, Samantha, according to HO and PE findings, I didn’t find anything that is serious. Your BP is
OK, breast exam is fine. Heart and lungs normal. And I didn’t see any conditions we need to start
u on HRT. U dun have any menopausal symptoms like hot flushes, night sweats, irritable mood,
right? (RP nodded her head). So, at this moment, u dun need any HRT. But what I am a bit
concerned is ur body wt which is higher than it should be. So, I like u to stick to life style
measures which can be memorized by beautiful short form : SNAP (I wrote down SNAP on
paper)
S - Smoking avoided
N - Nutrition : take balance diet. I can get involve dietician who can give u proper diet plan to
help u lose wt.
A - Avoid alcohol if possible
P - Physical exercise : do regular exercise at least 30 min per day, 5 days per week.
So far, do u understand my explantion. (Yes, Dr)

Ok, another thing is lining of ur down below is dry and thin coz of menopause. In menopause,
female hormones are no longer produced from ovaries which are essential to maintain ur lining
of down below to be healthy. That’s why u have pain during sex and need to use lubricants. For
that, I ll give u oestrogen creams to apply ur down below.
Are u with me so far? (Yep) What else do u want to know? (What else should I know?) Ok. Let
me check ur profile again (I read the PE findings again and read the tasks. OMG!!!, I am about to
miss one task which is investigations)

At the same time the bell rang. I told quickly that I will check female hormone levels in ur blood
and imaging of ur tummy called USG (I couldn’t tell about pap smear, other blood tests,
mammogram)

Then examiner told me “Thank you, Dr. U can go out now”


AMC Feedback - Menstrual complaint : Pass (Global Score - 4)
Approach to patient - 4
History - 5
Counseling/Education - 4

Feedback (29/11/2017)
( Feedback – Menstrual complaint!!)
You are GP, 52 year old lady come for requesting HRT. Her LMP was 18 mths ago.
Tasks - Take relevant HO
- The examiner will give u PE findings on card
- Explain patient initial management and investigations that u want to do
History taking
Greeting
Dr, I want to know if I can take HRT
Ok, what is ur concern to take HRT? (My friends recommended it to me)
Alright, let me ask u a couple of questions in order to know if u are suitable candidate for HRT.
(Sure Dr)
Menopausal symptoms
Physical : At the moment, do u have any night sweats, tiredness and hot flushes? (No)
Psycho : Do u think ur mood is irritable and easily get angry? (No)
Urinary : Any burning pain when u pass urine? Do u need to pass it more frequently? (No)
Genital : Any itchiness and dryness in ur down below? (Yes), I wanna ask u sensitive and
private Q, is that ok for u? (Yep). Are u sexually active (yep). Are u in stable relationship (Yep).
Do u have pain during sex intercourse (yep Dr. I need to use a lot of lubricants for sex)
Alright, I see. We ll fix that problem surely.
Musculoskeletal : Any aches and pain? (No)
(4B- Bone, Breast, Bowel, Bladder, 2p- Prolapse, Post menopausal s/s)
CI for HRT
- Have u ever been Dx with CA, uterus breast? (No)
- I understand that ur LMP was 18 mths ago. Do u have any bleeding from down below after
that? (No). Did u notice any mass in ur tummy? (RP : what’s that Dr? seems like she doesn’t
know the term mass) Ok, any lumps or bumps in ur tummy? (No)
- Any liver problem before like yellow coloration of skin and urine? (No)
- Any previous heart ds or heart attack? (No)
- Any Stroke?
- Any HTN?
- Any SLE?
- Any migraine HA? (No)
- Any blood clotting problems before? (No)
- - Contraindications of HRT:
- o Estrogen-dependent tumor (endometrial,
- breast cancer)
- o Recurrent thromboembolism
- o Acute IHD (absolute)/history of CHD
- (relative)
- o Uncontrolled hypertension
- o Active liver disease
- o Pregnancy
- o Undiagnosed vaginal bleeding
- o Otosclerosis? Intermittent porphyria
5P - Any previous preg and miscarriages? (Yes, Dr I have two children with normal delivery).
What about pap smear (uptodate)
What about ur mammogram? (I haven’t done it before) Any bulging from u down below? (No)
General health, SADMA - all clear
PE findings (card)
GA - well. BMI - 32, Vital signs - All stable.
CVS, Resp, Abdomen, Breast - all normal
VE - Dry atrophic vagina, all clear on BME.
Management with pt
Well, Samantha, according to HO and PE findings, I didn’t find anything that is serious. Your BP is
OK, breast exam is fine. Heart and lungs normal. And I didn’t see any conditions we need to start
u on HRT. U dun have any menopausal symptoms like hot flushes, night sweats, irritable mood,
right? (RP nodded her head). So, at this moment, u dun need any HRT. HRT are hormones, so
they have their own side effects.
Investigations
(only mentioned a few hormone and blood tests.)
but what I am a bit concerned is ur body wt which is higher than it should be. So, I like u to stick
to life style measures.SNAP
S - Smoking avoided
N - Nutrition : take balance diet. I can get involve dietician who can give u proper diet plan to
help u lose wt.
A - Avoid alcohol if possible
P - Physical exercise : do regular exercise at least 30 min per day, 5 days per week.
Refer to dietacian.
Ok, another thing is lining of ur down below is dry and thin coz of menopause. In menopause,
female hormones are no longer produced from ovaries which are essential to maintain ur lining
of down below to be healthy. That’s why u have pain during sex and need to use lubricants.
For that, I ll give u oestrogen creams to apply ur down below.
Review- after seen by dietacian
Redflag – pv bleeding, lump in breast, something coming down PV
Case 3
Another case: patient has clear indications for HRT, but patient had breast cancer 10 years back,
fully treated

HRT should not be started here

Advise the patient lifestyle modifications:


-healthy balanced diet
-decrease spicy foods
-limit coffee should to 2 cups/day
-alcohol safe limit, no smoking
-Advice to wear light cotton clothing.
- Exercise like swimming (because the water can keep the body cool).
-Ask her to sleep in cool rooms.

Put her on medications (alternatives to HRT):


-First choice: Pregabalin (Lyrica)- derivative of Gabapentin, can bring down all the vasomotor
and psychological symptoms
or
Gabapentin itself
or
SNRIs/SSRIs - venlafaxine/paroxetine


Case 4
47 year old Jane is your next patient at your GP. She has come to you to discuss the options of
starting HRT. She is on combined oral contraceptives for the past 5 years.
TASKS
-Further relevant history
-Counsel accordingly

History
1-approach and why (menopausal symptoms)
2-HRT indications questions (+ve)
3-5Ps questions
-LMP and regularity
-Stable relationship/ STI
-Pregnancy
-type of pill, since how long, any SE
-HPV and mammogram
4-HRT contraindications questions
5- PMH and PSH and lifestyle

Note/ As long as the patient is on combined OCPs, usually they do not get menopausal
symptoms, and if they do, it is during the dummy pill period. But if the patient on combined
OCPs get menopausal symptoms, it tells you that the estrogen content in the pill is not sufficient
to replace the loss of estrogen that she is having.
Note/ Normally, the option here is to increase to high dose COC. BUT never increase the dose if
the patient is perimenopausal. If she is and she presents with perimenopausal symptoms on
combined pills, stop her on combined pills, estimate her hormones (FSH, LH, estrogen --usually
FSH is the main predictor of menopause; FSH and LH high, estrogen is low), then start her on
HRT and advice on alternative method of contraception like condoms.

Counselling
-describe what HRT is
-what OCP is
-tell her that the estrogen of the OCP is not enough to cover the loss that is why you have these
symptoms.
-we need to stop OCP and start you on HRT
-explain risks of HRT
-need to do complete physical examination, basic Ix and hormonal essay especially FSH
-prescribe cyclical or sequential HRT and tell all side effects
-IMPORTANT: tell that HRT is not a contraception so you need to use alternative methods of
contraception.
-lifestyle modifications
-reading materials
-HRT is hormone replacement therapy where we replace the hormones that become deficient as
you go towards menopause, and also after menopause. The symptoms that you are having is due
to a decrease in the secretion of estrogen from the ovaries, as the ovaries start to become less
functional as you go towards menopause, and this estrogen is replaced by HRT.
-Even though the combined pills contain estrogen, the estrogen in the pill does not replace the
estrogen that is deficient from your body.
-At this age, we do not recommend you to be shifted to a high dose combined OCPs. What we
can do however is to stop the combined OCPs that you are currently taking, and start you on HRT
instead as you have clear indications, and no contraindications.
-But I must also advise you that HRT carries multiple risks. It can lead to thromboembolic disease
or increased clotting in your veins, stroke, womb cancer or endometrial cancer, ovarian cancer,
breast cancer, and also gallbladder disease.
-The benefits of HRT are: you can get rid of perimenopausal symptoms, decreases the incidence
of bowel cancer, and there is a possible decrease in the incidence of Alzheimer's disease and
other forms of dementia.
- Blood tests including hormones need to be done like FBE, UEC, LFT, BSL, lipid profile, FSH, LH,
and estrogen levels.
-After a complete physical examination and a pelvic examination, I will start you on HRT on a
cyclical or sequential manner. Taking the menstrual cycle for 28 days, we will put you on
continuous estrogen, and progestogens during the last 14 days, and after finishing the
progestogens, you will expect to have withdrawal bleeding.
-You may experience a bit of nausea, abdominal bloating, headaches, and breast tenderness, and
sometimes a breakthrough bleed.
-We may stop you on HRT once you become cleared of the symptoms, and it is usually advisable
to take HRT for only 2 years, and by any chance not more than 5 years.
-HRT cannot act as a method of contraception, so you need to use alternate methods of
contraception like condoms.
-I will give you reading materials regarding HRT for further insight and I will arrange a review
with you regularly.
Case 5
Another case: She is on combined pill, no symptoms of menopause, she is coming to you for HRT
because her friends tell her that you feel more feminine if you start on HRT.
Counselling
-describe what HRT is, indications and risks
-at present you have no indications so we cannot start you on HRT.
-Ask her to go on with the COC and stop the combined COC at 50 years old. If she has no periods
after that, she is menopausal. If she is getting irregular periods, estimate her hormones (FSH, LH,
estrogen) and then at that time, if she has clear indications with no contraindications, you start
her on HRT.
-If she is menopausal after 50 years, always ask her to use alternative methods of contraception
like condoms for 1 year after menopause.
-If she attains menopause before 50 years, she needs to use condoms for 2 years.

-As from the information that you have given me, there are no indications in you to be started on
HRT. The indications for HRT are when you experience perimenopausal symptoms like mood
changes, irritability, sleep disturbances, muscle aches and pains, history of bone fractures,
vaginal dryness, burning or itching, and problems with your water works.
-HRT should always be started with caution, as there are lots of risks associated with HRT. The
risks are…
-Because of these risks, HRT is never started if there are no clear indications for it. HRT will never
act as a contraceptive.
-What is advisable this time is that you continue on your combined pills if you are comfortable
with that, until 50 years of age. After then, you can stop your pills, because as long as you're on
the pills, you will go on getting your periods, and we will not know whether you've hit
menopause or not. Once you stop the pills by 50 years old and if you do not get your periods
anymore, that could mean that you are already menopausal. However, if you get irregular bleeds
after stopping the pill, and there are indications in you for HRT, we will estimate your hormones
like FSH, LH, and estrogen. If FSH, LH are high and estrogen is low, that means that you are
menopausal, and at that time if you have no contraindications for HRT like…, we can consider
starting you on HRT with your informed consent. We will do a full physical examination and
pelvic examination on you before we start you on HRT. Make sure that your pap smear and
mammogram is up to date as well by doing it every 2 years. It is advisable to use alternate
methods of contraception like condoms for 1 year after menopause, if the menopause happens
after 50 years of age.
-I will give you reading materials regarding HRT for further insight and I will arrange a review
with you regularly.
Feedback 22-6-2018
10. 52 yo patient with no periods for the last 18months coming in to consult for HRT.

Task:
hx,
PEFE ( PE card given),
Explain to pt what you found,
ask for any necessary investigations and mx.

Approach
Greeted patient
Open ended question
She said her friends are tsking HRT so she came in for advice
I told her that It is therapy that is not required for every post menopausal women as there are
certain side effects of these medications.
I will asses her situation first and then will advice on that

Confidentiality
Asked about Hot flush, night sweats, sleeping problem, mood changes, dryness or discharge
from down below ( all neg) asked about sexually active or not, post coital bleed, dyspareunia
Past hx or family hx of breast, womb or ovary cancer
Past or family hx of fractures
Any co-morbidities
Any lumps, bumps or unintentional weightloss.
Asked about screening tests like Pap’s done or not ( 5 years ago) so mentioned will arrange a
HPV screen for her
Asked about mammogram (not done) said will arrange
Asked SADMA
Past hx of any known medical illness
PEFE—showed card
Very detailed
Positive was for atrophic vaginitis and cystocele

So I mentioned I will arrange for all the screening tests to be done. (Mentioned HPV screen,
Mammogram, Asked about any heaviness or bulging down below or any involuntary dribbling of
urine she said no

Then explained as she has no indications she is not a candidate for HRT. Explained atrophic
vaginitis and said will give local estrogen cream and jelly to use during intercourse.

Mentioned will refer to gynaecologist for cystocele.


65-Breath holding attack
you are a GP when a 2-years old boy has lost his consciousness an hour ago. An hour ago the
child had a finger jammed by the door then he stopped breathing and started twitching. He is Ok
now except for a bruise and swelling around his fingers. Father is concerned about epilepsy.
Tasks
-History
-provisional diagnosis and differentials
-explain diagnosis to mother
-management and further advise

Differential diagnosis
1-breath holding attack
2-trauma
3-infection
4-febrile convulsion
5-seizure/ epilepsy

History
1-ask if any pain in his fingers and offer painkillers

2-Attack
Before the attack
-did he behave differently? Looking past you or as he is flying?
-has he had any trauma?
-any fever?
-any recent infections?
-During
-ask about if anyone witness the attack or if he has lost his consciousness (if they are not
mentioned in the stem but here the stem mention the father and LOC an hour ago)
-as you witnessed the attack can you tell me for how long he has lost his consciousness? (less
than 1 minute)
-ask if he noticed any jerky movements? (if not mentioned in the stem here there is twitching)
-So you noticed that he started twitching so was it localized or generalised? For how did it last?
-did he turn pale or blue?
-has he bitten his tongue?
-did he wet or soiled himself?
-have you noticed any up rolling of the eyes?
-any trauma?
-After
-how long did he regain his consciousness?
-was he drowsy?

3-past and family


-has this happened before? Any past episodes?
-any family history of epilepsy? (yes)
4-BINDS
Birth
-any problem during birth or after delivery?
-did he need resuscitation?
Immunisation
nutrition
-how is his feeding

Development
-is he thriving and growing normally?
-Has he started talking?
Social
-are you happy family? Any kind of stresses in the family?
-how is his relationship with other siblings?

5-Behaviour
autism
-doe he emotionally interact to you?
-does he like playing with other children?
Temper tantrum
-have you noticed that he cries excessively when he need something?
-does he accept no easily
-any history of head banging?
-how does he behave in the supermarket? When you say no what does he do?

Explanation
Condition
-from history and examination your child most likely had a condition called breath holding attack
have you heard about it?
-it is a common condition in this age group. It is a type of behaviour disorder. Let me assure you
that this is not epilepsy.

Cause
-it can be precipitated by pain, emotion or frustration.

Complication
-it is completely harmless and does not lead to epilepsy or brain damage.
-usually resolved by 3 years of age

Other possibilities are:


-trauma to head is unlikely as no history of that.
-infection or febrile fits but unlikely as no fever or recent infections.
-fits is also unlikely as no history of tongue bite, wetting or soiling himself….
*always say unlikely, as the most likely one is Breath holding attack
Management and further advice
-as this condition can happen again, let me give you some advice of how to avoid and manage
similar situation:
1-try to avoid situations which can precipitate the attack.

2-when your child starts crying ignore him, but once he has breath holding spell or fit you need
to follow these steps:
-lie him on his side and watch him
-keep their arms, legs, and head from hitting any sharp objects.
-do not shake the child and do not put anything into his mouth
-do not splash your child with water
-no need to help you child with his breathing as your child will start breathing again on his own
after the spell subside

3-After the spell:


-make sure your child gets plenty of rest and feels secure.
-treat the child normally and try to act as if nothing has happened.
-try not to punish or reward him.

4-no medical treatment is required if your child has frequent spells, we may test for low iron
levels in the blood as this can increase the frequency of spells.

5-reading materials, red flags (attacks last more than 1 minute, Loss of consciousness, bite his
tongue or wet himself), Review.

Note/ always reassure the parent and check his/ her understanding.

Case (7/3/2017)
2 years old child lost his consciousness an ago he is ok now except for a bruise and swelling
around the fingers
history diagnosis further management ..
I asked about the history of the attack then she said that the car door closed on his fingers then
he had loss of consciousness lasted for less than 1 min.
he was stiff at that time but no wetting or passing stool or tongue bite or jerky movement.
she gave me family history of epilepsy
I told her is he in pain but towards the end of station so offered him pain killer and refer to
hospital to do x ray.
I told her it’s most probably breath holding after pain so that why he lost his consciousness and I
do not think it is epilepsy (I was not confident enough so I do not know if I will pass this station )
Case (4/10/2017)
Loss of consciousness – Pass
2 year old. Loss of consciousness 30sec. Mother comes to see you. Child now well.
Tasks:
•History
•Relevant PEFE
•Explain diagnosis to mother
•Explain management
It was a typical history of breath holding attack- jammed finger on car door/ cried / turned
blue / LOC / no convulsions / better soon after / now quite well. First episode. No family or
personnel history of epilepsy. No behavioural/ social issues. No fever or significant illness
recently. BINDS normal
Did explanation and management tasks as in
http://www.health.vic.gov.au/edfactsheets/downloads/breath-holding-in-children.pdf
please refer to it. Got 6s and 7s for all domain scores here, as this one was very straightforward.
PEFE: Examiner said all normal and child is happily playing now.
Mother had concern about epilepsy. Need to address that and reassure.

Feedback march-2018
3 yr old boy brought by concerned mother because of sudden loss of consciousness and
trembling of hands 1 hour ago. On examination he's well and active and you noticed bruises on
two fingers of the hand.
Task :
History
Most likely diagnosis
Immediate management to the mother
No generalized jerking of limbs, no rolling of eye, no incontinence, cried after the
episode,healthy child, cousin has epilepsy
Blue fingers because his fingers got caught in the car door and he started crying loudly and he
went blue and started shaking. This was what the mother told me. So it's an anoxic seizure
secondary to breath holding spell.

Feedback march, 2018


Breath holding spell child. Child turned blue when finger was accidentally struck by the car door.
Tasks: 1. History 2. Diagnosis to father 3. Acute management 4. Long term management
No need to do PE.
Ruled organic disorders like seizures, cardiac problems and acute brain injury.
Also ruled out abuse and explored on home situation. Apparently normal child.
Advised ice and panadol for pain. Then gave repeated reassurance that a serious condition is
unlikely. I placed the red flags at the end with emphasis on seizure episodes.
Key steps: YES to all, Approach:6 History:6 Diagnosis: 6 Management:4, Patient
counselling/education: 5
Global score: 5 (Pass)
Feedback Loss of Consciousness: Fail
2 mins outside: Typical history of Breath holding attack (BHA). Child of 4-5 yrs jammed his 3 (I
think) fingers in door. Became unconscious. Now In ED. Pain killer given. X ray done, no fracture.
His father wants to know what happened.
Tasks: History, Dx, management and counselling.
Thinking outside: Straight forward case of BHA. Rule out causes of unconsciousness such as
convulsions, head injury, meningitis, hypoglycaemia etc.
Inside: Pt’s father was young male, having hoodie, long hair and beard. Not very well groomed.
Me: I am so sorry to hear about your son. How is your son at the moment?
Pt: he is ok as he got painkiller. But what happened to him?
Me: We will find out soon. Can you pl tell me what happened exactly?
Pt: told me the whole story how he jammed his finger and got unconscious.
Me: Then what did u? Pt: I took him here. Me: Ok.
Me: Is it the first time? Pt: Yes.
Me: How long did he loss conscious for? Pt: for few seconds, half a minute.
Me: Any event apart from finger jamming before LOC? Such as headache, skipping of meal? Pt:
No
Me: During attack, did he move his limb or limbs? Pt: no.
Me: Rolling of eyes? Saliva coming out of mouth? Pt: No
Me: after attack, was you son confuse? Pt: No.( all to rule out Grand Mal epilepsy).
Me: does he have any fever, SOB, any heart condition? Any early morning headache, visual
blurring? Pt: no.
Asked about system questions, such as CNS, known thyroid, diabetes, CVS, respiratory,
abdominal, pee and poo. All negative.
Me: asked about behaviour prolems. Does he take ‘No’ easily for his demands Pt: yes (I was
expecting no for answer).
Me: does he demand things very often? Pt: No (I was surprised).
Me: asked about past history, F/H of convulsions, any known diseases? Birth, vaccination, growth
history. All negative.
Me: Who looks after child at home? Pt: M and my wife.
Me: I got suspicious because of his look (big mistake), asked about any chance of child abuse? ( I
think I asked directly) Pt gave me surprised look. I think he felt offended. I did not stop here and
asked about repeated fractures, he said no.
DD: I gave breath holding attack, explained him about white and blue attack. This was white one
due to pain. It’s a part of tantrum and behaviour problem (according to JM).
Management: we need to address two types of management. 1. Immediate management: we
will look at the fingers again and I would like to do full body examination of your child if its ok for
you to rule out non accidental injuries ( Pt felt offended again, another big mistake again). So we
may need to do more x rays. ( if in stem x ray of finger not done, then I think we need to do
finger x ray). If everything ok and my seniors happy, he can go home. Further management: I
explained everything according to JM and karens.
Global score: 3 Key steps: 1 yes. 2 & 3 No Approach to pt: 3 History: 4
Dx/ DD: 4 (I don’t remember whether DD were there or not, IF yes then I may have explained
dds of unconsciousness) Management and Pt counselling/ education: Both 3
66-Lichen sclerosis
Michelle, a 54 year old lady is you next patient at your GP. She complaint of severe itching in her
vulva for the past 2 months.
TASKS
Further relevant history
Examination findings from examiner
Investigations
Management

Differential Diagnoses
1-Atrophic vaginitis / menopause
2-Infections/ candidiasis/ UTI
3-Lichen sclerosis
4-Skin conditions (Dermatitis, Eczema, Psoriasis) / Skin allergies due to cosmetics or
undergarments
5-Diabetes/ Steroids

History
1-Itchiness questions
open ended question
-I can see that you are complaining of itching in your vulva can you tell me more about it?
Onset & duration
-for how long have you had itching in your vulva? (past 2 months)
-has it started suddenly? Is it constant or does it come and go? Is it getting worse? (continuous,
worsening)
-has this happened before?
Site
-is there itching anywhere else?
Severity
- do you think it is interfering with your daily activities?
- does the itching affect your sleep?
Aggravating and relieving.
-Does anything make it better or worse?

2-Differential diagnosis
- Do you have any bleeding or discharge from you vagina? (Atrophic vaginitis/ candidiasis)
- Any colour changes over your private area? (lichen sclerosis)
- Any fever? (Infection)
- How’s your waterworks? Any burning or stinging on passing urine? (UTI/ Lichen sclerosis/
menopause)
- Any rash, ulcers, or vesicles around the area? do you have any skin condition like eczema,
psoriasis? (Skin conditions)
-Have you changed your cosmetics like shower gel or soups? do you use any vaginal douches?
Any allergies? (Skin allergies)
-any medical illnesses like diabetes? (Diabetes)
-do you use any medications? (Steroids)

3-5 P’s Questions


Menopause (= period in young age)
-When was your menopause age or last period?
- do you feel your vagina is dry or thin?
-any symptoms like Hot flashes, excessive sweating or mood changes?(menopause symptoms)

Partner (sexual)
-Are you sexually active?
-any pain during intercourse? Any bleeding post coital?
-any history of STI

Pregnancy (not relevant here)

HRT (= pill in young age)


-do you use any HRT?

Pap and Mammogram


is your pap or hpv and mammogram up to date?

4-General (if enough time)


-diet rich in calcium
-SAD
-exercise
-good support

Physical exam from examiner

1- General appearance
- BMI
- pallor, lymph node enlargements, generalized rash, eczematic or psoriatic patch?

2- Vital signs:
Especially temperature (infection)
BP HR with rhythm, RR with saturation

3-Quick CVS, R/S

4- Abdomen
-Visible distention, any mass?
-Palpate for any mass, tenderness (soft and non-tender)

5- Pelvic exam (with the consent of the patient and presence of a chaperone)
Inspection of the vulva and vagina:
-visible discharge, any bleed?
-Any other skin lesions? (White, shiny, wrinkled plaques in the vulva and perivaginal areas in a
laced-like pattern) ask specifically if not given by examiner
Inspection of the anal area:
is there any lesions, excoriation marks, plaques?

Speculum exam:
- is the cervix healthy or not? Any discharge or bleed?
-Does the vagina appear thin or atrophic?

Per vaginal exam:


-uterus size and tenderness (normal, no tenderness),
-adnexal mass and tenderness

6-Office tests:
UDT, BSL (rule out diabetes)

Investigations
- FBE, UEC
- TFT (autoimmune thyroiditis)
- refer to specialist for multiple punch biopsy (critical error)

Explanation (4C)
Condition
From the history and examination, most likely you have a condition called lichen sclerosis. It is a
chronic inflammatory skin condition. It is not infection or contagious.
Clinical feature
this usually presents with severe itching and causes white, wrinkled plaques in your genital area.
Cause
Exact cause is unknown but thought to be an autoimmune disease. The immune system of your
body usually protects the body against infections, but in autoimmune conditions, the system can
get confused, and it starts attacking your own body cells rather than protecting it.
Complication
It can result to scar formation and it can join up with the surrounding genital skin leading to
adhesions. Occasionally or 4% of the lichen sclerosis can turn nasty or be pre-cancerous. (I prefer
not to give percentages)

Outcome
because this is an autoimmune condition, there is no permanent cure for lichen sclerosis. But we
can keep the condition under control.
Management
-You needs to be seen by the specialist because the multiple punch biopsy needs to be done.

-the treatment is with local steroid creams which you need to apply twice daily for the first one
month then once daily for the second month and then depending upon your response the
strength and number of applications can be reduced.
You need to be put on a maintenance therapy of a lifelong 1-2 applications per week.
-If not responding to steroids, we can use retinoids or ultraviolet therapy.

- When do you go for surgery?


If there is scar formation or adhesions, and also if there is any malignant change or cancer.

-maintain a good genital hygiene


-keep your HPV and mammogram up to date

-you needs to be on a lifelong surveillance because of the malignant change to begin with 6
monthly intervals, and then annually.

- Red flags: in case you experience any bleeding, abnormal discharge, or if the itching is
becoming worse, please report back.

- I will arrange a review with you in around 1 monthtime and see how you are doing.
- I will give you reading materials for lichen sclerosis.

Is it due to menopause?
Lichen sclerosis has nothing to do with menopause. It is an autoimmune condition, not
associated with any hormones. It can happen at any age, not only in menopause.

….For the exam, the roleplayer may tell you that she's been having this for one year. She has
seen some doctors who have prescribed her with vaginal estrogen creams which is not helping.
(estrogen cream is for atrophic vaginitis)
Feedback 5-7-2018
77 years old lady present with itching.
Task: 1- hx

2- PE from the examiner

3- management plan

Outside the station I thought it’s the recall of lichen sclerosis, however I thought about ddx
(allergy, DM, STD, atrophic vaginitis and vulvar CA). when I entered a nice old lady is sitting on
the chair.
I introduced myself and start taking hx: for how long? Continuous or intermittent (was
continuous)? Anything makes it worse or better (nothing)? Any discharge (none)?
Any problem with water work (none) Any Hx of recurrent UTI or vaginal infection (none)?
Any Hx of DM (no)? any discharge(no)? Any lump from below?
Any change in wt or appetite(no)?
Last LMP (I think she says 20 years ago and no bleeding since that time)?
Any HRT ( I think she said yes for 2 years I’m not sure)? Not sexually active?
I asked for 4 B (bone, breast, bladder, bowel) and 2 p (post-menopausal symptoms and
prolapse)? Pap smear (not done)?
Any Hx of STD? SADMA

The examiner gave me a picture of lichen sclerosis and asked me to describe, I said the pic
showed the valvular, perineal and anal area of a lady, there is redness and scratch marks on
both sides of labia with some white patches.

I turned to my pt, told her there is a lot of possibilities, the most likely one is lichen sclerosis, I
drew a diagram and tell her its autoimmune disease where the immune system in our body
which originally fight infection, start to attack certain parts of the body in ur condition, the skin
of vulva. Reassure the pt its not infection and not a cancer.
Start her on steroid cream with follow up if unresponsive refer to gynecologist.
Need to be treated sometimes nasty growth (cancer) develop in scaring tissue. Other less likely
causes infection, allergy and atrophic vaginitis (she asked about that and I explain)

Passes: global score 5


Feedback 5-7-2018
a 77 year old female c/o itchy vulva for long duration
Task
Relevant historyf
Examination finding from the examiner
Explain condition to the patient
The rash has been there for many years, when asked she said she tried
estrogen cream and fungal cream but did not help her. No other positive
finding I could elicit.
Asked for vaginal discharge – no
Urine problems –no
LOW, LOA, lumps or bumps? – NO
Itching anywhere else? - no
Any soaps or anything you put down below? – she said she washes with
non-soaps.
5 P’s > menapaused, no partner, no HRT or pill, no children.
SADMA > no medication, occasional alcohol. .. no medical or surgical
condition done.

On examination , the examiner gave a pic of the vulva and asked me to


describe it. I said so this is urethra, this is vagina, labia majora and
minora, this is anus .. I can see scratching mark, redness, some white and
purple discoloration along with purple papules… the skin looks thin. This
is lichen planus or sclerosus ( I don’t know if there is difference!!)
(explanation: define / cause / risk / complication / prognosis)
I explained to the patient that this is lichen sclerosis. Its likely an
autoimmune condition, common in older people. Loss of hormones could
be a cause.
Risks of becoming a nasty growth (she asked what is nasty growth?) – I
told her: cancer. She said (OH). I told yes that could happen that’s why I
want to make sure that you’re ok so I’ll refer you to a gynaecologist to take
a biopsy and check if everything is okay. How about that? – tes doctor.
I can give you some steroids and calamine lotion for the time being to
lessen the scratching and try not to scratch because that irritates the skin
further.
Wear loose cottor undies and make sure the area is clean.
I will give you some reading material and hope to see you soon in 2 weeks
to see if everything is ok.
Feedback 5-12-2018
77 years old lady with itchy vulva,
Task
1 History taking
2 examiner will give picture
3 Describe the photo to examiner and give him ur provisional dx on the picture
4 Tell Dx to patient and manage her accordingly.
D/D atrophic vaginitis, Lichen sclerosis, Infections (candidiasis, Bact vaginosis, Eczema Psoriasis, Allergies
Approach: greetings, How are you feeling right now? Would be asking some personal q Is that Ok?
Positives in HX: Pt is having itching for 2 yrs. previously managed my local estrogen, lubricants. She does
not know any specific dx for that.(Can’t remember now rx worked for he or not).She mentioned of some
bleeding probably from scratching). No constitutional s/s of Cancer. No discharge down below.
Mammogram was normal PAP was normal. No sex activity. Examiner gave a big photo of vulva and asked
to describe that.
HX :
HOPI Q: Pt is having itching for 2 yrs. previously managed my local estrogen, lubricants. She does not know
any specific dx for that.
Associated symp Q: No pain, no fever, no ulcer or discharge, No itchiness in anywhere else, No rash she
thinks (she mentioned she can’t see there properly). She mentioned of some bleeding probably from
scratching)
DD Q: No allergy,. Have not applied any creams or perfume No dryness down below, no incontinence, Any
other skin cond. in her or family like eczema or psoriasis, No lumps or bumps or wt. loss.
5P and menopausal Q: Menopause 20 yrs ago. Have not been sexually active for long. No menopausal
syndrome or use of HRT, Up-to-date with mammogram and PAP.
Went to second task: Told examiner want to do focused pelvic examination. Examiner gave me photograph
and asked to describe what I can see._ there was shiny wrinkled skin and some whitish plaque in vulvar
area with no ulcers or discharge, most probable dx is lichen sclerosis of vulva. Asked for speculum exam.
Examiner said rest are normal or said unavailable – can’t remember. Then asked me to talk to Pt.
3rd task: Described Lichen sclerosis, in 5C manner (condition, clinical features, Commonality, cause,
coarse, complication)
Will refer to gynecologist for confirmation with punch biopsy. Reassured her its quite manageable, also
told unfortunately it’s a chronic condition need lifelong surveillance.
Rx: initially gym will start steroid 2 times daily for 1 month and then 1nce daily for 2nd month. Then
depending on response dose will be reduced. Even after control she need to continue application for 1 or
twice a week for life long. We call it maintenance therapy. (Did not mentioned about SX, or UV therapy or
retinoid)
Advised about good genital hygiene, regular HPV and mammogram. And lifelong surveillance 6 monthly
for 1 year and then yearly.
Key steps 1234: No yes, yes, yes
Approach to pt : 5
HX: 4
Accuracy of the examination: 5
Dx/ DD:5
Management:5

67-Transient synovitis
6-year-old child with knee right knee pain refuse to walk.
Tasks
1-take history from mother
2-physical examination findings from examiner
3-dx and ddx to patient
4-Investigation
5-treatment.

History
1-Pain questions
-I can see from the notes that your child is complaining of right knee pain. May I know his name?
Is he with you (because I could not see any child inside just the mom)? (Yes)
-how severe is his pain from 1 to 10 1 is the least 10 is the most? Offer painkiller after allergy
question (does not need painkiller)
-when did it start? Sudden or gradual? Constant or come and go? Is it getting worse? (Since
yesterday, sudden, constant)
-where exactly the pain is? Any pain in the right knee or anywhere else? Does it go anywhere
else? (Right knee, not radiated, no pain on left knee or anywhere else)
-how does he describe the pain?
-anything make it better or worse? Have you tried anything to relive it? (Relieved bit with
Panadol)
-is he able to walk or bear weight? (No, refuse to walk)
-has this happened before? (No, this is the first time)

2-Differential diagnosis questions (HOST M)


-any tummy pain or rash? (HSP) (No)
-any fever? (if yes ask fever questions and dehydration but here will be no) (OM) (No)
-any joint pain or swelling? (SA) (No)
-any recent flu or viral infections? (TS) (Yes, 2 weeks ago but now recovered)
-has he had any trauma? (T) (Very active boy at school so not sure if he fell down or had any
trauma)
-any loss of weight, appetite or lumps and bumps? (Malignancy) (No)

3-past history
-any history of thyroid disease, asthma or any illnesse? (No)
-is he taking any medications (steroid) or chemoradiotherapy? (No)
-any family history of joint or bone problems? (did not ask in the exam but other feedback said
grandma has arthritis or mother has joint problem when young)

5-BINDS (All ok)


especially growth and development

Physical examination from the examiner


1-General appearance (DR PJL)
dehydration, rash, pallor, jaundice , LAP (Normal)

2-vital signs especially fever (all normal)

3-Growth chart (can ask even if 12 y.o) (normal for age)

4-Quick chest, heart and abdomen examination (Normal)

5-lowe limb focus exam


-with adequate exposure and in the prescence of mom I would like to examine both legs
including hip, knee and ankle joints
-Gait (not assessed)
-look (SSSDW) scar, skin color change + bruises, swelling, deformity, wasting (normal)
-feel: tenderness, temperature, joint effusion (normal).
-move: any restriction in rang of movement (the examiner said normal ROM both knees the
asked what do you want in hip joints? I told the same look and feel with range of movement
abduction and etc. so give +ve painful abduction and internal rotation).
-measure: limb length (examiner tell how to measure limb length so told how to measure the
true and apparent limb length )
-roll test (positive)

Note/ in the exam even if you asked would like to examine all hip and etc.. the examiner will give
you only knee findings then he will ask you what do you want from the hip so you need to tell
again look …, feel…, move; first ask any restriction in ROM if examiner ask you which movements
then you need to tell for hip abduction, adduction, extension, flexion, internal and external
rotation. The examiner will help you through

6-office tests (negative or not available)


-UDT
-BSL
Explain
-from history and examination there are several possibilities why your child has right knee pain.
From the examination, it appears that the problem actually is in the right hip joint and goes
down the knee causing knee pain.
-most likely, he has a condition we call transient synovitis or inflammation of the lining of the hip
joint Causing pain, limping, and restriction of movements.
-It usually caused by viral infection he had 2 weeks ago causing buildup of fluid in the joint space.

-It is a common cause of limping, it is not serious and it is a self-limiting condition within a week
or two.

- Other possibility could be Perthes disease (no enough blood supply to the head of the
thighbone, which is important to rule out. Could also be due to trauma.
Others could be osteomyelitis (infection of the bone) or septic arthritis (infection of the joint but
unlikely because he has no fever, no swelling, his general health is good and examination does
not suggest them.

Treatment
-to confirm diagnosis and rule out Perthes and others I need to arrange blood tests like FBC and
inflammatory markers and refer for X-ray and Ultrasound.

-I will give painkiller.

-important to rest the limb and not allow him bearing weight use crutches for that till resolution;
Normally we expect to see the resolution of your symptoms in 7 days.

-we will repeat X-ray in 6 months to rule out Perthes

-Red flags (rest pain, persistent or increase pain, fever, joint swelling)

-follow up within 3 days


Case (13/10/2017)
A 6 Y/O Peter was brought by Ben( Indian roleplayer) with severe knee pain from 2 days before ,
due to which the pt could not bear weight and walk.
Task :
Hx, PEFE, Investigation to patient, Most likely diagnosis and ddx with reasons to pt.
-There was a history of viral infection few days back and trauma during a fall in a basketball
field.
-No fever
- no Abdominal pain no joint pain or bone pain on other parts of the body.
- no rash
- BINDS NL
- no specific PMH his grandma had RA.
Px:
-GA: Pallor, Jaundice, dehydration(-), no LAP
-V.S no fever,
-Growth chart NL
-Systemical exam focusing on the limbs :
inspection- comparing both but on right leg is there any swelling or erythema or bruises or
bowing of legs?(no)
On palpation of knee tender to touch?Patellar tap? Buldge? no
In Movement? knee ROM restricted?(no)
then I remembered the hip…I said in hip again in inspection the same questions. ROM in
internal rotation and abduction was restricted. I mentioned about Roll's test and it was positive.
-The Urine dipstick and BSL was not available.

I said look from the Hx and Px the most likely condition that you have is called tenosynovitis
have you heard of it before? No
actually it is inflammation of the outer layer of bones of hip joint. That is why I want to
investigate it with U/S and X-ray and FBE to rule out infection with inflammatory markers.
Other possibilities are Perthes disease (which is important to be ruled out) and SCFE and
Trauma and OSD which are less likely due to the age and the hx and px of your child still u/s can
help us to find the cause.
For now I kindly ask you not to move and bear weight with the affected leg and use the
crutches for it and I prescribe pain killer as well as cold compress.
Normally we expect to see the resolution of your symptoms in 7 days.
Bell rang I said I want to see you in 6 month as well to rule out Perthes!
(Still I am not 2 sure about Perthes and T.S in this case so I look forward to my result and
feedback)
Feedback: Joint Pain, PASS( G.S:7)
Key steps:1,2 and 3: Yes
History:6
Choice and technique of examination and organization and sequence:7
Choice of investigation:7 Dx/DDx:7 Mx:6

Recall of 23-2-2018
Child with acute knee pain. Limping. Had a history of a viral URTI. Unable to int and ext rotate
with pain n restricted abduction.

Recall of 21-6-2018
your next patient (I guess it was GP) is a 6-year-old boy with intermittent knee pain for the last 2-
3 days.
Take focused history.
Ask the examination findings from the examiner.
Explain the provisional diagnosis and your management for the mother.
In examination it turned out to be some issues with the hip joint. Probable Dx and then Mx Joint
pain: Pass/ Global score: 4

Entered the room, after introduction, asked the mother how I could help her. She explained that
her son mentioned knee pain and did not come down from the couch (refused to walk). I asked
about duration of the pain. Any history of trauma, any signs of trauma in him, any obvious
swelling in his knee. And joint swelling in the knee or elsewhere Also about previous illness, and
his general health condition since birth, also asked if the son is very fat or very thin (he was
average and healthy, but mum clearly mentioned twice that he had had a viral infection like a
cold two weeks before. Also checked his allergy, home situation, vaccination whether is up-to-
date or not. Then proceeded to physical examination. In general appearance he refused to walk
so assessment of his gait was not possible, his vital signs, especially temperature was normal.
Heart and lung were Normal. Knee examination: inspection: obvious swelling, Negative, local
temperature, signs of arthritis, signs of trauma, negative, tap sign: negative, swipe test, negative,
then the examiner said that knee looks normal. So I proceeded to Hip examination: obvious signs
of trauma or inflammation in hip from inspection: negative, asked about the roll test, the
examiner’s non-verbal behaviour showed that he is not sure what to answer, or maybe he asked
what I was looking for, so I mentioned that I am looking for signs or effusion or inflammation or
any Articular pathology in the hip. So he said that there is reduction in internal rotation and
probably adduction f the right hip joint. Checked the vasculature of the leg, also checked the
genitalia to be on the safe side (mentioned to the examiner that I do not expect to find any
abnormality with the genitalia, only want to make sure it is normal), I am not sure whether I
asked for office test, but It did not add to my information. (maybe the results were not available)
so I explained to the mum that according to my examination knee is fine but the problem is with
the hip. I explained that I was thinking of infectious reasons such as septic arthritis and
osteomyelitis, but as he is not febrile, it is very unlikely. Also there is no finding in the history and
physical examination to suggest trauma as the cause. So I think most likely the hip pain is related
to a condition called “Reactive arthritis”. She asked explanation, I explained the condition, and
explained that it is self-limited and only need s pain control and avoiding weight bearing. And
will subside in about 7 to 10 days. However, there is another diagnosis called Perthes which very
unlikely, but as it is important for Perthes to be diagnosed at an early stage cause the treatment
is difficult and time-consuming, I would like to order an X-ray to rule it out.

Feedback 5-7-2018
2 year old girl , brought in by her mom to your GP, unable to walk since this
morning
Tasks
Take relevant history from mother
Examination findings from the examiner
Explain the d/d to the patient and management

Mom did not look anxious , unable to walk since this morning, cried when
woke up from bed. She said her daughter was limping afterwards not able
to move her KNEE.
What happened last night? Nothing. Did she trip over anything? Fall?
Injury? Went to garden and stung by insect? All no . she was home nothing
much done.
Fever/ looks sick? No
Is she active girl? Oh yes she runs all day.
General health? All good
BINDs – good.

Examination
General appearance from examiner: girl is sitting on floor playing. Not
anxious or distressed.
Vital have to say all one by one.
Leg examination. He said examine what? I said knee. (mentioned
everything) he said normal even by comparing to another leg
Then hip .. he said restriction of movement.
Foot .. normal

Then I turned to mum and told her most likely her daughter has perthes.
Explained what it is .. showed her with my hands how hip joint is like ball
and socket and repetitive movement causing inflammation. Then I
remember to ask about flu .. she (oh yes had a flu 1 week ago now
recovered).
So I said yes perthes could be excacerbated by flu or injury.. could also be
synovitis .. some inflammatory markers from flu affect joints.. could be
viral or bacterial we call it septic arthritic/ osteomyelitis.
No other positive finding
I reffered to orthopaedic surgeon to have a look.
Review in 2 weeks.

Feedback 5-7-2018
2 years old girl brought by her mom because inability to walk.GP setting
Task:
1- Hx
2- PE from the examiner
3- Dx and MX

Inside the station young nice mother was sitting on the chair.
I thought its limping case I need to exclude ( trauma, tenosynovitis, perthes, septic arthritis,
osteomyelitis, HSP, insect bite) .. on Hx I asked about my DDx

The mother gave me short hx, she woke up crying and refuse to walk, I asked if she active? She
said usually very active but today not, Hx of URTI 1 week ago, no fever or fall, no rash asked
about BINDS and home situations to exclude child abuse.. all negative. I forget to ask about
smoking in family.

On examination there was limitation in movement of hip joint, I asked about knee 1 st then the
other knee all negative. And foot as well

I drew a diagram explain the hip joint, then told her most likely Dx is tenosynovitis,
inflammation affection the capsule surrounding the joint as a cross reaction due to recent
URTI.
avoid wt bearing, use cruches, pain killer, review in 1 week
if still have the pain refer to orthopedic surgeon, cause sometimes it could be perthes disease

at the moment we need to CBC, ESR, xrat and US of the joint to confirm the DX and exclude
other causes.
I shortly explain perthes and other causes, told her if everything went well we need to repeat
the xray in 4-6 months to exclude perthes and the bell rang.

Passed: global score 5


Feedback 20-7-2018
Transient tenosynovitis
Gait disturbance pass , Global score 4
key step 1, 2, 3 Yes
key step 4 No
history 4
choice and techique of examiantion, organisation and sequence 4
Dx/DDx 5
choice of investigation 4
patient counselling /education 3

2 yr old baby , wake up this morning unable to walk ,


History , PEFE , Dx and DDx and Mx
history to mother
unable to walk since this morning , first time it happen like that , complained of pain around
the knee , no radiation ,
no redness , no swelling , no fever but recently flu like symptoms + one week ago , no
treament at that time , no history of trauma or fall or twisting leg , no previous joint pain
problem , no pain or swelling in other joint areas ,
no rashes or spot on the skin, no bleeding problem
well baby questions are normal
BINDs .... NAD
past history ... NAD
family history ... NAD
Social history ... NAD , no one smoking around baby

PEFE - GA fine, no dehydration , no pallor , vital sign normal but temperature is 37.4,
growth chart normal ,
lower limb examination .... I comparing both lower limbs , no redness , no swelling , no
deformity , no bruise ,
on palpation ... no local rise of temperature , tenderness.... asked specifically where to
examine , i said as the child complained of pain in knee, i will started knee and then hip
and ankles and along thigh and shin ----no tenderness
movement .... knee movement ( flexion and extension ) normal
hip movement (reduced abduction external rotation and internal rotation)
( examiner asked detailed where to examine and what movement to examine in each joint ) ,
no other abnormal findings

Explanation
Lilia , there are a few things that can make a child unable to walk. based on your story and
examination findings , most likely john have a condition called transient tenosynovitis at his
right hip , it is inflammation of covering of hip joint , even tho he complained of pain in his
right knee , the problem is in his hip as hip and knee has the same nerve supply , the pain
may sometimes show up at knee but there is no problem in his knee . this condition is
strongly associated with flu or viral infection he has a wk ago, the viral infection can also
affect on joints .but this is not serious , we can do effective mx for it ...
There are other possible dx i have ruled out like
septic arthritis which is bacterial infection of joint( serious one ) but he doesnt have high
fever suggesting for it ,
trauma or fracture of thigh bone , but less likely , no trauma history
bleeding disorder that can affect the joint like hemophilia or immune related problem like
henoch scholen purpura but less likely
perthes disease , which is a condition due to lack of blood supply to head of thigh bone that
definitely need to be ruled out
Mx
i will run investigation like
full blood examiantion and inflamatory markers, to rule of bacterial infection of joints
USG to hip joint to check any fluid collection in joint space
X ray to hip , to differentiate with perthes disease or fracture
for treatement , mainly is supportive care , i will give effective pain killer , let him rest ,
dont let him wt bearing during this period ,the inflammation should have resolved in 7-10
days time . can put ice packing on the hip joint to reduce swelling and pain . I will review him
when the results are back as well . Red flags , if the child has high fever , generally sick ,
pain get worse , immediately see me or bring him to ED , or otherwise , i will review in 2
wks to check with another x ray again to rule out perthes .
Feedback 26-10-2018
Station 19: Gait disturbance

Passed (GS – 5)

You are a GP. A Child, brought in by dad, is refused to walk since this morning.

Hx, PEFE, explain DDX & MX

Hx: unable to walk this morning, pain Q (role player pointed out medial side of knee when I
asked site of pain), trauma, recent flu (yes), redness/ rash/ insect bite, fever, feel drowsy,
previous injury to joint, well baby Q, home situation, BINDS (didn’t ask much)

PEFE:

 GA, VS (temp)

 Inspection of LL – any redness, swelling, insect bite

 Knee exam (affected side) – FEEL (any point and generalised TDN), range of movement –
all N

 Hip exam –any TDN (yes) , ROM (limitation of movement in IR and Abduction)

 Other joints – Normal

Dx & DDx

DX- transient tenosynovitis aka irritable hip, usually followed by viral infection.

Other DDx- Perthes disease, infection of bone or joint, trauma to joint, abuse (I asked social
history but didn’t mention abuse as my DDX to patient)

Mx:

-Reassure that it will go away in 1-2 weeks.

-I will give him Pain killer.

-Meanwhile, avoid weight bearing.

-Will review in one week.

-I will also arrange Xray to rule out Perthes disease. will refer to specialist if necessary.
-Red flags

68-Hypoglycemia
You are an HMO in an ED where a 47 yr old lady was brought in by an ambulance after she lost
consciousness at a shopping mall. Her BSL was 1.8. Given glucagon and She has now been
stabilised.
Tasks:
1. Take history from the patient
2. Tell her the cause for loss of consciousness and the possible reasons for it

History
1-approach
-Hi I am dr--- I am on of the doctor her.
Patient: what is going on doctor?
-you have lost your consciousness this morning and your BSL appeared to be low so you had
been given glucagon injection and now BSL level is normal and you are stable.
So sally can I ask you a few questions in order to unravel the nature of the problem?
2-open ended
-can you tell me more about what happened? (this morning I was hurrying to the shopping mall
when I felt unwell and lost my consciousness)

3-Chief complaint questions


-did you lose your consciousness completely? How long?
-has this happened before? How often?

4-causes of hypoglycemia questions (MEDIKA: Medication, exercise, diet, infection, kidney,


alcohol)
Diabetes medications and interactions
-have you had a history of DM? (Yes)
-when were you diagnosed with Dm? (3 years ago)
-what medications do you take for it? (Metformin but recently; 3 months ago; my GP add
giclazide because of recent infection)
-do you take any other medications? (Yes will show you card with timisartan for Hypertension
and cholesterol lowering drug)
-do you take your medications regularly? (Yes)
-do you regularly measure your BSL? (Yes)
-have you had regular check up with your DM physician? (Yes)
-have you been hospitalized due to very low or high BSL? (No)
Diet and exercise
-so you said you were hurrying to the shopping mall. Have you missed your breakfast meals or
any morning snacks before that? (yes I missed my midmorning snack)
-have you had any change in your exercise routine like excessively exercising?

Others
-have you had any recent infection or fever?
-any kidney problem or decrease in urine output?
-do you smoke or drink alcohol?

5-Complicatin of DM questions
-any chest pain, BOV, weakness or numbness?
-any skin or UTI?
-do you feel thirsty or passing large amount of urine?

6-PMH, family hx, occupation, support? (hypertension, hypercholesterolemia, family history of


DM)

Explain reasons.
-there are several possibilities why you have LOC but most likely, you have a condition called
hypoglycemia have you heard about it?
-this is where you have a low BSL of 4 mmol or less.
-this condition can be caused by diabetic medications like insulin or other oral medications like
gliclazide. Other causes could be due to excessive exercising, recent infections, and kidney
problems.
-However, most likely cause in your case is due to skipping your midmorning snack. Because you
are taking diabetic medications, you need to have regular meals and health snacks to keep your
BSL under control.
-someone with hypoglycemia can have mild symptoms like feeling hungry, sweaty, hand shakes
or funny racing of the heart. They can also have sever symptoms like Loss of concentration,
confusion, fits or even Loss of consciousness.
-reading materials+ review

Case 5-5-2018
Male pt was transfered to the hospital bc of LOC. BSG 1.8. Take history and explain the diagnosis
with reasons. This case is hypoglycemia on diabetic pt .

Case 5-6-2018
lady admitted with hypoglycemia,now stable after giving glucagon,take history,tell reason why
she had hypoglycemia.(missed morning meal,went shopping centre)..

Case (11/11/2017)
case 1/ a women 47 years old brought to the ed where you work as an hmo/ or intern because
she lost consciousness and her sugar level was 1.8 ,
Task
a-take history.
b-tell the patient possible causes of the condition.
She was diagnosed with dm , hypertension and hypercholesterolameia and has obviously missed
a meal in the morning, Was started to medication 6 months ago a gliclazide I think.
Gave reasons of her hypoglycaemia

Case (11/11/2017) Altered conscious state


You are an HMO in an ED where a 47 yr old lady was brought in by an ambulance after she lost
consciousness at a shopping mall. Her BSL was 1.8. Given glucagon and She has now been
stabilised.
Tasks:
1. Take history from the patient
2. Tell her the cause for loss of consciousness and the possible reasons for it
Introduction
Role player : What’s going on?
I started by building a rapport. How are you doing Ms…. ? Are you ok to talk to me? You have had
quite an eventful day, I explained briefly what was already given in the stem, then asked her if I
can ask a few questions? She was ok to talk.
- She has been a diabetic on medications since 3 years and recently new medication added 6
months back.
- Never had low sugars in the past. Good control and visits to her GP regularly.
- Gave list of her medicines when asked- metformin, gliclazide ( 6 months ), temisartan
- She said she remembers hurrying in the morning and getting to the mall.
I asked what she means by hurrying ?
- Did she miss her breakfast? She said No.
- I asked- what she had ? Toast and orange juice at 7:30 AM – Usual breakfast
- Then I asked what time she got to the mall – said 12 PM.
- I said that quite a long gap, do you usually take anything between breakfast and lunch? She
said she missed her mid-morning snack.
(Note: Morning candidates were given different history, simply missing morning breakfast)
- Further I asked about recent fever, UTI symptoms, decrease in wee, any renal troubles, over the
counter medicines.
Past history, Family history, SADMA, Social situation
Explained to her that she had Hypoglycemia, her blood sugar level was dangerously low and that
led to loss of consciousness.
Described mild, moderate and severe symptoms of hypoglycaemia.
Prompt response to glucagon makes other possibilities of unconsciousness unlikely.
The reason for low sugar was due to missing the mid-morning snack, however there a lot of
causes why I diabetic can have low sugars such as- gliclazide, infections, kidney problems,
interaction with other drugs, insulin.
We have done all the necessary investigations here in the department and I will be reviewing
those to see if there is any other cause.
I’m going to give you written information of hypoglycaemia its causes and how it can be
prevented and treated.
Key steps: 1,2,3,4- Yes Global score: 5 ( Pass)
Approach to pt/relative: 5 History:5 Diagnosis/ differential diagnosis: 5

69-Assessemnt of comatose patient


Unconscious patient primary survey done
tasks
-assess glasco coma scale (chart given)
-do other relevant examination
-tell the most likely diagnosis and four differential diagnosis to the examiner.

Note/
There will be a young patient lying unconscious on the bed covered with a sheet.
On the table beside the patient, there will be these instruments: tendon hammer,
ophthalmoscope, otoscope, pen torch, cotton wool, and stethoscope

1-Glasco Coma Scale


Eye
spontaneous (4)
to voice (3)
to pain (2)
nil (1)

Verbal
Oriented (5)
Confused (4)
Inappropriate word (3)
incomprehensible sound (2)
Nil (1)

Motor
Obey command (6)
localized to pain (5)
withdraw to pain (4)
abnormal flexion (3)
abnormal extension (2)
Nil (1)

suggestions
*The question for eye is can you open your eyes for me?
*the question for verbal is can you tell me your name and where you are now?
*the question for motor is can you bend your elbow for me?/ or can you move your fingers for
me?
Let us start with task 1
Task 1
feedback approach
washed hands and started with the GCS. The chart was on the table so I take it and started like
this:
-Hi my name is dr. can you open your eyes for me? ( no response)
Now I’m gonna press on your shoulder now. ( he opened his eyes, tell me one inappropriate
word and move his left hand toward his right shoulder)
-checked again the verbal response by asking him ; if you could tell me where are you now? (no
response )
Now I’m gonna press on your shoulder then while pressing asked the same question again so he
answered with inappropriate word.
I ticked the chart with ( open eyes to pain=2, inappropriate word=3, localized to pain=5 )
then I told the examiner that his GCS is 10

the Better approach is


-wash your hands and start with the GCS. The chart will be on the table or the examiner will
hand you the chart so take it and start like this:
-Hi, my name is dr.
-Eye: can you open your eyes for me? (No response)
-Verbal: if you could tell me your name and where are you now? (No response)
-Motor: can you move your elbow for me? (No response)
-Now I am gonna press on your shoulder. (The patient will open his eyes, tell one inappropriate
word and move his left hand toward his right shoulder)

Tick the chart with ( open eyes to pain=2, inappropriate word=3, localized to pain=5 )
then tell the examiner that his GCS is 10

Now Task 2
Task 2
feedback approach
-after exposing the patient I started with :
-GA: no bruising, laceration or trauma , no raccoon eyes or battle sign. Rhinorrhea or Ottorrhea,
no scars, normal skin color, no swelling, deformity or wasting.
-VS: I would like to check the pulse can I have a watch --- examiner said normal
I would like to check his blood pressure----- examiner normal
respiratory rate and temperature----- examiner said normal

-Started with eyes


normal pupil size and shape
using the pen torch: direct light reflex normal
corneal reflex--- examiner said normal
the ophthalmoscope was unassembled.. Now I would like to examine the back of eyes with
ophthalmoscope so after assembling it--- examiner said normal
-then took the otoscope again after assembling it the examiner asked what do you want to
check: I answered for any discharge, CSF leak and ear drum condition---- then he said normal.
-Then I examined the neck---- neck stiffness +ve
-I said I would like to examine the peripheral nerves tone and reflexes.
I examined the biceps reflex with tendon hammer twice unable to notice anything so I told the
examiner that I cannot elicit the reflexes --- he said normal
-I want to examine chest and heart with my stethoscope--- examiner said normal
-Abdomen--- examiner said normal.

I think the stepped wise approach is like this (examiner will help you like above)
1-general inspection
-raccoon eyes, battle signs, Ottorrhea, rhinorrhea.
-trauma, injury, bruising, rash.
-insulin injections especially on tummy / IV drug injection sites on arms
2-Vital signs
3-Neck
-stiffness
4-Eyes
-pupil size and shape
-light reflex
-corneal reflex
-fundoscopy
5-Ears
-otoscopy
6-Chest
-breathing pattern
-heart sounds
7-abdomen
-insulin injection
-other aspects
8-Neuro: tone and reflex
9-office tests
-BSL -ECG

Now Task 3
Task 3
feedback approach
-So I told him I finished the examination
the most likely diagnosis is meningitis viral or bacterial
other DDx are
-subarachnoid hemorrhage
-Trauma
-Tumour
-hypoglycemia or hyperglycemia so I need to do BSL office test
-electrolyte imbalance—hyponatremia, hypomagnesemia.
I had time so I arranged for Ix like FBE, UCE, LFT , BSL, etc.. (However it was not a task though)
Wash hands

Differential diagnosis are


1-meningitis
2-encephalitis
3-subarachnoid hemorrhage
4-hypoglycemia
5-electrolye disturbance
6-drug/ alcohol
7-trauma
8-CVA
9-Epilepsy

If Investigation
-FBC, ESR/CRP, Blood culture
-UCE, LFT
-BSL
-serum urine, drug screen, blood alcohol level
-brain CT

If treatment
-admit to be seen by a pediatric registrar
-TONG
thiamine (to cover for possible alcohol intoxication)
oxygen (if O2 less than 93%)
naloxone (possible opioids overdose)
glucose (possible hypoglycemia)
further management depends on Ix
70-GastroOesophageal Reflux
Mother of 4month old baby, comes that baby throws up milk all the time
Tasks
1. History
2. Physical examination findings from examiner
3. Counsel mother

History
1-Reflux Questions (= Vomiting)
-I can see from the notes that your baby is throwing up milk all the time? Since when did it start?
How often? (Since birth)
-is it getting worse?
-what is the color? (Just milk)
-what is the amount?
-is it related to feeding?
-is it related to change in position?
-anything make it better or worse?

2-associated symptoms (Normal)


-how is his bowel motions?
-is he crying a lot while feeding? Does he draw up his legs while crying?
-any fever, rash or recent viral infections?
-any cough? Any difficulty in breathing?

3-Dehydration questions (normal)


-does he look drowsy or irritable?
-any change in the number of wet nappies? Does he cry on passing urine? Any smelly urine?
-do you breastfeed or bottle feed him? Any problems with feeding? Have you introduced solid
food?

4-BINDS
-any complications during the birth?
-is his immunization up to date?
-is he thriving and growing normally? (will give you growth chart 50% percentile)
-any siblings with similar symptoms?
-do you have good support?
-anyone smoke at home?
Physical examination from examiner (all normal)
1-General appearance (DR PJL)
2-V/S
3-growth chart (no need if mother gives you)
4-ENT + neck stiffness.
5-Full systemic focus on abdomen
*Inspection (distension, masses)
*palpation (tenderness, masses, RIF emptiness, rigidity)
*auscultation (bowel sounds)
*Hernia orifices
*genitalia and anal inspection
6-office tests

Explanation
condition
-most likely he has Gastroesophageal reflux which is when the content of the stomach are
brought back up to the food pipe or even into the mouth after each feed in otherwise healthy
and thriving well baby.

Cause
in babies it is common for the valve at the top of the stomach to be quite loose that is why the
reflux happens.

Outcome
let me assure you that as your baby grows older this valve will become stronger. It gradually
improves with time especially after introduction of solids. It is a common condition, and natural
process will resolve by itself with time.

Management
advice
- frequent smaller feeds (not less than 3 hourly)
-feed him in upright position
-burb him after each feed
-do not put him immediately to sleep after each feed
-put him in cot with head elevated 10-20 degree

Red flags
vomit forcefully, not thriving, not feeding well, fever

Review+ reading materials


Feedback 7-7-2017
Mother of 4month old baby, comes that baby throws up milk all the time
Tasks
1. History
2. Physical examination findings from examiner
3. counsel mother

WHAT I DID
hello examiner, hi laura, Im dr..looking after you and your baby. may I know the baby's name
John
since when..since birth
is it just milk..yes
blood or green color..no
any problems with poop..no
no. of wet nappies..same
hows his weight gain and growth...gave me the growth charts..(they were at the 50th percentile)
these are fine nothing to worry about
BINDS HX..all normal

I will just speak to my examiner. Examiner can I know the general Appearance...hes playful
happy. Vital signs..all normal..Signs of dehydration? none.. neck stiffness rash? no..focused
abdominal examination..normal. THANK YOU EXAMINER, I WILL SPEAK TO MY PATIENT
Laura, John seems to be doing pretty good. his weight is good and so was the examination. He
probably has something we call as Reflux..drew the esophagus plus stomach..explained lower
end of foodpipe not well developed, will get better with time. give him feeds in the upright
position, burp after every feed
RED Flags, not gaining weight, vomitting profusely, fever, tummy bloated, goto ed
Thanked Laura and examiner

PASS
keystep 1 2 3 yes
approach 4
history 4
choice of examination 6
diagnosis 6
Feedback 5-7-2018
1 4 months old baby boy, brought in by mom to your GP for vomiting.
Task
Take history – Examination from examiner
I started by introducing myself “ Good morning, I’m ---- one of the doctors working in this …….
(GP/hospital)…. Nice to meet you. What is concerning you?

Asked mom what’s concerning her? “She said her baby is vomiting. Everyone is saying she’s fine
but I’m concerned”
So I said “I’m glad you came. Mum’s know best ”
Vomiting Questions: When did this start/ color/ is it after meals? Is it projectile or just spits?
General health: Gaining weight? Sleep/ feeding/ wet and dirty nappies as usual?
Any jaundice/ looking sick/ fever / rash /// ((all no))
All good at home? (yes) Anyone sick? (no)
BINDS

Examination: general appearance .. vitals .. tummy examination …. Growth chart (examiner gave
me sheet .. it was normal)

Explain d/d to mom


Management
My approach
Excluded my differential like pyloric stenosis, UTI, gastroenteritis, bowel obstruction. Came to
the most possible diagnosis of GORD, explained this to mom with the help of a diagram that
babies don’t have developed valve between food pipe and stomach.. so when they grow, it
developes better.
She asked when? (told her around 6 months – 1 year). and briefly explained the management

–reassurance, burping , positioning , explained the red flags. Gave her reading material or
review , mother looked happy and satisfied.
71-OCP request
Sample case/ Your next patient at your GP is a 19 year old, university student Jessica, asking you
for a prescription of oral pills, as she is now planning to become sexually active.

TASKS
Focused relevant history.
Examination findings from examiner.
Counsel the patient accordingly.

History
-Hi Jessica, I'm Dr----- one of the GPs of the clinic, how can I help you with today? (Well doctor,
I'm going to Uni soon, and I'm considering starting on oral contraceptive pills.)

Only 5 P’s Questions


period
-When was your last menstrual period? (3 weeks ago.)
-Are they regular? (yes)
-how many days of bleeding and how many days apart? (28 days cycle)
-any pain or heavy bleeding during menstruation? any pain in between the periods? (Just
moderate, and no problems with periods or bleeding in between periods.)

Sexual history
-Have you been sexually active before? Do you have a stable partner? (No, I haven't. But my
boyfriend and I are going to Uni together)
(If yes, do a urine PT to confirm if patient is not pregnant at the moment)

Pregnancy (not relevant)

Gardasil vaccine
Have you received the Gardasil vaccine? (Yes, I received 3 shots of that already)

Pill (Contraindications to OCPs)


BC (bleeding, clot)
MSHL (migraine, stroke, heart, liver)
BOW (breast, ovary, womb)
DDH (diabetes, depression, hypertension)
SAD (smoking, alcohol, drugs)
- any abnormal vaginal bleeding? (Bleeding), any history of clotting in the veins of legs? (Clot)
- Any previous history of migraine, stroke, heart diseases, liver disorders?
- Any history of breast, ovarian or womb cancer?
-any history of diabetes, depression, high blood pressure?
-smoking, alcohol and recreational drugs?
-Do you take any prescription or over the counter medications? Any allergies?
Thank you for those information, Jessica. I will just talk to my examiner and I will get back to you
shortly.

Physical Exam from examiner


1- general appearance
-What is the BMI of my patient?
-Any pallor, icterus, cyanosis, lymph node enlargement, edema, poor skin turgor, dry mucous
membranes, delayed capillary refill time?
- Any edema or tenderness of the lower leg?

2-Vital signs
What is the BP, PR, RR, Temp and Sats of my patient?

3-quick chest and heart


how are the heart sounds? Is the rhythm regular? Any murmurs?
Is air entry equal? Any abnormal breath sounds?

4-breast
Any palpable breast lumps, tenderness or visible distortion or dimpling?

4-CNS
How is the motor and sensory exam of the upper and lower limbs? (stroke symptoms)

5-Abdominal examination
Is there any visible distention or mass of the abdomen? Is there any hepatosplenomegaly, any
mass or tenderness?

6-Pelvic examination (just inspection)


Are there any visible lesions in the vulva and vagina? Any discharge or bleeding?

7-office tests
I'd like to do a urine dipstick test and a blood sugar level.

Thank you for those information. I'd speak with my patient again.

Diagnosis and Management


1-OCP mechanism of action
-Okay, it seems like it would be possible for you to start on combined OCPs, so I would like to
give you information about it.
-Combined OCPs contain 2 hormones, estrogen and progesterone which is normally present in
your body which regulates your periods.
-it inhibits ovulation, the release of egg from the ovary. To a lesser extent, it increases the
thickness of your cervical secretions so the sperm will find it difficult to get through. And just in
case fertilization happens, it changes the lining of your womb so that implantation does not
happen.
2-Dose
(TAKE PILL PACK)
In a pill pack, there are 28 pills, 21 are hormonal pills, 7 are sugar-coats or dummy pills. Starting
taking the hormonal pill from the 1st day of your next period, 1 pill a day, at the same time
everyday.
-Continue the hormonal pills for 21 days and then on starting the sugar pills, you get your
periods.
-However if you want to start taking the pill right away without waiting for your next period, you
may, but use alternate methods like condoms for 7 days.

3-Side effects (minor and major)


While taking the pill, you may experience side effects such as:
-Minor SE like nausea and vomiting, abdominal bloating and breast tenderness.
-Breakthrough bleeding or bleeding in between periods will usually settle in 3-4 months.
-Major side effects such as DVT, stroke and MI could happen but are rare with low dose pills,
such as what you will be taking.

4-Advantages
Advantages of the pill include:
- periods become more regular, lighter and shorter.
- There is less dysmenorrhea.
- There is decreased incidence of benign breast lumps and pelvic inflammatory diseases,
- decreased incidence of endometrial and ovarian cancer, and thyroid disorders.

5-Disadvantages
However, you must remember that OCPs do not protect against sexually transmitted
infections, so you must use condoms along with it just in case you're concerned about STIs.

6-Missed pill (imp)


-there will be a leaflet that come with the pills
-if you missed a pill of less than 24 hours just take the recently missed pill As soon as you
remember and keep going with the rest even if it means taking 2 pills on the same day.
-if you miss a pill for more than 24 hours, take the recently missed pill and just keep going with
the rest even if it means taking 2 pills on the same day.
-If you keep going with the rest, and the dummy pill period falls within 7 days of missing the pill,
skip taking the dummy pills and start the hormonal pills from the next pack. This will mean that
you will miss having your periods. Alternate methods of contraception like condoms should be
used for 7 days after missing the pill.

-I will give you reading materials for your further information, and if you experience any
problems with the pill, report back to me.
-Remember to do a pap smear every 2 years.
-I will review you in 3 months time, and then yearly after that. Do you have any questions at this
point?

Feedback 7-9-2018
Station 20 – OCP request – PASS
OCP counseling
- 15 –year-old girl , wants to get OCPills , as gonna have sexual relationsh with her boyfriend
aged 16-year-old , at the same school ,
Not getting along with her parents, but living with her grandmom. LMP – 10 days ago .
Task:
1. ask history ( 4 minutes )
- explain about the patient’s age and explain about the definition of Minor for contraceptive pills
Ask about absolute and relative CI to COC pills
- all normal
- understanding of attitude to the pregnancy , understanding about the pills MOA after
explanation
2. Focus PEFE ( Specifically ) – ask about breast lumps, BP , hepatomegaly , Jaundice , UDS - all
normal this task 2 , ask about secondary sexual characteristic – examiner slowly mention that
tanner stage 2 , pubarche – normal , adrenarche for axillary hair - normal for age
- I don’tknow what I need to ask for this PEFE for OCP,
2. Provide counseling to the patient
- explain the eligibility to the pills , but encourage to involve with family members esp: with nana
.
( Patient said that , in next visit, she would try to visit me with nana – okay doctor ? )
explain MOA and before explanation about miss pills, diarrhea – bell rang
- so quickly explain about need to know miss pills and vomiting – examiner said that – relax –
time’s up 
Feedback – Contraception request – Pass
Global score – 4
Key step 1 to 3 – YES
Key step 4 – NO
Approach – 4
History – 5
Examination , sequence – 5
Patient counseling/education – 4
Feedback 7-9-2018( Contraception request- PASS)
15 year old girl comes to you know about OCP. She is going to start sexual relationship
soon. She lives with her granny.
- HX
- PEFE
- Counsel accordingly.

FEEDBACK-PASS
Global score: 5
Key steps: 4/4
Approach to pt: 5
HX: 5
Choice of technique of examination, organisation and sequence: 5
Patient counselling: 5

2mint thinking-
5P, Partner's age, contraindication for OCP, missed pill, follow up after 3 months. Red-flags.
side effects of ocp.

There was an smiling young girl sitting on the chair. after introducing myself, I said
confidentiality. I said I am very glad that you are here to ask for OCP. I'll give you all the
information that you need, but I need to ask few personal questions as well. Would it be
alright with you? (YES) Have you talked about this to your parents? to your granny? (No)
How older is your boyfriend?(16) Is he your family relative or teacher by any chance? (no,
my senior in school) Have you started sexual relationship yet?(no) How much do you know
about OCP? ( Pills, need to take every day, got info from internet) Well, I'm going to ask you a
bit embarrassing question now, tell me what could happen if you haven't used any
contraception? ( She smiled and said pregnancy) I smiled and said good. Now tell me what
else is associated with unprotected sex? She became confused and looked at examiner I
guess. Then, I said, have you ever heard about Sexual Transmitted Infections? Then, She
smiled at me and said no. I said , no problem, I'll make sure that at the end of this
conversation you will know all info.
Now, I ask about her periods(2 weeks back), any bleeding, pain, discharge, gardsil vaccine,
any pregnancy symptoms? any medical problem?
Contraindication of OCP- migraine, spotting, liver problem, clotting problem, fx of cancers.
Then I said her that I'm going to talk to my examiner and will come back to you soon.
I asked examiner-
VITALS- all, especially BP
BMI
PICKLED
Any abnormality in breast
Hepatomegaly
Pelvic exam with presence of a chaperon - any discharge? Hymen intact? (IF NO- then PT )
thanked him.

I said to her, Okay Jenny, everything seems normal to me and I'll prescribe you OCP. There's a
starter pack available to make it more convenient for you . I draw a rectangle box and said
there is a red line in the starter pack. You will start from this line and 7 tablets are included
in this red block. This medication will be effective after 7 tablets.

There are 21 hormonal pills and 7 sugar or iron pills. They usually change your female
hormone level in your body to prevent pregnancies. You have to take this medicine every
day, preferably at the same time. Make a specific time, such as 9 o'clock in the morning or at
night. Make sense? (yes).

Now, you must be thinking what if I missed a pill? Right? (She smiled at me). I started to
explain if one pill within 12 hours- no problem, take it as soon as you remember. More than
48 hours, any vomiting, diarrhoea- Use condom. Any unprotected sex - go to pharmacy for
emergency contraception.
Always use condom to protect yourself from any STI.

Then, I ask her to repeat all the info I just said. She did.

I said, any time you feel headache, unwell, yellowish discolouration, spotting- come back to
me.
I will give you all reading materials, give it a read. Come back to me after 3 months for some
blood tests.(LFT) I would like to talk with your boyfriend as well if you are okay with it. She
said alright.

I did finish every points in time in this case. I was worried for my time mx about this case
but I did it.
Feedback 18-9-2018 OCP request 15 years old
15 yr girl ask for OCP, boy friend 16 yrs old. Task: Hx, PEFE, counsel the patients
same as Karen
HX: Said confidentiality.
Ask contraindications for OCP– none of them
HEADSSS: She said her parents know about the bf but not sexual activity. She argues with
parents and moved out after that. I encouraged her talk to parents and she agreed.
A bit stress from the exams in school but manageable.
PEFE: everything normal
Counsel: explained the need to assessed ability of mature minor.
The stem said the patients are aware of how to take the OCP so I didn’t say that part. Said
complications, risk, failure rate, not prevent from STD.
Score: 4 Key step; yes/yes/yes/no
Approach: 4, Hx: 5, PEFE: 4, counselling: 4

Feedback 18-9-2018 OCP request


You are a GP and your next pt is a 15 year old Uni student who came to you with
request for a script of OCP. She knows how to use the OCP. She has been living
Independently with her BF for the last 6 months and recently they decided to
start sexual relationship. Her BF is 16 years old and studies in the same uni. She
doesn't have good relationship with her parents, she used to live with her grand
mother but recently move out to live with her BF. She doesn't have any known healt
issue and her last check up with regular GP was 6 months ago and everything was
normal. Her immunisation is up to date. You are seeing this pt for the first time.
Your task is to
1. Take further history from the pt to assess the competency of the pt regarding
OCP use. ( you should spend no more than 4 mins in this task)
2. PEFE from the examiner ( examiner will only give findngs of what you have asked
for)
3. Consel the pt accordingly.
The pt was a teenager and she was not helpful. She didn't wanna talk much , she just
wanted the script and go. So I explained her that yes I will give you the script but we need to
follow some protocol as you are 15 years old. So I will ask you some questions and then I will
explain everything and then give you the script. She agreed with that.
So I asked all past Hx, surgical history, O&G history, recent infection, breast ca history, Migrain,
SADMA, 5PS all were normal.
On PEFE I asked GA, BMI, Vital signs, Breast , abdomen, Pelvic examination, Urine deepstick,
ecg and BSL
Explained her that you can qualify for OCP as you are living independently and your BF is just 1
year older than you. Then I was running out of time. So i rushed and quickly told her what to do
if she misses the pill in different days of her cycle. I didn't talk about how to take the pill as it was
given in the stem that she knows how to use it. I told that it will prevent conception but not STI
so you still have to use condom for that. But I forgot to check her understanding as I was rushing.
Then talked about break through bleeding and some side effects. Gave her 4Rs and bell rang.
AMC feedback: Station 04: Contraceotion Request Grade: Pass Global score: 4

72-Recurrent fall Diabetes

60 yrs old man comes with recurrent Fall within a month. He recently had a fall yesterday and he
is frustrated about the fall and he wants to discuss with you.
Task:
-History
-Physical examination findings card from examiner
-Explain the patient the causes of the Fall.

History
1-check hemodynamic stability, show empathy and ask of he has any concern?
2-Fall questions
-How many times have you fallen? (3 times this month)
-when did you fall?
-what were you doing at each time you fall? (All 3 times tripped over at garden & grounds)
-where did you fall In the house or outside? (no need if answered in the garden)
3-Before
-did you get any warnings like strange smell or feeling nauseous?
-did you have any dizziness, chest pain or palpitations?
-any headache, blurring of vision, weakness or numbness? (Numbness in foot)
-have you skipped your meal?
-Do you black out or feel dizzy when you stand up quickly?
4-During
-has anyone seen you when you fell?
-have you lost your consciousness?
-have you passed urine, bitten your tongue or has jerky movements?
-did you hurt your head?
5-After
-were you able to get up yourself?
-any confusion, N&V, headache or any weakness after the event? Any speech difficulty?
6-General questions
-PMH; have you had any past illnesses like:
DM? (Yes) what medication do you take for it? (Metformin) do you take it regularly? Do you have
regular check ups? (No)
Hypertension?
Heart problems?
Arthritis? (Has OA and joint pain)
Parkinson’s?
How is your vision and hearing? (Checked his eye 1 year ago and it was normal)
Have you had any fractures?
-Do you take any medications?
-do you smoke? Drink alcohol?
-do you have good support?

Physical examination findings from the examiner (Card will be given)


1-General appearance
pallor, dyspnea, cyanosis, signs of trauma
2-Vital signs
-BP + postural drop…… PR + regular or not….. Temp….. RR
3-CVS
-JVP + carotid bruit
-heart sounds+ murmurs
4-Respiratory
-air entry
-breathing sounds
-added sounds.
5-Neuro
-peripheral (tone, power, reflex, sensation) (glove and stocking sensory loss in the lower limbs
with loss of perception and sensory loss with monofilament)
-crania nerves + fundoscopy and vision (fundoscopy revealed silver wiring and cotton wool
exudates)
6-Joints
any tenderness or restriction of movements? (severe restriction in movement of his knees due
to severe pain)
7-GIT
-tenderness, organomegaly
8-ENT
-ear was, tympanic membrane
9-Office tests
-UDT
-BSL (15 mmol/l)
-ECG

Differential diagnosis
1-polypharmacy
2-vision problems
3-arrythmia, valvular heart disease, postural hypotension
4-stroke
5-DM neuropathy, hypoglycemia
6-UTI
7-arthritis
8-ENT (earwax, BPPV)
Explain Dx and Ddx
-there are several possibilities why you have recurrent fall.
-I am suspecting it most likely due to a condition called diabetic neuropathy which is sensory loss
due to uncontrolled DM as your BSL is high and sensation is affected in both lower limbs from
examination.

-could be due to vision problem as on fundoscopy there is some changes in the back of the eyes,
which again could be a complication of your DM.

-could be due to joint pain or arthritis as there is severe restriction of movement due to pain.

-could also be a mini stroke as your BSL is high and can be a complications of DM.

Feedback 4-7-2018
67 yr old lady with a hx of fall
Task hx, PEFE, explain diagnosis with possible causes.
Hx-had fall 3 times-inquired about each incident-all 3 times tripped over at garden & grounds
Asked all DDs for fall
No LOC/no seizure features/no head injury
No cardiac/CNS sym,
Initially denied vision/hearing
Knee pain
PMHx-DM on metformin long yrs
No regular check up-never hypo sym
Numbness in foot
SADMA-Nothing positive –as far as I remember.
PEFE-card given
BSL 15mmol, diabetic retinopathy & neuropathy finding-loss of proprioception
Knee OA changes
Explained with DM & OA causes

Feedback 19-7-2018
CASE 14 Fail( Key steps 1,5 yes 2,3,4 no score 4,3,3)
Recurrent fall ( single line scenario of a55 yrs old man come to you after a fall yesterday.
Role player was nice but the examiner is too old , I cant imagine I failed this case coz I have done
every thing same as my friend did and passed.
Case (5/10/2017)
Station 4 Falls (Repeated Fall (DM, OA, Diab Retenopathy)
Pass

60 yrs old man comes with recurrent Fall within a month. He recently had a fall yesterday and he
is frustrated about the fall and he wants to discuss with you.

Task: History

Physical examination findings card from examiner

Explain the patient the causes of the Fall

Introduced myself and told him i would like to check his vitals first and asked the examiner the
vitals ( had to ask specifically and all were normal). Then asked him if he has any pain ( i asked
this for building up rapport) and he answered he has pain in his knee joints. I asked if he would
like pain meds and he said he had it before he came to me. He said he has OA thats why he is
having jt pain. No vision impaired and he told me he checked his eyes 1 year ago and it was
normal. Then asked about the fall ( before, during, after episode). Had 3 episodes of fall within
this month, no dizziness or chest pain or racing heart or SOB, No jerky movement during episode
and no loss of consciousness. No slurred speech, no muscle weakness, no headache after the
episode. No head injury, no fever, no rash, no vomitting, no cancer features. Has DM on
Metformin and compliance with meds and followed up regularly. SADMA. From PEFE cards,
fundoscopy revealed silver wiring and cotton wool exudates, Glove and stocking sensory loss in
lower limbs with loss of proprioception and monofilament test ( sensation loss) and he has
severe restriction in movement of his knees due to severe pain. RBG was high ( dont remember
the exact value).

Causes of fall
- could be due to vision impaired as on examination, Fundoscopy to look at your eyes revealed
some changes in back of ur eyes, and could be due to complications of DM.
- could be due to Jt pain.
- could be due to sensory and jt sensation loss in ur jt due to complication of DM.
- could be due to ministroke as ur sugar level is high and one of the complications of DM.
- could be due to nasty growth in ur brain but it is very less likely as u dont show any symptoms
and signs of cancer.

Covered 3 key steps out of 5


Approach 4
History 3
Dx and DDx 4
Global score 4
Feedback 20-9-2018 Recurrent falls (PASS)
70 years old male comes to your GP clinic because of fall which happened yesterday.
Tasks : Focus History.
PE card will be provided.
Dx and DDx with reasons.

Approach :
I greeted the patient & asked him whether he’s comfortable or not and is there any concern?

History :
This is the 3rd time of fall. No loss of conscious in each time. All of them are similar. He says
once, he slipped on the small stones & fall. The other time was he bumped into sth and fall.
Don’t remember the other one but he’s not injured. In asking general health, he’s got Type 2 DM
for many years & he’s on Metformin. No changes in dose or medication. No history of recent
illness. BSL is not regularly checked, do not go to specialist regularly, seemed not well controlled.
When asked about complications, he denies all. He also tells he’s got knee pain (OA) and he’s on
OTC pain killer. Apart from that, there is no problem with hearing, vision, no h/o stroke, heart
problem, other joint problems, no features of postural hypotension.

PE Card :
GC – well
VS – Stable (BP on lying, standing not given)
Eyes – microaneurysms +
Heart – normal
Sensation on limbs – loss in gloves and stocking appearance
Both knees – acutely inflamed, tenderness +, reduced active and passive movement
Office tests – BSL – 18 or 24 mmol/L
I don’t remember whether UDS and ECG given or not.

Dx and DDx :
I explained that DM is not well-controlled and caused complications like loss of sensation in all 4
limbs and eye problems which can lead to slip and fall. But reassured that u don’t have other
complications like ur heart is fine. Ur kidneys are doing well. Another condition is OA knee. Then
I gave other DDx that I excluded in h/o.
73-Cerebral palsy
Case 1/ GP, 18 months old baby girl b/in by mom coz of not using her left side of hand & legs.
Task:
-take history including developmental history.
-Examiner will give u PE card.
-Dx with reasons.

Case 2/ mother of 12 month old baby comes to you complaing that the baby doeznt seem to be
using his right side and sometimes left side
TASK
1. History
2. you will be given physical exami ation findings card
3. explain condition to the mother

History
1-ask her about her concerns?

2-since when have you noticed that he is having this problem?


(Case 1-does not use left side since 5 weeks of age)
(case 2 (right version)- Since the beginning but it is more obvious now since he started holding
things and walking as he falls to his right side so thats why i am concerned now)

3-Developmental history (key)


(case 1-all milestones delay; can’t sit or roll or walk or stand)
(case 2- only 1 of the milestone like rolling over or sitting was delayed, Everything else was
normal/ another said can walk but fall to right side and others normal)
Gross motor
-is he able to walk?
-is he able to stand or sit by herself?
-can he roll over?
Fine motor
-does he reach and grasp things with his left hand? (usually at 5 months)
-can he use his 3 fingers to grab things? (usually at 9 months)
Social
-does he maintain eye contact? Does he smile or laugh?
Language
-doe he say at least one meaningful word? (usually at 1 year)
-if he is not able to speak one word then go backwards and ask, is he able to speak baba/ dada?
Babbling?
4-BINDS
(case 1: breathing nd feeding difficulty, needed resuscitation at birth, forceps delivery)
(case 2: mum had fever post-partum for the right side version/ BINDS fine for the left side)
Birth (key)
*pregnancy
-any infections during pregnancy? Any medications taken?
-Did you have regular checkups like US, sugar test and down screen?
*Delivery
-was your delivery a normal vaginal or CS and why?
-was it assisted? Did they use forceps? Was the labour difficult?
-how long did the delivery last?
*After birth
-term or preterm?
-did he cry immediately after birth?
-did he need any resuscitation?
-any trauma or injury?
-any infections?
-has he had a formal hearing test and heel prick test?
Immunisation
Feeding (is he eating and drinking well?, how is his waterworks and bowel motions?)
Home situation and support
family history of similar problem or any other illnesses? Key

5-Differential diagnosis questions


-Past medical history
-medications and allergy
-Arching of back? Scissoring of legs or crossing of legs when you hold her? Weakness or stiffness
of her limbs? (r/o cerebral palsy)
-Have you observed any fits or jerky movements in her? (r/o epilepsy)
-High pitched cry, decreased activities, dry skin, lumps or swelling around belly button (r/o cong.
Hypothyroid)

Physical examination from the examiner


(case 1-On PE card, head circumference reduced, jerks increased in left side.)
(Case 2- PEFE (card): weight was on the 50th percentile. Tone was increased and hyperreflexia on
left side. / Another hypertonia right side)
1-GA (DR PJL)
2-dysmorphic features
3-Vital signs
4-Growth chart
5-ENT
6-meck (thyroid)
7-CNS (tone, power, reflex, sensation) Key
8-musculoskeletal

Diagnosis and management


-Most likely your child is having a developmental delay, which means delayed development in
major milestones.
-it can happen due to different reasons, but in his case its most probably due to cerebral palsy.
Have you heard about it?
Condition
-it is a chronic, lifelong, non-progressive locomotor disorder that can cause problems with
posture and movements in addition to vision, hearing, speech and learning abilities problems.
Can also cause feeding problems or fits.
Cause
-the cause is mostly unknown but may be related to:
TORDH infections
placental infections or insufficiency
brain malformation
birth asphyxia
prematurity or hypoglycemia
-This condition cannot be predicted before birth therefore no prevention present yet.
Complications
-only predicted after assessment by the specialist and over a period

Management
-I would like to refer him to a paediatric specialist to confirm the diagnosis.

-before diagnosing CP some tests may need to be done like metabolic screen, TORCH screen,
EEG, Karyotyping, CT/MRI

-Once it is confirmed, we will work as a MDT composed of physiotherapist, occupational


therapist, speech therapist, orthopedic, nutrition specialist and learning support.

-Children with CP usually grow normally and if there IQ is normal they can attend regular
schools, otherwise special schools are also available.
-support group also available, centerlink for money support.
-social worker.
-Red flags: If the child develop fits, unable to eat or drink please come back immediately.
-reading material, review regularly.
-I understand this is tough for you but medical science is improving and improvising and a lot of
research is going on in this field.

-Risk for future pregnancy is not increased because of this child.

Differential diagnosis
1-Cerebral palsy
2-brain tumour
2-child abuse
4-head trauma
5-infections

Recall 5-4-2018
Cerebral palsy
GP, 18 months old baby girl b/in by mom coz of not using her left side of hand & legs.
Task: take history including developmental history. (6 minutes)
Examiner will give u PE card.
Dx with reasons.
On history, no h/o injury, infection, does not use left side since 5 weeks of age, all milestones
delay, BINDs had a h/o resuscitation at birth, feeding problem + at first but
Okay now, concern about development.
On PE card, head circumference reduced, jerks increased in left side.

Recall 8-3-2018
Health Review ( Cerebral Palsy)
GP , a 12 month old baby girl was referred by the child health nurse as she isn’t moving her Left
arm and leg as compared to the right. Her parents are concerned.
Task: Hx, Pefe, Dx, D/D.

Positive Findings: only 1 of the milestone like rolling over or sitting was delayed. Everything
else was normal. BINDs normal.
PEFE(card) : weight was on the 50th percentile. Tone was increased and hyperreflexia on left
side.

Recall 5-5-2018
Mom concerns his 12 month old son has problem with the left side, arm and leg. Pregnancy and
labour was normal. The boy was born maturely and eventful. You will get pe card from the
examiner. Take history and explain the diagnosis with reasons. Im not quite sure as I explained
cerebral palsy.

Recall 13-7-2018 Cerebral palsy


Feedback 7-7-2017
mother of 12 month old baby comes to you complaing that the baby doeznt seem to be using his
right side
TASK
1. History
2. you will be given physical exami ation findings card
3. explain condition to the mother

-Hello examiner, hello Venessa may i know your baby's name...Adam


-since when did you observe Adam not using his right side
Vanessa: since the beginning but it is more obvious now since he started holding things and
walking as he falls to his right side so thats why i am concerned now
-asked about other milestones which were normal.
-birth history was also normal except mother had fever post partum but baby was fine had an
elder daughter..who seems to be perfectly fine
-has adequate support at home, Binds normal
-no history of trauma, hearing vision test normal, heel prick test also fine at birth

I will speak to my examiner now, examiner gave me the card


right limbs...hypertonia
left limb...normal tone

Vanessa...it seems that the tone in Adams right side is increased... What does tone mean doctor.
Me: aaaaaa...how do i explain tone....i moved my arm...this is normal tone....tone in Adams right
arm and leg is not normal
Vanessa...what do you mean doctor
Me: aaaaaa, aaaaa....vanessa im not able to explain what tone is
Vanessa laughed...alright doctor.
but it occurs in a condition called cerebral palsy...in which there is incordinatio between
impulses sent from brain to the muscles of the arm and legs
it could be due to multiple causes like infections, trauma but in most cases the cause is unknown
like in Adams case.
but it needs to be confirmed by the specialist who will do a detailed assessment and if
confirmed alot of help is available
bell rang
thank you examiner vanessa
PASS
keystep 1 2 3 yes 4no
approach 5
history 5
diagnosis 4
Feedback
18 month old child cant use her left arm and leg.
Task: -history.. -PEFE.. -Diagnosis… -MX
(cant use her left arm and leg since 5 month old. Forceps delivery, didn’t have breathing when
was born, was resuscitated. Cant sit, roll over, stand. Speaking-babbling. No family history. PEFE:
increased tone in left arm and leg, brisk reflexes in left arm and hand, growth chart-25) Cerebral
palsy
DDX for kid unable to use one side of body
1. cerebral palsy ( hemiplegic , spastic CP)
2. SOL in brain (tumor)
3. head injury
4. spinal injury
5. meningitis
6. brachial plexus injury
7. meningomyelocole (less likely)
8. hypothyroid (less likely)
but those are questions to be covered. and don't forget child neglect , abuse and family stress.
So I started by asking about pregnancy (normal)and delivery making emphasis if there was any
complication and she mentioned that the baby needed resusitation because he was not
breathing properly, hospitalizations during neonatal period, then inmunizations, nutrition,
sibilings and development. Development was deleyed in the motor aspect.. he was 18 moths
and barely crawling because he was dragging his left limbs. Finally asked about frequent
infections problems eating, family hx and any known pathology everything normal... asked
physical exam and it was given on a card what was positive was rigidity of left limbs. When
explaining the diagnosis i said more or less the following thing: i'm sorry to tell you this but i
think that your baby is having a condition called cerebral palsy, its a disorder of posture and
movement that probably happened because of the period during birth when he didn't receive
enough oxigen causing some brain damage... i want to assure tou that this is not your fault..
another thing is that if he is having this condition the good thing is that it won't get worse
because is a non progressing condition. However i cant confirm tje diagnosis right now because
it can only be diagnosed after the 2 years of age and I'll have to refere him to several specialist.
The neurologis to start investigating the condition, speech pathologist, perform a formal
audiology test because this condition can be related to eye and hearing problems. Ill call a social
worker for her to help you and your husband cope with the situation and also to evaluate the
house enviroment to make it more suitable for the baby..you don't have to go througj this alone
and i can give you the referal for a psychologist that will help you and your husband to cope
better and also to social groups... gave reading material and follow up visit
Feedback
Amotherof8months old boy cameforvaccination, missed the 6month
schedule vaccination because of her grandma died. Baby not sitting; arches when held.
Task: take history.
Do physical examination. And diagnosis.
Amc feedback: Cerebral palsy(Passed)

Thoughts before entering the room :The word “ arches “ is the key word in the question. Arching
in a baby – we have to rule out increased tone. Increased tone – possible CP. Entered the room
and found an young lady. Wished her. Introduced myself. How should I call you. What is the
name of the child. ( from now on, let us use that name when refering to the child, not “child “.
“What can I do foryou today. Forvaccination. Missed it because Grandma died. That’s allright.
But remember to do it without much delays as some of the vaccine preventable diseases are
coming back. I noted from the notes that he is not sitting. Lets start hx from birth hx. Maternal
infections, how the pregnancy & delivery went – ante natal check ups, smoking, alcohol,any
concerns ? Pre eclampsia, disbetes, diff. labour, resucitaion at birth, neonatal period. In this
scenario, baby was born preterm. Did not cry soon after birth. Resuscitation +. Was in Special
Care nursery. Development : Gross Motor Delay. Language good. Personal Social appropriate.
PE : Growth Chart : Microcephaly at birth. Continuing in same centile, Lower limb reflexes hyper.
Clonus : Nil. What happens to lower limbs when baby is held : Scissoring. Examiner repeated 2-3
times what else you need ? I kept thinking. He advised me to start counselingmother .
Retrospectively, he might have meant – Contracture oflower limb joints. I am not sure whether
contractures can appear as early as 8 months. Counselled the mother : Your child has acondition
called Spastic Diplegia, a type of Cerebral Palsy, but the most favourable among the different
types. Because his mentation is normal ( as indicated by normal language and personal social
dvpt ) . There are lots of things to be done at this stage which can really make a difference to the
future outcome. Physio, Occ. Therapy, Stimulation exercises. Mention support groups, regular
follow ups, lots of recent developments in the field. Now, who will care for him ? We have to
make a decision. Supports available. I am happy to guide you. I shall refer to Occ., Physio, Social
worker, Shall meet with family next week. Think about arrangements and working
schedules of both of you so as to put a system forcontinued care for the boy.

A referral to Paediatrician . I did not mention it. Missed it. Thought it would have been a critical
error and awaited the result. Passed. Still I think that would be a mistake not to mention it.

Examiner was very helpful. Roleplayer did not seem to have shaken hearing the diagnosis. May
be it was towards the end of exam and she was tired of shaking to every candidate.
Feedback 5-12-2018
You are intern at GP, Mom of a 12 month old child came as he is not using one of his side.
1. Task take hx
2. PEFE on CARD (Head circumference 50%, Hypertonia and hyper reflexes)
3. Tell Dx and counsel
Entered the room Thanked examiner. Introduction and greeting. I understand X you have come
to see me regarding Peter’s movement, I can see that you are bit concerned. Let me assure you
that I will try my best to help you and Peter. Can you pls tell me bit more about it? (She told she
is concerned as he is not using one side since he was born.) Then I Took detailed developmental
Gross motor-When he started elevating his head? (3 months) Sit with support? (Can do 5
months), sit without support (cant as he tends to fall on one side), Crawl? While doing it use to
drag one side and started 8/9 months, He tries to stand up by holding things but sometimes falls.
Any scissoring of legs? (says I don’t understand, so didn’t ask further)
Fine motor- could reach and grasp things with normal hand. Pincer grasp normal, can transfer
object from one hand to other but always use one dominant side to pass.
Social- all good
Language- Interact, says BA BA da da.
Sick Baby Q- feeding (well), pee, poo (constipation –hypothyroid)
Birth Hx- During preg- Infection, complication?
During labor- difficult labor, prolonged, instrument used?, vaginal/cs?
Post-delivery- any resuscitation, Jaundice, late cry?, When you were discharged( next day)? Heel
prick test done or not? (not sure)
D/D Q- Hoarse cry? Excessive cry? Any lump around the belly button- r/o hypothyroid
Any trauma/ fall- r/o spinal injury
Fx of metabolic problem? Same problem any other members? (no)
Are you happy family? Any stress at home? Financial problem? (no) –r/o abuse
Any shakiness in the body- r/o Epilepsy (no)
Immunization? (Up-to-date) any medication? (no) any medical help so far? No
On PEFE: All typical finding of CP- hypertonia and hyper-reflexes on one side, all other seemed
normal.
Thanked examiner turned to Mom and said look Jenny from the story and after getting the
findings from examination I have found that John is not utilizing that side and its bit stiff as well
now there can be various reasons but most probably its due a condition called CP, which is
disorder of movement and posture, drew a picture of brain, Its permanent damage to immature
brain Im afraid, and bit concerned as well, but let me assure you that whatever it is we are here
with you. At this moment she said. Are you sure Doc? I said from your hx and findings it seems
most probable but it could be due to other conditions like Hypothyroidism, trauma, epilepsy,
metabolic problem, muscle problem but I couldn’t find anything from ur story but to be on safer
side will run some investigations. Moreover if we diagnose him as such then There are lot of help
available and MDT will take acre of him composed of child specialist, GP , Speech therapist,
physiotherapist, so on and so forth.
Key Steps ( 1, 2, 3, 4)- Yes, no, yes, yes Global score- 4

74-Spontaneous pneumothorax
Case 22-6-2018
15yo Patient brought in by ambulance from school to ED with complaint of chest pain and
shortness of breath. Your college already did primary survey, took x-ray and put him on O2.
Task
-brief history
-PE from examiner, he will only give you what you ask.
-Ask for x-ray and explain in to your patient
-dg with pt.

Case (9-3-2018) (10-2-2018)


Pneumothorax, sudden onset of dyspnoea and left upper chest pain. x ray available upon
request.
Task
-history,
-PE from examiner
-explain differential diagnoses to patient.

History
1-ask hemodynamic stability (vitals and O2 sat.)

2-pain questions
-severity+ painkillers and allergy
-duration, sudden or gradual, constant or come and go, getting worse
-site
-radiation (tip of shoulder)
-character (sharp, stabbing)
-aggravating (deep breathing)
-relieving (None)
-1st episode

3-Shortness of breath questions


-sudden or gradual?
-on rest or exertion or both?

4-What were you doing when it started?

5-Associated symptoms
-cough, fever? (Pneumonia)
-Palpitations
-sweating

6-General questions
-past medical history (underlying lung or heart disease) (secondary pneumothorax)
-PSH
-trauma
-travel history, clotting problems (pulmonary embolism)
-SADMA (especially smoking)
-occupation (athlete)
-family history

Physical examination findings from the examiner


1-General appearance
-cyanosis, dyspnea, Oedema
-pallor, LAP
-BMI
2-Vital signs
3-chest examination
inspection: chest movement with respiration
palpation: tracheal position, chest expansion, apex beat
percussion: dullness (hyper resonance)
auscultation: air entry, breathing sounds, wheeze or crackles, vocal resonance (decreased breath
sound)
4-CVS
-heart sound and murmurs.
-JVP
5-quick abdomen
6-office tests
-UDT, BSL
7-Any X-ray available

Differential diagnosis
1-Spontaneous pneumothorax
2-pneumonia
3-pulmonary embolism
4-pericarditis
5-ACS
6-trauma
7-anxiety
Explain X-ray
-Explain x-ray views and markings
-explain lung shadows
-then tell about air surrounding the left lung
-tell about lung collapse or diminish in size
-tracheal position
-normal heart, normal right lung
-no need to mention percentages

Explain diagnosis and differential diagnosis.


-from history and examination, most likely you have a condition we call spontaneous
pneumothorax, where there is a rupture in one of air filled sacs on the surface of the lung called
blebs leading to escape of air from lung to the space surrounding the lung causing pressure or
compression on lung and causing pain and SOB.
-the cause is unknown and it is usually occurs in healthy young persons.
-let me assure you that…
-other possibilities like above but less likely from history, examination and x-ray findings.

Management
-admit for observation
-doing serial x-ray to monitor if there is any increase in the air escaping or lung collapse.
-seen by registrar to decide upon management if need chest drainage or not.
-once discharge:
avoid smoking
avoid flying or dying
red flags
review
Feedback 22-6-2018
15yo Pt brought in by ambulance from school to ED with complaint of shortness of breath.
Your college already did primary survey, took xray and put him on O2.
Task
brief hx,
PE from examiner, he will only give you what you ask.
Ask for xray and explain in to your pt,
dg with pt.
Approach
Greeted patient ( seated prone with oxygen mask)
Said sorry you are feeling this way.
Are you in any pain.. would you like some pain killers he said already given
Then I asked before I proceed with the task I would like to check your vitals
Asked examiner said as given in the stem.. proceed with task
Hx
Asked SIQORRA
DD qs for
Trauma, Pneumonia, DVT, Tuberculosis
Asthma, Pericarditis ( with siqorra)
Anaemia ( on PE only)
Arrythmia, GORD, Pancreatitis, Hepatitis (on PE only), Inorganic causes

PEFE
General Appearance, Palor, Cyanosis, Icterus, Edema, Lymphadenopathy
Vitals any change
Growth chart tall for his age and lean
Respiratory system in details ( left side reduced movement, trachea central but apex beat
shifted, fremitus reduced, percussion hyper-resonant, reduced breath sound.. normal rt
side)
CVS only s1, s2 added sound or murmur
Office test- ecg, blood sugar

Xray—Left sided pneumothorax


Explained to pt.. did not mention about amount as it was the patient

Diagnosis-
Drew picture and mentioned most likely as pneumothorax
But gave all the dd’s by drawing and in layman term
Bell rang
Case (30/3/2017)
Acute SOB. Since 3 hours
Tasks.
History. Was jogging. Felt SOB. Nothing else positive.
PEFE. Auscultation reduced breath sound left lower zone. Percussion not done.
Investigation. Cxr given poor quality.
I initially mis judged xrays. Said pleural effusion. But I was confused. said sorry
my apologies. It is pneumothorax. Need to admit and chest drain. Could not judge the response
of examiner.

Case (28/4/2017)
15yr old SOB left side chest pain. Task history, PEFE, X-ray explanation , Dx to patient . Got
history of something pop in chest, sudden onset, parents coming now, X ray mod pneumothorax.

Case (28/4/2017)
chest pain.
16 years in Ed with sob n chest pain. now stable with oxygen.
Explain X-ray and diagnosis,dd to patient.
on history healthy athlete, no problem with lungs in past,no previous chest pain or Heart
problem,no smoking.
on X-ray pneumothorax.I explained X-ray,diagnosis and dd like pneumonia,pulm
embolism,pleurisy,pneumonia,acs,pericarditis,anxiety etc as task was dd.
Passed

Handbook Condition 062


Sudden onset of chest pain and breathlessness in a 20-year-old woman

CANDIDATE INFORMATION AND TASKS

This young woman has presented to the Emergency Department of the local hospital with the
sudden onset of right sided chest pain and breathlessness while walking to work. She is
otherwise in good health and is a nonsmoker. Physical examination of the chest showed no
definite abnormality. Her breathlessness is less now. A chest X-ray has been taken, and is
illustrated below.

YOUR TASKS ARE TO:

• Examine and interpret the patient's chest X-ray.


• Explain to the patient the diagnosis and how she should be treated.

There is NO need to take any further history from the patient NOR repeat the physical
examination.

AIMS OF STATION

To assess the candidate's ability to recognise the right-sided pneumothorax on the chest X-ray
and explain the diagnosis to the patient. The candidate needs to reassure the patient and then
explain how the problem will be managed.

EXAMINER INSTRUCTIONS

The examiner will have instructed the patient as follows:

You developed chest pain while walking to work. A chest X-ray has just been done at the local
hospital and you are about to be informed of the result of this.

Opening statement

• 'I've got a bit of a pain in my chest and I feel a bit breathless' — indicate site of pain which is on
the right below the clavicle and at the back in the same area.

You were walking to work when the pain came on suddenly. You were also breathless. The pain
is:
• sharp and stabbing (if asked it is not tight, heavy or gripping)
• not made worse with every breath

• Worse if you take a deep breath.

• Also radiating to your shoulder tip; and

• Moderately severe at onset, but easing now you also have:

• A feeling of breathlessness (not severe) at rest as well as on exertion.

• An irritating dry cough - not severe or distressing

You are a nonsmoker and drink alcohol on social occasions only (2-4 standard drinks).
You have no known drug sensitivities. You live 2 km from the hospital and there are several
others at home most of the time.

Indicate area where pain is felt, upper chest, both back and front. Also indicate that you are
concerned about the cause of the pain and what the doctor will do to relieve it. Be cooperative
and answer the doctor's guestions without evasiveness

You have a moderate-sized pneumothorax, a partial collapse of the lung.


The candidate must make the correct diagnosis, explain it to you and how the problem will be
managed inserting a catheter to take the free air out of the chest is a possible response.
Admission to hospital may be recommended. If so, ask if you could be treated at home.
EXPECTATIONS OF CANDIDATE PERFORMANCE Candidate
should respond along the following lines:

Response to patient
(Task 2 management)
•Pneumothorax is the diagnosis, confirmed by chest X-ray. A pneumothorax of this size may
not need active treatment. It would be prudent and reasonable to admit the patient overnight
for observation and serial X-ray. Sending the patient home if she lives nearby is a less
acceptable option with a pneumothorax of this size. Recurrence is possible after spontaneous
pneumothorax, and the recurrence rate is approximately 35% on the same side and 10-15% on
opposite side. Most recurrences occur within 12 months.

• The general consensus regarding the need for intercostal drainage is as follows:

~ If 25-30% or less lung collapse and no symptoms — observe. This is a reasonable option in

this patient.

~ If 25-30% or less lung collapse and persisting symptoms — drain. The patient may fall into

this category with observation. The pneumothorax is around 30% and her symptoms are

currently mild.

~ If greater than 30% collapse, whether symptomatic or not — drain.

Displaying clinical knowledge and skills

• Aetiology of pneumothorax — rupture of bleb on surface of lung.

• Nature of pain associated with pneumothorax — possibly due to tear of adhesion as lung

collapses.

• Associated breathlessness — depends on size of pneumothorax.

• Confirmatory investigation — chest X-ray diagnostic. This is a moderate (25-30%)

pneumothorax. It may need a formal chest drain. Inserting a catheter with a Heimlich valve is an
option to be discussed should the pneumothorax increase in size.

Demonstrating Communication skills

• Reassuring approach to patient anxious about the cause of the pain.

• The pain should be recognised as being of respiratory origin, rather than cardiac.
KEY ISSUES

• Correct interpretation of chest X-ray.

• Explaining the diagnosis and appropriate management to the patient.


CRITICAL ERROR
• Failure to identify the pneumothorax on the chest X-ray.
COMMENTARY

Spontaneous pneumothorax is usually due to the rupture of a previously nonsymptomatic bleb


on the pleural surface of the lung. Symptoms of breathlessness and local discomfort are
proportional to the size of the pneumothorax which is often small, in which case no active
interventional treatment is required. Elective intercostal catheter drainage is indicated for a large
(> 30% chest volume) initial pneumothorax or progressive increase in size on serial X-rays.

The common smaller size pneumothoraces are often difficult to identify on plain X-ray, even with
erect films and magnified views. Larger pneumothoraces, as illustrated, are usually easy to
identify.\

75-Threatened Miscarriage
You are at your GP, when 27 year old Susan presents with bleeding from the vagina since the
past 1 hour. She has done a home pregnancy test which has turned out to be positive (2 days
ago)

TASKS
1.Take a further history
2.Get the examination findings from the examiner
3.Discuss your diagnosis and further management with the patient

Differential Diagnosis:
1-Miscarriage (Threatened, incomplete)
2-Ectopic pregnancy
3-H-mole
4-Trauma
5-Infections
6-Bleeding disorders/ Blood thinning agents
7-Cervical polyps

APPROACH
History:
1-Is the patient hemodynamically stable?

2-Bleeding questions (Duration-Action-Trauma-Amount or severity-colour-odour-content-


dizziness- bleeding disorder or blood thinner)
- for How long have you been bleeding? Is this the first time?
- What were you doing the time you passed blood?
- have you had any trauma to your tummy? (Trauma)
- How many pads have you used so far? Is it (are they) fully soaked?
- What is the colour of the bleed?
- is it smelly? (infection)
- Are there any clots or tissues? Any vesicles or grapes? (Molar, Incomplete miscarriage)
- do you feel dizzy or tired? (Effect of bleeding)
-have you had any bleeding disorders or take any blood thinner medications?

2-Associated symptoms (tummy pain, fever, N&V, discharge)


- do you have any pain in your tummy? (Mild pain)
ask only severity + pain killer, site and radiation.

- Any fever? Nausea or vomiting? Abnormal vaginal discharge?

3-5 P’s questions


Period questions: (only ask LMP and regularity)
-When was your last menstrual period?
-were they regular?

Partner or sexual (only support and STI)


-do you have good support?
-have you or your partner ever diagnosed with STI?

Pregnancy (very important)


- are you trying to become pregnant?
- have you had any previous pregnancies or miscarriages?
- do you have any vomiting, breast tenderness?

Pill
how long have you been off the contraception?
Pap or Hpv
is your pap or Hpv up to date?

4-Early Pregnancy questions


-Any antenatal checks you'd done so far?
- Have you taken your folic acid?
-are you aware of your blood group?

5-General questions (Diet-SAD, OTC- pets, PMH-PSH-Family hx)


- Diet: any intake of raw meat? (predisposed to toxoplasma), how many coffee do you take in a
day?
- Do you smoke, drink alcohol or take recreational drugs?
-Any prescription or over the counter medications?
- Any pets at home? (toxoplasma in cat litter)
- Any other medical or surgical illness?
-Family history of miscarriages?

Physical Exam from examiner


1-General appearance:
pallor, dehydration, LAP (PDL) , Bruises or bleeding
2-Vital signs
Blood pressure (+ postural hypotension) , Tachycardia, Temperature
3-quick Systemic exam
4-focus on Abdomen
Inspection (distension, Mass)
palpation (tenderness, mass)
Bowels sound
5-Pelvic exam
consent of patient and presence of chaperone:
Inspection: colour of the bleed? Tissues, clots?
Speculum: look if bleeding is coming from cervix , cervix closed or open? (Key)
Per vaginal: CMT, uterine size, position, tenderness adnexal mass and tenderness (key)

6-Office test: urine pregnancy test, UDT, BSL

Explanation
-Most likely what you're having is a threatened miscarriage. It is a bleeding from the vagina
before 20 weeks of pregnancy. More than half of the women stop bleeding and they continue to
have a normal and healthy pregnancy.

-Exact cause is unknown but there are certain risk factors associated with this like smoking,
alcohol and recreational drugs, excessive coffee intake, infections, trauma, could be due to
problems in the placenta feeding the growing baby, or could be due to genetic abnormalities in
the baby.
-From the details that you have given me, I have not found any risk factors in you, so there is
nothing that you have done that has caused this miscarriage, and there is no way by which we
can predict a miscarriage.

Management
You need to be referred to the hospital now and seen by the specialist.

Blood investigations needs to be done such as:


-FBE, UEC, ESR/CRP, blood group and Rh typing, coagulation profile, vitamin D, antibodies against
rubella and varicella, STI screen, and also a TORCH screen.
-Urine needs to be given for microscopy and sensitivity.
-Ultrasound will also be done to check if the pregnancy is viable or not and also to rule out other
causes of bleed.

As you are not bleeding heavily, and the opening of your birth canal is closed, if the ultrasound
shows a normal viable pregnancy, then the specialist might advise you to return home.

Once you are at home, you need to:


-avoid overexerting yourself. No activities like no sports, lifting heavy weights.
-Rest is not usually advisable, because rest will not prevent the miscarriage from progressing.
-Do not insert tampons into the vagina for the bleed, but you should use pads for the bleed.
-No sexual intercourse until the symptoms have gone completely for 1 week.

Red flag: Seek urgent medical advice in the emergency department if the bleeding becomes
heavy, any passage of tissues to the bleed, and if the cramping worsens, or if you develop fever.

Follow up: A repeat ultrasound needs to be done after 1 week. As the pregnancy progresses, you
need to come for regular antenatal checks and do a Down syndrome screening.

I will start you on folic acid which you need to take for the first 3 months of pregnancy. (Key)

Refer to high-risk pregnancy clinic, Another ultrasound will be done at 18 weeks, sweet drink
test at 28 weeks, a repeat ultrasound at 32 weeks if needs, and a bug test at 36 weeks.

Maintain a healthy diet, do not eat any raw meat, no smoking, alcohol or recreational drugs and
limit coffee to 2 cups per day.

I will arrange a review with you once you are out of the hospital.

***if woman is Rh negative, a threatened miscarriage up to 12 weeks, there is no need to give


anti-D, unless patient goes in for a complete miscarriage.

But if it is any other kind of miscarriage, you need to give the anti-D.

Feedback Case (29/11/2017)


Young lady (24-27years) with vaginal bleeding.
Tasks
-Take history
-Ask for Physical examination findings.
-Explain reasons and arrange investigations.
Positive findings on history
-Bleeding is bright red,
- 2-3 pads per day. - First time happening.
- LMP was 6 weeks back. - Periods are regular.
- Had been trying to get pregnant. - Contraception history was negative.
- Not bleeding from anywhere else. - Pregnancy s/s – nausea, breast tenderness was positive.
Positive findings On examination
- no pallor, Retroverted uterus, os closed, no s/s of bleeding).
Key steps: 5/5
History: 6
Choice, organization, sequence of examination: 6
Choice of investigations: 4
Global score : 4/5- pass
Went inside, Greeted examiner and patient
haemodynamic stability?
Any chance Pg?
Bleeding ques
Bleeding from anywhere else (bleeding disorder)
Passage of grape (molar)
Passage of tissue, still feel n, v, breast tenderness (incomplete/complete)
Anything started it? (trauma, sex..)
Anemia ques
5ps
Blood group
Folic acid
SADMA

PE- General appearance, vitals, Anaemia, postural drop


Abdominal examination, pelvic examination,other systems
Office test- UPT, blood sugar, urine dipstick
Investigations -USG- to see fetal condition , FBE, Blood group, U&E
Explained dds to the patient
1. delayed period 2. Trauma 3. Threatened 4. Incomplete 5. Complete 6. Ectopic 7. Molar
8. bleeding disorder

Feedback 16-8-2018
Station 7. Threaten abortion
GP, 24 years old lady came to see you for bleeding per vagina.
Task: Take history
Ask PEFE
Give diagnosis
Differential diagnosis

Pt has no eye contact.
Reassure. Check stability: stable.
Bleeding per vagina: start morning. 3 pads soaked with blood, no vesicle, tissue and clot. No
dizziness, SOB. No abd pain.
LMP: 8 wks ago. Regular.
Stable partner. No history of STI.
Contraception: Use condom & OCP. OCP stop 12 weeks ago. No pills problem. I asked trying to be
pregnant (she didn't answer exactly)
Pregnancy: breast tenderness (+), no history of pregnancy & miscarriage. Her bld gp is O+.
Pap smear: forget when & result. I offered I will do after this section.
General health: no history of bld disease. Not taking bld thinning medication. No symptoms of
hypothyroid, SLE, Cancer and injury.
PEFE: All normal except VE: I: dark bleeding, Os close, No adnexae and cervical motion
tenderness, ut size is 6-8 wks size & no tenderness. No lymph node. Normal BMI.
Office urine dipstick: normal
Urine pregnancy test: positive

I said she has pregnancy. Draw a picture. Now bleeding fr downbelow: Threatening miscarriage. I
said other differential diagnosis.

Grade: Pass
  Global score: 4
Key Steps: 1- Yes
2- Yes
3- No
  4- Yes
Approach to patient: 4
History: 3
Choice & Technique of examination, organization and sequence: 5
Diagnosis/ Differential diagnosis: 4

Feedback 7-12-2018

30 yrs old with complain of vaginal bleeding since morning.

task :history, pefe from examiner, investigation, diagnosis with reasons.

entered greeted both of them, asked vitals-stable.

history - bleeding started- yesterday, slight, red in color , no other discharge. no vesicles, no
pain, no trauma, no itching, no ulcers , not dizzy, no SOB, 5 Ps - LMP-8 weeks back ,pregnancy
test positive, primi, did not use any contraceptive before.
no STD, PArtner- supportive,

family history-no history of special babies, twins, molar .

past history- no HTN, DM, bleeding disorders, abortions

SADMA-nil

pefe - all normal

pelvic examination- inspection-slight bleeding, no signs of trauma,

per speculum- os closed, vagina healthy

bimanual- uterus normal in size (was confused as not enlarged)

no adnexal tendenness, no CMT.

BSL,UDT-normal.

UPT+ve

i asked ultrasound -examiner said fetal sac present along with heart beat.
explained it could be threatened miscar, (reassured it is harmless, fetal heart beat seen , which is
good sign )there could be many reasons like trauma, intercourse, problem with baby, problem
with placenta - asked her can you relate to any of these - then she noded could be intercourse
related. i explained other dd names but less unlikely as ultrasound confirmed diagnosis.
scenario-vaginal bleeding,

GLOBAL-4( approach-4, history -5, examination -5,investiagation-4, d/dd-4)

Feedback 7-12-2018
Vaginal bleeding - 4
26 year old female with painless vaginal bleeding after 8 week of amenorrhoea. The stem is a bit
long.

Task: History

PEFE
Ask investigation

Dx

2 min thinking: incomplete/complete/threatened miscarriage; h.mole; ectopic; bleeding


disorders; if tissues were found  to skip UCG test and USG

History

Patient looked impatient. Greeted the patient. Checked stability.

(+)ve findings – LMP 8 weeks ago; regular period; no nausea and vomiting. Using contraception
ocasionally but no plans to get pregnant. No vesicles in the blood; no pain. GH – good; no
bleeding disorders; no thyroid problems; blood grp – O(+)

PEFE

GA – as you can see

Vitals – normal

Abdominal examination – nth significant

Vaginal examination – on inspection, no active bleeding; SSE shows os is closed; no tissues or


clots around the os; on BME, uterus is 6 week pregnancy size, no tenderness; no adnexa swelling
or pain

Investigation

FBE (you need to be more specific in what you are searching cuz the examiner remained silent
until I specifically said what I wanted to know) – Hb – 11%; platelet count – normal. When I
asked grouping and matching, he didn’t seem to understand me. So I repeated and he still didn’t
get me. After saying grouping and matching for 3 times, I finally told I would like to check blood
group of my patient – O(+) ve. Then I asked UCG – the examiner said “do you mean serum beta
hCG?” Yes. It was around 80,000 while the serum beta hCG at 6 week of pregnancy should be
around 60,000. (I had no idea what it means, maybe H.mole?) USG – baby of 6 week size;
intrauterine gestation; FHR – 80/min

Dx

I explained my patient what threatened miscarriage is. Things got awkward when I tried to
reassure my patient that this is not dangerous because suddenly, I remembered that it is not
intended pregnancy. The conversation came to abrupt halt, with plenty of time left. After a
while, I asked my patient if she wanted me to inform about this to her partner. “NO. So, doc am I
having a miscarriage?” No, about 90-95% of woman with threatened miscarriage go on to have a
successful pregnancy and a healthy baby. This is a fairly common condition in pregnant ladies in
their early time of pregnancy. To my rescue, the bell rang.

GS – 4

Key steps 1 and 4 – No; 2 and 3 – Yes

Approach – 4

History – 4

Choice and technique of examination, organisation and sequence – 5

Choice of investigation – 6

Diagnosis and DDx – 4

76-Bacterial vaginosis
AMC case 21-6-2018
A young lady come to GP with grey, foul smelling vaginal discharge for 3 months or so. Had
doxycycline and antifungal but didn't work.
Tasks
- history
- PEFE
- diagnosis
- management

History
1- discharge questions
-how long have you had this discharge form vagina?
-how many pads have you used? (Profuse)
-what is the colour? (Grey)
-is it smelly? (Yes, fishy)
-what is the consistency? (Thin) is it bubbly? (Yes)
-any blood stained?

2- Associated symptoms.
-any itchiness or soreness? (yes a bit)
-any fever, tummy pain, vaginal bleeding? (No)
-any burning or stinging on passing urine?
-how is your bowel habits?
-any ulcers, rash?

4-5Ps questions
periods
-when was your LMP? are they regular?
Partner or sexual
-are you sexually active? Are you in a stable relationship? (stable relationship)
-have you or your partner ever been diagnosed with STI?
-any pain during intercourse?
-any similar symptoms in the partner?
-do you practise safe sex? (Yes)
Pill
-what type of contraception do you use? for how long?
Pregnancy
-any previous pregnancies? Any miscarriages?
Pap or HPV

5-general questions
-past medical and surgical history
-SAD
-diabetes
-do you feel thirst? are you passing large amount of urine? Do you need to go to toilet more
frequently? Any history of diabetes?
Immunosuppressive disease
-any loss of weight? Loss of appetite? Lumps or bumps around the body?
Antibiotics or steroids
-do you take any antibiotics or steroids medications
vaginal douches/ pessaries/ shower gels change
- Any vaginal pessaries or douches that you have used?
-have you changed your shower gels?
Tight clothes
Do you use tight clothing or tight jeans, panty hose

Physical Exam from examiner


1-General appearance:
BMI, pallor, lymph nodes, rash, evidence of infection anywhere in the body
2-Vital signs: temperature, BP
3-CVS/Respi
4-Abdomen:
inspection: visible distention, mass,
palpation: palpable mass and tenderness
5-focus Pelvic examination
Inspection of the vulva and vagina:
-discharge, color of the discharge, is it smelly. (no discharge visible)
-any swelling or erythema of the vulva or vagina (mild erythema)
Speculum:
cervix healthy or not, discharge or bleeding, erythema of the cervix (mild to moderate greyish
smelly discharge)
Per vaginal: DON'T to avoid spreading the infection
6-Office test: urine dipstick and blood sugar level (both imp), UPT

Explanation
-you most likely have a condition called bacterial vaginosis. It is caused by imbalance of the
bacteria normally present in your vagina and this happens when the normal healthy bacteria is
suppressed or replaced by an overgrowth of other unhealthy mixed bacteria.

-the exact cause is unclear but could be sexually transmitted and this is one of the most common
causes of abnormal vaginal discharge in women.

-It usually produces a watery, white or gray discharge, and has a strong unusual fishy smell.

Investigations
-In order to confirm this, I need to take a high vaginal swab which is given for microscopy and
gram stain, and it will show clue cells, which is a normal vaginal epithelial cell with bacteria
attached all around.
-Another test is an amine whiff test where 10% potassium hydroxide is added, and it will give a
pungent fishy smell.
-Another one is to estimate the pH of the vaginal fluid and usually the pH will be greater than 4.5
if it is bacterial vaginosis.
-I would also like to take some blood tests (FBC, UCE, LFT) and urine for MCS
-STI screen with your consent (key)

Treatment
-Even after treatment, in about half of the women, it can sometimes recur in the next 6-12
months.
-Management of this is by prescription of antibiotics, metronidazole wither orally for 7 days or as
a gel intravaginally for 5 days. * if pregnant clindamycin
-Avoid vaginal douching because that can also alter the bacteria in your vagina. Follow good
genital hygiene.
-You partner does not require any treatment as of the moment, but always practice safe sex.
-I will give you reading materials regarding bacterial vaginosis.

Feedback 21-6-2018
Young to middle-aged woman with profuse greyish smelly discharge unresponsive to antifungal
and doxycycline for 3months. Take history. Ask for examination findings from examiner. also
explain your management. [ stable relationship and safe sex—bacterial vaginosis including asking
for management] Vaginal Discharge: Pass: Global Score: 4
-I asked about characteristics of discharge and possible burning sensation or itching down below.
Also asked about previous medications. Asked about previous medical conditions such as
diabetes. Also asked about hormonal contraception and her relationship (whether she practiced
safe sex. And history of possible previous STIs. She did not have any risk factor for any sexually
transmitted infection and no risk factor for developing candida infection.
-In examination findings from the examiner, I asked for General appearance, (she looked
concerned) vital signs especially temperature Normal. Lung and hearts equal air entry and
normal heart sounds, abdominal examination no distension and no tenderness in palpation. In
genital examination in the presence of a chaperone and with the consent of the patient, no
obvious discharge from outside, in speculum examination, mild to moderate greyish smelly
discharge. I do not remember the presence of the inflammation in vagina, but I guess cervix was
healthy. I took a high vaginal swab sample for gram staining and microscopy and did not proceed
to bimanual examination. So I put to her that her symptoms looked like a bacterial vaginosis. So I
explained that as she has not had any improvement with previous medications, I suggested
waiting for the results of swab sample and in the meanwhile, I suggested improvement in the
genital hygiene, wearing cotton underwear, changing underwear often, not using any pessaries
or vaginal douches, and after the results were available we discuss further management which
would be a suitable antibiotic or antifungal medication.

77-Psychogenic cough
A mother has come to your GP clinic concerned about her 12 year old daughter. She has had
cough for 6 months on a puffer and had antibiotics.
Tasks
A) History
B) PEFE
C) Order any investigations
D) DX and DDS

History
1-Cough questions
onset
-when did it start? Is it constant or come and go? Is it getting worse?
Character
-can you describe it for me? Is it loud, harsh, barking, honking?
-does she cough up anything (is it dry or wet)?
Aggravating/relieving
-does anything make it better or worse?
Timing
-does it occur at day or night mostly or both?
-is there any specific time when she cough? (in one case they said she is coughing mostly when
surrounding by people)
severity
-does it affect her sleep or wake her up at night?
1st episode
-is this the first time?
-has she been diagnosed with asthma before?
-has she been hospitalized before?

2-Associated symptoms questions (asthma, pneumonia, allergic rhinitis, sinusitis)


-any chest pain or shortness of breath?
-any fever or rash?
-any noisy sound in her breathing?
-any headache or bad breath?
-any past history of allergies? Is it associated with sneezing?

3-Trigger or risk factors questions


-any recent flu infection?
-anyone in family or school has similar symptoms?
-any pets, carpets, or soft toys?
-does anyone smoke at home?
-does she take any medications?
-any family history of asthma, hay fever, eczema?

4-psychogenic questions
-any stressors at home? Any recent changes at home? (In one case husband divorced 6 months
ago)
-how is her school performance? Any stressors at school or bullying?
5-BINDS
especially Immunisation and development

Physical examination from examiner


1-General appearance
-alert, irritable, restless, drowsy
-pallor, cyanosis
-respiratory distress signs
*nasal flaring
*accessory muscle use
*subcostal or intercostal recessions
-DR PJL

2-Vital signs especially RR + O2 saturation

3-Growth chart

4-ENT (sore throat, runny nose, ear discharge)

5-chest examination
inspection: deformity, chest movement
palpation: chest expansion, tracheal position
percussion: dullness
auscultation: air entry, breathing sounds, wheeze or crackles

6-CVS, abdomen

Investigations
1-Blood tests (FBE)
2-chest X-ray
3-Lung function test

Differential diagnosis for chronic cough


1-habit cough (psychogenic cough)
2-asthma
3-post viral
4-allergic rhinitis
5-chronic sinusitis
6-GERD
7-medications
8-pertusis

Explanation
-well, from history and examination. Let me assure you that your child is physically healthy. I
could not find anything serious.
-she most likely have a condition called habit cough or psychogenic cough have you heard about
it?
-this is a repetitive cough that usually occurs in the absence of any underlying disease. It is
common in this age group and not serious. It can occur many times in a minute as cough can
irritate the airways causing more coughing.
-it is often related to stress or anxiety. Body and mind are connected together so whenever the
mind is stressful the body can express this by causing symptoms like cough in her case. children
are usually not very good at expressing their emotions.
* in case of public cough add (In her case, it worsens when other people are around.)
* In case of parents divorced add ( it seems from the history that your child has been through a
lot since the divorce, she must feel confused as there has been a change in her life)

-let me assure you that this will go away by itself.

Management
-it is important that we try to identify the situation that trigger the cough and isolate the stress.
-praise and encourage her when the cough episodes diminish.
-refer you to a pediatrician as she has cough more than 3 months for review.
-if it does not work then I can arrange a referral to psychologist which will do talk terapy and
exposure-response therapy.
-it would also be a good idea to see your daughter and arrange family meeting.
-reading materials
-review
-red flags (failure to thrive, feeding difficulties, wheeze, barking cough, SOB, hemoptysis)

Case (31/3/2017)
A mother has come to your GP clinic concerned about her 12 year old daughter. She has had
cough for 6 months on a puffer and had antibiotics.
Tasks
A) History
B) PEFE
C) Order any investigations
D) DX and DDS
On hx there was no exposure to dust pollen smokers, non productive cough,effecting her school
performance and many days absent from school. Np family hx of asthma or travel hx. At the end
asked about stressors. Parents separating
PEFE was normal
I ordered FBC and CXR they were normal too
DDS psychogenic cough Asthma

Case (21/4/2017)
A 12y child presented with long time cough. She was not responsive to asthma medication and
all investigation for asthma were negative....
Task:
1. Take history( all normal except the cough was when he was talking in front of others)
2. Tel the mother the likely cause of the cough
*12 yr old with cough
Hx exclude organic causes, seems she coughs on special occasions when she is under stress
Dx psychogenic cough.
Reassure tge mother Psych referral Feed back cough: Pass

Case (2/6/2017)
Cough for 6 months – 12 years old girl – Honking cough
Treated for Asthma, In History - Parents divorced 6 months ago. PEFE + Investigations
Case (24/6/2017)
Chronic cough for 6 month in a (about 8 yr old ) young girl.
Tried inhalers and antibiotics with no effect.
Couldn't figure out what. Pe and investigation all normal.
Other candidates said psycho genic. Husband just moved out

Case (25/7/2017)
Psychogenic cough-12 yr old , has been treated for asthma with no relief- history and diagnosis
with reason (don't give DD or management)

Case (21/9/2017)
12 yr girl with cough for 6 months. Not responding to ABs and asthma medication
Hx, Ex, Ix, Explain dx to mother

comment
The stem was like
12 year old girl came to your GP with coughing for a long time. Asthma treatment was given
before and it wasn't relieved.
Task :
further history from mom
Explain the dx to mom
I asked about all the possible causes like infections, pschosocial and etc. But when I asked is
there any specific time the girl cough, the mother said she coughs when she is surrounded by
people. So I pressed on that point and asked more to make sure that it is truely psychogenic and
there is no other psychological impairment. And I explained about that to the mother. When i
offered treatment like CBT, the examiner interrupted that no treatment in the task. So i finished
the case early and I passed.
Sometimes when we are stressed, our body express it in other way such as pain. So this is a
same situation. The crowd make her stressful and this is expressed as cough. And reassured.

Recall of 16-3-2018
recall of 11-7-2018 Recurrent cough in a 12 yr old(mother said barky cough, everything was
normal-psychogenic)

https://www.rch.org.au/clinicalguide/guideline_index/Cough/

Feedback 7-12-2018
14 yrs old presenting with persisting cough on & off,
TASK - further history from mother, pefe and relavent investigation from examiner , dd.dx to
mother .
2 min thinking - looks like psychogenic , but need to exclude others , investigation which can
provide clue.
HISTORY- greeted both of them , mother was very coopertaive , i reassured her saying that, it
might be difficult to see child coughing but we will help. cough - SOCRATES since 6 months, on
& off , Honking type , no sputum , no chest pain , no aggravating and relieving factors. no allergy,
no asthma not post viral

BIND- normal, school performance - normal, not affected because of coughing, sibling -one ,
5yrs, family history -no one else coughing no asthma , no known allergies , no smoking (told her
just asked because some times this might also be cause).
(reassure her in between that im just trying to exclude important cause of cough) no
medications.
social- some stress in family (forgot actually - father left or mother working long )( asked her
would you relate cough to this stress -as both started same time - she said i can't say ).

PEFE GA, VITALS, PICKLE


growth chart- normal
chest -equal air entry both sides heart - normal
abdomen- soft
UDT&BSL- normal

INVESTIGATION - examiner asked what you want i asked CXR-normal,


inflammartory makers -normal,blood test-normal,
swab for pertussis - normal , Peak Expiratory FR- normal.

explained there could be many reasons , we have some tests to exclude most of them ,but could
not find clue- like whooping cough , asthma, allergic , and other , but after your history just
thinking can we relate stress to cough. (dont tell psychogenic cough from starting ) .
scenario-cough

Global -6 (approach 6, history 6, examination 6 , investiagtion 6 , dd/d 5)

Feedback December 2018 FAIL


12 year old boy brought in to the clinic by mother because he has been coughing for 6 months.
He had alrdy been given antibiotics and asthma medications which were not working.
Task: History (5 mins)

PEFE

Investigations

Dx
2 mins thinking: Psychogenic cough; asthma; pertussis; TB; repeated viral infections; allergic
rhinitis
History
Started with cough:-

 Nature - Honking

 Dry or wet - Dry

 Timing - no special relation (smokes on both weekends and


weekdays)

 Aggr/Relieve - unremarkable

DDx -- viral – fever? Rash? Runny nose?


-- pertussis – red eyes? Vomit after coughing? Any particular sound at the end of cough?

-- TB – travel recently?

-- asthma – anything he is allergic? Nosiy breathing? Have you ever have the use of puffer
checked with health care personnel? (YES, they say I am using accurately). Are there anyone who
smokes at home?
GH/BINDS (I think I forgot immunization, but again not sure)/Family history of asthma
Didn’t have time to explore home situation as the bell rang (I knew there is sth going on at home
of the child, but I just pretended I didn’t and wanted to exclude organic problems first)
PEFE – GA; Respiratory distress; any stridor or wheezing; cyanosis (I checked all of these in GA);
Vitals; then I proceeded to ENT examination (here, I just asked the examiner I would like to
examine ENT to exclude infection and was expecting the examiner to give me the findings
straight away but he smiled and told me to be a bit more specific. So I asked, any reddened
tympanic membrane, nasal discharge, tonsillitis.) Then did cervical enlargement which was
normal. On asking respiration examination, again, I just said I would like to do respi exam and
was waiting the examiner to tell me the findings, and again, was told to be more specific, so I
just asked the resp exam in details (I still skipped inspection and palpation and went straight to
percussion and ausculation for percussion note and rhonchi/crackles)

Investigations – FBE; (then i said I would like to know if there is any eosinophilia); Xray – every
tests were normal

Dx – *name*, before I explained you the cause for his symptoms, I would like to ask a bit more
questions. Then asked home situation – mother and father split 6 mths back; child coughing
since then; has a strong relalation with father
So, *name*, sometimes the children cough not because of the pathologies inside their airway or
lungs, but because of the stress they experience in their lives. Your child has been using puffer in
correct way and antibiotics and clearly, they arent improving the symptoms so I think the fact
that you broke up with your partner contributes a lot to his symptoms. Don’t worry, this can be
managed by a different approach. (bell rang)

GS – 3 KEY STEPS 1 – YES; 2 and 3 – NO

Approach – 4 History – 3 Choice and Technique of examination, organisation and sequence – 3

Choice of investigation – 2 Dx and DDx – 3

(in fact, this is my first station and is also the one I belived I did best out of 20 stations but failed
miserably. In investigaion, perhaps they want the candidates to ask SPIROMETRY result which I
didn’t. I have no idea what I could do more to pass so just check with another candidate who
passed this station)

78-Pertussis
Recall 1-6-2018
Cough for 16 days in a 8 month old child.
Tasks
-history
-PEFE
-Management and arrange necessary Investigations. .

Recall 7-9-2018
10months child with cough for 2wks x ray provided outside
Tasks
-take history,
-explain x ray (they are saying it was normal from a candidate passed this day)
-pefe on card.
-Explain diagnosis and differential diagnosis.
-management.

History
1-Cough questions
onset
-when did it start? How often? (12-14 days ago, occurring in bouts)
Character
-can you describe it for me? (Whooping like cough cough cough then vomit)
-does he turn blue?
-is your child healthy between these episodes?
-dry or wet? (Dry)
Severity.
-does it affect his sleep or feeding?
Timing
-does it occur at day or night mostly or both?
Aggravating/relieving
-does anything make it better or worse? (Nothing)
1st episode
-is this the first time?

2-Associated symptoms questions (all negative)


-any chest tightness or shortness of breath?
-any fever or rash?
-any noisy sound in his breathing?
-any runny nose?
3-Trigger or risk factors questions
-any recent flu infection?
-anyone in family or school has similar symptoms? (History of contact with brother or father with
cough)
-any pets, carpets, or soft toys?
-does anyone smoke at home?
-does she take any medications?
-any family history of asthma, hay fever, eczema?

4-BINDS
-Birth
-Immunisation (His immunization was up-to-date.)
-Feeding
-Development
-social

Physical examination from examiner (All normal)


1-General appearance
-alert, irritable, restless, drowsy.
-dehydration, rash, pallor, jaundice, cyanosis, LAP
-respiratory distress signs
*nasal flaring
*accessory muscle use
*subcostal or intercostal recessions.
2-Vital signs especially RR + O2 saturation
3-Growth chart
4-ENT (sore throat, runny nose, ear discharge)
5-chest examination
inspection: deformity, chest movement
palpation: chest expansion, tracheal position
percussion: dullness.
Auscultation: air entry, breathing sounds, wheeze or crackles
6-CVS, abdomen

Explanation
-From history and examination your child most likely have a condition called pertussis have you
heard about it?
-it is an infection of the lung, usually caused by a bug called Bordetella pertussis. It is contagious,
spread by tiny droplet of fluid when coughing or sneezing, usually presents with coughing
followed by whooping and vomiting.
-other possibilities are pneumonia; lung infection by another bugs, Bronchiolitis; viral infection
causing inflammation of small airways that delivered air to the lung or upper respiratory tract
viral infections, but unlikely from history and examination.
Investigation
-I need to order FBC (increased lymphocyte)
-take a nasopharyngeal swab by collecting a sample from the back of the nose and throat.

Treatment
-first you need to keep hydration by maintain his fluid and water intake.
-I will prescribe him antibiotic (azithromycin 10 mg/kg day 1 then 5 mg/kg for 4 days) to be taken
orally for 5 days.
-any close contacts need to take antibiotic as well (rifampicin)
-this is a notifiable disease so I need to notify to department of health services.
-look after his hygiene and keep him away from other children at least 5 days after starting
antibiotics.
-after completion of antibiotic, we will discuss about his immunization.
-Red flags (high fever, not eating, looks unwell)
-reading materials.

Feedback 1-6-2018

c/o Cough. Task Hx, PEFE, arrange appropriate Inv, management


+ cough cough cough then whooooop….. h/o vaccination +, Day 16. Father had h/o cough
I forgot to ask if mother had cough too, some say that she did.
PE pxt coughs on trying to check throat. All the rest normal.
I said it’s pertussis and gave all ddx and told why others are unlikely.
Mistakes I made: I mentioned admission and antibiotics. Forgot to arrange Inv, nasopharyngeal
swab. Things I did correct: Contact prophylaxis may be needed, mentioned it’s reportable
disease. Check RCH for treatment guidelines.
Feedback 1-6-2018
Cough for 16 days in a 8 month old child.
Hx
Pete
mx
necessaryIx
( pertussis. Baby is well between cough. Contact hx+. Mother has cough. No SOB
otherwise.Feeding well.
I opened this station with hemodynamic stability like a knee jerk reflex but got scolded by
examiner that have u read your task “it’s a chronic cough” I tried to handle by saying although its
chronic cough but I am just worried if the baby is unstable right now and want to ensure his
stability :P (examiner was not impressed btw)
Any ways asked about onset it was 12-14 days, occurring in bouts, not becoming worst, and
chronic in onset, character was of a whoop like cough cough cough and vomit. Associated with
dry cough. Vomiting no nausea, no fever, no flu, no aggrevating or relieving factors . hx of contact
with his brother who had similar cough , all the family members including the baby were
vaccinated no hx of contact with immunocompromised ppl , infants, school or day care centre.
Well baby questions were all fine baby was healthy and well
. no allergy, no exposure issues related to pets, carpets, food, no hx of travelling , no night
sweats, wt loss, tb contact
PEFE
Vitals: stable
GPE: no anemia, jaundice, dehydration, rashes, clubbing, peripheral or central cyanosis, LAD
eyes: no subconjunctival hemorrhage
resp: normal abd: no hernia urine dipstick: not available
MX:
I told him that the baby is suffering from whooping cough which is a contagious condition
speread through a bug call B pertussis. Although ur baby and other members are immunized
against this bug but it dosent give 100% proecttion and that is y he is showing very specific
symptoms. The condition is not very common and need to be notified to department of health
and needs to be treated by erythromycin, is ur baby allergic to any antibiotics, he said no then I
told him that we will treat your baby as well as contacts with the antibiotic, although he is not
going to day care and school but please take care of hygine as its transferred through droplet
infections, other wise I would have asked you to exclude him as well.gave DD’s (asthma,
tb,pneumonia, allergy, gerd, , viral infections etc) Gave him red flags and asked to do the
investigations (forgot to utter nasopharyngeal swab) and review him and his brother for
nvestigations, gave loads of reassurance and reading material
Feed back: pass
key step 1,2, 3 yes, step 4: no (probably I didn’t utter nasopharyngeal swab) Global score :5
HX:5 PEFE: 5 Dx&DDX: 5 mx plan: 4 pt counseling:5

Feedback 7-9-2018
10months child with cough for 2wks x ray provided outside take hx, explain x ray, pefe on card,
explain dx and ddx, mx (pertussis)
you mean case of pertussis???
It was normal vital just temp increase, no use of accessory muscles auscultation normal, growth
chart normal
79-Indigestion+ weight loss
GP, 42 years old woman complaining of indigestion.
Tasks
-history
-PEFE
-dx and ddx
-explain investigations you would like to do

History
1-Indigestions questions
-I can see that you have troubled by indigestion, so what do you mean by digestion? Can you tell
me more? (Discomfort in tummy)
-do you have any pain in your tummy? Any heartburn? (No, but like burning)
-for how long? Sudden or gradual? Constant or come and go? Getting worse?
-site and radiation (pointed to epigastrium)
-does anything make it better like sitting up position or antacids? (Yes antacids Mylanta initially
worked but now not)
-anything makes it worse?
-is it associated with food? (yes)

2-Associated symptoms (Dysphagia, vomiting, distension, bowels, bladder, LOW, LOA, L&B,
jaundice, fever)
-any difficulty or pain during swallowing?
-have you had any vomiting or vomited blood? (yes but no blood)
-any abdominal bloating or distension?
-has your bowels habit changed recently? Any diarrhea, constipation or alternating D&C?
Have you seen blood in your motions? Have your bowel motions been black? (yes no blood but
black motions)
-how is your waterworks?
-have you lost weight recently without dieting? Any loss of appetite? Any lumps or bumps? (lost
5 Kg)
-any change in the color of the skin or eyes?
-any fever?
-chest pain, SOB, palpitations, dizziness?

3-General questions
-PMH
GERD: (since when, on what medications, regular checkups) (long time history of GERD)
peptic ulcer
colon cancer
hepatitis
-PSH
-SAD (heavy smoker and alcoholic)
-diet (spicy food, fatty food, coffee) (+ve takeaway food)
-family history
PEFE card
1-General appearance DR PJL, BMI , Oedema
2-V/S
3-Focus abdomen + DRE
4-office tests (UDT, BSL, ECG)
(positives are pallor, Left supraclavicular LAP, DRE dark stool; in one feedback just pallor)

Explain
-from history and examination there are several possibilities why you have indigestion:
(Draw diagram)
-It could be esophagitis or an inflammation of the food pipe due to long standing GERD.
-Could be due to ulcer developed in the stomach or food pipe as you have vomiting, black stool
and discomfort in your tummy.
-Could be due to nasty growth of the food pipe or esophagus, as you have a history of weight
loss, stool color change and examination showed LAP, pallor, and this what I am suspecting the
most likely cause. However, I am not telling that you definitely have it because we still need to
do some investigations in order to know more about the cause of the indigestion.
-Others could be liver disease, inflammation of the bowels or pancreas but less likely.

Investigations
1-upper GIT endoscopy: is a procedure that allows to look at the inside lining of your esophagus,
your stomach, and the first part of your small intestine (duodenum). A thin, flexible tube called
an endoscope is used. inserted through your mouth and then gently moved down your throat
into the esophagus, stomach, and duodenum. if they saw anything suspicious they will take a
sample from

2-Colonoscopy: is a tube attached to a camera pass through the back passage to check the inside
of the bowels if they saw anything suspicious they will take a sample from it to be examined
under microscope.

3-FBC, LFT, UCE, Lipase and amylase

4-ECG
15-3-2018
GP, 42 yrs old lady complaining of indigestion.
Tasks: history, dx and ddx, explain invx u would like to do (CA oesophagus, history of long
standing heart burn, 5kg loss in last mth, now indigestion is persistent, no more time to ask abt
any problem with swallowing, smoking +, alcohol +, no spicy food, no coffee, no family history)
Approach: GERD And its complications.
This case of a 42 yr old man with indigest and long history of GERD i dont
remember anything else.it was history PEFE ddx and inx. No Mx . outside i
thought about GERD and its complications and thought this is mot likely ca.
When i eneterd this person was holding his stomach. I asked if there is pain he
said no just bad indigestion.
Started with HOPC about indigestion( Onset, severity, course, continous or
intermittent, anything makes worse better --+ve was antacids which intially
worked not working anymore) then asked about associated GI symptoms + ve
change in colour of stool so explored more about bowel motion , FOBT done
ever keeping ca colon in mind.
Then asked about consitutional symptoms no fever but positive loss of weight
which was significant5 kg then asked about what treatment received for GERD
and who was following him up and any complications.personal history was
positive for significant smoking and alcohol . Said its detrimental to gerd and
will have to address this. Nothing positive in past or family history .
In PEFE + ve was left supraclavicular LN and DRE showed dark stools. dont
remember anything else .
Ddx said sorry looks like nasty growth most likely ca oeosphagus due to long
stnding GERD but could also be ca colon need to refer you urgently for upper
gi endoscopy and colonoscopy . Explained about it .will also do some bloods
like FBE to see if you anemic etc. had plenty of time so told him i will advise
you about lifestyle changes examiner said that is not in task so said i will give
something for indigestion. Examined laughed and told me thats management
again dont go there. So thanked and left ,

Feedback 17-8-2018 Station-7(fail)


Abdominal pain,Task-hx,dx dd,inv
for me pt wasn’t cooperative from the very beginning,she was answering my question lookin
around the room or examiner all time,I wasn’t able to built good rapport with her due to her
aussie accent,I had to ask one question by many way to know my exact concern.
Hx-asked all SORTSARA questions.abdominal discomfort for 2mnths mainly after eating,was
taking take way food now a days,previous hx of reflux positive bt now ok,now taking myalanta
bt not helping,recent wt loss positive,no other medical condition,no nsaid,no coffee tea,alcohol
and smoking positive strongly(I said will take another session regarding this as its not good for
health,she was like whatever!),no s/s of reflux now,no fhx of cancer(score-2).
dx-what I did wrong I said most likely GORD,thn said cancer as wt loss positive,pt was bit
worried so I said not sure now, drew picture of upper abdomen and told all dd
liver,pancreas,stomach problem(forgot to tell gallbladder)(score-3).
I think I would have tell all dd 1st thn most likely cancer,from my experience I have decided I
wouldn’t tell cancer 1st,thn it takse time to calm down patient.
Inv-I said urea breath test,endoscopy,colonoscopy(suddenly I remember I did nt mention PUD,at
the end I said could be pud which is due to irritation of the stomach wall as u r taking take away
food,smoking,alcohol positive,need to check by endoscopy thn bell rang)score-4.
I think I was messy in this case due to role player behaviour,as a result I lost this case.

17-8-2018
ABDOMINAL PAIN
take history for 4 minPefeDd’s tp patTell the investigations you want to do
I started by asking what do you mean by indigestion ? can you please elaborate more. Then he
started telling me that he is having discomfort in his upper tummy he pointed towards
epigastrium. I asked do you have any pain over there ? he said not actually the pain I have
burning here he gave this type of history pointing towards heartburn.
But to be on safe side I asked if this is going anywhere else he denied that , but I asked
SORTSARA questions to make sure its not the pain .he said sometimes this is associated with
foods , he was not sure whether leaning forward or lying down makes any difference , then I
asked aggravating factors for heartburn by individually naming them like spicy foods, fatty food,
eating just before going to bed, smoking , alcohol he gave history of heavy smoking and drinking
5 6 standard drinks, he frequently takes take away foods, aked about lifestyle , not very healthy
regarding diet, in relieving factors he takes mylenta that gives temporary relief but not
anymore, asked about vomiting , he gave me history of vomiting as well , in details of vomiting
nothing was significant I specifically asked about blood in vomit , was negative , his of wt loss
was there , no history of difficulty of swallowing , black stools were positive but no fresh blood,
no hist of diarrhoea or constipation, no family history of GI disease no skin color change no
other significant positive history. I found It difficult during history to understand roleplayer and
was bit confused while performingThen I moved to examiner for pefe he gave me a card all were
normal there except mild pallor I remember. PR was normal.Then I explained the patient that
from you’re his and exam which I have done there could be few possibilities of your symptoms, I
drew pic and explained about inflammation of esophagus due to reflux, it could be ulcer
formation in your food pipegave reasons for that, it could be cancer /nasty growth but we have
to do further investigations to confirm that, but I am suspecting this to be most likely because of
your history and wt loss I named all triggers here,but in very carefull way not threatening the
patient, then told only one sentence about it could be due to pancrease but highly unlikely or
due to liver damage also unlikely, Then I mentioned I would like to ref you for endoscopy
(explained in layman terms in details)and colonoscopy ( layman and explained) here bell rang ,
I wanted to say amylase ,lipases for pancrease , LFT’s as well because of alcohol and FBE for
anemia but couldn’t complete . thought I lost the case but thanks to ALLAH I passed.All key steps
covered 3/3History 5Dd’s 6Choice of investigations 5Global 5
Feedback 27-10-2018

40+ year old man came to GP with the complaints of indigestion.

Task

- history

- Dx/DDx

- Investigation History

- duration (a couple of days)

- not the first time (3 - 4 similar episodes before)

- took Mylanta for previous episodes

- but getting worse lately

- burning epigastric pain (+) along with indigestion

- radiation (-)

- relieved after taking Mylanta in the past

- worse after having meal

- severity of pain (moderate) Associating symptoms

- nausea (-), vomiting (-)

- swallowing difficulty (-)

- waterworks (normal)

- when I asked about his bowel, he said normal.

- then I asked specifically “ is there any change in the color of your


stool?”

- he said there has been passing of black color stool for the past 3
days.

- lost 4-6kg over 3 months, but he said it is due to his stress at work
and been really busy these days.

- lumps and bumps in the body (-) other DDx

- chest pain (-)

- cough (-)
- change in color of skin (-) risk factors

- enjoyed eating spicy meal in the past, but not eating them anymore
as they made the pain worse

- not taking any analgesic

- smoking (+), alcohol (+)

Dx/DDx

Drew the stomach

- it could be due to

- nasty growth

- sores (ulcer)

- gastritis

Drew liver and gallbladder

- could be due to this organ

- but I’m concerned that you might be having nasty growth

- the patient looks upset at that time.

- told him that

- you have similar episodes in the past

- your condition is getting worse lately

- losing weight

- bleeding from your stomach

- then role player told me “ I’ve been really busy at work these days.
that’s why I lost some weight”

- I told him “I understand that you are quite upset about this.

- These are the possible conditions.

- I will arrange further investigations to confirm the Dx and to rule out


other possibilities”

- I asked him whether he has heard of endoscope

- he told me “ yes, Is that the tube that I need to swallow from the
mouth”
- I told him “yes, that tube have camera at one end which you have to
swallow it down the throat.

then, it reaches the stomach and we will be able to see clearly whether it
is sores or nasty growth inside your stomach”.

- I told him I will arrange FBE as well to see if he is anemic or not.

- reassure him at the end that after all these investigation, I will liaise
with specialist to manage your condition depending on the result.

Grade - pass Global score - 4 Key steps 1,2,3 - No,


Yes, Yes

History - 4 Dx/DDx - 4 Choice of Invx - 5

80-Sore eyes
27 year old woman complain of eyes pain (in some red eyes), fever and unwell (patient wearing
glasses)
Tasks
-History
-Diagnosis and differential diagnoses

History
1-pain questions
-severity and painkillers+ soothing drops.
-duration (2 days)
-sudden or gradual? Is it getting worse?
-site? (Both eyes)
-radiation?
-anything make it better or worse?
-has this happened before?

2-Associated symptoms (Local)


-do you have any redness in your eyes? (Yes)
-I can see that you are wearing sunglasses. Are your eyes sensitive to light?
-have you had any swelling or discharge?
-are your eyes stuck when you wake up in the morning?
-any pain or restriction of eyes movement?
-any blurring of vision?

3-Associated symptoms (systemic)


-any fever? (Yes)
how high is it, did you take the chance to measure it?
Is it constant or come and go?
Is it associated with chills or shivers?
Any night sweating?
Any recent viral infections?
-any rash (yes)
when did it start?
Where exactly?
Has it spread?
What is the colour?
Can you describe it for me?
Is it itchy?
-any LOW, LOA, lumps or bumps?
-any headache or neck stiffness?
-any nausea, vomiting or tummy pain? Any change in bowel habits? How is your waterworks?
-any aches or pains around your body? (Yes)
-any joint pain? (Yes) which joints? Any swelling or stiffness?

4-General
-PMH (especially of arthritis)
-PSH
-medications and allergies + SAD
-do you wear contact lenses?
-any injury or trauma to your eyes?
-have you travelled recently? (Yes)
where?
How long did you stay there? When did you return?
Any insect bite?
Have you had vaccination?
-are you sexually active? Do you practise safe sex?
-any contact with someone with similar symptoms?

Explain diagnosis and differential diagnosis


-from history, there are several causes why you have these symptoms. It could be travel related
infections, which are usually viral infections like dengue, zika or lyme. Usually transmitted by a
mosquito or insect that commonly present in tropics and subtropics areas.
usually presented with tiredness, aches, joint pain, sore eyes, rashes and fever. As you have
recently returned from travel that is why I am suspecting. They are usually self-limiting
conditions.

-it could be due to acute viral infections like measles, CMV or EBV.

-could be due to some joint problems like arthritis or connective tissue diseases as they can
cause similar symptoms sometimes.

-I need to do Ix to rule out these causes:


FBE (platelet for dengue), BSL, LFT, UCE
thick and thin blood film
hepatitis serology
serology for rose river fever
serology for dengue fever, lyme, zika

Recall of 16-3-2018

Case (27/4/2017)
a 22y/o lady comes with 2 days history of red sore eyes.
1- take history from patient
2- explain the most probable diagnosis for her.
when i entered the room, a young asian lady was sitting and wearing sunglasses. i started
introducing and ask her 'what brings you here today'. i didn't ask about her sun glasses.
any pain, any assocition with eye movement, any redness or swelling around the eyes, any
discharge, any effect on vision, any fever, any ache and pain, tiredness, racing of the heartbeat,
any rash, general health, travel history , 5 p and SADMA.
*PF (red eyes +, watery+but not associated with eyes movements, no effect on vision, no
swelling are noted, high fever +, she also had ache and pain and feel tired when do some activity,
at last she told me she also had rash).
i explained it to her that it could be kinds of viral infection, since you just recently came back
from south africa and got fever, ache and pain and rash, it could be due to dengue infection
which is caused by dengue virus. Reassure her that don't be much worry as it can go away with
some supportive measures. It could be due to Ross River fever but it is usually occured after
coming back from Queensland. it could be due to other infection like infection of the eyes like
cellulitis, periorbital cellulitis and CMV infection. Reassure her again and give her reading tips
and said that i will regularly see you to check your progress and condition.
FB-PASSED ( OVERALL-4, APPROACH-4, HISTORY-3, DDX-4)

Case (5/10/2017) Fever and Rash Pass


Middle aged woman comes in your GP due to feeling unwell and red eyes. ( Patient is wearing
sunglasses)
Task: History
Dx and DDx to patient with reasons
When i entered the room, she was wearing sunglasses. Introduced myself n asked her she has
any pain or not. Then i asked her to take off the sunglasses so that i can see the eyes. But i asked
if the light hurts her eyes or not cus i m asking her to take off n she said No n she apologised me
for wearing sunglasses in the office.
Positive findings....
redness of eyes but no pain just discomfort, no discharge, no visual disturbances.
Fever + abt the same time with sore eyes ( subjectively measured) relieved by panadol.
Rash + all over the body, not itchy, red colour, she doesnt know the pattern of spread.
Joint pain n stiffness + but no special time to have those pain.
Travel history + ( a country in central africa) cant recall the name of country. She got yellow fever
vaccine for that. No contact history n not on any regular meds. No unsafe sex during the trip.
Mosquito bite ++
DDx -- told her suspecting she has viral infection caused by mosquito bite called Zika n Dengue
N it could b Normal Flu. Conjunctivitis. Glaucoma ( build up pressure in the back of ur eyes ) but
very less likely as u have no painful eyes n no visual disturbances n it doesnt have symptoms like
Rash.
Covered 4 key steps out of 5 Approach 5 History 3 Dx and DDx 4 Global score 4

Feedback
SORE EYES Recall
22 yr lady
bilat sore eyes
positive features
all for 2 days .
bilat red sore eyes, irritable eyes
waterry eyes n nose
sore throat
visual problem not noticed
fever, generalized rash
muscle ache joint pain
no jt swelling
no lymph node
no GI symptoms
no neck stiffness
no bruises.
travel to brazil n america so latin america for 2-3 wks
came back 1 wk ago.
ho of severe RA in mother
i gave ddx in main topics n examples
1. acute viral inf also measles, rubella,
2. travel related infections s/a lyme ds ( we can add zika ).
3. other infections with fever rash sorethroat n bilat red sore eyes. i will try to exclude.
said u may have concerns for meningococcal meningitis but i tried to exclude it n it's quite
unlikely a/t history n pe features
4. first n early presentation of RA or other connective ts ds
or autoimmune disorders
pass
fever and rash
global score 5
approach only 2, can't understand why
history 5
dx ddx 5.
all 5 key steps yes.
so i hope this is helpful.
may be due to i did not provide any pain killers or soothing eyedrops or sth they want. pt is asian
lady wearing sun glasses in the room.

Feedback 25-10-2018 (Important)


Scenario: fever and rash
Stem: young woman coming with generalised body pain and sore eyes.
Tasks:
Hx
Explain probable dx and ddx with reasons

I entered room greeted the pt. she was a friendly lady wearing sun shades inside the room.
Asked her about body pain and tiredness. She said she feels hot but didn’t measure temp. and
its been for last 3 or 4 days and today morning she saw this rash (no pic shown, u just have to
assume there some rash) I asked fever qs. Then rash qs (it was on whole body pink colour no
itching or oozing, no bleeding or bruising) asked photophobia (-ve , wearing glasses cz eyes are
red and shes embarrassed bcz of it)
Asked her travel hx. Positive for brazil. Chance of mosquito bite (yes) did u take travel advice
(yes), sick contact (no)

Sexual hx (stable partner no casual relations) i asked her any chances of hanky panky at all n she
was laughing n examiner smiling too

Past med, immunisation (not vac against mmr)

Sadma (-ve)

Explained her that there are certain dx more common in some parts of the world than other. As
u travelled to brazil im suspecting most likely it could be zika virus or even dengue both of these
are spread by mosquito bite. But there are other possibilities as well like chikungunya.ross
river fever, yellow fever, meningococcemia, measles mumps or rubella. Could also be som
bleeding dx like itp, hsp, leukemia or some close contact related EBV. Bell rang
Grade: pass
GS : 5 5/5 key steps covered Approach to pt: 6 Hx: 5 Dx/ddx: 7
Feedback 25-10-2018 FAIL
A young woman with sore eyes.
Task:
History
Dd
I entered and saw a girl wearing sunnies, she wouldn’t take them off. I was asking politely,then
examiner interrupted “ I ll tell you , she has bilateral bacterial conjunctivitis”!!!!!!!!!!
I was sooo confused, as the examiner just told me the diagnosis?!
So I kept taking history accordingly and did not ask to remove sunglasses anymore.
She said mild fever, no contact with other with similar symptoms, I asked of insect bite around
eyes..said no. asked about wearing lens…all negative
Gave dd: all types of conjunctivitis, multiple sclerosis aVd whatever I could think of as eye pain
Other candidates got hox for rash, travel joint pain, photophobia
My scenario is written: “FEVER WITH RASH” so…I completely lost this one

81-Breast reduction surgery request


Case 1
Female patient, coming for breast reduction surgery referral
Tasks
-History
-explain the reason that motivates her request.
-further mx plan

Case 2
30-year-old woman wants to have a breast reduction surgery.
TASK
-Take relevant history,
-Explain to her about her condition, If you refer her to the surgeon give reasons why

1-Approach
-patient is crying from the start, she is lean and thin lady, sitting on the chair. So give tissue and
water

-I can see you are very sad. Would you like to tell me what is it about, so that i can help you (Dr
my breasts are so uncomfortable and big).
Note/ in some feedback she will tell you dr I want to get rid of my breasts so ask why? are they
bothering you? (yes, always bothering me but recently too much).

2-chief complaint questions.


- All right jenny, I will certainly try my best to help you, I just need first to ask you a few questions
and let me assure you that everything we gonna discuss will be confidential unless there is a
harm to yourself or others.

- Since when you are having this problem? (Few months now )
- do you have any lump , redness, discharge from the nipples? ( no) so you just concern only for
the breast size.

- does any one comment about it? (No)

- do you check yourself in the mirror often? (I just check it regularly)

- Do you do anything to hide it (no).

- do you concern about other body parts (no)

-How does it affect your life? If anything happened recently? (She started to cry more and telling
I have so many problem, I broke up with my husband few months back  give another tissues)
I can see it must be very distressing to you.

3-HEADS
-who are you living with now? (I live alone)
how about your parents? (I lost my father 6month back, my mom passed away at my age of 13)
sorry to hear that

-What do you do for living? (Work at a company). Any stress at work or any financial problem?
(No)

-have you lost interest in things you used to enjoy?

-how is your social activity? (I go out with my friends and I feel good when i go out with them)

4-psych history
mood
-how is your mood? For how long? (Low)
-have you ever thought to harm yourself or others? (No)
-how is your appetite? (eat normal)
-have you put on weight recently? do you have any excessive concern about body wt.? (No)
-how is your sleep? Any nightmares or vivid dreams? (Disturbed)
-any difficulty concentration?
-do you feel guilty? (No)

Delusion and hallucination


-do you see, hear, feel things that others do not? (no)
-do you feel someone is following you, trying to harm you or spying on you? (No)

5-Differential diagnosis questions


-PTSD: any flashbacks, nightmares or vivid dreams? (No)
-GAD: Do you worry excessively about lots of things? (No)
-Thyroid: weather preference
-SADMA (alcohol drinking (+).
- past or family history of mental illness (No)

Explanation
-Jenny , I can see you have been through a very tough time. You lost your father and relationship
problem as well, and have a lot of stress, and you are not coping well.

-I think your problem is more than breast reduction surgery request, as due to these stressors,
you do not know where to vent it all and you go for breast reduction as seems to be the way.

-most likely you have a condition called Body dysmorphic disorder which is an anxiety disorder
due to stress. Usually it is normal to be concerned about one’s appearance, but in your case you
are over concerned about your body image which makes you anxious.

-And I think you have depression as well , as you have low mood , early morning awakening
(critical error)

Management
-so you want a breast reduction surgery and off course, it is up to you it is your body and your
choice. However, surgery can have many risks like infection, bleeding, anesthesia and we dont
want to put you at risk unless it is necessary.

-so how about we try some other things first; I would like to refer you to a psychologist who will
do talk therapy who will help you to change negative thoughts to positive thoughts.

- After that, if you still think that, you need breast reduction surgery, I can refer you to a plastic
surgeon or psychiatrist for second opinion. How does that sound to you ... she nod ...

-support group where you can meet people with similar situation.

-I will do regular follow up, but if you feel very low, have harmful thoughts towards yourself
please come back to see immediately.

Case (29/11/2017)
My station 16
BDD ( Feedback – surgical consultation)
30 year old woman wants to have a breast reduction surgery.
TASK-Take relevant history, Explain to her about her condition ,If you refer her to the surgeon
give reasons why

She seemed tearful and didn’t have big breast , so I introduced myself in a low tone
Me : How are you doing
started crying, had tissues in hand so only offered glass of water. ensured confidentiality
I just want to get rid of these breasts..sobbibg
why are they bothering you
Breasts have always bothered her!! But recently too much
Anything changed in ur life?
i dont know, my husband is asking for a separation and my father died 6 months back. i dont
know what to do
I am sorry for your loss. I need to ask you a few more questions if thats alright.
sleep was disturbed, early morning awakening, increased apetite, mood low, goes to work, goes
out with friends, no support, no smoking, alcohol, drugs
do u think having this surgery might be able to save ur marriage?- don’t know
lets see. There is a lot going on in ur life.it seems to me is that you are depressed, the loss of
your father and your husband asking for a trial divorce adds up to all this and you do not know
where to vent it all out and go you a breast reduction seems to be the way. U want a breast
reduction, and of course I can give u a referral its ur body ur choice.as a surgery has few side
effects. So what if we take a detour?
how about if we try some other things first, a psychologist probably who will tell you techniques
to think positive, maybe arrange a meeting with your husband and a marriage counsellor to sort
things out ( she said i dont think he will agree) and some support groups, if it doesn’t work then
we can opt for surgery. How do u like the plan?
ok doc lets do this then

Feedback 20-7-2018 (important)


Body dismorphic disorder
Surgical consultation Pass Global Score -4
key step 1,2, 3 yes yes yes
approach to pt 4
History 4
Dx and D/Dx 4
patient counselling /education 4

Female patient , coming for breast reduction surgery referral ,


History ,
explain the reason that motivates her request ,
further mx plan

(((patient is crying from the start , she is lean and thin lady , sitting on the chair , with very
low neckline blouse ))))
I gave her tissues , jenny i can see you are very sad . would you like to tell me what is it about
, so that i can help you . Dr my breasts are so uncomfortable and big , yes jenny , i will
certainly try my best to help you , i tell confidential before asking further question ........would
you tell me since when you are having this problem ( few months now ) do you have any
lump , rednesss, discharge from the nipples ...... no , so you just concern only for the breast
size , does any one comment about it .. no , do you check yourself in the mirror often .... i just
check it regularly, do you do anything to hide it .... no , do you concern about other body
parts .... no , does it affect your daily activity ......she started to cry more ........and telling i
have so many problem , i broke up with my husband few months back ....i gave another
tissues, i can see it must be very distressing to you .... so who are you living with now , i live
alone , any family ... i lost my father 6month back ... sorry to hear that , Jenny , were you
very close to your father .... she said , yes since my mom passed away at my age of 13, he
was the only family member ...... pt work at a company , no financial problem ,
Depression question low mood, not sleep well , wake up early morning , eat normal , she
thinks she is gaining wt ( at that point , i asked her do you have any excessive concern about
body wt , are you afraid of gaining wt ..... no) , do you enjoy doing fun activities when you
are free .... yes i go out with my fris and i feel good when i go out with them ..... no sucidal
and homicidal ideation , do you feel guilty of something you have done before ... no ,
she denies halluniation question ,
PTSD ... no nightmares , vivid dreams , flashback memories ,
GAD .. are you easily anxious person ... no , SADMA .... alcohol drinking (+) , no past or family
history of mental illness,

Explanation
Jenny , i can see you have been thr a very tough time . Things are very difficult to you . you
lost your father and relationship problem as well , and have a lot of stress, and you are not
coping well . I think your problem is more than breast reduction surgery request , Many ppl
on the world concern about their body image , it is natural but when it become excessive ,
like in your case , it can make you so much distressed . i think you are having condition called
BDD , which is anxiety disorder , the stress you are having push you to have this excessive
concern about the breast size, and i think depression as well , as you have low mood , early
morning awakening ,
so i would like to refer you to specialist , psychiatrist who will do further assessment on you.
and the psychologist , who will do talk therapy who will help you to change negative thoughts
to positive thoughts
i personally think your breast size are normal , you dont need breast reduction surgery , cos
surgery can have many risk ( infection , bleeding , anaesthesia ), we dont want to put you at
risk unless it is necessary , even though you do the surgery , if we dont fix the actual
problem , things can come back again and you may not be satisfied with the result either
At that point she said , my breasts are too uncomfortable , i said , i utd , how about we try
the talk therapy first , i believe it will be very helpful to you . but after that , if you still think
that , you need breast reduction surgery, i will refer you to plastic surgeon for second opinion
. How does that sound to you ... she nod ...
i will do regular follow up , but if you feel very low , have harmful thoughts towards your
self .... pls come back to see immediately
i will arrange family meeting ( she started to cry again ... i know i said wrong , she has no
family ) i gave her tissues ,said i mean will do meeting with your fris if you agreed so that
they can give you support ......
This station is a lot of talking and reassurance station , I involuntarily gave her tissues 3 times
whenever she accelerated to cry :D bec i dont know what to do any thing else

Case (1/3/2017)
Request for breast reduction surgery, burst in to tears when asked for reason

Second psych case is body dysmorphic disorder. the patient want to do surgery as she said I have
very big breast and it effect my life a lot. I separated from my husband recently and my dad
passed a way 6 months a go. I try to asked her if there is any relation between her breast shape
and separation. she said no. I try to ask more questions about depression but there is non and
she start crying and said I miss my dad and we use to be very close. this case I didn't give dx
even though I know.
I said we usually don't prefer to refer patient for cosmetic surgery if she don't need there is risk
for anaesthesia and surgery but if you insist, I will give you a refer for second opinion.
I will refer you to psychologist for CBT who help you to change this negative feeling with positive
one and if that doesn't work might be refer to psychiatry for medication.

Feedback 7-12-2018
35 years comes for breast reduction surgery.

task: history for 4 min , explain findings to patient, counsel.

2 min thinking- history in 4 min is very difficult- need to exclude depression and suicide, NO
need to tell Diagnosis as not task but never know.
history - patient cried for 2 min i have got 2 min for history, i tried to reassure and counsel her ,
gave tissue , water and did all efforts to calm her, which took good 2 min,( i was almost 7th
candidate- but still role player reading history from paper in her hands -donno whether they do
intentionally or something else) rest 2 min, quickly asked history since when -6 months, what
happened-husband left, any other BDD-no, any anxiety-no, just ruled out any discharge, any
discolouration of skin, any lumps and bumps-no, no suicidal thoughts - bell rang .
tried counselling her with words like brain body axis and all (i guess it did not work)(resisted
myself not to tell diagnosis- but told its Body dismorphic disorder)
scenario-surgical consultation

Global -3 (approach 3, history -3, dd-3, patient counselling 3)


Handbook case 84 (just reading)
CANDIDATE INFORMATION AND TASKS

You are the duty Hospital Medical Officer (HMO) in a busy city hospital clinic attached to the
Emergency Department. It is early evening.
The patient you are about to see is a neatly dressed, well-groomed 29-year-old man who has
brought an envelope containing some hair strands to the front desk, asking if they can be
'examined under a microscope'. He appeared to be quite anxious and restless whilst waiting to
be seen by you and the triage nurse has told you that he has visited the toilet facilities for
lengthy periods of time on several occasions.

The triage assessment states that he is worried that he is suddenly going bald because he has
begun to lose his hair. He has brought some of his hair to the hospital to be examined to find out
what the problem is and have treatment urgently' because he believes that his hair loss is
affecting his 'prospects for promotion at work'.

He is single and lives at home with his parents. He has worked as a financial analyst in a
merchant bank for the past six years. He admits to not having any social life and is a nonsmoker
and nondrinker.

He appears to have a normal full head of hair as illustrated below.

YOUR TASKS ARE TO:

• Establish rapport.

• Take a sensitive, focused and relevant history.

• Reach a diagnostic conclusion, and discuss this with the patient.

• Discuss management briefly with the examiner.


AIMS OF STATION

This is primarily a diagnostic and communication skills station, assessing the candidates ability to
take an empathie and relatively quick psychosocial history and to rapidly reach the correct
diagnostic conclusion.

EXAMINER INSTRUCTIONS

The examiner will have instructed the patient as follows:


I can see from the notes that you have hair in this envelope and you want me to take a look at
it under the microscope? Could you tell more about your problem?

You are a 29-year-old financial analyst in a large city merchant bank. It is a competitive
environment. You have been with the company for six years since graduation from university, but
your career progress has stalled.
Why do you think that your progress has stalled? (if stopped talking)
Although you are conscientious and reliable at work, it is three years since your last promotion.
Fellow cadets from your intake cohort seem to have left you behind. You have just had a
performance appraisal interview and have once again been passed over for promotion.
What do you think the cause is? (if stopped giving information)
Your explanation for this is that you are losing scalp hair and must be going bald and that it is
your hair loss that has cost you your promotion.

Well, 'Apart from your hair, have you ever been very worried about your appearance in any
other way?'
Yes

Ok so 'Can you tell me what your concern was?'


Since your adolescence you have been concerned about your appearance and grooming. It
began with a belief that your face was asymmetrical which you believed was obvious to other
people
'Did this concern preoccupy you? Do you think about it a lot and wish you could worry about it
less?'
this led to you checking your facial features in a mirror several times a day. Then you began to
notice facial skin flaws and different shades of pigmentation. If you developed a pimple or
shaving rash you would touch, pick at and constantly inspect the lesion in a mirror on an hourly
basis.

'What effect has this preoccupation with your appearance had on your life?'
Checking your facial appearance on a regular basis during the course of a day has now become
part of your daily life. Each mirror checking episode lasts several minutes with you having to
reassure yourself that no new blemishes have appeared or that any existing blemishes are
improving or fading. Whilst at work you can only do this every two hours for a few minutes, but
at home and on weekends it may take you at least half an hour to complete a thorough
inspection of your entire head and face region, which you repeat three times a day.
What is the total amount that you spent checking your appearance?
If asked, you will concede that the total amount of time you spend checking your appearance in
front of the mirror, touching, examining and picking at almost every skin pore or hair follicle,
could be four hours a day on weekends. The amount of time you spend monitoring your
appearance is slowly increasing.

'Has it affected your social life, family relationships, friends, job or other activities?'
You still live at home with your parents, who are both school teachers approaching retirement.
Your mother suffered from agoraphobia during her twenties before her marriage. They have
become accustomed to you constantly asking them 'How do I look?' or, 7s there something
wrong with my face/skin/mouth/eyes/hair?' Their unfailing reassurance that there is nothing
wrong with your appearance does not reduce your concerns as you are sure they are only saying
that to humour you.

Your concern for the care of your skin means that you do not like to socialise at parties or clubs
where people may smoke. You avoid crowds and public transport to avoid embarrassment of
strangers. Subjecting your features to close scrutiny. You avoid direct sun exposure. You do not
smoke or drink alcohol.
Have you done anything about your concerns? Have you tried to hide it in the past decade, you
have spent a small fortune on male beauty treatments, face packs, facial massages, hair styling
and allergy-free soaps, shave creams and cosmetics in an attempt to treat or camouflage your
skin defects. It is your main interest.

Over the past few months you have been monitoring the number of your scalp hairs you have
found on the floor of the shower cubicle after a shower. Although it may only be one or two, you
have come to believe several things. The first is that the cumulative loss of hairs means that you
are going bald and that the resultant change in your hair density and thickness is obvious to
other people. You are certain that this obvious hair loss has influenced your employers not to
promote you because to have employees with thinning hair is not good for the bank's image
when dealing with clients.

Finding out that you have yet again not been promoted has driven you to seek advice, about
diagnosis and treatment for your hair loss.
Do you wish to do anything about your concerns?'
As you do not trust your family general practitioner, who has dismissed your concerns and said
there is nothing wrong, you have sought a second opinion from an unbiased doctor at the city
hospital closest to where you work.

Mood questions

How to play the role

You must be neat, well dressed and have a full head of hair. You will have an envelope with a
couple of strands of your hair in it. You will be anxious and somewhat irritable. If there is7 a
mirror in the consulting room, then insist on showing the doctor your 'receding hair line' at an
early stage of the interview. (It would be useful to have a small mirror as a prop)

Opening statements

• ‘Doctor, you've got to find out why I'm losing my hair!'


• ‘I want these hairs of mine examined under a microscope by a specialist! '

• 'Let me show you where I'm going bald'.

• 'You've got to do something!'

Your subsequent behaviour and emotional reactions will be shaped by the way the interview
unfolds. If the doctor rushes to judgment and dismisses your concerns without tact, empathy or
appropriate discussion, then your irritability and exasperation may increase.

If the doctor realises what your underlying problem is. effectively establishes rapport, and the
extent of your difficulties and hypochondriacal concerns is realised, then be defensive and
sceptical, but be prepared to listen and interact appropriately.

Do not willingly volunteer history of your rituals or checking behaviour until asked. These have
been behaviours you have kept secret for years, but you may be relieved that at last someone is
able to encourage you to talk about them.

After six minutes, the examiner will interrupt the consultation and ask 'What is your provisional
diagnosis? Describe briefly possible management plans to me. '

EXPECTATIONS OF CANDIDATE PERFORMANCE

The patient has a form of Body Dysmorphic Disorder (BDD) presenting with the conviction of
impending baldness, when objectively and clinically, there is no supporting evidence.

BDD is a form of hypochondriasis which is part of the anxiety disorder spectrum. In this instance
there are many obsessive compulsive disorder features. BDD is a condition that affects about 1%
of the population, but is infrequently diagnosed because of the lack of awareness by clinicians
and patients' secrecy about their bodily preoccupations.

The candidate will be expected to establish the diagnosis, the associated behaviours and the
complications in this case, as well as being aware of the common comorbid psychological
disorders. Knowledge about the effectiveness of the serotonin reuptake inhibitor
antidepressants and cognitive behavioural therapy in this condition would be desirable.

Key questions the candidate should ask of the patient would be:

KEY ISSUES

• Ability to take a focused psychosocial history and to come to an appropriate diagnosis.


• Ability to communicate with a patient with body dysmorphic disorder.

CRITICAL ERRORS - none defined

COMMENTARY
'Body Dysmorphic Disorder, or dysmorphophobia, is a chronic preoccupation with an imagined
defect in one's appearance. Even if a slight physical anomaly is present, the person's concern is
markedly excessive. The preoccupation causes significant distress or impairment in the person s
social, occupational and other important areas of functioning.

Typical complaints commonly involve imagined or slight flaws of the face or head such as
thinning hair, acne, wrinkles, scars, vascular markings, paleness or redness of the complexion,
swelling, facial asymmetry or disproportion or excessive facial hair.

Other common preoccupations include the shape, size or some other aspect of the nose, eyes,
eyelids, eyebrows, ears, mouth, lips, teeth, jaw, chin, cheeks or head. Any other body part may
be the focus of concern (the genitals, breasts, buttocks, abdomen, arms, hands, feet, legs, hips,
shoulders, spine, overall body size or body build and musculature).

The preoccupation may focus simultaneously on several body parts and although it may be often
specific: 'a hooked nose'; it may also be vague: 'a flat chest'; or general: 'I'm just ugly'.

From Diagnostic and Statistical Manual 4 - Text Revision

Most individuals with this disorder experience marked distress over their perceived deformities.
They find their preoccupations difficult to control and may make little or no attempt to resist
them. Many hours of the day may be spent thinking and worrying about their 'defect' and these
thoughts may dominate their lives, leading to significant impairments in functioning and
avoidance of work, social and public situations. Repetitive and time-consuming behaviours are
undertaken to reduce their distress, which have no or only minimal benefit. Reassurance that
there are no visible defects has no lasting effect on their abnormal beliefs.

The core irrational belief in BDD is that the person is somehow defective and unattractive and
this is accompanied by low self esteem, shame, embarrassment and fear of rejection. The
condition is common if it is looked for and asked about.

The most common associated behaviours are mirror-checking, touching, comparing the defect'
with other people's body parts either directly or with pictures in magazines, excessive grooming,
camouflaging, constantly seeking reassurance and questioning others about their alleged defects
or ugliness and then seeking dermatological or cosmetic surgical treatments.

Social impairment is the norm. They are socially avoidant and will not willingly visit restaurants,
shopping centres, beaches or go to parties or functions because of their self-consciousness
about their appearance. Insight is usually poor or partial and their beliefs may become
delusional. It is their self-referential ideas, i.e. that other people are taking special notice of their
'defect' and will talk and gossip and laugh about it, that contribute to their social isolation and
intensify their suffering to the point of despair, self-harm and sometimes suicide.

Psychiatric comorbidity is universal. Major depression is the most common (60%) but social
anxiety/phobia, obsessive compulsive disorder, substance abuse and avoidant personality
disorder are highly prevalent.

There is a roughly equal sex incidence and similar clinical features. Perhaps women are more
likely to focus on their skins, lips, and weight, whereas men are more preoccupied with overall
physique, their genitals and hair loss or excess. The condition typically begins in adolescence, but
may not present or be diagnosed until the thirties. The course is chronic and relapsing.

Most patients with BDD seek costly dermatological or cosmetic surgical consultations and
treatments, but remain dissatisfied with the results. They may then become litigious or violent.
Rarely patients perform their own procedures after consulting internet web sites.

Management

The specific serotonin reuptake inhibitor antidepressants and clomipramine are often
effective. The dosages need to be in the higher range and it may take three months to get a
response, but 70% of patients report improvement. Augmentation with antipsychotics may
increase the response rate. Treatment must be continued longterm as relapse is common if
treatment is discontinued.

Cognitive behavioural treatments including psychoeducation, cognitive challenge and


restructuring, exposure and response prevention, as well as anxiety management training, can
supplement drug treatment and increase response rates to over 80%. Severe comorbid
depression may need hopitalisation and/or lithium carbonate augmentation with
antidepressants.

Trying to convince patients with entrenched ideas that their beliefs are irrational or that they
look normal is unlikely to persuade them to accept psychiatric treatment or referral. With the
patient's consent, family involvement in psychoeducation and treatment planning and
supervision of response prevention strategies and the removal of mirrors from the family
home may be valuable.

Karin case
Case: You are an HMO and your next patient is a 29-year-old male who came to see you. He
brought an envelope containing hair and he asked you to examine it under the microscope. He is
worried that he is getting bald and he believes that this hair loss will affect his promotion.

Task
a. Focused history
b. Diagnosis
c. Management

Case 2: Female wants to have a breast reduction surgery

Case 3: Man concerned about penis size

Case 4: You have a 24-year-old male student coming to your GP clinic asking for referral to plastic
surgeon because he thinks his nose is too big. On examination, you find that his nose is
completely normal.

Task
a. Further focused history
b. Examination not necessary
c. Advise patient

Features
- Preoccupied
- Try to HIDE it
- Frequent mirror checking

History

- I can see from the notes that you have hair in this envelope and you want me to take a look
at it under the microscope? Why? Is it the first time? How is it affecting your life? Apart from
hair loss, are you concerned about any other part of your body? Does this concern preoccupy
you? Do you try to hide it?
- How is your sleep? Restless? How is your mood? Appetite? Weight? Anhedonia? Guilt? Do you
think life is worth living? Have you thought of harming yourself or others? Do you have any
strange experiences?
- Social history: how are things at home and at work? Do you have a lot of friends? Do you
socialize?
- SADMA?

Examination: HAIR

Diagnosis and Management:

Case 1:

- Your hair looks normal and let me reassure that I do not see any hair loss. It is normal to lose
up to 100 strands of hair a day which is the natural process of hair regrowth. I also understand
that it is normal to be concerned about one’s appearance, but in your case, you are over-
concerned which is making you anxious and affecting your function. This is known as body
dysmorphic disorder which is a type of somatoform disorder. In your case, it is accompanied
by anxiety as well. Unfortunately, such an irrational belief can lead to severe self-doubt and
low self-esteem. Your problem has been worsened by your work stress.

- Don’t worry. You made a decision to come up and talk about it. At this stage, I would like to
refer you to a psychiatrist who will do talk-therapy. He may also put you on some medications
(SSRI and antipsychotics) for your anxiety and concern.

- I would advise you to remove the mirrors in your home.


- Offer family meeting/support/social workers
- Review, reading material, referral.
Case 2:

History

- Confidentiality
- History: patient information  HPI
- Family history
- Personal and social history

- MSE
o Appearance: eye contact; psychomotor agitation, how patient dresses,
o Speech: pressured, normal, soft, loud, monotonous
o Affect

o Thought: Form and Content

o Cognition
o Suicidal Risk
o Insight/Judgment

Management
- I think you have what we call BDD. This means that you have a preoccupation with a certain
part of your body even though it is normal and this is causing you distress and anxiety.
- I can give you a referral to a plastic surgeon if you want, for a second opinion, but honestly at
this point, I don’t find it necessary.
- I would, however, like to refer you to a psychologist for cognitive behavioral therapy so that he
can talk things through and identify any stressors and help you cope with them.

Feedback 7-12-2018
Surgical consultation - Pass
Old recall of 30 something woman who wants a referral letter to surgeon so that she can have
her breast reduced/removed.
Task: History (4 mins)

Explain motivational factors for her concern

Further Management

2 min thinking: I couldn’t think clearly because of the previous station

History
-Greeted the patient. What brings you here today? “Please write a referral letter for me, I need
to see a surgeon” May I know that reason? “i want my breast to be removed/reduced. They are
so large to the point that it has become quite embarassing for me” Right. So, do you often have
comments or remarks about it from other people? “No” Ok. Any other parts of your body that
you are not satified with? “No, just my breast”
-Are you happy with your weight? “Yes”

-Do you have any rituals that you think you need to do everyday? “I don’t understand” Like
checking yourself esp your breast? “No”
-So, are your enlarged breast affecting your daily routines in some ways? “… (I don’t remember
what the patient said)”

-Home situation – broke up with partner 4 mths back and father died 6 mth back. Do you think
your current idea about large breast started after that? “Yes, I think so”

-Financial status?; No depression; not enjoying life; no suicide ideas; not using drugs; nor
smoking or drinking a lot;

-GH – is good; have you ever had any issues with ur breast in the past? Any redness, heaviness,
pain or swelling in ur breast these days? “No” Do you have to change the bra size? “Yes”

Explanation of motivational factors

*name*, i can see you feel quite frustrated about your large breast and you want surgery to
reduce/remove it. However, from my point of view, I would like you to try a different method
first. From your story, apparently, you have been going through a difficult phase of your life; your
dad passed away 6 mths back and split with your partner 4 mths back. So, I wonder if these
factors might be responsible for your desire to have surgery. That is why I want to go with a
different approach that does not need an operation at first place. How does that sound? “….
(remains silent, bowing her head down)”

What I would like to do is to refer you to trained psychologist first for you to have a little nice talk
therapy for everything that has been going in ur life. We will help you to overcome these first,
and then see how things will go from there. Hopefully, after this therapy, your perspective on
your breast may change, but if not, we will definitely consider surgery then and refer you to
surgeon. How does that sound to you? “ok”

If you wish, I can contact social support group and see how they can help with you as well. Bell
rang.

GS – 4

Key steps 1, 2, 3 – all covered Approach – 4

History – 4 Dx and DDx – 3 Patient counselling/education – 3

(Here I think my overall performance was good. I thought I would get great marks for that but
obviously, I barely passed this one. The task didn’t tell us to tell dx/ddx so I just avoided the term
BDD intentionally. But according to the feedback, we should at least mention this briefly as AMC
gave only 3 for this category. For counselling, I don’t know where I went wrong though I believed
I did pretty well. The rapport was good. The patient looks depressed throughout the consultation
and did cry sporadically but at the end, she smiled and showed thumb up)
82-Irritable baby (Irreducible Hernia)
3 months old baby (or any age) girl brought in by mom to rural hospital ED concerns about her
baby crying being irritable and crying a lot since 6 hours. Vomited twice. Brother has gastro.
Tasks
-history
-PEFE
-dx/ ddx
-management

History
1-Crying questions
-since when? How often?
-is he crying more in the day, night or both?
-how does it affect his sleep?
-is it getting worse?
-is it related to feeding?
-is he drawing up his legs while crying?
-does he turn pale or blue while crying?
-anything make it better or worse?
-is this the first time?

2-Vomiting questions
-How often
-amount?
-colour? Greenish or not?
-content?
-is it forceful?

3-Bowel questions
-has he passed poo? Any diarrhea or constipation?
-any red stools?

4-any fever, rash?

5-Dehydration questions
-is he alert, drowsy or sleepy?
-are the number of wet nappies as usual or reduced? Any change in the colour of urine? Is it
smelly? Crying on passing urine?
-Is he feeding well? Is he breast or bottle feeding? How often?

7-specific questions
-has he had any recent flue or infections? (intusseption)
-any lumps or bumps in the body? (hernia)

8-BINDS and support


Physical examination from examiner
1-General appearance (DR PJL)
2-V/S
3-growth chart
4-ENT
5-Full systemic focus on abdomen
*Inspection (distension, masses)
*palpation (tenderness, masses, RIF emptiness, rigidity)
*auscultation (bowel sounds)
*Hernial orifices (examiner will give you picture of lump in the groin) so ask: reducible or not,
tender or not, any overlying skin redness.
genitalia and anal inspection
6-office tests

Differential diagnosis
1-Inguinal hernia (reducible or irreducible depends on the case )
2-intusseption
3-volvolus, malrotation
4-pyloric stenosis, duodenal atresia
5-hirschuprungs
6-gastro/ UTI

Explanation
-from history and examination, your child most likely has bowels obstruction due to inguinal
hernia.
-this is our tummy wall and if there is a weakness in the wall of the tummy, tummy content can
protrude out of its position,
-sometimes the swelling can go back by itself but if it doesn’t go back then it can lead to bowels
obstruction leading to tummy pain, vomiting, swelling we call this obdtructed inguinal hernia.

-it is good that we picked it up eary as if left untreated, the blood supply to the gut can also be
blocked and if that happened the gut becomes damaged and the condition becomes life
threatening. We call this as strangulated inguinal hernia. There will be also risk of perforation
and serious infections.
-other possibilities are intusseption (abnormal folding of the gut), malrotation, narrowing of part
of gut like pyloric stenosis, duodenal atresia but these are less likely from hx and Examination.

Treatment
-this is an emergent condition, need surgical intervention to repair the hernia.
-I need to refer him to tertiary hospital to be seen by pediatric surgeon.
-in the meantime, please do not feed him anything by mouth. I will secure IV line and start him
on fluid, get some blood tests as well. I am going to insert a tube through the nose ti his gut to
release the pressure building up inside.
-let me assure you he is in safe hands and he will get the best possible care. Outcome is very
good with treatment.
Case (4/10/2017)
Abdominal pain –Pass
You are a HMO - ED (in a rural or regional hospital). 3-month-old infant had vomited twice
today and cries excessively. Father has brought the baby.
Tasks
History
Relevant PEFE
Give diagnosis/DD
Explain management to father
Approached with stability question as this is an emergency. Results were quite ok as I
remember.
-In history you find a previously healthy infant being increasingly irritable from yesterday, crying
excessively 6 hours. (onset and duration)
-Asked pattern of cry, drawing up of legs? no. (pattern of crying)
-Vomited twice. Once greenish as I remember. (vomiting)
-Refuses feeding now. Last feed few hours before (exclusively breastfed). (feeding)
-No fever. Not lethargic or drowsy. No rashes. (fever/ rash/ drowsiness)
-Bowel habits not changed. Urinary habits- normal. When asked about number of nappies and
whether they were wet like usual – he said all that is normal. (bowel and waterworks)
- BINDS normal /thriving well (BINDS)
-previously had infrequent posseting like symptoms but never had this type of vomiting.
(PREVIOUS EPISODES)
-Did not notice any abdominal distension. (abdominal distension)
- Brother had gastroenteritis recently. (gastroenteritis in family)
PEFE:
-Went on starting with does he look ill? Irritable/ drowsy? VS?
- Asked specifically for CRF/ cold peripheries/ mottled skin. I faintly remember he had latter two.
Not too sure. Got no other signs of dehydration.
- No rashes.
-Abdominal exam: Inspection -distension??? (Not sure). Palpation /percussion / auscultation /
Genitalia- all normal.
When mentioned on hernial orifices examiner gave a picture that looks like inguinal hernia.
Asked is this reducible? Examiner said no.
Other systems normal.
Explained diagnosis as inguinal hernia that has got obstructed causing symptoms of bowel
obstruction. DD: can’t remember what I mentioned at the time. May be Gastro / bowel
obstruction due to volvulus- malrotation/ pyloric stenosis/ GERD/ UTI / intussusception…..
Management: Mainly stressed that this is potentially fatal if not treated surgically ASAP.
Explained ischemia/ bowel necrosis on obstructed site / perforation / peritonitis very concisely,
but I think this was too much as I ran out of time for this case.
Reassured that we will send to nearest tertiary hospital with necessary facilities as soon as
possible after stabilising the patient.
Prognosis very good with prompt treatment. Started explaining IV cannula/ Ix etc. in
immediate management but the bell rang. 4/4 key steps covered.
Recall 6-6-2018
GP, (3-6 months old cannot remember now)irritable child, hernia. Lil girl child, Examiner shows a pic , n
lump in her er groin, not reducible

Recall 11-7-2018
Mother with crying baby(hernia pic given)

Recall 6-8-2018
Inguinal Swelling in child�##Inguinal Hernia##�

Feedback 6-6-2018
Station 12 incarcerated hernia-pass

3 months old baby girl, mother concerns, baby irritable and crying since 6hrs ago, vomiting
2episodes already, brother has GE recently.

Task: hx, PEFE, DX, Mx or initial mx I don’t remember

History: check with examiner for HD stability. child crying for last 6 hrs, first time, ? intermittent,
drawing legs on crying nil, pale on crying nil, crying on touching tummy not sure, no abdominal
distension, vomiting present colour content( I could not remember), last stool this am, normal.
No lump bump on the body, lump on tummy on feeding nil, not feeding well since this am, no
urinary symptom, no fever, rule out infection ,rash nil, happy family. BIND normal. ( could not get
the hernia from the history, mother did not notice any lump on groin or tummy)

PEFE : general –irritable , moderate dehydration, Temperature a little bit elevated, pale on
looking , no jaundice, cvs ,respiratory system normal, abdominal ex – no abdo distension,
palpation : examiner said tenderness not sure because of the child is crying, no organomegaly,
bowel sound not sure because the child is crying.

(started to explain the mother, I don’t know what is the dx, decided to explain the mother about
all ddx of crying baby, then I remembered suddenly and turned to examiner again and asked for
hernia orifices –examiner gave me a photo at that time, of inguinal hernia, not reducible)
Explained mother about incarcerated hernia with diagram and NPO, IVF, blood tests, need for
urgent transfer to hospital, specialist review, investigations and possible need of urgent sx.
Reassure the mother.
Feedback 26-10-2018
Station 12: Abdominal Pain (Hernia) Passed (GS – 6)
You are a GP. Parents are quite concerned about their irritable girl, who vomited twice.
Hx, PEFE, Dx and DDx, Mx
Hx:

 irritable baby (how long and how frequently she cries)

 infection (fever, jaundice)

 intussusception (draw up legs or turn blue when she cries, any mass in tummy)

 hernia (any mass in groin) – Yes there is mass in groin, noticed this morning.

 Trauma

 Severity – feeding well now? Any colour change or blood in pee and poo

PEFE

 GA (any pallor, J, dehydration status)

 VS

 abdomen exam (N)

 hernia orifices (examiner gave me a pic – lump on left groin, I asked examiner whether it
is reducible or not and he replied “no, it’s irreducible”)

DX explain inguinal hernia, which is irreducible type.

Mx:
It’s emergency condition. I need to transfer baby (name) to hospital immediately. otherwise, can
lead to serious complications (bowel obstruction, septicaemia).
Appreciated that you brought in baby on time. She is safe now.
explained briefly possible Mx in hospital. (admit, seen by child specialist, may do some blood
tests and imaging such as USG. Once it’s confirmed as irreducible inguinal hernia, surgery is the
first line Mx. You will be explained details every step at hospital. Please go to hospital
immediately, I will write referral letter for her.)
83-Infantile colic
6 weeks old baby crying and his mother concerned
Tasks
1-take history
2-physical examination card examiner
3-diagnosis and differential diagnosis

History
1-Crying questions (= tummy pain)
-since when? How often? (Since he was 2 weeks of age, every 3-4 hours)
-is he crying more in the day, night or both? (All the day but mostly in the evening)
-is it getting worse?
-how does it affect his sleep?
-is he drawing up his legs while crying? (No)
-does he turn pale or blue while crying? (No)
-anything make it better or worse? (No)
-is this the first time? (Yes)

2-Vomiting questions (normal)


-any vomiting?
-does he throw up milk all the time? (to rule out reflux)
3-Bowel questions (normal)
-any problems with motions? Any diarrhea or constipation?
-any red stools?
4-any fever, rash? (No)

5-Dehydration questions
-is he alert, drowsy or sleepy? (Normal)
-are the number of wet nappies as usual or reduced? Any change in the colour of urine? Is it
smelly? Crying on passing urine? (Normal)
-Is he feeding well? Is he breast or bottle feeding? How often? (She thinks it is enough but her
mother in law saying that she did not feed the baby well, so she stressed out)

7-Key questions
-has he had any recent flue or infections? (intusseption) (No)
-any lumps or bumps in the body? (hernia) (Not sure)

8-BINDS
-any problem with birth? (No)
-is his Immunisation up to date? (Yes)
-nutrition (already asked)
-is he thriving and growing well? (Yes)
-any siblings with similar symptoms? (no he is my only child)
-do you have enough support? (Live with husband who is working and not getting help from
him)
-how is your mood? Your sleep?
Physical examination card from examiner
-thanks for the information, I just need to examine your child now I’l ask the examiner. (Examiner
will give you card all normal except birth mark behind the neck)

-I examined your child let me assure you that I could not find anything serious, all normal. I could
find a birthmark behind the neck but this is a normal.

Note/ Infantile colic: episodes of crying more than 3 hours a day for more than 3 day a week in
healthy baby.

Explanation
-most likely your baby have a condition called infantile colic. Normally babies cry on an average
of 2-3 hours a day, in colic they cry excessively. Let me assure you that it is not serious and it is
common in this age group. Some babies are extra sensitive to various stimuli either internal like
if baby is hungry, tired, or external such as noise, over heating or lighting. it is completely
harmless and self-limiting condition, most likely disappear by 3-4 months of age.

-other possibilities could be:


reflux but less likely as he is not throwing up milk.
Urinary tract infections, gastro or infection of the gut, ear or throat infections, or any viral
infections but unlikely from history and examination.
Hernia or weakness in the tummy wall but no lumps in examination so unlikely.

Management (not a task but just in case)


-it is advisable to maintain a cry diary and see if it is related to food or not.
-you can minimise environmental stimulation by having low-level background noise, soft lighting
and comfortable temperature.
-you can also do the 5 S techniques:
swaddling,
side or stomach position
shushing
swinging
sucking
-take him for a ride in car as sometimes help.
-continue breast feeding and space feeds to 3 hourly or more as sometimes it might be due to
lactose overload
-family meeting + social worker
-Red flags: any vomiting, fever, rash, bloody stool, looks unwell seek medical help.
-Review regularly.
-reading materials

I think the error:


1-not being reassuring
2-blaming the breast milk and agreed with her mother in law or say colic could be due to
inadequate feeding.
Feedback 21-2-2018

CASE 1: PAEDS -IRRITABLE BABY- PASS

6 Weeks old baby brought by mom with iritable cry


tasks:
history, PEFE, management with mother
Asked about the cry pattern- intermittent lasting 30- 45 minutes, both day and night mostly
night, not associated with symptoms like colour change, fever, altered bowel or bladder; no
cough, runny nose, pulling of ear, rash, swelling; active child otherwise.
Breastfeeding- her mother-in-law thinks inadequate breastmilk with mother; so, I asked about
the feeding pattern-how long , both breasts, engorged while feeding, feeding techniques. Asked
whether she is comfortable with the techniques, she was not happy; mentioned midwife
available to check and help her, will arrange shortly.
BINDS normal; mom mood good and support needed as shifted to new place –arranged for help
PEFE: card given , everything normal some pigmented lesion not remembering exactly like the
birthmarks ? port wine stain

MANAGEMENT: reassured her, feeding technique, support. When finished examiner asked
anything else, so I mentioned the birthmark is normal

Feedback 6-6-2018
Station 15 irritable baby/ post partum blue- pass
6 weeks old baby, crying and irritating mostly in the evening, cry for 3 to 4 hours. (long scenario,
I don’t remember detail, however it suggested of irritable baby/infantile colic) Mother still
concern.
Task: history, PEFE card, dxddx
History: mother teary from beginning, reassure her. Confidentiality. Mother gave typical history
of irritable baby, asked other causes of crying baby like infection, bowel pblm- none positive.
Breastfeeding, she thinks it is enough but her mother in law saying that she did not feed the
baby well, so she stressed out. Tear out, reassure her again. First baby, no complications during
pregnancy, delivery and puerperium.
Live with husband who is working and not getting help from him, no financial pblm. Mood (dont
remember how did she answer but she is not depressed), appetite normal, sleep impaired
because the child crying at night too, no depression symptoms, no psychosis symptoms, no
weather preference, insight /judgement normal.
PEFE card given by examiner: of baby, everything normal.
Explain about irritable baby, common condition, child at this age is very sensitive to external
stimuli like…. explain the other possible causes of crying baby, all of which are unlikely in ur child
case based on the history and examination finding. Also , I realize that it is really hard for you to
manage this situiton by yourself , common , with first child and no enough family support, might
have a condition we called it postpartum blue. Will refer you to social worker , center link. You
are not alone.
Feedback 19-7-2018
STATION 17 PASS ( key steps all yes Score6,6,6,5)
Irritable Baby( again long scenario that mother came for 6 week checkup and worried as her
baby cried too much)
Task Hx /PEFE /DX.
I started with greetings and support at home than asked all question of Infantile colic And DDs of
excessive cry, pregnancy, delivery, BINDS, and depression in mother.

PEFE card was given by examiner, I read loud with the patient that I examined the baby and all
normal.

Gave Dx of infantile colic and gave other DDs as well and explain 5Cs of irritable baby.
84-Drug-induced Hepatitis
45 year old nurse who works in the same hospital as you, you are the HMO. About two weeks
ago, she had sore throat, and now she develops yellowish skin. She also complains of pruritus
and dark urine.
TASKS
-Focused history

-PE from examiner/ Perform PE for 4 minutes

-Diagnosis and Management


AMC case
A forty something man 2 weeks after a sore throat yet feels ill and tired. Today he has noticed
that his urine has darkened.
Tasks
-Take focused history.
-Ask for examination findings from the examiner.
-Explain the differentials to the patient.

Differential Diagnosis:
1-Viral hepatitis

2-Alcohol-induced

3-Drug-induced

(Phenytoin, Carbamazepine, Lithium, Erythromycin, Flucloxacillin, Methotrexate, Methyldopa,


Sodium valproate, OCPs, Paracetamol overdose, Azathioprine

4-Chronic liver disease

5-Biliary tree disorder (Choledocholithiasis, Cholangiocarcinoma)

6-Cancer of the pancreas

7-EBV

History (AMC case)


can you tell me more? (I am not getting better doctor)
1- Sore throat

-How is your sore throat now? (good)

-Are you feeling too tired? (yes)

-Did you take any medications for it? (Yes, I went to his GP and he prescribed some medication
and the sore throat is gone)
-do you remember what type of medication did you take? (He will show you a hand written
prescription, Augmentin was prescribed)
2-Dark urine

-since when? (today, if not present in the stem)


-What do you mean by dark urine? Is it dark yellow, or reddish? Any fresh blood?

-Any burning or stinging sensation?

-Any excessive exercise? (rhabdomyolysis)

3-associated symptoms:

-Any yellowish discoloration of skin, or change in colour of urine or stools?


-Any nausea, vomiting? Fever?

-Any abdominal pain?

-Any loss of appetite? LOW, lump or bumps?

-any joint pains, any rash?

-Any Itchiness? When did it start? Did it start together with the yellowing, or after the yellowing?
Is it itchy at any particular time of the day, especially after bathing?

4-risk factors questions

-Any recent intake of possible contaminated food and water?

-Any recent travel?


-Any blood transfusion?

-Any past history of tattoos, piercings?


-Any IV drug use?

-Confidentiality statement; Are you sexually active? Are you in a stable relationship?
Before this, did you have any other partners? Do you practice safe sex?

What type of sexual activity do you engage in? What is your sexual preference?
-Any recent exposure to sick persons or known hepatitis cases?

-Any past history of any liver problems?

-Any conditions that run in the family, especially affecting the liver?

SADMA

-Do you smoke?

-Do you drink alcohol

-Do you take recreational drugs?

-Do you take any over the counter or prescription medications?

-Any known allergies?


Physical Exam from examiner

1-General appearance:
- jaundice, pallor, icterus, rash, LAP
-IV drug marks
-parotid enlargement, fetor hepaticus, spider nevi

2-Vital signs: pulse, BP


3-ENT especially throat and thyroid
4-chest and heart
5- Abdomen:
-signs of portal hypertension
-hepatomegaly (yes so ask: size, surface, margin regularity, consistency, tenderness)
-ascites

-Inguinoscrotal: hypogonadism (hemochromatosis)


6-Office test: UDT, BSL (bilirubin +ve)

Diagnosis and Management

-Based on the history and physical examination, most likely you are having a condition called
drug induced cholestatic Hepatitis; which is inflammation of the liver with obstructive pattern
that could be caused by the medications that you took for your sore throat. Like any medication,
this medication also has side effects. We always weigh the risk and benefits before giving the
medication, and in your case, treating your condition with this medication was more important.

-it could be other causes like viral hepatitis, gall stone, pancreatic cancer, pancreatitis but they
are less likely.

-stop taking the medication if he still taking it and I would like to do further investigations to
confirm and rule out other causes like:
*FBE, ESR/CRP.
*LFT, Hepatitis serology A,B,C,D, HIV serology, PCR viral load.
*hemochromatosis studies/iron studies, alpha-1 antitrypsin deficiency (congenital deficiency of
this is associated with liver disease), autoimmune hepatitis screening (ANA, antimitochondrial
antibodies, anti-smooth muscle cell antibodies),
*EBV and PSGN tests as it can rarely affect the liver.
*I would also like to do an ultrasound of the abdomen.

-Please stop taking the medications and stop OCPs until we get to know what is happening.
-It is very important to do lifestyle modifications.
-Please do not drink alcohol at all as it can affect the liver.
-And please practice safe sex always.
-And be careful in eating contaminated food and water, especially shellfish.

-If you develop fever, worsening abdominal pain, or the yellowish discoloration progresses,
please come back immediately.

-reading materials. Review once results come back.


Recall 1-3-2018
Gp setting, Guy who took tt for tonsillitis: amox and clavulanic, presented with dark urine.
When asked only gave h/o peripheral itching. Vitals normal. No proteinuria, urine billirubin
positive, neg urobilliongen, no proteiburia.
Looks like a case of drug induced hepatitis secondary to amoxycillin.
Task was take hx, diagnose and mx.

Recall 21-6-2018
A forty something man 2 weeks after a sore throat yet feels ill and tired. Today he has noticed
that his urine has darkened. Take focused history. Ask for examination findings from the
examiner. Explain the differentials to the patient. (Drug induced hepatitis Augmentin

Feedback 21-6-2018 Malaise/itch :Pass: Global score:5


A forty something man 2 weeks after a sore throat yet feels ill and tired. Today he has noticed
that his urine has darkened.
Tasks
-Take focused history.
-Ask for examination findings from the examiner.
-Explain the differentials to the patient. (Drug induced hepatitis Augmentin)
the role player complaint that he is not getting any better, in spite of using medication. I asked
him to explain more about “not feeling well” he had change of colour of his urine, so I asked
more details about change of colour, whether it was like fresh bleeding or very dark yellow, or
coca cola coloured. Then asked about the stool colour change. [ so confirmed that the change of
the colour of urine is most likely related to hepatic reasons rather than renal reasons.] then also
asked about dysuria or any problem with urinating. Then also asked about itching, change of
colour, loss of appetite, abdominal pain, nausea, also asked about his sore throat, he said that
sore throat is gone, so I asked if he had used any medication for it and about the accompanying
symptoms of sore throat. Then he said that he went to his GP and he prescribed some
medication and the sore throat is gone. Then I asked about medications. He showed me a
hand written prescription, Augmentin was prescribed. I asked about duration of the use of
Augmentin, and the time of the onset of itching and malaise. Also asked him about his
occupation, smoking, alcohol and recreational drug use, recent travel, contact with anyone with
similar symptoms. I forgot to asked about sexual history (safe sex, stable relationship). Then also
asked about previous medical conditions and medications and allergies. then asked the examiner
about examination findings:
In general appearance, jaundice, anaemia, dehydration and lymphadenopathy. vital signs,
especially temperature and blood pressure
in chest, heart and lung examination equal air entry and normal heart sounds.
In abdomen, obvious distention and mass negative/
In palpation, the examiner said that there is no tenderness in light palpation, then I asked him
what about tenderness in deep palpation, especially in Right Upper Quadrant, then he asked
what I was looking for, so I remembered that I should Rule out Morphy sign as well. So I said I
want to rule out morphy sign and I am also looking for signs of inflammation in the liver. So he
answered that Morphy sign is negative, and said every detail about size, tenderness, consistency,
regularity of the liver. I also checked the spleen, and the skin rash on the abdominal wall. Also
checked for gynaecomastia, ten checked the legs for possible oedema. I do not remember
whether I checked the hands, or I did the per rectal examination, in the office test, I am not sure
about the results. But I think blood and nitrite were negative and bilirubin was positive.
Finally explained to the patient that his symptoms are compatible with a hepatic inflammation.
It could be due to many reasons such as viruses, or obstruction because of gall stones, and also
it could be due to immunologic reasons. But as well, it could be due to certain drugs or toxins,
and according to his history it is very likely that it be related to Augmentin as Augmentin can
cause a hepatitis with obstructive pattern.
So I asked him to stop taking Augmentin and offered him more investigation for confirming the
diagnosis and more importantly ruling out other possibilities for hepatitis.

Feedback 15-8-2018
Scenario: Malaise/ Itch
Stem: Patient coming with dark urine and had sore throat 2 weeks ago.
Tasks:
~take history
~pefe
~dx and ddx to patient.
In the room a man in late 30s was sitting on chair. Greeted and asked to tell me more. He said I
am unwell since few days. I had this sore throat etc and 1 or 2 days back I noted dark urine.
Took history about dark urine(ruled out ddx that I had uti, psgn, hsp, and cancer(pancreas,
bladder), viral hepatitis, gallstones, trauma, blood thinners, bleeding disorders, medicines etc ).
Took history of sore throat ( there he told he took antibiotics) explores more about antibiotic.
He said I have stopped taking it and now my sore thorat is better. Asked about pale stools,
itchiness, anorexia etc (he said yes to everything).
(Pls note no antibiotic use and clay stools given in stem,it was meant to be explored by
candidate. Iam writing this cause I came across few discussions on the group with these findings
given in stem)
Pefe:
Asked in usual format from start and found icterus + Liver enlarged Urine dipstick: bilirubin +
Gave dx of drug induced hepatitis and explained it.. and ddx of viral hepatitis, psgn, uti, hsp etc
all that mentioned above in history…
Grade: Pass

Feedback 1-3-2018
dark urine post amox for tonsillitis. PASS
i dont know how i passed this as i spent 4 min thinking its post strep glomerulonephritis lol
i read the stem it said guy had sore throat n took amox now developed dark urine and i went in
with tunnel vision !!!worst thing that can happen to you!!
after wasting time i finally realised my mistake and then made up for it quickly ruled out all
causes of conj hyperbilirubinemia
in pefe there were scratch marks , and dipstick had bilirubin
i explained the diagnosis sayin its hepatitis which can cause this picture or it could be duct
blockade too but you dont have pain so its unlikely .
85-Splenic trauma
Case 1
30 yr old female , met with an accident, another car crashed into her car from side, patient was
retrieved, brought to the ED, primary survey done, reviewed by surgical team, found to have
multiple rib fractures on left side and a spleen laceration, decided to manage conservatively. 3
hrs later , patient complained of increasing pain and you are asked to review the patient.
Observation charts give inside the room , patient lying on the bed with a drip running
Tasks
-Explain the d/d to the patient
-Management

Case 2
A woman had RTA, CT chest and brain done normal. Cervical spine imaging normal. Fast scan
done showing grade 2 splenic lacerationand minimal fluid in abdomen. Patient shifted for
observation. Chart given shoeing BP falling down and HR increasing
tasks
-explain condition to patient
-management

Approach
1-Check stability
-Ask about hemodynamic stability (one case the patient is unstable from the stem and other
case the patient is stable but when you entered and ask it turns unstable).
-I would like to secure IV Cannulae, take some blood samples for FBC, blood group and cross
matching, LFT, UCE, Clotting profile, BSL. Start him on IV fluids and hook to monitor.

2-Inform patient about current condition


-Hi John, my name is ---- I am one of the doctors here. I can see that you have had a road traffic
accident. I am sorry to hear that. Are you in any pain at the moment? (yes) do you have any
allergies to any medications? all right, I am gonna arrange painkillers for you.
-look john, on observing your chart, it seems like your BP is falling down and heart rate is going
up means your heart is working faster. I was a bit concerned about these changed as it indicates
you are losing some blood.
I attached IV cannula, took blood samples and started you on IV fluid as well and I informed the
registrar to come and see you.
Let me assure you are in safe hands and we will try to give you the best possible care.

Note/ another approach is to ask the examiner first then give you chart after that inform patient
about the chart and assure he is in safe hands then tell him about the need of fluid and Ix etc…

3-talk about causes of bleeding


-well john the bleeding could be from anywhere in the body. It could be injury to the lung,
spleen, any body organs or vessels.
-you have had an accident and imaging has been done for you. CT chest and neck were clear.
However the abdomen CT showed an injury to the spleen which I am suspecting the most likely
cause of the bleeding.
4-Explain and Management
-spleen is an organ of fist size located in the left side of your tummy and it is part of our immune
system. It has multiple roles. It filters the blood from old RBC, helps fighting infections and bugs.
-it is prone to injury during blunt or penetrating trauma to tummy. However due to its important
role we usually keep you for observation as there is a chance of bleeding to stop itself.
-in your case the bleeding reoccur and make your condition unstable that is why I informed the
surgical registrar who will take you to operative theatre for immediate removal of the spleen in a
procedure called splenectomy. They might decide to do some imaging as well.

5-talk about splenectomy


-surgery is either key hole or open. It is performed under GA. By experienced staff.
-like any surgery complications are infection, bleeding, injury to other structures, risk of
anesthesia but as this will be done by experienced staff so such complications will be unlikely.

6-Long term management


-body can live without spleen, but as it helps to fight infections so you will be at high risk of
catching infections. For this reason, you will be given vaccines and antibiotics to prevent that.
-other complications are clotting problems.
-you will stay a while in hospital and other advice will be given by the specialist.

Case (2/6/2017)
Grade 2 spleenic trauma
laceration in spleen , vitals were ok outside , Inside - O2 sat were going down 88% , BP going
down , Pulse going up.
Check the pt in the resuscitation cubicle, call for surgical reg consult, under their supervision,
pass IV line , take blood group and cross match, do base line investigations, make pt NPO, pass
NGT if needed, start IV normal saline, pre op antibiotics. Arrange for cxr and abdomen x ray, or
fast USG scan. Give adequate analgesia, call OT, and arrange for laprotomy, as patient is unstable.
Also catheterize the patient after checking for blood at urethral meatus. Monitor input and
output.
DDs :
splenic rupture
Lung contusion
Rib fracture
Diaphragmatic rupture

Case (10/2/17)
MVA 7 hr ago. Fast US +, CT scan shows left ribs fracture and spleen (laceration) . The Mx was
concervative , Nurse called you, PX complaining on the severe chest pain. The vital chart is given
outside but (blurry pix ) Task: explain findings, Mx and DDs. Vitals-BP trending down and HR
trending up, tachycardia, tachypnea. Chest pain DDs: haemothorax, pneumothorax, pleural
effusion, spleen injury , cardiac tamponade. Ix: ECG, Xray, repeat CT.I failed this case ( I cant help
u with it
comment pass
The one I had was a trauma patient who had initially been stabilised and diagnosed with a
splenic laceration. The surgical team had decided to treat him conservatively. You were given a
chart outside the room showing deteriorating BP and tacychardia. The task was to talk to the
patient about his further management and possible cause and differentials.
Went in introduced myself my role. Explained that I was concerned about his changing vitals and
that he might be loosing blood. He was hooked up to an IV. I mentioned that we would be giving
him continuous fluids, didn't really get into which kind. Mentioned that we were hooking him up
to a heart and oxygen sat monitor. Also that we would be giving him oxygen if required. Said that
I was going to chase down his labs check the group, crossmatch and arrange for blood of
required. I also mentioned that we we're going to put in a catheter to monitor his output. I then
told him that once he was stabilised we would consider imaging but if he doesn't the surgical
team might need to take him directly to the OR. I then went on to talk about the possible causes
while all these steps were taking place, mentioned a bleeding from the laceration, damage to
the lungs, colon or a hidden abdominal bleed. pointed each structure out to him and explained
where the bleeding could be coming from. I made sure that his family was present, kept
reassuring him and emphasising that we would stabilise him first before any imaging. Because I
had time left I then focused on the most likely cause, told him the possibility of a splenectomy
and briefly told him what a spleen was, why it tends to bleed and get injured. I then quickly went
over the short term and long term complications.
I mentioned that I would continue with the fluids he was on at the moment and keep monitoring
his vitals. I mentioned we might need to transfuse if he didn't respond. Didn't go into the details
of which fluids. Just glanced at the IV I just mentioned CT. I mentioned infection bleeding
anesthesia reaction intially. Told how after the operation he would be at more risk to certain
infections and that we would immunize as well as give antibiotic prophylaxis. This was fairly brief
because I did this after I was done with the rest of the counseling and I had time left.

Feedback 5-7-2018
30 yr old female , met with an accident, another car crashed into her car
from side, patient was retrieved, brought to the ED, primary survey done,
reviewed by surgical team, found to have multiple rib fractures on left side
and a spleen laceration, decided to manage conservatively. 3 hrs later ,
patient complained of increasing pain and you are asked to review the
patient. Observation charts give inside the room , patient lying on the bed
with a drip running
Tasks
Explain the d/d to the patient
Management
Patient lying on bed.
I started asking patient to scale pain and she said severe .. so asked any
allergies? And then gave painkillers.
asked about hemodynamics the examiner gave me the charts which
showed hypotension and tachycardia. My differential were , rebleed from
the laceration, pneumothorax and heamothroax. Bowel perforation.
in management I explained I need to call surgical registrar, admit patient.
Make sure she is stable. send blood for cross matching. Do ultrasound, x-
ray., if its pneumo/hemothorax a tube will be put in,chest to drain. If from
spleen it should be removed and given antibiotics and also briefly talked
about post splenectomy –vaccination,antibiotics, travel guidelines..
Feedback 8-2-2018

Recall of 7-3-2018
Spleen injury ( fast US included)
Feedback 5-12-2018
Splenic laceration/ splenectomy
Pt after MVA was on conservative mx for splenic laceration. C/O now left
Upper Quad pain and chest pain also. Observation chart given all vital
stable. BP110/70, slightly downgrading from previous reading.
Task
1. Tell the possible reason.
2. management (can’t remember specified immediate mx or not)

Now its bit different than old recall, which used to come as shock. So we
used to start with DRABCD protocol but here there is no shock.

SO I went in and introduced, I understand X you having tummy pain and


chest pain. How you feeling right now? She said I still having bit pain (In
face there was no sign of much pain or discomfort, here I did not ask to
rate pain or offer pain killer). Don’t worry we will take care of it. Now X,
along with your pain, I also observed your chart showing your bp Is bit
falling. That’s making me concerned that probably there’s some ongoing
bleeding inside your tummy. It can be from any organ in your upper tummy
and chest. As you know from the earlier scan we found an organ called
spleen in your upper left tummy was injured, I am suspecting its spleen
where its bleeding. Let me show you. (I drew and showed). Here is your
spleen, there’s some other organ like gut, liver, kidney as well I showed.
Now bleeding can occur from these organ or also from fractured ribs in
your chest. To be confirmed we need to do urgent scan .I am calling my sx
team. If it is your spleen that is bleeding , most probably they do an opn to
try to stop bleeding if not possible unfortunately Sx team may decide to
remove your spleen.( here I should mention the mx if its haemothorax
which I did not.. may be that reason lost my key step)

If spleen is removed there is some immediate and long term


consequences. Like every sx has risk there is risk of injury to surrounding
organ and bleeding. Removal of spleen will weaken our immune system
you will need to have AB for long time.. (here examiner told me “ I
recommend examinee to review the task again..I turned to examiner
thanked her.. I looked at the task .. tell the probable MX plan. There was no
mention of any specific immediate or long term mx). I was bit confused but
I did not ask any thing to examiner.. I started again from immediate Mx
(thought she probably did not hear my immediate mx). I finished saying
longterm MX AB. Vaccination, Precaution against infections. Don’t worry
we will try our best. I still had time thanked examiner and pt and went out.

Key step 1234: yes yes no yes

Approach: 4

Dx/ddx:4

Mx:4

86-Stroke CT interpretation counselling


You are HMO in ED seeing an old man who lost consciousness a few hours ago with weakness of
left side of his body. He has long standing hypertension and on ACEI and aspirin. There was no
HO of fall. CT scan of brain is done which is similar to following one but in exam it was on rt side.
Wife has come to discuss about her husband condition.
Tasks
Explain CT scan findings to wife
Discuss about possible causes
Explain about prognosis (No management)

-Does the wife has consent to discuss her husband condition?

-Hi Mrs---- I am dr----- I can see that you are here to discuss about john’s condition. You might be
worried about him. Let me assure you that he is now stable and in safe hands, we will do our
best to help treating him.
Before we start do you have any particular concern?

-well, I will explain everything and if you have any confusion please let me know. John had lost
consciousness a few hours ago and presented with some weakness in his left side.
Ct scan has been done to check for the possible cause. The result is now with me let us see it
together.

-this is the head and this the brain. This is the left side and that is the right side. In CT the grey
colour means soft tissue and the white colour means solid structure.
As you see the left and right side are not similar as there is a large circular white area in the right
side. Possibilities are bleeding in the brain, infection or collection of pus inside the brain or nasty
growth of brain tissue.

-from history and examination, most likely he has a condition called hemorrhagic stroke which
most likely happens when a blood vessel inside the brain bursts and leaks blood into the
surrounding brain tissue causing the affected part of the brain to stop working properly.

-there are several causes behind this bleeding. First, he has a history of high BP so it exerts
pressure on vessel wall and cause it to rupture. Could also be due to head injury, aging vessels,
and malformed blood vessels, transformation from previous stroke or even nasty growth.
Moreover, he is taking aspirin, which is a blood thinner medication that can cause further
bleeding.

-as our brain control the opposite side of the body so when the right side stops functioning
properly there will be weakness in the left side of body.

-for hemorrhagic stroke, he will be seen by a specialist neurosurgeon who might consider a
surgical intervention to remove the bleed and might put him on medications to control his high
BP.

-in term of outcome, usually it varies from person to person. With hemorrhagic stroke although
the survival rate is quite low but the outcome for the survivors is generally good.
So careful management is ensuring good clinical outcome.

Q/ but we do not want him to lie in his bed as useless body; we would rather kept him die he
told me?
- Look the outcome will remain guarded but too early for comment on whether he will be in
vegetative state. I totally understand your view but we will also do our best to let him live as an
independent person.

-After acute management and for such patients there is ongoing management when he will be
under care of MDT. We call this rehabilitation phase.

-The team will include physiotherapist who will work with patient to improve his strength,
movement and physical condition, they can also assist him in fitting and using wheel chair.

-Occupational therapist who will help improve performance in daily activities, help him perform
tasks in their daily living and working environment.

-Speech therapist will assess and treat his communication and swallowing problems.

-He will be followed up by his GP to prevent infection or clotting problems also seen by
psychologist and neurosurgeon.

-He need to follow LSM and control his BP.

-so we will try to give him the best quality of life by all possible means.
Recall of 15-3-2018
ED, CT scan of haemorrhagic stroke. Pt taking perindopril for high blood pressure and aspirin. No
injury or trauma history. Been living together happily with husband
Tasks: explain the findings to the husband, dx and possible risk factors in the patient including
the prognosis with reasons. (Pt complained that his wife has told him to make her dead if she
ends up in vegetation)

Approach: 5 C approach to counselling

Karens case. Prognosis will remain guarded but too early for comment on whether she will be
in vegetative state

Case (22/2/2017)
You are HMO in ED seeing an old man who lost consciousness a few hours ago with weakness of
left side of his body. He has long standing hypertension and on ACEI and aspirin. There was no
HO of fall. CT scan of brain is done which is similar to following one but in exam it was on rt side.
Wife has come to discuss about her husband condition.
Tasks
Explain CT scan findings to wife
Discuss about possible causes
Explain about prognosis (No management)

2 mins thinking

- I haven’t read this recall before, this is new for me.

- Anyway, It is all about counseling, I have plenty of time.

- DDx - Haemorrhagic stroke, injury, brain abscess, tuberculoma, tumor

Talking with roleplayer


When I stepped into the room, a very nice while granny with facial wrinkles was sitting on a
chair.
I introduced myself and she asked me “Dr, is my husband gonna be alright?”
We are gonna discuss about it, now I d like to ask my examiner for something. I turned towards
examiner and said “Examiner, does wife have consent to talk about pt’s condition”
(Examiner - Dr, Pt has lost consciousness. This is his wife. All the decisions is up to u now)

Then I turned towards role player, she said “Dr, I have authority to discuss about my husband’s
situation” and smiled at me.

Ok Jenny. I understand ur husband has lost consciousness few hours back. We did the CT scan of
his brain and the results are with me right now. Let’s see it together.

This is ur his CT scan of brain. Basically, in CT scan, white things are solid structures and grey
things are soft tissues. This is the head bone. This is the brain. If we compare two sides, u see
there is a large white thing on right side. (CT scan has the marking R and L) I told R means right
and L means left.

So far, are u following me? (Yes, Dr) Just let me know if u have any confusion, I am happy to
explain it to u again.

So, this white thing is not supposed to be there. Considering his clinical history, it could be a
blood clot from bleeding in his brain we call haemorrhagic stroke.

There are other possible causes, like it could be brain abscess which is infection and collection
of pus in brain. It could be nasty growth brian tumor. Or it could be TB infection of brain we
call tuberculom which is very rare in Australia. But they are very unlikely.

This type of bleeding and blood clot can be caused by head injury but according to his notes,
he didn’t have any. So, most likely cause is stroke which is rupture of blood vessels that supply
brain and blood come out from them. He has high BP and it exerts pressure on vessel wall to
cause it rupture. Moreover, he is taking blood thinner and it can make more bleeding. Our
brain control the opposite side of the body and blood clot is in rt side, so he has weakness in
left side of arm and leg.

I will inform my senior who will come and look at him. He might consider doing another scan
with contrast in which is we ll inject chemical through his vein and take CT scan again. (Coz CT
picture states that it is non contrast, I dun know I am right or wrong)

So far, are u with me? (Yep Dr)

In terms of outcome, it might not be good coz as u see, the blood clot is quite large. But we
still have hope that he can recovers from it. Specialist will decide to do surgery to remove
blood clot and hopefully, he can get well. He may have residual weakness when he recovers,
for that, I will get involved physiotherapist who will teach him some exercise regimen to get
his muscle strength back. On the other hand, unfortunately, he may get into prolonged
unconsciousness.
(RP : If it happens, we dun want him to be lying on bed as a useless body. I want him to die
rather than going this way)

I really understand ur concern. We ll respect ur descision. Let me reassure u that, we ll try our
best to treat his condition and frequently monitor him.

AMC Feedback - Altered Conscious State : Pass (Global Score - 4)


Approach to relative - 4
Interpretation of Invx - 4
Diagnosis/ DDx - 5
Counseling/Education - 4

Feedback 7-9-2018
Station 4 –Altered conscious state - FAIL
.. Intern at ED at the hospital .
CT is given with ( obvious big right frontal lobe grey-white opacity haematoma ) ,
The patient started with headache 6 hours ago and quickly cannot move the whole left side of
the body . . Wife called the ambulance and comes to ED . Taking Aspirin in the stem ( reason , I
forget – heart or previous chronic medical condition ) , denies Head injury in the stem .
TASK:
1. explain patient’s wife about CT result ( ask examiner about power of attorney from the
patient and he said , go on your task ) – explain CT with lay-term slowly as role player is crying
intermittently , provide tissue and water , pause and listen
2. explain possible dx and ddx to the patient’ wife
- most likely intracranial bleeding
3 explain initial treatment plan to the patient’s wife
- blood tests routine – including bleeding profile , including LFTs
Refer to the brain specialist immediately
-possible plans from the specialist – surgery – SEs of the surgery in brief – bell rang

Feedback - Fail

Key step 1 – yes

Key step 2 – No

Key step 3 – Yes

Key step 4 – NO

Approach to patient – 3
Interpretation of invx – 3

Dx and DDx – 4

Patient counseling / education – 2

Feedback
Introduce, then i said sorry to hear the event occurred to your wife and right now our medical
team is trying our best for your wife. We just did imaging test CT scan over your wife
head and the result is with me and i want to explain it to you. Before that do u have any
particular concern?
Here is CT scan result and lets see together. You can white circular line is skull bone, blackish grey
area is our brainand you can already see a white or grey large area here. According to the
symptoms and history, the most likeley cause for the lesion is haemorrhagic stroke, u can say
bleeding inside the brain.
Is it serious? Yes its quite serious. Our team is currently trying theit best. When we say stroke, we
think of two conditions, block stroke and bleeding stroke andunfortunately, in bleeding stroke,
the condition can rapidly deteriorate especially in such kinds of large stroke (yes its large in ct
scan) and patients of 72 yrs age like ur wife
Before that, my wife has once told me to put her deas if she ever ends up in vegetation. Alright
mr., i understand your concern and also your wife's. But rifht now its too early to decide. Our
doctor team will try our best for your wife. Yes, but i will mark your words on my mind. We can
discuss further about it after the result of treatment.
Why did it occur to my wife when she has been quite well till now? I said itd bcoz of long
standing hypertension andaspirin is also a possible risk but not major one because normal
people has been taking it quite well and ok.
Whts thr plan of mx? Now we are trying to lower BP very carefully and when she is stable, the
surgeon team will do surgery. Making a hole right around here ( pointing my temple
area) and then we will suck out the blood clot. If ur wife made it through, there can be residual
muscle weakness and for that we have rehab program. It sometimes takes several months to
several years depending on individuals.
More than 1 minute still left, i asked understanding andarea u want me to cover again. He said ur
explanation is quite straightforward and i can understand. So i told him abt hypertension on
stroke.

Feedback 7-9-2018
CT stroke counselling
Altered Conscious State- PASS)
A CT scan was given- A large white patch at right cerebral cortex. Pt was hypertensive, taking
aspirin, no head injury. You are intern. (Stem was big though, I just only looked for causes of
bleeding in the stem)
- Explain CT to his wife.
- Explain your diagnosis and causes to her.
- Initial MX.
FEEDBACK-PASS
Global Score: 4
Key steps: 3/4
Approach to patient: 4
Interpretation of invx: 4
DDX: 5
Patient counselling: 4
I entered the room, there was an old lady sitting with a tissue paper in her hand. I
introduced myself. After that, I asked her if she had permission to know info about her
husband's condition. (She blankly looked at me. After few seconds, then, she said yes. )
Then, I asked her, if she had any question before I got started. She said I wanted to know
what had been happening with him. ( Her reaction was very slow. I took a pause and looked
at her, she then asked me to start :D )
There was an paper with the stem. I took it and pointed towards the CT. I explained to her;
Look, we have the CT scan with us. This is the picture. This circular part is the bone, skull.
This is our brain. Can you spot this white patch over here? (yes). Most likely, there is some
accumulation of blood in this area. This has cause the symptoms that your husband is
having now.
This condition is called haemorrhagic stroke. Regular blood thinning medication and aspirin
medication can lead to this. As I can see, your husband was taking aspirin for a long time.
However, it is also associated with long standing hypertension, high lipids in blood; all that
can lead to distension of small arteries of our brain. This condition, we called, aneurysm. It's
like a over-expansion of balloon. ( I gestured with my hands) Anytime, they can rupture and
lead to accumulation of blood inside our brain. That can lead to unconsciousness and
altered mental function.
(At this point she asked me to speak slow!! Well, my voice was low. That point, I realised that
I was speaking fast!! May be her slow reaction made me hurried a little bit!!)

I said: I'm really sorry that you have felt like this. I would repeat all as many times as you
want. I can only imagine how difficult this situation for you now. I can come back later on if
you like. I know, I'm giving you a lot of information and it's a lot to take in right now. Please
be reassured, I will give you printed copies regarding this, so that you can give it a read,
anytime you want.Then, I offered her tissue .I paused for few seconds then. I looked at her,
then, she gestured me to start again.
Then, I asked again, would it be okay with you if I explain a little bit about why this has
happened with your husband? She nodded her head. Then, I scribbled ABCDEF in the paper
and started to explain the risk factors.
Alcohol- Would you mind me asking, do your husband drinks alcohol daily? (no,
occasionally)
Blood pressure: was your husband regular with his GP regarding BP measurements? Was it
controlled? (she said yes!) Then, I said we will review all medications again and will contact
his GP as well.
Cholesterol: Do you know, if he has ever checked his blood lipids level? (no)
Diet: how's his diet? Was it a mixed diet? (yes). BMI? (normal)
Exercise: I'm just wondering if he does few stretching/yoga/walking everyday? (yes)
Family hx(no)

Then, I told her, look jenny, I'm just an intern, if you have any further questions, I can
arrange a meeting with consulting doctors and my seniors. We will always be with you and
will explain you everything as we proceed. Your husband is in safe hand. There's a MDT in
our hospital, they are expert to take care of the people with similar symptoms. Alright?
Then, I didn't go for further explanation as she was talking very slowly. I didn't explain
further about mx. I was thinking; Don't say too much! I have already said more than
enough!! this is a counselling case!! I might fail if she didn't like me!! That moment, I
thought, I had failed this case . Then, I started to ask about her.
Is there anyone with you at this moment? Any family members? ( Yes, my sister and brother
in law are coming) I said, very good. If I have permission, we can discuss this issue with all. I
would like to meet with them as well. Anyone lives nearby? Would you like me to call anyone
atm? (no, I'm fine!)
Are you going home now? ( No, I'll wait for my sister) Alright. I just want to give you this
information; if you like, I can refer you to counsellor in our hospital as well. This will help
you to unwind your worries. ( now, she was interacting with me with a good vibe) Would
you mind me asking how older are you? (70 ) Then, I said please take care your health as
well. Please be regular with your check ups.
Finally, Bell rang!!
(Even though, she was speaking slow, I had enough time, I repeated few questions
twice/thrice, especially when I was asking about her.)
87-Hypoglycemia counselling
A truck driver in ED admitted with episode of?? Syncope. Long term Type2DM on treatment with
Metformin. Some weeks before had an infection and found to have increased blood sugar levels,
therefore specialist added another drug gliclazide. Already received treatment and now blood
sugar normal.
Tasks
-Explain to patient diagnosis
-further management.

1-approach
-Hi I am dr--- I am on of the doctor her.
-you have lost your consciousness this morning and your BSL appeared to be low so you had
been given glucagon injection and now BSL level is normal and you are stable.
So sally can I ask you a few questions in order to unravel the nature of the problem?
-how do you feel at the moment?

2-History
(I do not know if history a task or not )

3-Explain reasons.
-there are several possibilities why you have LOC but most likely, you have a condition called
hypoglycemia have you heard about it?
-this is where you have a low BSL of 4 mmol or less.
-this condition can be caused by diabetic medications like insulin or other oral medications like
gliclazide. Other causes could be due to excessive exercising, recent infections, and kidney
problems.
-However, most likely cause in your case is due to skipping your midmorning snack. Because you
are taking diabetic medications, you need to have regular meals and health snacks to keep your
BSL under control.
-someone with hypoglycemia can have mild symptoms like feeling hungry, sweaty, hand shakes
or funny racing of the heart. They can also have severe symptoms like Loss of concentration,
confusion, fits or even Loss of consciousness.

4-Management
-I need to refer you back to your DM physician for further assessment of your medications and
current diabetes. And you will be seen now by the registrar as well.

-so to prevent further episodes of hypoglycemia, you need to check your BSL every day. If
normal, take regular dose of your medication followed by regular meal and healthy snacks.

-if the level below 4 or you experience mild symptoms, you need to take simple sugar like 6-7
regular size jelly beans or a cup of fruit juice and wait for 15 minutes. If still below 4 take another
simple sugar. If this corrected then you can take glycemic snack like a slice of bread or fruit. If the
mealtime is after 1 hour then take your regular meal.

-there is a hypo kit containing glucagon this is needed to be carried with every person having
DM. because if severe symptoms developed, you may need to have injection of glucagon to
correct BSL and call 000.

-I will refer you to DM educator or nurse to teach you how to use glucagon and also your family
members.

-regarding driving, it is important to check your BSL before you drive and make sure it is above 5.
If you were driving and having symptoms of low BSL, you need to pullover and stop your engine
then take your snack and check BSL.

-reading materials, red flags, review.


Case (4/10/2017)
Altered conscious state - Pass
Scenario
A truck driver in ED admitted with episode of?? Syncope. Long term Type2DM on treatment with
Metformin. Some weeks before had an infection and found to have increased blood sugar levels,
therefore specialist added another drug. Already received treatment and now blood sugar
normal.
 Explain to patient diagnosis and further management.
Did not take breakfast today. Has a habit of skipping meals due to work.
Explained management:
Hypoglycaemic drugs, if taken without meals, will invariably cause this problem. So, prevent it by
taking regular meals/ keeping a healthy snack near him all the time.
Explained warning symptoms of hypoglycaemia (hunger, sweating, shaking, palpitations…) and
once detected need to act quickly.
Patient is used to checking blood sugar with glucometer, so if level below 4, asked to take 15g of
simple sugar -with a fruit drink/ snack (keep these in pocket) as soon as hypo detected. Wait for
15 min and check blood sugar again, if still below 4 take another 15g. If more than 4 then take
low glycaemic snack (wholemeal sandwich/ fruit or regular meal if meal time).
Explained that prolonged hypoglycaemia can cause coma state/ death. His family members
should know how to use Glucagon injection if he is unconscious.
Explained sternly the risk of driving the truck with hypo attacks, risk to himself and other road
users. Needs to check his BSL before driving (should be more than 5). Need to keep sugary
snacks to use if he becomes hypo…
He asked can I drive home? I got in trouble here so thought for a while and said we will monitor
BSL hourly, in the meantime I will discuss with my seniors regarding his fitness to drive, will get
back to him later….
Early follow up afterwards. His previous high sugar levels may have been due to infection at the
time, so asked him to regularly check and record his BSLs and will refer to his usual
endocrinologist to reconsider medication and further assessment.
 All 4 key steps covered here.
88- lymphoma
30-year-old lady who work at agency complaining of fever, weight loss and night sweating
tasks
-history
-PEFE
-diagnosis and ddx
-management

History
1-can you tell me more?

2-fever questions.
-since when? (3 months)
-is it constant or on and off?
-how high is it? Did you have the chance to measure it? (37.6)
-any night sweating? (Yes)
-any chills or shivering? Recent infection?
Note/ if night sweats only in the stem without fever then ask since when and after that ask if any
fever and continue.

3-Weight loss
-how many kg and for how long?
-are you on special diet or exercise?
-any loss of appetite?

4-associated symptoms
-any lumps or bumps?
-any rashes? Bruises?
-any itching? (Yes). all the day or at specific time?
-any nausea or vomiting?
-tummy pain?
-yellowish discolouration of the skin?
-how is your bowel motions? Urine?
-chest pain, cough or SOB?
5-General
-Past medical history
-past surgical history
-SADMA
-contact, travel, sexual history
-family history of similar condition?

Physical examination from the examiner


1-General appearance
-pallor
-bruises, rashes
-jaundice
-scratch marks, pruritus
-BMI

2-Vital signs

3-Hands and arms


-nails: spoon shaped nails, dry or brittle, pale nail beds
-joints swelling
-purpura, petechial, ecchymosis

4-Face
-eyes: jaundice, pallor, hemorrhage, injection
-mouth: gum hypertrophy, gum bleeding, glossitis

5-Lymph nodes (site, size, consistency, mobility, tenderness)


-epitrochreal LN
-Axillary LN
-cervical LN
-inguinal LN

6-Neck
-stiffness
-bone tenderness (spine, ribs, sternum and clavicle)

7-Abdominal examination
-distension, tenderness
-hepatomegaly, splenomegaly
-genitalia

8-CVS and chest


-air entry, breathing sounds
-heart sounds, murmurs
9-neuro
upper and lower limb 9tone, power, reflex, sensation)

10-legs
-bruising
-pigmentation
-scratch marks
-leg ulcers

Diagnosis and differentials


-based on history and examination, there are several possibilities why you have weight loss,
night sweating and itchiness what ia m suspecting is a condition we call ?Hodgkin lymphoma or
nasty growth of the lymph glands; as there was enlargement of LN in the neck area in addition to
spleen. Both are part of our immune system to fight against infection and filter body from toxin
and trap cancer cells.
-other possibilities are
*could be nasty growth of the blood or leukemia.
*it could be due to infections like glandular fever (EBV), tuberculosis, liver infection (hepatitis),
lung infection (pneumonia), HIV, CMV.
* Connective tissue disease or SLE.

But we still need to do further Ix to rule out these:


-Basic blood tests (FBE, ESR/CRP. LFT, UCE, BSL)
-viral serology.
-chest X-ray, CT.
-refer to specialist for lymph node biopsy which is to take a sample of tissue or cells from the
lymph glands using fine needle or partial or complete removal of Lymph glands to be examined
under microscopy.

Feedback 15-3-2018
I can only remember the second case faintly as there was lot of lymph nodes and lymphomas.
This was a man with lump in neck of about 3 to 4 months cants remember for sure. Either you or
colleague has examined and found out that it is cervical lymph nodes enlargement. Ask PEFE,
Ddx and invx. Approach: All causes of LN enlargement
Asked examiner everything that was possible in step wise format. general apperance.. examiner
you are looking at him right now. Vitals normal ,PICKLE asked abt pallor, icterus and lymph nodes
. When i mentioned lymph nodes he asked what in lymph nodes so had to mention all groups in
cervical, axillary, inguinal lymph nodes . cant remember so clearly but he had only bilateral
cervical nodes. Then asked specifically about cervical in detail like largest node, consistency was
firm , mobility were mobile nor fixed, tenderness non tender, any skin changes above them
none, then asked about BMI and wt of patient . Asked if i have previous wt to compare if any
loss of wt, skin rash none. Then asked about focused abd exam any tenderness any
hepatospelnomegaly none. Then said with consent, privacy and chaperone would like to know if
testes size is normal as lymphoma can involve testes and lastly asked to listen to heart and lungs.
ddx was most likely lymphoma but cannot rule out infectious etiology so in investigation asked to
do FBE.. inflammatory markers EsR, CRP, baseline LFT and RFT, LDH and b2 microglobulin as
prognostic markers for lymphoma, but said the most important investigation would be LN biopsy
which would help in diagnosis and also tell me about the type of lymphoma as they all have
different prognosis. Also said if it turns out to be lymphoma i will sent to specialist who will do
furthur investigations like CTscan for staging chest x ray etc . If it is not lymphoma we will have to
look for infectious etiology by doing viral serology. Drew diagram and expalined a bit about LN
and lymphomas

Recall 5-5-2018
Female pt has night sweats and weight loss recently.
Take history, PEFE and explain diagnosis with reason. I didn’t do this case well.
On PE, there are cervical lymph nodes and splenomegaly. I talked about viral infection, might be
EBV but no rash, no ENT signs and I realised I were wrong as there was no travel history and
recent infection in the stem. Think about it! Is it Hodgkin lymphoma?

Case (7/3/2017)
GP, 30 yr old lady who work at agency complaint of fever.
Task-history, askPE, give diagnosis and differential.
Fever 3months, of random, night sweat+, itchiness+ ( I forgot to ask), no chills and rigors, no
travel, poo and pee ok, not noticed lumps and bumps and rash, noLOW and LOA. I offer
confidential ask about sexual history-has stable partner for 2 years, all types of sex, not use
condom, use OCP, no discharge, not at tooor ear piercing, no blood transfusion.
Then examiner gave PE card-temp-37.6C,vital-stable, one lymph node+ on each side of
sternocleidomastoid, splenomegaly+, scratchmark+ on limbs, no rash. I hesitated to tell
diagnosis and told that it may be EBV (I know it is less likely), other differential-HIV, Lymphoma
and leukemia.
* Examination card pallor, no jaundice, 2 enlarged cervical LN firm, scratch marks on chest and
abdomen, throat normal.

Case (8/6/2017)
Hodgkins lymphoma – I don’t remember the stem , wt loss + sweating
Feedback 7-12-2018
:40 y old female comes to your GP , with history of weight loss,night sweats , loss of appetite,
tired,long history , no travel, no contact history.
TASK : physical examination findings from examiner, explain your daignosis and investigation to
patient .
2 min : as no history , need to be organised in taking findings from examiner as might miss

ALWAYS KNOCK THE DOOR.


I entered in greeted examiner and patient , (i forgot the name of Patient so just called as Mrs)
taken permission of my patient that i read about her problem need to do some examination
before i can explain her whats happening with her . i followed haematology examination
sequence (as my DD were lymphoma,leukemia , HIV , DM. ) started with
General appearance, not malnutritions, pallor +,ICKE, VITALS, BMI, any weight loss past 6 months
(6 kg i guess ),
Then HANDS- NAILS - pallor +, crt - normal, no cyanosis, rest negative. WRIST- no tenderness.

ARM - scratch marks positive, (need to ask each lymph node specifically ), 3-4 axillary nodes +ve.

FACE- NO frontal bossing


EYES- no Subconjuctival haemorrahage
MOUTH - no glossitis, angular chelosis, gum hypertrophy, bleeding,
PALATE TONGUE , TONSIL,

NECK- nodes +ve,


CHEST AND HEART - auscultation normal
ABDOMEN- MILD spleenomegaly positive, no hepatomegaly
GROIN
LEGS all _ve.
UDT _ve, BSL - normal.
To patient : After doing examination and from history there are couple of things running in my
mind , it could be lymphoma, (cancer of some infection fighting nodes), leukemia, HIV(pt looked
worried - i told its just my working diagnosis need blood tests to confirm- she said ok ), DM (but
blood sugar normal -unlikely), EBV,CMV, TOXO ( i remember writing all these abbrevations on
paper but told her these are some names of bugs that can cause symptoms like this ( chapter
28,26 of john Murtagh)
Told her ill arrange Investiagtion like blood test - to see any bugs in blood , hb,fighting cell
number, USG to look for tummy problem all organ test like LFT,KFT,esr,cue and all
if could not find any answer need more advanced like ct scan and bone marrow aspiration
(explained that procedure would be done by experts under local anaesthesia and cells seen
under microscope will help us to know diagnosis)
finally reassured that we can support her what ever results are.

GLOBAL :7( approach -7, choice of examination, organisation and sequence 6, dd-6,invest-6 )

Karin case Hodgkin Lymphoma


Case: You are a GP and 33-year-old male came to you complaining that he has not been able to
shake off his flu which started about 7 weeks ago. It was not too bad initially, and it improved
after a few days. He was fine for a week but then started again and was repeated five times. He
is asking if he should have some antibiotic or vaccine.
Tasks
a. History
b. Physical Examination
c. Investigation
d. Management
Case: Sheila, a 22-year-old female, comes to see you with a 2 week history of intermittent fever
and drenching night sweats. She has been experiencing lethargy, anorexia, and generalized
pruritus for 4 weeks. She has been dieting but has lost 5 kg in weight during the past few
months. She has been taking aspirin for the fever but no other drugs.

Case 3: History  8 weeks ago developed runny nose and sore throat prescribed doxycycline for 2
weeks but did not help enough. + history of travel to Malaysia and did not have any bushwalking
or any risks to acquire infections. Before traveling, prescribed with doxycycline and had vaccine
against hep a and typhoid; came back one week ago and had 37.5, cough, yellow sputum, weight
loss x 8 weeks; mild smoker, diabetic, uses condoms; no TFTs, CTD, diarrhea or vomiting; (+) loss
of appetite; PE: 37.5, clear lungs on auscultation, no LAD, slightly enlarged liver.
Task
a. Further focused history
b. Examination Findings from Examiner (tired, dehydration, BMI 27, PR 90, T38, RR 16, BP 90/60,
bilateral non-tender cervical LAD, liver and spleen palpable 1cm below)
c. Possible Diagnosis and further Advise

Differential Diagnosis
- Glandular fever
- CMV
- Hepatitis
- HIV
- SLE
- Tuberculosis
- Sarcoidosis
- Secondary metastasis from other sources
- Leukemia/lymphoma
Hodgkin’s lymphoma
- Generalized lymphadenopathy
- Rubbery consistency
- Pruritis
- Recurrent infection
History taking
- What do you mean flu? Fever? Any night sweat? What did you do for that? Does it help? Any
runny nose? Have you lost weight recently? Any changes in appetite? How is your energy level?
Are you still able to go to work? Do you feel itchy anywhere on your body?

- How is your general health and your past medical and surgical condition? Have you any lumps
on your body? YES. Are they painful? Did you come into contact with anyone having a similar
condition? How is your bowel habit? How is your urination?

- Social history? SADMA? History of travelling? Any family history of similar condition or any
blood related cancer?

Are you sexually active and any history of STD? Have you recently migrated to this country?

Physical Examination
- General – BMI, rash
- Vital signs –stable
- Lymph nodes examination (bilateral cervical LAD)
o Axilla, cervical, inguinal
- Skin – scratch marks
- ENT examination: Signs of infections; including throat
- Auscultation: Chest and heart
- Abdominal examination (liver enlarged 2cm below ribs)

o Tenderness o Masses o Organomegaly


- Urine dipstick, BSL

Diagnosis and Management

- The most likely diagnosis at this stage is unfortunately a serious condition which is a malignancy
of lymphatic system. It is a bit unusual called Hodgkin lymphoma where the lymphatic system
produces abnormal lymphocytes.

- Lymphatic system consists of glands and interconnecting channels functioning as a filter of the
body to get rid of any bugs, toxin and harmful substances. Sometimes, due to unknown reasons,
these cells start to multiply rapidly and abnormally causing painless enlargement of lymph
glands.

- There are some other symptoms such as fever, malaise, night sweat, itching and sometimes
liver and spleen can be enlarged. If undetected, these cells will spread involving other lymph
nodes groups in the armpit, around the windpipe and nodes in the pelvic area. The diagnosis
needs to be confirmed and rule out other possibilities.

- The investigation includes: FBE with peripheral smears, LFT, U and E, FNAC, ESR/CRP. However,
the definite diagnosis can be obtained by excisional biopsy to find Giant Reed Sternberg cell.
Bone scan, CT scan (chest, abdomen, pelvis) and bone marrow biopsy will also need to be done
after consultation with specialist hematologist.

- Fever >38, weight loss >10% of body weight and night sweats means patient is worse
- From now on, the management and prognosis depends upon the staging of the disease.
Generally speaking, stage 1 is where only one group of lymph nodes is involved, stage 2 is where
2 lymph nodes group are involved but they are on the same side of the diaphragm; stage 3 is
where 2 involved lymph nodes group are on different sides of the diaphragm; stage 4 is where
there is involvement of liver, spleen and lymph nodes across the body.

- For stage 1 and 2, 5 year survival rate is 90% while stage 3-4 has 5 years survival rate is 50%.
- Mainstay of treatment is chemotherapy
o ABVVD
▪ Adriamycin ▪ Bleomycin ▪ Vincristine ▪ Vinblastine ▪ Decrabazin
- The treatment will be given over a period of weeks to months. There are some complications
related to treatment such as nausea, vomiting, bone marrow suppression leading to anaemia,
painless hematuria (do not need to worry, will resolve itself), hair loss (which is usually reversible
and you can use wigs), abdominal pain with or without diarrhea (good fluid intake is necessary),
cardiologic problems as Adriamycin is cardiotoxin, interstatial lung fibrosis with Bleomycin,
opportunistic infections, and infertility might result after the treatment. For that reason, I will
refer you to specialist to store your sperms/eggs for later use.
- Radiotherapy is usually followed to kill remaining and undetected cancer cells in localized area.
The side effects are confined to the area treated such as tiredness, sore throat, taste changes,
muscle cramps, diarrhea and localized hair loss.
- You will be monitored throughout the treatment by specialist. You will need to have regular
follow-up every 3 months for 2 years and regular 6 months for 5 years. I will write an urgent
referral to specialist oncologist and reading materials, contacts of support services such as
cancer council organization and centerlink for your knowledge and motivation.
- For NHL: bone marrow suppression  pallor, anemia, pancytopenia - 1 cycle = day 1 and day 15.

Feedback dec 2018


Station 9: Fever - PASS

I don’t remember this case much. 30/40 something female patient complaining of tiredness and
fever(?) for some time. No travel history, no significant medical history.

Task: History

PEFE

Dx and DDx

Ix
History – I seriously don’t remember what I asked in exam.

PEFE – positive findings were fever (+) – around 38; cervical lymph node enlargement – 2cm;
non-tender; firm; fixed; no redness over the lymph node; no axillary and epitrochlear lymph
node; splenomegaly (+);

Dx and DDx – Lymphoma; leukaemia; I don’t remember what I gave other ddx (maybe DM;
thyroid)

Ix – FBE; ESR/CRP; TFT (?); RBS (?); biopsy of the lymph node

GS – 6

Key steps 1, 2, 3, 4 – covered

Approach – 6

Choice and technique of examination, organization and sequence – 7

Diagnosis and DDx – 6

Choice of investigation – 6
89-Peripheral vascular disease
Gp setting, 62 years old women with right calf pain.
Task
-History
-PEFE
-Diagnosis
-Investigation and management

History
1-Pain questions
-severity + allergy and painkillers
-how long? Sudden or gradual? Constant or come and go? Getting worse?
-site (show me exactly, one or both legs) + radiation
-type?
-anything makes it worse like walking or exercising? How much distance is necessary before you
get the pain? (pain on walking 200m )
-anything makes it better like rest? (relieve by rest)
-has this happened before?
-how does it affect your life?

2-associated symptoms
Local
-any swelling?
-any skin colour changes? Like redness or rash?
-any ulcers?
-any trauma or injury?
Systemic
-fever?
-LOW, LOA, lumps and bumps?
-SOB, chest pain, palpitations?
-joint pain?
-tummy pain?
-Any weakness, numbness or tingling?
3-General
-past medical history (hypertension, DM, high lipid, heart disease) (some cases hypertension +ve
and on B-blocker/ other not)
- What medication do you take for it?
-Past surgical history?
-Smoking and alcohol? (some case smoker but others not)
-Travel, occupation? (painter)
-family history

Physical examination from the examiner


1-General appearance
pallor, jaundice, LAP, BMI.
2-Vital signs
3-lower limbs examination
-inspection (scars, skin colour changes or ulcers, swelling, deformity, varicosities)
-palpation (tenderness, temperature, CRT, oedema pitting or not) + pulses (diminished DP, TP even
the popliteal)
-feel and auscultate for arterial bruit. (Femoral, renal, aortic)
4-neuro sensation and power
5-special tests (burger + ABPI) (ABPI not done/ burger test in some cases not done and other cases
dusky color one leg and pallor the other)
6-CVS (apex beat, heart sounds and murmurs). + Carotid
7-abdomen (distension, tenderness, organomegaly).
8-office tests: UDT, BSL

Explain the diagnosis


-from history and examination, you most likely have a condition called peripheral vascular
disease which is narrowing of blood vessels supplying the legs due to build up of fat causing no
enough blood or oxygen supply. So at time of walking there is increase body demand to blood
and O2 so this lead to pain while at rest the demand is less so that there is no pain.

Treatment (according to history findings)


1-Investigations
-take blood for FBC, ESR/CRP, UCE, LFT, BSL, lipid
-arrange Doppler ultrasound to look at the vessels of the legs

2-SNAP
-advise to maintain healthy lifestyle
-stop smoking + arrange another consultation
-healthy balanced diet (low fat, no junk food) + refer to dietician
-limit alcohol intake to safe level
-gradual exercising to the level of the pain to increase collateral circulation+ refer to physio
3-Chronic diseases
-control hypertension, cholesterol and diabetes
-start aspirin and statin
-stop B-blocker and replace with ACEI
-control DM + foot care

4-Referral
-to vascular surgeon to decide upon any need for surgical intervention + CT angio
Intervention options
*endovascular stent: stent inside the vessel to dilate and improve blood flow
*bypass graft

Feedback 30-5-2018
Station 19
‘Leg and buttock discomfort’:
PVD –all CV risk factors +ve. CC: pain in leg, classic ischaemic pain story.

Task: Hx, PEFE, advise

Feedback: Pass

 HOPC
o PainQs
 Specifically differentiate neurogenic vs intermittent claud
o CVS risk fx assessment
o Other Sx
o DdxQs: LVANS (lymph veins arteries nerves skin)
o PHx FHx SHx SADMA
 O/E
o GA VS BMI
o Systems
o LL vascular exam
 Trophic changes
 TPRCS
 Buerger’s
 +ve: pulses diminished
 Dusky colour one side
 Pallor in the other
 Asked ABI: not done
 Ax & P:
o Most fitting with something called PVD
 Basic picture of vessel>narrowing 2/2 fatty deposition much like you
may have heard of in CAD
 Normally vessels dilate in exercise to incr bld supply to incr O2
 This diseased vessel can’t dilate to same extent>dcr bld supp>decr O2 to
muscle>pain
 It’s a systemic phenomenon and can eventually occur in other vessels
too
 CVA/MI
 Need ABI
 Mr Vasc assessment
 CTA

Feedback 1-3-2018
calf pain PASS
60 yr old with pain in leg
hx pefe inx mx
in hx she gave a v clear hx of intermittent claudication, no rest pain no swelling or tortuous
veins ,she didnt know if she had bp or dm etc i took a thorough hx she was a nice old lady.
on pefe abi wasnt given , pulse of DP and popliteal was absent and weak femoral pulse was
palpable on right side however no ulcers or discoloration ,buerger was neg
inx i said color doppler, my diagnosis was PAD. i explained it to her .
i asked if she was a smoker or on any med , she wasnt .
in mx i said il refer to the specialist and before that run some blood tests to look for
comorbidities like cholesterol ,blood sugars etc

Feedback 7-9-2018
Station 2 - PASSStation 5 – PVD – not assessed – PILOT
Peripheral Arterial disease
In stem - taking betaloc ( beta-blocker ) taking over 5 years for HTN not having right femoral
pulse , LL normal –
Previous surgery for DVT 5 years ago ( not sure how many years ago ) ,
Comes here as he got pain on both legs while walking , no pain on rest.
Buerger’s test – NEG, ABI 0.7
Task:
1. explain Dx and DDx to the patient –
2.Arrange appropriate invx to the patient and explain
3. Explain mgmt to the patient.
Task 1
- PVD ( explain with a diagram , explain causes and complications )
- others – artherosclerotic plaque in lay-term with a picture
- others – thrombophlebitis , cellulits , Venous blood clot like recurrent DVT again .
Task 2 – explain routine blood tests – including blood clot screening tests what we called
coagulation screening test , imaging tests like doppler U/S to your both legs – both arteries and
venous Doppler ( explain with arteries – coming from heart to lower legs with a picture , veins –
flow from peripheral LL to heart ) . ask the patient – are you still with me ? he said – YES

Task -3 – refer to the specialist – he said who ? – specialized to blood vessel what we called “
vascular surgeon “ , - stop beta-blocker.

Feedback 7-9-2018 (PVD)


GP clinic, 65year age old male with calf pain at left leg while walking uphill. No pain at rest.
He is hypertensive, taking b-blocker for last 5 years. He is a smoker for last 50 years, smokes 20
cigs per day. You have already done PE. Bmi-29, BP- 140/80, Gait-normal. No pulse until
femoral pulse. sensation-normal. right leg normal. ABI- 0.7 at right leg. Burger test-negative.
(Basically everything was given to indicate PVD)
tasks-
1. Give your diagnosis to patient.
2. Give your differentials to pt.
3. Invx .
4.Further MX .
(no hx)
FEEDBACK: Assessed, not score.
First 2 mints- This was my first station. I was thinking in my mind- no shaky voice or shaky
hands( I did horrible in first station in my trial exam). It's clear cut PVD.
Key points-
1. DDX- LVANS (explained later)
2.stop b-blocker
3. stop SMOKING.
4. Doppler usg to exclude DVT
5. other general tests and general mx, especially weight loss.(high BMI)
6. Finish it within time ( My weakness was time mx)
I entered the room. Examiner introduced me to my patient and said that " Do you
understand your tasks? You can have a look at the screen and the paper again. "
[ I would say some examiner took my time to describe all of these . Most of them were very
slow speaker. This introduction would take time around 30 secs to 50 secs. It's better to
practice and finish all tasks within 7 mints while doing role-play. No more talking after the
beep at 7 mints. I had poor time mx, so my friend suggested me to practise like that. I would
say this idea helped me a lot.]
I didn't introduce my name again as I have seen this patient earlier according to stem. Pt
was old and overweight male: Are you in pain atm? (no) . Okay. Then, I said that as I had
already taken history and examined you, now I would like to say what could be the causes
and reasons associated with it. Would you like me to address any specific concern first,
before I proceed further? (doc, I want to know why I'm having this?) . Okay. Most probably
you have a condition called Peripheral Vascular Diseases, Have you ever heard about this?
(NO)
I drew a leg picture and drew some lines to indicate blood supplies. I said there are few
vessels like different pipe-lines that kind of act as a drainage system of our body. One of
them, we called arteries, supply oxygen and blood to our muscles. But you know there are
few factors that can alter their functions, such as high blood pressure for long time, some
medication such as B-blocker, high levels of fats in the blood and smoking ; these factors can
make your vessels narrow ( I did a gesture by my hands :D) and block the blood supply just
like dirt in water can block water-pipes. Whenever you are walking, there is actually more
need of oxygen in your muscles, which your arteries are unable to carry through. That can
lead to damage of your muscle cells which in turn leading to pain while walking. Does any of
these make any sense to you? ( Pt was nodding his head) . I'm sorry to say that I couldn't
even feel the pulses in your right leg until your groin. But let me reassure you this condition
is manageable and we will do our best to help you.

However, there are few things in my mind that could be associated with it as well. As I
mentioned earlier, there are few other vessels in our body. I scribbled again some lines in
the paper.
L- (lymph nodes)-lymphangitis ( no fever, no insect bite, no trauma)
Lymphedema ( no swelling around legs)
V- (Vein)-any clots in your vein, we called it DVT
A- (Arteries)-PVD
N- (Nerve)-neuropathic pain
S- (Skin)- cellulites, ulcers, insect bite.

I would like to do some invx to exclude these causes-


1. some blood tests to know if there's any infections are going on or not, any problem with
your kidney and liver, fat levels in your blood .
2. Doppler usg to check any clots in your leg.
3. Angiography to check the narrow arteries.

Further MX
1. I said; I'm really worried about your long standing smoking habit (Pt mentioned himself ,
yeah 50 years) . As we discussed earlier, smoking is one of the major reason for your calf
pain. Have you ever thought about quitting smoking? (I tried to quit). I said excellent. This
time you can definitely do it. I will do another consultation regarding this. I'll give you some
reading materials regarding how much beneficial is quitting smoking.

2. I would liaise with specialist regarding your anti-HTN medication(b-blocker) as we have


to stop it and put you on another medication.

3. Please opt for some healthy diet as well as we know that high fats in blood are associated
with your calf pain. I would like to send you to dietician for your diet chart.
I was going to talk more about exercises, weight loss but Bell Rang!!
I thanked the pt and examiner and came out of the room.
Feedback 25-10-2018 Scenario: leg pain
Stem : old man with pain in right leg. ( cant recall other details although stem was long)

Tasks:
Hx
Pefe
Dx ddx with reasons
Inx

Greeted the pt. he was a very friendly person so I had established a good rapport with him from
beginning and I really enjoyed consulting him.
Asked all pain qs. Pain more on walking yphill downhill. After a set distance. When he stops pain
goes away. No night time symptoms.
Asked qs for cellulitis, dvt , pad, sciatica, trauma etc
Past hx. Positive for COPD d/t chronic smoking but stopped 5 years back. Hx of back pain n
osteoporosis positive.
Pefe: all negative except peripheral pulses upto femoral absent. Bergers negative. Abi not
done. Don’t know why did I asked for chest findings and examiner gave me findings and took
good 15 secs there saying in the end all of these findings go with his copd n smiled at me :D

Well then I turned to pt explained her pad n other ddx n reasons


He asked me what do u mean by peripheral arterial dx can u break this term n explain. I said
yeah sure, peripheral means at the ends. Its like ur body n head are in centre n so ur legs and
arms are your peripheries, artery means a vessel carrying blood and oxygen n nutrients to your
peripheries which is now diseased meaning obstructed due to chronic smoking and plaque
formation due to which oxygen n nutrients cant reach ur muscles n u experience pain bcz ur
muscles feel weak after a while(he was happy with this explanation) bell rang . I look at the task
lying on my table it says INX. While moving out of room I said I woukd do some inx like abi
doppler arteriography.(but it didnt work for me as i got 0 in inx)

Grade: pass
GS: 5
3/4 key steps covered
Hx 6, choice n tech of ex 6, dx/ddx 7, INX 0
Im shocked to see I passed this station even though I didn’t complete all tasks and got 0 in it.

Feedback 25-10-2018
Stat 7 -pass ( got 7 global score :D, but examiner was so stern, unexpected!)
57 yr old lady complaining of calf pain
Task :
History
Pefe
Diag/dd
Investigation

I took all history for pain, offered analgesics, trauma hx, clauidcattion hx, chest racing/pain
Med hx( she was saying she took spireva- I did not understand, asked examiner-he said its not a
medication :/), then asked all that I could remember for pvd and dvt.
Pefe- was straightforward,started with inspection, temp, then all pulses in right leg were absent
upto femoral. I asked for bruits in carotid, cvs exam . also asked for SLR to exclude neurogenic
claud.
Then explained diag of pvd, other dd like dvt, cellulitis, and said ABPI plus Duplex scan is needed.

Feedback 11-12-2018
57 year old man has come to your GP clinic complaining of pain in his calf. Known case of
diabetes, smoker, hyperlipidaemia. Blood results show that lipids on higher end and glucose 7
Take Hx,
PEFE,
Dx

HX- SOCRATES(PAIN)
Pain present on walking, relieved with rest,character(crampy)
ASSOCIATED SYMPTOMS- redness, itching, skin changes, ulcers(present), night pain
Ddx- dvt, cellulitis, leirche syndrome(ask about erectile dysfunction , always do sign posting
when get personal like sensitive questions), anemia
For dx – pvd – always ask risk factors- blood pressure, lipids(+), smoking(+), obesity,
diabetes(+)controlled
System review- GIT (pain after eating food) , heart(chest pain, sob, dizziness)
Pefe—GA- BMI(HIGHER SIDE), Pallor (-)
Vitals- stable
Local examination- inspection--skin changes(redness), ulcers present(+), shiny skin
Palpation-Tenderness- _negative, pulse absent whole right leg(no femoral pulse on right side)
Auscultation- (I forget)
Cvs- normal, carotid(normal) Urine dipstick-negative
p.s- examiner ..i don’t like him , even he was Indian but he is acting like it is pilot station(rest of
my examiners were very cool),so be prepared and handle with care.

DIAGNOSIS- pvd…n explain like you all know..also mention all ddx with reasons and lay man
terms(I advise even ddx is not asked for, always give dd) ,AND ALSO SAY I WILL Further evaluate .
(EVEN THERE IS NO MANAGEMENT )
Key steps covered, global score-5

Feedback11-12-2018
Station 15 ( Leg and buttock discomfort) Global score 4 PASS

57-year-old man comes to your GP clinic with complaints of pain in the calf. He is a known case of
diabetes, hyperlipidemia and smoker. His blood tests show high glucose and high lipids.
Tasks: History, PEFE, Dx and implications to the patient.
When I was outside I was puzzled about this “implications to the patient” task because it made me
think I would have to look for something else.
I entered the room and greeted both examiner and patient.
The examiner was very nice.
I introduced myself and started with open question: “Can you tell me more about your calf pain?”
He just told me that it was a calf pain when he walked.
I kept asking:
What kind of pain? A cramp.
When did it start? For the last 2 months
Is it on and off or continuous? On and off.
Is it progressive? Yes
What makes it worse? When I walk or climb stairs.
What makes it better? When I stop and rest.
Does it happen during the night? Yes. I have to wake up and hang my leg on the bedside.
Any chest pain? No
Any SOB? No
Any abdominal pain? No
Any recent travel? No
Any fever? Yes (I got alarmed!)
Any change in urine color? No
Any trauma? No
Any back pain? No
Any recent surgery? No
Smoke? Yes. (I offered further counseling if he wanted to quit it)
Allergy? No
Drugs? No
Alcohol? No
Medication? Only for diabetes and hyperlipidemia. No blood thinners.
Home situation? Good support
I forgot to ask family history.
I excused and started taking history from examiner:
General appearance: “ As you see”
Vital signs all normal.
PICCLED (clubbing)
BMI: 28
ENT: normal
RS: Normal
CVS: S1 S2 no added sound.
Abdomen: all normal. Pulsing mass? No. Any bruit? No. Femoral pulse? Absent on the right side (I
told her: I assume popliteal and dorsalis pedis are also absent. She confirmed)
Leg exam: Pallor? Yes. Shinny skin? Yes. Loss of hair? Yes. Temperature? Cold. CRF? More than 3
seconds. Cyanosis? Yes
I said: I would like to do the Buerger test. She said it was positive on the right side.
(By this time I was sure about PVD but I was still thinking about the fever).
I asked: Any ulcer? Yes. Any sign of infection? Yes. Redness, presence of pus and offensive odor.
Office test: presence of glucose.
I asked about Doppler: Not available.
I thanked her and went back to patient.
“ John, according to your history and PE, you have an obstruction in the arteries that take blood to
your leg(I Draw it.). It is what we call PVD. Certain conditions rises the risk of PVD like
hyperlipidemia, diabetes, high BMI and smoking. It is a common condition but if untreated the
prognoses is not good.
I am very concerned about the ulcer in your leg because it is infected.
Are you following me? Yes.
I wouldn’t like to tell you this, John, but if infection doesn’t get any better and treatment doesn’t
improve the blood flow to your leg, I am afraid an amputation might be needed. (3sec pause). Are
you with me? Do you want me to continue?
You need to be admitted and we will do whatever possible to save your foot and leg.
Bell rang, I thanked them and left.
Examiner seemed happy.

Key step 1:yes. Approach to patient/relative: 4


Key step 2: no. Choice & Technique of exam, organization and sequence: 5
Key step 3:yes. History. 4
Key step 4:yes. Diagnosis/ Differential diagnoses. 5

Feedback 13-12-2018 (important)


Health review: Pass
This was a PVD case ( or could be multiple issues). This was a headache case as patient was
giving me vague history, u would know why.
2 mins outside: Woman aged about 65 yrs complaint of foot pain and discoloration (picture
given).
Tasks: History, PEFE Dx and DDS to pt.
Thinking outside: Honestly picture was not good. Picture of one foot blue color but swollen. I
was thinking PVD (but PVD don’t come with swollen foot), DVT ( but complaint for 4-6 months),
Reynaud’s (possible, but age does not match), Chronic venous insufficiency ( generally comes
with gaitor area brown discoloration). Anyway so made my DDS.
Inside: How can I help you?
Pt: my friend says I m having gangrene in my foot.
Me: Don’t worry, we will find out and help you whichever way I can.
Me: is it the one foot of both foot are affected?
Pt: Both, but right more than left.
Me: Do u have pain at the moment? Pt: No
Me: do u get pain when u walk? Pt: Yes.
Me: how long do u walk before u get pain? Pt: 3-4 blocks (I think)
Me: what do u do then? Pt: I stop. Me: do u get pain while sitting? Pt: No
Me: what type of pain is it? Pt gave me vague answer.
Me: Do u get this pain while walking on hill. Pt: not really (I got confused here as PVD typically
give more pain on walking hill).
Me: I have noticed your both feet are swollen, since when? Pt: 4-5 months.
Me Does this swelling occur at morning or as day pass by? Pt: all the time
Me: Do u have short of breath (to rule out heart failure). Pt: Yes due to my smoking habbit.
Me: Do u have any chest pain or funny heart beat feeling: Pt: no
Me: any cough, sputum, fever? Pt: No
Me: Do u have big lumpy veins in legs? Pt: No, varicose veins surgery done in both legs.
Me: Any othe symproms apart from pain, discoloration and selling of legs: Pt: No.
Me: any known diabetes, HTN, known thyroid disease, kidney or liver diseae? Pt: no
Me: in past did you have chest pain, racing of the heart, known heart attack? DVT? This type of
symptoms before? Pt: No
Me: SADMA? Pt: Smoking (don’t remember since when?)
My examiner was old and partial deaf.
PEFE: General appearance: as u see
Vitals? (I asked all individuals) Exa: What? Me: (loudly) vitals? Exa: Normal
Respiratory and CVS examination: Normal
Femoral pulse bilatrally: Exa: What? Femoral pulse (loudly): Palpable
Popliteal Pulse bilaterally? Exa: What? Popliteal pulses(loudly): palpable.
PTA and DP pulses bilaterally (I asked loudly): Two non palpable on one leg and one not palpable
on other (I forgot which one which?)
ABPI?: Exa: What? Ankle brachial pressure index: Not done
Doppler study to rule out DVT and PVD: None of them done.
Dx: from history and physical examination, I think u r suffering from peripheral vascular disease.
Pt: whats that?
Me: peripheral vessels means blood vessels in your limbs. In your case some blood vessels which
takes blood from heart to feet in both legs are blocked, That’s why u having pain.
There could be other diseases that can explain the swelling such as DVT means blood clot in your
legs which we will keep in mind and send u for proper investigations. Other could be heart
failure which is less likely as u don’t have any other symptoms. Other could be valves in your
veins not working. There could be hidden kidney disease which we will find out by investigations.
Honestly, I was not happy with pt giving me vague history. I thought I would fail this case. Other
problem I realise that if examiner cant hear me properly while talking to him facing him, how
could have he hear me taking history while facing opposite to him. He was sitting behind me
while I was taking history. So guys pl speak loudly.
Global score: 4
Key steps: 1,2 no. 3,4,5 yes
Approach to pt: 4
History: 3
Choice and technique of examination, organisation and sequence: 4
Dx/ DDs: 4

90-Cellulitis + PVD
Patient age somewhere between 50 to 60 years coming with pain in one leg. Picture of the one
leg given outside showing redness and swelling close to ankle joint and above up till mid shin.
Task:
history
PE from examiner
Diagnosis and ddx with reasons.

History
1-Pain questions
-severity + painkillers
-how long? Sudden or gradual? Constant or come and go? Getting worse? (Pain since 12 months)
-site (show me exactly, one or both legs) + radiation
-type
-anything makes it better or worse? Exercise or rest? How much distance is necessary before you
get the pain? (pain earlier walking abt 500mts now even on little walk. Pain while walking up
stairs or walking uphill)
-has this happened before?
-how does it affect your daily activity?
2-Rash questions
-duration and oncet?
-1st time?
-rash anywhere else?
-what were you doing before it appeared? Any trauma or insect bite?
3-associated symptoms
Local
-itching?
-swelling?
-warmth or hot to touch?
-any ulcers or discharge?
Systemic
-fever, night sweating? (fever +ve)
-LOW, LOA, lumps and bumps?
-SOB, chest pain, palpitations?
-joint pain?
-bowel motions, tummy pain?
4-General
-past medical history (hypertension, DM, high lipid, heart disease) (hpt +ve) for how long? What
medication do you take for it? (aspirin, perindopril, statin)
-Past surgical history
-SADMA, travel, occupation (ex-smoker 2 years ago)
-family history

Physical examination findings from examiner


1-General appearance
-pallor, LAP, oedema, dehydration, BMI
2-Vital signs (fever and tachycardia +ve)
3-quick CVS and carotid
4-lower limbs examination
-inspection from the pic (redness +ve up till the shin) ask if rash anywhere else?
-palpation (temperature, tenderness, pitting oedema, CRT and pulses (hot and tender, absent
pulses up to popliteal artery)
-sensation (light and deep touch, vibration and proprioception)
-tone. Power, reflexes
-Burger test + ABPI
5-Office tests
-UDT
-BSL (imp)

Explain diagnosis
-most likely you have a condition called cellulitis, which is an infection of the skin and the tissue
under the skin usually by a bug; a bacteria staph or strept commonly affecting the lower leg.
it enters the body through broken or normal skin and can spread to the underlying tissue.
Smoking or diabetes can precipitate it.

-I am also suspecting that you are also complaining of a condition we call peripheral vascular
disease. Which is a reduction of blood supply to your legs due to narrowing of blood vessels
supplying your legs leading to pain on exertion due to increase demand of oxygen on walking.
smoking, high blood pressure or high fat level, DM are all risk factors.

-other possibilities are trauma, insect bite, allergies, clotting of vein of the legs but less likely

Management
1-admit to hospital to be seen by a surgeon.
2-Investigations:
FBC, ESR/CRP, LFT, UCE, TFT, lipid, BSL, clotting, blood culture, swab culture.
Doppler ultrasounds
3-demarcate rash + observe
4-IV antibiotics then switch to oral
flucloxacillin or cephazolin (IV)
flucloxacillin or cephalexin (oral)
5-painkiller
6-rest+ elevate the limb

Feedback 10-5-2018
5- can you pls tell us the detail of cellulitis with PVD case ?what was the tasks
For this case the question was patient age somewhere between 50 to 60 years coming with pain
in one leg. Picture of the one leg given outside showing redness and swelling close to ankle joint
and above up till mid shin.
Task:
history
PE from examiner
Diagnosis and ddx with reasons.

I started with pain questions and then offered pain killer. Asked for any recent injury, insect bite,
any allergies, fever recent URTI...Then asked Claudication pain questions like the distance in
which pain comes and asked risk factors like smoking any medications, atherosclerotic
conditions, DVT questions. Past medical/surgical history. SADMA.

History positive: fever, pain since 12 months. pain earlier walking abt 500mts now even on little
walk. Pain while walking up stairs or walking uphill. She quit smoking 2 years ago. She was some
medication for high BP but i forgot the medications.

PE from examiner: GA,Vitals, systemic examination then came to local examination.


Positive in examination was temp 39, peripheral pulses lost uptil popliteal. Femoral pulse felt.
Local rise in temperature. Buergers test negative.

So then explained tht cellulitis and told her i am suspecting PVD as well and explained tht too
with risk factors also. While explaining condition i always correlate it to the patients complaints..
Feedback 7-7-2017
50 yr old woman, presented with rash on leg with the picture given (pic of cellulitis on lower leg)
TASK
1. take history
2. physical examination from examiner
3. tell the patientyour differential diagnosis with reasons
WHAT I DID
hello examiner, hello helen
I've just seen your leg, is it painful? i will arrange for pain relief
so when did it start
helen: yesterday night, painful, nowhere else on the body, no insect bite, no h/o allergies, no
new product used on the legs,
Me: fever? helen: i havent checked but i feel hot, no lumps or bumps around the body
Me: Sad hx
Helen: used to smoke, quit 2 yrs back as had a heart attack and has a stent and on medication for
that
Me: which medications? THE PATIENT GAVE ME A PAPER WITH A STATIN, ACE AND aspirin
Any sob, pain while walking? helen: no sob but there is pain when i walk uphill
Me: is that recent onset with the rash
Helen: no it was present before the rash
Helen i will speak to my examiner now. Examiner
VITAL SIGNS..was febrile, tachycardic
CVS EXAM: all normal
FOCUSING ON LIMBS..I HAVE SEEN THE APPEARANCE...EXAMINEr: yes thats how the rash is as in
the picture
PALPATION: calf tenderness..no temperature: hot to touch
pulses..absent dorsalis, post tibial, popliteal in one leg and absent dorsalis in the other
THANK YOU EXAMINER
ALRIGHT helen after examining you, it seems there are a few problems though nothing serious
that cant be managed.
I have ruled out any allergic reaction or contact dermatitis (any reaction to a new product) most
likely you have cellulitis that is infe tion of the skin on your leg
AND PERIPHERAL VASCULAR DISEASE...which means the blood supply to your legs have been
compromised. the blood vessels are blocked in your legs most probably due to the same
condition affec ting your heart but we need to investigate further
I just need to ask one more thing from the examiner ( i wanted to ask for ulceration on the foot
but the bell rang) then said thankyou to both.
( I called the patient by the wrong name twice, as soon as I realised I apologized alot)
PASS keystep 1 yes 2yes 3no 4yes 5yes
approach 4 history 4 choice of exami ation 4 doag osis 4

91-Lithium in pregnancy

Sample case/ You are at your GP, when 30 year old Lisa comes to you for Prepregnancy
counselling, she has a history of bipolar disorder and is on lithium for the past 2 years.

TASKS
Take a further relevant history
Counsel the patient accordingly

History (she is well controlled on lithium)


1-Ensure confidentiality
2- disease questions (Bipolar Disorder)
-for how long have you had Bipolar disorder?
-how many episodes have you had during that time?
-When was the last episode of your bipolar disorder? She is symptoms free (if she has not had an
episode in 1 year, you can stop Lithium)
- are you able to identify early warning signs of bipolar disorder?

3-Medication questions (Lithium) DISCO (duration, indication, side effects, compliance, and
other medications)
-how long have you been on lithium? (Duration)
-indication (already given bipolar disorder)
-have you ever had any SE like dizziness, tremor, weather preference, diarrhoea? (SE)
-do you regularly take lithium? (Compliance)
-do you tale any other medications? (Others)

4-well control questions


-have you had regular check up with your psychiatrist? When was the last visit?
- How often are your Lithium levels monitored? When was the last check?
- have you ever hospitalized due to bipolar disorder?
5-Brief Psychosocial history (as this case is combination psych and obs)
-How is your mood?
-Do you have passing thoughts of harming yourself or others?
-How is your sleep? (sleep deprivation can lead to bipolar disorder)
-do you hear, see or feel things that others do not? Do you feel someone is following you or
trying to harm you?

6-5Ps questions
Period history
-When is your LMP? Is it regular?

Partner or Sexual history


-Are you into a stable relationship?
-Do you have a very good support from your partner?
-Any history of STIs?

Pregnancy
-Have you ever been pregnant before?
-Any previous miscarriages?

Pill
what type of contraception do you use?

Pap or HPV
When was your last pap smear or HPV, and what was the result?

7-general questions
-Any previous history of any medical or surgical illness, mental illness?
-SAD

Counselling
1-reassurance
many women who have bipolar disorder and on medications for bipolar disorder go for a normal
and healthy pregnancy and deliver healthy babies.

2-effect of pregnancy on bipolar and lithium on pregnancy


-Bipolar disorder can sometimes worsen during pregnancy.
- The medication of lithium that you are taking can have some effects on the baby especially if
you take it during the 1st 3 months of pregnancy. It can cause
neural tube defects sometime
heart defect
liver problem
respiratory difficulties once the baby is born
hypothyroidism
developmental delay

3-prepregnancy plan (psychiatrist role and GP role)


Now, I need to reconsider the lithium in you. For this I need to Refer you back to the psychiatrist
for review of your medications. There are three strategies that the psychiatrist can do,
depending on the severity of the bipolar episodes.

First strategy is a medication-free pregnancy. The specialist might decide on a safe and
supervised withdrawal of Lithium.
**It is usually done if a patient has a few episodes of the disorder, long periods of mood stability
(at least 1 year), low risk of self-harm, good support, and if the patient is able to identify early
warning signs. (No need to mention this point **I think)

Second strategy is to go for a medication-free first trimester after discontinuation of lithium now
and monitoring you further. Lithium will be reintroduced after the 1 st trimester.

Third strategy is to give a mood stabilizer throughout the pregnancy. The specialist will consider
the option of changing Lithium to another mood stabilizer carrying less risk like Olanzapine and
if necessary (If the bipolar episodes are not controlled with these medications) Lithium will be
reintroduced after in the 2nd trimester.

-I will do all the baseline antenatal blood tests like FBE, UEC, Blood group and Rh, coagulation
profile, rubella and varicella serology, hepatitis serology, and TFTs.

-I will prescribe you folic acid for 3 months before and for the first 3 months of your pregnancy

4-during pregnancy plan


-Once you become pregnant, you will be monitored in the high-risk pregnancy clinic, and will be
seen by a multidisciplinary team composed of the obstetrician and the psychiatrist.

-if on lithium at any stage you requires regular check of your kidney and thyroid function and
also lithium level (lithium levels monthly in the first half then weekly in the second half).

-daily dose of lithium will be broken down into small frequent doses to avoid lithium level
peaking.

-During your pregnancy you need to go for more frequent antenatal checks, a Down syndrome
screening, ultrasound at 18 to 20 weeks, fetal echo or heart scan need to be done by 22-24
weeks, sugar test at 28 weeks, repeat ultrasound at 32 weeks will be done, and a bug test at 36
weeks will be done.

5-labour, delivery and postpartum plan


-usually, the dose of Lithium has to be decreased by 38 weeks to reduce high levels of Lithium in
the baby.

-you may go for normal vaginal delivery but Delivery should be in tertiary hospital under
specialist guidance with continuous monitoring of your lithium level and also the baby with CTG.

-once labour is finished and when you go to postpartum period, you might get a relapse of your
bipolar disorder so lithium level need to be increases to the dose was before introduced.

- You cannot breast feed your baby while you are on lithium.

6- Ending
- Our goal during your pregnancy is to maintain maternal wellbeing, ensure your baby's safety,
and also prepare you for the post-partum period.
-I will give you reading materials regarding bipolar disorder during pregnancy for further insight,
and will review you regularly.
-give you referral to the psychiatrist

Feedback9-5-2018
Case: Medication in Pregnancy
Possible pregnancy + Bipolar on Lithium
Young female in her twenties with history of bipolar in management with Lithium a year ago she
has multiple recurrence of psychosis but she has been well for the last 10 months I think. She
comes because she believes she is pregnant. A bit long stem talking about her bipolar.
Task
Take history no more than 2 minutes.
Counsel her about her pregnancy
I managed this case as many other doctors have posted before.
Passed. Global score 4.
Bipolar well controlled, on lithium n THINKS she is pregnant and what's to find out what's the
effect of lithium in general pregnancies. Task counselling /mx

Outside- my goodness, i did not remember what i should say, may be ask some questions and
mentions about risk for mother and baby, how to manage during pregancy.However, she may
not pregant so i have to do pregnacy test first as on the question, patient has irregular period
only , she did not do any PT test.However, they did the thyroid test, KFT and CBC test for her on
questions given.

Inside: talk with patient, ask her about her health, when she was diagnosis with bipolar, what
medication. Ask her about 5 P, intention to have baby or not and what did she prepare like folic
acid. I asked about complication of lithium.I told her i will do pregnancy test for her now. There
are 2 ways. If she is not pregnant, it may due to side effect of lithium on her thyroid. If she
pregnant, there are some risk factor for mother( relapse) and baby( heart disease, thyroid
disease, death inside the womb). I told her about MDT and all investigations for antenatal check
up, folic acid need to be use...Then i ask examiner any test result back, he said no. i talk again
and again just all of this information including 5R.

Feedback 13-12-2018 Medication in pregnancy: Pass


2 mins outside: Pt is on lithium for bipolar since last ? 1-2yrs. She has no symptoms currently and no
problem with lithium. She missed her period thinks she is pregnant. She come to see you regarding that.
Tasks: history

Management

Outside thinking: Thought about what need to ask in history as I knew management.
Inside: Me: I come to know from the notes that u come to see me regarding your possible pregnancy and
wether you should be on lithium or not.
Pt: yes
Me: start with history od presenting complaint.
Pt: none
Me: asked full psy history for bipolar such as symptoms of bipolar and side effects of lithium. Also asked
about full psy history such as delusion, hallucination and depression. Also asked about psy visits. She did
not see psy since missing period.
Pt: none.
Me: special history of periods and pregnancy tests. He did not have pregnancy test done at home. Then I
asked about remaining 4 Ps. Quickly ased about current medical problems, which were none.
In all cases, I kept my history short and sweet as I knew u don’t need perfect history. It is better than
missing tasks.

Me: First of all we will do pregnancy test. If it comes negative then we will do some hormonal tests to see
why u r having irregular periods. ( to do pregnancy test is very important even if pt has done pregnancy
test at home, according to HB, in threaten abortion case it is a critical error if u don’t do pregnancy test
even pt has done this test at home.)

If it comes positive then we need multi-disciplinary approach as this would be a high-risk pregnancy
(scared her here, I should have said as this is a specialist area). As you know that some medications have
side effects on person and on foetus if person is pregnant. There are certain side effects of lithium on fetus
such as heart problem in fetus and blah blah blah ( I mentioned all the side effects, 8-10, which I read
form internet, my biggest mistake, scared the pt here too. did not need to mention them. Just needed to
say such as heart problem). Because of this issue I will need to refer u to obstetrician and psychiatrist. The
current approach is for mild case of bipolar: May be psychiatrist will withdraw the lithium and monitor u
regularly. For moderate case: lithium may be withdrawn for 1 st trimester and resume after that. For sever
case: keep pt on lithium throughout the pregnancy. In either case we will keep monitoring you and your
baby by regular visits, regular ultrasound and any necessity investigations. Again, it is a specialist area and
we need to adopt an balanced approach according to your psychiatry condition. Don’t worry, many pts
with this condition have successful pregnancy and we will try our best to get good results.

Me: do u have any que?


Pt: no, is there anything else I need to know?
Me: OMG, what have I forgotten? I checked my tasks again. Bell rang.
Key Steps: all yes. Global score: 4
Approach to pt: 2 ( I think I scared her, that why)
History: 3 ( I kept my history short)
Pt Counselling/education: 4 Management Plan: 4

92-Toddler’s diarrhoea

3yr old boy with diarrhea since the past 3-4 months
Tasks:
-History
-PEFE on card
-DDx
-Mx

History
1-diarrhoea questions (ACC CSF) = (amount, colour, consistency, content, smell, flush)
-for how long has he had diarrhoea? How often? (Everyday/ multiple times a day)
-how much? What is the amount?
-what is the colour?
-is it watery or loose? (Watery)
-does it contain any blood, mucous or undigested food particles? (No mucous or blood)
-is it smelly? (Foul smelling)
-is it hard to flush?
2-Tummy pain
-does he have any pain in his tummy? (No)
3- Vomiting.
-Does he have any vomiting? (No)
4-any fever or rash (No)
5-Dehydration questions (all NORMAL)
-does he look drowsy?
-how is his urine output? Is it same as usual or reduced? Any change in the colour of urine?
-how is his feeding?
*Can you tell me about his diet, meals and drinks? Is he drinking milk? ( Had stopped giving him
milk completely thinking it might be the cause, not even lactose free milk. Was having a glass of fruit juice
with every single meal, but she claimed that’s what he has been doing since long before the diarrhea
started. Picky eater)
6-BINDSMA (all good)
-birth history all
-Immunisation
-nutrition (already asked)
-is he growing and thriving well?
-anyone at home with similar symptoms
does he attend a childcare? Anyone there has similar symptoms?
have you travelled recently? (No)
do you have good support?
-does he take any medications? (antibiotics) Any allergies?
7--any family history of diarrhea related illnesses or any illnesses? (celiac) (No family history)

PEFE card but just in case (all normal)


1- General appearance (DR PJL).
-dehydration (sunken eyes, dry mucous membranes, skin turgor, CRT)
-rash
-pallor, jaundice, LAP
2-Vital signs
3-growth chart
4-full system but focus on abdomen
-inspection mass, distension
-palpation-tenderness, mass, organomegaly
-auscultation bowel sounds
-genitalia, hernia orifices, anal inspection for excoriation
5-Office tests
-urine dipstick, BSL

Differential diagnosis
1-toddlers diarrhoea
2-Giardiasis
3-celiac
4-lactose intolerance
5-antibiotics
6-gastroenteritis (unlikely)

Explain
-from history and physical examination, your child (mention his name always) has chronic
diarrhea as it is more than 2 weeks in duration.

-there are various causes behind it, it could be due to infection we call giardiasis or due to food
allergy or sugar intolerance. Could also be a problem with the absorption of nutrients from the
gut we call celiac or even due to medications like antibiotics. However, these are unlikely as your
child is growing and thriving well, now weight loss, pallor, rash or fever. Also there is no blood or
mucous in the stools. Reassure her about examination findings all normal.

-most likely he has a condition called toddlers diarrhea, which is a common type of diarrhea in
this age group, not serious or infectious, it is also self-limiting so let me assure you about that.
It usually presents with runny diarrhea, more frequent with smelly stool.
The cause is unknown but it thought that diet might be related to it, as children’s diet is not ideal
in this age with low fibers, low fat, and high sugar intake like fruit juice in his case

Management
1-I still need to run some investigation to rule out other causes
stool MCS (ova/ occult blood, fat or fatty acid crystals)
FBC, ESR/CRP
LFT
??Celiac serology and total IgA

2-you need to modify his diet and ensure healthy balanced diet with high fibers. Try to avoid
giving him fruit juice and instead let him drink water to quench his thirst. You can reintroduce
milk intake, as it is important part of his diet and important for his growth.

3-I will follow him regularly


-reading materials
-red flags (increased diarrhea, loss of weight, not eating or playing well)
-if these measures does not work, I will refer him for further evaluation

Feedback 21-7-2018
Diarrhoea – PASS (G.S- 5)
Stem: 3yr old boy with diarrhea since the past 3-4 months
Tasks:
-History.
-PEFE on card
-DDx
-Mx.
3yr old brought in by mum coz of diarrhea, everyday, watery, unformed stool, multiple times in
the day, foul smelling, no mucus or blood, no distension of abd, vomiting, crying with pain, fever,
active kid otherwise, BINDS ok, no celiac history, travel, family, daycare- no similar history Had
stopped giving him milk completely thinking it might be the cause, not even lactose free milk.
Was having a glass of fruit juice with every single meal, but she claimed that’s what he has been
doing since long before the diarrhea started. Picky eater. Had undergone multiple tests- all
normal.
- PEFE – all normal
- Said it must be toddlers diarrhea, due to excess fruit juice. Gradually reduce it. Reintroduce
milk. Its important for growth. Find out his favourite foods. Give him small frequent meals.
Reward him for finishing meals. Will review in 4-6weeks

Case (1/9/2017) toddler's diarrhoea


3 years old boy brought by a parent for diarrhoea for six weeks. She has tried restricting foods
and increase fluid intake but it is not improving
Task:
history not more than 4 min
Ask PE ( card will be given)
Explain about the provisional diagnosis with reasons

Case (9/5/2017)
Toddlers diarrhoea / Malabsorption ??
3yr old boy comes with diarrhoea for chronic duration. parents tried restriction of food and
increase fluid intake, no improvement
Task: H/O Ask PE, Dx and Mx incl inv
smelly and greasy poo, 7times a day
no increase milk intake, normal BMI

Case (22/4/2017)
Toddlers diarrhoea - this case both role player and examiner are gud..
2 year old have diarrhoea.. binds ques all normal.. happy child.. growth normal..
examination findings examiner gave to hand..all normal..
so explained all findings are normal it's more common for kids in this age group they like to play
a lot and they have more temper tantrums and have wat they like .. as having fruit juices he is
not having food properly.. give him 3 meals... if ur concerned I will do some tests like celiac
serology it's for gluten sensitive problems but it's not nessessary for him..if anytime u feel he is
not growing well.. not playing well.. no eating come back to me.

Case (3/3/2017)
Diarrhea in 4 yr old for about 4 weeks. Patents have restricted food and increased fluids buy still
it's there.

Task: hx, PEFE card ( all normal ), Dx, Mx to mom

Stool not floppy not hard to flush not sticky not frothy, no food particles.
No increased consumption of milk or juice.
no ulcers rashes no loss of weight.
No antibiotics used
Healthy child and active
I gave ddx of Toddlers, Celiac, giardiasis, antibiotic and arranged investigations including Celiac
Case (7/3/2017)
GP, 4 yr old child with loose motion for 5 weeks. Previous GP treat as acute Gastroenteritis but
still present. Physical examination all normal.

Task-history, PDx, Mx.

Finding- foul smelling diarrhea, 5 or 6 times a day, no blood and mucus, no Nausea and vomit
right now, can eat well, no food restriction, no similar problem

at home, no undigested food in stool, BIND all normal, no travel history, no FH of special diet.

To be honest, I don’t know the diagnosis and thought block in 2 min thinking. I can’t think of DDx
of chronic loose motion. So, I said it is GE (which is wrong) and Treat as acute GE. But, I also told
that to exclude other possible conditions like

Coeliac disease and infection, I want to do blood test including antibody test and Stool tests. ( I
failed this case. In my exam, most are just history, diagnosis and DDx, No PE. So, I have to tell
diagnosis in most cases just by history.)

Case (26/7/2017)
2 years old diarrhoea persistent. I could not find any positive history. Task history, PEFE card from
examiner, explain diagnosis and management to mother

comment
Stool culture was already done which was negative, so no need for stool study. But I arranged
CBC because diarrhoea was for 2 months. I passed this one

in giardiasis she gives h/o bubbly diarrhoea & weight loss with travel history with similar
symptoms in family, in this nothing of the above and a very clear h/o high intake of fruit juices
with stools containing undigested food particals
93-Post cholecystectomy bleeding angry partner counselling
Patient had Urgent Lap Cholecystectomy 6 hrs ago for Acute cholecystitis.
The gall bladder was very badly inflamed. Everything was fine and surgeon
finished the operation. Then suddenly BP was dropping to 80/40 and HR 120.
Surgeon had to urgently reoperate as he thinks the cystic artery clip may
have come off. They will try to go for key whole surgery but if required they
might do an open too. Patient is going to the theatre and his partner wants to
speak to a doctor. Surgeon has sent you to talk.
Task
–explain the situation
–address her concerns

1-Approach
when you enter the husband or wife will be standing and angry asking you why did this happen?
-Mr John I understand that you naturally must be worried about what happen. If you could have
a seat and I will explain in details your wife’s condition.
-Introduce yourself.
-before we talk would you like to have a glass of water?
2-explain what happen
-First of all, I would like to reassure you that your wife is in stable condition now, the doctors got
everything under control. She is in good hands now. Do not worry; we are keeping eyes on her
continuously.
-your wife has gone through a challenging operation to take out the gall bladder, After the
surgery, Your wife’s condition suddenly deteriorated her BP dropped significantly and her heart
rate increased in a short period of time. The surgical team suspected that there was a bleeding
inside the tummy. They already gave your wife fluid and blood to settle everything down.
However, they are considering a re-operation to find out the source of the bleeding.
3-Explain the possible source of bleeding
- Most likely, the source of bleeding is coming from the cystic artery which supplies blood to the
gall bladder. Then, draw the picture of the liver, gall bladder and cystic artery.
The surgeon cut the blood supply to the gall bladder and clip both ends off. Because the gall
bladder is inflamed so the surrounding tissue is very soft, soggy, and slippery which may cause
the clip to slip off. However, the bleeding can come from adjacent organs that we need to find
out.
4-Answering patients questions
Q1 was there any mistake of the surgical team?
-I am not making excuses by saying this is not the surgeon’s fault. This is one of the possible but
uncommon complications, which can happen on any surgeries. There are many other possible
complications after the surgery, such as bleeding, infection, damage to other organs, anesthetic
reactions. Unfortunately, the bleeding happened to your wife. We are very sorry about that. We
are trying our best to help your wife.
I believe that the period after the surgery is very important when the surgical team can detect
and control any possible unwanted outcome and it is good that they picked it up early and they
are trying to control it.
Q2 Should we have been told about this before the Surgery?
-We always explain all the possible complications as well as the positive outcomes of the surgery
before we carry out any invasive procedures to get the patients consent. This is part of surgical
protocols not only in Australia but also in the world.
Q3 what are you doing next?
- they will take your wife to the theatre again to perform an exploratory key-hole surgery to
clean up the blood and identify the source of the bleeding and stop it. However, they may have
to open her tummy up if they cannot handle the bleeding via the keyhole surgery.

5-Ending
- Definitely after the surgery, I will arrange to meet your husband as well as the surgeon.
-do you have kids at home? I can arrange social worker to look after them.
-I will keep you updated about your wife’s condition.
-call me if you need any help.
Feedback 23-6-2018
Feedback: Post-operative compications: a female pt just came through a LAP-COLE, the surgical
picked up the bleeding, gave the pt blood, pt is stable now and ready for a exploratory
laparoscopic operation to stop the bleeding. Counsel the pt with the possiblity of open
operation.
I walked into the room, seeing the husband is standing up and angry. He didn’t let me to say
any greetings and said  why did it happen? I approached and calmed pt down:
- Please calm down and take a seat, I will explain the situation to you and answer all your
questions.
- First of all, I would like to reassure you that your wife is in stable condition now, she has just
gone through the ordeal but the doctors got everything under control. She is in good hands now.
Don’t worry, we are keeping eyes on her continuously.
- I said I totally understand your feelings at the moment, If I was in your situation I would act the
same way. However in situations like this, we need to compose ourselves to fix things properly.
I explained the situation and the procedure:
- Now I will explain your wife’s scenario: your wife has gone through a challenging operation to
cut off the gall bladder, After the surgery, Your wife’s BP dropped and her heart rate increased,
the surgical team suspected that there was a bleeding inside the tummy. They already gave your
wife fluid and blood to settle everything down. However, they are considering a re-operation
to find out the source of the bleeding.
- Most likely, the source of bleeding is coming from the cystic artery which supplies blood to
the gall bladder. Then, I drew the picture of the liver, gall bladder and cystic artery. The surgeon
cut the blood supply to the gall bladder and clip both ends off. Because the gall bladder is
inflamed so the surrouding tissue is very soft, soggy, and slippery which may cause the clip to slip
off. However, the bleeding can come from adjacent organs that we need to find out.
The pt asked me: was there any misake of the surgical team
- I said: This is not the surgeon’s fault. This is one of the possible but uncommon complications
which can happen on any surgeries. There are many other possible complications after the
surgery, such as bleeding, infection, damage to other organs, anesthetic reactions,…
Unfortunately, the bleeding happened to your wife. We are very sorry about that. We are trying
our best to help your wife.
The pt continued asking me: Should we have been told about this before the surgery:
- I said: We always explain all the possible complications as well as the positive outcomes of the
surgery before we carry out any invasive procedures to get the pt’s consent. This is part of
surgical protocols not only in Australia but in the world.
The pt asked me: what are you doing next?
- I said: We will take your wife to the theatre again to perform an exploratory key-hole surgery
to: (1) clean up the blood and identify the source of the bleeding and stop it. (2) However, we
may have to open her tummy up if we can handle the bleeding via the key-hole surgery.
- I will arrange a meeting between you and the surgeon right after the operation is done.
- Then I provided counsellor talking, water,…
- Finished my tasks.
Grade: PASS with GS 5 Key step 1,2,3,4: Y (all covered) Key 5: N
Approach: 5, Explanation of the procedure: 5, Pt Counselling: 5.
Feedback 28-11-2017
Station 17: Post-op complications - pass
Patient had Urgent Lap Cholecystectomy 6 hrs ago for Acute cholecystitis. The
gall bladder was very badly inflamed. Everything was fine and surgeon finished
the operation. Then suddenly BP was dropping to 80/40 and HR 120. Surgeon had
to urgently reoperate as he thinks the cystic artery clip may have come off. They
will try to go for key whole surgery but if required they might do an open too.
Patient is going to the theatre and his partner wants to speak to a doctor.
Surgeon has sent you to talk.
Task-
–explain the situation
–address her concerns
I went inside the room
Greeted the examiner
Role player was standing at the door and was very rude. She shouted why her
husband was taken for another surgery. I tried to calm her down. I told her
politely that I would explain to her in detail everything. After so much
reassurance she took her seat.
She was asked why she was not told. I explained to her what exactly happened.
I explained to her why it was important to take him to theatre for another
surgery.
I explained how such complications sometimes happen during surgery and how it
is tackled by our expert surgeons
She asked if she could meet him….I reassured I will make necessary
arrangements after the surgery for her to meet her husband as well as the
specialist.
She was worried as she had to pick her kids from school. I said I would arrange
for a social worker.
A lot of reassurance was needed for this case.
I told her she would be updated about her husband’s Condition. Asked her about
any more concerns she had. More empathy and reassurance were given
When the bell rang she seemed to be satisfied.
Key steps: 4/5
Approach: 4
Explaination : 4
Pt counselling: 4
Global score: 4 – pass

Feedback 8-11-2018
you ,re , intern in surgic,l unit , middle ,ged l,dy underwent emergency l,p,roscopic cholecystectomy
,surgery w,s uneventful ,nd moved to post op observ,tion w,rd.post op observ,tion indic,ted
t,chyc,rdi, ,nd ste,dy drop in BP. He w,s tr,nsfused with PRBC ,nd Pl,telets ,nd now she she h,s been
shifted to Ot ,nd Surgeon h,s discussed with you most likely suture h,s slipped ,nd ,sked you to t,ke
consent from wife.
T,sks
Arr,nge consent for procedure from husb,nd
My -ppro-ch w-s,when I entered the room husb-nd w-s there -nd he w-s re-dy to le-ve the room(-
nother door w-s there opposite side),I introduced myself -nd he s-id Iʼm very concerned -bout my
wife no one h-s informed -bout her -nd th-tʼs y I donʼt w-nt t-lk to -nyone. I showed emp-thy,I
underst-nd th-t ur concern but donʼt Iʼm here to help you -nd my sincere -pologies to you bec-use
del-ying inform -bout your wife -nd he c-lm down c-me to ne-r. I s-id John h-ve se-t, ru comfort-ble to
discuss -bout your condition? Yes dr,c-n you tell me how much do you know -bout your wife
condition? He s-id,dr she suddenly developed p-in in her tummy -nd c-me to hospit-l,dr told me she
required surgery.i -sked,did the surgeon expl-ined -bout the procedure -nd itʼs complic-tions? Yes
expl-ined with my wife but I w-s not th-t time bec-use my children were crying. Then I expl-ined –
bout hep-tobili-ry system -nd I drew - di-gr-m -nd expl-ined.then expl-ined –bout wh-t is cholecystitis
-nd th-tʼs y she required surgery -nd -fter the procedure she shifted to post op room for observ-tion.i
-sked ,h-ve you understood so f-r? Do you w-nt me repe-t -nything? He s-id no dr.During observ-tion
her blood pressure w-s going down -nd he-rt r-te is incre-sed th-tʼs me-ns le-k-ge of blood inside the
tummy but we st-bilised your wife by giving blood. He -sked me y dr my wife developed this complic-
tions -nd is this one bec-use of surgeon mist-ke? Iʼm going to sui the dr. I told him I underst-nd th-t ur
wry -bout your wife but see every surgery h-s it own complic-tions,unfortun-tely youʼre wife
developed one. Your wife is in s-fe h-nd -nd we c-n t-ke c-re of her.he -sked me wh-t is next step dr? I
s-id we need to loc-te the bleeding mostly bec-use of slipp-ge of sutures -nd we h-ve to fix th-t one
otherwise pooling of the blood inside the tummy. After the procedure surgeon will come -nd expl-
ined -bout the procedure -nd they fixed but donʼt wry she is in good h-nds -nd for this procedure u
need to sign on the consent form. He s-id ok dr. Fin-lly I s-id I would like to -rr-nge the soci-l worker to
t-ke c-re of ur kids during this time.
Bell r-ng Glob-l score 4
Key steps 1 2 4 5 yes 3 no Appro-ch 4 Expl-n-tion of procedure 4 Pt counselling 4

94-primary amenorrhea
Sample case/ you are at your GP when 17 years old Maya presents to you with complaint of not
starting her periods yet.
Tasks
-take further history
-Physical examination from examiner
-DDX with the patient
-further Ix with the patient

Differential diagnosis
1-hypothalmus
-eating disorders
-exercise induced amenorrhea
-stress induced amenorrhea
-chronic illnesses like liver disease or diabetes
-severe depression
-kalman syndrome
2-Pituitary
-hyperprolactinemia
-thyroid
3-ovarian
-turner syndrome
-Premature ovarian failure
-PCOS
-chemotherapy
4-uterine
-pregnancy
-mullerian agenesis
-androgen insensitivity syndrome
5-Vaginal
-imperforated hymen
-transverse vaginal septum
6-adrenal (Cushing’s)
7-delayed puberty

History
1-ask her regarding her concerns?

2-Differential diagnosis questions


-do you have monthly basis pain or cyclical pain? (Imperforated hymen, TVS)
-have you developed secondary sexual characteristic like having your breasts developed, having
pubic and axillary hair? (Turner syndrome)
-any hot flushes, heavy sweating, mood changes? (Premature ovary failure)
-any weight gain, hirsutism, acne? (PCOS)
-any weather preference? How’s your bowels habit? (Thyroid)
-any headache, blurring of vision, milky discharge from nipples? (Hyperprolactinemia)
-do you think you are excessively overweight? Do you try to lose weight through crash dieting or
exercising? (Eating disorder)
-how much exercise do you do? How many hours? (Exercise induced amenorrhea)
-any stress at home? Are you working or studying, any stress at work or UNI? (Stress induced
amenorrhea)
-are passing large amounts of urine? Do you feel thirsty? (Diabetes)
-any past history of liver or kidney disease? (Liver/ kidney problems)
-are you on any medications? (Chemotherapy)
-any change in the sense of smell? (Kalman syndrome)
-are you sexually active? Have you ever been sexually active? Any acne problem for which you
take OCP? Have you had Gardasil vaccine? (Pregnancy)
-SAD
-when your mother or sister first had their periods? (Delayed puberty)

Physical examination from the examiner


1-general appearance
-BMI
-Dysmorphic features (short stature, web neck, wide carrying angle, wide spread nipples)
-verilisation
-acne/ hirsuitism
-Tanner staging (pubic and axillary hair, breast)

2-Vital signs

3-all systems
-abdomen for mass
-thyroid
-pelvic examination (just inspection) looking for imperforated hymen. Atrophic vagina)

4-Office tests
-urine dipstick
-urine pregnancy test
-blood sugar level

Explain the diagnosis


-there are several reasons why you have not started your periods yet.
-let me explain to you using a diagram (Draw hypothalamic pituitary ovarian axis). There is an
axis in our body we call HPOA that control your normal menstrual cycle. Your periods usually
regulated by hormones secreted from the brain (hypothalamic-pituitary) then act on the ovaries,
which in turn produces more hormones, which act on the womb causing the bleeding of the
periods.
Hypothalamus:
-sometimes lifestyle factors can cause absence of periods like eating disorder because of
excessively low body weight, extensive exercise or any stress at home or work all these can alter
the functioning of an area in your brain called hypothalamus causing n periods. Certain medical
conditions like liver, kidney problems or diabetes can also cause this.
Pituitary
-it could be due to overactive or underactive thyroid glands.
-may be due to a small benign overgrowth from pituitary causing over secretion of a hormone
called prolactin causing no periods.
Ovary
-can also happen in a condition called PCOS when multiple cysts developed in the ovaries due to
relatively high and sustained level of hormones rather than the fluctuating level needed for
normal periods.
-it could be due to genetic conditions like turner, premature ovary failure, or medications.
Uterus
-could be a structural problem like absence of uterus, cervix or vagina in a condition called
mullerian agenesis or due to outflow tract obstruction like imperforated hymen or septum.
Delayed periods
Look from history and examination I couldn’t find anything to suggest one of the above
conditions so it could be just due to a constitutional delay of the periods. But need to do Ix to
make sure it is just normal delay in your periods and rule out others.
-Basic blood tests (FBC, UCE, TFT, BSL, prolactin)
-pelvic US
-LH and FSH
-may need karyotyping according to the results

Note/
in pelvic ultrasound:
-if uterus is present it could be either t outflow tract obstruction like imperforated hymen. If no
outflow tract obstruction it could be because the uterine lining is not responding to hormones so
we need to do another test called pregnancy challenge test.
-if uterus is absent need a chromosomal analysis or karyotyping to rule out mullerian agenesis or
androgen insensitivity syndrome

In LH and FSH
-if low could be due to eating disorder or exercise or stress induced and sometime in
constitutional delay.
-if high need karyotyping for turner syndrome

95-Permanent Sterilisation
1-sample case
47 years old lady presented to GP clinic asking for tubal ligation.
Tasks
-History.
-Counselling

History
1-why
-why you want tubal ligation to be done? (I have heavy periods and my friend told me that it
helps stopping the bleeding)

2-Menorrhagia questions
-How long have your periods become heavy? (6 months)
-when was you last menstrual period? Are they regular?
-how many days of bleeding and how many days apart?
-how severe is the bleeding? how many pads a day do you use? (15 pads)

3-Differential diagnosis questions


Fibroids
-do you feel any lumps or swelling in your tummy or heaviness?
Cancer
-any LOW, LOA, lumps or bumps?
-is your mamo or HPV up to date?
-any bleeding in between the menstruation? Any bleeding after intercourse?
PID
-any fever, vaginal discharge? Any tummy or back pain?
-have you or your partner ever been diagnosed with STI?
Endometriosis
-any pain during intercourse
thyroid
any weather preference? How is your bowels habit?
Bleeding disorder/ blood thinner medications

4-pregnancy
-how many pregnancies have you had?
-any previous miscarriages?
-are you quite sure you do not want any children in the future? Have you discussed this with
your partner?

5-Pill
-what type of contraception do you use? Any difficulty with OCP or SE?

6-General
-PMH, PSH, medications and allergies

Counselling
heavy bleeding.
-from the details, I see you are having heavy bleeding. there are several possibilities: could be
fibroid or benign growth of the womb, endometriosis or when the lining of the womb present at
some other parts, pelvic inflammatory disease or pelvic infection. Could be due to nasty growth
of womb or cervix. Could be due to other medical conditions like thyroid or bleeding disorders or
even medications.
-I could not find any cause from the history so I need to do full examination and also do further
Ix like blood tests, hemoglobin, RFT, TFT, clotting factors and blood group. Arrange for
transvaginal ultrasound.

-if no pathology could be seen in examination or Investigations then this could be a dysfunctional
uterine bleeding which is a quite common condition that happens towards the menopause due
to hormonal imbalance.

Tubal ligation.
-coming to your concern about tubal ligation.
-First tubal ligation is not going to help with the heavy bleeding. It is just a permanent method of
contraception and should be considered irreversible, as the success rate is very low with
reversion. So you need to be sure that you do not want any child in the future.

-Tubal ligation is usually done as a key hole procedure under General anesthesia where 2 or 3
cuts will be made around your belly button. A camera will be passed with tube through one cut
and through the other cut the surgical instrument. The tubes are then identified and will be
blocked using either clips or rings as this will prevent the sperm from reaching the egg.

-there is risk of anesthesia, bleeding, infection, injury to surrounding structures but rare as it is
usually done by expert specialist and staff.

-the advantage is that it is permanent method and also does not interfere with menopause or
sexual desire.
-look as tubal ligation is not helped with heavy bleeds, you have another option like mirena
which is an IUCD having hormone progesterone and once inserted inside the womb it controls
heavy bleeding and also provide long term contraception for 5 years. And by this time you will go
into menopause. Do not worry about contraception as it is almost similar to tubal ligation but its
course is reversible.

-ask if your partner is willing to do a vasectomy as this will carry less complications than tubal
ligation.

2-AMC exam Case


GP, 47 years old woman came to request for sterilisation. She has 3 children and the youngest is
17 years old. She has been taking OCP since the 3 rd child.
Tasks:
-History
-Physical examination findings from examiner
-Explain about the sterilisation method
-Explain further management

History
1-sterilisation questions
-I can see that you are here requesting for Sterilisation. Which type are you thinking about?
(tubal ligation).
-why you want tubal ligation to be done? (I do not want to take OCP anymore and my friends
had tubal ligation and it was good)
-all right can I appreciate you come her to discuss about tubal ligation but before that can I ask
you a few questions?

2-5Ps questions
Pill
-for how long have you been taking the pills? (30 years)
-what type of pills do you take? (Microgynon 30)
-any difficulty with OCP? Any side effects? (No)
Periods (normal)
-LMP? Regularity?
-heavy bleeding or pain during menstruation?
-bleeding in between menstruation?
-any hot flushes, night sweating or mood changes? (No)
Pregnancy (normal)
-how many pregnancies have you had? (She has 3 children and the youngest is 17 years old).
-any previous miscarriages?
-are you quite sure you do not want any children in the future? Have you discussed this with
your partner?
Sexual (normal)
-support from partner?
-STI?
Pap and HPV
-when was your last pap or HPV vaccine (4 years ago) it is important for your cervical screen to
be up to date I can arrange another consultation to discuss about it.

3- General
-PMH (hypertension, diabetes, high cholesterol, stroke, heart, liver, clotting problems)
-PSH (laparoscopic appenicectomy)
-Medication and allergies
-SAD
-Family history

Physical examination findings from examiner (I doubt it would be a task really but one feedback
said and other if it was just do like any gynecological exam case )

Tubal ligation Counselling


- First I want to mention that tubal ligation is a permanent method of contraception and should
be considered irreversible, as the success rate is very low with reversion. So you need to be sure
that you do not want any child in the future.

-Tubal ligation is usually done as a key hole procedure under General anesthesia where 2 or 3
cuts will be made around your belly button. A camera will be passed with tube through one cut
and through the other cut the surgical instrument. The tubes are then identified and will be
blocked using either clips or rings as this will prevent the sperm from reaching the egg.

-like any surgical operation it carries some risks so with tubal ligation there is a risk of anesthesia,
bleeding, infection, injury to surrounding structures but to happen is rare as tubal ligation is
usually done by expert specialist and staff.

-the advantage is that it is permanent method and also does not interfere with menopause or
sexual desire.

Further management
-As you are 47 now, so in a 3-4 years you could go for menopause, If menopause happen there
will be no risk of pregnancy.

-In the meantime I can give another option for contraception that you can consider, which is a
copper IUCD; a small plastic device with copper wrapped around its stem. Once inserted inside
the womb it can provide long-term contraception for 5 years. And by this time you will go into
menopause. And it is almost similar to tubal ligation but its course is reversible.
-ask if your partner is willing to do a vasectomy as this will carry less complications than tubal
ligation.

Ending
-Assess patient’s understanding.
-Dear ---, you do not have a make a decision today. I will give you reading materials for tubal
ligations and other forms of contraception so you can read at home. When you decided which is
best for you we can make another appointment and we will proceed from there. I
- will refer you to the specialist if you would like to have tubal ligation. I will also assist you if you
think about other form of contraception.
-it is important to have regular cervical screening if you want

Feedback 20-2-2018
GP, 47 years old woman came to request for sterilisation. She has 3 children and the youngest is
17 years old. She has been taking OCP since the 3 rd child.
Tasks:
-History
-Physical examination findings from examiner
-Explain about the sterilisation method
-Explain further management

2 min thinking: need to clarify what kind of sterilisation? Ask reasons. Take a normal gynae
history. Remember to give very brief explanations and other form of contraception in
management.

History:
Introduce myself. Patient stated that she is thinking about tubal ligation as she does not want to
take OCP anymore and her friends had tubal ligation and it was good. I expressed to her it is very
good for her to come here today and we can talk about tubal ligation in a short while. Get
consent to ask a few more questions about her general health.
PMHx: unremarkable
5Ps: pills – Migrogynon 30, missed some pills recently, pregnancy – unremarkable, PAP smear – 4
years ago, I explained there is a new cervical screening program and we can talk about it during
next consultation, Partner – unremarkable, periods – regular, no signs of premenopausal
symptoms
FHx: unsure about when did her sisters/mother attain menopause
Past surgical history – laparoscopic appendectomy.
SADMA – unremarkable
Physical examination: – Unremarkable (forgot whether it was given in a card or have to ask
verbally)
Explanation:
Drew a diagram and explain very briefly about tubal ligation (key-hole surgery as what you had
for your appendectomy, clip/suture/ensure coil on both the fallopian tubes). It prevents the
ovum and sperm from meeting each other, however does not disrupt hormonal production by
ovaries. A form of permanent sterilisation, very good success rate. Done by gynaecologist, risks
includes surgical and anaesthetic. Assess patient’s understanding.
Further explanation:
As you are 47 now, which you might become menopause in a few years’ time, there are other
forms of contraception which you could consider. I explained Implanon (lasts 3 years) and
Mirena (5 years). Assess patient’s understanding.
Dear Jane, you do not have a make a decision today. I will give you reading materials for tubal
ligations and other forms of contraception so you can read at home. When you decided which is
best for you we can make another appointment and we will proceed from there. I will refer you
to the specialist if you would like to have tubal ligation. I will also assist you if you think about
other form of contraception.
AMC Feedback – Contraception request: PASS
Feedback 20-4-2018
7-9-2018
47yo woman come for sterilization she is on ocp ask hx pap smear was
before 5 years ago , pefe all normal , counsel about sterilization, mx plan
96-Advanced age pregnancy
42 years old woman comes to your GP clinic after her Home pregnancy test appears positive
tasks
-history
-PEFE card
-counsel the patient

History
1-approah
-I can see that you are here for checkup as your HPT is positive. Is it a planned pregnancy? (yes)
-congratulation!

2-5Ps questions
periods
-when was your LMP? (6 weeks ago)
-were they regular? (yes)
-any pain or heavy bleeding during periods? (No)
Partner
-are you in a stable relationship? (yes)
-is your partner supportive? (yes)
-have you or your partner ever been diagnosed with STI? (No)
Pregnancy
-is this your first pregnancy? (yes)
-have you had any previous miscarriages? (No)
-do you have Nausea, vomiting or breast tenderness?
-any tummy pain, vaginal bleeding or discharge? (No)
Pills
-what contraceptive method were you in? (OCP)
-how long have you been off the pill? (since 1 years)
pap or HPV
-is your pap or HPV up to date? (yes)

3-General questions
-do you take any medications?
-do you start taking folic acid?
-Past medical history (HPT, DM, SLE, Epilepsy, heart) (No)
-SAD (does not smoke or drink)
-family history of birth defects?
-diet and vaccination?
-blood group?
PEFE card (everything normal except BMI 32)

Counselling
-I finish examining you let me assure you that everything looks fine. I could not find any serious
problems.
-your vitals are normal. Your heart is fine and tummy as well. There is one thing, which is your
BMI which is thing we use to assess your weight which appears to be higher than it should be.
Normally it should be less than 25 but yours is 32.

-it is good that you are here so that I can discuss about your pregnancy.
-firstly, I need to confirm your pregnancy by doing an office pregnancy test.
-I would also like to arrange some routine blood tests for you: FBE, Blood group and RH, BSL,
UCE, urine MCS, LFT, vitamin D, serology for chicken pox and German measles, and STI screen
with your consent.
-you need to follow some life style measures:
* take a healthy balanced diet rich in fruits and vegetables, cereal and bread,, avoid soft cheese,
raw meat,
*increase your fluid intake
*limit coffee to 2 cups a day. I appreciate that you stop smoking and drinking alcohol as it is good
for your pregnancy.
* do regular non-contact exercise
*I can also refer you to a dietician Will help you attain the ideal weight.
- I would you to continue with your folic acid but I will prescribe a higher dose, to be taken in the
1st 3 months of the pregnancy.

-Because of your advanced age, I need to refer you to high-risk pregnancy clinic where you will
be seen by a specialist and will have frequent regular ANC checkup with regular visits up to your
delivery.
-during each visit, your body weight and BP will be recorded along with baby wellbeing.
-let me assure you that many woman at your age can go through normal pregnancy and have
safe delivery. But for further precautions we offer close monitoring as there is a risk of having
complications during pregnancy with increasing age for example: DM, high BP, PROM, Preterm,.
Any of these if detected can be managed properly by specialist at high-risk clinic.
-for the baby there can be a risk of birth defects and the most important one is Down syndrome.
So that we offer screening tests at 11-13 weeks of pregnancy which is a combined test; blood
tests and us in addition to confirmatory test in the 1st or 2nd trimester; CVS in 1st trimester when a
needle inserted through the tummy to take a sample of cells from placenta. And amniocentesis
in 2nd trimester; when needle passed into your tummy to take portion of fluid in the bag of
water surrounding the baby. Risk of miscarriage with CVS is 1% while 2% for amniocentesis.
-you will have Ultrasound imaging at 18 weeks and repeated ultrasound at 32 weeks, sugar test
at 28 weeks and bug test at 36 weeks.
-lots of things but Do not worry I will give you reading materials about all of these.
-red flag. (Tummy pain, bleeding, discharge)
-review.

97-Epiglottitis
18 months old baby with high fever and difficulty breathing
Tasks
-history
-PEFE
-DX, Ddx
-Immediate management

Differential diagnosis
1-Epiglottitis
2-pneumonia
3-peritonsillar abscess
4-croup
5-foreign body
6-anaphylaxis

History
0-hemodynamic stability
1-shortness of breath questions
-when did it start? Sudden or gradual? Constant or come and go?
-can you describe it for me? is it rapid or slow? does he turn blue?
-any accessory muscle use?
-does anything make it better or worse?
-is this the first time?
2-Fever questions
-how high is it? Did you have the chance to measure it?
-any recent viral infection? Recurrent infections?
-any contact with someone with similar symptoms?
-any rashes?
3-Associated symptoms questions
-any cough?
-any noisy sound in his breathing? Can you describe it for me? (Yes) (Mother acted the voice as
stridor so you will know it is epiglottitis case)
-any mouth breathing? (Yes)
-any drooling? (Yes)
-is he sitting still?
-any difficulty swallowing or pain during swallowing? (Yes)
-any hoarseness of voice?
4-dehydration questions
-does he look drowsy ?
-any difficulty eating or sleeping?
-any reduced in the number of nappies?
5-BINDS
-any problems during birth of the baby?
-is his immunization up to date?
-nutrition(already asked)
-is he thriving and growing normally?
-contact (already asked)
-anyone smoke at home?

Physical examination from the examiner


1-General appearance
-alert, irritable, restless, drowsy.
-position (is he sitting still, in tripod position, drooling saliva) (all positive)
-dehydration, rash, pallor, jaundice, cyanosis, LAP
-respiratory distress signs
*nasal flaring
*accessory muscle use
*subcostal or intercostal recessions.
2-Vital signs especially temperature and O2 sat. (High temperature)
3-growth chart
4-Ear and nose only (NO THRAOT CRITICAL)
5-chest examination
inspection: deformity, chest movement
palpation: chest expansion, tracheal position
percussion: dullness.
Auscultation: air entry, breathing sounds, wheeze or crackles, STRIDOR (yes)
6-quick CVS, abdomen

Explanation
-from history and examination your child most likely have a condition called epiglottitis.
-epiglottis is a flap in the throat made up of cartilage mostly. Works as a valve to prevent food
and liquids from entering the windpipe when we eat or drink.
-in epiglottis there is inflammation and swelling of epiglottis usually caused by a bug bacterial
one called Hib, causing airway blockage and breathing difficulty.
-it is a emergency condition, which needs immediate attention as it can block the airway
completely.
-other possibilities are infection of lung, foreign body, pus collection around the tonsils, allergies
but less likely from history and examination.

Treatment
-reassure the patient.
-he needs admission to hospital to be seen by a specialist ENT.
-they will monitor O2 level, secure airway if O2 low, then O2 by mask.
-they will give IV fluid if dehydration
-they will give antibiotics.
-they will do Ix if needed

Feedback 30-5-2018
as i entered asked examiner how the child looked like nd vitals with saturation.. he said child
looks very sick..
I asked any drooling saliva/tripod position.. he said leave that to examination, u can talk to mum
So i intro to mum nd reassured her.. took hx for presentjng complain nd all ddx. Positives were
breathing diff , drooling saliva, fever nd stridor(mother acted to show me stridor)
In exam.. pos were child sitting still not moving head with tripod positon drooling saliva ., fever
tachy.. ear nose clear. I mentioned i ll not examine throat.. nd did chest .. he told soft expiratory
stridor. Rest exam N
Then i thanked nd returned to mum.. explained her epiglottis by a diag.. reassured her again..
called senior.. told IV line, bloods, uv antibiotics nd O2 in lay man terms nd also that i ll call
specialist anesthetist to pass ETT in lay man terms.. reassured her again. I finished before time.
Hope it helps
98-Pre pregnancy DM counselling
AMC Case (9-2-2018/ 1-6-2018/ 6-6-2018)
Woman Type 1 DM on insulin. Plan to become pregnant. Her husband is a farmer and they are
living in a rural area 2.30 hr from tertiary hospital. Would like to conceive but wants to have care
and delivery in local rural hospital.
Tasks:
-take history
-PEFE
-convince to have ANC is tertiary hospital.

History
1-Approach
-Hi my name is --- I’m one of the doctors here I understand from the notes that you have DM and
you’ve come to discuss about pregnancy planning. Is that right?
-when are you planning for the pregnancy? (in 6 months)
-it is a great start from you to come here and discuss about it. So we can address it together.
-First, can I just ask you a few questions to assess your current health condition?

2-Diabetis Mellitus questions


-since when have you been diagnoses with DM? (Since 9 years old)
-do you feel thirsty? Are you passing large amounts of urine? Any trouble seeing? Any skin or
vaginal infections? (No)

3-Medications questions
-what medications do you take to control you DM? (Insulin)
-do you take it regularly? (yes)

4-Well control questions


-have you had regular check-up with your physician? When was the last one? What was the
result? (7 months ago and HBAIC is 11%)
-do you regularly measure you BSL at home? What was the most recent result?
-is there any time when your BSL was too low or too high that you had fainting episodes or need
hospitalisation?

5-5Ps questions
-Periods: LMP, regularity (2 weeks ago and regular)
-partner: support, STI? (Normal)
-pregnancy: 1st or not, previous miscarriage (1st)
-pill (No)
-pap or HPV (up to date)

6-investigations
-have you had any blood tests like (blood group, rubella or varicella serology)
7-May I know the reason why you want to deliver in rural hospital?
(I wants my husband and family members to stay around me)

Physical findings from examiner


1-GA (PODL)
2-VS
3-BMI
4-chest and heart:
air entry, breathing sound, dullness, adventitious sounds, apex beat, heart sounds and murmurs.
5- focus abdomen
inspection: distension, mass
palpation: mass, tenderness
auscultation: bowels sounds
6-pelcvic
inspection of vulva and vagina: discharge, bleeding, rash, vesicles
speculum: healthy cervix, discharge
Bimannual: uterine size and tenderness, adnexal mass and tenderness.
7-office tests
-urine dipstick (glucose 2+)
-BSL

Counselling
Effect of DM on pregnancy (mother and baby)
-all right, first let me assure you that many woman with diabetes can have normal pregnancy and
can get through safe deliveries with healthy babies. However, you need to be closely monitored
during pregnancy as there are some issues that might arise during pregnancy.

-because of DM, you could have increase in BP or increase amount of fluid in the womb called
polyhydramnios. Having DM can increase the risk of birth defects or having a big baby, it can
cause breathing problems to the baby. However, these risks can be minimised by keeping your
BSL during pregnancy under control in addition to monitoring you closely with the help of MDT.
So if any complications happened it can be dealt with appropriately.

Prepregnancy
-from history and examination it seems like your blood sugar is not well controlled so I would like
to perform some blood tests like BSL, HBAIC to assess DM control.
-I would also like to refer you to diabetic physician for complete assessment of nerves and
kidneys and your DM.
-refer you to eye specialist to check your eyes.
-once all investigations are normal then it will be safe to attempt pregnancy.
-I will start you on folic acid now, (2.5 mg high dose)to be taken prior to pregnanct and continued
for the 1st 3 months of pregnancy. And will also arrange all routine blood tests usually performed
at 1st antenatal visit.
During pregnancy
-you will be managed at high-risk pregnancy clinic by MDT
-you might need increase In insulin requirement to keep BSL under control.
-need to go through more frequent antenatal checkups

Delivery
-will be in tertiary hospital under specialist guidance. Usually planned at 38-40 weeks

After delivery
-insulin dose will be return back to Prepregnancy level.
-you can breastfeed your baby.

Address rural concern


-I understand that the presence of your husband or family members is very important and
comfortable to you.
-the reason why you need to be managed in the tertiary hospital is that rural hospitals have
limited facilities as well as specialists, and due to all possible complications that can happen
during pregnancy like big baby, high BP, PROM it would be best for you and your baby to be
managed in tertiary hospital where facilities are available there.

Feedback 1-6-2018

Pre preg counselling. T2 DM. Task: Hx, Counseling


6 months ago last HbA1c 11%. Presently RBS 8. Under insulin. Asked DM questions, pre preg
questions. Explained need to repeat HbA1c, complications in preg (to mother and to baby) and
what to expect, increased insulin demand etc….

Feedback 1-6-2018
Pre preg counselling. T2 DM. Task: Hx, Counseling
6 months ago last HbA1c 11%. Presently RBS 8. Under insulin.
Asked DM questions, pre preg questions. Explained need to repeat HbA1c, complications in preg
and what to expect, increased insulin demand etc….

Feedback 1-6-2018
Type 1 DM on insulin. Plan to become pregnant in 6 months.husband is a farmer and they are
living in a rural area 2.30 hr from tertiary hospital.Prepragnancy counsling. C wants to deliver at
rural hospital.
Hx, PEFE,advise.(DM since her 9 years. Couldnt remember her blood sugar. Last HbA1c 8.last
specialist visit 7 months ago. On Ex all normal except urine sugar2+.)

It was a confusing station as she didn’t have good control ps wanted to deliver in rural set up

Any ways in hx asked her about her DM control(not controlled, HB1AC deranged, last visit to
specialist 6-7 months back, regular with her meds i.e insuln don’t remember how much she was
taking , no compli on hx like BOV, numbness tingling, sob, excessive weight, frothy urine, urine
frequency, no discharge, recurrent infection over all she didn’t give any complication of dm from
hx).
Then I asked about 5 Ps: last period 2 weeks back, previous preg: no, pap: recently normal, pills:
no , partner: healthy no std no health issues and is supportive, Blood gp B +ve, no other chronic
ailments.
I asked her reson to deliver in rural set up and she said that she wants her family members to
stay around her and no financial and other issues

PEFE: quckly asked examiner for stigmata of dm complications and examiner said al normal

MX:
I told her after seeking hx I don’t think it’s the right time to conceive as well controlled DM it self
make a pregnancy high risk for mum and the baby and in your situation ur blood sugar is not
controlled . first we need to run fresh set of investigation, seek specialist review for DM
management and adjustment of insulin accordingly, maintain DM diary and started her on folic
acid then told her compli of dm on pregnancy , risks to her and baby . and told her that I know
its comforting to see all the loved one around you once u r deliverling the baby but because of
high risk issues and complications I would suggest you to review your decision and arrange
family meeting with ur consent as she needs frequent monitoring during pregn and at the
delivery even after delivery there are risks for baby like RDS. I said how do u feel about mx plan
she said I will do what ever you will suggest I said good lets work as MDT (referral to DM
physician, high risk preg clinic ince she is pregn, but ran antenatal tests, and dietitian referral)

It was a long station could cover rest


Feed back: pass Global score: 4
Key step 1&2 : no, 3 & 4 : yes Approach to pt: 3
Hx: 5 Choice and technique of exam :5 Pt education: 5
Feedback 10-11-2018
pregnancy advice DM type 1

She was dx since 9 yo so far no hx hospitalised due to her illness, routine checking up her BSL ,
last time to see sp 3 weeks ago all were normal , blood test normal eye normal, no
bowel/urinary problem , no weight loss no vaginal problem no hx of STI in stable relationship
.Partner supoortive , living in rural want to be deliver in local hospital , periode , papsmear were
normal never pregnant before didn’t take any contraceptive

SADMA only, diabetes + finding .

PEFE : I was confused what to ask in here I asked from head to toe BMI normal VS normal

Ix : bsl 11 , glucosuria + 2 , UPT not provide

Counselling : reassurance that she can have normal pregnancy like other women but need close
monitor as there’s some issues that may arise during pregnancy : such from from baby like …

From your pregnancy it self such as …..those could be happen as well in normal pregnancy .

From delivery as well …that’s why delivery will be planned and the facility that has all supports
you need.

Looking at those issues thast why we need to control your BSL before you start falling pregnant
that why it a great start from you to come here so we can address all those and put everything in
place .I will refer you to diabetes specialist for that as well as dietitan that could help control
your BSL once reach 5 to 7 its good environment for baby to grow and yourself. I will give you
script for high dose folic acid to minimise the complication to baby as well that u need to take
before pregnant and I will see you again after you coming back from specialist

Address about her wish deliver to rural hospital that is quite understanding that she need to her
support , unfortunately due to all those possibilities that may happen and we need to prepare
and rural hospital has limited facilites as well as specialist that would be the best for her in
bigger hospital as you and your baby well being is our most priorities .

My head thinking should I do ANC now or not but time tickling as her BSL just too high so I
decided I didn’t say it and address that 1 st

Give some reading material

Feedback passed

Approach /hx/pefe/counselling/ 6/54/6


Feedback 6-6-2018
type 1 DM prepregnancy advice- pass

28 years old , type 1 DM since childhood, controlled well. Now she wants to get pregnant.

Task: history

Explain possible cx to patient if she gets pregnant

Explain possible cx to baby if she gets pregnant

History: appreciate the patient coming for advice prior to pregnant. Any specific concern?

Type 1 DM, seen by specialist regularly, on insulin, compliant with meds, kidney check, eye
check done 6 mths back, was normal. No tingling and numbness. HB A1c within normal limit,
no admission to hospital for DM emergency, diet , exercise, 5Ps, social hx, family hx of
complicated pregnancy no

Cx to mother: uncontrolled DM, DM can cause complications in pregnancy as well as pregnancy


can complicate ur DM, need tight control of sugar to prevent cx, might need more insulin than
usual, kidney, eye pblm, PIH, polyhydramios, pprom,

Cx to child: miscarriage, birth defect, prem, IUFD, big baby, difficult labour, hypoglycaemia,
neonatal jaundice

Don’t worry, I am not meaning all those cx will happen to you. Just explained you of all the
possible cx. We can prevent those by tight control of your DM and close monitoring during the
pregnancy, you will be manage by MDT team. (Please try to reassure the patients after
explaining cx in all case, they looked so worry after explaining about a list of complications)

I had time, so talked to patient about mx briefly ..like taking folic acid from now on , AN blood
tests from now, to see specialist before she gets pregnant, continue diet and exercise .
99-Chronic venous insufficiency
77-year-old woman with bluish discolouration and swelling of both feet for 5 years
Tasks
-history
-Pefe
-Diag with reasons.
-Dds with reasons.
Worried about gangrene.
Pefe- Dorsalis pedis absent, tibialis post weak. Remaining normal. Buerger negative.

History
1-Approach
-I can see that you are complaining of bluish discolouration and swelling in both legs.
-How do you fell at the moment? Do you have any pain?
-Do you any concerns? (Yes, I am worried about gangrene) I can understand your concerns can I just
ask you a few questions first to unravel the nature of the problem?

2-swelling questions
-since when? (5 years)
-constant or come and go?
-is it getting worse? (Yes getting worse for a year)
-any time of the day when the swelling is more or less prominent? (less In the morning)
-is there swelling anywhere else?
-anything makes it better or worse?
-How does it affect your life and daily activities?

3-Associated symptoms
-do you have any pain that happens while walking or exercising and relieved by rest? (PVD, DVT)
-any redness? Is it hot to touch? Any ulcers or discharge? Fever? (Cellulitis)
-have you had any trauma or injury ? (Trauma)
-any tingling, numbness or weakness? (if affecting nerves)
-any chest pain, SOB, palpitations? (Heart)
-how is your urine? Any reduced urine output? (Kidney)
-Any change in colour of urine? any tummy pain, yellowish discoloration of skin? (Liver)

4-General (all imp)


-PMH (diabetes, hypertension, hypercholesterolemia, heart, kidney or liver problems)
-PSH (varicose veins)
-Medication
-travel
-occupation
-SAD
-family history
Physical examination findings from the examiner
1-General appearance
pallor, jaundice, LAP, BMI.
2-Vital signs
3-lower limbs examination (Picture given by examiner showing pitting oedema with blue feet)
-inspection (scars, skin colour changes or ulcers, swelling, deformity, varicosities)
-palpation (tenderness, temperature, CRT, oedema pitting or not) + pulses (dorsalis pedis absent,
tibialis posterior weak)
-feel and auscultate for arterial bruit. (Femoral, renal, aortic)
4-neuro sensation and power
5-special tests (burger + ABPI)
6-abdomen (distension, tenderness, organomegaly)
7-CVS (apex beat, heart sounds and murmurs).
8-office tests
-UDT, BSL

Differential diagnosis
1-chronic venous insufficiency
2-heart problems
3-kidney problem
4-liver problem
5-Peripheral vascular disease
6-Cellulitis
7-DVT
8-Trauma

Explanation
-from history and examination you most likely have a condition called chronic venous insufficiency
which is when your veins have trouble sending blood from your lower limbs back to the heart, so
blood does not flow back properly to the heart causing the blood to pool in the veins of your legs
leading to swelling and bluish discolouration.

-other possibilities are:


heart problems but less likely as no SOB, chest pain or palpitations and heart ex was clear.
Kidney problem less likely as urine output is normal
liver problem less likely as liver ex was normal and no change in colour of skin or urine
PVD less likely as no leg pain
cellulitis less likely as no fever or redness and leg was not tender
trauma less likely as no history of injury.

Treatment
-Investigations (FBE, LFT, UCE, BSL, lipid) (UDT) (ECG) (Doppler ultrasound)
-compression stocking + elevate legs
-SNAP (regular exercise and others)
-refer to specialist
Feedback 4-4-2018
77 yr old lady with bluish discolouration and swelling of both feet for 5 yrs, on antihypertensive
and statins
Task- history
Pefe
Diag with reasons.
Dds with reasons.

On hx condition getting worse for a yr. less swelling in morning. Worried about gangrene.
Pefe- pic was given, pitting edema with blue feet. Dorsalis pedis absent, tibialis post weak.
Remaining normal. Buerger negative.
i passed but with very low score. I said PVD and it can be anything either arterial, venous or
lymphatic insufficiency. Can be DVT and can be secondary to medication, HF, or RF.
100-Abdominal aortic aneurysm
Case 1 (AMC 2016)
An elderly man john came to visit you again after you have organised an urgent abdominal
ultrasound. The ultrasound was organised at the first visit after you detected a pulsatile mass on
his abdomen. He came back after the US confirmed an aneurysm on the abdominal aorta below
the renal arteries a size >5.8 cm. he came to check the situation and he is planning to go on a
caravan trip.
Tasks
-explain the diagnosis
-advise further management
-explain consequences and answer his questions

Counselling
approach
-Hi john, how are you feeling today?
-you must be very exciting for this trip, have you made all the arrangements?

Explain results.
-do you know why US has been done for you? (yes you tested me to find out about the lump you
found on examination the previous day)
-that is right and I have the result with me now. The results showed that you have a condition
called abdominal aortic aneurysm have you heard about it?

Condition
-all right, I will explain it to you with a diagram for clear understanding. (very imp to draw a
diagram)
-this is a swelling or enlargement of a large blood vessel coming from the heart down into the
tummy to supply the organs and legs.
-the result showed a size of 5.8 cm, which is larger than it should be because the normal size
should be below 3 cm.

Cause and risk factors


-it is usually due to weakening of the blood vessel’s wall.
-there are many factors that can predispose to it, like high blood pressure, high level of fat in the
blood, smoking, increasing age in men and family history as well.

Complications
-because the swelling is significant, treatment should be ASAP as if left untreated it could burst
inside the tummy leaking blood and becomes life threatening. But it is good that we have picked
it up at this stage so that we can manage it appropriately and put it under control. So let me
assure you that you are in safe hands.
Management
-I would recommend that you delay the trip and I apologize for the inconvenience it may cause
but I also appreciate that you came to see me before the trip considering your health as a
priority.

-I would like to refer you to the nearest tertiary hospital where you will be seen by a vascular
surgeon who would confirm the diagnosis by doing a CT scan and will then plan further for
surgery most probably.

-if this condition is dealt with in timely fashion it will be easily treatable with modern methods
including open repair and endovascular repair which involve sending a probe down the artery
from inside to repair it.

Long-term advice
-you need to quit smoking and I can arrange another consultation to discuss about it with you.
-we will regularly check your BP, blood fat
-It would be advisable to check other male family members as this condition can run in families.

Question: can I go home and get my wife before going to the hospital?
-it is better that you can go straightforward to the hospital. Meanwhile I can inform your wife
and let her know about it.
Case 2 (AMC 2017+2018)
60 years old male whom u have seen last week and ordered an ultrasound. Report is here and
showing AAA size 3.9 cm ( there was no picture or ultrasound just this description was written).
He is smoker with high bp. He wants to go on a caravan trip or holiday.
Tasks
-explain the result
-management

Counselling
approach
-Hi john, how are you feeling today?
-you must be very exciting for this trip, have you made all the arrangements?

Explain results.
-do you know why US has been done for you? (Yes, you tested me to find out about the lump
you found on examination the previous day)
-that is right and I have the result with me now. Do you have any concern before we begin?
The results showed that you have a condition called abdominal aortic aneurysm have you heard
about it?

Condition
-all right, I will explain it to you with a diagram for clear understanding. (very imp to draw a
diagram)
-this is a swelling or enlargement of a large blood vessel coming from the heart down into the
tummy to supply the organs and legs.
-the result showed a size of 3.9 cm, which is larger than it should be because the normal size
should be below 3 cm.

Cause and risk factors


-it is usually due to weakening of the blood vessel’s wall.
-there are many factors that can predispose to it, like high blood pressure, high level of fat in the
blood, smoking, increasing age in men and family history as well.
-you can ask here questions about these risk factors to reveal if he has any and counsel later
about them.

Complications
-although the size is not very significant but it is good that we have picked it up at this stage so
that we can manage it appropriately and put it under control. So let me assure you that you are
in safe hands.
so with this size management is regular surveillance and monitoring by the specialist; the
vascular surgeon because it can increase in size and if goes more than 5 cm it becomes
significant and could burst inside the tummy leaking blood and becomes life threatening. In that
case, urgent surgery will be required.
Management
-I appreciate that you came to see me before the trip considering your health as a priority.
-I would recommend that you first need to be seen by the specialist surgeon who might do some
other tests and put you on 2 yearly surveillance.

Long-term advice
-you need to quit smoking and I can arrange another consultation to discuss about it with you.
-we will regularly check your BP, blood fat and general health to make sure everything is fine.
-It would be advisable to check other male family members as this condition can run in families.

Any questions??

Reading materials and red flags (imp)

Feedback 15-8-2018
Scenario: Abdominal Mass
Stem: 60ish male whom u have seen last week and ordered an ultrasound. Report is here and
showing AAA size 3.9 cm ( there was no picture or ultrasound just this description was written).
He is smoker with high bp. He wants to go on a caravan trip.
Tasks
~explain the result
~management
Inside there was a man sitting on chair.
I greeted by saying hello (name) how have u been how was ur week ( keep in mind he was not a
new patient).
So the reports are here for the test which we ordered. Whats ur expectation?any concerns? To
all he said ( no) I found the role player a bit confused as he was looking to examiner most of the
times when I asked question.
So I will be asking few questions then I will explain the result.is that okay? He said ok.
Asked questions about chest pain, tummy pain , nack pain, sob, racing of heart, blackout etc.
Then asked about risk factors, alcohol, bp, smoking, cholesterol, dm, dvt, lifestyle, fhx, stress
etc
Explained the condition in the best possible layman term like heart is a pumping organ sitting in
chest there is a big blood tube that comes out of heart and provides blood to the body.
( diagram) called aorta. Normally it has a certain size but sometime size increases. Told risk
factors and symptoms etc
Management depends on size. Though we have found this condition but the good thing here is
size is 3.9 cm( bad news with good news always). With this size we do regular surveillance
with specialist because if the size goes more than 5cm it carries risk of opening up which we
don’t want…in that case a surgery is performed etc.
Told him that he needs to be seen by specialist first who will do some other tests and he will be
put on 2 yearly surveillance. Would be discussing about family screening too.
Told about his smoking that I will talk about this in another consultation.
Any question? Want me to repeat? (no)
Review back. Readin material. Red flags. Referral letter.
Grade: Pass
Feedback 30-5-2018 ‘Abdominal mass’:
AAA 3.9cm –post test counselling, same recall
Task: explain USS report, advise further Mx.
Feedback: Pass
Same approach as recalls.
Established rapport/semi breaking bad style: Ideas Concerns Expectations.

 Remind me why we did that/how much do you know


 I’ve got your results, we’ll go through it step by step
o Just use the report/path/radiology etc to populate your talking points
o Drew a pic –described aneurysm
o Spoke about risk fx/Sx/Complications
 Drew the table of guideline recommendations for surveillance
o 4cm(Q2yr)
o 4.5cm(Q1yr),
o 5cm(q6mnthly), [**call MrVasc]
o 5.5cm(q3mnthly),
 Said he can go on his holiday, but for due diligence I think just touch base with spesh
(who’ll likely say the same thing)
 Asked if he’s got family –may need to consider family screening etc
 Finished this case early

Recall 28-2-2018
Abdominal aneurysm : size 3.9cm, below the renal arteries. Man comes back for USG results. Is
going for a caravan trip in 4 weeks. Explain USG to him and advise management.

101-Head collision and brief LOC


15 yr old boy was brought by worried father after a head collision injury during a game on the
field. Boy had lost consciousness for less than 10 minutes.
Tasks:
-History
-PE from examiner
-Explain condition with reasons
-Management

History
1-reassure
-let me assure you that your boy has been looked after by experts, primary survey has been
done and he is stable at the moment. I just need to ask you a few questions…

2-Before
-can you tell me more?
-before the event did he complain of any strange feeling like aura, strange smell or tunneling of
vision?
-when did he have his last meal?

3-During
-has he lost his consciousness? And for how long? (Ask if not mentioned in the stem)
-has anybody witnessed the event and noticed any jerky movements of his body or limbs or
rolling of eyes?
-has he wet or soiled himself?
-does he have any injury to his head or any other parts of his body? (Yes, swelling in head)

4-After
-how soon did he recover?
-Did he remember the event? Any memory loss? Is he able to recognise you?
-any headache, vomiting or blurring of vision? (vomited once so ask about amount, colour and
content)
-any residual weakness in his body? Any difficulty speaking?
-any chest pain or shortness or breath?
-any problems with his bowel motions or urine?

5-Has this happened before?

6-General questions
-PMH, PSH, history of bleeding disorder
-medications, allergies, SAD.
-Family history
-school performance and stress at school
-home situation and stress at home

Physical examination from the examiner


1-GCS (oriented)
2-Vital signs
3-Head
-bruising, laceration, swelling (swelling in forehead so ask about size, site, fluctuation)
-eyes: ptosis, swelling, raccoon eyes, pupils shape and size
-ears: discharge or CSF leak, battle signs
-nose: CSF leak, swelling bleeding
-mouth: teeth, soft tissue injury
-face: feel for tenderness and crepitus.
4-Neck: tenderness, muscle spasm
5-Cranial nerves 2,3,4,6 especially light reflex and fundoscopy
6-power, tone, reflex, sensation, any lateralising signs.
7-chest, heart and abdomen
8-office tests

Management
1-call supervisor or registrar
2-observation
3-CT/ Blood tests
4-IV fluid if dehydrated

Explain
-from history and examination, your child most likely has a moderate head injury because the
child had a brief LOC and memory loss of only a few minutes and tell any other positive finding
you will have from the history.
-Let me assure you there is no neurological signs to worry about.
-At this time we need to admit him for observation and monitoring.
-I will give him adequate painkillers for his pain
-I will consult with the ED consultant and paediatric neurosurgeon, to come and see him. He
might need CT but the specialist will decide this.
-if he does not take oral feed or vomit. Then He might need IV fluid.
-so in the meantime, try not to stress yourself he will be fine. We are just keeping him for
observation because sometimes whenever there is an injury the impact might be seen later.
-regarding the swelling in his head, it will usually resolve with time, there is no other problems
apart from this.

Examiner will ask for how long will u keep him under observation? 24hrs!!
Case (22/4/2017)
Paeds collision with child in playground
A 15 years old child was playing football and collided with his friend and was brought by his
mom to the ED.
the positive findings were that there is hematoma on the head and the child is unable to
recognise his parents and the vitals are stable (I added myself).
Task:
Hx,
PEFE,
Immediate Mx

Head collision
history
- boy had collision when playing football..
- so asked any injuries anywhere else she said no..
- did he lose consciousness she said yes how long she said less than 10 mins
- wet his self - no,
- he remembered event afterwords- yes but little he forget ..
- how is his school performance- he is doing gud is said..
- home situation is gud happy family..

Pefe:
- asked GA: active, bruise on head he said..
- vs: wat u want he asked bp, pulse then he was telling slowly..
- asked bruises anywhere else- no,
- cranial nerves examination- all gud.. was asking soon he said slowly u have to tell and more
specific
- I asked cardiovascular exam - normal..
- neurological exam - tprcs- normal.

Management
Then talked to her after talking to u and examination findings from examiner ur son is doing well
no prob but as he has bruise and loss consciousness will admit him and will inform specialist and
will do ct scan for him to make sure everything is fine with him.. and once he is fine u can take
him.
Don't worry he will be fine..
Thanked and went out.
Case (29/11/2017)
Child came with collision Hx with a child drowsy. Brief loss of consciousnessthen regaied
consciousness and loss of memory but recognizes her mom.
Task
-Hx from mom,
- PEFE ( bump / swelling on forehead , don’t know about this ),
- Dx and Mx to mother.

comment pass all key steps 4


15 yr old boy got hurt in head while playing football at school. had brief LOC, vomitted once.
brought in ed.
- primary survey done. hemodynamically stable.
talk to mom, history, pefe, diagnosis, management
history
- i asked all about unconscious pt, most of the time mom said i dont know.
pefe
- i started with gcs. he was confused. but examiner said he walked in the room.
- cervical collar removed no findings there.
- chest heart neuro abd,
- fundoscope, pupil light reflex,
- no csf or blood from orifice, all normal, only swelling in front of heaf asked about swelling.
Explanation.
- i said pt had head injury, but there is no neurological sign to worry about
- now,will keep him for observation. monitor frequently. call my senior to come and see. if
things go wrong he may need referral to neurosurgeon and may need ct,xray head but will be
decided by specialist.
if he doesn’t take oral feed or vomit may need iv fluid. the swelling in head mostly resolves if no
other problem with time.
the role player said ‘thank god ‘when i said i will ask my senior to come and see her son.my role
was an intern

Feedback 10-5-2018
-I gave diagnosis of concussion.
-Management i said we will admit and keep under observation and will do a CT/MRI for safer
side. Rest there is no need to worry as he is doing fine for now. Paracetomal for pain, local ice
packs and rest is the mainstay of treatment. Child was oriented but was a little confused so i said
its jus coz of the injury and its common to happen in concussion but there is nothing to worry.
With good rest he will get better. Gave red flags. At the end examiner asked me for how long
will u keep him under observation, so i said 24hrs.
To understand it better u can read through rch guidelines for concussion or head injury.

Concussion= mild head injury!!!

Note/ according to the criteria in RCH this is a moderate head injury but the candidates are
giving different diagnosis with different approaches and still pass. I just chose to do like RCH
https://www.rch.org.au/clinicalguide/guideline_index/Head_Injury_Guideline/
102-HSIL
32 years old woman comes to your GP for the result of cervical screening.
HPV is positive and pap showed HSIL
-take history
-Explain result and treatment

History
1-approach
-I can see that you are here for the results of your cervical screening is that right?
-Before that, can O just ask you a few questions in order to assess your current health and
correlate the history with the results?

2-5Ps questions
periods
-when was your LMP? are they regular?
-how many days of bleeding and how many days apart?
-any pain or heavy bleeding during menstruation?
-any bleeding in between menstruation?
Partner/ sexual
-are you sexually active? (Yes)
-are you in a stable relationship? (Yes)
-have you had multiple partners before? (Yes)
-what type of sexual activity do you prefer?
-do you practise safe sex? Do you use condoms?
-have you or any of your partner ever been tested for STI?
-any pain during intercourse or bleeding after intercourse?
Pregnancy
-have you ever been pregnant before?
-are you planning for pregnancy?
-any previous miscarriage?
Pill
have you had Gardasil vaccine

3-Symptoms questions
-any tummy pain, vaginal bleeding or discharge?
-any Loss of weight, loss of appetite, lumps or bumps?

4-General
-medications and allergies
-PMH, PSH
-SAD
-occupation
-family history of any cancer?
Why you ask me this question? We need to ask such questions as a routine to assess the general
and reproductive health.
Explain results
-here is the results with me, let me explain it and I am gonna draw a diagram to be more clear. If
you have any question or confusion feel free to interrupt me.

-First, these tests are part of cervical screening program. The aim for this screening is to identify
any early changes in the cervical cells before becoming nasty or cancer. So it is not a test to
diagnose cancer.

-this is the womb and this is the neck of the womb or the cervix. In cervical screening we scrape
a sample of tissue to detect the presence of HPV which is a type of virus that can cause
abnormal growth and changes in the cervix. The result shown to be HPV+ve which mean that
you have this virus.
this virus usually transmitted by direct skin to skin contact usually from sex and any sexually
active woman can contract the virus.

-this sample of tissue also been examined for the presence of abnormal cells as there are levels
of cervical cells abnormality. The test showed that you have a high grade squamous intra
epithelia lesion which is a moderate to sever abnormal cells of the cervix.

Is it cancer doctor?
-the presence of HSIL does not mean that you have cancer; some cells are growing and dividing
abnormally, and it is good that we picked it up because, if not treated these can turn into cancer.
So we can prevent this with early treatment.

Management
-as you have abnormal cells so further tests need to be done for you:
UPT to rule out pregnancy
FBE, BSL, UCE, LFT

-would like to refer you to a specialist for colposcopy which is a procedure when the doctor use
light and magnification to see the vaginal and cervical tissues more clearly. (Critical error)
and in some cases they might take a sample of your cervical tissues in a procedure called biopsy.

-treatment will be decided by the specialist. Options could be


local destructive therapy by cryotherapy or electrotherapy
local excision od suspected segment
cone biopsy if the upper segment cannot be seen which is taking a cone shaped segment.

-what if I become pregnant?


it can cause miscarriage in the 2nd trimester or having labour prior to the exact date called
preterm labour. But all of these will be managed and prevented accordingly by frequent
checkups.
Feedback
32 yr old female with hpv +, hsil
-take hx
-explain result and tt

Regarding the case of HSIL


During the two minutes outside I asked myself is this breaking bad news but how come pap
smear is only screening test . So don't forget asking about gardasil vaccine ,pregnant or not ,
Good morning my name is ahmed am your GP today can I get your name ... I understand you are
here to get the results for your pap smear ..
Yes doctor what do you have for me ?
I would like to ask you few questions before we talk about the results .. Why do wanna ask me is
there anything wrong ? I just need to ask you few questions ..ok

Are you sexually active ? Yes when did you start your sexual life ? She said around 16 I think
Are you in stable relationship? Yes
Practise safe sex ? Yes using condom
Multiple partners before ? Yes long time ago
Ever been diagnosed with STD? No
Pain during sex .. No
Bleeding after sex.. No
Have you had gardasil vaccine ? What ? Gardasil vaccine .. No what is that ? Then I explained
quickly about it
Asked about her periods :
Regular .. Yes
Pain ? No
Bleeding in between ? No
LMP: 2weeks
Could be pregnant ? I don't know
Then I asked about signs of pregnancy.. All no
Any pregnancy or even miscarriage? Never
General health ? Ok
Smoke ..no
Drink .. Occasionally
Family hx of cervical cancer ?? Why doctor what is wrong and she started to cry
I offered her water and tissues she didn't take and just tell me what is wrong

I said as you know pap smear is just screening test for cervical cancer .. What screening means ?
Means not confirmatory means of it shows any abnormality we need to do further investigations
..
What this has to do with me doctor (crying )
your pap smear shows you have some abnormal cells .. What abnormal cells mean ?
So i realised am going to have hard time so i draw diagram of the cervix quickly with some cells
and said this is the cervix and this is the lining during the pap smear we took some cells and we
tested it under microscope and it shows some abnormal which means different in shape than
expected for the lining of the cervix we call it HSIL of course she said whaaaat ! I said don't worry
about the name .
Then she said oh my god am I having cervical cancer ? I said no we need to do more further
investigation to confirm the presence if these cells .
Firstly we need to make sure you are mot pregnant so i will run blood test to confirm or
exclude pregnancy .
Then I said i will refer you to gynaecologist he will do colposcopy.. What that ? it is small tube
with camera he will go and look at the cervix looking for the abnormal cells and take a biopsy ..
What is the biopsy ? (�)
take the abnormal cells out and test it and the treatment will depend on the results ..
What could be the ttt? If is confirmed they may do cone biopsy and i talked briefly about the
complications .
If it is LSIL I talked about ablation .
If he has to do the cone will i be able to be pregnant ? Yes but as i mentioned there is increase
risk of premature labor and PPROM .
The bell rang before I mentioned future follow up or mentioned gardasil vaccine because she
kept asking about everything . I was so upset and I said I will not make it for this station ..
But fortunately passed it.

Feedback 1-3-2018
HPV PASS
32 year old female with pap showing HSIL and HPV
hx explain result tx
frankly i hadnt done pap smear as i tought its obsolete now so they wont give it but its AMC you
should expect anything !!
as soon as i read the task i knew im screwed and was cursing myself for leaving it anyway i went
in
i took 5p and sexual hx and gardasil vacc hx ,previous pap,fam hx of gynaecological cancers
explained the result by drawing the epithelium and explaining that some cells are growing and
dividing abnormally however is not cancer but can convert into it so needs to be treated
tx i said il refer her for colposcopy (explained what it is) and then i said they might excise the
area if needed or treat by cautery...vell rang i thanked the roleplayr and examiner
103-Recurrent genital ulcer
Woman with recurrent vulvar ulcer this is the fourth attack in the last 12 months
Tasks
1-take history
2-dx and ddx
3-management

History
1-chief complaint, Ulcer questions
-I can see that you are complaining of recurrent ulcer on the vulva. So for how long have you
been suffering from this? (It is the fourth attack within the last 12 months)
-All right do you have an ulcer at the moment? (Yes)
-Anywhere else (no)
-Has it appeared suddenly? Increasing in size? for how long does each attack last?

2-associated symptoms questions


-do you have any pain? Is it painful? (Yes there is pain but no discharge)
how severe is the pain from 1-10 (severe enough) ask allergy and arrange painkiller
does the pain go anywhere else (no)
anything alleviate or aggravate the pain? (Panadol gives bit relief)

-Any itching, redness, vaginal bleeding or discharge? (no)


-any burning or pain during urination? (No)
-Any fever (No)
-any LOW, LOA, lumps or bumps around your body? (No)

2-5Ps questions.
Period
-LMP and regularity (2 weeks ago and regular)
-how many days of bleeding and how many days apart?
-heavy bleeding or pain during menstruation (no).
Partner/ sexual (critical)
-are you sexually active (yes)
-are you in a stable relationship? (Yes)
-how long have you been in this relationship?
-have you had any previous multiple partners? (Yes)
-do you practise safe sex? Do you use condoms? (No)
-have you or your partners ever been diagnosed with STI? (Not sure the answer what! mostly
No).
-does your partner have similar symptoms?
Pregnancy
-are you planning to become pregnant? (no)
-Pills (no)
-Pap test or hpv vaccine (up to date)
3-general questions
-Medications or OTC (no)
-PMH or PSH (no)
-smoking or alcohol and recreational drugs
-family history
-Travel history, occupation.

Explain
-from history and examination there could be few possibilities why you have recurrent painful
vulvar ulcers

-the most likely cause is due to a viral infection called herpes simplex virus 2. Firstly, it enters the
body and stays in the nerves in your body then it is activated might be due to exhaustion,
infection or pregnancy, sometimes even the pill and high weigh can precipitate it.

-it presented usually as a painful genital ulcer sometimes associated with itching, rash or
discharge, and it can recur again and again.

-this is a sexually transmitted disease; transmitted through unprotected sexual activity, and from
the history it seems like you have history of multiple partners with no use of condoms which
makes transmission most likely.

-Others could be syphilis but unlikely as it is painless or could be other STI like gonorrhea or
chlamydia but unlikely as there is no discharge or tummy pain.

Treatment
-I would like to examine you and arrange some Ix like FBE, BSL, UCE, and urine dipstick, urine
pregnancy test.

-I would like to take swab from the ulcer for M&C.

-I also after your consent would like to test for other STI like Syphilis, HBV, HCV, HIV , take 1 st pass
urine for chlamydia, endo cervical swab for gonorrhea.).

-I will prescribe you oral acyclovir to treat viral herpes infection and painkiller.

-try to rest in warm sitz bath, wear loose cotton clothes and underwear.

-try to avoid sex until active lesions clear and symptoms go away, and It is important to practise
safe sex using condom in order to protect against STI.

-I need also to see your partner and organise STI screening with his consent.

-If you develop bleeding, tummy pain, fever report

-review once result appear and follow you up regularly to ensure everything is all right
gave reading material.
Feedback (13/10/2017)
Young woman came with 4 episodes of recurrent ulcers on vulva over 12 months

Tasks: Hx, Tell the pt the Most likely diagnosis,Csl (Very kind Asian examiner!)

In hx, the pt was having unsafe sex with several men,


No symptoms at the moment no rash, ulcer, pain, discharge or bld.
Pap smear 18 month before (NL) (period was regular and no bld between no pregnancy before
no PID she is on pill (COC).
So I said the most likely condition you are having is called Genital Herpes ulcer have you heard of
it before?
- It is kind of viral infection. Firstly, it enters the body and stays in the nerves in our body thence
it is activated due to the exhaustion or infection or pregnancy or even with pill and obesity and
- it gives the symptoms of rash ulcer itchiness and discharge.
- This can be happened in your partners as well as this is sexual transmitted disease so I need to
visit your partners to examine and treat them as well.
- Meanwhile, I want to run some investigation to rule out other STI like HIV, Syphilis, HBV, HCV
and take swab for bugs like chlamidia and gonorrhea and obviously confirm the Herpes with T-
zanc test if there is any.
- At the moment I will not start medication for you I wait for the result and review you once
result back if any was positive we treat you with 3 days of Acyclovir and then once weekly for 6
month-period.
- It is important to over this period have safe protected sex with condom.
- be cautious about the hygiene and wipe yourself from front backward.
- Have warm bath sitz.
- if symptoms occurred, apart from medication I prescribe lignocaine gel which can help to
decrease the amount of your pain.
- I don't notify DHS now but I need to notify if any of STI screening came back positive.
- We can later talk about other options of contraceptive methods like Implanon or devices and
- so forth about which I can give reading materials if you want…Feedback: Vulval complaint,

PASS(G.S:5)
Key steps:1,2,3 and 4: Yes
Approach to patient:6
History:5
Choice and technique of examination and organization and sequence:5.
Dx/DDx:6
Patient counseling and education:5.
Feedback 25-10-2018 pass
Scenario : vulval complaint
Stem: young female had 3rd episode of ulcers on genitalia.
Task:
Hx
Ddx
Counselling
Inside the room when I asked her if shes in pain or anthing, she said right now she doesn’t have
anything at all. But she had previous episodes of ulcers on genitalia . noe she wanted to get
herself checked that why did she have those ulcers and any other serious problem going on
because she has started a new relationship recently. So I took hx of the ulcers (1st episode was v
painful later episodes were not that bad) Had 3 4 sexual partners previously. Was on the pill so
didn’t use condoms all the time. No sexual relation for last 6 months. All other things negative
Explained her the ulcers are related to HSV and its an STI she said none of her partners had such
a condition. Told her sometimes men are carrying it without any symptoms. Then told other
sexually transmitted infections and that need to do complete STI screen, including some blood
tests, urine tests. Cervical swabs and if any active ulcer swab of that she said no I don’t have any
at the moment. Explained in blood will be doing hiv as well and that I need your written consent.

Safe sex education and told her I can arrange another appointment to discuss about
contraception. Reading material.

GS: 5 5/5 key steps covered Approach to pt :6 Hx:5 Dx/ddx:5 Pt counselling:5

Feedback 25-10-2018 pass


22 year old woman came with recurrent ulcers in genitals, at the moment not there.

Task”
History
Diag/dd
Counselling

I had herpes in mind., told confidentiality, Asked about nature of ulcer- pain discharge, fever,
itchy,anywhere else similar ulcer. Partner? Safe sex? Wt loss, lumps bumps, periods, pregnancy
dd- said herpes, chancre, chancroid. Forgot dermatitis.
Counselling: advised antiviral for next time, all STI check up, warm bath for soothing, safe sexual
practices,

Got 3 for dd
104- Smoking Counselling
There are two cases in AMC
1-ready to quit 2-asthmatic who does not want to quit

Recall 10-2-2018
counselling asthmatic pt fr smoking cessation.role player says he doesnt want to quit it helps him
with stress etc.
Recall 23-2-2018
Smoking counselling. Pt ready to quit
recall 5-5-2018
Counselling smoking cessation for a female lady with well controlled asthma
Recall 5-6-2018
smoking counselling

Handbook case (suitable for the case who wants to quit smoking)
You are working in a general practice. The next patient is a 30-year-old man who has returned to
your practice for follow-up following a recent chest infection. He is a smoker (20 cigarettes per
day). On his previous visit, you had told him that the 'best thing that he could do for his health
would be to stop smoking'. You have examined his chest which is clinically normal.
At this visit, you are expected to follow up his response to your previous advice and counsel him
further about tobacco cessation.
TASKS
1. Assess his motivation to stop smoking.
2. Counsel him appropriately.
3. Discuss treatment options and general resources.
4. Respond to any questions he may have.

APPROACH
If ready to quit
-Hi John. How are you today?
-John, I really appreciate that you are here to let me help you about quitting smoking. As I've told
you before, the best thing that you could do for your health is to stop smoking. Let me just ask
you a few questions about your smoking habits so we can appropriately make a plan for you, will
that be alright?
ASK
(For both ready and not ready to quit)
-Do you still smoke tobacco?
-How many cigarettes do you smoke every day?
-For how long have you been smoking?
-What time of the day do you smoke more?
-How soon after you wake up do you have your first cigarette?
- Do you find it difficult not to smoke in a non-smoking area?
-Is the first cigarette of the day the hardest to give up?
-Do you smoke even when you are sick?
-Have you tried to stop smoking for good in the past but found you could not? What happened?
What happens if you don’t smoke?
-Do you drink alcohol? Do you take any illicit drugs?
-How is your blood pressure? Are you taking the medications regularly?
-What is your occupation? (check for sedentary lifestyle)
-Any past history of epilepsy, kidney disease, or any mental illness? (contraindication to nicotine
replacement therapy)
(If not ready to quit)
-ask about asthma history and if well controlled.

Assess
(for both)
-How motivated are you to quit smoking on a scale of 1 to 10, 1 being the least motivated, 10
being the most motivated?

Advice
(if ready to quit)
-To encourage you more on your journey to quit, I would discuss with you the risks associated
with smoking and the benefits that you will experience when you quit smoking.
(For both ready and not ready to quit)
-Are you aware of the risks of smoking on your health?
-Tobacco smoke is made up of thousands of chemicals that cause cancer.
-Increases the risk of high blood pressure, heart disease, stroke and lung disease.
-smoke can irritate your lungs, increasing the amount of mucus in your chest and restricting your
breathing thus it predisposes you to frequent upper respiratory tract infections.
-If you are smoking at home, it can affect your partner or your children as well.

-Did you know that as soon as you stop smoking your body begins to repair itself?
-it will decrease the chance of having health issues related to smoking.
-Your food will taste better and your sense of smell will improve.
-You will also be able to save more money
-Even if you have smoked for a long time, you will still be able to experience the benefits of
quitting smoking.
-It is never too late to quit smoking.
(If not ready to quit)
-Do you ever feel as if your smoking controls you?
-When you start to regularly smoke more than 10 cigarettes a day, most people will become
addicted to nicotine. Nicotine levels drop every hour or so, so often the urge to smoke is about
your body recognising that it needs another hit.
-Regaining control over smoking feels good.
-This in turn can make you feel more confident about tackling other positive goals, and shifting
to a healthier, happier lifestyle in other ways.
-How do you feel now about the smoking? Do you want to quit?
-The decision to quit smoking is a personal one and in the end you have to want to do it for
yourself. It’s important, though, to remember that your smoking also affects those around you.
Assist
(if ready to quit) (talk briefly if does not want to quit just to inform about methods will have)
-I will suggest you to set a quick quit date within the next 2 weeks.
-Choose an easy day to stop smoking, one when you won’t be under much pressure and will
have plenty to occupy you. Very important is to stick to your quit date.
-Try to aim for total abstinence as it is more effective than cutting down.

-It is important for you to expect to have symptoms of nicotine withdrawal after starting to quit
smoking and to prepare for it. Withdrawal can be unpleasant especially in the first four days, so
it’s useful to think of the symptoms as signs that your body is recovering from smoking.
-It is common to have strong cravings, irritability, trouble concentrating, restlessness, anxiety,
low mood and trouble sleeping.

-Focusing your attention on something that gives you pleasure or is relaxing can help with
withdrawal, such as exercise, which can reduce cravings and withdrawal symptoms. It can also
lower stress and help keep your weight down. Get involved in new or favourite hobbies
-Also focus on relaxation by getting a massage or spa, try deep breathing exercises, listen to
music, or take yoga or Pilates classes.

-Inform your family and friends about your plan to quit. (Feedback: here I asked him if he lives
alone, he said with his partner so I asked him if she smokes, he said yes. So I offered to bring
here on the next visit)

-Take a healthy balanced diet, and take citrus fruits like orange, lemon which will reduce your
craving for smoking. Drink alcohol to safe limit, which is an important trigger for smoking, and
similarly reduce your caffeine intake.

-Based on my assessment, you may benefit from medications called nicotine replacement
therapy. It doesn’t contain chemicals that causes cancer, and you can use it as gum, patches or
inhaler to help you with your cravings.

Arrange (if ready to quit)


-I will refer you to quit line, and support groups to keep you motivated. I will also review you
regularly and monitor your progress, and give you reading materials about this for further
insight.

*If not ready to quit


-reading materials.
-Think about it
-anytime you feel you want to stop come to me and we can make an appropriate plan for you.
Smoking Cessation (2nd materials)
45-year-old Steve is visiting your surgery to have a regular check-up of blood pressure and
cholesterol levels. He is a known hypertensive for the past 3 years and on Coversyl (perindopril)
and rosuvastatin. While shaking hands, you noticed nicotine staining on his fingers and smell of
tobacco when he talks to you.
TASKS
1. Take history from the patient
2. Counsel further about quitting smoking

 Diagnosis and Management


o Ask
 How many cigarettes do you smoke everyday? For how long have you
been smoking? What time of the day do you smoke more? How soon after you
wake up do you have your first cigarette? Do you find it difficult not to smoke
in a non-smoking area? Do you smoke even when you are sick? Have you ever
tried to quit smoking? What happens if you don’t smoke?
o Assess
 How motivated are you to quit smoking on a scale of 1 to 10, 1 being the
least motivated, 10 being the most motivated?
o Advise
 So I can see that you are smoking for a long time. Are you aware of the
effects of smoking on your health? Cigarette contains cancer-causing
chemicals, and smoking increases the risk of high blood pressure, heart
disease, stroke and lung disease. Also it predisposes you to frequent upper
respiratory tract infections. If you are smoking at home, it can affect your
partner or your children as well. At the moment, are you experiencing any
cough or chest pain? Once you quit smoking, it will decrease the chance of
having health issues related to smoking. You will also be able to save more
money, your food will taste better, your sense of smell will improve. Once you
quit, within 24 hours you may develop some symptoms like anxiety and
difficulty in concentration, which usually improves within a month.
o Assist
 I will suggest you to set a quick quit date within the next 2 weeks and
aim for total abstinence. Inform your family and friends about your plan to
quit. Take a healthy balanced diet, and take citrus fruits like orange, lemon
which will reduce your craving for smoking. Based on my assessment, you
need medications called nicotine replacement therapy. It doesn’t contain
chemicals that causes cancer, and you can use it as gum, patches or inhaler. [if
patient has epilepsy, kidney disease, psychotic disorders, pregnancy are
CONTRAINDICATIONS to nicotine replacement therapy]
o Arrange
 I will refer you to quit line, and support groups. I will review you
regularly and give you reading materials about this for further insight.
Feedback 5-7-2018 (ready to quit)
Young man coming to your GP as she wants to quit smoking
Tasks
Take relevant history
Management
This is typical handbook case , I followed its steps
Smoking Hx and counselling (sorry I don’t remember the exact stem)
When I entered the station a young man was sitting on the chair, I introduced myself and asked
him how can I help today? In Hx I assessed the pattern, dependence, withdrawal symptoms, any
previous attempt and why did they fail? Health problem like cough or recurrent URTI? Assess
motivation? Few Qs about mood and SADMA? In management I start by telling him it’s a very
good decision to come here today I explained the harmful effect of smoking shortly and benefits
like saving money, better breathing and smelling, reduce risk of cardiovascular disease like
hypertension and heart disease and risk of lung cancer. I took the paper and told him that we’ll
have 2 line of Mx : general measure like reviewing previous attempt, avoid situation that
increase the risk of smoking, trying new ways to ventilate ur stress, inform ur friends and
family( here I asked if he live alone, he said with my partner, I asked him if she smoke as well, he
said, I offered to bring her on next visit). I talked about lines of NRT and withdrawal symptoms
and how-to mange it.

Passed: global score 7

Feedback 5-12-2018 (important)


Health review
Smoking Counselling( Dr. Muhit)
He recently diagnosed and released from hospital due to Bronchitis. Used to have 30
sticks/day. He came to GP quite a few times before but that time he was not motivated
now he is here to seek help.
Task
1. Assess, dependency, Tolerance and motivation level
2. Tell the process of withdrawal and counsel him accordingly
[+ve- motivation9/10, no SOB/ Chest pain or any other s/s, within 30 min, even when
he’s sick, difficulties in non smoking zone, 1st ciggy hard to give up. Withdrawal effect
none though he tried quite a few times to quit but due work stress he got that back.
Recently not increased the cig No. no other drugs.]
Approach:
Confidentiality statement.
Jayson I really appreciate that you have come to see me again, now I understand that
you have been suffering from Bronchitis and you are given some treatment, how are you
feeling now?(Fine now doc, as you told me before to quit and benefits of quiting but I
was not ready at that time now I am thinking of get rid of it) I said Nice Jayson, really
appreciate that So can you please rate your motivation at this point from 1-10 for me 1
being lowest 10 being highest? He said 8/9, I said awesome Jayson.
Now before going into further discussion I want to ask you some question, is it alryt with
you? (said Ok)
How long you are smoking? ( Since teenage years)
How much you smoke everyday ( 30 sticks /day).
Whom do you smoke with (alone)
Do you drink or take any drugs with it? (no but alcohol occasionally)
Dependence: How long you can go without it( not a single day)?
Have you ever tried to quit? (no)
How long after getting up from bed you have to smoke (said within 20 min)
Do you find it difficult to be in the nonsmoking area? (yes)
Do you even smoke when you are too sick? (yes)
Tolerance: Have you recently increased your smoking amount or frequency? Yes, Why ?
(due to work stress). What about home? Whom do you live with? (partner), Happy with
life? (Yes, no problem in the family)- I said that’s great.
Qickly asked chest pain, leg pain, problem with sex, ( all no)
I asked do you know what are the risks of smoking and what are benefits if you quit? He
said yes doc u told me before. I said great , quickly drew a picture and told harmful
effects coz of Tar, nicotine and other chemicals in it and benefits like food taste better,
more energy, breathe well which is imp for your lung condition as you have already got
bronchitis., you can smell better, save money, prevent serious condition like stroke, heart
attack, bowel problems so on…
Now as you are determined which is good, please set a definite date and stop
completely from that day, it is called going to ‘Cold Turkey’. He said alryt. Inform your
family and friends that you are quiting smoking so that the can support you and also
avoid those people or environment that tempt you to smoke again. He nodded.
Now Jayson when you will quit you may feel restless, sleep problem, dizzy, sweaty we
call it withdrawal symptoms, for that we can consider you giving some therapy called
NRT where 40% of your of your body nicotine will be replaced hence relieving you from
those symptoms. It also decrease the craving.
Types of NRT- Gum , patch inhaler- told quickly how to use it, for 6-10 weeks, gum, patch
may cause some burning in the throat and rash over skin respectively. And inhaler is
quite expensive.
Are you with me so far? (yes doc)
I fall these fails we can use special medication called Champex.
I am arranging some Gov services like Quit line for you., I can see that you r quite
stressed at your work so I will arrange counsellor service for you, Meanwhile try to relax,
engage yourself in yoga/ exercise , fun activities you like.
Try to have plenty of water and fresh lemon juices to decrease craving. If withdrawal S
gets worse I will consider NRT according to your choice. How does it sound to you? (fine
doc)
I will review you frequently probably after 1 week of quitting ans here is some pamphlet
for you about quitting smoking. Bell rang.
Key steps- 1-5( all covered)
Global score- 4
105-Herps Zoster
Recall 7-9-2018
man with shingles pic provided outside
Tasks
-ask history
-pefe
-explain the condition with causes (shingles due to lymphoma)

Case/ 55 year old man complain of rash on his chest a few days ago

History
1-rash questions
Onset.
-how long? Sudden or gradual? Getting worse?
Site+ spread
-where exactly? If rash anywhere else?
Type
-is it painful? (yes) so ask severity and painkillers
-is it itchy
-any discharge
relieving and aggravation
-anything bring on the rash?
-anything relieve it?
1st time
-has this happened before

2-Associated symptoms
-any fever? Night sweating? (night sweating +ve)
-any LOW, LOA, lumps or bumps? (LOW +ve so ask how many Kg and if on special diet)
-chest pain? SOB? Palpitations?
-cough?
-bowel motions and urine, yellowish Discolouration?
-headache, confusion

3-General
-contact history? Family history? Travel history?
-past medical history (any illnesses in the past especially chicken pox)
-PSH
-SADMA
-occupation
Physical examination from the examiner
1-General appearance
-describe rash in the pic (site, extent, colour, scratch marks, vesicles , oozing or discharge)
-pallor, jaundice
-LAP (site, size, tenderness, consistency, mobility) (axillary and cervical LAP +ve)
-BMI

2-Vital signs

3-chest and heart


-air entry, breathing and added sounds, dullness
-apex beat, heart sounds, murmurs

4-abdomen
-tenderness, organomegaly (splenomegaly +ve)

5-office tests
-UDT
-BSL

Explain the diagnosis


-from history and examination most likely you have a condition called shingles or herpes zoster
-it is an infection with a virus called varicella zoster virus that can cause chicken pox. The virus
enter your body and goes to your nerve cells, stay there in an inactive form.
At any time, your immunity becomes low because of any reason it activates the virus and
appears like your case with this rash distribution.

-as it flares up if you have low immunity, so I am concerned that there might be something
happening, and as from the history you have loss of weight, night sweating, LN enlargement and
also enlargement of spleen.

-I need to do some Ix to rule out possibilities like any infections and to any nasty growth like
leukemia or lymphoma.
Arrange for FBE, ESR/CRP, LFT, TFT, UCE, BSL
Urine MCS
Chest X-ray, Ultrasound
refer for LN biopsy

-I will prescribe you antiviral medication acyclovir to reduce the duration of this infection.
-will give you painkillers, emollient, loose cotton clothes, avoid scratching
-avoid contacting with infants/ children especially those who have not had chicken pox, any
people with low immunity system or those taking chemo
-offer Immunisation to family members
-medical certificate

-Red flags (worsening of rash, high fever, headache, confusion, neck pain)
-reading materials
-refer
Case (13/7/2017)
Herpes zoster
62 yo man, painful rash for 1st time. H/ o lumps n bumps.
LOA, LOW. No h/o DM, medication, steroid, STI.
Other h/o unremakable.
Tasks. H/o, PEFE, Pdx n DDx with reasons to patient.
LN in Cx, axillae and inguinal bilateral symmetrical. Spleen enlarged.

Case (22/4/2017)
Herpes zoster- picture given same hb case hx- severe pain.. gave pain killer .. no other rashes ..
stable partner.. Pefe: no other rashes.. told its shingles rash it's herpes infection and as u got it
first time it's very painful.. this virus stays in the body and comes out when u have fever or any
infections he asked oh it stays I said yes but don't worry Will give painkillers and creams for
rash .. no itching he said..antiviral drugs I said and offered vaccination for his wife.. he said it will
be gud.. u will be fine I said.. thanked and came. That's all I remember guys. All the best for all of
you..I just did wat I know.. and didn't scar if I made a mistake just left that part went on to
further leaving mistakes back no more thinking of them

Handbook case 46
A painful rash on the trunk of a 45-year-old child-care worker

CANDIDATE INFORMATION AND TASKS

You are a medical officer in a hospital primary care clinic. A 45-year-old child-care worker
presents with a painful rash on the trunk, as illustrated below.

YOUR TASKS ARE TO:


Take a history about the presenting problem.

Explain your diagnosis and the nature of the condition to the patient.

Advise the patient about management.

(Near the end of the time allotted, the examiner will ask you one or two questions).
AIMS OF STATION

To assess the candidate's approach to a patient with a dermatomal rash from herpes zoster plus
weight loss and tiredness, which could be incidental but may be associated with underlying
malignancy. These symptoms need to be further assessed.

EXAMINER INSTRUCTIONS

The examiner will have instructed the patient as follows:

You are a child-care worker in a kindergarten. You are single and live by yourself.

Task 1
Opening statement:

'I've had a pain in my lower chest and now there is this rash. '
Answer questions about your condition as follows:

• You have been feeling a bit unwell for a few days.

• You have had a burning pain over your lower chest and flank for a few days.

• You noticed today that you have developed a blistery rash that runs in a line around your
chest and abdomen in the area where the pain started.

• In addition, you have lost 6 kg in weight over the past few months and have been feeling
more tired than

usual (you have nothing to add to this general statement. You have noted no disturbance to
any body system function).
• You have had no serious past illnesses, there is no relevant family history. You have no
allergies and are on no medications.

Describe the pain and the skin rash without prompting. Do not volunteer the weight loss and
associated recent tiredness unless questioned first about How has your health been in
general?'or something along those lines. You have been considering having a checkup but have
no other symptoms, and you had not considered that there might be something seriously wrong
until the pain and this rash appeared.

EXPECTATIONS OF CANDIDATE PERFORMANCE

• History

~ Typical history and rash (see Figure 1) of herpes zoster with prodromal preherpetic neuralgia.

Task 2
• Diagnosis

~ Must make diagnosis of herpes zoster. Must show concern over recent weight loss and
tiredness.
Task 3

• Initial management

~ Treat the rash with symptomatic measures such as calamine or cold compresses and a drying
lotion.

~ Use analgesics with or without codeine.

~ Treat with antiviral medications if patient presents (as in this instance) within firs! 72 hours
of the rash — aciclovir, famciclovir or valaciclovir.

Monitor for the development of postherpetic neuralgia which may require further
management.

~ Examine patient and perform investigations for any possible precipitating cause. In this case,
the weight loss and tiredness demand further investigation (no details are required at this
stage).
• Patient education and counselling

~ Explain the cause to the patient (i.e. relationship of herpes zoster to varicella [chicken pox]).

~ Explain that the condition is only mildly contagious, but that chickenpox can be acquired by
those persons in close contact with the patient who have not previously had chicken pox.
Therefore appropriate infection control measures need to be taken including management of
occupational and community contacts (for example, she is a child-care worker in a
kindergarten and young children and babies should not be exposed to vancella zoster virus).

After 5-6 minutes, if the candidate has not discussed these issues, the examiner will ask ~ 'Are
there any unusual features of the condition in this patient?

~ 'How would you manage this particular patient?'

KEY ISSUES

• History-taking must elicit weight loss and tiredness.

• Diagnose herpes zoster/shingles

• Management must consider use of aciclovir or other related antiviral drugs.

• Must advise further assessment regarding weight loss and tiredness and discuss implications of
infectivity.

CRITICAL ERRORS

• Failure to diagnose herpes zoster.

• Failure to consider the possibility of an additional underlying cause in this patient.

• Failure to assess implications for contacts in community and work settings.

COMMENTARY

Task 2
Herpes zoster (shingles) is caused by reactivation of varicella zoster virus (VZV) acquired
originally through primary infection with chicken pox.

• The condition is more common in people over 50 years of age.

• The virus is found in the dorsal root ganglion. In most cases the reason for reactivation is
unknown, although occasionally this can be related to an underlying malignancy such as a
lymphoma, leukaemia or immunosuppression including HIV infection.

• Occasionally patients may get rare complications including meningoencephalitis.

• Post-herpetic neuralgia is an important sequel. The incidence of post-herpetic neuralgia


increases with age, affecting around 30-50% of adults aged 70-79 years.
Karin case
Herpes Zoster (CLL with Herpes Zoster)
Case1: Your next patient in GP practice is a 45-year-old man working in childcare. Since
yesterday, he has had painful rash on the tummy and that patient has lost 6kg of weight for the
last 2 months.
Task
a. History (2 days ago started with pain, redness then rash yesterday in abdomen; 6/10, felt tired
for the last 2-3 months; rash was for the first time; no HTN, DM, surgeries, malignancies, no
problem with waterwork and bowel motions)
b. Diagnosis and Management

Case 2: Your next patient in GP practice is a 55-year-old male complaining of rash on the chest
Task
a. History (painful few days ago and feels like spasms of muscles, no fever, no work, does not
radiation, not feeling well for the last 4 months and had weight loss and night sweats).
b. Physical Examination (not unwell, no pallor, rubbery cervical and axillary LAD, enlarged spleen)
c. Diagnosis and Management

History
- I understand you came to see me because of your rash. Is it only there? How did it start? Did
you take any medications for it? Is it happening for the first time? Did you apply any creams,
perfumes or chemicals to this area? SORTSARA?
- Did you have any cold or flu before it started?
- Have you noticed any children in childcare with similar problems?
- Any fever or do you think you’re hot?

- I understand that you lost weight recently, are you on any special diet?
- Do you feel tired?
- How is your mood? Do you still find things pleasurable?
- Any lumps and bumps in your body?
- Anyone in the family having similar condition?
- Any contacts with animals or pets?
- Whom are you staying with? Are you a happy family?
- SADMA?

Management
- From the history and the rash that I can see, you have a condition called herpes zoster or
shingles. It is an infection from the virus that can cause chickenpox. Most likely, you got it from
the workplace. The virus enters your body and goes to your nerve cells, stays there in an inactive
form, and anytime your immunity is low because of any reason, it activates and appears like this
condition.
- At this stage, I would give you a medical certificate to have rest and give painkillers. Since your
rash started yesterday, I will give you acyclovir which is an antiviral. I would advise you to use
cooling emollients. Do not scratch the lesions because it might spread to other parts. Wear
loose cotton clothing, and avoid contact to other people.
- As I said, it flares up if you have low immunity, so I am concerned there might be something
happening, so I need to check your lymph nodes for any enlargement and we will do some
investigations to exclude some nasty things such as leukemias, lymphomas, infections, etc. We
will do FBE, ESR/CRP, urine MCS, CXR, abdominal ultrasound, LFTs, RFTs, BSL and if you have any
lumps and bumps, we might need to get a sample of tissue for investigation in the laboratory.

- Reading materials. Review once results are back.

- Red flags: worsening of rash, high fever, headaches, neck pains, confusion

Features

- Typical shingles case PLUS: constitutional symptoms like malaise, weight loss, fever; LAD of
neck, axilla, groin (80%), moderately enlarged spleen and liver (50%), mild anemia,
lymphocytosis >15x109/L.
- Management
o Explain shingles and need to investigate LAD.
o Need for urgent referral for biopsy
o HZ is mildly contagious but avoid contact with infants and young children who have never
had chickenpox and avoid contact with the immunocompromised and those undergoing
chemotherapy
o Consider giving VZIG to those immunocompromised contacts who have no history of
varicella.
o Medications: calamine lotion, analgesics, antiviral therapy to reduce duration
▪ Treated within 72 hours of onset of vesicles
▪ >50 years of age ▪ Immunocompromised ▪ Acute severe pain
▪ With involvement of special areas (eye, perineum)

106-Urticaria
Young female with itchy rash. Pic given. Role player is scratching a lot
tasks
-History
-Diagnosis and Differentials
-management

so I do not know much about this case but the urticaria could be idiopathic or post viral so
depends on the history.

History
1-rash questions
-Duration (12 hours)
-sudden or gradual (sudden)
-if rash anywhere else? (Back, trunk, over forearm as the pic)
-has this happened before? (No)
-do you think anything brought on the rash?
-have you used anything to relive it?
-can you describe it for me? colour? Is it palpable? (Red, raised)
-if it Is blanch on pressure? (yes)
-is it itchy (given in the stem if not ask)
-any discharge or oozing?
-is it painful?

2-Associated symptoms
-any swelling on face, lips or eyes?
-any fever? Joint pain? red eyes?
-any cough? Chest pain? SOB? Wheeze?
-any difficulty swallowing? Nausea or vomiting?

3-General questions (to look for triggers)


-past medical history (allergies, eczema, asthma)
-medications
-SAD
-recent viral infection?
-travel?
-contact?
-any new food, cosmetics, soaps, lotion, detergent, new clothes, pets
-trauma, insect bite
-is it related to heat, cold, sunlight
-sexual history
-stress
-family history

Diagnosis and differentials


-from history, most likely you have a condition called urticaria or hives.

-it is a condition that affects the skin and the mucous membrane causing this red raised itchy
rashes.

-it occurs due to release of a chemical substances including histamine from mast cells in the skin.
These mediators activates sensory nerves and cause small blood vessels to dilate or widen
causing redness, oozing of fluid.

-The cause or the trigger for it is unknown in majority of cases but other possibilities could be:
associated with various types of infections which is also common. Other triggers could be insect
bites, reaction to food or medications. Could be due to physical trigger like skin exposure to heat,
cold, exercise, water or sunlight.

-it is a common condition that tends to come and go and can last for a variable period of time.
-it is not serious let me assure you about that and in most cases it is self-limited and resolve
spontaneously within 6 weeks.

Treatment
-during this period treatment with oral antihistamine is advisable to relief the itching and
swelling. It is very safe and needs to be taken regularly.
-refer to specialist for allergy test.
-red flags (anaphylaxis)

Case (4/5/2017)
1- Rash(picture) for 12 hr with itching in young female, nothing in Hx positive (No food allergy,
no exposure to perfumes or new clothes..etc, no contact with sick ppl, no drugs no travel, no
new food, no sexual thing no fever no arthralgias no wt loss... only FHx of anaphylaxis (IT WAS
HARD ONE.
2- Rash over the back and trunk and over the forearms ( photo ), itchy, sudden onset, no allergy
history, no angioedema, no fever, no Fs of HSP, no recent change with soap/lotion. All negative
findings apart from itchy rash.
TASK: Dx/D.Dx with reasons.

Case (9/5/2017)
Itchy Skin rash, photo given.(with swelling and redness on forearm and back), no personal and
family history of allergy. no change in cosmetic and clothing (Rp is scratching a lot).
Task:
explain the finding,
Dx,DDx
explain the mechanism of this finding

Feedback comment pass overall 6


ok friends, it was my exam question.
it was a lady and pic was given typical of hive in back, it was intense itchy and red raised rash.
initially I got confused about was it contact dermatitis or scabies or allergy or hive..
when I entered the room, I asked everything to exclude my DDs, nothing was positive, no allergy,
not happened due to any food (I excluded anaphylexis as well). no h/o astham, eczema, allergy..
nothing was positive, I specifically asked some question to exclude scabis, like any itching to any
skin fold, or any other family member with same problem.
then I diagnosed it as urticaria, treated with anti histamine, give red flag about anaphylexis, call
for review if it wont heal/help with anti histamine. and told her to avoid allergy contact like
carpet, pet, grass, food etc. I don't know wheather I did right or wrong..that's what I did, don't
know about result yet. hope this will help u..
btw, there was no recent URTI history, as I asked her specifically.one more thing, i asked do you
have thia type of attack in past to exclude chronicity, it was acute without any positive history..
I also referred for allergy test as well.
in the counsel part, I told her about mast cell, histamine mechanism, and how u got this rash n
itching and everything.

Case (10/5/2017)
Young female with itchy rash. Had some viral resp illness few weeks back. Rash for 12 hrs. ( I
think then it faded away) no hx of allergy eczema asthma hay fever contact irritant n FHx. No
need to take
further hx
-explain most likely diagnosis to pt, cause of the condition,
further management

107-Secondary Post-partum hemorrhage


You are at your GP clinic, 30 year old Jane presents to you with bleeding per vagina since today
morning. She had a normal vaginal delivery, 10 days back.
TASKS
Relevant history
Get the examination findings from the examiner
Discuss your diagnosis and reason for diagnosis with the patient
Management

Differential diagnosis of secondary post-partum hemorrhage


1-Retained product of conception: placenta or membranes
2-Endometritis
3-Laceration, tears, episiotomy wounds
4-Bleeding disorder
5-blood thinner medications
History
1-Bleeding questions (from here you can rule out retained product of conception)
-Is the patient hemodynamically stable?
-do you feel tired, dizzy or have funny racing of the heart?
-how long have you had bleeding form vagina?
-how many pads have you used so far? Are they fully soaked?
-what is the colour of the blood?
-is it smelly?
-have you noticed any clots or tissues?
-are you aware of your blood group?
2-Endometritis (fever, tummy pain, discharge)
-do you have any fever? (slightly flushed since yesterday)
how high is it? have you had the chance to measure your temperature?
Is it constant or does it come and go?
Any chills, shivers?
Have you taken any medication for your fever?
-any rash?
-do you have any pain or discomfort in your tummy? (yes)
ask severity and give painkillers after allergy
site and radiation
-any abnormal offensive vaginal discharge?
3-pregnancy questions (you can rule out breast problem and laceration/ episiotomy here)
-any condition you had during the pregnancy?
-congratulate her and ask:
how is the baby doing? Are you enjoying your motherhood?
Do you cope well with the baby? Do you have enough support?
Do you breastfeed the baby? Any problems with breastfeeding
-any cuts made at time of delivery? Is it healing well or not? Any tears or lacerations

4-general questions (you can rule out other sources of fever)


-any coughs or cold recently? (respiratory infections)
-how is your waterworks? Bowehabits? (UTI)
-SAD
-any history of bleeding disorder? (bleeding disorder)
-do you take blood thinner medication or any medications? (blood thinner meds)

Physical exam from examiner


1-General appearance:
pallor, dehydration, Oedema, lymph node enlargements, rash, bruises, skin petechia

2-Vital signs: temperature, blood pressure and postural drop

3-quick CVS and respiratory

4-Abdomen:
inspection:
-visible distention,
-visible mass,
palpation:
-palpable mass (yes you can feel a mass)
-can I get below the lower border?
-is it the uterus that I can feel in the abdomen?
what is the size?
Is the uterus tender? (yes)
is it contracted or laxed? (slightly laxed)
Auscultate for bowel sounds

5-Pelvic exam
Inspection:
-bleeding, color of the bleed, tissues or clots in the bleed
-episiotomy wound is well healed or not
-any other lacerations or hematomas,
Speculum:
-is the os open or closed? (slightly open)
-any other offensive discharge that you can see
-Is there bleeding from the cervix?
-any cervical laceration?
-I want to take swab from vagina for MCS
Per vaginal exam:
-CMT
- uterine size, tenderness (tender, enlarged to 14 weeks)
-palpate adnexa for mass, tenderness

6-Office test: BSL, UDT

Explanation
-from history examination you have a condition called secondary post partum hemorrhage which
is heavy bleeding that occur after 24 hours up to 6 weeks after delivery.
-there are several causes:
*1st if you have products of pregnancy retained inside like bits of placenta or membranes but in
this situation you might noticed tissues coming with the blood and will not have fever or tummy
pain unless get infected.
*another could be episiotomy wound that also bleeds but the pain is usually coming down when
the wounds heal completely.
*could be due to laceration or tears but examination shows nothing.
*tears in the birth canal could also be a cause but I am not suspecting this in you.
*bleeding disorder and blood thinner medications are also possibilities but history does not
suggest them.
*what I am suspecting is endometritis: the normal lining of the womb called endometrium and
infection of the endometrium called endometritis. This can lead to heavy bleeding with fever
and tummy pain and when I examined you, I could see the uterus size larger than it should be. It
was tender to touch and the neck of the womb was open.
this is common after delivery because at time of delivery the normal organism inside vagina can
get disturbed and climb higher into the uterus causing infection of the uterus especially at the
site of placental attachment which is a bit raw.

Management
-You need to be immediately referred to the hospital. I will arrange an ambulance for you and I
will liaise with the ED of the hospital and I will make them aware of your condition.
-Meanwhile, I will start you on an IV line, and start you on IV fluids, take blood for all
investigations such as FBE, UEC, ESR/CRP, blood group, cross-matching and hold and Rh typing,
coagulation profile, and blood culture. A urine sample needs to be sent for microscopic culture
and sensitivity.

Once you are in the hospital:


-you will be admitted, seen by the specialist.
-An ultrasound will be done to look for any retained product of conception.
-If there is no retained product of conception, you will be started on IV antibiotics (triple
regimen: co-amoxiclav + gentamicin + metronidazole) and usually responding win 2 days then
will be shifted to oral antibiotics to be continued for 10 days. Panadol for fever.
-You will also be given IV oxytocin to stop bleeding +/- IV or IM ergometrine (if she is not
responding to oxytocin, give ergometrine).
-ask if there is somebody with her to look after the baby.
-Review once out of the hospital

Just Note/ If there is a retained product of conception,


-an exploration under anesthesia will be done
-gentle blunt curettage to remove the products of conception under antibiotic cover.
-after cervix dilated give IV oxytocin

Feedback 20-2-2018
GP, young woman 10 days puerperium came because of bleeding from down below.
Tasks:

- Take history
- Physical examination from the examiner
- Explain the probable diagnosis and differential diagnosis to the patient

2 min thinking: assess patient haemodynamic stability, ask mode of delivery in history, and ask
temperature in examination findings

History:
Introduce myself, ask the patient how does she feel? How long has she been bleeding and how
many pads? Then explain to her I need to make sure she is stable by measuring her blood
pressure and other vital signs and turn to the examiner. (Examiner said she is haemodynamically
stable).
More questions about the bleeding – colour, any clot, foul smelling, any pain at lower part of
belly, fever (she felt flushed), any problem with BO/PU (no pain/burning sensation).
Ask mode of delivery (normal vaginal delivery), no instrumental, there was episiotomy done, is
she aware of the placenta delivery (the midwife said it is complete), duration of hospital stay (2
days). Pregnancy course (unremarkable), any previous pregnancy (1 st one?), bleeding problem in
the past (no)
Quick question about support for her and baby (yes), any concerns about baby?(no)

Physical examinations:
Temperature around 38/39, other vital signs within normal range,
P/A: soft, non-distended, uterus? 10cm, forgot whether there is any tenderness
Pelvic examination: inspection unremarkable, episiotomy wound well healed
Speculum examination: vaginal wall normal, cervical os closed
Bimanual examination: uterus? 10cm, no adnexal tenderness, positive cervical motion
tenderness
Urine dipstick: blood stained (contaminated), the rest (nitrate, leucocyte) negative

Explanation:
Most likely you are having some infection in the womb due to retaining of fraction of
placenta/after birth. Other possibility would be bleeding from the episiotomy wound but the
wound is well healed on examination, urinary tract infection but it is less likely. Don’t worry too
much it is very good that you are here, I will arrange you to go to the emergency department
because you need further investigation and treatment. (Did not elaborate further as it is not my
task). Assess understanding, ?reading material (I forgot whether I said so)

AMC Feedback – Postpartum bleeding: PASS

Feedback 31-5-2018
secondary PPH,GP ,a lady who delivered the baby 10 days ago ,now presenting with vaginal
bleeding.placenta was complete.baby is well and takes breastfeeding well.
Tasks -H/O,PEFE,Dx and Ddx

when I went into the room a lady was lying on the bed.i asked vitals BP 110/70 P around 100 T
38 or 39.bleeding 2-3 days after delivery,no clots ,no tissues,2-3 pads per day, heavy, tired ,no
dizziness no SOB.Feeling hot.
No problem at the time of delivery,no prolong assisted,no episiotomy. Pain at the lower
abdomen the same day with bleeding ,(ask pain scale and offer pain killer, also ask a few
questions about pain).5Ps normal with good support,blood group O+,no family history of
bleeding.no trauma.

PEFE-other systems normal.abdomen tenderness at SPA, uterus still enlarge . Mid way between
umbilicus and symphasis pubis.Pelvis examination Bleeding present,no clots ,no tissue.no cut or
tear. Sterile speculum no vaginal and cervical tear.
Bimanual uterine tenderness present no cervical and ednexa tenderness

Explained- endometritis by drawing,don’t worry treatable with antibiotics,other possible causes


RPOC ,even though placenta was complete we still need to do USG to make
sure.trauma,tear,bleeding disorder all unlikely.arrange ambulance and refer to hospital do some
blood test and take swab for culture.

Key 5/5 approach 4,history 4,choice and technique of examination,organisation and sequence
2,accuracy of examination 4,dx and ddx 5 .global 4

Feedback 27-10-2018
30+ year old patient came to your GP with bleeding per vagina. She has given birth to her baby
10 days ago. She had a normal delivery and placenta was complete. Task

-take relevant history

-perform physical examination on the patient

-explain the Dx and DDx to the patient History

a young lady is sitting in the chair looking anxious.

introduced.

checked the vitals with examiner (examiner told me to stick to the task)

asked the role player

are you feeling dizzy?

details about bleeding?

- Duration (does not remember)

- color (bright red)

- amount (soaked 2-3 pads)

- smelly (no)

associating symptoms

- pain in the lower tummy (present)

- fever (present)
- asked her blood group.

- ruled out other causes of bleeding

- blood thinning medications? (-)

- bleeding disorders? (bleeding elsewhere in the body) (-)

- cut made down below (episiotomy) (present, but pain becoming less and less)

- ruled out other causes of fever

- breast pain? (no)

- waterworks and bowel habits? (no)

- calf pain? (no)

- cough? (no)

- asked her about the baby whether baby is doing fine? (normal)

Examination

- asked the patient to lie on the couch and started the examination Abdomen

Inspection - normal

Palpation - tenderness in the SPA

Percussion - normal

Auscultation - normal Pelvic

inspection - bleeding (+), smelly (-), episiotomy wound healing well

sterile speculum - os (open), no tear on the cervix bimanual

- I told the examiner that i will skip the bimanual examination ( hence only 3 in accuracy
of examination )

- but examiner gave me the findings

- CMT (+)

- size of uterus is 14 weeks

- tenderness over the uterus

- adnexa normal office test

urine dipstick - RBC (4+++)

Dx/DDx

- most likely due to endometritis (drew and explained) other possibilities


- tear in the birth canal and cervix

- bleeding disorder

- placenta retained in the womb, but these are less likely. Grade - pass

Global score - 4

Key steps

1,2,3,4,5 - Yes, No, Yes, Yes, Yes

Approach to patient/relative - 4

History - 4

Choice and technique of examination, - 4

organization and sequence

Accuracy of examination - 3

Dx/DDx

Feedback 13-12-2018
Postpartum Bleeding: Pass
2 mins outside. I don’t remember the stem but it was the case of postpartum bleeding. I m not sure fever
was mentioned outside or not.

Tasks: history, Dx and DDs to patient

Thinking outside: in History, need to ask about presenting complaints, associated symptoms. Then full last
preg with full delivery history. Then past pregnancy. Then other present, past and f/h.

Inside: asked about bleeding, such as since when, how much, colour (bright red), any blood clot (for DIC),
any tissue (for retained placenta).

Any bruising anywhere (for DIC, any blood disease). Then asked for associated symptoms specially fever
(present, this gives our diagnosis). Then I asked about abdominal pain and all que to rule out dds of
bleeding and postpartum fever. Then I asked about full history as I mentioned above including blood
thinners but did not ask much about irrelevant past and present history. Please read history from Karen’s
as it is good to know all questions but need to know when to apply relevant questions.
I think it is good idea to ask many questions in history but because of time problem I kept my history to
relevant questions.

PEFE: same as karen. Positive findings were temp 37.8, lower abdominal tenderness, mild bleeding seen
on speculum, no clot or tissue. Episiotomy scar well healed.

Dx and DDs: from the history u have provided to me, I think you have a condition called endometritis
because of fever and bleeding plus pain in lower abdomen. There are many conditions that can be
possible such as retained product of placenta but less likely as it is generally does not cause fever unless
infected. Could be medicine but u r not on any blood thinners, could be DIC where person have bleeding
at the same time blood get clotted, this is less likely as u don’t have any symptoms of it. I think she had
episiotomy but it was healing okay. I think I have mentioned 4-5 DDs. When it comes to explain to pt, it
takes too much time so please hurry up.

Key steps: all yes. Global score: 5, History, Choice& technique of examination, organisation and sequence
(which was not in task I think) and dx/dds: all 5.
108-Post CS fever
You are an HMO in the maternity ward. You are asked to see Mary 32 years old Jane who had
underwent an elective lower segment C-section 3 days ago for breech at 38 weeks. This is her
first baby. The birthweight of the baby is 3.5kg and the baby is doing well.
She wants to get discharged today. She's taking paracetamol for pain. She was given 1 dose of
ceftriaxone intraoperatively and enoxaparin 2 doses given post-surgery. The patient's vitals chart
is given outside the door:
BP range 120/70 - 130/80
PR normal
RR normal
O2 sat normal,
temperature on day 1 is normal, but on day 2 it is 38.0C and day 3 it is 37.7C.

TASKS
Take a further history
PE from examiner
Talk to patient about her findings
If you think she can be discharged discharge her otherwise give reasons

Causes: (genitourinary, chest and breast, limbs)


1-wound infection
2-Endometritis
3-UTI
4-mstitis/ breast abscess
5-URTI
6-DVT/thrombophlebitis

History
1 Approaching
-Congratulate her on her delivery.
-are you coping with your baby well?
-are you enjoying the motherhood?
-do you have good support?
-Look Jane, I can understand that you keen to be discharged today. Before that can I just ask a
few questions?

2-chief complaint and Differential diagnosis questions


fever
-I have seen in the chart that your temperature has been high for the past few days. Are you
feeling feverish? (slightly flushed at times)
-Any rash (no)
wound infection
- have you noticed any discharge form the wound?
-is your wound’s pain coming down?
Endometritis
-do you have tummy pain in the upper part away from the site of the wound? (Generalised pain)
-How about your bleeding? is it coming down or becomes heavy? How many pads have you
used? Are they the same number you were using? (Minimal bleeding)
-any smelly bleeding?
-Any other offensive vaginal discharge? (no)
Urinary tract infection and bowels
-how is your waterworks?
-any burning or stinging on passing urine>
-is it smelly?
-any change in the colour of the urine?
-have you opened your bowels after the surgery?
Mastitis/breast abscess
-do you breastfeed your baby? Any problems with breastfeeding? (day 1 she cannot breastfeed
properly but day 2 latching very well)
-any sores or cracks in the nipples?
-any painful lumps in the breasts?
Upper respiratory tract infections/ atelectasis
-any coughs or cold that you having? Ant sore throat?
-are you short of breath? Any chest pain?
DVT/ thrombophlebitis
-any calf welling or pain?
-any pain or redness at the site of Cannulae?
-any excessive pain at the site of anaesthesis?

3-general questions
-Any other medical or surgical illness?

Physical Exam from examiner


1-General appearance: PODL
pallor/ oedema/ dehydration/ LAP

2-Vital signs (as you seen in the chart)

3-ENT (rule out URTI)

4-CVS and respiratory


-air entry/ added sounds
-S1, S2 and murmurs

5-Calf:
tenderness, swelling.
6-Abdomen:
Wound:
-wound covered with bandage that is minimally soaked with blood.
-I would like to remove the pad with the consent of the patient and have a look at the wound,
looking for any erythema (+ve), discharge (+ve) , wound dehiscence
Uterus:
-what is the size of the uterus? (Enlarged)
-is the uterus contracted or lax? (Contracted)
-any uterine tenderness towards the upper pole? (Tender)
-bowel sounds

7-Pelvic exam:
Inspection:
-bleed, abnormal vaginal discharge.
Speculum:
cervical os open or closed, bleeding from the os? (Cervix still open)
bimanual examination:
size and tenderness of uterus
adnexal mass or tenderness

8-breasts
mastitis/ abscess

9-Office test:
Urine dipstick, BSL

Explanation
-I am a bit concerned as you have a rise in temperature yesterday and also today.
there could be several causes of having fever after CS:
*could be due to wound infection but the wound site is healing well and the pain is coming less.
*could be UTI but you have no burning or stinging on passing urine and the urine dipstick come
out normal.
*another cause is URTI but you do not have coughs or cold
*another is atelectasis where your lung expansion becomes affected after surgery but you do not
have chest pain or SOB and your lungs are clear on examination.
*other possibility is DVT or clots in your legs vessels that lead to blockage of the blood supply
but this usually cause pain and swelling in the calf muscles that you are not having. Also you
were given 2 doses of blood thinner at time of delivery so it is unlikely.
*could be infection at the site of the cannulae but no evidence of this.
*could be mastitis or abscess…
*the most likely one is a condition we call endometritis where the inner lining of the womb
becomes infected. It is common in post CS patients. That is why you have fever, uterine
tenderness or pain in your tummy and also discharge later.
Management
-As I could not confirm the cause I would still like to keep you in hospital (critical error) for
further monitoring, assessment and management. further investigations need to be done such
as FBE, blood culture, urine microscopy culture and sensitivity, ultrasound to see if there is
anything happening in the uterus or not.

-I need to have a talk with the specialist and if it is Endometritis, it can be managed by giving IV
antibiotics. Once you start responding to IV antibiotics, you will be shifted to oral antibiotics.

-So it would be better for you to stay in the hospital till at least 24 hours when you have no fever.

Feedback 4-7-2018
post partum D3-has fever-planning to go home

Observation chart given-T-mild fever/ I guess PR/RR

Task hx, PEFE, Mx

Hx-all infective foci-mastitis/endometritis/wound infection/UTI/DVT/thrombophelebitis

Asked about coping with baby/family a support/feeding/ bld gp/bleeding

Antenatal complication

PEFE- asked what not given in the chart-can’t remember exactly

But apart from mild fever-nothing very significant

Mx-explained as having mild fever need to do some Ix –blood/urine to exclude infection

Fluid/PCM

Will r/v with obs reg


109-Endometritis Fever
110-Post-partum mastitis
Material Case/
28-year-old Emily, mom of a 5-week-old baby, comes to your GP with complaints of tiredness and fever
since the past 2 days.
TASKS
1. Focused history
2. PE from examiner
3. Diagnosis and Management

Differential Diagnosis:
 Breast: Mastitis/Breast abscess
 Birth canal: endometritis, episiotomy wounds, laceration that has become infected
 Bladder: UTI
 URTI
 DVT

History
Congratulations on the pregnancy. How is your baby doing?
1-fever questions
-Since how long are you having the fever?
-Is it a continuous fever or an on and off fever?
-Have you recorded the temperature?
-Any rash? Any chills or rigor?

2-DDX questions
URTI
-Any runny nose, cough or colds?
Mastitis/ abscess
-Are you breastfeeding your baby? Any problems with breastfeeding?
-Any lumps that you have in your breast? How long have you been feeling the lump?
-Is the lump increasing in size?
-Is it warm and painful to touch?
-Any other lumps that you can feel in the same breast or in the opposite breast?
-Do you have a sore or cracked nipple on that side?
-Any blood-stained or purulent discharge from the nipple?
-Is the baby being positioned to the breast correctly? Has somebody taught you the correct positioning of
the baby during breastfeeding? (key issue)
Endometritis/ lacerations
-Did you have any conditions during your pregnancy or was your pregnancy uneventful?
-What type of pregnancy did you have?
-Any cuts made down below? Any tears that you had at the time of delivery?
-Any abnormal foul smelling discharge from down below? Have you stopped bleeding? Are you having any
tummy pain?
UTI
-Any burning or stinging while passing urine?
-Any constipation that you are having? Do you open your bowels regularly?
DVT
-Any calf pain or swelling?
3-Lifestyle and support
-Rule out depression: Do you have a good support from your partner to look after the baby? Are you
enjoying your motherhood?
-Do you eat a healthy diet?
-Do you smoke, drink alcohol or take recreational drugs?
-Have you done your pap smear? What was the result?
-Any medical or surgical conditions in the past?

Physical Exam
1-General appearance: pallor, edema, lymphadenopathy, is it tender? dehydration
2-Vital signs: temperature
3-CVS, Respi, CNS
4-Breast: key issue
-compare right with the left breast.
-look for any visible lumps in the right breast, size, shape of the lump, color of the skin over the lump.
-inspect the nipples for any cracked nipple.
-palpate for local rise in temperature, confirm the site, palpate the borders if well-defined or not, palpate
the consistency, mobility of the lump, any fluctuations, examine the same breast for any other lumps, and
the opposite breast as well
5-Abdomen:
any abnormal distention, any mass, on palpation, do you still feel the uterus in the tummy or has it
involuted already? Any other mass or tenderness in the tummy? Auscultate for the bowel sounds
6-Pelvic exam
-Inspection of the vulva and vagina: abnormal discharge, bleed, episiotomy wounds or lacerations
-Speculum: cervix healthy or not, OS open or closed, discharge or bleed from the cervix
-Per vaginal exam: uterine size, CMT, tenderness, adnexal mass and tenderness
7-Office test: UDT, BSL

Diagnosis and Management


-Most likely you've got a condition called Mastitis of the right breast. This is the cause of your fever and
tiredness. Mastitis is an infection of the connective tissue of the breast caused by bacteria, and the
bacteria that usually causes this is staph and e. coli.
Where did these bugs come from? It usually enters into the breast if there are any cracks in the nipple,
from the baby's mouth or from the surrounding area of your skin and this condition will worsen if there is
poor milk drainage from the affected breast. The usual cause of the cracked nipple is due to poor
positioning of the baby to the breast.

-You need to continue breastfeeding from the affected side, the milk is not affected by the bugs. Put the
baby on the affected side first so the breast will be drained completely. Before breastfeeding, you can put
some hot washers on the breast, so that the milk ducts will dilate, and during breastfeeding, you can
massage the lumps towards the nipple. And after breastfeeding, you can put some cold washers on the
breast.
Take plenty of fluids, and take adequate rest.

-I need to put you on antibiotics such as cephalexin, or flucloxacillin 500mg QID, for 7-10 days, and
analgesics such as Panadol. I will give you reading materials regarding proper breastfeeding techniques.

-Just in case that there is no improvement in your symptoms, the lump is increasing in size, becomes more
painful, you have high spikes of temperature, report to ED immediately.

-I will arrange review with you in 1 weeks time.


**Breast abscess management:
Do an ultrasound if you have doubts whether it is mastitis or breast abscess. If it is breast abscess, she
required referral to the hospital.
If it is a small breast abscess: do an aspiration of the abscess
If it is reasonably big: do surgical drainage under anesthesia
Curve-like incision over the breast to drain the abscess. Discharge will be sent for microscopic culture and
sensitivity. Put in a drain for 2 days.
Antibiotics and painkillers. Go in for a temporary weaning from breastfeeding. Make sure the breast is
empty by using a breast pump.

Note/ in mastitis case, I am not sure If you need to add ultrasound as Ix or not.

Recall of 11-4-2018
111-Breast cancer breaking bad news
You are at your GP, and is about to see 58 year old Susan who had a lump in her left breast. She
was seen by another GP who had ordered a core biopsy and the biopsy result has shown invasive
ductal cancer with progestogen and estrogen receptor positive. The lump was 1cm in size and
she has come to collect the result of the cone biopsy.
TASKS
-Take further history
-Discuss the results with her
-Discuss your further management with the patient

History (not a task in the exam)


Lump questions (Lump, nipples, LN, secondary sites)
-How long has the lump been there?
-Is it a single or multiple lump?
-Where is the site of the lump? Is it towards the inner or the outer side?
-Is it increasing in size?
-Any pain in the site of the lump?
-Any skin changes that you have noticed over the lump like dimpling or puckering?
-Any ulcers or any discharge?
-Is the lump fixed or is it moving freely?
-Is it soft lump or a hard lump?
- Any ulceration or distortion of the nipples? Any blood stained discharge?
- Any lumps or bumps in your armpits or the neck?
-Have you noticed any weight loss?
Any back pain? (spinal metastasis)
Any shortness of breath? (lung metastasis)
Any yellowing of the skin? (liver metastasis)
Any headaches? (brain metastasis)

Risk factors questions


-period: when was your first and last period (early menarche and late menopause)
-pill: have you used HRT and for how long? (HRT> 5 years)
-sexual history: are you sexually active? Have you been sexually active before? (nuliparity)
-SAD (alcohol)
-personal history of any cancers
-PMH/ PSH/ medications and allergies
-Family history of breast or ovarian cancers
FIRST- Breaking bad news:

1-Knoweldege about the test


-I have got the results of biopsy with me. Do you know why the biopsy has been done for you?

2-knowledge about the disease


-have you been told about your medical situation so far?

3-How much information they want


-How much do you want to know about the results?

4-breaking bad news


-the news that I’ve got for you is a little bit concerning. Unfortunately, the sample of tissue that
we have taken from the lump in your breast has shown that it is a nasty growth or cancer.

5-silence, tissue and water


-the patient will start crying so offer box of tissues and glass of water if available, wait for her to
settle down.

6-after few seconds break your silence


-would you like me to continue the consultation today?

7-offer a call for someone


-would you like me to call anyone for you?

8-showing empathy
-I know that this is not what you wanted to hear, and I also wish the news were better. I can see
that this is an upsetting news for you and also very difficult situation to face.

9- showing support to reassure she is not alone


-but we will be working as a team with you, supporting and monitoring you along with your
family to take you through this difficult situation and the survival rate has significantly increased
overtime due to the better diagnostic tests and scans and improvement in the treatment
methods.

10-ask if she is ready


-Are you ready to discuss a treatment plan now, or should I arrange another appointment with
you?
SECOND- Talk about the disease if needed
-in this case mostly no need.

Third- Management
1-MDT and Refer to the specialist for Ix to check for spread
-from now on the treatment is by a multidisciplinary team.
-I will make an immediate referral to the cancer specialist. Further investigations needed to be
done to check if it has spread to other parts of your body, which we are not suspecting at the
moment but need to look for.
-They might need to do:
FBC and blood group,
UEC
LFTs
Coagulation profile
Bone scan
If necessary CT head

2-Treatment according to the disease (usually talking about surgery, radio and chemo and LN)
-Treatment will be decided by the specialist
-if it is stage 1 usually it is breast conserving surgery. The surgery will either be: a lumpectomy
where the cancerous lump is removed with a border of normal breast tissue, or partial removing
of the breast.
-the surgery is usually followed by radiotherapy.
-Meanwhile the LN which are adjacent to the breast also will be checked to see if there is any
cancer and would be removed.
-other treatment will depend on receptor positivity, in you it showed estrogen receptor positive
so it indicates good prognosis so you will most likely put on adjacent hormonal therapy with
tamoxifen

3-Follow up (usually blood tests and imaging)


-yearly mammogram is important (till 70 years old)
-1st degree relative need to be screened 2 yearly after 50 years with mammogram
-can talk about plastic surgeon and reconstructive surgery

Note/ if it is diagnosed less than 50 years old so screen 1 st degree relatives from 35 years old
every one year till 49 years then 2 yearly till 70 years
Case (7/3/2017)
GP, 55 yrs old lady comes for ultrasound and FNAC biopsy result. She noticed a lump on her left
breast and you did investigation. USG shows suspicious of malignancy.

Core biopsy shows Invasive ductal carcinoma stage1. Estrogen receptor positive.

CTchest–no metastasis (not lymphatic not blood invasion) .She is mother of two children and had
hysterectomy 10yrs back.

Task-explain about the result, further management including implication of her condition.

I approached with breaking bad news, give her time, she looked sad but not cried, Tissue box
and water present but I didn’t offer, I explained about MDT and

Encourage her as it is early stage and no spread, we can expect complete cure.

She looks better. Then, I told treatment options including surgery +reconstructive surgery
followed by RT, chemotherapy, hormonal treatment, Pain clinic and regular follow up. I forgot to
tell her about self-breast examination. When I asked

About if she has children or not, examiner said you are not allowed to ask history.

Then, I told her about genetic counselling. Reassure her again and arrange family Meeting for
you. She said fine. Implication-ulcer, nipple discharge, spread to Other organs.

23-6-2018 FAIL
Feedback: Breast Lump: Deliver the bad news with biopsy result: Invasive ductal carcinoma., estrogen +
Approach: when I walked into the room, a grumpy lady stay there.
How are you doing?
I am aware that you got a lump on your L breast and the surgeon did biopsy on that lump and you came
here today for the biopsy result. Before I deliver the result, do you need anyone to be with you or you
are ok being alone? Do you drive today or anybody drive you here today?

Yep, the result came back, I revealed that you got an invasive ductal carcinoma which is a breast cancer.
(Stop for a little while waitng for pt’ reaction).
The pt was not sad but look very grumpy, she did not cry
so I was not supposed to give her tissue.
Do you need a cup of water? No
I said are you alright to continue or you want to stop here? Continue.
Do you want nay family member to present with you now? No.

I started explaing the result: this cancer is a common type of the cells in the milk duct which produce
milk. Now It has been growing uncontrolled. And the biopsy said: Invasive  it means the cancer
already spread around areas nearby or further, but we are yet to know how far I has gone.
I need to check your Lympho nodes and your lung, abdomen as well.

Recently do have any headache? SOB, chest pain? Tummy pain? Any lumps and bumps in the other
breast? LOW? LOA? LOS? Any body in your family has cancer?
Now I would like to:
- Urgently order some investigations to stage the tumor: Head, chest, abdo/pel CT scan; bone scan.
- Refer you to the breast surgeon asap right after the results come back (1 day).

Treatment:
- So far, main therapy is total mastectomy, Lns + additional radiotherapy +/- chemotherapy if needed.
- The surgery has some complications, such as bleeding, infection, anesthetic effects, cosmetic
appearance. The plastic surgeon may recontruct your breast by using muscles nearby.
Radiotherapy will be carried out to kill off the remaining hidden cancerous tissues.
Do you have any questions? No, what am I supposed to know more.
I would like to reassure you that we work in MDT, we will take a good care of you. If you have any concern
just let me know. Do you have any financial problems? Family problems? (no)
Now is there drive you home? Where do you go now? Do you need me to call your partner of friend to
pick you up? I also can arrange a taxi for you. (no)
Here is the materials, you can read at home. And here the referal for the scans, you come back to see me
after the scan done and I refer you to the specialist.

Grade: FAIL, GS: 3 WHY????


Key steps 1,2,3,4: Y (all covered)
Approach: 3, management: 4, counselling: 3
I don’t know why AMC failed me this case. Maybe I can not handle the pt well, she was not happy, and
insensitive throughout the talk, despite all key steps were covered.
112-Hyperkalemia
Middle aged woman has come to ED due to pre-dialysis assessment. Complaining of palpitations.
Nurse did ECG which shown in the stem. Bp: 130/80, Pulse 60 and irregular, temp: NL, RR: NL.
Tasks:
-Take relevant hx,
-Explain the ECG to the examiner,
-Ask from relevant investigation from examiner,
-Tell the diagnosis to the patient with the reasons.

History
1-Approach
-Hi I can understand that you are here for pre dialysis assessment.
-do you have any concerns?

2-Current conditions
-Palpitations? (Yes)
Duration? Sudden or gradual? Tap it? Regular or irregular? 1 st time?
-confusion, dizziness?
-chest pain, SOB, cough, ankle swelling?
-nausea and vomiting? (feel nauseated)
-urine output?
-diarrhea? (has diarrhea)
-headache, LOC, BOV?
-weakness, tingling or nymbness?

3-Chronic renal failure


-since when
-what medications do you take? Do you take it regularly? Any SE? Any other medications?
(perindopril, ca bicarbonate, calcitriol, erythropoietin)
-have you ever increased the dose? (yes recently)

4-General
-past medical history (DM, lipid, HPT, renal stone, heart disease)
-smoking and alcohol

Explain ECG to examiner


this is 12 lead ECG and I can say the rate is 60 and sinus as there is p wave before QRS complexes
and it is narrow.
Axis is NL and I can appreciate that T wave is tented and there is a strain of ST interval.
The most likely condition regarding this ECG is Hyperkalamia
Ask Investigations
1-Full blood count
2-Kidney function urea and creatinine (KFT impaired)
3-Electrolyte Na, K, Ca, Mg (Na =145, K= 6)

Explain condition
-from history , ECG findings and investigation it seems that you have a condition called
hyperkalemia which is increase level of potassium in the blood which is one of the mineral body
salts. Ix showed that your kidney function is also impaired.
-this can be due to several possibilities:
*could be due to effect of medication perindopril that can inhibit an enzyme called ACE which in
turn suppress the secretion of aldosterone hormone which is responsible for excretion of
potassium out of the body. This causing retain of potassium in blood.
*could be due to diarrhea causing more fluid loss leading to dehydration.
*other is acute kidney failure as your kidney function is impaired.

Case (13/10/2017)
Middle aged woman has come to ED due to pre-dialysis assessment. And ECG has been taken
shown in the stem. Bp: 130/80, Palse 60 and irregular, temp: NL, RR: NL.

Tasks:
Take relevant hx,
Explain the ECG to the examiner,
Ask from relevant investigation from examiner,
Tell the diagnosis to the patient with the reasons.

Task1
I entered and introduced myself. There was a nice RP.
No Vomiting but the patient was nauseated.
NO cp or sob or cough
no problem in waterworks but the patient has diarrhea.
No AB no other PMH and FH.
Her medication: Prindopril, Ca bicarbonate , Calcitriol.

Task 2
I told the pt if you are OK I want to talk with my colleague regarding your problem.
So I said dear examiner, This is 12 lead ECG and I can say the rate is 60 and sinus as there is p
wave before QRS complexes and it is narrow.
Axis is NL and I can appreciate that T wave is tented and there is a strain of ST interval.
The most likely condition regarding this ECG is Hyperkalamia( examiner said OK)
Task 3

Then I asked for investigations. Examiner said what you are looking for? I said: Na, K, Mg, Ca
level and RFT. Na was 145 and K was 6 and RFT was impaired( I don't remember the exact
numbers). Thanks examiner.

Task 4
I turned to the patient. I said your ecg has some problem which is due to high amount of
potassium in your blood which are the electrolyte and chemicals in your body. Also Na is
decreased and your kidney function is impaired. This is due to many causes.
- One of them due to kidney failure that you are having( I did not mention ARF on CRF!!:(..)
- the other condition is due to your medication which is perindopril and
- the other one is due to diarrhea that u were having causing your symptoms of nausea and
heart arrhythmia. Bell rang.

Feedback: Occasional palpitation, PASS(G.S:4)


Key steps:1 Yes2 No ,3 Yes 4 Yes
Approach to patient:5
Hx:5
Interpretation:4
Dx/DDx:4

Case (29/4/2017)
HMO at renal unit to see CRF patient came for dialysis complaining of palpitation. She is taking
perindopril, calcipotirol and erythropoietin when asked about medication to her. ECG and blood
test were done.
Tasks
- Explain ECG to examiner
- take history
- explain condition and most likely diagnosis to patient with reasons
patient has history of diarrhea and increased perindopril dose recently ECG showed tall T, and U
and C raised, potassium 7.1

Case (26/7/2017)
50 + yr old on regular dialysis for Renault failure. Feeling unwell since few days ago. Today having
palpitations.
ECG showed tall tented T waves, I did not get the history of change in medication!.
Task Examine ECG to examiner, explain diagnosis to patient with reasons can't remember if the
was management

Recall 7-9-2018
man with renal failure come for dialysis he was complaining of palpitation and feeling unwell
have diarrhea nurse did ECG provided outside hyperkalaemia explain ecg to examiner, ask hx pt
will give you his medication only if u ask there was increase the dose of antihypertensive, ask ix
from examiner increase k, decrease NA, urea cr increase, explain the condition to pt
113-Spiral fracture
You are a GP, mother brings her 2- 3 year old boy due to pain in leg. The child is refusing to walk.
Tasks
-Take relevant history
-ask physical examination finding from the examiner. You will only get the answer you ask for.
-Further mx

History
1-pain questions
-Pain severity and painkiller
-Can you tell me more?
-Pain Qs
-Is he able to walk or bear weight?

2-HOST-M
-Tummy pain, rash
-Fever, recent infection, joint swelling, redness
-Trauma (+) fall on lego in daddy's home
When? How? Any Bruise?
-LOW, LOA, lumps, bumps

3-General questions
-BINDS: Eating, immunisation, G&D
-Pmh? Psh? Medication
-Family Qs ( social hx)

4-Imp question
-Confidentiality( to r/o child abuse )
-Who is looking for the child most of the time?
-Any stress at home (happy family )
-Father? Siblings (divorce 6 m ago )
-Child care? ( mum takes care of baby)
-Relationship with father & mum? (spoke with dad, didn’t mention any problem with child)

Differential diagnosis
1-Trauma( 1st )
2-OM/Septic arthritis
3-Bleed in joint
4-Transient synovitis
Pefe (like transient synovitis) Here Normal examination
-G/A DRPJL
-V/S fever (-)
-Growth chart
-ENT
-Focus limb ex
( pain, swelling, bruise, skin redness )
( point tenderness )
( movement )

-X-ray (AP, lateral, oblique views) given showing spiral fracture of tibia

Explanation
-Pain can be due to a lot of reasons:
Infection of joint, bone or bleeding in the joint, but no fever, joint swelling.
The most likely cause is fracture. As you can see the Xray. This is the leg bone called tibia.
Showing thin, spiral fracture called Toddler fracture.
- Cause: It is very common in this age. It can be resulted from a rotational force (twisting)
- C/F: presenting with pain, swelling with or without deformity. The child usually refuse to bear
weight on the injured leg.
- Outcome: Most fractures will heal well without complications in 8-12 week

Management
-Supportive
-Apply cast for 4 wks
-Review in 24 hr

Red flag (increase pain, swelling, tingling, color change)


Review in 2 week.
Reading materials
Feedback 29-11-2017- (acute limp)
You are a GP, mother brings her 2 – 3 year old boy due to pain in leg. The child is refusing to
walk. Take relevant history and ask physical examination finding from the examiner. You will only
get the answer you ask for. Further mx
Greeting
Hx- limp history- since last night, went to meet his dad, came back home then started crying,
doesn’t wanna walk, 1st time, in one leg, tripped over a lego at dads place
No fever, rash
BINDS normal
Feeding, pee, poo good
Family- mom divorced 6 months back, no problem coping, child has good relation with dad and
mom. She spoke with the dad last night and he didn’t mention any prob with child, no money
problem
Doesn’t go to childcare, mom takes care of baby
No FH of bleeding disorder
PE- vitals normal sp temperature, makes eye contact
Growth chart normal
No signs of injury anywhere, no rash
Limb exam doesn’t have any bruise, swelling, no restriction of joint movement, joint was normal
No office test available
X ray was given _ showed spiral fracture of tibia
Mx – pain can be due to a lot of reasons infection of joint, bone, bleeding in joint, most
commonly fracture. See the xray. This is the leg bone. Thin fracture. Mentioned toddler fx, very
common at this age, we call them terrible twos. Don’t worry. Will put a cast. May need 3 – 4 wks
to heal. No followup xray needed. Didn’t mention anything abt child abuse or CPA.
Followup in 24 hours.
Red flag- pain in limb, tingling, numbness, blue color limb
Feedback 8-11-2018
4 your , intern ,2 yrs old boy brought by mother complaining of
114-back pain
A young male comes to your GP due to severe back pain after lifting an object 2 hours ago.
Task:
-take Hx for 4 minutes
-PEFE.
-explain ddx and most likely dx

History
1-Back pain questions
-I understand that you are complaining of back pain so how severe is it from 1-10 1 is the least
10 is the most. I would like to give you painkillers so do you have any allergy to any medication?
-I understand that the pain started 2 hours ago. Can you tell me more about it?
Has it started suddenly or gradually?
What were you doing before the pain started? (He lifted a rock without squatting instead he was
bending forward.)
Is it constant or does it come and go?
Is it getting worse
-could you show me exactly where the pain is? (Buttock, thigh, leg)
does it go anywhere else?
-can you describe it for me?
-does anything make it worse? (Coughing, sneezing, leaning forwards)
-is it getting better with rest?
-has this happened before? (2 years ago)

2-associated symptoms
-is it associated with stiffness? Is the pain and stiffness worse in the morning?
-have you had any tingling, numbness, or weakness in your legs?
-any fever, LOW, LOA?
-any trauma?

3-Cauda equine symptoms


-have you lost control of your urine? Or felt inability to empty your bladder?
-have you lost control of your bowels motion?
-any numbness in the lower back or private area?
-have you had difficulty obtaining or maintain an erection?

5-General
-do you take any medications? Steroid use?
-PMH (joint problems)? PSH? FHx?
-have you travelled recently?
-what do you do for living?
-how does the pain affect your life or job?
Differential diagnosis
1-mechanical back pain (gradual onset, no neuro signs or symptoms, no recent minor injury)
2-sciatica (pain radiates down leg beyond knee)
3-cauda equine (severe pain, urine retention or incontinence, faecal incontinence, saddle
paraesthesia, leg weakness).
4-fracture of vertebral body (sudden onset, trauma, steroid use, known osteoporosis, tenderness
over vertebral body)
5-spinal canal stenosis (pain on walking, improves by bending)
6-ankylosing spondylitis (pain at rest)
7-abscess (immunosuppression, fever, worse at rest)
8-malignancy( LOW, LOA, worse at rest, sleep disturbed, persistent > 4 weeks)

Physical examination findings from examiner


1-GA, VS
2-Gait
3-look (SSSDW).
4-feel (tenderness).
5-move (ROM)
6-neuro (tone, power, reflex, sensation)
7-SLR.
8-DRE (anal tone)

Explanation
-from history and examination, you most likely have a condition called sciatica, have you heard
about it?
-draw diagram; these are your back bones and between each bone there is a cushion we call
disc, in your case the disc slipped out of its position causing compression on a nerve passing here
that what causes your back pain.
-This usually happens due to stress on your lower back after heavy lifting or sudden
inappropriate movement.
-let me assure that this is a common, not serious condition and its usually self limiting and goes
away in 2 weeks.
-there is no need to do any further Ix or refer to a specialist.

Management
-I will prescribe you painkillers to help with your pain, refer to specialist for MRI.
-what you need to do is some bed rest just for a short period not more than 2 days, just to
relieve the pain because prolonged bed rest actually prolongs the pain.
-the most important thing is to stay gently active and this will help your back to recover, stop
your back becoming stiff and your muscles from weakening and allow you to return you to
normal activity ASAP.
-refer you to physio for short course of stretching and strengthening muscles exercise. Once the
pain is ceased.
-Lifestyle modification;
-reading materials, review in 2 days.
-work: advise about proper bending and lifting heavy objects.
Case (13/10/2017)
A young male comes to your GP due to severe back pain after lifting an object 2 hours ago.
Task:
take Hx for 4 minutes!
PEFE,
explain ddx and most likely dx.
Having entered I introduced myself and asked whether the pt needs painkiller? He said no I am
fine now.
He lifted a rock without squatting instead he was bending forward.
No trauma.
No PMH of joint disease.
No medication.
No Fhx of joint diseases.
He has tingling in his left leg. No trauma to leg.
No stiffness in morning.
He had the similar attack 2 years ago.
He is worker in mine… and it is affecting his job as he can’t work now.
No involuntary BM and urination.
PEFE:
Gait ( heal walking toe walking squat NL)
Inspection: no stepped deformity, no muscle wasting, no scoliosis, no erythema, lore doses is
normal.
In palpation; tenderness on lower part of lumbar spine, no inc temp.
Slump test and Schober test is not available.
ROM is decreased in all movements.
I want to do neurological exam of lower limb.
In inspection, No muscle wasting no bruises no fasciculation.
ITPRCS:
Tone is NL,
Power is NL,
Reflexes of Knee is compromised.
Sensation of little toe is not intact.
Coordination is NL.(I forgot to mention SLR!!!!!).
I said I want to check DRE to check to anal tone with consent of the patient( not available)
I said the condition that you have is called Disc prolapse have you heard of this? (Drew pic),
there is a cushion between your lower back bones called nucleus pulposus which is most likely
between L4-L5 bones prominence.
It is ruptured some time due to sudden lifting and bending movements like your case thence it
puts pressure on the nerves surroundings and causes this tingling and pain you have in your legs.
Other condition like inflammation around that cushion or just mechanical back pain or
spondyloartheropathy which is chronic condition not related to your case here.

Feedback:PASS(Global score:4) Key steps:1,2,3 Yes…but 4:No History:4,


Choice and technique of examination and organization and sequence:6, Dx/DDx:4
Handbook condition 47
Acute low back pain and sciatica in a 30-year-old man
You are working in a general practice. Your next patient is a 30-year-old self-employed landscape
gardener who is complaining of disabling left sided low back pain. The pain came on suddenly
yesterday whilst lifting a heavy rock. The pain is also felt down the side of the left thigh and leg
and the outer side of the foot. It is made worse by coughing and movement. The patient could
not sleep last night despite taking two Panadeine® tablets (paracetamol 500 mg codeine
phosphate 8 mg per tab). The patient has previously been in excellent health and has no other
relevant past or family history.
Abnormal examination findings are: He has difficulty standing or walking on his toes on the left
side. He has severe limitation to left straight leg raising, with a positive stretch test, diminished
left ankle jerk and diminished sensation to light touch on the outer aspect of the left foot, and
painful limitation of lumbar spine movements, particularly flexion/extension and left lateral
bending.
YOUR TASKS ARE TO:
• Advise the patient of the most likely diagnosis and management required.
• Counsel the patient about when he can return to work and any necessary modifications that
may be required.
There is no need for you to take any additional history, nor request any further examination
findings. All the information you need is detailed above.
AIMS OF STATION
To assess the candidate's ability to diagnose and treat the problem of acute exertion-related low
back pain and sciatica.
EXAMINER INSTRUCTIONS
The examiner will have instructed the patient as follows:
You are a self-employed landscape gardener aged 30 years. You have consulted this doctor
because of the sudden onset of severe disabling pain in your lower back yesterday which moved
down your left thigh and leg into your foot. It came on when you lifted a heavy rock and you
have not been able to work. You could not sleep last night despite taking Panadeine® tablets. It
hurts to move and to cough.
You usually keep in excellent health with no serious medical problems in the past.
The doctor has taken your history and examined you. He will explain the problem and what you
have to do.
• Show concern about how you are going to be able to work now and in the future.
• Appear to be in severe pain - sit uncomfortably be restless.
• State dissatisfaction with level of pain relief — you could not sleep last night.
• Expect the doctor to 'do something' to get rid of the pain.
• Resist advice (irrationally) not to go to work even in a supervisory capacity because of
important jobs needing to be finished.
• Become compliant if the doctor explains the situation and gives appropriate advice.
Questions to ask unless already covered (candidate's likely response is detailed in brackets):
• What has happened to my back?' (Explain 'slipped disc' — intervertebral disc prolapse with
herniation of nucleus pulposus — use of a diagram can be helpful).
• How does this happen?' (Very common, related to stress on back whilst lifting).
• 'Do / need to see a Specialist?' (Not at this stage, will be arranged if symptoms persist!
• 'Can't I have an operation to fix it and relieve the pain?' (Usually not necessary, but wil
depend on progress).
• 'What else can I take for the pain?' (Panadeineforte03).
• How long will I be away from work?' (Depends on progress. Usually settles rapidly with I
adequate rest. If so, off work for 1-2 weeks. If pain does not settle, must be investigated by CT
orMRI).
• 'Should I see a chiropractor?' (Definitely not at this stage; manipulation may worsen the
condition).
• Will I be able to lift heavy objects in the future?' (Give advice on how to lift with gooi self-
maintenance strategies).
• 'Will I always have a bad back?'{No, likelihood of recovery is good).
EXPECTATIONS OF CANDIDATE PERFORMANCE
Diagnosis and explanation of condition
• Anatomy of lumbar spine (this is an L5/S1 level problem, involving the S1 nerve root)
• Causes of pain particularly disc prolapse with nerve impingement/irritation (radiculopathy).
• Expected course both short and long term — most resolve completely
• A diagram would assist.
Immediate management
• Adequate rest is essential (3-4 days rest at home, but up and about as tolerated)
• Pain-relieving medication — Panadeine®, Panadeine forte® or similar (including NSAID).
• Subsequent physiotherapy and back-strengthening exercises.
• Physiotherapy — stretching and arching active mobilising exercises appropriate once initial
symptoms ease.
• Orthopaedic, neurological or rheumatologic consultation — will be required for lack of
resolution.
• Indications for further investigation — lack of, slow, or incomplete resolution. Then needs CT
or MRI.
• Need for investigation — this is particularly important if there is no improvement, or there is
continuing evidence of neurologic or muscle weakness (CT acceptable, MR I preferred, plain X-
ray gives limited information only).
• Avoidance of manipulation.
• Gentle traction may have a place in treatment if progress slow — would be advised after
specialist referral.
• Emphasis on positive approach. Prognosis for recovery within a few weeks is good despite
ankle jerk being affected.
Preventive measures 'Back education' including advice regarding bending and lifting, and the
value of walking, swimming.
Future management
Reassessment in short term (2-3 days). This is essential.
key issues
• Ability to determine the likely cause of the sciatica and to explain the cause to the patient.
• Adequate knowledge of the management of a patient with acute sciatica including what
further investigations or referral are required and when these should be done.
• Ability to advise the patient about work practice modifications required to prevent a
recurrence of the problem.
• Ability to advise early rest and short term review.

CRITICAL ERRORS
• Failure to make correct diagnosis of a likely disc lesion.

COMMENTARY

Low back pain is a very common problem in Australian society. The incidence increases with age
and is more common in manual workers than sedentary workers.

A major problem in spinal assessment is the fact that there is often a poor correlation between
clinical presentation (the patient's history and examination finding) and the imaging findings.
Imaging abnormalities will be found with increasing frequency in individuals with or without
accompanying symptoms from their third decade onwards.

Back pain, acute and chronic, is thus one of the most common of all conditions encountered but
precise pathology is very frequently lacking. The portmanteau and nonspecific term 'Mechanical
low back pain' is useful in that it codifies a very common condition from which almost all
individuals will suffer at some time of their lives. In such instances the precise pathology is
indeterminable and no specifically diagnostic imaging or other test is available.

Back pain may (or may not) follow an identifiable injury or strain as occurred in this patient
Pain is usually self-resolving over a period of days or weeks, but may become recurrent,
relapsing or chronic, and is influenced by cultural, psychological, socioeconomic and other
personal factors in its incidence and persistence. Against such a background, it is hardly
surprising that the condition and its preferred treatment remain controversial.

The outcome of physical treatments such as massage, manipulation, heat, light,


sound/ultrasound, electricity and magnetism (and surgery) are each difficult to separate from
placebo and are prone to fashion and fetish. Clinical studies are possible and literature search
and meta-analysis can be helpful and reveal (for example) that laser treatment of low back pain
is free of concerning side-effects, but gives short term outcomes no different from placebo, and
is expensive and not cost-effective.

Distinguishing true radicular sciatic pain ('sciatica'), due to nerve root compression requires
symptoms of pain of lancinating or cramping type, extending usually from low back and
buttock down the leg to foot and toes corresponding to sensory disturbance within the
dermatomal distribution of appropriate nerve roots (most commonly L5 or S1 ). Exacerbated
by straining or coughing. with positive nerve tension signs, and sometimes with objective
motor weakness and sensory loss corresponding to the appropriate motor nerve root.
Such a constellation of objective signs (as in this patient) is virtually pathognomonic and
diagnostic of nerve root foramen compression (from intervertebral | disc prolapse, facet joint
arthropathy or other encroachments on the relevant nerve root or spinal canal). Confirmation of
the diagnosis can usually be made by noninvasive imaging, of which MRI is the most accurate.
Persistence of unrelieved pain after one month is an | indication of the need for a full history
and examination (including diagnostic imaging), concentrating on the search for pointers of
more serious pathology (malignancy, referred back pain from intra-abdominal lesions, bone
infections, or cauda equina symptoms such as interference with bowel or bladder control).

This case scenario has been chosen to exemplify the classical syndrome of nerve impingement
radiculopathy, the most likely diagnosis being compression from an intervertebral disc prolapse
between L5 and S1.

By contrast, most cases of simple mechanical back pain due to musculoligamentous soft tissue
strain injuries will resolve within one to two weeks. with explanation and encouragement,
early mobilisation without bed rest ( 'don't take back pain lying down), and simple analgesics,
aided where indicated by a short course of physical therapy concentrating on early
mobilisation and an active exercise program, and patient education regarding good back
strategies.

Plain radiographs for patients with persisting chronic pain rarely are of clear cut diagnostic value,
but may show loss of disc height, gas formation in the nucleus pulposus, adjacent vertebral
marginal sclerosis and osteophytes, or other radiological signs of lumbar vertebral spondylosis
affecting the facet joints. However, similar radiological signs or evidence of minor spondylolysis
or spondylolisthesis are also present commonly in nonsymptomatic middle-aged or elderly
people.

MRI is the investigation of choice for defining spinal pathology when surgery is being
considered. Surgery is, however, indicated in only a very small percentage of patients with low
back pain and it is quite rare to demonstrate treatable new pathology in patients with chronic
low back pain, which has lasted for more than a year.

Associated job dissatisfaction, depression, obesity and socioeconomic deprivation are commonly
found in such instances. Long-term treatments with laser, shortwave diathermy, ultrasound,
acupuncture, transcutaneous electrical nerve stimulation, formal physiotherapy or chiropractic
have not convincingly been demonstrated to have other than placebo effects.

The effects of repeated image-guided facet joint, epidural or nerve root foraminal injections of
local anaesthetic or corticosteroids are also disappointing in the long-term. Percutaneous
semisurgical procedures (radiofrequency rhizolysis) also seem of little convincing long-term
value.

Surgical techniques have improved in the small group of patients requiring surgery, and release
surgery for focal major nerve root compressions confirmed by imaging can be dramatically
effective. By contrast, spinal fusion techniques for chronic low back pain are various, results can
seldom be guaranteed and persisting pain after surgery is common.
115-Heroin Addiction
Sample Case
a lady comes to your GP she has heroin addiction and wants to cut it down as her husband is
going to leave her
TASKS
-take History
-Counsel the patient

AMC exam Case


11-4-2018 heroine counselling, tell dx to examiner
20-7-2018 heroin addiction history and diagnosis to examiner

History
1-Appreciate him and make Confidentiality
- Hi ----I appreciate your coming to see me today. I can see from the notes that you’ve been
taking Heroin and it’s good that you are trying to cut it down.
-can I ask you a few questions first to assess your current condition and let me assure that
everything we gonna discuss here will be confidential, unless there is a harm to yourself or to
others.

2-Heroin questions
-Since how long have you been using Heroin?
-how often do you take it?
-can you tell why you started using it?
-How do you use it as injection or inhale it?

3-Tolerance question
-since you started using Heroin, have you increased the number of time or dosage to get the
same effect?

4-Dependence question
-how long can you go without taking it?
-have you noticed any withdrawal symptoms if you didn’t take it? For example nausea and
vomiting, abdominal cramps, agitation, insomnia or inability to sleep, sweating, chills, goose
bumps, rhinorrhea or excessive nasal discharge, cravings for heroin
-do you think you are dependent on Heroin?
-have you tried to quit heroin in the past? Why do you think you could not?

5-Motivation question
-On a scale of 1-10 how motivated are you to stop Heroin?
6- Psychosocial history and HEADS
Mood questions
-how is your mood?
-have you ever thought of harming yourself or others?
-how is your sleep? Do you find it hard to get to sleep or you wake up early in the morning and
find it hard to get back to sleep?
-how is your appetite? Have you lost or put on weight recently?
-any difficulty in concentration?
Delusion and hallucination
-just a routine questions I ask to my patients. Do you see, hear or feel things that others do not?
-do you think someone is trying to harm you or spying on you?
Insight and judgement
-do you think you need a professional help?
-if we supposed there is a fire in this room what would you do?

HEADS.
Home
-Who do you live with?
-any stressor at home?
-do you have good support?

Education/ work
-do you work?
-any stresses at work?

Anhedonia (you can ask this here or with mood questions as it is one of the 2 major criteria of
depression)
-have you lost interest in things you used to enjoy?

SAD
-do you smoke, drink alcohol, take recreational drugs aside from Heroin?
-are you sharing needles?

Social activities
how is your social life? Do you go with your friend quite often?

7-past mental history + general


-Any past history or family history of mental illness?
- Any problems with law so far?
-Any financial or money problems
-Past medical history
Counselling
Talk about Heroin
Condition and clinical feature
-as you know heroin is derived from medication called morphine, which is a drug of addiction
that can cause tolerance and dependence on it.
-When you use it, you can have a clouded mental functioning and you will be in an alternatively
wakeful and drowsy state and feeling like warm flushing in your skin and dry mouth.

Complications
-Because of your cloudy mental state, it can affect your decision making abilities making you
more prone to accidents.
-if you inject heroin especially if you shares needles then there will be high risk of contracting
hepatitis C and HIV, infection of heart valves, repeated injections can lead to collapses, infections
and pus collections.
-If you inhale heroin there is a high chance of lung problem like pneumonia.
-heroin can cause permanent damage to heart, kidney and brain.
-just in case you overdose it breathing suppression can occur lead to coma and permanent brain
damage.
-can cause trouble with law, impairment of social activities and financial problems.

Management
-you require a referral to the drug addiction centre and the usual preferred mode is complete
abstinence from Heroin.
-once you cut it down completely you can get withdrawal symptoms like nausea and vomiting,
abdominal cramps, agitation, insomnia or inability to sleep, sweating, chills, goose bumps,
rhinorrhea or excessive nasal discharge, cravings for heroin.In order to minimise these symptoms
you will put on medications like methadone or buprenorphine to help relieve cravings and other
physical symptoms.
-you will be given CBT by psychologist to help you stop drug use and enhance self-control as
well.
-other is motivational enhancement therapy, which will motivate you and help you engage
better in treatment.
-with your consent, family based therapy can also be arranged.
-support groups are available
-Life style advice+ sleep hygiene+ relaxation technique
-reading material and refer to drug rehab centre.
-STI screen if heroin taking by injection
-any specific concern I’ll be here to help you.
Note/ in the exam there will be 2 cases one is a counselling case and management and the other
one which is 2018 is to take history for 6 minutes then tell your diagnosis to the examiner.

Your history will be like this in the lecture.


And the diagnosis is heroin addiction or substance misuse and tell the examiner all positive
findings from history that lead you to the diagnosis.

Feedback comment
I passed this case (but I didn’t do it satisfactorily to me) Rp wanted to quit drugs and asking for
help as she desperately wants to save her family.
I asked heroin addiction questions and the psychosocial hx, but nothing was positive.
At the end I was confused for few seconds then rp told doctor I wanna keep my children with
me.
Then finally I told I'm not very sure abt the dx but I think this is heroin addiction.
I passed with global score of 6

Feedback 20-7-2018 (very important)


Heroin
Substance use pass , global score 5
key step 1,2,3,4 all yes
approach to patient 5
history 4
Dx/DDx 5

Patient who is your occasional patient , has 2 children , husband find out she is using heroin ,
now very distressed cos husband said if she not stopping , will leave her and take her children
away from her , debt on credit card and from family and friends as well
History for 6 mins
Tell examiner the diagnosis with reason

Jasmmine, i utd that you would like to stop heorin using , i really appreciate that you have
come to see me today , i am sure that you made the right decision . Before that would you
mind if i ask you a few question , Every thing we discuss here is confidential , i will not
disclose any formation unless it is dangerous to you and other people ,

since when you use the heroin , i used it since my 20 , but i stopped using it , but lately i
restarted using it 5 years ago , is there any reason why u started to use it again , my fris use
it , so i just happened to use it , where do you get it from my fris .. where do you usu use
it .. sometimes alone at home , sometimes at fris place , in what method do you prefer using
it ... injection , she does not share needles

have you been increasing the dose of using lately ... no but i dont use very often before but
now i use all week days .. only in the weekend , i dont use as i have to work as online
marketing ... How long can you go without using it ... she does not give specific answer, do you
have any symptoms if you dont use ... feel anxious and somtimes , craving for it ... do you feel
relieve when you use it again Yes

how is your motivation level to stop using it on the scale from 1 to 10 , its 10 , great i
appreciate that , do you use any other drugs no , other SADMA is negative ,

how is your general health , good , any fever ,wt loss , lumps and bumps , yellow
discolouration of skin , Running nose , watery eye ,yawing , drownsiness.... no how is your
mood , good appeptie , sleep ..... normal ... psychosis do you hear or see thing that other ppl
can not see or hear no have you ever been troubled with law and crime or accident ... no ...
would you tell me more about family life .... live with husband and two children , by any
chance , when your husband find out that you have been using it, does he do anything
violent to you ... no , he's good to me , how about when you are on drugs , any violence to yr
children ... no , financial problem + , feeling stressed about that ,

I started to counsel some , rehabilatation center, medication for withdrawal sym, family
meeting , centerlink ,to stay away from ppl who use drug.......... after some times , examiner
stopped me , you are not supposed to tell anything to pt ......

I looked at the question , i turned to examiner , and said

based on the story , my dx is my patient has heroin addiction , the reason is she has
tolerance ( even tho she is not increasing the dose of heroin , she has been increasing the
frequency of using ) , and dependent on heroin ( as she has withdrawal symp if she does not
use it ), her motivation level is very high to stop heroin , she does not share the needle so she
does not increase the risk of getting HIV , Hep B and C ,

she does not have any symp of depression or psychosis or overdose, but she has family
problem and financial issue that make her stress a lot

in this station , i left long time .....


116-Patellofemoral syndrome (Chondromalacia patella)
You are in GP, 18 year old girl who is hockey player (not professional) present with right knee
pain for 2 months which is worse on climbing up and down stairs and less pain on flat floor.
Tasks
-Ask further history (3 minutes)
-PE findings from examiner
-Management with patient

Recall 21-2-2018
Pain in RT knee, hockey player, meniscus and ligaments ok: Anterior knee pain(22F/ sports).
Recall 4-7-2018
Knee pain in hockey player.
Recall 20-7-2018
chondromalacia patella. Hx, pefe, provisional diagnosis and dds, management of provisional.

History
1-Pain questions
-how severe is your pain from 1 to 10 1 is the least 10 is the most? (4-5) would you like me to
give you painkillers?
-I can see that you have pain for 2 months so is constant or come and go? Is it getting worse?
-where exactly the pain is? Any pain in the left knee or anywhere else? Does it go anywhere
else? (Pain in front of knee, no radiation)
-how do you describe the pain?
-anything make it better or worse? Have you tried anything to relive it? (aggravated by walking,
relieved by rest)
-has this happened before? (No, this is the first time)
-How does it affect your life? (affect daily life, I cannot play hockey now and the becoming game
is in the next 2 weeks, I cannot go to university because of pain) here show empath like I see it
must be very distressing for you I will try to figure it out and help you with this.

2-Differential diagnosis questions (HOST M) + meniscal and collateral ligament tear


-any tummy pain or rash? (HSP) (No)
-any fever? (OM) (No)
-any joint pain or swelling? (SA) (No)
-any recent flu or viral infections? (TS) (No)
-have you had any trauma? (T) (No)
-any loss of weight, appetite or lumps and bumps? (Malignancy) (No)
-do you have difficulty straightening your leg? (locking for meniscal tear)
-are you able to walk and bear weight without giving way? (giving way for collateral lig tear)

3-past history
-any history of thyroid disease, asthma or any illnesse? (No)
-any family history of joint or bone problems? (No)

4-SADMA all ok
Physical examination from the examiner (Will be card) but just in case
1-General appearance (DR PJL)
dehydration, rash, pallor, jaundice , LAP (Normal)

2-vital signs especially fever (all normal)

3-BMI (normal for age)

4-Quick chest, heart and abdomen examination (Normal)

5-lowe limb focus exam


-with patient consent I would like to examine both legs including hip, knee and ankle joints and
start from the normal side first.
-Gait (not assessed)
-(Left knee normal)
Right knee:
-look (SSSDW) scar, skin color change + bruises, swelling, deformity, wasting (normal)
-feel: tenderness, temperature, joint effusion (when you press on patella there is pain).
-move: any restriction in rang of movement (passive movements with minimal restriction)
-measure limb length

6-office tests (negative or not available)


-UDT
-BSL

Explain diagnosis and differentials


Diagnosis
-well, according to History and PE findings, there are a couple of causes, which can cause your
right knee pain.

-Draw a diagram of knee joint. This is your thighbone, this is shinbone. and this is knee cap bone.

-One of the causes is chondromalacia patellae. (What’s that Dr)


Ok, this is inflammation of undersurface of knee cap bone as it rubs against with underlying
bone and cartilage.

-Other cause is patellar tendonitis, which is inflammation of tendon that attach knee cap bone

-these are usually due to overuse of knee joint or trauma and they are common in Athlete.

-presenting as pain in front of the knee, which increase with exercise or bending of knee like
climbing, running, jumping.

Differential diagnosis
-It could be due to infection of joint like septic arthritis, infection of bone like Osteomyelitis but
they are unlikely, as you do not have any fever.
Could be ligaments injury or meniscal tear, which are supporting structures around your knee
jt, they are unlikely too from examination.
-could be Osgood schlatter disease which is inflammation of bony prominent in the shinbone
but they are also unlikely.
Investigation: X-ray to rule out injury
Treatment
1-PRICE
-So, what we need to do for now. First I am gonna give you painkiller to relieve the pain
(ibuprofen)
- is I’d like you to have a rest at home.
-you can apply ice over the site of pain
- put elastic bandage to support the knee joint, try to use shoe inserts to support the ankle and
feet.
2-refer to physio for stretching and strengthening exercises

3-in the future try an alternating activity program for example high impact activity one day
(running, playing sports) the low impact activity next day (swimming, ridding bicycle) (if you have
time)

4-Medical certificate to get off day from university.

***(RP : Can I involve in upcoming game?) do not forget to cover this question because she told
you about it in the history
I am sorry that, at this point of time, you can’t engage in active sports because it can cause more
damage of your joint and bones. But if u recovers from pain, u can continue your sports.

5- I’ll review u after 2 days, this is reading material and told her red flags symptoms

Case (22/2/2017) (important )


You are in GP, 18 year old girl who is hockey player (not professional) present with right knee
pain for 2 months which is worse on climbing up and down stairs and less pain on flat floor.
Tasks
- Ask further history
-PE findings from examiner
-Management with patient
2 mins thinking
-Usually, knee pain comes with PE. Now it is changed to HO. (:p LOL)
-My DDx are chrondromalacia patellae, patellar tendonitis, injury, Osgood-schlatter ds,
ligament and meniscal tear
-Dun forget to offer pain killer and ask affect daily life

History taking
When I stepped in, A young girl was sitting on a chair and she is keen to know what happened to
her. I greeted and asked to grade pain and offer pain killer.
Pain Q - Pain in front of knee (I asked her to point with one finger), Severity - 4-5, No radiation,
aggravated by walking, movement, relieved by rest. Affects daily life - she cant play hockey now
and the upcoming game is in next 2 weeks, she cant go to university too bcoz of pain
No joint swelling, no rashes, no fever. No HO of fall or injury. No similar problem in left side.
Past HO and family HO of joint problem – No
SADMA - all clear
PE from examiner (He gave me a card)
GA, VS – normal
Left knee - all normal
Right knee - no swellings or redness, full range of movement, when u press on patella, pt has
pain.
(So, I asked examiner “I d like to know the name of special test which is mentioned on card, is it
clark test or patellar tilt test” (Examiner, this card is all I can give u).

Explain to roleplayer
Well, Samantha, according to HO and PE findings, there are a couple of causes that can cause ur
right knee pain.
I drew a diagram of knee joint. This is ur thing bone, this is shin bone. And this is knee cap bone.

One of the causes is chondromalacia patellae. (What’s that Dr)


Ok, this is inflammation of undersurface of knee cap bone as it rubs against with underlying
bones. It is seen in active persons like who runs or do sports a lot.

Other cause is patellar tendonitis, which is inflammation of tendon that attach knee cap bone
It could be due to infection of joint like septic arthritis, OM but they are unlikely as u dun have
any fever.
Injury like ligaments and meniscal tear, which are supporting structures around ur knee jt, they
are unlikely too.

So, what we need to do for now is I d like u to have a rest at home. PRICE - give u pain killer, MC
to get off day from university, Ice compression, elevation. Dun move excessively during these
days.

(RP : Can I involve in upcoming game?)


I am sorry that, at this point of time, u cant engage in active sports coz it can cause more damage
of ur joint and bones. But if u recovers from pain, u can continue ur sports. I ll arrange
physiotherapy for you who will teach u exercise regimen to get ur muscle strength again.

I ll review u after 2 days, this is reading material and told her red flags symptoms

AMC Feedback - Knee pain : Pass (Global Score - 5)


Approach to patient - 5
History - 5
Dx/Dx - 6
Management plan - 4
Feedback 21-2-2018
CASE 10: kNEE PAIN- PASS
adolescent male , hockey player with knee pain .Tasks: history,pefe and ? Dds and treatment
with patient
pain questions, painkiller, localised knee pain, not radiating more on playing , decreses with rest,
no able to wal at times, no swelling, other joint involvement, trauma, fever, past history, family
history, not on medications.
Asked about the hockey practice, warmup before playing,etc.

Pefe: positive was patellar tilt test, everything else normal just some pain in
passive movements with minimal restriction

draw a picture explained due to repeated stress,and strain along with dds – fractures, arthritis,
trauma
treatment; RICE , physio,etc

Feedback 4-7-2018
22yr girl with R/knee pain x 6/12
Task-hx, PEFE, DDx, Mx
Start with pain-no pain now
Hockey player-pain over 6/12 ,no particular triggered event, worsen with activity Walking ok, no
radiation to leg/no hip pain Knee mvt good Otherwise well-PMHx-nil significant Not going into
too detail unrelevant hx

PEFE-card given-lot of detail on the card Only +ve finding tender patella, normal menisci

DDx-explained with diagram


Patella tendonitis/patella femoral syndrome
Unlikely ligament /menisci damage as all normal-

Mx
PRICE in acute stage
Rest from offending activity
When told pain relief-was told in high pitch voice that’s a good idea
Physio for strengthening- stressed this as she is a hockey player
Feedback 20-7-2018 (very important)
Chondromalacia patella
Knee pain pass global score 5
key step 1 , 2 Yes
key step 3 No
approach to patient 4
history 5
Dx/D/Dx 5
management plan 5

22 yr old , Female patient with knee pain , interfere with her sports
History , PEFE card , Dx and DDx , and Mx
Positive finding - history hockey player , pain on going upstair and downstair , pain 6
months ,PEFE card -

pain killer offer


pain questions , pain on the right knee for few months ....said at my knee cap pointing
towards it , severity .../10 , worse with going upstair and downstairs , never consulted to
doctor before ..., history of hockey playing , .... hours a day , i asked any increase in training ..
she said no , but routinely she has to play a lot , she has got 3 games in every week , no
redness , no swelling , no fever , no history injury or fall , no twisting , no change in foot
wear recently , no joint pain in other areas , no preexisting joint problem , no family history
of bone and joint disease , no rashes in body , SADMA NAD
concern .. conerned that she can't play sports well

PEFE card
long findings
Right knee pain on pressure on patella , meniscus and ligament normal , normal range of
movement ,
( yes , they didnt mention any name of special test)
Left knee .... they wrote findings ... all normal
Explain Dx and D/Dx
jenny , based on the examination finding , most likely cause of your knee pain is due to a
condition called condromalacia patella , have you ever heard of it before ,( drawing ) it is the
inflammation of undersurface of knee cap , as you are very active and play sports , sometimes
it may lead to repeated stress and strain , the undersurface of knee cap rub against the
thigh bone repeatedly leading to inflammation . thats why you have the pain esp when i press
over the knee cap .
I also think about tendon , meniscus and ligament injury which are the supporting strutures
of your knee , but they are less likely , because their examiantion finding were normal , i
think about fracture , but unlikely .
as you dont have preexisting joint disease before and there is no limmitation of range of
movement , the arthritis is less likely , infection of joint is far away from my dx as you dont
have fever

Management
Jenny , i know its discomfort but it is not serious condition , we can help you
i will prescribe you effective pain killer , you take it after your meal to prevent stomach burn.
you can apply ice packing when you have the pain ,
you will need to rest during this painful period , i will give you certificate . ( patient said she
wants to play sport )
i said jenny i am sorry , during this inflammation , i wd advise you to rest , not involving in
sport as this may cause further damage to your knee joint , however i wd refer you for
physiotherapist who can help you to do effective exercise so that you will have quick recovery
and can return back to your normal sport activities as usual gradually .
i tell red flags ....fever , persistent pain , redness and swelling

Feedback 5-12-2018
Knee pain in hockey player ( Dr Muhit)
1. Task HX and Pefe examiner will give u only the findings on the card on request.
2. DX and DDX
3. management

( no H/o trauma.. pain while playing and using stair. No pain on rest, no stiffness, no other joint
problem, On PE card tenderness while pressing patella, other ligaments and meniscus normal)
HX: Pain scale 6/7, I offered pain killer she said she is fine, already took it.

S( RT knee all around it) Onset( after play) Radiation ( no) Type of pain( sharp pain) S(already
asked) Aggravate( play , running) , Relieve(Rest, panadol sometimes) , associated( fever-no,
Nausea, Vomiting), Is it 1st time?(yes)
How’s affecting your life? (badly)
D/D Q:
Fever (already asked), redness, swollen/lump around there?- Septic arthritis/ osteoarthritis—no
Previous cold/ rash/ abdo pain- HSP—no
Previous joint problem?- no
Bleeding disorder-p Haemarthrosis (bleeding disorder)—no
Trauma? –no
Constitutional s/s- LOA, LOW, ---Tumor –no
How’s general health? –ok
Occupation- Hocky player, every day she is playing still now but it’s painful and sore.

SADMA- Negative.

PEFE:
Gen appearance- N
Gen examination-pallor/ Laym node, Edema- negative
I said I want to focus my examination to local knee examination then examine gave me PE card
where I found pt has got slight pain when you put pressure on patella ( but no specific test was
not mentioned positive like Clarkes test) all other findings were normal.

Managemt-
I said from Hx and P E findings there could be different possibilities for your knee pain , I drew a
picture Pointed out what is what, then said could be inflammation of tendon called patellar
tendinitis, could be patello femoral Syndrome where knee cap comes is touch with thigh bone as
you play hocky long term wear and tear can cause some damage to bone covering present as
pain. Could be OSD where continuous pulling of tendon over bone can cause inflammation and
micro fracture (avulsion fracture).. could be Septic/osteoarthritis but unlikely as u have no fever,
could be injury to muscle/ligament/bone/ shock absorber disk called menisci( pointed out
drawing).
She nodded, said what to do—I said 1st Rx is Rest then follow PICE , she said what rest, I have
game tomorrow, I said sorry but its your prime treatment and it will take 3 to 6 months to heal
properly but if u continue play It will not resolve. She was not happy , I said I will try my best to
help you and thinking of referring to Physiotherapist who might consider special strapping for
you before you play once your pain is settled down, and I will contact with your coach as well.
Bell Rang. That’s it.

Key steps-1,2,3- yes, yes, no

Global score- 5
117-Testicular torsion
Case 1 ?????????
Young boy 10 years was riding bicycle and started having pain in lower tummy and vomiting. He
had same episode few days back which resolved by itself.
Tasks:
-History for 2 mins,
-Ask specific findings from the examiner,
-DDX,
-Management

Case 2 (most common scenario)


11-yr-old pain in scrotum
Tasks
-Ask history,
-Do pe (examiner gave a poor quality photo),
-Explain Dx with reasons

Recall of 16-3-2018/ 6-4-2018/ 20-7-2018


Testis torsion. Hx, pefe, explain diagnosis

History
1-hemodynamic stability

2-pain questions
-severity+ pain killers and allergy
-onset
when did it start (last 3 hours)
sudden or gradual? (Sudden)
constant or intermittent? (Constant)
-radiation (groin)
-character
-anything make it better or worse?
-has this happened before? (No)

3-associated symptoms questions


-nausea and vomiting
any nausea or vomiting? (Yes)
how many episodes?
Colour and content?
-fever or rash
-bowel and bladder
how is your bowel motions?
Any pain on passing urine or frequent urination?
4-others
-any lumps in your tummy? (hernia)
-did you hurt yourself? (trauma)
-any swelling or redness of scrotum? (torsion)

5-BINDS

Differential diagnosis
1-acute abdomen (acute appendicitis, bowel obstruction)
2-infections (gastro, UTI, epididmoorchitis)
3-hernia
4-teticular torsion

Physical examination from examiner


1-General appearance (DR PJL)
2-Vitals especially fever
3-focus abdomen
inspection (distension, mass)
palpation (tenderness, rebound tenderness, rigidity, guarding) mcburney
auscultation (bowel sound)
hernia orifices.
4-with consent of mom or him according to role player examine scrotum
inspection (swelling, redness, horizontal lie).
palpation (tender to touch)
(thick tender spermatic cord)
(prehn sign)
5-Office test (urine dipstick

Explain (check 20-7-2018 feedback as well)


-from history and examination john most likely has testicular torsion. The testis is usually
suspended by a cord, which contains blood vessels. In this condition, the cord is twisted around
for unknown reason compromising the blood supply to the testis causing these symptoms.

Treatment
-this is a medical emergency, I need to arrange an ambulance to send him to hospital
immediately where he will be seen by a surgeon who will perform an operation called
orchidopexy to untwist the affected testis and fix it in place. The optimal time to operate is 4-6
hours or the testicle might get damaged and become non-viable.

-In the operation they will fix the other testis as there is high chance of occurrence of torsion to
the other testicle.

DDX
-other possibilities are appendicitis, bowel obstruction, UTI but unlikely.
Questions if asked
1- does he require any Ix
no Ix required because it can cause undue delay in the treatment.

2-does it affect fertility


no, they will try to save the testis and also the other testis will be still functioning.

3-will it cause cancer


No

Case (30/3/2017) (23/6/2017) (27/7/2017) (24/8/2017)


Torsion of testes (11-yr-old pain in scrotum)
Tasks
-Ask history,
-Do pe (examiner gave a poor quality photo),
-Explain Dx with reasons

Case (29/3/2017) (????????)


Young boy 10 years was riding bicycle and started having pain in lower tummy and vomiting. He
had same episode few days back which resolved by itself.
Tasks:
-History for 2 mins,
-Ask specific findings from the examiner,
-DDX,
-Management

2 mins:
DDX= hemodynamic stability, testicular torsion, appendicitis, bowel obstruction, meningitis, uti
*Asked vital signs- stable, pain killers- Reassured the mother that her son was stable.

History:
-Pain questions- severe pain in the lower abdomen, sudden, first episode, not radiating, no
aggravating or relieving factors.
-Vomiting- 1 episode
-no problems with passing urine
-Red and tender testes,
-Hernia swelling (groin swelling)- said her son does not allow her to look at his private parts
-Not sure about trauma.
-No fever, sick child questions negative
-BINDS especially tetanus immunization- normal
PEFE:
-general appearance: child in distress
-vital signs again
-Abdomen: inspection- no mass, distension, bruising

Palpation: no superficial and deep tenderness, No organomegaly, appendicitis signs-


negative,
-Hernia orifices- normal,
-bowel sounds: normal
-Testes with the consent of the mother: red, exquisitely tender right sided testes, made worse by
elevation. The other side was normal.
-Office tests: urine dipstick

Diagnosis:
Testicular torsion, explained it (the testes has twisted upon itself thereby compromising its blood
supply.
If not operated upon immediately it can lose its viability.
Gave other ddx.
Told her the risks of an operation, that the surgeon might fix the other testes as well.
Asked for support.
Global score- 5
Feedback 20-7-2018 (very important and detailed feedback good to follow)
Testicular Torsion

 Groin pain Pass , global score 6


 Key step 1, 2, 4 Yes , Yes , Yes
 Key step 3 No
 history 5
 choice and technique of examiantion , organisation and sequence 5
 Accuracy of examination 5
 Dx/DDx 6

11 yr old presented with scrotum pain ,


History , PEFE , dx with reason

Father asked what's wrong with his child and showing that he is very worried.
i calmed him down and tell him i will give pain killer and started the history
scrotum pain ... started 4 hrs ago , was the first time , on the right side of scrotum , no
radiation , was crying a lot because of severe pain , no relieving factor , he was playing
soccer 4 hrs ago, he said no hit or fall or injury to scrotum at that time
when asked about vomiting has vomited one time , the food that he ate, no greensih colour ,
has noticed redness and swelling on right side of scrotum as well
no problem with his pee and poo
no swelling at groin area or no tummy distension
has no fever
other history unremarkable
PEFE
child is crying and painful distress, Vital Stable
no pallor , no jaundice , no sign of dehydration
abdominal examination I no distension , no scar no bruise
P non tender
A normal
Hernia orifices intact
genital examination with the father consent
examiner give me the picture and tell me to ask for other findings
Sample picture

i described it to examiner and started to ask


palpation -
started with normal left side
on right side , any tenderness on testes?.... very tender
how is the testes position ...horizontal or longitudinal?.... horizontal
any high riding ? ... yes
how about prehn test..... examiner asked what is prehn test?
i described .... does the pain relieve on elevation of scrotum .... no
can get above the swelling? yes
no discharge from penis ,
any inguinal lymhadenopathy ... no
any sign of trauma ... no
urine dipstick and BSL not available

Explanation (Very good explanantion)


John, based on the story you gave it to me and examination findings , most likely david has a
condition called testicular torsion. Have you ever heard of it before ? No . i explained by
drawing , This is scrotum which is a sac that holds the testes . the testes is connected with
the cord like structure which contains blood vessel that supply O2 to the testes . in case of
testicular torsion, the testes is twisted along its stalk , cutting off its own blood supply , the
exact cause of twisting is not known but sometimes it may be preceded by minor trauma . as
you mention he was playing soccer before this happened. This is surgical emergency , because
if we dont fix this immediately , the testis is no longer getting the blood and can die ...... but
John ,i really do appreciate you brought him in time. try not to worry , david is in safe hands
, i will call surgeon to review and fix with surgery , we need to untwist it ( Mx was not a
task )
so i also continue some D/Dx ....as time left
i also think about direct major trauma to testes but no history like that, or infection of testes
it is unlikely , infection of testes is rare in his age and usu not sudden onset like this
i also think about hernia that is stucked in the groin but PE not suggesting for it .
i also think about blockage of bowel , urine infection , but they are very unlikely , tummy
examination is fine.

118-Child abdominal trauma


AMC Case 20-7-2018
10 years old boy with pain in abdomen for 3 days, getting worse , your colleagues check vital
sign and urine dipstick ( ketones +++) no WBC , no RBC, BSL normal.
Tasks
-History
-Physical examination findings from examiner
-Most likely diagnosis
-Investigation with reasons

AMC Case 11-4-2018


Child 10 yrs with vomiting & abdomen pain for 3 days , increased in last hrs
Task
-hx (child has trauma to abd fall from bicycle)
-PE from examiner (bluish sicoloration at hypogastrium , slight abd destintion wirh sluggish
bowel sounds)
-Further igx & mx

AMC Case 6-7-2018


Duodenal perforation - child bicycle accident

AMC Case 11-7-2018


Boy on bicycle with abdominal injury

Note/
Patients at risk include those with:
-High impact / deceleration injuries;
-Direct blows to the abdomen;
-Evidence of injuries above and below the abdomen, suggesting the abdomen is unlikely to have
been spared.
-Seat-belt injuries (duodenum or pancreas).
-Bicycle handlebar injuries to upper abdomen (duodenum or pancreas).
-Straddle injuries, astride a bar or beam (perineum, vagina or urethra).

Note/
-When trauma arrived
In all aspects of trauma management, the primary survey is the first priority.
-PRIMARY SURVEY
Airway with c-spine stabilisation , Breathing , Circulation assessment and management
Secondary survey
Perform a thorough back & front / head-to-toe examination for other injuries.

History
1-Hemodynamic stability check

2-Pain details
-How sever is his pain from 1 to 10? Offer pain killer after allergy question. (Very severe)
-when did it start? Suddenly or gradually? Constant or come and go? Is it getting worse? (For 3
days and getting worse)
-can you point out exactly where the pain is? Does it go anywhere else? (The whole tummy)
-can you describe it for me?
-anything make it better or worse?
-has this happened before?
-what was he doing when the pain started? Any history of trauma or injury? (Injury with bicycle
crash few days back to his upper tummy)
- was he wearing helmet, protective clothes?
-have you consulted any doctor at that time? (Pain was minor at that time that is why I did not
consult to doctor immediately)

3-nausea and vomiting


-any nausea or vomiting? (Yes vomiting)
-since when? (Last evening)
-how often? (Multiple times)
-what is the colour? Is it greenish? (No)
-any blood in it? (No)
-does it affect his feeding? (Cannot tolerate food)

4-Fever and rash (No)

5-bowels and bladder


-how is his waterworks? Any change in color of urine? (No)
-how is his bowel motions? (Normal)

6-Ass. s/s:
-any Loss of consciousness, shortness of breath

7-General (negative)
-medications
-PMH, PSH

PEFE
1-General appearance (DR PJL)
-dehydration (Painful distress, dehydrated)
-rash, bruising, signs of trauma or injury (bruise in epigastrium)
-pallor (look pale)
-jaundice
-LAP
2-V/S

3-Abdomen
-Inspection: wound, bruising, seat belt or handlebar marks, distension (bruise in epigastrium)
-Palpation: tenderness including pelvic tenderness/instability, guarding (tenderness + on
epigastrium, no guarding or rigidity, murphy sign – negative, no liver dullness obliteration, no
organomegaly).
-Auscultation: bowel sounds (Bowel sound is minimal)

4-Inspection of the back, hernia orifices and genitals (hernia orifices intact , genitals normal)

5-Urethral meatus for blood

**urine dipstick already given in the stem .. so dont need to ask again.

Diagnosis and differential diagnosis


-Acute duodenal injury most likely
-Other D/Ds or accompanying injuries to look for:
Pancreatic injuries
Spleen
Liver
Bowel
Renal injury

Explanation
-from history and examination, I suspect that at the time he got accident, he might have injured
any organ inside the tummy.
-There are some organs around there (draw a pic): solid organs like liver on the right, spleen on
the left, and pancreas in the middle and back, hollow organs like stomach, duodenum small
intestine. Most likely, the 1st part of the small bowel or the duodenum is injured. Other
possibilities could be an injury to the pancreas, spleen, liver, bowel or kidney. The bowel might
have perforated.
-I need to order some investigations to find out more about problem. And call the specialist
surgeon to come and see him.

Investigations
1-Bloods: FBC, lipase/Amylase & LFTs, Crossmatch, BM
2-Urine analysis: All abdominal trauma patients should have urine analysis done.
3-CT scan is the imaging modality of choice.
Intravenous contrast, preferably supplemented by simultaneous oral contrast, is essential.
Patients must be stable enough to move to CT scan.
4-X-ray. Only require supine and decubitus lateral / right-side-up abdominal X-rays if there is a
suspicion of intestinal perforation.
Will show air under diaphragm if intestinal, Will show ground glass appearance because of
retroperitoneal haemorrhage in case of duodenal perforation.

Not much role of U/S, FAST U/S, DPL, IVP

Management
1-ABCDE
2-Maintain 2 wide bore I/V lines
3-Fluid resuscitation with 20 ml / kg normal saline or Hartman’s.
Second bolus of fluid as above, if required.
If further boluses of fluid are required, use blood.
4-Immediate further SURGICAL review.
Pass orogastric tube.
NPO
I/V antibiotics?
All patients with free intraperitoneal air require a laparotomy.

Note/
Duodenal injury 2 types:
1-Duodenal intramural haematoma can be seen on CT scan.
This can be treated conservatively with nasogastric drainage, but may take up to 3 weeks to
resolve.
2-Acute duodenal perforation
-needs to be a diagnostic consideration at initial presentation.
-It can be detected by presence of free air. This air may be retroperitoneal, rather than intra-
peritoneal. See X-Ray with ground glass appearance.
-It may also be detected by extravasation of contrast on an upper GI contrast study or CT scan.
-Perforation can be delayed if there is an area of devascularisation with the trauma.
-Acute perforation is treated with laparotomy.
-Late presentations may be managed conservatively with NGT, TPN & antibiotics

Feedback 20-7-2018
Abdominal Pain Pass global score 4
key step 1, step 2 Yes key step 3 No
approach to patient /relative 5
choice and technique of examiantion , organisation and sequence 5
Dx/DDx 3 choice of investigation 4
**10 yr old boy with pain in abdomen for 3 days , getting worse , your colleages check vital
sign and urine dipstick ( ketones +++) no WBC , no RBC, BSL normal
History, PEFE, Dx, Further investigations
History to mother after pain killer offer
pain in whole abdomen , getting worse over 3 days , very severe,
vomiting from last evening, multiple times, whatever he ate , no greenish color, no blood ,
can't tolerate any food since then, no change in pee and poo
History of injury with bicycle crash few days back to his upper tummy, was minor pain at
that time , thats why she didnt consult to doctor immediately ,
no fever , no other past medical or surgical history
PEFE
Painful distress, dehydrated, look pale , bruise in epigastrium , tenderness + , no RT , no
guarding or rigidity , murphy sign - negative
no liver dullness obliteration , no organomegaly , Bowel sound is minimal
hernia orifices intact , genitals normal, other examination unremarkable
urine dipstick already given in the stem .. so dont need to ask again.
Explanation
Rose , i did examination , based on the findings , i found the bruise over the upper tummy ,
where he has the most pain. there i strongly suspect that at the time he got accident , he
might have injured to organ inside the tummy. there are some organs around there..... i draw
the pic ( solid organs like liver on the right, spleen on the left, and pancreas in the middle
and back) , ( hollow organs like stomach , duodenum small intestine ) ..... these organs may
get bruised or injured , or the bowel might have perforate but perforation is less likely based
on the PE findings .... so we need to run some investigations to find out the exact problem
here
i will call senior specialist for further assessment
we will run some base line testing , full blood examination , inflamatory marker, liver func
test , kidney function test , ( i forgot pancreatic enz) ... CXR to rule of perforation of bowel...
We will also need to do CT scan , special imaging to tummy to check these organs as well
and reasure her
PS , i didnt tell most likely pancreatitis , i guess that put me to get D/Dx to be 3
i forgot to tell the examiner about resuscitation , may be that is one key step

119-Haemopneumothorax CT
A young man has just been in MVA. Primary and secondary survey done are both done and the
patient is now haemodynamically stable but he is still feeling breathless. A truck crashed into his
car and the door swung onto the left side of his chest. On Physical examination there is dullness
on the left side of chest. CT scan transverse section is given.
Tasks
-explain CT scan to the patient
-Explain further management

1-approach to patient
-Hi my name is --- I am one of the doctors here I can see that you have involved in MVA I am
sorry to hear that. We have done the primary and secondary survey and let me assure you that
your stable at the moment all your vitals are fine.
-do you have any pain? if yes offer painkillers after asking for allergies.
-A CT scan imaging of your chest has been done, let see it together to discuss about the result.
Please if you feel uncomfortable at anytime let me know.

2-Explaining CT scan
-Here is your CT scan. These are your lungs.
-the CT showed a condition we call Haemopnemothorax in the left side of your chest, which
means the presence of fluid and air in the chest cavity. (point to the black and white portion
indicate fluid and air)
- It is most likely that your lungs were injured due to the accident you had something ago. fluid is
most likely blood and air is entrapped in between your left lung and its coverings compressing
the lung, that is why you have SOB. (no need to draw any diagram.)

3-Management
-arrange for painkillers
-need to admit you to hospital and need to be seen by a specialist surgeon.
-Do underwater seal drainage at 5th ICS AAL
treatment is by draining this fluid and air out of the chest. For this, a chest tube will be placed.
Which is a hallow plastic tube to be placed in between the ribs to the area around the lung in
order to drain air and blood.
I will explain to you the procedure, possible risks and outcomes. Feel free to ask me any
questions. if you agree to undergo this procedure I will ask you to sign a consent form.
-the procedure is done under Local anesthesia, after cleaning the skin with disinfectant, the area
of tube insertion will be infiltrated with local anesthesia using a thin needle to numb the area.
After insertion of the chest tube, it will be fixed in place with a suture. The tube then will be
connected to a machine to help with drainage and you will feel better after the procedure.
Auscultation and X-ray will be repeated to check if everything is fine.

Risks.
as any medical or surgical procedure a chest tube carries some risks.
-pain at site of insertion but we can manage the pain with painkillers if required
-bleeding-to decrease the risk we always insert the tube on the upper border of the rib
-infection-to decrease the risk we always use sterile technique
-very rare complications are chest or abdominal organ injury.
-Usually the chest tube stays 2-3 days, sometime longer depending on the situation. When there
is no longer fluid coming out, we will repeat the chest x-ray to ensure drainage and the tube can
be removed.

-indication of thoracotomy: excessive blood in the tube.

-Ix (blood group, cross matching)


-keep monitoring him
-if need to call someone
-reassurance he will get the best possible care.

Case (8/3/2017)
30 years old guy motor vichle accident.. Giving a chest, CT..explain to patient with mx
-Interpret and explain CT scan and explain management to patient.. CT scan after trauma-
showed Hemopneumothorax
Case (29/11/2017)
Patient had a RTA. Was driving, seat belt was on and a car hit the side onto his door at 80km/hr.
Rest all fine, but has difficulty breathing. On O2 6 L/min. HR,BP stable. RR 30/min. Breath
sounds on right decreased but trachea central and heart sounds not displaced.
Task:
-Explain CT scan findings
-Explain management to the patient
( CT scan showed Haemo-pneumothorax. Patient will ask, what is chest drain, where will you do
it, what is the complication of it, what is the outcome of it)

comment
this the ct scan of your chest... usually fluid/blood appears white and air appears black in the
scan.... from your ct scan it shows that there is blood and air present in your lungs which we
medically call it as haemopneumothorax... have you heard about this condition? as you had a
trauma to your chest, the chest wall might have punctured the pleura(the outer layer of the
lungs) whereby blood and air have entered your lungs...
its important that we need to remove the air and blood from your lungs so we need to admit
you, and put a iv drip and take blood for basic blood test and start you on fluids and i will call my
senior. we might do a procedure called chest tubethoracstomy ( a chest tube is inserted between
your chest wall to remove the fluid and the blood from the lungs and its connected to the
underseal drainage), its done under anaesthesia.
after the procedure we will do a rpt X-ray to see if the tube is in place... the chest tube should be
there for few days.

Comment
The patient was on O2 and dyspnoic when I entered . I explained myself and asked about the
pain levels and offered in this way, I am Dr X and I am really to sorry to hear that u were involved
in a MVA and ended up in the ED and it's really good that u have already been assessed by the
trauma team ( according to the Stem ) I would like to explain the CT of the chest with u . Do u
think u could b comfortable for me to proceed with that or are you happy to have some more
pain relief ? He said its all been given . He nodded his head that he is happy to proceed . Did not
ask for any vitals . I think those counselling cases the first min or so is fully patient driven and the
key is to take it in to fully candidate driven mode is to ask the Q of are u happy for me proceed ?
The dyspnoic guy was very calm quiet and very supportive. Explained every basic thing on the CT
. I said I am suspecting and based on the examination findings things are certainly very
convincive , how ever we are working on the radiology team for confirmation. I clearly said that
we are team of all core disciplines and the aim is to make sure sure u r safe and u will be given all
the support. What we normally do is under these circumstances after the admission constant
observation, pain relief and fluid management , ABs with the specialist advise , IC tube insertion ,
performing more blood and images to further assess and then a long term follow up which
includes Out Patient assessment and Physiotherapy. Then the R P asked about the IC tube and I
drew it simply explained it and the time was almost gone and when I asked are u comfortable
still and he said yes and there are many more things to be discussed and the whole team will b
with u constantly and specially myself being a junior doctor I will in touch with all the other
specialists to give u the best care that we could possibly ever do . He was happy and the bell
rang

Hi , I started with the word of CT which means a computed tomography and it helps us to have
look at your chest in coronal sections it's like I am looking at your chest from above to get a
better understanding. Then explained what black and white means started with the periphery
and just compared the 2 sides and did show him the differences. Always did show him the
normal structures first. Explained the likely possibility of a rib fractures. I could not see multiple.
But I said it may be a possibility specially with the hemothorax which was evidenced by a
collection of fluid in one side and there was a pneumothorax as well. I said some times a
fractured rib can damage the lung tissue and can even cause a tear leading to a small bleed
which over times get bigger causing DIB which might be the cause case with you. To reverse that
we undertake some acute measures and explained what IC tube and how to insert. I also
explained how to preserve the nurovascular bundle . He was happy at the end and the examiner
was just standing near me and watching every thing very carefully. Because the PT was on the
bed and I was standing. Just do not look at him. Make sure you show the precise structures and
if not sure just tell this might be some thing significant but would like to confirm with the
radiology team.

Feedback 13-12-2018 (important)


Injury to chest. PASS
2 mins outside: There was a long stem. A person with MVA and multiple injuries. Primary survey was
done. The patient is stable (vitals were given and normal except tachycardia due to pain). I was looking for
tension pneumothorax (bcs I love this topic) but all the parameters were given that shows it was not
tension pneu. CT chest was done.

Tasks: Explain CT to pt.


Provide management.

Outside thinking: CT of hemopneumothorax. There was no emphysema. I forgot to look for Ribs.

Inside: The PT was lying on bed with Mask and IV fluid.

Me: I am so sorry for the accident. U must be in lot of pain. Are you in pain now, do u want some pain
killer?

PT: yes, I am having pain and shortness of breath, why? No for pain killer. thanks, he just had it.

Me: Ok. It will take some time to kick in. I will explain

Me: I have yr CT scan which we just did. This is the CT of yr chest. As u can see here the black part is lung
blah blah and explained most of the parts of chest. Then I asked him can you see this black area here
(pointing to pneumothorax) and not on other side (pointing to normal side).

Pt: Yes

Me: that means the air which should be normally in the lungs has come out of lung into between two
pleurae. There are two covering around our lungs we call pleurae and the air is between them. This is one
of the reason for pain and breathlessness.

Pt: ok.

Me: Now can you see this white area (pointing to hemothorax) and not on other side (normal side).

Pt: yes

Me: this means fluid and that’s why it is white. This could be any fluid but in your case most likely it is
blood as u r involved in accident.

Pt: ok
Me: So to relieve pain and breathlessness we need to suck the air and blood out from these area. We will
need to treat this both conditions separately ( I read on internet this, pl check regarding management in
details). We will put two small cuts around chest (showing roughly around 5 th ICS), pass the small tubes
and put the other end into bag full of water (this is very important as per AMC clinical book, specially
written that candidate should mention this, different case though). I had some time, so I said we will keep
monitoring your vitals. I will call my senior and chest physician. Lots of sympathy. Reassure him that you
will be ok. Then bell rang. Thanked both of them.

Key steps: 1 No, 2 and 3 yes. Global core: 5


Approach to pt, Interpretation of investigation and management plan: all 5

120-pulmonary embolism
Case 1
A 27-year-old lady, comes into GP clinic with SOB since last night
Task:
-Focus history (4 minutes )
-PEFE
-Diagnosis and differential to patient
History
1-check hemodynamic stability (ask the examiner for vital signs and O2 saturation) (all normal
you can proceed)
2-shortness of breath questions.
-can you tell me more?
-for how long have you been short of breath? (Since last night)
-has it begun suddenly?
-is it constant or come and go?
-is it getting worse? (Yes)
-what were you doing?
-are you short of breath at rest or exertion or both? (If constant no need to ask)
-can you lie flat without feeling short of breath?
-do you wake up at night short of breath?
-is this the first time? (Yes)
3-symptoms related to chest questions
-Cough
-Chest pain
-Wheeze.
-Palpitation
-Fever and recent infection
4-Symptoms not related to chest questions
-any ankle or leg swelling?
-any LOW, LOA, Lumps or bumps?
5-General questions
-Past medical history (HPT, DM, LIPID, heart and lung problems, Clotting)
-Past surgical history
-medications
-travel history (long flight from USA 2 days ago, took sleeping pills -sleeping all the journey on
the airplane)
*have you travelled recently?
*where and for how long?
*did you walk or move your legs during travel?
-Trauma
-occupation: what do you do for living and what have you done in the past
-smoking? alcohol?
-contact and family history

Physical examination findings from the examiner


1-General appearance
-cyanosis, dyspnea, pallor, jaundice, LAP
2-Vital signs and O2 sat (tachycardia +ve 110)
3-chest examination
inspection: chest movement with respiration
palpation: tracheal position, chest expansion, apex beat.
Percussion: dullness
auscultation: air entry, breathing sounds, wheeze or crackles, vocal resonance.
4-CVS
-heart sound and murmurs.
-JVP
5-quick abdomen
6-Legs: oedema, calf tenderness, redness, varicose
7-office tests
-UDT, BSL, ECG
8-Any X-ray available

Explain diagnosis and ddx


-you most likely have a condition called pulmonary embolism.
-these are your lungs; your lungs are supplied by blood vessels. In your case, there is clot
blocking the blood vessels supplying the lungs.
-it is quite common after long flight due to improper circulation of blood in the legs leading to
clot formation in the legs. It can then travel to up and block the blood vessels of the lungs
leading to SOB etc…
-other possibilities are
*pneumothorax: less likely as no chest pain or chest findings on examination
*pneumonia; less likely as no fever or chest findings
*heart failure less likely

Feedback 7-9-2018
Station 19 – SOB – PASS
Pulmonary embolism-
A 27-year-old lady , comes into GP clinic with SOB since last night
Task:
1. Focus history ( 4 minutes ) –
Haemodynamically stable – YES and proceed
( thanks examiner for your instruction )
Open-ended question first – how are you feeling ? – feeling difficult to breathe
How long – since last night – get worse
Associated with chest pain – NO
First time – YES
Any infection like fever , N&V –NO
Any recent weight loss , loss of appetite – NO
Any injury to the chest – NO
Regular medication – no
Regular medical problem – like heart or lung problem – NO
Previous surgery to chest – NO
Travel history – YES .
long flight from USA 2 days ago , took sleeping pills – sleeping all the journey on the airplane .
Denies injury to the chest , denies chronic medical conditions
SADMA- no for all Smoking and alcohol
( I didn’t ask all COSTVMPF in this scenario, as I got travel history at the end of history )
forget to ask LMP , OCPills , Malignancy , Varicose Veins , BMI

2. PEFE ((not given any investigation or ecg or imaging like CXR ) , all normal for PEFE apart from
HR 110 (regular) - tachycardia -)

3. Explain Dx and DDX to the patient.


- explain Pulmonary embolism with a drawing picture , explain risk factors – long flight from USA
and not moving makes blood stasis in the blood vessel ( drawing with a picture )
Others – pneumothorax , infection of lung ( pneumonia ) ,
- heart failure , nasty conditions in the lung – lung cancer ( lease likely in your age and history )

Feedback – SOB – pass


Global score – 5
Key step 1 to 4 – YES
Approach – 4
History – 4
Examination and sequence – 4
Dx and DDx - 6

Case 2
young male with chest pain
tasks
-history
-PEFE
-management

Please check pneumothorax scenario it is like any approach of chest pain and here the travel
history will be positive and also past history of clotting problems
explain PE like above with all ddx of chest pain

Management
1-admit (if in ED) refer (if in GP) you to hospital to be seen by a specialist.
2-arrange Ix like:
*FBC, UCE, LFT, D-dimer
*ABG/VBG, coagulation profile
*If female urine pregnancy test
*Chest X-ray, CTPA
3-specialist might decide to start you on blood thinner medications (warfarin+ LMWH)

Feedback 21-2-2018
CASE 13: CHEST PAIN -PASS

young male with chest pain, tasks;history and mannagement ,not sure whether pefe

in this station

pain questions and painkiller – severe central chest pain, not radiadting, bit difficult in
breathing, mentioned patient will be shifted to the resuscitation cubicle with all monitors
attached and then further history. anxious patient, no palpitations, sweating, trauma, cough,
fever, first episode. smokes and drink occasionally, recent history of travel from london past
history of clot in legs with travel present, not on any medications, no family history.

Mentioned about pulmonary embolism, admit in hospital, arrange investigations blood, ecg,
imaging, xray etc, call specialist will start on blood thinning medications and further
investigations for blood clotting problem.

Feedback 7-9-2018 (pulmonary embolism)


Shortness of breath-PASS
ED, An young lady with SOB for last 2 days, increasing with time. No past history, no medical
condition. Non-smoker.
-HX
-PEFE
-DDX
FEEDBACK-PASS Global score: 4
Key steps: 4/4 Approach to patient: 4
Hx: 4
Choice of technique of examination, organisation and sequence: 4
DDX: 4
I was thinking fever? - Then pneumonia or infection
If fever positive, will ask TOSI- travel, occupation, sexual hx, immunity or contact hx. Plus
night sweats, rash, weight loss.
If no fever- other chest cause, heart cause , anaemia.
If Pulmonary embolism- COSTVMPF
DDX:
Chest: Pneumothorax
Pneumonia
Pleural effusion
Any infections
Pulmonary Embolism.
Heart failure
Anaemia
Malignancy
I entered the room and asked vitals first. Examiner asked me back what I wanted to know
specifically. I asked respiratory rate & SPO2. I think, RR was high and SPO2 was low. I
transferred the pt to resuscitation cubicle and started oxygen with BP & ECG monitoring.
Then, I asked her if she was comfortable to talk . Asked her details about SOB, same like
ODIPARA.
First time? any fever? (no- okay, I didn't ask any follow back fever questions to save my
time)
cough, sputum, weight loss? (all no)
Chest pain, swelling, racing of heart? (no)
Then ask travel? (yes) I got a long air travel hx . Then n there, I wrote COSTVMPF in the
paper and started to ask accordingly.
C- Contraception ( I asked about pills)
O- Obesity ( I didn't ask it, she was a thin lady)
S- Smoking ( already given- non-smoker, I didn't ask), Surgery hx
T- Travel, tumor(already asked about travel)
V- Varicose vein (no need to ask, will get it in PEFE)
M- Malignancy (already asked weight loss, just asked lumps and bumps)
P- Pregnancy ( I asked about her period and heavy bleeding )
F- Family hx.

In past hx- any previous lung problem? (no)


I didn't ask SADMA.

PEFE:
GA: any accessory muscles use? facial deformity?
BMI
Vitals: asked one by one.
PICKLED
Respiratory details: Inspection, Palpation, percussion, auscultation, trachea position.
Apex beat position, dual heart sound?
Any organomegaly?
Calf tenderness.
Varicose vein.
ECG? Chest Xray? Pregnancy test?
(Everything was normal)

DDX:
I then drew lungs picture, heart: explained her one by one. I said, there could be number of
reasons that could lead to SOB. Sometimes, infection, fluid collection, collection of air in
between the coverings of your lungs, growth of mass in your lung and some previous lung
problems: all can lead to SOB. But, no fever, no cough, no past hx, contact hx, no weight loss :
so, very unlikely those reasons are associated with your symptoms now. Most probably,
there's a clot in your lungs which in turn making your symptoms. This is usually associated
with long travel hx. We called it Pulmonary embolism. Have you ever heard of it? Then, I
explained the risk factors of it. Then, explained, sometimes heart problems and anaemia
also participate similar symptoms. I will admit you right now and call my seniors. We will do
some invx to rule out other causes which we discussed. She smiled and said okay.

121-post-partum Check up
Sample Case
23-year-old Amy has come to your GP clinic for a 6-week post-partum check-up. She had a
normal vaginal delivery and she delivered a 3.3 kg healthy baby boy.
TASKS
-Relevant history
-PE from examiner
-Counsel Amy accordingly

AMC Exam Case 20-7-2018


Women come for 6 week post-partum checkup , child was examined by your colleagues
yesterday , normal , didn’t bring the child today , on the stem , pregnancy and delivery was
uneventful nor episiotomy or laceration at delivery , normal vaginal delivery
tasks
-History
-PEFE
-counsel

History
-Congratulate her on her delivery. ? Are you enjoying your motherhood? How is your mood?
(Motherhood totally fine, mood is great)
1. Baby (normal)
- Is the baby doing well? Any concern that you have regarding your baby?
- Was he given the first dose of hepatitis vaccine?

2. Breast (normal)
- Are you currently breastfeeding your baby? (give the exclusive breast feeding 4 hrly)
- Any sore or cracked nipples? Any lumps or pain in your breast?
- Do you know the proper breastfeeding technique? (if the baby is not latching well, it can lead
to sore and cracked nipples, leading to mastitis and breast abscess)

3. Bladder/ bowels (normal)


- Any problems with your waterworks?
- Any burning or stinging while urinating? Any incontinence?
- Are you opening your bowels regularly?

4. Birth canal/ delivery/ systemic (normal)


- Any conditions that you had during pregnancy or was it uneventful? (Mentioned in the stem)
- Any complications during your delivery? Any cuts made down below or any tears that you had
at that time? (Mentioned in the stem)
- Have you had excessive bleeding right after delivery? (Primary postpartum haemorrhage)
- have you stopped bleeding now? (Secondary postpartum haemorrhage)
- Do you have any tummy pain? Any fever? Any abnormal, offensive vaginal discharge?
- Have you felt any dryness, itching and burning down below?
- Any calf swelling or pain, SOB, chest pain?

5. 5Ps (Critical)
- Have you resumed your sexual intercourse? Any problems that you are having e.g dyspareunia?
Do you have good support in looking after your baby? (Yes doctor, it is embarrassing, I restarted
4 days ago, I have pain with intercourse)
- What contraception were you on before you planned for you pregnancy? (Microgynon 30)
will you be interested in talking about contraception again? (Yes, i am interested doctor, but i do
not like minipills)
- When was you last Pap smear done? What was the result? (I did it 5 yrs. back ... haven’t done
one since then)
6.SADMA, PMH, PSH

Physical Exam from examiner


1-General appearance: BMI, pallor, calf for swelling/tenderness
2-Vital signs: temperature
3-Abdomen: visible mass, tenderness, Palpable mass (usually the uterus goes back to the pelvis
in 2 weeks time) has the uterus involuted or not? Tenderness
4-Pelvic examination
-Inspection of the vulva and vagina: any bleed or discharge, thin, dry atrophic vagina
-Speculum: arrange for a pap
-Bimanual examination: size of uterus, tenderness, Adnexal mass and tenderness
5-Office tests: UDT, BSL

Counselling
-When I examined you, you are generally healthy but I could see that your vagina is dry and thin
we call this; atrophic changes. Most likely, you have a condition called lactational atrophic
vaginitis.
-It is quite common after the delivery because as you are breast-feeding, sometimes it can cause
reduced female hormone in the body leading to dryness. As the prolactin or the milk secreting
hormone acts against the estrogen.
-Normally, it resolves spontaneously in 2 weeks once the hormones come back into place, but
you can use vaginal moisturizers during this time. If it becomes severe, then we can use vaginal
estrogen creams or tablets.
-It is not advisable to take a combined pill if you are breastfeeding as the estrogen content in the
pill can suppress the breastmilk production. But there are other methods that you can use, such
as progestogen only pills but you don’t like it, others are: Depo-provera, Implanon, or
intrauterine contraceptive device.
Depo is by injection to your buttock every 12 wks , its effective , but there are some side effect ,
risk of delayed returning to fertility after stopping it , risk of wt gain , acne.
Another one is IUCD , it last long 5 years , very effective , no delay in returning to fertility when
you stop it, but the side effects are increase risk of infection , ectopic pregnancy.
Another option is implant , that can last 3 yrs , which is very effective as well , but it needs minor
procedure done by specialist to put under the skin of your arms.
-As your pap smear is already due, I will arrange one for you now
-I will give you reading materials regarding contraceptive options so you can think about these
options.

Feedback 20-7-2018
6 wks post partum

Health review Pass , global score 5


key step 1,2,3,4 all yes
history 6 ,
chocie and technique of examination , organisation and sequence 6
patient counselling/education 4
Women come for 6 wk post partum check up , child was examined by your colleagues
yesterday , normal , didn’t bring the child today , on the stem , pregnancy and delivery was
uneventful nor episiotomy or laceration at delivery , normal vaginal delivery
History , PEFE , counsel
hello , Karen , i do utd you have come for 6 wks post partum check up , how are you doing
these days ... i am fine , how is your motherhood .... totally fine , do you get enough
support ... yes ... how your mood ... great ...
she said she didnt bring the child today who has been already checked out and normal.....
any specific concern ... no
i asked systemic question ... no fever , no cough , no sOB , no chest pain , no abdominal pain ,
appetite good, no pee and poo problem , no discharge or bleeding from down below
breast feeding the baby ... fine , no soreness , baby suck well , give the exclusive breast feeding
4 hrly
so i said i would like to ask private and sensitive quesiton
5 P .... have you restarted sexaul activity ... said yes doctor , its embarassing , i restarted 4
days ago, any problem ... pain on sexual interourse , but no contact bleeding , no discharge
have not returned any period
pill ... have you ever use contraception pill before .. yes i use microgynon 30 , will you be
interested in talking about contraception again...... ... yes i am interested doctor , but i dont
like minipills .......
pap smear ... i did it 5 yrs back ... havent done one since then
do you have any plan to get pregnany in near future .... i dont have plan yet but if i get it , i
dont mind
SADMA NAD, no PMH or PSH
PEFE
GA , VS , CVS , RESP , BREAST .. all normal
abdomen normal
pelvic examiantion .... atrophic changes , no dischagre
Explanation
Karen , i did examination on you , you are generally healthy , only finding is there is some
dryness and thining of your down below . what we called atrophic changes . it is quite
common after the delivery because as you are breast feeding , sometimes it cause reduced
female hormone in the body leading to dryness. but its not serious , it will resolve with time .
i will prescribe soothing cream and if oestrogen cream to apply, if not relieved , will refer you
to specialist .
as your pap smear is already due , i will arrange one for you
now in terms of contraception , microgynon is not suitable for your breast feeding , as it can
suppess breast feeding
there are other options like depo injection to your buttock every 12 wks , its effective , but
there are some side effect , risk of delayed returning to fertility after stopping it , risk of wt
gain , acne
another one is IUCD , it last long 5 years , very effective , no delay in returning to fertility
when you stop it, but the side effects are increase risk of infection , ectopic pregnancy
another option is implant , that can last 3 yrs , which is very effective as well , but it needs
minor procedure done by specialist to put under the skin of your arms
i am going to give you reading materials so you can think about each options
bell rang
i didnt have time to talk about non hormonal method

Feedback 30-5-2018
‘Health review’: Pass
6/52 post-partum atrophic vaginitis.
Mum & bub well, pregnancy and L&D benign, completely w/o concern.
Task: Brief Hx, PEFE & counsel
 HOPC

o Basic rapport/Mum & Bub Qs

o Lactation consultant/BF issues/support/pelvic floor regime etc

o Any concerns

o Has anyone had a chat to you about contraception

 We’ll touch on that

o Have you resumed sexual activity

 Any trouble in the bedroom

 Pain

o Further Sx evaluation

o Talked about ANC/L&D hx/Cx [all NAD]

o Basic screener for other Sx: Br Wind Wound Water etc

o PMHx FHx SHx HPV IUTD SADMA

 O/E

o Usual spiel

o GA BMI VS>Br>CNS>RS>CVS>Abdo>chaperone>pelv>spec>biman>Wards: uDip BG

 Typical atrophic vag

 Asked specifically re: other injury/perineal tear/wound/Ut changes

 Ax & P

o Atrophic vag (quick description of why it happens 2/2 lo female hormone etc)

 Moisturise

 No good  local oest cream

o LSM SNAP *Pelvic floor ex resources etc

o Bub  arrange immunization + *Offer paeds check [already done]

*ran out of time in this case & so couldn’t go further into POP etc; honestly thought I bombed
this case

Feedback11-12-2018 Station 11 Pilot Case (Assessed, not scored)


A woman comes for her 6-week postpartum check. She has no complaints.

Tasks: Hx, PEFE, Dx and Mx.


I came in the room and greeted the examiner and the patient.
I introduced myself and congratulated the mom for the baby. I asked how he was, if he was thriving, how
she was coping with it, if she had enough support.
Than I went to more specific questions:
How was the delivery? Vaginal
Was it at home or at the hospital? At home
Was it necessary to have a cut down below? Yes. Is it healing well? Yes
Did the baby have any complication? No. Did he need CPR or go to ICU? No.
Did you have any bleeding? No.
Were you both discharged together from hospital? Yes
Do you have any fever? No.
Did you have gestational diabetes? No
Are you breast feeding? Yes Any breast tenderness? No Any bleeding? No Any nipple fissure? No
Any vaginal discharge? No, only on the first week after delivery.
Are you back to your sexual life? Yes. Are you on any kind of contraception? Condom
Any pain during sexual intercourse? Yes, doctor. And it is very dry. But it is ok if I use lubricants. Any
bleeding? No.
Any edema of the legs? No
SADMA
Last HPV screening? (I don’t remember the answer exactly but I think she was due for a screening.)
( I skipped family history)

I excused and asked PEFE:


Everything was normal except for vaginal atrophy on speculum exam.
Urine dipstick and BSL were normal.
I thanked the examiner and returned to the patient:
“ Mary, according to your Hx and PE, you have a condition called atrophic vaginitis. It is a common
condition and it is not serious. It happens because the hormone that is responsible for the milk production
acts lowering the production of estrogen by the ovary. This comes to normal in a few weeks. Meanwhile
you can continue with the lubricants. If it doesn’t get any better we can start you on estrogen vaginal
cream.
As you are back to your sexual life, we should discuss the contraceptive options because you can only have
pills or other devices with progesterone…”
The bell rang.
I thanked both as I left.

122-Cyclical mastalgia
GP, 22 years old female with bilateral breast pain for 2 weeks or 2 months. Her mother was
diagnosed with breast cancer and currently under treatment.
Tasks:
-Relevant history
-Physical examination from the examiner
-Explain to the patient probable diagnosis and differential diagnoses

History
1-pain questions
I can see that you have pain on both breasts for about 2 weeks
-do you have pain now? How severe is it from 1-10? (Yes so ask allergy and offer painkillers)
-can you tell me more?
-Is it constant or does it come and go? Is it getting worse?
-can you point out exactly where you feel the pain?
-does the pain go anywhere else?
-does anything make it better or worse? (Worse in periods and relieve by wearing bra)
-how the pain related to periods? Does the pain come down when the period starts? (Pain worse
before periods and relieved after the periods)
-does it interfere with activities of daily life?

2-associated symptoms
-have you noticed any lumps in your breasts? (Yes)
-One or both breasts? (Both breasts)
-have you noticed any increase in the size?(Not sure)
-Any skin changes of the breasts?
-Any nipple discharge?
-Any lumps or bumps in your armpits or the neck?
-Have you noticed any weight loss?
-any trauma to the breasts?

3-5Ps and general


-period: when was your first menstrual period? Any problems with periods? (No)
-pill: do you use any contraceptive methods? What is the type? (Use OCP for 5 years)
-sexual history: are you sexually active? (Not sexually active)
-SAD and coffee
-PMH/ PSH/ medications and allergies
-Family history of breast or ovarian cancers (mother has breast cancer, had surgery, radio and
chemo, and she is worried that she has cancer)
when was your mother diagnosed with cancer? (at/before 50)
have you had any imaging before ?

Physical examination from the examiner (may be card but just in case)
Inspection (normal)
-any asymmetry, Scars, any skin changes (erythema), Puckering or dimpling of the skin
-nipples (retraction, distortion, ulceration, discharge)
Palpation
if I can feel any lump or lumps?
-Site and size
-Surface and border: smooth surface or not/ regular or irregular border
-Consistency and tenderness: if soft, firm or hard/ tender or not
-Mobility and fluctuation: are they fixed or mobile/ fluctuant or not
note/ in the exam there will be multiple small lumps with one dominant lump. Not tender and
all other normal.
Lymph nodes on neck or armpit + quick systemic examination

Explain
-from history and examination most likely you have a condition called cyclical mastalgia, which is
a pain in your breasts that is related to periods, usually due to hormonal changes during
menstruation.
-on examination, I can feel small lumps as well. Most likely, are due to fibrocystic disease or
fibroadenosis, which is a common and benign breast condition that can cause cyclical breast pain
and lumpy breasts.
-lump may also be a benign growth of breast tissues called fibroadenoma.
-Infection, pus collection or injury can also cause lumps in the breast but less likely in your case.
-Lumps may also be the result of fatty tissues accumulation called a lipoma.
-could be nasty growth but unlikely from history and examination.

Management
1-Triple tests as lumps always need to be evaluated
-send her for ultrasound
-refer to specialist may need FNAC to confirm diagnosis
-reassure cancer is unlikely
-self breast examination monthly + review yearly by GP doing breast examination
-later discuss about mammogram

2-painrelief
-Panadol or NSAIDS
-supportive bra
-evening primrose oil

3-review medications
-if on OCP (advise stop and change it in another consultation)

4-lifestyle modifications
-low fat, high fibre diet, exercise, weight reduction
-decrease coffee and alcohol intake
-avoid smoking

Case (1/3/2017)
Bilateral breast pain
( In this case there was another lump consistent with fibroadenoma so I send for us+fnac )
Cyclic mastalgia ok this case again young girl,27.
Pain on bilateral breast.
worse around period, relieve by wearing bra.
Mum has hx of breast cancer. I said sorry about that. is she feeling alright now? she said yes.
she is taking OCP for 5 years but she is not sexually active? I said why who prescribe it for you?
do you have any problem with period? she said no in case if start any relation.
okay the examiner handed paper of examination typical for cyclic mastalgia ( multiple fibrocystic
I think bilaterally ).
I said I know you are concerned about breast cancer lump in the breast could be cancer the
patient face sad or fibroadenoma but from the hx and examination most likely cyclic mastalgia.

But again it could be related to pill I want you to stope the pill, wear supporative bra, NSAIDs and
hot pad and see you again.
As I remember the patient on ocp but not sexually active so I advised her to stope the pill,
wearing supportive bra and take NSAD for pain and reassure her is it a normal condition. ( EPO,
Vit E, Bromocriptine ).
Feedback 20-2-2018
GP, 22 years old female with bilateral breast pain. Her mother was diagnosed with breast cancer
and currently under treatment.
Tasks:
-Relevant history
-Physical examination from the examiner
-Explain to the patient probable diagnosis and differential diagnoses

2 min thinking: worried patient needs reassurance. Usual gyne history taking (5Ps, PMHx, FHx,
SADMA)
History:
Introduce myself. Empathy with patient having pain on both breast, offered stronger pain killer
after asking allergic history.
Asked more about chief complaint (pain question: severity, type, aggravating/relieving factors,
worse/better, associated symptoms) – as far as I could remember, had breast pain since 2
months ago, pain is constant, more severe during menstruation, feels lumpy on both breasts. No
recent injury or signs of infection on the breasts.
5Ps: regular period, on Microgynon 30, forgot the answers for partner, pregnancy, PAP smear.
PMHx: unremarkable
FHx: mother, 50 years old, had breast cancer diagnosed recently, had mastectomy, ? LN excision,
on chemotherapy/tamoxifen?? She is unsure about BRCA status.
SADMA – unremarkable

Physical examinations:
Positive findings: multiple lumps on both breast, no skin changes, no nipple discharge, forgot
whether there is any tenderness on palpation

Explanation:
Dear Mandy, there are various reasons for a person to have breast pain. It could be due to
infection to the breasts, injury but they are unlikely in your case. Sometimes it is related to
female hormones which could be normal. As you are having breast pain for two months now
and it is quite disturbing for you, we can help you by giving stronger pain medication, some heat
pack to reduce the pain. I would like to reassure you that it is not cancer, but as you are having
family history of breast cancer and it increases the risks of you having breast cancer. We could do
some basic test, I would liaise with your mother’s physician, if she is having the BRCA test, a
genetic test then we might suggest for you have the test as well. Otherwise I would recommend
you to start breast screening at the age of 40. Assess patient’s understanding. Reading material.

(I do not think I do well in this case and my management might be wrong, please counter check
with John Murtagh and RACGP)

AMC Feedback – Breast pain: PASS

(Approach to patient/relative 4, History 5, Diagnosis/Differential diagnoses 3, Management


plan 4)
Feedback 11-12-2018
...27 year old lady comes complaining of pain in both her breasts.

TASKS-a.History (Not more than 3 mins)

b. PEFE( from examiner in form of card)]

c Dx, DDx

d. Mx

-Hx-introduce yourself(should be done in a good way as patient lookS very concerned)

Reassure her in the beginning(as from stem I know… it is cyclical mastalgia)


Ask pain question(SOCRATES)--patient says pain increases before her periods and is relieved
after.
Then ask questions about local (breast )- skin shanges, lump, nipple discharge
Ddx- always start from benign to malignant cause( fever,weight loss, night sweats, Loss of
appetite,) 5 P(TAKING OCP)
Review of systems- genitourinary system, GIT
Past medical hx- not significant
Family history-I asked any history of similar condition, she said none
SADMA.
Then I rephrase my question- any family history of breast cancer or any other cancer(need to
rule ovarian cancer , colon cancer as it can predispose someone to breast cancer)- at this time
she said mother got cancer and underwent treatment—then always show your concern for
mother..like I am sorry and how she is now? Then if she is doing good…then say that’s great.

Thanks to patient for history then turn to examiner(he gave me a card of physical findings)-PEFE
– multiple lumps in both breasts and one dominant one. All lumps are rubbery, mobile, not
fixated, not hard.(findings were inconsistent with cancer but consistant with fibrocystic
disease(cyclical mastalgia)
dx- always start like based on history and examination, first of alI, i would like to reassure you
that it is not cancer so please don’t worry..then explain cyclical mastalgia as diagnosis

ddx- abscess, fibroadenosis, mastitis , cancer(all less likely )

mgt- TRIPLE ASSESSMENT AS YOUR ARE CONCERNED AND POSTIVE FAMILY HX.

A- ANALGESICS
B- -GOOD SUPPORTIVE BRA
C- I STOPPED OCP
D- DANOZOL(D
E- -EVENING PRIMROSE OIL, VITAMIN E.
Passed- global score- 6, KEY STEPS- ALL
123-Perimenopause
AMC Case 7-7-2018, 14-3-2018 and 7-9-2018
47 year old woman with hot flushes, irregular periods, pap smear 3 months ago was normal
TASKS
-history
-most likely diagnosis and differentials to patient
-Send Investigations and Counsel

History
1-Open ended question
-I can see from the notes that you are complaining of hot flushes and irregular periods can you
tell me about it?

2-HRT indications Questions


Vasomotor symptoms:
--For how long have you had hot flushes? Is it constant or does it come and go? Is it getting
worse? (Her main problem was hot flushes, especially at night which. It started almost a year
back and was getting worst.)
-any heavy sweating at night?
Psychological:
-Are you having mood changes?
-Do you feel irritable, anxious or depressed?
-any sleep disturbances?
-have you lost weight recently? Any loss of appetite?
-do you have difficulty concentrating on things?
(She had become moody these days with some poor concentration, was interrupting her sleep
so she couldn’t sleep)
Somatic symptoms:
-any muscle aches and pains? Any bone or joint pain? (No)
Reproductive symptoms
-Any vaginal dryness, itchiness, discharge? (No)
Urinary symptoms
-Any burning or stinging while passing urine? Do you have to go to toilet more frequently? Do
you fell any lump down below? (No burning in pee and no prolapse)
history of osteoporosis
-any self or family history of fractures? (No)
-How does these symptoms interfere with your life and daily activities? (Her sleep was badly
affected so she felt tired through the day.)
3-5Ps questions
Periods
-when was your last menstrual period?
-How often do you get irregular periods?
-have you had any pain or heavy bleeding during menstruation?
-Any bleeding in between the periods?
(Her periods became shorter and lighter but no intermenstrual bleeding or pain during periods)
Partner/ sexual
-Are you sexually active?
-Have you ever been sexually active or not?
-any pain during intercourse?
(She was married, had no dyspareunia or bleeding after intercourse.)
Pregnancy
-Have you been pregnant before?
-how many pregnancies have you had?
-are you planning to become pregnant?
(Had children all deliveries were uneventful. not planning and her husband had a vasectomy.)
Pill
-do you use any contraceptive methods? (No)
Pap/ HPV
-is your pap/ HPV up to date?
-is your breast mammogram or ultrasound up to date?

4-Differential diagnosis questions


Thyroid
-any weather preference?
-any change in bowels habit?
-do you have shaky hands?
Infections
-any fever?
Malignancy/ Phaeochromocytoma
-any lumps or bumps around your body
anxiety disorders
-about any nightmares or bad memories from the past, but there was none.
-any stresses. at home or work?

5-Contraindications for HRT questions (None)


- Any undiagnosed vaginal bleed?
-Any history of clotting in your leg veins?
-any history of cancers (breast, womb or ovaries? Lumps or bumps?
-Any history of stroke
-Any recent heart problems like angina or heart attacks?
-Any history of active liver disease?
-Diabetes, uncontrolled blood pressure, high cholesterol?
-Any other medical or surgical conditions in the past?
6-Lifestyle
-healthy balanced diet (Her diet had enough Ca)
-regular exercise
-SAD (Not smoker)

Diagnosis and differentials


-There are several possibilities why you are experiencing these symptoms, most likely you have
what we call perimenopausal symptoms, means that you are most likely going through a natural
phenomenon or a perimenopausal phase of your life which is the periods before menopause
when your estrogen and hormone levels begin to drop. So because of these hormonal changes
in your body you are having hot flushes, mood changes, irregular periods.
-Other possibilities could be a thyroid problem, which is a butterfly shaped gland in neck but you
do not have weather preference or change in bowel habits, could be due to any infections but no
fever, could be due to anxiety or medications.

Investigations
So I would like to order few tests to know more about the diagnosis and rule out other
conditions
-Basic blood tests like FBC, ESR, CRP, BSL, lipid profile, UCE, LFT, TFT.
-I would like do a hormonal test to look for FSH and LH levels.
-ECG to make sure your heart is fine because the decline in female hormones can affect it to.

Treatment
-I would like to refer you to the specialist for further evaluation.
-we might also consider giving you a combined sequential hormone replacement therapy to
replace the hormones that become deficient.
-advise life style modifications.
Feedback 14-3-2018
47 yrs old lady with irregular menstrual cycle and hot flush.
Tasks
HX , send investigations and counsel
2 min thinking..
Menopause/ perimenopause,
so menopausal h/o including 4Bs especially osteoprosis, 2 Ps, mood, cardiovascular risk and Ca
Indications and Contraindications for HRT
R/O medical cause like thyroid, infection, pheochromocytoma
R/O psychiatric cause

History
1-Menopause questions
-After introduction, I asked her to tell me what exactly was happening.
-vasomotor sx: Her main problem was hot flushes, especially at night which. It started almost a
year back and was getting worst.
-psych symptoms: She had become moody these days with some poor concentration, was
interrupting her sleep so she couldn’t sleep
-somatic: No breast tenderness or bone pains, She had no joint pain.
-reproductive: No discharge or itching, or dry feeling in private area.
-urinary symptoms: No burning with pee. No prolapse.
-bowel habit
- I asked how it was affecting her life and she said her sleep was badly affected so she felt tired
through the day.
2-5Ps
-Her periods became shorter and lighter but no intermenstrual bleeding or pain during periods.
-She was married, had no dyspareunia or bleeding after intercourse.
-Had children (forgot the number) all deliveries were uneventful.
On my asking if they had any plans for any other children, she replied no and that’s why her
husband had a vasectomy.
-Wasn’t using any contraception.
3-HRT CI
-She was never diagnosed with heart problem or any Cancer nor was there any family history
suggestive of it.
- Not troubled with headaches, clot in veins, non smoker, no liver disease. (CI for HRT)
4-Lifestyle
-Her diet had enough Ca and had no FH of fractures.
-She had never used any medication, I asked about Tamoxifen and she denied.
-Never diagnosed with PCOS.
-She had no weight-loss or any change in her appetite.
5-DDX
Asked about any tremors or weather preference or any change in bowel motion?
There was none. R/O pheochromocytoma and infections. Regarding her sleep problem I asked
about any nightmares or bad memories from the past, but there was none.
No stresses. SADMA nothing positive.
Counselling
-I told her that she was going through a natural phenomenon where there were some hormonal
changes in her body and that her periods might stop sometime soon. She was, what we called as
Perimenopausal phase of her life and her body was responding to the changing hormonal levels.
-It could also be a problem due to a butterfly shaped gland in neck called thyroid.
-infections and anxiety
Investigations
So I would do few tests to confirm and rule out any complications because of it.
-I would do a hormonal test too look for FSH and LH levels.
-Also included FBE, ESR, CRP to r/o any infection and anemia.
-ECG to make sure your heart is fine because the decline in female hormones can affect it to.
-We could do a bone scan but a specialist would organise that so I would write a referral.
-And to R/O thyroid disease, I would do some TFTs.
I referred her to specialist who would consider her for HRT use.
And because her sleep was disturbed, I told her about ways and lifestyle changes to improve her
sleep. Bell rang!
Scenario.. Health Review
Grade… Pass
Global score… 4
Key steps 1,2,3,4… yes
History… 4
Diag/ D/D… 3
Management plan…. 4

Feedback 7-7-2017 FAIL


47 year ild woman with hot flushes, irregular periods, pap smear 3 months ago was normal
TASKS
1. History
2. most likely diagnosis and differentials to patient
3. management to patient
WHAT I DID
asked about durATION of hot flushes...since 4 months
mood changes...no
on ocp...no
sexually active...yes
dyspareunia...no
were periods regular before...yes....any problems with periods...no
when did your mother had her menopause....i dont know
any aches and pains...no
any weather preference....no
smoking...yes
no alcohol
no medications
Mary you might be experiencing early menopausal symptoms and these could also be caused by
changes in thyroid gland functioning or due to some problems with ovary or womb so we need
to do some investigations...but probably its fue to menopausal symptoms
So will arrange for some baselines like fbe, crp.
Some hormones like thyroid functions and femake hormones estrogen, lh, fsh
ONCE RESULTS ARE BACK,specialist will decide whether any treatment is required or not
IN THE MEANTIME ADOPT SOME LIFESTYLE MODIFICATIONS LIKE HEALTHY BALANCED DIET,
EXCERCISE, STOP SMOKING, LIMIT ALCOHOL INTAKE
THANK YOU MARY AND EXAMINER.
FAIL
keystep 1 2 3 4 NO
history 3
diagnosis 2
management plan 1

Note/ Why he fail because his diagnosis is wrong he said early menopause not perimenopause.
his management is poor and did not mention she is a candidate for HRT.

Feedback 14-3-2018
Amenorhoea in 47 years lady – with flushing – counsel and send investigations.
Started with history – 5 Ps – mood changes – regularity – medications..etc – it was an obvious
premenopausal symptoms – explained that to her and offered her HRT to minimize her
symptoms and told her that I would give her reading material ( was an easy straight forward
case ) . told her that we might need to send for FSH/LH/Prolactin/U/S.

Feedback 14-3-2018
47 yrs old lady with irregular menstrual cycle and hot flush HX only irregular period ,hot
flushes,no mood swings on no pills now but she was on OCP for 5 years and then stopped,no
weather preference no lumps and bumps no contraindications of HRT.
counsel the pt (anovulatory cycle,we have to exclude other causes ,referral to gyn,might
consider giving combined sequential hrt)
send investigation(hormonal study fsh lh estrogen progesterone thyroid function test )
Another modification of this case came in 10-5-2018
-the same but the presenting complaint is sleep disturbance and tiredness. So start with all sleep
questions like duration, difficulty getting to sleep, wake up early in morning and difficulty getting back to
sleep.
then continue the same with indications and contraindications and lifestyle.
in DDX you can ask tiredness ddx questions HEMIFAD will be good

Feedback 10-5-2018
Can you plz share your approach for primenopausal wome along with sleep problems
Her presenting complaint was sleep problem and tired during day so I asked detailed sleep problem
questions, then few causes for tiredness like diabetes thyroid any infections, then asked period History
sexual history and detailed history of menopausal symptoms.
I forgot the exact task... but I remember taking history and talking abt diagnosis and ddx and abt
management part also... sorry but I don’t remember the exact task

So in summary 2 cases of perimenopause the same age


*first chief complaint is hot flushes and irregular periods
*second chief complaint is sleeping problems and tiredness

Feedback 7-9-2018
Station 15- peri-menopausal symptoms – PASS

. Peri-menopausal Symptoms
In STEM: Hot flushes and difficult to sleep
47-year-old lady , comes in . have 2 children ,
Not complaint about abdominal pain ,
TASK
1. history ( very disturbing night time , can’t sleep , please help me to get rid of it , - patient request )
patient herself - -nil known migraine headache, breast cancer , active liver disease , abnormal bleeding
apart from dyspareunia with her partner , blood clot or calf muscle pain .
5Ps- LMP was few months ago , few and few over time.
Family history – not all with those conditions as well .
Nil family history of bowel cancer , osteoporosis ( protective factors )
2. Explain INVEStigations to the patient ( routine , hormonal studies ( explain in details slowly FSH/LH ,
estrogen , prolactin , ) 2 times apart to confirm menopausal stage.
3. Explain possible causes to the patient
( DDx – peri-menopausal symptoms – draw with a diagram of vagina , uterus , fallopian tubes and ovary ) –
told quickly about ovary function – estrogen – which is deficient in the patient )
- other metabolic – thyroid dysfunction , sugar problem like DM ,

Need to rule out – least likely – infections


3. explain mgmt plan to the patient
- for sexual performance – lubricant gel or estrogen cream
- will refer to the specialist and they might start HRT – as no contraindication to the patient
Before explanation about mammogram , the bell rang.

Feedback – Health review – PASS


Global score – 4 Key step 1 to 4 – all YES
History - 5 Invx and DDx – 4 Mgmt plan – 2
Feedback 7-9-2018 (perimenopause) PRE-MENOPAUSAL SYMPTOMS- FAIL
47 year age old lady having hot flushes and mood changes, difficulty to sleep(I'm not sure about
this complaint??) Her BP -120/80, all other vitals normal. She has two kids. You have done pap
smear , result - normal. few other invx were also mentioned normal.
tasks are-
Hx
Ddx
Invx
MX
FEEDBACK: Fail. HX 4, Dx/ddx: 2 , Mx: 2
This was my second station. I can't remember now but definitely I missed ddx or couldn't
explain properly. Please check the mx as well as I can see from another feedback that one
candidate from same date got 2 in mx in this case. I'm going to explain my thought process
during the exam. To be honest, I think I couldn't deliver in that 8 mints what I thought in 2 mints.
Maybe, I was anxious, that's why I missed points!!! However, for your convenience, I put all the
ddx of this case.
2mints thinking-
5P- pregnancy, pap smear given. Have to ask details about period, contraception, dyspareunia ,
4B(Bladder, bowel, bone, breast) 2P( prolapse, incontinence, other peri-menopausal symptoms).
SADMA important. Plus need to ask HRT contraindications. Finally some general measurements.
DDX:
1. peri-menopausal symptoms or already reached menopause??
2. Hyperthyroidism
3. Pheochromocytoma
4. Generalised anxiety disorder
5. life-style stress
6. high caffeine intake
There was an thin old lady sitting on the chair. Confidentiality (could be a psychiatry case). Ask
her about hot flushes in details, mood problem, sleep problem, 4B, 2P , ddx questions,
SADMA(nothing positive). Everything was normal except period was scanty , dyspareunia and
she is not using any contraception. No stress as well. I forgot to ask HRT contraindication
questions in HX time.
I started to explain her ; most probably due to peri-menopausal symptoms because of low
female hormones in your body and you are close to menopausal age. I'll do few invx to rule out
other causes as well.
- blood tests- TFT, hormone levels, LFT, RFT
-urine tests
- pap smear already done
- trans-vaginal usg to look for width of uterus
(I guess I explained ddx while I was talking about invx. As far as I remember, Invx was part of the
task but it's not mentioned in the feedback)
Then suddenly it came to my mind that I forgot to ask about HRT contraindications. I asked
those questions.(no hypertension, BP was given normal, no migraine, heart problem, liver
problem, clots, no fx of cancers) everything was normal. I said I would do few tests for HRT
before referring her to specialist. Then, I explained general measurement, physical exercise,
yoga, meditation, sleep hygiene. Reading materials regarding post menopausal symptoms. Plus
oestrogen creè me for dyspareunia.
I had enough time in this case. I asked her to repeat what I said. I was also thinking in my mind I
guess I had missed something!!
124-Myocardial infarction
This is just a collection. For approach refer to lecture 12 pericarditis
Feedback (11/11/2017)
55 yr old man presents to ED with chest pain where you are the HMO.
Tasks:
-Take history
-Ask examiner for examination finding card and ECG. Interpret ECG , explain findings to examiner

-Explain to pt about the likely causes for his chest pain

Introduction
I asked examiner for vitals- He said the pt is stable
History: Started with pain questions (SORTSARA)
Scale 5/10
Central
Chest tightness
No aggravating or relieving
Never experienced similar pains
Associated with sweats, vomiting and pallor.
No h/o angina
No cough, SOB, abd pain, no change in bowel or bladder
Risk factors- Diabetes, Hypertension, Hypercholesterolemia, past h/o smoking
I don’t remember family history
Asked for his medications- He removed a list from his pocket
Briefly asked about his home situation

Examination card: Vitals stable,CVS- normal, Basal reps on chest

ECG: Sinus rhythm, HR 100, normal axis, ST elevation I, aVL, V1-V6 with poor progression of R
waves suggestive of Acute anterolateral myocardial infarction

I explained to patient that based on my findings I believe that he has a serious problem,
checked his response and then said that he has a heart attack and that he will be looked after
well by an experienced team of skilled doctors. He had a lot of expressions but felt relieved that
he will be looked after well. I explained that he has typical cardiac chest pain and with the
background of DM, HTN , lipids, smoking its unlikely to be other causes- Clots in lungs,
inflammation of the covering of the heart, infection of the lungs, ulcers in tummy, pancreatitis.

I had time remaining and this was my first station, certainly dint want leave early( desperate to
pass) so I asked pt to tell me what he has understood so far. He repeated everything very well
and the bell rang.

Key steps 1,2,3,4 – Yes Global score : 6 ( Pass )


Assessment domain Approach to pt/relative: 6
History: 6 Interpretation of investigations: 5 Diagnosis/ differential diagnosis: 6
Feedback 12-6-2018
A 47-year-old man in ED. He has chest pain. His ECG was provided in the stem. There was ST
elevation in anterolateral leads. Vital signs were given in the stem. Blood pressure was 100/70
and the rest were normal.
Task
-history taking,
- explaining the ECG to the patient
-talk about management. (I am not sure about management.)

Chest Pain: Pass: Global score: 5

Entered the room, introduced myself. Asked the patient bout his pain and the severity. Then told
him that” I think you need some pain killer, as I can see you are in pain. What do you think?” he
agreed. I asked the examiner to move the patient to resuscitation cubicle, take an IV line, give
morphine and Aspirin. Also said that as the BP is borderline we might be able to give Nitro-
glycerine, His O2 saturation was good, so I did not mention O2 supplement.

The started asking the patient about characteristics of pain (duration, severity, location,
radiation, quality, previous episodes like this or milder, the explained that his pain characteristic
looks like a typical ischemic cardiac pain, but to be on the safe side I would like to ask a few more
questions. Whether the pain was increased with respiration, if any cough was present, also
asked about calf pain or history of recent long flight. Then asked about past medical history,
Diabetes, hypertension, hyperlipidaemia. He said he has never had, then I asked has he ever
checked for these and the results were negative, or he has not checked. So he said that he has
never been assessed or screened. Also asked about smoking, and alcohol. And also about family
history, especially heart attack at an early age in the parents, he said he was adopted.

So explained the ECG to him, explained the ischemia or lack of oxygen to the heart muscles.
Explained that I could not confirm whether the heart muscle is damaged due to lack of oxygen or
not (differentiating between Unstable angina and ST elevation MI), and I need more investigation
(I did not mention Troponin) but this ischemia can happen due to a long term obstruction of the
vessels due to diabetes or hypertension or hyperlipidaemia, or can be due to a bleeding and
haematoma inside the vessels which as an acute reason.

Therefore, we need to start him on medications that can dilate the vessels in short term, and
some blood thinning agents. Also I need to involve the Cardiology Registrar and they will
possibly take him for PCI which is a procedure that can visualize the vessels and enables the
cardiologist to get rid of the obstruction inside the vessel. And the outcome is usually very
good. After getting rid of this acute emergent pain, we need to find the underlying reason for
this obstruction. So he will need a complete workup for cardiac risk assessment.
Feedback 25-10-2018
Scenario: chest pain
Stem. Was as old recall. Middle aged lady with chest pain in ed.
Tasks: hx, ecg interpretation, dx/ddx.

So I entered greeted the pt who was sitting on the bed putting hand on mid of chest n showing
pain. I asked pain scale and offered pain killer. (I was standing through out this station n so was
the examiner just besides me while the pt sitting on the bed)
After pain killer I asked pain SOCRATES. She had central chest pain radiating to jaw. She had
some previous episodes of pain but this time it was worst than ever before n not resolving for
last 30 min.
Then I asked ddx qs for pericarditis, aortic dissection, pneumonia, pul. Embolism, pneumothorax,
gerd, psychogenic.
Past hx family hx sadma (heavy smoker quit smoking 3 or 4 yrs back)
Medicine hx she handed me a paper, showing 4 meds daily.
Then I asked examiner for the ecg and interpreted it to him.
Then explained MI to patient that there is some blockage of one of the vessel supplying to your
heart muscle and that’s why you are aving this pain something we call myocardial infarct, in
common words HEART ATTACK she gave a surprised/scared expression. I told her no don’t worry,
relax u r in the hosp we will take care of you now, take a deep breath don’t worry n she was fine
then. Then I took the pen n paper n drew and explained all other ddx asked her inbetween about
the pain she smiled and said no im much better after the pain killer u gave me doc. Bell rang.
Grade: pass
GS: 4
3 out of 4 keysteps covered
Approach to pt: 5
Hx : 6
Interpretation of inx: 4 (I didn’t do good in it, I didn’t tell rate rhythm. I started with name n time
and straight to p wave and so on, I realised this when I came out of the room)
Dx/ddx: 6

Feedback 25-10-2018
Ed, man with chest pain came.
Task:
History
Ecg interpretation to examiner
Diag/dd
It was a straightforward case. Asked hemodynamics, took history sortsara for chest pain, asked dd
questions for PE, Peptic ulcer, prev hx of heart attack, palpitation, medication( he gave me a slip)
Smoking, fam hx., blood loss hx
Ecg showed: anterolateral mi, with 90 pulse.
Told pt tht its heart attack( with empathy), but could be angina, peptic ulcer, musculoskeltal,
Pulmonary embolism
Feedback 5-4-2018
Chest pain: Your next patient in the emergency department is a 62 year old woman presenting
with central chest pain which started about 4 hours ago. He is anxious and sweaty.
Task:
-Take a further history for 4min pain om
-PEFE (a cad w findings were given)
-Ask for an ECG from examiner and explains to the examiner
-Discuss the diagnosis and ddx with the patient

Copied the cases from previous recalls but it was exactly like this scenario.
Introduction�I asked examiner for vitals- He said the pt is stable
History: Started with pain questions (SORTSARA) Scale 5/10�Central�Chest tightness playing
golf
No aggravating or relieving�Never experienced similar pains�Associated with sweats, vomiting
pallor bit of SOB�No h/o angina�No cough, abd pain, no change in bowel or bladder�Risk
factors- Diabetes, Hypertension, Hypercholesterolemia, past h/o smoking �I don’t remember
family history�Asked for his medications- He removed a list from his pocket�Briefly asked
about his home situation

Examination card: Vitals stable,CVS- normal, Basal reps on chest

ECG: Sinus rhythm, HR 100, normal axis, ST elevation I, aVL, V1-V6 with poor progression of R
waves suggestive of Acute anterolateral myocardial infarction

I explained to patient that based on my findings I believe that he has a serious problem, checked
his response and then said that he has a heart attack and that he will be looked after well by an
experienced team of skilled doctors. He had a lot of expressions but felt relieved that he will be
looked after well. I explained that he has typical cardiac chest pain and with the background of
DM, HTN , lipids, smoking its unlikely to be other causes- Clots in lungs, inflammation of the
covering of the heart, infection of the lungs, ulcers in tummy, pancreatitis.

Recall 5-4-2018
MI
ED, 56 year old pt c/o chest pain.
Task: history taking
Examiner will give you PE card.
Ask ECG from examiner & explain it to examiner.
Dx & Ddx to pt with reasons.

Recall 11-4-2018
pt with epigastric pain pale sweaty.
Task hx frim pt , ivg from examiner ECG showed inferoseptal MI
Explain most likely dgx & ddgx with reasons
Case (9/6/2017)
acute anterior MI with ECG . ST elevation on v3 and v4
Task: hx, examination findings from examiner
Hx of DM, HT on prendopril , statin for hypercholestromia
Chest pain radiate to neck the pain started while he is plying golf 2 hrs a go
Pepe bp 150/90, bilateral basal crackle (HF)
Ddx MI, pulmonary oedema, pneumonia,pericarditis

Case (9/2/2017)
MI on ECG, crepitation present • Ask PEFE and examiner gave sheet • Ask for ECG & interpretate
it- Anterolateral ST elevation • Explain DDx & reason • No management

Case (7/9/2017) silent MI


Acute MI - 60 yrs in Mall staarted feeling unwell - went to GP.
Hx tell what happened to you. No chest pain or SOB. No nausea and vomiting. PEFE BSL 7.5, ECG
- explained to examiner. HR 50-60 with P waves and T inversions V3-V5 shows ischemia.
Explained to patient.

Pt had diabetes well controlled

Note/
-first check stability and even if they said stable move the patient to cubicle and attach him to
monitor then give morphine and aspirin. (sometimes the examiner will tell you he is already in
the cubicle and attached)
-ask all pain questions + ass. Symptoms+ risk factors
-PEFE will be card
-EXPLAIN ecg either anterolateral or inferior
-management is to admit and call the cardiologist. Do blood tests and troponin and say the
cardiologist will decide to give him blood thinner meds and send to lab for PCI and etc..
Feedback 12-2018
Station 4: Chest Pain - PASS
HMO at ED of tertiary hospital; 30 something male patient complaining of chest pain. ECG was given
showing anterolateral MI. All vitals signs are given and good.
Task: History (2mins)
Explain ECG
Possible cause for this condition
Immediate management plan
2 mins thinking: symptom analysis + risk factors (ABCDEFS); management plan – MONA plus PCI
History
As soon as I went in, the patient started shouting. So, after greeting the examiner, introduced the patient
and told him that we have done some tests to find the cause for your chest pain. Before I explain this, I
would like to do some things first to relieve your pain so please excuse me a moment. Then turned to
examiner and said “I’d like to shift my patinet to resuscitation room, give MONA”. Before I could complete
my sentence, examiner said just do your task. “Thank you” and then asked about the pain in details –
duration (1 hour), onset (sudden), 1st time (to which the patient said no, I had previous bouts of chest pain
but none of them were as bad as this one), better or worse (not sure), severity (7 out of 10), associated
symptoms such as sweating and palpitation (yes). Then asked risk factors – BP, cholesterol, DM, family
history. I totally forgot anaemia/bleeding and smoking as I slightly panicked as the patient kept on
shouting continuously. (bell rang)
Explanation of ECG
Just showed the ECG card to patient and pointed out ST elevation (initially, I thought about drawing and
explaining but the patient didn’t open his eyes throughout the 8 mins so I just gave up the idea, but
pointing out ST elevation was needless anyway)
Possible causes
What you are having is heart attack, due to narrowed arteries that supply the heart. (didn’t say AMI).
Previously, u have been having less severe chest pain which might be warning signs but now the pain has
got more intense and maybe the arteries have beee completely or severly blocked. And there are some
factors that can lead to narrowed artery of the heart. High calorie diet and fatty meals, DM, raised fat
level, high BP, stress (missed smoking again). “So far, I cannot find the cause yet, but I will make sure we do
everything including tests and examination to find it out and treat it so that you wont have another attack
in future.
Mx plan
I can see this is so painful for you, but I really appreciate that you are now in the ED within 1 hour since
the chest pain began, so if we start the treatment right away, the outcome is excellent. I will inform the
specialist first and attach some leads on you to continuous monitoring. For your pain, we will give you
painkiller as strong as morphine. Oxygen together with medications to open the arteries will be given –
called aspirin and nitrates. Most importantly, the specialist with proceed with a procedure called PCI. This
involves inserting a deflated balloon from the artery around your groin. This balloon will travel up to your
chest and when it reaches the narrowed artery, it will be inflated to widen the artery. All of this will be
done under imaging scan. Understand?
(finished all of the tasks 1 or 2 mins ahead of the alloted time, so just spent the rest of the mins summing
up the condition, mx, reassuring, and blah blah blah…)
GS – 5
KEY STEPS 1, 2, 3, 4, 5 – all covered
Approach – 5 History – 3
Interpretation – 4 Dx and DDx – 5 Mx plan – 6
(TBH, I thought I would fail this one as I couldn’t find out the cause for the AMI and interpretation was a
bit off as the patient didn’t even open his eyes throughout 8 mins. Should have asked medications used
during the previous attack, smoking, and bleeding history)
Handbook 65
Acute chest pain in a 60-year-old man

CANDIDATE INFORMATION AND TASKS

You are working in a hospital Emergency Department. You are asked to see a 60-year-old man
complaining of acute chest pain.

YOUR TASKS ARE TO:

• Take a concise, relevant and focused history.

• Present a summary of the patient's history for the examiner, who will then give you the
findings on physical examination which you request.

• Tell the examiner your provisional diagnosis and the reasons for this.

• Interpret the ECG to the examiner (the ECG will be given to you at about 7 minutes into this
consultation).

• Institute emergency treatment.

AIMS OF STATION

To assess the candidate's ability to take a medical history in an older male patient presenting to
the Emergency Department with chest pain of two hours duration. The candidate needs to be
aware of the potential seriousness of the situation, the importance of taking a focused history to
distinguish between cardiac and non-cardiac sources of chest pain, whilst being aware of the
patient's discomfort and the need to take steps to relieve this. As in clinical practice, the early
performance of an ECG and its correct interpretation is a key step in the assessment of this
patient.

EXAMINER INSTRUCTIONS

The examiner will have instructed the patient as follows:

You have acute and worsening chest pain. You are 60 years of age. Opening statement

'I have a very bad tightness in my chest.'

Characterisation of symptom:
Site: central, retrosternal, radiating to lower jaw.
Severity: severe 8/10, Time course: Context: came on two hours ago, steadily getting worse
recent angina for two months; this pain started when playing third set of tennis

Aggravating factors: none, no association with respiration


Relieving factors: anginine, oxygen when given in Emergency Department
Associated symptoms: sweating, nausea, breathlessness
Other health problems: overweight, diabetes (5 years); hypertension (3 years): high cholesterol
(3 years); pain in left leg on walking 500 metres (1 year).
Systems review: central chest pain when walking on cold mornings for the past two months:
short of breath on exertion and breathless at night (three days); tiredness (two days). No
epigastric pain, oesophageal reflux or dysphagia. Recent black bowel motions (five days). If no
questions about your bowels, volunteer this information.

Past history: Myocardial infarct three years ago


Drugs: For blood pressure (enalapril/hydrochlorothiazide); diabetes (metformin); aspirin; lipid-
lowering agents
Habits: Smoker until three years ago; alcohol intake 1 glass wine per day.
Family History: Nil relevant

The examiner will provide physical examination findings to the candidate as follows:

He is an overweight man of stated age who is in acute distress with pain and who is anxious
and sweating. Blood pressure is 150/96 mmHg, pulse rate 96/min and regular. Heart sounds
dual rhythm, no murmur. There are no signs of heart failure. Examination otherwise
noncontributory

Approach to patient

The candidate is expected to demonstrate professionalism, empathy and to seek relief of the
patient's discomfort with use of oxygen whilst taking the history. Awareness of the potential
seriousness of the situation as the history evolves still requires the candidate to be calm,
confident and reassuring.

History-taking skills

The candidate is expected to fully characterise the chest discomfort, its time course, the context
and associated symptoms. This needs to be done in a sensitive and focused way. using a mixture
of open-ended and direct questioning. The cardiovascular risk factors must be determined,
including the past history of myocardial infarction. The occurrence of this pain in the context of
recent chest pain on exertion and breathlessness needs to be defined.

If the history of recent 'black bowel motions' is not obtained by the candidate, the patient has
been asked to bring this to the candidate's attention.

What should the doctor be thinking?


Meeting the patient: an overweight, anxious-looking, sweaty older man with chest pain: urgent
assessment needed: focus on key questions relating to possibility of ischaemic heart disease.The
presenting problem: fits pattern of acute myocardial infarction.
Check out his medication list: indicates he is diabetic, hypertensive, high cholesterol Other
cardiovascular risk factors: previous acute myocardial infarction; claudication: smoker.
Other medical problems: Type 2 diabetes; hypertension; central obesity; hypercholesterolaemia;
probable melaena.
Physical examination: fits pattern of acute myocardial infarction
Treatment starts immediately: This is a medical emergency requiring management
by an expert team (what is the candidate's role in the team?); commence oxygen therapy,
aspirin, glyceryl trinitrate and morphine; monitor pulse, blood pressure. ECG; assess for
thrombolytic therapy.

Ability to provide a concise clinical summary

This should be along the following lines and reflect the manner in which a junior doctor would
describe the key features of the history to a registrar or consultant.

The patient is an overweight, anxious looking, sweaty 60-year-old man with chest pain
described as 'a very bad tightness', 8/10 severity, in the central, retrosternal region, radiating
to the lower jaw but not to the arms. The pain came on when playing tennis and has been
increasing steadily over the past 2 hours, associated with shortness of breath, sweating and
partly relieved by anginine and oxygen. In addition he has had exertional chest pain over the
past 2 months and shortness of breath, orthopnoea and tiredness over the past few days. A
concerning symptom is his 5 day history of passing black bowel motions, which is suggestive of
gastrointestinal blood loss.

He is at very high risk of acute coronary ischaemia, having had a prior myocardial infarct, and
with risk factors of diabetes, hypertension, high cholesterol. EC G findings need to be checked
and anaemia considered as a precipitating factor.

Diagnosis

The most likely diagnosis is acute myocardial infarction. The key features that suggest this
diagnosis are the characteristics of the chest discomfort in a patient with significant risk
factors and prior myocardial infarction.

Interpretation of ECG

The ECG shows the following features:

• Sinus rhythm, rate 96/min.

• There are features of acute inferior myocardial infarction shown by the Q waves in leads II,
III, AVF and ST segment elevation in these leads as well.

Tests: confirm acute myocardial infarction; assess anaemia

KEY ISSUES

• Approach to patient — sensitivity to the patient's discomfort and a calm and professional
manner.

• The ability to take an appropriate and focused medical history showing an awareness of the
likely causes of chest pain and the main characteristics that distinguish cardiac and noncardiac
sources of chest pain. The candidate needs to show an appreciation of cardiovascular risk
factors, and an efficient ability to characterise associated symptoms and to define the context in
which the symptom of chest pain has arisen.

• Commentary to examiner — a succinct summary which brings together the key features of the
presenting complaint, the context in which it has arisen, the associated symptoms and the
cardiovascular risk factors. The candidate should identify the potential significance of the history
of melaena.

• Diagnosis/Differential diagnosis — the candidate must consider the diagnosis of acute


myocardial infarction and why noncardiac causes of the chest pain are less likely.

• Interpretation of investigation — the most important findings on the 12-lead ECG must be
defined: sinus rhythm, acute inferior myocardial infarction.

CRITICAL ERRORS

• Failure to consider the diagnosis of acute myocardial infarction on the history.

• Failure to define the main cardiovascular risk factors — prior myocardial infarction. Type 2

diabetes mellitus, hypertension and hypercholesterolaemia, smoking.

• Failure to correctly interpret the ECG features of myocardial infarction

Acute anterolateral myocardial infarction

Features indicating acute anterolateral infarction are:

• ST elevation in leads I, aVL, V2-V6; and

• Q waves in aVL, V2, V3 and loss of R waves across chest leads.

Acute inferior myocardial infarction

Features of acute inferior myocardial ischaemia/infarction are:

• ST segment elevation in II, III, and aVL; and

• The slow rate is also common in this condition.

Acute posterior-inferior myocardial infarction

Features of posterior-inferior myocardial infarction are:


• Q wave and ST elevation in inferior leads (II, III, aVF); and

• the prominent R waves in V1 (labelled C1) and Q waves with ST elevation in

V5, V6 indicate postero-lateral infarction.

Central chest discomfort is a common presentation of cardiac disease, but it may also be due to
disease of the gastrointestinal tract, lungs or a musculoskeletal disorder. The features of the
chest discomfort/pain, the context in which the symptom occurs, the associated symptoms and
the patient's predisposition to cardiac versus noncardiac disease based on an assessment of
cardiovascular risk factors, must all be considered.

In taking a history relating to chest discomfort, a number of key descriptors needs to be defined
to determine if its origin is cardiac ischaemia. A common sequence of enquiry would be as
follows:

• 'What is the discomfort like? Describe it in your own words. '

• 'How severe is it — e.g. a score out of 10?'

• 'Show me where you feel it? Does it go anywhere else — the abdomen, the back, the neck,

the jaw, the arms?'

• 'When did it start? How has it progressed? How long has it been present or how long did it

last?'

• 'Does anything make the discomfort worse? Does anything make it better?'

• 'Do you have any other symptoms? Shortness of breath? Dizziness? Palpitations?

Sweating? Nausea or vomiting?'

In addition there are a number of questions that will be used in trying to determine if there is a
non-cardiac cause:

• 'Do you get acid indigestion or reflux?'

• 'Was the onset of the discomfort related to a meal?'

• 'Does it hurt to take a deep breath?'

• 'Is the chest sore to touch?'

Karin case
Acute Myocardial Infarction/Cardiac Arrest
Case: You are an intern in ED and are about to see Mr. Smith who is a 55-year-old man brought
by his wife because of severe central chest pain. He is also breathless and sweaty.
Task
a. History
b. Provisional diagnosis and differential diagnosis from examiner
c. Manage patient’s condition
d. Answer examiner’s question

Case 2: You are in ED where you’re working as an HMO. A 50-year-old man comes complaining of
sudden onset of chest pain 1 hour ago.
Task
a. History (ECG: VF)
b. Manage the patient

- Is my patient hemodynamically stable enough to proceed?


- I would like to put my patient in a resuscitation cubicle. Before starting my history I would
like to check his oxygen saturation, hook him to high-flow oxygen (8L) and cardiac monitor and
I will ask the nurse to do ECG while I take my history.
Is he in severe pain?
- I would like to offer the patient morphine (2.5-5mg every 4 hours) and metoclopramide.

History
- You are in experienced hands. I am here to help you.
- Do you have any gastric bleeding, gastritis or allergy to aspirin? If not, then give aspirin 300
mg.
Are you ready to talk?

- SORTSARA: Where exactly is the pain? When did it start? How severe is it on a scale of 1-10? Is
the pain constant or does it come and go? Does the pain travel anywhere? Can you describe to
me the character of pain? Squeezing, stabbing or constricting? Are you aware of any factors that
relieve or aggravate the pain like when stooping forward or aggravated by deep breathing? Have
you had this pain before?
- What were you doing before the onset of chest pain? Do you have associated symptoms like
SOB, diaphoresis, palpitations, cough, leg swelling, stress, fever, trauma, heartburn or
indigestion, soreness of the chest?
-Any problems with bowel motions (r/o hematochezia and anemia) or waterworks?
- CV Assessment Risk: Smoking, DM, hypertension, hyperlipidemia, stress and sedentary lifestyle,
FHx,
- Past Medical history that I need to know such as previous MI, PVD, cardiac disease? Any history
of bleeding disorders, previous stroke, surgery or head injury, or any medications such as ASA or
warfarin or viagra?

Provisional diagnosis and Differential Diagnosis


- The most likely diagnosis is AMI. I have built my assumption upon the description of the pain
which indicates a cardiac origin. It is a central chest pain, squeezing in character, 8/10 in severity,
lasting for about an hour, radiating to the shoulder and jaw and associated with diaphoresis.
DDX
- My other differential diagnoses are other cardiac disease such as ACS, aortic dissection and
pericarditis.
- Others include respiratory causes such as pneumothorax, Pleural effusion, and pneumonia.
- Less likely are GI causes such as esophagitis, gastric reflux, esophageal spasm and PUD.
- Musculoskeletal causes include herpes zoster, costochondritis, and fractured rib.
- Psychiatric condition such as anxiety and panic disorder.

Drugs:
- Oxygen 8L (4-6L)
- Asprin 300mg
- GTN (Anginine)  sublingual (300mcg or ½ tablet) or spray repeat every 5 minutes x 3
doses systolic BP should be >100 and monitor BP
- Morphine sulphate (2.5mg every 10 minutes up to 10 mg in the first hour) with
metoclopramide 10mg (Maxolon) OR 2-5mg at 1mg/minute max 15mg

Management

- Blood will be sent for cardiac enzymes, FBE, and other baseline workups, CXR and possible
echo.
- I will call registrar or cardiologist and send him to cath lab ASAP for reperfusion therapy
Percutaneous Coronary Intervention (angioplasty) or fibrinolysis/thrombolysis (recombinant
Tissue Plasminogen Activator [rTPA] – alteplase) or CABG

- Indication: new-onset chest pain and persistent ST-segment elevation of >1mm in 2 contiguous
limb leads or > 2mm in 2 contiguous chest leads or new LBBB

- PCI is treatment of choice. It provides a good outcome especially if done by experienced


hands. Maximum acceptable delay is 60 minutes within 1 hour of symptom onset and 90
minutes if patient presents longer

125-Melanoma Breaking bad news


A 42 y/o IT specialist has came back to discuss about the result of a newly biopsied neck skin
lesion that is clark 2 superficial spreading melanoma. A picture was given outside.
Tasks
-Explain the investigation to patient.
-counsel the mx plan
-explain the risk of possible treatment you suggested.

A Pass approach
1-Approach
-introduce yourself
-yes ---, I have the result with me now. How are you doing these days
-any pain at biopsy site?

2-Breaking bad news:


*Knowledge about the test
-Do you know why the biopsy has been done for you?
*Knowledge about the disease
-have you been told about your medical situation so far?
*How much information they want
how much do you want to know about the results?
*Breaking bad news
- now I am going to explain about the result
-The biopsy was done and I am afraid to tell you the result comes back as skin cancer.

*If there is reaction do like this:


-the patient will start crying so offer box of tissues and glass of water if available, wait for her to
settle down.
-would you like me to continue the consultation today?
*If there is no reaction say like this:
-how are you feeling on hearing this result? You can tell me what’s going through in your mind?
Are you worried? (Yes I am worried)
*offer a call for someone:
-would you like me to call anyone for you? (No)
*showing empathy
- I do understand that this comes as a shock to you. I can see that this is an upsetting news for
you and also very difficult situation to face.
*Showing support to reassure she is not alone
-But we can do a lot of things. You don’t have to go through this alone, if this is in early stage and
if we start treatment as early as possible it can have a good outcome.

*ask if she is ready


are you ready to discuss a treatment plan now, or should I arrange another appointment with
you?

3-Talk about the disease


so the type of skin cancer you have is malignant melanoma. This is one of the commonest types
of skin cancers. There are factors that can increase the risk of having this e.g past hx of malignant
melanoma, presence of many moles, fhx, sun burn, tanning treatments.
In your case, the thickness is 0.6 mm. the thickness is important to determine the outcome and
as it is less than (0.76 or 0.75 not sure), then it can have a good outcome with treatment if it is in
early stages. It is superficial spreading, Clerk 2 which means …..

4-Management
you will be managed by MDT
-you will be taken up by a MDT including surgeon and cancer specialist, specialist nurse, support
groups and me as the Gp.
-the specialist will decide to do further assessment for staging , like blood tests and CT scan
and other imaging e.g CXR to your body to make sure it’s not spread.
-the surgeon will decide upon the surgery which most likely to be safe margin excision (wide
local excision of skin surrounding the already excised area and mostly of about 1cm. However,
this will be decided by the specialist. Further treatments will also be decided by the specialist.
- Follow up (usually blood tests and imaging)
After treatment , regular frequent follow up is required with blood tests, regular blood tests and
imaging.

-Advice on sun protection (put sunscreen, long sleeves, wide brimmed hat, wear sunglasses, try
to avoid sun exposure at its peak hours)

5-Ending
we are here to help you. You are not alone. Do you have family member, would you like to bring
them in next appointment, so that they can understand your condition and can give support.
When you go home if you have any concern, you can contact me anytime. If you feel stressed
about this condition, i can refer you to psychologist as well. There are a lot of support gp
available for you.

Feedback 19-7-2018 PASS(Key steps all yes score6,7,7,6,5)


Skin lesion ( Melanoma 0.6 mm ) very long scenario with a lot of details ,bx done and now
patient is here for result.
Task explain investigations and further MX
I Started with breaking the bad news SPIKE approach , explain the result , melanoma and
significance of 0.6 mm, asked few questions in the middle about symptoms of spread and then
told the management as in Karen.

Feedback9-5-2018 Skin Lesion Melanoma biopsy report


A punch biopsy have been done on a mole and you got the report: Melanoma. Barlow 0.6mm.
Clarke II. Superficial spreading with ulceration. Many more details I can remember.
Task
Explain the report of the biopsy to the patient
Further management to the patient
No further history
I thought I failed this case but I didn´t. I managed this as a case of braking bad news. however I
got very confused with my explanation as I didn´t practice this case before. Don´t want to
mislead you even though I passed better find the case from someone who was confident in their
approach.
Passed. Global score 4
melanoma "punch"biopsied, 0.6mm beslow, task, explain result, further management
Outside- This is cancer case, all cancer cases i will talk about SPIKES, risk factor, grade, pronosis,
complications, treament, investigations, prevention, most important is follow up.Inside- I think
this is easy case and many candida wrote about it so i should not tell it then. But the thing is the
patient is scare when they heard about cancer, i told her that it is just stage 1 and localize only.
Treatment is symple but follow up and prevent are most inportant. i said that is is good when we
figure it out early. I think i finish this cases early and talk again and again just make patient
comfort.

Feedback 5-10-2018
A 42 y/o IT specialist has came back to discuss about the result of a newly biopsied neck skin
lesion that is clark 2 superficial spreading melanoma. A picture was given outside as usual inside
also. Your task is:
 Explain the result of the biopsy
 Tell the management to the patient
This was my worst station. 1st mistake I did was I called him once in wrong name during the
discussion though I quickly corrected it. 2nd thing is I totally forgot to talk about sun exposure
and sunscream, long sleeved dress, hat etc etc. But I managed to pass this case somehow.
Feedback: Passed Key step 1,2,3: All Yes Approach to pt: 5 Interpretation of inv: 5
Management plan: 4 Explanation of procedure: 4 Global score: 4

Feedback 5-10-2018 A FAIL approach


Question: It’s frequently asked melanoma case (Clark staging 2 and size is 0.6mm, radical spread
and lesion is on the neck)
Tasks: Explain the biopsy findings to the patient. Tell the future management plan.
Score: I failed this case with global score 3 (I even expected to get good score as I did pretty well
and practiced this case pretty hard)
Key steps 1, 2, 3: All YES Approach to patient: 3 Interpretation of investigation: 2
Management plan: 3 Explanation of procedure: 4

My performance:
Hi.. Mary… Nice to meet you again. I’m Dr MM, one of the doctors in this clinic. I knew that you
came here for your biopsy result. But I’m sorry that I don’t have good news for you today. Is
there anyone, family members or friends coming along with you today? Do you want someone
to be with you during my consultation? Is there anything you already knew about possible
causes of your skin lesion? (patient replied NO to all these questions)
Ok, Mary, I’m afraid to tell you that your skin biopsy result showed nasty skin condition we called
melanoma. In simple terms, it’s a mole cancer. Are you okay right now? I knew that this might
shock you. Do you want a cup of water? Are you okay to continue our discussion? (patient
looked surprised but she said she’s okay to continue)
Ok, Mary. Let me tell you detailed biopsy results. It’s lucky we caught your skin cancer at early
stage because biopsy result showed stage 2 among 5 stages. And also your skin cancer didn’t
invade deeply as the depth is only 0.6mm but it’s spreading superficially. Are you with me so far
now?
Ok, Mary. Let me tell you future management plan. Firstly, you need to see skin specialist
urgently. You need to have a minor operation again. It’s very important to undergo surgery again.
I’ll draw a picture for you. This is skin cancer area with the depth of 0.6mm. We need to have
1cm from the margin between skin cancer cells and normal skin cells. And it’s very important to
have follow up visits every 6 months for 2 years. Right now I’m going to order some basic blood
tests and chest X ray as if there’s any spread, the lungs is usually the first site. We’ll do yearly
chest X ray after the proper treatment, too. And you might need some advanced imaging like CT
or MRI to exclude any spread to the other parts of your body but it’ll be the specialist’s decision.
And in some cases, radiation and cancer medication might be added but it’ll be very least likely
in your case with early staging. Are you with me so far now?
And also, you have some risks of having another skin cancer because patient diagnosed with
melanoma have higher risks than normal people. So it’s very important to look for any suspicious
skin lesions in you even after the treatment. Don’t worry. We’ll teach how to examine your skin
on other parts of body and we’ll do so in your follow up visits.
And it’s important to follow some general advice. Sunlight is one of the risk factors and so you
should keep inside during the time of highest UV rays especially from 10 in the morning to 4 in
the evening. Try to use some SPF creams when you have to go out during these times. Don’t
worry. I’ll give you some written materials about these general advice.

And there’s some risks of having skin cancer in your family members, too. So if there is any
suspicious skin lesions in your family members, come and seek our medical advice immediately.
If you’re happy, I can arrange some family meeting, too. Are you with me so far? I’m afraid I
might give you loads of information in this one visit. So is there any points you want me to
repeat again? (patient said no)
So let me summarize again. First you need to see skin specialist and have an operation again and
have follow up visits on 6 monthly basis for 2 years and yearly chest X ray. It’s important to avoid
sunlight on highest UV rays time. Then the bell rang.
Comment: I thought I failed this case because I spoke very quickly in this case. And this is my last
station and I felt quite tired which might affect my performance. I felt the patient was tired, too
and didn’t give much attention during my discussion. I forgot to mention about sentinel lymph
node taking and physical examination of whole body for any spread. I forgot to give some
reading material about melanoma. I’m afraid my speech having much force might threaten the
patient. I forgot to talk about prognosis and reassure the patient. I shouldn’t describe about
radiation and chemotherapy in this case.

126-Croup
AMC CASE
you are at GP when mom brings her 15 months child with difficulty in breathing since last night
tasks
-history
-PEFE
-DX, DDX

Case: You are in a GP clinic and your next patient is 9-month-old John who was brought by his
father saying that he’s having noisy breathing.

Sample Case: You are working as a Junior Doctor in a metropolitan hospital and you are about to
see Tom a 15 months boy brought to the Emergency Department by his mother due to coughing
and breathing difficulty for one day.
TASKS
1. History
2. Physical Exam
3. Diagnosis and Management

Differential Diagnoses
 Viral URTI:
 Bronchiolitis
 Pneumonia
 Croup
 Laryngomalacia
 Whooping cough
 Foreign body: any choking episodes
 Cardiac cause
 Epiglottitis

Positive findings in the history: barking cough, fever can be present, runny nose, symptoms are
often worse at night.
Rao developed a flu like illness two days ago with runny nose, cough and fever, last night he
came down with a spasmodic, harsh brassy (barking) cough. He has been a previously well child.
He is still drinking fluids and has eaten alright yesterday

Positive findings in the physical examination: Irritable behaviour, stridor (high pitched stridor)
tachypnoea, increased work of breathing, lethargy, hypoxemia is a late sign.
Vital signs normal except for mild tachypnoea and a temp. of 38 degree Celsius.
Mildly unwell looking boy with a loud audible inspiratory stridor, followed by a harsh barky
cough. Some retraction at rest. His larynx looks quite red. Inspiratory stridor on auscultation
with mild expiratory wheeze.
No other pathological findings.

History
1-Is my patient hemodynamically stable? (in the exam all good)
-I would like to check the GA (alert, conscious, irritable, cyanosis, pallor, position)
-I would like to check vitals(BP, PR, RR, T) and O2 saturation
-Respiratory distress(nasal flaring, grunting, tracheal tug, accessory muscles, recessions)

*If GP I would like to assess my patient in treatment room. If ED proceed to history.

2-Breathing difficulty questions


-can you tell me more? For how long?
-any noisy breathing? (yes)

3-Noisy breathing questions


-can you describe it for me? (high pitch sound while breathing) (in the exam they will give you a
sound record)
-Onset (sudden or gradual/ continuous or intermittent)
-does it occur at rest? During sleep?
-does anything make it better or worse?
-have you noticed that she is breathing too fast?
-any particular posture the child prefer?
-any chance of foreign body inhaled?
-has this happened before?

4-Fever questions:

-Any recent fever? have you checked it at home? Is it continuous or intermittent?


-Have you given any medication?
-Rash? Fit or seizure? Vomiting?

5-Cough questions

-Onset: for how long is she having cough? Sudden or gradual? What time of the day?
-Type: can you describe it for me? Is it like barking? Is th cough followed by vomiting? Does she
turn blue when she cughs? Does the breathing stop for a while? Any drooling of saliva?
-Pattern of cough (cough which is absent during sleep is suggestive of habit cough).
-anything makes it better or worse?
-runny nose
-Exercise tolerance/ playing (how does it affect)
6-Dehydartion
-Activity? Sleepy or drowsy?
-Eating or drinking well?
-Any change in the bowel motion or urine? Number of wet nappies?

7-BINDS
-Birth history
-Immunisation
-growth and development
-any other kids at home? Sick contacts?
-anyone smoke at home? Pets/ carpets? (None)
-past medical history (asthma, allergies, eczema, recurrent infections)
-family history (asthma, eczema, hay fever) + contact with someone with similar symptoms
(positive family history of asthma)

Physical examination from the examiner


1-General appearance
-alert, irritable, restless, drowsy
-position (is he sitting still, in tripod position, drooling saliva)
-dehydration, rash, pallor, jaundice, cyanosis, LAP
-STRIDOR AT REST
-respiratory distress signs
*nasal flaring
*accessory muscle use
*subcostal or intercostal recessions.
2-Vital signs especially temperature and O2 sat. (all normal just temperature high)
3-growth chart
4-Ear and nose only
5-chest examination (clear)
inspection: deformity, chest movement
palpation: chest expansion, tracheal position
percussion: dullness.
Auscultation: air entry, breathing sounds, wheeze or crackles,
6-quick CVS, abdomen

Explanation
Children with croup should have minimal examination. Do not examine throat. Do not
upset child further.
From history and examination, your child most likely has a condition called Croup.
Condition
-it is an infection or swelling of voice box and windpipe making the airway narrower so it is
harder to breathe.
-common condition in children 6 months-5 years
Cause
-usually caused by viral infection (parainfluenza)
Clinical features
-usually begins like a normal cold then the cough change to become harsh and barking.
-when your child breathes in they may make a sneaky, high pitched noise called stridor.
-often begins without warning in the middle of the night worse on the 2 nd or 3rd night of illness.
Complications
-can last for 3-4 days or up to 3 weeks prolong cough
other possibilities:
epiglottitis, foreign body, asthma, bronchiolitis, pneumonia then say less likely with reasons.

Note/ I am not sure if treatment is a task you can decide upon the findings as they can change it
so treat as mild or moderate or severe according to findings.
Treatment
Features Mild Moderate Severe

Behavior Normal Some/intermittent Increasing irritability and/or


irritability lethargy

Stridor Barking cough Some stridor at rest Stridor present at rest


Stridor only when
active or upset

Respiratory Normal Increased respiratory Marked increase or decrease


rate rate Tracheal tug
Tracheal tug Nasal flaring
Nasal flaring

Accessory None or minimal Moderate chest wall Marked chest wall retraction
muscle use retraction

Oxygen No oxygen No oxygen requirement Hypoxemia is a late sign of


requirement significant upper airway
obstruction

Management Manage at home -Admit to ED  -Admit to ED


symptomatically  Nebulised
-Rest,  -Oral adrenaline (1 mL of 1%
-adequate fluid, Prednisolone adrenaline solution plus
-minimal handling 1mg/kg, AND 3ml Normal Saline, or 4ml
of the child, prescribe a second of adrenaline 1:1000.)
-Panadol for fever, dose for the next  Give
-2 doses of steroid evening. 0.6mg/kg (max 12mg)
(1 given now, then OR IM/IV dexamethasone
one given at a single dose of Oral  If
home; 1mg/kg), Dexamethasone good improvement,
-small frequent 0.15mg/kg. observe for 4 hours
feeding (Oral dexamethasone post adrenaline.
suspension ONLY Consider discharge
BUT if mother is available in hospitals, once stridor free at
not capable of NOT available at rest.
taking care of the commercial  If
child at home, pharmacies) there is improvement
ADMIT. then deterioration,
-Observe for half an give further doses of
hour poststeroid adrenaline. Consider
administration. admission/transfer as
appropriate.
-Discharge once  If
stridor-free at rest there is no
improvement,
reconsider diagnosis.
Case (23/2/2017)
ED, 15 mth old child by anxious mom d/t noisy breathing, woke up in middle of sleep. Hx,
recorded sound played (inspiratory Stridor with barking cough), PEFE, PDx.

Case (2/3/2017)
Croup ( child with inspiratory stridor, but with drooling. I told epiglotitis as the ddx )

Case (10/5/2017) Croup

Case (7/6/2017)
15 months baby suddenly had noisy breathing. Task - hx, pefe, dx to mother. (Mom played a
sound on mobile which seemed like as wheeze, childs father has asthma. Pe fever, signs of resp
distress present, but lungs were clear and no added sound !)
Croup ( Feedback – breathing difficulty)
You are a GP . mom brings her 15 month old child with difficulty in breathing since last night.
Task-Hx, physical exam from examiner, dx .
Asked about vitals in beginning _ all good
Typical croup case.Sound was recorded
Details of breathing difficulty, no FB inhalation, BINDS, positive FH of asthma, mom takes care of
baby, unable to feed till last night, nobody smokes at home
PEFE
Normal vitals (only temperature high), growth chart.
Heart and lungs- no problem, clear
No office test available
DX- breathing difficulty can b due to – bronchiolitis ( usually smaller child), asthma ( but lungs
clear), pneumonia, heart cause, FB but most likely here croup. Viral cause. Very common.will
admit the baby. Steroid is the rx. Don’t worry will give in measured dose. May need nebulise
adrenaline. Seen by pedi specialist. We will be following him up regularly. Need to call anyone?
Support at home?

Recall 12-4-2018
a mom comes to you worried about her 15 m old child as having noisy breathing started from
last night woke him up at night.
tasks:
* take relevant hx
* ask p/e from examiner
* mx

Mom has recorded breathing of her child and it was croup.


child was stable but has retraction of accessory mucsels, O2 sat=98%, temp: 38.2,
no cyanosis, immunization was uptodate, no recent infection, no one else with this, father was
smoker but not at home, no allergy, no symptom of anaphylaxis or angio edema.
I did not ask about dehydration and if irritable and counscious or not unfortunately.
127-Angina
Case 1 (27-06-18)
56 year old man complaining of chest pain
Tasks
-take history
-diagnosis and differentia diagnosis

History
1-checked hemodynamic stability ( Examiner said stable you can proceed

2- Pain questions (SOCRATES)


-are you in pain now? (No)
-can you tell me more about it?
-How severe was the pain from 1-10? (Pain was 5-6 in severity when it comes)
-what were you doing when the pain started? (Was going uphill)
-How long did it last? Suddenly or gradually? (Suddenly and lasted for 1 minute)
-can you show me where the pain is? Does it go anywhere else? (In the centre of the chest and
not radiating to shoulder, jaws or arms.)
-can you describe it for me? (it is like dull or tightness)
-anything makes it better or worse? (Got it when was going uphill relieved by rest for few
minutes.)
-has this happened before? (Not happened before)

3-Associated symptoms
- is it associated with shortness of breath? Funny racing of the heart? Dizziness or Sweating?
(No)
- Any cough, fever? Recent infections? (No)
- Any vomiting, nausea, heartburn or tummy pain? (No)
-Any leg pain or leg swelling? (No)
-Bowel motions change or colour? (Normal)

4-General questions
-past medical for hypertension, diabetes, hypercholesterolemia, heart or lung problems (Never
had checkup and does not know about his BP, lipid or sugar levels)
-PSH (no surgical history)
-do you take any medication? (Not taking any medications)
-Family history of any illnesses? (Father died of heart attack)
-SAD? (Heavy smoker and drinks alcohol occasionally and arrange another consultation)
- Any stress at work or home?
-Travel?
DX and DDX
-From history, there are several possibilities why you had chest pain. most likely it is a condition
called angina have you heard about it?
-Heart is a muscular pump, which is supplied by vessels called coronary blood vessels. In angina,
the heart is not getting enough blood supply due to narrowing of these blood vessels. At rest.
The body’s supply is enough to supply the heart. While on exertion, the body’s demand goes up,
so the heart is unable to cope with that causing low oxygen supply leading to chest pain.
-from history there are also several risks that might be associated with it. HPT, DM, high fat level
in the blood, previous heart conditions and as you have never checked for these before so these
are possible risks for angina. Others are smoking as it shows heavy smoking form history, positive
family history of similar problems)

-other possibilities are:


heart attack: less likely as the pain is different it should be constant and very severe, not relieved
by rest.
Pericarditis or inflammation of the coverings of the heart, less likely as the pain is not sharp.
Pneumonia or lung infection: less likely as no fever or cough.
Pulmonary embolism or clot in one of the vessels supplying the lung: pain is not of lung type and
no travel history.
Pneumothorax or accumulation of air around the lung also less likely.
GERD or stomach ulcer but no heartburn, no tummy pain or indigestion.
Muscular pain unlikely form history.

-It is good that you have come today I really appreciate it so that we can manage your conditions
appropriately and put it under control. Let me assure you are in safe hands.
Case 2
ED, patient having chest pain characteristic of angina from time to time. Long stem given. Getting
pain with exercise, walking uphills and not relieved with omeprazole. Pt. taking omeprazole and
ibuprofen for back pain. Family history of heart attack in dad and stroke in mum.
Tasks:
-ask pefe
-dx and ddx
-give management

Physical examination from examiner


1-general appearance
pallor, cyanosis, dyspnea, oedema (pedal and sacral oedema), dehydration, LAP (pallor +ve)
2-vital signs (all)
3-JVP+ carotid bruit
4-Heart
-apex beat? Thrill and heave?
- Heart sounds and murmur? Any pericardial rub?
3-chest
-air entry/ breathing sounds/ any basal crackles
4-Abdomen + DRE (DRE blood +ve)
-distension, mass
-tenderness, organomegaly
5-fundoscopy
6-office tests
-urine dipstick/ BSL
-ECG

Note/ so this case could be angina due to anaemia but also do not forget upper GIT bleeding
due to ulcer.

Diagnosis and ddx


1-Angina (explain it like before and stress on positive features presented in history and risk
factors stress on blood on DRE and anemia signs so causing low oxygen and precipitate angina.
2-also could be ulcers in the stomach or first part of blood complicated by bleeding.
3-give other possibilities of chest pain like before and say unlikely.

Management
-admit to hospital
-seen by specialist cardiologist and GIT
-secure 2 IV line, take blood Ix (FBC, ESR/CRP, UCE, LFT, coagulation, blood group and cross
matching. Start fluid
-troponin enzymes and ECG, chest X-ray.
-endoscopy and colonoscopy
-assurance
Feedback 23-6-2018
Feedback: Chest pain: very long stem giving you everything, chest pain during exertion and hx of
GI bleeding. Tasks: PEFE, DDx, Management.
Approach the pt: Vital signs? Pain? SOB? Reassure the pt.
PEFE: pallor, signs of anemia, CVS and pulmonary ex were normal, abdo ex was normal, rectal
examination: blood +.
DDx: Angina due to Anemia. Explain to the pt: bleeding causes lack of blood, anemia  cause lack
of oxygen to the heart during exertion  angina.
Bleeding most likely come from the stomach or first part of your small intestine.
Management plan:
- Admit pt to the hospital for fluid and blood tranfusion.
- Carry out investigation: FBE (severity), upper and lower endoscopy, ECG, chest Xray, heart Usd,
abdomen USD.
- Call enterologist and cardiologist to see the pt.
Grade: PASS, GS: 5
Key steps 1,2,3,4,5:N,Y,Y,Y,Y
Approach: 5, PEFE: 6, Dx: 4, Management plan: 5

Feedback 15-3-2018
ED, pt having chest pain characteristic of angina from time to time. Long stem given. Getting pain
with exercise, walking uphills and not relieved with omeprazole. Pt. taking omeprazole and
ibuprofen for back pain. Family history of heart attack in dad and stroke in mum.
Tasks: ask pefe, dx and ddx, give management
Approach: My approach was either this two things chest pain and Malena are related or not
related
yes i will not forget this case as it was my first station . Lot of history given outside with chest
pain which radiated to jaw etc suggesting ischemic pain and also had Malena and was taking
painkillers. My approach was either this two things chest pain and Malena are related or not
related and I have to do focused cvs and abdominal exam so for PEFE asked everything started
with vitals to assess hemodynamic stability yes had postural drop , pulse was normal not
irregular rest ok CVS no positive findings and abdomen not sure but I think there was tenderness
. drE had blood Or dark stools don’t remember.

so for ddx to patient I said that from my findings most likely this is chest pain angina due to
anemia as you are taking painkillers since long time and have Malena or it could also be that
your chest pain and dark stools are two different problems ie chest pain is due to blood supply
getting compromised due to block somewhere and we will have to investigate the bleeding also.
mx said I will move to resus area and do a quick ECG . If there are ischemic changes will start
MOnA and I am also giving cardiology and GI referral for your bleeding.will also send bloods as
you may be anemic and drew something while explaining. es also the usual admission start on iv
fluids since he had postural drop so was hypovolemic
Feedback 11-12-2018
47 year old man in ED with complaints of chest pain and feeling dizzy. He has also passed some dark
tarry stools recently. Recent use of ibuprofen for 3 months for joint pain. Take hx, PEFE, Dx DDx

Asked about hemodynamic stability (postural drop). patient was painted white on face, look kind of severe
anemia

For this case refer handbook.chest pain history in form of Socrates,rule out ANGINA , MI, PERICARDITIS,
AORTIC DISECTION,PULMONARY EMBOLISM,GiT SYMPTOMS(MELENA WAS THERE) osophagitis, peptic
ulcer, spicy food, nsaids(+),I think patient was taking PPI before , now he don’t take, smoking, alcohol,
stress,cancer don’t forget to rule out.
-PEFE – postural drop. Epigastrium tenderness. DRE – dark blood on examination. No haemorrhoids.

Dx and ddx-- angina induced by anemia, gave cause of chest pain , and anemia due to benign or malignant
growth in stomach, or intestine , bleeding complicated peptic ulcer,nsaid induced ulcers, bleeding
disorders,

Passed- global score-4

Another feedback 11-12-2018


Station 1 (Chest Pain) Global Score 6 Pass
47-year-old man comes to the ED complaining of chest pain and feeling dizzy. He had had chest pain
before but it is getting worse. He passed some dark stools and had been on Ibuprofen for knee pain for the
last 3 months. Tasks: Hx, PEFE, Dx and DDX.

When I read this case outside, the first thing that caught my attention was: NO Mx! What do I do now?

I came in the room, greeted the examiner and the patient. Asked about vital signs (I don’t remember them
now) and said the examiner that due to the dizziness I would like to lay patient down on the resuscitation
cubicle, start IV fluids, attach him to monitors and give him morphine. Examiner told me to proceed.

I asked the patient how he was feeling and if he was ok to proceed.

Than I started asking about the chest pain. He told me it was a strong pain in the chest, traveling to the
neck and arm. He had had that before during exercises and pain would go away at rest. But that time the
pain was not going away and it was more intense ( 8/10).

Than I asked about the stools. He told me it was very dark and no red blood. First time it happened to him.
Very mild epigastric pain.

I asked questions to rule out other causes: cough? Fever? Weight loss? Recent flu-like symptoms? Relation
of pain to meals? GORD? Rash? Lumps? SADMA? FH? Previous diseases?

On PEFE I asked about the vitals again. There was postural drop and tachycardia.

Positives: pallor, sweating, CRT more than 2sec. RS normal. CVS S1, S2 no murmurs.

Abdomen: mild epigastric tenderness on palpation, abdominal sounds normal. Gained consent and asked
for DRE. Examiner said: “ No hemorrhoids, dark stools with blood on the glove. Positive test for blood”. I
asked about office tests and ECG (not done). I thanked him and went back to the patient (always calling
him by his name).

I explained to him: “ John, most likely you have a bleeding ulcer due to the medication you were taking for
the knee pain. This ulcer most likely is in your stomach or duodenum (I draw it!). The bleeding is causing
your dizziness because of anemia and low blood pressure.
But my biggest concern at the moment is your chest pain. Most likely you had a stable angina that is when
you don’t have enough oxygen coming to the muscles of your heart. Anemia makes angina worse and can
lead to an AMI. That’s why your pain is not getting better now when you rest.

I started saying that we would have to admit him and run some investigations and that he might need
blood transfusion. Than I remembered it wasn’t my task.

I continued explaining my DDx for chest pain: pericarditis but no recent flu, pneumonia but no cough and
fever, pleural effusion but no dullness on chest percussion, GORD but no relation to meals, gastritis but the
pain stopped with exercises, Zoster but no vesicles. Than I gave quick DDX for the anal bleeding:
diverticulitis, hemorrhoids, anal fissures, colitis, polyps and tumors.

Bell rang. Both patient and examiner looked happy.

Key step 1: no. Approach to patient/relative: 7

Key step 2:yes. Choice & Technique of exam, organization and sequence: 5

Key step 3:yes. Diagnosis/ Differential diagnoses: 6

Key step 4: yes. Management plan: 5

Key step 5:yes

128-CVS exam physiological murmur


You are in GP, 30 year old man came for check up before he increase his health insurance. He was
told by his previous GP one year ago that there was abnormal heart sound murmur in his heart.
Tasks
-Do relevant PE excluding abdomen and lower limbs
-Explain findings to pt
-Discuss about Mx

Examination steps (Talley o Connor 7th)


Consent
I can see that you are here for health insurance. Today I would like to examine your heart and
vessels system this will involve having a look, feeling a pulse in your neck and wrists and listening
to your heart and back. Is that ok for you?

1-WIPE
*position: patient lying in bed with enough pillow at 45 degree

2-General appearance
-dyspnea
-cyanosis
-oedema
-cachexia

3-Hands
I would like to have a look at your hands if you could bring them up in front of you please?
-clubbing.
-signs of infective endocarditis: (you do not have to mention them just say no signs of IE)
splinter hemorrhage
Osler’s node (red tender nodules on pulp of fingers)
janeway lesion (non tender erythema on palm)
-Exanthomata (optional)
-cyanosis
-pallor

4-Arms
pulse:
-rate
-rhythm
-radio radial delay (examiner will say no)
-radio femoral delay (examiner will say no)

*in the exam just check for 15 seconds and multiply by 4 . I think that either the examiner will
give you the result straight away or there will be a stop watch inside to count 15 seconds.

Blood pressure: (examiner will give you the results in this case so you will not have to measure)
-feel: put the cuff on the right arm and feel the radial pulse. inflate it till the radial pulse
disappear and increase 20 mm hg above then start deflating and when the radial pulse returns
this is the SBP.
-Measure: with the stethoscope on the brachial artery of the right arm start inflating the cuff 20
mm hg above the previous SBP then start deflating every 3-4 mmhg and when the pulse appear
this is the SBP, then continue deflating and when pulse disappear this is the DBP.
-Measure the BP of the left arm only with stethoscope
-Measure BP now with patient standing (only for postural hypotension):
>15 mm hg decrease in SBP
>10 mmhg decrease in DBP

5-Face
-eyes (pallor, jaundice, exanthelasma) I am gonna pull down your lower eyelids
-malar flush
-mouth (central cyanosis). Can you open your mouth for me? And For central cyanosis ask Can
you lift your tongue to the roof of the mouth?
6-Neck
-feel the carotid pulse (one artery per time; not two carotid together)
-measure JVP: >3 cm increase JVP. Ask patient to turn his head to left.
-hepatojugular reflux: press on tummy for 10 sec. ask if any tummy pain before you press.

7-Heart examination
inspection
-scars (median sternotomy, lateral thoracotomy) ask patient to lift arm to look for lat. Scars.
-deformity
-visible pulsation
-pacemakers

Palpation
-apex beat (5th intercostal space mid clavicular line 1 cm medial and below the nipple) heel off
-heave: left sternal edge fingers off
-thrills (palpable murmurs): 3 positions: apex beat, left sternal edge, upper chest

Auscultation
-bell on apex
-diaphragm on apex
-diaphragm on tricuspid area (5h intercostal space left sternal edge)
-diaphragm on pulmonary area (2nd intercostal space left sternal edge)
-diaphragm on aortic area (2nd intercostal space right sternal edge)
-axilla (radiation of systolic murmur of MR)
-right neck (radiation of systolic murmur of AS) + carotid bruit.
-left neck (carotid bruit) as patient to hold breath during listening of carotid bruit.
-lean forward for AR
-Left side turn for MS

Notes
bell=low pitch sounds= S3 or diastolic murmur
diaphragm=high pitch sounds=S1, S2, S4, systolic murmur
S1  loud=MS/ soft=MR
S2 loud=AS/ soft = AR
P2 loud= pulmonary hypertension
S3physiological(young, pregnancy, thyrotoxicosis)/ heart failure/ MR, AR, VSD, PDA
S4 AS, HTN, IHD
opening snap MS
Ejection systolic click AS, PS

8-Back
inspection: kyphoscoliosis
palpation: sacral oedema
percussion: base of lungs
auscultation: basal crackles (imp)

9-Abdomen (lie flat) (no need in this case)


inspection: distension
palpation: (liver) (kidney only in HPT) (aorta only in Hpt)
percussion: ascites
auscultation (aortic and renal bruit only in HPT)

10-legs (no need in this case)


inspection: color and swelling
palpation,: tenderness, oedema, hot, pulses

11-office tests (if you have time)


-urine dipstick
-fundoscopy
-ECG

Explanation.
-I have examined you very carefully but I could not find any murmur. I reassure you that it was
completely normal cvs exam. Maybe at that time, you might have some fever or illness;
sometime we can find some temporary murmurs during illnesses because of increased blood
flow and work of the heart. We call it physiological murmur, which I think it was the most likely
in your case.
-For more reassurance, I can do ECG to read your heart.

-Are u with me so far? Any questions?

Case (22/2/2017)
You are in GP, 30 year old man came for check up before he increase his health insurance. He
was told by his previous GP one year ago that there was abnormal heart sound murmur in his
heart.
Tasks
Do relevant PE excluding abdomen and lower limbs
Explain findings to pt
Discuss about Mx

2 mins thinking
- This is typical recall of CVS examination for murmur. But here he is not coming for travel.
- I ll start from hand, pulse, BP and then face, JVP and chest exam. Dun miss to feel carotid
during auscultation and use both bell and diaphragm.
- Explain that murmur is physiological

Examination
When I stepped into the room, a young guy is lying on couch who seems to be a medical student.
Bcoz when I finish one step of examination, he gently nods his head. :P :-) There was an Asian
examiner who was not smiling at all.

Introduction
- Greeting – Good morning, I am Dr Khine. I am one of the Drs in this GP.
Taking consent
- Today u come here for ur medical insurance and I d like to examine ur heart and vascular system.
It will involve having a look at ur hands and chest, feel and listening ur chest. Is that ok for u? (Yes
Dr)
washing hands, asking to take off clothes
- Now, I ll wash my hand. Can u take off ur top for me?
GA
– A young man with average body built is lying comfortably on couch with cardiac bed. He is not in
distress.
Face
– There is no dysmorphic face. No cyanosis, no accessory muscle working. Teeth and gum are
healthy.
Hands
– There is no clubbing, splinter haemorrhage, Janesway lesion, no palmar erythema.
Radial pulse
– I feel the pulse and ask examiner for watch. (He said it is 80 per min, regular). Then I checked
radioradial delay “It is equal on both sides” I asked permission to RP “Can I feel ur groin area?”
“There is no radio femoral delay”
BP
– I d like to check BP in both lying and sitting and ideally in all 4 limbs. (There is a BP machine on
the table but examiner said BP was 120/80, no postural drop, equal in all limbs)
JVP
– I ased RP “Can u turn ur head towards left hand side?” “JVP is not raised”

Precordium exam
Inspection
– Shape of the chest wall is normal. No previous surgical scars, no dilated veins, no diffuse
pulsation.
Palpation (I d like to feel ur chest, Is that Ok?)
- Apex beat is palpable in left 5th ICS within left mid clavicular line.
- No thrill in all 4 areas
- No parasternal heaving and epigastric pulsation
Auscultation
I started with bell on mitral area and then asked RP to turn left side and breathe in and out and
hold. With bell, I listened tricuspid, aortic and pulmonary areas.
At first, I forgot to put my left fingers on carotid but I knew it in time and listen it again.
Then, changed to diaphragm and listened all 4 areas with left two fingers on carotid.
I asked RP to sit up and lean forward. Listened on lt sternal edge with diaphragm for EDM. Checked
carotid area radiation and carotid bruit.

Back
– No bilateral basal creps and no presacral oedema.

I presented my findings then “On auscultation, normal S1 and S2 are heard in all 4 areas, no added
sound, no carotid bruit, no bilateral basal crep and sacral oedema”
(Examiner checked each and every step of my examination and method without blinking)
Thank you John for ur cooperation, I ll wash my hands again.

Explanation to pt
John, today I ve examined ur heart and vessel system and I couldn’t find anything that is abnormal.
Everything is perfect. But I understand from ur notes that u were told by previous Dr that there
was some abnormal sound in ur heart 1 year back.
Do u remember that u had any cough or cold at that time? (RP : I dun really remember Dr)

Ok, no problem. There are some reasons that ur heart can produce abnormal sound we call
murmur. I drew a diagram of heart and valves. And explained if there is structural defect in our
heart, it can cause murmur. Or sometimes, if there is increased blood flow in ur heart like stress,
drinking too much coffee, alcohol, cough and cold, etc. it can produce murmur as well, we call
physiological or harmless murmur. In ur case, it is most likely physiological murmur as I don’t find
anything abnormal now.
To make sure that ur heart is alright, I will do some tests in my GP, ECG of heart, CXR and if I find
any concerning feature in ECG, further tests like echocardiogram will be done if needed and get
involved the heart specialist if it is abnormal.
Are u with me so far? (Yes)
Ok I ll give u red flag symptoms, review u when invx results come back.
AMC Feedback - Exam for insurance purposes : Pass (Global Score - 4)
Approach to patient - 4
Examination technique and choice- 5
Accuracy of examination- 3
Dx/ DDx 4
Feedback 1-3-2018
health assessment CVS exam PASS
old recall where a guy came for insurance checkup, do complete cvs examination exluding
abdomen and leg
i did complete examination with the dynamic maneuvers . everything was normal
which i then explained to the roleplayer

Feedback 17-8-2018 pass


Cvs PE with previous murmur-
I don’t know how they passed me in this case,all key steps covered,accuracy-3,approach-
3choice-4dx-4,bt global-4.
Dental brassier was there,I asked any rheumatic fever b4? any long time antibiotic b4, he
couldn’t remember, I found osler node janeway lessions positive, bt I wasn’t sure so I said I can
see some nodes, some haemorrhagic spots around nails.
I wanted to check radiofemoral delay, examiner said can u plx check ur paper again? so guys if its
murmur exam no need to check any hypertension pe part.
thn I became shaky as examiner interrupted me, when I was checking normal murmur bell rang,
examiner told go to ur next task, I said I need more time to check dynamic menuver,I checked
quickly, thn told him I didn’t get any murmur still I want to send u to the specialist to check any
murmur.

Feedback 17-8-2018
EXAMINATION FOR INSURANCE PURPOSES
A pat has presented to you his previous doc found murmur last year. Now he is here perform cvs
exam excluding abdominal and legs.
Done similar to geeky medics but very accurately. Didn’t done abdominal aortic and renal art
bruits. As well as pedal edema. Because of task was to exclude abdomen and legs. But did
hepatojugular to see jvp.
I couldn’t find any murmurs. So I explained , I have examined you very carefully but I couldn’t
find any murmur . I reassure you that it was completely normal cvs exam. Maybe at that time
you might have some fever or illness , sometime we can find some temporary murmurs during
illnesses, it could be due to a gland in front of your neck that is thyroid but that is also unlikely .
but for your reassurance I would like to do ecg. Then I asked examine that I would like to do
thyroid examination as well but she said not required. And bell rang
¾ key steps covered
Approach to pat relative 7
Choice and technique of exam organization and sequence 6 Accuracy of exam 7
Diag and dds 7 Global6
Feedback 16-8-2018
GP, 40 years old man want to increase his health insurance level. Previous doctor told him that
he has heart murmur. He is generally healthy. No history of taking medication.
Task:
Perform CVS examination excluding abdomen & lower limbs
Counsel the patient
I can't do well this case. Examiner disturbed me many times.
GA: no cardiopulmonary distress
Vitals: I said pulse rate: examiner said you measure yourself. I checked it: rate, rhythm, volume,
radio-radial delay. When I assess radio-femoral delay, examiner said you are not allowed to check
inguinal area. When I checked BP, examiner stand up and comes to see me. I think the cuff is a
bit small, so I planned to measure the cuff. After wasting a few seconds, Examiner said BP is
normal. I said I measure the temperature: he said do you think it is needed? I said no. I checked
hands, Anaemia & Jaundice. I said I checked fundoscopy: he said do you think it is needed? why?
I said I check Roth spot for IE. Examiner said: Ok, if you think it is necessary, you find out the
fundoscopy on the table and do it yourself. I said I skip it (I think waste about 30-40 sec). I check
chest (Inspection, palpation, auscultation). I listened carotid bruit. Bell ring. Examiner said move
to next task. (I haven't done signs of CHF).
I explained routine. During explanation, the patient cough. I help the patient wearing gown. I
can't do anything except this. Reassurance for patient. Continue explanation.
Grade: Pass
Global score: 4
Key Steps: 1- Yes 2- Yes 3- Yes 4- Yes
Approach to patient: 4
Choice & Technique of examination, organization and sequence: 4
Accuracy of Examination: 4
Diagnosis/ Differential diagnosis: 4

Recall 8-5-2018
CVS EXAM murmur last insurance case recall findings all normal Task CVS EXAM abdomen LL
exam diagnosis DD
129-Headache examination (Temporal arteritis)
50-year-old man, complains of right sided headache. and also pain while chewing food. bp is
stable. cant remember what else was there
Task
1. relevant head and neck exam
2. tell your differentials with reasons to the patient

Examination steps
1-ask for pain, painkillers after allergy

2-check vital signs (pulse, BP, temp)

3-Look
-face: asymmetry, bruises, laceration, rash
-eyes: ptosis, redness, discharge
-nose: runny nose
-ear: discharge
-mouth: dental caries, enlarged tonsils, red pharynx
-temporal area: temporal artery (prominent, beaded, site)

4-Feel for tenderness


-Frontal bone
-temporal bone (+ve tenderness)
-temporal artery (tenderness, thickness, pulseless)
-scalp (+ve tenderness)
-zygoma
-maxilla- mandible
-temporomandibular junction (TMJ) (tenderness and crepitation)
-sinuses (maxillary, frontal, ethmoid)
-teeth (tenderness on percussion)
-Neck (spine and Para spinal muscle)

5-Move
-neck movement + neck stiffness

6-Cranial nerve (2,3,4,6)


-all especially eye movements and fundoscopy

7-otoscopy!!

8-shoulder and hip movement for polymyalgia rheumatica myopathy (active and passive
movements)

9-complete with Upper and lower limbs neuro, other cranial nerves, respiratory and CVS
Diagnosis and differentials
1-temporal arteritis (most likely): is a condition in which the temporal arteries, which supply
blood to the head and brain, become inflamed or damaged. The cause of the condition is
unknown; it may be linked to the body’s autoimmune response. Usually our immune system
produce antibodies to fight infections but in temporal arteritis they are fighting our own body
tissues in your case the temporal artery. Causing your symptoms.

2-tension headache, migraine, cluster


3-sinusitis
4-TMJ dysfunction
5-cervical spondylosis
6-meningitis
7-space occupying lesion

Case (10/2/2017)
65 y old man , c/o headache , pain while chewing and shoulder pain .
Task perform PE not written what.
I started with general appearance of my pt He was elderly sitting on the chair Inspection of the
face , no asymmetry , no rashes ,no eye ingestion or excessive lacrimation , no exophthalmoses
Palpation of the sinuses , ear.
temporal area , 1st he said no pain as I was papating wrong area , Ikept on manipulated my
palpation until I got the tendernessI said I will do fundoscopy the bag was there I tried to open
it he said consider it normal.
Then I decided to do v quick cranial (was super quick ) as I couldn’t see any instrument , he told
me u can pretend that u have the tooth prick.
I mentined I would do full upper nad lower CNS I didn’t do as Iknew no time.
The I started to expain the diagnosis and why not other differentials( SOL,sinusitis ,ear or
dental ,cluster, migraine) , I PASSED THIS CASE

Case (10/2/2017)
Headache for few month probably, cannot remember detail stem, Jaw claudication was there
most probably.

Task :Do PE with commentary to patient*Dx and DDs

It was a total mess.i had no idea what to do,so asked the pt a few qs,and did
inspection,palpation like facial trauma case and did all Cranial nerve exam till 7.
No snelles chart ,no red pin,no tuning fork so skipped those and didn’t do fundoscopy, otoscopy
bcz examiner said ‘I cant dim the light and there is no dialator,do u think u could do it?’ I told
patient all the d/d s and didn’t explain anyone and she kept saying Really!OMG :O

I was too embarrassed so left the room early. WAS very depressed because of the examiners
look of utter disgust !

FB: Passed with GS 4 so those who r going to appear dnt rely on examiners look, attitude to
judge how u did. CZ the examiner on my 1st station looked very cheerful and I failed that.
Case (29/4/2017)
GP, 72 year old man with headache for 3 months, Jaw pain while chewing for 2 months and
weight loss for 2-3 kg in a few months
Tasks
- Perform relevant examination focusing on head and neck
- Explain Dx and DDx with reasons to patient

Patient was a very cooperative Aussie chap. The only positive finding was the patient scream out
loud when I feel his left temporal area. No dilated vessel was observed. When I examined the
eye and ENT, the examiner assessed me how to assemble and use fundoscope and otoscope but
no need to examine the patient. (some candidates needed to do those) Both were normal.
Visual acuity, visual field and LAMP were normal. I saw tongue depressor but I had no idea
where to examine. (thought it was for tenderness over teeth)

Dx- Temporal arteritis

Case (29/4/2017)
Old man with headache since few months, jaw pain since 2 months, weight loss since 3 months ,
tasks are history and examination diagnosis :
prominent vessel on forehead is what i found,tenderness in the temporal area,my diag was
temporal arteritis,cannot confirm anything before getting the feedback.

Case (9/11/2017)
Temporal arteritis
Task –PE of head & neck ( tenderness in left temporal area, +ve shoulder and hip girdle pain ),
Dx, DDx

1- Vital signs:pulse,BP,temp,RR
2. General appearance: Uncomfortable, at ease,etc.
3. Scalp: swelling and tenderness. The ipsilateral temporal artery is palpated both
temporomandibular joints: tenderness and crepitance while the patient opens and closes the
jaw
3. HEENT: eyes and periorbital area are inspected for lacrimation, flushing, and conjunctival
injection. Pupillary size,PEARL, extraocular movements, visual fields .If patients have vision-
related symptoms or eye abnormalities, visual acuity is measured fundi are checked for
spontaneous venous pulsations and papilledema.If the conjunctiva is red, the anterior chamber
and cornea are examined with a slit lamp intraocular pressure is measured.
Sinuses tenderness
The nares for purulence.
The oropharynx for swellings, and the teeth are percussed for tenderness.
ears
4. Neck stiffness: Check for stiffness, rigidity, tenderness to palpation. The cervical spine is
palpated for tenderness.
5. Neurologic exam: The whole thing.** For a new headache, unless you have a very good
reason not to, you should do the entire neurological exam, including mental status, individual
cranial nerve strength, sensation,reflexes, cerebellar, Romberg, and any special neurological
tests.
Feedback 7-7-2017
50 year old man, complains of right sided headache. and also pain while chewing food. bp is
stable. cant remember what else was there
Task
1. relevant head and neck exam
2. tell your differentials with reasons to the patient

Hello examiner, hello John. I have been asked to examine you, that would involve looking at your
head and neck, feeling for any tenderness, and some ne k movements
washed my hands. saw on the table ophthalmoscope, otoscope, tongue depressor
Im going to inspect your head and neck now...on inspection no bruises, scar marks, soft tissue
swelling
im going to touch now, if there is pain anytime just let me know, tenderness right temple, non
tender sinuses
can you do neck movements.
i will check your vision, looked around the room, examiner said chart not there...went to get the
torch to see pupillary reflexes, normal
assembled the ophthalmoscope, couldnt get the light on ( examiner was a nice lady, helped me)
as i approached the patient, she said its normal
went to get the otoscope, assembled it..examiner said ear is fine
got the tingue depressor....told the patient im going to look inside your mouth, he opened the
mouth, asked him to say aaahhh....uvula central, tonsils not enlarged, throat not congested
Thank you John. so i have examined you...you have this pain and tenderness in your right
temple...most probably its called temporal arteritis, which means inflammation of a blood
vessel near your temple
I checked for other causes as well that could cause headache. like any problem with your
vision..thats normal
PASS
keystep 1 yes 2 no 3 yes 4 yes 5 yes
approach 5
choice of examination 4
accuracy 4
diagnosis 4
Feedback 20-9-2018 Headache PE (PASS)
60 years old male c/o left sided headache for months, he also had aches & pain and loss of 3 kg.
His VS today are Temp normal, BP 140/90 mmHg, the rest normal.
Tasks :
Perform PE of head and neck. Explain your findings to examiner.
Dx and DDx with reasons to patient.
( There were tongue depressor, torch, otoscope and fundoscope on the table. )

Approach :
I greeted the patient and asked him whether he’s in pain and offered him painkiller. I told him
that I’m going to examine your head & neck, and so on…

PE :
Washed my hands.
Inspection - Eyes, ENT (with torch and tongue depressor), face especially temporal area –
unremarkable.
Palpation – I started on eye balls, then patient c/o pain when I press over left temporal region,
no pain over sinuses, and on opening the jaws. No cervical spine tenderness as well.
I proceeded to examine 2nd CN. I looked for Snellen chart & asked examiner, he said VA normal.
Then I did VF which is normal. I took fundoscope and explained patient what I’m going to do, but
I couldn’t find the switch. xD after some time, examiner said normal. Then the small bell rang.

Dx : GCA, explained about it. (didn’t tell about PMR)

DDx : it could be due to raise BP but yours is not that high. Can also be due to TMJ dysfunction,
dental problems, ENT infections, eye problems, migraine, brain tumor but less likely.
130-Respiratory examination peak flow meter
You are in GP; your next patient is 45-year-old man. He has shortness of breath after recent viral
URTI.
Tasks
- Perform respiratory system examination (3 or 4 minutes)
- Teach patient how to do peak flow meter.
- Explain what u are testing to pt.

Respiratory examination (talley O connor 7th)

note/ in the exam when you will start looking at the hands the examiner will tell you that the
task is to examine chest only. So just double check the first task if it is just chest then start fom
chest as the time is only 3-4 minutes.

1-WIPE

2-General appearance
-look at surroundings (oxygen, MDI, medications, sputum mug)
-dyspnea
-respiratory distress (signs of COPD)
*accessory muscle use
*intercostal or subcostal recession
*elevation of shoulder on inspiration
*pursed lip breathing
*leaning forward with arms on knee
*tracheal tug

-Cyanosis
-Cough

3-Hands
-Nicotine staining
-Cyanosis
-clubbing
-Muscle wasting
-flapping tremor

4-Arm
-pulse
-Blood pressure

5-Face
-eyes (pallor, horner signs)
-Mouth (central cyanosis, red/ enlarged tonsils)
-signs of allergic rhinitis (nasal crease)
6-Neck
-JVP
-Trachea position

7-Posterior Chest
Inspection
-chest shape, deformity
-scars
-prominent veins (only from front)
-chest wall movement
if unilateral decrease: collapse, fibrosis, effusion, pneumonia
if bilateral decrease: COPD, severe asthma

Palpation
-chest expansion (should be >=5 cm)
-cervical LN

Percussion (fold the arm and elevate the elbow)


-dullness
-hyper resonance

Auscultation
-breathing sounds
-added sound
-vocal resonance

8-Anterior chest
inspection (the same)
palpation (expansion + supraclavicular LN)
percussion (over clavicle, upper chest, axilla and laterally (middle lobe)
auscultation (upper chest, axilla and laterally)

9-Heart
-apex beat
-parasternal heave
-sounds and murmur

10-Abdomen
-liver

11-lower limbs: Cyanosis, oedema, rash

12-Others: spirometry, PEF, temperature chart


Now summary of what one would do in these short 3-4 minutes
Respiratory examination (12 steps) focus on chest only
1-inspection (front and back then comment together about: Normal shape, No deformity, No
scars or prominent veins and no RDS signs, normal chest movements)
2-palpate the trachea (central position; not deviated).
3-palpate chest expansion from the back
4-vocal fremitus from the back
5-percussion from the back.
6-aucultate breathing and added sounds from back including upper lobe and laterally.
7-auscultate vocal resonance from back (99)
8-chest expansion from front
9-vocal fremitus from front
10-percussion from front.
11-auscultate breathing and added sounds from front.
12-auscultate vocal resonance from front.

Explaining Peak flow meter


1-Now I would like to take some peak flow measurement this would involve breathing out
through this peak flow meter (show him)
this is used to measure how well the air can flow out of the lungs.
2-in term of setting the equipment first ensure that this mouthpiece is inserted in the peak flow
meter.
3-Ensure the dial is set to zero
4-please make sure to sit up nice and straight
5-take a nice deep breath in as far as you can
6-ensure that your lips are sealed lightly around the mouthpiece
7-then breath out as fast and hard as you can.
8-then I will take the best of three measurements

Is that all clear to you?


Now if you could do it t see how is your breathing?

**I think you do not need to show the patient how to do it just tell the steps and the patient will
do them after that!!!

Explanation
Told him we will do this test three times and take the best of 3 as the reading then examiner
gave me the normal reading for the patient. So i told him ur lungs r doing good. And chest
examination was also normal
The examiner gave me 700 as the normal reading for the patient and when I performed the test
on the patient the reading was 800. So I told patient that the normal value according to your
weight and age should be around 700 and yours is 800 so it’s really good

Differentials could be asthma or COPD


Case (22/2/2017)
You are in GP, your next patient is 45 year old man with. He has shortness of breath after recent
viral URTI.
Tasks
- Perform respiratory system examination
- Teach patient how to do peak flow meter.
- Explain what u are testing to pt.
2 mins thinking
-Though respiratory exam is usual, teaching peak flow meter is new task.
-It should be similar to spirometry method and take the best of 3.
Examination
When I stepped into the room, middle age man is sitting on a chair.
I greeted to examiner and roleplayer.
Good morning Andrew, I am Dr Khine. I am one of the Drs in this GP. How are u feeling today?
(Good Dr).
Today, I would like to examine ur breathing system and lungs which will include having a look,
feel ur chest and listening with stethoscope. I will try to be as gentle as possible, and during the
examination process, if u have any pain or discomfort, just let me know. OK?
Now, I wash my hand. Would u mind to take off ur gown?
I started from GA – Middle age gentleman of average body built is sitting comfortably on chair.
He is not breathless. I asked vital signs which are normal.
Then examiner interrupted me to do only focus respiratory exam.
I asked pt to brace his arms. And examined from back.
Inspection – Shape of chest wall is normal. No deformity. No scars, no aspiration marks.
Palpation
- I palpated cervical LN (examiner said do only focus resp exam)
- Chest wall movement is equal in all three zones
- (I didn’t do the vocal fremitus as I ll do resonance later on).
Percussion
– Normal resonant percussion is noted in all zones in both sides (I forgot to percuss from side)
Auscultation
– normal vescicular breath sounds in all 3 zones including apex. vocal resonance is normal.
Then, I asked “Examiner, would u like me to perform the same examination from front” “Yes”
At that time, the bell rang and he asked me to move to next task.
There was a peak flow meter on table. Examiner said it is not single use item. (I dun know why
he said this, but to be safe, I wiped the mouth piece with tissue from box given on table :p).
I pick it up and told pt “Andrew, this is a device called peak flow meter, I would like u to take
deep breath, put it in ur mouth and seal it with lips, then breathe out as much as u can”
Pt did it and the result was 640L/min. I told examiner that I want to repeat it for total 3 times
and took best of 3. He said, assume this is best of 3.
I told the roleplayer that this is basically to test how much u can blow out and to check patency
of ur smaller airways.
The results showed normal, which means u have no problem with breathing out. And I also
examined ur chest which is also perfect.
Some time left. I asked examiner if I can go back to first task. He said as u wish.
I told pt to lie down on bed to examine from front. I was about to start it and the bell rang.
(I didn’t examine trachea and apex beat from front)
AMC Feedback - Cough : Pass (Global Score - 6)
Approach to patient- 6
Examination technique and choice- 5
Accuracy of examination- 5
Explanation of procedure - 6
Dx/ DDx - 6 (Examiner is very good natured :D : ) :p)

Feedback 10-5-2018
plz can u tell about ur approach in Pefe meter and relevant examination case
It was respiratory CHEST examination ONLY.... PEFE meter i explained to the patient tht it is used
to check how ur lungs are functioning, how well they r doing right now. Then explained the
procedure tht u have to make sure the reading is on zero before u start, sit upright take few
normal breaths then take a deep breath as much as u can and then blow out in it while keeping a
tight seal of ur lips around the mouth piece. Told him we will do this test three times and take
the best of 3 as the reading then examiner gave me the normal reading for the patient. So i told
him ur lungs r doing good. And chest examination was also normal
The examiner gave me 700 as the normal reading for the patient and when I performed the test
on the patient the reading was 800. So I told patient tht the normal value according to ur weight
and age shld be around 700 and urs is 800 so it’s really good.

Feedback 27-10-2018
50+ year old man with chronic cough which features suggestive of asthma.
Task
-perform respiratory examination
-instruct him to use peak flow meter
-tell him what peak flow meter is for

-50+ year old male patient wearing a gown was sitting in the chair.
-introduced, washed hands, explained and got the consent.
-asked him to take off the gown Respiratory examination Inspection - normal

Palpation - normal
Percussion - normal
Auscultation - normal (normal vesicular breath sound, no wheeze, no rhonchi)

instructions on how to use peak flow meter


- device exactly like this was found lying on the couch right next to him.
- asked him whether he has any idea on how to use this.
- He said “no”
- Picked up the peak flow meter and started explaining to him.
- hold it with your hand
- put the scale to ‘0’.
- open the mouth piece
- completely cover the mouth piece
- breath in and out deeply and as much as you can
- asked him to do it and it was 600 ml.
- told the patient that we usually ask the patient to do it 3 times and take the best
reading out of 3.
- then, examiner told me “consider it is done 3 times and you get the same value”
- then he handed me a chart with normal PEFR value on it (625ml or sth)
- I explained to the patient that the cough that you are having is most likely due to
asthma based on the history (according to features given in the stem).
- asthma is a chronic condition where your airways are narrowed in response to
allergen.
- peak flow meter is used to assess how narrowed your airways are.
- Told him that the finding is normal in his case (both examination and PEFR value)
mentioning his condition is well controlled.
- Then I asked him to do the PEFR again without my instruction
- He did it perfectly.

Grade - pass Global score - 5

Key steps 1,2,3,4,5 - Yes, Yes, Yes, Yes, No

Approach to the patient - 6


Choice and technique of examination
organization and sequence - 5
Accuracy of examination - 6
Explanation of procedure - 4
Dx/DDx - 5
Feedback 8-11-2018
19)30yrs old m,le pt c,me to ur clinic compl,ining of cough ,known c,se of h,y fever,childhood
,sthm, but l,st 15yrs no ,tt,cks,no f,mily history , Vit"ls "re norm"l

T#sks
Do relev#nt ex#min#tion 4mins
Pe"k flow meter expl"n"tion
Di-gnosis -nd dds

When I entered the room,young m-le pt sitting on couch ,I introduced myself -nd st-rted ex-min-
tion with h-nds ,ex-miner s-id focus only on chest ,I s-id th-nk you ex-miner,then I performed
inspection,p-lp-tion,percussion –nd -uscult-tion -ll -re norm-l,tr-che- position midline,I st-rted
from b-ck,concluded with front -nd cvs ex-min-tion.

Then expl-ined -bout Pe-k flow meter with new mouth pe-ce, -fter expl-n-tion he performed
wrong w-y but I corrected it,the score w-s700 ,I s-id excellent. Then I expl-ined to pt John v-lue is
norm-l,lungs -re good I didnʼt he-r –ny -bnorm-l sounds,he -sked me th-t me-ns I donʼt h-ve -ny
-sthm- dr,I s-id –t present you donʼt h-ve -ny symptoms of -sthm- but it doesnʼt me-n you wonʼt
get -ny -sthm- -tt-cks future, C-uses for ur cough h-y fever,irrit-nt cough,smoking is one of the c-
use,simple cold,pneumoni- but less likely in your c-se.

Glob-l score 5
Appro-ch 4
Choice of ex-min-tion 4
Accur-cy of ex-min-tion 5
Expl-n-tion of procedure 6
Di-gnosis -nd dds 4
Key steps 1 2 3 4 5 yes
131-Prostate cancer counselling
55-year-old man who initially presented to your GP with frequency, urgency and pain when
passing urine was referred by you to a urologist because you found on the DRE examination an
enlarged prostate.
The urologist sends you a letter with the results of a range of investigations he has initiated:
1.MSU showed growths of e.coli
2.PSA 6 ng/ml ( <4ng/ml normal, 4-6 ng/ml intermediate, >10 ng/ml high)
3.The core biopsy with 8 samples from the prostate was positive for adenocarcinoma
4.Gleason grade 7.
5.A cystoscopy did not reveal a bladder neck obstruction
6.the whole body radioisotope scan did not show metastases
7.CT pelvis and spine did not show any tumour outside the prostate capsule (stage T2)
The urologist mentions that he explained to patient that there was cancer but he suggested to
see you for further counseling.
Tasks:
1.Explain results and its implications
2.Discuss treatment options

Counselling
1-Approach
Role player: Doc, please explain the results after I heard the word ‘Cancer’ everything was a
blur!
-I understand that you came here for the explanation of your results. I am sorry I can only
imagine what you are going through but we will be here to help you and support all the way.
-all right so let me explain the results for you and if you have any questions please let me know.

2-Explain what prostate and prostate cancer


-(draw a diagram) first; the prostate is a walnut sized gland that forms part of the male
reproductive system. It secretes fluid that carries sperms. It surrounds the urethra or the canal,
which carries urine from the bladder out of the body.

-prostate cancer is one of the most common cancer in men. It is rare before age of 50 and
increasing incidence with age. There is a family connection similar to breast cancer in women.
it is considered to be a slowly growing cancer and can be asymptomatic even if extended beyond
the prostate or can cause symptoms like….

3-Explain results
-The urologist had done some Ix to determine its nature, potency and spread.

-PSA test is to measure a protein in the blood called PSA that help to detect prostate cancer or
any prostatic abnormalities; this showed a moderate elevation in PSA level.

-So usually if DRE is suspicious or PSA elevated then a procedure we call core biopsy needed to
be done under ultrasound guidance in which several tissue samples are taken to be examined
under microscope for histological evaluation using a scoring system called Gleason score.
-I will explain what and how they measure Gleason score
First there is a Gleason grade to help determine how quickly is the cancer to grow and how
likely to spread outside the prostate. This ranges from 1 to 5 given based on the appearance of
the cancer cells. 1 is when the cells look much like normal and 5 indicates a very abnormal and
aggressive cell type
-then the pathologist adds the two most common grades together to make a final Gleason score,
so a total of rating from 2 to 10 is given.
-the Gleason score determines if the tumor is aggressive or slowly growing which in turn is
important for management decision.
-In your case 7 means a moderately aggressive so it does not lie in the highly aggressive
category.

-In term of staging, and to determine if cancer has spread to adjacent organs. Bone scan showed
that there is no spread to bone. In addition, CT scan of abdomen and pelvis showed that the
cancer is confined to the prostate and there is no spread to adjacent structures.

-urine microscope and culture showed E coli infection of the urine so I would like to prescribe
antibiotics to be taken for 14 days.

4-Management
-now the good thing is that we picked it up early so it can be managed properly.

-treatment will be by a MDT including urologist, cancer specialist and GP.

-urologist will decide upon the treatment. Options depend on age, PSA level, Gleason score and
staging. Options are:
1-watchful waiting: usually for those > 70 years old
2-total prostatectomy usually for tumor that confined to prostate and if patient is < 70 and PSA <
20 ng/ml.
3-External beam Radiotherapy
4-brachytherapy: another form of radiation with tiny radioactive seeds inserted directly into the
cancer.
5-androgen or hormonal suppression: mainstay for metastasis or spread or locally advanced
disease. Can include bilateral removal of the testicles, hormonal injections and antiandrogen
cyproterone acetate to suppress the growth of the cancer.

-will follow you up regularly with DRE and PSA blood test. To determine the progress of the
disease and the response to treatment.

-support

-family meeting with consent


Feedback 22-6-2018
57yo male with hx of recurrent dysuria, micturia had been seen by your colleague who on PR
found enlarged prostate and sent to urologist. The pt has gone to urologist who did:
Urine c & s: e coli, cystoscopy: obstruction of urethral neck, prostate bp: adenocarcinoma
(gleason 7), PSA- I can’t remember the level. bone ct: no tumors, pelvic and abdominal ct with
no presence of tumors other than the prostate.
Task: patient knows about cancer but needs further explanation.
Explain the examination results
Explain treatment options.
Approach I went in and asked how the patient is feeling. He said he needs more explanation as
he went blank the moment the urologist mentioned cancer. I said I am sorry I can only imagine
what you r going through but I want to say that we will be with you every step of the way and I
will arrange support for you.
I said before I proceed do you have any question.
He said he wants to know about gleason score
So I drew prostrate with the kidney, ureter and bladder.. explained that it is a walnut shaped
organ which produces a part of semen. You have the cancer in this organ. Gleasons score means
how invasive this cancer is. 7 means it is moderately invasive. I asked whether understood and
he stared blankly. So I explained again ( wasted my time there)
Then explained the rest of the investigations quickly
Then asked about co-morbidities- heart, liver, kidney condition ( all normal)
Then said as it is moderate according to gleason score will probably go for surgery.
But I will give you all the treatment broad headings
1. Surgery
2. External beam radiotherapy
3. Brachytherapy
4. Watchful waiting with regular followup—rare as he is young 57, no co-morbidities and
moderate grade of cancer.
I said we will approach your treatment plan in a multidisciplinary team manner.
Then went for details
Surgery is key hole under anaethesia. We will do an anaesthesia eligibility check up for you
before we proceed with surgery
EBRT—localized radiotherapy with covering of other body parts
Brachytherapy—injection of anticancer drugs locally
SE-
Surgery
Injury to surrounding organs rare as expert surgeons will perform
Retrograde ejaculation- explained and mentioned about sperm preservation for infertility
Then bell rang—I said I will arrange support and another appointment with urologist and said
thank you to both patient and roleplayer
(could not mention about urinary incontinence, orchidectomy, family meeting, social worker so I
felt upset about this case)
Case (11/11/2017)
Prostate cancer - deliver the bad news tell him the lab result and implication on him.
Urinary frequency
Stem same as Wenzel notes: Prostate cancer
55 yr old man who initially presented to your GP with frequency, urgency and pain when passing
urine was referred by you to a urologist because you found on the pr examination an enlarged
prostate.
The urologist sends you a letter with the results of a range of investigations he has initiated:
1.MSU showed growths of e.coli
2.PSA 6 ng/ml ( <4ng/ml normal, 4-6 ng/ml intermediate, >10 ng/ml high)
3.The core biopsy with 8 samples from the prostate was positive for adenocarcinoma
4.Gleason grade 7.
5.A cystoscopy did not reveal a bladder neck obstruction
6.the whole body radioisotope scan did not show metastases
7.CT pelvis and spine did not show any tumour outside the prostate capsule (stage T2)
The urologist mentions that he explained to patient that there was cancer but he suggested to
see you for further counseling.
Tasks:
1.Explain results and its implications
2.Discuss treatment options
Introduction:
Role player: Doc, please explain the results after I heard the word ‘Cancer’ everything was a blur!
I dint follow breaking bad news pattern here.
Empathised with him. Said I’m sorry about the diagnosis of cancer.
Started off explaining about Prostate gland with a diagram then said that it is a Slow growing
tumor, the urologist has done investigations to determine its nature, potency and spread.
Briefly explained about each result as in Wenzel notes.
Treatment options:
1.Needs antibiotics for UTI
2.Wait and watch-approach for elderly pt with prostate cancer
3.Radical prostatectomy
4.Radiotherapy- External and brachy
5.Hormonal for advanced cancer
I told him that urologist and the oncologist will decide on treatment options best suited to
you. Provided written information. Every step asked for his understanding
Key Steps 1, 4- No Key steps 2,3 –Yes
Global score: 4 ( Pass)
Approach to pt/relative: 5 Interpretation of investigations:6 Pt counselling/ Education:4
Feedback 8-11-2018
Your next p-tient in GP pr-ctice is - 68-ye-r-old m-le who h-s h-d recurrent urin-ry tr-ct infections.
Investig-tions were done -nd he w-s di-gnosed with prost-te c-ncer by urologist. P-tient c-me to
your clinic for more expl-n-tions Some of the result from the investig-tion: PSA 6ng/ml, MSU:
E.coli, Biopsy: Adenoc-rcinom-, Gle-son score: 7 Cystoscopy: No outflow obstruction of bl-dder,
Bone sc-n: No met-st-sis, CT chest/-bdomen: Norm-l.
T-sk
-. Expl-in the results
b. Discuss further m-n-gement pl-n
c. Answer p-tientʼs questions

Appro-ch w-s,-fter entered the room I introduced my self,i told th-t r u comfort-ble to discuss
-bout the results,he s-id yes! Then I -sked wh-t speci-list told -bout your condition,he s-id I h-ving
c-ncer -nd Iʼm bit stressed -bout the condition,then I told him,I underst-nd th-tʼs very shocking
news but donʼt worry we r here to help you!Iʼmsorry for th-t, now Iʼm going to expl-in the
results,if you h-ve -ny doubts during my discussion donʼt hesit-te to stop me ok,

Then I -sked do you know wh-t is prost-te.? He s-id no,ok let me expl-in,I drew - di-gr-m -nd expl-
in which is - w-lnut sh-ped gl-nd present lower p-rt of bl-dder which helps to secret p-rt of
semen. In your results shows there is - infection of urin-ry system but donʼt worry we c-n
successfully tre-ted with -ntibiotics, In blood tests Prost-tic specific -ntigen is -lso r-ised th-t me-n
it is specific for prost-tic gl-nd,not for c-ncer ,it usu-lly r-ises in the c-se of infection,infl-mm-tion
-nd c-ncer.in your it not too high. We h-ve done use -nd ve t-ken some s-mples -nd re-d under
microscope it shows some -bnorm-l cell ch-nges we c-lled -denoc-rcinom- -nd Gle-son score
helps to how tumour is growing it is 7 in your c-se th-t me-n itʼs not f-st growing, h-ve you
underst-nd so f-r, he s-id yes,I -sked do you w-nt me repe-t -nything?he s-id no.we -lso did ct sc-n
to ruled out spre-d of c-ncer to surrounding structure,fortun-tely in your ct sc-n is norm-l.th-t
me-ns tumour is present only in prost-te itself.

Now Iʼm going to tell -bout the m-n-gement,now onw-rds we c-n m-n-ge multi discipline te-m
,like me -s gp ,c-ncer doctor,Surgeon -nd psychologist. Speci-list c-n w-it -nd follow, they regul-rly
do blood -nd im-ging studies but they mostly do on -ged more th-n 70yrs but every thing decide
by the surgeon.

Next one they remove -ll prost-te gl-nd we c-lled tot-l prost-tectomy -nd it h-s some complic-
tions like bleeding,infection.
Next one is Turp ,they p-ss sm-ll tiny tube -nd remove the prost-te gl-nd,it -lso h-s some complic-
tions -nd expl-ined. R-diother-py -nd -lso br-chyther-py -nd itʼs complic-tions -lso expl-ined.-nd I
-lso -sked h-ve you understood so f-r ,-nything do you w-nt me repe-t. He s-id no.

One more option is -lso there, chemother-py -nd they remove testis but itʼs for if tumour spre-ds
to surrounding structures. I told him,but donʼt wry we h-ve excellent support,if ur wish I would
like to -rr-nge - f-mily meeting those -re good person to sh-re ur p-in ,psychologist -ppointment -s
well. Then I give you re-ding m-teri-l,red fl-gs -nd referr-ls –s well. I g-ve support groups -s well

Glob-l score 5 Key steps 2 3 4 yes 1 no


Wenzel case
INFORMATION FOR CANDIDATE:
You recently referred a 68 year old Mr. Higgins who initially presented with frequency, urgency
and pain when passing urine to a urologist because you found on the pr examination an enlarged
prostate.
The urologist sends you a letter with the results of a range of investigations he has initiated:
1.MSU showed growths of e.coli.
2.PSA 6 ng/ml ( <4ng/ml normal, 4-6 ng/ml intermediate, >10 ng/ml high)
3.The core biopsy with 8 samples from the prostate was positive for adenocarcinoma (Gleason
grade 5)
4.A cystoscopy did not reveal a bladder neck obstruction
5.the whole body radioisotope scan did not show metastases.
6.CT pelvis and spine did not show any tumour outside the prostate capsule (stage T2)

The urologist mentions that he explained to Mr. Higgins that there was cancer but he suggested
to Mr. Higgins to see you for further counseling.
Mr. Higgins comes today for explanations.

YOUR TASK IS TO:


•Explain the results
•Discuss the diagnosis and management options with the patient

HOPC: As above. Mr. Higgins also tells you that he is still sexually active. His main problem is that
he got quite a shock when the urologist mentioned cancer and he really did not understand
anything else at the time and would like to get information from you.
PHx,: unremarkable
FHx.:NAD
SHx: married, retired accountant although still engaged in family business activities, non drinker,
non smoker, NKA, no medication.

DIAGNOSIS:
1.UTI
2.PROSTATE CANCER

Understanding the Prostate:


The prostate is a walnut-sized gland that forms part of the male reproductive system. The
prostate secretes fluid that carries sperm. It surrounds the urethra, the canal which carries
urine from the bladder out of the body.
With age and time, the prostate enlarges causing pressure on the urethra, similar to a clamp on
a garden hose. This causes problems with urination.

Prostate cancer is the most common cancer in men and constitutes a significant health issue.
Even though the saying “more men die with prostate cancer than from it” is still valid, there is
considerable morbidity! Rare before the age of 50 and increasing incidence with age.
There is a family connection similar to breast cancer, i.e. the younger a family was when
diagnosed with prostate cancer, the higher the risk for the patient. However, it is considered to
be a slow growing carcinoma and can be asymptomatic even when it has extended beyond the
prostate! Although, if a patient develops symptoms (lower urinary tract obstruction, bladder
outlet obstruction (BOO), back pain, haematuria, uraemia, tiredness, weight loss and perineal
pain) it usually is at a stage where the carcinoma has already metastasized (bone!)! There a no
early symptoms!
The prevention is therefore a must with regular digital rectal examination (DRE) and prostate
specific antigen (PSA) blood testing, although it’s usefulness as a screening test is still debated
because it is neither sensitive or specific enough. It is more useful as a test over time as well a
ratio of free to total PSA. PSA serves as a very sensitive indicator of progress of disease and
response to treatment!!!

Signs of abnormal prostate on DRE:


•Hard lump
•Asymmetry
•Induration
•Loss of median sulcus

BIOPSY: usually if DRE is suspicious or PSA elevated under transrectal ultrasound (TRUS)
guidance taking several core biopsy samples (6-12) for histological evaluation using the
Gleason score.
A score from 1 to 5 is given based on the appearance of the cancer cells, where 5 indicates a
very abnormal and aggressive cell type.
By grading the appearance of the two most common cell types and adding the scores together,
a total rating from 2 to 10 is given. The Gleason score determines if the tumour is aggressive or
slow-growing, which in turn is important for management decisions.
Fast-growing cancers which are more likely to affect a man’s health and lifespan are called
‘high-grade cancers’ with a Gleason score of 7 to 10. High grade cancers usually need to be
treated more radically, either by surgery or radiotherapy, since they grow more quickly and
may spread to other parts of the body. (Gleason score: 2–6 Low, 7 Intermediate, 8–10 High!)

The radionuclide bone scan is used for staging purposes looking for bone metastases.

In addition the normal TNM staging system is used:


T1 The tumour is found only in the prostate. It cannot be felt during a digital rectal examination.

T2 The tumour is located within the prostate only. It can be felt during a digital rectal
examination.
T3 The tumour has spread from the prostate to nearby tissues such as the seminal vesicle
glands, which produce semen.
T4 The tumour has spread beyond the prostate to the bones or lymph nodes.

N1-3 means the cancer has spread to the lymph nodes (glands) near the prostate. N0 means the
cancer has not spread to any lymph nodes.

M followed by 1a, b or c shows that the cancer has spread to the bones or other organs of the
body.
MANAGEMENT:
The management options should really be discussed with a specialist but include:

1.“WATCHFUL WAITING” recommended for patients over 70 years with no symptoms

2.TOTAL (RADICAL) PROSTATECTOMY: one of the two potentially curative therapies


(radiotherapy being the other one) for tumours confined to the prostate. Usually
recommended for patients under 70 years with a PSA <20ng/ml. It offers 90% 10 year disease-
specific survival for organ-confined tumours. Incontinence rate is about 7% and impotence rate
at least 30% or up to 70% (different authors).

3.RADIOTHERAPY: perhaps slightly less survival but also less mortality and incontinence and
impotence rates. Other side effects are diarrhoea and urinary frequency.

4.BRACHYTHERAPY: another form of radiotherapy with tiny radioactive seeds inserted directly
into the tumour. Probably less side effects!

5.ANDROGEN SUPPRESSION: the mainstay for metastatic or locally advanced disease. This
methods can include bilateral orchidectomy, depot injections of long-acting luteinising hormone-
releasing hormone (LHRH) analogues (goserelin = Zoladex, leuprorelin acetate = Lucrin) and anti-
androgens ( cyproterone acetate = Androcur, flutamide = Eulexin, bicalutamide = Cosudex).
All these have significant side effects ranging from an initial testosterone surge to loss of muscle
mass, osteoporosis, anaemia, hot flushes and adverse cognitive changes.

Specifically in this patient the PSA is only slightly elevated and the biopsy has confirmed a cancer
with low aggressiveness and there is no further spread of the tumour.
At this stage in the AMC exam it is not necessary to come up with a definite treatment rather
to put the options to the patient!!!!
However one also has to deal with the UTI which should be treated with empiric antibiotics or
in accordance with the MSU result (sensitivity).

132-Down syndrome baby counselling


You are an HMO and you are doing post-natal ward round. The nurse calls you and asks you to
explain to mother whose baby is suspicious of having Down's syndrome after examination by
one of your colleagues. Mother is very upset and she wants to talk to one of the doctor. The
chromosomal test is pending.
Task
Counsel the mother

1-Approach
-when you enter the mother will be on a bed sitting up right with baby in her hand and crying
with tissues and water next to her. So you need to be silent first and offer tissue and water.
-Introduce yourself.
-I can understand you got the news that your child has been suspected to have Down syndrome.
it is understandable that you feel like this, it must be upsetting for you.
- Do you have any special concern at this moment? Do you know what Down syndrome is?
(Yes I googled it last night. They have a very low life expectancy. Almost <30.)
-I can understand your concerns. How do you cope with the baby? (not coping well)
-do you have enough support from your partner or family? (yes my partner is very supportive)

2-Explaining Down Syndrome


I believe you need support at this stage. Let me first explain to you about down syndrome to give
you a better view about your baby’s condition and what management is available for him. If you
have, any questions please interrupt.
Condition
Down syndrome is a genetic condition in which the person has an extra copy of chromosome 21.
Chromosome are the blueprint for body’s development. The usual number is 46 in each cells but
in Down they have 47 in each cell instead of 46.
Cause
having this extra chromosome happens by chance. It is nobody’s fault. It means nothing that you
did before or during pregnancy has resulted in this.
Babies with Down could be born to parents of any age, race or social status.
Clinical features
babies with down clinically presented with upward slanting eyes, small fold of skin on the inside
of eyes, rounded face with flat profile, flat nose, low set ears, short neck, reduced muscle tone,
are usually smaller and weigh less at birth than others. They usually tend to grow slowly but
eventually meet many milestone.
They can develop communication skills but might take a little longer.
Complications
they are at higher risk of problems with vision, hearing, gut or heart defects, hypothyroidism,
bones problems and some degree of learning difficulties.
We will get your child reviewed by the spec and they will rule out all of these conditions. Blood
tests (TFT), Echo, USD.
3-Management
-look you are not alone we are with you.
-let me tell you that medicine in Australia is very well developed. The number of health facilities
giving to down are expanding and promising.
-At this point, although it is a clinical Diagnosis we need to confirm it as well.
-let me assure you that your baby will be taken care of by MDT:
*we will regularly testing his vision and hearing. He may need speech and language therapist to
improve communication skills and use language effectively.
*physiotherapist may arrange exercises to improve muscle strength, posture and balance.
*emotional and behavioral therapist, psychologist all will help in addition to GP.
*there is also Down syndrome association of Australia. Social worker and financial support from
centerlink.
-At the end, what you read on the Internet is not necessarily true. Down's patients can live up to
70 years or more. Down syndrome babies are socially happy and If they are provided with
appropriate stimulation and encouragement they can become productive members in our
society.
-allow yourself sometimes to adjust, talk to your partner about how you feel.
-Reading material thanked her
Feedback 7-9-2018
Down $ counseling – PASS
Mother age is 24-year-old patient . Uneventful pregnancy , other doctor checked the baby and
explain to the mother after just delivery that , the baby ‘s features are similar like Down
Syndrome. confirmation test ( karyotyping ) was taken and on the way to get the result .
- simulated patient’s mom is sitting on the bed with a gown.
Started to cry while introducing
Explain about confirmation test , and explain about meaning of Down $ , causes and
complications to the mother ( with a picture on the white plain paper ) – I explained 2 times as
simulated patient was crying intermittently . ( pause conversation and listen to her , taking time
a lot this station ) .
will treat as a MDT – GP , Paediatrician , OT , child psychologist , Speech therapist , social workers
and so on .
Further plan about pregnancy – need to discuss with genetic counselor – bell rang
Feedback – Counseling – Pass
Global score – 4 Key step 1 to 4 – all YES , Key step 5 – NO
approach – 4 Counseling- 4 Mgmt plan – 4

Case (10/2/2017)
-Down syndrome
the women with a new born kid Clinically Down, now counsel the mother. Had absolute no idea
what to tell, did as a breaking bad news but failed, so better to follow someone who passed

-3 days old baby brought by the mother with a provisional diagnosis of Down syndrome , genetic
testing is pending .
task is counselling:Y oung lady was holding a wrapped doll , and crying .Wants to know more
about the disease .
I started with trying to calm her down , I told her down syndrome is not that bad as you
think !,
I want to reassure you that nowadays the medicine in Australia is v well developed and the
number of health facilities the give to down pat is expanding and very promising. I will expaine
in details about the syndrome if you have any concern please share with me .
The disease is ( 5C Condition , cause , commonality , clinical pic and complications )Her
concern was that he has heard that they die early and she is scared .
I told her in depends on the grade of down but most of them they live long especially with the
amount of care that is Australia is offering )
We need to do further investingation to role out the complication that I told u about.
I said x ray abdomen .As your child is feeding well im not thinkinh of any abnormal connection
between the food pipe and the wind pipe .We need to do echo to role out heart problems .I
finished early I felt like it was only 3 min , I passed this station
Case (5/10/2017)
Station 14 Health review
Fail
You are an HMO and you are doing post natal ward round. The nurse calls you and asks you to
explain to mother whose baby is suspicious of having Down's syndrome after examination by
one of your colleagues. Mother is very upset and she wants to talk to one of the doctor. The
chromosomal test is pending.

Task; Counsel the mother

When i entered the room, the examiner checked my ID and then he couldnt find the Case stem
card and he kept on looking for it even i told him it's ok that i remembered the task. It took
about 1 minute and finally we found it. Then i went to the patient who was lying on the bed with
the dummy baby and she had very upset looking and didnt look me in the eyes. I introduced
myself and approached as a breaking bad new case. Then, i explained her what is Down's
syndrome and reassured her that it's not confirmed yet and we are waiting for the blood test to
confirm. Then explained to her the complications and conditions that can occur in Down's
syndrome babies like ( congenital heart defects, intellectual disability, growth retardation,
Leukemia). If it's confirmed, we have multidisciplinary team including specialists to manage and
follow up the baby. We will do physiotherapy, speech therapy, occupational therapy and even we
have special school for baby. And i told her that Down's syndrome babies are happy and cheerful
babies and with early and certain intervention, they can live as near normal life as possible and
they will be helpful to the community. I asked her what more she wants to know and she asked
me what more she should know. I asked her about home situation and asked her how she coping
with the baby now, n she answered she is not coping well but her partner is supportive. I offered
her psychologist to have talk therapy as i understand u have so many stress right now and also
told her that i can include social worker if she has any financial issues or home situation. And i
mention Down's syndrome support group where she could go. I told her i will give her reading
materials about Down's syndrome. I reassured her so many times as far as i remember.
Covered all 5 key steps
Approach 3
Patient counselling 3
Management plan 4
Global score 3

I didnt mention details about specialists. But I got 4 for the management part. I asked her if
someone is outside and whether she wants someone to be beside her while i m explaining to
her and she said it's ok. I asked her how she felt and she said she is feeling very upset and i
addressed her concern. Dont know how to approach better.
Feedback
Hey guys sorry for the delay. Here is my approach to the Down's syndrome case:
I had in m mind what I wanted to say, As I read the stem outside.
Once two mins were finished I walked inside, and basically the mother was on a bed sittin up
right with baby in her hand And crying with tissues and water next to her.
I got really startled as that was MY JOB! to give her the water and tissues.
Nonetheless, I proceeded by saying. Jenny I understand you got the news about your child
having downs ? How do you feel. She kept crying throughout the 8 mins.
How do you feel ? Is there anyone I can call for you ?
I understand you are very worried. Do you know what Down syndrome is ?
Have you heard of it before ?
She said. Yes I googled it last night. They have a very low life expectancy. Almost <30.
And I said okay do you know anything else ? What are your concerns
What do you expect from this condition ?
And she said. Just want my son to be alright
And I want him to live longer.
I then told her I can't do that I'm not GOD haha just kidding didn't say that.
Okay so then I preceded with the 5Cs
Condition ; Down's every cell has pairs of chromosome however in your child's case on of the
pairs has 3 chromosomes.
And then I proceeded. There's can be many factors such as
Age , Family Hx And genetic changes
And then I told her look you are not alone we are with you.
At this point in time it's not confirmed. Although it's a clinical Dx we need to confirm it as well.
I then told Her look Down syndrome is also associated with heart defects, gut defects
Neck bone defects and intellectual disability as well as Demenita and she started crying. I said
just because they are associated doesn't mean your child will have them. It's just association
nothing more.
We will get your child reviewed by the spec and they will rule out all of these conditions.
Furthermore. You are not alone.
We Will have a whole team supporting you morally physically medically financially and
emotionally
We call it MULTIDISCIPLINARY TEAM. And then I numerated the entire team.
SW ot
Psychologist , Speech pathologist , Centerlink , Down's Australia, Down's support groups.
Neurologist, Specialists
And then explained to her don't worry, wat you read on the Internet is not necessarily true.
Down's patients can live upto 70 years or more.
They live healthy and comfortable lives with today's medical advancements.
What's more is, that I have many patients with Down's
And then go to Uni and work and they are a very contributing part of society.
4Rs
Reading material thanked her and got the hell out of there.
Feedback 7-9-2018
DOWN SYNDROME COUNSELLING -PASS
You are an intern in a hospital. 22 year age old female delivered a boy. Her first baby. The
paediatrician suspects that the baby has down syndrome. The pregnancy and delivery of this child
were uneventful. Paediatrician has already informed her about her baby. She wants to talk to with
you.
- Explain mom about down syndrome
- further mx
Global score: 4 key steps 4/5 approach to pt:4
patient counsellling:4 management plan:4
IN 2 mints I was thinking - All I have to do:
1. showing empathy( it's not easy to do when you are stressed ) speaking slowly( time mx??) ,
low voice as pt is going to cry.
2. have to ask risk factors for down: family history (age ? 22, first child?)
3. Down syndrome screening during ANC check up ?
4. Confirmation test
5. talk about next pregnancy
6. further help
An young lady, wearing an hospital gown , was lying in a bed with a tissue in her hand. She
started to cry from very beginning. I introduced myself as an intern working here and I'm here to
talk to you about your baby. I directly asked her 'are you feeling okay to talk right now or I can
come back later'.( she started to sob again, said yes and I handed over the tissue box to her)
.Then I asked her would you like me to call your partner right now to be with you? what about
your family? (she mentioned something about it, I forgot) Then, I asked her; would you mind me
asking about how much you know regarding down syndrome? She mentioned that she had
checked about it on the internet. "Okay, alright. What's your main concern before I explain more
details?" She started to cry loudly saying down syndrome people's short life span made her
worried. I replied ; I'm sorry to hear that. It must be a difficult situation for you and internet
provides a lot of wrong information as well. Let me reassure you that down syndrome people
can live a long life, more than 55 years. Medical science has improved a lot over the years and in
Australia and we make sure to take extra care for them. We will be with you in every step and
every need. I'll provide you all the information regarding down syndrome and you can have a
quick read. If you have any question I'll explain it you again.( She nodded her head)
[It took my time to soothe her before I started to bombard all information]
Then, I went on explaining what's down syndrome; It's a common genetic disorder that delay
mental learning ability because of an extra chromosome at 21. Normally we have half from
mother and half from dad, but something has caused an extra cord that can lead to physical and
mental abnormalities. We usually ask every women to do some tests at first trimester to screen
the baby regarding DS. Have you done any tests? (No). May I know why? (I didn't think it's
necessary) Do you have any family history?(no) . I gave her comfort by saying that's alright, you
are young, it's your first baby and you don't have any family hx , to be honest , you were not at all
in high risk group for DS. We will ask you to do the screening test in your next pregnancy.
Well, right now we still have to confirm the diagnosis. We will do some other tests to look
through any associated defects. I'm not here to scare you but sometimes it could be associated
with heart defects and some problem in eyes, thyroid and in tummy. Please don't get scared, I'm
only providing you the information, it's not like your baby is going to be affected by all of these.
MDT will take care of your baby and monitor him regularly. There are special child care, support
group, social worker and special school as well. I'll provide you all the details of these support
groups later. Usually they are very compliant and cheerful and happy child.
[ while giving her all info, I reassured her couple of times, repeated some info couple of times
and took pauses couple of times as well. I had enough time in this case.
133-Morton neuroma ex
10-2-2018 and 1-6-2018
Foot pain. Sever during end of day. (Ex- pain at 2nd metatarsal space.Morton
neuroma).

AMC Exam case


Male or Female presented with foot pain
Tasks
-History 2 minutes
-Examine the foot
-Diagnosis with reasons

Focused History
1-Pain questions
-how severe is your pain from 1-10? (3-4/10) would you like me to give you painkillers (no it is
not that bad)
-can you point out exactly where you feel the pain? Does it go anywhere else? (Forefoot/ not
radiating)
-how long have you had the pain? Sudden or gradual? Constant or come and go? Is it getting
worse? (Chronic for 4 months)
-can you describe it for me? (Dull)
-does anything make it better or worse? (Worse towards the end of the day/ relieved by rest)
-has this happened before?

2-Differential diagnosis questions


Morton neuroma
- do you wear tight or high heeled shoes?
-do you feel that you have a pebble in your shoes?
Plantar fasciitis
-what do you work? Do you stand for a long period of time
Fracture
-have you any trauma or injury? (No)
Arthritis
-any history of joint or bone problems?
Tarsal tunnel syndrome
-any tingling or numbness? (Yes)
Foot Examination
1-WIPE (compare both feet)

2-Gait (Normal)
-walking
-turning
-walk on toes
-walk on heel
look for antalgic gai

3-Look (SSSDW) (Normal)


-scars
-skin colour.
-swelling.
-deformity (bunions, crowded toes, arch of foot).
-wasting

4-Feel (TTP)
-Temperature
-Tenderness (do this as group)
1st group
*lower end of tibia (6 cm above the medial malleolus) *medial malleolus-*navicular bone.

2nd group
*Lower end of fibula (6 cm above lateral malleolus) *lateral malleolus*base of fifth
metatarsal.

3rd group
*Calceno-fibular lig.*Ant.talo-fibular lig.*Post. Talo-fibular lig.

4th group
*Calf muscle- *squeeze Achilles tendon- *squeeze heel-plantar fascia

5th group
*Squeeze forefoot-*between 1stand 2nd and between 3rd and 4th digit space. (In the exam pain
on second metatarsal and sometimes between 3rd and 4th)

-Pulse (normal)

5-Move (only active movement) (exam all normal)


-Dorsiflexion
-plantar flexion
-eversion
-inversion

6-Sensation (normal)
7-Special test (based on palpation findings)
-Mulders click +ve in Morton neuroma (Mulder’s Sign is a physical exam finding associated with
Morton's neuroma, which may be elicited while the patient is in the supine position on the
examination table. The pain of the neuroma, as well as a click, can be produced by squeezing the
two metatarsal heads together with one hand, while concomitantly putting pressure on the
interdigital space with the other hand. With this technique, the pain of the Morton's neuroma
will be localized strictly to the plantar surface of the involved interspace, with paresthesias
radiating into the affected toes) (positive over 3rd metatarsal)

-tap on tarsal tunnel to rule out tarsal tunnel syndrome.

Explain
-from history and examination you have a condition called Morton neuroma
-normally you have a small nerve going in between each two toes called interdigital nerves.
These usually supply skin of the space and has a sheath around it.
-So for example if the toes are crowded together may be because of tight shoes or high-heeled
shoes this can cause thickening of the tissue around one of the nerves leading to your toes, and
cause a sharp, burning pain in the ball of your foot. Your toes also may sting, burn or feel numb.
-talk about reasons from history and examination positive findings.

Differential diagnosis (less likely)


-stress fracture
-Arthritis
-Tarsal tunnel syndrome

Investigation (Not a task)


-usually clinically diagnosed
-ultrasound (if not getting better)
-MRI (may be done by specialist if surgery being considered)

Management (Not a task)


PRICE
-Pain killer (NSAIDS)
-rest
-Ice
-elevation
-compression
-Advice good footwear (flat wide shoes)
-orthosis (special pad/ insole in between affected toes to take pressure out of the nerve.
-Physio
If does not work
-Injection (steroid)
-surgical excision of nerve.
Reading materials and review
Feedback (5/10/2017)
A painful foot – Pass
A middle-aged lady c/o foot pain.
•Focussed history
•PE
•Diagnosis to patient with reasons

Shooting pain in forefoot first noticed on a jogging trip. Can’t remember duration. No trauma/
swelling/ joint pain. No pain in other joints. Can’t remember much as I took a hasty history.

PE:
Look/ feel/ move/ special tests/ vascular /sensory
Positive findings: Pain on squeezing metatarsals, ……………test for Morton neuroma positive
Showed that I am excluding stress fracture of 5th metatarsal/ plantar fasciitis / tarsal tunnel
syndrome, mentioning these during examination.
Checked light touch sensation (looked around for cotton bud examiner was amused and asked
me to do with finger) and checked distal pulses as well.
Diagnosis: Morton neuroma. Explained with a diagram.
Said I have excluded other possible diagnosis (above). 2/2 key steps covered

Feedback 1-6-2018
Foot pain . Sever during end of day. (Ex- pain at 2nd metatarsal space.Morton neuroma).
Very difficicult old lady!!! Was wearing sandle with straps on over the normal foot and was
adamant not to remove the sandle

For hx as it was of 2 min asked pain questions, location: forefoot, intensity: 3-4/10, quality dull,
not going any where, onset chronic, duration: for 4 months, aggrevation on walking and at end
of day, relieving factor: rest , associated numbness and tingling : no, no trauma

Any ways
I entered, washed hands, introduce. Asked hr if she is in pain and if she wants any pain killer and
she said no its not that bad.
I said I need to explore the reason for pain for that I want her to expose her foot so please roll up
your trouser and keep foot out side bed end and remove the slipper but she said but I have pain
over right foot, I tried to counsel her that I need to compare my examination with nrmal but she
was adamant that she don’t want to remove, that confused me I excused from her asked the
examinier that I want to compare the abnormal foot with normal one but examiner said proceed
like the pt is asking you to…..

Any ways I
Asked her to walk, heel and toe walk: normal, no antalgic gate, heel and toe walk normal.
Checked her foot wear it was flat open toed sandle.
inspection: no scars swelling, redness, rashes, no evidence of pitting of nail, no joint deformity,
transverse and longitudinal arches normal, no musle wasting
feel: tempr: normal, pulse palpable dorsalis pedis and post tibial, tendeness: pain over 2 nd
metatarsal, rest all normal including Ottawa rule, sensations: normal.

move all normal


special test: squeeze metatarsal test, calcaneal squeeze test and high ankle squeeze test and
acchilies test : all negative
mudddlers click : positive over 3rd metatarsal
planter fasciitis test negative
told the examiner that I would like to conclude by performing the examination of one joint
above and neurology lower limb

told pt that it could be


metatarsalgia :( as she had pain while palpation over 2 nd metatarsal)
morton neuroma (muddlers click positive>>>> abnormality of nerves travlling between foot
bones secondary to excessive wear and tear, and wrong choice of foot wear)
planter fasiatis (special test negative)
stress fractures: (compression test negative).
RA, OA, PA , GA: (no visible features)
Uncommon ligament injry over ankle, aschilies tendititis or rupture unlikely cuz of location of the
pain

Thanked and reassured the patient

Feed back : passed


Global score: 5
Key step 1 &2 : yes
Hx: 5,
Choice and technique of exam: 4
Accuracy of exam: 5
Dx, DDX: 5
Feedback 1-6-2018

C/o foot pain and difficulty walking. Task: Relevant Hx, Perform PE, Dx with reasons,
(DDx not in task but still, always mention to be safe)
Pain relief offered.
Hx of foot pain with pins and needles sensation aggravated on walking. Rest normal.
PE: Moulder’s click +ve…. Bell rang when I finished my PE. Said Morton neuroma before I
left the room. Examiner gave me big smile and ticked a box happily. No time to explain
other possible causes or why Morton neuroma. I guess he was ok with that.

Note/ if you compared this feedback with the previous one you will see the difference in their
scores but why the latter got the highest score even she didn’t tell DDX and just explain the
diagnosis.
1-she made the examiner happy because the technique is 7
2-accuracy of examination 7
so this is the most important part the examiner look at in examination cases

3-her approach to patient is 6 the previous one was challenging the patient and did not follow
her.

4-even she just explained the diagnosis but she was following the task which is diagnosis with
reasons. The task did not mention any DDX so DDx are optional. And sometimes if you explained
the diagnosis only very well would be more beneficial than explaining it shortly and mentioning
other DDx.

However, both candidates pass this station.


Feedback 13-12-2018 (Important)
Foot pain: Pass
2 min outside: I don’t remember the stem exactly. But there was a young lady probably 35 yrs having
unilateral foot pain.
Tasks: History, PE, DD and DDX to patient

Thinking outside: DDS of foot pain. Forefoot (Morton’s neuroma, stress fracture, trauma, osteomyelitis),
backfoot (calcanitis, plater fasciitis, Achilles tendinitis, ruptured Achilles) etc.

Inside: I am so sorry to hear about your foot pain, is it ok if I ask you few questions and examiner your foot
so we can come to conclusion and help you to relieve pain.

Pt: ok.

Me: Do u have the pain at the moment? Do u need any pain killer?

Pt: No, thank you.

Me: Can u pl tell me with one finger where exactly is the pain? (I ask this que in early stage to narrow
down my DDS and to come to the conclusion very fast).

Pt: (Pt was very smart and must have been told not to show with finger) Showed me pain in entire
forefoot.

Me: Then asked all the questions of SOCRATES. (typical of Morton’s such as worst with tight shoes, better
when shoes taken off, worse at the end of day, pt said cant wait to take the shoes off.)

Me: pain when u wake up from bed? Pt: no

Me: do u have pain anywhere else in foot such as in back of foot or under sole (all negative).

Me: Any Injury (no), any insect bite(no), do u think there is raised local temp? (no).

Me: Pain when u touch (only at specific part and position).

Me: do u do lots of exercise or jumping? Pt:no

Me: Any similar pain in left foot? (No, only right).

Me: any other symptoms such as swelling, feeling heaviness? (no).

Me: Any known diabetes, HTN, kidney disease, thyroid disease? (no)

Me: Any similar past history? (no) Any blood clot in past (no)

Me: Any f/h of gout? (no) Any relevant family history. (no).

PE: Now, is it ok If I examine your foot, that includes looking at it, touch and to do some special tests. If u
fell any pain during examination, pl let me know.

On inspection: I do not see any obvious dissimilarity between feet, no swelling, redness, scars, obvious
deformity or any obvious calf problem.

Now I am going to touch your calf and foot, is it ok? Pt: Ok

I started to touch both midcalf downwards to toes for temperature. I said no change in temp noted.
Then I palplate both pulses, DP and PTA. I said both pulses palpable. (Be careful here, it is difficult to
palpate PTA in normal but fat people, but u just need to assume as its normal according to age and
symptoms).

Then I started to compress gently from midcalf (only rt leg) to toes and I asked for tenderness. I kelp doing
commentary as no tenderness noted on tibia, fibula, medial and lateral malleolus. Then I said no
tenderness on Achilles tendon, calcaneum, talus bone blah blah blah (I remember all tarsal bone names
so I mentioned them, don’t remember now). Then I individually compressed metatarsal bones to look for
stress fracture on 2nd metatarsal (negative)

Then I quickly but individually palpate toes (negative).

Now I am going to do some special tests, so let me know about pain. Pt: Ok.

Me: Did Mulder’s click. I pressed all inter digit space even though she gave me classic pain between 3 rd and
4th. Pt gave me classic Morton’s pain between 3rd and 4th toe pain.

I did not stop here and palpate under the sole for planter fasciitis and did special test for it (both negative).

Then I did tests for anterior and posterior talofibular ligaments and calcanofibular ligament ( I kept giving
commentary) All negative.

The I said Ideally, I would like to do tests for Achilles tendon but due to time restriction I will skip it.
Examiner; Ok.

Dx: I started explaining to examiner, then examiner said not to me, to patient or u can check your tasks.
The I turned around and started to tell pt.

From history and examination, I think u r suffering from condition called morton’s neuroma, do u know
about it?

Pt: No

Me: (after quickly drawing anatomy) Here is the nerve we called inter digital nerve, there is a fibrous
sheath around it, when it gets inflamed and swollen, u get pain the one you r getting nowadays. It gets
worse with tight shoes and relieved by removing them.

Other less likely possibilities are infection of skin (negative because no raised temp) infection of bone
(negative because no raised temp and no entry point), stress fracture (negative as generally it occurs to
them who does lots of jumping). Other very less likely are calcaneum bone inflammation, planter fasciitis,
Achilles tendon injury and swelling, osteoarthritis and trauma. I kept showing her the site as well when I
was giving her to DDs.

Global score: 5
Key steps:1,2 yes
History: 5
Choice& technique of examination, organisation and sequence: 4
Accuracy of examination: 5
Dx and DDs: 5

134-Plantar Fasciitis examination


22-2-2018 heel pain examine plantar fasciitis.
23-2-2018 PE: Foot examination. Windlass +ve
Exam case
A man or woman complains of foot pain for past 3 months, worst in morning when he steps out
from bed
Tasks
A)Physical examination
B)Diagnosis with reasons

Differential diagnosis of foot/heel pain


-Plantar fasciitis
-Achilles tendonitis
-ankle joint arthritis
-heel spur
-flexor tendon inflammation
-nerve entrapment

Foot Examination
1-WIPE

2-Gait
-walking (pain on walking)
-turning
-walk on toes
-walk on heel
look for antalgic gait

3-Look (SSSDW) + Compare


-scars
-skin colour.
-swelling.
-deformity (bunions, crowded toes, medial and lateral arch of foot).
-wasting

4-Feel (TTP)
-Temperature
-Tenderness (do this as group)
1st group
*lower end of tibia (6 cm above the medial malleolus) *medial malleolus-*navicular bone.
2nd group
*Lower end of fibula (6 cm above lateral malleolus) *lateral malleolus*base of fifth
metatarsal.
3rd group
*Calceno-fibular lig.*Ant.talo-fibular lig.*Post. Talo-fibular lig.
4th group
*Calf muscle- *squeeze Achilles tendon- *squeeze heel*plantar fascia
5th group
*Squeeze forefoot-*between 1stand 2nd and between 3rd and 4th digit space.

-Pulse

5-Move (only active movement)


-Dorsiflexion (may be +ve)
-plantar flexion
-eversion
-inversion

6-Sensation

7-Special test (based on palpation findings)


-Windlass test for plantar fasciitis
https://www.youtube.com/watch?v=fg0PtnoAzSs

-Mulders click to rule out Morton neuroma

Explain the condition


-I think you most likely suffer from plantar fasciitis, which is inflammation of fascia in the soles of
the foot.
-plantar fascia It is a band of connective tissue that runs along the sole from the heel to the ball
of the foot.
-It is the most common cause of heel pain and is usually due to overuse and stress on foot.
-Explain reason from the stem and examination findings.

-Other less likely causes of foot/heel pain are heel spur, plantar bursitis, flexor tendon
inflammation, Achilles tendonitis, ankle joint arthritis, nerve entrapment etc.

-I will do an x-ray to rule out any other causes and complications like spurs

Management (not a task)


Explain immediate management
-I will prescribe NSAIDs for you and you will need to take them as they help reduce the
inflammation.
-I would like you to rest your foot and avoid prolonged standing for now.
-You can also apply ice to it daily
-I will also refer you to the physiotherapist who will tell you a few exercises as well as how to
apply tape to reduce the stress on the foot
-Also please avoid wearing high heels or completely flat footwear like flip flops.

Long-term management
-Most cases resolve in a year
-If the pain is not relieved in 6 weeks we need to refer you to sports medicine specialist who.
might advise you on some orthosis and night splinting to stretch out your fascia
-If nothing else works we do have options of steroid injection and surgery as well.
-It would be good if you could maintain your weight as it has also known to help with treatment

Reading materials
History (not a task and if you want you can follow Morton neuroma approach)
Pain questions
-How bad is the pain on a scale of 1-10? Do you need a painkiller?
-Where exactly is the pain?
-Does it go anywhere else?
-Associated with movements or touch?
-Does anything relieve or aggravate the pain like rest or walking?
-Can you please describe the nature of pain
-Did you take any medications? Did it help?
History of risk factors / effect
-Occupation?
-Do you need to stand/walk for long periods of time?
-How does the pain affect your life?
History of DDs
Have you ever suffered from joint problems like arthritis before?
Any history of trauma to the area?
Any family history of joint problems like arthritis?
Is this the first time you’ve had this problem?

Risk factors
-Sports
-running, dance and aerobics
-Flat-footed or high arches
-Age
-Obesity
-Pregnancy
-Poorly designed shoes

Symptoms
-Intense sharp heel pain with the first couple of steps in the morning or after other long periods
without weight-bearing.
-Pain is experienced primarily on the plantar surface of the foot at the anterior aspect of the
calcaneus
-Symptoms typically are relieved by unloading the affected foot
-Symptoms may actually worsen by the end of the day

Signs
-Palpating the plantar-medial calcaneal tubercle at the site of plantar fascial insertion to the
heel bone – tenderness
-A tight Achilles tendon – reduced dorsiflexion
-Wind lass test
Ankle in dorsiflexion passively
Toe extended passively
Palpate the fascia on the sole
Tenderness +

Investigations
-X – ray to rule out other causes / heel spur

Immediate management
-NSAIDs
-Rest
-Ice
-Taping
-Physiotherapy
-Avoiding heeled footwear

Long term
-after 6 weeks / referred to sports medicine specialist
-Night splints – hold plantar fascia lengthened overnight
-Orthotics – e.g. heel pad distribute pressure over feet more evenly
-Physiotherapy
-Injections – intralesional steroids
-Surgery

135-Post MVA blurring of vision


Patient has involved in MVA. LOC + for 3 mins. P survey is done and all normal. Cervical spine
injury is excluded and collar is already removed. Patient complaint of blurred vision
Task
-perform physical examination on head and vision.
-Report the findings to the examination while u r examining
-Explain the findings. Give reasons; dx and ddx to patient

1-Wash hands + introduce yourself + approach with empathy like say sorry to hear you have
involved in MVA.
2-reassure patient of hemodynamic stability
3-take consent
4-ask for pain and provide painkillers if required

5-Inspection (all normal)


Face
-facial asymmetry, deformity
-scars, swelling
Eyes:
swelling
redness.
Raccoon eyes
pupil size and shape
Nose:
-obvious fracture
-depressed nasal bridge
-discharge or blood
ears:
-blood or fluid discharge
-lacerations
-battle sign (Discolouration on mastoid)
mouth:
-loss of teeth or damaged teeth
-tongue depressor for tonsils
neck and scalp:
-swelling
-bumps

6-Palpation (all normal)


always ask the patient if they feel pain at any time let you know and will stop.
-start feeling from the scalp frontal bones-temporal-orbital- zygomatic-maxillary-
mandible nasal bones.
Feel for any fractures (depressed bone), tenderness, swelling or crepitus.

-Then feel for cervical spine and Para spinal muscles (tenderness, spasm)

7-neck movements (normal)


-flexion, extension, lateral flexion, rotation)
8-Eye examination (all normal)
-visual acuity
-visual field
-light reflex
-accommodation
-eye movements (not painful and no diplopia)
-fundoscopy (examiner will give you a photo to comment on)

9-5th cranial nerve examination


-Sensation
-Motor
-Jaw Reflex
-Corneal reflex

10-Facial nerve
-wrinkle forehead
-close eyes tightly
-puff cheeks
-smile
11-conclude with all cranial nerves

Note/
- Regarding fundoscopy, one feedback said fundoscopy showed papilledema and other said it
was normal the first one got 3 in counselling the second not mentioned. However, both passed
because the examination scores were 4 and more.
-I think you need first to explain the picture to the examiner  after that talk about the findings
to the patient- complete your examination if you have time left say management, which is
admission, seen by specialist and CT scan

*Feedback of papilledema
At that moment examiner gave me the photo which showed Papilledema. Swelling of optic disc
and blurring of optic margins. Fovea and macula were normal and healthy.
The examiner told me to explain that to your patient. I told him that I looked inside his eyes and
had found some changes in there. (I pointed to them on the photo) and told that normally there
is a disc with well defined margins, but in his case, as he could see, there was a blurred margin
and that area looked raised and swollen. (This occurred due to increased pressure inside the
brain and that was why he had blurring of vision). This might be a result of his accident.
So I admitted him in the hospital where he would be seen by the specialist who would
recommend doing some tests like CT scan or MRI to look for the cause in his brain.

Note/ for this feedback, I think it is better not to show the photo to the patient as you have
examined him by fundoscopy so how you got a photo so you can explain it oral or use papers
and draw. The second thing say it might be an increased pressure in the brain.

*Feedback of normal fundoscopy


he gave me beautiful picture of fundoscopy and said read it to me I said here is the nerve,
arteries, vein and fovea all are normal there is no bleeding, no cupping then I said I need to do
eye movement the examiner didn't said anything it was normal too. he said talk to your patient
I said look I am happy all the eye examination went normal but because you have problem in
your vision I inform the specialist who will come and check you again and might be do further
Invx such as ct scan.

Feedback 14-3-2018
Post MVA with blurred vision. Tasks,
Do eye examination
Tell DX and DDX.
2 min thinking.. Inspection, palpation for signs of trauma, Eye examination. Look for signs of
cervical injury.
After introduction, I told the role player that I had read about his road accident in his notes. Was
sorry about it.
I asked if he had any pain or vision problem at the moment. He told about blurring of his vision.
I took consent for the performance of PE and asked him to bear with me as during examination I
was about to use some medical terms for the examiner.
I started with inspection of the head and face. I commented what I saw. There were no signs of
trauma or fracture on the head or facial bones. There was no drooping of eyelid or any blood in
ant chamber. No inequality in size of pupils was seen along with no abnormality in the position
of the globe. No raccoon eye. There was no discharge from the nose or any abnormality in the
shape of the nose. I asked him to open his mouth and with the torch looked into it for any loose
dentures or broken teeth or bleeding but there was none. Looked on ears for any sign of trauma
but there was no battle sign or any discharge.
I asked him about any pain in the back of his neck and he said there was none so I asked him to
bend his chin to touch his chest which he did without any pain.
I palpated his head, sinuses, face and neck for any tenderness but there wasn’t any significant
finding.
Moved on to do Eye examination. Visual acuity, Visual fields were normal. Normal
accommodation and pupillary light reflexes. There was no afferent pupillary defect. Normal eye
movements without any pain or double vision. I told him I would use an instrument, called
fundoscope, to look at the back of your eye ball. At that momet examiner gave me the photo
which showed Papilledema . I explained that to the examiner about swelling of optic disc and
blurring of optic margins. Fovea and macula were normal and healthy.The examiner told me to
explain that to your patient. I told him that I looked inside his eyes and had found some changes
in there. I pointed to them on the photo and told that normally there is a disc with well defined
margins, but in his case, as he could see, there was a blurred margin and that area looked raised
and swollen. This occurred due to increased pressure inside the brain and that was why he had
blurring of vision. This might be a result of his accident.
So I admitted him in the hospital where he would be seen by the specialist who would
recommend doing some tests like CT scan or MRI to look for the cause in his brain. And he would
be treated according to the cause. Bell rang!
Scenario … Head Injury
Grade… Pass
Global score.. 4 Key steps 1,2,3,4… yes 5.. No
Approach to patient…. 5
Choice of exam…. 4
Familiarity with test equipment… 4 Accuracy of Exam…. 4 Patient counselling… 3

Another Feedback 14-3-2018


MVA – Blurred vision – PE and DDx – the wrote Do not perform Corneal Reflex as it was normal.
Young man –said had MVA 4 hrs ago – had blurred vision – now better- asked him quickly about
his general health/past medical/SADMA – all Normal.

Then inspection (was talking to examiner while performing PE ) – then palpation for tenderness
– then VA/VF/then Ocular movement/ Light reflex – grabbed the fundoscope then examiner
said no need then handed me a big photo showing normal retina/optic disc/macula told him
that the inside of the eye was normal ( then said some medical terms to examiner like normal
fundus/macula/disc/no flame hameorrhage..etc )

Then said ideally I need to do slit lamp to check anterior chamber – examiner said we do not
have one !

Then for DDx – told patient that everything is ok – nothing to worry about – might be due to
some concussion – need to refer you to specialist and follow up again – will give you reading
material then bell rang.

Note/ look 2 feedback on the same day one papilledema and other normal. I know another one
on the same day and passed who call papilledema as well.

Case (1/3/2017)
Post MVA, exam regard to blurred vision.
Ok trauma case beautiful young girl she had MVA and complained from blurry vision. Tasks- to
check the vision I asked if the patient has any pain. look she sit comfortably, no sob, no change in
conscious then I inspect the head, eye, nose, ear, neck then I explain to her what I need to do.
today I start with VA, Pupil, VF, color vision – all normal. then I hold the ophthalmoscope I asked
to dim the light the examiner said we can't do that then I said I will adjust the light and start the
examination. the examiner said okay examination done. and the finding there.
he gave me beautiful picture of fundoscopy and said read it to me I said here is the nerve,
arteries, vein and fovea all are normal there is no bleeding, no cupping then I said I need to do
eye movement the examiner didn't said anything it was normal too. he said talk to your patient I
said look I am happy all the eye examination went normal but because you have problem in your
vision I inform the specialist who will come and check you again and might be do further Invx
such as ct scan.

comment.
I had this case and j passed this case too so its basically a man who had mva now viatlly stable
has conplain blurres vision your task is to examine him.
so starting with inspection then eye exam all components then i said i want to c back of eye and
pik up fundoscope and ask to dim light so examiner said u know its u r not going to perfom and
then he gave me. A pic of fundoscopy and asked me to explain it it was almost normal just the
margin seemed little blurred but i was confused so wasn't sure so examiner asked me leave it
tell what u going with this patient so i said im going to admit him and observe himm also eye
exam was totally normal no blurring found.
did you use snellen chart?
No it was not there so i used the paper lying there and ask him to read at arm distance.
how to start the examination?
After confirming that vitals are stable and primary and secondary survey done will start with eye
exam - will start with inpection looking for racoon eye, any obvious deformity, fractures, site of
bleeding, facial asymetry, battle sign, nasal bridge ...ENT exam , eye exam will strt with CN 2,3, 4
and 6 which will include fundoscopy...is it right ..anything to add???

136-Pleural effusion
Case 1
45 years old man complains of shortness of breath for 3 months
Tasks
-History
-PEFE
-explain the reasons for his shortness of breath

History
1-check hemodynamic stability (ask the examiner for vital signs and O2 saturation) (all normal)
note/ do not give oxygen if saturation more than 93%

2-shortness of breath questions


-for how long have you been short of breath
-has it begun suddenly? (pulmonary embolism)
-is it constant or come and go?
-is it getting worse?
-are you short of breath at rest or exertion or both? (On walking but relieved by rest)
-if on exertion ask how much distance is necessary before you get SOB?
-can you lie flat without feeling short of breath?
-do you wake up at night short of breath? How many pillows?

3-
Cough
-have you had any coughs? (Yes)
-do you cough up anything? Or you can ask dry or wet? (Dry)
-do you cough up blood? have you seen any blood in the phlegm? (TB, cancer)
-is it smelly? How much? (Bronchiectasis)
note/ if it is dry do not need to ask all about sputum.
Chest pain (No)
-do you have any pain in your chest? (heart disease, pneumonia, pneumothorax, embolism)
-can you show me where?
-can you describe it for me?
Wheeze. (No)
-any noisy breathing?
Palpitation (No)
any funny racing of the heart?
Fever (No)
-have you had any fevers? (pneumonia)
-how high is it?
-any chills or shivers? Any night sweats?

4-other Symptoms
-any ankle or leg swelling? (heart failure)
-any LOW, LOA, Lumps or bumps? (cancer)
-urine output? (kidney)
-weather preference and bowel motions? (thyroid)
-yellowish skin colour? (liver)

5-General questions
-Past medical history (HPT, DM, LIPID, Clotting)
-Past surgical history
-medications
-travel history (embolism)
-trauma (pneumothorax)
-occupation: what do you do for living and what have you done in the past (occupational lung D)
-smoking? (Cancer) alcohol? (smoker for 25 years)
-contact and family history

Physical examination findings from the examiner


1-General appearance
-cyanosis, dyspnea, oedema, pallor , jaundice, LAP
2-Vital signs and O2 sat
3-chest examination
inspection: chest movement with respiration
palpation: tracheal position, chest expansion, apex beat
percussion: dullness (dullness on right lower zone).
auscultation: air entry, breathing sounds, wheeze or crackles, vocal resonance (absent breath
sound right lower zone)
4-CVS
-heart sound and murmurs.
-JVP
5-quick abdomen
6-office tests
-UDT, BSL
7-Any X-ray available

Pleural effusion causes


Transudate
heart failure, kidney failure, liver disease
Exudate
infection (pneumonia)
infarction (pulmonary embolism)
inflammation (CTD, SLE, RA)
malignancy (lung cancer, pleural membrane cancer)
pyothorax (lung abscess)

Explain diagnosis and differential diagnosis


from history and examination. There could be several possibilities why you have SOB:
-could be pleural effusion, which is accumulation of fluid between 2 membranes surrounding the
lung. As there is some dullness and absent breath sound in the lower zone of the right lung.
-could be fibrosis, lung collapse or reduce in lung size.
-COPD smoking history
-could be pneumonia or lung infection, pulmonary embolism or clots in one of the vessels
supplying the lung, pneumothorax or air around the lung but the symptoms are for a few
months and there is no fever, cough.
-could be nasty growth or lung cancer smoking history which is just a possibility at the moment
but we need to rule out.
-could be due to heart failure but no SOB at rest or lying flat no PND, chest pain. Kidney problem
but normal urine output.

Feedback 8-2-2018

Case 2 ?????
The middle aged man has come to you because he has sob and you have the chest x-ray with AP
and Lateral view of the patient.
Tasks:
-take Hx,
-explain the x-ray to the pt
-tell dx and ddx with reasons

History
-same like above
-Positive findings
he has not travelled to anywhere since 15 years ago
He had been working in coal factory however he is now retired.
He has HTN and Hyperlipidemia for which he is taking medications (some people were shown his
medications but he did not show me his medications however they were alendronate and
perindopril and atorvastatin which were not really specific to his problem now)

Explain X-ray
-I have got your X-ray let us see it together
-there are 2 views one from the front and the other form the side
-her is your rib cage, collar bone, wind pipe, and these black shadows are air in the lungs.
-as you see the left and right lung not similar as there is large white shadow on the lower part of
the right lung
-we call this pleural effusion which is accumulation of fluid between 2 membranes covering the
lung. I can tell this from the appearance also the costophrenic angle here usually can be seen but
not in this x-ray. The midline structure also pushed out to the other side.
-the fluid can be clear, blood or pus.
-possibilities are either heart, kidney or liver problem. Could be lung infection, clot in the blood
vessel supplying the lung, inflammation or nasty growth.
-what I am suspecting is a nasty growth of the pleural membrane covering the right lung we call
mesothelioma because of long history exposure to coal..

-however we still need to arrange several Ix to reach the diagnosis


1-FBE, ESR/CRP/UCE. LFT, TFT
2-refer to specialist
3-diagnostic tap: aspiration of fluid or take a fluid sample to be examined under microscope.
4-CT scan for better view and to rule out cancer.

Feedback
Old man with SOB.u did the following xray (showing P.effusion )
Task* Take relevant focused history
*Talk about the possibilities.
The guy was very friendly ,he told me he had SoB for last ¾ months,gradually increasing
,previously could walk long now not even being able to do his own shopping,has cough,no
hemoptysis,no fever,chest pain,chills,no travel history ,contact history,long term smoking history
was there..
I talked about all the d/ds of PE esp focused on lung CA with lots of reassurence.
Feedback:failed ! with 2 in history and diagnosis and d/d!
Have absolutely no idea why!

Case (13/10/2017)
The middle aged man has come to you because he has sob and you have the chest x-ray with AP
and Lateral view of the patient.
Tasks:
take Hx,
explain the x-ray to the pt
tell dx and ddx with reasons.

I went in introduced myself. The RP was nice middle-aged man and I think he looked like a real
patient.
(PMH lung problem)
- He had no hx of lung disease.
(SOB questions)
- His sob is getting worse while he was walking.
(lung cancer)
- No cough, no cp
- no LOW and LOA.
(pulmonary embolism)
- no hx of travel to overseas (he has not travelled to anywhere since 15 years ago)
- no leg(cuff) pain.
(lung infection)
- no hx of contact to anybody with viral infection or respiratory infections.
(smoking and alcohol)
- no smoking, no alcohol,
(heart failure)
- he doesn't need pillow while sleeping ( normally use 2 pillow but it is not increased)
- no sob during sleep, - no leg swelling,
(waterwork and bowel motion)
- no problem with waterworks and BM were present.
(occupational lung disease)
- He had been working in coal factory however he is now retired.
(PMH)
He has HTN and Hyperlipidemia for which he is taking medications (some people were shown his
medications but he did not show me his medications however they were alendronate and
perindopril and atorvastatin which were not really specific to his problem now)
(medication history)
he is regular with his medications.
(Family history)
No family hx of lung cancer.

(X-ray explanation)
- I said I have your cxr in my hand these are 2 views of your chest taken. One is from front and
the other from lateral.
- These are your shoulder, rib cage and collar bones.
- Black area is normally due to air in lung.
- This whitish area you can see is abnormal due to its specific pattern.
- There are some LN which are responsible for fighting against the bugs in the helium part of
your lung (showed the area) which is enlarged as well.
- This can be due to some conditions like tissue disorders called sarcoidosis.
- But what I am thinking of is Mesothelioma as you have being exposed to coal previously so
there are some membranes around your lung called pleural membrane which are affected in
this case due to that lack of sharpness in this area (showed on pic).
- Normally there is an angle at the base of lung which is called costophrenic angle which in your
case is not that sharp that we expect it to be and it is blunted actually. It can be due to nasty
growth there or accumulation of fluid or blood which can be due to infection or mesothelioma
or sarcoidosis and so on.
Feedback: SOB, PASS(G.S:4) Key steps:1 No2 Yes ,3 No 4 Yes
Approach to patient:6 Hx:4 Interpretation:4 Dx/DDx:4

Another 2018 recalls similar


7-3-2018 SOB working in timber industry.
comment pass
Its was a DD case , the roleplayer and the examiner were nice , he had sob for 3 month continuous
progressive , dry cough, no wt loss or fever or...all the rest of Hx normal only for Fx of COPD
PE
No sign of copd , CHF, Anemia, but there was dullness (not stony)in the right lower lobe ..
Give dd
Copd , ILD , Effusion do to systemic or local lesion ..
Pass

8-3-2018 Old person with Sob lt lung opacity with steel industry exposure
comment pass
- Had this case in my exam
- Was it mesothelioma or ca lung ?
- DDx with reason I think or DDx but yeah he has history of hypertension as well
- And work in steel factory
- The X-ray was a DDx X-ray ( some consolidation and slight pleural effusion
- Anterior/ posterior and lateral X-rays were provided
- My performance was bad
- Still passed with only 2 key steps covered.
- Can u give a bit details dear?did ask mx?
- did u specically say mesothelioma?
- No management
- All but 1 case had management
- Not so I said this can be due to you underlying hypertension or a bug infection etc
But what I’m concerned about is your work history and this can be a nasty growth we call
mesothelioma.
- Reading material, Guarded reassurance ( further tests required) that’s all
- Didn’t think I would pass this case but realised later it was an actually easy case the key is the
history, if well prepared you can finish this case in 5 mins and get global score of 6 or 7.
- So if you ask PMHx he talks about hypertension that’s all but I had to stop him and ask about work
history etc travel / contact history , fever , appetite / loss of weight etc
- First time, during exercise, at night or morning , how many pillows do you use etc any trauma ,
relieving and aggravating factors.
- You can even ask more in 5 mins
- And X-ray will take u 30 seconds to explain quickly and and DDx 1 min
- You can say DDx while reading the X-ray.
- Nope the detrusor instability one was and the U/S + mittelschmerez one also
- how did u correlate underlying htn with opacities???/
- DDx case , correlates withthe pleural effusion the heart on the X-ray looked normal to me but fellow
candidates said there is cardiac hypertrophy and didn’t mention nasty growth just mention we need
to do more investigations and passed

Feedback 18-9-2018 Pleural effusion


SOB
Male complain of SOB for a few months, chest X-ray given outside, PA view and lateral view.
Task: HX, explain X ray, Dx/D/D
Hx: works in motor vehicle company, use a number of medication, has hypertension. Sometimes had
leg swelling. Always sleeps on a few pillows but never wake up from sleep or has to sit up because of
shortness of breath.
very confused history.
Xray: pleural effusion, pleural thickness
D/D: I gave all the DD about pleural disease related to occupation
Pleural effusion: infection, nasty growth, heart failure, liver and renal disease.
Score: 6
Key step: yes/yes/no/yes
Approach: 4, Hx: 5, Interpretation of Ix: 5, Dx/D/D: 6

Feedback 18-9-2018 Pleural effusion


The next pt in your gp clinic is a 42 year old man who complains of SOB which is
increasing day by day. He has SOB during walking and doing day to day work. He has
leg swelling as well but no calf muscle tenderness. You ordered chest Xray for this
pt and the illustrations ( PA & Lateral view) are attached below.
Your task is to
1. Explain the Xray to the pt
2. Take focus history to find out cause for this presentation
3. Discuss Dx and DDx with reasons with the pt
My approach:
The Xray was Pleural effusion in the left lung. So I explained that first. Then I started taking
history. He was a motor vehicle worker, he is hypertensive, on medication history he gave a list of
medication in paper containing Aspirin, Hydrochlorthiazide, Enalapril, Esomiprazole,
Atorvastatin. He has no travel hx, no family hx , no Asthma or COPD, no diabetis, NO recent
infection, SADMA was normal. He has night symptoms and use 2 pillows at night but he never woke
up from sleep due to the symptoms.
So I gave Dx as pleural effusion due to Heart failure
My DDx were Nasty growth, infection, liver disease, renal disease
AMC feedback:
Station: 19
Scenario: SOB
Grade: Pass Global score: 4
Key steps: 1,2,3,4 no,yes,no,yes
Assessment domain:
Approach to pt/relatives: 4
History: 4
Interpretation of investigation: 5
Dx/DDx: 5

137-Occupational lung disease


70 years old man with SOB ,he was seen by a doctor and came back with chest x rays, both AP
and lateral.
Tasks
-explain the x rays,
-history
-possible dx and ddx

I can see from the notes that you feel SOB. Do you feel comfortable now?

Explain X-ray
-I can see that chest x-ray has been done for you and you are here for the results. Let us see it
together and if you have questions at any time please let me know?

-this is the x-ray of your chest. These are two vies; this is taken from the front and the other from
the side. The x-ray is showing a whitish shadowing surrounding the right lung field while the left
lung field is clear. In addition, it showed that the windpipe is deviated towards the right side (if
you really see it mention it)

-there are several possibilities it could be pleural effusion or accumulation or fluid around the
lung but it is unlikely as the windpipe in that case should be pushed to the other side. It could be
infection, pneumonia but the windpipe usually in centre in such cases.
-what I am suspecting is due to either collapse or fibrosis of the right lung with reduced and
diminished lung volume or size.

-However I still need to ask you a few questions in order to know more about the diagnosis is
that ok with you?

History
1-SOB questions.
-for how long? (2 months)
-has it started suddenly or gradually? Constant or come and go? Is it getting worse?
-are you short of breath on exercise or rest or both? (on walking)
-how much distance is necessary to get SOB? (the distance gets shorther)
-do you feel SOB on lying flat?
-do you wake up at night short of breath?
-is this the first time?
2-Associated symptoms questions
-do you have any cough? (Yes)
for how long?
Do you cough up anything? (No its dry) do you cough up blood?
-any fever or night sweating?
-any chest pain, funny racing of the heart or leg swelling?
-any LOW, LOA, lumps or bumps?
3-General questions
-PMH (DM, HTN, lipid, heart D. Lung D.)
-PSH (ca prostate surgery) do you have regular checkups (yes)
-do you smoke? (Smoking 40 years 20 cigarettes per day ,stops for 5 years)
-do you drink alcohol?
-do you take any medications or OTC? (take ACEI, Ventolin, steroid inhaler) what would be used
for?
-have you travel recently?
-any trauma or injury to chest?
-any contact with patient with similar symptoms?
-occupation: what do you do for living and what have you done in the past? (he is retired but
was working as a boiler for 30 years)
-family history heart and lung disease

Diagnosis and differential diagnosis


-from history and x-ray findings you most likely have an occupational lung disease. Which is
repeated and long-term exposure to certain irritants in the job that can lead to a range of lung
diseases that may have lasting effects even after exposure ceases.

-from history you were working as a boiler for 30 years in the past, so you most likely exposed to
an inorganic dust particles called asbestos leading to asbestosis; which is a condition caused by
inhalation of microscopic fibers of asbestos.

-other possibilities could be nasty growth of the lung or mesothelioma; is a cancer affecting the,
the pleura or the outer membrane of the lung. Do not worry I am just telling possible causes it
does not mean you have it, we still need Ix to be done.
-Secondary spread of cancer cells from other organs like the prostate although this is less likely
from the history and you have regular checkups.
-Could be pleural effusion, COPD.
-others could be infections or heart problems but less likely.

Feedback 31-5-2018
70 years old man with SOB ,he was seen by a doctor and came back with chest x rays, both AP
and lateral.
Tasks
-explain the x rays,
-history
-possible dx and ddx

*Picture is resemble as shown in the comment, but trachea is deviated to Rt side mildly. (the Rt
side lung is surrounded by white area from top to bottom, I was also thinking pleural effusion
but no tracheal shift to opposite side,it should be deviated with that large amount of fluid,left
lung quite normal)
[because of trachea shifted to same side mildly I chose pulmonary fibrosis or collapse]

Patient is sitting
Greet the patient and ask vitals-examiner said u can proceed,I said I am sry about his SOB,asked
is he ok with that position.let me know if u have any discomfort during conversation.
I explained X rays like fibrosis and collapse.he is nodding his head and listening carefully.

On history he said SOB 2 months(I felt relief because of that duration).he mentioned SOB worse
on exertion and walking distance becomes shorter and shorter,No SOB at rest,no orthopnea ,no
PND, dry cough present,no wheezing, no fever,no LOW,no LOA,no lumps or bumps,no night
sweat,no chest pain. no history of travel,no leg swelling,

Smoking 40 yrs 20 cigarettes per day ,stops for 5 years ,(appreciate him for stopping)no
alcohol,no drug allergy.
Medical history HTN I asked what medication (he looked at the examiner and gave me the
medication card,ACEI,ventolin and steroid inhalers)I asked those inhalers help him,he said
sometimes.
Surgical history-history of CA prostate got surgery,he said it is ok now.(I asked him do u get
regular follow up,he said yes,how long ago,he looked examiner,so I realized he has no answer so
I said it is ok if u have regular check up.) Occupational-retired,when I asked previous job-boiler
worker for 30 years.

Explained-occupational related lungs problem,because of longstanding inhalation of dusts and


chemicals.He asked what kind of Chemicals-(I did not expect like that,so I chose Asbestos and
told him,I was also thinking silicosis in my mind,but patient agreed with me oh,ok,ok asbestos).
so I followed him like,as you know at your time ,the rules and regulations about occupational
health was not not fully developed. 

I also give COPD,lung cancer,Mets from prostate but unlikely because of regular check up,heart
failure,chest infection,chronic PE. When I said lung cancer ,he raised up his eyebrows ,I said
,please don’t worry I am just telling possible cases,it does not mean u have it,we still need to do
many investigations to confirm your problem.i will refer you to specialist.
Keys 4/5 Approach,Interpretation of investigations,history,dx and ddx 5,5,4,4 Global 4

138-Back examination
Examination steps (In general)
1-ask pain and painkillers
2-Gait
-assess walking
-walking on toes (S1)
-walking on heels (L4-L5)
-squatting (L2-L3)
3-Look
-Behind: scar, skin colour changes, swelling, deformity (scoliosis)
-lateral: lordosis
-Front: pelvic tilt
4-Feel
-Tenderness
Vertebra R+L paravertebral muscles both sacroiliac joints.
(Spasm or disc prolapse can cause pain on the affected side)
5-Move (for pain and limitation) (do active only)
-flexion
-extension
-Right + left lateral flexion
-right + Left lateral rotation
6-Lower limbs (roots only)
-sensation (cotton wool only)
*L2= Upper lateral thigh
*L3= medial side of knee
*L4= medial side of leg
*L5= big toe
*S1= little toe.
-Power (resisted movement only specific not all)
*L2= hip flexion
*L3= knee extension
*L4= ankle dorsiflexion
*L5= big toe extension
*S1= plantar flexion
-Reflexes: knee (L3-L4) and ankle (L5-S1)
7-Special tests
-SLR
*patient lying down
*lift the straightened leglimited by pain (< 60 degree).
-Schober (optional)
*identify post iliac spine and put a mark then measure 5 cm below then 10 cm above.
*ask patient to touch toes- if <20 cm this means restricted ROM.
8-PR examination with consent for anal tone and cauda equine lesion

Differential diagnosis (could be)


1-mechanical back pain: spasm of muscles of the back due to abnormal posture or heavy lifting.
2-disc prolapse: when the Cushing between the backbones get out of its position causing
compression on nerves passing on sides.
3-trauma
4-discitis (inflammation or infection of the intervertebral discs)
5-osteoporosis

Notes/
-Exam cases
Case 1
a worker comes to your GP c/o back pain. Started suddenly few minutes ago after lifting a heavy
box. Has not have any problem before.
Task:
1- examine him(with running commentary)
2- explain the most probable diagnosis for the patient

Case 2
Young man, sudden onset of low back pain while lifting heavy weight, pain radiating down the rt
thigh and leg.
Tasks:
-P.E.
-Give DDx

-if the case was just mechanical back pain then finding may be
* gait painful and limited.
*there will be tenderness +ve on the left or right back side.
*limitation and pain of movements may be all will be +ve .
*neuro all normal.

-if the case was Sciatica or S1 radiculopathy then may be


* gait painful and limited.
*there will be tenderness +ve on the left or right back side.
*limitation and pain of movements may be all will be +ve .
*decrease sensation in S1 root (lateral side of foot)
*weak plantar flexion (when you ask the patient to push foot down against your hand)
*absent or weak ankle and plantar reflexes
*SLR will be +ve

-In case of L5 radiculopathy then may be


* gait painful and limited.
*there will be tenderness +ve on the left or right back side.
*limitation and pain of movements may be all will be +ve .
*decrease sensation in L5 root (middle 3 toes)
*weak big toe extension (when you ask the patient to push big toe up against your hand)
*reflexes all normal
Feedback 15-3-2018 unknown status (Sciatica)
back pain examination. Dx and ddx.
Back exam—look , feel , move and special tests and neurology
+ve S1 sensation loss and SLR +ve, protective posture +ve, restriction of ROM , Mentioned
acute mechanical back pain with disc prolapse as likely cause

Feedback 5-4-2018. (mechanical back pain)


Mechanical low back pain GP, 27 years old patient c/o low back pain, which radiates to LL after lifting
heavy weight.
Perform PE, Dx, Ddx.
1.PE ruling commentaries to examiner ( it was knee hummer and turning fork on the table
examiner told me vibration, propioseption and sensation are intact, pte complaint of pain on
walking and ROM and doing SLR even when i was lifting the leg but no pins and needles)
2. DX and DDX to pte

Feedback 21-7-2018 Back pain – PASS (G.S- 5)


Young man, sudden onset of low back pain while lifting heavy weight, pain radiating down the rt
thigh and leg.
Tasks:
-P.E. -Give DDx
-The pt was standing taking support of the table. Mentioned it. Offered pain relief, sympathy.
Performed full examination of back as in Geeky medics, lower lumbar spinal and paraspinal
tenderness, he was exxxxtremely slow, restricted movts due to pain, still tested all movts,
mentioned that all movts restricted due to pain(didn’t do the Schobers test as it was irrelevant
and time consuming) Straight leg raise test positive rt side. Lower limb neurological exam- the
examiner ran me through everything saying ‘normal’ as and when I started tone…power…
sensation…allowed me to do reflexes.
-DDx- IV Disc Prolapse, paraspinal spasm, sciatica- nerve impingement (drew pic), fracture,
tumour

Case (27/4/2017)
LBP a/f lifting box.
Perform pe, explain the possible causes to pt. *PF *(pt can't walk on toe and pain in all
movement, SLRT positive on rt side. Jerk, sensation and PR are nl)
i explained pt about the condition. it could be due to mechanical low back pain that muscle at
your back got strained due to sudden weight lifting.
i drew a pic about back bone that it could be due to prolapsed intervetebral disc. it could be due
to bone break or injury but less likey.
most likely mechanical low back pain, no involvement of nerve.
eventhough, there was no mx task, i explained about some mx like PRICE and regular review.
give reading tips and reassure.

FB-PASSED, OVERALL-4, APPROACH-4, EXAMINATION SEQ-5, TECHNIQUE-4, DDX-4


139-Hip Examination
Case 1 (Hip osteoarthritis Karin)
A 64 years old man with a history of pain on his right hip joint for the last 6 months comes to
your GP clinic. The pain is worse with activity. He tried Panadol but did not get relieved from the
pain.
Tasks
0relevant hip examination+ running commentary
-diagnosis
-management

Differential diagnosis
1-osteoarthritis
2-rheumatoid arthritis
3-Trauma
4-greater trochanteric bursitis
5- avascular necrosis of femoral head

Examination steps
1-Gait
-if you could walk to the other side of the room for me the turn and walk back. Look for:
antalgic gait (painful gait or limping) (+ve on the right side)
Trendelenburg gait (+ve on the right side)
-Now walk on your toes and then on your heels (stand beside the patient) (unable to walk heels
or toes.)

2-Trendelenburg test
-I am just gonna assess the muscles surrounding your hip joints.
-if you could stand up straight for me
-I am gonna stand behind you and just feel the muscles of your hip.
-if you could lift your left leg for me (the unaffected one): test is +ve if the pelvic drop on the side
with the foot off the ground
(Trendelenburg test +ve on the right hip)
Note/ for more understanding this patient has pain on the right hip so the problem is in the right
hip joint. So the only support for walking is his left hip. When you asked the patient to lift the
unaffected leg off the ground which means the left side then there will be no support and the
pelvic will drop towards the left or the normal side; which means the right or the affected side
elevate while the left or the unaffected side drop. But when you say +ve need to give where the
problem is; mean for the right hip or right hip problem.

3-Look (good exposure standing front and back) (usually –ve)


-Scars
-skin colour changes
-swelling
-deformity (fixed flexion)
-muscle wasting
4-Thomas test (Fixed flexion deformity +ve on the right side)
-put your hand behind the back of the patient (the lumbar area)
-if you could bring your left knee as far to the chest as possible and hold it afterwards.
-test is +ve when the extended leg is lifted off the table
-test the other leg.
Note/ in this test you are looking for fixed flexion deformity that might be visible on inspection
but sometimes not. So here ask the patient to lift the normal side here the left one all up and
hold it by his hands. Then notice the right leg if they elevate or raise then the test is +ve as in this
case)

5-Feel (for tenderness 4 points)


-ASIS
-femoral head (distal to the mid point of inguinal ligament) (+ve in OA)
-adductor tendon (inner side if the thigh adductor tendon) (+ve in adductor tendonitis)
-greater trochanter (+ve in trochanteric bursitis)
(+ve tenderness on right femoral head)

6-Move (6 movements)
-Active flexion (+ve limited)
-passive abduction
-passive adduction
-passive external rotation; (when foot medial)
-passive internal rotation; (when foot lateral) (+ve limited)
Note/
-when you do passive abduction or adduction; put one hand on the heel and the other hand on
ASIS.
-when you do ER or IR; put one hand on knee and the other hand on foot on flexed leg.

7-Measure (true and apparent)


-true leg length (+ve short right leg)
-apparent leg kength (normal)
Note/
true leg length= from ASIS to medial malleolus)
apparent leg length= from umbilicus to medial malleolus.
Difference in true leg length indicates hip disease on the shorter side
Difference in apparent leg length indicates tilting of the pelvis.

8-Special tests
-SLR test (+ve) (<30 degree joint/ >30 degree sciatica)
-FABER test (to rule out SIJ pain) (+ve).
Flexion of the knee then abduction then external rotation= figure of 4)
*put the R leg on the knee of the left leg
*apply pressure on the right leg as to push it down while one hand on the opposite ASIS.
*+ve test is when there is pain or discomfort: if on hip= OA/ if on back=sacroiliac joint
*test the other side

9-Conclude with joint above and joint below.


Diagnosis and management
-most likely have OA of the hip. It is resulting from wear and tear as a result of excessive use of
your joint over the years or due to old injuries in the affected joint.
-the cartilage that covers and protect the end of the bone gradually wears away causing the joint
to become rough and stiff.
-most cases are mild and with treatment you can cope with it well.
-other possibilities are trauma, RA on inflammation of hip joint due to autoimmune disease. No
enough supply to head of thighbone and etc…

Investigations
-I will arrange an X-ray of hip joint

Treatment
1-relative rest during acute pain
2-painkiller
3-crutches
4-weight loss
5-refer to physiotherapy for strengethening of mucles
6-occupational therapist for walking aids
Case 2 (Adductor tendonitis Karin)
your patient in GP is a 20 year old man complaining of pain in the right groin.
Tasks
-Physical examination
-diagnosis and management

Differential diagnosis
1-adductor tendonitis
2-osteitis pubis
3-hernia
4-trauma
5-OA
6-refered pain from lumbosacral spine

Examination steps
1-Gait
-if you could walk to the other side of the room for me the turn and walk back. Look for:
antalgic gait (painful gait or limping) (-ve)
Trendelenburg gait (-ve)
-Now walk on your toes and then on your heels (stand beside the patient) (-ve)

2-Trendelenburg test (-ve)

3-Look (good exposure standing front and back) (–ve)


-Scars
-skin colour changes
-swelling
-deformity (fixed flexion)
-muscle wasting

4-Thomas test (-ve)

5-Feel (for tenderness 4 points)


-ASIS
-femoral head (distal to the mid point of inguinal ligament) (+ve in OA)
-adductor tendon (inner side if the thigh adductor tendon) (+ve in adductor tendonitis)
-greater trochanter (+ve in trochanteric bursitis)
(+ve tenderness on right medial groin)

6-Move (6 movements)
-Active flexion
-passive abduction
-passive adduction (+ve painful)
-passive external rotation; (when foot medial)
-passive internal rotation; (when foot lateral)
7-Measure (true and apparent) (-ve)
-true leg length
-apparent leg length

8-Special tests
-Squeeze test (+ve)
*ask patient to flex both knees and hips
*put your fist between flexed knees
*ask patient to push on your fist.
*+ve test when there is pain.

9-Conclude with joint above and joint below.

Explain diagnosis and differentials


-most likely have a condition called adductor tendonitis also called groin strain which is when the
tendons near the groin become stressed and tensed and can tear causing pain.
-common in sport activities.
-other possibilities are……

Investigation
X-ray and USD to rule out pain from bone or tendon.

Treatment
-avoid activity until pain settled
-apply ice every 3-4 hours for 20 minutes until pain free
-compress the thigh with help of elastic bandage or tape.
-Pain killers
-physiotherapy
-not relieved- steroid injections
-come back if pain persists.
Case 3 (Trochanteric bursitis AMC case)
A 45 year old female complaining of pain in the right outer hip that travels down her leg since
last week.
Tasks
-Physical examination
-diagnosis and management

Differential diagnosis
1-trochanteric bursitis
2-osteoarthritis
3-Trauma
4-iliopsoas tendinitis
5-avacular necrosis of femoral head
6-lumbar spine radiculopathy

Examination steps
1-Gait
-if you could walk to the other side of the room for me the turn and walk back. Look for:
antalgic gait (painful gait or limping) (+ve)
Trendelenburg gait (+ve)
-Now walk on your toes and then on your heels (stand beside the patient)

2-Trendelenburg test (+ve)

3-Look (good exposure standing front and back) (–ve)


-Scars
-skin colour changes
-swelling
-deformity (fixed flexion)
-muscle wasting

4-Thomas test (-ve)

5-Feel (for tenderness 4 points)


-ASIS
-femoral head (distal to the mid point of inguinal ligament) (+ve in OA)
-adductor tendon (inner side if the thigh adductor tendon) (+ve in adductor tendonitis)
-greater trochanter (+ve in trochanteric bursitis)
(+ve tenderness on greater trochanter)

6-Move (6 movements)
-Active flexion
-passive abduction (+ve painful)
-passive adduction
-passive external rotation; (when foot medial)
-passive internal rotation; (when foot lateral)
7-Measure (true and apparent) (-ve)
-true leg length
-apparent leg length

8-Special tests
-SLR (-ve)
-FABER (-ve)
-Squeeze (-ve)

9-Conclude with joint above and joint below.

Explain diagnosis and differentials


-most likely have a condition called trochanteric bursitis which is hip pain caused by
inflammation of fluid filled sac called bursa on the outer edge of the hip.
-can results from falls, trauma, overuse.
-other possibilities are..

Investigation (to rule out other possibilities)


-X-ray and USD

Treatment
-you need to rest and reduce activity for a few days.
-put ice pack on the painful side
-painkillers to relieve the pain.
-try to avoid sleeping on the affected side
-you can use a small pillow to elevate the involved area.
-refer you to physiotherapist for strengthening exercises.
-Lifestyle modification
-if the pain is sever or affected your daily activities- refer for steroid injection
140-Lung cancer
72 yr old lady , history of living in nursing home , present with SOB and LOW and cough with
blood streak , Your colleages did examiantion …. Crepitaion in lung , Spo2 92% , other vital
normal, X ray given in stem ( left upper lobe opacity )
Tasks
-History (2-3 minutes)
-Explain the x ray to patient
-DDx based on the x ray findings

Patient was on the bed , with nasal canula , pretending getting oxygen

History
1-Check hemodynamic stability (vital signs and O2 saturation) (same as the stem)
introduce yourself and say let me assure you are stable I have just checked your vitals and all are
fine. I just need to ask you a few questions if you feel uncomfortable anytime please let me
know.

2-shortness of breath questions


-for how long have you been short of breath (for a few months now)
-has it begun suddenly? (Gradual)
-is it constant or come and go?
-is it getting worse? (Worsening over 3 weeks)
-are you short of breath at rest or exertion or both? (worse with activity)
-if on exertion ask how much distance is necessary before you get SOB? (Walking distance is
reduced)
-can you lie flat without feeling short of breath? (no orthopnea)
-do you wake up at night short of breath? How many pillows? (No PND)
3-symptoms related to chest questions
Cough
-have you had any coughs? (Yes)
-How long? (For a few months now)
-do you cough up anything? Or you can ask dry or wet? (Productive of yellow sputum)
- have you seen any blood in the phlegm? (lately blood streak in the sputum)
-is it smelly? How much?
Chest pain (No)
-do you have any pain in your chest?
Wheeze. (No)
-any noisy breathing?
Palpitation (No)
any funny racing of the heart?
Fever (No)

4-Symptoms not related to chest questions


-any ankle or leg swelling? (No)
-any LOW, LOA, Lumps or bumps? (yes weigh loss and loss of appetite) how many Kg? (10)
-any change in your voice or hoarsness? (No)
-any weakness, tingling or numbness? (No)

5-General questions
-Past medical history (HPT, DM, LIPID, Clotting) (No)
-Past surgical history
-medications
-travel history (No)
-trauma
-occupation: what do you do for living and what have you done in the past (used to work as
Retail for many years)
-smoking? (Cancer) alcohol? (Used to be heavy smoker but stopped 5 years ago) appreciate that.
-contact and family history (No)

Explain X-ray to patient


- you can see this is the x-ray of your chest, these are your collar bones, ribs, chest bone,
windpipe and your heart.
-the blackish grey area on each side of your heart is your left and right lungs.
-on comparing right and left lung, we can see white rounded shadow in left top part of lung , this
is abnormal findings. It is most likely the cause of your symptoms.
-otherwise your heart size is normal, your lungs are increase in size but I don’t see any
abnormality in other parts of your lungs.
Explaining possible dx and ddx
-well from history, examination and x-ray findings there are few possibilities why you have
weight loss, coughing red phlegm and SOB.
-Could be infection; a bacterial infection in the lung but less likely as you don’t have any fever
-Tuberculosis infection which is possible cause we need to rule out , however it is rare in
Australia , you don’t have travelling history and contact history , but as you are living in nursing
home , we need to consider it as well.
-what I am suspecting these symptom could be due to a lung cancer but we still need to do
Investigations for definitive diagnosis.

-Other possible causes are Benign or non cancerous growth in lung but very less likely
-it could be pus collection in the lungs we called abscess , but again less likely.
-could be a blood clot blocking in blood vessel of lung but your symptoms are not acute onset , it
is unlikely.
- I am also thinking about COPD , which is inflammation and narrowing of airways strongly
associated with smoking , it may coexist but its not the main cause of your symptoms
-other cause of SOB like heart condition , anemia are less likely , but I can do further testing to
rule them out

Feedback 17-8-2018
Station-1(pass)
Weight loss,Task-hx,explain x ray,dx,dd)
Hx-lung cancer all positive finding given in the stem with pe findings of lung,hemoptysis,w toss
10 kilo in 3mnths,sob positive,no chest pain,no other medical condition,living innursing home for
10 yrs,single mother,no fhx of cancer,lost appetite,no leg swelling,no lump bump,no travel hx

Xray-whitish area in the upper left lobe where lung dullness is present,people said
costo/cardiophrenic angle was lost due to mild fluid collection,I didn’t notice that,I said in lay
term this is ur wind pipe,this is collar bone,these are chest bone,2 lung shadows,heart shadow
and this is tummy,here in ur case black shadow turns whitish could be due to pneumonia(bug in
lungs) or TBwhich is bug also bt no travel hx,fluid or nasty growth that means cancer,(pt told me
good….i was like u have cancer u r saying good!!she was happy with my dd or performance may
be ),I gave all other sob dd too-heart ,lung,anxiety,anemia,injury all…
Feedback 5-4-2018
lung cancer x ray given long stamen. old lady moved into a nursing home 12 months ago. had
lost 10 kg in this 12 months now blood on sputum, smoking +ve, osteoatritis taking painkillers,
no travel hx, no fx hx as she was adopted (sorry i can’t remember anything else)
Task: 1. Ask focus hx for 4min
2. explaining the X-ray (bad quality no well differentiated mass, more like diffuse infiltrated in the
apex similar to this one.
3. Dx and Ddx to pte

Feedback 17-8-2018
STATION 1 – WEIGHT LOSS
An old lady living in age care facility has presented to you with history of 10 kg wt loss, there was
hist of red sputum and on examination Rt upper lobe crackles were present, patient was smoker
5yrs ago but now stopped.

Your task is to take a short history no more than 2 3 min, explain invest. to pat, tell the dd’s to
the patient.

I started taking history about presenting complaints and ruled out all possibilities i.e lung ca,
pneumonia, previous lung disease,T.B, pulm. Embolism ( because of blood in sputum) cardiac
failure(because of pink sputum and sob.so I took all recent travel history , occupational history
and asked about where she lived previously to rule out any exposure to chemicals. Took history
of smoking but she quitted 5years ago. No travel hist was positive. No exp of chemicals at work
or residential area. No fever no rigors or chills to support pneumonia or T.B. No long travel ,no
recent major surgery . no SOB while lying down ,no extra pillows needed at night. No other
significant things positive apart from wt loss of 10 kg over last 3 to 4 months. History of COPD
was positive previously.

Then I took the x ray from the examiner and explained it to the pat by showing that you can see
this is your x ray of lungs ,here you can see your collar bones, here is your wind pipe which is in
the centre, here are your lungs which are blackish gray but here you can see on the rt side on the
top there is whitish infiltration and we have examined you and found some abnormal breath
sounds over this area. Here you can see the blood supply of your lungs and rest of lungs fields
look normal.

Then I started giving dd’s according to priorities in my mind at that time

It could be some nasty growth in your lungs , like ca but this is just my suspicion we have to do
further investigations and you will be seen by the specialist to counter check all the things I have
done, it could be pneumonia as she was in age care( gave the reasons why its less likely),
T.B,chronic lung disease(interstitial lung disease)Pulm. Embo, cardiac failure(gave the reasons
why less likely all of them.
In this station I covered 4 out of 5 key steps
Approach to pat 4 Hist 4
Interpretation of invs 4 Dds 4 Global 4
Feedback 20-7-2018
Weight loss pass Global score 6
approach to patient 5 history 5
interpretation of investigation 7 Dx/DDx 7
72 yr old lady , history of living in nursing home , present with SOB and LOW and cough
with blood streak ,
Your colleages did examiantion …. Crepitaion in lung ( not mentioned site in stem)
Spo2 92% , other vital normal
X ray given in stem ( left upper lobe opacity )
History (2-3 minutes) , Explain the x ray to patient , DDx based on the x ray findings

Patient was on the bed , with nasal canula , pretending getting oxygen. i asked examiner to
check vital sign , said the same as the stem .....
told the pt if anytime she is not comfortable during consultation , please let me know
history taking .... SOB for a few months now worsening over 3 wks , worse with activity
,walking distance also reduced , no PND , no orthopnoea , no leg swelling , no chest pain ,
no palpitation , cough for few months now , productive of yellow sputum lately blood
streaks in sputum , no wheezing , wt loss + , apeptitie loss + , no lumps and bumps in the
body , no one comment that you look pale , no blood loss , no change in voice , no weakness
or tingling and numbnesss of finger , no history of travelling to oversea , no contact with ppl
who are coughing , no blood clot problem in the past or family history ,
SADMA used to be heavy smoker , stopped 5 yrs ago , i appreciated, no alcohol drinking , no
Recreational drugs, not on any medication , Past medical history - no HTN , no DM , no ht
disease , PSH - nil occupation . used to work as Retail for many years
I explain the x ray to pt , lets look at the xray , the white shadow in the middle is your heart
, the black area on each side of your heart is your right and left lungs , these are your ribs. on
comparing right and left lung , we can see white rounded shadow in left top part of lung ,
this is abnormal findings , most likely the cause of your symptoms , other wise your heart size
is normal , your lungs are increase in size ,( might be related to smoking you had before ) but
i dont see any abnormality in other parts of your lungs
( examiner came behind me while i am explaining the x ray )
there are few reasons the can cause this ....
like infection bacterial infection in the lung but less likely as you dont have any fever
Tuberculosis infection which is possible cause we need to rule out , however it is rare in
australia , you dont have travelling history and contact history , but as you are living in
nursing home , we need to consider it as well.
i am afraid , i am concerned it might be the nasty growth in lung ( patient asked what is
it ) , it means cancer in the lungs , patient pretend to worry, but it is my working
diagnosis , we are going to help you , you dont have to go thr this alone ,
other possible causes are B9 growth in lung but very less likely ( pt asked what is B9) i said
the growth that is not cancerous , it could be pus collection in the lungs we called abscess ,
but usually it wd be in lower part of lungs , so it less likely. , i think about blood clot blocking
in blood vessel of lung but your symptoms are not acute onset , it is unlikely.
i also think about COPD , which is inflammation and narrowing of airways strongly
associated with smoking , it may coexist but its not the main cause of your symptoms
other cause of SOB like heart condition , anemia are less likely , but i can do further
testing to rule them out
Time left .... so i talk further testing like CT scan , admit and review with specialist( even
tho its not asked) , biopsy
141-Facial trauma
A young female came to ED where you are working as an HMO after she had MVA. She was
sitting in the passenger seat and had her seat belt on. She had injury in her face. Patient is now
hemodynamically stable and neck clear.
Tasks
-examination
-diagnosis and differential diagnosis

1-Wash hands + introduce yourself


2-reassure patient of hemodynamic stability
3-take consent
4-ask for pain and provide painkillers if required

5-Inspection
Face
-facial asymmetry, deformity
-scars, swelling
-in the exam there is bruising on the left side of the cheek!
Eyes:
swelling
redness.
Raccoon eyes (picture provided)
pupil size and shape
Nose:
-obvious fracture
-depressed nasal bridge
-discharge or blood
ears:
-blood or fluid discharge
-lacerations
-battle sign (Discolouration on mastoid)
mouth:
-loss of teeth or damaged teeth
-tongue depressor for tonsils
neck and scalp:
-swelling
-bumps

6-Palpation
always ask the patient if they feel pain at any time let you know and will stop.
-start feeling from the scalp frontal bones-temporal-orbital- zygomatic-maxillary-
mandible nasal bones. (Tenderness in lower orbital margin)
Feel for any fractures (depressed bone), tenderness, swelling or crepitus.

-Then feel for cervical spine and Para spinal muscles (tenderness, spasm)
7-Eye examination (do as Amaurosis fugax lecture)
-visual acuity
-visual field
-light reflex
-accommodation
-eye movements (in exam there will be diplopia on upward +/- lateral gaze and painful eye
movement)
-fundoscopy

8-5th cranial nerve examination


Sensation
-now I’d like to test the sensation in your face. First of all I’ll start with this cotton wool then
neuropin would that be ok?
-this is what the cotton feels like. If you could just close your eyes and let me know when you
feel it. (temporal - zygomatic-chin both sides)
-now with this pin it feels like this.
Motor
-now I’m gonna feel some of the muscles around your face
-clench your jaw for me (temporalis and masseter)
-open your mouth and do not let me close it (weakness or deviation)
Jaw Reflex
-now I’m gonna test your reflex. Just relax your jaw for me. I am gonna place my finger on your
chin.
Corneal reflex

9-Facial nerve (like bell’s palsy lecture)


-wrinkle forehead
-close eyes tightly
-puff cheeks
-smile
10-neck movements
-flexion, extension, lateral flexion, rotation)
11-ear otoscopy!!
12-complete with all cranial nerve

Diagnosis and differentials


-talk about positive findings
-there are several reasons that can cause double vision. As it happens even when you close one
eye, it is called unilateral diplopia. Can be a problem in muscle, bone or even nerve. But most
likely in your case is fracture of orbital floor  Need to admit you to be checked by the specialist
and to have some Ix:
routine FBC
x-ray skull
Ct head and neck
do you want me to call someone for you?
Reading materials
Facial Trauma
You are working in an emergency department when the ambulance brings in 25 year old James,
a driver of a car who was not restrained by a seat belt and hit a tree at medium speed. He
injured his face by hitting the steering wheel. His right eye was quite swollen and he could hardly
open it and he was bleeding from his nose but had no other injuries. Primary survey was done
by the paramedics and initial resuscitation provided. Secondary survey also done by you and
revealed no injuries of rest of the body.
Tasks:
take a focused history
Do a focused relevant examination
Explain your findings to the patient
Arrange appropriate relevant investigations and tell management

History
-MVA details. How? When? What speed? Seat belt? Side of impact?
-LOC?
-Vomiting?
-Discharge from ears/ nose?
-Bleeding/ bruising anywhere?
-Pain anywhere?
-Double vision?
-Quick systemic inquiry.
-Drinking? Past Med? Social support?

Investigation
Baselines
Mainly x-ray skull and facial bones including orbital floor
Special x-ray OPG
CT head and face
CT cervical spine if appropriate

Management
-Admit + analgesia, nasal decongestant sprays, broad-spectrum oral antibiotics, and ice packs
should all be administered.
-Call registrar
-Urgent consultation by maxillofacial surgeon for exploration and elevation of orbital floor within
24 h if CT shows entrapped muscle or tissue with signs of diplopia and gastrointestinal
(nausea/vomiting) or cardiovascular symptoms (heart block, bradycardia or syncope).
If no entrapped muscle is suspected, surgical repair of the orbital fracture can be delayed for 1-2
weeks, and is indicated in cases of cosmetic deformity or diplopia.
-D/W ENT regarding nasal fracture and need of reduction
Feedback 29-11-2017
Orbital Floor Fracture (feedback – facial trauma)
You are a MO in ED , Young man had an accident, primary survey done. Vitals stable. Only a
bruise over the left cheek. Do relevant examination and tell the patient your dx and mx.
Greeted the pt, Asked about pain and painkiller, Any injury anywhere else
Problem in seeing?
Then Look, feel, special test- head neck region look, Temp, tenderness, Examine 2, 3, 5, 6 cranial
Positive finding- bruise over cheek
Tenderness in lower orbital margin
Diplopia when looks upwards
Numbness over cheek
I said there are several reasons that can cause double vision. As it happens even when you close
one eye, its called unilat diplopia. Can be a problem in muscle, bone or even nerve. But most
likely in ur case it is fracture of orbital floor. Pic. Usually Involves a nerve. Common. Don’t worry.
Admit. Ct scan. Will be seen by specialist. Need someone to call?. Thanks
142-Gynecomastia Examination
1- A 52 y/o man who goes to bodybuilding center has come with c/o large breasts.
Task:
- explain for the patient the examining steps that you want to do (then the examiner gives you
the result of physical examination)
- explain the most likely diagnosis and give reasons

2-40 years old, self -conscious, think he got breast enlargement. He used steroid for the gym,
BMI - 27.
Tasks
-Perform PE
-explain the most likely cause.

Examination steps
1-General appearance
-weight, height.
-comfortable etc…

2-Vital signs (all)

3-Hands and arms


-clubbing, cyanosis (CLD)
-palmar erythema (thyroid+ CLD)
-sweating (thyroid)
-tremor (fine tremor when hand outstretched and using paper+ flapping tremor when hands
cocked back) (thyroid+ CLD)
-spider nevi, scratching marks, petechial, bruising, rashes (chronic liver D).

4-Face
-moon face (steroid)
-pallor, jaundice (Chronic liver D)
-thyroid eye signs (thyroid)
-visual field + Fundoscopy (pituitary)

5-Neck
-thyroid
-cervical Lymph nodes

6-Breast
look (no need to position the patient).
-asymmetry
-Scars
-skin changes: (erythema, puckering or dimpling of the skin)
-visible lump or not
-nipples (retraction, distortion, ulceration, discharge)

Feel (like female breast ex) + axillary LN


7-Abdomen
inspection
-central obesity, distension, dilated veins
palpation
-superficial and deep tenderness
-liver and kidney

8-Genitalia with consent and chaperone


-testicular mass, hypogonadism + inguinal LN

After doing examination the examiner will give you card with only breast tissue enlargement
bilaterally and all other normal.

Diagnosis and Differential diagnosis


1-steroid induced (most likely in this case)
2-phsiological
3-chronic liver disease
4-thyrotoxicosis
5-pituitary tumor
6-breast cancer
7-liver, adrenal cancer
8-testicular cancer
9-other medications (estrogen, spironolactone, cimetidine, digoxin, marijuana)
10-klienfilter syndrome

Sometimes the examination preceded by history for 2 minutes? Focus on:


1-gynecomastia questions.
-since when?
-one or both breasts?
-any pain, visible lump, nipple discharge? skin colour changes?
2-DDX questions
-yellowish skin colour, change in urine or motions colour?
-weather preference? Change in bowel habits?
-LOW, LOA, Lumps or bumps?
3-General
-medications
-PMH, PSH, SAD
-family history
-if not revealed ask specifically if going to gym and use anabolic steroid?
Feedback 31-5-2018
gynaecomastia d/t anabolic steroid
Tasks examination ,dx and ddx
U will get finding from the card on the table.i did all examination as required and examiner said
it is good to do but u can get findings in the card. after finish all examination I looked at the card,
just breast tissue enlargement ,all r normal.
Explained dx and ddx- steroid use. No Cushing disease,no liver problem,no thyroid.
Key 4/4 , 4,4,4 global 4.

Recall 16-3-2018
Gynecomastia (hx, Ex, ddx and dx)

Case (27/4/2017) failed


Gynaecomastia exam
40 yrs old, self -conscious, think he got breast enlargement. He used steriod for the gym, BMI -
27.
Perform pe, explain the most likely cause.
i entered the room and introduced myself and ask about pt conern today.
i took consent and explained him that i need to examine his breast which will involve looking,
feeling your breast and ask u to do some movement. Then, ask the pt to undress and wash the
hand. i started looking any features of cushion syndrom or steriod side effects but found
nothing.
*PF (during pe, pt breast is a little bit enlarged but i think it is normal , so i just said no breast
enlargement, no swelling or mass are noted. when do palpation, pt said he got some pain
around nipple but no other findings are noted.
when i asked the pt to lie down to feel the abdomen, examiner asked me what to examine, i told
the exminer that i will feel the liver and looking for any features of col,
examiner asked me again, anything else. i said i also would like to do private area examination
and looking for any testicular atropy.
after exam, examiner give me pe card, breast nl, abd nl, no features of col and testicular exam
also nl)
i explained the pt that exmaination findings are normal. since you think your breast is increased
in size eventhough it is normal. i think you might have a codition called body dismorphic
disorder. i couldn't find any features of side effect of mediction, steriod but i want u to stop using
it. sometimes, it can be due to the medication. it could also be due to problem with liver like
liver failure and also could be due to problem with testis.
Reassure him that i couldn't find any features of liver failure and problem with testis.
check his understanding and reassure him.
i still have some time, so i told more about body and mind and reassurance.
FB-FAILED , OVERALL-3, APPROACH-4, EXAMINATION AND TECHINQUE-3, DDX-2.
Case (27/4/2017)
a 52 y/o man who goes to bodybuilding center, has come with c/o large breasts.
Task:
1- explain for the patient the examining steps that you want to do (then the examiner gives you
the result of physical examination)
2- explain the most likely diagnosis and give reasons

Feedback comment pass


Gynecomastia in a male
1-25yr male concerned & worried about breast enlargement. He's attending gym & is on .....( i
cant remember the name, as i can remember it wasn't the generic name, but it was an anabolic
steroid)
Task
1)PE
2) examiner will give a sheet of findings
3) explain dx to patient
This was my first station, fully anxious, my 2 mins thinking,
1-steroid induced
2- R/O Ca
3- liver failure very unlikely in this young patient
( to present with only gynaecomastia).
Insp- facial, axillary, pubic hair+,no central obesity, no moon face
Voice- NL
Breast- not enlarged/ colour change/puckering/ nipple deviation/ ulcers.
Palpation- tender+, no masses, no gynaecomastia, no nipple discharge, no axillary LN.
" i would like to Ex abd to exclude liver pathology, testis for atrophy"
Finished PE in 3 mins, ( almost same in the Ex card, but gynaecomastia was there)
Told its steroid induced, not a cancer as no features. (Finished in 5 mins, Passed)

Case (8/3/2017)
gynecomastia pt using anabolic steriods perform relevant examination, reg case of gynecomastia
with anabolic steroid use.
1- Vitals,
2-abdominal (central obeisty, striae)
3-moon face
4-upper back hump)
5-breast exam( unilateral/bilateral, tenderess or no,masses, discharge and FH of Ca) .
Other dds of gynaecomastia: CLD, Drugs s/a anabolic steroids, Digoxin, Cimetidine, etc,
Klinefelter syndrome, testicular tumours
Gynecomastia Talley connor
Gynecomastia
Gynaecomastia is ‘true’ enlargement of the male Breasts. Careful examination will detect up to
4 centimetres of palpable breast tissue in 30% of normal young men; this percentage increases
with age. These men are unaware of any breast abnormality.
Gynaecomastia occurs in up to 50% of adolescent boys, and also in elderly men in whom it is due
to falling testosterone levels.

1-Weight- Fat deposition (‘false’ enlargement) in obese men can be confused with
gynaecomastia.

2- Examine the breasts for evidence of localised disease (e.g. malignancy, which is rare),
tenderness, which indicates rapid growth, and any discharge from the nipple. Detection of
breast tissue in men is best performed with the patient sitting up.
Squeeze the breast behind the patient’s nipple between the thumb and forefinger.
Try to detect an edge between subcutaneous fat and true breast tissue.

3-Examine the genitalia for sexual ambiguity and the testes for absence or a reduced size. Note
any loss of secondary sexual characteristics.

4-Look especially for signs of Klinefelter’s syndrome These patients are tall, have decreased
body hair and have characteristically small, firm testes.

5- Look also for signs of panhypopituitarism or chronic liver disease. Thyrotoxicosis can
occasionally be a cause.

6-Finally, examine the visual fields and fundi for evidence of a pituitary tumour.

Causes of pathological gynaecomastia are diagnosis (causes) o

Increased oestrogen production (liver/ thyroid/ testis/ lung/ kidney)


Leydig cell tumour (oestrogen)
Adrenal carcinoma (oestrogen)
Bronchial carcinoma (human chorionic gonadotrophin)
Liver disease (increased conversion of oestrogen from androgens)
Thyrotoxicosis (increased conversion of oestrogen from androgens)
Starvation
Decreased androgen production (hypogonadal states) (klinefilter/ trauma/ orchitis)
Klinefelter’s syndrome
Secondary testicular failure: orchitis, castration,
trauma
Testicular feminisation syndrome.
Drugs:
Oestrogen receptor binders: oestrogen, digoxin, marijuana
Anti-androgens: spironolactone, cimetidine
143-Polycystic Ovarian Syndrome
Sample Case
You are at your GP clinic when 20 year old Amanda presents to you with history of irregular
periods over the past 1 year after no periods at all for the past 3 months.
TASKS
-Take a further history
-PE from examiner
-Investigations with examiner
-Advise the patient regarding further management

Exam Case (4-4-2018) Was Unscored


23 yrs with mentrual abnormalities
Task
-hx
-Pefe
-Most likely diagnosis and and investigations.

Differential Diagnosis:
Hypothalamus
-Eating disorder
-exercise induced
-stress induced
-liver/ renal disease

Pituitary
-hyperprolactinemia
-Thyroid
-antipsychotic medications

Ovary
-PCOS
-premature ovarian failure
-post pill amenorrhea
-chemotherapy and radiotherapy

Uterus
-pregnancy
-asherman syndrome

Note/Amenorrhea: no periods for 6 months if she is having regular periods, and no periods for 3
months if she is having irregular periods
History
1-Can you tell me more (if chief complaint was just menstrual abnormality)

2-5Ps questions
Periods
-when was your last menstrual period?
-When did you have your first period?
-Were your periods regular until the past 1 year? How often do you get irregular periods?
-have you had any pain or heavy bleeding during menstruation?
Note/ in the exam she has 2 cycles for the past 1 year, before normal with no heavy bleed.
Partner/ sexual
-Are you sexually active?
-Have you ever been sexually active or not?
Pregnancy
-if sexually active ask if by any chance she is pregnant? (Pregnancy)
Pill
-do you use any contraceptive methods? What is the type? (Post pill amenorrhea)
Pap/ HPV
-Have you taken the Gardasil vaccine?
-pap/ HPV up to date

3-Differential questions:
PCOS
-have you put on weight recently?
-any excessive hair growth? any acne?
-do you feel thirsty? Pass large amount of urine? (DM complication of PCOS)
POF
-do you have any hot flushes, heavy sweating, mood changes
Thyroid
-any weather preference? Any changes in bowel habits?
Hyperprolactinemia:
-any headache, blurring of vision? Any milky discharge from the nipple?
Eating disorders,
do you think that you are overweight? Do you try to lose weight through crash dieting or
excessive exercise?
Exercise-induced amenorrhea:
-do you exercise regularly and how many hours do you exercise?
Stress-induced amenorrhea:
Any stress at home? Any stress with your family?

4-General questions
-Do you smoke? (Smoking predisposes to PCOS)
-Any family history of a similar condition especially in your mom or sister?
-Any previous medical or surgical conditions? Any medications that you are on?
-any medications that you are on?
Note/ in the exam Acne positive, overweight positive. Remaining hx normal
Physical Exam from examiner
1-General appearance: hirsutism, acne, BMI, pallor, LN
2-Vital signs: BP
3-CVS, Respiratory
4-Thyroid exam
5-Abdomen:
any visible mass, distention
6-Pelvic: consent and chaperone
-Inspection of the vulva and vagina
-Speculum and Per vaginal exam CAN be done in PCOS, but NOT in this case because she is
virginal.
7-Office test:
BSL, UDT, urine pregnancy test

Note/ in the exam bmi 31, acne and hirsutism positive.

Explanation of the Diagnosis:


-there are several possibilities leading to abnormality of the periods but from history and
examination, you most likely have PCOS have you heard about it?
Condition
-Normally during each menstrual cycle, even though several follicles develop in each ovary, one
of the follicles becomes mature, and then will break open to release the egg. PCOS is complex
condition in which, the ovaries are larger in size, and they develop many small immature follicles
or cysts, none of which grows to maturity and breaks open to release the egg. In other words,
ovulation does not happen. That is why your periods have become scanty and finally stopped.
Cause
The exact cause of PCOS is unknown but it has been found to run in families.
Complications
-There are also other hormonal disturbances in PCOS, like the body becomes resistant towards
insulin, which is a hormone that keeps your blood sugar level in check. This causes a rise in the
blood sugar level, leading to Diabetes.
The ovaries also secrete a small amount of male sex hormones called testosterone which gets
out of balance leading to weight gain, hirsutism and acne.
-This can also lead to high blood pressure, high cholesterol levels, infertility and even depression.

Investigations
I need to run some Ix to confirm the diagnosis and to rule out others
-FBE, UEC, ESR, CRP, LFT, serum lipid profile
-TFT
-FSH/LH ratio (Normal 2:1)
-Testosterone levels
-Pelvic ultrasound - greater than 10 follicles less than 10mm in size, this is PCOS
Management (according to the feedback it is not a task)
-I will refer you to the specialist.

-You need to folow lifestyle modifications. I will refer you to a dietician for proper diet chart. You
need to follow the advice of the dietician as even a 10 percent reduction in the body weight can
normalize your cycles. Along with exercise 30 minutes 5 days a week. So once the cycles are
normalized, all the other hormones can also be put back to place.

-We will also start you on low dose combined OCPs to bring back your menstrual regularity,
decrease excessive hair and acne.

-We will also start you on Metformin. It can increase insulin sensitivity so you will start losing
weight and once weight loss started it restores the menstrual regularity.

-If medical management does not work, we think about surgical management.

**What we do is laparoscopic ovarian drilling. Under general anesthesia, using a keyhole


surgery, laser or electrocautery is used to break through the thick outer layer of the ovary, and
then destroys small portions of the ovary. When the small portions of the ovary are destroyed,
the testosterone production comes down drastically. If you break any part of the circle, all the
rest will fall back into place. (I do not think it is necessary to talk about all of these)

-Meanwhile, because these medications take some time to act, you can go in for cosmetic
therapies, like laser or electrolysis. She can be put on combined oral contraceptive which
contains the 3rd-generation progesterone (has anti-androgenic property) cyproterone acetate
(Dyne 35)

-review and reading materials

Note/ PCOS + infertility


1-lifestyle modification for 6 months
2-if doesn’t work- ovulation induction medications like clomiphen and add metformin with
that
3-if not work laprascopic ovarian drilling
144-Tiredness anaemia
57-year-old woman presented with tiredness for 6 m. Ix done low Hb, low MCV, low iron, low
ferritin
Task
-Explain result
-History
-Ddx

Explain results
-These are the results of FBE showing decrease in the Hg level, which, is iron rich protein carried
on red blood cells responsible for carrying Oxygen to body organs. That is why you feel tired.

-It also showing low Mcv, which means your red blood cells are small in size

-However, WBC cells to fight infection & platelet that help blood clotting are Normal.

-So this is suggesting of anemia either IDA, thalassemia or inherited blood disorder or chronic
illness anemia.

-So we arranged also for iron studies and it shows that Iron level in the blood is low and iron
stores or ferritin as well low

-the result suggests that the most likely cause behind anemia is deficiency in iron level.

History
I would like to ask you some questions to help me reaching the cause behind IDA is that alright?

1. Diet and exercise


-Can you describe your daily diet for me? (Eating all food 3 times)

2. Celiac, malabsorption, cancer, PUD, thyroid


-How is your bowel habit? Diarrhea, constipation or alternating? Any change?
-Is it hard to flush? Smelly? Sloppy? Any blood? Black stool?
-Any LOW, LOA, lumps or bumps?
-Fever, rash
-N/V, blood in vomitus, tummy pain?
-Weather preference

3. Anemia Qs
Dizziness, palpitation, Sob, on exertion
Bleeding anywhere

4. General Qs
Pmhx( bleeding disorders, PUD, heart, cancer, CTD)
Pshx, trauma
SAD
Medication (NSAIDS, blood thinners) using NSAID for shoulder pain for 9 months +ve
Fhx of cancers
Gynecological hx e.g all period qs

Explain

There are several possibilities behind your IDA.

Most likely could be due to irritation & inflammation of stomach ( gastritis) or development of
ulcers in stomach or 1st part of bowel both mostly due to analgesic that can cause bleeding.

Other ddx are


-diet
-cancer bowel
-Gerd
-Poor absorption ( IBD, celiac, infection)
-Thyroid disease

Mx

1. Ix
-Refer to specialist for upper GI endoscopy & colonoscopy to r/o any benign or nasty growth
in the bowel

-Blood tests for celiac screen

-Stool MCS

-Blood tests e.g TFT, etc…

2. Start on iron tablets ( S/E black stool, constipation so advice to increase water intake)
3. Iron rich food

Feedback 31-5-2018
GP(I forget the age)Tiredness for ?6 months. Inx Hb low, MCV low, iron study Fe low.
Tasks explain result,h/o 5 mins and ddx.
Greet and explained result. Microcytic anaemia and IDA
H/o diet-she is eating all foods 3 times daily.all r normal except she is using NSAID(?name) for
shoulder pain for ?9 months. gynecological questions are normal.Pap smear also normal.

Most likely d/t analgesic that cause irritation and inflammation of stomach ,can cause
bleeding.other ddx diet,Ca bowel,poor absorption causes IBD,celiac,and infection.thalassaemia.
All unlikely but need to refer you to specialist for further investigations and treatment.

Keys 4/5 approach,interpretation of result,h/o,dx and ddx 3,0,5,5


Global 4 (I dunno why I got 0 in interpretation of result may be error or non scored in that
task,because I did pretty well in explaining the results).

Feedback 18-9-2018 Anemia tiredness


fatigue - IDA
52 year old lady feel tired and something, blood test found IDA. Rest stain normal I think,
PE also normal
task: explain the blood result
further history
Dx and D/D with reasons

Explain the result


History: just tired, no weight loss, lumps bumps, no fever, no stool colour change, no
hard to flush stools. Diet is normal eat red meat 3 times a day (this lady is very nice, she
told me all this without I ask much)
Take neurophen for shoulder pain for 5 years, but not abdominal pain no indigestive
symptoms.

(I think I didn’t ask colonoscopy history/ FOBT… )

Dx: iron deficiency can be caused by reduced intake, malabsorption and increased blood
loss.
In your case, the most like cause is related to neurophen use, NSAID can cause gastrotitis
or peptic ulcer even you don’t have symptom now.
Other cause: diet- unlikely
Malabsorption: celiac, crohn’s disease, IBD – need to rule out
Nasty growth- need to rule out
Do endoscopy and colonoscopy
Score: 5
Key step: yes/yes/yes/yes/yes
Approach: 4, Interpretation of Ix: 4, Hx: 5, Dx/D/D: 5
Feedback 18-9-2018 Tiredness anaemia
Your next pt in your Gp clinic is a 52 year old lady with a complaint of weakness and
tiredness for the last 6 months. You are seeing this pt for the first time. Last
week one of your colleague ordered some blood test and the results have come back
as following;
There was a big list of result but only findings were low HB, Low MCHC and Low
Iron store, the rest was normal.
Your task is to
1. Explain the result to the pt
2. Take further focused history.
3. explain Dx and DDx
My approach:
I explained that one of the element of our blood is HB which carries oxygen to all of our
body cells, in your blood report we found that HB is low. Also I told her about low iron.
There could be number of reasons behind this so I need to ask you some questions.
Then I asked about diet, any bleeding, recent infection, any blood loss, surgery, bleeding
or clotting disorder, family history of blood disease, Medication history, SADMA, Post
menopausal history. everything was normal except for she uses Neurofen for for joint
pain.
Then I explained IDA due to NSAID use as my Dx.
For DDx I explained Thalassaemia, Bleeding disorder, malabsorption, less intake and
explained why they are not the cause in her case.
AMC feedback:
Station 16:
Scenario: Fatigue
Grade: Pass Global score: 4
Key steps: 1,2,3,4,5 yes,yes,no,no,yes
Assessment domain:
Approach to pt/relative: 5
Interpretation of the investigations: 6
History: 4
Dx/DDx: 6
Case (3/5/2017)
60 yo lady came to know her blood result. She visited your GP practice 1 week ago complaining
of tiredness and ordered FBE.(anemia)
Task: -explain results, -history, -Dx and DDX with reasons, -Mx
Case (9/11/2017)
anaemia tiredness case, book case 111
Task-tell invx to pt, suggest further invx, and deal with her concerns.
Case (26/8/2017)
lady came with SOB & tiredness Ibuprofen long time use
MCH low, MCHC low, HB low, WBC normal Task : interpret results, Dgx & Ddgx
Case (9/3/2017)
Cbc report (hb low and mcv low ) . and blood film poikilocytosis, anisocytosis
Task
1) .interpret results,, tel her the different reasons for this and als0 the reason in her case
positive findings
1)postmenopausal but no bleeding , taking ibuprofen)

Karin case Iron-Deficiency Anemia


Case: You are a country GP and yesterday saw a 45-year old woman who was feeling weak and
tired, and you arranged for a FBC to be performed. She has now come back for the result, which
is as follows
- Hb 8.5 g/dl (12-24) - MCV 75 fl (80-95) - WCC 8,000 (7000-9000) - Platelets 255,5000 (150-250)
Task:
- Explain the result to the patient
- Answer her questions
- Describe to her your planned future management
Counseling
- This means that your body is lacking iron which is needed to produce blood cells. This is the
reason why we need to make a few more tests is to determine why you have decreased iron
either due to inability to absorb iron or because of bleeding.
- Do further iron studies to confirm cause of anemia. Colonoscopy/Gastroscopy to ascertain the
cause of bleeding.
Causes of IDA:
- Blood loss: menorrhagia, Gastrointestinal bleeding (e g. carcinoma, haemorrhoids, peptic ulcer,
hiatus hernia, GORD, NSAlD therapy, frequent blood donations, malignancy, Hookworm
(common in tropics)
- Increased physiological requirements: Prematurity, infant growth, adolescent growth,
Pregnancy
- Malabsorption: Coeliac disease, Postgastrectomy
- Dietary: Inadequate intake, Special diets (e.g. fad, vegetarianism)

Read Handbook Case 111 (reading material)


145-Peripheral vascular disease
62 year old man with calf pain on walking uphill and night pain but no pain at rest. Has a history
of DM, chronic smoker for 20 years
Tasks
-perform Physical examination
-diagnosis to patient
-management

Examination steps
1-Inspection
-Scars
-skin (colour, dryness, thickness) (atrophic changes: shiny skin, hair loss, ulcers)
-swelling, oedema
-deformity (joint deformity, amputated limbs)
-muscle wasting
2-Palpation
-Temperature
-non pitting oedema
-CRT (compress for 5 seconds)
-Pulses: Femoral artery (take consent)popliteal arteryposterior tibial arterydorsalis pedis.
3-Auscultation
-Aorta bruitfemoral bruit
4-Neuro examination
-sensation (MVPR)
-motor
5-say Complete with CVS, upper limb and abdomen
6-special tests
Burger tests
-elevate both legs 60 degree then look for pallor
-if pallor do dependency
-if dependency +ve repeat again in 20 degree.
ABPI (in exam not available or they will ask you to briefly explain what it is)
-place approximately sized cuff around the arm and locate the brachial pulse then apply gel at
this site.
apply the hand held probe at 45 degree and locate the best signal. Then inflate the cuff until the
signal disappear then deflate it slowely and record the pressure at which the signal reappear.
repeat it on the other arm
take the higher of two readings
-place approximately sized cuff around the ankle immediately above the malleoli. Locate dorsalis
pedis. Angle the probe of Doppler at 45 degree. Locate the possible signal and inflate the cuff
until signal disappear then deflate it until reappear.
repeat again on posterior tibial artery.
repeat again on the other leg. take the highest of two readings.
-now divide ankle by brachial (>0.9 is normal)
Explanation
-from history and examination, I do not suspect the presence of clot in your blood vessels. But
there is a compromised blood supply due to narrowing of the arteries supplying the lower limbs.
So on walking there is a demand of high blood flow that is why you have pain, while at rest there
is no such demand so that the pain gone. We call this condition intermittent claudication.
-it is usually associated with DM, HTN, hypercholesterolemia, smoking.

Management
1-Investigations
-take blood for FBC, ESR/CRP, UCE, LFT, BSL, lipid
-arrange Doppler ultrasound to look at the vessels of the legs

2-SNAP
-advise to maintain healthy lifestyle
-stop smoking + arrange another consultation
-healthy balanced diet (low fat, no junk food) + refer to dietician
-limit alcohol intake to safe level
-gradual exercising to the level of the pain to increase collateral circulation+ refer to physio

3-Chronic diseases
-control hypertension, cholesterol and diabetes
-start aspirin and statin
-stop B-blocker and replace with ACEI
-control DM + foot care

4-Referral
-to vascular surgeon to decide upon any need for surgical intervention + CT angio
Intervention options
*endovascular stent: stent inside the vessel to dilate and improve blood flow
*bypass graft

Feedback 4-7-2018
DM for 10yrs calf pain walking uphill & night pain do LL examination
(got confused outside whether to do PVD or diabetic neuropathy from the hx)
?need to check gait/not got confused
Not satisfied with my performance.
When I went inside about 60yr old man was lying on the bed-
so I started with inspection/palpation/vascular-warm/CRFT/
when I checked peripheral pulse Ex asked me whether it is present or not
pt looks like real pt-on one side DP could’t palpate so said it-not sure right/wrong
vibration + at leg –can’t remember now
when asked ABPI-told not available
when I about to go out ex asked me to wash my hand-but I did before she didn’t observed-I told
I did it-she said ok
146-Secondary amenorrhea - Exercise-induced
AMC case
A young female who is not having her periods since 12 mnths .she had her pregnancy test done
which was negative. Pulse was 47 beat/min her husband is concerned so he asked her to seek
consultation.
Tasks:
-History
-dx, ddx
-mx (no pefe)

sample case
Maria presents to your GP with complaints of absence of periods since the last 6 months.
TASKS
-Further history
-PE from examiner
-Discuss management with the patient

Causes:
 Pregnancy
 PCOS
 Premature Ovarian failure
 Post-pill amenorrhea
 Asherman's syndrome
 Thyroid disorders
 Hyperprolactinemia
 Eating disorders
 Exercise-induced amenorrhea
 Stress-related amenorrhea

History
1-5Ps (pregnancy/ post pill amenorrhea)
Periods
-What are your concerns?
-Do you get spotting at the time of your normal periods?
-How were your periods before it stopped?
-How do you quantify your bleed before it stopped?
-Any pain at the time of periods?
Sexual history
-Are you sexually active? Are you in a stable relationship?
Pregnancy
-Were you planning for a pregnancy?
-Have you ever been pregnant?
Pill
-What contraception were you using?
2-DDX questions (PCOS/POF/ THYROID/ HYPERPROLACTIN/ EATING D/ EXERCISE/STRESS)
-Have you noticed any weight gain? Any excessive hair growth or acne?
-Any menopausal symptoms like hot flushes, heavy sweating, mood changes?
-Any weather preferences. How are your bowel habits?
-Any headache, blurring of vision, milky discharge from the nipples?
-Are you much bothered by your weight and appearance?
Have you ever thought that you are overweight?
Do you try to lose weight by crash dieting or self-starvation, vomiting or water pills?
-What is your occupation? (key)
How many hours of ballet dancing do you do per day?
Any other exercise that you do?
-How is your home situation? Any stress at home? Any stress at work?

3-lifestyle questions
-How is your diet?
-SADMA history
-Past history
-Family history

Physical Exam
1-General appearance:
BMI, pallor, dehydration
PCOS: any excessive hair growth, acne
Elasticity of skin - low estrogen can decrease elasticity
2-Vital signs
3-Thyroid
4-CVS/Respi/CNS
5-Abdomen: visible distention, mass, palpate for mass and tenderness
6-Pelvic examination
Inspection of the vulva and vagina: any discharge or bleeding, look for atrophic vagina
Speculum exam: cervix healthy or not, any discharge/bleed
Bimanual exam: CMT, uterine size, tenderness, adnexal mass and tenderness
7-Office tests: UPT, UDT, BSL

Diagnosis and Management


-Most likely you have a condition called exercise-induced amenorrhea.
Amenorrhea is when you have a loss of periods for around 6 months after established periods.

-Draw the HPO axis.


The estrogen and the progesterone that is released by the ovary, under the influence of the
hormones secreted by the hypothalamus and the pituitary is responsible for a normal menstrual
cycle.
These sex hormones could be affected by a range of factors like excessive exercise when certain
exercise-related hormones like beta endorphin and catecholamines are released. The high levels
of these hormones suppress the areas in your brain which alters the secretion of estrogen and
progesterone.
The other contributing factors could be eating disorders and low levels of body fat and also the
emotional stress that is associated. So an imbalance between the exercise and increased
nutritional demands along with stress can bring about amenorrhea, as it starts interfering with
the normal hormonal pathways.

-If this is not treated, this can lead to long-term complications, like the decrease in the fertility
or decrease in the bone density and the bones become brittle and break easily, increased
cholesterol levels and also premature aging.

- But first, we need to rule out all other possible causes of amenorrhea.
We have already done a pregnancy test, and it has come out to be negative.
All the basic bloods need to be done like a FBE, UEC, LFT, RFT, TFT, vitamin D (25-OH vitamin D),
serum prolactin, serum lipid profile, estimate all the hormones like GnRH, FSH, LH, estrogen,
progesterone, and also a pelvic ultrasound.

-You will be referred to the specialist.


Adopt lifestyle modifications:
Try to put on around 2-3 kilos of weight. I will refer you to the dietician who will give you proper
dietary advice.
I will put you on calcium and vitamin D supplementation.
Limit exercise to a maximum of 8 hours per week.

If these measures are not working in 6 month's time, then we can put you on combined oral
contraceptives.

I will give you reading materials regarding exercise-induced amenorrhea and I will arrange a
review with you in 1 month.
Feedback 19-7-2018
Menstrual complaint (exercise induced amenorrhea) Pass 4 key steps : yes
Case: a young female who is not having her periods since 12 mnths .she had ger pregnancy test
done which was negative. Pulse was quite low (i dnt remember exact value) her husband is
concerned so he asked her to seek consultation.
Tasks : hx,dx ,ddx, mx no pefe
when i entered the room there was a young female smiling n quite unconcerned abt her not
having periods . Consent for private qs and then I asked her 5ps in detail and then dds... pcos,
thyroid, hyperprolactinimia,pof,all were negative. Had a D&E some yrs ago (2 or 3) Then i asked
her any change in her diet ir routine or anything around that time since she has this complaint.
She said she has a marathon coming up in few mnths for which she was preparing since 15 /16
mnths . Exercise for 5 hrs n swimming for 2 n half hr . There i got my dx . Dx : exercise induced
amenorrhea.
Ddx: pcos, Asherman's, thyroid, pof,
Mx: i explained her why she is not having periods.then said with her consent i need to include
her trainer in my team so we can midify her time and pattern of exercise bcz it is affecting her
normal body functions. Will also add a dietitian to modify and plan a good diet bcz she lacks the
essential nutrients which are important for normal body functions due to this heavy exercise.
I also said that i will do some basic investigations (blds and u/s) and will refer her to a
gynecologist. i didn't ask ger to quit the exercise because i thought i can't stop a person who is
just few mnths away from her goal and was preparing so much for so many mnths so i just said
we need to modify things .and if that doesn't work i will refer her .

Feedback 19-7-2018 PASS( KEY STEPS 1,2,3,4 YES,Score 5,5,3)


Menstrual complaint (Secondary Amenorrhea 2 to excessive exercise)
Task : Hx , provisional Dx and DDS and Mx, NO PEFE.
Long scenario outside .25 year old female came to u because of not having period for last 1
year ,did preg test which was negative, vitals showed pulse -47/min rest all normal.
When I entered the room young lady was sitting ,seems quite unconcerned, told me that she is
here because her husband is worried. I told her confidentiality and asked consent about private
questions, then started with periods, covered 5Ps then PCO,POF ,Pituitary, thyroid , diet and
exercise. She had a hx of D&E 3 years back, no kids as don’t want to, doing exercise 5-6 hours
day because preparing for marathon, diet was good.
Most likely Dx was sec. amenorrhea due to excessive exercise, DDs Asherman, POF ,PCO,
Thyroid.
MX multidisciplinary team as not ready to quit or dec exercise, need councilling as its affecting
her body badly, exercise instructor and me. I told her that I want to do hormonal assays(blood
test and US to make sure every thing is fine) After dec exercise ur period will return in couple of
months, if not then referral ,reading material and fu.
Feedback 13-12-2018 Menstrual Complaint: Pass
Young lady around 28 yrs with history of amenorrhoea since last 3-4 months. ( this is the case of
secondary amenorrhoea)
Tasks: History, Dx and DDX
Thinking outside: I remembered the causes of secondary amenorrhoea from karen’s.
Inside: Me: I am so sorry to hear about your problem.
Pt: Oh Don’t worry, I am not worried about not having periods. Its my husband who is worried that’s
why I am hear.
Me: Is it okay If I ask you few questions so we can find the cause of it.
Pt: Okay
Me: 1st please tell me more about your periods.
Pt: they were regular until 4 months ago, every 28 days, moderate not heavy last for 4 days. She gave
all the history including menarche as well.
Me: Is there any chance you could be pregnant? Pt: No
Me: Did you do pregnancy test at home? Pt: No
I asked about remaining 3 Ps. All negative.
Me: Then I asked questions regarding all the causes as per karen’s. (All negative except heavy
exercise).
Me: Do you do lots of exercise? Pt: yes
Me: pl tell me more about it. Pt: I am preparing for triathlon. So I do lots of exercise. Me: okay.
Then I asked about anorexia nervosa such as what is your perception regarding your body? Pt:
means? Me: do u consider yourself fat? Pt: no way
Me: do u use laxatives of water pills by any chance? Pt: No
Then I ask about SADMA, present, past and family history.

Dx: Me: there could be many reasons why your periods are stopped.

I think it is your heavy exercise that is causing it. It is called exercise induced amenorrhoea. When
you do heavy exercise body thinks that you r losing lots of energy and not having enough energy
and body produce some changes that causes stopping of periods.

Others reasons such as thyroid problem ( but u done have thyroid symptoms), high hormone
production such as prolactin ( but u don’t have milky discharge from nipple and no visual blurring),
early menopause ( but u don’t have mood swing and hot flushes), pregnancy ( but u don’t have
symptoms), psychogenic causes (less likely bcas u don’t have symptoms), certain medications ( but
u don’t take them), certain problem with ovaries or womb.

Global Score: 4
Key Steps: 1&2 No, 3 yes
Approach to pt: 4 History: 5 Dx/ DD: 5
147-Bulimia nervosa irregular periods
Recall 14-3-2018
Young woman with menstrual irregularities. (There were clues that told it was eating disorder
which I can’t recall now)
Tasks
-History
-Diagnosis and D/Ds

Recall 5-5-2018
17 year old girl concerns about irregular periods. In her past medical history, she was admitted
to hospital because of hypotension and overuse laxatives. She still use laxatives and makes her
sick as she feels she is fat.
Tasks
-Take history
-explain diagnosis and cause for her irregular periods.

History
1-Can you tell me more

2-5Ps questions
Periods
-since when have your periods been irregular?
-when was your last menstrual period? (3 weeks ago)
-How often do you get irregular periods?
-have you had any pain or heavy bleeding during menstruation?
-any bleeding in between menstruation?
Partner/ sexual
-Are you sexually active? (No)
-Have you ever been sexually active or not?
Pregnancy
-if sexually active ask if by any chance she is pregnant?
Pill
-do you use any contraceptive methods? What is the type? (No pill)
Pap/ HPV
-Have you taken the Gardasil vaccine?
-pap/ HPV up to date

3-Differential questions:
PCOS
-any excessive hair growth? any acne?
-do you feel thirsty? Pass large amount of urine? (DM complication of PCOS)
POF
-do you have any hot flushes, heavy sweating, mood changes
Thyroid
-any weather preference? Any changes in bowel habits?
Hyperprolactinemia:
-any headache, blurring of vision? Any milky discharge from the nipple?

4-Confidentiality+ eating disorders.


-do you think that you are overweight? (Yes, I am fat)
-anybody told you that you are overweight? (Yes)
-can you tell me briefly about your diet? (I think I am eating healthy diet)
-have you ever lost control over how much you eat? (Yes, I take a lot of food at times and lose
control)
-Do you feel guilty after that (yes)
-do you try to vomit or use any medications in order to lose weight (yes vomiting)
-do you exercise regularly and how many hours do you exercise? (30 minutes/ day)
-do you know your BMI? (21)
-dizziness, palpitations, dental problems?

5-Psychosocial history and HEADS


mood questions:
-how is your mood
-have you ever thought of harming yourself or others
-how’s your sleep
-do you find it hard to concentrate on things.
Delusion and hallucination
-I am gonna ask you certain questions that could be a bit sensitive but theses are just routine
questions I ask to some of my patients.
-do you feel, hear or see things that others don’t?
-do you think someone is spying on you or trying to harm you?
Insight and judgment
-do you think you need any professional help?
-if there is fire in this room what would you do?
HEADS
-any support at home, any stress?
- What do you do for living, any stress at work?
- have you lost interest in things you used to enjoy?
- SAD
- how is your social life? Do you catch up with friends quite often?

6-General questions
-Any family history of a similar condition especially in your mom or sister?
-Any previous medical or surgical conditions?
-any medications that you are on?
Explaining diagnosis and differentials
-From history and examination, there are several possibilities why you have irregular period.

-what I am suspecting it could be related to a condition we call bulimia nervosa; It is one of the
eating disorders when you think that you are overweight but in reality it is not as your BMI is
falling within the normal range. because you think you are overweight even though you go for a
control diet, at times you lose control and eat a lot of food then feel guilty about this, so try
compensatory behaviors like vomiting or others It can cause irregularity of your periods.

-other could be anorexia nervosa; another type of eating disorder.

-could be thyroid, pituitary, PCOS etc….. but less likely.


Feedback 14-3-2018
Young woman with menstrual irregularities. (There were clues that told it was eating disorder
which I can’t recall now)
Tasks
-History
-Diagnosis and D/Ds
2 min thinking.. Bulimia and Anorexia Nervosa but always rule about medical reasons. Causes of
Eating disorders, predisposing factors, complications of these condition, depression symptoms
and drug abuse.
After introduction I started by building some rapport and asked how was she feeling and what
was she concerned about. She told about her menstrual irregularity.
I asked about how it all started? Since how long and how have her periods changed through this
time? She said they had become more irregular only. Last one was three weeks back. She
wasn’t sexually active, on no contraceptives. No chances of being pregnant (I still asked about
morning sickness and breast tenderness). Asked about any headache or visual problem (pituitary
cause) any tremors or weather preferences (thyroid cause) or any gynaecological procedure, but
all was negative.
I gave the confidentiality statement and asked about her perception of her weight. She said she
thought she was fat. Here I asked if anybody told her about this and she started wrapping her
cardigan around her body saying yes some guys commented on my belly. I further asked if she
was worried about any of her body parts.. she said yes I’m fat and guys commented on that.
During further questioning she accepted that she was fearful of getting fat and tried to vomit
after food especially chocolates and that helped her feel better. She also had episodes of
uncontrollable eating that end up making her feel guilty . Her diet was normal and gave me a
long description. She ran 30 mins per day. There was laxative abuse previously but no diuretic
use. Her BMI was 21 (she told it to me herself without a question)
Further asked about complications like if she had any abnormal sense of rapid heart beat or
dizziness or dental problem but there was none.
Asked about Predisposing factors like anybody in the family had similar condition or she had any
bad experiences from her childhood or wether she had a perfectionist personality but none was
positive. She had no signs of depression and wasn’t using any drugs or smoking.

I started by explaining and writing the diagnosis and D/Ds. I told her that after talking to her
there were a couple of conditions that are in my mind that present like that. It could be Bulimia
Nervosa.. because her BMI was normal and she was vomiting and had uncontrollable episodes
of eating.
It could be Anorexia nervosa as that causes menstrual irregularity.
It could be Body Dysmorphic Disorder as she was concerned about her body and thinks guys
comment on that.
Other causes can be pituitary, thyroid etc but were unlikely in her case.
She didn’t look happy so I asked if there was anything bothering her and why she looked
unhappy? She smiled ‘no everything’s fine.’ End of it!
Bell rang. Scenario… Menstrual Problem Grade.. Pass
Global score… 6 Key steps 1,2,3,4… yes Approach to patient… 5 History…. 7 Diag/ D/D…. 6
148-Seizure
7-year-old boy had a seizure at school. Teacher witnessed the event. Now he is at the hospital.
Teacher wrote a note saying the symptoms (loss of consciousness for 1 min, jerky movement
generalised, rolling of the eyes, wet himself. He was complaining of sickness then had a fit) after
the fit, he was drowsy. Now v/s stable. Mother is here to discuss. BSL 6.5
Tasks
-History
-Most likely Diagnosis
-Management

Differential diagnosis
1-Unprovoked seizure
2-Provoked seizure
*Trauma
*Infection
*Tumor
*Electrolyte imbalance (NA, CA)
*Hypo or hyperglycemia
*Drugs, medications, alcohol, toxins

History
1-Approach
-When entered the room, mother or father came to ask what’s wrong with their child and
walking around the room anxiously.
-I can understand how concerned you should be about your boy. Let me assure you that he is
stable now and he is in good hands. I want to ask you few questions about your child so that I
can figure out why this happened to him. Can you have a seat please? Let me introduce myself,
my name is ….
-is this the first time?
2-seizure Qs
Before
-How was his behaviour? Any Strange behaviour or recent personality changes?
-Was he drowsy or alert before the episode?
-any trauma to the head?
-Headache, vision problems?
-Fever, recent infection, rash?
-When was his last meal?
-General health, vomiting, diarrhea, cough, allergies?
-are you aware if he is doing SADMA esp drugs, toxins, alcohol if old?
During (they might say talk to teacher as I was not there at that time)
-Loss of consciousness
-if Fell down, injury
-Jerky movements of limbs, Rolling of eye, tongue bite, Wet or soiled himself
-Turned blue, breathing difficulty, noisy breathing
After (they might say as you can see him)
-How long the entire episode lasted?
-How did he recover? Did he require any assistance?
-Drowsy, memory loss
-Any weakness, numbness, headache, N/V
-Was he able to walk in here or did he require any assistance
3-General + BINDS
-Pmh e.g. DM
-Fhx of seizure or epilepsy
-BINDS esp. Support, stress, sleep deprivation

Explain the Diagnosis.


- What the child had is seizure or fit, which is an abnormality within the electrical activity of the
brain.
- It could be due to several underlying causes; we called provoked fit. Like Could be from simple
temperature rise as fever or infection, injury or head trauma, or lack of oxygen or it could be due
to allergic reaction, salt and minerals imbalance, issues with blood sugar either decrease or
increase in the blood sugar, but here the BSL is normal.
-But from the history, I could not find any of these so sometimes it can happen without any
cause; we call that unprovoked fit.
-Unlikely to be epilepsy because it happens for the first time and no Fhx. However it can be
suspected if recur again.

Management
- I am going to keep him admitted in the emergency department for observation if any other fit
occurs or not .he will be seen by the specialist as well. Although we don’t do investigations in
first fit in which no cause could be found but it depends on specialist he might consider doing
investigations to rule out underlying causes like
*Blood tests (FBE, CRP,ESR) (UCE, LFT, TFT, BSL)
*Urine MCS if UDT +
*X-ray chest
*+/- CT decided by specialist
*Drug screen with consent of parents.

- after ruling out these underlying causes if nothing appeared positive then it most likely
unprovoked seizure, which could be precipitated by many factors like sleep deprivation, hunger,
stress, infection, bright flashing light.

-Reassure that their son will be taken care well. And we will let him/ her know what to do in
those situations if he has this again just relax.

If you have time say -after discharge he most likely will be


*Referred to neurologist.
*referred to 1st seizure clinic they might do EEG
*General measures in case if happen again. Need to know what to do:
Do not allow him to swim alone, no baths, no sea diving, heights to prevent recurrence.
Feedback 17-8-2018 (IMP)
A 7 year old child had first seizure at school he was alright when arrived at school. Can’t
remember all exactly. There was a note written by teacher in which he explained the situation he
had generalized fit at school before that he was perfectly alright then he started feeling unwell
and sick and fell from the chair and had a fit involving whole body and passed urine and bitten
his tounge.
Tasks
-take history from father
-Pefe from exam
-Management to father

Father was standing at the door. Looked very anxious. I tried to calm her by saying I can
understand you are worried about your child but don’t worry I will try my best to find out the
cause why this happened to him. I need your help regarding that so that I can better understand
his condition . can you please have a seat so that we both can have a talk about him and find the
solution. He agreed. And sat down on chair then I started history as in Karen.

Ruled out hypoglycaemia he said he has done his breakfast. Any recent illness any fever he said
no. any trauma no. any previous trauma to head, no. any history of headache early morning
vomit no . I tried to rule out SOL by this. Any recent vomitings or diarrhoea no. ( electrolyte
abnormality) any history of fits previously , he said no.
After asking pre seizure questions I then moved to the seizure episode but the father didn’t
answer that, he replied all questions by saying that you can talk to teacher as I was not there at
that time and I don’t know.
Post seizure question he replied as you can see him.
I asked family history of seizure there was no history positive. Took birth history any trauma to
head at that time how was the preg with this baby nothing significant stress at home school
performance, nothing was significant.

Then on pefe exam gave a card that was completely normal except B.P 90/60 so I mentioned in
children it might not be significant but I will observe this as well.

Then I explained the father that there are few things which can cause seizure. And name all of
above one by one but I have ruled out all on history I couldn’t find any of them in your child ,
sometimes it happens without any cause we call that unprovoked fit, as it has happened for the
first time we cant say that he is having epilepsy. Its not epilepsy at this stage . but I am very
concerned about him , I am going to keep him admitted in the emergency department for
observation if any other fit occurs or not .he will be seen by the specialist as well. Although we
don’t do investigations in first fit in which no cause could be found but it depends on specialist
he might consider doing investigations , these are ct brain , mineral levels of blood, blood sugar
levels etc. then again I reassured him that his son will be taken care well. And we will let him
know what to do in those situations if he has this again just relax. He looked happy
2 of 2key steps covered
Approach to pat relative7
History 6 Dds 6 Management plan 7 Pat education and counselling 6 global 6
Feedback 17-8-2018
Loss of consciousness(7yrs old with 1st time seizure),Task-hx,mx(“pefe provided)
One hand written sheet was provided with question which was from school teacher-generalized
seizure with tongue bite, leaking of urine, b4 fall down he was feeling unwell. mom was standing
inside was very worried, I gave reassurance he is in safe hand now, can we have a sit and thn talk
more?
Hx-mother didn’t give much info about b4 fall as she wasn’t there, no fever, no urti, no muscle
spasm or neck stiffness, no previous hx, no fhx of epilepsy, no head trauma, no vomiting or wt
loss, school performance very good, no bulleying hx, no stress in life, no behaviour change,
healthy baby with good social interaction, pbinds in short-fine.
mx-all reassurance,I said this is seizure,could be due to epilepsy (in lay term-abnormal electrical
discharge from brain cell), meningitis, head trauma/tumor, febrile convulsion(they r unlikely why
I explained )1st time seizure.
we cannt say epilepsy bt his s/s shows kind of epilepsy, bt need to confirm, so admit in ED,s
pecialist will do EEG, other basic ,blood test to see any infection and imaging of brain.” and said if
epilepsy thn treatment –medicine,f/u,long time medication,all precaution to take, some
restriction to job bt he will have normal life, it will nt cz any brain damage,not affect intellectual
activity,red flag.
at last I said again we r not sure if it is epilepsy or not, so all this info I am giving to u is only if he
is epileptic(score -5).
bt I got low mark in pt conselling(score-2), may be I was not empathetic as always,I used to talk
too much(dnt do this plz),I wanted to finish all my key point first thn empathy sympathy,bt at last
pt didn’t give me any anxious look bt gave a vacant look…bt see as I I covered all my key points
they passed me.
i found from my previous experience if they dnt get their all point they dnt pass people.,though I
was very empathetic, smiling face, communicating in good way in some stations they didn’t give
me pass as I missed some key point.

Feedback 5-10-2017
The child had seizure at school n brought to ED. Now he is awake but little bit drowsy. Vitals r
stable. The writing report from teacher abt the incident was given on the monitor ( describing
GTC seizure) .
Task history explain Dx n Ddx to parents
When i entered the room, father just came n asked me whats wrong with his child n he was
walking around the room. I reassured him abt his son condition n calm him down n asked him to
sit. Then explained him that i would like to ask him questions abt his son so that i can figure out
why this incident happened.
Positive findings-- first time, no fever no headache or rash seen before, no head injury, happy
family n happy at school, no developmental concern, no cancer features ( LOA, weight loss,
lymphadenopathy ) , no vomiting, immunization up to date.
Couldnt figure out provoked factors that cause seizure (last meal was before he went to school,
not on any meds, no medication at home that he could possibly take by himself )
DDx explained to him abt unprovoked fit, then provoked fit
Then epilepsy. He asked me he heard that epilepsy only occurs in brain damaged child. I
explained to him abt epilepsy. N meningitis n brain tumours.
149-Diabetic neuropathy Physical examination
A middle age woman presented to you with burning and tingling sensation of feet and leg calf
pain. She is known history of diabetes.
Tasks
-do sensory neurological examination (may be relevant or Lower limb exam as well)
-dx and ddx to patient

1-wash hands
2-Introduce yourself : Hi my name is --- I am one of the doctors here
3-permission: I can see from the notes that you have diabetes and you are complaining of pain
in your calf and some burning and tingling sensation of feet is that right?
In order to unravel the nature of the problem I would like to examine you this would involve me
having a look at your lower limbs, checking the sensation and reflexes of your legs , check your
pulses and watching you walking as well. Would that be all right?
Note (on the table there will be monofilament, tendon hammer and tuning fork)

4-Gait
-now if you could come off the bed and walk in front of me towards that wall (for example) and
turn around then if you can walk one foot in front of the other and now if you could walk on
your toes and now on your heels. perfect thank you.
Note/ keep yourself near the patient while examine the gait.
Look for
Symmetry
broad based gait, antalgic gait
foot drop
turning (slow or quick)
tandem gait
-now I would like to do a test called Romberg test to test your balance. If you could stand, put
your feet together. I am here to hold you if you fall. If you close your eyes for me now. And now
open them. Thank you
Note/ in the exam when you just start doing the gait and Romberg test the examiner will say
normal.

5-look (SSSDW)
-if you could lift your shorts up for me. Thank you
-I am just gonna have a look from the end of the bed
-I am just gonna have a look between your toes, bottom of your feet. I f you could lift this heel
up. And the other one.
-look for
scars, missing toes or limbs
-skin (color, ulcers, caluses, atrophic changes like shiny skin and hair loss)
-swelling
-deformity (charcot joints)
-muscle wasting
note/ in the exam inspection will be normal.
6-feel (TCP +MVPR)
-I am just gonna feel along your limbs (do not forget to compare both limbs)
feel for
TCP
-Temperature -capillary refill time -dorsalis and posterior tibial pulses
note/ in the exam all TCP will be normal.
MVPR (sensation)
-Monofilament
I am just gonna test the sensation in the bottom of your feet with this monofilament. It feels like
this
now if you could close your eyes and let me know when you feel it.
Allow light pressure for the monofilament to bend… test 5 points on the bottom of each foot
then go upwards. Same if cotton wool and pin prick were present.
note/ in the exam there could be monofilament or cotton wool and pin/ tooth pick for examine
the sensation. If both of them present I would choose to test with monofilament only.
note/ in the exam there will be loss of sensation both light and pain from the middle of the legs
down bilaterally a glove and stock.
-vibration
I would like to test the vibration on your feet with this tuning fork. It feels like this.
If you could close your eyes and let me know when you feel the vibration and when it stops.
note/ in the exam there will be both 256 and 128 HZ so choose 128 HZ.
note/ in the exam there will be loss of vibration same area like light and deep touch
-proprioception
I am just gonna test your joint position ao with this big toe.. this is up and this is down.
if you could close your eyes and let me know whether it is up or down. Ok
-reflexes
I ‘d like to test the reflexes with this tendon hammer. I just gonna tap gently with it.
test ankle reflex.
note/ in the exam proprioception and reflexes are normal.

7- Tone, power, coordination (usually no need you can skip because the task is to do sensory
neuro exam. However if the task was relevant exam or lower limb exam do say I’d like to examine
the tone, power and the examiner may say normal)

8-others
examine the shoes (any holes, correct size)

Differential diagnosis
1-diabetic neuropathy: form the history and examination the most likely cause why you are
having----- is diabetes you have when it is uncontrolled it can affect the nerve and vessels
supplying the lower limbs causing problem with sensation and calf pain.
2-alcoholic neuropathy
3-B12 deficiency

9-Wash hands and thanks both the patient and the examiner
Feedback 8-2-2018

feedback 9-5-2018
10 Case: Painful feet
Diabetic Peripheral Neuropathy PE.
ED setting. Middle age male, with history of uncontrolled Diabetes in the hospital because
glucose was on 11. He has been diabetic for the last 10 years or so. Now glucose is controlled
with insulin and you have been asked to perform neurological PE of his lower limbs because loss
of sensation.
Task:
Perform PE of lower limbs (I can´t remember if it was running commentary to examiner)
Give diagnosis and differentials with reasons to the patient.
Not need of taking further history.
On the trolley were 2 tuning forks (128 and 512), cotton ball, tooth pick and hammer.
Patient was lying down on the bed (a big guy on his 50s) already legs exposed, when I asked him
to walk the examiner said don´t worry perform examination with the patient lying on the bed) so
I started with inspection, then temperature, pulses, then focus to neurological, the only
abnormal finding was impaired of sensation from middle leg down (absent light sensation, pain
and vibration). I mentioned most likely diabetic peripheral neuropathy on his background of his
uncontrolled diabetes. As a differential, I mentioned a problem with the spine like a disk
prolapsed which can cause a compression in the nerves producing similar symptoms but also
absence of reflexes which wasn´t present in his case so this was unlikely. I didn´t have time for
more differentials. Passed. Global score 4

Feedback 6-6-2018
Station 4 DM foot examination- pass
Patient is middle age, male presenting with tingling, burning in the feet. Known DM. RBG today ?
18mmol/l.
Task: sensory neurological examination to lower limbs
Dx, ddx.
WIPE then general condition of the patient then I started with gait however examiner don't let
me do it and he said gait is normal then I asked examiner what about Romberg test and he said
‘ok, do if u want to’, so I thought that he doesn't want me to do it not to waste the time, so I told
him I will skip it . then I started with inspection , then sensory.. cotton wool, pin prick, vibration,
proprioception.( I forget to do monofilament test) then I told examiner, to complete my
examination, I want to check motor and co ordination, he said tone and power normal, so I
asked him what about reflex, he told me ‘check it by yourself’..I checked knee and ankle ,then
bell rang for 7 mins. so, I moved to next task... dx and ddx. (i started with sensory coz the task is
to do sensory neurological examination then i have extra time ,so I asked motor and others.)

Feedback 22-6-2018
Diabetic foot examination.
Task Sensory neural examination, Dx and ddx
PE
Consent, wash hands, GA, Gait (examiner said no need), asked to see the sole of shoes (LOL..
was not available)
Monofiament, fine touch, Course touch, Vibration, Propioception, reflexes

DDs
Your peripheral neves are getting damaged. There could be number of reasons
DM (most likely)
Other causes could be
Alcohol
Vitamin deficience
Medication SE
Feedback 9-5-2018
Dm foot exam, you will be hinted that it's Dm related with poor control Task- Perform PE,
report to examinor, report to patient and talk about dd to patient

Out side: the patient just have weird sensation on the leg and there is 3 tasks. This is a long cases
so i need to be quick. DD is DM neropathy, PVD, alcohol, B12 deficiency, stroke.

Inside: I want to do monophilament in the first step but they did not have so i go with the cotton
and stick. ( There are some unclean cotton and stick they put on the table), you have to use all
the hygien one which was inside the box- that is the trap for candida. I used all instrument like
turning fork, hammer. I check from Gait- inspection- palpation- movement and special test like
sensation, vibration... I finish it really quick before the small bell rang and i report to examiner,
also i told that i need to check heart, eye and upper limb, fundoscopy as well. I jumb back to
patient and say my dd and tell that he got this due to uncontrol DM, i will refer him to specialist,
advice him drink medication regularly. I still have time( i do not know why, i may be do
everything so quick as i afraid time shortage), so i talked about take care for DM food.....until the
bell rang.
Feedback 7-9-2018
Station 6 – PASS ( Bilateral Diabetic neuropathic pain)

Diabetic neuropahty (Diabetic foot examination)


Comes into your GP , poor controlled with Diabetes , less HbA1C 11% , RBS – 21? ( forget )
Feeling pins and needle sensation on both lower legs .
Not given UDS- result
TASK
1. Perform PE –
2. Explain Dx and DDX to the patient

Dont provide monofilament in the room. Gait - normal (verbal report) Explain shoe wear check(not
available)
Hand wash – Explain procedure , asking about pain or not . Then started with
Look ( expose to mid-thigh with rolling up the shorts –
both legs – hairy legs ( :D )
skin – nil skin ulceration , color changes ,
m/s – nil muscle atrophy or visible fasciculation
bone – nil bony deformity
check – hidden ulcers – between every toes , and back of heel of both feet .

Feel – TTPSPR
Temperature – nil warm , but slight cool at the distal toes bilaterally
Tenderness – nil bony tenderness on all joints bilaterally
Pulsation – CRT < 2 secs both , Dorsalis pedis – reduced ( Bilateral ) , Posterior tibial ( reduced )
bilateral
Popliteal arteries – got bilaterally

Sensation – looking around for 5 seconds – monofilament – after 5 seconds, examiner said that –
they don’t have monofilament today ( Love the examiner as waiting for around 5 seconds )

Start with cotton wools – can’t feel up to ankle ( both pathways )


Pin-prick ( lateral spinothalamic pathway ) – can feel dull only on skin below ankle joints
Vibration – given 256 tunning fork only – cannot feel until proximal bony prominence of bilateral
tibial
Proprioception – intact
Power- examiner told me straight away – NORMAL.
- Reflex – knee reflexes ( lying down ) – initially stiff the joints, tell him to relax and floppy –
Intact bilateral knee and ankle jerks.
Tell the examiner that – would like to do – Gait and assessing foot-wear – not available

TASK 2:
- explain diabetic neuropathy – explain causes – long term poor-controlled sugar and nerve is
damaged
Complications – can have skin infections, prone to get injuries
Other possible causes - alcoholic neuropathy , thrombophlebitis , cellulitis , could be mini-stroke
Feedback – Painful feet – PASS Global score – 5 Key step 1 to 5 – all YES
Approach to patient , examination , accuracy of examination , Dx and DDx – all – 5

Feedback 7-9-2018 diabetic foot examination Painful feet- PASS )


45 years age old male,comes into your GP , poor controlled with Diabetes , less HbA1C 11% ,
RBS – 21? ( forget )
Feeling pins and needle sensation on both lower legs .
TASK
1. Perform relevant PE (not sensory)
2. Present your findings to your examiner
3. Explain Dx and DDX to the patient
Feedback: PASS Global score: 4
Key steps: 5/5 Approach to pt: 4
Choice of technique of examination, organisation and sequence: 5
Accuracy of examination: 5 DDX: 4
2 mints thinking- Even though it's relevant, not specifically written sensory- I would
perform sensory first, then reflexes. Backwardly ITPRS- Inspection, then sensory, reflexes, If
I had time, would go for power & tone.
DDX:
1. Diabetic neuropathy.
2. Alcoholic neuropathy.
3. Vit B12 deficiency.
4. Referred pain from back.
5. Nerve entrapment
After introduction
WIPER
GA
Vitals
Asked for gait and romberg's test- examiner said all normal, no need to do it.
There were cotton balls, toothpick, 2 tuning forks(256 hz and another one with no marking
on it), no mono-filament.
1.I started with Inspection
2. Feel- temp, pulses, no need for checking tenderness in this case I guess.
3.dermatome sensation with cotton and toothpick on both legs- L4, L5, S1 lost
4.stocking pattern on both legs- sensory lost
5.Vibration ( I chose the fork with no marking)- lost at great toe, present at middle malleolus
-same on both legs
6. proprioception- normal
(I checked sensation at chest first, then started at leg)
I forgot to do monofilament test as that was not on the table.
Reflexes- Ankle reflex, knee reflex- normal
All these steps took my time, as I had to explain all these steps to patient and I repeated all
these steps in both legs as both of them were affected. I was also giving running
commentary as I was afraid that I might not get enough time later to explain findings to
examiner. I just mentioned to pt; I'm going to use some medical terms while examining you.
Please excuse me for this, I'm will explain all my findings to you later on.
After sensory exam, I directly talked to examiner that I would like to finish my examination
by checking tone, power, saddle sensation, anal reflex, SLR and fundo-scopy to check any eye
changes due to DM.
Then, I explained my findings to pt and said ddx. Pt was really co-operative. I had enough
time at the end. I asked my pt if he understood whatever I had said. He said yes. Then, I
asked examiner if I would present any summery to her or not. She replied no need, I got it.
Then, bell rang !!!
Feedback 26-10-2018 DM foot Passed (GS – 4)
Chronic DM patient. Her RBS given is quite high.
Task: perform sensoryneurological exam, tell DDx
Monofilament, cotton wool, jerk, tuning fork were given, no tooth prick was given.
Examination of lower limb
I started with Gait (examiner said N),
Then asked pt to lie down and about to test Tone and Power (examiner told me to skip).
Then, I checked sensory using:
 Monofilament to the sole of foot. (There is sensation loss in over 5 points esp
upper parts of sole, patient said Yes to some points of lower part of soles (mayb
only 5 points were instructed to patient, I was confused.)
 cotton wool from distal to proximal parts of legs, starting from dorsum surface of
foot to upper part of the leg (loss of sensation up to ankle on both limbs)
 toothpick: When I asked for toothpick, examiner said you can skip as patient
requested not to test with tooth prick.
 vibration sense (on both limbs, there is loss of sensation on big toe, sensation was
intact on malleolus).
 Proprioception
Short bell rang. Examiner told me to move next task. I said “ideally, I’d like to complete with
testing proprioception.”
DDX:
Then, I explained DDx with findings what I got (peripheral neuropathy, most likely due to
chronic uncontrolled DM, or vitamin deficiency such as B12, folic acid, alcohol related
neuropathy.
Feedback 8-11-2018
Feedback 13-12-2018 Health Review - PASS

GP; 72 year old who is a diabetic regular patient to your clinic is now complaining of burning pain in his
toes and pain in the calf which gets worse with walking uphills. (didn’t mention about diabetes being well
controlled or not)
Task: Now, you have been asked to do physical examination before your senior comes
Explain Dx and DDx

2 min thinking: I don’t know if I should do PVD examination for calf pain or lower limb neurological
examination for burning pain. So I just decided to play along after seeing the instruments given.

Physical Examination

Greeted both examiner and patient.


I started with general appearance and moved on to lower limb. On inspection, no muscle wasting, hair
loss, skin colour discolouration, fungal infection, nail changes. Then did temperature assessment, and
palpated the pulse (dorsalis pedis) which was palpable. CRT was within 2 secs.
Then I quickly had a quick look at the metal tray and saw monofilament, tuning fork and hammer. So I
decided to start with monofilament test. Checked 10 points on the right side first. Perhaps due to
miscommunication, there is loss of sensation on 2/3 points. The left side is normal. I said “there is loss of
sensation at 3 points on the right side on monofilament test”. Then said I wanted cotton wool and pin
prick to check fine and pain and was told just do the examiation with the available instruments. After that,
checked vibration – there was loss of vibration sense below knee joint bilaterally. As I started giving
instructions for propioception, examiner told me to skip. Both ankle and knee reflex are normal.

Dx and DDx
Sensation is normal but loss of vibration sense below your knee. Your burning pain is due to damaged
small nerves in ur feet and toes probably due to poorly controlled diabetes. Sometimes, alcoholic patients
can present with similar symptoms. Sometime, patients with Vitamin B12 and folic acid defeciency can
have this symptoms. Other possibilities for you symptoms are individual nerve injury (I was referring to
mononeuropathy), back pain, tarsal tunnel syndrome. For your calf pain, can be due to narrowed artery
inside the leg or calf muscles called PVD. Bell rang.

GS – 5
Key steps – 1, 4, 5 – YES; 2 and 3 – NO
Approach – 6
Choice and technique of examination, organisation and sequence – 4
Accuracy of examination – 4
Dx and DDx – 5

(On monofilament test, I made some mistakes regarding technique of the examination so maybe that is
why the patient didn’t say YES on 2 points on the right side but I was foolish to say there was loss of
sensation at 3 points as I was thinking about previous cases. During explanation, I corrected that by saying
the sensation is normal, though)
150-Spider bite
Note/ unfortunately, there is no feedback yet for this case and it is made depending on a
comment of someone sat that day. So this lecture is just an outline of what it could be. Real
exam will reveal the complete picture.
Case/ Young boy presented to your GP with leg pain
tasks
-history
-PEFE
-DX, DDX

History
1-Pain questions
-severity + painkiller after allergy question
-can you tell more about what happened? (child woke up from sleep with leg pain)
-when did it start? (since this afternoon)
-has it started suddenly? Constant or come and go? Is it getting worse?
-where exactly the pain is? Does it go anywhere else?
-is he able to walk or bear weight?
-can you describe it for me?
-anything make it better or worse?
-what was he doing before the pain started? (was in the shed with his father)
-has this happened before?
2-Associated symptoms questions
-any recent viral infections? (HSP, Transient synovitis)
-any fever or night sweating? (Cellulitis, septic arthritis, osteomyelitis)
-any rash?
-any joint swelling? (Septic arthritis)
-any swelling in the site of pain? is there any redness? Is it hot to touch? Itchy? (Cellulitis)
-any pain in his tummy? Nausea or vomiting?
-any loss of weight or appetite? Lumps or bumps?
-any trauma or insect bite? (Trauma, bite, cellulitis)
3-Anaphylaxis questions
-any shortness of breath, noisy breathing, cough?
-any difficulty swallowing?
-has he had any allergies
4-Dehydration questions
-is he drowsy
-how is his waterworks?
-is he eating well?
5-BINDS
-immunization
-is he thriving and growing normally
-contact
-PMH

Physical examination from examiner


1-General appearance
-Dehydration
-rash, signs of bruising, bleeding or trauma
-pallor, jaundice , LAP
2-Vital signs
3-Growth chart.
4-Quick CVS and chest (wheeze)
5-Leg examination (compare both limbs)
-inspection (examiner will give you a picture of swelling and redness just above the ankle)
-palpation: if it is hot to touch, tenderness, point tenderness
-move: any restriction of range of movements
-complete with the knee and hip joints as well.

Explanation ??? (Not sure as no feedback for it)


- from history and examination there are several causes why your child has leg pain.

- I think most likely your child has been bitten by a spider while he was in the shed causing local
reaction. His symptoms could be due to cellulitis, which is infection and inflammation of the
skin and tissues under the skin, usually by a bug; a bacteria that entered through a crack or
break in the skin, leading to swelling, redness, warmth and tenderness of the affected area.

- Other possibilities could be due to infection of the bone we call osteomyelitis. Could be
infection of the joint we call septic arthritis. Could be due to direst injury or trauma but he has
no fever, no joint swelling and no signs of injury so they are unlikely.

-let me assure you that most spider bite in Australia are not serious and not poisonous.

Recall 9-2-2018
Young boy with leg pain since this afternoon, wake up screaming from his nap , not feverish,
doctor will give you a photo of localized redness & swelling above the ankle joint , he was at the
shed with his dad before sleeping
151-Lymphadenitis
Case 1 (post viral Lymphadenitis)
4 year old child mother anxious because of lump in his neck
tasks
-take history
-physical examination findings from examiner
-dx and ddx to patient
-management

History
1-Lump questions
-when did you notice it? Has it appeared suddenly or gradually, have you noticed any change in
shape or size? (either suddenly recognised it yesterday or for 2 weeks),
intermittent or constant? (Constant)
-where exactly is it? (Front of the neck),
-is it painful? (No)
-have you noticed any change in the colour of the overlying skin? (No)
-is this the first time (yes)

2-Differential diagnosis questions


Post viral Lymphadenitis
-has he had any recent viral infection or any other infections? (Common cold 2-3 weeks ago).
Bacterial Lymphadenitis
-any pain, fever, itch (no)
Lymphoma
-any night sweats? (No)
- is he eating and drinking well? Any change in weight recently? (No)
-any lumps or bumps around the body? (No)
Leukemia.
-bruising, bleeding or rashes? Joint swelling? Any recurrent infections? (no)
Tuberculosis and IMN
-Travel, contact, sore throat (no)
Cat scratch
-any pets at home?

3-General questions and BINDS (his general health was otherwise very well)
-Past medical or surgical history
-medications and allergies? (Imp)
-Contact history? (Imp)
-family history of similar problem? (Imp)
-BINDS (nothing positive)
Dx and Ddx
1-viral or bacterial lymphadenitis
2-lymphoma
3-leukemia
4-infectious mononucleosis
5-congenital lumps

PEFE
1-GA (no dehydration, rash, pallor, jaundice )
2-VS (all normal)
3-Growth chart (normal for age)
4-lump: site, size, consistency, mobility, tenderness ( in the anterior part of neck, size of 2cm ,
mobile, soft, not tender, no axillary or inguinal LAP)
5-ENT (tonsils enlarged but not inflamed).
6-abdomen (no organomegaly)
7--others all normal.

Diagnosis
-from history and examination most likely lymphadenitis which is inflammation of lymph glands
of the neck probably due to viral infection he had 2 weeks ago, lymph glands persist enlarged
after infection cleared
-let me assure you that it is self-limiting and may take time to return to normal.
-other possibilities are …… but less likely.

Management
-I will give you reading materials
-Any time you these lumps are increasing in size, developed fever, night sweating, change in
weight or appetite please let me know.
-Follow up
Feedback 21-2-2018
CASE 3: PAEDS- REACTIVE LYMPH NODE –PASS
7 years child with swelling in neck , brought by mother, she is anxious as the childs cousin
diagnosed with leukemia.
Tasks: history, Pefe, management.
Greeted mother and child, swelling history- few days, not progressing rapidly, no ulcer, no skin
changes, no other swelling no trauma, recent flu , no ENT problems now, no weight loss/
appetite, active child, sleep good , no bleeding or recurrent infections.
BINDS normal
PEFE: no PICKLE except for the neck node no rash, growth chart- some low percentage asked
about the previous charts for more than two standard deviation- normal.
ENT normal, no skin lesions in head or neck no hepatosplenomegaly
joints normal
reassured her that this is not leukemia and why I am ruling out, explained lymph and drained
by nodes which filter bugs, might be due to recent illness he is having the node now.
mentioned redflags: increase in size, pain, ulcer, fever arranged follow-up

Feedback 18-9-2018 Lymphadenitis


5. neck swelling - lymphadenitis
18 months old baby, father noticed a neck swelling
Task: further history, PEFE, Dx and Ix (not sure included Mx or not)
Hx: flue like symptom 2 weeks ago, waterlike nasal discharge, had fever till now, this few
days father noticed a lump in the neck and grows bigger, father concerned it was
lymphoma (I didn’t reassure the dad much.. hence approach score is 2, I think…)
PE: ill looking, temperature 38.9
neck mass 3 cm, tender movable, consistency soft
ear: some sort of congestion,
red in throat, no exudate
I forgot rash, or neck stiffness and so on….
Dx: lymphadenitis (can be viral or bacterial infection) D/D nasty growth less likely
Ix: FBE, electrolyte, LFT, renal function, blood culture and throat swab
I said a lot of things that may not be relevant, I said your baby is currently on high fever
and it had been like that for two weeks,
I am concerned that the infection had spread to other part of the body. To be safe I will
send him to hospital to rule out sepsis.
Nasty growth is unlikely because it is usually not tender.
Score: 4
Key step: yes/no/yes/yes
Approach: 2, Hx: 4, PEFE: 4, Dx/D/D: 4, Mx: 4
Feedback 18-9-2018 Lymphadenitis
You are a GP and the next pt in your clinic is an 18 months old baby who came with
her mother. The mother noticed a lump on the right side of his neck 2 days ago.
Since then she noticed that the size of the lump is increasing. There is no redness
around the lump but tenderness present. Mother is concerned because a baby of
her friend has recently been diagnosed with Lymphoma.So she wants to know more
about the lump and what to do from here.
Your task is to
1. Take further history regarding the cause of the lump. ( you shold spend no more
than 4 mins in this task)
2. PEFE ( Examiner will give findings only what you ask for)
3. Discuss the Dx with the pt.
4. Discuss about Mx plan for this condition.
My approach:
I went in and their was a worried mum so I assured her that she is in expert hand. Then I asked
all the history. First with the lump , She just said its in the right neck and she doesn't know
anything
else coz she didn't touch it. Then I asked recent infection, fever history. The baby was sick 1
week ago for fever and runny nose. The baby is well alart, but a little irritable lately. BINDS,
family history, home situation all normal. Bowel bladder normal. No other lumps and bumps in
anywhere in the body except for the neck one.
On PEFE: GA and growth chart was normal.
Vitals: Temp was 38 others normal
I Asked everything about the lump: site, size, colour, tenderness, hot to touch, skin change,
border, mobility, fixed or not, consistancy. Everything was pointing towards lymphadenitis.
then I asked about bone tenderness and scratch mark over the body ( as mother was concerned
about lymphoma)
Then I explained it as Post viral lymphadenopathy( why didn't I say Lymphadenitis I don't know.
exam pressure I guess) And explained everything about it.
Regarding Mx plan I didn't know much or I forgot then. So I just said, give panadol as he still has
fever, keep him well hydrated , rest and this will go away spontaneously. But still I would
arrange some investigations to rule out other dds including Lymphoma.
Gave her 4Rs.
AMC Feedback:
Station 05: Neck swelling
Grade: Pass Global score: 4
Key steps: 1,2,3,4 yes,yes,no,no
Assessment domain:
Approach to pt/relatives: 4
History: 4
Choice & technique of examination, organisation and sequence: 4
Dx/DDx: 4
Mx Plan: 3
Karin case
Posterior Triangle Lump (Postviral [Reactive] Lymphadenitis)
Case: A mother of a 3-years-old boy is in your GP clinic as he found a lump on his neck. She is
worried about it because her nephew was recently diagnosed with lymphoma.
Task:
a. History
b. Physical examination (well looking child, posterior triangle of neck 1x1.5cm, soft, mobile, non-
tender, movable; mild redness of the throat)
c. Diagnosis and management
Neck lumps in children - 80% benign and 20% malignant
- If <2cm in diameter + hard + soft + mobile  postviral
- If >2.5 cm in diameter + fixed + hard/firm  suspicious  may do biopsy and further examination

- Posterior triangle: bronchial sinuses/cyst, cystic hygroma, tuberculosis, lymphoma.


History
- Where exactly is the lump? - When did you notice it? - Is it painful?
- Did he have a recent URTI or other infection?
- Did you notice any change in size or shape of the lump?
- Change in color of overlying skin?
- Is it hard or soft? Is it fixed or mobile? - Any other lumps and bumps in the body?
- - Any previous history of fever, night sweats? - Is he pale or lethargic?
- Any symptoms of tiredness, recurrent infections, bleeding, bruising?
- Any change in his appetite? - Did you notice any mass in his tummy?
- Waterworks and bowel motions?
- Is he in childcare/schooling? Contact history? Anyone else at home having a similar problem?
- Any recent travel history interstate or outside Australia?
- BINDS
- FHx: other history of lymphoma or cancers in the family?
Physical Examination
- General appearance: pallor, jaundice, dehydration
- Vital signs
- growth chart - Skin: bruising, pallor or signs of infection
- ENT with LAD
- Chest and heart – Abdomen
Investigations: FBE, ESR/CRP
Diagnosis and Management - Most likely it is postviral lymphadenitis and it is a common
condition after a viral infection.
It is a benign condition and not a cancer. It is self-limiting and no treatment is needed.
- Antibiotics? No.
- If concerned may do investigations but if persistent, refer to pediatrician.
- Review after 1 week to see if the lump is decreasing.
- Red flags: lump is increasing in size, becoming hard/fixed/immobile/painful and if he is getting
pale or lethargic, weight loss etc.
Case 2 (Bacterial Lymphadenitis)
Case(10/2/2017)
21 month girl Tender LN lt side of the neck
Task
History ,
PEFE,
Diagnosis and management,
HOPC:
- swelling in the left side of the neck the mother is concerned it might be lymphoma or
leukaemia .
- For 3days ,
- preceded bt URTI 1 w ago ,
- pain ,
- no respiratory distress
- febrile ,
- d/ds no wt loss no lumbs and bumbs , no night sweats .
- no recurrent infections - no bleeding anywhere - no SOB with exertion or easily getting tired
- no travel history
- no history of contact.
- well baby qs not feeding well ,tired lethargic , wee and poo are ok
- BINDARS :
- allergic to penicillin.
PEFE:
GE: lethargic sick looking no pallor no jaundice
Vitals fever high.
Growth chart normal.
ENT: congested tonsils , ears I forgot what he said ,
Neck : left sided mass a lymph node like surrounding redness , hot and tender to touch .
Chest, CVS : N.
P/A : no organomegaly .
Diagnosis:
Reassure mother not leukemia or lymphoma (why), Not viral as your child is sick looking , not
feeding , and inflamed lymph nodes .It is most likely bacterial .
Management:
Admition as not tolerating food and sick ,Start on antibiotis (don’t forget the allergy as I did first
and the remerbred ) mother was trying to help by insisting to know the name of antix in order
for me to remember to ask ;) IV FULIDS and paracetamol for the fever

Thank god I PASSED


152-Clavicular fracture examination
Recall of 10-2-2018
recall of 5-6-2018
young male clavicle fracture,focused pe,explain x ray finding to pt,do management with given
material.
Recall of 5-5-2018
Male pt had a fall. You will explain XR, perform examination on clavical then apply a sling.

Explain the x-ray.


-Hi john how are you feeling now? Do you have any pain?
-I have your x-ray result with me to discuss with you. This is the x-ray of your shoulder area
showing the right collarbone and it showed that your collarbone is broken here (point to the
place with finger) in the middle with just minimal displacement.
-this is quite common in athlete individuals usually follows a fall onto the shoulder and causing
symptoms like pain, swelling, and some tenderness. You can expect a small bump on that area
but it is just of no significance.
-this fracture usually managed conservatively by applying a sling without the need of any
operation.
-Before that, I just need to examine you.

Examination
1-General appearance (if in pain)

2-look (SSSDW)
-scars
-swelling
-skin (colour, bruising)
-deformity
-muscle wasting

3-Feel (TCP) + sensation


-temperature
-CRT
-pulse
-C5, C6, C7, C8, T1

4- Check C8-T1 motor function:


Ask patient to hold your finger and do not let it go, spread finger and do not let me push them
together

5-chest examination (IPPA)


-to rule out pneumothorax
Talk about the sling
-I am gonna apply this sling to elevate and support the arm so that to allow healing and also to
help with pain and minimise displacement,

- Perform the sling like this (if it was the one in handbook)
1-the affected arm is 45 degree high elbow.
2-perform like broad arm sling
3-knot

Advice
- you need to keep it for 4 weeks. I will prescribe you some painkillers.

- I need to see you again after 7-10 days to assess the healing process and the function of your
arm. X-ray might need to be repeated.

-during that time, make sure to exercise your elbow joint, wrist within the range of pain to
prevent stiffness and you can take off sling.

-I will also refer you to physio to teach you some active exercises.

-try to avoid any contact sports or lifting heavy things. Do not put any weight on this arm to
allow appropriate healing process.

-you can take off the sling during the sleep but you need to support the arm on a pillow by your
side.

-you can return to work once the arm function returned to normal which might take 2-4 weeks
and depending on the process of union.

-red flags: numbness, decrease sensation, bluish colour hand, significant movement of bone
report to ED.

Any questions?

Case (25/8/2017) (5/5/2017)


Clavicle # - apply triangular bandage. Xray displaced fracture no pneumothorax.
Management.
Case (23/2/2017)
Clavicular # ( handbook case ) ( sling on table )
Explain XR & its implication to the pt. Perform relevant PE. Initial Mx with material given
What is the correct way of putting a sling for clavicular fracture? Is it still the same as Handbook
p563?
Feedback 11-12-2018
Patient had a recent bicycle accident where he fell on this right shoulder. Explain X-ray to patient
(clavicular fracture and its implications PE and and mx with available prescription in the room.5
tasks<so please switch fast>
I advise you don’t miss pneumothorax and brachial plexus injury in this particular case and you
will be safe.
Give painkiller.All things provided for neurological examination , so did upper limb neurological
examination. Explain patient xray. I refer this patient as it look displaced to me(to be safe side),
then use cloth provided to make a triangular bandage. check crt after applying sling. While
doing sling keeps patient comfortable. And then bell ring.
Passed with global score- 4, key step no. 4 missed.

Feedback11-12-2018
Station 6 (Shoulder injury) Global score 3 Fail

A male patient had a bicycle accident and fell over his shoulder.
Explain the X-ray to the patient, perform PE and management.

I totally ruined this case.


And the examiner was not very friendly.
First I washed my hands.
The approach to the patient wasn’t bad.
I started explaining the X-ray. But that was when I had my first doubt. The clavicle fracture had many
fragments and it seemed dislocated from the shoulder. All the cases I had seen before were of aligned
fracture. So, I wasn’t sure if the Mx would be only immobilization and painkiller.
I told the patient what I would perform and gained consent. I started telling him that I would not move his
arm due to the fracture.
Than I started the PE: on inspection I mentioned the bruise (there was a make up over the collarbone) and
that skin was intact. Than I felt the temperature on the hands, the pulse and the CRT. When I told that I
would gently touch the clavicle the examiner shouted: “Don’t touch the fracture!” and seemed angry. I
was so nervous that I checked for the hands movements and totally forgot to check sensation.
Than I assembled the arm sling asking patient if he was comfortable.
I explained him that for most clavicle fractures a sling, painkiller and physiotherapy would be enough. But
in his case I would ask for the opinion of an Orthopedist because his fracture seemed unstable and he
would decide if a surgery would be necessary.
The bell rang and I was glad I left.

Key step 1:yes. Choice & Technique of exam, organization and sequence: 4
Key step 2:yes. Diagnosis/Differential diagnoses: 4
Key step 3:yes. Management Plan: 2
Key step 4:no. Performance of procedure: 4
153-Tremor examination
Case 1 (Benign Essential tremor AMC)
55 y/o man have come to your GP with shakiness in both hand more on left…he has this symptom for
many years getting worse recently. He takes alcohol which can helping a bit. He is concerned about
having Parkinson disorder as his family succumb to one.
Task:
-PE upper neurological
-relevant neurological exam,
-tell dx and ddx with reasons to the patient by giving running commentary to the examiner.

Case 2 (Risperidone induced Karin + AMC)


Case: A 32-year-old woman comes to your GP practice. She has history of depression with psychotic
features. She is on risperidone and mirtazapine. She developed tremor three weeks ago and she
comes today for prescription renewal.
Task
a. focused History
b. perform Physical examination
c. Explain condition and Management

Examination steps
1-Gait
-assess walking (Parkinson)
*posture (stooped posture)
*initiating movement (slow)
*shuffling gait/ festinate gait
*reduce arm swing
*slow turning
-Tandem gait (cerebellar)
-Romberg test (cerebellar)
-pull test (stand behind the patient and tell him that you gonna pull him backward so try not to let
me do it ok) (Parkinson)

2-Hands
-resting tremor/ pill rolling tremor (Parkinson or parkinsonism)
-fine tremor by stretching arm+ place paper (thyroid)
-finger nose test (intention tremor)
-move both hands towards your face (benign essential tremor) (if you do not how to do it forget)
-flapping tremor (chronic liver disease or any organ failure)
-feel palm for sweating

3-Arms
-Tone
*wrist (cogwheel rigidity)
*Elbow (lead pipe rigidity)
-Power
-reflex
4-Face
-look
*mask facies
*thyroid eyes
-eye movements + nystagmus
-glabellar tap

5-Neck
-thyroid mass if only present then palpate

6-Special tests
-writing micrographia
-open and close hands as fast as you can
-piano tapping
-twiddling (one hand around other)
-ask him where do you live?  hypophonia

7-Complete with lower limbs and cranial nerves, chest, heart and abdomen

Differential diagnosis
1-benign essential tremor
2-parkinsonism
3-parkinson
4-hyperthyroidism
5-cerebellar disease
6-liver disease
7-dementia
8-vascular stroke

Note/ for risperidone induced - refer to specialist who might change or reduce the medication.

Note/ for BET say may be familial, anxiety, stress, increase coffee intake, then talk about ddx , then
rule out ddx by doing Ix especially (FBC, TFT, LFT)
Case (13/10/2017)
55 y/o man have come to your GP with shakiness in both hand more on left…he has this
symptom for many years getting worse recently. He takes alcohol which can helping a bit. He is
concerned about having Parkinson disorder as his family succumb to one.
Task:
-PE upper neurological
-relevant neurological exam,
-tell dx and ddx with reasons to the patient by giving running commentary to the examiner.
Introduced myself washed hand asked for consent.
RP was sitting behind the table.
Gait+ Romberg test
- I asked to walk few steps examiner said it is NL don't assess gait and don't make patient stand. -
I said OK I want to check Romberg test as well examiner said NL.
hand examination- I started by hand.
- No muscle wasting, no nicotine stain, no palmar erythema, no sweaty hand.
- Obvious tremor on both hand. No flapping tremor. Radial Pulses were NL bilaterally.
pulls test
- In Head I asked the patient to stand I did Pull's test I said negative as in parkinsonism pt will fall
back with few steps backward with this test.
Eyes
I said now I want to check eyes. No exophthalmos, No lid lag,
mask face + glabella tap
I forgot to mention mask face and Glabella tap!!!!!:(
Thyroid examination
Then I went for thyroid inspection I said there is no enlargement no skin changes.
Then I saw a glass of water. I asked the pt to take a sip of water. His hands were severely shaking
I said sorry I annoyed you let me help you. Then with consent of patient I went to the back
starting palpating the thyroid nothing was there positive no lump no nodule no pain no
warmness asked pt to swallow. Then I said now do tap on table like playing piano examiner said
what is that I said piano tapping test!
finger nose test/ micrographia
And said finger to nose and twiddling and micrographia however mistakenly I said this is
micrographia instead saying there is no micrographia!!!:(
I forgot Dysdiadochokinesia!!
I want to finish my examination with Abdominal examination and lung and heart and cranial
nerve examination.
So I said look the problem that you have is called benign essential tremor have you heard of it?
It is muscular problem not serious which is running in family and not parkinsonism. Still other
condition like alcohol consumption as well as hyperthyroidism and liver disease can be there
but I have ruled them out in physical examination. However I want to rule them out with
investigations as well.
Feedback: Shaky hands, PASS(G.S:4) Key steps:1,2;NO..3 and 4 Yes
Approach to patient:5 Choice and technique of examination and organization and sequence:4,
Accuracy of examination:3 Dx/DDx:4
Feedback 28-11-2017
Station 11: health review – pass
Patient came concerned as friend said he has Parkinson’s. Came for a check. Task:
-Do examination for this reason / to check if he has Parkinson’s or not.
-Talk about the reason behind this
( Positive pill roling movement, micrographia, glabellar tap and loss of swinging of arms)
Key steps: 4/4 Approach: 5 Choice and technique of exam: 4 Accuracy of examination: 4
Dx and DDx: 4 Global score : 4 – pass

Parkinson’s examination:
-WIPE
GA=Mask face, dropped question mark posture, abnormal movement
Vitals- may be postural drop, ask for the BP specially
Speech.
Ask the patient to get out of the chair ( observe if he can stand up easily )
Gait : could you plz walk to the wall ( observe the gait , short shuffling gait, festinate gait and lack
of arm swinging / if it is hard for him to initiate walking .. Stop and turn around quickly (takes a
big turn) and walk heel to toe
Romberg sign- Assess coordination by asking him to close his eyes and make sure to put your
arms for the patient to not fall
Inspection :
Hands : resting tremors , pill rolling ,
Ask him to act like playing piano with both hands
Bamba

Opposition
Ask him to swing his hands over each other fast ( bradykinesia )
Write his name ( look for small crowded letters )
Ask him to relax his arms and passively flex it ( cogwheel rigidity )
Face : mask face, Dripping of saliva, eye brows for sweatiness and seborrhea
Weak upward gaze.
Glabellar reflex : by taping on the forehead and ask him to try to not blink ... If he blinks too
much ( positive sign )
To complete my examination I would like to do upper and lower limb neurological examination,
complete neurological xam
mx- withdrawal of the precipitation factors
neurology opinion, rule out other condition causing tremor- LFT, TFT,
Case (22/4/2017)
Risperidone induced tremor- examiner and patient was gud in this case.
- they gave in stem he is psychotic and now he don't have symptoms he has given risperidone and now he
came for prescription renewal and tremors
- he was gud actor resting tremors he gave
- asked focused hx- after this medication u got this periods then he said yes. Smoker- no , alcoholic- no.
- Examination
*asked to walk him examiner said no need neurological exam
*only hand , wrist and arm exam then I checked temperature, pulse, crt, tone- rigidity was there..
*told him to write I am great he wrote no micrographia they were laughing for it..
*then tap on the table he did well no bradykinesia I said and
*forgot reflexes and did them again was normal..
then said this shakes are due to medication so I will refer u to specialist they will reduce or change the
medication called atypical antipsychotics which have same affect like typical ones with less side affects and
asked him is it ok he said yes..
then don't worry once we stopped this medication u will be fine.. thanked both of them and came out.
They were happy only.

Recall 15-3-2018
GP, pt coming to you because his frds are telling him he is hving parkinsonism.
Tasks: perform examination (yes no specific mentioned), give dx and ddx to patient (no resting tremor,no
head tremor, no intentional tremor, fine tremor +, mask like facies +, walking well, finger nose good, no
micrographia, slow movement, slow speech +, tone a little bit increased, glabellar +).

Feedback 25-10-2018
Scenario: health r/v (considering it as the P.E station)
Stem. Old man comes to you because his friend think he has got parkinsons dx.
Tasks: do relevant p.e
Explain dx ddx

I entered room. Greeted the pt. explained him that because his friend has shown concern and I appreciate
that he has come to me so we can find out if any issues. Asked any pain. Did the WIPER
Did all the steps of parkinsons p.e
Only positive was short shuffling gait with loss of arm swing. Rest all negative, no tremor all special test
negative.
Took pen n paper and jotted down the ddx and why I don’t think he has them.
GS: 6
4/4 key steps covered Approach to pt: 6
Choice & technique of examination: 6 Accuracy of exam: 5 Dx/ddx : 4

Feedback 25-10-2018
The scenario for a man whose friends told he might have Parkinson, so came for check up.
Task:
PE relevant.
Give DD
I was confused, whether to examine for tremor or direct Parkinson.
I did all for Parkinson, checked reflexes. Gave dd, then thought should do cerebellar signs all of a sudden-
mentioned it and the bell rang.
Thanked pt.
GS 5
Feedback 27-10-2018 Shaky hands
50+ year old male patient with complaints of shakiness in his hands. He noticed that it gets better with
1- 2 drinks of alcohol. He is concerned about having Parkinson disease as he has family history of PD. He
has no issue with walking or maintaining his balance. BP, PR were given in the stem which are normal.
Task

- Perform upper limb examination and other relevant examination

- Explain the differentials to the patient

- 50+ year old male patient was sitting in the chair.

- WIPE

- introduced

- washed hands

- get the permission / consent to perform the examination

- explained what I’m going to do.

Examination

Hand

- asked the patent to put out his hands in front of him

- found coarse tremor (then I skipped looking for the fine tremor)

- pill rolling tremor was absent

- looked for

- warm and moist hands (negative)

- pulse (normal)

- BP (mentioned that I will skip the BP as it was done already.)

- there was no tachy/bradycardia or postural hypotension

- then proceed to upper limb neurological examination

- tone (looking for cogwheel and lead pipe rigidity) (normal)

- power (normal)

- reflex looking for hyperreflexia (normal)

- asked the patient to do hand twiddling movement (normal)

- asked him to write down his name on the paper (micrographia). There was some shakiness
while writing his name, but there was no micrographia. I mentioned that there is action tremor present
in my patient.

- did finger nose test and dysdiadochokinesis. (normal)


Face

- asked the patient to take off his glasses

- looked for exophthalmos (negative)

- anxious look (negative)

- glabellar tap (negative)

- asked him to say ‘ hippopotamus’ and british constitution (normal)

- forgot to mention masked like faces and neck swelling.

I’d like to complete my examination by doing abdominal examination looking for features of chronic
liver failure as my patient is a chronic alcoholic.

Differential diagnosis

- Mr. ——-, after going through thorough examination, I haven’t found any features of Parkinson
disease.

- Your symptoms are most likely due to a condition what we called benign essential tremor.

This kind of condition tends to be relieved by drinking alcohol, but this is not the right solution for this
condition. I’m concerned about your drinking habit.

- there could be other possibilities that can lead to shakiness such as

- chronic alcohol drinking

- cerebellar problem (which is the part of the brain that control the balances)

- Parkinson disease

- thyroid problem

- certain medications but these are less likely in your case. Grade - pass

Global score - 5

Key steps

1,2,3,4 - Yes, Yes, Yes, Yes

Approach to the patient - 5

choice and technique of examination - 5

organization and sequence

accuracy of examination - 6

Dx/DDX - 5
Talley o Connor Parkinson’s disease (7th)
This is a common extrapyramidal disease of middle to old age (1% of people older than 65)
where there is degeneration of the substantia nigra and its pathways. This results in dopamine
deficiency and a relative excess of cholinergic transmission in the caudate nucleus and
putamen, which causes excessive supraspinal excitatory drive.
There may be a history of insidious and asymmetrical onset. Non-specific symptoms (sleep
abnormalities, constipation, depression and dementia) may precede or accompany the classic
tremor.

A-INSPECTION
1- Note the lack of facial expression, which leads to a mask-like facies.
2- The posture is characteristically flexed and there are few spontaneous movements.

B-GAIT AND MOVEMENTS (bradykinesia + postural instability)


- Ask the patient to rise from a chair, walk, turn quickly, stop and start.
1- The characteristic gait is described as shuffling— there are small steps, and the patient hardly
raises the feet from the ground.

2- There is often difficulty in initiating walking, but once it begins the patient hurries
(festination) and has difficulty stopping.
- The Parkinsonian patient seems always to be trying to catch up with the centre of gravity.

3- There is a lack of the normal arm swing.

4- Walking heel-to-toe will be difficult.

5- Testing for propulsion or retropulsion (propulsion involves pushing the patient from behind
and retropulsion pushing from in front) is of uncertain value and must be done with some
caution because the patient may be unable to stop and may fall over. You can stand behind the
patient and pull him or her backwards, but should stand braced to catch the patient.

6- Bradykinesia (a decrease in the speed and amplitude of complex movements) may be the
result of a lesion in the nigrostriatal pathway (a dopaminergic pathway), which affects
connections between the caudate nucleus, the putamen and the motor cortex, causing
abnormal movement programming and abnormal recruitment of single motor units.
Two simple tests for this are finger a- tapping and b- twiddling.
Ask the patient to tap the fingers in turn onto a surface repeatedly, quickly and with both
hands at once.
Twiddling is rotating the hands around each other in front of the body.
These movements are slow and clumsy in Parkinsonian patients but obviously depend on motor
and cerebellar function as well.
c- Difficulty getting out of a chair can be another sign of bradykinesia
d-patients often have difficulty turning over in bed.
- Kinesia a| radoxical is the striking ability of a patient to perform rapid movements (especially if
startled) but not slow ones; for example, the patient may be able to run down the stairs in
response to a fire alarm but be unable to stop at the bottom—this is not a recommended test.
C-TREMOR
1- Have the patient return to bed. Look for a resting tremor,
2-which is often asymmetrical.
3- The characteristic movement is described as pill-rolling. Movement of the fingers at the
metacarpophalangeal joints is combined with the movements of the thumb. Various attending
movements may also occur at the wrist.
4- Distraction tests a- On finger–nose testing the resting tremor decreases, but a faster action
tremor may supervene.
- Tremor can be facilitated by getting the patient to perform b-‘serial 7s’—take 7 from 100,
then 7 from the answer and so forth (mental stimulation) —or to c- move the contralateral
limb (e.g. by rapidly opposing the contralateral thumb and fingers).

D-TONE (rigidity)
Test tone at both wrists. The characteristic increase in tone is called cogwheel (wrist) or plastic
(lead pipe -elbow-) rigidity. Tone is increased with an interrupted nature, the muscles giving way
with a series of jerks. If hypertonia is not obvious, obtain reinforcement by asking the patient
to turn the head from side to side or to wave the contralateral arm. Cogwheel rigidity occurs
because the exaggerated stretch reflex is interrupted by tremor.
Remember, the signs are often asymmetrical early in the course of Parkinson’s disease.

E-FACE
1- inspection
- titubation (tremor) of the head,
- absence of blinking,
- dribbling of saliva and
- lack of facial expression.

2- Test the glabellar tap (reflex): keeping your finger out of the patient’s line of vision, tap the
middle of the patient’s forehead (glabella) with your middle finger. This sign is positive when
the patient continues to blink as long as you tap. Normal people blink only a couple of times
and then stop. The glabellar reflex is a primitive reflex that is also frequently present in frontal
lobe disease.

3-Assess speech, which is typically monotonous, soft and faint, lacking intonation (dysphonia).
Sometimes palilalia is present; this is repetition of the end of a word (the opposite of stuttering).

4-Now test ocular movements, particularly for weakness of upward gaze. Isolated failure of
upward gaze is a feature of Parkinson’s disease.
There is a separate group of patients with marked rigidity and paralysis of gaze who should be
diagnosed as having progressive supranuclear palsy rather than Parkinson’s disease. These
people develop loss of downward gaze first, then loss of upward gaze and finally loss of
horizontal gaze.

5-Feel the brow for greasiness (seborrhoea) or sweatiness, due to associated autonomic
dysfunction. Orthostatic hypotension may also be present for the same reason.
6-The palmomental reflex is commonly present in these patients and tends to be more
prominent in those with severe akinesia. Dementia develops in 30% of patients.
F- WRITING
Ask the patient to write his or her name and address. Micrographia (small writing) is
characteristic. The patient may also be unable to do this because of the development of
dementia, a late manifestation. Test the higher centres if appropriate.

Other extrapyramidal movement disorders (dyskinesia)

G- CHOREA
Here there is a lesion of the corpus striatum, which causes non-repetitive, abrupt, involuntary
jerky movements. These may be unilateral or generalised. Often the patient attempts to
disguise this by completing the involuntary movements with a voluntary one.
In this disease, dopaminergic pathways dominate over cholinergic transmissions.

Chorea can usefully be distinguished from hemiballismus, athetosis and pseudoathetosis.


Hemiballismus is due to a subthalamic lesion on the side opposite the movement disorder. It
causes unilateral wild throwing movements of the proximal joints. There may be skin excoriation
due to limb trauma. These movements may persist during sleep.
Athetosis or dystonia is due to a lesion of the outer segment of the putamen and causes slow
sinuous writhing distal movements that are present at rest.
Pseudoathetosis is a description given to athetoid movements in the fingers in patients with
severe proprioceptive loss (these are especially prominent when the eyes are shut).

If the patient has chorea or if chorea is suspected, proceed as follows.


- First shake hands. There may be tremor and dystonia superimposed on lack of sustained
hand grip (‘milkmaid’s grip’).
Ask the patient to hold out the hands, then look for a choreic (dystonic) posture. This typically
involves finger and thumb hyperextension and wrist flexion.

-Go to the face and look at the eyes for exophthalmos (thyrotoxicosis), Kayser-Fleischer rings
(Wilson’s disease) and conjunctival injection (polycythaemia). Ask the patient to poke out the
tongue and note frequent retraction of the tongue (serpentine movements). Look for skin
rashes (e.g. systemic lupus erythematosus, vasculitis).
If the patient is a young girl, examine the heart for signs of rheumatic fever (Sydenham’se
chorea).

Test the reflexes. The abdominal reflexes are usually brisk, but tendon reflexes are reduced
and may be pendular (due to hypotonia).

Assess the higher centres for dementia (Huntington’sf chorea).

H-DYSTONIA
The patient manifests an involuntary abnormal posture with excessive co-contraction of
antagonist muscles. Dystonia may be focal (e.g. spasmodic torticollis), segmental or generalised.
Other forms of movement disorder may be present (e.g. myoclonic dystonia). The acute onset of
dystonia is seen most commonly as a side effect of various drugs (e.g. levodopa,
phenothiazines, metoclopramide).
A classification of nonphysiological
Handbook 33
Tremor in a 40-year-old man
You are working in a general practice. Your next patient is a 40-year-old man who is consulting
you because of a tremor ('the shakes').
YOUR TASKS ARE TO:
• Take a focused history about his condition.
• After completing the history, discuss possible diagnoses with the patient.
• The examiner will then question you about the physical findings you would check to clarify the
diagnosis, and any investigations you would arrange.
AIMS OF STATION
To assess the candidates skill in defining a presenting symptom of tremor, making a probability
diagnosis from the history and selecting with discrimination which aspects of physical
examination and investigations will clarify the diagnosis.
The examiner will have instructed the patient as follows:
You are a 40-year-old man who has consulted a general practitioner complaining of the shakes'.
The doctor will seek details of your symptoms and will ask questions about your health status,
medical and social history.
Opening statement
'I've got the shakes doctor. I spill my drink sometimes'.
Provide the following information without prompting:
‘I have had the shakes since my early 20s. It happens when I get nervous about something, but
it hasn't been a problem until recently (6-12 months). Sometimes my head shakes and often I
spill my drink when I put a glass to my mouth. I feel much better after a couple of beers.
Recently I heard something about Parkinson's disease which can cause the shakes, so I thought I
should see a doctor'.
In response to further questioning:
'It doesn't seem to bother me when I get up in the morning and doesn't stop me going to
sleep. It can go away for a few days then comes back. Seems to be when I am doing something
with my hands like using a knife and fork, or writing. The newspaper shakes when I am trying
to read it. Sometimes I have trouble lighting a cigarette. My right hand is the worst. I don't have
any stiffness and I don't have trouble moving from one position to another, nor in walking. I
can control the shakes by gripping things firmly. '
• You have noted that your hands and fingers shake if you hold your arms out in front of your
body. You may ask the doctor if your voice could be affected because you have noticed some
shakiness in your voice, but only occasionally and of minor degree.
• Shakes were first noted in your dominant right hand. Left hand is less affected and not until
recently.
• You have always been a tense and nervy person, stress makes the shakes worse. Youlike to
have a 'few beers' especially at the weekends but never get 'drunk'.
• Deny any other symptoms affecting the central nervous system, cardiovascular system,
respiratory, gastrointestinal or urinary systems, but admit to a diminished libido and difficulty in
maintaining an erection if asked directly or if the doctor provides an opportunity for you to do
so. No loss of weight.
• You work as a storeman. You are married with two teenage children. You smoke 10-15
cigarettes daily and drink up to five 375 mL cans of full strength beer on most days. Youhave had
no serious illnesses or operations.
• Family history: Father died from lung cancer aged 58 years. He also used to get the 'shakes'.
He was a non-drinker. Mother died of stroke aged 62 years. Her father had Parkinson disease.
Your brother, sister, wife and two children all keep in good health.
• You are anxious about the cause of your symptoms. You are ignorant of long term harmful
effects of alcohol. If the doctor facilitates your story and maintains an open-ended approach
whilst you are giving details of the shakes', continue to amplify your symptoms. If, however, the
doctor controls the interview too early, by only asking questions and not listening to your story,
just answer the questions asked. If asked, admit to concern about Parkinson disease because of
your grandfather. The same applies to your alcohol use: be reluctant to confirm or reveal the
true level of your alcohol intake. You have never been charged with exceeding .05, and have
never had an injury or motor accident associated with alcohol use.
EXPECTATIONS OF CANDIDATE PERFORMANCE
The candidate should assess the patient's tremor as follows:
History
By the process of listening (using an open-ended approach followed by direct questioning), to
develop the two most likely diagnostic pathways: benign tremor and tremor associated with
heavy drinking. Other differential diagnoses such as an anxiety state. Parkinsonism and
thyrotoxicosis are much less likely from the history. Past, family and social histories should be
sought.
Discussion with patient after the history
This may or may not be specific. It is included to assess the candidate's diagnostic approach to
the patient. Knowledgeable candidates will be reassuring because of their confidence in the
likelihood of a benign tremor.
Questions from the examiner after six minutes with expected responses:
What are your differential diagnoses?'
Benign or essential tremor (familial); effects of heavy drinking: Parkinson disease: cerebellar
disease; thyrotoxicosis
'At this stage what do you consider to be the most likely diagnosis?'
Benign tremor or alcoholic tremor
"What are the essential physical signs you would look for in this patient?'
Hepatomegaly and any stigmata of chronic liver disease, cerebellar signs, increased muscle
tone, tachycardia, cardiomegaly
'What investigations would you advise assuming the physical examination to be normal?'
~ Full Blood Examination (FBE), liver function tests, possibly thyroid function tests.
• Approach to patient — establishing trust and confidence by having a non-judgmental attitude,
listening to patient's concerns and being reassuring.
• History — comprehensive but focused, using appropriate communication skills Diagnosis must
include benign tremor and alcoholic tremor.
Physical examination should include checking for hepatomegaly and stigmata of
chronic liver disease, tachycardia and cerebellar signs.
Investigations — must advise liver function tests, but thyroid function tests not
essential.

CRITICAL ERRORS
Failure to indicate the most likely diagnosis is essential tremor.
Failure to advise liver function tests.

COMMENTARY
Essential tremor is one of the most common neurological disorders, with prevalence increasing
with age. An autosomal dominant family history is present in 50-60% of patients and the genetic
basis is unknown. Functional imaging reveals abnormal cerebellar activity and no histological or
structural changes have been identified. Age of onset is bimodal. with the largest peak in the
second decade, and a smaller peak in the fifth decade. The characteristic finding is a postural
and kinetic tremor of the upper limbs which interferes with fine manual tasks. Head tremor is
also present in 40%. Less commonly legs are involved or there is voice tremulousness. With
advancing age, the tremor frequency often slows and amplitude increases, leading to a coarse
tremor which can be disabling, although this is uncommon.
Patients with benign essential tremor often drink as a means of controlling the tremor as alcohol
has an ameliorating effect in 50% of cases.
In this case, the patient may also be suffering from the effects of prolonged heavy drinking. This
would require further assessment with the investigations recommended above.

Focus History
1-tremor
-onset and duration
-time (get up in morning, during sleep)
-getting better or worse
-intermittant or constant
-site (one or both, the worst)
-exacerbating factor- stress
-relieving- alcohol
-type- rest, action (reading, writing, drink water, lightening cigarette)
2-parkinson (moving from one position to another, walking, stiffness)
3-hyperthyroidism.
4-systematic (decreased libido, erection)
5-other common part of history

154-Anorexia induced amenorrhea


Amenorrhea for 12 months. BMI 17. Swimming 1.5 hour daily. On strict diet control.
Tasks
-History
-diagnosis and Ddx to pt.

History
1-5Ps (pregnancy/ post pill amenorrhea)
Periods
-can you tell me more about your concerns?
-when was you last menstrual period? (1 year ago)
-How were your periods before it stopped? (Before that, it gradually became irregular)
-Do you get spotting at the time of your normal periods?
-Any pain at the time of periods? (No)
Sexual history
-Are you sexually active? Are you in a stable relationship?
Pregnancy
-Were you planning for a pregnancy?
-Have you ever been pregnant? (Never been pregnant before)
Pill
-What contraception were you using? (Used OCP for long time but periods were normal while
popping the pills, currently using condoms, Taken pills until 2 years back)
pap/ HPV screening

2-DDX questions
PCOS
-Any excessive hair growth or acne?
POF
-Any hot flushes, heavy sweating, mood changes?
THYROID
-Any weather preferences. How are your bowel habits?
HYPERPROLACTIN
-Any headache, blurring of vision, milky discharge from the nipples?
Confidentiality+ eating disorder questions
-How is your diet? (Just veggies, no meat)
-do you think that you are overweight? How do you think about your appearance? (No)
-Do you try to lose weight by crash dieting or self-starvation, vomiting or laxatives? (No)
-any reason of strict diet control? (I want to stay healthy and no other perceptional issues at all)
Exercise induced
-do you exercise regularly and how many hours do you exercise? (No excessive exercise)
-What is your occupation?
Stress induced
-How is your home situation? Any stress at home? Any stress at work?

3-Psychosocial history and HEADS


mood questions:
-how is your mood
-have you ever thought of harming yourself or others
-how’s your sleep
-do you find it hard to concentrate on things.
Delusion and hallucination
-I am gonna ask you certain questions that could be a bit sensitive but theses are just routine
questions I ask to some of my patients.
-do you feel, hear or see things that others don’t?
-do you think someone is spying on you or trying to harm you?
HEADS
- have you lost interest in things you used to enjoy?
- SAD
- how is your social life? Do you catch up with friends quite often?

4-lifestyle questions
-medication
-Past history (medical and surgical)
-Family history
-childhood history (While exploring her child hood hx she said she used to be a fat and chubby
kid I asked her how does her feel about it and she said it didn’t upset me)

Explaining diagnosis and differentials


-from history and examination, there are several reasons why not having periods which we call
secondary amenorrhea.

-Draw the HPO axis.


The estrogen and the progesterone that is released by the ovary, under the influence of the
hormones secreted by the hypothalamus and the pituitary is responsible for a normal menstrual
cycle.

-Our hypothalamus is usually very sensitive to change in body environment and I’m concerned
about your body mass index which is quite low 17 so it could be diet induced amennorhea,
stress induced but u don’t have any stress.

-Although you are not fitting in the eating disorder criteria, but it could be eating disorder;
anorexia nervosa or beginning of this condition.

-Although you do not have concerning symptoms but you are not having periods and your child
hood obesity history and that is making me feel if we take the help from psychiatrist. Who is
going to conduct a formal interview and help form dietitian and nutritionist who will design your
work out schedule and diet plan in such a well-balanced way that it will not affect your body.

-It also could be


Pregnancy (not PT –ve)
Prolactinoma(tumor of pituitary, but no head ache BOV , and milky discharge although we will
run basic investigations).
PCOS(not obese, no facial hairs, no acne)
Hypothyroid(no lump In front of neck, no hot or cold intolerance, wt is quite low)
Stress: (no stressors, except for child hood obesity which also didn’t bother her)
Exercise induced(she didn’t give me anything red flags).
Ocp induced (but periods were normal while she was on ocps)
POF: (no hot flushes, no dysperonia, no dryness of vagina)

Feedback 1-6-2018
amenorrhea for 12 months. BMI 17. Swimming 1.5 hour daily. On strict diet control.
-History
-diagnosis.
-DD to pt.
( never been pregnant before. Taken pills until 2 years back. After than had period only once.
Pt doesnt like to limit swimming and she told she want to be healthy. She know she is slim but
want to maintain health by diet and exercise)
This was the most controversial case but thankfully I passed it , I was very fixed headed on eating
disorder from the very beginning so tried to concentrate on psychosocial rather then
amenorrhea
She told me that her last period was 1 year back and before that it gradually became irregular
no abd pain, she don’t think she is pregnant, used ocps: for long time but periods were normal
while popping the pills, currently using condoms, cervical screen / pap : normal, partner, stable
healthy no health issues no family stressors.
no stressors, no BOV, head ache, no thyroid related complaint, no acne hirusitism, no
gynecological sx, no previous or current pregnancy, no pid, no hot flushes or dryness of vagina.
then I moved on to psych hx stressors: no , eating habit: just veggies, no meat , no excessive
exercise, no purging, no laxatives, no body image issues, no frequent mirror check, no mood
issues, no see, hear or feel thingy, no family hx of eating disorder.
While exploring her child hood hx she said she used to be a fat and chubby kid I asked her how
does her feel about it and she said it didn’t upset me I explored any particular reason for strict
diet plan she said I want to stay healthy and no other perceptional issues at all. No hx of child
hood abuse or molestations.Explored HEADSSSS: all satisfactory

As it was DD station I told her there could be loads of reason for her not having periods wuch we
call secondary amenorrhea in our language, then drew pituitary ovarian axis on paper and
started enumerating the reasons.
Told her our hypothalamus secretes some hormaones which act on pituitary and inturn pituitary
secreats the hormones which act on the ovaries and that’s how the female experience regular
periods on monthyly basis.

Our hypothalamus is usually very sensitive to change in body environment and I m concerned
about your body mass index which is quite low:17 so it could be diet induced amennorhea,
stress induced (but u don’t have any). I m not a psychiatrist and although u r not fitting in the
eating disorder criteria. but it could be eating disorder; (anorexia nervosa or beginning of this
condition.

although u don’t have concerning symptoms but ur not having periods and ur child hood obesity
hx is making me feel if we take the help from psychiatrist who is going to conduct a formal
interview and help form dieatetion and nutritionist who will design your work out schedual and
dieat plan in such a well balanced way that it wont affect your body.
Then went on to tell other DD’d
Told her I was thinking of
Pregnancy (not PT –ve)
Prolactinoma(tumor of pituitary, but no head ache bov , and milky discharge although we will
run basic investigations).
PCOS(not obese, no facial hairs, no acne)
Hypothyroid(no lump In front of neck, no hot or cold intolerance, wt is quite low)
Stress: (no stressors, except for child hood obesity which also didn’t bother her)
Exercise induced(she didn’t give me any thing red flaggish)
Diet (most likely).
Anorexia nervosa (querry , not sure but 1st DD)
Ocp induced( but periods were normal while she was on ocps)
POF: (no hot flushes, no dysperonia, no dryness of vagina)
No gyneacologic sx on ovaries or uterus
PID (no hx)

Feed back : passed Global score : 6 All key steps cpvered


Approach to pt: 5 Hx: 6 Dx, DDX: 6

Feedback 1-6-2018
Amenorrhea for 1 year. Task: Hx, explain cause of amenorrhea Anorexia nervosa.
DDx Mnemonics 5PTEAS (Pregnancy, Pills, POF, Prolactinoma, PCOS, Thyroid, Exercise and diet,
Asherman, Stress) which I gave as DDx and told why others are unlikely.
BMI 16 I think. Excessive exercise. Diet restrictions. Both started 1 year ago, no specific stressor
in hx.
Patient wasn’t concerned about the condition. So I explained briefly what it is and how it can
adversely affect her health. Explained HPO axis. Other possible causes and they are unlikely.
Mentioned possible adverse effects of anorexia

155-Anniversary grief
Case 1 AMC
around 55 years old woman whose husband died few mnths back. now his birthday is around so
she is feeling gloomy and tearful because they had plans for that birthday, booked tickets etc.
Previously she was all fine and was coping well after the death. These changes are just recent.
Kids are in same state but living independently. make herself busy in book club and volunteered
herself in look school canteen.
Tasks:
-explain dx and ddx to patient.
-Further mx

Case 2 AMC
GP, a gentleman comes for regular checkup of his HTN, controlled and compliant. Suddenly
tearful, when u asked questions he said his wife died? Months ago? MI, next month will be her
birthday. They also planned to go somewhere. They did all activities together, involved in
charities. Psychiatric assessment no features no depression, mood normal, sleep, appetite good,
and no suicidal, homicidal ideas.
Tasks
-explain dx and ddx to patient.
-counsel the patient and management.

Sample Case
Gp, next patient is 57-year-old Maria who has come for regular health check she has
hypertension since past 3 years and on ACEI. Today the nurse recorded the BP and it is 130/82.
Told that her BP is normal but suddenly become tearful and start crying. She has no history of
previous illnesses apart from high BP. You also not noticed any emotional disturbances in her so
far.
Tasks
-explore her current condition
-discuss likely diagnosis
-counsel management.

History (just for sample case)


1-Approach
-give tissue and water
-confidentiality
-can you tell me why you are feeling like so at the moment? (my husband has passed away 1
year ago same time)
-so sorry to hear about your loss. It is very heard time for you. Are you happy for me to carry the
consultation with you today.
2-Differential diagnosis questions
-since how long have you been feeling like this? (2 days when I was cleaning the wardrobe) (to
rule out depression)
-do you think you have coped well after your husband passed away? (yes) were carrying on your
activities after that time? (yes) (to rule out abnormal grief)

3-HEADS
-any other recent stresses that you have? (No)
anybody else to share your feeling with?
anybody else at home who is living with you? (lives alone but daughter nearby living)
do you visit your daughter often?
do you have good support?
-are you working? (No)
any financial problem?
-anhedonia
have you lost interest in things you used to enjoy? For how long? (yes for 2 days)
-SAD (-ve)
-social activity (not good to go out with friends now)

4-psych history
mood
-how is your mood? For how long? (low for 2 days)
-have you ever thought to harm yourself or others? (No)
-how is your appetite? (good but not feel like eating)
-have you lost weight recently?
-how is your sleep? Any nightmares or vivid dreams? (disturbed)
-any difficulty concentration?
-do you feel guilty?
Delusion and hallucination
-do you see, hear, feel things that others do not? (feel him sleeping beside her but on waking up
no)
-do you feel someone is following you, trying to harm you or spying on you? (No)
Insight
-do you think you need a professional help? (no)
Judgement
-let us suppose there is a fire in this room what would you do?

5-general questions
-past medical history
-menopausal symptoms
-thyroid weather preference and bowels habit
-compliance with BP medication
For AMC exam cases
1-Approach
-give tissue and water
-so sorry to hear about your loss. It is very heard time for you. Are you happy for me to carry the
consultation with you today.
-confidentiality (let me assure you that everything we gonna discuss today will be private and
confidential.

2-Explain diagnosis and differential diagnosis


-what I am suspecting is that you are experiencing an anniversary grief reaction which is a feeling
of intense sadness which occurs at the time of the anniversary of a loved one passed away. In
your case his birthday.

-it is a normal expected human psychological behaviour. Feeling of sadness or helplessness are
quite normal at this time and these feelings will go off once you get over the anniversary grief
period. Grief is just like a bruise that heals with time.

-other possibilities are:


*could be depression but less likely as your symptoms are just for short time. And no other
suggestive criteria.
*could be adjustment disorder but also less likely in your case.
*could be hypothyroid disorder or menopause but less likely as no weather preference or
symptoms of menopause.
* complex grief but less likely as you have coped well after your husband passed away.

3- Counselling

-I acknowledge your grief and allow yourself to get upset, do not bottle up your emotions, share
your thought and feeling and your future with your family or any of your good friends. With your
consent, I can even arrange for a meeting with any of your family members so that they can
support you during this time. (you can ask if she has any support before talking about this)
-support group also available where people with similar experience come together and share
their feelings.
-during this time, I can refer you to psychologist for a counselling session.
-you do not need any medications at the moment. Just look after your health with healthy diet
and regular exercises, relaxation techniques like yoga or meditation. And follow sleep hygiene
practices.
-warnings signs (just in case your feelings become more intense, any idea of self-harm please
report.
-I will review you in 2 weeks time and see how you go.
-Reading materials
-come back if you need any further help.
Feedback 19-7-2018
TEARFULNESS(ALL 3KEYSTEPS Yes, Score 4,4,4,5)
55 year old female whose husband died a couple of months ago feel tearful as his birthday is
coming soon and they had plans for that, living alone, kids are interstate , make herself busy in
book club and volunteered herself in look school canteen. Whole history was outside.
Tasks
-Give Dx and DDx.
-further management.
When I entered a lady was sitting and crying badly, took 2 mins to console her and after that she
stops crying, I told her most likely what she is having is Anniversary grief which is an expected
and understandable response on a loss of someone we love. but it could be adjustment disorder
,depression, hypothyroid, post menopausal.
Give MX as written in KARREN , 4Rs.

Feedback 19-7-2018
around 55 yrs old lady whose husband died few mnths back. now his birthday is around so she is
feeling gloomy and tearful bcz they had plans for that birthday, booked tickets etc. Previously
she was all fine and was coping well after the death. These changes are just recent. Kids are in
same state but living independently.
Tasks:
-explain dx and ddx to pt
-Further mx
When i entered the room, the lady was crying and had some tissues in her hand . I offered glass
of water n more tissues. Asked her how she feels at the moment.. does she wants to continue or
wants to arrange another consultation (she said she feels better so continue) .
Then i explained her that i understand that she is feeling gloomy n tearful these days and there
could be few reasons for that but my most preferred diagnosis is anniversary grief reaction bcz of
( i explained all the anniversary grief reaction) .

Then i said there could be some other reasons for having these symptoms which are unlikely but
these are hypothyroid, menopause, adjustment disorder etc .explained each of them
separately. Then management as in previous notes /karen etc ..

Feedback 4-4-2018
Station 7- tearfullness- FAIL
Partner died few months ago, now has sad mood for a month. This month partner had birthday,
on birthday they had lot of plans. He goes out with friends. Visits his children often.
Task. Explain diag and dds with reasons
Further mx plan
Inside room, male was teary and faking alot.
Feedback 31-5-2018
GP, a gentleman comes for regular checkup of his HTN,controlled and compliant. suddenly
tearful, when u asked questions he said his wife died ? Months ago? MI, next month will be her
birthday. They also planned to go somewhere. They did all activities together, involved in
charities. Psychiatric assessment no features no depression, mood normal, sleep, appetite good,
and no suicidal, homicidal ideas.
Tasks
-explain dx
-counsel the patient and management.

[No need to ask questions]

Patient was sitting and crying inside.greet and give tissues to him at same time.pull the chair a
bit close to him. said sorry for his wife.is it ok for you to explain about the condition? He said yes.
Is this place comfortable to have conversations. he said yes.(he already stopped crying since I
gave tissues).

I said it might be difficult for you because of lost of your better half. Explained anniversary grief.it
is normal reaction. but it is important to take care of your health. also important to adjust to it
by taking things slowly. share your feelings with yr children and closed friends, spend time with
them.in the future, to prevent this sudden overwhelming sadness, celebrate intentionally with
your children and friends by praising her good things when anniversary is near.
i am also thinking about acute stress disorder, adjustment d/o because of changes in your life
and yr environment, I also noticed that u and yr wife enjoyed and participated in all activities
together.
i am also thinking about mild depression and anxiety but unlikely.
i will also refer u to psychologist to have good talk therapy,also share yr feeling with specialist,life
style modification is also important,healthy diet, reduce alcohol and stop smoking, adequate rest
and regular exercise, reduce anxiety and stress. follow up frequently, if you have any ideas to
harm yrslef or see or hearing strange things, don’t make decision quickly. come and talk to me at
any time.

Keys 3/3 approach,dx and ddx,counseling and management 3,4,4,5

Global 4
Feedback 25-10-2018
Scenario : tearfulness
Stem : old man, ur old patient whose wife passed away few months back. Now wife’s bday in next week or
month maybe. All of the hx was given which didn’t show any depression. And he works at some café for
special kids (something like voluntary work)
Tasks:
Explain probable dx and ddx with reasons
Future mx plan

When I entered the man was crying heavily, I calmed him down and offered water but not tissues ( he was
holding one in his hand) I told him you are doing a great job by helping kids and blah blah. Asked few qs
like any kids friends he was just nodding not saying any words.
Then I told him I can see its very unfortunate for you to go through this situation as spouse is a person very
close to heart and life but im going to tell you that this isnot something uncommon. And your tears and
feelings are natural donot think you r going crazy, if it was me in your place I would feel exact the same as
you are esp when its her bday coming soon and you must hav planned something great for her. I gave my
ddx that it could be depression, ptsd, adjustement, but its something we call anniversary grief anf it’s a
part of normal grief so don’t worry.
I would like to call your kids and arrange family meeting with them. Would like to arrange frequent
consultations with myself unless you feel good in urself. Can put to in contact with social and spiritual
worker if u have any financial issues? He nodded in no. so I said if anytime you have that let me know
theres so much help available for you and you are not at alone. Will give you numbers of some support
groups and reading material. Bell rang

GS: 5
3/3 key steps
Approach :6
Dx/ddx :4
Pt counselling: 4
Mx plan:6

Feedback 25-10-2018 pass


65 yr old woman, well known to you comes with concerns about being tearful n sad. Husband
passed away 6mo/or 1yr ago. She is concerned about her being in tears easily as previously she
was a calm composed person. Birthday is nearby,had plans for holiday. Has grandchildren, visits
them every week. Lives alone
Task:
Take history
Counsel about her condition and give reasons
I did this as “ anniversary grief reaction” She was crying, gave her tissues she accepted.
Then began empathic questioning, her health, menopause, thyroid, organic illness.sadma.
Forgot to ask previous mental illness.
Counselled about possible cause being anniversya grief, or postmenopause, or depression, or
thyroid problem. Therefore some investigations are needed. Gave her ideas about recreation,
family meetings, talk therapy, diet, exercise.

GS-4 Counselling: 3 ☹ I thought I did well, as she smiled in the end.


156-Thyroid examination
Case 1 (Hyperthyroidism)
Your next patient in GP practice is a 27-year-old lady with restlessness and weight loss. The
patient is on propranolol 20mg for prevention of migraine headache. The migraine is well
controlled.
-Perform relevant physical examination
-explain your findings to the patient.
-tell the possible differential diagnoses.

Examination steps (talley o connor)


1-WIPE

2-General appearance (in exam the patient will be thin and restless)
-anxious -restless -thin -weight.

3-Hands
look and feel together
-nails (clubbing, Onycholysis)
*in the exam there will be a white polish to mislead you about leukonychia so say just polish and
cannot elicit nail changes
*do clubbing test by put both hand’s index finger upside down against each other and see if
there is a space or no.
-palmar erythema
-warm, sweaty (will be positive)
-Fine tremor
*ask patient: if you could bring your arms in front of you like this and splay your fingers. I am just
gonna lay a piece of paper on your hands. (this will be positive)
-pulse (rate and rhythm)

4-Face (in exam the patient may pretend to make eye signs but you need to decide if it looks real
then say it if not say could be)
look
-thyroid stare (it is like patient staring on you with all eyes open)
-ptosis (drooping of eyelids)
-proptosis (protruded eye).
-Exophthalmos (I think if the lower sclera was obvious and not covered by the lower eyelid)
-lid retraction (I think when the upper sclera was obvious and not covered by upper eyelid)
-conjunctivitis
-corneal ulceration
Move (all –ve in the exam)
-lid lag
*keep your head still and follow my finger.
-eyes movement (opthalmoplagia) (like H shape movement) ask patient if blurring of vision
present.
5-Neck (all negative in the exam but sometime they give real patient)
look
-scars
-dilated veins
-skin redness
-swelling
-ask patient to swallow water
*if you could just take a sip of this water and hold it in your mouth…and swallow.
-ask patient to protrude her tongue
*if you could just stick your tongue out.
Feel (from behind)
-flex the neck slightly and start palpating the mass
*can you flex neck slightly forward and relax.
site, size, surface, regularity, consistency, tenderness, temperature, mobility
-ask patient to swallow water and repeat palpation
-palpate the inferior border of the mass.
-palpate Lymph nodes
-tracheal position (from front)
*now I am just gonna feel the windpipe it might be a little bit uncomfortable.
Percussion (tap on the upper part of sternum 2 sites with your fingers horizontally over the
sternum)
*I am just gonna tap on your chest. (retrosternal goiter).
Auscultation
-over each thyroid lobes (bruit)
-stridor (apply mild compression on the lateral lobe) no need if there was no swelling.
Special pemberton signs for retrosternal goiter.
*can you raise your hands above your head like this.

6-Chest
look: gynecomastia
Feel: apex beat
listen: heart sounds and murmurs

7-others
-reflexes (biceps) (hyperreflexia +ve) (may be a critical error if we did not examine the reflex)
-pretibial myxedema
-proximal myopathy (leg): *can you cross your arms and stand up for me.

Diagnosis and differentia diagnosis


1-hyperthyroidism (most likely): thyroid gland is over active with increase hormone secretion
causing your symptoms.
2-graves disease (autoimmune disease) (young woman with eye signs and thyroid bruit)
3-thyroiditis (infection or inflammation) less likely no tenderness, warmth or fever.
4-anxiety
5-medications less likely as she does not take any such medications.
6-nasty growth or cancer less likely from examination.
Feedback 21-6-2018 Nervousness: Pass:/Global score:4
Your next patient in GP practice is a 27-year-old lady with restlessness and weigh loss. The
patient is on propranolol 20mg for prevention of migraine headache. The migraine is well
controlled. Perform relevant physical examination, explain your findings to the patient and tell
the possible differential diagnoses.
When I entered the room started to introduce myself, in the middle of my greeting, the role
player asked: Aren’t you going to examine me?” and I said of course, if you don’t mind.
Then I washed my hands and explained about the examination, and asked for consent, then
started with general appearance:
In general appearance, she is a young, thin and lean lady, she looks fidgety and restless,
however she does not look cachectic or ill. On inspection of the eyes, I addressed that I cannot
notice a real lid retraction or exophthalmos, in conjunctiva inspection, there was no pallor, as
well no other signs of anaemia such as angular chelosis, or glossitis in the mouth. Then
proceeded to the examination of the movements of the eye, which were normal and no lid lag
was present. On inspection of the neck, there was no swelling. I explained to the patient that I
need to stand behind her and palpate her neck for the examination of the thyroid. I could not
feel the thyroid gland, as it was hardly palpable, and there was no lymphadenopathy in the
neck. Also no bruit in the auscultation of the thyroid gland. During the examination I asked the
patient to swallow her saliva twice and palpated each side of the gland.
Then proceeded to the auscultation of the heart. I listened to four spots on the chest, and
mentioned that I can hear S1 and S2 clearly, without any added sound. Then the examiner said
heart is normal. Then I asked the role player to get up from the chair without using her hands or
arms, and sitting again, there was no signs of proximal myopathy.
I checked her legs and feet for pretibial myxedema. then proceeded to hands, checked her
pulse, I did not count the pulse, but mentioned that pulse rate is regular and its count looks
like being around 70 to 75, however because the patient is taking propranolol, I do not expect
to find tachycardia. Then checked her tremor, (I had almost forgotten to check the tremor, the
role player Guided me herself, by keeping her hands outstretched, there was a fine to slightly
coarse tremor, in the hands, the hands were sweaty, but no palmar erythema, and no swelling.

Her nails were done with white polish, ( I do not know whether I was supposed to take that
artificial nails as clubbing or not, however, I asked her to put her finger nails against each other,
and mentioned that this was not a real clubbing, and to me it looks like a normal hand, without
signs of thyroid acropachy.
I also asked the examiner to give me a blood pressure cuff to measure BP, he said, it was 125/
75 , also asked for medical hammer for checking the reflexes, I checked biceps reflexes on both
sides which were obviously increased.
In my differential and explanation, I explained that she does not show the full picture of
hyperthyroidism, however, some of these signs appear late during the disease course, and also
as she is taking propranolol I do not expect to find anaemia or heart problems in her, then I
added that by approaching nervousness or tiredness I can give several different diagnoses, but
by considering all her symptoms together I cannot really think of any diagnosis other than
Hyperthyroidism, which is caused by more than normal secretion of the thyroid hormones, so
she needs to have a blood test so that we can exclude thyroid problems. Then the bell rang and
I exited the room.

Feedback 4-4-2018
Station 1- nervousness- FAIL
22 yr lady with 1 month of nervousness, tired, and weight loss of 3 kg.
Task
- do relevant examination
-Tell most likely diagnosis and dds with reasons
On exam- thin lean girl with brittle hair, Muscle wasting, palmar erythema, and no sweating,
proximal muscle weakness, White nail polish on nails.
There was a cat scratch mark on neck, and 1 on arm.
No lymphadenopathy present. She was pretending to be exophthalmos, and lid lag.
Thyroid exam was normal.
Examiner didnt tell anything
I couldnt comlete full thyroid exam.
Told all DDs related to thyroid and others which cause nervousnes (dds score- 5)

Feedback 23-6-2018
Feedback: Neck examination: pt nervous, tired, loss of weight. TFT suggest pt has
hyperthyroidism
I did: thyroid examination: hand, face, neck, reflexes, proximal muscular,…
I thought this case we all know too well.
Grade: PASS, GS:5

Feedback 7-9-2018
PE Hyperthyroid examination
In the stem - Hyperthyroidsim examination
FT 4 - 17 slightly high TSH 0.3
Task
1. -Examine the patient and –
2. Explain ddx to the patient with reasons .
(fine hand tremor with paper test only , others – all normal )
Greet the examiner and greet the patient, washed hand and wipe out with paper towel
attached to the wall …
Asked any pain at this moment . And explain the procedure very briefly within 5 – 10 secs
GA – looking and shouting by myself , to be heard enough to the examiner ( running
commentaries myself ) – nil anxious looking , nil sweating in the A/C room
Started with hands – nil hand sweating, nil palmar erythema , nil Onycholysis.
When examine her with a paper test on outstretched hands – patient has fine hand tremor – I
claimed that fine hand tremor is present .
PR – nil tachycardia – checked collapsing pulse – examiner asked how do you check collapsing
pulse – Explain that Pressing hard enough to occlude the RA , then raised the right hand above
heart level and. In my patient , there is no rebound pulsation ; hence nil collapsing pulse.
Face- nil sweating , nil proptosis from front , then , I stand up and checked from side of eyeballs
and from above – explained to the examiner that there is nil proptosis , nil conjunctivitis
Examined – lid-lag from front , H-Test for OCM tests – asking the patient whether double vision ,
eye pain on H-Test or not – she said – NO –

Neck –
Inspection –
Nil scar, nil dilated vein , nil masses are seen from front ( didn’t do swallow water and
protusion of tongue in Inspection )
Palpation ( from behind of the patient’ neck ).
- quickly told the patient – if there is a pain , please do let me know ( name of patient )
Started checking with lymphadenopathy – running commentaries of – submentle ,
submandibular , anterior cervical , supraclavicular , posterior cervical , pre and post auricular ,
and occipital areas -
- claimed that there is no lymphadenopathy
- Checking thyroid swollen – nil glands are palpable .
So , while palpating the isthmus of thyroid gland, ask patient swallow water , protrude the
tongue – nil lump is palpable .
Going to front - checking retrosternal extension by percussion on bilateral sternal bone to
upward direction – no retrosternal extension
Auscultation – told the patient the instruction , breath in and breath out then hold breath for
few seconds. – check thyroid bruit bilaterally – nil
Check – Lahey test – told the patient if there is a pain , please let me know – nil posterior lobe of
thyroid was palpable
Told the patient about pemberton’s signs – examiner told me “ do you think it is necessary
without gland enlargement “ , then I smiled to him . So , I will skip this test.
check – proximal myopathy for lower limb ( cross-arm and stand up ) – normal
Jerk – tried to do jerk to archilles tendon- examiner said that any tendon u can check for reflexes
– then smiled to him and told him “ thanks for your instruction “ and then , I changed to right.
bicep tendon jerk – examiner stand up and come to check the technique – then he said –
NORMAL jerk reflexes.
- then I told the examiner ( without really checking the patient ) – Would like to complete my
examination by checking - pretibial myxedema or not – he said – NO
Task -2
- provide DDx –
Condition – high thyroid hormones –
Possible causes –
1. Hyperthyroid condition what we called Grave’s disease
2. Inflammation of thyroid gland – what we called thyroiditis
3. Own immune system attacking to your thyroid gland – what we called autoimmune thyroiditis

4. sometimes drug can cause thyroid disorder such as drug –induced thyroiditis
5– need to exclude nasty condition such as thyroid cancer
6. last but not least – other possible masses – such as fat tissue called lipoma or other soft tissue
swelling.
Feedback - Pass Global score – 4
Key step 1 – yes Key step 2 – yes Key step 3 – no
Approach to patient – 4 Technique of examination – 5 Accuracy of examination – 5
DDx – 4

Feedback 18-9-2018 Thyroid examination


The next pt in your GP clinic is a 26 year old girl Who has come to you due to
restlessness and Nervousness for couple of months. She complains that something
is wrong with her nerve and she is always anxious about small things. You are seeing
this pt for the first time.
Your task is to
1. Perform required physical examination with running commentary regarding the
cause of her nervousness.( You should spend no more than 7 mins in this task)
2. Discuss the Dx with the pt.
My Approach:
I went in and there was a girl sitting on a chair with her glasses on.She was moving all the time, and
very restless. She was keeping her eyes wide open , she was trying to give me exopthalmus.
She was a great actor though. Anyway so I commented on everything I saw. Then I started with
hands. Pulse was around 90, She had fine tremor , I used a paper just like gicky medics, no Flapping
tremor. She had palmer erythema and her hands were sweaty.
Then I went to eye and asked her to take off her glasses. I did lid lag test and it was positive, I forgot
to do eye movements. Then I did neck examination. I asked her to swallow the sip of water but I
forgot to ask stick your tongue out. Then I felt the thyroid gland and did purcussion and auscultation.
There was thyroid Bruit( I heard some abnormal sound I am not sure but told that its
a bruit). Then I did reflexes and all of them were increased, She moved her hands very strongly when
I tapped with the hammer. She didn't have any pretibial myxoedema but there was proximal
myopathy coz she couldn't stand up with crossed hands. My examination organisation was not
good and I forgot to do CVS and Pamberton sign.
Then I explained with a diagram about the thyroid gland and told her that this glads secrets some
chemical what we call hormon. In your case its secreting more hormone and thats why you are
having this symptoms. Then I explained Hyperthyroidism in easy terms.
AMC Feedback:
Station 07: Nervousness
Grade: Pass Global score: 4
Key steps: 1,2,3,4,5 no,yes,yes,yes,yes
Assessment domain:
Approach to pt/relative: 4
Choice & technique of examination, organisation and sequence: 4
Accuracy of examination: 4
Dx/DDx: 4
Feedback abdomen examination
Case 10/2/2017
-Thyroid exam PE Hyperthyroidism PE. Male with lethargy. Blood test done shows low TSH, T4
raised. Task: perform PE and DDs with reason.Normal young pt sitting on the chair . Tools are on
the table (hammer , stethoscope ),General apearane of the pt .not restless , properly dressed for
the weather , Start with hands no tremor , the nail , palmer erethema ,pulse , no watch , he said
what do u want to check ? rate rhythm volume pattern , then do muscle tone , reflexes ,
shoulder m power (chicken wings ), Then face , opthalmopathy ? eye movements , Neck inspect
with swallowing any mass any dilated veins, protrude tongue out .Palpate with swallowing RT,LT
and middle isthmus ,Percuss for retrosternal extension . Auscultate for bruit in the neck ,Check
lower limb reflexes and oedema ,Then explain the condition 5C ( layman language I advise from
myoclonic)THANK GOD I passed this case ( was easy no interruption from both the examiner or
roleplayer )

- Thyroid exam ,girl with lethargy ,TFT was given


*Do PE*State dignosis and further mx
Did PE,findings weren’t not clear and very confusing.
I used to had the impression that when u will do examination the examiner will
gv u finding,which is not right,at least not with me.In no PEs,none of the
examiner esp this one uttered a single word.So I said Hyperthyroid based o TFT even though the
c/C was lethargy and feeling sleepy :/ Passed

THE THYROID GLAND


Examination anatomy
Even when it is not enlarged, the thyroida is the largest endocrine gland. Enlargement is
common, occurring in 10% of women and 2% of men and more commonly in iodine-deficient
parts of the world. The normal gland lies anterior to the larynx and trachea and below the
laryngeal prominence of the thyroid cartilage. It consists of a narrow isthmus in the middle line
(anterior to the second to fourth tracheal rings and 1.5 centimetres in size) and two larger
lateral lobes each about 4 centimetres long. Although the position of the larynx varies, the
thyroid gland is almost always about 4 centimetres below the larynx.

Inspection
The normal thyroid may be just visible below the cricoid cartilage in a thin young person. 1,2
Usually only the isthmus is visible as a diffuse central swelling. Apparent enlargement
(pseudogoitre) can occur as a result of the presence of a fat pad in the anterior and lateral part
of the neck. This finding is more common in people who are overweight but can occur in those
of normal weight. Enlargement of the gland, called a goitre, should be apparent on inspection
especially if the patient extends the neck.
Look at the front and sides of the neck and decide whether there is localised or general
swelling of the gland. In healthy people the line between the cricoid cartilage and the
suprasternal notch should be straight. An outward bulge suggests the presence of a goiter.
Remember that 80% of people with a goitre are biochemically euthyroid, 10% are hypothyroid
and 10% are hyperthyroid.
The temptation to begin touching a swelling as soon as it has been detected should be resisted
until a glass of water has been procured. The patient takes sips from this repeatedly so that
swallowing is possible without discomfort. Ask the patient to swallow, and watch the neck
swelling carefully. Only a goitre or a thyroglossal cyst, because of attachment to the larynx, will
rise during swallowing. The thyroid and trachea rise about 2 centimetres as the patient
swallows; they pause for half a second and then descend. Some non-thyroid masses may rise
slightly during swallowing but move up less than the trachea and fall again without pausing.
A thyroid gland fixed by neoplastic infiltration may not rise on swallowing, but this is rare.
Swallowing also allows the shape of the gland to be seen better.
Note whether an inferior border is visible as the gland rises.
The thyroglossal cyst is a midline mass that can present at any age. It is an embryological
remnant of the thyroglossal duct. Characteristically it rises when the patient protrudes the
tongue.
Inspect the skin of the neck for scars. A thyroidectomy scar forms a ring around the base of the
neck in the position of a high necklace. Look for prominent veins. Dilated veins over the upper
part of the chest wall, often accompanied by filling of the external jugular vein, suggest
retrosternal extension of the goitre (thoracic inlet obstruction). Rarely, redness of the skin over
the gland occurs in cases of suppurativa thyroiditis.
Palpation
Palpation is best begun from behind but warn the patient first. Place both hands with the pulps
of the fingers over the gland. The patient’s neck should be slightly flexed so as to relax the
sternocleidomastoid muscles. Feel systematically both lobes of the gland and its isthmus. Feel
one side at a time; use one hand to steady the gland and the other to palpate. Consider the
following:
● Size: only an approximate estimation is possible Feel particularly carefully for a lower border,
because its absence suggests retrosternal extension.
● Shape: note whether the gland is uniformly enlarged or irregular and whether the isthmus is
affected. If a nodule that feels distinct from the remaining thyroid tissue is palpable, determine
its location, size, consistency, tenderness and mobility. Also decide whether the whole gland
feels nodular (multinodular goitre).
● Consistency: this may vary in different parts of the gland. Soft (but firmer than a fat pad) is
normal; the gland is often firm in simple goiter and typically rubbery hard in Hashimoto’s
thyroiditis. A stony, hard node suggests carcinoma, calcification in a cyst, fibrosis or Riedel’s
thyroiditis.
● Tenderness: this may be a feature of thyroiditis (subacute or rarely suppurative), or less often
of a bleed into a cyst or carcinoma.
● Mobility: carcinoma may tether the gland.
● A thrill: this may be palpable over the gland, as occurs when the gland is unusually
metabolically active (e.g. in thyrotoxicosis).

Repeat the assessment while the patient swallows.


Palpate the cervical lymph nodes. These may be involved in carcinoma of the thyroid.
Move to the front. Palpate again. Localised swellings may be more easily defined here.
Note the position of the trachea, which may be displaced by a retro sternal gland.
Causes of neck swellings
Midline
Goitre (moves up on swallowing)
Thyroglossal cyst (moves on poking out the tongue with the jaw stationary)
Submental lymph nodes
Lateral
Lymph nodes
Salivary glands (e.g. stone, tumour) • Submandibular gland • Parotid gland (lower pole)
Skin: sebaceous cyst or lipoma.
Lymphatics: cystic hygroma (translucent)
Carotid artery: aneurysm or rarely tumour (pulsatile)
Pharynx: pharyngeal pouch, or brachial arch remnant (brachial cyst).
Parathyroid gland (very rare).
Differential diagnosis of thyroid
Nodules
1. Carcinoma (5% of palpable nodules)—fixed to surrounding tissues, palpable lymph nodes,
vocal cord paralysis, hard, larger than 4 centimetres (most are, however, smaller than this)
2. Adenoma—mobile, no local associated features
3. Big nodule in a multinodular goitre—palpable multinodular goiter
Goitre
Causes of a diffuse goitre (patient often euthyroid)
Idiopathic (majority)
Puberty or pregnancy
Thyroiditis • Hashimoto’s • Subacute (gland usually tender)
Simple goitre (iodine deficiency)
Goitrogens—iodine excess, drugs (e.g. lithium)
Inborn errors of thyroid hormone synthesis (e.g.
Pendred’s* syndrome—an autosomal-recessive condition associated with nerve deafness)

Causes of a solitary thyroid nodule


Benign:
• Dominant nodule in a multinodular goiter
• Degeneration or haemorrhage into a colloid cyst or nodule
• Follicular adenoma
• Simple cyst (rare)
Malignant:
• Carcinoma—primary or secondary (rare)
• Lymphoma (rare)

Percussion
The upper part of the manubrium can be percussed from one side to the other. A change from
resonant to dull indicates a possible retrosternal goitre, but this is not a very reliable sign.
Auscultation
Listen over each lobe for a bruit (a swishing sound coinciding with systole). This is a sign of
increased blood supply, which may occur in hyperthyroidism, or occasionally from the use of
anti-thyroid drugs.
The differential diagnosis also includes a carotid bruit (louder over the carotid itself) or a
venous hum (obliterated by gentle pressure over the base of the neck). If there is a goitre,
apply mild compression to the lateral lobes and listen again for stridor.

Pemberton’s sign
Ask the patient to lift both arms as high as possible. Wait a few moments, then search the face
eagerly for signs of congestion (plethora) and cyanosis. Associated respiratory distress and
inspiratory stridor may occur. Look at the neck veins for distension (venous congestion). Ask the
patient to take a deep breath in through the mouth and listen for stridor. This is a test for
thoracic inlet obstruction due to a retrosternal goitre or any retrosternal mass.3 (Lifting the arms
up pulls the thoracic inlet upwards so that the goitre occupies more of this inflexible bony
opening.) Examination of the thyroid should be part of every routine physical examination.

HYPERTHYROIDISM (THYROTOXICOSIS)
Hyperthyroidism is a disease caused by excessive con centrations of thyroid hormones. The
cause is usually over production by the gland but it may sometimes be due to accidental or
deliberate use of thyroid hormone (thyroxine) tablets—thyrotoxicosis factitia. Thyroxine is
sometimes taken by patients as a way of losing weight. The cause may be apparent in these
cases if a careful history is taken. The antiarrhythmic drug amiodarone, which contains large
quantities of iodine, can cause thyrotoxicosis in up to 12% of patients in low iodine-intake areas.
Many of the clinical features of thyrotoxicosis are characterised by signs of sympathetic nervous
system overactivity such as tremor, tachycardia and sweating. The explanation is not entirely
clear. Catecholamine secretion is usually normal in hyperthyroidism; however, thyroid hormone
potentiates the effects of catecholamines, possibly by increasing the number of adrenergic
receptors in the tissues. The most common cause of thyrotoxicosis in young people is Graves’c
disease, an autoimmune disease where circulating immunoglobulins stimulate thyroid-
stimulating hormone (TSH) receptors on the surface of the thyroid follicular cells
Examine a suspected case of thyrotoxicosis as follows.
General inspection
Look for signs of weight loss, anxiety and the frightened facies of thyro toxicosis.
Hands
Ask the patient to put out his or her arms and look for a fine tremor (due to sympathetic
overactivity). Laying a sheet of paper over the patient’s fingers may more clearly demonstrate
this tremor, to the amazement of less-experienced colleagues.
Look at the nails for onycholysis Plummer’sd nails. Onycholysis (where there is separation of the
nail from its bed) is said to occur particularly on the ring finger, but can occur on all the
fingernails and is apparently due to sympathetic overactivity. Inspect for thyroid acropathy
(acropathy is another term for clubbing), seen rarely in Graves’ disease but not with other
causes of thyrotoxicosis.
Inspect for palmar erythema and feel the palms for warmth and sweatiness (sympathetic
overactivity).
Take the pulse. Note the presence of sinus tachycardia (sympathetic overdrive) or atrial
fibrillation (due to a shortened refractory period of atrial cells related to sympathetic drive and
hormone-induced changes). The pulse may also have a collapsing character due to a high
cardiac output.
Arms
Ask the patient to raise the arms above the head and so test for proximal myopathy. Tap the
arm reflexes for abnormal briskness, especially in the relaxation phase.
Eyes
Examine the patient’s eyes for exophthalmos, which is protrusion of the eyeball from the orbit.
This may be very obvious, but if not, look carefully at the sclerae, which in exophthalmos are
not covered by the lower eyelid.
Next look from behind over the patient’s forehead for exophthalmos, where the eye will be
visible anterior to the superior orbital margin.
Now examine for the complications of proptosis, which include
(1) chemosis (oedema of the conjunctiva and injection of the sclera, particularly over the
insertion of the lateral rectus);
(2) conjunctivitis;
(3) corneal ulceration (due to inability to close the eyelids).
(4) optic atrophy (rare and possibly due to optic nerve stretching).
(5) ophthalmoplegia (the inferior rectus muscle power tends to be lost first, and later
convergence is weakened).
The mechanism of exophthalmos is uncertain. It occurs only in Graves’ disease. It may precede
the onset of thyrotoxicosis or may persist after the patient has become euthyroid. It is
characterised by an inflammatory infiltrate of the orbital contents, but not of the globe itself.
The orbital muscles are particularly affected, and an increase in their size accounts for most of
the increased volume of the orbital contents and therefore for protrusion of the globe. It is
probably due to an autoimmune abnormality.
Next examine for the components of thyroid ophthalmopathy, which are related to sympathetic
overactivity and are not specific for Graves’ disease.
Look for the thyroid stare (a frightened expression) and lid retraction (Dalrymple’se sign),
where there is sclera visible above the iris.
Test for lid lag (von Graefe’sf sign) by asking the patient to follow your finger as it descends at a
moderate rate from the upper to the lower part of the visual field. Descent of the upper lid lags
behind descent of the eyeball.
If ptosis is present, one should rule out myasthenia gravis, which can be associated with
autoimmune disease.

Causes of exophthalmos
Bilateral
Graves’ disease
Unilateral
Tumours of the orbit (e.g. dermoid, optic nerve glioma, neurofibroma, granuloma)
Cavernous sinus thrombosis
Graves’ disease
Pseudotumours of the orbit
Neck
Examine for thyroid enlargement, which is usually detectable (60–90% of patients). In Graves’
disease the gland is classically diffusely enlarged and is smooth and firm. An associated thrill is
usually present, but this finding is not specific for thyrotoxicosis caused by Graves’ disease.
Absence of thyroid enlargement makes Graves’ disease unlikely, but does not exclude it. Possible
thyroid abnormalities in patients who are thyrotoxic but do not have Graves’ disease include a
toxic multinodular goitre, a solitary nodule (toxic adenoma) and painless, postpartum or
subacute (de Quervain’sg) thyroiditis.
Patients with de Quervain’s thyroiditis typically have a moderately enlarged firm and tender
gland.
Thyrotoxicosis may occur without any goitre, particularly in elderly patients. Alternatively, in
hyperthyroidism due to a rare abnormality of trophoblastic tissue (a hydatidiform mole or
choriocarcinoma of the testis or uterus), or excessive thyroid hormone replacement, the thyroid
gland will not usually be palpable.
If a thyroidectomy scar is present, assess for hypoparathyroidism (Chvostek’sh or Trousseau’si
signs. These signs are most often present in the first few days after the operation.

Chest
Gynaecomastia occurs occasionally. Examine the heart for systolic flow murmurs (due to
increased cardiac output) and signs of congestive cardiac failure, which may be precipitated by
thyrotoxicosis in older people.
Legs
Look first for pretibial myxoedema. This takes the form of bilateral firm, elevated dermal
nodules and plaques, which can be pink, brown or skin-coloured. They are caused by
mucopolysaccharide accumulation. Despite the name, this occurs only in Graves’ disease and not
in hypothyroidism. Test now for proximal myopathy and hyperreflexia in the legs, which is
present in only about 25% of cases.

Feedback 4-4-2018
22 yr lady with 1 month of nervousness, tired, and weight loss of 3 kg.
Task- do relevant examination
Tell most likely diagnosis and dds with reasons
On exam- thin lean girl with brittle hair, Muscle wasting, palmar erythema, and no sweating, proximal
muscle weakness,White nail polish on nails.
There was a cat scratch mark on neck, and 1 on arm. No lymphadenopathy present. She was
pretending to be exophthalmos, and lid lag.
Thyroid exam was normal.
Examiner didnt tell anything
Case 2 Hypothyroidism
middle age woman with tiredness and all TFT give in the stem pointed toward hypothyroidism
Tasks
-perform examination of thyroid gland
-diagnosis and differential diagnosis

Examination steps (talley o connor)


1-WIPE
2-general appearance
-tired
-physical and mental sluggishness
-overweight
3-Hands
look and feel together
-peripheral cyanosis (BLUE)
-palmar erythema (RED)
-yellowish discolouration (YELLOW)
-hand swelling
-cool and dry
-pulse (rate and rhythm)
4-Face
look
-yellowish discoloration
-puffiness.
-loss of outer third of eyebrow
-hair loss
-exanthelasma
Feel
-dry, cool skin
-thin, dry hair
-tongue swelling
-ask patient to speak (hoarseness, slow)
5-Neck
-same like hyperthyroid
6-Chest
-pericardial effusion
-pleural effusion
7-others
-reflex (biceps reflex): (critical error if you did not examine the reflex)
-non pitting oedema (leg)
-proximal myopathy
-peripheral neuropathy (sensation).
Diagnosis and differential diagnosis
1-hypothyroidism (most likely from history, examination and Ix)
2-see lecture 32 of hypothyroidism
Feedback 5-12-2018 AMC feedback: health review HYPO THYROID PE
All hypo feature given in stem> no Blood test.
Task: 1.Do PE
2. Tell your finding relate with your dx (something like that)

Greetings approach (WIPE)


• Hello X I understand you are having some symptoms like …… Today I have been asked to
examine you to find out probable cause • during my examination I will be looking at you from
head to toe and examining a gland in your neck called thyroid. • If you have any discomfort,
please stop me is that ok? • X I need adequate exposure neck and hand for my examination
which I already have. Thank you. • Hand wash.

General look • my patient is sitting comfortably, Looking for any tired dull looking apathic face/
puffy face. • Dressed appropriately to the weather.

• Hands: Looking for dry / cool / periphery/ : hands are warm well perfused, skin seems dry and
coarse but no paronychia or nail changes., I asked the pt to stretch out hands- No tremor

• Pulse (for 15 sec I calculated 64): But I said 60 as its hypo pt I know.

BP : examiner told me the BP was normal.

Now can you tell me your full name: I am looking for husky voice?

Now I will be looking at you face X. will you please take off your glass?

Looking for thinning of hair,

In eye: loss of outer 1/3 of eye brow, periorbital edema , xantheleshma which I cannot
appreciate. No signs of hyper Keratinemia. I looked for anemia.. No anemia

Now X I will be looking at your neck. Can you please pretend to swallow for me and stick out
your tongue.. I cannot appreciate any thyroid lump moving.

Now x I will be going behind you and have a feel of your neck glands. Is that oK? Let me know if
its painful (thyroiditis).I will stop. Went behind palpate the gland and asked for swallowing.
Palpate the LN . Came in front Told examiner as I don’t suspect hyper thyroid and there is no
gland swelling I want to skip auscultation and percussion. Then I squat down and looked for
edema and jerk. Interesting thing is I know I had to de jerk for delayed relaxation of ankle jerk
But I was thinking I will lose my time if I ask the pt to lie in bed as she was sitting in chair. I
checked the jerk in knee and said I was looking for hyoreflexia and delayed relaxation of jerk.

Then I said to Pt from the symptoms I am suspecting hypothyroidism where a gland in your neck
releasing less hormone. But truly speaking in my examination could not find any specific and
obvious sings for that except dry course skin and your heart rate is at lower limit. But don’t
worry there is some blood test for dx it.I will also refer you to my seniors to review you.
HYPOTHYROIDISM (MYXOEDEMA)
Hypothyroidism (deficiency of thyroid hormone) is due to primary disease of the thyroid or, less
commonly, is secondary to pituitary or hypo thalamic failure.
Myxoedema implies a more severe form of hypothyroidism. In myxoedema, for unknown
reasons, hydrophilic mucopolysaccharides accumulate in the ground substance of tissues includ
ing the skin. This results in excessive interstitial fluid, which is relatively immobile, causing skin
thickening and a doughy induration.
The symptoms of hypothyroidism are insidious but patients or their relatives may have noticed
cold intolerance, muscle pains, oedema, constipation, a hoarse voice, dry skin, memory loss,
depression or weight gain.
Examine the patient with suspected hypothyroidism as follows
General inspection
Look for signs of obvious mental and physical sluggishness, or evidence of the very rare
myxedema madness. Hypothyroid speech is a feature in about one-third of patients. This is
characteristically slow, nasal and deep in pitch. Obesity is no more common than in euthyroid
people.
Hands
Note peripheral cyanosis (due to reduced cardiac output) and swelling of the skin, which may
appear cool and dry. The yellow discolouration of hypercarotenaemia (there is slowing down of
hepatic metabolism of carotene) may be seen on the palms.
Look for palmar crease pallor—anaemia may be due to: (1) chronic disease; (2) folate deficiency
secondary to bacterial overgrowth, or vitamin B12 deficiency due to associated pernicious
anaemia; or (3) iron deficiency due to menorrhagia.
Take the pulse, which may be of small volume and slow.
-Test for sensory loss, as the carpal tunnel is thickened in myxoedema.
Arms
Test for proximal myopathy (rare) and a ‘hung-up’ biceps or Achilles tendon reflex.
Face
Inspect the patient’s face. The skin, but not the sclerae, may appear yellow due to
hypercarotenaemia. The skin may be generally thickened, and alopecia may be present, as may
vitiligo (an associated autoimmune disease). Inspect the eyes for periorbital oedema.
Loss or thinning of the outer third of the eyebrows can occur in myxoedema but is also
common in healthy people. Look for xanthelasmata (due to associated hypercholesterolaemia).
Palpate for coolness and dryness of the skin and hair. There may be thinning of the scalp hair.
Look at the tongue for swelling. Ask the patient to speak, and listen for coarse, croaking, slow
speech. Bilateral nerve deafness may occur with endemic or congenital hypothyroidism.
Thyroid gland
A primary decrease in thyroid hormone results in a compensatory oversecretion of TSH. A goitre
will result if there is viable thyroid tissue.
Many cases of hypothyroidism are not associated with an enlarged gland as there is little thyroid
tissue. The exceptions include severe iodine deficiency enzyme deficiency (inborn errors of
metabolism), late Hashimoto’s disease or treated (with radioactive iodine) thyrotoxicosis.
Chest
Examine the heart for a pericardial effusion and the lungs for pleural effusions.
Legs
There may be non-pitting oedema. Ask the patient to kneel on a chair with the ankles exposed.
Tap the Achilles tendon with a reflex hammer. There is apparently normal (in fact, slightly
slowed) contraction followed by delayed relaxation of the foot in hypothyroidism (the ‘hung-up’
reflex.
Examine for signs of peripheral neuropathy and for other uncommon neurological abnormalities
associated with hypothyroidism.
Neurological associations of hypothyroidism
Common
Entrapment: carpal tunnel, tarsal tunnel
Delayed ankle jerks
Muscle cramps
Uncommon
Peripheral neuropathy
Proximal myopathy
Hypokalaemic periodic paralysis
Cerebellar syndrome
Psychosis
Coma
Unmasking of myasthenia gravis
Cerebrovascular disease
High cerebrovascular fluid protein
Nerve deafness

Thyrotoxicosis and hypothyroidism


Causes of thyrotoxicosis
PRIMARY
Graves’ disease
Toxic multinodular goiter
Toxic uninodular goitre: usually a toxic adenoma
Hashimoto’s thyroiditis (early in its course; later it produces hypothyroidism)
Subacute thyroiditis (transient)
Postpartum thyroiditis (non-tender)
Iodine-induced (Jod-Basedow* phenomenon— iodine given after a previously deficient diet)

SECONDARY
Pituitary (very rare): TSH hypersecretion
Hydatidiform moles or choriocarcinomas: hCG secretion (rare)
Struma ovarii (rare)
Drugs (e.g. excess thyroid hormone ingestion, amiodarone)
Causes of hypothyroidism
PRIMARY

Without a goitre (decreased or absent thyroid tissue):


• Idiopathic atrophy
• Treatment of thyrotoxicosis (e.g. 131I, surgery)

• Agenesis or a lingual thyroid


• Unresponsiveness to TSH

With a goitre (decreased thyroid hormone synthesis):


• Chronic autoimmune diseases (e.g. Hashimoto’s thyroiditis)
• Drugs (e.g. lithium, amiodarone)
• Inborn errors (enzyme deficiency)
• Endemic iodine deficiency or iodine-induced hypothyroidism

SECONDARY
Pituitary lesions
Tertiary Hypothalamic lesions
Transient
Thyroid hormone treatment withdrawn
Subacute thyroiditis
Postpartum thyroiditis
Feedback 7-9-2018
Thyroid examination- FAIL
23 year old lady feeling weakness for last few weeks. Her TSH- 0.02, T3 & T4- 25/35 (very high).
- examine the pt
- explain the condition / possible causes to her. (I'm not sure which one was the task.)
Global score: 3 Key steps: 2/3
Approach to pt: 4 Choice of technique of examination, organisation and sequence: 5
Accuracy of examination: 3 DDX: 3
I was shocked to see that I failed this simple station !!!
I can see that I got good mark in Choice of technique of examination, organisation and sequence, but poor
mark in accuracy. May be I hurried a little bit and didn't finish all the steps properly. I only mentioned
thyroid ddx, forgot to say other reasons for her weakness!! Anyway, I am going to jot down my experience
of this case in the exam.
WIPER
General appearance- sitting comfortably, no respiratory distress, no facial deformity.
Vitals- I started to check pulse by myself and asked examiner if I could get any watch. (no). Then, I just said
pulse rate feels regular to me. I turned to examiner and asked if I could measure BP or not? Examiner said
vitals were not checked yet. I said okay, I would like to do it later on. ( I got scared a little bit that time
because vitals are important in hyperthyroidism.)
Hand changes first- thyroid acropachy changes? -no
fine tremor?- yes
palms- sweaty?- no
(I have already checked pulse)
Arm- hair distribution normal, dry,pale skin?
hair?- dry, fine hair?
Face- eye brow loss? (usually in hypo) exophthalmos? lid retraction test, H-test (opthalmoplegia?) At that
moment I was looking for a red-top pen in the tray. Examiner asked me what I was looking for. I said
nothing and picked up a normal pen and did H-test.
All normal.
Neck- Inspection
Palpation from behind- explained to pt before doing it. - normal
Asked for sipping of water and sticking out tongue this time- normal
cervical lymph nodes- normal.
percussion over sternum
auscultation for thyroid bruit
Special tests-
Proximal myopathy- normal
Pemberton's test-normal
Reflexes-checked knee.- normal.
oedema- while I was pressing over medial malleolus , examiner said no oedema.
Then, I started to explain to the pt- Remember last time, we did a blood test about your thyroid function?
The result is with me atm. According to it, your thyroid is over functioning at this time. There are few
reasons why we get this finding- ( I started to scribble on the paper.)
1. Inflammation of thyroid tissues, can happen after a flu or viral infection and can change thyroid function
temporarily, we called it thyroiditis.
2. Sometimes, our body react against thyroid tissues as well, we called it autoimmune thyroiditis. It's very
common in this age group.
3. Sometimes, these could be associated with some growth of thyroid tissues. There are few cells in
thyroid that can presents with hyper growth or accumulation of fluid; such as colloid cyst, papillary cells
growth, follicular cells growth. I couldn't find anything though . However, I would like to further
investigations to know the cause.
157-Anemia in pregnancy
Case (21-6-2018)
Pt 30ws pregnant, comes to review the results you ordered the day before. Hb:9.0 microcytic
hypochromic anaemia. glucose: normal.
Task:
-hx (4min).
-Explain investigation
-Dx
-Management

Feedback 22-6-2018
Pt 30ws pregnant, comes to review the results you ordered the day before. Hb:9.0 microcytic
hypochromic anaemia. glucose: normal.
Task:
hx (4min). Explain investigation Dx Management
Approach Greeted patient
Asked if she is feeling light headed, dizzy would like to lie down. She said she is alright she wants
to know what the results mean So I explained the result first. Then told her number of causes
and risk factor and cause these so I would like to take a through hx.( I did not do well in this case)
So I asked about the any tummy pain, baby kick, discharge from below.
Asked headache or leg swelling, racing in the heart
Then antenatal checkup done or not, Folic acid taken or not, Asked diet—vegetarian
Bleeding diseases in her or family, Colour of urine, Stickiness of stool to pan, On any blood
thinning medication Known kidney or liver condition Ethnicity.
Previous pregnancies—she said 4 ( I completely forgot to ask about the gaps in the pregnancies
Periods—any heavy bleeding Blood group
The I said as I mentioned earlier you are having Anaemia. Then I said most likely it is Iron
Deficiency. Thought about chronic diseases, thalassemia seemed unlikely To confirm the
diagnosis I would like to take blood to do Iron profile
Mx
Refer to High risk preg clinic
Start on iron tablets
Will give stool softners
Check iron profile again in 2 weeks
Continue treatment for at least 2-3 months.
Asked her any questions—will this anaemia harm my baby
I said complication in mother is heart failure and in baby is IUGR
But don’t worry in the High risk pregnancy clinic you and your baby will be frequently monitored
for any complications.
Bell rang Said thank you to pt and roleplayer ( Most likely the cause was reduced gap in
pregnancies which I forgot to ask.)
Case (15-3-2018)
30 weeks pregnant, GP, 4th pregnancy. Tests show hypochromic microcytic anaemia.
Tasks:
-take further focused history
-ask pefe
-give dx
-mx
(vegetarian, children 5, 3, 1 respectively)

Feedback 25-10-2018
22 yr old woman, 8 wks of preg came with blood test results: Values were for microcytic
hypochromic anemia with normal iron levels.
Task:
History
d/d
further inv

I took history for possile causes of anemia. Asked ethnicity and finally explained possibility of
thalaseemia n said ALL antenatal inv plus hb electrophoresis.

Approach
Explaining results
-Some blood tests have been performed and the results are now with me to explain to you so let
us see the results together.
-We have three types of blood cells the red blood cells, which contain an iron rich protein called
Haemoglobin, which is responsible for carrying oxygen to the tissues. White blood cell to help
fighting against infection and platelets to help with blood clotting.
-Platelets and WCC are normal.
-But there is a decreased level of hemoglobin we call this anemia.
-Also this MCV is less than the normal range this mean that the size of the red blood cells are
small.
-There are few possibilities but first I need to ask you a few questions in order to reach the
diagnosis.

History
1-Late pregnancy complications questions
-How’s your pregnancy so far?
-Any tiredness, dizziness?
-any chest pain, palpitations, shortness of breath?
-any tummy pain, vaginal bleeding or discharge?
-any headache, blurring of vision or leg swelling?
-any fever, nausea or vomiting?
-is the baby kicking well? (Baby problem)
2-recurrent visits questions
-Have you had regular antenatal checks?
-How were the blood tests? Are you aware of your blood group?
-have you done down syndrome screening?
-US at 18-20 weeks gestation? Is it single baby? Any birth defects? What is the position of the
placenta?
-Sweet drink test at 28 weeks?
-Repeat ultrasound at 32/34 weeks?
-Bug test at 36 weeks?
-did you take your folic acid?

3-Past medical history


-Any bleeding disorders, are you bleeding from anywhere in the body like gums, black stools?
-celiac disease or any chronic diseases?

4-past surgical history

5-past obstetric history


-is this your first pregnancy? How many pregnancies have you had?
-when was your last pregnancy?
- What are the ages of your children?
-How were your previous pregnancies and deliveries? Any complications during those?
-Did you have anemia before you got pregnant or anytime during your previous pregnancies?

6-Past gynaecological history


How were your periods before you became pregnant? Any excessive bleeding?

7-Social
-What is your usual diet? Does it include meat and green leafy vegetables? Are you on any
special diet?
-SAD -Medications
-family history and origin

PEFE
1-General appearance: pallor, skin for petechial or bruising
2-Vital signs
3-CVS/Respi/CNS
4-Abdomen: fundal height, FHR, lie, presentation
5-Pelvic examination
Inspection of the vulva and vagina: bleed, discharge
Speculum: cervix for any bleed or discharge
6-Office test: UDT, BSL
Diagnosis and Management
-What I am suspecting in you is an iron deficiency anaemia. It is a common condition that
happens in pregnancy because there is an increased demand during pregnancy because you
need to supply iron for yourself and your baby.
-The causes for this condition are multiple. It could be due to the diet, or the pregnancies that
are close together, and as you had a history of anaemia.
-other possibilities are thalassemia, chronic disease, infection, bleeding disorder but unlikely.

- Iron level is correlated with the oxygen supply, which is important for the baby's growth and
development. It is good if it diagnosed and treated early, to prevent any problems during
pregnancy.

-If it is untreated, it can cause lethargy, tiredness, exhaustion and dizziness, it can lead to heart
failure and also postpartum haemorrhage or bleeding after delivery. In the baby, it can result in a
low birth weight and a growth retardation.
Management
-I would like to arrange blood test; iron studies including ferritin. (Critical error)
-Refer to High-risk pregnancy clinic.
-Start on iron tablets.
-The iron tablets should not be taken along with dairy products, but you can take orange juice
along with it as vitamin C increases the absorption of iron.
-After you start on iron, your haemoglobin levels will go up in two weeks' time, but you need to
continue taking the iron tablets still for at least 3 months after your iron comes back to normal
to replenish the iron stores. Will Check iron profile again in 2 weeks.
-Once you start taking the iron tablets, you may experience a bit of abdominal discomfort, and
constipation, your stools may turn black. You should take a lot of water to avoid constipation.
-Include green leafy vegetables, red meat, beans and lentils in your diet, and cut down on
coffee, tea, soy products as all these can decrease your iron absorption (key point).
-If no improvement is seen after taking the supplements, then I will refer you to the
haematologist for further assessment and management.

Read handbook condition 71


158-Delayed PTSD post colonoscopy
Sample Case
You are a HMO in the psychiatric department of the major hospital. You have been called to see
27 yo , Jane , who just had a colonoscopy for her recurrent abdominal pain. The result was
normal and during the procedure, she was given extra dose midazolam, as she was a bit
agitated. In the recovery room, she woke up quite anxious and you were called because she said
that she needs to talk someone.
Tasks
-talk to patient
-explain the most probable cause of the symptoms
-discuss further management

Exam cases
1-(27-4-2017) 27 y/o female has had colonoscopy because of recurrent abdominal pain which
was normal.after that she is given midazolam because of pain.
You are working as an intern in GI department.the nurse told you that the patient "has
remembered something " that wants to talk to someone.
Task:
-take further history from the patient
-explain the most probable diagnosis for the patient
2- (1-3-2018) PtSD: Hospital, post colonoscopy. Experienced ptsd of childhood sexual abuse. Task
hx and mx.
4-(15-8-2018)

History:
1-after you enter she starts crying so offer tissue/ water
(the patient saying I can feel him, see him, smell him)

2-Ensure confidentiality

3-Open ended question


-can you tell me whom you are talking about? (is my Neighbour)
-can you please tell me more about your Neighbour? (Was sexually abused by Neighbour when
10 years old)

4-sexual abuse questions


-sorry to hear that, have you reported this incident to your family members, police or any
friends? (no because he is known to the family)
-are you feeling safe at home now?
-is this Neighbour still living nearby your house? (no he moved away and she feel safe at home)

5-PTSD questions
-do you have any nightmares or flashbacks?
-any other recent traumatic events other than this?
6-Delirium questions
-do you know where you are right now?
-what time of the day is this?

7-colonoscopy questions
-do you know why you have had colonoscopy? (recurrent abdominal pain and the GP requested
it)
-well the results of the colonoscopy has turned out to be good so how do you feel about that?

7-Psychosocial History and HEADS


Mood
-how is your mood in general?
-have you ever thought of harming yourself or others?
-how is your sleep?
-how is your appetite? Have you lost or put on weight recently?
Delusion and hallucination
-just a routine questions I usually ask do you usually feel, hear, see things that others do not?
-do you think someone is trying to harm you or spying on you?
Insight and judgement
-do you think you need a professional help? (no)
-if we supposed that there is a fire in this room so what would you do?

HEADS.
Home
-who do you live with?
-do you have enough support?
-any stress at home?
Work
-what do you do for living?
-any stress at work?
-any difficulty concentrating at work?
Anhedonia
-have you lost interest in things you used to enjoy?
SAD
Social activity

8-General questions
-have you had any past or family history of mental problems?
-any recent infections, head injury or thyroid problems?
-how is your periods?
Explain Delayed PTSD
Condition
Based on the history, most probably, you have a condition called delayed PTSD. It’s an anxiety
disorder, now it’s included under the trauma and stress related disorder commonly experienced
by people who had gone through or witnessed a severe distressing psychological events which in
your case is the sexual assault you had at 10 yo.

Clinical features
in this condition you start getting distressing recollection of the events in the form of flashbacks
and nightmares, which could be recurrent and you tried to avoid similar situation.
we call it delayed if you have got these symptoms any time after 6 months of the traumatic
event.

Cause.
Why you having these symptoms now may be because of extra dose of midazolam you had
received during the procedure and as you come out of the effect of the midazolam, you can get
memories that have been in your subconscious mind.

Management

•would you like to make a formal complaint against this person so he could be persecuted. This
is your decision so if you want I can assisted you.

•Refer to the Psychologist for CBT a talk therapy we call trauma-based psychotherapy.

•would like to refer you to a psychiatric as well.

•I will ask the senior to have a look at you.

•PTSD support groups

•with your consent, I can arrange family meeting.

•Relaxation techniques (yoga/ meditation), Reading materials regarding to the delayed PTSD ,
Review by her GP.
Case (27/4/2017) fail
post colonoscopy lady, unknown Dx

20 yr woman feel anxious and uncomfortable after post colonoscopy which was nl. She need to
use more sedation during the procedure. She thinks she need to talk to someone about the
things in her mind.

H/o, explain the possible cause to pt.

When I entered the room, a young lady was lying on the bed. I introduced myself and start
greeting.
Ask her name and ask her feeling. I explained her that colonoscopy results were good. No abnl
findings.
She started to tell me that she is not feeling well and feeling frightened. I offered confidentiality.

She told me that it was like a bad dream and feeling frightened. When I asked her about the
dream.
She just said it was like a bad dream. Any similar feeling before, she said what do you mean. I
mean feeling like that frighten, anxious. she said no. anything you want to talk. Then, I asked
HEADS,

*PF (HEADS clear, when asked further; she asked what do u mean? When asked about any rape
or childhood accidents or events or injury or surgery, she said no, she said it was like a bad
dream, ask about any flashback or dreams -no), past mental problem, past medical problem and
SADMA-clear.
I am a little bit confused. I just remained calm and told her that I understand how you feel.

It can happen sometimes, it can be anxiety problem or can be seen in patient with PTSD, if they
had events in the life in the past, they can present like that. If you want to talk to me anything,
you can talk to me at any time, so that, I can help you as much as I can. Or if you want I can refer
u to psychologist, you can talk to him or her. Or you can talk to your gp. Plz take your time. I want
u to know that I am here to help you. …just reassure…. Time up….
FB-FAILED, APPROACH-3, HISTORY-3, DDX-1.

Case (27/4/2017)
27 y/o female has had colonoscopy because of recurrent abdominal pain which was normal.after
that she is given midazolam because of pain.
You are working as an intern in GI department.the nurse told you that the patient "has
remembered something " that wants to talk to someone.
Task:
1-take further history from the patient
2- explain the most probable diagnosis for the patient

Patient says that can hear the sound of her neighbour in her childhood period that asks her sex
Dx delayed PTSD
Feedback 1-3-2018
Hospital, post colonoscopy. Experienced ptsd of childhood sexual abuse. Task hx and mx.
Abdominal pain PASS
recurrent abd pain for which colonoscopy was done, and she rememebered something wants to
talk
old recall
i did complete psychosocial hx. asked about hyperarousal, nightmares,flashbacks etc and
avoidind situations that remind her of that
made sure she was safe and felt safe now and the abuser wasnt around,asked if she wants to
involve police she said no, offered continuing support in case she changed her mind,
mx> CBT as i said its delayed PTSD.
Explained that recurrent abd pain could be due to the same stress and cbt will help with that too
as colonoscopy was normal

15-8-2018 Feedback
Scenario: Abdominal Pain
Stem: Young lady who had a colonoscopy for recurrent abdominal pain is in the recovery room.
She is anxious teary on waking up and wants to talk to someone.
Tasks
~history
~explain most probable dx
A lady in 30s was sitting on couch and looked dull, down and teary. I introduced myself,
reassured her and told about confidentiality. She was not saying anything. Again I reassured and
talk about confidentiality. She started sobbing but didn’t say anything. There was a cup of water
and tissues on table.i offered both. She sipped water took tissues but didn’t open up.
so I started taking history as I felt running out of time and her not connecting to me.
Ptsd questions
Colonoscopy question
Ruled out organic
Detailed psychosocial history ( she was mumbling throughout “ I can still feel him, that was
horrible I was just a child etc) tried taking history about the event happened and has she
informed anyone etc.
Gave dx of delayed ptsd and explained it… ddx of anxierty, depression,stress, drugs, thyroid etc (
missed out delirium)
Grade: FAIL
( Now I feel I failed that station because of not able to patint make feel safe and engaged or
partly missing out on other impotant differentials). This is very famous recall of delayed ptsd
guys and I knew it by heart so pls look for the important points.
Feedback 16-8-2018 pass (important)
Station 11. Delayed onset of PTSD
HMO, 32 yrs old lady recently underwent colonoscopy due to abdomen pain. The result is
normal. During colonoscopy, she was needed to use more midazolam. After recovery, she feels
something and want to talk with doctor.
Task: history
Explain the condition to patient


Confidentiality. The patient did not open up anything yet. (Other candidate said the patient told
all after confidentiality). I gave reassurance again & again. She said happened something at the
age of 8 yrs. She smelled him right now. (I reassure again and ask who is he). She said her
neighbour. What kind of things happened? she said sexual abuse and then cry. I gave water &
tissue (I didn't see tissue box which is placed far away from patient, so I gave rough tissue on the
wall which is used after hand washing. The patient said it is ok, I don't use tissue but she drinks
water. After that, I saw tissue box on the chair far away from patient. I went to take it and gave
her). I am silent until she said it is ok. I asked: similar feeling before: no. No vivid dream, no flash
back, no avoidance. No inform to police at that time because she is afraid of her parent knowing
about this. (I offer do you want to inform police-? sorry, I forgot answer).
HEADSS: Live alone. Work- forgot. No financial problem. Not sexually active. No smoke, no
alcohol, no recreational drugs. No features of depression. Not easily anxious person.
General health: no fever, no thyroid p/b, no head injury, no weakness, normal poo & pee apart
from abdomen pain.
Explain: Delayed onset of PTSD. Bad memory in unconscious mind. After using sedative
medication for the procedure, suppress the conscious mind & the event came out from
unconscious mind.
Other possibility: SE of medication, organic p/b (Infection, thyroid, drugs, heart)

Grade: Pass
     Global score: 4
Key Steps: 1- Yes
2- No
3- Yes
Approach to patient: 5
History: 5
Diagnosis/ Differential diagnosis: 3
159-Ear examination
Brief history would be
FDD NVV
WOO MM ATP

1-Ear pain questions


-Duration (already mentioned in the stem)
- where exactly the pain is? Any pain in the other ear?
-Radiation
-severity.
-character.
-anything make it better or worse?
-has this happened before?

2-FDD NVV
-any fever?
-any ear discharge? colour?
-any hearing problems? One or both ears? Do you hear better in noisy environment? Have you
increased the TV volume?

-any nausea or vomiting?


-do you feel room spinning around you? (Vertigo)

3-WOOO MM ATP
wax: have you had problems with wax in your ears in the past?
Otitis media: have you had recent infection? Any recurrent ear infections?
Otosclerosis: is there a history of deafness in your family?
Occupation: does your job or hobby expose you to loud noise?

Meniers disease: any ringing in your ears?


Medication: do you take any medications that affected hearing?

Acoustic neuroma: any problems with walking or balance


Trauma: any injury to your ears
past medical history: have you had any history of serious infections like meningitis?
Talley o Connor ear examination (7th)
Examination Method
Ear examination consists of inspection and palpation, otoscopy, tuning fork assessment, testing
hearing and peripheral vestibular examination. In some cases lower cranial nerve assessment is
indicated.

Inspection
1-Inspect the position of the pinna and note its size and shape.
2-Look for deformity an obvious accessory auricle (separate piece of cartilage away from the
pinna), cauliflower ears (haematomas from recurrent trauma, which obscure the normal
anatomical features of the pinna) and bat ears (protrusion of the ears from the side of the head)
3-Note any scars or swelling around and behind the ears.
4- Look for erythematous(redness) inflammation externally and any obvious ear discharge.
5- Inspect the auditory meatus and outer ear.
6-Then look for signs of gouty tophi (nodular, firm, pale and non-tender chalky depositions of
urate in the cartilage of the ear, specific but not sensitive for gout).
7- Look to see if the patient is wearing a hearing aid; if so, remove it.

Palpation
1-Palpate the pinna for swelling or nodules.
2-Pull down the pinna gently; the manoeuvre is often painful when there is infection of the
external canal.
3- Feel for nodes (pre- and post-auricular)

Otoscopy
1- Otoscope examination of the ears requires use of an earpiece that fits comfortably in the ear
canal to allow inspection of the ear canal and tympanic membrane.
This examination is essential for any patient presenting with an upper respiratory tract infection,
any symptom related to the ears, dizziness, facial weakness or head injury.
2- Always examine both ears! The correct technique is as follows.
3- Ask the patient to turn his or her head slightly to the side,
4- then pull the pinna up, out and back to straighten the ear canal and provide optimal vision.
5- Stretch out the fingers of your hand holding the otoscope to touch the patient’s cheek, to
steady the instrument and to prevent sudden movements of the patient’s head.
6- When examining the patient’s right ear, the otoscope is preferably held in a downward
position with the right hand, while using the left hand to pull the pinna. An alternative position
involves holding the otoscope upwards, but there is a risk that if the patient moves suddenly
injury is more likely to occur.
7- Look at the external canal for:
- any evidence of inflammation (e.g. redness or swelling) or discharge.
- There should be no tenderness unless there is inflammation.
- Earwax is usually white or yellowish, and translucent and shiny; it can be moist or hard and
impacted. It may obscure the view of the tympanic membrane.
- Blood or cerebrospinal fluid (watery, clear fluid) may be seen in the canal if there is a fracture
at the base of the skull.
- In patients with herpes zoster, there may be vesicles (fluid-filled blisters) on the posterior wall
around the external auditory meatus.

8- Inspect the tympanic membrane (ear drum) by introducing the speculum further into the
canal in a forward but downward direction.
- The normal tympanic membrane is a pearly grey colour.
- It is ovoid in shape and semi-transparent.
- The upper fifth is called the pars flaccida and the lower four-fifths are called the pars tensa.
- The handle of the malleus is often visible near the centre of the pars tensa.
- From the lower end of the handle a bright cone of light should be visible: the light reflex. The
presence or absence of the light reflex is not a sensitive or a specific sign of disease
- Note the colour, transparency and any evidence of dilated blood vessels (hyperaemia—a sign
of otitis media.
- Look for bulging or retraction of the tympanic membrane. Bulging can suggest underlying fluid
or pus in the middle ear. Retraction means a reduction in pressure in the middle ear and is a sign
of a blocked Eustachian tube.
- Perforation of the tympanic membrane should be noted.

Hearing assessment
1-Wisper test
- to test hearing, whisper numbers or words such as ‘mark’ or ‘park’ into one of the patient’s
ears while the other ear is distracted by movement of your finger in the auditory canal or the
canal is occluded by pressure on the tragus.
- It is important to stand behind the patient to prevent lip reading.
- Then repeat the process with the other ear.
- With practice, the normal range of hearing is appreciated. If a patient says he or she is deaf, or
the whisper test is positive, formal hearing testing (audiometry) is indicated.

2- Traditionally, Rinné’s and Weber’s tests are performed if deafness is suspected, but these are
not very useful screening tests:

A. Weber’s test: place a vibrating 512 Hz fork at the centre of the patient’s forehead. Nerve
deafness causes the sound to be heard better in the normal ear, but with conduction deafness
the sound is heard better in the abnormal ear.

B.Rinné’s test: place a vibrating 512 Hz tuning fork on the mastoid process. When the sound is
no longer heard move the fork close to the auditory meatus where, if air conduction is (as is
normal) better than bone conduction, it will again be audible.
Hearing tests
1-whispering test
- A simple test involves covering the opposite auditory meatus with a finger, and moving it
about as a distraction while whispering a number in the other ear.
- This should be standardized by the use of set numbers for different tones. For example, the
number 68 is used to test high tone and 100 is used to test low tone.
- Whispering should be performed towards the end of expiration in an attempt to standardise
the volume and at about 60 centimetres from the patient’s ear.
- Your larynx should not vibrate if the whispering is soft enough.
- If partial deafness is suspected, perform Rinné’s and Weber’s tests:

2- Weber test (is the buzzing louder on one side?(


—a vibrating 512 Hz tuning fork is positioned on the centre of the forehead.
- Normally the sound is heard in the centre of the forehead.
- Nerve deafness causes the sound to be heard better in the normal ear.
- A patient with a conduction deafness finds the sound louder in the abnormal ear.

3-Rinné sx test (where does it sound louder?)


—a 512 vibrating tuning fork is placed on the mastoid process, behind the ear, and when the
sound is no longer heard it is placed in line with the external meatus
Normally the note is audible at the external meatus.
- If the patient has nerve deafness, the note is audible at the external meatus, as air and bone
conduction are reduced equally, so that air conduction is better (as is normal). This is termed
Rinné-positive.
- If there is a conduction (middle ear) deafness, no note is audible at the external meatus. Bone
conduction is better. This is termed Rinné-negative.

Causes of deafness.
Unilateral nerve deafness may be due to:
(1) tumours, such as an acoustic neuroma;
(2) trauma, such as fracture of the petrous temporal bone; or
(3) vascular disease of the internal auditory artery (rare).
Bilateral nerve deafness may be due to:
(1) environmental exposure to noise;
(2) degeneration, such as presbyacusis;
(3) toxicity, such as aspirin, gentamicin or alcohol;
(4) infection, such as congenital rubella syndrome, congenital syphilis; or (vi) Ménière’s disease.
Brainstem disease is a rare cause of bilateral deafness.

Conduction deafness may be due to:


(1) wax;
(2) otitis media;
(3) otosclerosis;
(4) Paget’s disease of bone.
Feedback 14-3-2018
Ear Pain in a young man.
Tasks were Hx not more than 2 mins, Physical examination, diagnosis to the patient.
After introduction, I asked where exactly he had the pain? He told about his right ear. Left ear
was fine. The pain was deep in the ear, not going anywhere and increasing. Nothing helped to
relieve or aggravate it.
It was followed by ear infection although he didn’t have any discharge. He also had some
hearing loss.
He couldn’t tell if he heard better in a crowded or noisy environment and he was increasing the
volume of TV.. He had a history of ear infections previously for which he took antibiotics. He
was fine between episodes.
He didn’t swim a lot. No air travel recently. No use of ototoxic drugs or exposure to loud music.
No FH of deafness.
Non smoker and non drinker.
Took consent for PE.
Started inspection on the role player and commented on his right ear and compared with his
left ear.
There was no abnormality, no signs of trauma, no redness or swelling or any discharge. No
swelling in-front of ear and looked for any signs of mastoiditis. Palpated ear, parotid and
mastoid.
Did rest of the examination on the mannequin, whose left ear was exposed. Held the otoscope
in left hand and started inspection by commenting that I was pulling the pinna upward and
backward to straighten his ear canal.
No wax, discharge or any foreign body was present. Tympanic membrane was ruptured in the
Attic area and there was deposition of white pearly material in the tympanic membrane. TM
was grey in colour with normal light reflex and handle of malleus. I withdrew it and looked for
any discharge on the otoscope.
I told the role player that because of the repeated infections he had a ruptured ear drum in the
upper part which is known as ‘unsafe perforation’ being in that sensitive part of ear drum. I
was more concerned because he also had developed a condition called Cholestetoma, due to
abnormal deposition of skin as a result of repeated ear infection.
I told him I I was afraid it had some complication where it involved nerve that supplies the
muscles of facial expressions and could also cause infection of the covering of the brains. I
further asked If he minded if I do a couple of further examination on you.
He cooperated well. I did examination for his facial nerve and asked him to touch his chin with
his chest to rule out neck stiffness in meningitis. Everything was fine.

I completely forgot about doint tuning fork test, plus I wasn’t left with enough time either. So I
concluded by saying I would refer him to the specialist for hearing assessment by audiometry
and he might do some CT scan to see if there was any extension of this growth towards his
brain. Bell rang.
Scenario …. Ear Pain Grade… Pass

Feedback 14-3-2018
Ear Pain for months with discharge- PE and Dx
There was a head model in the room with a table on which there were 2 tuning forks – Otoscope

There was a young pain – apparently in pain – not very cooperative – said he had pain with
discharge for months – then took brief history – asked him about loud noises/trauma/infection/
tonsillitis / said no – then he said he has been having this since childhood !

The consented for PE – inspected external ears on both sides – scars – discharge – trauma -
palpated for LN – wanted to examine throat then examiner said no tongue depressor – then
used the otoscope and examined the ear on the model – there was obvious TM rupture with
discharge and visible ossicles. – I told him that he had ruptured TM may be du to trauma –
infection-loud noises and I need to refer you to ENT for management – I told him about TM
grafting then examiner said no management then bell rang ! I DID NOT perform HEARING TESTS
with TUNING FORK.

Feedback 14-3-2018
ear pain PE
hx 2min
reccurent otitis media with discharge (yellowish thick discharge with blood),
PE (CONDUCTIVE hearing loss in the left ear)
Otoscopic exam (perforated ear drum with yellowish debris on it)

Explain DX to the pt(I said recurrent infection cause damage to the small bones in the middle
ear could chloesteatoma need hearing test and ENT referral)
Feedback 6-6-2018
unscored- not sure , please check with other candidates.. ear examination
Middle aged man, hearing pblm since last year. (in the scenario, not given left or right, not given
what kind of hearing pblm)
Task: physical examination, explain dx,ddx

Patient is sitting. dummy with only half head on the bed


WIPE, inspection to the patient, palpation to patient, otoscopy to the dummy.. wax impaction,
could not visualize the tympanic membrane, whispering test normal both sides, Rinne and
Weber.. conductive deafness.
Explain it could be because of wax impaction, ddx : cholesteatoma, otitis media .however,
could not see ear membrane and inner part at this moment because of wax impaction, will
check it later.

Feedback 9-5-2018
Ear Pain. Failed
Young patient with left ear pain.
Task:
Take history no more than 2 minutes
Perform Ear exam, give diagnosis and differential with reason to patient.

History of ear pain, Socrates, no fever, no other symptoms when ask about hearing loss he said
his girlfriend has been complaining he doesn’t hear well. I think he said he has ear infection
before or ear pain before in two more occasions, sorry can´t remember well.
PE: first I explained what I was doing to the patient. Gross hearing assessment: reduced hearing
in the left ear. Rinne and Weber tests to the patient: Rinne test negative in left ear, weber test:
the patient said he wasn´t sure.
Otoscopy to the manikin (examiner asked me to put the gloves on to manipulated the doll). I
explained inspection of the ear lobe and the used the otoscope I said collection of pus in the
middle ear with erythema of the tympanic membrane, most likely middle ear infection.
Then I said in the background of ear pain and a conductive hearing loss in the left ear the most
likely diagnosis is acute middle ear otitis with effusion. I mentioned other causes of ear pain
and conductive hearing loss.
I failed so look for someone who passed this one.

Feedback 9-5-2018
Ear exam, cholestoma -Task- history in 2 m, perform PE and explain to patient
2 Munites- DD of ear pain- ostitis media, cholestetoma, trauma, FB. Most important is 12
cranionerve invole or not in cholestetoma cases.This cases i will need to run around to take
instrument so i should do everything quickly.

Inside: I think this is easy case, you can have look on other candida notes. The thing here is you
have to remember what turning fork you have to take and remember to refer the patient to
specialist for audiometry and CT scan.PE_ Insepction, palpation- lymphonote and ouside the ear,
movement then special test.
Feedback 19-7-2018
Case : a young female having progressive hearing loss in left ear which is getting worse .
Tasks : ear examination, Dx n ddx to pt
Started with wipe etc .did the ear examination according to the format as we do . On otoscopy i
saw lots of wax in the left ear which had the hearing loss so i told her that what she is having is
a conducive hearing loss and gave dds of conductive hearing loss but told her that these are
unlikely as ... then told her abt my otoscopy findings and explained about presence of wax in ear

feedback 19-7-2018 Fail (Key steps 1,4-no SCORE 4,3,4,3,3)


EAR Examination and DX and DDs to the patient.
I was Shaky in this stations, and I was tired as well ,its my second last station, I forgot CN 7)
I started with Wiper ,Vitals, G. INSP,palpation ,otoscope( dummy with half face was there, white
crystals with perforation at 11o clock position was seen)

Feedback 22-6-2018
Ear Examination for left ear pain. Task hx, PE with running commentary to examiner, explain
diagnosis to pt
Asked about pain, discharge, fever, diving or flying, recent flue, swimming, ringing, clean ear
often or not, foreign body, trauma, first time or recurrent
Consent, Wash hands, GA, Inspection of the ear, Palpation.
Auroscope in dummy—for me it was tympanic membrane rupture, TM looked swollen and
inflamed
Hearing test—Whisper test
the 512 tuning fork was not working
I did the first time pt could not hear.
Then I kept banging on my hand and listening to it, I only gave to pt when I could listen (I ended
up hurting my hand the tunic fork was very old and faulty, some of the other examinees directly
told examiner it is not working but I stupidly kept banging it to my hand).
So found out Conductive deafness of left ear (bone>air conduction, Webber lateralized to the
affected left ear).
To examiner I said ASOM with ruptured tympanic membrane causing conductive deafness (I
mentioned abbreviation stupidly!!!!)
Bell rang
Said to pt you have middle ear cavity infection with rupture of ear drum.
Said thank you with body outside and head inside.

Recall 16-3-2018
ear examination(chlesteatoma)
Recall 10-4-2018
Ear Examination
Young Male – Recurrent Otits Media
Do a PE and Explain to the patient what you see
Hyperaemic ( Red Hyperaemic Mass behind the Tympanic Membrane)
No Perforation
Hearing was Ok as reported by patient . Tender over Left Mastoid Bone
Tuning Fork given X 2 so choose the correct one
PE Rinne & Weber and Explain to the patient exactly what you are about to do and why and
commentate as you go along.
Recall 9-5-2018
Ear exam, cholestoma
Recall 10-5-2018
Ear examination with conductive hearing loss (otitis media with effusion)
Recall 6-7-2018
Ear exam

Recall 12-7-2018
Hearing Examination (was conductive hearing loss) I couldn’t find what is the wrong with the
tympanic membrane! Examination, DDx to the patient

Feedback 13-12-2018 Ear pain : Pass


2 min outside: Young lady with 2nd time left ear pain.
Tasks: History, PE, Dx and DDS

Thinking outside: DDs of ear pain plus if there is deafness then add dds of ear pain, such as acute
otitis media, otitis externa, wax, injury, H. zoster vesicles. Trauma, intracranial mass etc.
Inside: I am so sorry to hear about your pain, do u need pain killer? Pt: No
Me: is it ok if I ask you few questions and look at your ear to help you better?
Pt: Ok.
Asked all questions of pain such as SOCRATES. Positive was 2nd time of left ear. Throbbing pain,
no discharge, no other positive findings about pain.
ME; any associated complaints such as hearing loss? Pt: yes, in left ear.
Me: any other symptoms such as nasal discharge? Throat pain, neck pain, headache, eye pain or
discharge? Pt: No.
Me: Pl tell me more about last ear pain attack. Pt: took medicine and gone.
I quickly asked about present medical history including medicines, past history and family
history.
I kept history short and sweet as there was a PE and DX and DDS.

PE: Me: I will look at your ears, feel and do some special tests, is that ok? Pt Ok.
Me: On inspection, (with comparing to right side) I do not see any redness, obvious discharge,
trauma, vesicles, battle signs or scars.
Feel: I palpate the right and left ear, no positive findings.
Hearing tests: Me: (rinne) this is a tuning fork. I am going to put behind your ear and once u
stop hearing pl let me know so I will bring it in front of ear and u need to let me know which one
u can hear better. Positive was left side BC>AC.
Then I did weber tests: positive was she can hear better on affected side. Classic left side
conductive deafness.

Now I am going to look inside of your ear, if you feel pain then pl let me know we will stop there.
On Dummy, I forgot to change the black tip and even I forgot to mention. Ideally u need to
choose the widest possible. First I hold the otoscope on right hand, then I quickly realised that
it’s a left ear. Then I changed to left hand.
Then I started to mention from outer ear, I do not see any vesicles blah blah blah, at this stage
examiner asked me to mention what I see in one sentences ( I think he was bored hearing
these). Positive finding was red tympanic membrane without perforation and discharge. I got
little bit suspicious as I thought it cannot be that simple case, so I look at attic, there were few
white line running throughout on TM (actually they were just the part of dummy, not a
cholestetoma), so I mentioned there could be a possibility of cholestetoma ( big blunder).

Dx: I think you have an inner ear infection we called acute otitis media, features are ear pain,
hearing impairment in repeated cases, perforation of ear drum and discharge.

Other less likely possibilities are cholestetoma, which is a skin growth in ear, I suspect this as
well due to some white lines in ear so we will keep an eye on it, wax (but I did not see any),
infection of outer ear ( but symptoms does not support it), otosclerosis which is a growth of
small bone in ear (less likely as it does not cause pain). Bell rang ( I could have add many DDs
but time finished)

Global score: 6 Key steps: all yes History: 5


Choice and technique of examination, organisation and sequence: 6
Accuracy of examination: 6 Dx/ DDs: 6
Case (11/2/2017)
Female with pain in left ear.
Task:
Take history,
do PE on patient and mannequin.
Explain diagnosis to patient.

This role player was kind of irritating. I thought I was asking questions politely and I wasn’t rude.
Anyways, she had this pain before, had been given antibiotics and the issue resolved but then
comes back. Has some hearing loss on left side. No discharge from ear. No foreign body into
ear. No sore throat etc.

Did whisper test, Rinne & Weber- Rinne negative on left side and Weber lateralised to Left side
so I said there is conductive hearing loss on left side.
Proceeded to do otoscopy on mannequin, told that I am going to insert this otoscope into your
ear to have a look. nothing in the external ear canal, are you ok? any pain? then saw the
tympanic membrane, it looked red & a little opaque to me (didn’t do too many otoscopies in my
work experience) and there was no perforation.
It didn’t look like the classic cholesteatoma which has been described in books.
THEREFORE, I told the patient that based on what she told me and what I got during
examination, it could be one of two things, something we call chronic otitis media or
cholesteatoma. I can't tell clearly which is it but since antibiotics helped the last two times, it
may be chronic otitis media.
And the bell rang.
Feedback: Ear pain, Pass, Global score: 5

Case (29/3/2017) WAX


Ear exam and otoscopy on a dummy. Tuning fork and whisper test exam on roleplayer.
Tasks: PE and Dx [Wax on otoscopy - conductive unilateral loss of hearing on the side of wax]

Case (29/3/2017)
Ear exam, deafness of left ear, specific tests and one special test
2 mins: ear exam with facial nerve exam
Hi, I am Dr. Maria. I am required to examine you. That would require me to ask you to do certain
manoeuvres. Is that all right? I will try to be as gentle as possible, but if at any time you feel
uncomfortable, please let me know. Before I begin, do you have any pain?
I washed my hands while saying all this and wore gloves.
Inspection: general appearance, both ears, mastoid process, parotid, pinna
Palpation: temp, tender, lymh nodes, mastoid process and pinna
Otoscopy: I was unable to turn it on and asked the examiner for help.
Only one half of the dummy was there
Pull the pinna lateral, backwards and upwards. Comment about the external canal, any wax,
tympanic membrane- perforation with some pearly flakes behind it (when I asked the other
candidates they did not see it, so check with others)
Rinnes and webers- conductive hearing loss
Whisper test: reduced on the left
Facial nerve tests: frowning, close eyes, blow air, smile.
Diagnosis: cholesteatoma with reasons- explained the results of the above tests
Dds- I forgot about the dds
Global score-3

Case (13/10/2017)
The young patient has come to your GP as he has ear pain for 2 days.
Task: HX, relevant examination, most likely diagnosis.
The Asian role player (RP) that I thought was medical student was there.
I started history.
- No discharge
- no loss of hearing
- obviously no trauma
- no swimming frequently
- no spinning around
- no dizziness
- occupation was secretor in bank.
- Previous infections? Yes he was treated with amoxicillin couple of time (I should have asked
about increase of the hearing in noisy area and noticing the TV sound and exposure to loud
music and…)
- No PMH.
As for PE,
having introduce myself as well as washing my hands,
- I started by inspection. No skin tag erythema discharge on helix antihelix tragus and pinna area.

- I started by palpating mastoid and tragus and drawing back the ear to look for any pain none
was there.
- Otoscopy exam: perforated ear drum+ erythematous and whitish materials in ear.
- Weber test: localized to left ear. Rhinne test was the same on both part and AC was better than
BC!!!!!!! I did check that double time with 2 different techniques but the result was the same.
- Facial nerve quick screening was NL no LAP.

I mentioned the most likely condition is chronic infection of your middle ear due to previous
infections that you have. it can be also Chlosteatoma which is again infection of the middle ear
with skin changes in the middle ear that can go to other structures in brain which is risky.

Feedback: Ear Pain, FAIL (G.S:3) !!! I thought definitely I passed the case!
Key steps:1 &2: Yes…but 3&4:No

History:4,
Choice and technique of examination and organization and sequence:4
Accuracy of examination:2
Commentary to examiner:4
Dx/DDx:1
Karin case Cholesteatoma
Management
- Most likely you have a condition called cholesteatoma and a conductive hearing loss. It is an
abnormal skin growth in the middle ear behind the ear drum. When this growth increases in
size, it destroys the delicate bones of the middle ear causing the hearing loss and when
infected, it can lead to ear pain and discharge.

- I will refer you to the ENT specialist for further treatment. You will undergo surgery with
removal of the mass. The specialist might decide to do a CT scan and audiogram (hearing test)
before deciding on surgery.

-Initial treatment will be careful cleaning of ear (ear toilet) and some topical antibiotic drugs
like ofloxacin.
The main aim of the surgery is to remove cholesteatoma and create a safe and dry middle ear.
- Advise on risk factors: smoking
- Later on, you can undergo a second surgery for reconstruction of damaged middle ear bones.
- This is potentially serious as it can lead to facial nerve injury and intracranial extension such
as brain abscess and meningitis.
- Review regularly every 3-6 months.
- Reading materials.

Notes Summary
there are 3 cases for ear examination
1-Choleasteatoma
2-otitis media with/without effusion
3- wax

Regarding history: is the presenting complaint is ear pain do like above. If hearing loss then start
from hearing loss questions then ask if any ear pain and continue like above
So pain questions + FDD, NVV+ WOOO MM ATP

Steps summary:
1-Consent: I need to examine your ears this would involve having a look, feel and using some
instruments would that be alright?
2-inspection or roleplayer:
-shape and deformity
-scars and swelling (always look fron and behind the ear)
-redness and discharge, vesicles
3-palpation on roleplayer
-tenderness
-swelling
-LN (pre and post auricular)
*If you want to pull the ear take consent I am just gonna pull your ear is that ok?
4-Otoscopy
-When you use otoscopy take consent: Now I’m just gonna uses this instrument called otoscope
to look inside your ears, checking the ear canal and the ear drum. I will be as gentle as possible if
you feel uncomfortable at any time please let me know.
-Inspect ear canal:
-redness, discharge, swelling, wax, foreign body. (in one case you will see lots of wax which will
be your most likely diagnosis as you can’t see the ear drum clearly).
-Inspect the eardrum:
-redness, bulging, retraction, perforation, light reflex.
(in one case there will be intense red TM +/- bulging with fluid behind it so your most likely
diagnosis will be otitis media with/without effusion; infection of the middle ear with pus
accumulating behind the ear drum)
(in another case the there will be perforation in the unsafe zone of ear drum with deposition of
whitish materials so cholesteatoma will be your most likely diagnosis; because of repeated ear
infection you have perforation of the ear drum that happens in the upper part of the drum or
the unsafe zone we call unsafe perforation. This lead to development of cholesteatoma, which is
an abnormal skin growth in the middle ear behind the eardrum. When this growth increases in
size, it can destroy the delicate bones of the middle ear causing the hearing loss and when
infected, it can lead to ear pain and discharge. I will refer you to the ENT specialist for further
treatment. You will undergo surgery with removal of the mass. The specialist might decide to do
a CT scan and audiogram (hearing test) before deciding on surgery.

5-hearing tests (there are different methods apart from the following)
-whisper test (60 cm behind the patient say and number 55 or 99 any you like)
I am just gonna stand behind you and whisper a number or a word. Is that ok. Please tell me
what I have said.
-tuning forks
When you use tuning fork: I am just gonna use these tuning forks to test your hearing.
choose 512 HZ
weber test (I am putting this on the middle of your forehead tell me if the buzzer is louder on
one side or the other side or in the middle.
Rhinee test (I am gonna place it once on a bony part behind your ear then in front of your ear
and tell me where does it sound louder?)

(I think In all cases there will be conductive hearing loss so whisper test positive on the left ear.
Weber lateralized towards the left ear. Rhinee; left ear bone conduction> air condution.

6-if you have time


-cervical and supraclavicular LN
-Cranial Nerve 7
-nose and throat
160-TIA lower limb
60-year-old man with transient weakness of his right leg, known history of diabetes and
hypertension. You already complete upper and lower limb sensory examination.
tasks
-lower limb neuro examination
-other relevant examination
-dx and DDx to patient

REMEMBER: GITPRCS

1-WIPE
-wash your hands
-Hi my name is ---I’m one of the doctors here. I can see from the notes that you have weakness
in your leg. Is that right?
-all right so in order to know more about the problem now I just need to start by examining your
lower limbs this would involve me having a look, feeling and moving your legs and asking you to
walk a bit. Would that be all right?

2-ask for pain


do you have any pain at the moment?

3-Vital signs
pulse for irregularity/ blood pressure for hypertension

4-Lower limbs (GITPRCS)


-Gait
-walking
-turning
-walking on toes
-walking on heels
-tandem gait
-Romberg test

.Now I’m just gonna assess the gait so if you could come of the bed and walk towards the wall
for me?
.And turn?
.Now walk on your toes? And on your heels? And now if you walk one foot in front of the other
like this?
.Perfect thank you
.I’m just gonna do a Romberg test to assess the balance so if you just stand and put your feet
together, your hands by your side. I am here to catch you if you feel unsteady. Now if you could
close your eyes for me. And now open them. Thank. You can lie down on the bed.
-Inspection (SWIFT)
-scars
-wasting
-involuntary movement
-fasciculation
-tremor

-Tone
-leg roll
-knee lift
-knee clonus
-ankle clonus

I’m just gonna check the tone of your legs now. Can you just let your legs go floppy for me?
I’m just gonna lift your leg
I’m just gonna move your ankles now. Could you just let your foot go floppy?

-Power (resisted movement) you can test both limbs together and compare
-hip movements
flexion (push up against my hands)
extension (push down against my hands)
abduction (push outwards against my hands)
adduction (push inward against my hands)

-Knee movements.
Extension (if you could bend your knees and now kick out against my hands)
flexion (push inwards inward the bottom against and don’t let me straighten your legs)

-Ankle movements
dorsiflexion (push up against my hands)
plantarflexion (push down against my hands)

-Subtalar joint movements


eversion (push outward against my hands)
inversion (push inward against my hands)

Note
L2= hip flexion
L3= knee extension
L4= ankle dorsiflexion
L5=toe extension
S1= ankle plantar flexion
S2=knee flexion
-Reflexes
-knee reflex
-ankle reflex
-plantar reflex

Now I’m just gonna test the reflexes this would involve me tapping with this tendon hammer on
parts of your leg. Is that ok?

Just let your leg go floppy for me?


just bend thi leg and flop it out to the side just gonna tap on the back of your ankles?
just gonna run the blunt end of this neurotip against the sole of your foot it might be a little
ticklish.

-Coordination
-heel shin test
-tapping

.I’m gonna test the coordination now.


if you could put this ankle on the other knee. Run it down your leg. Lift it off and get back to the
knee and keep going like this as quick as you can. Thank you

.tap your feet against my hands as quick as you can.

-Sensation (no need to do it because it is not a task however I just put it just in case)
-cotton wool
-pinprick
-vibration
-proprioception

.I want to test the sensation with this cotton wool it feels like this. Cam you close your eyes and
let me know when you feel it?
Now with this pin prick. It gonna feel sharp but won’t pierce the skin. It feels like this.
do you feel the same on both sides?

.now gonna tst the vibration sensation of your legs with this tuning for. It feels like this. Can you
close your eyes and say yes when you feel it and when it stops.

.gonna test the joint position. So with this big toe this is up and this is down.
can you close your eyes and let me know whether it is up or down.

Note
L2 = upper thigh
L3= medial side of knee
L4= medial side of leg
L5=medial side of foot (big toe)
S1= lateral side of little toe and sole of foot
Other relevant examination
1-auscultate for carotid bruit
2-cardiovascular exam
feeling for apex beat if displaced and auscultate heart sounds and murmurs
3- complete motor nerve examination of upper limbs and cranial nerve examination

Note/ in the exam all will be normal and no focus deficit

Dx and DDx
1-TIA: from history and examination you most likely have a condition we call transient ischemic
attack or mini stroke. This condition happens when the blood supply to your brain is blocked by a
clot and because this is a small clot it can resolve spontaneously and signs disappear within a
short time. There are certain risk factors associated with it like diabetes , high blood pressure…
other possibilities
2-complete stroke unlikely
3-Migraine
4-space occupying lesion
5-focal epilepsy

Feedback 19-7-2018
Pass ( ALL Key steps yes , score 7)

PE of TIA( very very long stem outside the elderly male DM/HTN brought by daughter because of
weakness of one side , now resolved .

U have already examined upper limb and lower limb sensory system now complete ur exam by
doing lower limb neuro and relevant exam.

Task PE, DX ,DDs

I started with wiper , did vitals( watch n bp app was there) than did lower limb neuro( ITPRC) and
the checked eyes(fundo) , auscultate carotid and heart.

Explained DX AND DDS.


161-Osteomyelitis/ septic arthritis
Feedback 22/2/2017
You are HMO in metropolitan ED, 5 year old boy is brought in by mom as he had knee pain (it
mentions clearly that pain is under the knee) for few days. He also had high fever (38 C) and
cannot walk at all. There was no history of fall or injury.
Other findings were given like no allergy, not on regular medications. growth and development is
normal.
Tasks
-Ask PE findings from examiner
-Explain diagnosis/ diagnoses to mom.
-Initial Mx with mom.

PE findings from examiner


-When I stepped into the room, there was an anxious mother who asked me “what happed to
my son, Dr”
Ok, I understand that u are concerned about ur son. Let me introduce myself first. My name is Dr
Khine and I am one of the Drs in this ED. We ll figure it out together what happed to ur son and
treat him accordingly. Ok? Now, I d like to ask my examiner for some findings and I ll get back to
u soon. (Sure)

-Examiner, I d like to give pain killer to my patient first and offer him wheelchair or crutches and
allowing him not to bear wt.

-Examiner said “Dr, all I can give you is only PE findings”

GA - Child is ill and toxic, skin survery for rashes - no

VS - T : 38 C, BP - 100/70 mmHg, other vitals – normal

Signs of dehydration - No,

BMI - normal

Focus examination of Lower limb (I d like to compare both limbs)


-Look - Any swelling, redness, signs of injury. Any abnormal groin crease (Examiner - redness +
around rt knee jt, no swelling, Lt knee Is normal)
-Feel - Temperature, tenderness over knee jt, patella, tibial tuberosity. (Examiner - tenderness
around knee jt, point tenderness is negative)
Move - Active and passive (Examiner - U can do full range of movement of knee jt)

-I d like to check hip jt, ankle jt, back examination and full neurological examination of Lower
limb (Examiner - they are all normal)
-CVS, Resp - normal
-Bedside tests - not available
Explain to mom
-Well, Jenny, according to HO and PE findings, most likely condition ur son is got (as I forgot to
ask son’s name) is septic arthritis. Have u ever heard of it? (RP waved her head)
Ok, Let me draw a diagram to explain u. (I drawed a diagram of knee jt with capsule)
This is thigh bone and this is shin bone. Septic arthritis is basically an infection of joint caused by
bug.
It can also be osteomyelitis, which is bug infection of bone and its inner cavity we call bone
marrow. (I scribbled on bone and its marrow)
-These two conditions are usually caused by a bug we call staph aureus.
-They are most likely conditions in his case and because of infection, ur son has got high fever
and pain which makes him unable to walk.
(I was not very sure, so I talked both Dx) So far, are u with me Jenny? (Yep Dr)
-Ok, there are also other possible things that can cause this condition, like injury to leg, viral
infection of joint, Osgood schlatter ds (RP : what’s that?) It is inflammation of this bony tubercle
(I pointed to tibial tuberosity). But they are unlikely.

-RP asked me “What is he actually having Dr?” At this point of time, most likely cause is septic
arthritis or osteomyelitis. (RP : Ok, how do u treat him?)

-Alright, (I looked at the stem, I am HMO in ED), now I am gonna admit ur child coz this is serious
infection and we need to fix it immediately.
-In the hospital, child specialist and bone specialist will come and look at him.
-We ll do some blood tests like basic blood check, chemicals in his blood, inflammatory markers,
renal function and liver function. Considering his high temperature, we ll do culture of his blood
to find of what bug is causing this.
-Specialist might consider taking out some fluid sample from his jt and examined it under
microscope we call jt aspiration. And if it is needed, Xray and further imaging like MRI will be
done.

-In terms of treatment, we ll give him antibiotics through his vein which he may need to receive
it for a couple of weeks probably 2-3 weeks. If he improves, AB will be changed to oral one. We ll
continuously monitor his condition during his hospital stay.
-We ll make sure to give him enough pain killer and Panadol to lower down his fever (RP said
thank you Dr)
Do u understand my explanation? (yes dr)
Ok, Jenny. Let me reassure u that though this is very serious condition, u have brought him in
time which I really appreciate it (I saw some facial grimace in her face) and this is well
manageable condition with antibiotics. If it is properly treated, he wont have any residual
disability and he can walk again normally after his recovery. (She seemed happy with my
explanation) Then the bell ranged.

AMC Feedback - Painful leg : Pass (Global Score - 6)


Approach to relative - 6
Examination - 6
Dx/ Dx - 6
Management plan - 6
Recall of 13-7-2018
5 yo boy, knee pain, cannot walk, fever, PE positive for osteomyelitis

Feedback 21-4-2018
5Y old Boy brought in by the mother due to pain in the right lower limb since yesturday.
Tasks
Hx from the mother for 2 mints
Tasks examination findings from the examiner
Diagnosis and differentials to mom
Management to mom

- Fever present , He plays a lot outside but has no scratch marks or bruises. Not sure about falls , He's not
able to bear weight for the past one day

– point tenderness over the upper tibia , Febrile , unable to weight bear , no bite marks , UA negative

– Most likely Osteomyelitis , Others -Cellulitis , Septic arthritis in knee joint unlikely , Juvenile rheumatoid
arthritis ( unlikely due to no joint involvement)

– Non weight bearing , Bloods , IV cannula with ABx , Analgesics, Send to ED now for Paed Ortho review ,
Social worker arrangement for household activities

Global score - 5 ( Approach 5, Examination 5 , DDs – 5 , Management – 5)

My comment - if not able to weight bear , then its critical – Septic arthritis , OM, Perthers , SUFE
possible

Feedback 21-4-2018
Osteomyelitis- pain in the right lower limb. Of a 5 year old boy along with fever. He plays a lot outside but
has no scratch marks or bruises. He's not able to bear weight for the past one day
Tasks examination findings from the examiner. Diagnosis and differentials to mom. Management to mom.
Station 1 Painful leg
Paeds Osteomyeltis
Hx: As per Karen
Key point in PE: any point tenderness for OM, ROM of joint in septic arthritis, previous URTI in transient
synovitis
Mx: admission, FBE blood culture CRP Xray, IV abx, nonweight bearing

Dx: osteromyelitis; DDx: septic arthritis, fracture, cellulitis

I also reassured parent the outcome was usually very good, then parent appeared relieved and happy.
Approach to pt: 5
Choice and technique of exam, organisation and sequence: 5
Dx/DDx:6
Mx: 5

Global Score: 5 pass


Feedback 7-12-2018
6yrs old boy complaining of limping since 2 days. no fall , slight fever, no past history of joint problem .

TASKS:
history from mother,
pefe from examiner,
dd,,
investigations.

2 min Thinking : need to be empathetic, AS GP need to refer though not task, all dd .mother name.

i entered greeted both of them, mother was bit tensed counselled her, history started 2 days back pain
increasing, no swelling, no redness, she can't remember any injury, slight fever. trying to walk but painful,
no vomiting , not dull, urine -not coloured, no tummy pain, bowels normal
past history :not significant
family history :not significant
BINDS :normal no smoking in family (don't know why i asked this )

pefe: need to ask one at a time GA- dull, not lethargic,


VITALS -temp 38.5, pr-bit high. PICKLE-noraml. chest and heart - auscultation normal .
Inspection of LOWER limb -comparing with opposite side -all normal (knee,hip, ankle) no redness no
swelling , no deformity or any protected posture.
palpation- tenderness of tibial tuberosity . movements - all normal .
UDT &BSL-normal

DD- explained it could be infection of bone what we call osteomyelitis, other could be infection of joint
space-but unlikely, irritable hip, injury-fracture-can't say untill xray . ,HSP,referred pain,
invest- ill refer to hospital , he will be seen by children doctor and will arrange investi like blood test to look
for infection and X ray to see any bone injury ultrasound to see joint space, aspiration if needed (will be
decidec by specialist , bone scan to confirm.
scenario :leg pain Global 5(history 5, choice and tech of examination 5, dd-5, choice if investigation -4)
Feedback Station 12: Leg Pain - PASS
A 5 year old boy was just brought in to the ED where you are working as EMO with the complaint of
limping.

Task: History
PEFE
Provisional Dx/Diagnoses
Mx

2 min thinking: Wth is diagnosis/diagnoses? Do I give one dx or a tons of ddx?

History
-Intro
-Limping – duration (forgot); onset (sudden); 1st time (yes); do you think the child is limping because of the
pain? (maybe cuz he kept pointing his inner-side of the knee) cry a lot especially when that are is touched?
(yes) any redness? Swelling? (no)
-DDx – any injury recently? (nil); any fever these days? (forgot); recently has he caught a cold? (no); home
situation? (harmonious family; no paticular issues going on); any bruises over the body? Any pre-existing
history of bleeding disorder? (no)
-BINDS (immunization is updated); GH (good)

PEFE
GA + Vitals (Fever present with around 38.5);
Leg examination – inspection (no redness; no swelling; no bruises over the legs and knee); palpation
(localised tenderness on medial side of below-knee); movement of knee joint – (normal); hip and ankle
(normal);

Dx/DDx
Here, I just gave osteomyelitis and septic arthritis as my dx
(other ddx should you want to mention include – perthes; irritable hip; juvenile arthritis; haemoarthrosis;
bone problems like osgood schlatter although the patient is young; non-accidental injury)

Mx
Admit; inform
Blood tests – FBE; Blood culture and sensitivity;
USG joint and bone
Aspiration
After these tests – antibiotics

GS – 4
Key steps 1, 2, 3 – all YES
History – 3
Choice and technique of examination, organisation and sequence – 4
Dx/DDx – 5
Choice of investigations – 4
Note/ there are two cases of the same scenario 5 year old boy presented to ED with right lower
limb pain since yesterday
the first is without history and the other is with history ….However the approach will be the
same.
these feedback mentioned above are very good to follow.

In summary
1-regarding history you can follow the same approach of pain questions then HOST-M then past
then BINDS
**here fever will be positive so you need to ask: how high is it? If it is constant or come and go?
If associated with chills and night sweating?
and as there is fever you need to ask dehydration questions like if look drowsy, loss of appetite,
urine output.
then continue for tummy pain, rash, recent infection, trauma, joint swelling, LOW and lumps
ask about contact and family history.
keep asking till the 2 minutes hx finished

2-regarding examination
the same approach as other lectures but add ENT
here the positive findings is in the knee not hip and there will be point tenderness, redness,
warmth. No swelling.

3-Diagnosis (here is the confusion in this case because of the first feedback but I think the
most likely is osteomyelitis)
most likely is osteomyelitis or infection of the bone by a bug a bacteria from blood stream
causing fever, and pain etc…

Differential diagnosis are


-septic arthritis (I will not say unlikely or less likely just could be and need to rule out )
-cellultis
-trauma

4-Treatment
-admission to be seen by child and bone specialist
-Ix (FBC, ESR/CRP, UCE, LFT) blood culture, x-ray, specialist might do joint aspiration as washout if
fluid is milky and suggest pus.
-antibiotics ( IV flucloxacillin till symptoms improved, no temperature and no CRP then oral for 3-
6 weeks need to be taken regularly and correct dose.
-Painkillers (to reduce pain and fever)
-non weight bearing so will rest and put splint to minimise pain and deformity.
-reassurance in safe hand
162-Knee examination
Case 1 (Osgood schlatter disease AMC exam case)
1-16 year old boy presents with bilateral knee pain on playing football. Training to join
professional team soon.
Tasks
-Perform PE
-Diagnosis and ddx
-advise management.
2-17 year old boy complaining of bilateral knee pain, doing a lot of exercise, pain on climbing
stairs..etc long stem.
Tasks
-Perform examination
-explain diagnosis to patient

Examination steps
1-Gait (normal)
2-Look (normal)
-front scar, skin colour changes, swelling, deformity, wasting.
-lateral: fixed flexion deformity
-back: popliteal fossa swelling

3-Feel (lying)
-temperature (Normal)
-quadriceps muscle bulk for wasting. (Normal)
-Tenderness (feel for tenderness with knee in semi-flexed position) (the most important step).
quadriceps tendon
 patella (if tender here mean chodromalacia patella)
 patellar tendon (if tender means patellar tendonitis)
 Tibial tuberosity (if tender mean Osgood schlatter)
fibula head and neck (if tender mean fracture)
joint line (if tender mean osteoarthritis)
popliteal fossa.
-Knee Effusion (normal)
Bulge sign (sweeping test)
patellar tap test

4-move (active and passive; extension and flexion) (normal)


-active movements: look for limitation, excessive movement
-passive movements: look for limitation, crepitus

5-special tests
-Anterior drawer test (ACL injury) -posterior drawer test (PCL injury)
-valgus stress test (MCL injury) -Varus stress test (LCL injury)
-appley’s grinding test (meniscal tear) -clerk test (chondromalacia patella)

6-complete with joint above and below


Diagnosis and ddx
-you most likely have a condition called Osgood schlatter. This is a painful knee condition
common in young athlete and not serious.
-often occurring during a growth spurt associated with physical activity.
-it is thought that the tendon attaching the muscles at the front of the thigh to shin bone
become tight and causing inflammation and micro fracture of the Tibial tuberosity (the bony
bump in the shin bone.
-the condition usually resolved by itself by time (6-18 months).
-other possibilities are osteomyelitis but no fever, septic arthritis but no joint swelling, trauma
but no signs of trauma, meniscal or collateral ligaments tear which are a supportive structure
around knee joint but unlikely from examination.

Management
-first need to arrange x-ray to rule out other possibilities and confirm the diagnosis. Also FBC to
rule out osteomyelitis.
-you need to avoid activities and sports for at least a couple of weeks or till pain free followed by
gradual return to activities guided by symptoms level.
-Give you painkillers
-apply ice to decrease local pain and swelling
-physio (strengthening and stretching exercises)
-the knee may remain uncomfortable until growth finishes.

Case (8/3/2017) (9/3/2017)


case 1/ young teen with bilateral knee pain..
perfom PE ( Osgood Schlatter’s, pain in tibial tubercles bilat )
OSD. Positive findings: bilateral tenderness on palpation of tibial tuberosity r>l; bilateral
crepitations on passive ROM.

Case 3/ young teen with bilateral knee pain..perfom examination..


-Its Osgood schlatters. Teenager with tibial tuberosity tenderness. -Knee case- 17 years old plays
soccer. Has started to get below knee cap pain. She said on the right knee. It was my first station.
- I asked her to walk and assessed gait- normal
- Did flexion/extension all good
- Drawer test/ Collateral ligament test/ Apleys grinding - all normal
-Patellar tap test, swipe test all normal
-Then I kept my finger on her tibial tubercle and specifically asked if that was tender.... the role
player started looking at the examiner... I asked twice and she stared at the examiner... then
examiner said yes it's tender
-Patellar tendinitis is in basketball players not soccer -Soccer player usually have chondromalacia
Feedback 11-12-2018
Young 17 year old boy complains of knee pain. He’s a soccer player at school and has been
practicing for the soccer championship. He has this knee pain on and off. Now recently he has
knee pain again especially on running, jumping and passing the soccer ball.
Tasks-
1. Perform knee examination with running commentary
2.explain diagnosis and give differential diagnosis
Try to do all examination in LOOK , FEEL , MOVE PATTERN EXCEPT THYROID AND LUNGS
EXAMINATION
I did first GA, GAIT, LOOK, FEEL JOINT, MOVEMENTS
Postive findings- tenderness on patellar tendon and tibial tuberosity.
SPECIAL TESTS- When I m going to check for cruciate ligaments , examiner said its negative, then
I went for grind test for meniscus.. she said negative, then I did Clarke test which was negative ,
tibial tuberosity tender on both knees, I did a test for patella tendon also which is given in
murtagh.
DX- PATELLAR TENDINITIS
DDX- OSGOOD SCHLATTER DISEASE, CHONDROMALACIA PATTELA, cruciate ligaments injury,
meniscus injury
Global score -5,key steps 4/5 covered

Feedback 11-12-2018 (Knee pain) Global score 5 Pass


17-year old male complaining of knee pain. The pain is on and off but it has been getting worse
because he has been training harder for a soccer championship. It is worse when he runs or
jumps.
Task: Perform PE and explain DDX.
I had the same case on my first exam attempt in nov 2017.

I got in the room and greeted the examiner and the patient. He was sit on the chair by the table
wearing shorts.
I introduced myself and explained him what I was going to do: “ My task here today is to perform
a PE on you. It will involve me taking a look, touching your legs and perform some special tests. I
will be very gentle. If you feel any discomfort let me know and I will stop. Is that ok for you?”
Patient consented.
“ I will be giving running commentaries to the examiner. If you feel like asking something, feel
free to interrupt me.”
I washed my hands and asked patient to stand up, and offered help. I asked if he was in pain at
the moment and he denied.
Appearance: no mobility aid. Standing upright.
I commented: “ I see no scars, swelling, erythema and asymmetry. No genu valgus, no genu
varus and no quadriceps wasting.
I moved posteriorly and commented: no scars, no asymmetry and no Baker’s cyst.
Than moved laterally and commented: no genu recarvatum.
I asked patient to walk towards the bed and commented on the gait: normal heel strike, not
antalgic, no limping.
I asked patient to lay down in bed and offered him help.
I felt for temperature and pulse.
Using both hands and one knee at a time, I palpated the quadriceps, suprapatellar pouch looking
for effusion, tested for patellar fluctuation, did the bulge test. I palpated the head of the fibula
and the tibial tubercle (patient complained of pain). I did the Clark test and patellar
apprehension test.
Than I checked the active movements: flexion, extension, internal rotation and external rotation.
I performed the anterior drawer test (cruciate ligament testing), the LCL and MCL testing. I asked
the patient to lay on his tummy and performed the grinder test(meniscus exam).
I thanked the patient and washed my hands again.
I explained the patient: “Most likely like you have a condition called Osgood Schlatter Disease. It
is not uncommon and it is not a serious condition. It happens because in your age bones grow
faster than tendons making tendons to pull out the tuberosity. That’s why you have pain when I
palpate this area.
It could be due to other reasons like patelo-femoral syndrome, osteomyelitis, septic arthritis,
fracture and meniscal injuries.” (for each of them I gave reasons why I thought it wasn’t that).
The bell rang patient seemed convinced and the examiner had a poker face.

Key step 1:yes. Approach to patient/relative: 5


Key step 2:yes. Choice & Technique of exam, organization and sequence: 5
Key step 3:yes. Diagnosis/ Differential diagnoses: 4
Key step 4:yes. Accuracy of Examination: 5
Key step 5:yes

Case 2 (Knee patellofemoral syndrome; chondromalacia patella AMC exam case)


Case (28/4/2017)
case 1/ Knee pain 1 month, about 18 yrs old. Task, examination of knee and explanation of
diagnosis to patient. Clarke's test positive Lt>Rt.

Case 2/ knee pain


Teen age boy at Gp with both knee pain.c hondromalacia patella case.simple examination,tell
diagnosis and management.on examination patelofemoral compression test positive.v
cooperative role player and examiner. simple performed examination.
Pass

Examination steps
1-Gait (normal)
2-Look (normal)
-front scar, skin colour changes, swelling, deformity, wasting.
-lateral: fixed flexion deformity
-back: popliteal fossa swelling

3-Feel (lying)
-temperature (Normal)
-quadriceps muscle bulk for wasting. (Normal)
-Tenderness (feel for tenderness with knee in semi-flexed position) (the most important step).
quadriceps tendon
 patella (if tender here mean chodromalacia patella)
 patellar tendon (if tender means patellar tendonitis) (this may or may not be positive)
 Tibial tuberosity (if tender mean Osgood schlatter)
fibula head and neck (if tender mean fracture)
joint line (if tender mean osteoarthritis)
popliteal fossa.
-Knee Effusion (normal)
Bulge sign (sweeping test)
patellar tap test

4-move (active and passive; extension and flexion) (normal)


-active movements: look for limitation, excessive movement
-passive movements: look for limitation, crepitus

5-special tests
-Anterior drawer test (ACL injury) -posterior drawer test (PCL injury)
-valgus stress test (MCL injury) -Varus stress test (LCL injury)
-appley’s grinding test (meniscal tear) -clerk test (chondromalacia patella) (+ve)

6-complete with joint above and below

Diagnosis/ ddx and treatment- see patellofemoral syndrome lecture.

Case 3 (Knee osteoarthritis AMC exam)


52-year-old with knee pain. No history of trauma. Cannot remember the stem much.
Tasks
-Examine the knee giving a commentary to examiner
-Diagnosis and differentials to patient

Differential diagnosis
1-osteoarthritis (most likely)
2-rhumatoid arthritis
3-gout
4-psoriatic arthritis
5-trauma

Examination steps
1-Gait (+ve antalgic gaitl)
2-Look (normal)
-front scar, skin colour changes, swelling, deformity, wasting.
-lateral: fixed flexion deformity
-back: popliteal fossa swelling

3-Feel (lying)
-temperature (Normal)
-quadriceps muscle bulk for wasting. (Normal)
-Tenderness (feel for tenderness with knee in semi-flexed position) (the most important step).
quadriceps tendon
 patella (if tender here mean chodromalacia patella)
 patellar tendon (if tender means patellar tendonitis) (this may or may not be positive)
 Tibial tuberosity (if tender mean Osgood schlatter)
fibula head and neck (if tender mean fracture)
joint line (if tender mean osteoarthritis)
popliteal fossa.
-Knee Effusion (normal)
Bulge sign (sweeping test)
patellar tap test

4-move (active and passive; extension and flexion) (may be limited knee flexion)
-active movements: look for limitation, excessive movement
-passive movements: look for limitation, crepitus

5-special tests
-Anterior drawer test (ACL injury) -posterior drawer test (PCL injury)
-valgus stress test (MCL injury) -Varus stress test (LCL injury)
-appley’s grinding test (meniscal tear) -clerk test (chondromalacia patella)

6-complete with joint above and below

Diagnosis and differentia diagnosis


-most likely have osteoarthritis of knee joint which is a degenerative condition or wear and tear
type of arthritis that occur most often in people 50 years of age and older.
-the cartilage in knee gradually wears away, it becomes rough and the protective space between
the bone decreases, so resulting in bone rubbing and produce more painful bone spurs.

-other possibilities like above…

Management
1-run Investigations like FBE, ESR/CRP, X-ray
2-rest (acute phase)
3-painkiller
4-lifestyle advice
5-physio
6-occupational therapist

Case (4/10/2017)
Osteoarthritis of knee – Examination – Pass
52-year-old with knee pain. No history of trauma. Cannot remember the stem much.
Tasks
Examine the knee giving a commentary to examiner
Diagnosis and differentials to patient
Patient was standing in the room. So, started with gait which was normal but patient complained
of pain while walking.
Did the knee exam as in the Geekimedics youtube video.
Positive findings:
On inspection he had a red colour very minor skin rash which I mentioned just to show I am a
very keen observer �, but on palpation when he complained medial and lateral joint line
tenderness, I looked more closely and found soft swellings on either side of right knee- left knee
normal… so I admitted my mistake and commented on this.
No other positive findings. Special tests also negative but tenderness was quite generalised in
the joint.

Diagnosis/ DD
Osteoarthritis is most probable diagnosis. Told him it’s a degenerative condition……
Gave gout, pseudo gout, RA, patellofemoral pain syndrome, septic arthritis (said this is very
unlikely) as DD.
All 5 key steps covered.

Case 4 (medial meniscus tear Handbook case 53)


Examination steps
1-gait (painful)
2-look
3-feel (joint line tenderness +ve)
4-move (passive: limited extension)
5-special tests (apleys grinding test +ve)

163-Atrophic vaginitis/ Endometrial hyperplasia


Sample case/ 62 year old Mary, presents to your GP, with complaints of vaginal bleeding since
the last 2 days. She does not have any other medical or surgical illness and is not on any
medication.
TASKS
-Focused, relevant history
-Examination findings from examiner
-Order relevant investigations
-Further management of this patient.

Differential Diagnosis
1-Atrophic vaginitis 2-Endometrial hyperplasia
3-Endometrial or cervical polyps 4-Endometrial or cervical cancer

History
1-bleeding questions
-Is the patient hemodynamically stable?
-How long has your bleeding been going on? (2 days)
-How many pads have you used so far? Are they fully soaked (2 pads)
-What is the colour of the bleed? Are there any clots?
-Is it smelly?
-has this happened before?
2-Associated symptoms/ DDX
-Do you have any abdominal pain, fever, nausea and vomiting?
-do you feel dizzy or tired at the moment? (Anaemia)
-any dryness from down below? Any burning or itching sensation down below? Any abnormal
discharge? (Atrophic vaginitis)
-any bulging from down below? (Polyps)
-any weight loss? Any lumps or bumps around the body? (Cancer)
-any bleeding disorder? Do you take any blood thinner medications?
-are you aware of your blood group?
3-5Ps
Menopause (instead of period)
-When did you have your menopause?
-do you experience any hot flashes, mood swings, heavy sweating? Do you experience burning
or tingling when you pass urine? Have you experienced any incontinence? Do you open your
bowels regularly?
Sexual (partner)
-are you sexually active?
-Do you experience painful intercourse? (Atrophic vaginitis)
-Any bleed after intercourse? (Cervical cancer)
Pregnancy
-how many pregnancies have you had?
HRT (instead of pill)
-Are you on any hormone replacement therapy? (Can predispose to endometrial hyperplasia)
PAP and Mammogram
- Have you done your pap smear? What was the result? Mamogram?
Physical Examination
1-General appearance: BMI + PODL
-BMI, pallor, lymph node enlargements, dehydration, oedema
2-Vitals: BP with postural drop, temperature
3-CVS, R/S, CNS
4-Abdomen: Visible distention, mass, tenderness (abdomen is soft and non-tender)
5-Pelvic exam
Inspection of the vulva and vagina:
-bleeding, discharge, rash, vesicles
Speculum:
-does the cervix appear healthy?
-Any bleeding from the cervix?
-vaginal wall (vagina is thin, dry and atrophic with diffuse erythema)
Bimannual examination
-uterine size and tenderness of the uterus, adnexal mass and tenderness (uterine size is normal,
no tenderness)
6-Office tests:
UDT, BSL

Investigation
1-FBE, UEC
2-Transvaginal ultrasound and diagnostic pap
-Thickness of endometrium (in postmenopausal women, thickness should be less than 5mm)
-Polyps in the endometrium or cervix
-Any other abnormal growths

Management
Atrophic vaginitis
-From the history and examination, most likely you are having atrophic vaginitis. It is a condition
where the lining of the vagina becomes thin and dry, and breaks down at times, leading to a
bleed. Normally, the lining of the vagina is dependent on estrogen for strength and integrity. At
time of menopause, the ovaries shut down completely so that only very low levels of estrogen
are formed, so when the estrogen levels go low, the vagina becomes thin and atrophic.

-You can use local estrogen therapy in the form of vaginal estrogen cream, vaginal estrogen
tablets (Vagifen), or pessaries. You can also use vaginal lubricants, which is also effective to
control dryness especially during intercourse.

Should I be put on hormones doctor?


For atrophic vaginitis alone, HRT is not indicated. This condition can be well treated by local
therapy but it might take some time to act. However if you have other symptoms of menopause
like hot flushes or sweating then you need to be started on it.as HRT can carry some risk to you.

Reading material of atrophic vaginitis


review in 2 weeks’ time
red flags
Endometrial hyperplasia
-From the history and examination, most likely you are having endometrial hyperplasia.
-The endometrium is the normal inner lining of your womb. After menopause, the endometrial
thickness should be less than 5mm. But in your case, the thickness has gone up to 7mm. When
there is an increase in the endometrial thickness, it is called endometrial hyperplasia.
-It is due to an unopposed stimulation of the endometrium by estrogen in the absence of
progesterone. After menopause, the ovaries shut down completely, so that only very low levels
of estrogen are formed, with no progesterone at all. However, estrogen is still being produced in
the body through peripheral conversion of fat tissue and this estrogen causes the stimulation of
the endometrium.
-In endometrial hyperplasia, the endometrium becomes quite unstable and breaks down at
times leading to bleeding. The other complication of endometrial hyperplasia is endometrial
cancer which could happen in around 7% of the patients.

-I will refer you to the specialist who will do a hysteroscopy and endometrium tissue sample
-once sampling is done treatment will be given depending on the type of endometrial
hyperplasia so if it is typical, you will be given progestogens either orally or an intrauterine
contraceptive device like Mirena will be placed. If the tissue sample shows atypical changes, the
specialist may decide to do a hysterectomy or removal of the uterus.

-I will give you reading materials regarding endometrial hyperplasia for further insight.
-I will arrange a review with you after the specialist consult.
-Life style modification and dietician if BMI is high

notes/
I think. There are 2 presentations for atrophic vaginitis case the first one is postmenopausal
bleeding for 2 days.
The other would be a persistent vaginal discharge for 3 weeks
History
-You will do the same history like bleeding or discharge no difference then same differential and
5Ps or general.
-in post menopausal bleeding the patient will say it is brownish discharge in lay term I think. And
her husband will be dead, if so no need to ask pain or bleeding after intercourse. Also she might
not done pap or mamo so arrange one if so.
-in the case of discharge 3 weeks as k importantly about the colour.
Examination
-for both there should be atrophic vagina, vaginal discharge or bleeding according to the case.
-there might be high blood pressure as incidental findings so again do LSM and arrange
consultation if so. But be careful they might be wrong with this feedback.
Management
-for both cases talk about atrophic vaginitis.
-in sample case they put Ix before discussion because the tasks are different. In the exam after
talking arrange Ix especially to rule out endometrial cancer.

Case (4/10/2017)
62 year old F with vaginal bleeding
- atrophic vaginitis
Vaginal bleeding – Pass
Scenario
Post-menopausal bleeding 2 days.
•History
•Focused examination from examiner
•Explain most probable diagnosis to patient.

C/O brownish discharge 2 days.


Menopause 15 years before no bleeding till 2days back. Used 2 pads so far. No abdominal pain.
Last pap smear 2 years back. (Told her we’ll have to do it today.)
Not sexually active for years. No previous vaginal discharge. No fever/ LOA/LOW
Waterworks and bowel habits normal.
Never been on HRT. No menopausal symptoms. Never had breast lumps/surgery.
Assessed for risk factors of endometrial ca. 3 children. No family history of endometrial (womb)
or colon cancer. Not taking any meds. (esp. tamoxifen), no evidence of PCOS in the past
history. Not used OCP…
SAD-no

PEFE:
All normal except blood in vagina/ not coming from OS. Atrophic vagina. Said I will do a PAP
smear during speculum exam. When asked for BMI, examiner said she is as you see her -role
player was quite lean.

Diagnosis:
Atrophic vaginitis is most probable diagnosis. Explained the effect of estrogen on integrity of
epithelium………. And explained her examination findings…
At her age, endometrial carcinoma is the most important diagnosis we want to rule out with
this presentation. (she got anxious- so added as she does not have any of the risk factors apart
from being post-menopausal and age, so atrophic vaginitis is the most probable), but cancer
should be excluded with a transvaginal USS, and PAP smear test before giving her above
diagnosis.

• 4/4 key steps covered.

Feedback comment
I had this case in June. I passed it too. In the history, you have to rule out Endometrial/Cervical
cancer as the presenting complaint is brownish discharge in a post menopausal woman.
Ask detailed menopausal history about last menstrual period, any sexual problems (sexual
history, my patient's husband has passed away 10 years ago and she said she wasn't sexually
active then), vaginal dryness, emotional problems.
Then ask Cancer questions, like weight loss, appetite, night sweats, lumps or bumps.
(My task was history, ask examiner for phys exam findings and most likely diagnosis and DD)
Ask about her general health. When was her last Pap smear and mammogram? (This is very
important, my patient was due for a Pap smear, so I told her that I will perform one now and
also refer her for a mammogram).
Then asked the Phys Exam findings, it was consistent with atrophic vaginitis. I told her it is likely
to be atrophic vaginitis, but just to be sure I'd like to run a few more tests like an ultrasound to
see how thick her uterus lining is. (It wasn't necessary for the task)

My patient had an incidental high BP reading.


So I asked her if she is taking BP meds? She said no, she wasn't aware, so I told her that I will
kee an eye on her BP readings and if it was consistently high I will consider giving her
medications in the future.

Feedback 20-7-2018 (important)


Post menopausal vaginal discharge
Pass , Global score 4
key step 1, 3, 4 Yes
key step 2 No
approach to pt 4
history 4
choice and technique of examiantion , organisation and sequence 5
Dx/Dx 5
Mx plan 4

62 yr old femal , persistent vaginal discharge ,


History , PEFE , DX , Mx plan
history
discharge .. yellowish or brownish vaginal discharge for 3 wks , no foul smelling , no blood ,
no need to use pap, not itchy , just irritation , especially when she passed urine , no rash in
her down below , no swelling from her down below, no urinary symptoms , no abdominal
pain , no fever , no LOW , no LOA , no lumps and bumps in the body ,
period last time at age of 52 , no menopausal symp , not on HRT , has 3 children , normal
vaginal delivery , pap smear 18 mths ago , ( result were good) , mamogram done , good ,
husband died? , not sexually active , no PMH or PSH , no family history of gynaecology
cancer , SADMA not relevant
PEFE
BMI as you see , normal
vital sign BP 140/90, PR 80 min , RR 20 min , temp normal
no pallor , no jaundice
CVS normal
REsp normal
abdomen normal
pelvic inspection - no discharge , no mass , coughing ... no urine leakage , no prolapse
SS atrophic changes + , os normal
BME normal
urine dipstick , BSL not available

Explanation
Julia , based on the examination finding and history taking , most likely the cause of vaginal
discharge is due to a condition called atrophic vaginitis , ,which is very common in women
after menopause , causing thining and dryness of vagina due to lack of female hormone after
menopause .that can lead to irritation easily and discharge as well. this is not serious
condition . however , whenever a women in her post menopause has discharge we should also
consider and rule out other condition , like nasty conditon of neck of womb ( CA cervix )
nasty condition of womb ( CA endometrium ) , even tho it is less likely , infection of womb is
unlikely as you are not sexually active ,
so i would like to refer you to specialist , who will do further assessment , take swab for
discharge , doing usg to your belly to check thickness of the womb and pap smear , if
anything suspicious , biopsy ,
i will prescribe lubricant and oestrogen cream to apply in your down below to relieve the
condition , and will review you regularly .
as your blood pressure is a bit high side , i will make another appointment to recheck your BP
as well

Recall 12-4-2018
a 67- year old lady comes with vaginal discharge from 2 m ago.
Tasks:
* further Hx
* ask p/e from examiner especificly
* mx
during hx these were found:
husband died 6 years ago , her LMP was 15 years ago , no any bleeding
befor, was on pills and HRT for 2 years , no any other symptom, no
itchiness, no wt loss.
not sexualy active, last pap was 2 years ago and was nl. not smoker.
in p/e:
just vagna was thin and cervix was pale and dry. no any other symptoms.
Feedback 31-5-2018
8)atrophic vaginitis,GP,57 yr old woman with one day blood stained vaginal
discharge.vitals r Normal
Tasks- H/O,PEFE,dx and ddx
History-one day bleeding ,just pink spotting on the pad.no discharge,no itchiness,no
smell,no urinary symptoms no trauma,no abdominal pain.
No menopausal symptoms,no HRT,husband died 6 yr ago.no sexual activity since
there.Cervical screening test normal,mammogram normal.no LoW,LOA, No family
history of cancer
PEFE-typical atrophic vaginitis feature was given by examiner.
Explained-by drawing ,other ddx endometrial hyperplasia,gynecological
cancers,infection all less likely.
Keys 3/4 approach 3,examination 4,dx and ddx 4 ,global 4

Feedback 26-10-2018
Station 5: Vaginal bleeding Fail (GS - 3)
Post-menopausal woman with Hx of vaginal discharge.
Hx, PEFE, Dx & DDx

Hx:

 discharge (2-3 days, colour is period-like blood), other symptoms (fever, abd pain, no features of
UTI, no features of Cancer)
 menopause 15 yrs ago. no bleeding since then. No other menopausal symptoms, no HRT
 Sexual Hx – not active, (skip dyspareunia), STI screening (not done)
 Screening (did Pap smear 2 years ago), mammogram
 SADMA

PEFE:

GA, VS, abdomen exam (Normal),

Pelvic exam: normal on inspection (bleeding ??)

Speculum exam: atrophic changes in vagina, bleeding on areas where speculum is touched

Bimanual exam: uterus and adnexa – normal size and no TDN

Dx & DDX:

Explained Atropic vaginitis as my main Dx.

DDx: infection (candidiasis, Bacterial vaginitis, Trichomoniasis), Ca cervix and Ca endometrium

(patient looked worried as I mentioned cancer. But I reassured her that those are less likely as she was
healthy and she had no features of cancer. we just want to rule out cancer by doing some basic screening.)
Feedback 8-11-2018
Endometrial hyperplasia feedback
Feedback 21-6-2018
A postmenopausal lady, (maybe 57-year-old) has come to see you for vaginal bleeding. Her
Ultrasound examination report is provided.
Tasks
-take history
-Explain the results to her.

(endometrial hyperplasia Ultrasonography 7-mms) I guess there was also talking about
management as well. Vaginal Bleeding:
Pass: Global score 5

I entered the room, introduce and greeted the role player. Took a history regarding her bleeding.
Asked about characteristics of bleeding. Any concurrent pain, or vaginal discharge. Also about
the time of menopause, contraception, HRT, osteoporosis, or any bone pain or fracture, also
sweating, sleep disturbance or mood change. And also asked about her BMI, recent weight gain,
balanced diet and routine exercise.
Then explained about menopause, hormonal change, and unopposed oestrogen secretion in
menopause especially among the overweight women, and also about endometrial hyperplasia
and risk of cancer. Explained that this sort of bleeding is not a definitive sign of endometrial
cancer, however, as there is a risk of malignancy, it is wise to do more investigation on her by
endometrial sampling. So she needs to be referred to specialist for further investigation. And the
rest of the management depends on the results of the investigations.

Feedback 11-12-2018
58year old presents back to your GP clinic for her vaginal ultrasound report. It shows
endometrial size of 7mm.
Take hx,
Examiner will give you result and you have to Explain the results to patient
DD
Ask hemodynamically stability, ask bleeding history- CCVO, Bleeding anywhere else. Onset,
duration, progression, 5 Ps, RULE OUT DDX- ATROPHIC VAGINITIS,CERVICAL POLYP ,CERVICAL
CANCER,(ASK ABOUT ANY ABNORMAL PAP SMEAR), ENDOMETRIAL CANCER, OVARIAN
TUMOR,ANY OTHER CANCER IN PATIENT AND FAMILY, BLEEDING DISORDER ,LIVER DISEASES.,
trauma
RISK FACTORS- obesity, smoking, nulliparity(you will cover in obstetric history)sadma-negative
positive findings--patient has had a proper period for the first time since LMP 5 years ago. Bled
for 5 days, used a couple of pads, no clots, not fully soaked. AT the moment, she is not bleeding
but she is worried..so have a guarded reassurance.
RESULT—USG FINDINGS—Endometrial thickness was 7mm, ovary and cervix- healthy, so explain
each part of results and also give above ddx with the result. So said will do further investigations.
Try not to scare the patient..
Global score- 5, all key steps covered
Feedback 11-12-2018
Station 5 (Vaginal bleeding) Global Score 4 Pass
58-year-old woman comes to GP Clinic for the results of her ultrasound. It shows a 7mm
endometrium.
Tasks: History, explain the results and DDx. (Relevant PE will be given on a chart)
I came in the room and greeted both the examiner and the patient.
I introduced myself and noticed patient was distressed. She said she wanted to know the results.
I told her I would be pleased to explain her the results but I would need to make some questions
first. She agreed.
I asked her to tell me more about her bleeding. When it started? Last month. How long did it
last? 5 days. How many pads? A couple. Fully soaked? No. Any clot? No. Any abdominal pain?
No. Any fever? No. When was your last period? 5 years ago. Did you have hot flushes back than?
Yes. Were you on HRT? No. Any symptoms now? No.
I told her I would have to ask some sensitive questions. She nodded. I asked if she was sexually
active? Yes. Any bleeding during or after intercourse? No. Any pain during intercourse? No. Any
STD? No.
Any weight loss? On the contrary, doctor, I have been putting on weight. (that caught my
attention).
Any lumps or bumps anywhere in the body? No.
I asked about her HPV screen she said she had done last year and was normal.
I asked about mammogram and she said the last one was three years ago. I told her I would
arrange one for her.
Smoke? No. Any medication? No. Any Blood thinners? No. Any drug? No. Diet? Normal and rich
in calcium.
Home situation was unremarkable. Family history: unremarkable Previous conditions: none.
I asked for the PE chart. Everything was normal but BMI was 30.
I showed her the ultrasound result and explained it to her. I draw the uterus and showed her
what the endometrium was. I told her that normally it is 5mm but hers was 7mm and that was
the cause of bleeding. I named the condition: endometrial hyperplasia.
I explained that when menopause occurs, the ovaries stop producing hormones like estrogen
and progesterone. But her body was producing estrogen because she was overweight.
She looked at me and said with half a smile: “ you don’t need to be this direct, doctor!”
I said: “ I am sorry, let me rephrase it. As you told me earlier, you have been putting weight and
your BMI is 30. That’s the main reason why your body is producing estrogen. It acts on the
endometrium making it thicker as you don’t have progesterone to oppose its action.”
“Other causes of postmenopausal bleeding could be: atrophic vaginitis but it is unlikely because
there was no atrophy on the specular exam and you have no bleeding and no pain during sexual
intercourse; coagulation problems but you have no family history and you are not using any
blood thinner; polyps but it did not show on your ultrasound; cancer but you have no weight
loss, no family history and no lumps or bumps in your body.”
Key step 1:yes. Approach to patient/relative: 5 Key step 2:yes. History: 5
Key step 3:yes. Interpretation of investigation. 4
Key step 4:yes.

164-Neonatal Examination
GP, 10 days old baby brought in by mom for routine checkup. Baby growth weight (given), height
(given), head circumference (given)
Tasks:
-Take history including pregnancy and delivery history not more than 2 mins
- Perform physical examination
-Tell mom what you are going to do
- Tell the findings to mom

History (2 min )
-congrats
-How are you coping with the baby? Any support?
-BINDS
*Birth

 Pregnancy (Complication)

 Delivery (Type, prolonged, trauma/forceps, need resus)

 Baby ( weight, term or preterm, Heel prick test, Hearing/vision)

*Immunisation
*Feeding (well? What type of feeding? Sleeping well?)
*Siblings, Symptoms (jaundice, fever, cough, noisy breathing, wet nappies, 1 st pass stool)

Examination
1-G/A
- Active, alert
- No signs of respiratory distress
- No bruises, pink, jaundice, rash
2- Auscultate the chest and heart sounds while sitting in mum’s lap. Then put on the couch
3- Hands
-Dorsum: nails, cyanosis, syndactyly, polydactyly
-Palm: no single palmar crease, Grasp reflex
-Pulse: regular, RR delay, RF delay
4- Arms
-Tone, movement
-palpate clavicles
5-Head and Face
-Fontanelle (bulging or depression) and sutures
-Eyes (epicanthic fold, squint, pallor, jaundice, red conjunctivae, Fundoscopy red reflex, )
-Nose (shape and discharge)
-mouth (cleft lip, cleft palate, tongue, cyanosis, Rooting reflex, suckling reflex with gloves)
-Ears (ear tag, low set ears, otoscopy)
-Mandible (prognathia, retrognathia). (You will just say normal jaw position.)

Note/ with fundoscopy use just its torch and point to the eyes as red reflex in babies very
obvious with just light so no need to look through the fundoscope.

6-Neck
-swelling, web neck, trachea position

7-Chest
-Deformity, wide spaced nipples

8-Abdomen
-LOOK (distension, umbilicus)
-FEEL (organomegaly)
-Hernia orifices

9-Pelvis and Lower limbs


-Asymmetrical creases
-Barlow and Ortolani (DDH) = hip stability tests
-Tone and movements of the legs

10-Feet
-club foot, nails, space between 1st and 2nd toe

11-Back passage and Genitalia (Take consent from mum plus wear gloves)
-Imperforated anus, opening of urethral meatus, testicles

12-Back
-tuft of hair, swelling

13-Moro reflex

14-Ask for Growth Chart ( weight, length, head circumference)

Comment for mum


*Your baby is well and nothing to be worried about
*Encourage immunisation
*Continue breast feeding
*Follow up

Some glossary in lay term


Jaundice=yellowish discolouration of skin
clubbing= increase curvature of nail
cyanosis=bluish discolouration of finger tips
syndactyly= joining of the fingers
polydactyly= extra fingers.
Fontanelles= spaces between the bones of the skull
sutures= lines between skull’s bone
epicanthic fold= skin fold of upper eyelid covering the inner corner of the eye
squint= convergence of eye balls.
Cleft lip= opening in the lip
cleft palate= opening in the roof of the mouth
prognathia/ retrognathia= the lower jaw positioning forward or backward

Feedback 16-8-2018
Station 17. Newborn examination

GP, 10 days old baby brought in by mom for routine check up. Baby growth wt (given), height
(given), head circumference (given)
Task: Take history including pregnancy and delivery history not more than 2 mins
Perform physical examination
Tell mom what you are going to do
Tell the findings to mom
  Delivery: normal Vg delivery, no fever, no crack nipple
Baby: no fever, No jaundice, normal poo and pee
PE as usual: all are normal. (Measure tape, otoscope, fundoscope and paediatric stepthoscope
are given)
 I told that baby is healthy & no abnormal signs. Bell ring
I said br feeding & immunization b/f I left room.
Grade: Pass
  Global score: 4
Key Steps: 1- No 2- Yes 3- No 4- Yes 5- Yes
Approach to patient: 4
History: 4
Choice & Technique of examination, organization and sequence: 3
Commentary to Examiner: 4

Feedback 20-2-2018
GP, a mother brought in her 10 days of newborn for a checkup.
Tasks:
-Focused history (not more than 2 min)
-Perform physical examination on the newborn and explain what you are doing to the mother
-Explain your findings to the mother

2 min thinking: short history, check head to toe (my mind was blank for a while, I just let it be)
History:
Congratulations, ask baby’s name, ask support, breast feeding, briefly about pregnancy and
delivery, is this the first baby (yes), any concerns about baby and herself?
Physical examination:
Asked the mother to undress her newborn, mannequin is without nappy. Examiner asked to
wear gloves. Perform as what I learn from this youtube - https://www.youtube.com/watch?
v=hf6Znnusix4

Explain to the mother what I am doing along the way. She was a bit concerned when I did the
barlow and ortolani tests. Reassure her.
Forgot to said I would like to finish my examination with measuring the head circumference,
length and weight.
Explanation:
Explain to the mother that her baby is well and nothing to be worried about. Encourage her to
have immunisation for her baby and reading materials.
AMC Feedback – Health review: PASS
Feedback 20-4-2018 (good)
165-Assessmnet of dehydration
Case 1
Middle age man with watery diarrhoea, v+, headache and brown stool for 1 day.
Tasks
-Do Ex to assess dehydration
-differential diagnosis to pt.

Case 2
Patient had history of acute diarrhoea.
Task:
-History
-Perform PE to assess hydration status
-Most likely Dx and DDx.

Assessment of dehydration Examination steps (Talley o Connor)


1-Do you feel thirsty? General appearance: tired, drawn

2-Vital signs
-pulse rate (rate and rhythm)
-BP including postural hypotension

3-Hand and arm


-CRT
-look for dry skin, temperature (if cold)
-skin turgor

4-Face
-sunken eyes+ pallor, jaundice
-dry mucous membrane + central cyanosis.

5-neck
-JVP

6-Chest and heart


-just quick auscultation of back and heart sounds.

7-focus Abdominal examination


-inspection
-palpation (when you will ask about site of pain he will tell it is not actually pain but a discomfort
all around my tummy)
*superficial and deep palpation
*organ palpation; liver, spleen and kidney
*you can do percussion here as there is no pain but only discomfort
-auscultate bowel sounds

7-office tests (UDT, BSL)

Diagnosis and differential diagnosis


1-Acute viral gastroenteritis (most likely)
2-salmonella
3-giardiasis
4-food poisoning, food intolerance or allergy
5-UTI (normal UDT)
6-DKA (BSL)
7-pyelonephritis (renal angle tenderness)

If history a task
1-do you feel thirsty?
2-diarrhea questions
-duration and frequency
-amount
-colour, blood, content, smell, consistency, hard to flush
3-vomiting
4-tummy pain
5-fever, rash
6-LOW, LOA, lumps or bumps
7-able to eat or drink + urine output.
8-contact, travel, medications esp antibiotics
9-if on special diet, if ate food outside
10-PMH, PSH, Fhx
Feedback 1-6-2018 (IMP)
Middle age man with watery diarrhoea, v+, headache and brown stool for 1 day.
Do Ex to assess dehydration and DD to pt.
( examiner told BP and there was a clock to measure Pulse rate)
PE:
Asked hemodynamic stability as a knee jerk reflex and examiner told to do it my self. Any ways
introduced my self washed hands, asked for exposure and position
General inspection foot end: lying comfortably, not in distress , resp movements are abdomino
throcic, no adjuvants attached,
Hands: warm, looks well perfused, capillary refill time normal, no anemia , jaundice , peripheral
cyanosis, clubbing . rashes, petiches, bruises, skin turger normal, hands not cold and clammy
Check Pulse: with stop watch just checked 15 sec and told him that its good vol, regular and
around 60
Face: eyes: no anemia , jaundice, tears are present no sunken eyes
Mouth: no kusmuells breath, no anemia , jaiundice and central cyanosis, tongue not dry
Chest : resp: clear.
heart: s1&S2 normal no gallop rhythm
Abd: inspection from bed end : no scar, swelling redness , rashes, distention and discolourations
Plapation: soft non tender no organomegaly

Percussion in all 9 quadrents normal


Didn’t do liver and spleem span but checked kidney by ballotment method
Auscultation: bowl sounds normal not exaggerated
Limbs: normal, temp normal, nails no peripheral cyanosis, nor CRT, no insect snake bite or
bruises.
Told examiner would like to conclude by examining hernia orifices, DRE, ECG, urine dip stick for
nitrites, leukocytes, hematuria, ketones, RBS she said all normal
Pt education:
Told him that although you had few episodes of N&V but thankfully u r not dehydrated
For Dx & DDX said
Most likely
Food poisoning
Viral gasto enteritis
Bacterial GAE.
DKA (but sugar is normal and not dehydrated)
UTI (urine dip stick normal
Pyelonephhritis( renal punch negative)
Brain tumor ( hx is chronic)
Feed back passed
Global score :6
Approach to pt: 5
Choice, technique of exam , organization:6
Accuracy of exam : 6
Dx, DDx: 6

Feedback 1-6-2018

Patient had history of acute diarrhoea. Task: History, Perform PE to assess hydration status, Most
likely Dx and DDx.
r/o all ddx of diarrhea from hx. Diarrhea was for 24 to 48 hrs (can’t recall). Rest normal.
Hydration status all normal. (CRT, Pulse, BP, Skin turgor, JVP, oral mucosa etc…)
Forgot to ask if pxt feels thirsty, check bowl sounds and mention DRE.
Gave Dx AGE and other possible DDx and told why it’s unlikely that it could be anything else.

Feedback 18-9-2018 Dehydration assessment


2. health review - abdominal pain and dehydration PE
2 young man vomited and had watery diarrhoea for a few times for the last two days.
Taks: PE and D/D

I have to measure the pulse, and I found it to be 54!


I did a full abdominal examination plus dehydration exam. Everything is normal except slight discomfort on
deep palpation.
(Forgot murphy sign and Mcburney’s tenderness. )
Dx: acute gastroenteritis. Then my mind went blank and could not come up with other DD. Then I said it
could be appendicitis then I released that I didn’t do any sign for acute abdomen. So I said I would also like
to do Murphy sign and Mcburney tenderness. Then the examiner said it was normal… The bell rang so I
quickly ran away….
Score: 5
Key step: yes/yes/yes/yes
Approach: 4, choice of the examination: 5, accuracy: 5, Dx and D/D: 4
Feedback 18-9-2018 Dehydration assessment
You are an HMO and the next pt is a 28 year old man who is suffering from
diarrhoea for the last 2 days. Today he had about 10 motions and vomited two
times. He came to the hospital with a complain of nausea, vomiting and headache.
After primary survey the findings are given below
Temp. : 38*c RR: 18brths/min
per abdomen: tenderness present, no organomegaly, no visible pulsation
Genitalia: Normal
DRE: Normal
Hernial Orifices: Normal.
Your task is to
1. Do relevent Physical Examination to assess the dehydration status of the pt.
2.Discuss the Dx and DDx with the pt.
My Approach:
I went in and greeted the pt , he said Doc I am having tummy pain and headache. I asked if he want any
painkiller. He said no I am ok to continue.But whats happening with me doc? Then I intrduced myself and
explained the examination and gained consent, washed hands and start commenting on GA. Pt was lying
and anxious. abdomen was moving normally with respiration and no scar mark.
Then Started with pulse , there was a clock to count the pulse. I got pulse around 55. Then I did CRT which
was less than 2 seconds,Skin turger was nomal, then I went to tongue (Dry) and oral mucosa, everything
was normal. No sunken eyes. Tried to do BP and she said its 110/72 Then I quickly palpated the abdomen
superficial and deep. He had a generalised abdomial pain .Didn't do murphys or macburneys sing test. I
forgot to address the headache as well. On Dx I said gastroenteritis with mild dehydration which
explains your headache too.
On DDx I said Malabsorption syndrome, bowel obstruction, constipation, diverticulitis, infection, PID
AMC Feedback: Station 02: Health Review Garde: Pass Global score: 5
Key steps: 1,2,3,4 yes,yes,yes,yes Assessment domain: Approach to pt/relative: 4
Choice and technique of examination, organisation and sequence: 4
Accuracy of examination: 4 Dx/DDx: 4

Case (3/3/2017)
Middle aged lady present with tummy pain, loose motion and vomiting a few times containing
food materials. Stool was dark brown no blood. No recent travel H.
Task :
Assess dehydration status of the patient,tell examiner about your findings.
Perform abdominal exam, Mention pdx, ddx to patient.
(positive findings : skin turgor sluggish, cap refill normal, BP normal, no sunken eyes, no dry
tongue, patient feeling thirsty a bit. Abdominal examination deep tenderness present on whole
abdomen, no signs of perforation, hepatosplenomegaly absent, PR and VE Examination normal,
bowel sound normal, even did fluid thrill also negative) Mention Gastroenteritis,Typhoid, non
specific viral diarrhoea
comment
Hi passed yhis case i began mh examination from general appearance. then vitals in which
examiner asked me to calculate pilse my self they gave me a watch however bp was given by the
examiner. then i comment about mucous membranes skin pinch capillary return then i stated
abd examination dehydration status was mild to moderate and abd has no specific findings just
discomfort all over finished with pr which examiner gave the finding
And did u measure the Pr for complete 1 minute??
No just for 15 sec then multiply by 4 just comment about rate and rhythm

Case (4/10/2017)
GIT exam- aslo assess dehydration. I think the pt was mild dehydrated. give findins as you go
with the exam . the case was abd pain of a 27yr old man. i did not get any positive finding here.
just non specific abd pain. Give DDs
Health review – Pass
Young lady, c/o diarrhoea for 2 days- vomited twice today.
Task
History
Do physical examination looking for hydration status and abdominal examination with
presentation to examiner
Diagnosis and DDs to patient

Watery loose stool. Cannot remember much details in history, but it all sounded like a viral
diarrhoea. Nothing alarming in history. No travel history. SAD-no
No positive findings on checking for hydration status, but when asked if she feels thirsty she said
yes/ dizzy-no.
Normal vitals given by examiner
Abdomen- Only positive finding was generalised mild tenderness on deep palpation. Bowel
sounds normal.

Diagnosis: Gastroenteritis most probably viral, with mild dehydration.

DDs: Salmonella/ Giardiasis/ Food poisoning/ Food allergy / Intolerance / Side effect of
medication…..Bell rang
¾ key steps covered.
166-Breast Examination
Case 1/
You are in GP and 55 year old came with complaint of lump in right breast. Her mom was
diagnosed with breast CA and she is concerned about it.
Tasks
-Do relevant physical exam with running commentary to examiner
-Explain about possible causes to pt
-Discuss what investigations u would like to do

Case 2/
27 yr old lady found lump in her Rt breast and visits your GP for advice. Her mother was
diagnosed with breast cancer recently and has undergone treatment, is well now. She is anxious
about that she may be having cancer too.
Tasks:
-Conduct examination of breast on the manikin and present your findings to the examiner.
-Counsel and give differential diagnosis based on your findings to the patient.

Case 3/
breast lump in a 27 years old lady worried as her mother is just being treated for a breast cancer
at 52 years old
Tasks
a- take consent from patient
b- examine the breast on the manikin
c- tell to what investigation needed
d- give dx and ddx to the pt

1-Introduce yourself and wash your hands

2-take permission and exposure


-today I need to examine your breasts this would involve me inspecting the breasts then I will be
placing my hands over your breast tissues. I will start with the normal breast first then I will
examine the breast with the lump. The lump also will be examined and finally I will examine the
glands in your neck and armpits.
-At times I need to reposition you and if you and if you have any pain or discomfort feel free to
stop me at any point.
-a female staff will be present as a chaperone throughout the examination.
-are you happy for me to carry out the examination?
-now if you could go behind the screen undress yourself up to the waist then cover yourself with
the sheet provided and sit on the couch and when you are ready please call me.
3-Inspection (4 positions all sitting)
sitting with her hands on her thighs relaxed
-asymmetry (if you noticed any gross asymmetry ask if it is latest or not).
-Scars (lumpectomy, mastectomy).
-skin changes:
erythema ex. Infection/malignancy
puckering or dimpling of the skin ex breast cancer
peau d’orange ex aggressive breast cancer
-visible lump or not (if you can see it then you need to talk about the site, size and others in the
exam unlikely you will see it)
-nipples (retraction, distortion, ulceration, discharge)

Hands on hip pressing inwards


-if lump visible (look if it is moving or fixed)

Both hands behind her head


-puckering or dimpling become more evident
-lower side of breast clearly
-if any lymph nodes in the armpits

Hands still behind head and ask the patient to lean forward
-if lump falling forward with lump tissue (again if not then it is attached to chest wall indicating
malignancy)

4-palpation
-wear gloves
-if you could lie down on the bed for me and ask her to raise the hand of the side that will be
examined and put it behind the head (for example if you start with left breast ask her to raise her
left hand and put it behind her head).
-ask about pain then start with the normal breast first.
-palpate in a spiral way starting from the nipple then proceeding outwards and always make sure
to palpate the axillary tail.
-then go to the abnormal breast the same maneuver and if you found a lump leave it complete
the whole breast the go back to it.
-Lump examination
Site and size
-site is in what quadrant and the distance away from the nipples.
-size is to take a tape measure and find the length and width
Surface and border
-surface if even or not (smooth or nodular)
-border if regular or irregular
Consistency and tenderness
-consistency if soft, firm or hard
-tenderness
Mobility and fluctuation
-ask the patient to squeeze her nipples
-palpate the lymph nodes of the axilla and the neck
-recover the dummy with the sheet

-Differential diagnosis are


1-breast cancer (nasty growth).
2-fibroadenoma (benign or non-cancerous overgrowth of connective tissue of the breast)
3-fibroadenosis (lumpy-bumpy breast)
4-breast cyst (fluid-filled bubbles, similar to blisters, in the breast tissue)
5-fat necrosis or traumatic injury to the breast.
6-breast abscess (localized collection of pus in the breast tissue)

- talk about triple assessment


-for any breast lump we need to do a triple assessment or tests in order to confirm the diagnosis.
-The first part is a clinical examination, which I have done on you.
-The second part is imaging for which I would like to refer you for Ultrasound (young)/
Mammogram (old).
-The third part is cytology or FNAB, for which I would like to refer you to a specialist surgeon.
Where a fine needle is introduced into the lump, and cells from the lump is aspirated and
observed under the microscope for any nasty growth.
-I would like to take some blood tests as well like routine FBC, UCE, LFT.

Notes
-consistency: when you press down to feel the consistency and ask the patient if it is painful or
not.
-fluctuation: 2 fingers of the one hand around the lump and one finger of the other hand to
press. +ve if 2 fingers move apart.
-mobility: 2 directions; horizontal and vertical
-5 lymph nodes Group site
1-along the anterior axillary fold
2-apical
3-along the posterior axillary fold
4-along the chest wall
5-along the Humerus
Case (22/2/2017)
You are in GP and 55 year old came with complaint of lump in right breast. Her mom was
diagnosed with breast CA and she is concerned about it.
Tasks
-Do relevant physical exam with running commentary to examiner
-Explain about possible causes to pt
-Discuss what investigations u would like to do

2 mins thinking
-This is new PE which was asked yesterday
-I am happy that I watched geeky medics video yesterday and all the things are fresh in my mind
-Not to miss to take consent, chaperone, inspection, plapation including axillary tail and LN
-Triple assessment is important in invx

Physical Examnination
When I stepped into the room, middle age lady with a big smile was sitting on a chair.
I greeted to examiner and roleplayer.
Good morning Magaret, I am Dr Khine. I am one of the Drs in this GP.
I understand from ur note that u have lump in ur breast. May I know which breast? (Rt breast
Dr)
U might be worried about it, I can understand.

Today, I would like to do examination of ur breast which will involve having a look, feel ur breast
and glands in ur armpit and neck as well. Is that ok with you? (Yep Dr)

I am gonna call a female staff and she will be with us as a chaparone thorought our examination.

I wash my hand now and I would like u to take off ur top including bra. U can wear this towel on
ur upper part. (RP : Sure Dr. She intimated to remove her dress. Actually there was no towel
inside the room)
During my exam, I ll try to be gentle as much as possible, just let me know, if u feel any
discomfort or pain. I ll stop my exam.

Then I started my examination

GA – My patient is middle age lady of average body build who is sitting comfortably on a chair.
She is not in distress.

Inspection
1- There was a manikin of upper half of female body sitting on the couch. I approached to it and
about to start examination. Examiner interrupted me saying “Dr pls put on gloves pointing
towards the glove box on the wall”. I quickly went there and put on gloves. Luckily my hands
were not too sweaty and it was not difficult to put on them.

I asked vital signs which were normal.

On inspection, Comparing both sides, both breasts are symmetrical in size. There is no erythema,
no ulcer, no surgical scars, no dilated veins, no radiation marks. Nipples are healthy and no crack,
no ulcer, no erosion, no retraction. I cannot see any lumps, no Peau d orange, no skin dimpling as
well.

2- Margaret, can u put ur hands on ur waist and press them? (RP followed my instruction, she is
sitting on a chair throughout examination)
I cant appreciate any mass, no skin dimpling.

3- Margaret, can u put ur hands behind ur head? (At the same time, I showed her how to do) I
can’t see any mass, no dimpling. (I should have told no mass in axilla region)

4-Ok, good. U can put ur hands down now and lean forward. Both breasts are hanging down
symmetrically.

Thank you. Margaret.

Palpation
Now, I am gonna feel ur breast. Is that ok? (Yes Dr)
The breast of ur concern is right, so I ll start from left breast. Do u have any pain in both of ur
breasts? (No)

I started palpating Lt breast in manikin in radial manner from outside towards nipple saying “ Lt
breast is soft, temperature is normal, no mass, no tenderness (I look at the roleplayer’s face)”
and palpating axillary tail “ Axillary tail is clear”

Then, I did the same on Rt breast. There was a small mass in upper outer quadrant. I said “There
is a mass which is about 1cm x 2 cm in size in upper outer quadrant of rt breast, it is firm, not
tender, not mobile (I looked at the examiner’s face, he didn’t tell me about positive findings, so I
said what I feel on manikin), temperature is normal. It is not attached to overlying skin.
Then I asked RP to use her hands to press against waist and checked muscle attachment and
moved the mass in two directions. “The mass is not attached to underlying muscle”

I asked RP, would u mind to press ur nipple gently. She intimated it. “There is no nipple discharge
on pressing nipple”

Then I do axillary LN examination on both side. I asked RP to put her arm on me and keep
relaxed for me. I examined anterior, posterior, lateral, central, apical groups in manikin.

Then, I checked supraclavicular and cervical LN.

The bell ranged and examiner said to move to next task.


(OMG, I forgot to do palpation in lying position, I turned towards examiner and said “Examiner, I
would have done plapation in pt with lying position” He nodded his head which made me
relieve.)

Thank you Margaret for ur cooperation and i removed gloves.


Explanation to patient
Well, Margaret, I ve examined ur breasts today and found out a lump in ur rt breast.
There are a couple of conditions like Fibroadenoma which is non cancerous proliferation of
breast tissues. It could be fibroadenosis which is lumpy bumpy breast, or abscess, or traumatic
injury to ur breast, or nasty growth. (I drawed diagram of breast with lump on rt side)

So, We need to do further tests. I ll check ur blood for basic counts, chemicals, renal and liver
function.

I ll get involve the specailst and he will do imaging of ur breast we call mammogram. He take a
small piece of tissue from it with needle and examine it under microscope we call FNAB. So far
are u with me? (yes dr)

Ok, all of them are called triple assessment : clinical exam, imaging and tissue diagnosis.
According to investigation results, we ll treat u accordingly.

The bell ranged. I smiled and thanked to examiner and role player and came out of the room.
AMC Feedback - Breast lump : Pass (Global Score - 5)
Approach to patient - 4
Examination technique and choice - 5
Accuracy of examination - 5
Dx/ DDx - 4
Invx - 5

Case (29/3/2017)
Breast exam, mother had hx of CA
Hi, I am Dr. Maria. I am required to examine you. That would require me to ask you to do certain
manoeuvres. Is that all right? I will try to be as gentle as possible, but if at any time you feel
uncomfortable, please let me know. Before I begin, do you have any pain?
I washed my hands while saying all this and wore gloves. I said I want my patient properly
exposed in the presence of a chaperone and the examiner asked me to proceed. Talk to the role
player and examine the dummy.
Inspection:
with arms at the side: breasts, nipples, skin, scars, swelling
With arms pressing at the hips, hands raised up behind the head and then bending forward
Palpation: axillary lymph nodes, sc and infra claviculae lymph nodes. Rough palpation of the
breasts
Then with the patient lying down and i/l arm raised behind the head start from the normal
breast (see JM for examn) and the abnormal breast , cover all the quadrants and then come to
the mass.
Mass: site-right outer upper quad, size-2*3cm, shape- circular, surface- smooth, skin over the
swelling-normal, consistency- firm, temp, not tender,
Dds: cysts, fibroadenoma, tumour, traumatic fat necrosis, cancer. Talked about triple tests and
screening of other first degree relatives
Global score- 4
Case (11/11/2017) FAIL
27 yr old lady found lump in her Rt breast and visits your GP for advice. Her mother was
diagnosed with breast cancer recently and has undergone treatment, is well now. She is anxious
about that she may be having cancer too.
Tasks:
-Conduct examination of breast on the manikin and present your findings to the examiner.
-Counsel and give differential diagnosis based on your findings to the patient.

Introduction-
Roleplayer abruptly said – Tell me what I should do?
I introduced myself again and said I will be doing an examination of your breasts, I will arrange a
female chaperone during the examination. She said it is not necessary and please wear gloves!!

Exposure- Asked to change to a gown while I wear gloves


Started off with look- Asked the roleplayer to put arms on thigh, back of head, lean forward. She
was very reluctant to do it. But I politely waited for her to do it and then looked at the manikin
each time. I described what I could see- Both breasts appeared symmetrical, skin normal, nipples
normal, no obvious lumps visible
Palpation- I had to put the manikin in supine position myself. Followed geeky medics steps, I
really struggled to find the lump and at first dint find it.
The examiner told to check again- found a lump in the outer lower quadrant which slipped and I
could barely make out size, shape, consistency. So I just said it feels like the rest of your breast in
consistency and moves within the breast ( Some pple said upper outer quadrant- which confirms
movement ), it is not attached to any structures underneath or to your skin.

Change manikin position to sitting- Palpate 5 groups of axillary nodes on both sides, then neck
nodes.
I gave running commentary so when I started to present to the examiner he said he heard me as
I went so move on to next task.

Task 2
Using a diagram I explained differentials- Most likely fibroedenoma, other fibrocystic disease
of the breast, hematoma, abscess, cancer. I reassured her its unlikely to be cancer.

I FAILED this station because although the task asked for the differentials only( I remember very
well that there was no mention of management) I think the key word is counsel pt so it is
essential to mention that you would do an ultrasound and FNAC. The assessment domain has
choice of investigation. You may want to ask other candidates on this day if there was mention of
investigation in the task. I clearly don’t remember.

Key steps 1, 2 , 3 – Yes Key 4- No Global score: 3 ( Fail)


Assessment domain
Approach to pt/relative: 6
Choice and technique of examination, organisation and sequence: 4
Accuracy of examination: 5
Diagnosis/ differential diagnosis: 4
Choice of investigation: 0
Case (5/10/2017)
Breast lump Pass
57 years old woman comes in with a Lump in her Right breast. Her mother died from breast
cancer and she is concerned about it.
Task:
Relevant examination
Explain to patient the Investigation you want to do
Dx and DDx to patient

Introduced myself and explained her about breast exam and why i m going to do it. Then asked if
she has any pain or not and then reassured her that i will have chaperone throughout my
examination. Then perform Breast exam. Patient was sitting on a chair and there was dummy on
the bed. It was very confusing as i had to do inspection on role player as she is sitting and
dummy is lying on the bed. Then palpation on dummy, there is a mass in Right upper outer
quadrant area, firm, not sure about mobility. I commented the mass ( site, size, surface,
consistency, tenderness, skin attachment, mobility). And then i had to ask the role player to
squeeze her nipple, she pretended to squeeze on her nipples which was very weird for me. Then
i asked her i forgot to do some and asked her to raise her arm above her head and inspect about
accentuation of skin dumpling or nipple retraction. I even did axillary lymph node palpation on
lying dummy ( which is not correct i think). Then, i explained to her that i would like to arrange
Mammogram and FNAC ( small needle to put in the mass ).

Dx - Fibroadenosis DDx- Fibroadenoma, Nasty growth ( she asked me what is nasty growth
and i told her cancer), Trauma ... I couldnt squeeze my brain enough to tell other DDx at that
time. I explained thoroughly about all the Dx and DDx though.

Covered 3 key steps out of 4


Approach 4
Choice and Technique 5
Accuracy 4
Dx and DDx 3
Choice of InVx 4
Global score 4

Case 8-2-2018
Breast exam: 52 yo female, noticed lump in R breast. Examination, tell patient finding and
management

Case 7-3-2018
PE breast for ca

Case 6-4-2018
Breast examination
Case 12-4-2018
A 27- year old lady comes with a breast lump found 2 w ago and so
worried. her mum recently diagnosed as having breast cancer. tasks:
*do p/e of breast
* explain DDX & MX.
Case 5-5-2018
Breast examination for a female with a lump. She has family history of breast cancer and in menopause
now. Give Investigation and explain with reasons
Case 8-5-2018
Breast exam cyclical mastalgia diagnosis dd found 2-3 lumps not sure firm to hard mobile
Case 5-6-2018
female ,mom had breast cancer,task-take consent from pt,do pe,dx dd,inv u will do.
Case 6-6-2018
Breast examination, hard fixed lump 57 yr ol, mum had cancer
Station 6 breast examination – pass
?57 years old lady, Breast lump, mother had breast cancer.
Task: get the consent from the patient to do breast examination
Do breast examination ( need to wear glove.)
Possible dx and ddx
Explain the patient what inv we are going to arrange for her.
Reassure, sorry to hear that your mother had cancer .
WIPE. Chaperon.
Do examination on dummy and give instruction to patient.
Inspection: nothing abnormal
Palpation: hard lump at upper outer quadrant, fixed, not tender,
Lymph node: axillary and supraclavicular nil
covered back the dummy with the sheet.
Case 4-7-2018
Breast exam
57 yr old lady with breast lump. FHx-mum had breast CA
Task-PE-commentary to ex /DDx/ Ix
3 position inspection
palpation-firm hard lump at R/UOQ
ALN-manikin has very short arm asked examiner in whom to do LN-said to do on manikin
DDx-explained with positive hx & ex findings-malignancy, abscess, less likely breast
cyst/fibroadenoma/fibroadenosis-as prevalent in Menstruating women
Ix mammogram & biopsy.
Case 11-7-2018
Breast exam with DD
Case 19-7-2018
57 yr old lady with breast lump . PE , dx withddx and investigation
STATION 19 PASS( KEY Steps all yes score 5)
Breast PE( elderly female with + FHx came with lump in breast
Task PE , DX AND DDs ,Investigations
Solid small lump was present in the dummy, axillary and cervical LN done on dummy , explained most
likely diagnosis with empathy and dds as well. Told her triple test.
Case 15-8-2018
Breast exam.

Feedback 20-9-2018 Breast examination


Breast Lump PE (FAIL)
27 years old lady c/o breast lump on Rt breast. Her mother was Dx with breast Ca.
Tasks :
Examine the breast on dummy provided.
Dx, DDx
Further investigations

Approach :
I greeted the patient, asked her is there any concern? She says what would u like me to concern
with a serious facial expression. I was surprised.

PE :
Then I took consent from her and started examination. ( needed to wear gloves ) I did according
to what I practiced, and I found a lump on Rt upper outer quadrant which is around 2*3 cm in
dimension, hard in consistency, not mobile. Then, a small bell rang.

Dx/DDx :
I told her that I suspect Breast Ca according to nature of the lump. Then gave other DDx like FA
breast, fibroadenosis, traumatic fat necrosis, breast cyst, breast abscess which are less likely.

Inx : USG, FNAB

Then, I forgot to examine Axillary and cervical LN and was counseling her about CA breast. Then
in the last minute, it just pop up in my mind so, told the examiner that I’ll examine axillary LN
and time’s up.
Feedback 26-10-2018
Station 7: Breast exam
Passed (GS – 5)

-Introduction and get consent from patient

-Wear gloves!

On inspection by 3 positions:

 Hands by side: no ulcer, no scar, no obvious mass, skin appears normal. Nipples are in same level.
 Press your hand on your waist: no new changes or tethering
 Raise hands over head: no new dimpling or tethering

On palpation

Breast (ask patient to raise hand over the head, examined from normal side):

Findings: left breast is normal. No tenderness. On right side, I can feel small mass on Rt upper outer
quadrant, which is 2*3 cm in diameter, round, seems to be smooth, margin is well-defined, firm in
consistency, no tenderness, no fluctuation, hardly mobile (mannequin is hard to assess mobility).

Lymph nodes: axillary and supra & infra-clavicular lymph nodes (on mannequinma)

Findings: NO LAD

Then, I covered mannequin back.

DDX

Explained my findings and DDx

i. Fibroadenosis (given the location and consistency of lump),


ii. Breast cancer
iii. Fibroadenoma
iv. Breast cyst (less likely as it is a solid mass and no fluctuation),
v. abscess (no fever, no tenderness).

INVx:

Refer to specialist for triple assessment (patient asked me what it is?). Then, I explained further about
physical exam, imaging which is mammogram, FNAC.
Feedback 27-10-2018 STATION 7 BREAST LUMP
50+ year old lady with lump in her right breast. she is concerned about having a breast cancer
Task
- perform examination
- Explain DX and DDx to the patient
- Arrange investigation
- performed the examination according to TALLY O’ CONNOR
- asked for chaperone and told her that the room is safe and private

- found a hard, fixed, non-tender lump (2x2cm) in the right upper outer quadrant of the right
breast. (consistency could be wrong!!!)

- no skin dimpling or peau d’orange appearance

- no changes in nipple or discharge

- no axillary LN palpable

- left breast normal

DDx
- breast cancer
- fibroadenoma
- fibroadenosis
- breast cyst

(breast cancer wasn’t the correct diagnosis as i got only 3 in DDX. Besides, it has a well defined
margin !!!!)

Investigation
- biopsy
(forgot to mention the imaging test (mammogram). Hence only 3 in investigation) Grade -
fail

Global score - 3
Key steps 1,2,3,4- Yes, No, No, Yes
Approach - 3
Choice and technique of examination, - 4
organization and sequence
Accuracy of examination - 3
Dx/DDx - 3
Choice of investigation - 3
Breast examination Mastalgia
(see mastalgia lecture as well)
Feedback 15-8-2018
Scenario: Breast Pain
Stem: Young lady comes with complain of breast lumps getting painful with menstruation and
had a fhx of breast cancer…
Tasks:
~Perform Physical examination
~Tell your dx and ddx to the patient

In the room young lady was sitting on chair with a breast mannequin next to her covered with
sheet. Started with usual WIPE approach and call for chaperone. It was written in the stem that
use gloves so I wore gloves.

Inspection: from front, then hands on waist and behind the head with lean forward. ( comment
on size symmetry, skin changes, swelling, scar, veins, dimpling, pseudorange and any
accentuation of mass etc etc) ( All normal)

Asked her to lie down and again took the consent again and asked which breast is of concern,
she said “right” so I said I will start with the left one ( hoping that examiner would stop me and
would ask to examine the concerned breast) but he did not. So I palpapted the left breast with
hands behind head. Checked all 4 quadrants, tail and asked her to squeeze nipple ( ALL
NORMAL)

Moved to the concerned breast. Did same and found a lump in outer inner quadrant ( don’t
hesitate to push hard as mannequin is not going to feel and respond but u might ending up
missing lumps) keep looking at patients face. Comment on lump (site, size, surface, consistency,
fluctuation, fixity, temderness etc). looked for other lumps and found another in outer upper
quadrant with same features almost. Complete the palpation, tail and nipple squeeze.

Thanked and asked her to sit so I can check glands in arm pits..Checked all group of lymph nodes.

Asked about checking cervical. Examiner said skip it.

Thanked the patient and covered the mannequin. Took gloves off.

Gave dx of cyclical mastalgia and explained it… ddx, fibroadenoma, fibrocystic, fibroadenosis,
lipoma, carcinoma( ductal, papilloma, in situ) least likely mastitis abcess pagets phylloides etc
( try giving as many ddx as possible)

Grade: Pass
Breast examination talley and connor (for reading )
Breast examination is a vitally important part of the general physical examination. In women
aged 40 and older, a screening physical examination for breast cancer may be advised (e.g.
monthly self-examination by the patient and yearly examination by the doctor), but there is no
convincing evidence of the value of breast self-examination.

Examination anatomy
The attachment of the female breast extends from the clavicle superiorly to the ‘bra line’ (sixth
rib) inferiorly and from the sternum to the midaxillary line.
The area covered is more rectangular than circular. The axillary tail is an extension of the
superolateral part of the breast up towards the axilla. This tail extends along the lower border of
the pectoralis major muscle and may pass through the deep fascia to approach the axillary
lymph nodes. Breast tissue has fatty, glandular and fibrous components. Except during lactation,
most of the bulk of the breast is adipose tissue. There is a surrounding superficial fascia attached
to the skin and pectoral fascia by ligaments. The normal breast does not have a homogeneous
feel but is somewhat lumpy.

There are a number of benign abnormalities of the breast that change the texture or
appearance of the breast, including:

● fibroadenomas caused by an overgrowth of connective tissue surrounding the ducts

● cysts due to obstruction of collecting ducts

● intraductal papilloma, causing a palpable lump

● mammary duct ectasia, which causes a nipple discharge

History
The history is imperative. Essential questions to ask include
- the length of time any mass has been noticed,
- the presence of pain,
- any change in size or texture over time,
- relationship to the menstrual cycle, and
- any nipple discharge.
- Ask whether the patient has conducted regular self-examinations and discovered a change in
her breast.
- Ask about previous cyst aspirations.

Find out about risk factors for breast cancer, including


- any family history of breast or ovarian cancer (and age affected),
- previous personal history of breast cancer,
- obesity,
- early menarche and late menopause,
- late first pregnancy,
- mantle radiation,
- heavy alcohol use and
- use of oestrogens

post-menopausally. A personal history of atypical hyperplasia (ductal or lobular) increases the


risk of breast cancer 3 to 5 times. However, three-quarters of patients presenting with a breast
cancer have no known risk factors.
The breast cancer genes BRCA1 and BRCA2 are associated with a strong risk of breast (and
ovarian) cancer, as well as breast cancer in men. Patients with a strong family history of breast
or ovarian cancer should be asked if genetic testing has been carried out. Screening
mammography is generally recommended for all women from the age of 50 years.2
Enlargement of the male breasts (gynaecomastia; is often a result of hormone or drug
treatment (e.g. for carcinoma of the prostate or with aldactone). It is often tender. Bilateral
enlargement is usually benign but unilateral enlargement may be due to malignancy.
Since malignancy is more common in men with gynaecomastia, asymmetrical enlargement
should arouse concern. Carcinoma of the male breast is over 100 times less frequent than
female breast carcinoma. Since there is less breast tissue present in men and the condition is
often not thought of, disease tends to be locally advanced at the time of diagnosis.

Examination
When it is done properly, the examination takes some time to perform (about 3 minutes per
breast).
The need for examination must obviously be explained to the patient at the start. The patient
should be offered a chaperone.
The examination is only just over 50% sensitive for carcinoma but specificity is as high as 90%.
The likelihood ratio of a positive examination is 14.1 and the LR of a negative examination is
0.47.

INSPECTION
Ask the patient to sit up with her chest fully exposed.
There is controversy about the value of inspection of the breasts as part of the examination, but
advanced cancers may be obvious at this stage.
1-Nipples
-Look at the nipples for retraction (due to cancer or fibrosis; in some patients retraction may be
normal, so it is important to ask if it is long-standing)
-Paget’s disease of the breast (where underlying breast cancer causes a unilateral red, bleeding
skin).
- A persistent erythematous plaque in the areola area may be contact dermatitis or skin
irritation, but if asymmetrical or it has not responded to treatment this may be the malignancy
Paget’s disease of the breast.
2-Breasts
-Asymmetry of breast size is common, and it is important to ask the woman if any asymmetry is
new.
3-Skin
- Look for visible veins (which if unilateral suggest a cancer),
- skin dimpling and peau d’orange skin (where advanced breast cancer causes oedematous skin
pitted by the sweat glands).
-Scars

4-Visible swelling.

Ask the patient to raise her arms above her head and then lower them slowly. Look for
- tethering of the nipples or skin,
- a shift in the relative position of the nipples or
- a fixed mass distorting the breast
- Note whether there are any obvious visible masses in the axillae.

Next ask her to rest her hands on her hips and then press her hands against her hips (the
pectoral contraction manoeuvre). This accentuates areas of dimpling or fixation.

Conditions unrelated to the breast may be found incidentally, such as pectus excavatum or
carinatum, distended veins related to inferior vena caval (IVC) obstruction and signs of
virilisation such as acne and hirsutism.

PALPATION
1-Examine both the supraclavicular and the axillary regions for lymphadenopathy.
I t may be difficult, however, to distinguish an axillary fat pad from an enlarged lymph node.

2-Then ask the patient to lie down.


The examination can be performed only if the breast tissue is flattened against the chest wall. If
the breasts are large, it can be helpful to have the patient place her hand behind her head for
palpation of the lateral aspect of the breast and bring her elbow up level with the shoulder for
palpation of the medial side of the breast.

Palpation is performed gently with the pulps of the middle three fingers parallel to the contour
of the breast. Feel the four quadrants of each breast systematically. Do not pinch the breast as
you may mistakenly think you feel a mass.

The total examination should involve a rectangular area bordered by the clavicle, the sternum,
the midaxillary line and the bra line.

Start in the axilla and palpate in a line down to the bra line inferiorly.
The pattern of palpation is like that of mowing a lawn, a series of vertical strips that cover the
whole of the rectangle.

Palpation may also be performed by quadrant, in a spiral fashion from the nipple outwards or in
a ‘flower petal’ pattern radiating around the breast from the nipple outwards.

Each area is palpated three times, using small circular movements and slightly increasing
pressure.

Palpation is more difficult when a breast implant is present.


It is probably best to examine such a patient in a supine position and to keep the ipsilateral arm
down at her side.
3- Next feel behind the nipple for lumps and note if any fluid can be expressed: bright blood
(from a duct papilloma, fibroadenosis or carcinoma), yellow serous fluid (fibroadenosis) or
serous fluid (early pregnancy), milky fluid (lactation) or green fluid (mammary duct ectasia).

Do not mistake normal breast structures for a mass. You may feel a rib or costochondral junction
normally on deep palpation.

4- The inferior ridge of breast tissue (inframammary fold) may be felt and is symmetrical.
You may feel normal rubberytype plaques (fibroglandular tissue), especially in the upper outer
quadrant. It is normal to feel firm breast tissue at the areola border, and the upper outer
quadrant of the breast is often dense.

Evaluation of a breast lump


The following five points need to be carefully Elucidated if a lump is detected.

1. Position: the breast quadrant involved and proximity to the nipple.

2. Size, shape and consistency: a hard, irregular nodule is characteristic of carcinoma; a hard,
smooth regular nodule is more likely to be a fibroadenoma, especially in a young woman.

3. Tenderness: suggests an inflammatory or a cystic lesion; breast cancer is usually not tender,
although inflammatory breast cancer is associated with tenderness.

4. Fixation: mobility is determined by taking the breast between the hands and moving it over
the chest wall; in advanced carcinoma the lump may be fixed to the chest wall.

5. Single or multiple lesions present: multiple nodules suggest benign cystic disease or
fibroadenosis.

Remember that lumps found on breast examination may not involve breast tissue: these include
lipomas and sebaceous cysts. Remember also that many normal breasts have palpable lumps
and that although benign lumps tend to be soft, moveable and regular, they can also have the
characteristics of malignant lumps.

1. A palpable breast mass is likely to be significant (called a dominant mass) if it is:


– clearly three-dimensional
– distinct from the surrounding tissue
– asymmetrical compared with the other breast
– persistent throughout a menstrual cycle
– not smooth, well-demarcated or mobile.

2. A palpable breast mass is more likely to be malignant if it has the following characteristics:
– It is very firm.
– The margins seem poorly defined or have an irregular edge.
– It is immobile or fixed.
– There is associated skin dimpling.
– There is associated retraction of the nipple, or nipple scaling.
– There is a bloody nipple discharge.
– The draining lymph nodes are palpable.
167-Bee sting

Child stung by bees today, sting taken out, uncomfortable and crying kid, had been bitten by bee
before... At time.. Just local reactions... Pefe given in stem, vitals - normal, CVS resp - normal -
Tasks
-History
-current and future mx

History
1-Hemodynamic stability
-(already given in the stem and stable Bp=90/60, RR=25, PR=90)
-let me assure you that your child is stable and his vitals are fine.

2-chief complaint questions


-can you tell me more?
-how has he stung by the bee? What was he doing?
-Local reaction (they said it was swelling on arm) so ask
-since when? Is it getting bigger?
-anywhere else?
-is it itchy? Is it painful? Does it affect his sleep?
-I can see that you have taken the sting out. Have you done anything else?
-has this happened before? (2 nd time)

3-Associated symptoms
-redness
-fever
-SOB, noisy breathing
-difficulty swallowing, hoarsness of voice
-nausea, vomiting, tummy pain

4-BINDSMA
-any problems during birth?
-immunization?
-how is his eating?
-is he thriving and growing normally?
-does he take any medications?
-had he had any allergies
-family history of allergy, asthma, smoking
-where do you live? (if in area with bees and bee hives)
Explain
-your child most likely has a local reaction to bee sting.
-usually presented with redness, local swelling at the site of the sting.
-let me assure you that it was not a severe one in your child’s case and was not life threatening.

Current management
-would like to keep him in ED for observation and to be seen by a specialist.

-such type of bee stings can be controlled by simple measures:


*usually at first scrape the stinger with your finger nail ASAP and you have already done it.
*wash with soap and water
*apply ice and elevate the limb + rest
*give him oral antihistamine for 1-3 days to relieve itching.

Long term management


-Now I am gonna give you further advice about how to prevent further bee sting

-it is important to be aware of these measures as sometime bee sting can lead to reaction we
call anaphylaxis when the child’s condition becomes unwell with difficulty breathing, talking or
swallowing problems, tummy pain, vomiting. So if your child stung by bee again and presented
with any of these symptoms please come to ED.

-to reduce risk of getting stung by a bee


*avoid areas full with bees or bee hives.
*wear close-toed shoes when walking outside
*wear long sleeves, avoid wearing Colourful loose clothes or perfumes which can attract bees.
*tightly cover food container and trashcans.
*when driving keep your windows rolled up.
*have hives or nests near your home be removed by a professional

-I can give you allergic action plan as well….

4R (imp)
Feedback 9-5-2018
Child stung by bees today, sting taken out, uncomfortable and crying kid, had been bitten by
bee before... At time.. Just local reactions... Pefe given in stem, vitals - normal, CVS resp -
normal -
Hx, current n future mx
Outside- question is long, they give all information including PE. so i need to focus on
management
Inside: talk with the parent and reasure that the baby is stable. No need to worry. ask about risk
factor for bee sting as this is the second time. Ask about asthma, eczema and dust, smoking,
allergy risk factor, family history. It is non of these positive. Then explain this is local reaction due
to bee sting but tell the father that there is a severity reaction as well- tell him all anaphylaxis
symptoms but say that this is not his child case. Immiditerly treatment now- histamin, pain killer,
take stiger out. Give for him advice about further prevent bee sting- not go to garden, not wear
colorful cothes, talk about RASH test, anaphylasix kit and how to use it. Remind about red flag
again , 5R. I talk so fast on this case as i afraid i will run out of the time the i finished it so early. I
this not know what to do so i repeat all this information 2 times and tell that he is the great
parent to bring the child in time .
168-Pancoast tumor examination
Young patient with a lump in the neck. Biopsy confirms SCC. Patient knows about the results.
Now has tingling and numbness in right UL.
Task:
-Do upper limb neurological examination
-Do the examination of primary site/source
-Reason behind this/causes/arrange investigations

Examination steps
1-General appearance
-weight loss; cachexia

2-Upper limb neuro (ITPRCS) especially:


-inspection (SWIFT)
-power
-sensation

3-Face
-eye examination (Horner signs: ptosis, myosis, and anhydrosis)

4-Neck
-LN+ thyroid

5-Chest
-inspection
-palpation
-percussion
-auscultation

6-complete examination of skin, mouth and nose, abdomen, DRE, genitalia.

Differential diagnosis
1-pancoast (apical lung cancer)
2-oral cavity cancer (lip, tongue)
3- nasal cancer, esophageal cancer, thyroid cancer
4-kidney cancer
5-uterine cancer

Investigation
-Chest x-ray (2 views)
-CT??
-tumor markers??
-endoscopy??

Karin case
Neck Swelling (Pancoast tumor)
Case: A 55-year-old woman is in your GP clinic, the surgeon has excised a lump (lymph node) on
the left side of her neck and sent some specimens for histopathology that revealed squamous cell
carcinoma.
Task
a. Relevant physical examination
b. Primary and differential diagnosis
c. Order relevant investigations

Head&neck: >90% of cancers are SCC Physical examination


- Inspection: cachexia, obvious weight loss
- Hand: T1 involvement (abduction/adduction), HPOA,
- Head for swelling, nose, ears, mouth
- Eye: Horner syndrome
- Neck: ask patient to swallow and palpate thyroid and palpate LN including axillary and inguinal
- Face: hoarseness and bovine cough (RLN paralysis)
- Chest: palpation and auscultation (reduced expansion/breath sounds/dullness of upper lobe)
- Neurological examination: power and sensation o C5: abduction
o C6: elbow flexion
o C7: elbow extension
o C8: wrist flexion, finger flexion (grip)
o T1: spreading of fingers

- Skin, abdomen, and DRE, genital examination

Diagnosis and differential


- Oral cavity cancers such as tongue, lip, esophagus, mouth, nose, sinus, kidney, uterus

Investigations
- Chest xray, CT scan of neck, chest, abdomen and pelvis, brain,
- Bone scan, tumor markers
- Bronchoscopy or mediastinoscopy
- Endoscopy

Case (7/3/2017)
55 yr old came after FNAC of right neck lump. Chronic smoker for a long time.
Has noticed some weakness of right hand
Task:
-perform focused upper limb neurology
-Perform focused examination to find the cause of the mass,
-Explain dx to pt

C8T1 power reduced


Respiratory exam normal
I said I want to check vision, examiner gave me pic of Horner's.
Case (8/3/2017)
60 yr old man, chronic smoker, has one lymph node on the right side of the neck, Already done
biopsy- shows SCC. He also has right arm weakness.
Task
-perform neurological examination of upper limbs,
-do relevant examination to find out the Primary source.
-explain the possible causes.
Finding-reduced power and hand grip in right side, and I did respiratory examination but while
doing percussion, examiner said respiratory is normal and Moved to another task. I forgot check
ENT and also Horner’ssign (other said when they check for it, examiner showed photo of
drooping of eyelids). I said primary source may be from your lungs, also from skin ulcer in head
and neck, from liver and bowel as well. ( I think I might fail this case because I missed important
points but luckily, I passed.

Feedback 28-11-2017
Neck swelling - pass
Young patient with a lump in the neck. Biopsy confirms SCC. Patient knows about
the results. Now has tingling and numbness in right UL.
Task:
-Do upper limb neurological examination
-Do the examination of primary site/source
-Reason behind this/causes/arrange investigations (cant remember for sure)
(after neuro exam , ask for eye signs, examiner told all other findings are normal,
no need to do resp exam, all normal)examiner showed picture of horners
syndrome
key steps: 2/2 Approach: 4
Choice and technique of exam, organization, sequence: ¾ Accuracy of exam: 4
Diagnosis: 4 Global score: 4 – pass
Examination steps:
-WIPE
-Vitals
-UL exam: ITPRCS
-Skin survery, hairline, scalp,
-Eye examination: Checking for horner syndrome (examiner gave me picture of
horner)
-oral cavity- for growth
-thryroid- normal
-RS: normal
-complete with abdomen ?
Examiner asked primary sites: growth in skin, lungs( apical), thyroid
Feedback 5-7-2018
32 years old female , heavy smoker, had biopsy done for neck LN and
showed SCC , she also complains of tingling sensation of the right arm
Tasks
Do focused neurological examination of the upper limb
Do relevant system examination to locate the primary
Explain the provisional diagnosis to the patient
Young lady with neck mass, biopsy was taken showed SCC , presenting with numbness in the
right arm and armpit.

Task: 1- do neurological examination of upper limb and any other relevant exam

2- tell DDx.

When I entered the room, a young lady was sitting on the chair, I washed my hands, took a
consent from the pt and start neurological exam as in geeky medics then did face inspection,
mouth inspection, asked for eye findings (were of horner syndrome) then neck exam including
the thyroid. The examiner told me that I need to move to the next task, I said that I need to do
resp and abdominal exam, he said nothing. I said the possible origin is either skin cancer or
thyroid or lung cancer.

Passed: global score 4


169-Achilles tendonitis
Refer to lecture 133 & 134 for examination steps

Young lady history of cyclist presented with pain in right ankle for 4 weeks .
Tasks
-Do PE
-explain D/DX to patient

Feedback 8-11-2018 (important)


04/04/2018 recalls
Cyclist and runner having ankle pain for few days.
Task- perform PE
Diag and dds with reasons
Exam for Achilles tendinitis +

Feedback comment
(Pain killers first as he has pain ) so I just told him to walk he had pain I said it’s ok please sit
(a lot of empathy when a patient has pain). then inspection and then check neurovascular
integrity, then check for any tenderness and squeeze the calf from up to down and did
movements and then did THOMPSON test for Achilles.
I didn’t do any test for Morton neuroma or plantar or tilt test as it was acute pain and for sprain
he wasn’t complaining of any pain on the ligaments on palpating so thought better to leave that
too.
Ddx
As he is a professional cyclist most probably it can be Achilles rupture, but the pain could also
be due to Achilles’ tendon inflammation (tendinitis).
it could be due to Metatarsalgia which is also common in cyclist and Stress fracture and Sprain
and plantar fasciitis but the special test most probably is pointing towards the rupture so I’ll
refer you to specialist for ultrasound and in the mean time prescribe some painkillers to
reduce the pain.

Feedback comment
Only tenderness around the tendon. Negative thompson’s effectively excluded tendon rupture
so tendonitis it was. Which you explain as inflammation and injury to the tendon at the ankle
(point it out) because of repeated friction
Muhammed Ali Examination
These are the things I will prefer to do in exam but i have already done with my exam so please
counter check rather then blindly follow me. I don’t want to be a notorious person among IMGs..
just kidding
I will summarize all the cases for PE by one case every day.. good luck to all of you
Examination for plantar fasciitis/ Morton neuroma/ Achilles tendinitis
How m going to perform in exam if I had the case..
Start with gait( look for any pain antalgic gait can be present) ask to walk on heel and toe.. can
be pain full
Inspection: comment on things all of you know about it..
Palpate.. following points are important to palpate as positive findings will be pain in multiple
AMC cases
Calf muscles
Achilles’ tendon ( painful in tendinitis)
Medial malleus and lower end of tibia
Navicular bone ( stress fracture tenderness positive)
Squeeze heel .. calcaneal fracture
Palpate at plantar fascia for tenderness
Lateral malleus and fibulae lower end
Base of fifth metatarsal
Squeeze forefeet
3rd and 4th digit space Morton neuroma
Then active movement
If normal no need to do passive or resisted movement . In case of painful active I will advice no
need to do passive or resisted as well.. like in compartment syndrome case
Based on palpation findings do special tests
Windlass plantar fasciitis
Mulder click Morton neuroma
Simmonds squeeze to rule out rupture
170-Chronic liver disease
55 yr old man comes to your GP for check up. You did some blood tests. Liver function results are
given all normal Alb 32 and GGT 500. Significant alcohol consumption history in the stem
Tasks:
1.Relevant examination of the patient
2.Explain your findings and give reasons

Examination steps
1-General appearance
-mental state
-jaundice
-IV line, drains

2-Vital signs

3-Hands
-clubbing, leukonychia
-palmar erythema, duputryn contracture
-flapping tremor

4-Arms
-spider nevi
-scratch marks, bruising, petechial
-proximal myopathy

5-Face
-eyes (pallor, jaundice, keiser Fleisher rings)
-mouth (fetor hepaticus)
-parotid enlargement

6-Neck
-lymph nodes
7-Chest
-gynecomastia
-spider nevi, telangiectasia (multiple small red swelling)

8-Abdomen
-inspection: movement with respiration, abdominal distension, scars, bruising, scratches, dilated
veins, visible pulsation or mass.
-palpation
superficial tenderness
deep mass
liver, spleen, kidney+ liver span
-percussion ascites
-auscultation bowels sounds, bruits
-complete with genitalia(testicular atrophy), DRE (blood), inguinal LN, hernia.

9-legs
-bruising
-oedema

10-complete with peripheral neuropathy, CVS, temp chart

Differential diagnosis of CLD (do not forget to explain result here)


-alcoholic, non alcoholic
-viral, autoimmune, drug induced hepatitis
-cirrhosis

-These values reflecting the function of your liver. Long term alcohol consumption has caused
some damage to liver. Although the rest of LFT are normal but GGT raised which is very common
in person with history of heavy alcohol drinking.
-Ix: viral hepatitis serology, US + arrange meeting to talk about alcohol.
Feedback 1-6-2018

Multiple lab results given. LFTs mostly. All normal. Only GGT elevated. Hx of chronic alcohol
intake. No subjective complaints. Tasks: PE, Explain lab results and PE findings.
Examination: hands, face, neck, abdomen, legs. Tape measure present for liver span. Mentioned
testicular exam, DRE, Neuro exam to r/o alcoholic neuropathy. I fumbled a bit with measuring
liver span.
All exam was normal. (Some said there was tenderness on deep palpation over hepatic region)
I missed the elevated GGT when I scanned the labs outside…. So I explained all tests are normal,
including examination. Failed this one

Case (29/3/2017)
An alcoholic is here, he said that he will quit alcohol last time but hasn't been able to i guess,
drinks 8-10 SD daily, do CLD examination with findings to the examiner and summarize the to
the patient in the last two minutes
2 mins: recall cld signs
Instruments there: tape, pen torch.
Hi, I am Dr. Maria. I am required to examine you. That would require me to look for certain signs
all over your body and ask you to do certain manoeuvres. Is that all right? I will try to be as
gentle as possible, but if at any time you feel uncomfortable, please let me know. Before I begin,
do you have any pain?
I washed my hands while saying all this. I said I want my patient properly exposed and the
examiner asked me to proceed. The role player was a real patient, I suppose.
General appearance, vitals
Hands: clubbing, erythema (positive), dupytren’s contracture, flapping tremor,
Proximal myopathy (positive)
Face: eyes- pallor, icterus, kf rings…..Parotid swelling, temporalis muscle wasting
Mouth: icterus, fetor, pigmentation
Chest: spider naevi, telangiectasia (there were multiple small red swellings, I was not sure and
mentioned them to the examiner), gynaecomastia
Abdomen:
inspection- distension, umbilicus, dil veins, discolouration, peristalsis.
Palpation: superficial and deep, liver, spleen, bladder, kidneys
Measured liver span with the tape (totally forgot about shifting dullness and fluid thrill)
Told the examiner about checking hernia orifices, testes and pr
Global score- 4

Case (11/11/2017)
Health review
55 yr old man comes to your GP for check up. You did some blood tests. Liver function results are
given all normal Alb 32 and GGT 500. Significant alcohol consumption history in the stem
Tasks:
1.Relevant examination of the patient
2.Explain your findings and give reasons
Introduction…... WIPE
Role player was real patient
Examination of abdomen as in geeky medics / Tally O’Connor

Spider nevi present, sparse body hair, no tremors, flapping tremors, jaundice, parotid
enlargement, fetor hepaticus, gynaecomastia
Liver span 13cm
No free fluid
Spleen, kidneys not palpable
Bowel sounds present, Few veins visible on the abdomen not dilated or tortuous- no venous
hum
I said I would examine testes, spine, supraclavicular nodes.
•Make sure to mention what you see- this pt had a bandage on little finger and had redness in
sun exposed part of the body.
Explained to patient that he has findings of chronic liver disease. Most likely cause is his
drinking, briefly mentioned about fatty change, then hepatitis and final stage is cirrhosis. At
this stage bell rang.
Feedback shows low score for diagnosis/DD: may need to mention other causes for CLD like viral,
autoimmune, drug induced and not just assume drinking as the cause. I feel even if task don’t
explicitly mention DD please take words like reasons, counsel as hints for DD or give DD
whenever possible.
Global score: 4 ( Pass ) key steps 1,2,3,4 pass
Assessment domain
Approach to pt/relative: 5
Choice and technique of examination, organisation and sequence: 5
Accuracy of examination: 5
Diagnosis/ differential diagnosis: 3

Case (11/11/2017)
a man came before to another gp for check up and he is here to check results it was all liver nad
but increase ggt he drinks alcohol 4-8 glasses a day
Task :
a-examine
B-Tell finding to examiner or pt can't remember
Or had many anguima rubi
Asked about testicular atrophy said beg. But I could not feel the liver , I don't know why ? Could
it be cirrhosis? With only elevated ggt? I acted as if I felt it
Handbook condition 70
Recent haematemesis in a 50-year-old man CANDIDATE INFORMATION AND TASKS

You are an intern at the hospital Emergency Department. This 50-year-old patient has presented
having had a haematemesis of about 500 ml_ of fresh blood two hours ago, accompanied by a
transient feeling of light headedness and sweating.

The patient has given you a past history of a previous admission six months ago with a similar
episode of haematemesis which settled spontaneously. An endoscopy was done and the patient
was told there were dilated veins at the lower end of the gullet and was advised not to drink
alcohol. The patient tells you that he has been trying to give up alcohol with limited success.

On the basis of the history you have just finished taking, and his prior episode, you believe that
the patient may have had a haematemesis from oesophageal varices with portal hypertension
and chronic liver disease as the explanation for the current problem

YOUR TASKS ARE TO:

• Perform a relevant and focused physical examination of the patient.

• Explain your actions and what you are looking for to the examiner.

• Describe your findings as you proceed.

You are not required to take any further history.

Aims OF STATION

To assess the candidate's clinical perspective in examining a patient presenting to the Emergency
Department with an acute haematemesis.

To check abilities to examine for evidence of chronic liver disease.

EXAMINER INSTRUCTIONS

The examiner will have instructed the patient as follows:

The candidates have been told that you have vomited up a large quantity of blood two hours ago
and they have been instructed to undertake a relevant physical examination. They are not
expected to take a further history from you. They have just finished taking a history.

On this occasion you vomited a large amount of fresh blood two hours ago — you think it might
have been a pint or so (500 ml_). You felt temporarily faint and broke into a sweat. You have not
vomited since. Your wife has driven you to hospital where the Hospital Medical Officer (HMO)
has taken your history You are lying on the couch undressed to your underclothes and wearing a
hospital gown. The HMO who has taken your history is about to examine you.

You have given a past history of a previous admission six months ago with a similar episode of
vomiting blood which settled spontaneously, and you were discharged after a few days. You had
an endoscopy through the mouth and you were told you had dilated veins at the lower end of
the oesophagus leading into the stomach. You were warned about the effects of continued
drinking. You've been trying to give this up but you have had limited success. It is likely that the
candidates will want to measure your blood pressure and feel the pulses in your arms. They will
also examine your hands, face, chest and abdomen.

In a patient with liver problems, the findings will be evident on examination, and will have been
previously checked by the examiner.

EXPECTATIONS OF CANDIDATE PERFORMANCE

Candidates should first look for evidence of haemodynamic compromise (looking for evidence of
hypotension and postural drop, pulse, peripheral perfusion). Once candidates indicate they
would take the blood pressure, they can be told that the BP is 110/70 mmHg and pulse 90/min.
As the bleeding occurred only two hours ago, this assessment is particularly important.
Candidates should indicate that they would do a rectal examination looking for a melaena stool,
and will be informed there is a fresh melaena stool on the glove.

After assessment of stable haemodynamic status as first priority, the candidate should:

• Put the patient at ease, correctly position him supine with appropriate exposure to examine
the whole abdomen, groin, head and neck and upper limbs.

• Make an appropriate examination looking for evidence of chronic liver disease (examination of
hands — liver palms, leuconychia, spiders, clubbing), easy bruising, spider naevi elsewhere,
gynaecomastia, parotid enlargement, oral cavity and tongue, ascites, portal hypertension
(dilated veins and splenomegaly), testicular atrophy.

• Examine for liver flap.

• Palpate the abdomen adequately for hepatic and splenic enlargement.

• Check for evidence of ascites, by palpation for shifting dullness or fluid thrill.

• Percuss for evidence of liver and splenic enlargement.

• Auscultate abdomen for venous hum, bruit, bowel sounds.

• Provide a logical description concerning the examination.

• Perform the examination in a logical sequence.

KEY ISSUES

• Performing a satisfactory physical examination pertinent to an episode of acute haematemesis


in a patient in whom evidence of chronic liver disease should be sought

• Accuracy of examination will be a key issue for the mark sheet when a real patient is involved.

• Satisfactory commentary to examiner.


CRITICAL ERRORS

• Failure to assess the haemodynamic state of the patient.

• Failure to look for evidence of liver failure and portal hypertension.

COMMENTARY

In this important and common emergency room setting there are three issues the doctor must
focus on:

• Checking the ABC (Airway, Breathing, Circulation) of immediate resuscitation.

• Assessment of the cardiovascular state of the patient and provision of prompt resuscitation,

if necessary.

• Identification of the cause of the haemorrhage.

This patient has had a large haemorrhage and the airway couid be compromised.
In this scenario the patient appears fully conscious and is able to give a detailed history, so it is
unlikely that there is a major problem with the airway or breathing.

Thus the physical examination must start with measurement of the blood pressure (lying and
sitting, if necessary) and pulse, and a clinical evaluation of how well the periphery and vital
tissues are perfused. Is the patient shocked, cold and clammy, with a shutdown peripheral
circulation?

If the patient is shocked, the physical examination must cease at this stage and the patient
must be resuscitated.
Provided the patient is stable, a methodical examination may be undertaken to look for the
cause of the haemorrhage. Although the patient's past history has suggested that the cause is
alcoholic liver disease, portal hypertension and bleeding oesophageal varices, this should not be
assumed as many patients with known varices will be bleeding from another cause. The
examination should look for:

• signs associated with chronic liver disease: • signs of possible liver failure;

• signs of portal hypertension;


• any other clues suggesting a different aetiology for the haemorrhage.

Signs associated with chronic liver disease


hepatomegaly
nail changes (leukonychia),
salivary gland enlargement,
testicular atrophy,
gynecomastia
spider naevi.
If the liver is failing,
ascites
encephalopathy changes. Encephalopathy may have a variety of presentations, ranging from
minor mental impairment and flap, through to coma.

Portal hypertension may be manifest by the signs of hypersplenism (purpuric haemorrhage),


splenomegaly
collateral venous channels (may be visible in the anterior abdominal wall as communications
between the umbilical vein and the epigastric venous channels flowing back into the systemic
circulation. Of more sinister import are the oesophageal mucosal collaterals that form between
the portal and azygos systems through decompression along the left gastric (coronary) vein.

Occasionally, the physical examination will reveal other signs that might be associated with
haemorrhage, for example, the hereditary haemorrhagic telangiectasia associated with the
Osler-Weber-Rendu syndrome.

Variations on this theme are also used, in which a real patient with liver disease is to be assessed
after admission to the ward and institution of an intravenous drip while blood is being typed and
cross-matched. The instructions will in that case state that the patient is now haemodynamically
stable and the emphasis of the task is to assess the patient for evidence of chronic liver disease,
which is expected to be present. The assessment now concentrates on the technique and
accuracy of physical examination as key issues.

In this emergency department scenario the emphasis is FIRSTLY on assessment of stable or


unstable haemodynamic status in a patient with recent haematemesis.

171-Acute abdomen examination


Examination steps
1-ask examiner for hemodynamic stability and check vital signs

2-ask patient for pain severity and offer painkillers

3-Consent
-I understand that you are in severe pain. What I am gonna do now Is to examine you to fine out
the reason why you are having this pain. this would involve me having a look, feeling your
tummy and listening to your bowels sounds.
-I will be touching the painful site towards the end and I will try to be as gentle as possible.
-if you could undress yourself from above to the waist, please?

3-General appearance
-lying comfortably or in pain
-difficulty breathing
-pallor, jaundice
-IV line attached
4-Inspection (from right side of the bed)
-tummy moving or not with respiration
-abdomen distension
-skin discolouration, scars, dilated veins
-visible pulsation, visible mass or peristalsis

5-if upper abdomen pain (then hands and face important) if lower then can skip
-hands (temperature, CRT, pulses)
-face (pallor, jaundice, dehydration)

6-palpation
superficial palpation
* to look for tenderness, guarding and rigidity)
*before touching the painful site tell the patient: I am just gonna touch the painful site very
gently)

Deep palpation
*look for mases
*I am gonna feel your tummy a bit deeper
*if pain only in one of the quadrant do deep palpation for all quadrants except the painful site.
*if pain was generalised skip deep palpation due to pain.

Rebound tenderness
*for localized peritonitis
*I am gonna press deeply on your tummy slightly away from the painful site and then I will
release it quickly let me know if the quick release cause you any pain.

Special tests (3)


*murphy sign (for upper abdomen pain esp. RUQ)
I am gonna keep my hand under your ribs, I want you to take a deep breath for me, let me know
if that hurts you.
*micburney point tenderness, Rovsing, psoas and obturator if lower pain esp RIF pain
*Renal angle tenderness (for all)

7-Percussion and organ examination


-skip percussion and organ examination in acute abdomen cases. But do it in patient with
dehydration assessment as he has discomfort not pain.

8-Auscultation
-bowels sound +/- aortic bruit

9-Complete examination with PR, pelvic, genitalia, LN, hernia orifices.

Differential diagnosis for acute abdomen cases


1-Right upper quadrant pain
-cholecystitis
-cholangitis
-pancreatitis
-hepatitis
-Duodenal ulcer
-MI
-pneumonia

2-Right iliac fossa pain


-ovarian cyst torsion
-appendicitis
-ectopic
-Pelvic inflammatory disease
-pyelonephritis
-kidney, ureteric stone
-crohn IBD

3-Epigastric
-pancreatitis
-Duodenal ulcer
-Gall bladder, liver
-MI
-pneumonia

4-Left iliac fossa pain


-diverticulitis
-all RIF pain causes except appendicitis

5-Peritonitis (inflammation of membrane surrounding organs and the abdominal wall)


-ruptured viscus (ruptured diverticula, perforated ulcer)
-pancreatitis
-mesenteric ischaemia
Feedback 18-9-2018 acute abdomen PE
Young girl companying of lower abdominal pain both left and right side for two days. Have passed bowel
since and few flatus. LPM is 3 weeks ago and it was normal.
Task: PE, Dx and D/D
WIPE
Asked vital sign and give pain killers
Check from hands, hands and head are normal no paler jaundice.
Abdomen: inspection: normal
Palpation: tenderness on superficial palpation, most severe if RLF
More tenderness on deep palpation
Mc burning point rebound tenderness +, rosving sign +
Murphying sign -ve
Ask from examiner:
Hernia orifice, DRE normal
Pelvic examination: both side adnexal tenderness

Dx and D/D: appendicitis, ovary cyst, ovary cyst torsion, ovary cyst rupture, bowel obstruction. PID
Score:5
Key step: yes/yes/yes/yes
Approach: 4, choice of the examination: 4, accuracy: 4, Dx and D/D: 4

Feedback 26-10-2018 Abdomen exam Passed (GS – 5)


Man with abdominal pain, acute onset. History of appendicectomy.
Task: perform relevant exam. Explain DDX
PE:
Offer pain killer
GA
VS (temperature is increased)
Dehydration status: CRT, skin turgor, pallor, oral mucosa (Normal)
Abdomen –
Inspection – NAD
Palpation - localised TDN, guarding on LIF on both light and deep palpation
Percussion – As my patient is in pain and no features of abdominal distension, I’d like to skip percussion.
Auscultation – BS present.
Offer DRE exam: Examiner asked me what I am looking for.
On inspection, any ulcer, prolapse. (No)
on DRE, any mass, blood on my examined finger? (NO)
I finished my exam.
DDx
Diverticulitis, diverticular disease, acute GE due to bacteria or parasite infection, IBS, IBD.

Feedback 18-9-2018
You are an HMO working in ED and the next pt is a 32 year old lady who was
brought by her relatives for abdominal pain for the last 3 hours. She has taken
some painkiller but its not working. She has passed bowel once and also some flatus
since then. After doing the primary survey you started her with some pain killer
and ordered some investigation. The pt is NPO, and fluid is running.
Your task is to
1. Perform focused abdominal examination to find out the cause.
2. Do other relevent examination if needed.
3. Ask the examiner for any specific result if you need( examiner will only give you
findings what you have asked for)
4. discuss the Dx and DDx with the pt with reasons.
My approach:
As it was mentioned in the stem that I already gave her painkiller so I didn't offer any. But I told her that
the pain killer will take some times to kick in, can you bear the pain. she said yes. Then I asked the
examiner about GA and vitals which were normal. Then I started with abdominal exam.
On inspection there was movment restriction. I palpated and It was painful. So I asked that would you let
me do a little bit deep pressing over your tummy? she said ok. So I tried deep palpation and she showed
tenderness so I stopped and asked her to cough. Cough was painful so I commented that rebound
tenderness is present and thats why I am skipping deep organ palpation. Examiner nodded his head.
Then I quickly auscultated the heart and abdomen fro bruit and bowel sounds.
Then asked the examiner about the DRE and PV exam result which were also normal. Hernial orifices
normal.
Then I explained her with a digram of abdomen that there are a number of organs in our tummy and any
problem with any one of them could cause this symptoms.
Then I explained PID, Appendicitis,Small bowel obstruction, Constipation, ovarian rupure,
oavarian cyst.
But i gave Dx of Smal bowel obstruction ( i don't know why I did that stupidity. It was PID )
Anyway my DDx were correct so I was saved.
AMC Feedback:
Station 09: Abdominal pain
Grade: Pass Global score: 4
Key steps: 1,2,3,4 yes,yes,yes,yes
Assessment domain:
Approach to pt/relatives: 6
Choice & technique of examination, organisation and sequence: 5
Accuracy of examination: 5
Dx/ DDx: 3

172-Vasovagal syncope
Case 1 (2017-2018) 17-year-old girl brought in to ED following unconscious collapse- recovered
within 30 seconds. She was watching a horse parade at the time. At ED she is stable.
Task:
-History
-PEFE
-Diagnosis

History (similar to prolonged QT with little modification)


1-collapse questions
-assure the patient that she is stable at the moment. How she is feeling? Take consent to ask
some questions?
-can you tell me more?
-Did you lose your consciousness completely? How long for? (30 seconds)
2-Before
-what were you doing before it happened? (Watching horse parade)
CNS, Vasovagal
-did you get any warnings like dizziness, nausea, strange smell or light-headedness? (Migraine/
epilepsy/ vasovagal) (it is preceded by lightheadedness after prolonged standing)
-did you get any headache, blurring of vision, weakness? (TIA/ Stroke)
-Any fever or recent infections (Meningitis)
Cardiac
-have you had funny racing of the heart? (Arrhythmia)
-any chest pain, SOB, dizziness?
Endocrine
-did you skip your meals? When was your last meal? (Hypoglycemia)
any head injury
3-During (epilepsy)
-anyone witnessed and noticed any jerky movements? (Her friend)
-any change in colour of lips or skin?
-have you passed urine or soiled yourself?
-have you bitten your tongue?
-have you injured yourself?
4-After
-how long does it last until your recovery? (Uneventful)
-after recovery do you feel normal or drowsy? Any N&V or headache?
5-previous episodes
-has this happened before? How often? Can you tell me more like what were you doing at each
episode and are you aware of why they happened? (This is the 3rd attack. Previous 2 episodes
happened when I saw blood after a self-inflicted cut and when I had flu)
6-General
-PMH (are you DM, do you feel excessive thirst, polyuria), PSH, SAD
-medications
-family history of sudden death? + Stressors

PEFE
1-General appearance
-pallor, cyanosis, dyspnea
2-V/S + postural hypotension
3-JVP, feel carotid bruit
4-Heart
-thrill, heave, apex beat
-heart sounds (s1, s2) + added sounds
-heart murmurs (if any ask site and radiation)
-listen carotid bruit
5-chest
-chest movement, air entry, breathing sounds, dullness, crackle or wheeze.
6-CNS
7-abdomen
8-sacral and pedal oedema
9-office tests
-UDT
-BSL
-ECG

Explain
-from diagnosis, you most likely have a condition called vasovagal syncope when you have
sudden LOC with rapid recovery because of prolonged standing that is because the vagal nerve
one of the nerves of your body becomes over active to certain triggers such as sight of blood,
extreme emotional distress, prolonged standing, pain, fear, heat).
-other possibilities are
*hypoglycemia or low blood sugar level but you did not skipped meal and your BSL is ok.
*postural hypotension or sudden drop in BP from sitting to standing but less likely from the
history.
*heart disease, CNS disease but less likely from history and examination.

-Do’s & Don’ts


*Don’t skip breakfast esp prior to activity/ avoid prolonged standing
*Hydration
*get up from sitting/lying slowly

Management
-I would like to keep you in hospital for observation and seen by specialist.
-do some blood Ix like FBE, UCE, LFT, TFT, BSL, ECG

Feedback 30-5-2018 Loss of consciousness’ Vasovagal. Feedback: Pass


17yof c fainted 30sec attending a parade witnessed by friends. No ECG/BG & benign Hx. 3 rd
episode classic VV.
Task: Hx, PEFE, Dx
HOPC
-Basic rapport and asked if it’s ok to ask her mates some Qs too
-Syncope approach: pre/during/post
-Confirm LOC
Pre
*what were you doing
*any n/v/d/HA/SOB/CP/racing heart
During
*How long did it last.
*Any jerky movements/tongue biting/incont/injuries.
Post
*get back up yourself
*fully aware of your surroundings after
*injuries/incont
*any CP/SOB/racing
-1st ep/any similar PHx –yes asked further & specifically re postural
-Ddx Qs
*TIA: HA, blurring vision, difficulty walk/swallow/speak, dizziness, pins & needles, weakness
*any FHx of sudden (cardiac) death[HOCM, WPW]
*Trauma: any recent head inj
*Ca: wt loss, lumps.
*are you DM, do you feel excessive thirst, polyuria
*any recent v/d/bld loss from anywhere, friends say you’re pale, skip meals, colour change in
skin/gums/abdo pain + blackouts
*PSY: SAD  any stressors in your life, Anx, Dep
*PHx, SHx, FHx SADMA
*LNMP(any issues)/HPV etc

O/E
-GA Vitals[BP w post drop] BMI
-CNS>CVS>RS>Abdo>Wards: uDip BG ECG
Ax & P
-‘Vasovagal’…vagus nerve…
-Sometime we suddenly collapse…there’s a nerve in our body which when stimulated causes us
to collapse…exact cause unknown but it is a benign condition, I’ve assessed you thoroughly and
found no major problems
-Do’s & Don’ts
*Don’t skip breakky esp prior to activity/prolonged standing
*Hydration
*get up from sitting/lying slowly
*who’s picking you up/can I call someone for you
Case 2 (2015-2016)
15-year-old girl marathon runner syncopped after running 100 m in hot day.
Task
-History
-PEFE
-dx/ Mx

Positive in history (she felt hot and suffocated)


in examination BSL= 3.1

Explain
-based on history and PE findings, most likely why your daughter fainted is because of
hypoglycaemia or low blood sugar level. However, as she also felt hot and suffocated, it could
have been what we call a vasovagal syncope.
173-Funny turn alcohol
HMO in Ed when 22 years old girl brought in by worried parents that she had funny turn this
morning. Now she Is fine and vitally stable.
Tasks
-History (6 minutes)
-PEFE card
-Dx and Ddx with reasons

History
1-Approach
-how are you doing now? any dizziness or drowsiness? (feel good)
2-Before
-can you tell me what do you mean by funny turn?
-what were you doing at that time? (She felt grodgy and gazed this morning while she was
opening the fridge but do not remember anything)
-have you lost your consciousness? (Yes). for how long?
-has this happened before? How often? (I had 2 episodes of seizure when I was a little girl)
do you remember if you were febrile at that time? What was the diagnosis? (can’t remember)
-did you get any warnings like feeling nauseous or strange smell?
-any BOV, headache, fever?
-any weakness, numbness or tingling?
-any palpitations, chest pain or SOB?
-do you black out or feel dizzy when you stand up quickly?
-did you skip your meal? When was your last meal? (Missed a meal this morning but ate
sandwich in the hospital)
-any trauma to the head by any chance?
3-During
-anyone witnessed any jerky movements? (jerky movements in hands when fell on the floor
noticed by parents)
-for how long did it last? (30-45 seconds)
-did you passed urine or soiled yourself?
-have you bitten your tongue? Rolled up eyes?
-did you injured yourself?
4-After
-did you wake up feeling normal or drowsy?
-recovery time?
-nausea and vomiting, weakness, headache?
5-General
-PMH (DM, HTN, epilepsy, heart D)
-medications
-SADMA? (+ve alcohol wine 2-4 drinks a day)
-any binge drinking? (last night she was partying with her friends boozed 2 sherry, 1 full bottle
champagne and few beer)
-family history of sudden death, epilepsy?

PEFE card all normal

Diagnosis and Ddx


-most likely diagnosis is seizure or fits due to binge drinking. As you were boozing the previous
night. Binge drinking can cause imbalance of mineral body salts and fluid in the body and
dehydration effects leading to your symptoms. Binge drinking can also cause low sugar level in
the blood because leading to fit.

-could also be due to alcohol withdrawal causing fit; as it seems like you drink alcohol above the
safe limit for a period of time. This can be presented with fit after stopping alcohol within 36-72
hours.

-could be due to trauma , heart problems, infection but less likely as you have no trauma history,
no chest symptoms or fever.

Feedback 21-7-2018
Funny turn – FAIL
Stem: Young girl, had a funny turn this morning. Fine now.
Tasks:
-History for 6minutes
-PEFE – on card
-DDx
Please refer to the feedback of someone who passed. It was my last station. Came out with a
funny and incomplete feeling. Quite expected I’d fail this one. Thankfully this was the only one.

22 year old lady with funny turn at home 3 hours ago, now stable. Hx. Dx and dd

Feedback 1 (21-7-2018)
Dear friends i passed my clinical on 21 July. Heartfelt thanks to this group and all my study
partners.
I saw the detailed feedback given by my fellow friends on the same day.
I would like to add about the funny turn case as i passed it with 5 score.
Case: Ed setup, you are an intern and 22 yr female bib father who noticed funny turn today
morning.
Task : elicit history for 6 min, pefe on card and give the likely diagnosis and dds with reasons to
pt.
I entered room, greeted pt and asked how she is doing now. Any complaints of dizziness or
drowsiness.
She said she is feeling good. Asked her to explain what does she meant by funny turns.
Roleplayer explained everything in a flow, she woke up in the morning late, get out of bed and
went to kitchen, opened the fridge to get some water and suddenly fell down and her parents
noticed some jerky movements of the hands and she also had loss of consciousness for about 30
to 45 seconds.
I elicited history like pre, during and the post event. No significant history.
First episode of its kind, no relevant history. I missed febrile seizures but she gave a positive
febrile seizure history when asked.
I tried to rule out epilepsy, migraine, tumour, trauma, infections, arrythmia, hypoglycemia and
vasovagal.
On SADMA history she said she used to consume 2 to 4 drinks per day and the previous night
she had a late night party and boozed like 2 sherry shots, 1 full bottle champagne and few beer.
Not a smoker or drug abuse.
No relevant family history.
Pefe all normal.
I gave my above mentioned differentials and said the most likely diagnosis is syncopy because of
hypoglycemia and the binge drinking.
I explained my diagnosis like as you were boozing the previous night and it seems you didn't
have your breakfast yet most likely it could be due to low sugar because of the binge drink and i
will inform my senior regarding this as it seems very concerned to me.
Friends, later i discussed with other friends and came to know that the diagnosis must be said as
seizures due to binge drinking.
I got 6 in history and 4 in diagnosis and dds. I was bit confused between syncopy and seizures
during my exam but i said syncopy as my diagnosis. I hope this helps. Thanks again to everyone.

Feedback 2 (21-7-2018)
I passed that case!! The approach was similar ruling out a convulsion! Also asking for fever or any
cold previously!! My dx was a fit due to binge drinking and ddx was infection, illicit drugs,
“epilepsy” because she was having hx of febrile convulsion (however i explained was less likely
and the dx was not just with one episode) also hypoglucemia or trauma ! They score 6 in that
case and in ddx!

174-Giardiasis
Female has been having chronic diarrhoea for 3 months
Task
-Hx
-Ddx (giardiasis, celiac, IBD,IBS, medications, lactose intolerance, infection)

History
1-Diarrhoea Qs: Duration, frequency- amount, consistency, content- colour, smell, flush-
urgency, severity.

-Duration? (3 months)
-How often do you pass stool per day?
-Are you passing large amount of stool?
-Is the stool hard, loose or watery? Greasy or bubbly? (Watery, bubbly)
-Any blood, pus or mucus?
-What’s the color? Is it pale?
-Is it smelly? (Smelly)
-Is it hard to flush?
-Do you have to race to the toilet to pass stool?
-Does it wake you up at night?

2-Associated symptoms
-Tummy pain
-N/V
-Abdominal distension, bloating?
-Fever, rash, joint pain?
-LOW, LOA, lumps and bumps?
-Waterworks

3-general Qs
-PMH e.g celiac
-PSH
-Medications
-Diet, are you on special diet?
-Fhx of celiac
-Travel
*Ave you travelled recently? (Yes to bali)
*When did you return? (4 months ago)
*Did you drink unbottled water or eat local food?
-Sexual hx (safe sex)
-Contact – have you been in contact with similar sx?
-Occupation
-Psychological hx if suspect IBS

Explain
-From history there are several causes why you have chronic diarrhoea which means diarrhoea
for more than 2 weeks.
-The most likely cause is a condition called giardiasis.
*This is an infection of small bowel by bug parasite called giardiasis lamblia.
*This is one of the travel related infection and usually transmitted by drinking contaminated food
or handling anything contaminated by this bug.
*It usually presented with diarrhoea, which can last for week causing greasy bubbly smelly stool.
*It is self-limiting in most cases. Can prescribe metronidazole for 7 days

Other possibilities but less likely


*celiac disease or sensitivity to substance in food called gluten.
*IBD,
*IBS
*any other infections
*medications.
The most likely one is giardiasis.
Feedback 23-6-2018
Feedback: Diarrhea: female has been having chronic diarrhea for 3 months. Take Hx, DDx.
Hx:
Chronic Diarrhea History: Coeliac Disease? (no), infectious? (no), IBD (no), IBS (no), Giardiasis
smelly, bubbly, travelled to Bali right before the diarrhea).
Malignancy: low, los, loa, lumps and bumps, family hx.
Past hx: SADMA,….
Most likely Dx: Giardiasis  explain to the pt.
DDx: Coeliac, IBS, IBD, infectious diarrhea.
Grade: PASS, Gs: 5
Key steps 1,2,3,4,5: all covered (Y).
Approach: 5, Hx:5, DDx: 6
In conclusion, I passed with 11/14.

Feedback 13-12-2018 Diarrhoea: Pass


2 min Outside: Don’t remember the stem exactly, but patient has diarrhoea since few months.
Tasks: History, Dx and DDs
Thinking Outside: As this is a chronic diarrhoea, I need to put DDs of Chronic diarrhoea, such as Coeliac,
giarrdiaisis, amoebiasis, IBS, IBD, Causes of malnutrition and malabsorption such as previous gastric
surgery, hyperthyroidism and psychogenic.
Inside, young male was sitting comfortably.
Me: I m so sorry to hear about diarrhoea. Is it ok if I ask you few questions to come to diagnosis so we can
help you better. Pt: ok.
Me: Can you pl tell me more bout your diarrhoea?
Pt: Got this diarrhoea since last 2-3 months. ( that’s all he said)
Me: How many times a day (4-5) (I think pt said not every day, I may be wrong), color (yellow),consistency
(watery sometimes semi liquid), Any blood (no, to rule out amoebiasis, IBD), , float on water (yes), difficult
to flush (sometimes), any mucus (no), weight loss (mild, I don’t remember how much). Any food relation
such as wheat, barley, rye (no), any medication to relieve (no)
Me: R u taking any antibiotics (no), weather preference (no), alternative constipation with hard ribbon
pallet like stool and diarrhoea (no, to r/o IBS).
Me: any work or family stress (no, to r/o psychogenic and IBS)
Me: any gastric surgery (no).
Me: Any recent overseas travelling?
Pt: yes, Bali, 3 months earlier.
Me: Where did u stay? Pt: In good hotel.
me: where did u eat Pt: Mostly hotel.
Me: Did you eat street food or drink non- bottled water. Pt: no (here I got confused as I have made up my
mind for Giardiasis)
I asked about present, past and f/h according to Karen’s s here I had lots of time because of history and Dx
and DDS only.

Dx: Me: from the history, I believe you r suffering from condition called giardiasis, have u heard about it?
Pt: No

Me. It’s a infection from a bug called giardiasis. It is transmitted by faeco-oral route. You may have got it
from food in Bali. Although you have not eaten outside it is still possible to get it.
There could be other conditions, these are all less likely such as Amoebiasis, other bug but less likely as u
don’t have any blood in stool, Coeliac disease where u are sensitive to gluten (less likely because it does
not occur after certain food), IBD less likely bcas no blood in stool, IBS less likely due to lack of stress and
constipation, thyroid less likely due to no symptoms for thyroid disease.

Global: 6 Key steps: all 5 Yes


Approach to Pt: 4
History: 6 ( I think I have asked lot more in history as I have lot of time due to less tasks), but I don’t
remember now)
Dx/DDs: 6

175-Scrotal examination
Examination steps
1-Introduction+ explain, consent, chaperone, undress

2-Inspection (ideally standing position)


Inguinal area
-swelling and if +ve cough
-scars
Penis
-rash, ulcers, discharge
-urethral opening
-shape and deformity
-retract foreskin looking for ulcers
Scrotum
-skin rash, ulcers, scars
-varicocele
-position of testis
-asymmetry, enlargement

3-Palpation (down to top)


testicles epididymis spermatic cord LN
Mass
-can I get above the mass?
-if it is related to testis or not?
-description (site, size, surface, regularity, consistency, tenderness, mobility, temperature)
-LN

4-Special tests
-Transilumination test (+ve in hydrocele and epidydmal cyst)
-prehn sign (+ve in epididmoorchitis)
-cough (+ve if hernia)

5-complete examination with examination of tummy, hernia orifices and DRE.

Differential diagnosis
1-Hydrocele
2-spermatocele
3-epidydmal cyst
4-varicocels
5-cancer
6-epidymoorchitis

If epidydmitis
-look (ulcer, discharge, swelling, redness of skin)
-feel (mass, tender, soft, separated, can get above, warmth)
-special test (transillumination + or – depending on hydrocele / prehn test +
-office tests (urine dipstick, swab if discharge or ulcer, blood tests, US

If cancer
-look (asymmetry, enlargement)
-feel (mass, hard, fixed, non tender, can get above)
-special test (transilumination +ve)

Recall 2-3-2018
Guy concerned that he may have contracted STI. Take consent, do genital exam, further inv. Male
pelvis dummy. (?) Scrotal mass, +transillumination test (?)

Feedback 4-4-2018
Station 4-SCROTAL SWELLING- PASS
50 yr old man with scrotum enlargement for months, its painless, no problem in urination
Tasks- do relevant examination
Most likely diagnosis with Dds

Investigations.
On table torch and measuring tape was there.
On exam there was small nodule on top of left testes. It was hard and round but
transillumination was positive.
I said to examiner i want to measure the size of lump but he said that original size is much larger
than its present on manequin and exact value is not available. Transillumination was positive
when i did from behind.
i said Hydrocele but told all other DDs of lump as well. Passed case

Feedback 30-5-2018 ‘Testicular lump’: Feedback: Pass


It’s a supine mannequin, so say what you would do ideally etc…

• O/E

o Rapport/explain/pain etc

o Examine in 2 positions, standing then lying

o Inspection

o Standing:

 Expose>chaperone>gloves

1. Penile skin changes > retract – balanitis/phi/paraphimosis

2. Scrotal changes > Assym R vs L in size > overlying skin/ulcers > swelling more on R/L

3. Turn head > cough > bulge/bag of worms (varic)

o Palpation

 Temp – compare ea nut to thigh

 Scrotum

• I’m going to examine the testicle 1st

• Ant/Post(‘Ca mostly at this


pole)/Upper(spermato/epididymocele)/Lower/Medial/Lateral pole

• Now outside the testicle

o No swelling btw the testes

o My case had hydrocele

o **this was a WEIRD mannequin, the hydrocele felt asymmetrical and felt as if the left
testis had been pushed to one side…it was discrete and no obv lump could be felt on the testis:
so I didn’t dare comment on it thinking it was just the way that mannequin was. So I just stuck to
hydrocele approach.

 Lump
• Site Size Shape Surface Skin TT Consis

• Get above it

• Cough

• Transilluminate

o Lying

 Hernial orifices

 Inguinal LNs

 Femoral pulses (r/o fem aneurysm)

 Cremasteric reflex – lost in T Torsion

*My role player and examiner felt to me like they were a little uncooperative or mocking
(whatever), I felt like slapping the role player cos he’s getting paid for this sh*t. I managed my
frustration and stuck to the approach, did everything then summarized the findings to the pt w
clinical correlates and that most likely is Hydrocele –said it’s common etc w brief description of
types of scrotal lumps. Then gave 2 other less likely Ddx (growth/inflam) and said that I didn’t
mean to scare them, just being thorough. Said we’d enlist the help of USS and +/- specialist to
confirm my suspicion and for safety’s sake r/o other Ddx. I thought for sure that I’d bombed this
case and was furious (w a smile on my face for the Dbags assessing me) coming out of the
station.

My case (27-6-2018) pass


50 year old man with gradual painless swelling of left testis
tasks
1-perform scrotal examination
2-dx and ddx
3-management

Approach
there was a patient sitting on the chair. On the bed there was a dummy covered with a sheet.
on the table on the end of the bed there was a torch, measuring tape and gloves

Task 1
hi--- mu name is – I’m on of the dr here I can see that you are complaining of a painless swelling
of your left testis is that right?
In order to know about the diagnosis and possibilities I would like to examine your testis and
scrotum this would involve me inspecting your genitals, placing my hands and feeling the
testicles. Would that be alright?
a male staff will be acting as a chaperone during the examination
if you could now go behind the screen and undress yourself from the waist down and cover
yourself with the sheet provided and when you are ready let me know.
I’m gonna wash my hands
the examiner said he is ready and was exposing the dummy while I was putting on the gloves.
starting by inspection and once I was telling the finding the examiner asked me to tell the
findings to the patients
so by inspection
groin- no swelling
penis- no ulcers, deformity, discharge, redness
scrotum- swelling of the left side , no redness or ulcers

Palpation
- I can feel above the welling so it is unlikely that its origin from the tummy or above the testis.
- then started feeling the testis-epidydimis-- spermatic cord
I can feel a lump on the top of the left testis, its soft in consistency regular border not tender
normal colour of skin overlying the lump then measure the size using the tape measure
-feel the temperature.
-as I started feeling the Ln on dummy (examiner skip)

the first bell rang

Task 2
told the patient that there could various reason why you have a lump on the top of your left
testis it could be nasty growth then the patient asked what is that ? I told him could be a cancer.
It could also be due to infection or trauma so I need to do transillumination test which I couldn’t
do it because the bell rang for the task.
the examiner said to me do it then
so I took the torch and illuminate from behind the swelling transillumination test was +ve.

Examiner I would like to complete examination with examination of tummy, hernia orifices and
DRE.

so I told the patient that the swelling illuminated with light so it is most likely a hydrocele which
is accumulation of fluid it could be due to trauma, infection or cancer.

Task 3
let me assure you are in safe hands I just want to arrange for some Ix to reach the diagnosis and
rule out cancer.
I would like to refer you for US, do some blood test and see you again once the Ix results appear.
Feedback 5-10-2018 Scrotal ex hydrocele

Station 9 (Hydrocele PE)

Question: A middle aged man came with scrotal swelling for several months.

Task: Perform relevant physical examination to the dummy and explain what you are going to do
to the patient. Tell the possible causes to the patient and explain about investigations you would
like to order to the patient.

Score: I passed this case with global score 6 (which was totally unexpected for me, you can look
at my performance review why I thought like this)

Key steps 1,2,3,4 : All Yes

Approach to patient: 5

Choice and technique of examination, organization and sequence: 5

Accuracy of examination: 5

Dx/DDx: 5

Choice of Investigations: 5

My performance:
Hello…. David. I’m Dr MM, one of the doctors in this clinic. Nice to meet you. I knew that you
have some swelling in your private part. So I’m here to examine you today focusing your private
part. My PE will involve look, feel around your private parts and perform special tests as
necessary. Are you happy to go ahead, David? Ok, Now can you please take off your lower half of
clothing including your underwear and used this sheet to cover your private part for a moment.
Meanwhile, I’m going to call one of my nursing staffs to act as a chaperone through my
examination and I’m going to wash my hands and wear gloves. OK?

Ok, David. Let’s start the examination. Can you stand up? I will remove your covered clothing
now. (The dummy was in lying position and the examiner told me to examine at this position
only)

On inspection, I cannot see any prominent scrotal swelling but I can see right testis obviously. I
cannot see any rugosity on both sides according to this dummy. There is no redness or ulcer on
the scrotal skin. (I lifted up the scortum.) I cannot see any abnormality on the back of the
scrotum as well. Ok, David do you have any pain in your swelling. (No) Ok, if there is any pain, let
me know at once. OK? I started to palpate around both scrotum. Up to this point, I don’t know
which side is the problem. By the way, David, do you know which side of scrotum did you notice
swelling? (I don’t know doctor, you have to examine it). Ok, I will, David. On palpation on right
side, I can feel right testis which is firm to hard in consistency. On left side, I can feel left testis as
well which is seemed to be normal. Can you cough for me, David? (The examiner interrupted me
that expansile cough impulse is negative) Dear examiner, I want to examine inguinal lymph
nodes. (She told me no inguinal lymph node enlargement) Ok, David, I’m going to perform some
special tests, ok? I did fluctuation test on left side. Dear examiner, fluctuation test is negative. So
do I need to perform transillumination test, examiner since fluctuation test is negative. (You can
do it if you think it’s necessary) Ok, then, I’ll do it to make sure, examiner but I have to dim the
light very properly. (at that point, role player stood up and told me he would help. He folded the
sheet covering the dummy at the beginning of examination and cover it on the dummy again. I
had to look the dummy covered under the folded sheet and used the pen torch and found that
transillumination was positive on left side and negative on right side. (only at that time I realized
the case was hydrocele on left side) Dear examiner, since I was not sure about my previous
fluctuation test and I’ll examine the test again. I repeat the fluctuation test on left side again.
Dear examiner, fluctuation test is positive on left side.

Ok, David, according to my examination findings, you have a condition we called hydrocele on
left side. Let me draw a picture and explain more to you. This is the scrotum inside which lies the
testis. There is a fluid collection on the side of the testis like this we called hydrocele. Are you
with me so far now? Ok, David, other possible causes are collection of pus or blood or a blood
tissue like fluid we called lymph in this area. But they are less likely because when I shine the
light on the swelling, I can see clear fluid. There’s a structure we called epididymis in the
scrotum. Sometimes fluid filled sac formation in epididymis we called epididymal cyst can be the
cause but it’s less likely.
Regarding hydrocele, we have to find underlying causes like injury, infection or nasty condition of
testis. So I have to order blood test and urine test to rule out any infection in the swelling or in
private parts. And also you need ultrasound imaging on your private parts to rule out nasty
condition like testicular cancer. If none of the causes is found, it would be primary hydrocele
without underlying cause. (Doctor, do I need some advanced tests, as well?) Ok, David, that’ll
depend on the results of basic tests. If there is any suspicious findings like features of testicular
cancer on ultrasound, only then we’ll order some advanced blood test and imaging for you. Ok?
Any point you want me to repeat again? (No, doctor). Then the bell rang.

Comment: I swear my performance was exactly like above. I forgot to mention about scrotal
swelling on left side throughout inspection and palpation as I’m not sure which side is abnormal.
I even thought about testicular cancer on right side as right testis on the dummy is so obvious. I
got the diagnosis only when I did transillumination test. I had to say sorry to my examiner and
repeated some tests and corrected my mistakes during my examination. As I didn’t realize the
swelling on left side, I didn’t perform any detailed scrotal swelling examination, either. Gaining
such high score is a miracle for me. I truly, deeply am thankful to both examiner and role player
in this case.

Feedback 7-12-2018
30 years old male with pain in scrotum .

TASK - examination , explain finding to patients (im sure about this task- we have to explain what
we found ), invest, d/ddx.

ALWAYS KNOCK THE DOOR.

I entered in, greet both of them (forgot patient name this time also ) explained as your are
having some pain in private would like to examiner so that can be treated accordingly.
WASH HANDS AND WIPE WITH TISSUE PAPER

patient was sitting beside mannequin (which is very important ), started whether in any pain ,
vitals (no temperature)

i said ill examine you both in lying and standing position which position are you comfortable to
start with- examiner said do in lying .

started with inspection(examiner- what are you looking for -- i said scrotum (he said its swollen
and red , you can proceed)

then i started giving running commentary to examiner im looking at inguinal area for any
swelling(always start with ingunial area), he said no, then i said im looking at penis for any ulcer
, discharge,( examiner said - there is pus at end of penis, i asked ccvo(examiner was white
colour ,mild offensive)
i asked to retract glans (examiner said normal ) started
commenting in scrotum inspection red and rt is larger than left .

i asked patient to elevate scrotum so that i can see beneath the scrotum .i asked whether pain
reduced while lifting (pt looked at examiner - examiner said yes pain decreased ),.
Palpation - (as task was to explain to pt) was palpating and explaning him that right is little big
than left , can feel bumps (lobulations) (ups and downs )(tired every thing to explain him
epididymitis )testis felt separately, left was normal.
transillumination negative (rolled paper for scope)(examiner was standing beside me and
watching everything )

cough test (examiner said negative)

examiner was pushing me to explain patient , so started drawing picture testis, epididymis, said
inflammation of epididymis explained other like hydrocele, variocele, cyst of eidydimis .
investigation:cue, urine culture, pcr of first urine, other STI screening.

GlOBAL 4 (approach 4 , Choice and technique 4, accuracy 5,, d/ddx 4, choice of investigation 4)

Feedback Health review - PASS


22 year old male patient presents with scrotal pain.
Task: Take consent
Perform PE
Dx and DDx
Investigation

2 min thinking: DDx – epididymo orchitis; torsion; STI; UTI; mumps; trauma; to mention about chaperone
Physical examination
Intro
Took consent and mentioned about chaperone
-Wore the gloves (patient handed me 3 boxes and told me that “choose whatever fits you; I have to wear
large sized gloves; I chose medium ones as his hands looked big but medium sized gloves were too small
even for a person like me with small hand. Wasted about 20 secs taking off and wearing gloves)
-General appearance
-Now can you please stand up for me? I would like to have a look around scrotum first.
Examiner said “On inspection, there is a swelling on the left side of the scrotum. The swelling is tender and
temperature over the swelling is raised”
Thank you. Now, *name*, can you turn ur head towards the left side and give me a cough? Cough reflex is
negative. Overlying skin looks normal, no redness, no dilated veins. No high riding scrotum. There is no
discharge coming out from the penis. Then the examiner said “there is pururlent discharge from the penis.
What else do you want to look?”. I told him I wanted to check whether there are any ulcers or rash around
the penis as well. “Nil”. Thank you.
-Now, would like to do palpation. Examined the right side first. No positive findings. On the left side, I
could feel the testis, which is slightly tender, firm. There also is another swelling on the lateral border of
the scrotum which is about 2x2 cm, non-tender, firm in consistency, non-fluctuant; smooth; rounded; well
defined edge; not mobile). Asked the patient to cough, examiner told me cough reflex is negative. I can get
above the swelling and feel the cord.
-Next, told the examiner that I would do Prehn’s test. Said positive.
-checked inguinal lymph node on the dummy – negative
Dx and DDx
As mentioned above with lay terms for each of them
Investigation
To confirm the dx, would like to do some tests; Blood test – FBE; ESR/CRP; urine sample to check whether
there are infection in the urine; take sample of discharge from the penis; USG to check the swelling which I
felt over your left scrotum whether it is solid or fluid filled cyst. Btw, are u sexually active? (YES) do you
practice safe sex? (NOT ALWAYS) have you ever been diagnosed with STI in the past? (NO). Well, this
condition can be associated with STI, so if you agree, would like to add STI tests to the investigation lists.
(no problem, I agree) Understand? Any questions?

Told him about avoiding intercourse until the results come back and the need to use condoms in the
future.
I finished the case a few seconds earlier. So re-read the tasks again. Then examiner told me to check the
tasks again. I was startled cuz I was sure I had everything covered. He asked me what inx are u gonna do?
So, I had to repeat all the tests I just mentioned to the patient. Bell rang.GS – 4 Key steps – all covered
Approach – 4
Choice and technique, organization, sequence – 4 Accuracy of examination – 4
Dx and DDx – 4 Choic of investigation – 4

176-Hematology examination
Examination steps (in general)

1-General appearance
-pallor (anemia)
-jaundice (hemolysis)
- rash
-scratching marks (lymphoma)
-bruising
2-Vital signs (pulse, blood pressure)
3-Hands
-nails: spoon shaped, brittle nail, koilonychias (IDA)
pale nail beds (anemia)
-palmar crease (anemia)
-joints (PIP, DIP, MCP, Wrist): swelling (RA, CTD)
-purpura, petechial, ecchymosis
4-Arms
-Epitrochlear LN
-Axillary LN.
5-Face
-eyes: pallor, jaundice, subconjunctival hemorrhage.
-Mouth: glossitis, angular stomatitis, gum hypertrophy
use tongue depressor for tonsils
6-Neck
-Cervical LN+ supraclavicular LN
-Bone tenderness: tap spine-press ribsgently press sternum and clavicle
7-Chest and heart
8-Abdomen
-palpate liver, spleen, kidney
-DRE, genitalia
-Inguinal LN
-Para aortic LN
9-Legs
-bruising, scratch marks.
-Ulcers
-neuro (B12 deficiency).
10-Others
-fundoscopy
-urine dipstick
-temperature chart
Case 1 (anemia in pregnancy examination)

Feedback 27-6-2018 pass


pregnant lady 10 weeks presented for checkup you haven’t seen her before.
blood test results
HB 10
MCV 70
WCC and platelet normal
Tasks
1-perform hematological examination
2-dx and ddx to patient

Approach
Then tell that I have never seen this patient and ask me to start first with explaining the results
briefly to the patient before starting the examination.
Task 1
-I greeted the patient and introduced myself.
-I can see that you are here for a checkup. Some blood tests have been performed and the
results are now with me to explain to you so let us see the results together.
We have three types of blood cells the red blood cells, which contain an iron rich protein called
Haemoglobin, which is responsible for carrying oxygen to the tissues. White blood cell to help
fighting against infection and platelets to help with blood clotting.
Platelets and WCC are normal.
But there is a decreased level of hemoglobin we call this anemia.
Also this MCV is 70 less than the normal range this mean that the size of the red blood cells are
small.
-There are few possibilities but first I need to examine you in order to correlate the results with
the physical examination and have a better view.
This would involve me having a look, examining your eyes and mouth, feeling your lymph glands;
examining your chest and tummy would that be alright?

-Wash hands
-giving commentary to the examiner at the same time.
-started with GA
-then both hands (no Onycholysis , no spoon shaped nails, pale nail bed , no joint deformity, no
bruising, normal skin colour, no wasting)
checked the pulse---normal
-feeling for epitrochlear LN --- negative
-I would like to feel the LN of your armpit---examiner said not enlarged
-examining the eyes--- no pallor or yellowish discolouration.
-examining the mouth—no cyanosis, gum hypertrophy, no glossitis or angular stomatitis
-now I would like to feel the LN of your neck--- examiner said show me how you can do it. So I
asked the patient to sit while her legs still on the bed and I started feeling the glands from
behind ---examiner said normal , and the supraclavicular LN ---examiner said normal as well
-now I would like to examine the chest and heart with this stethoscope---examine normal.
-now I would like to examine your tummy this would involve me having a look pressing gently
and deeply, tapping and then listening… examiner said show me how to do it.
if you could expose your tummy for me
inspection from end and side of bed--- no scars, normal skin colour no distension, normal
movement with respiration etc…
now I’m just gonna press gently--- no tenderness or superficial masses.
now I’m gonna press deeply--- no masses
-the bell of the task rang---examiner said assume that all abdomen, leg examination normal
explain your diagnosis and ddx to patient.

Task 2
-I finished examination. It seems like you do not have any serious problem. Let me assure you
about that.
-So from examination and the results there are few possibilities why you have anemia . most
likely it is due to Iron deficiency which is decreased level of iron in the blood causing low level of
hg and small sized rbc
-let me assure you that it is a common condition in the pregnancy because you now have to
provide iron to yourself and the baby.
-it could be due to thalassemia which is no enough production of hemoglobin and inherited
condition but this confines to specific ethnic groups like Asian and mediterranian people and
also there would be a positive family history
-it could be due to chronic disease like kidney disease or infection but from examination I
couldn’t find anything suggestive of these.
it could be due to bleeding disorder but unlikely from Ix and ex.
-most likely iron deficiency anemia.

Feedback 21-2-2018
CASE 6 PE- ANEMIA IN PREGNANCY-PASS
10 weeks pregnant lady with blood reports showing anemia. Mixed diet, to do physical
examination and tell patient about the cause

Patient lying on bed, WIPES protocol.look for ethnicity; Started from hands -palor, clubbing-
schamroth sign, creases, cyanosis, pulse rate, rhythm and volume,rash in cubital fossa present
asked since when; examiner told just a scratch mark, checked bp ,eyes-palor,jaundice,mouth-
gum bleed,hypertrophy, waldeyers ring, forget to use tongue depressor first , examiner showed
then I used and mentioned and disposed the wooden spatula in bin.

Asked her to sit down in a chair and examined from behind all the lymph nodesincluding epi
trochlear. Palpated for uterus, liver and spleen , pedal edema. Mentioned iron deficiency usually
dietary deficiency unlikely with mixed diet was running out of time; mentioned will start on iron
pills, review in 2 weeks for tests, if no improvement considering other causes and further tests.
Bell rang.
Feedback 16-8-2018
Station 20. Anaemia in pregnancy
GP, 27 yrs old lady 10 wks pregnancy came to you for routine ANC. Previously you did blood tests
and showed below. Other tests are normal.
Hb: 10 g/dl
RBC: ?
MCV: 70
MCHC: ?
WBC: 7800
Platelet: 320,000
Task: Perform the physical examination
Diagnosis
Differential diagnosis

Patient lying on bed. Giving clock, Pen touch, tongue depressor.
PE: all normal
I explained diagnosis (most likely IDA). Other possibilities: Thalassaemia, Lead poison,
sideroblastic anameia, Chr disease.

Grade: Pass
  Global score: 5
Key Steps: 1- Yes
2- Yes
3- Yes
  4- Yes
Choice & Technique of examination, organization and sequence: 5
Accuracy of Examination: 5
Diagnosis/ Differential diagnosis: 5
Feedback
Ante partum care - PASS
23 year old pregnant lady; she just had her first AN visit last week and today she is at the clinic
for the blood test results. Urine, infection screening was normal. FBE result shows the following:-
Hb – 8.5%

MCHC; MCV; MCH – all 3 parameter were reduced;

WBC – normal; so was platelet count

She is not a vegetarian. This is her 1st pregnancy. No bleeding disorders.

Task: Perform physical examination

Explain Dx and DDx (i don’t remember the exact task)

2 min thinking: This is a case of HMA and routine haematological examination with SAIT causes
(sideroblastic anaemia, anaemia of chronic disease, IDA and thalassaemia)

Physical examination

Greeted both examiner and the patient. Asked permission.

But before I could proceed my examination, the patient asked me about the result first which I
explained in details about the count, size and the blood cells being faint.

Then did the examination as I did in previous case.

Bell rang while examining spleen though it was non-palpable.

Dx and DDx

<Explained the above DDx in lay terms>

“But the most common cause of this HMA in pregnancy is because of deficient iron in the blood;
this, in turn, can be due to pregnancy itself as the demand of the body rises or bleeding disorder,
dietary deficiency, or due to exposure to the chemical lead.”

GS – 4 Key steps 1, 2, 3, 4 – YES Choice and technique of examination – 4 Accuracy of


examination – 4 Diagnosis/DDx – 5

Case 2 (Lymphoma)
Old male came with fever, weight loss and night sweats. You have found 2 rubbery lymph nodes
in groin then u had to leave for some urgent call at ED. Now you are back to see him again.
Tasks
~perform Haematological exam
~dx and ddx

DDX for LAP in groin area


1-Lymphoma
2-Leukemia
3-Viral or bact infection
4-IMN

Feedback 15-8-2018
Scenario : health Review
Stem: Old male came with fever, weight loss and night sweats. You have found 2 rubbery lumph
nodes in groin then u had to leave for some urgent call at ED. Now you are back to see him
again.
Tasks
~perform Haematological exam
~dx and ddx
Old man was lying on the couch with sheet on. His face was flushed and during the examination I
found he was hottish as well
As I was already examining the patient and had to leave in between. I didn’t start with usual
greeting that we say with seeing patient for the first time.
Started the examination with WIPE approach. Did stepwise as mentioned in handbook.( positive
was some scratch marks. Couldn’t find the torch lying on table examiner handed over to me but
said he is a bit sensitive for gagging. So I wasn’t sure whether to use tongue depressor to see
throat. So I didn’t :P continued the examination and when came on inguinal nodes. Examiner
said no need to do that . Just 2 lymph nodes palpable.
Finished off . thanked and covered him.
Gave dx as lymphoma( in breaking bad news manner) and explained but said its still a suspicion.
Other ddx as infection( toxo, cmv, ) least likely hiv, hepatitis and leaukemia etc..
Grade : FAIL

( Here I think I should have organized myself more, reading the stem outside and already
knowing the case put me in overconfidence and despite of oraganizing my thoughts in 2 mins
and I got relaxed and lost the station)

Feedback 15-3-2018
GP, pt having night sweats, 2LNs palpable at groin, and some features of lymphoma. Very long
stem. Tasks: perform haenatological examination, give pdx and ddx (no positive findings except
groin nodes)
Approach: hematological system
Yes there was lots of lymph nodes, lymphomas and hemat exam that i actually thought i am
hallucinating. First case lady with bilateral inguinal lymph nodes. You have examined the lymph
nodes and gone out and come back to do rest of exam. long stem with negative night sweats, no
allergies, bites etc .case was perform hemat exam and give ddx . Took consent . for got to
handwash. The role player was supergeneral ap helpful . She gestured to me that wash your
hands and the examiner had a smile. So to cover up i said remember i have examined your groin
and then went out. I have already washed my hands and not touched anything but since you are
concerned i will wash my hands again...examiner smiling even more. follow hemat format.
General apperance . told examiner i would like to do vitals . Examiner said normal. start with
hands epitrocheal LN, then axillary both sides, Face frontal bossing, eyes , mouth , neck cervical
LN, clavicle tenderness, listened to heart, abd palpate for tenderness. Hepatosplenomegaly, then
examined legs for rash, skin changes etc, nothing positive gave ddx lymphoma, said cannot rule
out viral or bacterial etiology from exam And history. Will need investigations

Case (21/2/2017)
pruritus, night sweats, weight loss. Examine and tell DDs .Cervical lymphadenopathy +ve, Scratch
marks over chest due to pruritis
Do whole hematological exam and tell DDs and investigations.
Dear I don’t remember clearly now as it’s been a while but I do remember doing the thorough
hematological examination, actually giving running commentary to examiner, the one which is
given beautifully in handbook and then telling the dds from
Hepatitis
IM
Tonsillitis
Leukemia
Lymphoma
HIV !

Case (1/6/2017)
haemato examination
Already gave inguinal lymph node enlargement. Don't need to do inguinal LN check again
Perform other examination, and explain finding to pts.
*Row Korner Nirupama Anand The stem said: you were examining this middle aged woman who
has lost weight and you found two lymph nodes enlarged in her groin. Then you got a phone call
and now you're coming back to the room to finish your examination. (That was the stem!)
1. Perform haematological exam
2. Management plan to pt
There was a note that you shouldn't explore the groin nodes again!

Case 3 (ITP sinusitis)


Your next patient is a 27-year-old lady who is supposed to have a sinus surgery due to recurrent
sinusitis. Several years ago she had an episode of low platelet count.
Tasks
-Perform a haematological examination on her, with running commentaries to the examiner
-Dx and ddx

DDX for low platelet history presented before sinusitis surgery for check
1-ITP (in response to viral infection)
2-TTP
3-HSP
4-bacterial infection
5-medications
6-leukemia
7-hepatitis
8-high alcohol consumption

Feedback 21-6-2018
You are an intern in pre-admission ward (or something similar). Your next patient is a 27-year-old
lady who is supposed to have a sinus surgery due to recurrent sinusitis. Several years ago she
had an episode of low platelet count. Perform a haematological examination on her, with
running commentaries to the examiner. I am not sure about the differentials or not. Health
Review or Pilot/ Pass with Global score 5 or 6.

After entering the room and introduction and washing hands, I explained my examination and
took consent and started examining the patient. On general appearance the patient looked
comfortable and in no distress. Checked for pallor in conjunctivae and also jaundice in the sclera.
Also addressed that there were no other signs of anaemia like angular chelosis, or glossitis.
Examined lymphadenopathy in the neck. There were no enlarged lymph nodes in the head and
neck, but I could feel a small round, regular soft tissue enlargement in the lower neck, on the left
sternocleidomastoid, which was probably an accidental finding. Then proceeded to axilla and
also to supratrochlear nodes, then examined the abdomen for spleen and liver enlargement.
Also mentioned that I needed to examine the inguinal lymph nodes. The examiner said they
were normal. Checked for bone tenderness in sternum, clavicles, ribs and vertebrae, and also
mentioned that there were no abnormal skin rash such as petechial or purpura. So I explained
that there could be several reasons for low platelet count. Such as problems of the bone marrow
such as malignancies, or destruction of the platelets in the peripheral blood due to viruses or
immune conditions, such as ITP. But as she looked healthy and I could not find any abnormality
with her haematological system, her previous decrease in Platelet count should have been due
to some minor viral reason, and currently she is absolutely fine.
Feedback 7-12-2018
25 years male posted for sinus surgery , he had low platelets in the past .
TASK :physical examination .,dd to patient .

2 min THINKING: as same haematology should not forget things i forgot in my station 1(like
bony tenderness and all')

entered and greeted both of them, explain him that ,as he was posted for surgery need to
examine him to see every thing is alright involves feeling tummy , looking nodes and all. asked
him to undress, washed my hands and wiped it.
followed same approach, GA,VITALS , PICKLE, same as haemotolgy as mentioned in case 1.
abdomen examiner was standing beside me and looking while palpating for liver and spleen.
chest and heart :normal .

UDT and BSL -ve

all negative could not get any positive finding.

thanked patient.(don 't remember whether i asked him to undress or not as i was running short
of time, having bells for each task was very helpful)
said it could be ITP (unfortunately patient asked me explain - was in dilemma whether to explain
him or complete my other DD)(just explained because of some reasons some times platelets
may go down it might be that - i reassured him ill explain later in detail), said other dd TTP,
leukemias, drug reaction . Don't worry ill repeat blood tests again mostly platelets would be back
to normal.

scenario:health review

Global 5 (approach 5, choice of technique 5, accuracy 5 patient counselling 4)


Feedback Health review - PASS

Young patient is to have surgery for sinus problems. Previously he had low platelet level. He
didn’t have any follow up visit for that.

Task: Perform physical examination

Explain findings

Examination
Just routine haemato examination
• GA
• Hand: clubbing; koilonychia; leuconychia; palmar pallor; no rashes or bruises on the
arms
• Face: pallor and jaundice – nil; no epistaxis; no gum hypertrophy; no gum bleeding
(don’t forget to use tongue depressor and to discard)
• Cervical lymph node (I decided against wearing gloves though I saw a box of gloves was
on the wall) + Axillary lymph node + Epitrochlear lymph node – all non-palpable
• Bony tenderness – nil

• Asked the patient to lie down – did abdominal examination – normal


• Asked to check inguinal and popliteal – the examiner said normal
• Lower limbs – I saw some red spots which look like petechiae but didn’t mention about
them as I thought they were normal (now, I seriously think I should have mentioned about it)

Explain Findings (I don’t recall exactly what I told my patient)


Previously, platelet which is one of our blood cells involved in clotting process was reduced. But
everything is normal, at least on examination. So, it was probably due to viral infection or a
condition called ITP (explained briefly). Would like to arrange some tests to check the cells again.

GS – 4 All key steps – covered


Approach – 4 Choice and technique of examination, organisation and sequence – 4
Accuracy of examination – 4 Patient counsellin/education – 4
Case 4 (ITP leg rash)
Young male with hx of sorethroat t and flu like illness a month back now developed rash on
lower limb(pic given outside
Task,
-PE,
-Provisional Dx and DDS to the patient

DDX pic of non blanchable rash and petechial in lower limb


1-ITP
2-HSP
3-leukemia (unlikely)

Feedback 19-7-2018 STATION 6:PASS(ALL 4 KEYSTEPS YES, Score 6,5,5,5)


PE of RASH in lower limbs
Young male with hx of sorethroat t and flu like illness a month back now developed rash on
lower limb(pic given outside) maculo papular, non blanchable rash ,not painful ,no fever ,joint
pains nothing.
Task, PE, Provisional Dx and DDS to the patient
IN the room young male is lying on the couch with shorts and a blue gown ,after wiper I asked to
remove gown he refused as its cold. I Asked examiner about vitals he said not done. I asked the
pt to sit up started with G. Inspection then pulse(watch not there n examiner didn’t tell me the
rate as well) I just said regular and not fast then hands , epitrochlear n axillary nodes(with
gloves) then eyes nose, mouth, neck( nodes ad rigidity) then asked him to lie down followed by
abdomen, inguinal and popliteal nodes and commented on rash, end with genitals, DRE and
office test.
I gave DX of HSP BUT it could be ITP, DRUG induced, meningococcemia
( I think I was wrong,it was ITP as no joint pain, abdominal pain and there was petechial rash but
I was confused over there.)

Feedback 23-6-2018
Feedback: Rash: Hx in the stem revealed unblancheable petechia following after a cold with the
picture of petechia in lower limps. Perform relevant examination, DDx to the pt.
Perform Hematological examination: everything was normal apart from the unblanchable
petechia in the lower limps. URINE test: normal.
Dx: ITP  explain ITP to the pt
Other DDx: HSN, leukemia (very unlikely).
Grade: PASS, GS: 5 Key steps 1,2,3,4: Y (all covered) Technique: 5, Accuracy:5, DDx: 5.

Case (3/3/2017)
27 yr old female with maculopapular non blanching non pruritic rash in the lower legs. Viral URTI
last week.
Task : perform pe with running commentary to examiner, Tell likely dx and ddx to patient
Axillary epitrochlear cervical negative Had to check throat with spatula no findings
Abdomen no organomegaly
* This was my exam case. It was Itp. I did the hematological exam and differentials I gave were
IMN, HSP, meningococcemia, HIV and leukemia
177-Irritable baby reducible hernia
Feedback only (check lectures 82 and 83)

Feedback 16-8-2018
Station 14. Irritable baby and reducible hernia
GP, 6 wks old baby brought in by mom because baby is crying a lot. It has been about 3 weeks,
almost every day esp at night. Today morning mom noticed that baby right groin has a lump.
Task: Take history
PEFE
Possible causes

Check vital: stable.
Crying: 3 wks, almost everyday, don't know how many hrs. No blue lip, LOC, drowsy.
Breast feed well (3-4 times in day and 2 times in night). 1st baby. She think her breast well drain.
No complication during and after delivery. Heel prick test: normal. Normal Vg delivery. No poor
adjust of room temperature. I assess Mom mood: a bit agitated, sleep p/b (+) due to baby crying,
no appetite p/b, no feel worthless life, no suicidal ideation.
Lumps: notice this morning, other features of lump: don't know. Vomiting x 2 times this
morning: not go away, yellow color fluid.
BIND: good
PEFE: GA: active, Vitals: stable. GC: growing well to 90 centile. No signs of dehydration. ENT:
normal. Ht and lungs: normal: Abd: no bruise, no mass, no tenderness, no guarding, no rigidity,
no organomegaly, bowel sound: (+). I asked hernia orific: showed picture for inguinal hernia. I
asked it is reducible: Yes.
Urine dipstick: normal
I mentioned reducible hernia (explain tummy wall weak & tummy o/g came out). Refer
Another thing is irritable baby and mom is tired as well.

Grade: Pass
  Global score: 5
Key Steps: 1- Yes
2- Yes
3- Yes
  4- Yes
5- No
Approach to patient: 5
History: 5
Choice & Technique of examination, organization and sequence: 6
Diagnosis/ Differential diagnosis: 5
Management plan: 4
Feedback 4-4-2018
Station 19- pass
3 months old Irritable baby for 3 weeks. Today mother noticed lump on inguinal area.
Task
further hx
Pefe
Diag and Further mx.

On Hx, crying more in evening with reduce feed in evening.


Pulls leg up while crying, gets better with feed.
On asking hernial orifices pic was given of inguinal hernia with normal skin and reducible.
I said both irritable baby and inguinal hernia. Bcoz baby was irritable for 3 weeks while mother
saw lump only in morning. I managed case according to irritable baby and told mother what she
can do just as in karen.
And referred child to specialist as soon as possible. Told all red flags that if he cries more, vomits,
drowsy, fever, bluish coloration over lump , or lump doesnt go back pls go to emergency
department.

Feedback 15-3-2018
5) 8 mnth old child crying for 6 hrs father very worried task hx dx nd ddx
Approach: irritable child head to toe
Yes there was this lady with a doll and pretended to comfort the child and was
really worried. outside was the baby is irritable and crying for 4 hours . I dont
remember any other history. When i went in asked hemodynamic stability
examiner said you have to find that out.i thanked him and then comforted the
lady, started with crying questions, ( is it first episode, is it continous or
intermittent , was there any trigger factor any trauma , was the baby alright
before this episode started or was the baby sick, are there any associated
factors like drawing up of legs, baby turning blue or pale while crying, child
was turning pale while crying + ) irritable baby r/ o everything from head to toe
Including lumps or bumps anywhere .Head - meningitis, ENT, RS and CVS,
abdominal was in detail with poo details , urine details, including change in
colour, odour, consistency , number ) then asked well baby questions,
BINDS,travel history. brother was sick with gastro+ . Ya there was no PEFE
which was an indication for taking detailed and good history. I gave a ddx with
i need to r/ o intussusception first but could also be gatro which the child got
from brother, UTI less likely child does not have fever, could be strangulated/
incarcerated hernia i need to examine the child etc, mentioned child abuse as
ddox as well said I need to look for injuries on child, mentioned colic as ddx
but said unlikely As this is not age for presentation and this is a first episode
of crying
Comforted the lady saying whatever it is we will take good care of the child
and asked about the other brother as well. Ddx could be anything in this case
as the only + ve thing was child turning pale while crying

Feedback 18-9-2018 reducible inguinal hernia


irritable baby + inguinal hernia (this case is a baby girl but I keep saying HE )
6 weeks old baby cry for 3 weeks and this morning mom noticed hernia in the inguinal
area.
Task: Hx, PEFE, Dx and Mx
Mother noticed hernia this morning, she is pretty sure it was not there before,
Baby cries for three week, mother in law blaming it is because of her breast milk.
Cries draw up her leg, more severe in the evening, not turning pale or blue, no other
infection sign.
Mom stressed because husband does not help. No financial stress.
BINDSMA normal, baby is otherwise well, no sign of infection, fever, etc.….
PEFE: picture given of inguinal hernia, reducible
Rest normal
Mx: inguinal hernia: refer for surgery within 2 weeks
Irritable baby: it is normal. Common among her age, encouraged breast feeding, social
worker, family meeting to address mom’s stress
5 S approach: I said three S before bell rang
Score: 6
Key step: yes/yes/yes/yes/no
Approach: 5, Hx: 5, PEFE: 5, Dx/D/D: 5, Mx: 6
178-Breastfeeding jaundice
4 day old infant brought in to ED, not feeding well concerned by the nurse.
Tasks
-Hx
-Pefe card
-Explain dx/ddx
-Mx

Hx
1-Dehydration Qs
-feeding Qs
Type. (breastfeeding)
Frequency (every 3 hrs) ( if decreased feeding then ask why? Any problem with nipple, mum or
baby?)
Technique?
-Number of wet nappies? Crying on changing nappies? Color of urine and smell?
-Lethargic, sleepy, how is his sleep?

2-Bowel motion (color of stool, diarrhea) (greenish stool)

3-vomiting?

4-Drawing up legs while crying (if crying )

5-Fever, rash, yellowish discoloration of skin, lumps (yellowish skin +ve)

6-How long? Level of jaundice?

7-BINDS ( risk factors )


Birth hx
*cx during pregnancy, PROM, infection ( massive bleeding pph)
*Delivery, trauma
*term, preterm, weight
*screening test
*blood group mum and baby
Immunisation
Nutrition
Development
Contact, fhx of hemolytic disease, travel

Pefe card
-G/A DRPJL dehydrated and jaundice
-V/S
-ENT eye/cataract
-Growth chart. 13% wt loss
-Abdomen /hepatosplenomegaly?
-Office test ( UDT esp for bilirubin and urobilinogen, BSL)

Explain

-Breastfeeding jaundice is yellowish discoloration of skin and whites of the eyes due to
accumulation of pigment in the blood called bilirubin, which is a waste product resulted from
breakdown of used RBC.

-This is caused when the baby doesn’t get enough milk. As adequate amount of breast milk
increases the baby’s bowel movements which help secrete the build up of bilirubin.

-This can occur due to improper milk supply either due to improper technique of breastfeeding
e.g problem with latching or poor feeding by the baby.

-Other possibility is physiological jaundice due to normal immature liver when can’t filter
bilirubin enough, precipitated by inadequate feeding as well.

Mx
-Keep in ED to be seen by specialist
-Ix needs to be done
-Lactational consultant
-This jaundice often resolves by itself with increased feeding and help from lactation consultant
to make sure the baby is taking adequate amounts and latching well onto breast.

-Phototherapy if bilirubin is more than 15.20 mg and stop breastfeeding for 24 hrs
Feedback 20-7-2018
Feeding problem Pass Global Score 4
key step 1234 all yes ,
approach to patient 4
Dx/D/Dx 3
Management plan 4

ED , 4 D old brought by mom , as community nurse is concerned the child is not feeding well
History , PEFE card , DX with reason , Mx

Hello rose , i am dr yin , one of the intern doctor from this ED , Yes doctor , I am
concerned my baby's feeding . would you like to tell me more about it yes i am breastfeeding
the baby , but however she look hungry all the time after i stop feeding , how often do you
give feeding .... 3-4 hrs, how long in each time 15 mins, do you have enough milk supply ... i
am not sure doctor, this is my first baby, thats why i dont know much.... No problem , rose , i
will check out it for you. do you have any soreness, redness or nipple sores ... no , can the
baby suck well... yes i think so doctor , does the community nurse check out your milk
supply? what did she say? yes she just asked me to go to hospital , apart from feeding
concern , does the baby has fever / rash/ lumps bumps in the body no , yellow discolouration
of skin Yes , when did you start to notice it .. i am not sure , no ENT discharge ,no cough ,no
fast breathing , no tummy distension, navel fine , well baby questions ..... is the baby alert and
active ... yes he is active , any poo problem , what colour is it .... dark green , small amount ,
any smell when she pee ... no , what colour , yellow , do you think wet nappy numbers
reduced ... dont know doctor , its my first experience , last time i changed was 2 hrs ago,
BINDS-where did you deliver the baby , at hospital , how.. NVD , any problem at that time ...
the baby was fine but i got bleeding after delivery , was taken to do the procedure to stop
bleeding , baby got initial immunization, no sickness during pregnancy , regular AN care , no
family history of liver infection or jaundice , social . from a happy family , no scoical problem

PEFE card - was a long reading


baby is active and vigorous , vital all normal , no pallor , mild jaundice , baby sucking her fist
, dry lip+ ,
Body wt 13% reduced compared with birth weight , exact birth wt and current body wt was
given
Anterior fontanelle is normal
CRT is <2 sec , other examination normal ….
no urine dipstick is given

Rose , based on my examination finding and your story, i realized that baby looks hungry
even tho you feed her, she is sucking her fist while i examined her , her lips are dry and she
lose body wt compared with her birth wt , she has only small amount of dark green stool ,
these findings make me suspect that the baby has inadequate feeding. ( pt nod) , when the
baby has inadequate feeding, the baby can lead to dehydration ( loss of fluid from the body) ,
that can cause inadequate excretion of yellow pigment from the body , that inturn can cause
yellow skin and eye ( what we call breast feeding jaundice) .

Rose , there are a few reasons that can cause inadequate feeding
 mother cause , when the mother have insufficient milk supply , esp mothers in their
first experience of having baby, it is quite common to have it.
 baby cause , if the baby has infection in the body , sucking problem , food pipe
problem or blocking in the bowel ( however , i dont see any features suggesting for
infection and other cause and baby is active

Management
i will send the child to hospital ( the pt said why do you send me to hospital)
i look at the stem again, remembered that we are in Ed , so i will call my senior doctor to
come and review the baby . i will also call latational educator to help you with breast feeding
and to check whether you have adequate milk supply or not , we will use the pump ( i forgot
the name sucker ) to assist with your breast feeding , if the breast feeding is still not adequate
, We will also do the blood test to find out the cause of inadequate feeding . we may need to
give the baby formula feeding , baby needs close checking , so if the feeding then ok , i want
to review the baby every 2 days .
179-Congugated hyperbilirubinemia
Case 1 (5-10-2018)
ED setting. A mom comes with 4 weeks old infant, who has jaundice since the 3rd day after birth.
Baby is feeding well, growing well, no other complains except the yellow skin. You have
examined the baby and liver is not enlarged, PE unremarkable apart from jaundice. You order
bilirubin level shows: total bilirubin increased, conjugated bilirubin increased. Baby and mom’s
blood type same O RH-. Mom comes today for the result of Ix. Ix shows total bilirubin high 220
Conjugated bilirubin high 160.
Task:
-explain the result
-take focus Hx
-further management

Case 2 (13/07/18)
4 days old infant brought in having jaundice.
Tasks
Take history
Explain results
Dx/ddx

Explain the results


-There is an increase in bilirubin which is a pigment present in blood resulted from breakdown of
used RBVs. Accumulation of this pigment in the body causing yellowish discoloration of skin.
-There’re two types of bilirubin ( conjugated and unconjugated ) the one that is elevated in your
baby’s blood is the conjugated one which means Direct cause either problem with the liver, bile
duct or enzyme.

History (not all Qs needs to be asked as already given in the stem)


1-C/c jaundice Qs
-When have you noticed this yellowish discoloration? ( already given in the stem since day 3)
-Where did it start first?
-Is it progressing
-Color of urine and smell? (Dark tea color urine +ve)
-Bowel motion e.g color of stool, diarrhea (pale stool)
2-Dehydration Qs (normal)
*feeding Qs
Type. (Breastfeeding)
Frequency (good)( if decreased feeding then ask why? Any problem with nipple,mum or baby?
Technique (good)
*No of wet nappies? Crying on changing nappies?
*Lethargic, sleepy, how is his sleep?

3-Vomiting?
4-Drawing up legs while crying ( if crying )
5-Fever, rash, lumps
6-BINDS ( risk factors )
Birth hx ( most of these given in the stem )
*cx during pregnancy, PROM, infection
*Delivery, trauma
*term, preterm, weight
*screening test
*blood group mum and baby
Immunisation
Nutrition
Development (growing well given in the stem)
Contact, fhx of hemolytic disease, travel( all –ve)

Explanation
-Most likely having obstructive jaundice as presented with pale stool, dark urine along with
increase in conjugated bilirubin.
-Draw diagram and explain (this is the liver and these are the ducts or tubes that drain bile from
the liver. Bile is the substance that is required to digest fat).
-So the obstructive jaundice could be due to biliary atresia when there is block or absence of bile
duct, which is the tube drain bile to into the intestine and kidneys. So because of the block this
causing build up of bile in the liver and can cause damage.
-Could also be due to choledocal cyst which again causing obstruction of biliary tubes causing
retention of bile.
-Other possibilities that can be a cause of high conjugated bilirubin are neonatal hepatitis or
infection of liver by a virus.
-Or can be due to galactossemia or deposition of galactose in the liver due to lacking of enzyme.
-Sepsis but no signs of serious infection.

Management
-We need to keep him here in the hospital to be seen by specialist and arrange further Ix to help
with the dx.
-US imaging to check in the liver and ducts if there is any obstruction
-Blood viral serology to r/o neonatal hepatitis by virus, toxoplasmosis etc..
-Septic workup for sepsis
-TFT
-LFT, UCE,FBC,ESR,CRP,BSL all basic blood work, blood film
-Urine MCS, bilirubin and urobilinogen and reducing substances

-Note during the whole consultation, the mum was anxious so try to reassure her that her baby
is in good hands, etc..

Recall 10-4-2018
3 Day Old Baby – Dad brought child in concerned about the baby being Yellow

Explain the Investigation Results to Dad after a 3 min Hx

Total Bilirubin 227? Umol/l(<10mg/dl)

Child was Breastfeeding well , No irritability Gaining weight. Not unwell

Look at the Range that was given ? The unit of Conj Bilirubin was not given just the figure of
about 220-247 ( Range was in mg/dl)

Both Baby and Mother Rh Negative . No problems with Birth. Antenatal Screening all Blood test
were normal

Hx Stools Light Grey Dark Urine

Physiological – Breastmilk Jaundice

Conj Hyerbilirubinaemia - Biliary Atresia (Stools White )

Cyst Obstructing the Bile Duct

Neonatal Hepatitis

Recall 13-7-2018
Jaundice in neonate day 3, direct bilirubin 160!�
Feedback 5-10-2018 Congugated hyperbilirubinemia

Station 4 (Conjugated bilirubinemia)

Question: 2 weeks old baby brought by father came to you with jaundice. Pregnant history is
normal and baby and mother blood groups are Rh positive. You have ordered some blood tests.

Total bilirubin: high (forgot the exact number)

Conjugated bilirubin: obviously high (I think it’s 160)

Tasks: Explain the blood results to the father. Take a relevant history from father. Tell the
provisional diagnosis to father and your management plan.

Score: I passed with global score 6 (as I expected, I got high score in this case)

Key steps 1, 3, 4 Yes 2 No

Approach to patient/relative: 5

History: 4

Dx/DDx: 6

Management plan: 6

My performance

Hello….. David. My name is Dr. MM, one of the interns in this clinic. Nice to meet you. Can I
know you baby’s name? (you can call him George) Ok, I already knew that George is having
yellow skin problem and so I ordered some blood tests and I’m going to explain about it, ok?
(Yes, doc) I ordered the blood level of yellow pigment we called bilirubin in George. It’s quite
high and that’s exactly we can see obvious in George. And actually, there are two main types of
yellow pigment: one is water soluble and one is fat soluble. In George’s blood, water soluble type
of yellow pigment is raised. Are you with me so far?

Ok, David, I have to ask you some more relevant questions first. Would that be alright? Do you
think George is too yellow? (Yes, doc) How about George’s urine and stool color? (Yes, doc, urine
is yellow in color and stool is sometimes like pale and whitish) I knew that pregnancy is normal
but I have to confirm some of the facts. Is there any fever or rashes in your wife when she’s
pregnant? (No) I think there are some infection screening tests during pregnancy. I want to know
about hepatitis B and C infection screening in your wife. (it’s clear, doc). Any family history of
similar problem like George before? (No) Do you notice any tummy distension in George? (No)
How about immunization at birth? (it’s done) ok, thank you for the information, David.

According to blood tests and story of your baby, George is having increased level of water
soluble type of yellow pigment which has a number of causes. Let me draw a picture. This is liver
and there are some ducts arising from liver like this and they together form a larger one we
called bile duct. Most likely ,in your baby, there might be some developmental defect in this duct
we called biliary atresia. This duct should be patent but in biliary atresia it’s totally occluded so
that the fluid carrying yellow pigment can’t go down into the bowel. There’s another condition
we called choledochal cyst, which is a fluid filled sac formation in this duct obstructing the
drainage of the same fluid. Are you with me so far? There are some unlikely causes like having
viruses infecting the baby’s liver we called neonatal hepatitis. But according to history, it’s not
possible. There’s also very rare metabolic problem we called galactosemia which is unlikely too.

So to confirm my working diagnosis, I’ll order ultrasound imaging on your baby’s tummy to look
for any obstruction in this area. I’ll order some blood tests and urine tests to rule out unlikely
causes: hepatitis and galactosemia. If biliary atresia is confirmed, we need to admit your baby to
the hospital and there’ll be specialist surgeon to see your baby. Your baby might need some
operation to treat this condition but you’ll be explained in details about it by surgeon. Are you
with me so far now? Don’t worry, David. We’ll give your baby all the best care and you all are in
safe hand now. I’ll give you some reading material to read about this condition in details and
you’re always welcomed if you have anything to ask me more. Then the bell rang.

Comment: Some candidates appeared on same exam day posted as breast milk jaundice for this
case. But it’s conjugated bilirubinemia case. My history is quite short as the task described to
take only relevant history and so I only enquired about the possible causes of conjugated
bilirubinemia but I forgot to ask baby and mother condition in my history.
Feedback 5-10-2018 congugated bilirubin 2

A mother has come with her 4 weeks old child as she has noticed yellow coloration of the skin.
Local nurse has send her to your GP practice. Total bilirubin was 220 umol/L and Unconjugated
bilirubin was 160 umol/L. Your task is:

 Take history from mother not more than 5 mins

 Talk about your diagnosis and management

 NO PEFE

After checking my Id examiner introduced me with a very anxious mum with a baby doll in her
lap. After reading the stem i was thinking about breast milk jaundice. I started taking hx
regarding her main concern as like as Karen. She has noticed the yellowish coloration for 1 week
and there was some change in the colour of pee and poo. But she gave me hx of not gaining
weight which made me puzzle. Bcz in breast milk jaundice there will no problem regarding
gaining weight. There was no PEFE. So I could not take help from examiner.

So in my diagnosis I told her that I’m not sure about her dx though I’m suspecting it could be
breast milk jaundice or biliary atresia. I drew a diagram and explained both. Then talked abt
other dd as Karen and why these are not her baby’s case. When I talked about hospital
admission she cried. I was really scared as I was thinking may b I have told her something wrong.
I offered her tissue and water and reassured her that I’m not sure abt the dx thats why i’m
sending her to the hospital and there will be specialist paediatrician available who are very
expert and experienced dealing such type of case, If needed they will admit her and do some
inx. But at the end the mum was not that satisfied bt was not crying atleast .

Feedback: Passed

Key step 1,2,3,4: All Yes

In all domain: 5

Global score: 5
180-UTI in a child
Case 1 (2017) check feedback as well
18m girl has been brought by her mom.her mom believes that she is not well since 2days ago.
She does not eating and drinking well. T=37.7.A urine sample with bag has been taken that
shows leucocyte 3+, no other cells, no nitrite.
Task:
1-Explain for the mother the most probable diagnosis
2- talk about the management
3- talk about the further measures( if necessary)

Case 2 (2018)
GP, 6 months old, referred by the nurse because of leukocyte 3+ in urine. Alert, no dehydration,
examination normal.
Tasks
-h/o 3 mins.
-explain result.
-Management.

History
1-can you tell me more?
2-Urine:
-number of wet nappies?
-crying on passing urine?
-any colour change?
-is it smelly?
3-bowels
-any change in colour or frequent than usual?
4-vomiting, crying a lot, fever, rash
5-SOB, cough, runny nose
6-BINDSMA
-birth
-immunisation
-how is her feeding and sleep
-development and growth
-medication, allergies, PMH
Explaining results
-So here is the result of urine dipstick with me let see it together.
-it showed the presence of WBC which is a type of blood cell that usually responsible for fighting
infections and its presence indicates the possibility of urine infection by a bug usually a bacterial
one.
-However, I still need to confirm by urine culture to find out what kind of bacteria in the urine
because dipstick is not specific test.

-Even though your child is active and playful, I am going to specialist to decide for further
investigations and management.

Investigations:
-FBC, inflammatory markers, renal function test.
-Blood culture and USG of kidney and urinary tract.
- For taking urine sample, in babies of this age group, a sample or urine usually obtained through
aspiration from the tummy to look for possible infections. Please don’t be stressed it will be
done by a specialist. The nurse will give him some painkillers before the procedure. A very small
needle is passed through the skin into the bladder and the sample is withdrawn.
The purpose is to obtain a sterile urine, but specialist will decide.
- If necessary specialist will arrange repeat USG or put dye into urine pipe to see any structural
damage.

Treatment
-Treatment is mainly with antibiotic for 7-10 days and when the result come back, specialist will
change to specific antibiotic for that bacteria. Repeat urine culture 7 days after tx.
-I will follow up regularly.
-to prevent recurrence change diaper frequently and clean from front to back because of short
urethra and infection can go from back passage to front passage.
-Any financial problem. any support.

- Reassure her that it is a common condition and do not be much worry as baby will be under
safe hand very soon. With effective treatment, she will be fine. Give her reading tips and check
understanding.
Feedback 31-5-2018
GP, UTI 6 months old, referred by the nurse because of leukocyte 3+ in urine. Alert, no
dehydration, BIND -normal. Examination normal
Tasks
-h/o 3 mins.
-explain result.
-Management.

Greet, I did not why the nurse did the urinalysis, everything is normal, no fever, no decreased
appetite, no change in urine color or smell.no diarrhea, no rash, no vomiting, no rash no cough,
no SOB, no running nose. BINDSMA normal.

I explained what is leukocyte (WBC) in urine. Could be infection in urine.

Still need to confirm by urine culture to find out what kind of bacteria in the urine because
dipstick is not specific test.

Even though your child is active and playful, I am going to specialist to decide for further
investigations and management.

Inx:
-FBC, inflammatory markers, renal function test.
-Explained briefly about how to take Supra pubic sample and catheter specimen to get sterile
urine, but specialist will decide.(I did not say clean catch urine method).
-Blood culture and USG of kidney and urinary tract.
- If necessary specialist will arrange repeat USG or put dye into urine pipe to see any structural
damage.

Tx
-antibiotic for 7-10 days and when the result come back, specialist will change to specific
antibiotic for that bacteria. Repeat urine culture 7 days after tx.
-I will follow up regularly.
-to prevent recurrence change diaper frequently and clean from front to back because of short
urethra and infection can go from back passage to front passage.
-Any financial problem. any support. she said no.

Key steps 3/3, H/O, Dx and Ddx, choice of investigations, management 4,4,4,4
Global score 4.
Case (27/4/2017)
18m girl has been brought by her mom.her mom believes that she is not well since 2days ago.
She does not eating and drinking well. T=37.7.A urine sample with bag has been taken that
shows leucicyte 3+, no other cells , no nitrite.
Task:
1-Explain for the mother the most probable diagnosis
2- talk about the management
3- talk about the further measures( if necessary)

Child UTI invx counselling

*18 months old girl with fever and reduced feeding, mom is worried. Nurse did UDS -leucocytes
+++, others -unremarkable. OE done, T 37.8, others NR.

Explain the condition to mom


Explain the investigation to mom if u think necessary.
Further mx.

The role player was sitting but no signs of worry.

I started with introducing and greeting and asked her name.

I explained that it was infection of urine. That’s why baby got fever and reduced feeding. I asked
her that is it totally no eating. She said just reduced eating.

I explained mom that the infection could spread throughout the body, so, baby need to be
admitted.

Baby need to be seen by specialist and do further assessment. It will involve examination and
invasive investigation called suprapubic aspiration. I explained the procedure and the nature of
the test like painless and reassure. The aim is we need sterile urine and do C&S to find out the
causal organism.

I explained the sandwich approach. Give her advices about more fluid and healthy diet. And
explained her general vulva hygiene to prevent further infection.

Reassure her that it is a common condition and don’t be much worry as baby will be under safe
hand very soon. With effective treatment, she will be fine. Give her reading tips and check
understanding. 6R….
FB-PASSED, APPROACH-4, HX-4, DX-4, IVX-4, MX-4.

181-Alcoholic neuropathy
Case 1/GP, middle aged man came because having tingling and pain on his legs. He is a chronic
alcoholic and consumes ? drinks ? years.
Tasks:
-Perform lower limb neurological examination, tell the findings to the examiner (7min)
-Tell the diagnosis/differential diagnosis

1-WIPE
-wash hands
-Introduce yourself
-permission and consent
-exposure

2-Gait
-walking (symmetry, broad based gait, antalgic gait)
-turning (slow or quick)
-tandem gait
-walk on toes
-walk on heels
-Romberg test

3-Look (SSSDW)
-scars, missing toes or limbs
-skin (color, ulcers, caluses, atrophic changes like shiny skin and hair loss)
-swelling
-deformity (charcot joints)
-muscle wasting
note/ in the exam most likely to be all normal

4-Feel (TCP+ MVPR)


-Temperature
-capillary refill time
-pulses (dorsalis and posterior tibial)
note/ in the exam most likely be normal
-again cotton wool/ pinprick or monofilament (I think in the exam there will be cotton
wool/pinprick).
-vibration
-proprioception
-reflexes
note/ in the exam there will be loss of sensation and vibration up to….

5-Tone, power and coordination (I think we need to do them but again after inspection,
sensation and reflexes)

6-I would like to complete my examination by doing


-upper limb neuro exam
-cranial nerve exam
-cerebellar signs
-chronic liver disease signs

7-rewash hands and thanks the patient and the examiner

Note/ in the exam according to the feedback, I think the only positive finding will be sensory
involvement of light and deep touch and vibration on both feet up to (check feedback)

Dx and DDx
1-alcoholic peripheral neuropathy: from history and examination the most likely cause why you
are complaining of tingling and burning sensation in your legs is due to heavy and prolonged
alcohol consumption affecting the nerves of the lower limb. In examination you have a loss of
sensation from ankles below. Other possibilities could be.
2-diabetic neuropathy
3-B12 deficiency
Case 2/ 50 year old man comes to your GP as having problem with his gait. He is chronic alcohol
drinker, known history of hypertension. Lab value given abnormal GGT and normal B12 and
HBA1C
tasks
examination of lower limb
explain Dx and DDx to patient with reasons

On table you will find cotton wool, tooth pick, tendon hammer and tuning fork

GIT PR CS with modification

1-WIPE
-wash hands
-Introduce yourself
-permission and consent
-exposure

2-Gait
-walking (symmetry, broad based gait, antalgic gait)
-turning (slow or quick)
-tandem gait
-walk on toes
-walk on heels
-Romberg test
note/in the exam there will be ataxic gait and Romberg test will be positive.

3-Inspection (SSSDW)
-scars, missing toes or limbs
-skin (color, ulcers, caluses, atrophic changes like shiny skin and hair loss)
-swelling
-deformity (charcot joints)
-muscle wasting
note/ in the exam most likely to be all normal

4-TCP + sensation and reflexes


*-Temperature -capillary refill time -pulses (dorsalis and posterior tibial)
note/ in the exam most likely be normal
*-Sensation
-vibration
-proprioception
*-reflexes
note/ in the exam there will be loss of sensation, vibration and loss joint positions up to a certain
level (check feedback)
normal reflexes

5-Tone, power and coordination


note/ in the exam the examiner will say normal or you will do it and will be normal. But most
likely the examiner.

6-I would like to complete my examination by doing


-upper limb neuro exam
-cranial nerve exam
-cerebellar signs
-chronic liver disease signs

7-rewash hands and thanks the patient and the examiner

Dx and DDx
1-alcoholic peripheral neuropathy (most likely)
2-hypertensive peripheral neuropathy
3-diabetes neuropathy but HBAIC normal
4-B12 deficiency but B12 normal
5-nerve compression in the back

Case (30/3/2017)
Peripheral Neuropathy.
Hb A1C and B12 normal.
Positive Alcohol plus macrocytosis in history.
Task. PE
Diagnosis. Roomberg plus all sensations lost over both feet. Said Alcohol Neuropathy.

Case (16/9/2017)
lower limb tingling sensation. taking alcohol. lower limb examination

Case (2/6/2017)
Lower limb examination, Alcoholic neuropathy
B12 , folic acid was normal, GTT – deranged
Megaloblastic anaemia +
Feedback 20/2/2018
GP, middle aged man came because having tingling and pain on his legs. He is a chronic alcoholic
and consumes ? drinks ? years.
Tasks:
-Perform lower limb neurological examination, tell the findings to the examiner (7min)
-Tell the diagnosis/differential diagnosis

2 min thinking: alcoholic peripheral neuropathy.

Introduce myself. Gain consent from patient to perform physical examination. Wash my hands.

Perform as per Tally & O’Connor physical examination book. GA, gait, look, feel, neurological
(tone, power, reflex, coordination, sensation)

Examiner stopped me after every steps and asked for findings. Hence 7 min is just enough to
perform all the relevant LL examination.

Explanation:

Dear John, as you are having burning and tingling sensation on your legs, and during
examination I found out that the sensation on your feet till ankle are loss. Most likely it is linked
to the alcohol consumption. Otherwise it could be other possibility like DM (Bell rang)

AMC Feedback – Painful feet: PASS


Feedback 20-4-2018
Feedback 5-7-2018
Middle age man, heavy alcoholic, tingling in feet.

Task: 1- do PE

2- tell DDX

I did neurological examination as in geeky medics, the examiner told the power is normal, so I
move to sensation, vibration, proprioception and reflex. I checked the gait at the beginning,
Romberg sign was positive. And there was a sensory level so I did the exam in both dermatome
and the glove stock. My ddx was alcoholic neuropathy, diabetic neuropathy, vit b12 def and
cerebellar lesion.

Passes: global score 4

Feedback 5-7-2018
5. middle age man , heavy alcoholic , c/o tingling sensation of both legs, vit
b12 level normal,
Tasks
Do relevant examination
Explain the d/d to the patient
Patient appears chronic alcoholic (dishevelled, uncombed hair) sitting on
chair.
I asked him to walk few steps, then on heel , then on toe. Then told him to
lay on the bed.
I did the neuro examination of the lower leg, Just remember > Power, tone,
reflex, sensation (cotton and prick), coordination, vibration. (PTRSCV)
I did both dermatomal and grove and stocking sensation, there was a
sensory level at mid shin for light touch and pain and vibration. Remember
in this case don’t do all the movement at joints. Check myotomes only. You
have to comment what you find out as you do the examination. Don’t ask
finding from the examiner( he would say “ as you can see”).
When examining foot power, I told him push on your brakes (to test foot
extension) – the patient liked that and starting laughing. When I wanted to
go up to legs, examiner said all powe is normal.
I was very friendly with the patient.. I told him I will test your sensation
with this prick, promise I wont hurt you.
I did heel- shin examination
My d/d was alcoholic neuropathy but I mentioned that vit b12, diabetic. But
most likely alcohol.
When you drink lots of alcohol, above the ability of your liver to digest it, it
causes some problems with your nerves.

Feedback 5-10-2018 (Alcoholic neuropathy ex)

Physical examination (Alcoholic peripheral neuropathy)

Question: An old aged man came with pain and tingling sensation in his soles for several weeks.
Some findings of liver function test are given. (All enzymes are normal but GGT is slightly
increased)

Task: Perform neurological examination to lower limbs. Explain what you are going to do to the
patient. Tell the possible diagnosis (es) to the patient.

Score: I passed this case with global score 4. (I don’t expect to pass as I was very disorganized
and didn’t finish PE in time)

Key steps 1,2,3,4,5: All Yes

Approach to patient: 4

Choice and technique of examination, organization and sequence: 3

Accuracy of examination: 4

Dx/DDx: 4

My Performance:

The patient is sitting on a chair wearing shorts and T shirt.

Hello…. David… I’m Dr MM, one of the doctors in this clinic. Nice to meet you. I knew that you
have some pain and tingling sensation in your soles. Are you comfortable right now? (Yes) Ok,
I’m here to do physical examination on you focusing on your lower legs. Would that be alright?
(Yes) Ok, now I’ll wash my hand quickly. Can you please wait for a second?

Ok, let’s start examination right now.

(David, 55 years old gentleman is sitting comfortably on the chair.) Ok, David, can you please
stand up and take a few steps for me? Is there any pain? (He can walk normally and there is no
pain during walking) Ok, David, can you please sit down on the chair again. I’m going to have a
look first to your lower legs. (On inspection of lower legs, there is no deformity, no redness and
swelling, no ulcer, toe nails look normal, no muscle wasting) Then I lifted up both feet and looked
at the soles (there’s no pressure callus or ulcer)

Ok, David, now I’m going to feel it around your lower legs. If there’s any pain you can’t tolerate,
let me know at once. OK? Then I pressed all areas of both feet and soles and there was no
tenderness.

Ok, David, now I’m going to make you move your feet. At this point, examiner interrupted me
and told me to look at the task again. Only then, I realized it’s neurological examination. So I told
patient to lie down on the bed quickly. Ok, David now I’m going to examine your lower leg
muscle tone. You have to go floppy of your both legs. Then before I examined, examiner told me
tone is normal. Then I told the examiner that I’ll examine muscle power. He told me it was
normal. Then I told examiner I wanna test reflexes. He pointed me to take the hammer given on
a small table at the end of the bed.

Ok, David… this is clinical hammer and I’m going to tap with this to your knee and ankle. It won’t
hurt you. You have to go floppy of your legs. OK? (both knee and ankle reflexes are normal).

Now, David…. I have to test your sensation with this cotton wool. Let me give its sensation first to
your chest. Do you feel it? Now I’m going to touch this cotton wool to your lower legs. If you get
the same kind of sensation, please say yes for me. Ok? Now can you please close your eyes for
me? Then I examined it according to dermatomes (My patient has touch sensation loss starting
from ankles)

Now, David…. I’m going to test the same steps like before. This time I’ll use this tooth prick. Let
me give its sensation first to your chest. Do you feel it sharp? Ok, please say yes if you feel sharp
when I touch this to your lower legs. Ok, close your eyes again for me please. ( My patient has
pain sensation loss starting from knees)

Ok, David…. I’m going to test your vibration sense. I picked up both tuning fork and hammer and
struck tuning fork to hammer’s rubber end and gave the sensation to his chest. At that time,
short bell rang.

So I stopped my examination and told the patient: Ok, David, as far as I examined you, you have
a condition we called peripheral neuropathy. Sometimes diabetes mellitus can cause this. Other
possible causes like common peripheral nerve palsy but it’s unlikely according to my PE so far.
Sometimes a condition called Morton neuroma (inflammation interdigital nerves) can cause
these symptoms but I have to do special test to confirm it if I have more time. Then the bell rang.

Comment: See, how chaotic and disorganized my performance was. The examiner looked angry,
too. So I’m pretty sure I lost one station as soon as I came out of the exam room. I approached
the patient in the wrong way as I didn’t stick to the task. I didn’t have time to examine vibration,
proprioception and coordination. I forgot to do Romberg test, too (my study partners told me it
was positive in this case) But luckily, I passed this case. I’m really thankful to that examiner.
182-Hypertension examination
Patient had a high blood pressure recording of 170/90 while in a blood donation campaign Inside
the room patient was lying on the bed in 45Degrees angle
fundoscopy picture given in the stem
Task
-CVS ex
-Explained fundoscopy picture to examiner.
-Tell the Dx to pt

Examination steps
1-WIPE
2-General appearance
-cyanosis , dyspnea, Oedema
-weight
3-Hands
-cyanosis, clubbing, pallor
-nail changes
4-Arm
Pulse
-rate and rhythm
-radio radial delay
-radio femoral delay with permission (coarctation of aorta)
Blood pressure
-make sure cuff size is correct
-position patient correctly (elbow at heart level, lying)
-take the SBP on one arm by palpation method
-take the BP same arm by auscultation method both SBP & DBP
-then take by auscultation the BP on the other arm (difference between the arms of >10 mmhg
suggest vascular disease).
-take the BP in standing (ask permission first) to look for any postural changes).
Note/ the examiner will not let you do all the steps usually after measuring one arm the will give
you the other arm and the postural one)
5-Face
-eye (pallor and jaundice)
-malar flush
-mouth (central cyanosis)
-cushinoid facies (moonface, plethora, acne)
-eye puffiness, sallow skin (kidney px)
-verilisation (pcos).
6-Neck
-JVP
-carotid artery palpation be careful one at a time not both together
-Thyroids

7-Heart
inspection
-scar, deformity, visible pulsation)
palpation
-apex beat
-heave and thrill
auscultation
-heart sounds and murmurs
Mitral tricuspidpulmonary aorta
-radiation
axilla neck
-carotid bruit (breath in and out then hold)
8-Back
inspection (oedema)
palpation (sacral oedema)
percussion (upper and lower lobe)
auscultation (basal crackles).
9-Abdomen
Inspection (distension, visible bruit)
palpation
-liver
-kidney
-abdominal aorta
auscultation (diaphragm)
-aorta and renal bruit
10-leg
-pulses
-oedema
tenderness
11-others
-fundoscopy (showed AV nipping and silver wiring – said its grade 2 Hypertensive eye changes)
(other case with papilledema and grade 4 hypertension)
-UDT
-ECG

Explain
even though your BP normal now , u have hypertensive eye changes which means ur having
poorly controlled long standing high blood pressure

Differential diagnosis EAR M


-Endocrine (Cushing, conn, acromegaly, thyroid, pheochromcytoma)
-arteries (Coarctation of aorta, Aortic Stenosis)
-Renal (renal artery stenosis, polycystic kidney, DM)
-Medications (NSAIDS, OCP)

o Fundoscopy findings
▪ I – Arteriolar narrowing and silver wiring
▪ II – AV nipping
▪ III – Cotton wool spots and flame-shaped hemorrhages
▪ IV – Papilledema

Feedback 21-4-2018
Patient had a high blood pressure recording of 170/90 while in a blood donation campaign
Inside the room patient was lying on the bed in 45Degrees angle
Tasks
Perform relevant examination ( remember not only CVS ) - on the table – Clock , BP monitor
was present.
Did all the CVS examination including PR and BP – normal ( offered BL arm BP to rule out
coarctation or dissection of aorta – examiner said normal ) , Lungs bases , Pedal oedema and
calves, offered Proximal myopathy to check for Cushing’s , Thyroid examination and observe gait
to rule out complications ( ICH due to High BP ) and Abdo examination to rule out AAA and
Polycystic kidney.
When asked for fundoscopy findings gave the picture it showed AV nipping and silver wiring –
said its grade 2 Hypertensive eye changes.
Explain the condition to the patient - said even though your BP normal now , u have
hypertensive eye changes which means ur having poorly controlled long standing high blood
pressure( Rolepayer was happy with the explanation and didn’t ask any other questions.
Global score - 5 ( Approach 5, sequence Examination 5 , accuracy of Ex – 6 , Counselling – 5) My
comment - CVS examination and Hypertension examination are different

Feedback 4-7-2018
4.37 yr male gone to donate blood found to have BP 170/100
fundoscopy picture given in the stem
Task
-CVS ex
-Explained fundoscopy picture to examiner.
-Tell the Dx to pt
Ex- stated to RP examine to find out the cause & effect of HT on you
General inspection starting from hand-has clock check PR for 20sec
BP apparatus was there-so started checking with palpation- asked to do auscultation only
Observed very closely all the technique.
Face/neck /chest-as times goes told examiner as no murmur-not checking murmur with
positional change.
Told would like to complete-Lungs/liver/ankle odema-
Typical fundoscopy picture=silver wiring/AV nipping/papillodema/exudates & hae’ge
Told pt as complication of BP affecting eye-hypertensive retinopathy=bell rang

Feedback 21-4-2018
4.CVS examination.
Patient had a high blood pressure recording of 170/90 and fundoscopy picture was given along
with the stem. Tasks
Perform cvs exam. Tell the examiner the findings. Interpret the fundoscopy( it showed AV
nipping and silver wiring) explain your finding to pt.
Station 6 Periodic health review
Cardiology PE.
Need to measure blood pressure, two cuffs given, not sure need to change cuff, but I did not
I could not finish all steps in PE and examiner asked me to proceed to Fundoscope
interpretation, I told nipping and silver wire. Then times up. So no time to explain, I just mention
“you had hypertension”before I came out of door.
Approach to pt: 6
Choice and technique of exam, organisation and sequence: 3
Accuracy of examination: 4
Pt counseling: 3
Global Score: 3 fail

Case (7/9/2017)
CVS with fundoscopy pic. Man with 3 reeding of high BP. Came and did fundo - in stem.
Tasks:
do cardivascular examination
explain funduscopy to examiner and most likely condition.

CVS exam - including measuring BP with set.


Role player BP normal. 110/70. JVP normal, carotid pulse and bruit. Chest no impulses no
deformity, apex beat ….. all normal.
Funduscopy can see av nipping and signs of oedema - correlates with hypertensive
retinopathy. explained to examiner.
Explain to patient - changes at back of eye - and explained HT must be managed.

Case (28/4/2017)
1-37 yr man, send from blood bank as he come to donate, BP 170/ 100. Task examination of CVS.
Then explain patient his condition as well as explain him given fundoscopy findings. ( need to do
BP as well, need to change BP cuff size as it is Paedi cuff, fundoscopy show silver wiring for sure).
2-periodic health review
Thirty plus male went to blood donation twice noted high bp.referred for fundoscopy.now
comes to u at Gp.examine him and tell most likely cause of hypertension and explain fundoscopy
picture to him.hypertension examination case.some catch in bp measurements.I couldn't finish
this n failed

183-Frontal lobe dementia


Sister of a 40 years old man John is here to talk about John's behaviour change and memory
problem. CT scan has showed generalised cerebral atrophy (CT image is not given in this case),
John also had an episode of mania 5 years ago. His sister has the consent to talk on his behalf.
Tasks:
-take history from the sister (7 minutes)
-Most probable dx and ddx to the examiner.

History
1-chief complaint questions
-I can see from the notes that your brother has some behavioral changes and memory problem.
Can you tell me more?
Behaviour change
-when did he start behaving like this?
-has this happened before?
-can you describe what type of behavioral change is? (Mostly she will tell about from can you tell
me more)
Memory problem
-regarding memory, does he have problems remembering things he did earlier in the day or
things he did in the past?
-has he had any problems with managing things like paying bills or driving?
2-Organic causes questions
Trauma
-has he had any problems with falls or loss of balance?
-by any chance did he have any injury or trauma to his head?
Infection
-has he had any fever?
Malignancy or Tumour
-any headache or blurring of vision?
-any LOW, LOA, Lumps or bumps?
Electrolyte imbalance
-any dizziness, funny racing of the heart?
Thyroid
-any weather preference?
-Any change in bowel habits?
Past illnesses (DM, lipid, BP, stroke)
-any past illnesses of DM, high lipid, High BP, stroke, problem with vision or hearing
-past and family history of mental problems or Alzheimer? (Father has Alzheimer)
Mania and medications
-when was he diagnosed with mania? (5 years ago)
-how many episodes has he had? (one)
-is he taking his medication regularly? (Stopped taking from the psychiatrist)
-does he take any other medications?

3- psychosocial history
mood questions:
-how is his mood? (low)
-has he ever thought of harming himself or others?
-how’s his sleep?
Delusion and hallucination
-does he feel, hear or see things that others don’t?
-does he think that someone is spying on him or trying to harm him?
Insight
-does he think he need any professional help?
HEADS
- who does he live with? any support at home, any stress? (Live with sister and supportive)
- What does he do for living, any stress at work? (retired because of forgetfulness)
- has he lost interest in things he used to enjoy?
- SAD (imp)
- how is his social life?

Explain diagnosis and differentials


-I am suspecting he has frontal lobe dementia because from the history he has personality and
behavioral changes, short-term memory loss and poor insight.
Other possibilities are
-could be Alzheimer disease as he has positive family history of Alzheimer and generalised brain
atrophy on CT scan.
-could be head trauma, infection, tumor, thyroid, electrolyte imbalance, diabetes, stroke,
medications, depression, relapse of mania, drugs or alcohol induced brain injury but all less likely
from the history.
Management (if they change the questions and become counselling)
-I need to perform MMSE to confirm short term memory loss.
-do frontal lobe cognition tests
-do blood tests (FBE, BSL, UCE, LFT, TFT, vitamin B12) + urine MCS+ ECG
-vision and hearing tests
-refer to psychiatrist for full neuropsychological assessment.
-treatment by mental health team
To sister
-attend to his hygiene
-he need adequate nutrition
-regular home visit from relative or friends
-not to continue driving as he can put himself and others at risk.
-support group (Dementia self-help group)
*does he need to be at nursing home?
-it is advisable to be at home as the sufferers tend to be managed better in familial environment
and this can sometimes delay the worsening of his behaviour to a certain extent.
-if you think you need a break you can go for respite care where a trained professional staff can
take care of him for a short period of time.

Feedback 17-8-2018 pass


Sister of a 40 years old man John is here to talk about John's behaviour change and memory problem.
CT scan has showed generalised cerebral atrophy (CT image is not given in this case), John also had
an episode of mania 5 years ago. His sister has the consent to talk on his behalf. Tasks:
1. take history from the sister
2. Most probable dx and dds to the examiner.
(I really didn't like the role player. The accent was hard to understand. I couldn't understand some of
the things he said and I had to ask my questions repeatedly. My history was really not organised.)
I started to take history of presenting complaint. Can you describe what kind of memory problem and
behaviour change? When did all these start? Does he forget recent things? Does he misplace things?
Does he forget distant things? Does he remember names of family members and friends? What kind
of behaviour change? Any personality change and disinhibition? Any change in his language? Does he
use bad language? Few questions to rule out organic cause such as any fever any lumps or bumps any
weather preference.
Past history: I asked about mania. When was it?(5 years ago) What symptoms? Any medications?
(Yes) How many episode? (Only one, he used the medication for sometimes, then his doctor stopped
the medication) he tried to show that it's not related to his mania expisode. No other significant it
organic cause.
Family history: father had Alzheimer and died at age 50. Mood ok, appetite ok, no weight loss, no
sleep problem. Support ok. No partner. Sadma negative I guess.
Dds to examiner: I said frontal lobe dementia because of personality change but I want to assess for
any loss of executive functioning and planning. It could be Alzheimer's disease since there is family
history of early age Alzheimer's positive, and CT scan shows generalised cerebral atrophy. Or it could
also vascular or other kind of dementia. It could be relapse of his mania but unlikely. He needs a full
neuropsychologist assessment to confirm.

Feedback 17-8-2018 unknown status


hello. in this case, findings i got are
1- forgetfulness like where he put the keys and recent memory loss × few months
2- behavioural change × 1week.
3 -Agressiveness× 1 Week
4- no visual and auditory hallucinations
5- no depression, stroke, infection features, no hypothyroid , good nutrition, no history of fall, no
H/T , no D/M , heart disease, No alcoholic and drug
SADMA -no relevant finding.
Past mental history- Mania 5years ago, was hospitalized. took medication. symptoms free for years
and medication stopped by psychiatrist. i forgot to ask last time he met specialist.
Family history- dementia in his father.
social history - lives with his sister. supportive.
CT shows generalized cerebral atrophy.
DDx to examiner- i said
1. Dementia probably Alzheimer's . 2. stroke (vascular dementia)
3. Depression 4. delirium. buti tell examiner i need to check his insight, judgement, cognition
5-metabolic such as DM, hypothroid, 6 drug and alcohol.
I forgot to mention frontotemporal dementia. waiting for my feedback and i can elaborate more if i
get feedback.

feedback 17-8-2018 pass


adding behavioural illness-sister came with brother's memory problem and recent behaviour
change,pt had previous hx of mania which I missed, lost job 2/3yrs back due to memory problem,fhx
of alzheimers positive,hx of fall 2days back in shopping mall, no wt loss, fever, SADMA negative.no
other medical/surgical hx. no stress,mother and sister is taking care of him,sister had authority to talk
mentioned in stem, same as Karen case,
task-hx for 7mins and dx dd to examiner for 1mint.i asked all recent and long term memory loss
question,dd-frontal lobe dementia, stroke,head injury,alzheimers,mood problem,infection,head
tumor,alcohol induced brain injury-unlikely..(missed relapse of mania)

Case (10/2/2017)
Talking with sister of a patient who has recent behavioural changes and forgetful he gave permission
to her CT with atrophic changes
Short term memory loss for DD could be frontal lobe dementia vs Alzheimer.
I said I have to c the patient to do MMSE and to exclude organic causes ( dementia is a diagnosis of
exclusion)
Case (10/2/2017)
45 yo man, Sister came with consent from Px. He had abnormal behavior forgetfulness. CT shows
generalized cerebral atrophy.
Task: take Hx from the sister, DDs to examiner.
HOPI : I roled out organic d/ds ( I wasted a lot of time )
HEADSSSS : single lives alone , early retirement cuz he is forgetting things , he was scared to take
responsibility and forget at work ,job some thing like officer, big job (I don’t remember ). Yesterday in
the Shopping Mall he ( I didn’t understand the word ) but she said that was emaressing .
HEADSSS+ASEPTIC (negative) +r/o organic causes.
Positive: retired early- forgetting things-low mood-isolated-forget to pay bills.
I was confused ( I didn’t link the CT with the findings of forgetfulness )
My advise is spend more time on demntia qs and ask about frontal lobe qs (cuz I forgot ).
My D/DS were poor ( I gave depression , dementia , I got 3 in d/ds ) take the down d/ds and work on
them in the history ,Dds: depression, Alzheimer, frontal lobe dementia,senile dementia, brain
trauma (CT findings negative).
Dementia is a ds of exclusion, MMSE should be done for confirmaton.
I failed this case
Feedback 17-8-2018
Frontal lobe dementia with history of mania. History from sister and diagnosis and differentials to
examiner.
Sister had the consent to give history. Made confidential with her. Patient is 43 ? year old male.
History of mania 6 years ago which had been admitted for that. At the moment no sign and symptom
of mania( I couldn’t find anything positive for mania in the history) . CC: weird behavior. Aggressing.
Forgetfulness. Nothing positive for organic cause( no head truma. Meningitis. Fever. Rash. Hypo/
hyper th) . no alcholo. No drug. No medication. No hallucination. Asked all the mse. Fh: father had a
same problem in his 50. So sth familial. Wanted to expain ddx to the sister. She told me I have not
consent. Speak with examiner. So I checked my task and it was: ddx to the examiner ( accturly role
payer helped me!!)
So I mentioned: alchol indused brain damage but he is not taking alcholo / drug abuse which he is
not using / early frontal lobe dementia with positive FH/ alzheimer's/ stroke / lewy body dementia!! .
examiner was sooo poker face. Pass!

Feedback 2710-2018 STATION 11 BEHAVIOURAL CHANGE


50+ year old male patient with change in behavior and memory problem. His sister
comes to your GP to discuss with you. She has the consent to do so. CT of his brain shows some atrophy
(not given). He was diagnosed with mania a few years back.
Task
- further relevant history
- Explain the Dx/DDx with reasons to the examiner History
- Duration (a few weeks)

Behavioral change
- exhibit inappropriate behavior in shopping mall Memory problems
- does not recognize family member
- forgetfulness
- does not lose his way back home or misplace things or cause any domestic accidents Mania
- was diagnosed with mania a few year ago
- was on medications, but stopped under specialist supervision
- mood (normal) not elevated or low
- he looks rather withdrawn according to his sister
- feature of depression or mania (-)
- features of schiz such as hallucination or delusion (-)
- orientation (normal) HEADSS
- living with mom and sister
- getting along well
- not working
- spend most of his time at home
- no other stresses recently
- smoking (-), alcohol (-), recreational drugs (-) Organic causes
- father has died of dementia
- hypertension (-), DM (-)
- thyroid (-)
- head injury (-)
- fever (-)

DDx to the examiner


- frontotemporal dementia (most likely)
- vascular dementia (less likely)
- relapse of mania (less likely)
- Alzheimer (less likely)
- head injury (less likely)
• frontotemporal dementia wasn’t the right dx as I only got 3 in Dx/DDx
• Should have given Alzheimer as provisional dx or added pseudodementia due to depression in
the DDx

Grade - pass Global score - 4

Key steps 1,2,3,4 - Yes, No, Yes, Yes

Approach - 5 History - 4 Dx/DDx - 3


184-Incomplete miscarriage
You are an intern at ED. Your next patient is a young woman came with vaginal bleeding after 8
weeks of amenorrhoea. She looked pale.
Tasks:
-Take a history from the patient.
-Ask PE findings from the examiner.
-Tell the possible causes to the patient

Differential Diagnosis:
1-Miscarriage (incomplete, Threatened )
2-Ectopic pregnancy
3-H-mole
4-Trauma

History:
*ask patient if she has any pain, she said yes ask for severity and offer pain killer she said no
thanks she is ok now.
1-ask the examiner “Is the patient hemodynamically stable?”
*The patient unstable so need to stablise her by putting IV lines and start on fluid take blood
for FBE,UCE,LFT

2-Bleeding questions (Duration-Action-Trauma-Amount or severity-colour-odour-content-


dizziness- bleeding disorder or blood thinner)
- for How long have you been bleeding? Is this the first time?
- What were you doing the time you passed blood?
- have you had any trauma to your tummy?
- How many pads have you used so far? Is it (are they) fully soaked?
- What is the colour of the bleed?
- is it smelly?
- Are there any clots or tissues? Any vesicles or grapes? passed tissues +ve
- do you feel dizzy or tired? (yes)
-have you had any bleeding disorders or take any blood thinner medications? (no)

2-Associated symptoms (tummy pain, fever, N&V, discharge)


- do you have any pain in your tummy? (Mild pain)
ask only severity + pain killer, site and radiation.

- Any fever? Nausea or vomiting? Abnormal vaginal discharge?

3-5 P’s questions


Period questions: (only ask LMP and regularity)
-When was your last menstrual period? (Already given in the stem 8 weeks ago)
-were they regular?
Partner or sexual (only support and STI)
-do you have good support?
-have you or your partner ever diagnosed with STI?
Pregnancy
- are you trying to become pregnant? Said yes and UPT at home was +ve yesterday
- have you had any previous pregnancies or miscarriages? (no)
- do you have any vomiting, breast tenderness? (no)
Pill
how long have you been off the contraception?
Pap or Hpv
is your pap or Hpv up to date?

4-Early Pregnancy questions


-Any antenatal checks you'd done so far? (no yesterday I noticed myself pregnancy)
- Have you taken your folic acid? (no yesterday I noticed myself pregnancy)
-are you aware of your blood group?

5-General questions (Diet-SAD, OTC- pets, PMH-PSH-Family hx)


- Diet: any intake of raw meat? (predisposed to toxoplasma), how many coffee do you take in a
day?
- Do you smoke, drink alcohol or take recreational drugs?
-Any prescription or over the counter medications?
- Any pets at home? (toxoplasma in cat litter)
- Any other medical or surgical illness?
-Family history of miscarriages?

Physical Exam from examiner


1-General appearance:
pallor, dehydration, LAP (PDL) , Bruises or bleeding
2-Vital signs
Blood pressure (+ postural hypotension) , Tachycardia, Temperature ( low BP, high PR +ve)
(she is still unstable, so tell the examiner to continue giving the patient fluid and arrange for
blood group and cross matching)
3-quick Systemic exam.
4-focus on Abdomen
Inspection (distension, Mass)
palpation (tenderness, mass)
Bowels sound
5-Pelvic exam
consent of patient and presence of chaperone:
Inspection: colour of the bleed? Tissues, clots? (Tissues positive) so here tell the examiner that
you need to remove these tissues.
Speculum: look if bleeding is coming from cervix , cervix closed or open? ( OS open with tissues)
Per vaginal: CMT, uterine size, position, tenderness adnexal mass and tenderness ( size 6 weeks)

6-Office test:, UDT, BSL ( Avoid asking for UPT, coagulation profile, US)

Explanation
-Show empathy to the patient like I’m sorry to tell you that you most likely having miscarriage….
Do you want me to call anyone for you?
-Most likely what you're having is incomplete miscarriage. It is pregnancy loss presenting with
bleeding before 20 weeks of pregnancy, passing tissues and the neck of womb is open, the
womb size is less than age O/E.
-Exact cause is unknown could be due to genetic abnormalities in the baby.
-This usually managed by curettage(define it) by specialist….. you are in safe hands….

Other possibilities but unlikely are threatened miscarriage unlikely as in threatened misc the
neck of womb closed and no tissue passed. Molar pregnancy unlikely as no vesicles, and size of
womb usually larger than it should be. Ectopic unlikely as well

Feedback 15-8-2018
Vaginal Bleeding
Stem: Young lady with abdominal pain and vaginal bleeding after few weeks of amenorrhea (bp
80/40)
Tasks
~History
~PEFE
~Dx and ddx to patient
Inside the room there was a young girl in early 20s lying on the couch with sheet on. She was
acting as if she is in pain and looked anxious worried and breathing heavily.
Greeted her and address the pain first. Asked pain scale, allergy and arranged pain killers. Show
empathy.
Excused her to ask the examiner about vitals, he said already given. I asked if there is any change
since. He said no. So I told him to take patient to cubicle and give fluids before starting history.
He said sure.
Then took history ( You all know pain questions, bleeding questions.. keeping in mindtrauma,
intercourse, ectopic, molar, threatedned, imcomplete, complete abortion, appendicitis, pyelo
etc( she told she hasn’t seen any doctor and found out 1 or 2 days back with home pregnany test
that she is expecting). Asked about 5ps. And winded up.

PEFE
GA….Anxious breathing heavy
Vitals… Bp( still 80/40) even after fluids, PR Inc, RR Inc, Temp normal
Focused on Pelvic so asked for chaperone and patients consent ( examiner said offcourse you do)
Inspection: blood +
Speculum: blood +, clots +, POC +, OS opened ( I told examiner that I would remove poc as bp
isn’t coming up with fluids).
Gave dx of imcompplete abortion( in breaking bad news manner)and explained it.. ddx of
complete, threatened, molar, ectopic etc same as mentioned above.
Grade:Pass

Feedback5-10-2018
Station 5 (Incomplete miscarriage)
Question: You are an intern at ED. Your next patient is a young woman came with vaginal
bleeding after 8 weeks of amenorrhoea. She looked pale.

Tasks: Take a history from the patient. Ask PE findings from the examiner. Tell the possible causes
to the patient.

Score: I passed this case with global score 5

Key steps 1, 2 Yes 3 No

Approach to patient: 6

History: 5

Choice and technique of examination, organization and sequence: 6

Dx/DDx: 4

My performance

Patient is lying on the bed with some concerned and tired look.

Hello, Mary. I’m Dr. MM, one of the interns in this department. I knew that you might be very
concerned as you have bleeding from your down below. How are you feeling right now? (Yes,
doctor, I feel rather weak) I have to ask you some questions first. Would that be ok for you? (Yes,
doctor) Is it the first time to have bleeding from your down below after losing your period for 8
weeks? (Yes) Is the bleeding too much? (Yes) Any blood clots? (Yes) So I have to measure your
vital signs first from my examiner. Can you please wait for a minute? (Yes)

Dear examiner, as my patient has severe vaginal bleeding, I would like to measure blood
pressure and pulse rate first. (PR is 110/min and BP is 80/50 mmHg) My patient is having
hypovolemic shock and so I want to insert wide bore cannula on both hands and collect blood
for grouping and matching and basic blood tests. And I’ll run normal saline right now,
examiner. (Done) Let me get back to my patient.

Ok, Mary. I have done all the necessary urgent treatment right now. Can we continue for
history? (Yes) Previously, do you have regular periods? (Yes) Have you checked any pregnancy
when you lost your peroids? (Yes, doctor, pregnancy test was positive) Is this the first pregnancy?
(Yes) Do you have any recent injury history to your tummy or down below? (No) Have you seen
any tissue passed or grape like vesicles in your bleeding from down below? (No) Any tummy
pain? (Yes, doctor and it’s right here. The patient is pointing around lower part of tummy) Ok,
Mary, thank you for the information. I have to examine you right now. OK?

Dear examiner, I want to measure vital signs again. (It’s still the same)

Then I’ll look for pallor. (Present)

I’ll do focused abdominal examination. Is there any tenderness? (Yes, in lower parts) Is there any
abdominal distension? (No) Any palpable mass? (No)
Then I’ll move on to pelvic examination with my patient’s consent and in the presence of
chaperone.

On inspection, is there any bleeding at the moment? (Yes) Any blood clots? (Yes) Any foul
smelling? (No)

Any signs of injury or infection like redness and swelling? (No) Then, I’ll do speculum
examination. Is cervical os opened or closed? (open) Any tissue piece at the os? (Yes) Then I’ll
remove it now. I’ll do bimanual examination. I want to know uterine size, position and
tenderness. (uterus is 6 week size, RV position and slight tenderness present) Thank you ,
examiner. That’s the end of my examination.

Ok, Mary. I’m really really sorry to tell you that you’re having miscarriage now. Let me draw a
picture. This is your womb and here is neck of womb and here is birth canal. When I examined
you I saw some tissue piece at the opening of neck of womb and your womb size is reduced
now. This is a condition we called incomplete miscarriage. It’s very important for you to have a
procedure which will take out all the tissue pieces left in your womb. The other possible cause
might be a condition we called threatened miscarriage but it should present with only mild
bleeding and no tissue passed from your down below. There’s another condition we called
molar pregnancy but it’s unlikely as it usually presents with grape like vesicles passed. There’s
another unlikely cause we called ectopic pregnancy. Then the bell rang.

Comment: Examiner was a Chinese and looked very patient and good-natured. Role player was
very good at acting.

Feedback 5-10-2018 FAIL


You are in ED an young lady has come who has developed vaginal bleeding and she has 8 weeks
amenorrhoea. Your task is:
 Take further relevant hx
 PEFE , she will give you exactly what you want to know
 Talk with the pt abt ur diagnosis and dd
 No management is needed
This is the exactly the book case of Condition Number 79.
Nothing to add here. The roleplayer was an excellent actress. You need to show loads of
sympathy and empathy here as she she was losing her baby. Blood group was O+ve so no need
to talk about anti D.
Feedback: Failed (which I was v sure I’ll pass)
Key step1,2: Yes 3: No
Approach to pt: 5
History: 4 Choice & technique of examination: 5 DD: 3
Global score: 3

185-Large for date Uterus


Case 1 (Polyhydramnios)
Recall (2017)
You are in GP and 34 weeks pregnant lady come for antenatal checkup. Ur colleague saw her at
last appointment when she was 28 weeks and her FH at that time was 30 weeks. Now she is 34
weeks and SFH is 38 weeks. All the blood tests and USG have been done which are all normal.
Tasks
- Take further history
-Ask PE findings from examiner and investigations, where examiner will give u what u are asking
for
-Explain possible causes of this condition to patient

Recall 17-8-2018
a patient has presented to you at 32 or 30 weeks of gestation not sure about that, with increased
weight gain during last 4 weeks.
Tasks
-take history
-Ask the pefe and you can ask investigations what you want from examiner. And you will be told
only those which you will ask
-Explain the most probable cause to patient with reasons
-And mention investigations you want to repeat

History
1-Late pregnancy complications questions
-How’s your pregnancy so far? (everyone is saying my tummy is bigger than normal, its
uncomfortable)
-any tummy pain, vaginal bleeding or discharge?
-any headache, blurring of vision or leg swelling?
-any fever, nausea or vomiting?
-any burning or stinging on passing urine?
-any problem with your bowels?
-any dizziness, tiredness or funny racing of the heart?
-any shortness of breath?
-is the baby kicking well?

2-recurrent visits questions


-Have you had regular antenatal checks?
-How were the blood tests? Are you aware of your blood group?
-have you done down syndrome screening?
-US at 18-20 weeks gestation? Is it single baby? Any birth defects? What is the position of the
placenta?
-Sweet drink test at 28 weeks?
-Repeat ultrasound at 32/34 weeks?
-did you take your folic acid?
- Have you done your pap smear? What were the results?
3-General questions
-PMH (DM, fibroid)
-Family history (twins, big baby)
-SAD
-Pets, travel, raw meat

Physical Exam from examiner


1-General appearance (PODL)
pallor, oedema, dehydration, LAP

2-Vital signs (all)

3-quick CVS and chest

4- Abdomen:
-any tenderness
-any uterine contractions
-Fluid thrill
-what is the fundal height (fundal height is 6 or 7 cm large than dates)
-FHR
-lie and presentation
-is the lower pole of the uterus empty or not?

5-Pelvic:
-Inspection of the vulva and vagina: any bleed, discharge, rash, vesicles
-Speculum: cervical os open or closed, discharge or bleed from the os

Investigation you want from examiner


1-Office test: urine dipstick, blood sugar level (normal)
2-dating scan exam said normal, correct dates
3- torch screen ( normal)
4-18 week scan for *twin baby , *amount of amniotic fluid, *fibroids, *abnormal baby (all
normal)
5- Oral glucose tolerance test (normal)
6- repeated scan recently (no)
Explain diagnosis and Investigation
-From the history and examination, most likely you have a large than date uterus. As you are---
weeks but when measured the fundal height it is larger than it should be.

- There are several reasons why your uterus is larger than it should be:
*I am suspecting it could be due to polyhydramnios or an excessive amount of amniotic fluid
around the baby.
Infection, birth defects or DM can cause excessive amounts of fluid.
But still need to confirm it by referring you for USD.
Others are:
* Multiple pregnancy
* macrocosmic or big baby
* uterine fibroids.
*wrong dating,
however, these are less likely because your ultrasound at 18 weeks have confirmed your dates, a
singleton pregnancy, there is no evidence of diabetes, and there were no fibroids detected on
your ultrasound.

Management
-I need to refer you to the specialist

-arrange several Investigations


*FBC
*repeat OGTT
*viral serology (TORCHES)
*ultrasound to check amount of fluid around your baby.
*CTG to check the wellbeing of your baby.

-your pregnancy will be managed in high-risk pregnancy clinic for which you need to attend
frequent visit and follow-ups.

-Large than date uterus can cause problems in late pregnancy and labour including
malpresentation, premature rupture of the membranes, premature labour, and placental
abruption following membrane rupture.

-if at any time you have any water leakage from down below or bleeding or severe tummy pain ,
go to ED as soon as possible.

-I will give you reading materials about Large than date and polyhydramnios for further insight.
Case 1 Feedback

Feedback 17-8-2018 pass


Antenatal care (polyhydromnios handbook case),
Task-hx,pefe(ask any inv u want to review),d/d,inv
Hx-everyone is saying my tummy is bigger than normal, its discomfortable.
no hx/fhx of diabetes, no other medical condition, no fhx of congenital fetal abnormality, no
exposure to pets, vaccination uptodate, no s/s of dm or htn in pg,all antenatal test fine.
Pefe-uterus 6cm larger thn date, apart from that all r normal.
I asked for any inv available like bsl, usg(what pt told u that’s it) I asked again as task was any inv
u want to review, thn he said as I told u(bit angry I guess) office test-he asked me what u r
looking for specifically? i said bsl,urine dipstick-normal
dd-all 8 causes of hb
inv-same like hb USG,GTT etc.I said I need to send u to specialist,he might do some other
invasive test like amniocentesis(said how they will do this-taking some fluid from womb and
check whether any TORCH infection in baby)(score-5 here so enough inv).
I felt pt wasn’t happy as I had sob due to my physical illness, I had to take deep breath in
between and said sorry for that again and again bt still I had time-pt asked anything u want to
tell me? i said yes ur delivery is expected prior to due date, so red flag, stay near hospital, need
to seen by specialist within few days(all hb sentences)

Feedback 6-6-2018
large for gestational age- pass
? 29 wk pregnancy, regular patient at your clinic all AN screening tests were normal.
Complaining of abdominal discomfort and think her tummy is getting larger in size.
task: hx, PEFE, dx, ddx, arrange investigation for the patient
History: abdominal discomfort, no pain, no contraction, no nausea, no vomiting, pv
discharge/bleeding nil, no headache, visual disturbance nil, leg swelling nil, feel fetal
movement ,no urinary symptoms, no bowel symptoms, diabetes symptoms nil, pets nil, raw
meat nil. Sweet drink test done and normal, ultrasound done and normal, blood gp. LMP sure,
regular period before, DM hx nil, fibroid hx nil, family history of big baby present, no family
history of twin.
PEFE: fundal height ? 32 weeks size( 3wks larger than the gestational age given in the scenario),
single fetus, FHS normal.
Ddx :likely polyhydramios, explain causes of it, ddx: DM, big baby, wrong date , twins, fibroids,
placenta abnormalities.
Inv: baseline blood tests, TORCH screening, would like to repeat sweet drink test, USG, CTG.
(Explained all ,with layman term)
Feedback 17-8-2018 ANTENATAL CARE
A patient has presented to you at 32 or 30 weeks of gestation not sure about that, with
increased wt gain during last 4 weeks.
Your task is to take history
Ask the pefe and you can ask investigations what you want from examiner. And you will be told
only those which you will ask
Explain the most probable cause to pat with reasons And mention investigations you want to
repeat.
INSIDE
I started by asking about wt gain and little bit about diet and well preg questions, like any tummy
pain ,vaginal discharge, bleeding from down below, there was nothing concerning uptill now , all
was normal. Use of folic acid any supplements .she said her tummy is bigger than dates. I asked
about investigations like dating scan , first trimester infection screen (TORCH) baseline blood
and blood grouping, then 18 weeks scan that was normal single baby no mass in utrus or
abnormal location of placenta, or abnormality in baby, then I enquired about sweet drink test
that was completely normal ,no personal or family history of diabetes, no family history of big
babies was there. Nothing was positive in whole hist except wt gain and big tummy ( note that
uptill now we don’t know about fundal height) .
PEFE
Started from general appearance, vitals then moved to obs examination, fundal height was 6 or
7 cm large than dates . rest of exam was normal. Fluid thrill was negative. I specifically asked for
single baby or twin on physical exam as well. There was single baby with cephalic presentation. I
didn’t do pv in this case Then in investigatins I asked for dating scan exam said normal, correct
dates I asked torch screen that was normal
18 week scan , *twin baby , *amount of amniotic fluid, *fibroids, *abnormal baby, nothing was
present all normal. Oral glucose tolerance test, normal
Any repeated scan recently , exam said no. ( I was so panic at this stage to find some cause for
large for dates that’s why kept asking lol.)
Now moved to the pat and explained all the causes for large for dates first of all
polyhydroamnios told in layman and said its unlikely because no fluid thrill on examination but I
will refer you for USG so that we can see if its present . could be due to infection in pregnancy
they can also cause inc amount of fluid around baby , or any abnormality in baby. There could be
big baby due to diabetes in mum but also less likely in your case. Could be twins but less likely ,
or wrong dates , less likely. Then I said from the history and examination and the investigations I
could not find anything abnormal except big tummy I have tried to rule out all the possibilities
which can cause this but could not find. So at this time I am referring you to the specialist o that
he can also have a look at you and I am giving you a referral for usg so that we can check
amount of water around your baby and we can also repeat sweet drink test and infection
screen. I am referring you to high risk preg clinic and you will be taken care by them from now
onwards. Gave her red flags any water leakage from down below or bleeding or severe tummy
pain , go to ED as soon as possible.
3 key steps out of 5 covered History 4 Choice and technique of exam organization and
sequence5 Dds4 Choice of invest 4 Global 4
Feedback 14-3-2018
32 weeks pregnant lady with rapid weight gain over 4 weeks. (Don’t remember exact weight
changes and gestation).
Take Hx, PE from examiner and explain diagnosis to the patient.
2 min thinking time; look for causes.. Polyhydromnios, multiple pregnancies, fibroid, wrong
dates, placenta previa, fetal abnormalities, low Hb, choriocarcinoma, gestational diabetes,
preeclampsia, infections, family history of large babies etc.
After introduction, I told her that I saw in her notes that she had rapid increase in her weight in
these last few weeks. So I need to ask her few questions if she was okay with that. She agreed. I
asked her if she felt any difference recently? Or any rapid increase in the size of her belly? Any
difficulty breathing or swelling in legs? All negative.
The baby was kicking well and there was no discharge or bleeding from her private area. I said
let’s talk about her antenatal tests. I asked if there was any abnormality in her blood levels and
what her BG was. It was fine and she was O+ve. Wether her pregnancy was spontaneous or
assisted to R/O (multiple preg) Then I moved on to her 18 week growth scan to ask further about
multiple pregnancies, if the doctor commented to where her placenta was, or if there was any
abnormal growth in her womb, or if the baby was fine and growing normally? She hadn’t eaten
any raw or undercooked meat or didn’t come in contact with cats. Her OGTT was normal and no
previous or family history of DM or complicated pregnancies.
Her husband and herself were of normal built and height and nobody in the family was very
tall etc. She didn’t travel anywhere recently.
I took a moment from her to talk to the examiner. Vitals were all stable. No edema. Abdominal
exam was more important so I focused on that. Fundal height was 34/35cm above pubic
symphysis. FHR was normal. No fluid thrill present. Fetal head wasn’t engaged and fetal
presentation was cephalic. No proteins on urine dipstick and BSL was not available.

I got back to her and told along with drawing that after talking to her and the examiner it looked
like the rapid weight gain was due to a Larger for date gestation. There could be a lot of reasons
like polyhydraminos, placenta obstructing the mouth of womb, preeclampsia with increased BP
and release of proteins in urine but I couldn’t elicit any in history.
So I would refer her to the specialist to do furthur testing to find the cause. He may do an USG
to have a look in the womb and the baby. A CTG will be done to make sure the baby is doing
well. He may repeat OGTT if he suspected DM. And a couple of other blood tests. Asked she
understood?
Then told about complications of LDG mainly being preterm labour and malpresentation. So
the specialist may decide for an early delivery if any complications occurred. But from now on
there will be more frequent visits with doctor to keep her and her baby safe. Asked if she wanted
to ask any questions and the bell rang. Scenario … Antenatal Care
Grade… Pass
Global score… 5 Key steps 1,2,3,4…. Yes 5.. No
History…. 4 Choice of examination.…. 5
Diag/ D/D …. 5 Choice of investigations…. 5
Case (22/2/2017) (22/6/2017) (6/9/2017)
-polyhydramnios Karen case
- You are in GP and 34 weeks pregnant lady come for antetnatal checkup. U colleague saw her at
last appointment when she was 28 weeks and her FH at that time was 30 weeks. Now she is 34
weeks and SFH is 38 weeks. All the blood tests and USG have been done which are all normal.
Tasks
- Take further history
- Ask PE findings from examiner and investigations, where examiner will give u
what u are asking for
- Explain possible causes of this condition to pt
(You are not supposed to discuss about treatment)

2 mins thinking

- Oh, Is it new case? I didn’t see this in last year recall. Can it be transverse lie? No, it
should be reduced fundal height in T lie.
- It can be large for date uterus, my DDx are wrong date, polyhydramnios, big baby,
multiple pregnancy, fibroids, etc.
- In HO - I ll ask general Q and ask routine in PE. See, how it goes inside the room and I ll
play it accordingly.

History
When I stepped into the room, there was a re known AMC examiner whom I have seen in official
AMC videos. I greeted examiner and he checked my name card.

Greeting - Good morning. I am Dr Khine. I am one of the Drs in this general practice. Nice to
meet you Samantha. (Yep, Dr). I understand from ur notes that u are here for antenatal check up,
right? (Yep)
Antenatal Q -
-How’s ur preg going so far? (It’s going well).
-Did u do all antenatal check ups that u are supposed to do? (Yep), Are they normal? (yep)
-What about USG at 18 weeks? Do u have any concern for that? (No, Dr)
(I didn’t ask in detail like how many babies, placenta location, liquor coz the stem
mentions that it was normal)
-Sweet drink test at 28 weeks? (No problem). Ok that’s great.
Late preg Q -
-Any tummy pain, Any HA, blurring of vision, leg swelling (No)
-bleeding and water broke from down below? (No)
-Do u feel that ur tummy is larger than it should be? (I dun know)

5P - Is it first pregnancy (Yep), Any previous miscarriages (No), Stable relationship (Yes), Pap
smear (1 year back and it was normal)

SADMA, PMH (fibroid) , PSH - all clear


blood group and coombs test
PE Findings from examiner
-GA - Pallor, jaundice, oedema, lymphadenopathy (No)
-Vital signs - All normal
-Focus Abdominal exam
FH - 38 cm
Lie – Longitudinal
Presentation – Cephalic
-FHR - 140/min
-CVS, Resp – Normal
When I ask about PVE, examiner said no PVE has been done.
Bedside tests - not available
Then I asked USG report, examiner said “Ur patient has already told you”
(I was freaking out as the task mentions that ask invx results from examiner). I said “Sorry
examiner, my pt didn’t tell me anything.” Examiner said no USG report is available.
Any CTG report? (Examiner - what??) I said “Cardiotocogram”. (Examiner - for what?) (It widens
my eyes again) I said to check well being of baby. (Examiner - It s not available) Then examiner
told me talk to my patient. I thanked examiner
Explain to patient
Jenny, According to history and physical exam findings, I found out that ur tummy was larger
than it should be, we call it “large for date uterus”
It can be caused by a couple of reasons like wrong date, which means u wrongly memorise ur
LMP.
Then I draw a diagram of uterus with baby inside.
Other possible causes are big baby, increased amount of fluid inside bag around ur baby we call
it “polyhydramnios”, and if there are more than one babies inside ur womb.
Let me ask ur USG finding of 18 weeks, Did Dr tell u how many babies are there? (RP : it is
singleton pregnancy)
OK, that’s fine. So far, are u with me? (Yep, Dr)
Alright, the last cause is fibroid (I draw a fibroid in diagram) which is non cancerous growth of
muscle layer of ur womb.
Before pregnancy, did u notice any lumps in ur tummy? (No). (I should have asked any issue with
ur period like increased amount of bleeding)
So, we ll do a couple of investigations like repeat USG to find out the causes, and CTG which is a
special test to check wellbeing of ur baby. And we ll treat u accordingly.
(Mx is not task. So I didn’t talk about it)

Then the bell rang. (I should have told causes of polyhydramnios in this case but I didn’t)
AMC Feedback - Antenatal care : Fail (Global Score - 3)
History 4 Examination 4
Dx/ DDx 3 Invx 3
Feedback11-12-2018 ( Antenatal care) Global Score: 4 PASS
27-year-old female, 34w pregnant. She missed her last antenatal check. All her blood tests and US were
normal. Fundal height is 37cm.

Task: History, PEFE, Dx, DDx and Mx.

I came in the room and greeted the examiner and the patient.
I introduced myself and started by congratulating the patient for her pregnancy.
Than I started to slowly get into the questions:
How are you feeling right now? Any concerns?
Are you feeling any pain? Any vaginal discharge?
Is it your first pregnancy? Yes.
Was it planned? Yes.
When was your last period? Was it regular before? Yes. Any heavy bleeding? When and how did you
confirm your pregnancy?
Did you have early symptoms of pregnancy like morning nausea and breast tenderness? Yes. What about
now? No.
Were you on folic acid? Yes.
Did you make morphologic scan at around 20 weeks? Yes. Any concerns? No, it was normal. Was it only
one baby? Yes
Did you do the sugar test at 28 weeks? Yes. It was normal
Are you feeling the baby kicking? Yes
Any blurry vision? No. Headache? No. Tummy pain? No.
SADMA- unremarkable
When was your last HPV screen? (I don’t remember the answer!)
Do you have good support at home? Yes.
Is your partner the father of the baby? Yes. Is he a big man? Yes, doctor, he is a very big and bulky man.
Did anyone else in your family have big babies? Yes. My sister had three kids and they were all very big.
Was she diabetic? No.
Did you have any infection? No.
Are you coping well with the pregnancy? Yes
I excused her and asked the examiner for the PE:
I started with GA: “as you see”
Vital signs: normal
PICCLED
ENT: normal
RS: normal
CVS: normal
Abdomen: Soft, not tender. Fundal height: 37. Presentation and lying? Longitudinal and cephalic. Fetal
HR? 150. (I was a little confused at this part and the examiner did not seem happy)
‘With the consent of the patient and the presence of a chaperon I would like to perform the pelvic
examination. “ Examiner asked: “what do you want to know?”
On inspection: Any discharge? No. Any bleeding? No. Any rash or vesicles? No.
Sterile speculum examination: Any discharge? No. Any bleeding? No. Is os open or closed? Closed
Than I said that I was not going to perform the bimanual examination.
Asked for office test: Urine dipstick and BSL. (I don’t remember if it was normal or unavailable)
I thanked the examiner and went back to the patient: “ According to your history and PE, most likely the
cause of fundal height not compatible with the gestational date is because you have a big baby. That’s
what we call macrosomia. The reason is probably genetic because your husband is a big man and you have
family history of big babies. This is a very common cause.
Other causes could be wrong date, but you confirmed the pregnancy early right after missing your period;
big baby due to diabetes but your glucose levels are normal so far; multiple pregnancy but the US shows
only one baby, fibroids but you had no problems with your periods before pregnancy and it is not shown
on US; and polihydramnios.
You are due to a new US at 34 weeks, which I will arrange now.
On 36 weeks we will do the bug test and I will see you weekly.
When you go into labor, the obstetrician will follow you up. If they think baby is too big for vaginal
delivery, a C-section might be performed”
Bell rang and I had no time for red flags and reading material.
Key step 1:yes. Choice of investigation: 4
Key step 2:no. History: 5
Key step 3:yes. Choice & Technique of exam, organization and sequence: 3
Key step 4:yes. Diagnosis/Differential diagnoses: 5
Key step 5:yes.

Read Handbook 110


Case 2 (Large for date + measure SFH only yourself during PEFE + give only DDX without likely)
Large for date
GP, 27 years old lady 34 week pregnancy came to see you for routine ANC. It is the first baby for
her. She lives with her husband and he is supportive to her. She came regular ANC and test
results are normal. She underwent sweat drink test and her blood group is B positive.
Task:
Take history
PEFE and you do yourself SFH
Tell the patient possible causes

Appreciate. I asked ANC: bld test normal. USG normal & no fibroid. Pregnancy complications:
normal apart she feels tense in tummy and discomfort. Baby kicking (+).
No fever, no skin rash, jt pain, no exposure to animal poo & pee, no poo & pee p/b
No chronic medical p/b. No taking medication except folic acid.

Ask PEFE: No anaemia, no jaundice. Vitals: stable (normal BP). Ht and lungs: normal. Abd: soft,
no tender, no guarding & rigidity, no scar.
Examiner asked me to check pt abdomen. I forgot to wash hand & he remind me. Please make
sure the bed is completely flat (they intentionally elevate the bed upto 20'). I inspect abdomen
and palpate it but he said I want only SFH. I measured it (36 cm). Then I asked single fetus,
longitudinal lie, vertex presentation, not engaged yet. FHS (+) ?150/min. I defer VE.
Office test: urine dipstick: he said no UTI, no Sugar, no protein (I am sure he told one by one)
Blood sugar: NORMAL (I am sure I asked)
I asked patient LMP: she said she forgot.

I explained her Large for date: she asked what it is. Large the womb size compared with her
pregnancy age.
Possible causes: wrong date, polyhydramnios, big baby, Twin baby, baby problem, fibroid,
placenta problem.
Grade: Not scored
Case 3 (Wrong date)

CASE 2 Feedback
Feedback 18-9-2018
wrong date
A girl with 2 months pregnant, during PE found fundus height 30cm. Others normal
Task: Hx, explain the causes of her abdominal finding, order Ix
Hx: irregular periods, didn’t confirm the pregnancy, did take folic acid, never do antenatal check
ups. Not aware of blood group. It is not a planned pregnancy, but she decided to keep the baby
and so far happy with it. Started to feel baby kicking 10 weeks ago.
No other worrying symptoms during pregnancy. No trauma, infection, bleeding etc.
Explain: most likely, wrong date
Other possibilities: fibroids, polyhydramnios (all causes for this), chorioangioma
Ix: ultrasound
All test needed for first antenatal check ups.
Score: 5 Key steps: yes/yes/yes/yes Hx: 5, Dx/D/D: 5, Ix: 5

Feedback 18-9-2018 (????????? )


You are working in a GP clinic and your next patient has come to you with 20 weeks of
amenorrhoea. She did pregnacy test at home 1 month ago and it was positive. This is her first
antenatal check up so far. For the last 6 months She has been travelling overseas with her boy
friend and was out of Austrlia this whole time. She has no other abnormality or complain about
this pregnancy. On examination of this patient the following has been revild:
BP: 130/85 mmHg
Pulse: 88/min
Temp: normal
Per abdomen: SFH: 30 cm
Fetal parts felt and FHR : detected 142
Per vaginal: there was list but everything was normal.
Your task is to
-Take history from this pt regarding the cause of her syptom ( You should take no more than 5
mins in this task)
-Discuss the Dx and DDx with this pt with reason.
-Arrange necessary investigations.

My Approach:
I went in and there was a young lady sitting on a chair , she was so friendly and smiling all the
time. I greeted introduced myself then asked is it a planned pg? She said no. so I didn't
congratulate her. Then I asked ok but what is your view towards this pg? She smiled and said , ya
I am possitive and I wanna have this baby now. Then I smiled and appreciated her decision. Then
I asked all pg related questions in details, SADMA, 5P. All normal. asked about twins in family, big
babies and infection specifically. She wasn't immunised as well so I pointed it out. Her pap smear
was 3 years ago so I talked about cervical screening test and told that from now on you have to
repeat it in every 5 years if comes back normal. Her period was irregular , she wasn't sure about
her LMP.
Then I told it could be wrong dating as she wasn't sure abiut LMP.
As DDx I said twin pg, multiple pg, familial big baby, Polyhydramnios, Infections and explained
everything with a picture of the tummy.
As Ix I ordered all routine pg tests as it was her first visit.
FBE, Urine MCS, BSL, Urine deepstick, serology TORCH, Vericella, HIV, Hep B, A, and an
USG to find out the date and number of fetus.

AMC Feedback: Station 20: Antenatal care


Grade: Pass Global score: 4 Key steps: 1,2,3,4 yes,no,yes,yes
Assessment Domain: History: 4
Dx/DDx: 4 Choice of Investigations: 6

Case (7/3/2017)
20 weeks pregnant lady was overseas with her husband hasn’t seen anyone regarding her
pregnancy so when examined fundal height was 30 cm .. history and diagnosis and further
management
I took the history asked for any symptoms she denied then asked about first pregnancy or not
assisted or not any previous heavy bleed or fibroid.
any symptom of DM any family history of big baby or birth defect and also any contact with raw
meet any SLE she denied everything so I gave her all options and said high risk pregnancy US
viral serology and sle screen the bell rang but I think I should tell about the OGTT and the other
antenatal check as she has nor seen dr before

Case (7/3/2017)
GP, 30 yr old lady, she moved to another country with her husband 2yr ago and now came back
with 5 month pregnancy. She has never taken ANC before. On examination, u found that
temperature- 37.2C, fundal height-30cm, others are normal.
Task
-take history
-explain about your finding, possible diagnosis
-explain Investigation that you need to do to patient.
After reading the stem, I feel nervous a s it says possible diagnosis by taking only history. In
history, she has not taken ANC, don’t know blood group, no twin history, no USG and blood test
done, no fever, no rash, no BPV, LMP-5 months ago, Baby kick well, no known DM, also past
medical and surgical history-not relevant.
Pap smear not done and I said I will do it for you after our discussion then she said right now? I
said yes. Idon’tknow) Then, I explained that your temperature Is a little bit high( which I think)
and your pregnancy size is larger than it should be. (Until now, I don’t know what to tell about
diagnosis.) Then, I said may be due to polyhydraminion so I want to do ultrasound and check
baby, liquor index & placenta, also may be due to infection ( because I think she has low grade
fever) so Blood test for infection screening, blood sugar test to exclude DM. I also want to check
your blood group. Also, may be wrong date or twin ( which I forgot to tell her in exam). [I failed
this case.
Case 4 (Large for date early pregnancy) Feedback is good
Feedback 21-6-2018
11-weeks-old pregnant lady in her first pregnancy. She has done her first antenatal check-up, she was
supposed to do her first ultrasound tomorrow, but today had bleeding. So has come to see you. From 2 or
3 days ago she has developed sever nausea and vomiting. Take history, ask for examination findings, and
explain to her differentials.(Large for date 14 weeks, mild bleeding, no pain, no grape like structures, DDx)
First trimester complication: Pass/ Global score: 5
I asked the role player about the severity of her bleeding, and the number of pads she had used. Also
about the presence of any pain in the lower abdomen or down below. Then asked about the regularity of
her menstruation and concerns regarding her menstruations before getting pregnant. Asked about her
relationship, (in a stable relationship and having enough support from the partner) also asked about her
contraception before pregnancy, and the history of possible previous STIs. Also checked her symptoms of
pregnancy. She had nausea and mild vomiting, but she was capable of eating small portions of meals. And
she did not mention breast soreness. In examination, I checked general appearance, and anaemia, her
vital signs BP and PR, air entry in the lungs and heart examination, then in abdominal examination, asked
for tenderness, and any mass. In vaginal examination, checked for obvious bleeding from vagina, presence
of bubbles or grape-like structures in the blood. In speculum examination checked the cervix, cervical Os,
and presence of bleeding from cervix. (cervix was closed, no bubbles, but the origin of bleeding was from
the cervical os. In bimanual examination, there was no tenderness in uterus or adnexa, but the size of the
uterus was enlarged, (more than the age of the gestation). In office test, beta test was positive. I explained
to the role player that there could be several reasons for her bleeding. One reasons id that it is a
threatened abortion or just a minor bleeding due to implantation, but the reason for increased size of the
uterus could be firstly a twin pregnancy, and secondly molar pregnancy. And also talked about benign
lesions in her uterus called fibroids. So I explained that she needed to have an ultrasound examination to
confirm the reason for the increased uterine size and bleeding. [ I think in the stem her blood group was
something like O+, so I neither thought nor talked about Anti-D injection.

Station: 15 ( Non Scored)


You are an HMO and the next pt is a 10 wks pregnant lady came with a complaint of vaginal bleeding for 2
days. The bleeding started suddenly yesterday morning and she used 2 pads yesterday and 1 pad today.
Her all antenatal check up till now is normal. She doesn't have any tummy pain with it.
Your task is to
1. Take focused history form this pt in regard to find the cause of bleeding
2. PEFE
3. Arrange necessary investigation
4. Discuss Dx and DDx with the pt.
My Approach:
I went in and pt started crying, I told her that we are here to help, offered her tissue and water then went
back to examiner to ask about vital which were normal. Then I asked about PV examination, inspection
and speculum to make sure if there is any product of conception ( Incomplete abortion). OS was closed
and no product but bleeding present, no sign of trauma.
Then I asked about abdominal pain: Negative. Then asked Urin deepstivk, BSL .... Examiner said ketone
present. Then I told my pt about Threatened abortion and explained the DDx as incomplete abortion,
molar pg, twin pg, UTI. Arrange an USG, FBE, UEC, urine MCS. There wasn't any Mx task so I didn't talk
about it.
AMC Feedback: Assessed but not scored
186-Tension Headache

Feedback 27-6-2018 pass


Man complain of headache for 4 weeks
tasks
1-take history including psychosocial history
2-physical examination findings from examiner
3-dx and ddx to patient

Task 1
patient responded to each question with long answers so I needed to be listening carefully
showing some empathy.

History

1-Headache questions
-Hi—my name is --- I will be the doctor looking after you today. I can see from the notes that you
are suffering from headaches for 4 weeks is that right? (yes)
it must be very distressing for you so how’s your pain now from 1 to 10; 1 is least 10 is the most.
(5 or 6). do you want pain killers I can arrange one for you (yes please) do you have allergy to any
medication (no). I’m gonna arrange painkiller for you
-now I just want to ask you a few questions in order to unravel the nature of the problem would
that be ok? (yes)
-You say you have headaches for 4 weeks, is it constant or come and go (constant)
-has it started suddenly or gradually (gradually)
-is it increasing or decreasing in severity?
-can you show me exactly where you have it? (all around my head)
-does it go anywhere else?
-does anything make it better or worse?
-can you describe it for me? Is it like a band or tight? (Yes)
-Has this happened before?

2-Associated symptoms questions (to rule out Ddx)


-Any nausea or vomiting? is it associated with sensitivity to noise or light?
-Any fever? Recent viral infections?
-Any blurring of vision?
-any pain anywhere else? Neck or face pain?
-weakness, tingling or numbness?
-neck stiffness, rash?
-LOW, LOA, lumps or bumps?
-head trauma?
3-Psychosocial history
-now I just want to ask you a few other questions and let me assure you that everything we
gonna discuss will be confidential between you and I unless there is a harm to yourself or
others? (ok)

Mood questions
-how’s your mood? (low)
-have you lost interest in things you used to enjoy? ( yes)
-for how long have you been feeling low with lost of interest? (more than 6 weeks)
-how’s your sleep? Do you find it difficult to get to sleep or you wake up early in the morning and
find it difficult to get back to sleep? (wake at 4 am and find it hard to go back to sleep)
-have you lost or put on weight recently? (lost some kg don’t remember)
-do you find it hard to concentrate on things? (yes)
-do you feel guilty about something?
-have you ever thought of harming yourself or others? Any previous plan or attempt? (no)

-Now I’m just gonna ask you questions that would be sensitive but there are just routine to ask
all my patients
do you feel, hear or see things that others don’t? (no)
do you think someone is following you or trying to harm you? No

HEADS.
-who do you live with? Any stress at home? Any support?
-What do you do for living? Any stress at work? (lot’s of stress at work and can’t concentrate life
very hard)
-how’s your social activity? (hard to go out with friends)
-SAD (no illcit drugs but maybe drinking or smoking heavily so we need just to say it is important
to drink alcohol within the safe limit which is no more than 2 standard drinks a day I can arrange
another consultation to help you with it)

4-General
-PMH, PSH, Medications
-Family history
Task 2 (Ideally do like headache examination)
thank you I need to examine you for this I want to talk to the examiner to get the physical
findings.
-GA --- examiner as you see
-VS--- examiner all normal
-pallor, jaundice, LAP – examiner –ve
-what about cranial nerves, fundoscopy and peripheral verves –normal.
-neck examination--- examiner tenderness on the back of the neck +ve
thanked the examiner and returned to my patient.

Task 3
-from history and examination there are few possibilities it could be migraine but less likely as it
usually present on one side of the head and not persist for that long time like 4 weeks sometime
with nausea, vomiting, sensitivity to Nosie or light and family history +ve.
-It could be due to infection of sinuses or air sacs around the face also unlikely.
-most likely cause is tension headache which is headache like tight or band around the head
constant associated with stress as from the history you are going through lots of stress at work
and this affecting your mood as well and disturbing your sleep.
I can understand that you have lots of stress but you also need to deal with it by do some
relaxation technique like yoga or meditation even at work and also find some hoppy
-it could be due do spasm of neck muscles as there is some tenderness on the back of the neck
or it could be problems with backbone of the neck.

When bell rang just told quickly


-from the history, you have also low mood and lost interest with loss of weight and sleep
disturbance for more than 2 weeks so it seemed like you also have depression symptoms (not
you have depression or major depression) that may contribute to the headache you have.
(not sure about depression so do not make it a diagnosis. Not sure if you need to put it among
ddx)
Feedback 22-6-2018
32 male pt coming in because of recurrent headaches.
Task:
hx,
pe from examiner he will only give you
what you ask, dx and ddx with reasons
Approach
Greeted pt
Asked on a scale of 1-10 how bad is the pain and would u like painkillers.. he said 4 and no.
Then said can you tell me a little bit more about it.
Pt said he is worried he has brain tumor as his cousin had brain tumor. I said I am so sorry
about your cousin how is he doing now? He said he is getting treatment. I told him I
understand your concern, there are many causes of headache I will asses you thoroughly
and together we will come up with a management plan.
Then asked about siqorra
DDqs regarding (mixed in siqorra and PE to save time)
Migrane, Cluster, Tension
Temporal arteritis, Trauma, Tumor
Meningitis, Stroke, Sinusitis
Ear infection, Tooth infection, Blood pressure hx, Medication
Examiner prompted me as mini bell rang for next task
Quickly asked work and financial stress
Mood and anxiety
( positive for tension headache)

PEFE
GA, Palor
Vitals, BMI
Nervous system , Cranial nerve, Fundoscopy
Neck stiffness
Ulimb, Lower limb
ENT, Sinus, Ear and tooth
CVS just s1, s2 added sound or murmur
Neck musculo skeletal for cervicle spondylitis—said normal

Dx
Mentioned there can be number of causes and drew head
Said dx most likely tension headache as he has stressful work with evening headache and
worried about cousin
Then mentioned 4 dd’s with reason bell rang so I mention the name of 4 more
Examiner opened door I said thank you to role player and examiner
(headache time management has always been difficult for me and the roleplayer did not
make it easy but the examiner was very fast with the pe so I was grateful)
Feedback 13-12-2018
Headache: Pass
Again, don’t remember the task exactly. Complain of headache.
Tasks: History, PEFE, Dx and DDs
2 min thinking: DDs of headache: Migraine, tension headache, Intra cranial tumour, aneurysm, cluster
headache, Eye and ear issues, temporal arteritis, dental issue etc
Inside: Me: Sorry to hear about your headache, can u pl tell me about your headache.

Pt: I have this headache since many months, I am worried whether it is a tumour
Me: Don’t worry, we will find out. Please tell me more.
Pt: feeling tight around headache.
Me: can u show me where exactly? Pt: She showed around the head like a crown headache.
Me: since how many days? (since 1-2 months), whole day? (yes, more towards end of day), severity?
(4-5) Do u need any medication now? (No) have u taken medication (Panadol), does it relieve (yes).
Any aggravating and relieving factors (sleep and alcohol as relieving factor), any aura, weird smell, or
pain looking in bright light? (no), relieved by vomiting (No), Any water coming out from eyes or nose
(no), any pain on touching on side of head or cord like structure (no) and eye, ear or dental issue (no),
any neck issue (no). Any early morning headache and vomiting (no).
Me: pl tell me more about your family?
Pt: busy working whole day, have 2 kids and busy with them as well.
Me: what about your working life? Any stress at work?
She: Sort of, but too much work.
Me: R u a perfectionist? Pt: yes sort off, I spend many hours on work.
Me: asked about present, past and family history (all negative). Asked about SADMA (social drinker).
PEFE: General appearance: as u look at her
Vitals: all normal
Inspection: no abnormality (I asked about scarring, deformity, redness etc)
Palpation: Examiner: what do u want to know?
Me: any tenderness around head: Exami: tenderness at occipital area.
Me: Any tenderness around neck (no). Neck muscle tense? (Examiner gave me surprised look, She
said no. I read somewhere that in tension headache pt u get tense neck muscles).
Me: Eyes plus fundoscopy: Normal, ears: normal, Temporal area: normal, mouth and neck
examination: Normal. Respiratory system (Very important for metastasis): Normal. CVS system:
normal.
Dx: Don’t worry, according to history I think u do not have brain tumour. R u happy now? Pt: yes.
Me; There are many possibilities, but I think u r suffering from Tension headache, do u know what is
it? Pt: No.
Me: It occurs in young woman, who has stressful work plus who has perfectionist personality.
Headache like band around head occur more towards end of day.
Other less likely possibilities such as migraine ( no because, no aura, no photophobia, no F/h etc),
cluster headache where u get headache in night ( less likely as no features), Temporal arteries ( less
likely as no tender area on side of head, and no cord like structure), eyes, ear and dental and neck
issues ( less likely as no symptoms suggest that) and very less likely brain tumour ( very less likely as
no typical symptoms).
Global score: 4 Key steps: 1 No, 2 and 3 Yes Approach to Pt: 5
History, Choice and technique of examination, organisation and sequence and Dx/dds: all 4
187-Asthma severity assessment
32-year-old female with history of asthma admitted to hospital because of worsening of her
asthma. She is on budesonide and salbutamol and needs to use puffer 5-6 times a week and her
asthma frequently wake her up at night
nurse notes makes sure that
-the patient is taking her medication regularly
-knows how to use puffer correctly.
-no change in her home or work condition
tasks
-take history from the patient to know why her asthma not well controlled
-assess her level of asthma and tell the patient about the level
-tell her about acute management of her asthma

History
*when entered the room there was a patient sitting and there was a puffer on the table beside
us.
1-Asthma severity assessment questions
-I can see from the notes that you have asthma and been admitted to hospital because of
worsening of it. How do you feel now? Any concern? I just need to ask you a few questions to
assess your current asthma condition and know the reason beyond worsening. Is that alright?
-How many attacks of asthma do you have at daytime? (Said has 5-6 times a day although in the
stem it was per week)
-how many attacks do you have at night? (5-6 times a day including 2-3 times at night)
-do you wake up at night due to asthma? (Yes)
-how does asthma affect your physical activity and exercise? (Affect exercise)
-how many times do you need to use puffer? (Many times)
-how many timed have you had exacerbation and hospitalisation due to asthma? (just once)
-how does it affect your life and job? (the stem told there is no change)
*so from the history it appeared that her asthma level was severe (may be moderate)

2-Risk factors of asthma (most of them already mentioned in the stem except smoking which is
the most important point)
-previous viral infections
-smoking (smoke 10 cigarettes a day for 10 years).
-Pet, dust, carpet
-past medical history including history allergy to anything
-Medications
-alcohol
-Occupation + stress (do not remember but there was some stress at work)
--family history of eczema, hay fever, asthma (think in the stem family history was negative.
-do you use the puffer regularly? (in the stem mentioned that the nurse checked the technique it
was good and also mentioned she was using it regularly)
Counselling
-Thanks for the information. From the history the most likely cause why your asthma is not
controlled because of smoking history. You are smoking 10 Cig a day for a long period of time
and smoking can affect asthma and worsen it. So this is the main factor that I could illicit form
the history.
-also from the history, it seems that your asthma level is severe.
-Regarding management of your asthma and to prevent similar attacks in the future
1-Smoking
-it is very important to stop smoking and I can arrange another consultation for you to speak
about smoking its risks and methods to help you stopping it. (But smoking helps me with the
stress at work)
I know you are experiencing some stress at work but it is also important to stop smoking in order
to let your asthma be under control and avoiding recurrent severe attacks.
How does that sound to you? Do you want me to talk about smoking? (yes)
** told her briefly like this (there are NRT in forms of nicotine gum and patch and champix
(varnecilline) I give you 2 week to think about it with reading materials and then come back to
talk about it. There is also a quit line and lots of support group for you available.
2-specialist
-regarding your current inhaler asthma medication, I want to refer you to the specialist to adjust
it with the current level of your asthma, they might need to increase the dose, can also prescribe
or add oral steroid for more asthma control. (Other candidates told that increasing dose is not in
the current guideline)
3-Action plan
-I will also give write down an action plan for your asthma (what is that doctor??)
-it is like a paper with colours indicating levels of asthma like mild, moderate, severe or very
severe. It consists of some instructions to guide and help you what to do if you have asthma
attack of any stage and what to do in urgent situation.
4-Stress
-and for stress devote some of your time to Yuga and meditations. Attain health lifestyle and
there are also support group.
**other feedback talked about exercise (As for your exercise I put you some medication called
intal which you can use it before exercise as normal and regular exercises are important for you)
5-Spcaer (there is a puffer on the table).
- I can see that the nurse checked your technique and it was good. You also using puffer
regularly. (Other candidates just checked the expiry date and put it again on the table)
- you can ask her if she use spacer then you can advise her about using spacer as the benefit of
the spacer is to let the medication be used completely and won’t get wasted.!! (not sure about
this addition)
Case (13/10/2017)
A young woman comes to your practice due to her problem with asthma control. She is on
Budesonide 200 mcg twice a day. The nurse checked her technique which was good.
Task:
Take hx to assess her severity of her problem.
Address her problem regarding acute management!
I entered the room and introduced myself and she was really nice RP as she calm me with her
positive and cooperative attitude which really helped me as I was enormously stressed due to
previous stations.
History
- She had 2-3 times night attacks during the night.
- And daily attack every day
- but she can eat and talk with no problem
- however she has problem with exercise these days.
- She is smoking 10 cig per day for 10 years( I said that is too much we have talk about it next
consultation OK?) Yup…
- No allergy to pollens, linens, perfumes, food, no skin changes.
- She was a lawyer and has so much stress these days that is why she is smoking more
frequently. No family hx of asthma. No medications. She has enough support at home and no
problem socially, drinks alcohol occasionally.
(Action plan)
- I said regarding your asthma control it is very important to decrease the amount of smoking I
need to arrange another consultation to talk about your smoking habits. But do you want me to
briefly talk about it? She said yes….
I said Ok there are NRT in forms of nicotine gum and patch and champix( varnecilline) I give you
2 week to think about it with reading materials and then come back to talk about it.
There is quit line and lots of support group for you available.
- Meanwhile I want to add Floxitide to your puffer!!!( I totally forgot she is on Budenoside) as
your asthma is moderate to severe and needs to be contolled.
- As for your exercise I put you some medication called intal which you can use it before exercise
as normal and regular exercises are important for you.
- As for you stress devote some of your time to Yuga and meditations if you have specific
concerns I can refer you to psychologist as well..
- I said can you show me how you use puffer because there was a puffer on table she said your
nurse checked that and she was happy with that…I said so I am happy now as she is as good as
me!… bell rang!....
I did not talk about action plan and PFM!!! And I did not explain about asthma pathologies and
mechanism…and I think we should start prednisolone for this patient…but main things is
smoking here to be addressed. I am not sure for this case…
Feedback: Health review, PASS(G.S:4) Key steps:1 ,2,3 and4: Yes
Approach to patient:6 Hx:4
Dx/DDx:4 Patient counseling:4
Feedback 25-10-2018
Scenario: health r/v
Stem : young boy in her early 20s with frequent attacks of asthma
Tasks:
Hx
Reason for his current asthma condition
Future mx plan
In hx I asked since when he had asthma, asked qs about trigger factors, smoking hx positive. Rest
all negative (dust, seasonal, exercise etc)

Told her the reason is smoking. And he needs to quit it. ASAP and I’ll give him a good plan n
followup closely to see his progress. He was happy with it, I told him future mx will go according
to asthma action plan which I will discuss after telling him overview of quitting strategy (yes I got
confused should I take this station as smoking counselling or asthma mx) he asked me doc what
is this asthma action plan u just mentioned (I dropped my idea of going towards smoking
counselling and explained the asthma action plan as per hb)

Gs: 4 4/4 key steps covered


Approach:5 Hx: 4 Dx/ddx: 4 Pt counselling/education : 4
188-pregnancy induced hypertension
Sample case/ You are a GP when 31 years old mary who is 34 weeks pregnant comes for regular
antenatal check-up. During her last ANC 2 weeks ago with another GP her BP was 110/80 mm/hg
but today the BP is 150/110 rest otherwise normal.
Tasks
-History
-PEFE
-Dx and further Mx with patient

Differential diagnosis
1-preeclampsia
2-Pregnancy induced hypertension
3-chronic hypertension
4-secondary hypertension

History
1-regular visit questions questions
-How’s your pregnancy so far?
-is this your first pregnancy?
-have you had regular antenatal checkups?
-regular blood tests? Are you aware of your blood group?
-how about down screening 11-13?
-Ultrasound 18 weeks? And repeated US at 32 weeks?
-sugar test at 28 weeks?

2- Preeclampsia questions
-is there any high BP recording during any previous visits?
-any time you have leaked protein into urine?
-do you have high blood pressure before pregnancy
-any headache, blurring of vision, leg swelling?
-confusion, drowsiness, vomiting?
-tummy pain, vaginal bleeding or discharge?
-dizziness, palpitation, chest pain?
-is your baby kicking?

3-Secondary causes
-how is your diet?
-do you exercise regularly?
-SAD
-stress at work or home

4-Support

5-Past medical history (heart or kidney diseases)


Physical examination from examiner
1-general appearance PODL
pallor, oedema, dehydration, LAP

2-Vital signs (monitor once again)

3-CVS (all)

4-quick respiratory system

5-CNS (-ve)
-hypertonia, hyperreflexia
-fundoscopy (for papilledema)

6-Abdomen examination
-uterine tenderness, contractions
-fundal height (34)
-fetal lie and presentation
-fetal heart rate (normal)

7-pelvic examination
-inspection for bleeding, discharge, vesicles, rash
-speculum for cervical OS

8-office tests
-urine dipstick for proteinuria (-ve)
-BSL
-ECG

Explain the diagnosis


-today the blood pressure that was recorded showed a bit rise so most likely you have PIH or
gestational hypertension. Which is the onset of high blood pressure after 20 weeks of pregnancy
but not associated with protein leakage into the urine and also no end organ damage.
-exact cause is unclear but could be due to hormonal changes in pregnancy.
-if it is not well controlled it can lead to a condition called preeclampsia where there will be a
sharp rise of BP and protein leakage into urine. So a far as possible this need to be prevented.

Management
-we need to record your blood pressure in 4 hours time and if it is elevated then you have
gestational hypertension
-if you have pregnancy induced hypertension further blood tests need to be done like FBC, UCE,
LFT which will sometimes be repeated later.
-urine also need to be checked for protein creatinine ratio and urine protein need to be looked
for weekly.
-refer to high-risk pregnancy clinic to be seen by a specialist obstetrician.
-need to be out on oral anti-hypertensive (oral labetalol for example) (this is done at high risk
clinic not you)
-Ultrasound will be done now then at 2-4 weeks intervals
-CTG also for wellbeing of the baby

Warning signs
just in case any headache, blurring of vision, tummy pain, excessive swelling, baby not kicking
please report to ED ASAP.

Further management
-rest in lateral position if possible
-no added salt in your diet
-life style modification (healthy diet, regular exercise)
-our goal is to control BP and prevent preeclampsia and fit from happening and make pregnancy
and delivery safe for you and the baby.

Reading materials

Feedback (22/2/2017) FAIL


You are in GP, 34 weeks pregnant lady come to you for regular antenatal checkup, 1 st antenatal
visit BP was 110/80 but now BP is 150/100 and other vitals are given which are stable. Other
antenatal blood checks and USG were normal.

Tasks - Take relevant history

- Ask PE findings from examiner

- Discuss with patient how u will manage this case

2 min thinking time

- This might be typical recall of PE or Eclampsia

- The patient may throw fits when I ask PE from examiner

- Not to miss PE symptoms, late preg Q, BP before preg, family history and 5P in HO,
fundoscopy, CNS and urine dipstick on examination, and first aid measures if pt has fits

History

When I stepped into the room, the examiner checked my ID and introduced me the name of
roleplayer.

Greeting - Hi Good morning, I am Dr Khine. I am one of the Drs in this GP. Nice to meet u Lindsay.

(RP : Yes, Dr. nice to meet you. What happened to my BP?)


Yep, let me ask you a few questions in order to know what’s going on with your BP but before we
start I d like to talk to my examiner first and I ll get back to u soon. (RP : Sure)
(I didn’t congratulate pt about her pregnancy coz it s now 34 weeks, a lot of ppl has
congratulated her and u have seen this pt before, so I think it’s a bit weird to say it. But I will
surely congratulate her if it is early trimester preg)
Examiner, considering the high BP of my patient, I d like to move her to the treatment room, put
IV line just to maintain the lumen of vessel open and attach her to BP monitor.
(Examiner nodded his head)
Then I turned to roleplayer.

Antenatal Q –
-How’s ur pregnancy going so far? (RP : It’s going fine)
-Did u do all the antenatal blood checks? (Yes) Are they normal (Yes)
-What about USG at 18 weeks? (normal)
-Sweet drink test at 28 weeks? (no problem)
-Alright, I understand that ur BP is a bit high today. How’s ur BP in previous visits? (normal)
-Did the Dr tell u that ur urine contains protein? (No)

Pre eclampsia -
At the moment, do u have any headache, vision problem, tummy pain, N, V (No) Do u have any
oedema of legs? (RP : what do u mean?) Oh, sorry, do u have any legs swelling? (No)

(This was my first station and it was a bit hard to control my nerves but things become settled
and calm starting from second station)
Late preg Q -
Do u have any bleeding from down below? Any water broke? (No) Do u feel ur baby is kicking
well? (Yes) Ok. That’s good. Urine and bowel work? (Fine)
Past HO -
Do u have high BP before u get pregnant? (No), any other significant health problems like DM,
heart ds (No)
Family HO – Any family HO of high BP during preg? (No)
5P -
Is it ur first pregnancy? (Yes), previous miscarriages (No), stable relationship (yes), pap smear
(normal)
Thank you Lindsay for ur information. I ll ask my examiner for some findings and I ll get back to u
soon.

Physical Exam
GA - Any pallor, jaundice, leg oedema (No).
VS - T - normal, BP - 150/100, PR - 100, others – normal
Fundoscopy – normal
CNS examination esp reflexes – normal
Then, I d like to focus on abdominal exam, FH - 34 weeks, Lie - longitudinal, Presentation -
Cephalic, FHS - 140/min
I d like to complete my exam by doing resp and CVS exam (Normal).
I d like to check UDS for protein, nitrates, and leucocytes (All negative)
(Actually, I was expecting at least positive urine protein or pt throwed fits while asking PE, but
there was nothing. So this is just preg induced HT)
Management

Well, Ms Lindsay, according to HO and PE findings, most likely condition u are having now is
called pregnancy induced hypertension. It means high BP during pregnancy which is caused by
hormones secreted from placenta during pregnancy.

(RP : Is it due to hormones? Yes, it is caused by hormones that raise ur BP during pregnancy)

This is not very uncommon condition. Let me reassure u that it is well manageable and u can
have normal delivery and healthy baby if we treat it properly. (RP : Yep)

So, what we have to do for now is I will refer u to the high risk pregnancy clinic and manage u
with MDT including pregnancy specialist, specially trained nurses and me as a GP.
U need to go for frequent antenatal check up as it is high risk pregnancy. We ll check ur BP in
every visit and check protein in ur urine. Specialist will do USG of ur tummy and CTG to check
well being of ur baby. Depending on ur severity, specialist will decide how frequent we need to
do these tests.
In terms of delivery, it depends on ur condition, baby’s condition and the severity of ur high BP.
Specialist will decide it. It can be normal delivery or CS.

I will review you regularly. I ll give u reading materials to make u understand better ur condition.
I am gonna write down the referral letter for u to see specialist. And in the mean time, I want u
to be aware of some red flags features. (RP : what do u mean?) (seems like she doesn’t
understand the word red flags)

Ok, I ll let u know about some important symptoms that u need to be aware of. If u have any
severe headache, blurring of vision, leg swelling, bleeding from down below, u feel that baby is
not kicking well, just come back to see me or u can go straight forward to ED asap.

The bell rang. I thanked to roleplayer and examiner and came out of the room.

(What I should have done - Specialist will start u on medication to lower down ur BP)

AMC Feedback - Antenatal Care : Fail (Global Score - 3)

History - 6
Examination - 5
Diagnosis - 6
Choice of Invx - 3
Management - 1
Feedback 8-11-2018 (IMPORTANT)
189-Gestational hypertension headache
Check lecture 52 and 188 for similar approach

Feedback 23-6-2018
Feedback: Antenatal care: pregnant 30 weeks, with headache. Take Hx, PEFE, DDX, management.
Approach: vital signs? BP: 150/90. Any pains? Pain killers?
Hx:
Headache nature? nature, score, where, radiation, increasing and decreasing factors,…?
Pre-eclampsia possiblity: visual disturbance, shaking, vomitting, tummy pain, urine change,
seizure, legs
edema.
Pregnancy history: tests, ultrasounds, GSL.
6Ps and medicine previous HT: NO, surgery,…

PEFE:
Vital signs?
Fundoscopy?: normal
CVS and chest: normal
Obstetric examination: fundal height (30cm), cephalic, vertical, no pain, fetal heart positive.
Relfexes: normal
Urine test: normal

Dx: Gestational HTN


Management: refer to high risk clinic, MDT to control your HT and OB specilaist will make a
delivery
plan for you in controlled manner, do swab at 36 weeks.
Red flags: seizure, visual disturbance, tummy pain, water break, bleeding  Hospital.

Grade: PASS, GS: 5


Key steps 1,2,3,4: Y (all covered)
Hx:5, PEFE: 5, Dx: 5, Management: 5.

190-Concealed placental abruption


You are an HMO at the ED of a major hospital, when 25 year old Maria who is 32 weeks
pregnant, presents to you with severe abdominal pain since the last 2 hours.
TASKS
-Relevant history
-PE from examiner
-Diagnosis and management

AMC Case 21-2-2018


A young pregnant lady in her 32 weeks gestation presented to the ED with abdominal pain
tasks
-history
-PEFE
-diagnosis and management

Differential Diagnosis
1-Preterm labour.
2-Abruptio placentae.
3-preeclampsia
4-Trauma
5-Torsion of ovarian cyst
6-Medical causes: UTI, appendicitis, cholecystitis, pancreatitis

History
1-pain questions
when you will enter the room the patient will be crying and rolling because of pain
-is my patient hemodynamically stable?
-I read from the notes that you are having tummy pain. Can you score the pain from 1 to 10, 10
being the worst pain? (7/10) I will give you some painkillers, do you have any allergies?
-when did it start? Is it sudden or gradual? (for few hours )
-Where is the site of pain? (All over the tummy)
-Is the pain going somewhere else?
-Does anything make it better or worse?
-has this happened before?

2-late pregnancy complications/ DDX questions


Preterm labour/ PROM
-do you think that the pain is coming intermittently or is it a continuous pain? (Constant)
-Has the pain been coming at regular intervals or not? (if not constant ask)
-any Fluid leakage?
Placenta abruption
-Any bleeding from down below? (no)
Pre-eclampsia:
any headache, blurring of vision, dizziness, oedema? (no)

Trauma:
-any hit or falls prior to the abdominal pain? (No)
Acute abdomen (UTI/appendicitis/cholecystitis/pancreatitis): (normal)
-how are your waterworks? Any burning or stinging during urination? Any smelly urine?
-How are your bowels? Do you open your bowels regularly?
-Any fever or vomiting?
Baby kick
-Do you feel your baby kicking? How long have you noticed that your baby is not kicking? (Not
able to feel baby movements)

Recurrent visits questions


-Did you do your antenatal checks, blood tests and blood group?
-Have you done your down syndrome screening?
-Ultrasound at 18 weeks? What is the position of the placenta?
-Sweet drink test at 28 weeks?
-Did you take your folic acid?

4-General
-Any bleeding disorders?
-Any medications taken?
-Any past history of medical or surgical illness?
-Any smoking, alcohol or recreational drugs?
-Do you have a good support?

Physical Exam
1-GA
2-Vitals (postural drop present)
-Put in 2 large bore IV cannula, and take blood for examinations (FBE, UEC, Blood group,
crossmatching, coagulation profile, BSL)
-Give IV fluid.
4-Abdomen
-Look for uterine tenderness, rigidity and guarding, Feel for uterine contractions (generalised
tenderness, uterus -/ contracted or rigid on palpation)
-Fundal height: 36 cm (blood is collecting inside)
-FHR: not heard
-Lie and presentation: hard to feel the lie and presentation
5-Pelvic examination (normal).
-Inspection of the vulva and vagina: bleeding, discharge, rash, vesicles
-Speculum: bleeding from the cervix, if the cervix is closed or not
-DON'T DO PER VAGINAL EXAM, because the patient is in shock
6-Office tests:
UDT, BSL

Management
-From the history and examination, most likely you are having a condition called a severe,
concealed abruptio placentae.
-The placenta is the part that connects you to your baby, and which carries oxygen and food
from you to your baby. This placenta usually separates from the uterus after the delivery of the
baby. But if the placenta separates from the womb, during pregnancy, it is called abruptio
placentae, which could be revealed or concealed depending on whether the bleeding is coming
out or not. In your case, it is the concealed kind, the blood is collecting inside.

Breaking bad news:


-Unfortunately, the next news that I have for you is not very good.
-Do you want somebody to be with you while I discuss this with you? I can call your partner so
that he can be with you during this time.
-As you said, you have not been feeling the baby kick since this morning. And during my
examination as well, I could not hear the baby's heartbeat.
There is a probability that something nasty might have happened to the baby. But we need to
confirm this by doing an ultrasound to look at the condition.

-You will be admitted and seen by the specialist and the specialist might decide to do a C-section,
as the blood that has clotted inside needs to be evacuated or removed. Along with the placenta
and the baby, the leaking blood vessels needs to be seen.
-Once the emergency management is over, we can offer you and your partner counselling
services as well.
-I know this is a difficult time for you as you are hoping for this baby, but we are here to help
you.

Feedback 21-2-2018
CASE 5: OG- IUD WITH CONCEALED ABRUPTION –PASS
young pregnant lady with tummy pain .tasks history, PEFE, management

while entering the room ,patient was lying in bed and crying and rolling with pain.asked pain
scale, offered painkiller, all pain questions, throughout the station she was crying with pain.
Pain all over the tummy, acute, no bleeding or discharge from vagina; not able to feel baby
movements, no bladder or bowel problem, no trauma immediately I arranged for doppler to
check fetal heart rate,all antenatal history normal so far.

Pefe; vitals postural drop present , told examiner that I secure cannula and start fluids.
Abd exn- generalised tenderness, uterus -/ contracted or rigid on palpation, absent fetal heart
rate, pelvic exn normal

Explained patient IUD due to ? Bleed – Placenta separated before labour. Offered support , asked
for partner, etc
arrange blood investigations, blood group and hold, specialist might arrange immediate surgery
with stable vitals.

Feedback placenta abruption


Abdominal Pain (FAIL)
You’re HMO in ED, 27 years old lady with 32 weeks pregnany comes with severe abdominal pain
which started this morning. This is her 3rd pregnancy.
Tasks :
History
Ask PEFE
Dx with reasons.

Approach :
I greeted the patient. Offered her pain killer as patient seems very painful. Then I checked vital
signs and examiner said BP 90/50 mmHg, PR 100/min. When I told that I would like to do
resuscitation, examiner said no management needed.

History :
Asked about pain questions. This is the 1st time, the pain is very severe and she felt it all over
the tummy. No radiation. Nothing makes it better or worse.
Then asked DDx - Does not look like contraction. No bleeding or watery discharge from down
below. No history of injury. No fever, nausea, vomiting. No problem with poo and pee. No history
of fibroid. Then I asked about ANC which is unremarkable. I forgot to ask about fetal kicking. Past
Obstetric H/o - she delivered both babies normally and they’re fine. Everything was
unremarkable. PMH and PSH – PMH is unremarkable but she had laproscopic appendicectomy
when she was young. Social h/o – unremarkable.

PE :
GA – patient is in pain
VS – the same
EYES – slight pallor +, no jaundice
CVS, RESP – normal
ABDOMEN - Inspection - laproscopic scars in RIF.
OBSTETRIC exam - FH is 36 cm. The abdomen is tense & tender. Cannot feel the fetal parts well.
FHS cannot be heard.
VE – no bleeding or discharge. Bishop score – unfavourable.
BST – UDS shows protein 3 or 4 +
Reflexes – intact.

Dx : Abruptio placenta with IUFD ( i was explaining the condition to patient by drawing a pic that
her womb size was larger and which is tense and tender….then the bell rang. I need to rush &
tell her that it’s due to abruptio placenta and your baby might be probably dead. Then I came
out. )

191-Preeclampsia Tummy pain


For more information about similar approach check lectures 50, 52, 190

Feedback 22-6-2018
8. In the GP 36w pregnant, came for check up as she was having cramp like upper tummy pain.
Stem refers that in previous control BP:125/80, and slight swelling of ankles, rest of control
normal.
Task
Hx
PEFE
Dx

Initial management
Approach
Greeted the patient
Asked the score of the pain she said 5
I asked if she needs any pain killer she said no
I said before I proceed I would like to asses your vitals
Examnier said BP-180/100 and rest were normal.. I also asked hydration status he said normal so
I said I would like to shift pt to treatment room and start on labetalol and put on cardiac
monitors
Then went back to pt.. asked open ended question.
Then said will ask sensitive question and asked about bleeding from down below, can she feel
the baby kick, headache (positive), leg swelling that is not going away (positive)
DD qs
Eclampsia
Cholecystitis
UTI
Gastroenteritis
Asked about antenatal checkups
Period
Obstetric hx
Pap’s
Any hx of STI
Blood group.
Past medical hx of high blood pressure, any kidney or heart disease

PEFE
GA
Edema
Icterus
Hydration
BMI
Vitals
Nervous system
Fundoscopy
Reflex
Abdomen
Inspection
Palpation
SFH
Lie
Presentation
FHS

Pelvic-I think examiner said not done ( can’t seem to remember)


Office test—Urine dipstick positive for proteinuria

Dx- Pre-eclampsia—drew picture said some chemicals released from placenta (part connects you
to the baby) sometimes causes a sharp rise in blood pressure with protein in urine.

Initial Management
Shift to hospital
Call ambulance ask any help needed she said to inform partner
Talk to obstetric registrar

In the hospital they will admit you, OBG registrar will see you, start you on IV Hydralazine for
blood pressure control, Start Mgso4 to prevent complication like eclampsia and will do USG and
CTG to monitor baby’s condition.

Asked any question whether she understands


She said she understands
So I said the main management is delivery of the baby as she is 36 weeks the obstetrician might
proceed with it. But it depends on how she is responding to treatment, how baby condition is
( bell rang)

I thanked the patient and examiner and left

192-Rash in Children
Case 1 (Atopic dermatitis)
GP, a 2 or 4 years old boy brought in by his father because he is having rash on his body for a 4-6
months. A few Pictures given which shows maculopapular rash at the flexure of the elbows, back
of the shoulder, foot, hand and chest.
Tasks:
-Take history from father
-Physical examination findings from the examiner
-Explain the most likely diagnosis and Ddx to the father.
-management.

History
**Inside mom is very upset asking what happened to my baby? I can understand your concern, it
must be very distressing for you seeing your baby having these rash. Let me assure you…..
1-Rash questions
-since when have you noticed the rash?
-sudden or gradual? Constant or on and off? Getting worse? (on and off)
-where exactly is the rash?
Where did you notice it first? How has it spread? (if generalised)
has it spread and how? (if localized)
-what do they look like? Like blisters or fluid filled, flat or raised? Any scales?
-is it painful?( No) Is it itchy? (yes)
-does anything make it better or worse?
-does it affect sleep? (yes, last night was the worst night )
-has this happened before?
2-Associated symptoms
-Any fever?
-any cough, sore throat or runny nose?
-any watery eyes, sneezing?
-nausea or vomiting?
-poo and pee?
3-Triggers
-any recent viral infections?
-is it related to food? Insect bite?
-did you change the shower gel or body lotion recently?
-did he take any medications recently?
-anyone else at home has similar symptoms? Or any contact?
-any pets or carpets?
-personal and family history of asthma, eczema or hay fever? ( the child has history of hay fever
and his father has asthma, hay fever and dermatitis in childhood)
4-Others (ddx and BINDS)
-any problems with allergies or bleeding disorders
-any concern about growth and developments?
-who take care of the child most of the time?

Physical examination findings from examiner


1-General appearance (Dehydration, rash, pallor, jaundice, LAP)
so in rash describe pic to examiner
-distribution/ anywhere else -maculopapular, petechial -blanching or not, elevated.
-crusting, wet or dry
and in LN if any ask
-site and size -tender or not -consistency -mobility
2-Vital signs and growth Chart
3-ENT
-tonsils: enlargement, exudate, congestion)
-Tongue
4-meningitis signs
-neck stiffness
5-Chest and abdomen
6-Urine dipstick

Differential diagnosis
1-Atopic dermatitis (most likely).
2-contact dermatitis
3-allergic dermatitis
4-insect bite.
5-seborrheic dermatitis

Explanation of Dx and Ddx


-from history and examination, there are various reasons behind this rash. It could be due to
viral or bacterial infections, contact dermatitis etc… but less likely.
-However, the most likely is eczema or atopic dermatitis have you heard about it?
-this is a common skin condition characterised by red scaly dry and itchy rashes.
-The cause is unknown but it can runs in families and can be triggered by things like overheating,
dryness of skin, pollen, dust, soaps, detergent or viral infections.
-let me assure you that It is a common condition and not infectious. It is waxing and waning and
most children can grow out of it by age of 5.

Management
-for management I will give you a weak steroid ointment to apply on the involved areas for a
short period of time to control symptoms.
-I am gonna also prescribe you moisturizer to keep the area soft.
-it is also important to avoid any irritants that can trigger it.
-refer to specialist for allergy test.
-reading materials
-red flags (if getting worse, signs of infection like weepy, crusting or any broken areas)
-so in summary: keep your child cool and moisturize him frequently 4-6 time a day and keep him
away from things that irritate his skin.

Feedback 20-2-2018
GP, a 4 years old boy brought in by his father because he is having rash on his body for a few
months. A few Pictures given which shows maculopapular rash at the flexure of the elbows, back
of the shoulder, knees, one more can’t remember.
Tasks:
-Take history from father
-Physical examination findings from the examiner
-Explain the most likely diagnosis to the father and management.

2 min thinking: the photos shows typical atopic dermatitis rash. Have to rule out psoriasis,
allergic rash, insect bite, mites, bleeding disorders, infection – meningococcemia (less likely).
Briefly think about what management to tell the father

History:
Introduce myself. The father immediately asked what the rash on his child’s body is. I expressed
that I can see that he is a responsible dad and it is good that he brings him here. Asked for the
child’s name, whether he is here in the room (he is beside me, doctor).
History about the rash (few months, getting worse, spread to all the body, itchy, no
discharge/pus/vesicles, otherwise no fever/recent infection/medication usage. Child’s mother is
having hay fever, no pets at home, no new clothes/bed linen, no one smokes at home. BINDS
unremarkable)

Physical exam:
Apart from rash which are shown on the pictures, I asked whether it is present on scalp, and the
nappy area – none. Others unremarkable

Diagnosis and management:


Explain to the father it is most likely a common skin condition called atopic dermatitis. Runs in
family as his wife is having hay fever. It might go away as the child grows older. At the moment
we can treat by applying mild corticosteroid (1% hydrocortisone cream) on the rash. Keep the
skin moist by applying lotion. Give some suggestions on body wash and lotion (QV). Wear 100%
cotton clothes. Avoid scratching as it might cause secondary infection. Will review the child again
in a few weeks’ time, if does not get better we can consider referring to skin specialist for further
treatment. Assess understanding, give reading materials, red flags (fever, other form of rash)

AMC Feedback – Rash: PASS

Feedback 20-4-2018
Feedback 15-8-2018
4 year old child with rash around the mouth ( pic given).Father is here to talk.
Tasks: ~history ~PEFE ~Dx and ddx
Honestly I couldn’t reach to any disgnosis as it was a new case to me so I tried to cover all
possible questions in hisorty related to rash..Gave dx of food allergy, drug allergy, eczema,
contact dermatitis, cold sore, impetigo, Least likely sebrohic, psoriasis and gave few ddx of rash
that comes with fever( chcien pox, fifth, sixth disease, hand foor mouth etc) whatever came in
mind Father was sitting blank and didn’t ask any question
I left the room quite unsure and somehow got the idea of this station being unscored.

Case 11/2/2017 Feedback: Eczema, Pass, Global score 5


3 year old with rash all over body. Pictures given. Noted that the rash was worse on flexural
surfaces. Task: Take hx from mother, PE from examiner, Dx/DDx, Management
Mother was a really nice role player, said child had rashes past few months. tried whatever
pharmacist gave, not much help. Rashes been consistent, not sudden onset, not waxing or
waning. no fever, cough, RN, nothing else. The rash itches sometimes. quickly asked BINDS, all
good. Goes to childcare, happy there. No issues at home. Childs father had asthma when
younger. She is the only child. No other FHx of atopy. No known allergies.
Told mum that i think it might be something we call eczema. She has heard about it but doesn't
know much. Told here that it is common in children, esp in those with family hx of asthma and
other allergy associated conditions. Told her that dry skin makes it worse. I can prescribe low
dose of steroid cream to use for short periods of time to control symptoms. Need
emollients/moisturisers. Warm showers make the rash itch more. Bell rang.

comment AMC feedback: Eczema(passed)


You are a rural GP. Next patient is 3 yr old girl brought in by mother because of persistent skin
rash for 3 months. (5 photos of skin rash were given, 2 on elbow flexure, 2 on legs and 1 on back.
All photos look like typical eczema rash)
Task: history for 3 mins, PEFE, explain dx and ddx, discuss management
Global score:5 Approach to patient:5 History:5 Choice and technique of examination:4
Diagnosis/Ddx:5 Management plan:5.
What I did: Greeted examiner and roleplayer who looks frustrated due to this problem, ask
history by analyzing the symptom and focused on atopy history, family history, contact history
and aggrevating/trigger factors. Got positive atopy history and rash was really itchy even have to
scratch a lot at night, family history of atopy but couldnt find out any trigger factors. Asked PE
finding about temperature, rash, lymph nodes and organomegaly. Gave eczema as Dx and
contact dermatitis, allergic dermatitis, viral infection as Ddx. Mentioned emollients, low potency
steroid cream and educated on finding out and avoiding trigger factors, wrote down all these
facts in brief, also mentioned regular followup, review and redflags.
when asking PE finding i didnt ask much about the rash like scaly?oozing? Etc. i should have
added scabies,seborrhoeic dermatitis as Ddx. And i gave steroid cream only after roleplayer
mentioned it has been for 3 mths and not relieving at all. Should have given reading material
Case 2 (Seborrhoeic dermatitis)
8 weeks old - rash behind ear – and whole body ( maculo-papular rashes all around the body ,
including behind the ear )
Task
-History (4 minutes)
-Dx and Ddx
-Management plan to patient

History (same like above)

Dx and Differential diagnosis


1-Seborrhic dermatitis (most likely)
-inflammation of oil secreting gland in the skin.
-cause is unknown but can be related to familial or environmental factors
-presented as weeping skin rash, itchy, typical distribution around mouth (cheeks), behind ears
and groin.
-it is a quite common condition, not infectious let me assure you about it. Resolve usually by 18
months of age.

2-Infection (no fever, rash chronic)


3-atopic dermatitis (distribution is different, no history of atopy)

Management
-cream
-shampoo
-reading materials
-red flags (its rarely get infected but if it becomes then topical mupirocin ointment can be
prescribed)

**I think from feedback they will give a history of recent antibiotic use to make you say
medication related as a diagnosis and may be fail because of wrong diagnosis!!!
Feedback Seborrhoeic dermatitis (important)
Skin Rash- PASS)
6 weeks old child, has some rash, around flexor surface, behind the ears. Few pictures were
attached with it.
-HX
-PEFE on card
-DX
-MX
FEEDBACK-PASS
Global score: 4
Key steps: 2/3
Approach to pt: 4
History: 4
DDX: 4
Management plan: 4
2 mints thinking:
Well, to be honest, nothing more mentioned in the stem regarding rash. But, my main point was-
atopic dermatitis?/ or seborrhoeic?(I was quite sure it would be either of this, mainly
seborrhoeic because of the child's age.)
I differentiate this two from hx from this question?- Is your child crying/bothered/itching this
rash? --- If NO- DEFINATELY Seborrhoeic dermatitis.
Another point- any fever?> if yes> will go for fever questions plus rash questions.
Child abuse? Contact hx-childcare? Smoking at home?
DDX:
1. Atopic dermatitis.
2. Seborrhoeic dermatitis.
3. Infections.
4. Malignancies.
5. Child abuse.
6. Any medication.
There was an young lad inside with long hair and wearing colourful beads. I was quite surprised
to see him as he was smiling at me and quite happy all the time. I was expecting someone
serious!!! :D
There were few more pictures were given inside. I asked him to say more details about the rash,
he handed over me the full and big size pictures.
I asked ODIPARA.
rash morphology.- typical distribution of seborrhoeic rash.(check JM)
fever?- no
Is it itchy?- no
Is he crying all the time??bothered by this rash- not at all. (ahh!! good! I got my dx)
Is it affecting his sleeping, feeding activities? (No)
Any change in shampoo, clothes, contact hx, family hx, any new medication, anyone in the family
having similar symptoms, childcare, who take care of the baby, family hx of eczema, asthma?
( nothing was significant.)
BINDS.
Then, I asked PE card from examiner:
Everything were normal, typical seborrhoeic features.
I explained to father: Look, John, there could be a lot of reasons associated with a rash:
Sometimes because of viral or bacterial infections, sometimes some nasty growth, some changes
in shampoo or clothes, effects of some medication, allergies and bleeding underneath the skin-
all can lead to rash in a child. However, I think your child is having Seborrhoeic dermatitis, it's
kind a very common skin inflammation in this age group. Have you ever heard about this? (no)
Well, there's nothing to worry about, your child will grow out of this condition within 2/3 years. I
don't know exactly why your child is having this condition, but this is a well manageable and
treatable condition. I will give you all the pamphlets regarding SD, give it a read and come back
to me if you have any questions. Right now, I will prescribe a ketoconazole 2% shampoo ( rash in
scalp)and creme (nappy and flexor ares). Use it 2/3 times a week. Don't rub the skin after a bath.
As I can see, there are nappy rash, please try to give alex a free time out of nappy, so that skin
can breathe. One more to remember, if you see any changes in rash, any scaling, oozing from
rash- come back to me ASAP.

I had enough time in this case. Asked role player to repeat all the information. He was very
supportive.
Feedback 7-9-2018
Station 7 – skin rash – fail
Skin rashes pictures – 1 big pictures – generalized skin rashes , 2 nd small picture – rash –behind
the ear.
. Seborrheic dermatitis ( other candidates mention – but I gave main dx as – Drug-induced
dermatitis ) – too young to give dx . -
8 weeks old - rash behind ear – and whole body ( maculo-papular rashes all around the body ,
including behind the ear )
Task
1. History ( 4 minutes ) – father didn’t tell me initially until I asked specifically about “ any recent
abx or regular medication to the baby “ , he said that – Abx was given to him in the last week by
other doctor as he got some cough and cold , but not taking Abx at this moment by his baby .
2. Explain Dx and DDx to the patient’s father
(I explained the main one as “ drug-induced hypersensitivity reactions , )
But in DDx – explain seborrheic dermatitis , Viral Exanthem , least likely one is whole body
infection called septicemia
3. Explain management plan to the patient. - treat with General and specific measure to the
father
- explain red flag signs and review in next 72 hours .
-other Rs

comment
I passed this case, it came on 23rd of Feb,
the photo of rash as described was kind of typical of seborrheic dermatitis.
in history I ruled out all the ddx like infective and others like atopic dermatitis.
the mother said we don't have family of eczema, asthma or hay fever in the family and she kind
of insisted on that.
it couldn't be infectious as well, cuz there was no fever and the rash was chronic.
so i said this is most probable seborrheic dermatitis, a chronic skin condition, the cause is
mixture of genetic and environmental factors.
quite common don't worry, i'll give you a cream for the skin lesions (i didn't even mention the
name) and shampoo it will get better. i passed with a global score of 6.

comment
Hi, had this case on 23rd feb. stem was as stated here. Examination was given: rash as pictures.
(Google seborrheic dermatitis in neonates and the images are similar with minor weeping
lesions behind the ear which is classical for seborrhoea), afebrile and looks well, vital signs
normal. Rash involving areas in picture (face and behind ear), and upper chest and groin creases.
Tasks:
History and DD with management
comment
rash for the past 4 weeks, started first in scalp, then face and neck and behind ears, later spread
to chest and groin recently. Mum stated rash felt very rough and dry. Said Bub has not been very
bothered by it, feeding well, good activity, no fever at all, gaining weight well, no cough or URTI
symptoms. Mum insisted baby was very well and only concern was rash. Denied and change in
appearance of rash other than spread. For me, the mum mentioned baby's aunt has hay fever
but no FH of eczema or other skin conditions. Antenatal/perinatal history was completely
unremarkable.

Explained to mum DD was eczema or seborrhoeic dermatitis. Told most likely the latter but I
referred to paediatrician for confirmation of diagnosis due to age. Reassured not infective or
other acute rashes, and said I'll start on a emollient cream. Stated once diagnosis is confirmed
will start special cream and shampoo. Didn't mention and names of medication. Main thing
mum wanted was reassurance and possible causes and she was happy. Passed case

Case (7/6/2017)
baby has rash for 3 months. Photos shown maculo-papular rash on face and crack like ulceration
behind ear with rash.
Task
- hx from mom,
- dx and mx ( mom was saying flakes come out, no itchying/fever/allergy. Child's granddad has
asthma)
193-Hyperthyroidism nervous lady
Nervous lady d/t hyperthyroid, 30 yrs old woman, sth wrong with her nerves.
Tasks
-history
-PEFE
-explain Dx and ddx

History
1-chief complaint (nervous questions)
-what do you mean by something wrong with nerves? Anxious?
-onset?
-1st episode?
-affect your life?

2-Thyroid questions
-weather preference? Bowel motions?
-hands: shaky or sweaty?
-eyes: vision problem
-neck: any lump in neck if yes ask if painful or not and other lump questions? Any problem with
swallowing? Change in voice?
-heart: palpitations? Chest pain? SOB?
-period: LMP? Regularity?
-sleep; especially getting to sleep? Weight? Appetite?

3-Differential diagnosis questions


-Phaeochromocytoma: headache
-Infections: fever, night sweating
-GAD: do you worry excessively about thing or minor things?
-panic: Do you feel suddenly frightened, or anxious or panicky, for no reason in situations in
which most people would not be afraid?

4-General
-PMH (especially thyroid d)
-PSH
-SADMA + coffee
-Family history (thyroid or CVD)
-travel
Physical examination from examiner
1-General appearance
-anxious -restless -thin -weight.

2-Vital signs

3-Hands
-nails (clubbing, Onycholysis) -palmar erythema
-warm, sweaty -Fine tremor

4-Face
-ptosis -proptosis -Exophthalmos -lid retraction
-conjunctivitis -corneal ulceration
-lid lag -opthalmoplagia

5-Neck
look
-scars -dilated veins -skin redness -swelling
-lump on swallowing or protrusion of tongue.
Feel
-thyroid enlargement (site, size, consistency, tenderness, temperature, mobility)
-tracheal position
Percussion upper sternum
Auscultation
-bruit -stridor

6-Chest and CVS

7-CNS reflexes

8- leg: pretibial myxedema -proximal myopathy.

9-Office tests: UDT, BSL, ECG

Diagnosis and differentials


1-hyperthyroidism (most likely)
2-pheochromocytoma
3-GAD, panic
4-coffee
5-infections
Case (22/4/2017)
Nervous lady d/t hyperthyroid, 30 yrs old woman, sth wrong with her nerves.

H/o, ask PE, explain dx, ddx with pt.

Same as usual asked her ‘what do you mean sth wrong with your nerves’?
She got anxious and restlessness and racing of the heartbeat.

It was first time.


I asked about any headache, chest pain or SOB, she said no.
When I asked about weather preference, she prefer cold weather.
Sweaty+,
problem with poo, she said some loose motion 2-3 times per day.
Period got lighter and lighter.
Not aware of neck gland enlargement.
Past medical or surgical history-clear.
SADMA-clear.
Generally anxious, she said she was not like that before.
No coffee drinking.
Any panic attack, she said no.
No past mental problem.

Physical examination
GA-anxious
VS-normal.
Features of hyperthyroid, any eye signs-examiner said no. Any visible neck gland enlargement,
examiner said 3*3cm, yes. Is it move with swallowing, yes.
CVS-clear.
RS-clear.
Abd exam-clear.
NS- jerks are exaggerated.
BST are not available.

I explained her that she might have a condition called hyperthyroid according to my assessment.
There is a butterfly shaped glands called thyroid glands. They produce hormone called thyroid
hormone which is responsible for our body’s metabolism.
In your case, I think this gland is over functioning and produce more hormones and disturb our
body metabolism. That’s why you felt like that.
Do you understand my explanation, she said yes.
But, I need to run investigation for you to confirm the diagnosis. Sometimes, this kind of
condition can be caused by a condition called pheochromocytoma which is a kind of tumor that
produce hormone called adrenaline but it’s a bit less likely in you. It can also be due to anxiety
problem, panic attack, excessive coffee drinking but less likely in you.

Reassure the pt and 6R….


FB-PASSED, OVERALL-5, APPROACH-4, HX-5, PE-6, DX, DDX-5.
194-Pelvic examination
Case 2018
27 years old female with single episode of postcoital bleeding, on OCP, menstrual history
unremarkable. PAP smear 3 months ago was normal.
Tasks
-do PV
-most likely diagnosis with reason

Case (27/4/2017)
67 y/o lady comes with vaginal bleeding . Her LMP was 15 years ago and she has been treated 10
years with mixed estrogen and progestron . She has 2children. Her pap smear has been NL all the
time .the last one is 18 months ago.
Tasks:
1-take consent for examining her.
2- examine her (manikin) and give the results to the examiner.
3- explain the most probable diagnosis and other possible ones to the patient

Examination steps:
1-Introduce yourself and wash hands and put gloves.

2-take consent
-I will be doing a pelvic examination on you. This would involve me inspecting your genital area
for any discharge, bleeding, or ulcers.
-after that I will perform speculum examination with this instrument (show her). to view the
neck of your womb or cervix clearly
-after doing speculum examination the next step is bimanual examination feeling for vaginal
wall, the cervix, uterus, tubes and ovaries between my 2 hands.
-are you happy for me to carry out the examination?
-a female staff will be present to act as a chaperone during the examination.
-if you could go behind the screen and undress yourself from the waist down then lie down on
the coach with your both legs be bent with your heels towards your buttock and with knees
apart. You can cover yourself with the sheet provided and when you are ready please call me.
-I will try to be as gentle as possible but if you feel excessive pain at any time please you can feel
free to stop me.

3-Inspection
-ulcers, rash, vesicles
-discharge, bleeding.
4-Speculum examination
-Now I am gonna do speculum examination, First, I will lubricate it then this will be inserted
inside your vagina with minimal fore and once inside I will be slowly turning it round and once in
position the lips of the speculum will be opened and locked to view the neck of your womb or
cervix clearly. When I locked the speculum, you might hear a sound so not be frightened of that.
*I think the Procedure:
-lubricate the speculum
-your left hand separate the labia
-hold the speculum like a cigarette with your right hand
-hold in 45 degree to the right.
-then after insertion turn to 90 degree and tilt downward.
-open and lock it.
-see the cervix.

Taking a sample
-If HPV screen necessary to do: show her cervical broom. I will put another instrument called
cervical broom which will be inserted into the cervix and then rotating it 3-5 times and then will
be placing it in transport media.
*I think the procedure
-take a sample with the broom (rotate it 3-5 times) then take it out.
-put the broom on transport media (rotate it clockwise!)

Removing speculum
-slowly withdraw it until out a bit from cervix then unlock it.
-tilt it to 45 degree then hold it in this position all the way out.

5-Bimanual examination
-Now I am gonna gently introduced a gloved and lubricated 2 fingers inside your vagina feeling
for vaginal wall, the cervix, uterus, tubes and ovaries. While doing this my other hand will be
placed in the lower part of your tummy and these structures will be felt between my 2 hands.
*I think Procedure
-introduce your right hand’s middle and index finger like a pistol in 90 degree.
-once inside tilt it to horizontal.
-feel for vaginal wall both sides.
-tap on cervix (CMT)
-feel the uterus (press with upper hand)
-feel both sides (the ovaries)
-move all the way horizontal then 90 degree then move out.
For 2018 case
I think it is postcoital bleeding
1-cervical ectropion
2-cervicitis, vaginitis
3-cervical polyp
4-cervical cancer
5-cervical trauma.

-I think the most likely diagnosis is cervical ectropion (you see a red ring around the cervix os)
-explain: neck of the womb has 2 surfaces; outer ecto and inner endo
-ectocervix meet endocervix at transformation zone. In ectropion’ the rim of the cervix rolls
outwards exposing endocervix into the vagina, which has the appearance of red ring.
-commonly seen in teenagers, on OCP, pregnancy.
-treatment none but HPV screen should be up to date.
-if excessive discharge or bleeding after intercourse  refer to specialist who will obtain
procedure like electrocauterisation.

For 2017 Case


I think it is postmenopausal bleeding
1-atrophic vaginitis
2-endometrial or cervical polyp
3-endometrial hyperplasia
4-endometrial or cervical cancer
5-medication

-it could also be due to atrophic vaginitis but, there was no features of atrophic changes since u
r taking the hormone.
- it could be due to problem with the womb or cervix, i drew a pic.
-it could be due to the medication for osteophoresis as i couldn't find any abnl
-i told the pt to see gynaecologist to check it the condition for further assessment. if needed,
specialist will do USG, hysteroscopy and biopsy.
-reassure the pt.
Case (27/4/2017)
Pelvic exam PE
67 yrs old woman c/o brownish vaginal d/c few months, pt is taking female hormone like the pills
for many years for osteophoresis. last pap smear 18 months ago. Ask consent for examining of pt
Perform VE
Expalin the most likely cause to pt.

i entered the room, an old lady with an examiner were sitting on the chair. i introduced myself
and asked for the consent that i need to examine the down below. it will involve checking the
private area, putting some instrument and put my fingers to examine. it will not be painful and i
will also use lubricants during the examination. i will also call a medical person as a cheparone. is
that ok with u. she said ok.

Then, i told her that could u plz undressed for me and u can cover urself with the tower. i am
going to wash my hand. after washing my hands.

i start to wear the gloves, there are three boxes on the wall just near the hand gel. i tried to pull
out from the lowest box but a little bit difficult. so, i took a pair of gloves from the middle box, it
was the grey colour. i kind of noticing that it was written as sterile on the wall of the box. But,
just before i examined, i asked the examiner that is it sterile. examiner said just check it or sth.

There were many speculums, i think ,9 speculums on the table, there were also brushes and
some small box for collecting the smear. and lubricant pad just like kfc sauce.
i started with inspection, no abnl findings.
i took one of the speculum, i didn't choose the size and pad and teared it , there was lubricant. i
applied it on the speculum and turn to the pt and said now i am going to introude the speculum.
after putting speculum, i can't see anything. i asked examiner for the light, it was just next to me
like the small light used in ED setting. i asked the examiner to switch on the light for me. he said
do it urself. i tried to reach the power key. it was with the sensor, no need to open it actually.
*(no abnl findings on PE, no atrophic changes as well
i aslo mentioned to do pap smear but examiner said already done.
i pulled out the speculum and said i will do BME. examiner said nl.

i exlpained the conditon to pt that the discharge; it could be due to the medication for
osteophoresis as i couldn't find any abnl, it could also be due to atrophic vaginitis but, there was
no features of atrophic changes since u r taking the hormone. it could be due to problem with
the womb or cervix, i drew a pic. i told the pt to see gynaecologist to check it the condition for
further assessment. if needed, specialist will do USG, hysteroscopy and biopsy.
also, need to check medcation for osteophoresis, if needed, will change another medication for
osteophoresis. reassure the pt.

FB-PASSED, OVERALL-5, APPROACH-6, EXAM SEQ-5, PERFORMANCE OF PROCEDURE-5, DDX-5.

195-Post operative ?UTI


This was a pilot case on 27-06-2018 not sure if the diagnosis is right or wrong
Feedback
Patient day 3 post op cholecystectomy chart given outside want to be discharged
Tasks
1-take history
2-physical examination findings from examiner
3-address the patient request about discharge

Unscored

History
1-approach
I can see that you have had operation of gall bladder and you want to be discharged is that right?
All right so how do you feel at the moment? Are you feverish or in pain? (No)
ok, can I just ask you a few questions first to assess your current condition?

2-post operative complications differentials questions


chest (pneumonia, heart problem, pulmonary embolism) (all negative)
-any chest pain
-funny racing of the heart
-shortness of breath
-cough
abdomen (wound infection, peritonitis, drains) (all negative)
-any pain in your tummy (just a bit at the site of the surgery) severe? (No)
-have you passed your motions? How is it?
-have you noticed any discharge in the wound site? Is it smelly? Colour etc..
-have you noticed any blood coming through the drain?
UTI (all negative)
-any pain or burning sensation on passing urine?
-do you need to go to toilet more frequently?
DVT (all negative)
-any pain or swelling in your legs?
-have you travelled recently? immobilization?

3-General questions (I forgot to ask general questions in the exam hhh)


-PMH (lung problems or infections, heart attack, diabetes, Gout, clotting problems)
-PSH
-SADMA
do you smoke?
alcohol?
Drugs and medications and allergies.
-family history

Physical findings from the examiner (in the exam they give a card but just in case they do not)
1-General appearance
-in pain, look well, lying comfortable, pallor, jaundice, cyanosis, LAP
2-chart of vital signs from the stem explain it showing (could not elicit anything so explain it as
normal).
3-chest examination
Lung
-inspection: chest movement
-palpation: trachea position, chest expansion
-percussion: dullness
auscultation: air entry, vocal resonance, basal crackles or wheeze
Heart
-apex beat
-heave or thrills
-heart sounds S1+2 and added sounds
-murmurs.
4-Abdomen
-wound site (dressing, redness, pus, hematoma)
-tenderness
-bowels sounds
-drain (blood or bile leak)
5-leg examination
-redness, oedema, hot, and calf tenderness
6-office tests
-UDT, BSL (urine dipstick showing leukocyte, nitrite and trace of blood)

Explain diagnosis and differentials


-look john I finished history and examination. I could not illicit any symptoms you have at the
moment apart from a bit of pain in the wound site.
-your heart and lungs are fine, you have no wound infections or leg pain or swelling, no pain in
abdomen apart from the wound site.
-but urine dipstick showed some WCC which is type of blood cells that usually fight infection and
its presence indicate a urine infection, also there are a trace of blood and nitrite which lead me
to suspect that you have UTI. Which is one of the common complication to be expected after
operations.
-I understand you want to be discharged but I think it is better to stay in hospital at the moment,
I will call the registrar to have a look at you and check the findings they might do further
assessment. If everything is fine then you can be discharged.
-any questions? (anything else you want to say)
-can you tell me if there is anything important at home or work that you want to be discharged
for ? (tell something about his wife) bell rang

196-Post operative Heart failure


7th post op day, post hemicolectomy, the patient presented with SOB, had a history of HTN and
IHD, was smoker and some comorbid. Very long stem. There were charts outside temperature is
37.7, BP 150/? O2 sat 94%. No fluid or medication charts were provided.
Tasks
-Explain the observation charts to examiner
-PEFE (+ve Rt lung basal crackles, decrease air entry, raised JVP, bilateral leg edema)
-DDX to patient with reasons

Explain the observation chart to examiner


- A passed candidate said there was high blood pressure and tachycardia. (Others noticed fever
37.8 or 38 )

Physical findings from the examiner


1-General appearance
-in pain, look well, lying comfortable, pallor, jaundice, cyanosis, LAP (patient sitting in bed
uncomfortable SOB+ve)
2- vital signs (from the stem & charts )
3-chest examination
Lung
-inspection: chest movement
-palpation: trachea position, chest expansion
-percussion: dullness?
-auscultation: air entry, vocal resonance, basal crackles or wheeze (reduced air entry and
crackles on the base of the right lung +ve)
Heart
-JVP (elevated +ve)
-apex beat
-heave or thrills
-heart sounds S1+2 and added sounds
-murmurs.
4-Abdomen
-wound site: dressing, redness, pus, hematoma.
-tenderness
-bowels sounds
-drain: blood or bile leak
5-leg examination
-redness, bilateral leg edema+ve, hot, and calf tenderness
6-cannulae site
-tenderness and redness
7-office tests -UDT, BSL

Explanation
-from history and examination, it appears that your BP looks a bit high and there are some signs
on chest examination with legs edema and elevated jugular venous pressure. I think you most
likely have a condition called heart failure post op.
-it is more likely to occur in patients with preexisting heart disease with HTN and smoking.

-other possibilities are:


pneumonia or infection
pulmonary embolism
DVT
UTI

197- Postural hypotension


Case 1/ Female presented to ED following sudden blackout. Annual blood is done 2/52 showing
fasting BSL 8. All other results are normal.
Task
-Hx (+ve fhx of DM. patient on ACE and metformin)
-Dx with reasons ( LOC for 2-3 min, in office after stand up quickly)
-Mx (was not a task but just in case)

History
1-collapse questions
-Did you lose your consciousness completely? How long for? (for 2-3 min )
-has this happened before? How often?

2-Before
-what were you doing before it happened? (In the office after standing up suddenly)
CNS, Vasovagal
-did you get any warnings like feeling dizzy, nauseous or strange smell? (Migraine/ epilepsy/
vasovagal)
-did you get any headache, blurring of vision, weakness? (TIA/ Stroke)
-Any fever or recent infections (Meningitis)
Cardiac
-have you had funny racing of the heart? (Arrhythmia) if yes ask: how long did it last? Regular or
irregular? Do you have it now?
-any chest pain, SOB, dizziness?
Endocrine
-did you skip your meals? When was your last meal?
Any head injury
postural hypotension (if not answered before)
-do you black out or feel dizzy when you stand up quickly?

3-During (epilepsy)
-anyone witnessed and noticed any jerky movements?
-any change in colour of lips or skin?
-have you passed urine or soiled yourself?
-have you bitten your tongue?
-have you injured yourself?

4-After
-how long does it last until your recovery?
-after recovery do you feel normal or drowsy? Any N&V or headache?

5-General
-PMH (diabetes positive with regular check up) PSH, SAD
-Details of medications DSCO (taking metformin for DM and ACE for kidney related issues)
-family history of sudden death?

Explain diagnosis
-There are possible reasons behind your sudden LOC.
-Most likely due to postural hypotension especially you have DM which seems to be
uncontrolled as your BSL is 8 or due to ACE-I as well.

-It could be CVS, trauma, but less likely as you don’t have …..

-Could also be due to vision problem or mini stroke due to uncontrolled DM (BSL 8)

Mx if present
1.Keep in the hospital to r/o ddx and to be seen by specialist.
2.Arrange Ix to r/o causes (FBE, ESR,CRP, LFT, UCE, TFT, BSL, lipid, HbA1C, ECG, echo, US)
3.Once discharged refer back to DM physician for assessment of DM medication and GP to adjust
ACE-i.
4.To prevent or decrease such episodes from happening or getting worse:

 strict control of DM

 Increase fluid intake

 Get up slowly
Feedback 8-11-2018
Case 2
Feedback 5-12-2018 (important)
Dizziness/ Postural hypotention
53 years old lady, Pt having dizziness for few days , On anti DM , anti HTN and aspirin. BP and
DM well controlled.
1.HX
2. Most probable Dx
3.Tell the pt likely consequences of events of her problem.

( Now with this task I was confused is it asking mechanism of postural drop? Or asking
consequences or risk of post drop I decided to do both)

Greeting , Introduction. I understand you having dizziness lately, how are you feeling right now? (
I am ok now doctor I feel dizzy whenever I get up). Can you tell me bit more .. like what do you
mean by dizzy Do you mean head turning or black out/ faint?(she said kind of lightheadness))
Then I started proper Hx taking.( HOPI: keeping in mind kind of SIQORA, I kept in mind the fall
cluster while Hx taking as previous recalls used to come with postural hypo with recurrent fall,
Also causes of post hypo).

Gave typical Hx of postural drop every time while getting up from bed or sit for past few days, no
loss of consciousness, no Fall, No unusual feeling lately like Tingling numbness/ weakness, No
Chest P , Funny racing of heart.. Now when I asked SOB she said bit SOB sometime while
gardening only, not with other exertion or household chores), I asked again while having SOB any
Chest pain or tightness ( AS was in my mind). Pts ans NO.

Any recent illness or fever? She said recently having diarrhea.. I asked watery? Bit of loose not
watery ( I did not went detail of diarrhea)

Any nausea vomiting or tummy pain? She said she some time have epigastric pain. No acid burn
or regurgitation. Asked about any pain medication like NSAID pt said no, (At this point anemia
came into my mind and pt aslo having ASPIRIN). Forgot to ask tiredness. Started asking again
about bleeding Q: No easy bruising, menorrhagia?( pt stopped period 2/3 yrs ago can’t
remember), Asked of black stool or bleeding? ( Pt said cant say as she usually don’t notice poo)
Pt had regular check with DM AND BP well controlled.
No smoking or alcohol
(There was no PEFE)
Task 2:
Thanks X for all information. Now from all your hx and pattern of your dizziness most probable
cause is postural hypotension. Let me explain (took pen and paper).. So as the name apply its
related to posture change like you told me you always have this while getting up from lying or
sitting position. What happens is when you stand up your blood pressure falls hence brain
does not get enough blood that’s why you having dizziness. Are you getting my point? ( she
nodded) Now human body has basic mechanism to prevent this fall , sometime due to various
reason this can be hampered, truly speaking I could not pin point a single reason. It can be due
to dehydration as you said you having diarrhea, can be due to Diabetic neuropathy but your DM
is well controlled, Poly pharmacy or multiple drug unlikely in your case. However dizziness can
be also due to heart cause like AS. ( I drew heart ), where A heart vessel get narrowed, Can be
due to anemia as well as you said having upper tummy pain, Hence we need to evaluate for your
tummy pain as well. It can be simply due dehydration due to your recent diarrhea. Another
cause is stroke unlikely as you don’t have any weakness or numbness.

#3rd task: Now I am bit concerned that if this continues can lead to fall and fractures and head
injuries which can be deadly for you. So I need to refer you to fall prevention clinic. I will also
evaluate you thoroughly to find the exact cause and if needed will refer you to specialist.

Key step12345: yes, yes, yes, yes, yes.


Approach to pt :4
Hx:5
DX/DDX:5
Pt counselling:4
GS 4
198-Atypical Pneumonia
A young woman came with chest pain and cough episodically for 9 months.
Tasks:
-Take a history for 4 min.
-Ask PE findings from examiner.
-Tell the patient about DDx.

History
-Hi Mary. I’m Dr. .. one of the doctors in this clinic. Nice to meet you.
-How can I help you today? (I have some chest pain and cough for 9 months)
-Severity? Offer painkiller if she needs
-Ok, can I know where your chest pain is? (It’s here. The patient pointed around lateral side of
right lower chest) Radiation? (no)
-Ok, do you think your chest pain is worse with deep breaths? (Yes)
-Is it like a kind of dull aching or stabbing pain? (Stabbing)
-How about your cough? Is there any phlegm production? (Yes, and it’s greenish in color) any
blood? (No)
-Ok, do you notice any foul smelling in your phlegm? (No)
-Ok, you said it’s been nine months, right? Did you seek any medical help previously? (Yes, I had
and the doctors gave me amoxil but it didn’t relieve)
-Ok, Mary, did you notice any fever in you? (Yes, I have some mild fever too) night sweat (no)
-How about any shortness of breath? (Yes, sometimes)
-Any travelling history recently? (No) contacts with similar symptoms? (No)
-Do you have any underlying medical problem especially lungs problem before? Asthma? (No)
-History of allergies or family hx of allergies, asthma etc..? (No)
-Any regularly taking drugs? (No)
-Do you smoke? (No) drink alcohol? (Occasionally) stress (no) occupation?
-Pets, carpets at home? (No)
-By the way, do you think you have well ventilated house? (I think the patient was living in the
country and now moved to CBD)
-Ok, thank you for your information, Mary. Can you please wait for a while? I have to examine
you with my examiner.
Dear examiner, I want to start my examinations. Firstly, I will look at patient’s general
appearance. (As you see her now)
Then I want to look for signs of pallor or cyanosis, leg edema? (No) BMI normal
I want to measure temperature, blood pressure, pulse rate, oxygen saturation and respiratory
rate. (told me each finding and all are normal)
Then I’ll perform proper respiratory examination.
I’ll start with inspection. Any unequal chest wall movements? (No) any scars? (No)
I’ll do palpation? I want to know trachea position. (it’s midline) I want to perform vocal fremitus.
(it’s normal)
Then, I’ll do percussion. (Normal)
Then, I’ll do auscultation. (you can hear crackles in right middle zone)
Then I’ll quickly assess cardiovascular and abdominal examination. (All are normal)
Can I perform office tests like urine tests? (not available) Thank you, examiner. That’s the end of
my examination. Let me get back to my patient.

Hi again…. Mary. According to your history and my PE findings, I think you have some infection in
your right lung. I found some abnormal breath sounds right here. Are you with me so far? Since
it’s been 9 months and you took amoxil and you still get these symptoms and so I’m thinking
some atypical bug. Don’t worry. If we give you a different kind of antibiotic that kill these atypical
bugs, you’ll be fine.

And there might be other causes like fluid collection around your lungs (pleural effusion) here
talk about causes of Pleural effusion like heart, kidney, liver but less likely. Infection, CTD . Could
also be TB, but less likely as no travel overseas.

Feedback 5-10-2018

A young woman came with chest pain and cough episodically.


Tasks: Take a history for 4 min. Ask PE findings from examiner. Tell the patient about DDx.

Score: I passed this case with global score 4. (I don’t expect to pass this case because my
performance is on the margin and not good enough)
Key steps: 1, 3 Yes and 2, 4 No.
Approach to patient: 4
History: 4
Choice and technique of examination, organization and sequence: 4
Dx/DDx: 3
My performance:
Hi..Good afternoon… Mary. I’m Dr. MM, one of the doctors in this clinic. Nice to meet you. How
can I help you today? (I have some chest pain and cough for 9 months) Ok, can I know where
your chest pain is? (It’s here. The patient pointed around lateral side of right chest) Ok, do you
think your chest pain is worse with deep breaths? (Yes) Is it like a kind of dull aching or stabbing
pain? (I am not sure) Ok, how about your cough? Is there any phlegm production? (Yes, and it’s
greenish in color) Ok, do you notice any foul smelling in your phlegm? (No) Ok, you said it’s been
nine months, right? Did you seek any medical help previously? (yes, I had and the doctors gave
me amoxil but it didn’t relieve) Ok, mary, did you notice any fever in you? (yes, I have some mild
fever too) How about any shortness of breath? (Yes, sometimes) Any travelling history recently?
(No) Do you have any underlying medical problem especially lungs problem before? (No) Any
regularly taking drugs? (No) Do you smoke? (No) By the way, do you think you have well
ventilated house? (I think the patient was living interstate and going around for some months
but I’m not sure) Ok, thank you for your information , Mary. Can you please wait for a while? I
have to examine you with my examiner.

Dear examiner, I want to start my examinations. Firstly, I will look at patient’s general
appearance. (as you see her now) I want to measure temperature, blood pressure, pulse rate,
oxygen saturation and respiratory rate. (He told me each finding and all are normal) Then I want
to look for signs of pallor or cyanosis (No) Then I’ll perform proper respiratory examination. I’ll
start with inspection. Any unequal chest wall movements? (no) any scars? (No) Then, I’ll do
palpation? I want to know trachea position. (it’s midline) I want to perform vocal fremitus. (it’s
normal) Then, I’ll do percussion. (normal) Then, I’ll do auscultation. (you can hear crepitations in
right middle zone) Thank you, examiner. Then I’ll quickly assess cardiovascular and abdominal
examination. (All are normal) Thank you, examiner. Can I perform office tests like urine tests?
(not available) Thank you, examiner. That’s the end of my examination. Let me get back to my
patient.

Hi again…. Mary. According to your history and my PE findings, I think you have some infection in
your right lungs. Let me draw a picture for you. I found some abnormal breath sounds right here.
Are you with me so far? Since it’s been 9 months and you took amoxil and you still get these
symptoms and so I’m thinking some atypical bug we called pseudomonas. Don’t worry. If we give
you a different kind of antibiotic that kill these atypical bugs, you’ll be fine. And there might be
less likely causes like fluid and air collection around your lungs. And sometimes chest pain may
relate to your heart problems but very least likely in your case. Sometimes, patients may present
with these symptoms related with their mental problem but impossible in your case. And the
bell rang.

Comment: I had the exactly same case before when I took my clinical exam on May 31st for first
time and got retest. I passed with global score 4 in my first clinical exam, too. During 2 min, I was
thinking I would tell about chronic suppurative lung diseases like lung abscess or bronchiectasis
but I forgot to mention them in real exam. I thought my history was not good enough and PE was
not in details and DDx is not complete. But with some luck, I passed this case and I’m really
thankful to my examiner in this case.
199-IDA Cow milk
18 months old baby boy, came back from trip, was sick and doctor did the blood tests, come to
your clinic today to be reviewed with results according to previous doctors instruction, blood
test results are given which was: Hb:low, WCC: NL ;Neutrophil NL, Lymphocytes NL, Eosinophil
NL, Plt NL, Blood film was Hypochromia and Microcytosis. Iron level: Low, Ferritin: Low, Sat
Ferritin: low.
Tasks:
-Explain the Results
-further history
-PEFE (card will be given)
-Most likely dx and Management

Explain results
-These are the results of FBE showing decrease in the Hg level, which, is iron rich protein carried
on red blood cells responsible for carrying Oxygen to body organs.
-It also showing hypochromic microcytic, which means the red blood cells are small in size
-However, WBC cells to fight infection & platelet that help blood clotting are Normal.
- Iron studies showed that Iron level in the blood is low and iron stores or ferritin as well low
- The result suggests that the most likely have deficiency in iron level or iron deficiency anemia.
-would like to take further history
History
1-Symptoms questions
-open ended question
-lethargy and SOB
-fever, rash, recent infections.
-vomiting, tummy pain
-bowel motions (if pale, greasy, bulky or hard to flush)
-waterworks
-does he bleed easily.
2-BINDSMA
Birth
-any infection or bleeding during pregnancy
any twins
-was it term or preterm
-screening tests
Immunisation
Nutrition/ Feeding history
-can you tell me briefly about his typical daily diet (Fussy eater likes junk food) does he eat
enough red meat or green leafy vegetable (No)
-is he breast feeding, bottle feeding or cow’s milk. If cow milk how much (Cow milk 5-6 glasses/
day)
-at what age have you introduced solid into food? any change in bowel habits since then? Any
complaint of bulky, greasy, pale or offensive stools?
Growth and development
Social
-family origin (thalassemia)
-family history of chronic diarrhea or anyone on special diet.
-travel
-support and finance
medications and antibiotics recently

Explain diagnosis
-most likely IDA due to excessive cow milk intake and not eating iron rich foods such as green
vegetables and red meat.
-Toddlers can run into problem if they drink too much cow’s milk of more than 24 ounces/ day
and eat fewer iron-rich foods like red meat and green vegetables.
-cow’s milk is not a good source of iron. In fact, milk makes it harder for the body to absorb iron
and can contribute to IDA.
-other possibilities but less likely
*celiac or lactose intolerance (no diarrhea, no family history, normal growth)
*infection (no fever or recent infections)
*bleeding disorders, thyroid problems but unlikely from history.

Treatment
-Ix (urine sample, stool sample and celiac screening) (not sure)
-Iron supplement syrup.
-Limit cow milk to 500 ml/ day
-increase red meat, fish, green vegetables and pulses, advise iron fortified cereals in breakfast
-refer to dietician
-follow up
- if no improvement with treatment will look for other causes and may need a referral to
specialist.

Feedback 6-6-2018 Station 10 IDA secondary to cow milk- pass


18 months old baby boy, came back from trip, was sick and doctor did the blood tests, come to
your clinic today to be reviewed with results according to previous doctors instruction, blood
test results are given, hb low, wbc normal, platelet normal, fe study reveals iron deficiency.
Task: Explain the investigation result to mother, further history, dx, mx
Explain… function of each blood cell and explained the results and causes of low hb . iron is
deficient so, would like to ask further history .
History: symptoms of anaemia nil currently, was sick while he was on the trip but he is fine now,
what do you mean by he is sick ( I could not remember what mother said). Diet: cow milk, chips,
picky, doesnt like to eat what she gives him, bleeding hx nil, factory nearby nil, family origin
Australian, bowel pblm nil, greasy stool nil, hard to flush nil, infection symptoms nil, deworming,
well baby question, Social hx, BIND, family member on special diet nil, family member with
blood pblm nil.
Dx: IDA likely secondary to cow milk, explained other ddx, mx. Inv: stool test, coeliac screening
iron syrup, refer dietician, cut down cow milk, prepare fingers food, review with results, if no
improvement with treatment, will refer to specialist.
Feedback 13-12-2018 Pallor: Pass (IMP)
2 mins outside: Mother of a Young boy of about 4-5 yrs come to discuss the results of
investigations you have sent a week ago when the boy caught a cold or flu last week. Lab results
were given.The results were of typical iron deficiency of anaemia with anisocytosis and
poikilocytosis.

Tasks:
Discuss the results with her.
Take relevant history
Dx and management.

Thinking outside: DDS of IDA: same as karens such as nutritional, malabsorption, blood loss,
blood thinning medications, lymphoma, leukaemia or any bone marrow disease.

Inside: hi, we have got the results of the investigations of Andrew ( her son) which we sent last
week. Before we go ahead, how is Andrew, how is his flu?
She: He is good and improving.
Me: Good to hear. Let me discuss the lab results. The lab results suggest that Andrew is not
having enough haemoglobin in his blood, do u know what that means?
She: No
Me: our blood contains haemoglobin; its function is to carry oxygen to the different parts of
body. The most common cause is IDA. IN Andrew’s case it is the same, IDA. As u can see the
results (I was showing the results to her) these r the normal range and u can see his HB, Iron and
ferritin have been reduced. That means Andrew’s blood does not have enough iron and iron
storage which is ferritin. His result also shows anisocytosis means his RBCs, which is one type of
cell in blood, are different sizes and poikilocytosis which means his RBCs are of different shapes.
There are typical features of IDA. Don’t worry, it is very common. R u with me so far?
She: yes
Me: now IDA is not the cause, its an effect of something, which we need to find and that is
why I need to ask you few questions, is it ok with you. She: Ok
Me: does he have symptoms of IDA such as red ulcers in mouth, easy fatigability, any obvious
pallor? She: No
Me: pl let me ask u questions about his possible causes of IDA.
Me: first of all pl tell me about his daily diet.
She: Andrew is a fussy eater. He likes junk food.
Me: Does he eat red meat and green leafy vegetables enough? She: Not enough
Me: Anything else he eats or drink more? She: he loves to drink milk.
Me: what type of milk? Soya milk, cow milk? She: Cow milk.
Me: How much cow milk does he drink? She: 5-6 glasses per day (I don’t remember exactly what
she said, but it was too much)
Me: Then I asked about diarrhoea including blood in stool, anus itching, (all negative), All the
questions about all DDS such as coeliac, other type of malabsorption such as pancreatic disease,
previous surgery.
Me: Ask about night sweat, pruritus (lymphoma), gingival bleeding (leukaemia), any long term
medications, any bleeding clotting disease. All negative.
I have quickly asked about any present, past or family history including f/h of coeliac (special
diet), lymphoma and leukaemia ( blood related diseases), all negative.
I have asked history as per karens.

DX: I think in Andrew’s case, the cause of IDA is drinking too much cow milk and not eating
iron rich foods such as green vegetables and red meat. (pl be careful here, one of my friend
has given a dx of only cow milk anaemia and did not give fussy eater as an additional cause, he
failed this case).

Don’t worry, its easy to manage. 1st of all, I would advise to cut down his cow milk to 1 glass a
day. 2nd, try to reduce or stop his junk food and feed him green leafy vegetables and red meat
more. 3rd, I would advise to feed him iron fortified cereals in breakfast. 4 th I can prescribe him
an iron syrup. Pl give him once or twice a day, according to instructions as per bottle. I would
like to see him after 2 weeks and we will do more blood tests then. Most probably his iron and
HB level will rise. If he does not respond to our management, then we will look for other
causes of IDA.
Me: Do you have any questions for me? She: No
R u happy with management?
She: Yes doctor. Me: Thank you. Bell rang

Global Score: 4 (I don’t know why 4 and not 5)


Key steps: All 5 Yes
Interpretation of investigations: 5
History: 5
Dx/ DDs : 5 ( not sure whether DDS were there or not, if yes then I must have given)
Management plan: 5
Case (13/10/2017)
An 18 month y/o boy John was brought with his father Lindsy due to his pallor and investigations
which was: Hb:low, WCC: NL ;Neutrophil NL, Lymphocytes NL, Eosinophil NL, Plt NL, Blood film
was Hypochromia and Microcytosis. Iron level: Low, Ferritin: Low, Sat Ferritin: low.
Tasks:
Hx,
Explain the Results, PEFE(card will be given),
Most likely dx and Mx.

The RP was so careful about what I am saying (he was my previous exam RP in previous exams!:D
and his name was the same(Lindsy) and I really hated him( to somewhat due to PTSD) but I tried
to focus on my tasks but yet it affected my performance).
(History)
- I asked about causes of diarrhaea bld?no Diarrhea? No hard to flush? Stick to pant? Floating
pu?
- No waterwork was NL
- his diet was only chips and 5-6 bottle of milk that was all.
- No worm infestation no itchy back passage no change in skin of backpassage
- no N/V/D.
- BID apart from N were NL.
- Happy family.
- No skin lesion or rash. fe def anaemia( due to high intake of cow's milk,5-6 bottle per day plus
chips,only diet of a 18m old)

(PEFE)
I asked about the PE card: the examiner gave me the card which was NL completely.

(Explain and treatment)


I explained the result in a lay man term and said the condition that you are having is most likely
due to excessive consumption of bottle milk and it can cause diarrhea with some bleeding( he
said but he does not have any bleeding at the moment I said there can be tiny bleeding which is
not visible in stool.
And then said we must put you on Hydrolyzed formula and wean him off from milk and I need to
run some investigation to rule out other possibilities like celiac and hypothyroidism and bleeding
disorder and infections. (I should have talked about the iron supplement but I totally forget abt
that due to my PTSD I should say)

Feedback: Pallor, FAIL(G.S:3)


Key steps:1,2 no,3 Yes,4:Yes, 5:No…
Interpretation of Ix:3
History:2
Dx/DDx:3
Mx:3
Karin case Iron-deficiency Anemia (Coeliac Disease)
Case: You are a GP and a 15-month-old boy was brought in by father because the child had flu 1
week ago. You did some blood test. Results showed hypochromic microcytic anemia with low
iron levels.
Cases with anemia: cow-milk protein allergy, celiac disease, nutritional anemia

Task
a. Further history
b. Explain results to father
c. Discuss management

Differential
- Infectious diarrhea
- Protein losing enteropathy
- Toddler’s diarrhea
- CMPI
- Nutritional anemia
- IBD/IBS

Features
- Diarrhea + weight loss + iron/folate deficiency and abdominal bloating
- Malaise, lethargy, flatulence, mouth ulceration, alternating diarrhea and constipation, pale and
thin patient, no subcutaneous fat
- Associations: IDA, type I DM, pernicious anemia, PBC, subfertility, lymphoma, dermatitis
herpetiformis, IgA deficiency, autoimmune thyroid disease, osteoporosis, neurological (seizures,
ataxia, peripheral neuropathy), Down syndrome

History
(current condition)
- Please tell me more about your son’s condition? Has he recovered from the flu? Any fever,
cough, runny nose?
(Diet history)
- I need to ask you a few more questions that are related to the blood tests that we did. Can you
please tell me his typical daily diet?
- Does he take enough fruits, veggies, and meat?
- What type of milk does he take? How much milk does he take in a day (>1 year: 2 bottles a day;
>2years: 1 glass at night)?
- When did you start giving him solids? Any problems after the introduction of solids, like
diarrhea or constipation?

(BINDS)
- Do you remember your wife’s pregnancy and delivery? Any complications with that (preterm or
LBW are risk factors)? What was his weight after delivery? Was he breastfed?
- Do you think he has been gaining weight appropriately since birth?
- Did you bring his blue book?
- Any concerns about his development? Immunization?
(Bowel motion history, diarrhea questions/ water works)
- Any problem with waterworks? How are his bowel habits? Do you think he passes bulky, hard
to flush stools? Any diarrhea in between? Are the stools offensive? What is the color?
(Family history coeliac)
- Do you have a family history of anyone on a special diet?

Management
- Let me explain to you the results of the tests we did. It shows that your son has a low level of
hemoglobin. One of the causes of low Hb is low iron which is essential for the development of
Hb. That is why he looks a bit pale. From the history, it seems like he has a healthy balanced
diet, so that is not contributing to his anemia. So my main concern is that he might be
suffering from a condition we call as celiac disease or gluten-sensitive enteropathy. Have you
heard about this?

- It is a condition where there is poor absorption of certain food from the gut. As a result, the
child develops diarrhea with or without vomiting, tummy pain, distention, and sometimes loss of
weight. The exact cause is not known, but research shows that it has been linked with certain
autoimmune conditions as well as genes.
This condition is associated with a higher risk to develop diabetes later on in life, thyroid
problems, skin problems (dermatitis herpetiformis), and sometimes cancers of the bowel
(lymphoma).
Please don’t worry treatment is quite simple, but he will require long-term followup.

We still need to confirm the diagnosis by doing some tests that we call as celiac screening test.
We will be checking certain antibodies in the blood (anti-gliadin, anti-endomysial, tissue
transglutaminase antibody) as well as the level of proteins. We will keep on doing regular TFTs
for him.
- The definitive diagnosis will be done by endoscopic biopsy wherein we take a small piece of
tissue from the lining of the small bowel and we will check it on the microscope. Once the
diagnosis is confirmed, you will need to have regular follow-ups with the pediatric
gastroenterologist.
The treatment requires avoidance of gluten-containing foods (BROW  barley, rye, oats, wheat).
There are special food aisles in the supermarkets with food labeled as gluten-free for your child.
Please check all food items before giving it to the child. You will need to inform the childcare or
his school.
- I will refer you to the dietitian as well if you need a diet chart for him. There is a celiac society
of Australia that you can contact.
200-Oral thrush
6 months infant with History of oral thrush for 6 months and diarrhea.
Tasks:
-History
- PE from examiner
- Dx and Management

Ddx
-Milk residue
-Oral thrush candida
-Lactose intolerance
-Celiac disease
-Immune suppressive condition
-S/E of antibiotics

History
1-oral thrush
-Since when? (4 weeks old age)
-Constant or comes & go?
-Anywhere else? E.g Nappies, nipples, vagina mum (+ve nappies red only)
-Have you tried anything to make it better? Anything makes it worse? (tried some medications
which helped for some time)

2-dehydration Qs
-Irritable?
-Waterworks nappies
-Feeding Q
*Breast feed or bottle (breast feed)
*Does he cry while feeding?
*Has he started solid? (Puree recently)

3-associated symptoms
-Fever, rash
-Bowel habits, vomiting (+ve chronic diarrhea) so ask diarrhea questions
-Cough, recurrent infection

4-risk factors
-Family history of diarrhoea?
-Antibiotics use
-Does he use pacifier/dummy? How often do you clean?

5-BINDS
-especially thriving well (baby born vaginally, immunization up to date, weight loss)
Pefe
1-G/A
2-V/S
3-growth chart (declining)
4-ENT especially oral cavity (rubbing giving +ve underneath bleed/ can’t removed)
5-systematic
6-genitalia with consent (anal excoriation )

Explain
-recurrent Oral thrush is most likely caused by yeast infection, candida albicans.
-We all have yeast in our bodies. However, sometimes if the immunity becomes low or any
imbalance occurs in the body, it can multiply and produce infection.

-It can also happen as a result of antibiotics which destroy the normal bacteria in the gut. Your
child might also develop oral thrush if he sucks an object, which is already infected with thrush
like pacifier. But less likely from history.

-Persistent infection may indicate something wrong with the immune system, which is
responsible for fighting infection, and diarrhea could be caused by the same mechanism.
Could be due to immunosuppression state called IgA. IgA is a defensive structure for fighting
infection found in lining of bowel .Not having IgA can cause infections from flora that is already
present in our body.

-That’s why we need to do some blood test and imaging like us to know the exact cause

Management
-Investigation (FBE, ESR CRP, swab of thrush MCS, stool MCS, celiac screening)

-Refer to pediatrician

-Antifungal miconazole gel for 10 days and advice to wash nipples and sterilise objects in contact
with thrush.
Feedback 5-7-2018
6 months infant with Hx of oral thrush for 6 months and diarrhea.
Task: 1-Hx 2- PE from examiner 3- Dx and Mx
I was disorganized in this station, I asked about the thrush and I asked Qs of chronic diarrhea and
BINDS. The father was just saying its diarrhea and no other positive hx. On exam nothing positive
apart from anal excoriation. In Dx I said its fungal infection usually self-limiting, persistent
infection indicate something wrong with the immune system, and diarrhea could be caused by
the same mechanism. I explained immune system responsible for fighting infection and we need
to do some blood test and imaging like us to know the exact cause and screening for celiac and
the bell rang.
Passed: global score 4

Feedback 5-7-2018
a 6 months old boy , brought in by dad to your GP, having a rash in his mouth (pic of oral thrush
given with the task outside the room)
Tasks
Take relevant history
Explain diagnosis
The rash has been there since baby was 4 weeks old. Asked if he is on solids and the dad said yes
purees. Askd if mom had the same (vaginal or nipple candidiasis) – he said no, mum is fine.
BINDS – baby born vaginally, immunization upto date.
Baby had chronic diarrhoea, and weight loss. Rash in nappy area too but not white just red.
On examination finding from the examiner, general appearance / vitals / oral thrush and
abdominal examination and nappy area examination .. growth chart (declining)
I explained to dad that this is recurrent oral thrush likely due to immunocompromised state. It’s
called IgA immunoglobinopathy .. IgA is a defensive structure found in lining of bowel that
extends from mouth, including bowels and to the bowel opening…. Not having IgA is causing
these infections from flora that is already present in our body.
I need to refer you to paediatrician.

Recall 9-2-2018
Baby oral thrush: 6 m old , started since birth, tried some medications which helped for some
time, mum's nipple is free, baby is growing good , only finding is diarrhea, no AB or steroid , no
HIV Hx
Feedback 27-10-2018 ORAL THRUSH
6 month old baby came to your GP with white patches in his mouth (picture of thrush in the
mouth was given). Father is concerned about his son.
Task
-history -PEFE -Dx/DDx History

-Father told me the white patches has been there since 4-6 weeks of his age.
-He is not satisfied with his growth and thinks his son is not gaining weight since then.
-When i asked him if he tried to scrape away the patches, he said ‘no’.
-no similar lesion elsewhere in his body.
-no fever
-regarding baby’s nutrition, they have introduced him some solid food recently
-bottle feeding. (no)
-similar lesion in mother’s breasts or birth canal during delivery. (no)
-Mother is not taking any medication such as steroid or antibiotics.
-Asked about infection screening including HIV in mother, and it was negative.
-Mother is generally healthy
-BINDSMA (term baby)
PEFE
-asked for the growth chart and examiner handed me 2 growth charts.
-in both charts, baby’s growth pattern is in downward trend.
-the last measurement fell on the 3rd centile.
-whitish lesion in the mouth as shown in the picture
then i asked the examiner whether it can be removed to differentiate it from milk curd, examiner
said ‘No, it can’t’
-excoriation (+) in the back passage.
-the rest of the examination are normal.
office test blood sugar level (normal)
Dx
-I told the role player what your baby is having is most likely to be due to a condition called
candidiasis. then, i explained him what candidiasis is.
-appreciate him that he brought his son early.
-told him i’m also concerned about your baby’s growth. Showed him the growth chart and
explained.
-told him I need to refer your baby to the pediatrician to have further assessment.
-then, i wrapped it up. suddenly i realized i forgot to asked about baby’s pees and poos started
asking pees and poos.
-role player told me his son has been having loose motion for a few weeks.
-then, i asked details about his diarrhea and explained to him that this motion problem could
also be related to the lesion in his mouth.

Grade pass Global score 4 Key steps 1,2,3,4 Yes, Yes, Yes, Yes
Approach to the patient 5 History 4 Choice and technique of examination, 4 Dx/DDx 4
201-Mental state Examination (MSE Vet and DV)
Psychiatry history in general (when taking MSE) + findings of MSE vet
1-Confidentiality
everything we gonna talk will be between you and I unless there is a harm to you or to others.
2-Mood history
-How is your mood ? How do you feel today? (Low)
-have you lost interest in things you used to enjoy? (yes)
-have you ever thought of harming yourself or others? (Yes, I want to kill myself) any plan (yes I
have a date and a plan for it) any previous attempts? (Yes)
-how is your sleep? Do you have difficulty getting to sleep or you wake up early in the morning
and have difficulty to get back to sleep? (wake up in the morning and difficult get back)
-how is your appetite?
-have you lost weight recently?
-do you find it difficult to concentrate on things? Do you feel guilty?
3-Hallucination
-do you feel, hear, see things that others do not? (yes hear my dead cat) what does it tell you?
(to kill myself)
4-Dellusion
Paranoid
-do you think someone is following you or spying on you?
Persecutory.
-do you think someone is trying to harm you? (yes my wife wants to kill me)
Grandiosity
-do you believe that you have a special power
thought insertion/ withdrawal
-do you think thought are inserted in your mind? Withdrawn from your mind?
Broadcasting
-do you think your thoughts are broadcasting through TV or radios?
Ideas of reference
-do you think other people’s action or behavior are directed towards you?
5-cognition
-where are you right now?
-what time is it?
6-Insight: do you think you need a professional help?
7-Judgement: let us suppose there is fire in this room, what would you do?
8-HEADS
-who do you live with? Any stress?
-if alone: if ever had a partner? / if young how about your siblings or parents?
-what do you do for living? Any stress? / how about your performance in UNI? Any stress?
-SAD
-social activities
9-General
-previous and family hx history of mental problems/ PMH/ PSH/ medications
MSE summary steps in general (Karen)
1-short Intro
I would like to present MSE of a 40 years old lasy ho presented to me with complaint of (chief
compliant) for (duration) + any other positive findings that will not be included in MSE.
2-Appearnace
-dressing (hair, sunglasses, clothes, nail, shoes etc…)
-grooming
-relation to weather
3-Behaviour
-Body posture
-eye contact
-cooperation
4-Speech
-Rate (slow/ rapid)
-volume (low/ high)
-tone (monotonous/ low, high).
5-Mood and affect
-mood: what the patient tells you
-affect: what you observed in your patient
Congruent/ incongruent
6-Thought form
-monosyllabic (depression)
-flights of ideas (mania)
-loosening of association (schizophrenia)
-Tangentiality (schizophrenia)
-concrete thinking (schizophrenia)
-Circumstantiality (OCD)
7-Thought content
-delusions
-overvalued ideas (BDD)
-obsessions
8-Perception (hallucinations).
9-Congnition
-where are you right now? -what time of the date is?
10-Insight
11-Judgement
12-3R -rapport -reliability -risk
Summary of MSE VET
1-short Intro
Look at the stem + any positive findings not mentioned in MSE
2-Appearnace
-dressing: not appropriately dressed and wearing sunglasses.
-grooming: not well groomed as his hair is messy and untidy
-relation to weather
3-Behaviour
-Body posture: stooped posture
-eye contact: poor eye contact/ not marinating eye contact with me at all.
-cooperation: sort of cooperative with me.
4-Speech
-Rate: slow
-volume: low
-tone: monotonous
5-Mood and affect
-mood: what the patient tells you
-affect: what you observed in your patient
patient described his mood as low and the affect that I am observing is flat which congruent with
the mood.
6-Thought form
appears to be monosyllabic as most of time he answered yes or no and not giving further details
unless I asked for it.
7-Thought content
-delusions:
*delusion of guilt as he said that he is guilty because he killed it.
*persecutory delusion because he said that his wife wants to kill him.
8-Perception
-auditory hallucination as he hears the voice of the dead cat.
9-Congnition
-oriented to time, person and place
10-Insight
-poor insight ad he does not know what is happening to him
11-Judgement
-impaired judgement as when I asked….
12-3R
-rapport: maintain sort of rapport with me.
-reliability: history unreliable
-risk: at risk of suicide because of depression, low insight, current thought and plan, previous
attempts of suicide, hallucination, delusion, support? SAD?
Feedback 15-8-2018 (History then MSE)
Scenario: Mood Change
Stem: lady in 30s who is a vet by profession and during a surgery a cat diet in her hands ( that
famous vet recall)
Tasks
~ History
~present mse

Inside was a lady sitting on chair unkempt and hair untidy messy and she was wearing
sunglasses and had a drooped posture.

Reassure, confidentiality, Ruled out organic illness and took detailed psychosocial history.

(positive was low mood, low appetitie and sleep, guilt, anhedonia, previous attempt of suicide(
she tried to hang herself) and now thinking of it again, she had a date planned and access to
the weapons as well, delusion of guilt, and auditory hallucinations asking her to kill herself)
Took quick headsss history. Meanwhile I was making notes to present Mse (ABSEPTIC RRR).
Finishedoff and thanked her.
Guys this history took a lot of time as role player was very slow in replying with big pauses.so
practice pls, leave umimportant questions and don’t miss relevant ones)
Presented Mse to examiner In the usualy Abseptic rrr way… told actually whatever I saw in her
and asked her that time. Examiner looked happy.
Grade : Pass

Feedback 23-6-2018 (Video then MSE)


Feedback: Mood change: watch the VIDEO (VET with the death of a customer’s cat) for 4 mins
and present MSE to the examiner.
When I walked into the room, only the examiner and the clip. And some pieces of paper on the
table.  This was extremely easy case.
My way:
- I made up a form myself on the paper provided: ASEPTIC + risk assessment.
A: shabby, unkempt hair, restless, drooping posture
S: understandable, but very low tone, poor volume of words, depressing voice; cooperative,
answer all the questions, but avoid eye contact.
E: depressed, down, score 2-3 out of 10, mood consistent with affect.
P: auditory hallucianation (voice telling him to kill himself), no thing else.
T: no abnormal thought contents.
I & J: non assessed
C: non assessed
Risk assessment: high risk with suicidal ideation
 while I was watching the video and filling out the form in details at the same time.
 RE-READ the form I wrote to the examiner and give Dx (major depression with psychotic
feature) with risk assessment (high risk with suicidal ideation).
My tip for the VIDEO station: FILL OUT the form (self made-up) and RE-READ the form. Do not try
to remember anything.
Grade: PASS, GS:5
Feedback 9-5-2018
Vetnerian, whose pussy cat pt died due to complication.. Tried to hang herself ?today. Partner brought
her in for consult. Task hx and present mse .to examiner ( pt wearing sunglasses, reluctant to talk at first..
But keep offering reassures confidentiality, n continue asking MSEIGCAPS /HEADSSS questions..
2 minutes outside the room:this is well known case of MSE, should perform ABSEPTIC, confidential .( This
is my weakness areas and i speak quite slow compare to other candida as if i speak fast, the examiner may
not understand ).
Perform:
I get into the room, the patient sat on the chair with sun glasses. I saw he were depress. I thrown out the
confidential and ask all SEGEMCAT/headds. The patient studdenly cry non stop, i did not ask any further
questions and perform MSE to examiner.
However, i think that i will failt this station as i forgot to describe about EMOTION as the patient cry too
much then it distracted me.

comment VET
Scenario was a vet doc who was brought by his wife as he was trying to hang himself with the rope he was
feeling guilty since a cat was died during a surgery. Task was to perform mse and to present the examiner.
He was having delusions of guilt and auditory hallucinations.
A vet specialist who's upset comes to see you. You did surgery on a n during surgery died. He's
depressed has auditory hallucinations n persecutary delusion ( thinks wife wants to kill him )
What could be the dx?
I did MSE as patient with major depression with ASEPTICS mnemonic as in Karen's note. But have to show
lots of empathy, he will cry (so offer tissue) and he spoke so so slow with lots of paucity. Make sure you do
not rush him and be extra patient. His clothes and hair was messy n too tight he also wore sunnies, make
sure to note this on MSE. Also note his speech. What I found positive were delusion of guilt (he believed
he killed his patient, a cat, despite all evidence saying not his fault), some say there's no psychotic features
but I think he mentioned he could hear the cat's voice (auditory hallucination). I didn’t ask anything else,
just the one that is relevant to MSE n maybe a bit of Risk assessment, see if he really is suicidal n need
admission
To show empathy : I waited a lot as he cried n paused many time. It’s hard because our time is limited too.
But try not to rush him with questions after questions. Validation words like “it must be difficult to be in
your situation” or “that must be heartbreaking for you” etc helps to build rapport and made him talk
more. I didn’t manage to get all the ASEPTICS points but maybe this is more of a bedside manner station.

Feedback 18-9-2018 MSE depression VET


MSE video vet loss a cat
task: watch the video and present MSE finding to the examiner.
Patient is unkept, wearing sunglass, not groom well. No eye contact, withdraw body language. Speech is
slow. Mood is low, 1-2 out of 10. Cry a lot and eyes sore. Affection congruent with mood. Delusion of guilt.
Auditory hallucination, he shouts go away at one point (not sure if he sees something as well). Cognitive is
intact based on the stein. I did not hear the question of sight and judgement. (But I think they have related
questions, I most probably missed it)
Had suicide attempt and still very much want to go home and suicide. See no hope in the future.
I said rapport is poor because patient is hesitated to answer some questions; reliability is low because of
psychotic features. And risk is high.
Score: 4
Key steps: yes/yes/yes/no
Choice of examination: 5, accuracy of examination: 3
Feedback 18-9-2018 MSE vet
Your next pt is a Vetenarian who has been brought to you by his wife after being
discovered attempting suicide with a rope hanging form the ceiling. He had an
incident 3 months ago and a cat died. He was cleared by the coroner but since then
his wife noticed significant behavioural changes and this morning with suicidal
attempt.
You will see a video interview of this pt in this station.
Your task is to
1. Watch the video for 4 mins ( the video only be played once and you can't rewind
it)
2. Present the MSE of the pt to the examiner.
My approach:
In the video, the pt was unkempt, wearing sunglass, voice was so low and unwilling to talk, no eye contact.
He started with a statement that I a horrible doctor and a terrible husband. The interviewer told no david
see your wife loves you, and cares for you, thats why she brought you here. Then he started crying so
much. Doctor asked tell me whats bothering you david? He answered its the pain and started crying again.
( By this time around 2 mins gone because pt took lots of time to start talking at the begining)
what pain? Its the pain inside doc, I killed her , she was on my hand and I killed her.
No You didn't David it was an accident. No No doc you don't know, and now I can't sleep they tell me that I
am the killer and I need to die.
Who tells you what? Here he stops answering.He just looks at his side and starts shouting go away, for
God's sake just go away, ya I am gonna die I know go away.
who are you talking ?
He starts crying loud and says that the Forest is back with all the animals and they are telling him to die,
they are giving pain in her head and he holds his head with 2 hands and starts crying and screaming.
The video ends here.
I told the examiner that my pt david who is a veternarian has come after attepting suicide and my
colleague took an interview after watching that my MSE is as follows:
Then I went ASEPTICJRRR approach.
Cognition and judgement were not assessed and its a high risk case.
AMC feedback:
Station 17: Mood change
Grade: Pass Global score: 4
Key steps: 1,2,3,4 no,yes,yes,yes
Assessment domain:
Choice & technique of examination,organisation and sequence: 4
Accuracy of examination: 4
Feedback 25-10-2018 MSE domestic Violence
Scenario: physical injury

younf lady who was victim of DV and you previously adviced her to leave her partner and move out. She
is living with her friend now. her sleep, appetite is normal?? (cant recall properly but most points of hx
were given outside)
Tasks:
Mse
Report MSE

When I entered a young female bending forward in chair not maintaining eye contact was there. I greeted
her, appreciate her for taking advice and moving out of abusive relation. Uttered some kind words etc lol
don’t exactly remember now. then gave confidentiality statement. Asked ASEPTICS. Thanked her for being
co operative.

Turned to examiner and reported mse. While reporting I realised I didn’t ask any qs for delusion, admitted
to examiner that I didn’t ask any qs to comment on thought content , she blinked eyes, so i continued with
reporting mse. In the end i said I'll refer her for ful neuropsychiatric assesment, she told me to look at my
tasks again (no further mx needed) i appologised. Still bell didnt ring so i turned to my pt asked her
permission to ask few more qs she said sure, asked all delusion qs and quickly told my examiner that if she
allows I woukd like to add few points to my mse report she nodded head in yes so I reported that too. Bell
rang
Grade: pass
GS: 4
3/5 key steps covered
Approach to pt:6
Choice & technique of examination: 5
Accuracy of examination: 4

Feedback 26-10-2018 Passed (GS – 4)


Young lady with history of domestic violence. She was advised to move out previously and now living with
friend. Some features of depression are given in stems (since she moved out, she felt low, reduced
appetite, poor sleep, etc.)
Task- do MSE & present to examiner
Perform MSE:
I followed ASEPTICJ RRR, SADMA, organic causes, HEADSS
On questioning,
Appearance – dressed appropriate with weather, no abnormal posture or psychomotor retardation,
patient can maintain eye contacts sometimes.
Speech – low volume, quality is clear, make sense.
Mood- depressed, no episode that she feels high (rule out bipolar)
Perception – no visual and auditory hallucination (skip tactile hallucination as I thought it’s irrelevant)
Thoughts form – linear/ no abnormal thought form
Thought contents – There is delusion of guilt but she denies delusion of grandiose, delusion of
paranoid/persecutory (as she feels guilty about her incident and said to me that she should’ve done
better)
Insight and judgement – intact
Risk -no suicidal risk Rapport is good.
History is reliable but I’d like to take collateral history if possible.
She denies all organic causes (shakiness of hand, temperature intolerance, DM Qs), also denies
recreational drugs use and smoking, but she lately increases amount of drinking.
(I just write down any positive findings I got, as we all knows how to present MSE.)
Case (31/3/2017) DOMESTIC
Major depression mse
A lady came to see u as her GP, she had Domestic violence issues, moved out from her partner's house to
her friends house. She has reduced sleep appetite.
Task
A) MSE
B) Present MSE to examiner

Case (9/6/2017) DOMESTIC


MSE for depression
Patient has hx of domestic violence she lived with friend now. Loss appetite, problem with sleep, no
suicidal ideation, not interesting with any things insight and judgment intact she is oriented with normal
perception. She can't cope with work
Do MSE and presented your finding to examiner

Case (31/8/2017) DOMESTIC


History of domestic violence...now living with friend, features of depression are given in stems
Task- do MSE & present to examiner
202-MSE mania
Mania (1/3/2017) (6/5/2017)
MSE
My first case mania typical case beautiful young girl but she didn't comb her hair probably wearing shirt
and pant walking around the room. I need to follow her and keep saying I want to see the president you
are wasting my time okay . My task MSE , present finding to the examiner, PDx, ddx
Insight effect she keep saying you are wasting my time high energy no hallucination no suicide when I ask
her, she said do you think I am crazy normal cognition.
I said most likely mania. I need to exclude the rest such as drug, infection and space occupying lesion need
admission and check by psychiatrist

Feedback 22-6-2018
MSE video Task present MSE after seeing the video
X was a 37 year old lady brought in by partner because she is has been acting unusual and disturbed. MSE
finding according to the video.. X was dishevelved in apprearance her hair was unkempt.. her posture was
restless and she kept moving around the room and looking at her mobile phone. She did not maintain eye
contact throughout the session. She was talking very fast, her speech was pressured. She said her mood
was amazing.. her affect was congruent to her mood. She said she did not need sleep, did not need to eat.
She was sexually insatiable and her partner was exhausted. She is drinking alcohol more. So i think she
might be a risk to herself. But i need to assess more. She has no hallucination. She says all the brilliant
ideas are coming to her mind by herself. On thought form she had flight of ideas as she said many ideas to
make thw company turnover to billions in a week. In thought content she had delusion of grandiosity as
she believed she is very special. She had no insight as on asking she replied just write i am ok so i can put
my ideas to my ceo. I could not asses her judgment so i would like to talk to her myself. She was co-
ordinated to time and place. Her history was reliable. She was agitated so i think she could not build a
good rapport with the examiner.. all in all i think she is going through a manic episode.. i would like to call
CAT team get her assessed admitted and be seen by psychiatrist.

Feedback 21-6-2018 Behavioural Complaint: Fail/ Global score:3


A 22-year-old female student. She lives with her friends. Her parents have noticed her behaviour has
changes, she has not slept the previous night. And she wants to take flight ticket to London to share
certain important information with the president about a terroristic attack or something similar. Assess her
mental state, explain your findings to the examiner. Tell possible differentials to the examiner.
The patient was very talkative and hard to stop. Also her speech was in a fast pace. I addressed her
speech, mood, affect, delusions (grandiosity +, paranoia - and persecutory -) hallucination was negative,
insight was poor, I forgot to check judgement as she was not really cooperative and I was really anxious. I
absolutely forgot to address thought from, which was circumstantial and had flight of ideas. After
explaining the finding to examiner, and mentioning that the symptoms are compatible with mania and can
be due to drug or bipolar disorder, or schizoaffective disorders, I mentioned that I have forgot to address
judgement, the rest of the delusions (they were quite unlikely to be positive), and asked the examiner if I
could go back to patient and ask the rest of the history, he said no, that part us over, do you have anything
else to add? And I said no as I had absolutely forgot to mention thought form. In my feedback, the only
score that has caused me to fail has been the accuracy of examination. . I guess as I have forgot to address
thought form, I have failed the station, however, I can never be sure.
Feedback 7-9-2018
behavioral complaint by parents – PASS

Acute Mania
STEM:
Young girl – a university student , brought in by the parents because of odd behavior recently .
Nil known previous mental health illnesses , not mention about medical condition in the stem .
TASK:
1 . perform MSE to the patient within 4 minutes
2. explain MSE to the examiner
3. provide Dx and DDx to the examiner

Task 1
After just introduce myself, patient shouted me very loud that she is gonna take airflight to save the
President in London as terrorist attack is coming soon . She got the message from flowers and signs of
Omen to London . , before confidentiality ( I forget to explain about confidentiality , as the patient was
walking around the room and interrupted a lot , though I explain about , wanna help you as for your safety
and your parent’s concern. )
Can’t finish all EPTIC –
when ask suicidal or homicidal ideas – the patient laughed very loud, are you kidding me doctor? I am
saving the world now . ( didn’t answer nearly 30 seconds anything and looking at the other side of room )
and whispering herself.

P.S – I personally thought that , candidates might think that the role player didn’t co-operative with us . Of
course, she is in acute mania stage, will not co-operative with us and .
Though I didn’t finish all questions about ASEPTIC ( EPTIC – questions to the patient ) , the simulated
patient told everything with behavior , just explained to the examiner , what I didn’t ask in ASEPTIC and
what the patient did tell me with behavior .

Task 2 – Explain the examiner about all ASEPTIC


Appearance - ( Appearance– uncomb hair , not –co-operative , not wearing glasses , humping posture ,
clothing – messy , speech – pressure speech , tone – very high , volume – a lot of words , contents – not
reliable )

Emotion ( MAESS , WASS ) -

– obviously my patient has – high mood at this stage ,

the patient didn’t sleep at all for 2 days ,( disturbed sleep )


I need to ask collateral history from parents about “ appetite , weight changes , anhedonia ,
However, the good news is that my patient doesn’t have homicidal ideas or suicidal ideas at this moment
Perception – can see with her behavior – auditory and visual hallucinations are present
Thought – RCGGP.
Explain – my patient has delusion of persecution and grandeur – but cannot assess delusion of other
things such as reference , control , guilt .
Insight – nil insight –as ( name of the patient ) told me that she killed the fire in the room with her power
gotten from flowers .
Cognition – cannot assess at this consultation
( I FORGOT TO TELL CONFIDENTIALITIY TO THE PATIENT AT THIS STATION ) -
Task 3-
Most likely – acute mania state to my patient- Possible causes are –
1.acute relapse of schizophrenia
2. Manic episode of bipolar disorder
3.Depressive –psychosis
4. Drug-induced psychosis or drug-withdrawal stage
5. malignancy of brain parenchyma
6. other metabolic conditions to be ruled out – such as – thyroid dysfunction , severe glycemic dysfunction

Feedback – Behavioural complaint – PASS Global score – 4 Keyp step 1 to 4 – all YES
Appraoch to patient , organization or examination , Dx and DDx – all YES

Another feedback 7-9-2018 Behavioural complaint- PASS)


An young lady, uni student, c/o behavioural problem, brought by parents. She thinks she will save US
president from something. No past mental hx, family hx, no known medical condition.
- Take HX.
- Present MSE to examiner
- DX, DDX to examiner.
Global score: 4 Key steps: ¾ Approach to pt: 3
Choice of technique of examination, organisation and sequence: 4
Accuracy of examination: 4 DDX: 4
2 mints thinking: I thought it was OCD case!!( Don't know why!!) I was thinking about OCD questions and
ddx!! :D
I didn't read details stem, I guess!! Psychiatry cases, my strategy was : go with the flow!! Main important
part is suicide hx, perception, hallucination, thought problems. Thyroid and drug problem will go with
every psych case!
DDX:
1. Acute Mania 2. Acute manic phase of bi-polar disorder. 3. Acute psychosis.
4. Side effects of street drugs, hard drugs. 5. Myxoedemic psychosis/ Thyroid disorders.
( I forgot to say hypothermic personality disorder)
Examiner introduced me to the pt, and asked: are you ready?
After that, she started her role playing. As soon as she started, I realised, my god! it's mania!! I didn't get
time to introduce myself to her. She was blabbering about US president thingy. Even though, she was not
listening to me, I said confidentiality statement in between. She looked at me like: what did you say?? :D
I jotted down ASEPTICS in the paper first.
Anyway, I said her; I understand it's an important issue for you, can you please tell me, is someone ask you
to do so? Can you see or hear someone that I don't? ( She stared at me and somewhere between her
stories, she said me NO)
Then, I asked her ; Did you ever think about killing yourself? ( She was like, what? She told me, all she was
thinking about saving people, not harming anyone. I asked what about you? She said no.)
I asked her about hallucination, her thought problems, has she ever got high mood, low mood. (I forgot to
ask her current mood).
She talked a lot of things, She was also pacing around the room, told me how beautiful the red door knob
was. I also stood up along with her. At one point, she told me stupid. I acknowledged that, said : I'm sorry
that you are feeling this way, but trust me I will help you out.
Anyway, examiner stopped me after 4 minutes. I couldn't finish all.
I stopped then n there and started MSE presentation like ASEPTICS+3R then, said the ddx. I mentioned
what I couldn't finish as well. I got enough time. I sat around 2 more minutes. I asked examiner if I would
present MSE one more time? Examiner said up to you. I didn't do anything. Just sat there quietly.

203-MSE Delirium Post-Operative


HMO in surgical ward, middle aged patient who had a surgery done for dislocated right shoulder
currently is behaving oddly in the ward.
Tasks:
-History from the patient
-Present MSE to the examiner
-Likely diagnosis and causes to the examiner

History
1-Approach
-when you enter there will be a patient sitting and holding a pillow trying to hit cockroaches.
Calm her and reassure her. I will call the cleaner to clean the room.
-do you know why you are here? Do not worry I will explain to you. You had a shoulder
dislocation and just went through a surgery
-do you have pain right now? (No)
2-Confidentiality
everything we gonna talk will be between you and I unless there is a harm to you or to others.
3-Hallucination
-I can see that you are hitting cockroaches. Where are they? (All over, the place is very dirty)
- do the cockroaches crawl on you or on the table? (Yes, go away)
-do you hear things that others do not?
4-Dellusion (all negative)
-do you think someone is following you or spying on you?
-do you think someone is trying to harm you?
-do you believe that you have a special power
-do you think thought are inserted in your mind? Withdrawn from your mind?
-do you think your thoughts are broadcasting through TV or radios?
5-Mood history
-How is your mood ? How do you feel today? anytime you feel high?
-have you lost interest in things you used to enjoy?
-have you ever thought of harming yourself or others? (no)
-how is your sleep? Do you have difficulty getting to sleep or you wake up early in the morning
and have difficulty to get back to sleep?
-how is your appetite?
-have you lost weight recently?
-do you find it difficult to concentrate on things? Do you feel guilty?
-do you think other people’s action or behavior are directed towards you?
5-cognition
-where are you right now?
-do you know who I am? (Doctor)
-what is the day of the week?
6-Insight: do you think you need a professional help? (No)

7-Judgement: let us suppose there is fire in this room, what would you do?
8-HEADS
-who do you live with? Any stress? (alone)
-if alone: if ever had a partner? / if young how about your siblings or parents?
-what do you do for living? Any stress? / how about your performance in UNI? Any stress?
-SAD (couple of alcohol before she had a surgery)
-social activities
9-General
-previous and family hx history of mental problems/ PMH/ PSH/ medications

MSE summary steps


1-short Intro
I would like to present MSE of Jill a 57 years old woman who was behaving strangely post
surgery.
2-Appearnace
-dressing: wearing appropriate to weather. -grooming: unkempt.
3-Behaviour
-Body posture
-eye contact
-cooperation
4-Speech
-Rate: pressured/ fast
-volume: high
-tone: high
5-Mood and affect
-mood: what the patient tells you
-affect: what you observed in your patient
Patient described his mood as----and the affect that I am observing is congruent with the mood.
6-Thought form: is linear
7-Thought content: no delusion or obsessions
8-Perception
-experiencing visual and tactile hallucinations as she is seeing cockroaches and telling that they
are crawling on her.
9-Congnition: no oriented to time but oriented to place and person.
10-Insight: impaired
11-Judgement: impaired
12-3R
-rapport: I could build rapport with my patient all throughout the consultation.
-reliability: history is unreliable; I need to look into her case notes for PMH, PSH, operation chart
and Ix.
-risk: no risk of suicide (I think not sure)
Dx: most likely patient is having post op delirium. There are many causes it could be alcohol
withdrawal, which is the most likely. Need to rule out electrolyte disturbance, hypoxia, hypo or
hyperglycemia, medications, infections.

Feedback 20-2-2018
HMO in surgical ward, middle aged patient who had a surgery done for dislocated right shoulder
currently is behaving oddly in the ward.
Tasks:

- History from the patient

- Present MSE to the examiner

- Likely diagnosis and causes to the examiner

2 min thinking: most likely delirium, might need physical restrain. As usual ASEPTIC +PMHx +
SADMA

History:
Introduce myself. Patient is sitting and holding a pillow, trying to hit cockroaches. I tried to calm
her, reassure her I will call the cleaner to clean her room. I told her confidentiality statement.

Do you know where you are right now? (I am in the hospital and this place is filthy!) do you
know why you are here? (forgot her answer) Don’t worry Jill, I’ll explain to you why you are here.
You had shoulder dislocation and just went through a surgery. Do you have pain right now? (no)

(Emotion)How is your mood right now? (forgot what she said) anytime when you are feeling very
high? (no) very low/sad? (no) At any point of time do you think about harming yourself? (no),
how about harming others? (no)

(Perception - hallucination)Jill, I can see that you are hitting the cockroaches, where are they?
(all over the places, so dirty!) Do the cockroaches is crawling on you as well? (yes, go away!) (and
on your table) Thank you for helping me to swipe away the cockroaches, I’ll make sure this place
is being cleaned. (oh you better do!)

At any time does your thinking being withdrawn from your mind? (Thought withdrawal) (no) or
someone trying to insert some ideas into your mind? (Thought insertion) (no), to you think your
thinking is being broadcast? (Thought broadcasting) (no), does the news in TV or newspaper
hold special meaning to you? (Delusion of reference) (no) Do you think people is trying to spy on
you or harm you? (Delusion of persecution) (no)

Do you think you might need some medical help? (Insight) (no), if there is a fire in this room..
(Judgement) (where is the fire, I need to run) don’t worry Jill, there is no fire right now, please sit
down. I am making an assumption.

(Cognition)Do you know what is the day today (Wednesday), do you know who I am? (you are
the doctor)

Whom do you live with? (alone) what is your occupation? (sorry I forgot the answer), do you Do
you smoke/drink alcohol (a couple) when was the last time? (before I had the surgery)/use
recreational drugs? (no)
Thank you for the information you have told me. Now I would like to talk to my examiner.

MSE:
Dear examiner, I would like to present to you the mental state examination of my patient, Jill, 57
years old female who was behaving oddly post surgery.
She is wearing appropriate to time and season, unkempt.
She is responding to the examiner’s questions, however she is distracted by the visual
hallucination every now and then.
Her speech is rather pressured, high in tone and volume.
She described her mood as ___, which is congruent to the affect.
She is experiencing visual and tactile hallucination as she is seeing cockroaches in the room.
In terms of thought form, it is linear. She denies any thought insertion/withdrawal/broadcasting.
She denies delusion of persecution, no suicidal and homicidal ideation.
Her insight and judgement are impaired.
She is not orientated to time, but orientated to place and person.
I can built a rapport with the patient throughout the consultation.
In terms of risk assessment, she is having some risk to herself and the people around her.
Her history is unreliable, I need to look into her case notes for past medical history, surgical
notes, obs chart and investigation.
Most likely my patient is having delirium. There are many causes to her condition however I am
suspecting it could be due to alcohol withdrawal. I need to rule out other causes for example
electrolyte imbalance, hypoxia, hypo/hyperglycaemia, medication, infection.

AMC Feedback – Disturbed behaviour: PASS


Feedback 20-4-2018
Case (10/2/2017)
1- Px was admitted to the hospital for shoulder surgery. Management chart is given in the stem
and he has abnormal behavior now. Vitals- N.Task: Hx, Psychosocial assessment, DDs.
Positive findings: Px in arm sling with tremor . He see “cockroaches” all over her body.
I spent 1min asking her organic D/DS , I asked her about fever ? headache? Any weakness in her
limb ? any head injury ?wether she took alcohol? She was angry that nurses are not giving her
alcohol .I asked her wether she was drinking everyday ? any drugs ?
suicide risk ?ASEPTIC-tactile hallucinations.
Px not oriented in time/season but oriented in person/place, My patient is not fully oriented I
will shift to MMSE ( chart is not given u need to keep it on your heart )-ORAR (impaired) L (N) C
(impaired).
Then present ASEPTIC and MMSE .
DDs: acute delirium, infection, drug overdose, electrolyte imbalance, alcohol withdrawal.
I think the approach will be ( history taking then delirium workup then look at mx chart then
mmse and aseptic then ddx)

2- Post surgical delirium with cockroaches everywhere


Do MSE with differentials
followed ASEPTIC with few organic qs and psych qs.
Disoriented to time and place
Be prepared because the roleplayer threw the tissue paper box at me and I was about to fall
down from the chair when he suddenly stood up
Passed

Handbook case 149


Confusion and delirium after surgery in a 50-year-old man

CANDIDATE INFORMATION AND TASKS

You are the intern called to the ward to see a 50-year-old man who has become confused two
days after a left total knee replacement. A few hours earlier, he started to behave in an irrational
manner, became agitated and difficult to manage. Until this stage he had been making an
uneventful postoperative recovery. His confusion has now culminated in the patient being
disoriented, noisy and difficult to restrain.

The patient's wife is with the patient and she has been unable to help.

YOUR TASK IS TO:

• Assess the situation; formulate a management plan; and counsel the patient's wife as to the
cause of the current problem.

In this scenario format, your optimal approach is as shown in the performance guidelines,
indicating questions you should be asking yourself as you proceed. 1<2
The Performance Guidelines for Condition 149 can be found on page 773

How will you approach the problem?

-Your first action must be to ensure that the patient is not a danger to himself or to others
around him. This may involve a degree of physical restraint whilst you assess the situation and
obtain a history and examine him.
-You are unable to obtain a history as he is irrational and does not respond coherently to
questions.
- Before you examine the patient, it would be prudent to ensure that intravenous access has
been established and that he is on a high flow oxygen mask. Institute pulse oximetry if
available. At the same time as you insert an intravenous cannula, collect blood samples for
routine haematological and biochemical screens. In addition, collect samples for blood sugar
estimation, and blood culture, especially if he is febrile
Delirium workup
1-FBE , vit b12, folate, crp, esr 2-LFT, TFT
3-urea, creatinine, electrolyte 4-blood sugar
5-blood culture 6-urine microscopy and culture
-At this stage you should look at the case notes and the nursing observations charts. In
particular, you should note:

• Any recent change in the vital signs (blood pressure, pulse rate and temperature).

• Fluid balance.

• Recent drug administration (e.g. opiates, hypnotics).

• Details of the recent surgical procedure.

• Past medical history (e.g. problems related to diabetes or cardiovascular disease).

• Drug use (including alcohol).

• Comments in the medical and nursing record (e.g. sleep patterns, any abnormal behaviour).

~ Any abnormal laboratory investigations (e.g. anaemia, macrocytosis, abnormal liver function).

Once you have obtained as much information as possible from the case notes, nursing record,
patient, relatives and personnel involved in the immediate care of the patient, you are in a
position to undertake a physical examination.
How will you focus your physical examination?

Talk to the patient, reassure him and try to establish whether he is oriented and aware of what
is going on. He is clearly quite disorientated. Undertake the following:

• Check the vital signs of pulse rate and rhythm, respiratory rate, temperature and blood
pressure.

• Examine the cardiorespiratory systems.


• Look for evidence of sepsis (chest, wound).

• Look for evidence of venous thromboembolism (legs, chest).

• Look for any neurological deficits.

When you have completed your physical examination you can discuss the situation with the
nursing staff and family members and formulate a management plan.

It is difficult to take a history or to examine the patient as he is agitated and insisting that he be
allowed up to go to the toilet. He spends much of the time sitting on the edge of the bed.
The chest appears to be clear, but with his confused state, your examination is less than
adequate. Similarly, the examination of the abdomen is rather difficult, although you think
there may be some tenderness in the lower quadrants. The operation site around his knee
looks healthy.
The patient's wife tells you that the patient has been quite well up to the time of his recent
surgery, but had become rather anxious over the impending operation and had been drinking
rather more than usual.

- In the light of the history and examination:

What are the likely causes for the confusion and how you would go about differentiating them?

The likely causes of his confusion would include:

• Hypoxia: a very common cause, particularly in elderly patients. Arrange arterial blood gas
analysis (by this stage the patient should be attached to a pulse oximeter), chest X-ray and ECG
(possible pulmonary embolus).

• Hypotension: measure vital signs.

• Sepsis: investigations should include: white cell count, urinalysis, chest X-ray and blood
cultures.

• Metabolic abnormalities: serum electrolytes; blood sugar estimation; arterial pH.

• Cardiac disease: ECG to exclude myocardial ischaemia/infarction, arrhythmias.

• Cerebrovascular event; neurological examination.

• Pain: look at medication chart and nursing notes.

• Opiate overdose or effect of other drugs: look at medication chart. Cease any potentially
contributing medications.

• Drug withdrawal, including alcohol, benzodiapines and narcotics: seek information from
family and friends.

• Exacerbation of pre-existing medical conditions: for example dementia, hypothyroidism —


check past and recent history, arrange blood tests (thyroid function tests — TFTs).

The patient becomes quieter in response to intravenous fluids and oxygen by face mask.
How would you now counsel the family?

You need to reassure the family that:

• The situation is under control.

• Your investigations may yield a cause for the confusion.

• There could be a multitude of factors contributing to the patient's delirium.

• Alcohol withdrawal is a common cause of post-operative confusion and should be easily


controlled and the problem self-limiting.

• You will return to review the patient progressively, particularly when the results of the
investigations are through.
This clinical scenario was based on a real case. Little has been changed. Amongst the
investigations performed was a chest radiograph. This showed free gas under the right
hemidiaphragm. Further abdominal examination of the now calmer patient revealed clinical
evidence of a lower abdominal peritonitis.

An exploratory laparotomy revealed a widespread purulent peritonitis secondary to


perforated diverticular disease. The patient underwent resection of the diseased section of
sigmoid colon and made a satisfactory recovery.

In this instance, the patient's delirium was initially put down to alcohol withdrawal. By following
sound clinical practice, other causes of confusion were sought and the problem eventually
tracked down to intra-abdominal sepsis and acute diverticulitis with peritonitis.

204- Examination Tonsillitis


Refer to lecture 159 for examination approach
Feedback 28-11-2017

205-Obssessive compulsive disorder


Case 1
35 year old maria; a nurse comes to your GP clinic. She thinks her hands are always
contaminated and may transfer infection to her patients. She was seen by your colleagues 1
week back when she was sone certain Ix which seemed to be normal.
Tasks
-take history (or watch video)
-present MSE to examiner (or explain Dx and Mx to patient)

Case 2
A- Young Male University student referred by the student counsellor to the GP, because he had
falling grades and gradual reduction in his school performances. He is more keen in depth
unnecessary details rather than completing the tasks.
Tasks
-Ask further history
-Probable diagnosis and Differentials
-Answer patient’s questions

B- 32yom c behaviour change referred by uni prof, not finishing tasks and fixated on certain
topic.
Task:
-Present(2min) the mse after viewing (4min) it being performed.

Case 3 ??
A-30 years old lady who says she has some kind of thought problem in a health survey. Today she
came to your GP clinic to discuss about that.
Tasks:
-History taking
-Dx with reasons

B-a 25 year cleark girl just separated from husband few months ago. Now having thoughts of
taking pictures of her ex having sex with current spouse. She overcomes that by counting from 1
to 7
Tasks
-Take history
-dx and ddx
Case 1 (washing hands)
History
1-Approach
-Confidentiality
-ask about concern or further concern (they will tell you they have thoughts of hands are
contaminated)
-Can you tell me more?
2-OCD questions
-when you are having these thoughts what you usually do? (Wash hands)
-How long these thoughts are going on? (6 months)
-How often do you get it? Or how often do you wash your hands? (Every 15 minutes)
3-Trigger factors questions (anything happened + home questions)
-so 6 months ago does anything happen that related to this?
-do you live alone? Any children? Do you have good support from parent and siblings?
4-Insight
-do you think it is irrational to keep washing your hands? (yes)
-have you tried to resist these thoughts? (yes but unhelpful)
-any other thoughts keep coming to your mind? (No)
5-Personality
-how can you describe your personality? (if perfectionist)
6-affecting life + rest of HEADS
-how do these thoughts make you feel or affecting your life?
-any stress at work?
-have you lost interest in things you used to enjoy? (cannot enjoy hobby)
-SAD
-how is your social activities? (Impaired)
7-Psychosocial history
Mood history
-How is your mood ? How do you feel today? (Worried)
-have you ever thought of harming yourself or others?
-how is your sleep? (Difficulty getting to sleep)
-how is your appetite?
-have you lost weight recently?
Hallucination
-do you feel, hear and see things that others do not?
Delusion
-do you think someone is following you or spying on you? do you think someone is trying to
harm you?
Cognition
-where are you right now?
-what time is it?
Insight: do you think you need a professional help?
Judgement: let us suppose there is fire in this room, what would you do?
8-General question
Case 2 (KING)
History
1-Approach
-Confidentiality
-ask about further concern? Can you tell me more? (They will tell you they are failing grades)
-Why you are failing grades? (Worried about word KING) (Ask this if they did not mention his
thoughts in the stem)
2-OCD questions
-when you are having these thoughts what you usually do? (Walks 3 rounds around the chair
when hear that word)
-How long these thoughts are going on?
-How often do you get it?
3-Trigger factors questions (anything happened + home questions)
-so ---months ago does anything happen that related to this?
-do you live alone? Do you have good support from parent and siblings?
4-Insight
-do you think it is irrational to have such thoughts? (yes)
-have you tried to resist these thoughts? (yes but unhelpful)
-any other thoughts keep coming to your mind? (No)
5-Personality
-how can you describe your personality? (Perfectionist)
6-affecting life + rest of HEADS
-how do these thoughts make you feel or affecting your life?
-any stress at UNI?
-have you lost interest in things you used to enjoy?
-SAD
-how is your social activities?
7-Psychosocial history
Mood history
-How is your mood ? How do you feel today? (Worried)
-have you ever thought of harming yourself or others?
-how is your sleep?
-how is your appetite?
-have you lost weight recently?
Hallucination
-do you feel, hear and see things that others do not?
Delusion
-do you think someone is following you or spying on you? do you think someone is trying to
harm you?
Cognition (optional except in MSE)
-where are you right now? what time is it?
Insight: do you think you need a professional help?
Judgement: let us suppose there is fire in this room, what would you do?
8-General question
Case 3 (thoughts)
History
1-Approach
-Confidentiality
-ask about further concern? (had thoughts that troubling her)
-can you tell me more about these thoughts? (Had recurrent thoughts of her ex-husband having
sex with another woman and because of that, she couldn’t sleep well)
2-OCD questions
-when you are having these thoughts what you usually do? (Count 1 to 7)
-How long these thoughts are going on? (1 year)
-How often do you get it?
3-Trigger factors questions (anything happened + home questions)
-so ---months ago does anything happen that related to this?
-do you live alone? Do you have good support from parent and siblings? (Lives alone, divorced,
no family members)
4-Insight
-do you think it is irrational to have such thoughts? (Yes)
-have you tried to resist these thoughts? (Yes but unhelpful)
-any other thoughts keep coming to your mind? (No)
5-Personality
-how can you describe your personality? (Perfectionist)
6-affecting life + rest of HEADS
-how do these thoughts make you feel or affecting your life?
-any stress at work?
-have you lost interest in things you used to enjoy?
-SAD
-how is your social activities?
7-Psychosocial history
Mood history
-How is your mood ? How do you feel today? (Worried)
-have you ever thought of harming yourself or others?
-how is your sleep?
-how is your appetite?
-have you lost weight recently?
Hallucination
-do you feel, hear and see things that others do not?
Delusion
-do you think someone is following you or spying on you? do you think someone is trying to
harm you?
Cognition (optional except in MSE)
-where are you right now? what time is it?
Insight: do you think you need a professional help?
Judgement: let us suppose there is fire in this room, what would you do?
8-General question
Note/ If task was MSE you need to ask all about delusion and focus on MSE approach.

Explain the diagnosis


-OCD is an anxiety disorder characterised by recurrent and persistent thoughts or obsessions
that are intrusive and unwanted, followed by repetitive intentional acts or behaviours called
compulsions, which you cannot control even though you try to resist it.
-Later, it can become extremely distressing and time consuming for the patient. But OCD is a
treatable condition and seeking professional support is the 1 st step towards recovery and
wellness.
-exact cause is unclear but found to develop from a combination of genetic and environmental
factors.

Differential diagnosis (less likely with reasons)


-depression
-GAD
-adjustment
-thyroid
-head trauma
-substance abuse.

Management
-you need a psychologist referral for CBT (exposure and response prevention). Exposure involves
confronting what you fear while response prevention is a complete cessation of behavioral or
mental disturbance associated with it.
-you also need referral to a psychiatrist for SSRI.
-support group
-Lifestyle modifications+ relaxation technique
-reading material
-review in 1 month.

**Regarding nurse stem:


-regarding your home situation I can arrange a social worker who can look after your kids and
give respite to your husband.
-I can give you medical certificate to take some time off work.
-relaxation and meditation.
MSE (washing hands)
-I would like to present MSE of Maria; a 35 years old nurse, who comes to e to get help with her
constant and regular handwashing habit.
she had a divorce 8 months ago and having stress at work because of this habit and unable to
participate her hobbies and also social activities because of this.
-she appears well dressed according to weather and also well groomed.
-even though she is cooperative, she seems preoccupied with arranging the papers and
stationaries on my table and she settles a bit after arranging everything. She does not make good
eye contact. I could say she is a bit irritable.

-her speech is steady with good rate, volume and tone.


-mood according to her is worried and the affect I observed in her is anxious which is congruent
with the mood.
-her thought form is coherent and perseverant.
-thought content; she has obsessional traits of repetitive hand washing and orderliness. No
delusions.
-no auditory or visual hallucinations.
-she is oriented to time, place so cognition is normal.
-insight is good because she knows what is happening to her said her thoughts are quite
irrational.
-judgement is intact.
-history appears to be reliable and could achieve a good rapport with my patient.
She is not at risk to herself or others.
MSE (KING)
Feedback 13-12-2018 Behavioural Complaint: Pass.
2 min Outside: You will see the video interview. After the video You need to present MSE to examiner.
Video will be played only once.
I had no idea what is this case as there was no more history.
Thinking: Could be depression or Mania as there were only 2 cases came in Video in the past. I quickly
remembered the sequence of ASEPTICJRRR.

Inside: Examiner started the video. In the video interviewer was behind the scene means not shown in
video, just the voice. The patient was a university student with beard (I personally think he was bit older,
may be in late 20ss or early 30s). Pt was wearing shirt and was ok in terms of neatness. I think in video it
would be important to look at the pt and see what he is doing along with audio. He was fidgeting with
fingers with head down in early phase of interview (At this stage, I thought it was the case of depression).
All the psychiatry history was asked in good manner. Positive point was he was a bit of perfectionist and
takes hours and hours for assignment and project. He sometimes keeps revising them even though
nothing wrong with them. He missed many deadlines for them (typical of OCD). At this stage I made my
diagnosis and was looking for extra features of OCD or coexistence disease such as anxiety or depression.
In later phase he was quite open and happy as interviewer asked him what else bothers him. He said he
has this strange thought of his uncle is having sex with his mother. He can’t get rid of this thought and
makes him very nervous. Interviewer: What do you do to get yourself comfort or to get rid of this idea. Pt:
I think he said he does count to ten or walk around the room (either way it’s a typical of compulsive
behaviour). All other features were negative such as suicide, hallucination, delusions. He was oriented to
time place and person (not asked directly but can be assessed from interview). Question regarding
judgement was not asked.

MSE presentation: Young patient was interviewed today. The patient was groomed ok according to
situation. The pt was looking down and playing with his fingers in early phase of interview (I think
examiner liked this as he gave me some different look). Pt’s speech was hesitant in early phase but clearly
understood. The tone was ok. I did not notice any speech issues such as pressured speech. Emotions were
ok, pt was not crying and did not notice any strange emotions. The mood was ok and affect was congruent
according to mood. No signs of depressed mood were noticed. There were no signs of hallucinations or
illusions. No delusions were noted such as delusions of grandiosity or delusions of persecutions. There
were no signs of thought form disorders such as flight of ideas, tangentiality or circumferentially etc.
Insight can be judged as intact as patient knows what’s happening to him. Although no direct questions
were asked about cognition, it can be judged from interview that Patient was oriented to time, place and
person. Judgement was not assessed in this interview. Reliability was ok. Risk was low as pt has no
thoughts of suicide or harming others. The rapport was okay at the end of interview.
In my opinion, the patient is suffering from condition called Obsessive Compulsive Disorder (Never say
short form to examiner) as he has bizarre thoughts of his uncle having sex with his mother as in form of
obsession and these thoughts are interfering with his life. To combat this obsession, he has some form of
compulsion of walking in the room (or counting to ten). In support to this condition he has perfectionist
personality as evident from his university work.

At this stage examiner was going to ask me something but after 2-4 words bell rang and he told me you
may go now. I was feeling good about this case but did not get too good score.

Global score: 4 Key steps: All 4 yes.

Choice & technique of Examination, organisation and sequence: 5 Accuracy of Examination: 4


Feedback 8-11-2018

Feedback 21-2-2018 (KING)


CASE 15: MSE VIDEO OCD- PASS
video played for 4 minutes then we have to present the MSE to examiner.
In stem, patient young male difficulty incompleting his tasks as doing things repeatedly, lives
alone , no drug or alcohol history, some other history given, not remembering those.
Patient appearence- appropriate dressing, maintained eye contact only at times, anxious and
fidgeting while talking, speech- normal; obsessive thought present -mom having sex with cousin
but the patient says that's not true, do things again and again to be perfect and not completing
his tasks; count 3 times, googled about ocd and has insight for treatment, no hallucinations, no
risk of suicide or harming others, rapport good, reliable history, judgement question was not
asked in the video.
Feedback 21-4-2018 (KING)
Young Male University student referred by the student counsellor to the GP , because he had
falling grades and gradual reduction in his school performances. He is more keen in depth
unnecessary details rather than completing the tasks
Ask Futher history – worried about the word ‘’ King’’ , walks 3 rounds around the chair when
hear that word, tried to resist it and failed, no suicidal ideation, consider as a perfectionist ,
definitely asks for organic causes ( no thyroid problems , no infections, no drugs ).
Probable diagnosis and Differentials – OCD , other DDx – Generalized anxiety Disorder, Panic
attack, Depression, Organic pathologies ( Hypothyroidism , Sepsis, DM )
Answer patients questions – asked will u tell to my school teacher – I said yes , that is to assist
you and not to bully you – he was happy with my answer. (Be careful how you say yes and you
assured confidentiality first hhhh that is why counselling 3)
Global score - 4 ( Approach 6, Hx 4 , Examination 4 , DDs – 4 , Counselling – 3)
My conclusion - Read OCD King Case in Carrens Notes

Feedback 4-7-2018 (KING)


Uni student not good grades & delay in submitting assignment
More concern about neat work than the content
Task hx/explain Dx answer pt’s Q. No Mx
Typical OCD-King –but need to ask detail to get this information
RP asked what would you do to uni-said will given letter & with pt’s consent will having meeting
at uni student counsellor

When started to talk about CBT-Ex said no Mx

Feedback 30-5-2018 (KING)


‘Behavioural complaint’:
MSE video –OCD
32yom c behaviour change referred by uni prof, not finishing tasks and fixated on certain topic.
Task: Present(2min) the mse after viewing (4min) it being performed.
Feedback: Pass

 Classic OCD scenario

 ASEPTICJ as you view the MSE being performed

 When Presenting the case, I followed the typical ISBAR format: 32yom presents w xyz on
a b/g of abc etc etc [make note of everything you see…this guy had a row of pencils
perfectly lined up next to him on the desk]

 Stick to the task, I kept being tempted into talking about management

 I gave clinical correlates for each of the positives on MSE


Feedback (thoughts)
Health Check (PASS)
30 years old lady who says she has some kind of thought problem in a health survey. Today she
came to your GP clinic to discuss about that.
Tasks :
History taking
Dx with reasons
Approach :
I greeted the patient and asked her any particular concern? She said these thoughts are
troubling her a lot.
History :
I asked what kind of thoughts? She said she had recurrent thoughts of her ex-husband having sex
with another woman and because of that, she couldn’t sleep well. I asked more about sleep
problem. It was non-specific. I asked how long has it been going on? She said it’s been for a year
or so. This is the 1st time for her. I asked is there any other thoughts? She said no. I asked OCD
questions and all positive. When I asked did you try something to get rid of the thoughts? Then
she said when she count from 1 to 7, it goes away.
I excluded other DDx like other anxiety disorders, depression, PTSD and so on. No features of
depression and she did not have any major stressful events 1 year back.
When it comes to HEADDSS, she lives alone, can cope well, was divorced (don’t remember
when), no contact with family members? Father also passed away years ago, no problem with
job & financially secure, no smoking, alcohol drinking, no drug usage.
Dx : I explained her about OCD then said it can be treated effectively. Then told her other DDx
but they’re less likely in you with reasons.

Feedback 6-6-2018
Station 14 OCD –Pass
Middle age lady, came to your clinic for health check for ? insurance purpose.when she fill
questionaries, nothing abormal except she ticks yes in the question of “ do you have any pblm in
thinking”
Task: history, dxddx to patient
History: I understand you come here today for health check. Confidentiality. Any specific concern
or pblm at this moment? She started explaining she has this thinking pblm, want to take the
photo of her ex and his girlfriend. Same as old recalls……typical OCD case. Ask quickly about
general health and quick system review coz the patient comes for health check.
Explain what is OCD. Ddx : thyroid, generalized anxiety disorder, head injury , substance abuse
but not in your case

Case (30/5/2017)
MSE examination on OCD pt but they play video ) so am not sure if i mentioned everything.
they gave very long hx which i couldnt know wt was it finaly in the tasks they wrote u will watch
video for 4 mins ( which u cannt pause or repeat it ) then u have to do mse and present to the
examiner.

Note: u have no pt inside the room.

i comment of the video someone was doing mse on that guy so u have to be very clearly
mentioning wt r the positive things. don’t add anything for example( they asked him only if he
see or hear voices. so when i was commenting i dont know how i said no tactile hallu as welll
then i rememberd they didnt ask. so i just said sorry i cannt comment on this cause pt was not
asked for, and i noticed when i mentioned that examiner give some expression

Feedback comments
2. A female nurse has been dialoged with obsessive compulsive disorder. She wash hands many
time due to worry about contamination germs.
Task: Taking history
Management
Again the role player look like the real patient, she wears gloves.
My opening questions: how can I help you? She tell me about her worry of contamination germs
from hospital, she so anxious and can’t do anything including her work as a nurse and house
work.
Then I asked about depression, HEADSS, anxiety, social, family and SAD questions.
Then explain about the condition and management: CBT, SSRI and family meeting.
The question from the role player: Doctor, do you think I’m crazy? NO NO…then I explain again
about her condition call OCD which mean that…pla…pla...preoccupies…..
Bell ring….. I thank both of them but the examiner looked unhappy and says nothing….
Please refer to other recall. I fail this station for no reason, no idea……..

AMC Feedback: Obsessive Compulsive Disorder (Failed)

Young lady presented with ulcers in both hands for several weeks. One of your colloquy diagnose
that she is having obsessive compulsory disorder. She has come to your clinic today for taking
treatment. Manage the patient.
Hx
She is a bit anxious person.
She is a perfectionist.
She thinks her hands are dirty. She washes her hands frequently. No particular incident
happened before this.
She does not touch her two children as she is thinking she is dirty. Her husband looks after her
children and he goes to work also.
She is a nurse but she does not go to work these days as she is having ulcers and she thinks that
she would infect others.
She does not have hallucinations or delusions. no command hallucinations. No past Hx of any
psych diseases.
She is not depressed. No idea to harm others or harm herself. Her sleep and appetite are
normal.
Her main complaint is that she can’t go to work.

Mx
I explained about OCD. Referred her to psychologist and psychiatrist for her psychological
problem and referred her to dermatologist to treat her skin condition that was a result of OCD.
Ask to use moisturising cream to prevent dryness due to excessive washing. I explained about
CBT and medication (SSRI).
I issued a medical certificate and discus the matters with her working authority.
I forget to arrange social worker to look after her family and it is better to provide some respite
care to her husband to give some relief.
I failed this station. Please see others recalls.
This is my first station!! ( station no 7)
A nurse who claimed that she got some warts on her hands from patients . The warts
has disappeared already now. She washes her hands many times a day and has problems at work
since she is worried that she can get infections from patients. She has been diagnosed with
Obsessive compulsive disorder by your colleague already.
Tasks :
Take history
Address her concerns regarding her work
Management

I think I probably didn’t do well since this was my first station and not too familiar with OCD
counseling.
What I was thinking during 2 mins:
I need to ask a complete Psyh questions including suicidal risk even though the diagnosis
has been given as OCD.
8 mins performance:
RP was wearing gloves and also shaky.
I assured about confidentiality first .

History:
She has been having the problem for some months now.
She is afraid of getting infections from patients , so washes her hands multiple times.
The condition is getting worse. She has problems at work because of her concerns of getting
infection.
Me: Do you still have those warts on your hand? Can I see?
RP: No , it’s gone.
Me: how is ur mood?
RP: ok
Me: do u still enjoy the things u used to before?
RP: yes
Me: Can you hear strange voices and see strange things when no one is around?
RP: No
Me: have u ever thought of harming or killing yourself ?
RP: No
Me: Do u worry about everything?
RP: No , just worry about getting infection from patients.
Me: SAD?
RP: No
Me: any medical sickness?
RP: no
HEADSS questions : home situation is ok , has problems working as a nurse.

Then I told her that she has a condition called Obsessive Compulsive Disorder.
Body and mild are connected to each other ( I drew two overlapping circles). You have some
recurring thoughts due to anxiety in unconscious mind. To get rid of those thoughts , u do some
rituals like hand washings multiple times.
I want to refer u to a psychologist or psychiatrics for CBT in which the specialist will try to assess
your negative thoughts and replace them with the positives ones.
You may need some medications if it doesn’t work. ( I didn’t mention the name which is SSRIs ).
I realized that the RP kept asking about what can be done for her work.
I said , with your consent I would like to talk to your supervisor and have a meeting to try to
solve your concerns at work. I will also give you medical certificate and you should go on
vacations to relax yourself . You can also do some exercises and meditation to relieve your
anxiety. I also assure that you will be fine after appropriate treatment and relaxation , and told
her that I appreciate that you r doing very good job for the patients as a nurse and you will not
easily get the infections as you r helping sick people for their good.
I will review you regularly also.
No question from the examiner.
Finished on time.
AMC Feedback : Obsessive-Compulsive Disorder ( Failed )
(I failed in this station , probably because I couldn’t properly convince RP about her work.)

30 yo nurse came to you where you work as a GP. She was seen by your colleague a week ago.
She has repetitive intrusive and irresistible thoughts of washing her hands very frequently
because she thinks her hands might be contaminated and transmits infection to patients or
family member. PE and invs were done and she was diagnosed as having OCD (they give the
definition of OCD in the stem). She is feeling anxious and frustrated and now thinks whether she
can have some time off from work.
Hx, Management plan.

The RP was quite a nice lady, wearing rubber glove. she felt very anxious and frustrated about
this problem and as a nurse, she was afraid she might transmit infection to the patients. she
couldn’t help thinking about that and keep on washing her hands several time and keep on
wearing the gloves.
All the other psych Q, she responded No. Insight (+), Judgement (+), No suicidal idea, lives with
family and husband’s very supportive and concerns about her problems, no financial problems
Then, I started to explain about the management plan. (I think I did not explain the condition
well this case that’s why I failed)
For the management, I’d like you to relax and reduce the stress. Also, I’d like to refer you to
psychologist and do some CBT which is a talking therapy and he’ll also do special procedure
called exposure and response therapy. Then, the RP asked me what is that. I told her that he will
create a simulative environment and make you exposed to the situation where you would like to
wash your hands very much and he’ll help you to get over that situation. Also, I will regularly
follow you up and if your condition is not responding well, I can also refer you to the psychiatrist
to start you on antidepressant, SSRI.

-I had this case in my exam and role player was repeatedly asking me if she is required to see the
psychiatrist as it mentioned in stem that my colleague had referred her to psychiatrist. Because
she was so upset about the psychiatrist I mentioned I will refer her to psychologist first then later
to psychiatrist... I didnt get through this case. Probably I had to convince her that she must see
the psychiatrist as my colleague had made the referral and that was done in her best interest

-I asked all questions of ocd , her insight into prob,complete psychosocial hx , work related prob
sadma. then reffered her to psychologist for exposure n response therapy. explain her what is
this kind of cbt and how it ll work ,told her be calm dont bottle up things. relaxation tech,
offered social worker financial help if she want, lil bit family hx. offered her time off work for
some time n also told if prob wont get setteled with all these then ll refer you to psychiatrist who
will prescribe u ssri n anxiolytics which would definitly help n other LSM

206-Post traumatic stress disorder


Case 1 (SOB sample case)
28 years old Mike came to GP clinic with complaint of SOB. When checked his records, he had
MVA 4 months ago following which he had multiple ribs fractures on the left side for which a
surgery and drain needed to be done.
Tasks
-further history
-counsel patient accordingly

History
1-SOB questions
-do you feel SOB now? (No)
-check hemodynamic stability
-for how long? (past 2 months but now concerned)
-constant or come and go?
-associated with activity or even at rest? (at rest and activity)
-any relieving factors?
-do you feel SOB on lying down? Dow you wake up at night SOB? How many pillows?

2-Associated symptoms
-fever, coughs and cold
-chest pain, palpitation, sweating

3-past questions
-travel history
-asthma (key)
-allergies

4-accident questions
-confidentiality
-can you describe the accident for me? (I don’t want to remember)
-if somebody lost their life during the accident? (No)
-I see you had rib fractures so have you had regular follow ups? (Yes)

5-PTSD questions
-any flashbacks or recollection of events? Any nightmares? (yes)
-avoidance of similar situation
*have you started driving up to the accident site? (yes)
*do you find it hard to go to the accident site? (yes)
-hypervigilant state
*when you go out on the road do you always anticipate an accident? (Yes)
-since how long have these symptoms been going on? (2 months)

6-all psych questions


mood questions (worried, no suicide idea, good appetite, no weight loss, difficulty getting to
sleep and sleep disturbed by nightmares)
-no delusion or hallucinations. Poor insight and good judgment

7-HEADS
-home: living with who, support, stress (supportive partner and no stress)
-work: have you started working, any financial problems (no started, on pensions)
-anhedonia
-SAD (taking alcohol; 2 bottles beer a day, not smoking and not taking drugs)
-social activities

8-General: PMH, PSH, medications, FHx

Explanation
-your condition could be due to PTSD (not delayed here), it is an anxiety disorder now included
under trauma and stress related disorder commonly experienced by people who had gone
through or witnessed a severe distressing psychological events, like MVA you had with near
death experience.
-in this condition, you start getting distressing recollection of events in the form of flashbacks
and nightmares which could be recurrent and you tried to avoid similar situations.
-why you become SOB this time because mind and body work as a single axis so whenever you
have severe emotional disturbances, it can affect your body in different ways with different
physical symptoms.

Management
-I need to do a full examination and also some Ixs just to make sure you have no underlying
medical conditions.
-you require a referral to a psychologist for CBT called trauma based psychotherapy (talk therapy
in which you will gradually been exposed to your memories and will be taught ways to overcome
the disturbances associated with it.
-it is better to have a lifestyle modifications as well; Healthy balanced diet, regular exercise,
relaxation technique (yoga and meditation)
-your drinking habit Is more than self limits so it is important to keep it to standard less than 2
SD/ day
-sleep hygiene techniques
-with consent arrange family meeting with partner.
-PTSD support groups
-if these doesn’t work then require referral to psychiatrist who might start you on medications to
help you get over it.
-reading materials
-review in 3 weeks’ time.

CASE 2 (PTSD sleep recall)


patient presented to your GP clinic complaining of loss of sleep and tiredness
Tasks
-history
-diagnosis and Ddx

History
1-sleep questions
-for how long have you had sleeping problems? (6 months)
-do you find it difficult getting to sleep or you wake up early in the morning and find it difficulty
getting back to sleep?
-any nightmares or vivid dreams? (yes)
*can you tell me more about these nightmares? (events happened in the past)
*what happened in the past? (My comrade was killed in Afghanistan)
*sorry about that and when did this happen (10 years ago)
-are you taking any sleeping pills?
-is this the first time?
2-PTSD questions
-any flashbacks or recollection of events? (Yes)
-do you avoid all things related to wars?
3-Tiredness questions to rule out organic causes ddx (HEMIFADS)
-weather preference and bowel motions
-feeling thirsty, passing large amounts of urine
-LOW, LOA, lumps or bumps
-fever
-dizziness, palpitations
4-Psychosocial History
Mood
-how is your mood in general?
-have you ever thought of harming yourself or others?
Delusion and hallucination
-just a routine questions I usually ask do you usually feel, hear, see things that others do not?
-do you think someone is trying to harm you or spying on you?
Insight and judgement
-do you think you need a professional help?
-if we supposed that there is a fire in this room so what would you do?
5-other ddx questions
OCD
-do you have recurrent thoughts you have trouble controlling?
-do you find you have to do things repetitively such as washing hands? (Only if the first yes)
Phobia
-do you feel uncomfortable in crowded places?
GAD
-do you worry excessively about things or minor thigs? Do you have trouble relaxing?
6-HEADS.
Home
-who do you live with?
-do you have enough support?
-any stress at home?
Work
-what do you do for living?
-any stress at work?
-any difficulty concentrating at work?
Anhedonia
-have you lost interest in things you used to enjoy?
SAD
Social activity

7-General questions
-have you had any past or family history of mental problems?
-any recent infections, head injury or thyroid problems?
-how is your periods?

Explaining diagnosis
-Based on the history, most probably, you have a condition called delayed PTSD. It’s an anxiety
disorder, now it’s included under the trauma and stress related disorder commonly experienced
by people who had gone through or witnessed a severe distressing psychological events which in
your case is the witnessing your comrade killed 10 years ago.
-in this condition you start getting distressing recollection of the events in the form of flashbacks
and nightmares, which could be recurrent and you tried to avoid anything related to this event.
We call it delayed if you have got these symptoms any time after 6 months of the traumatic
event.
-why you have poor sleep and feel tired this time because mind and body work as a single axis so
whenever you have severe emotional disturbances, it can affect your body in different ways with
different physical symptoms.

Differential diagnosis (and explain why less likely)


-depression
-GAD
-adjustment
-phobia
-thyroid, infection, anemia
-substance abuse

we need to do some Ix to rule out organic causes.


Feedback 23-6-2018
Travel problem: very short stem with loss of sleep, and tired. Take Hx, DDx.
Hx: how long? 6 months, what set it off, progressive, what make it worse, better?
What do you mean by loss of sleep? How many hours? Do drink coffee? Any nightmares (Yes).
Psychological Hx: Tell me about your nightmares?  Yes, she said about events happened in the past
flashing back  could you tell me what happened back then? My commerade was killed in Afganistan 10
years ago….

 MSE hx
Tired hx to rule out: anemia, DM, Hyperthyroidism, polyralgia Rheumatica, chronic infection, infectious
endocarditis,…  Normal SADMA: smoking and alcohol  arrange appointments for those. Family, Friend,
Financial.

Dx: Delayed PTSD  Explain to the pt.


DDx: Depression, other organic cause we need to do investigations to rule out.

Grade: PASS, GS: 5


Key steps 1,2,3,4: Y (all covered)
Hx: 5, DDx:4

Recall 15-3-2018
GP, pt complaining of lack of sleep and irritable.
Tasks:
history,
give dx and ddx
(ptsd. Have been as a strategist in pakistan and experienced very bad, now avoiding all things concerning
abt war, flash backs and vivd dreams, lack of sleep for 6 mths, as a result very edgy and quarrel witb
husband, now working as a financial analyst, also some problems at work too, no depr3ssion, no thyroid,
no phaeochromocytom, lmp last week).

Feedback 7-12-2018
50 years old lady complains of not sleeping
Task : history 6min, D/ddx
2 min thinking : could be anything depression, heart problem ,GERD, anxiety, PTSD,.
i entered in greeted both of them, patient was very cooperative, without asking anything she told
complete story typical of PTSD.( like history of military veteran left job, now working in accountancy now,
satisfied with new job , but recently getting flask backs and night mares, irritable and also edgy , husband
not happy because of her behaviour).
excluded organic dd like GERD,heart condition and Thyroid issues-nil
past history -no mental disorders
ASEPTIC-not depressed, no hallucination, only anxious , excluded- GAD,panic attacks, did not ask
cognition and insight(ask very cooperative pt, knows whats her problem)
no smoking,alcohol-occasional.(SADMA)

explained her they could be many reasons for sleeplessness ,but mostly of her history it could be PTSD,
other dd- GAD, Depression, Panic attacks ,GERD, thyroid problems, and all .
scenario:travel problem

Global 7 (approach 6, history 7, d/ddx-6)


Feedback 2018
Travel problem

43 year old woman having sleep problems and irritable after Turkey trip. Typical PTSD case.
Task: History (6 mins)
Dx and DDx

2 min thinking: PTSD; Depression; GAD; perimenopause; thyroid; pheochromocytoma; OSA

History
-Hello, how can I help you today? “well, I cant sleep well at night doc”.
-Symptom analysis – duration (7 months), every night? (almost every night) by sleep problem, do u mean
you cant fall asleep or is it because you have been waking up from your sleep in midnight or in early
morning? (mixed) having nightmares (YES! About her past experiences when she served in military in
Afghanistan in her 20s. Went back to Turkey 7 mths back where a war memorial was held. Since then,
having nightmares). I m sry you have been re-experiencing these again. How have you been coping since
you returned from Afghanistan? (pretty well, no nightmares before 7 mths back)
So, how have you been feeling in yourself these days? (irritable and started explaining about how lack of
sleep has been affecting her routines; that her husband is quite patient and understanding)
-HEADDSS
-SADMA – clear; no psychotic symptoms; excluded other ddx. Period is normal.

Dx and DDx
Just like I mentioned above

GS – 6
Key steps 1, 2, 3, 4 – YES
Approach – 5
History – 6
Dx and DDx - 6
207-Normal pregnancy examination (by Riley Harisson)

GP practice, next patient 37 weeks of gestation, she has done all her antenatal care and
everything has been well. She is coming this time for a follow up consultation. It was more
information but everything was normal.

Tasks :

Perform PE to the mannequin describing to the examiner what are you doing.
The checklist will be like this

1. WIPEC (Wash hands, Introduce, position, exposure, consent)

2. GA and vitals

3. Face

4. Local examination (obstetric examination)

- Inspection

- SFH

- Palpation

WIPEC as usual and wear the gloves. You should say “Today, I need to examine your tummy to
assess your pregnancy. That will involve me looking and feeling the tummy and perform some
maneuvouers. It won’t be painful. If you feel any discomfort, please let me know but I will try to
be gentle as much as I can.

position is lying position


Exposure should be xiphisternum to pubic symphysis
In this case, ask the patient to empty the bladder for correct assessment.
General appearance –The patient is comfortable or not, also mention that you want to check
height, weight and vitals of the patient.

Then in the face, check for any conjunctival pallor for anaemia, Jaundice ( there may be obstetric
cholestasis) and facial oedema.

On inspection, we should check


Shape of the abdomen – oval transversely? longitudinally? (clue for fetal lie)
Visible fetal movements
Any previous scars
Linear nigra
striae gravidarum
Palpation
Light palpation should be done on all nine regions of the abdomen
Then palpate the uterus with ulnar border of your hand and mention the estimated week size.

Then measure the SFH. We have to measure between the fundus of the uterus and pubic
symphysis. Place the tape facing down to avoid bias, only turn it back when the position is
confirmed and noted with finger. This should be correlated with age in weeks. (+/- 2cm)

Then palpate the sides of the uterus to check for fetal lie. Mention the findings. (this is also
called lateral grip in obstetric usage)

On lateral grip of the left/right side, hard, broad, full surface is felt which is likely to be fetal
back/ multiple small knots which can be fetal limbs.

Then you know the types of fetal lie, Longitudinal/transverse/oblique


Then we need to check for presentation.
Warn the mother that this may feel uncomfortable.

Place your hands on either pole of the uterus and feel for the presenting part.
Mention what you found “small globular hard part which can be fetal head” (
or “ soft broad part which is fetal buttock” is felt.
So the presentation is cephalic/ breech presentation.
Place your hands on either pole of the uterus and feel for the presenting part. If we can feel the
whole head above the brim, we can say that it is five fifth palpable. (totally not engaged yet)

If the head cannot be felt abdominally, it is zero fifth palpable which means fully engaged.

Then we listen to the fetal heart sound with a trumpet if provided.(it is usually heard at the side
of the fetal back) or just mention it that you want to check it with a Doppler.

As the bedside test, we offer urine dipstick test.


Then cover and thank the patient.
Tell everything is normal.
Feedback9-5-2018 No scored
Normal pregnancy PE.
GP practice, next patient 37 weeks of gestation, she has done all her antenatal care and
everything has been well. She is coming this time for a follow up consultation. It was more
information but everything was normal I can´t remember more about the stem.
Task
Perform PE to the doll describing to the examiner what are you doing.
Please put gloves to protect the doll.

I went in and it was a doll on the bed just half chest and abdomen covered with a white blanket.
There was a trolley on the side of the bed with the tape measuring meter and a Doppler. I
started a complete PE and then focus on the abdomen examination for pregnancy. I explained
how to take the FH and it was 37 so I said it matched with a gestation of 37 weeks so it is
normal. I did the maneuvers to find out position and presentation of the baby. For me the baby
was cephalic and dorsum to the right of the mum, FHR was present 140 normal, I asked if any
contraction and examiner said no. I knew I had plenty of time so I said in the case of a history of
vaginal bleeding or discharge I would perform a vaginal examination with the consent of the
patient and the examiner said no need it.
I said this is a normal physical examination. I think I finished very quickly and examiner said you
can sit and you can add any information if you would like to do.
208-Urinary Incontinence
(Materials)
Tamara, 68 years old, presents to you at your GP clinic with complaints of inability to hold urine
for the past 6 months. She is quite worried about this and wants you to help her.
TASKS
1. Take a further history
2. PE from examiner
3. Investigations with the examiner
4. Diagnosis and Management with the patient
Types of Incontinence:
 Stress incontinence: happens due to weakness of the sphincter
 Urge incontinence: happens due to detrusor muscle irritability and small bladder
capacity
 Mixed incontinence: combination or urge and stress incontinence
Causes
 Rise in the intraabdominal pressure
o Cough
o Constipation
 Bad obstetric history
 Menopause
 Urinary tract infection
 Medications (cholinergic, tranquilizers, diuretics)
 Bladder irritants (smoking, alcohol, recreational drugs, excessive coffee)

Positive points in the history: leakage of urine for the past 6 months, leaks a large amount of
urine before I go to the toilet, feels stressed when I go out because I feel that I might pass urine,
leaks a small amount urine during coughing, sneezing, gets up around 2-3 times at night to go to
the loo, fears that she might wet her bed, had menopausal symptoms for 1 year then it went
away, has bad obstetric history
Positive points in the PE/Investigations: BP 110/80, Temp 36.7, PR 86 regular, RR 18, 99% sats;
BMI is 30, can see leakage of urine on cough test (stress incontinence)

History
1-Incontinence Questions (How long, how often, how much)
-Reassurance
-How long have you been experiencing the leakage of urine?
-How often?
-How much of urine do you leak at that time?
2-DDX questions
stress incontinence
-When do you experience this? Is it when you cough, sneeze, laugh or do exercise?

Urge incontinence
-Once you feel the urge to go to the loo, can you hold the urine back?
-Does urine leak out before you reach the loo?
-At night, do you need to get up quite often to go to the loo?
Prolapse
Any sensation of something protruding from down below?

3-past medical history


UTI
-How is your water works? Any burning or stinging while passing urine? Do you have any fever?

Constipation
How are your bowels? Do you have any constipation?

Chronic cough
-Any chronic cough that you are having?

4-Past surgical history


-have you ahd any surgical procedures or operations? (hysterectomy)

5-past obstetric history (pregnancy)


-How were your past pregnancies and deliveries? How many have you had?

-Any history of prolonged labor, obstructed labor, big babies, or any instrumental delivery?

6-past gynaecological history (menopause)


-When did you have your menopause? Any mood changes, heavy sweating, hot flushes?
-Did you take any HRT?

-When did you last have your pap smear?

-When did you last have your mammogram?

7-Social

SADC
-Do you smoke, drink alcohol, take recreational drugs?
-How much coffee do you consume in a day?

Sexual history

-Are you still sexually active?

-Do you have a good support?

Medications

Do you take any medication? Any allergies to any medications?

Diet and weight

8-how does it affect your life?


Physical Exam
1-General appearance: BMI, PICCCLED
2-Vital signs: temperature
3-CVS, Respi, CNS
4-Abdomen: visible mass/distention, tenderness, palpate for mass tenderness
5-Pelvic examination
-Inspection of the vulva and vagina: any bleed, visible discharge, prolapse, atrophic vagina.
-Speculum: cervix is healthy or not, discharge or bleed from the cervix, Cough test: after placing
a pad I would like to ask the patient to cough; any leakage of urine from the urethra, any
prolapse that you can see?
-Per vaginal exam: uterine size and tenderness, adnexal mass and tenderness
6-Office test: UDT, BSL

Investigations
1-Blood tests: FBE, UEC, ESR/CRP, LFT, urine MCS (check for asymptomatic bacteriuria)

2-Refer to urogynecologist for Urodynamic studies (check for urge incontinence)

Has 4 parts: (all 4 parts needs to be done in urge and mixed; only post void in stress
incontinence)

-Urethrocystoscopy: look at the urethra and bladder to look for any polyps and other lesions that
might irritate the bladder
-Uroflowmetry: measure the amount of urine that is passed and speed that it is passed

-Cystometry: look at bladder capacity, how full the bladder is when she feels the urge to void

-Post void residual volume: ask the patient to void and then view the bladder (most important
part of the urodynamic studies) If greater than 50mL, it is severe

note/ no need to talk about these urodynamic studies in details

Diagnosis

-Most likely you are having a mixed incontinence. It is a combination of stress and urge
incontinence.
Stress incontinence is leakage of urine when there is a sharp rise in the tummy pressure, usually
happens when you cough, sneeze, or when you strain during bowel movement, and during
exercise.
Urge incontinence is a sudden strong urge to urinate due to an overactive bladder, where the
bladder muscles contract at wrong times.

-The most important cause for incontinence is a pelvic muscle floor weakness. Pelvic floor
muscles and ligaments support the bladder and the urethra along with the uterus and the
vagina.
Pelvic floor muscle weakness can happen due to several reasons:
One is gaining extra weight or overweight, when the pelvic muscles starts becoming weak;

Second the a bad obstetric history, which causes stretching and damage to the pelvic floor
muscles;

But the major contributing factor is menopause. The estrogen or the female hormone is
necessary to maintain the stability and strength of the pelvic floor muscles. At the time of
menopause, the ovaries will shut down, so that very low levels of estrogen is being formed. This
estrogen deficiency contributes to pelvic muscle floor weakness.

Treatment
1-First step of management is lifestyle modifications.
-Your BMI is a bit high, so you need to bring down your weight to a more optimum level. I will
refer you to the dietician who can help you with a diet plan appropriate for your weight goals.
-Adopt a regular physical exercise as well.
-(if patient has cough and constipation, taking excessive coffee, smoking, alcohol, fizzy drinks,
address it here as well)

2-Maintain a bladder diary and record in it the times that you are having incontinence so that
we can correctly identify the type of incontinence and see whether the treatment is effective.

If it is a stress incontinence:
-Do pelvic muscle floor exercises (Kegel exercise). I will refer you to a physiotherapist who can
better advise you regarding that.

You can do the exercise in a sitting, standing or lying down position. Contract your bottom
muscles, for a count of 8, relax it for the same amount of time that you have contracted the
muscles, or count of 8 also, and do it 8 times at a go, three times a day.

-If conservative measures fail (or if there is no improvement in 6 months' time), I can refer you
to the surgeon who might do several options:

*a sling surgery via keyhole surgery. A U-shaped mesh tape in the form of a sling will be installed
to support the bladder and the urethra.

*Urethra-bulking agents. Substances like collagen will be injected into the bladder neck to make
it tighter and stronger.

*Colpo suspension. We use stitches to suspend or lift up the neck of the bladder and urethra,
and attach those to the pubic bone.
If it is an urge incontinence:
-Along with the pelvic muscle floor exercises, you have to do bladder re-training exercise. When
you feel the urge to go to the toilet, try to hold it back, initially for 5 minutes, then gradually
increase the span of time before you go to the toilet. Once you reach the toilet and start passing
the urine, try to hold back the urine at times, and then restart again. (Goal is to try to increase
the urine volume capacity of the bladder, and decrease the irritation of the detrusor muscle)

-Botox injections in the bladder wall can decrease the spasms of the bladder wall and thereby
can improve the urge incontinence.

-[surgery for urge incontinence is rarely done; it is called an augmentation cystoplasty: attach a
loop of bowel to the bladder to increase bladder capacity, but it is a complicated surgery that is
why it is not widely done]

Key point is to start the patient on medications (antimuscarinic agents)


Oxybutynin

Propantheline bromide

Initially for 1 month and if there is definite improvement, continue for 6 months.

If it is a mixed incontinence,
it is a mix of the management for stress and urge incontinence.

This is recall of 6-4-2018


209-Uterovaginal prolapse (Materials)
56 year old Maya comes to your GP complaining of a feeling of something bulging from down
below. She appears really concerned about this.
TASKS
1. Focused relevant history
2. Examination findings from examiner
3. Management

History
1-Prolapse questions + incontinence
-How long have you been having this? (Last 1 year)
-Is it there always, or just at times? (Assess degree of prolapse. If it is always there, it is 3rd
degree) (it is just there at times)
-When do you have the feeling of the bulge? (When she stands for a long time)
-What relieves you of the feeling? (When she lies down, she can feel the bulge going inside)
(Probable 2nd degree prolapse)
-Associated symptoms: bleeding, discharge, itching, rash? (Prolapse can get infected)
-any involuntary leakage of urine? (Incontinence)
2-past medical history
Chronic cough:
-any chronic cough?
Constipation
-how is your bowel motions?
-Any history of constipation? Do you open your bowels regularly?
UTI
ask about waterworks as well (can be associated with a cystocele or urethrocele, can have
urinary retention)
-any problems with passing urine?
-Is there any burning or stinging during urination?
3-Past surgical history (hysterectomy)
4-past obstetrical history:
-have you been pregnant before? how many pregnancies have you had? (3 pregnancies and 3
deliveries),
-any complications during the pregnancy?
-Were all your deliveries vaginal? Any C-sections?
-Any history of big babies? Any obstructed labor? Any prolonged labor?
-Any tears or lacerations that you had at the time of delivery?
-Any instrumental deliveries?
5-Past gynaecological history (Menopause)
-period history: have you had your menopause?
-Any menopausal symptoms like hot flashes, heavy sweating, mood changes?
-are you on any hormones or HRT?
-Any bleed after menopause?
-When was your last pap smear and mammogram? What was the result?

6-Social
-Sexual history: are you sexually active? Are you in a stable relationship? Any problems with
intercourse? Any bleed after intercourse?
-Do you smoke? How long have you been smoking? How many sticks per day?
-Alcohol drinking?
-medications and allergies

Physical Examination
1-General appearance: BMI (35, obese - can lead to weakness of the muscles), pallor
2-Vital signs
3-Systemic exam
Respiratory system (chronic cough is a cause) : air entry, adventitious sounds
CVS: normal S1 and S2, murmurs
Abdomen
Visible mass, distention?
Palpate for mass and tenderness (abdomen is soft and non-tender)
4-Pelvic exam
Inspection of the vulva and vagina:
-any discharge/bleed? Any prolapse? Any thin, dry atrophic vagina?
Speculum exam:
-is it healthy? Any discharge or bleed?
-Key point: ask the patient to cough or bear down
-Ask if there is a prolapse (cervix is coming up to the level of the introitus)
-Identify the degree of prolapse
+First degree: remains inside the vagina
+Second degree: cervix comes at or near the introitus
+Third degree: most of the uterus and cervix lie outside the vagina
-Ask for any incontinence that you can see (pelvic floor muscle weakness can result into a
prolapse or incontinence)
*always rule out incontinence in prolapse, always rule out prolapse in incontinence
Bimanual exam
-Uterine size (size is normal), tenderness
-Adnexal mass or tenderness
5-Office test: UDT, BSL
Management
-The condition you are having is a uterovaginal prolapse. It is a common condition where the
cervix or the uterus or both bulge into the vagina. For you it is a second degree prolapse as it is
not coming outside the vagina.
-The cause of a uterovaginal prolapse, is due to the weakness of the pelvic floor muscles and
ligaments, that support the pelvic structures like the uterus and the cervix.
There could be a lot of reasons for pelvic muscle floor weakness, like chronic cough,
constipation, but as far as your case goes, I can see that you are having a bad obstetric history.
So at the time of pregnancies, due to the extra weight gain, and the hormonal changes, these
muscles can become weak, and at the time of labor, when you push or strain, these muscles
become more weak. One of the deliveries that you had is a big baby, that is why you had a
prolonged labor due to that, and that might have contributed to your prolapse.

-Why didn't I had it then? Why am I presenting with the prolapse now? The contributing factors
for prolapse, is menopause and the extra weight gain. At the time of menopause, as the ovaries
shut down, estrogen is formed in very low levels. And this estrogen is necessary, to maintain the
strength, and the stability of the pelvic floor muscles.

-You need to be referred to a specialist.

-Start with lifestyle modifications. Your BMI is a bit concerning, so I would like to refer you to a
dietician who could give proper advice regarding your diet.
You need to have regular exercise, and maintain the BMI within the ideal range.

-You need to also do pelvic muscle exercises. I can refer you to a physiotherapist who could
advise you about this.
How do you do the exercise? You can do the exercise in a sitting, standing or lying down position.
Contract your bottom muscles, for a count of 8, relax it for the same amount of time that you
have contracted the muscles, or count of 8 also, and do it 8 times at a go, three times a day.
How long does the pelvic exercise take to work? Usually it takes around 3-6 months.

-Another option is a vaginal pessary, a device inserted into vagina. It will help to keep the
prolapse in place and prevent it from getting worse. But it is not a definitive treatment of
prolapse. You need to change it every 3-4 months.

-Surgical management/option is considered if the conservative management fails and if the


symptoms becomes worse. One is repair procedure and it is of two types. One is called
colporrhaphy where you reinforce the pelvic floor muscles by using stitches. Another type is you
put in a synthetic graft thereby strengthening the pelvic floor muscles.
The next procedure is a sacrocolpopexy. This is where you re-suspend the vagina, cervix and
other pelvic organs and secure it to the back bone or the sacrum.
The last option is a total hysterectomy.
***Another case is the patient has undergone hysterectomy and has prolapse: vaginal vault
prolapse. AMC EXAM CASE
Causes of vaginal vault prolapse:
Weakening of the suspension: due to extra weight gain, chronic cough, constipation
Management:
Referral to the specialist, lifestyle modifications (usually BMI will be high), Pelvic floor exercises,
vaginal pessary (but not a definitive treatment),
Surgery: sacrocolpopexy - resuspend the vagina and secure that to the sacrum

Management in summary
1-refer to gynaecologist for pessaries or surgery
2-lifestyle modifications (dietician, SNAP, physical exercise)

Karin case Uterine Prolapse


Case: An a 80-year-old lady comes to your GP clinic complaining of mass protruding down below
and rash around the private area for several months.
Task
a. Relevant history
b. Physical examination (BMI 29, maculopapular rash around introitus and inside of thigh, urine
dipstick + sugar, BSL 11.3mmol/L
c. Diagnosis and management
History
- I read from your notes that you have something bulging from your private area. Since when?
Can you tell how it happened? Is it increasing?
- Do you feel any abdominal discomfort?
- What is the effect of this bulging on your life?
- Is this swelling affecting your waterworks? Do you leak urine while you strain, cough, etc? Do
you have a strong urge to void on the way to the toilet or do you leak a large amount of urine on
the way to the toilet?
- Any discharge down below?
- Constipation? Waterworks?
- - Rash? Since when? Is it itchy? Can you describe the rash for me?
- Period: When was your last period? Any irregular bleeding after that? Hot flushes? Mood
swings? Breast pain? Irritable?
- Pregnancy: how many pregnancies? Were they big babies? Did you have any difficult labor or
prolonged labor?
- Partner: are you sexually active? Do you have a stable partner? Do you have painful
intercourse? Have you or your partner ever been diagnosed with STDs?
- Pap smear: When was your last pap smear? Result?
- Mammography?
- Past medical history: chronic cough, diabetes, asthma
- FHx: Osteoporosis, MI
- SADMA

Physical Examination
- General appearance
- Vital signs
- Abdomen
- Pelvic examination:
o Inspection for morphology of the rash (maculopapular rash around the introitus and groin
area), scratch marks, discharge, obvious bulge
o Speculum: wall of vagina, rash, discharge, blood, ask patient to cough (cervix comes up to the
introitus), leakage of urine, cervix
o PV: adnexal masses, CMT.
- PR: differentiate between cystocele and rectocele
- BSL and Urine dipstick

Diagnosis and Management


- You have a condition called uterovaginal prolapse with stress incontinence and candidiasis,
- Menopause resulting to lack of estrogen, difficult labor, big babies and constipation leads to the
laxity of the pelvic floor ligaments. It is a common condition among females in your age group.
- At this stage, I would like to refer you to the gynecologist. I would advise you to start with pelvic
floor exercises (contract pelvic floor muscles as if trying to hold urine).
- The specialist might insert a pessary which is a device inserted into the vagina to support the
uterus. They need to be changed every 3-6 months. They also advise topical estrogen to improve
the discomfort.
- Will it affect intercourse? Pessaries will not interfere with your sexual performance.
- If conservative measures do not work, the specialist might consider doing surgery to fix the
ligaments.
- How long will I be in the hospital? Usually 3-5 days. You can go home once you’re feeling well
and once you have started urinating without problems.
- Postop advice: For the first two weeks, restrict your activities. Rest. Avoid heavy lifting. Avoid
sports and swimming. For 1st 6 weeks abstain from sexual intercourse.
- Driving: It is not advisable to drive for the first 2 weeks.
- Complications: Pain, bleeding, injury to nearby structures, anesthesia complications
- For the candida, I will prescribe you antifungals. It might be related to high blood sugar. I will
give you referral to physician to investigate further
- Lifestyle modification: normal BMI, stop smoking, high-fiber diet

- Referral to specialist obstetrician. Reading material. Review.


- Advise OGTT.

Prolapse:
- I: cervix remains within vagina
- II: cervix comes up to introitus
- III: most of uterus lie outside vagina
Karin case Post-hysterectomy Prolapse
Case: You are a GP and a 52-year-old female comes to your clinic complaining of something
coming out from her vagina especially after straining

Task
a. History
b. Physical examination
c. Management

History
- Please tell me more about your problem? Since when have you noticed this lump? Is it present
all the time or does it come and go? Any changes with change in position like prolonged standing
or lying down?
- Do you have associated tummy pain or heavy/dragging kind of sensation in the lower tummy?

- Any urinary complaints like frequency, burning or leaking of urine? Any loin pain?
- Any history of prolonged cough, constipation, asthma or respiratory problems?
- Do you have any problems emptying the bowels?
- Any complaints of discharge or bleeding from down below?
- Any fever? Itching?
- When was your LMP? Any problems during or after menopause?
- When did you have the hysterectomy? Why did you have it? Any complications afterwards?
- Was it done at a tertiary care center? After the surgery, did you do pelvic floor exercises? Any
other surgeries that I should be aware of? Did you take any HRT afterwards?
- May I know are you sexually active at the moment? Any complaints of pain or discomfort
during sex? How many kids have you had? Any history of big babies? Difficult or instrumental
deliveries?
- SADMA?
- Have you recently noticed weight loss? Change in appetite? Night sweats? Lumps and bumps in
the body? Pap smear? Mammogram?

Physical examination
- General appearance
- Vital signs
- Chest and Lungs
- Abdomen: for tenderness
- Pelvic exam

o inspection: Obvious lump, discharge, ulcer, redness, discharge


o Sterile speculum examination asking the patient to strain looking for any visible lump while
straining; sims left lateral position (knee-chest position)  gradually withdraw while asking a
patient to strain  lump/bulge in the vagina (best way to detect cystocele and rectocele)

- Urine dipstick and BSL


Diagnosis and Management
- Most likely what you have is prolapse of the vaginal wall after hysterectomy. Once the uterus is
removed, the upper part of the vagina loses its anatomical support. Usually, during
hysterectomy, the surgeon will secure the upper part of vagina with the help of ligaments
attached to the backbone and pelvic wall. Some of these ligaments become loose because of: a.
loss of estrogen b. prolonged straining/coughing c. putting on weight.

- This phenomenon is quite common after hysterectomy. Up to 30% of patients might develop
this. It can affect the urinary system leading to frequent recurrent UTIs. It can also affect the wall
of the bowel causing constipation. Sexual functioning may be affected and might cause pain and
discomfort during intercourse.

- The treatment will be tailored according to your wishes, but you will need to see a specialist
gynecologist. The first option is conservative management which includes pessaries along with
pelvic floor exercises. Usually, this suitable for old, females who are not fit for surgeries.

- The second option is the surgical approach. It is called vaginal wall suspension surgery
(sacrocolpopexy). The surgeon will attach the upper part of the vagina to the strong tissues
within the pelvis usually to the lower backbone or sacrum. There are 2 options regarding the
approach: laparoscopic or keyhole surgery OR abdominal approach best decided by the surgeon.
- The recurrence rate after the surgery is very low therefore the surgery is mostly curative.
- Review. Reading material.
- Pelvic floor exercise (kegel): done to strengthen the muscles of the pelvic floor. The exercise can
be done either sitting or lying down. The patient needs to empty the bladder before exercise.
Contract the pelvic muscles, hold contraction for at least 5 seconds, release it slowly and repeat
3-4x and gradually build up duration for up to 10 seconds. She must not contract the abdominal,
thigh or buttock muscles. Exercises must be repeated 3x a day as many times as possible. Results
are usually apparent within 8-10 weeks. Safe to be done during pregnancy.

Case (28/4/2017)
Lump
Post menopausal with lump.examination n tell pt diagnosis.
on history
- lump in private part.
- no other menopausal symptoms.
- no hrt.no incontinence or constipation cough etc.
- on asking tells about hysterectomy few months ago.
- obs history not significant.

Examiner was standing beside me while doing per speculum on dummy.


On cough impulse patient pretends to cough.
After asking examiner tells no cervix.asked vault healthy? Lump is vault prolapse.
Finished examination with per vaginal biannual.explained patient diagnosis.advised
ammo,pay.examiner told management is not the task
Passed
Case (28/4/2017)
52 yr old woman with lump( didnt say where) .
task history, PEFE, DDx. On history she got vaginal lump and did hysterectomy 10 yrs ago. On
examination Vault prolapse.

Case (30/5/2017)
Uterovaginal prolapse with candida plus cyctocele and rectocele and having hight bsl obese (but
she was very thin RP)
hx ,pefe, tell possible dx , i think there was inx as well from the examiner.
- So she had mass down bellow which go up when she lay down and more when she cough or
sneeze.,
- she had no drippling or constipation.
- no hx of chronic cough,
- meanopasue lady.
- not on HrT
- she had 2 pregnancies i asked size of baby was big 4.5 with NVD no instrumenal
- i ask about wtt loss or lumps or bumps non.
- no VB no discharge
not DM and she dont know if she had when i asked her specifically so i said i will adress this later.

- Having no medical condition

About the rash only itchy for long time didnt take anything and she didnt use any new things
have no allergy as well i asked if u have rash else where as well she said no she describe it as
redness itchy only, i think she wsnt smoker or alcoholic

Pe:
well looking normal
Vs
BMI 37.
i asked abdominal wss normal.
then pelvic i asked i would like to do sims speculum in lt lateral so he gave me ut and vaginal wall
come all way out and normal looking cervix so i said it is grade 3 then ask about ant and post
wall and was postive for both.
- then i did urin dipstic was normal , she had high blood sugar level i asked mcu which wss
normal as well dont remember if i asked more than this.
I explain by saying u have all these things and we have to address each thing and u r in safe hand

*57 yo woman with vaginal lump feeling. Hx, PE, DDx, management.
Recall of 9-5-2018
Lump in vagina. Hx, pefe, explain ( hyresctomy 10 yrs ago, 3 vaginal births). ??Vaginal vault
prolapse

Recall of 6-6-2018
Prolapse with Rash in groin, Blood test result given- DM, stem lump coming out of vagina. Rash in groin
area. On PEFE, ant and post wall Vaginal Prolapse.

Recall of 12-7-2018
Vaginal vault prolapse, no signs of incontinence Hx, PEFE, Causes DDx

Recall of 15-8-2018
vaginal prolapse case

Feedback (vaginal vault prolapse) Important


A woman in her 50s (I cannot remember the exact age), comes to ur GP complaining of a lump
going down on and off mainly with sneezing and straining.
Task:
Hx,
ask examiner for examination findings
management.

History
-I do understand that u have been complaining of a lump from the down below? Could u please
tell me more about it? I have this lump on and off for months now but it is getting worse now.
-Is it there only when straining? Yes
-does it disappear by itself? Yes
Here the role player said “I had my womb removed 13 years ago”. Actually she surprised me
when she said that coz I was almost sure it is uterine prolapse. So I continued my history
taking...
-Can I ask why u had ur womb removed? Coz of heavy bleeding.
-Any complication after the operation? No, everything was normal until months ago when I had
this lump.
-Any problem with passing water? No, completely fine.
-Any bleeding? Discharge? No
-Can I ask some personal questions?Yes,please
-r u sexually active? Yes, I am married
-does this lump affecting ur sexual activity? Yes, it makes me feel uncomfortable.
-Any bleeding after sex? No
-Have u had any HRT before? No
-What about ur pap and mammogram? Done regularly and normal
-How is ur health in general? Fineany wt loss or loss of appetite? No
-do u have any cough? Constipation? No
- Do u have children? Yes 2(or three)
-What about the deliveries? All NVD and without any difficulties.
-Do u smoke? No
-Alcohol? Socially
-Some people used recreational drugs, have u ever used them? No
-Any chronic illnesses? No
-Any medication? Allergies? No

Examination
-General appearance? Normal
-BMI? 28
-Vitals? Normal
-Chest and heart? Normal
-Abdomen? Starting by inspection? Normal......Palpation, any masses, tenderness? No
-Vaginal examination after patient’s permission, starting by inspection, any lump? No...
-then by speculum examination, any discharge? No
-Then I would like to ask the pt to strain, any lump? Yes, the vagina comes down...
-Then I would like to use Sim’s speculum in left lateral position and ask the patient to strain
again to look for cystocele or rectocele? Normal
-Urine dipstick and BSL? Normal

Management
-Mrs x, from what u have told me and after examining u, it seems that u have vaginal wall
prolapse (Actually the right thing to say is “Vaginal vaultprolapse” and I knew that but it just did
not come out so I just said wall prolapse.
Also I drew to the patient and explain the condition but please read more about it as I have no
enough information and I am not sure if what I said is correct or not Coz u had ur womb
removed, this can leave the vagina unsupported and with age the ligaments become weak.

This is not serious but with time it can get worse and might lead to ulceration. We have
conservative treatment and surgical....so what treatment do u prefer?(The pt. says she wants to
find a final solution to the problem so I said:)so we need to refer u to a gynaecologist who will
do further assessment and most likely will decide an operation. but life style modification is
also important and u need to lose wt, walking for 30 mins every day, healthy diet( I am not sure
if we need to talk about pelvic floor exercise ,actually I did not have time as the bell rang!!)

AMC feedback –Vault prolapse after hysterectomy (O&G) -- PASSED


Feedback 15-8-2018
Scenario :Lump
Stem: Old aged female came with a “ lump”. It was not mentioned where. ( I thought could be
breast or vaginal)
Tasks:
~History
~pefe
~dx and ddx

Inside the room was an old aged female sitting on chair.


Greeted her and asked her to tell me more.She told about the lump down there.
Lump history:
since when, how started? Getting better worse etc .She said its been few months and it comes
and goes but getting worse with time and feels on strain cough sneeze.
Detailed 5ps 1. Pregnancies: how many kids, last pregnancy, baby sizes, prolonged labour , any
instrument? 2. Pills 3. Partner.:She was not sexually active as far as I remember 4. Periods: She
had her periods stopped few years ago. 5. Pap: don’t remember what she said.
Detailed 4bs 1. Bladder: any leakage? (no)uti (no) 2. Bowel (normal) 3. Breast ( normal) 4. Bone
pain (no)
Then asked about any menopausal symptoms.
Any hx of cough, constipation?
Any surgery.she said yes its been few years my womb has been removed.just a quick history
why was it removed? Any complication? How have u been since apart from this lump any other
complain related to that?
SADMA.

PEFE
Asked in the usual form starting from GA, I remember all was normal then came on pelvic
exam( consent and chaperone)
Inspection: blood, discharge, rash, vesicle , lump (no)
Speculum: cervical health ( normal) vaginal wall thinning, atrophy( no) . on bearing down, do I
see any lump. Examiner said yes. Any urine leakage (no)
Bimanual: dnt remember what examiner said.
Pr: any lump( wanted to ask abt rectocele) but he said no.
Ud and bsl : he told normal .

Gave dx of vaginal vault prolapse after hysterectomy and explained the condition. gave ddx of
rectocele, cystocele, any growth like cancer ( didn’t know much of ddx here)

No management was asked.


Grade: Pass
Feedback 9-5-2018
Lump in vagina. Hx, pefe, explain ( hyresctomy 10 yrs ago, 3 vaginal births). ??Vaginal vault
prolapse-
OUT side- this is the crazy case, when i out side the room, i understand the word" lumb" only
then i assum it is the breast lumb cases as they did not mention where is the lumb. All my dd
turn in to breast cancer, fibroadenoma....I think i got crazy in this case as the word:' lumb'( this is
language barrier)

Inside: the lady sat on the chair then i ask her about the symptoms, after asking a few questions,
i figure out that she got some thing protrude down below.
then i delete all my dd outside the room in my mind and try to find out what is this things.
She is the women so i after asking about presenting complain, I ask about 5P. Then i asked about
nasty growth question, risk factor, combination like menopause, surgery, delivery baby, how big
of baby, consipatient..I though she is really old so I ask about 4B- bone, breast as well. Then i ask
examinor about the PE for prolapse organ.
i told her dd- most likely it is vaginal prolapse , other can be cystocele, rectalcele, nasty
growth... i told her why she got this- menopause, delivery a lot of baby and surgery . i told her
about the refer specialist the exminer stop me and say that there is no management. i draw the
picture and tell her all the thing a gain. Bell rang.

Feedback 6-6-2018
Station 20 UVP, newly dx DM with candidiasis- unscored
? 57 /67 yrs old lady, new patient to your GP, presented with lump and rash( did not mention
the site in the scenario, so I was thinking about infection and haemato differentials)
task:
history,
PEFE card,
explain causes of each of her complaints.

history: site of lump , from her vagina ( so I realize at that time it is UVP or vaginal prolapse case)
coming out on walking or standing, no pain, no rash and ulceration on it, dragging sensation
present, no urinary and bowel symptoms. Causes…3 children, vaginal deliveries, first child need
forceps, no pelvic floor ex, no chronic cough, no constipation, BMI, (forget to ask abt other
postmenopausal symptoms) 5Ps not all.
rash… groin and down below, itchiness, ulcer, bleeding? Not on other area of body
no fever, no lump and bump, LOA,LOW nil. (I have no idea at that time why she has the rash)
No known chronic illness. No previous gyane sx. Smoking , alcohol?

Pefe card: UVP , rash in groin and vulva, RBG 18

Explain: 2 pblms, explain about UVP with diagram and causes of it.
Another thing, you might have DM because RBG is high, (not sure I asked her about DM
symptoms at that time) also the rash could be candidiasis which is strongly associated with DM .
will do further tests to confirm DM. don’t worry they are manageable conditions.
Feedback 9-5-2018 Fail
Vaginal prolapse.
47 year old or older woman comes to see you because a mass (didn’t say where) so during my 2
minutes outside I was thinking about an enlargement of a lymph node.
Task
Take history
PE from examiner

Diagnosis and differentials with reasons to the patient.

Nice lady. When asked why she was coming, she said she has a mass coming from her vagina so
took me for surprise because didn´t say that on the stem outside, any way I think I managed this
case well. History of 3 normal pregnancies, no pain, no fever, no any urinary symptoms I think
she said a bit of dry vagina. Medical history of hysterectomy I don’t remember if I asked why?
Last pap was normal. No any other relevant medical history. PE normal vital signs, didn´t ask
BMI, abdomen normal, genital examination: a mass from anterior wall coming through vagina.

I explained about vaginal prolapse and my differentials where all about prolapses, could be your
bladder or just the wall of the vagina, no your womb because you don’t have it. Didn´t have time
for more.

Failed
210-Headache Somatisation
Case 28-6-2018
30 year old woman with repeated headache. Diagnosed as tension headache and MRI fine for that
and neck spasm, which was normal. She is usual consumer to your GP practice over the last 10 year
with recurrent abdominal pain and dysuria on and off.
Task
-counsel the patient about the cause of current headache
-tell one more possible reason

History
1-any specific concern?
2-headache Qs (SOCRATES1) + pain anywhere else
4-associated sx (organic)
-Previous viral infection, fever, rash.
-N/V, photophobia, phonophobia, warning signs
-Watery eyes, runny nose
-Blocked nose, toothache
-Neck stiffness
-Jaw pain, vision problem
-Weakness, numbness, loss of balance
4-somatisation Qs
-Pain other sites
-N/V, bloating, food intolerance
-Loss of sexual interest, period irregular, erectile dysfunction
-Pseudoneuro: impair balance, paralysis, aphonia, urine retension.
6-psychological hx (confidentiality)
-Mood Qs (low mood)
-Delusion, hallucination, insight, judgment
-Suicide
-HEADS (divorce, living alone).
6-hypochondrial Qs
-Do you think you have underlying serious disease
-Family hx of cancer
7-FHx, PMH, period

Diagnosis
-First let me assure you that the MRI of head and neck is normal so it is unlikely that you have any
serious underlying organic cause.
-Your sx can be due to several reasons like tension headache with adjustment due to tighten muscles
and it can be due to depression and anxiety
-Your headache is real. However, what I’m suspecting is that you have more than headache. From hx
you seem unhappy and depressed and I’m wondering if we can relate these two together.
-Mind body axis… You are going through too much stress at work and at home.
-One other possibility is somatisation disorder, which is …
-Mx (Lifestyle, Psychologist, Family meeting, support)
Somatization Disorder (Materials)
You are seeing a 26 year old female, Jane who comes to you for review of the result of MRI brain
which was requested by the neurologist specialist. Jane undergone MRI as she has headache
associated with severe neck spasm. She has past history of abdominal pain and nausea.
Investigation with blood tests, CT and U/S were all normal. Colonoscopy and upper GI endoscopy
were normal too. Past history revealed she had a feeling that she had a breast lump on
investigation that was normal as well, history of chest pain investigation ECG stress test echo and
Holter and MRI revealed normal.

TASKS
1. Focused history
2. PE from examiner
3. Investigations
4. Diagnosis and Management

Somatization disorder
The essential feature of a somatization disorder is a pattern of many physical complaints in
personal younger than 30 years that occurs over several years and results in unnecessary
medical treatment and/or causes significant impairment in functioning. This diagnosis was
historically referred to as hysteria or Briquet syndrome. The somatic symptoms are neither
intentionally produced nor feigned and appear to be unconscious to the patient. All the
following historical criteria are required for a diagnosis.
 Four different pain sites (head, abdomen, back, joints, extremities, chest, rectum) or
painful function (menstruation, sexual intercourse, urination)
 Two GI symptoms other than pain (nausea, bloating, vomiting, or intolerance or several
different foods)
 One sexual or reproductive symptoms other than pain (erectile or ejaculatory
dysfunction, irregular menses, excessive menstrual bleeding)
 One pseudoneurological symptoms (impaired balance, paralysis, aphonia, urinary
retention)
Criteria:
 4 pain symptoms
 2 GIT symptoms
 1 sexual symptom
 1 pseudoneurological symptom

Positive points in the history:


Mood is flat.
Dad and mom left job and looking after her.
Met the criteria for somatization disorder
Anxious and worried.
Married before and now divorced.
Period regular, not sexually active (not interested) Been like this for 6-7 years.
APPROACH
 History
o Good morning, I am Dr. _____, your GP for today.
o Do you have specific concerns?
o Headache questions:
o Criteria of somatization disorder + duration
 Any other pain aside from the headache? Abdominal pain, back pain,
muscle pain, or joint pain?
 GI symptoms: nausea, vomiting, bloating, food intolerance?
 Sexual: loss of interest, irregular menses, erectile dysfunction?
 Pseudoneurological: impaired balance, paralysis, aphonia, urinary
retention?
o Differential questions
 Organic disease
 Hypochondriasis: are you more concerned about the symptoms or the
possible underlying disease that you might have?
o Psychosocial history
 How is your mood?
 Do you think that life is still worth living?
 How is your appetite?
 How is your sleep?
 Do you have any recent weight loss lately?
 Do you think that someone is spying on you?
 Do you see or hear things that others don’t?
 Do you think that thought are being inserted in your head?
 Do you think that your thoughts are being broadcasted in the TV, radio
or social media?
 Do you think that you need help?
 If there is a fire in this room, what would you do?
 Do you know where you are? Who I am? What date it is?
o HEADSS
 How is your home situation? Who do you live with? Any stress at home?
 Do you still talk to your relatives?
 Do you work? How is your work? Any stress at work?
 Do you still enjoy the activities that you used to before?
 Are you sexually active?
 Do you still go out and meet with your friends?
o SADMA
 Do you smoke?
 Do you drink alcohol?
 Do you take recreational drugs?
 Do you take any over the counter or prescription medications?
 Do you have any known allergies?
o Do you have any past history of any medical or mental illness?
o Do you have any family history of any mental illness?
 Diagnosis and Management
Jane, based from the history that I’ve gathered and the investigation results, it is highly
likely that you are suffering from the condition called Somatization Disorder. Do you know
what this condition means?
Before I explain it to you, I would like to tell you that you pain is real but we can relate it to
the condition that you have.
Somatization Disorder is part of the Somatic System Disorder. In this condition, the patient
experience pain, several problems related to your tummy area such as nausea and
bloating. It is also related to your loss of interest in sexual activity and problems in
urination.
The sadness that you have can manifest in your body and there is connection between this
and your symptoms.
The exact cause is unknown but it can be attributed both in familial and environmental
factors.
Management:
o Pain Management
o Lifestyle Modification
o Psychologist: CBT
o Family meeting with consent
o Support groups
o Reading materials
o Review
o Red flags

Case (3/3/2017)
30 yr old with repeated headaches. Diagnosed as tension headache and mri fine for that and
neck spasm which was normal. She is usual customer to your GP practice over the last10 years
with recurrent abdominal pain and dysuria on and off.
Task:
find the cause for the current headache
Tell her the likely cause and mx

No organic stuff, Husband left 3/12 ago, she's angry stressed and hurt because of that, no
depression no anhedonia no psychotic feats no suicidal ideation.
I said tension headache and adjustment, but check Karen's maybe it's somatisation

comment
Hello doctors this was my exam case. At that moment I did not think or say anything about
somatisation because the task was to find the cause of the 'current' headache. So I described her
dx as tension headache due to adjustment disorder. I passed this case.
Case (7/3/2017) fail
GP, 45 yr lady complaint of headache and neck pain. Last month, previous GP did MRI head and
normal, she was diagnosed with tension headache. Now, headache occurred again. 10 yrs
back(or for the last 10 years), she had history of tummy pain, urinary symptoms and some vague
symptoms.
Task
-History to find out causes of her presenting symptom,
- explain about her condition,
- give one more possible diagnosis to patient & Mx.

Her headache is typical nature of tension headache.


has a lot of stress-separated from her partner 3 months ago, works at book shop and worried
that she might be fired because she can’t concentrate well, financial problem+ and got help from
centerlink, lived with mother and sister, no alcohol, smoking and drugs, I forgot to ask about
other symptoms present in 10 yrs back.
Explain her condition related with her current problem and stress, she looks depressed but
mood is ok when asked, I reassure her and tell all the support that I can arrange for her. Another
Possible one I said was somatoform disorder and then bell rang [failed]

Case (6/7/2017)
You are a GP. Next patient is 57 yr old lady who complaint of headache. She has been diagnosed
with tension headache for 3 years. She also complaint of pain in neck previously and MRI was
arranged which turned out to be normal. She also has previous complaint of recurrent
abdominal pain for which no cause is found,and also dysuria,urine frequency

Task:take history,explain your understanding of cause of her headache and other possible
diagnosis
i also think somatization. but also treat and investigate the recent s/s . if
with headache exclude other causes, if has UTI s/s then Urine MCS

For me its somatization. Just mention a few organic nd psych dds like illness anxiety,
depression, adjustment
211-MSE Psychosis
Feedback 20-2-2018
GP, 19 years old man brought in by family members, he had recent behavioural change. He was
treated 18 months ago with likely Schizoaffective disorder and ? medication was given. She did
not take medication for a few months.
Tasks:

- History from the patient

- Present MSE to the examiner.

- No diagnosis and no management

2 min thinking: I had this case before, most likely patient will be wearing sun glasses, taking to a
voice, telling me about his special power in the new dimension. Just ask a proper psychosocial
history (ASEPTIC + HEADSS)
History:
Introduce myself. Patient is wearing sun glasses, talked to the wall before greeting me. I told him
confidentiality statement and asked him why he is wearing the sun glasses (to protect his eyes
and his eyes has special power). Oh alright Sam, would you mind telling me your special power?
(I can read people’s mind in the new dimension?) do you think you are special? (yes)

Sam how is your mood right now? (good) anytime when you are feeling very high? (no) very
low/sad? (no) At any point of time do you think about harming yourself? (no), how about
harming others? (no)
(Every now and then he would talk towards the wall)
Sam, I can see that you are talking to someone, could you tell me who are you talking to? (the
high authority) can you see him/her?(no), what is your conversation about? (sorry I forgot the
answer) do you see the person you are talking to? (no)
At anytime does your thinking being withdrawn from your mind? (Thought withdrawal)(no) or
someone trying to insert some ideas into your mind? (Thought insertion) (no), to you think your
thinking is being broadcast? (Thought broadcasting) (no), does the news in TV or newspaper
hold special meaning to you? (Delusion of referance) (yes, but sorry I forgot what special
meaning he told me) Do you think people is trying to spy on you or harm you? (Delusion of
persecution/paranoia) (no), are you afraid of certain thing or places? (Phobia) (no)

I can see from my notes you were diagnosed with some mental illness before, are you still having
the medication? (Insight) (no, I don’t need them) do you experience any side effect from the
medication? (no), do you think at the moment you need some medical help? (no), what will you
do if there is a fire in the room? (Judgement) (I will call for help)

(Cognition: orientation to time/place/person)Do you know where you are right now? (at the
surgery), what is the day today (Tuesday), do you know who I am? (you are the doctor)
(HEADSSS)Whom do you live with? (my parents), do you go to study? (sorry I forgot the answer),
do you enjoy your hobby? Are you in a relationship? Do you smoke/drink alcohol/use
recreational drugs? (no)
Thank you for the information you have told me. Now I would like to talk to my examiner.
MSE:
Dear examiner, I would like to present to you the mental state examination of my patient, Sam, a
22 years old male brought in by family members due to behavioural changes.
He is wearing appropriate to time and season, well kempt, however he is wearing a sun glasses
in the room. He is friendly to the examiner, answers questions appropriately and sometimes is
distracted by the sound he heard.
His speech is normal in tone, volume and speed.
He described his mood as good, which is congruent to the affect.
He is experiencing auditory hallucination as he is having conversation with the higher authority,
however he denies any visual hallucination.
In terms of thought form, it is linear. He denies any thought insertion/withdrawal/broadcasting.
There is delusion of grandiosity as he thinks he has special power in the new dimension. He
denies delusion of persecution, no suicidal and homicidal ideation. No phobias.
His insight is impaired, judgement is intact
He is orientated to time, place and person.
I can built a rapport with the patient throughout the consultation.
In terms of risk assessment, he is having some risk as he does not want any medical attention
and is having auditory hallucination.
Reliability – I could not finish it as bell rang

AMC Feedback – Behavioural change: PASS

Feedback 21-7-2018
Change in behviour – PASS (G.S- 4)

Stem: Previously diagnosed with schizophreniform disorder, was hospitalized last year, stopped
taking meds, brought in by dad coz of change in behaviour
Task:
-History
-Give your DDx
- Pt had no insight into his condition, on history he had auditory hallucinations, delusion of
reference, did the HEADSS questionnaire, no smoking alcohol or drugs. Had stopped meds as he
thought he was fine(don’t remember if I asked since when he stopped), but didn’t notice any
side effects with the meds. Judgement, orientation ok.
- DDx – relapse of the schizophreniform disorder, acute psychosis, personality disorder, alcohol
induced (but not likely as you don’t take alcohol). Just told that I would want to admit him, to be
assessed formally by a psychiatrist, though it was not a task. He agreed. Told I would speak to
dad. [P.S. felt funny explaining the possibilities to a psychotic patient. But he kept saying “yeah..
yeah.. yeah..”]
Feedback 20-4-2018
Feedback9-5-2018
Mood Change
Depression with psychosis and high risk of suicide
It was my last case. Female patient don´t remember her age. I don’t remember the setting if GP
practice or hospital. Patient was brought by her partner, she is a Vet, and recently she has lost a
patient I can´t remember how but it is mentioned in the case. She has been depressed since that
happened. Her partner caught her when she was trying to hang herself this morning and took
her to you.
Task
Take mental state examination for 7 minutes
Present to the examiner
Patient was wearing sunnies.
Offered confidentiality. After 1 minute or so patient finally started to talk. During that time I was
just reassuring her. I did all the mental health examination and then presented to the examiner
with ASEPTIC.
Passed. Global score 4.
Feedback 11-12-2018
Father brings 22 year old son for behaviour changes. He was diagnosed a few months back with
schizoaffective disorder but was not compliant with medication.
Do MSE and present examiner.

Patient was wearing sunglasses. I took history ASEPTICJ PLUS HEADSSS(DID NOT ASK ALL because
patient is responding to auditory hallucination . so have to wait in between asking questions. BE
QUICK. I GOT CHOICE AND TECHNIQUE OF EXAMINATION- 5..But story doesnot end here..i did
not do well in presenting mse because I miss important things in thought section…such as he has
Circumstantiality, lossening of association, flight of idea.. he showed in his behavior..as I have
almost waste my time in history. I don’t have any time say about 3 R-..So , my sincere advise ask
important questions in history to save ample time for presentation. I failed this .

Feedback11-12-2018
Station 17 ( Behavioral Change) Global score 4 PASS

22-year-old man brought him by his father for behavioral change. He was previously diagnosed with
schizoaffective disorder but has poor compliance with medication.
Task: Do MSE (6min) and present it to the examiner.

This station was my biggest deception but also my biggest surprise.


I had an MSE on my first exam. It was a 4-min movie in a patient with mania. I failed the case.
So I studied MSE like crazy until I knew it by heart.
Good thing I did not need to give diagnosis.

I came in the room and greeted the examiner. She was very nice.
I greeted the patient, introduced myself and assured confidentiality.
I took the paper sheet on the table and wrote the acronym ASEPTTICJ 3R.
It was very difficult to understand the patient.
But I kept taking quick notes in front of each item.
I did pretty well.
But when it came to presenting to the examiner, I was too slow and the bell rang before I could finish it.
I was very disappointed thinking I would fail this station.
What was my surprise when I saw that I passed it!

Key step 1:yes. Approach to patient/relative: 5


Key step 2:yes. Choice & Technique of exam, organization and sequence: 4
Key step 3:yes. Accuracy of Examination: 4
Key step 4:no.

Case (4/10/2017) Assessment of change in behaviour – Pass


22-year-old male brought by his father due to behavioural changes. Had psychotic symptoms 18
months before and diagnosed as schizophreniform disorder.
Tasks:
•Take relevant history
•Advice patient on probable diagnosis with reasons
Take relevant history
(Psychosis history)
Approached as to why his parents are upset. (I keep to my room.) Why? (I hear voices.)
How long has this been happening? (Quite a while… / he did not give a specific answer.)
What do they say? (Can’t remember his responses, but that was about galaxies and universe and
some change in the world in which he has a big role to play.)
Do they ask you to do something? (No, they just discuss among themselves.)
Any suggestions in these discussions, of hurting yourself or anyone? No
Do you see them? No
Feel, see or smell anything strange? No
(Mood question)
Do you think life is worth living? (Of course.)
Ever thought of suicide? No
(Thought questions)
No thought insertion /withdrawal. He thinks people in TV/radio are talking about him
sometimes.
(smoking/alcohol/drugs)
Non-smoker/ not using alcohol/ used (mentioned the name of a strange street drug -can’t
remember which) before the previous admission but never after discharge.
What happened at that time/ symptoms seems to be same. Antipsychotics tailored off and with
drown sometime back.
(HEADSSSS)
Home: Supportive parents and brother / no partner
Worked previously. Stopped because he has more important mission now.
(insight and judgement)
When asked what was his idea about medical treatment for this - showed insight, said the pills
worked well last time.
Judgement fire in the room question- showed good judgement.
(Patient was frequently distracted by voices during our conversation/ so I had to wait and ask my
question again. Noticed that he was not talking with the voices nor disturbed by them.)
Advice patient on probable diagnosis with reasons
You are having acute psychotic episode again probably due to relapse of schizophreniform/
schizophrenia as diagnosis depends on duration of symptoms. Psychiatrist evaluation necessary
for definitive diagnosis.
A common condition… Can be due to illicit drug use but he denied it.
Can’t remember much about the discussion though. He was surprisingly interested, even when I
described increased dopamine in brain as the cause.
• All 3 key steps covered.

Case (22/2/2017) FAIL


You are in GP, 18 year old girl, university student, is brought in by father for change in behavior.
She has been diagnosed with schizoaffective disorder about 2 year ago. She was on some
medications for that (I cant remember name) but stopped by herself.

Tasks - Do MSE

- Present MSE findings to examiner

(You are not supposed to tell Dx or Mx)

2 mins thinking

-This is usual MSE , dun need to worry too much


-Dun miss 4 key points in psy : Confidentiality, Mood, suicidality, substance abuse
-Then I recall pneumonic - HEADSSSS, ASEPTICJ

MSE
When I stepped into the room, a young lady was sitting and she was wearing sunglasses.
(I didn’t ask why she is wearing this, but if u ask it, she will tell u why)
Greeting and building rapport
1-Confidentiality - Everything we are gonna discuss will be private and confidential unless u and
other ppl are at risk (RP - Sure Dr)
I understand that u have been diagnosed with some mood disorder 2 years back. Are u on
regular medication for that?
(RP : Yep, but I am not taking them)
Why? Are u scared of SE or having SE? (No, I dun think I need them)
2-Mood - How’s ur mood during these days? (RP : Good)
Can u grade it from a scale of 1 to 10? (RP looked at the examiner and said I dun know Dr, it’s
good) Ok
3-HEADSSSS
Home - live with parents
Education - study at university, no problem with her studies
Appetite - Good, Activities - still enjoy
Drugs - SAID : Dun use
Sex - not active
Sleep – Good
Suicidality and homicidality – No
Social - support by parents

4-ASEPTICJ
-Perception - During consultation, pt is turning towards the wall and acted like she is talking to
someone.
I asked her “to whom are u talking?” “God” “What is it about?” “to save the world” “How will u
save?” “I dun know, he asked me to save the world”
There is no visual hallucinations
Do u think u are a special person? (God told me to save the world)

-Thoughts
Do u think ur thoughts are controlled by others? No
Do u feel like radios and TV are talking about u? (No)
-insight
If we restart ur medications, do u want to take them? (No, I dun need them)
-judgment
What will u do if there is fire breakout in this room? (I will run away)
-cognition
Do u know whom u are talking? (My Dr)
Rule out organic causes
Head injury, brain infection, hypothyroid - No

Presentation to examiner

Examiner, my patient is 18 year old girl, Jenny, who is brought in by father. She was Dx with
schizoaffective disorder but non compliant to meds. I ve done MSE on her and found out that
Appearance - She is well dressed according to weather and culture
Behaviour- She is a bit uncooperative at first, but she will later on
I am unable to assess eye contact as she is wearing sunglasses.
Speech - Her tone is OK. But she has flight of ideas. (Examiner winced and I realized that I am
wrong. It slipped out of my tongue)
Affect - Mood is good according to my pt and affect is congruent with mood
In terms of perception, she is exhibiting auditory hallucinations but there is no visual
hallucinations
She has delusion of grandiose
In terms of thought, form and contents are abnormal. (Coz I cant differentiate the exact form)
Insight is lost but cognition and judgement are intact.
In terms of risk assessment, she doesn’t have any suicidality and homicidality ideations and not
using any alcohol and recreational drugs.
AMC Feedback - Behavioural change : Fail (Global Score - 3)
Approach to patient - 4
Choice of exam and sequence - 4
Accuracy of exam - 3
212-Borderline personality risk assessment

young female came with multiple self-inflicted wounds/cuts on her thigh, assessed and managed
by CAT and followed by psychologist/psychiatrist and cleared for discharge.
Tasks:
-Before sending her home u need to take history to assess her mental state.
-Suggest future management she required.

Notes
SAD PERSONS
S sex (males are more successful but females more attempt)
A Age (more if less than 20 or more than 40)
D Depression (2 points)
P prior history of previous attempts.
E excessive drug use (including alcohol)
R rationality loss (psychosis) (2 points)
S separated (single)
O Organised plan (2 points)
N no supports (socially isolated)
S  sickness (other morbid illnesses)

*S (sex) + A (age)  from the stem


*D (Depression) + P (previous suicide attempts)+O (Organised plan)  from mood history
*R (rationality or psychosis)  from hallucination and delusion history
* S (separated) + E (excessive drugs)+ N (no support)  from HEADS
*S (sickness)  from general or PMH
History (with bit of empathy)
1-Approach
-Confidentiality
-how is your injuries? Do you have any pain?
-can you tell me why you injured yourself?
-is this the first time? How often

2-Mood history (all normal)


-How is your mood ? How do you feel today?
-have you lost interest in things you used to enjoy?
-have you ever thought of harming yourself or others? Any plan? Any previous attempts?
-how is your sleep? Do you have difficulty getting to sleep or you wake up early in the morning
and have difficulty to get back to sleep?
-how is your appetite?
-have you lost weight recently?
-do you find it difficult to concentrate on things? Do you feel guilty?

3-Hallucination (negative)
-do you feel, hear, see things that others do not? what does it tell you?
4-Dellusion (negative) need to ask all
-do you think someone is following you or spying on you?
-do you think someone is trying to harm you?
-do you believe that you have a special power
-do you think thought are inserted in your mind? Withdrawn from your mind?
-do you think your thoughts are broadcasting through TV or radios?
-do you think other people’s action or behavior are directed towards you?

5-cognition
-where are you right now?
-what time is it?
6-Insight: do you think you need a professional help?
7-Judgement: let us suppose there is fire in this room, what would you do?

8-HEADS
-who do you live with? Any stress? Do you have enough support?
-what do you do for living? Any stress?
-SAD
-social activities

9-General
-previous and family hx history of mental problems/ PMH/ PSH/ medications

10-Ending
-with whom have you come?
-where will you go after the consultation?
-is there anybody to take care or look after you?
Management
1-no need for admission (as the patient already cleared by psych team)
2-refer to psychologist for CBT and psychiatrist (not sure about psychiatrist)
3-regular follow ups
4-life style advice
5-support groups
6-centrelink
7- +/- family meeting
8-reading materials and red flags

Feedback 19-7-2018
Pass (all 5 key steps Yes, score 6,6,6)
Pre discharge risk assessment( borderline personality disorder)
Long scenario outside with young female came with multiple self inflicted wounds/cuts on her
thigh, assessed and managed by CAT and followed by psychologist/psychiatrist and cleared for
discharge.
Task:
Before sending her home u need to take history to assess her mental state.
Suggest future management she required.
When I entered ,a young lady was sitting in a withdrawn posture, pretending she don’t wanna
talk and tried to behave rudely . I started with confidentiality and ask abt the injuries. She said
she did that because it make her feel good, no intension of sucide. whenever she was upset she
did that than I quickly covered ASEPTICS and HEADS and SADMA.
Mx: I told her that what her boyfriend did was not good but the way she responded to the
situation concerns me. I want a happy cheerful life for her with stable relationships but for that
she need little modification in her behaviour and attitude. For that I refer her to psychologist for
talk therapy(CBT) giving u reading material regarding boderline personality and FU.
borderline personality
hx risk assessment mx old recall did exactly that complete psychosocial hx sad persons
approach
mx i said we will not keep you here and my senior will come and have a look at you and will
dischgre you arranged for some1 to oick her up.
cbt and family meeting

Feedback 19-7-2018
pre discharge risk assessment (borderline personality disorder)
Fail
Total 5 key steps , 4th is no ( i dnt know what was that ) Score : 5,4,3
Case : young female with multiple self inflicted cuts on thigh. Sutured n taken care of . Already a
diagnosed case of borderline personality disorder. Seen by CAT team and cleared for discharge.
Tasks :
history/ risk assessment / further Mx
I will not give any opinion about this case as i didn't pass it so dnt want to confuse anyone. I did
whatever I had practiced but dnt know the reason why i failed it .
Case 2-3-2017
-Borderline personality disorder young lady admitted after self inflicted cut injuries after fight
with boy friend. Patient was interviewed by Psych Reg over the phone and plan to DC. Do
necessary assessment including risk assessment and put up a plan.
- borderline personality- risk assessment ( Ask intention during self harm, Previous attempt to
suicide, Any plans, Any weapon possession, future plan, Aseptic qs briefly, Headss q )
Case (9/5/2017) (10/5/2017)
Borderline personality typical karen case. The role player was acting exactly the same way as in
notes and the scenario was also the same.
Case (25/8/2017)
Borderline personality cutting self to feel better. No suicidal tendencies. Has been in hospital
before in and out. Initial line of management.

Karin case
Risk Assessment (Borderline Personality Disorder)
Case: You are an HMO in the ED and your next patient is a24-year-old lady who has a history of
repeated self-harm. She has been diagnosed with borderline personality disorder. She cut her
thigh this time. She was found intoxicated and was brought to the hospital. Now she is okay. The
wound has been taken cared of and she wants to go home.
Task
a. Risk assessment
b. See if she’s ready to go home

Case 2: A 16-year-old girl was at a party and had benzodiazepine overdose yesterday after having
a fight with her boyfriend. She lost consciousness and was brought to you by her friends. Now,
she is ready to be discharged and your task is to do the risk assessment.
(on history she takes tablets from mom to be able to sleep  in this case advise on sleep
hygiene!!!!).

Case 3: You are an HMO in the ED and a 22-year-old female has been admitted with a number of
wounds in her arms and legs. Most of them were superficial cuts. She has been treated by the
registrar. The wounds have been sutured and she wants to go home.

Management
- Ensure confidentiality
- Appreciate that the patient is in stress
- I understand that you are going through a tough phase, can you please talk more about it. I
know I am sorry to hear that. Sometimes, we are very frustrated and some people do make us
sad, but
(suicidal questions)
-did you have any intention to kill yourself by cutting your thigh (no doctor, it just makes me feel
better)?
-Have you done this before?
-Have you ever thought of harming your boyfriend or any body else?
(Mood questions-depression,psychosis, mania)
- How is your mood? Do you feel sad?
-Do you think you have lost interest in things that you used to enjoy before?
-Any sleep problems?
-Change in appetite or weight?
-Do you think life is worth living?
- I’ll ask some questions which might seem funny but do you feel/see/hear things that others do
not? Do you have any strange experiences?
- Do you think I’m crazy? Sorry, I didn’t mean that, but these are routine questions for all
patients who are in your situation.
- Are there any times when your mood is really high?
(Relationship-living alone-stressors)
-Have you had problems with your relationship in the past as well?
- Whom do you live with? Do you get along with them? No issues? How about your parents?
-Do you have friends? Do you socialize with them?
-Do you work or are you a student? Any problems at work or at the uni?
-Is there anything else that is bothering you (financial, relationship, etc)?
- SADMA?
(Past history)
- PMHx: thyroid diseases? Drug history, child abuse, past history depression,Psychosis, smoking,
alcohol
(Ask about insight, cognition and judgment)
o Do you think there is something wrong? Do you think you need professional help? (Insight)
o Fire/envelope question (judgement)
o Do you know where you are, date, and time.(cognition)

- You can go home Mary. How would you go home? When you go home, what will you do? What
are your plans tomorrow? I would recommend you to call your family or friend to come and pick
you up. I would not recommend that you drive now.

-I am really concerned with the way you are coping up with stress, so I will refer you to the
psychiatrist and psychologist. They will do talk therapy and will teach you techniques on how to
handle stress without harming yourself.
-Organize social worker if you need support
- About alcohol we need to discuss about the safe level of drinking.
- If you have financial problems  centerlink
-There is also a 24/7 hotline number 1800187263 (1-800-18-SANE). Anytime you feel stressed,
you can give them a call and you can talk to them.
- If you agree to it, I would be happy to arrange a talk with your friends or family.
213-carpal tunnel syndrome
Factory worker has funny feeling in the fingers of her right hand. Sometimes wakes her from
sleep, having to shake it vigorously to go away.
Tasks
-Physical examination
-Dx and ddx
Instruments ( cotton, pinprick, tendon hammer)

1-Look
Dorsal aspect
-scars -skin colour changes especially bruising
-soft tissue/ joints swelling -Deformity -muscle wasting
palmar aspect
-scar (carpal tunnel release scar).
-thenar or hypothenar muscle wasting.
2-Feel
-Temperature + pulse (just present or not)
-Muscle tenderness and swelling
-feel for thenar and hypothenar muscle wasting
3-Sensation
*start with cotton wool, if reduced then pinprick, ask if it is the same on both hands.
*all 5 points
2 ulnar (lateral tip of little finer+ hypothenar section of palm)
2 median (lateral tip of index finger + thenar part of palm) (+/- loss of sensation over the index
finger)
3-MOVE (always compare)
Active:
-wrist flexion+ extension
-make a fist and open your hand (imp)
-abduction + adduction of fingers
-thumb movements.
Resisted
-grip (do not let me pull finger out)
-thumb abduction (point toward the ceiling and do not let me push it down)
-opponens (touch little finger and do not let me pull it apart)
4-REFLEXES
5-SPECIAL TESTS
-Phalen, tinel tests
-Neck + Shoulder & elbow look and feel to rule out other causes {all if thenar sensation is lost }.
DDX
-Carpal tunnel syndrome
-OA, RA
-Trauma
-Thyroid, DM
Case (29/3/2017)
A 50 year old factory worker is coming with the complaint of tingling in his right hand.
Tasks:
pe
diagnosis and differentials
2 mins: nerve entrapment case (median or ulnar)
Should check from neck, supraclavicular (sc), elbow and wrist
Instruments there: pin, cotton, knee hammer (so have to do peripheral neuropathy as well)
Hi, I am Dr. Maria. I understand that you have some tingling in your right hand. I am required to
examine you. That would require me to look at your hands, feel them and ask you to do certain
manoeuvres. Is that all right? I will try to be as gentle as possible, but if at any time you feel
uncomfortable, please let me know. Before I begin, do you have any pain?
I washed my hands while saying all this.
I said I want my patient properly exposed and the examiner asked me to proceed assuming it.
Inspection:
with hands on a pillow; both wrists and hands symmetrical, No obvious deformity, clawing of
hand, swelling, scar, trauma, muscle wasting, bruising, bleeding
Elbows: Normal, no swelling
Sc: no swelling
Neck: no deformity, spine curvature maintained.
Palpation:
peripheral pulse, CRT, temp, tenderness (from hand, wrist, elbow, sc and neck)
Movements:
wrist (symptoms elicited with both flexion and extension), fingers and thumb (I think I did both
active and passive, but I don’t remember)
Neurological examination:
ask the patient to compare both sides always.
sensation:
Median: C5-deltoid, C6- thumb, C7- middle finger, Ulnar: C8- little finger, T1- medial side of
forearm. Use both fresh pin and cotton and discard them.
Findings: absent sensation on the thumb and decreased on the middle finger
Power:
Median- pen torch Ulnar: card holding Shoulder: abduction and adduction
Elbow: flexion and extension Fingers: squeeze and spread Findings: normal
Reflex:
Biceps, triceps, supinator, finger jerk (normal).
Special Test: From the above findings it was median nerve so Tinel’s and phalen’s (positive)
Diagnosis: carpal tunnel syndrome, due to compression of the median nerve at the wrist

Dd: ulnar nerve entrapment, entrapment at neck, supraclavicular (sc), elbow, tumour pressing
on the nerve

Global score-6
214-Cubital tunnel syndrome
Factory worker, presented with T and numbness sensation of right hand.
Tasks
PE
Dx and D/Dx

1-LOOK (SSS DW )
-Especially wasting of hypothenar and claw hand deformity to r/o Injury
-elbow

2-FEEL
-Muscle (wasting, tender, swelling )
-Sensation at 5 points ( cotton and pinprick) … (reduced sensation in the R little finger)

3-MOVE (like CTS lecture)


Active (all movements esp fist, finger spread)
Resisted ( same as CTS )

4-Reflexes

5-SPECIAL TESTS
-Cubital tunnel tapping test (90 degree elbow position ) (+ve)
-Phalen, tinel tests

6-Complete with Upper Limb Neurological examination

Explain
increase pressure over ulnar nerve, which passes close to the skin surface in area of elbow,
usually due to

- Leaning on elbow on hard surface

- Bend elbow for a sustained period such as sleeping with your hands crooked under the
pillow, talking on phone.

- Abnormal bone growth or intense physical activity

Cubital tunnel = ulnar nerve compression


claw hand= ulnar nerve injury
Feedback 20-7-2018
PE
cubital tunnel syndrome

hand problems
pass , global score 4
key step 1 No
key step 2,3,4,5 all yes
approach to pt 4
choice and technique of examiantion, organisation and sequence 3
accuracy of examination 4
Dx/DDx 4

factory worker , presented with T and numbness sensation of right hand ,


PE and Dx and D/Dx

i did upper limb neurological examiantion


sensation was reduced in right little finger , motor and reflexes are normal
palen test and tinel test for median nerve was negative

i did tapping behind medial epicondyle , but patient didnt give me T and numbness , i am
pretty sure i am tapping the right place , so i tap again and again , still he didnt give me
finding , i was really confused , i asked whether he has neck pain ... he said no,

finally , i was just holding and rolling the skin behind the medial epicondyle before tapping,
suddenly patient gave me the positve result ,
so i guess , they want us to palpate the groove behind the epicondyle before tapping ,
I didnt have much time to do tooth prick sensation test , so i skipped it
these made me get 3 in choice and technique of examiantion
Case (11/2/2017) fail
A 40 yr old female, factory worker comes to you because she had been having some funny
feeling in the fingers of her right hand for some time.
Sometimes wakes up from sleep having that feeling. Shaking her hand vigorously helps
Task:
- Perform PE on patient
- tell examiner what are u doing and ur findings
- explain Dx and DD to patient.

Went in and introduced myself, washed hands and saw that patient was sitting on the bed
comfortably.
- Told her I’d like to examine her hands and see whats going on,
- put her hand on pillow and did brief hand examination.
- Inspection- nothing except she had this rash on her hand near the thumb so I mentioned it to
the examiner.
- Palpation- temperature and bulk of muscles of hand and forearm.
- Tested sensation and noticed loss of sensation in area of ulnar nerve.
- tested motor function of nerves.
- Phalens test and Tinel’s test on wrist and cubital fossa. Tapping at cubital fossa reproduced
symptoms.

Explained to her it is something we call cubital fossa syndrome.


So there is a nerve that passes in this area (pointed to cubital fossa) and during movement
especially prolonged flexion at elbow joint causes a strain/stretch of the nerve and resulting in
the symptoms.
Mentioned that initially i thought of carpal tunnel syndrome or some problem at the neck
causing symptoms but with my examination I’m pretty sure it is Cubital tunnel syndrome. Bell
rang.

Feedback: Hand problems, Fail, Global score 3

(Retrospectively, I think my approach to patient was okay, but maybe the sequence of the
examination I carried out might have been all over the place and I had to be reminded to give a
running commentary. Maybe that contributed to an overall low score)
215-Claw hand
Women fell down from tree. They were playing on tree and fell down. To lessen the impact of
fall, they tried to hold on to a branch while falling. Now came to you having this pic.
Task
-upper limb examination (I (look) T P (move) R C (think we can leave to end) S (feel)

Note
I think This case is a combination of hand ex+ upper neuro for the task

1-look (SSSDW) = inspection


-scar
-skin colour changes, bruising, bleeding
-swelling
-Deformity
*claw hand deformity +ve:
True claw hand if there is flexion of PIP and DIP joints
non true if loss of flexion of both PIP & DIP joints
-wasting
*wasting of thenar or hyopthenar
-elbow.
From feedback, they said look showed total claw hand may be they mean true one with flexion
of both PIP and DIP.
2-FEEL = sensation
-Temperature
-Pulse
-Muscle (wasting, tender, swelling)
-Sensation
*C5, C6, C7, C8, T1+ 5 points of hands (cotton and pinprick)
From feedback sensation show lost of C8-T1 mean the side of ulnar nerve and this mean
incomplete claw hand.
3-MOVE (active and resisted) = power
Active
-wrist flexion and extension
-finger flexion and extension
-finger abduction and adduction
-all thumb movements
Resisted
-the same like above
-you can also add shoulder abduction, adduction+ elbow extension, flexion.
Feedback showed power or resisted movements lost
C7 which is wrist extension
C8 which is hand grip
TI which is spreading fingers
4-SPECIAL TESTS
-Tone, reflex
-complete with neck and shoulder ex.

Anyway, on explanation all that you need to say is that you have injured a nerve called ulnar
nerve causing claw hand deformity. With ddx.

Recall 12-4-2018
A 27 year old man was playing with friends suddenly fall from tree around 2 meters. during fall
tried to hold himself by his hands. Comes with claw hand and no any other injuries.
task:
* do p/ e
* expkain to examiner what you are doing
* explain condition to pt
216-Hypocalcemia hand examination
35 year old lady Jane presents to you with some tingling sensation and severe burning pain in both her
hands. She had been diagnosed with hypothyroidism for the past 7 years and she has recently had a
surgery for goiter (thyroidectomy), she has been feeling like this since after the surgery.
TASKS
-Perform PE -Advise patient
Differential Diagnosis:
 Hypocalcemia due to removal/injury to parathyroid glands
 Carpal tunnel syndrome - median nerve compression
 Ulnar nerve neuropathy
 Cervical radiculopathy
 From the neck: neck pain, shoulder pain, hand pain
 From the elbow: wrist pain only
Hypocalcemia: pins and needles, paresthesia noted, sensation is intact, no nerve damage is noted

APPROACH

 Look
 Feel
o Temperature
o Tenderness
o Pulse
o Circulation
o Sensation with cotton and pin
 Sensations will be intact. Just a burning pain will be felt
 Move
o Median
o Ulnar
o Radial

 Special Tests
o Carpal tunnel syndrome
 Phalen's:
 Tinel's: tap on the middle of the wrist. Do you feel any shock-like pain, pins and
needles sensation over this area?
o Froment's sign
o Ulnar nerve entrapment
 Tinel: ulnar styloid side
o Chvosteks' sign
o Trosseau's sign

 To complete my examination, I would like to examine the elbow joints and do a full neurovascular
examination of the upper limb.

Explanation
I have examined you for all the nerve injuries, and I have found out that all your nerves are intact.
However sometimes, these symptoms may also be due to low levels of calcium in your blood. In your case,
you had this surgery and you had your thyroid gland removed. It is a common complication after this kind
of surgery to have low levels of another hormone in the body called as parathyroid hormone which is
responsible for regulating the calcium levels in the body. In your case, I suspect that you have had this
complication and that is why you are having these symptoms.
Summary
1-LOOK ( SSS DW ) NL

2-FEEL ( NL )
Muscle (wasting, tender, swelling )
Sensation at 5 points ( cotton and pinprick) ( intact )
TCP

3-MOVE ( normal )
Active & resisted

4-SPECIAL TESTS
Cubital tunnel tapping test ( 90 degree elbow position )
Phalen, tinel tests

Hypocalcemia test ( chovstick sign, trousseau sign +ve)

Explain
I have examined you, al nerves are intact. However, these symptoms may be due to low calicium
level in your blood.

In your case, you had surgery and you had your thyroid gland removed. It is a common
complication after this kind of surgery to have low level of another hormone called PTH which is
responsible for regulating Ca level in the body.

Case (30/5/2017)
1- Woman with cramps and pins and needles in both hands after 2 weeks of thyroidectomy.
Detailed PE, explain Ddx.

2- long scenario i just remember painful hand with tingling and numbness and pt had
thyroidectomy couple of weeks or days i think wss mentioned she is taking thyroxine and she
had 2 occasions of hand problem previously something like that.

Do pe and explain likely reason and other causes

Hand exam post thyroid


Hand examination post thyroidectomy i did everything but nothing postive as pt had tingling
and numbness sensation ,
then i saw sphegmo. I just ask to do it for and trousseaus sign and she gave me positive finding

so i said it is due to hypoCa. And give other ddx like carpal tunnel or cubital tunnel or due to
other arthritis problem but unlikely.

Hand examination i thibk she had scar at wrist and i said there was some muscle waisting but
nor sure i just comment wt i was able to see and feel , troussueos sign come positive so not sure
if she pretend that or was real
Case (1/6/2017)
Tingling and numbness examination (2 weeks after post thyroidectomy) Need to perform
trousseau sign
hypocalcemia hand exam

comment
I got this one , when i read the scenario i was thinking could be CTS or somthing like that so i
enter the room ask her how she is feeling now after the sx.
And also had a quick look at the wound mentioned no bleeding no signs of infection.
And then i did full hand examination nothing was positive.
i went to elbow tap to see if it is cubital tunnel again nothing positive.
so i asked her do u have any neck pain she said no.
last min i notice BP cuff i just told the examiner i want to check for trousseaus sign examiner told
me do it so i put the cuff inflate and pt gave typical carpopedal spasm.
so i just said u r having this feelings cause of low Ca and it is one of the complication of the sx
then i mentioned other ddx.
217- Hand examination
Case 1 (osteoarthritis/ rheumatoid arthritis AMC exam)
old patient with right hand pain
tasks
-physical examination
-dx and ddx

Examination steps
1-Approach
Introduce ask for pain and painkillers  wash hands consent- exposure+ elbow
exposure hands on pillow

2-Look
Dorsal aspect
-scars
-skin colour changes (may be redness in RA)
-soft tissue/ joints swelling
*DIP (heberden nodes) (OA)
*PIP (bouchard node) (OA)
*MCP swelling (RA)
*wrist
-Deformity
*swan neck, bottonaire, z deformity of the thumb, ulnar deviation (RA)
-muscle wasting
Palmar aspect
-scar (carpal tunnel release scar).
-thenar or hypothenar muscle wasting
-contracture
Elbow
-rheumatoid nodules, gouty tophi.

3-Feel
-Temperature
-tenderness (+ve) and swelling
normal hand then painful hand
wrist (RA)
metatarsals
MCP (RA)
PIP (RA&OA)
 DIP (RA&OA)
radial styloid (Dequevain tenosynovitis)
ulnar styloid (RA)
snuff box (scaphoid fracture)
-palmar: fee for thenar and hypothenar muscle wasting

Note/ when you feel PIP &DIP joints feel with both of your hands using the thumb and index
finger of each hand. One hand to hold the joint with the fingers in horizontal position and the
other hand to feel the joint with fingers position vertically when press.
Note/ ask always if they have any pain let you know and look at the patient’s eyes.

4-Move (limitation and pain)


Active
-wrist (4; extension, flexion, ulnar and radial)
move your wrists up then down then from side to side like this.
-fingers (4; flexion, extension, abduction, adduction)
make a fist then open your hand. Splay your fingers and push them together.
-thumb (5; extension, flexion, abduction, adduction, opponens)
point up to the ceiling, then down then outward then inwards then touch your little finger.

Passive (wrist and fingers only)

5-Function tests
-grip power (put your finger and ask him not to let it out)
-open jar
-key (imagine you open the door)
-safety pin or paper clip
-writing

6-special tests
-squeeze test (squeeze MCP joint) (RA)
-phalen test or tinnel (CTS)

Differential diagnosis
1-OA
2-RA
3-Trauma
4-Gout
5-Psoriatic arthritis
Case 2 (De Qurvein tenosynovitis AMC exam)
Your next patient is a 40 year old man who C/O pain in his right wrist.
Tasks
-history
-PE
-Dx and Mx

Differential diagnosis
1-De Qurvein tenosynovitis
2-trauma
3-ganglion

History
1-pain questions
-onset
-site
-severity
-radiation
-what makes it better or worse
2-asscoaited symptoms
-swelling (bursitis)
-redness (arthritis)
-numbness, tingling (CTS)
-fever
3-Trauma (fracture)
4-history of joint problems
5-occupation and general health

Examination steps
1-Approach
Introduce ask for pain and painkillers  wash hands consent- exposure+ elbow
exposure hands on pillow

2-Look
Dorsal aspect
-scars
-skin colour changes especially bruising
-soft tissue/ joints swelling
-Deformity
-muscle wasting
palmar aspect
-scar (carpal tunnel release scar).
-thenar or hypothenar muscle wasting.
-Contracture.
Elbow
-rheumatoid nodules, gouty tophi.

3-Feel
-Temperature
-tenderness (+ve) and swelling
normal hand then painful hand
wrist (RA)
metatarsals
MCP (RA)
PIP (RA&OA)
 DIP (RA&OA)
radial styloid (Dequevain tenosynovitis)
ulnar styloid (RA)
snuff box (scaphoid fracture)
-palmar: fee for thenar and hypothenar muscle wasting

4-Move (limitation and pain)


Active
-wrist (4; extension, flexion, ulnar and radial) (pain in ulnar deviation)
move your wrists up then down then from side to side like this.
-fingers (4; flexion, extension, abduction, adduction)
make a fist then open your hand. Splay your fingers and push them together.
-thumb (5; extension, flexion, abduction, adduction, opponens)
point up to the ceiling, then down then outward then inwards then touch your little finger.

Passive (wrist and fingers only) (pain +ve)

5-special tests
-squeeze test (squeeze MCP joint) (RA)
-fleinkstein test (+ve)
collect your thumb into your hand and make a fist then do passive ulnar deviation of the wrist;
test +ve if there is pain.
-phalen test

6-Function tests
-grip power (put your finger and ask him not to let it out)
-open jar
-key (imagine you open the door)
-safety pin or paper clip
-writing
Explain
-there are two tendons called abductor policis longus and extensor policis brevis pass through a
tunnel in the wrist covered by fibrous tissue called extensor retinaculum that hold the tendons in
place. When inflamed or swollen, the tendons become compressed against each other because
this retinaculum cannot extend this lead to resisted and painful tendons and thumb movements.
-possibilities: trauma, ganglion (nerve swelling causing compression)

Management
1-RICE
2-NSAID
3-Physio
4-if still symptomaticsteroid injection if not improved (surgery release)
Case 3 (Scaphoid fracture AMC exam)
1-25 year old man works as a chef, he had a fall a few days ago from skateboard and has pain in
his right hand. He came to see your colleague. X-ray was done and told it is a sprain. He come
back today because of persistent pain.

2-young man fell from bike 2 days ago. X-ray done in ED no fractures seen. Discharged with
painkillers now comed back to ED with ongoing pain.

Tasks
-physical examination of the hands
-explain x-ray to patient
-explain diagnosis
-further Ix and Mx

Examination steps
1-Approach
-Introduce and wash hands
- I understand from the notes that you are complaining of pain in your right hand so how severe
is it from 1-10. Would you like me to give you painkillers? All right before that do you have
allergy to any medications. I will arrange a stronger painkiller for you.
-explain examination and consent
-position hands on pillow and exposure

2-Look
Dorsal aspect
-scars
-skin colour changes especially bruising
-soft tissue/ joints swelling (swelling at snuff box)
-Deformity
-muscle wasting
palmar aspect
-scar (carpal tunnel release scar).
-thenar or hypothenar muscle wasting.
-Contracture.
Elbow
-rheumatoid nodules, gouty tophi.
3-Feel
-Temperature + pulse (just present or not)
-tenderness and swelling
normal hand then painful hand
wrist (RA)
metatarsals
MCP (RA)
PIP (RA&OA)
 DIP (RA&OA)
radial styloid (Dequevain tenosynovitis)
ulnar styloid (RA)
snuff box (scaphoid fracture)
-palmar: fee for thenar and hypothenar muscle wasting

4-Move (limitation and pain)


Active
-wrist (4; extension, flexion, ulnar and radial) (painful ext and flex and radial deviation)
move your wrists up then down then from side to side like this.
-fingers (4; flexion, extension, abduction, adduction)
make a fist then open your hand. Splay your fingers and push them together.
-thumb (5; extension, flexion, abduction, adduction, opponens)
point up to the ceiling, then down then outward then inwards then touch your little finger.

Passive (wrist and fingers only)

5-special tests
None

6-Function tests
-grip power (put your finger and ask him not to let it out)
-open jar
-key (imagine you open the door)
-safety pin or paper clip
-writing

7-Conclude joint above and below.

Explain X-ray
-this is the x-ray of your right hand, showing the bones of your right hand. It looks normal,
however from history and examination you most likely have fractured your scaphoid bone which
is one of the small bones of the wrist (show it on the x-ray) it is located on the thumb side of the
wrist.
Why I have been told no fracture despite severe pain?
-that because the nature of this fracture is such that initially may be so vague on x-ray so that it
is often missed at first.
Management
-what I am gonna do for you now is to apply scaphoid plaster from below elbow to MCP
including the thumb (arm pronation, wrist slight extension and radial deviation, thumb in mid
abduction and fingers in can holding position)

-I will give you painkillers and review you in 24 hours then I will do another x-ray after 7-10 days.

-if the 2nd x-ray come back normal then bone scan will be done. if the x-ray come back positive
then you will keep the cast for 6-8 weeks.

-avoid heavy lifting, pushing, pulling or any contact sport until fracture heals.
-maintain full movement of the fingers through the recovery period to avoid any stiffness.

-reading materials
-red flags (pain, numbness, tingling, swelling, colour changes, movement problems)

Case (28/4/2017)
Fall on hand 1 month, Initially X-ray normal, pain going on, come for check up. Task examination
hand, explain X rays, further investigations. Got scaphoid findings.

Case (9/5/2017)
Scaphoid fracture
middle age lady fall on skateboard 2 days ago, Visited GP and reassured as sprain. now pain not
relieved. Task: explain XRay, Perform , Dx and Invx

Case (10/5/2017) Scaphoid fracture pe typical Karen

Case (2/6/2017)
Scaphoid fracture
X ray given – difficult to see fracture
Examination of hand, Explain to the patient

Case (28/4/2017)
wrist injury
Young man fell from bike two days ago and X-ray done in ED.no fracture seen and discharged
with pain killers.now comes to ED with pain.task examine,explain ist X-ray to patient and
manage.
on examination tenderness at wrist and snuff box.no vascular or neurological problems.
role player wanted to know anatomy of bones especially where is fractured bone and y he was
told no fracture seen as he is having severe pain.
I explained scaphoid fracture,explained X-ray.told clinical diagnosis,plaster cast ,review in 24
hours then ten days if still pain bone scan to rule out necrosis etc.precautions like no wt
lifting,pushing,contact support etc

Passed
Case (11/2/2017)
25 year old man, works as a chef. He had a fall a few days back and had pain in his right hand so
he came to see your colleague in the same GP clinic. X-ray was done and he was told that its a
sprain. He comes back today because of persistent pain. Xray done on day of injury is provided.

Task:
Do PE (no more than 3 mins i think),
Diagnosis and what other Ix to do.

Washed hands and introduced myself, asked him how he was doing, if he was in pain and
needed painkiller.
Had a look at his hand in resting position.
palpated quickly, checked radial pulse, CRT, hand function(couldn’t do properly coz of pain), then
straight jumped to checking point tenderness in anatomical snuffbox, pressing of thumb on its
axis towards wrist. both reproduced pain.
Then I went on to explain to him what is on the X-ray and showed scaphoid bone and there was
a slight line, and plus with the positive findings I think it is a scaphoid fracture. Told him that the
nature of this fracture is such that initially it may be so vague on X-ray that it is often missed at
first.
Told him that he needs a thumb spica cast, i couldn’t remember the exact position in terms of
pronation/supination etc so i kind of showed him the position of the hand. Hopefully it was right,
my head was buzzing at that point of time.
I mentioned that I recently started working in this clinic so I’m not sure what is available easily to
us, but after about 7-10 days of immobilisation I’d like to arrange for bone scan or CT scan. Also
gave him red flags about when he has the cast on- pain, numbness, tingling in hand, to come
back to clinic

Feedback: Wrist injury, Pass, Global score: 6

Case (11/2/2017)
Hand exam
Boy had fall from skateboard. Did x ray yesterday and another gp said sprain and send him
home. Today came back coz of pain.
Perform physical examination (+ve signs for scaphoid fracture)
Diagnosis
Another imp investigation.
218-Primary PPH telephone case
sample case
HMO in obstetric department of major hospital. On call. Asked to see a 29 years old Lisa who
had just delivered a baby boy (3.5 kg) 15 minutes back. Baby is alive and healthy. This is her 3 rd
delivery. Informed by nurse over the phone that after delivery the cord has already snapped and
patient having sever bleeding.
Tasks
-talk to nurse over the phone
-advice about immediate management

AMC exam case


you are HMO in a rural hospital in duty for O&G ward. Nurse us calling you .A lady has delivered
recently and now is bleeding. From your room to ward take 5 min. before going there you want
to order what is needed.
your tasks:
* listen to her explanation and take history not more than 4 min.
* give advice and What she need do before you go

Approach
pick up the phone
-I am doctor---- the doctor on call today please May I know whom I talking to?
-which patient I will be dealing with?
-I can see that she is bleeding now so can you tell me the general appearance of the patient?
(Pale, anxious, sweaty)
-and what is the vital signs of the patient? (BP: 85/50, PR=120)
-I would like you to go ahead with DRABC protocol
*1st call for help
*put her on O2 mask 6-8 L/ min
*secure 2 IV large bore IV cannula
*take blood for all Ix  FBC, UCE, BSL, blood group and cross matching and coagulation profile.
*start infusing her Normal saline 20 ml/kg bolus till the BP becomes normal then put her on
continuous infusion.
*reassess her VS again after doing this.
 History: (all findings suggest uterine atony) (just exam case)
-just want to ask you few questions about her condition?
-can you tell me more about the bleeding? how much? Colour? Any tissues or clots? Is it smelly?
(1250 cc bleed)
-what was the mode of delivery? (Normal vaginal)
-any problem during delivery? was it prolonged? (had difficult delivery last 14 hours)
-what was the baby’s weight? Single or multiple? (4.2 kg, single)
-has the placenta delivered completely? (yes)
-any episiotomy done? Is it healthy?
-any complications during pregnancy?
-is she nulli or multiparous?
-PMH, PSH?
Advice and what she needs to do
-if you could perform abdominal examination for me?
*look if uterus is contracted or not (laxed in atony)
*if any uterine tenderness (No)
*if you could palpate the bladder

-ask the nurse to massage the uterine fundus


-start her on IV oxytocin 5-10 International unit IV as a bolus and then continuous infusion 40 IU/
hr.
-place urinary catheter

-proceed with pelvic examination


inspection
*bleeding
*if see cord outside the vagina (in sample case 2 cm cord outside vagina) if cord has been
clamped closed to introitus?
*episiotomy site
speculum
*laceration or tears in cervix

-is there bleeding anywhere else?

management
in sample case (please place one hand over the uterus and look for uterine contractions, once it
starts contracting give a gentle controlled cord traction and look weather you can deliver
placenta or not and if not possible shift her immediately to theater and specialist will do manual
removal of placenta under anesthesia.

In exam case
-Differential diagnosis
*uterine atony (most likely) causes could be prolonged labour, big baby, twin, multiparous
*retain product of conception
*laceration
*uterine inversion
-call specialist for manual examination under anesthesia
Feedback 12-4-2018
You are HMO in a rural hospital in duty for O&G ward.
nurse us calling you .A lady has delivered recently and now is bleeding. from your room to ward
take 5 min.before going there you want to order what is needed.
your tasks:
* listen to her explanation and take hx not more than 4 min.
* give advice and What she need do before you go
During the conversation via phone, pt has delivered a 4.2 kg baby, has had around 1250 cc bleed
and has had a difficult delivery 14 hours. a small episiotomy has been made and VS:
BP: 85/50, PR=120, others nl.
delivered placenta completely,no any bleeding from epi.
219-Encopresis
Feedback 18-9-2018 Encopresis (important)
bowel disorder –encopresis
10 year old boy soiling in his pants. 2 years history of constipation, few weeks history of soiling
Task: History, PEFE card, Dx, D/D and management
History: diet: don’t like vegetable and fruit

Pass stool every 4 days, big hard stool.

No previous history of anal fissure.

Just change school because moving, a bit of stress but is manageable.

Mother frustrated about his soiling and he is hiding his soiled underwear.

PEFE card: faecal mass palpable


Dx: encopresis and constipation
Management: explained the situation with a diagram
Laxative or enema to get rid of the faecal mass now. Patient asked which laxative, I said I will ask my
senior, but if laxative does not work we will use enema.
Long term management: diet: increase fruit and vegetable, make it colourful and change different type
Oral laxative for long time to soft stool and help the recovery of the bowl
Liaise with school teacher and family meeting to ease the distress.
It is not the boy’s fault so don’t punish him
Said it may take a few months for the condition to resolve.
Score: 5 Key step: yes/yes/no/yes Approach: 5, Hx: 5, Dx/D/D: 5, Mx: 5

Feedback 18-9-2018 Encopresis (Important)


You are a GP and You have to talk with the mother of a 10 years old boy. The mother is concerned because
the boy has recently started soiling his pants. For this reason the boy is always shy, he is losing friends at
school and doesn't wanna go out at all in recent days. His general health is good.
Your tassk is to
1. Take focus history from the mother.( you should take no more than 4 mins in this task)
2. Discuss the Diagnosis with proper explanation to the mother.
3. Outline management plan to the mother.

My Approach:
I assured the other that we are here to help and you are in expert hands as she was really worried. Then I
asked all history abot the boy. History of present and past illness, family history, diet, BINDS. Positive
findings: 2 years history of constipation, 2 weeks history of soiling pants and doesn,t like vegetables, only
eats chips and junk foods. So I drew a picture of the bowel and explained encopresis in easy terms.
In management plan I told we will first empty his bowel by any mean. So I talked about enema and then
lactulose. Then I talked about family support taht its not his fault. Then talked about diet. He has to have
more fibers in his diet. But I forgot to talk about bowel training and to encourage him to clear bowel
atleast 2/3 times daily. Then I gave 4Rs and finished the task.

AMC Feedback: Station 10: Bowel disorder Grade: Pass Global score: 4
Key steps: 1,2,3,4 yes,yes,no,yes Assessment domain: Approach to pt/relatives: 4 History: 4
Dx/DDx: 4 Mx Plan: 4
Handbook material Condition 091 Fecal soiling
Mark, a five-year-old boy, is brought to see you in general practice setting, because for the past six weeks
he has been soiling his pants, with increasing frequency, with foul smelling semifluid feces. It is now
happening almost every day and he is being teased at school.
His parent cannot tell you much about his bowel habits as he now attends to his own toilet needs when
he feels like it.
TASKS

1. Take a further focused history from the parent


2. Ask the examiner for the appropriate findings on examination of the child which would be relevant
to your diagnosis
3. Explain your diagnosis to the parent and advise on management
APPROACH

 [if parent is not calm] I know you have a tough time washing all of Mark's clothes, and I know you
are very concerned about him. But before we do anything to him, I need to ask you a few questions
first to assess him further, and then we can formulate a plan to help him. Will that be alright with
you?
 [if parent is just ok] I can see from Mark's notes that he's been soiling his pants recently. Can you
tell me more about it?
 HISTORY
o Fecal incontinence
 Since when is he soiling his underpants? How often? When did you first notice
it?
 How frequently does he usually open his bowels?
 Can you describe to me how his poop is? Is it loose or hard stools? Is it
associated with any blood or mucus?
 Does he complain of any tummy pain or pain elsewhere (anal pain) while
passing stools recently?
 If yes, Pain questions
 Does he have any previous episodes of constipation/hard stools/difficulty
passing stools/pain during passing stools? Any previous history of diarrhea?
 Is he toilet trained / I know he is toilet trained but did he have any similar
problems before?
 Did you have any difficulties during his toilet training before?
o Associated features
 Any nausea or vomiting or fever noted?
 Problems with his water works? Is he dry at night?
 How is his appetite? Any weight loss/weight gain?
o BINDSMA
 Birth (relevant): was he able to pass meconium (first stool) right after birth? /
do you remember if he had delayed passage of his stools after he was born?
 Immunization - not that relevant in this case.
 Nutrition: What does his diet usually consist of? Does he take a lot of fiber,
vegetables, and fruits? Does he drink a lot of water or not?
 Development: any concerns about his growth and development?
 School/Social: I'm sorry to hear that he's been being teased in school because
of this condition. Could you tell me more about it?
 If not sharing, you can reassure by using a confidentiality statement
 How long has this been going on?
 What does he usually do to cope with this?
 How is he as a student? Does he get along with his peers? Does he
enjoy school?
 How is your situation at home? Who else lives with Mark? How is his
relationship with him? Any concerns at home?
 Medications: does he take any medications or supplements?
 Allergies: does he have any allergies?
o PMH: Does he have any other medical or surgical illnesses?
o FHx: Any family history of bowel conditions?
 PHYSICAL EXAMINATION
o GA: PICCLED, does he look active/lethargic/withdrawn?
o Growth Chart**
o VS: temperature*, HR, RR, O2 sats
o FOCUSED ABDOMINAL PE: any distention or visible masses or peristalsis? Any palpable
masses or tenderness or rigidity? Any organomegalies?
o RECTAL EXAM with the parents' consent and with a chaperone?
 Inspection: any visible lacerations, fissures, bleeding, skin tags, discharge, fecal
staining?
 Per rectal exam: any palpable masses or tenderness? Is there blood on the
examining finger?
o OFFICE TEST: UDS, BSL
 PE FROM THE CASE
o A shy boy
o Normal height and weight on the 50th centile
o Abdomen is soft
o Fecal masses are felt in the lower quadrants
o No other abnormality
o Anus appears normal, with some fecal staining adjacent
o No anal fissure apparent
o On rectal examination, the rectum is packed with firm feces
 DIAGNOSIS AND MANAGEMENT
o From history and examination, it seems that most likely your child has a condition called
ENCOPRESIS, have you heard about it? it is a condition where there is an unknowing passage
of fecal material in the underpants.
o [ILLUSTRATE] The common cause is constipation or fear of pain during defecation. In
your child's case, a few months back he was constipated and had anal fissures which was
causing the pain on defecation. Even though the fissures were treated, David still fears that
it's going to be painful again if he goes to the toilet. So as he is refusing to go to the toilet, he
is now becoming constipated again. Do you understand so far?
o Because of this chronic collection of feces in the bowel, there is formation of liquid
stools and it tends to leak around the hard stools causing his soiling. However, the bulky hard
stools still remain inside, aggravating his constipation.
o Don't be stressed, it is a manageable condition. At this time the treatment is to empty
his bowel now by giving an enema. Once he empties his bowel, we can start him on stool
softeners.
o I also encourage you to start toilet training him again, and encourage him to go to the
toilet right after meals for a set period of time. You may use an egg timer for this.
o I can only imagine how distressing it is to wash his clothes every day but please do not
scold him. This takes a long time to get better, and needs a lot of patience on your part. You
can start a star chart to motivate him to empty use the toilet when necessary.
o I would also like you to talk to his teacher about his condition, and to address about
bullying. Please give him extra clothes which he can wear whenever he has soiling episodes
in school.
o Please do not scold the child, and talk to the teacher about bullying. This takes a long
time to get better and needs patience on your part.
o I will arrange a regular review with him to ensure that his constipation is not recurring.
o However, If the he develops severe pain, nausea, vomiting, or different pain from what
he feels now, please come back immediately for a review. Here are some reading materials
which can give more insight about your child's problem. Do you have any questions?
KEY ISSUES

 Explanation of diagnosis
 Initial emptying of the rectum and colon
 Need for prolonged treatment and follow-up

CRITICAL ERRORS

 Suggesting that sigmoidoscopy or colonoscopy is required at this stage.

IMPORTANT POINTS FROM THE COMMENTARY

 Majority of constipation cases are non-organic, that is they are related to an episode, in many
cases associated with the passage of a hard tool which may make the child wary of passing a bowel
motion subsequently
 Can be associated with a mucosal tear or anal fissure which distresses the child, further
compounding the problem.
o Recognize the pattern early and treat it with fecal softeners allowing the fissure to heal
 If the child remains fearful of going to the toilet because of anticipation of pain, this compounds
the situation which may lead to chronic constipation, toilet refusal, and in many cases, overflow
encopresis.
 Young children may develop fear of the toilet during toilet training if they are required to perch
on a toilet seat without support and this also may lead to subsequent constipation
 Constipation can also be secondary to emotional upset or trauma.
 AIMS OF MANAGEMENT
o Exclude any possible organic pathology
o Explore any precipitating features
o Provide adequate explanation of the processes involved for both parent and child with a
plan of action to alleviate the problem
 History SHOULD INCLUDE
o Thorough enquiry into the child's environment
 Diet, general health, growth pattern, the family dynamics, progress at school,
relationship with peers, and the like.
o Hirschprung disease at this age has usually presented with a history of constipation from
birth, often with a delay in the passage of meconium.
 If there is suspicion, referral to pediatric surgery should be sought to include
bowel biopsy
 Treatment
o Careful reassurance, explanation of the nature of the condition
o Comprehensive explanation about how constipation has developed, preferably with
illustrative drawings

Recall 10-5-2018
mother came with 8 to 10 year old boy as he was soiling his underpants. Mother is worried.
Task:
History
PE given on card
Explain condition with reasons
Management

220-Obstructive sleep apnea


Case 1 (5-5-2018)
Mum is concerned about son having difficulty breathing in hospital setting. The boy had cold
recently. Fine now
X-ray of lateral view head and neck given showing upper airway narrowing. Graph of SaO2
(polysomnography?) showing chart of sao2 going down to <92%
Tasks
-History (3 min)
-Pefe
-Explain x-ray
-Dx/ddx

History
1-breathing difficulty Qs
-What do you mean?
-What time day or night?
-Does he turn blue or pale?
-does he Stop breathing at night? Affect sleep?
-is he Sleepy during the day?

2-Associated symptoms
-Mouth breathing?
-snoring a lot?
-Runny nose?
-Recent viral infection, recurrent infection( tonsillitis or OM)
-fever?
-Cough?
-Poo& pee?

3- Others.
-Eating? Swallowing difficulty
-Growing?
-Performance at school and behavior?
-Hearing problem? (Does the child watch Tv with high volume. when call the child, does he
respond?)
-Contact hx
-Smoking at home
-allergy hx or Fhx
Pefe
1-G/A
-mouth breathing, adenoid facies
2-V/S
3-ENT
Ear- (TM congested, bulge, red and retracted +ve)
Nose (polyps, discharge, pale, swollen, boggy mucosa)
Mouth( tonsils enlarged, red and exudates)
LN – None

4-Growth chart
5-General system

Explain X-ray and diagnosis


-Point out the adenoids and nasopharyngeal airway and tell this structure here is enlarged called
adenoids, which is causing the airway to be narrowed.
-From history and Examination, X-ray provided with SaO2 chart, your child most likely having a
condition called OSA, where the child stops breathing for a short time when they sleep. It tends
to happen repeatedly during the night.

-Usually occurs due to the airways being blocked, and the cause most likely to be due to big
tonsils (soft tissues at the back of the throat to fight infection) or big adenoids (soft tissues at the
back of the nose to fight infection too)

-So this causing the child to have difficulty breathing mainly at night leading to mouth breathing,
snoring and recurrent infection and sleep disturbance with episodes of breathing obstruction.

Treatment
-Therefore, I would like to refer your child to e specialist ( ENT) who will decide upon the Mx.
Might consider taking the tonsils or adenoids out to relieve the obstruction. (key of all)

-Hearing test and speech therapist if problem with speech

-Avoid smoking exposure

Recall 5-5-2018
Mom concern about her son’s difficult breathing in hospital setting. The boy has had a cold
recently and is fine now. Take history, explain the XR which shows a narrow at the upper air way.
There is a graph for SaO2 and snoring related to low level of SaO2 you also need to talk about
that.
I came with the upper airway obstruction because of viral infection. It was case of obstructive
sleep apea with recurrent tonsillitis She said it was enlarged adenoids on lat x ray and oxymetry
chart was very low at night 70% On pe child was mouth breathing and big tonsils On mx refer to
ENT specialist

Case 2
Recall 6-9-2018
snoring in a child. Child had recurrent ear infections. Hearing not very good as per mother.
Speech good. Typical osa presentations

Recall 5-4-2018
GP, 4 year old girl b/in by mom coz of snoring a lot, she had recurrent ear infection as well.
Task:
take h/o not more than 3 min.
Ask PE from examiner.
Dx & Mgt.
On history, she had typical features of OSA, which affect her performance in child care. Difficulty
in swallowing +. Cannot hear well. BINDs good.
On PE, mouth breathing +, ear drums retracted, nasal d/c +, tonsils enlarged but no redness &
exudates, no cervical LN. systemic exam normal.

Material case.
A 5 year old girl with recurrent otitis media was treated with Amoxycillin by your colleague 3
times in 6 weeks.
TASKS
-History
-PE from examiner
-Tell the mother further management

Risk factors:
-ENT problems -Adenoid problems -Smoking at home -Pets
-Day care or school -Viral infections -Immunodeficiency states

Positive findings in the history: child has snoring and mouth breathing history. They come with
another relapse. Child is febrile; Positive findings in the PE: tympanic membrane bulging, red,

History
-How many episodes in the past 6 months? How long did the episodes last? In the previous
episodes, did the perforation heal, did the discharge come down? Did you have a review with
your GP? Was antibiotic given?
-What are the symptoms is the child having? Is there ear pain? Ear discharge? If with discharge
(type, color, amount, foul smelling, blood-tinged) sore throat? Fever? Runny nose? Headaches?
Rash?
-Any concerns about hearing? Any previous audiometry done? Does the child watch TV with high
volume? When you call the child, does he respond?
-Any complaints of the child about the child care?
-Any speech problem? Any learning difficulties? Any hearing problems? Any behavioral
problems? How is the school performance?
-Does he have frequent URTI? Any allergy? Any watering of eyes? Coughing? Sneezing?
-Any sleep problem? Does he snore at night? Daytime sleepiness?
-How is his eating and feeding? Is the child gaining weight or not?
-Is there anybody smoking at home? Do you have pets at home?

Physical exam
-General appearance and growth charts
-Vital signs
-Ear examination: tragal sign (if painful, otitis externa), any crusting in the external auditory
canal, how is the tympanic membrane, where is the perforation, type of discharge, mastoid,
hearing tests
-Throat: tonsillar enlargement
-Nose: polyps
-Lymph node examination

Management
From history and examination, your child is again having otitis media. Our ear has three parts,
outer, middle and inner ear. The infection is in the middle part as it contains pus. This is what we
call the eardrum and it is normally clear and flat. However on examination, it was found to be
reddish and bulging out indicating that there is an ongoing infections and it contains pus or
infected fluid. Ear infections are very common in children, and it is often preceded by a viral URTI
as was seen in your child. In your child it is happening frequently, so we all it as recurrent otitis
media. The child needs to take an antibiotic, but this time for a longer duration, which is
recommended at 4 months. But, as there is frequent ear infections, to rule out the causes of the
risk factors of the frequent ear infections, I will be referring you to the ear specialist and the
pediatrician. They might decide on continuing the antibiotics or not. In the meantime, do proper
ear toilette, no swimming for 2 weeks, and give Panadol for pain. Once the condition resolves,
we can do hearing tests to check his hearing. If there is speech problems, we can refer to the
speech pathologist.
If passive smoking history positive, please consider quitting as it is a risk factor for recurrent
infections in the child, and it will be beneficial for both you and the child.
I will review you after 2 days, then after 2 weeks again.
The causes of recurrent infections can be smoking or if the tonsils which are the glands present
at the back of the throat which is part of our immune system are enlarged, so the surgeon might
consider removing it. Like tonsils, there is also one more gland present behind the nose which
also can get enlarged due to repeated infections and cause symptoms. If that is the cause, it also
needs to be removed. Sometimes, the cause of the infection can be absent ventilation to the
middle ear part, so the specialist might decide to put a small tube in the eardrum which can help
in drainage of discharge and ventilation. It is done between the outer ear and middle ear.
However, it will be the specialist who will decide on the management of your child's condition.
And also, if the child is having snoring, daytime sleepiness, then we will refer you to the sleep
clinic.
Avoid smoke exposure, give pneumococcus vaccination, avoid group child care.
**Within 2 weeks of symptoms, not responding to antibiotic, or more than 6 months, it is
chronic otitis media.
**More than 3 episodes in the past 6 months or more than 4 in 1 year, it is recurrent otitis media

Case 3 (4-7-2018)
3.at GP-dad of 7yr old child comes to discuss about his son’s condition as his teacher told that
the boy is tired & easily distractable
Task
-History
-PEFE
-explain D/ Ddx
Outside –ADHD/OSA/anaemia

Hx -Started with ADHD sym-whether concentrating one task-not very positive history
So immediately moved to OSA hx
When ask about snoring-dad in loud voice agreed –that he sometimes hears from his room.
Effects of OSA sym
Day time sleepiness & tired /poor school performance-compared with his younger
sister/recurrent ear infection/hearing/speech
Poor diet, atopy hx +-allergic rhinitis, allergic triggers-pets/smoking
BINDS

PEFE- whatever you asked only will be given


Wt 10th centile( this was what I heard), ht 50th CLN+, b/l ear effusion
Enlarged adenoids & tonsil, nose –NAD, lungs-clear, other system-nothing significant
?pallor

D/DDx
OSA-explanied with diagram & in addition poor diet & anaemia
Need to refer to ENT ( forgot to mention about polysomnography)
221-Septicaemia
Septicaemia Handbook Material 113
You are an HMO in ED, 4 month old boy is brought to you by his parents at 3 am. The baby has been
unwell for past 24 hours.
TASKS
1. Further history
2. PE finding
3. Discuss the causes and management diagnosis

History
1. Ask for hemodynamic stability
2. Open ended questions: When did it start? How did it start? What was he doing when it
started? Is it getting worse? Has he ever had this before in the past?
What happened in the past 24 hours?
3. Well Baby questions:
a. Mental state of the baby: has he been really irritable, or has been hard to wake up?
b. Eating/Drinking: has he been eating and drinking fine?
c. Wet nappies: Has he not been producing wet nappies for the last 8 hours? (very
dehydrated), change in the number of wet nappies?
2. Differential questions
a. Rash questions
b. UTI: Any smelly urine or change in the color of the urine? Cry on passing urine?
c. Otitis media - any ear pulling?
d. Meningitis
e. AGE - change in bowel habits? Character of stool? Abdominal pain?
f. Pneumonia - fever, cough, colds, shortness of breath
2. Closure: BINDSMA (BIND - until 12 years old), Past history, Family history
a. Birth
i. Antenatal: what was your age when you had your child? Did you have any
infections or any medical condition during pregnancy? Did you take any drugs
or medication, or had any trauma? (these are important in heart diseases in
newborn, baby with vision problem, baby with hearing problem, child is
mentally retarded, child with developmental issues, child with slow school
performance)
ii. Delivery: was he a term baby? What is the mode of delivery? (any
instrumentation/forceps? [trauma, cephalhematoma leading to unconjugated
jaundice or prolonged physiological jaundice] What is the reason for the mode
of delivery? Did he require resuscitation or did he cry immediately after birth?
(Any premature rupture of membrane or artificial rupture of membrane?
[neonatal sepsis]
iii. Postpartum: did he spend any time in special nursery? Was heel-prick test
done?
b. Immunizations: Is the immunizations up to date? (Ask it is relevant in cases of fever,
rash, AGE, etc)
c. Nutrition: What is his typical diet? Any changes in appetite? (important in cases of FTT,
pancytopenia, developmental cases, slow school performance, constipation)
i. If infant: is he breastfed or bottlefed? When did you start weaning? Any
concerns about it?
b. Development: How do you think he is growing compared to kids of his age group? Are
you concerned his development when you compare him with kids of his age group?
c. Social history
i. Does the child have any siblings? Do they have the same symptoms? (fever,
rash, diarrhea, vomiting) Any family member with the same problem?
ii. Does the child go to school or child care? Any problems in school? How is his
behavior in school? Did the teacher say anything?
iii. Does he have friends in school? Is there any bullying?
iv. How is the situation at home? Are you a happy family?
b. Medications
c. Allergies
d. Past history
e. Family history

PHYSICAL EXAM
1. General appearance - alert/drowsy/irritable/lethargic
a. PICCLE
b. Growth charts (until 12 years old only)
c. Dehydration - CRT, skin turgor, sunken eyes, mucous membranes, anterior fontanel
2. Vitals signs
3. Systemic Exam
a. Musculoskeletal system
b. ENT
c. CVS
d. Respiratory
e. Abdomen
f. Neuro
2. Office tests

Explanation:
-From history and examination, your child has infection going on in the body. But in infants, we
cannot find out the cause of infection by symptoms and examination. So I will call the pediatric
registrar, and will admit the child and do investigations to find out the cause of infection. These set
of investigations we call it as septic workup or septic screen.
-We usually do blood test, we take a sample of the blood from the child's veins for FBE, UEC, blood
culture, to look for infection in the blood. It might cause a little bit of pain from the needle prick but
we will use a local anesthetic spray a few minutes before the procedure that will numb the area.
-To rule out infection in the lung, we will do x-ray of the chest. I understand that you might be
worried about possible risk of radiation will be tailored according to his age and weight to limit
overexposure. This test is very important to look for infections or pneumonias within the lungs.
-To rule out urine infection, in babies of this age group, we usually obtain a sample or urine through
aspiration from the tummy to look for possible infections. Please don’t be stressed it will be done
by a specialist. The nurse will give him some painkillers before the procedure. A very small needle is
passed through the skin into the bladder and the sample is withdrawn. Please don’t be alarmed if
you see tracs of blood in his urine after the procedure. It is totally harmless and commonly seen
after such procedure.
-To find infection in the brain, the other important test that I want to talk about is a lumbar
puncture where some fluid will be taken by passing a small needle through the space between the
lower spine. We will send this fluid for testing for infections. This procedure again I done by the
specialist. Sometimes, there is small amount of bleeding from the area. The child may be irritable
for some time but rest assured that it is unlikely to damage the spinal cord as the level is much
lower.
-We will start him on broad spectrum antibiotic once blood is taken for investigation. Specialist will
come and see. [If in country hospital and no specialist, transfer to tertiary hospital.]

Condition 113 Severely ill with fever


You are a junior Hospital Medical Officer (HMO) working in the Emergency Department. A 4-month-old
female infant is brought in by her parent at 0300 hours.
The baby has been increasingly unwell for 24 hours with the following history:
 Poor feeds — only one breast feed in that time.
 Feverish.
 Decreased urine output.
 Excessive drowsiness and difficult to wake for feeds.
 There has been no vomiting or diarrhoea, and no cough or wheeze.
The baby is the first born infant of healthy parents who are both currently well. She was delivered
normally at term after an uneventful pregnancy. She has been thriving and developing normally until now.
TASKS:
1. Ask the examiner for the clinical findings you would wish to elicit on examination.
2. Discuss the likely causes of the infant's illness with the very anxious parent.
3. Explain your plan of management to the parent.
No further history is required.
APPROACH
Physical examination
1-General appearance: what is the level of consciousness? Is he well-nourished? Any pallor, icterus,
cyanosis, signs of dehydration?
2-Vital signs: temperature, pulse rate, respiratory rate
3-Are the fontanelles soft or tensed?
4-ENT: what are the ear findings? Is the TM injected? Does the ear contain fluid or pus? What are the
throat findings? Is it hyperemic, inflamed? Is there any pus?
5-Are there abnormal lung sounds, heart sounds? Is the abdomen soft and non-tender?
Management
-From history and examination, I suspect that your child has an ongoing infection going in the body.
However, it can be difficult to determine the exact cause of the infant's illness without investigations as
babies present in a different manner from adults. I will call the pediatric registrar who will admit your baby
immediately for investigation and treatment. This set of investigations is what we call as septic workup or
septic screen to find out the cause of the infection.
-It involves doing a blood test wherein we take a sample of the blood from the child's veins for FBE,
ESR/CRP, and blood culture, to look for infection in the blood. It might cause a little bit of pain from the
needle prick but we will use a local anesthetic spray a few minutes before the procedure that will numb
the area.
-To rule out infection in the lung, we will do x-ray of the chest. I understand that you might be worried
about possible risk of radiation but rest assured that it will be tailored according to his age and weight to
limit overexposure. This test is very important to look for infections or pneumonias within the lungs.
-To rule out urine infection, in babies of this age group, we usually obtain a sample or urine through
aspiration from the tummy and submit it for urine microscopy and culture to look for possible infections.
Please don’t be stressed it will be done by a trained specialist. The nurse will also give him some painkillers
before the procedure. A very small needle is passed through the skin into the bladder and the sample is
withdrawn. Please don’t be alarmed if you see traces of blood in his urine after the procedure. It is totally
harmless and commonly seen after such procedure.
-To find infection in the brain, the other important test that I want to talk about is a lumbar puncture
where some fluid will be taken by passing a small needle through the space between the lower spine. We
will send this fluid for testing for infections. This procedure again is done by the specialist. Sometimes,
there is small amount of bleeding from the area, but this is expected. The child may be irritable for some
time but rest assured that it is unlikely to damage the spinal cord as the level is much lower.
-An intravenous line will be inserted as well and once the investigations and specimens are taken, we
will start him on broad spectrum antibiotics (cefotaxime/gentamicin) until the results of the tests arrive
for more specific cover.
KEY ISSUES
 Recognition of clinical findings indicative of acute life-threatening sepsis in infancy.
 Recognition of an extremely sick infant requiring immediate diagnostic workup.
 Recognition that investigation and treatment must proceed together.
 Recognition that parenteral (preferably intravenous) antibiotics (broad spectrum coverage to
cope with all possibilities) will be required.

CRITICAL ERRORS
 Failure to admit the infant to hospital.
 Failure to undertake urgent investigations.
 Failure to recommend immediate antibiotic treatment

IMPORTANT POINTS FROM THE COMMENTARY


 This scenario describes a 4-month-old baby girl who presents febrile and obviously very unwell.
The diagnosis is most likely to be severe bacterial sepsis with no signs suggesting the causal origin
of the infection.
 This infant requires urgent admission to hospital for investigation and treatment.
 In this situation a cause of overwhelming sepsis must be sought and this will involve a full work
up of investigations to determine the probable origin and spread of infection. This will include full
blood examination including blood culture and inflammatory markers, chest X-ray and examination
of urine, preferably by suprapubic aspiration, and cerebrospinal fluid via lumbar puncture.
 Infants present in a very nonspecific manner when infected: physical signs that may be present in
older children, for example neck stiffness, are absent. The medical attendant therefore must seek
out multiple possible infection sources. Having obtained these investigations rapidly, the safest and
most appropriate action is then to treat the infant with antibiotic therapy preferably intravenously,
providing a broad cover for appropriate organisms (e.g. cefotaxime and gentamicin). There is no
place in this situation for observation while awaiting results as the infant may deteriorate rapidly
and irretrievably. Under most circumstances one or other of the above investigations will reveal the
source of infection. Young infants become ill quickly with bacterial infection and will deteriorate
rapidly if not investigated and treated urgently. Appropriate treatment often gives rapid
improvement.
Case (23/2/2017)
GP, 4mth baby by mom, Fever, lethargy, not feeding well. You have examined the patient: T’
38’C, CRT 3 sec, ant fontanelle N, no rash, ? subcostal recession, BP, SO2 not done.
Tasks:
1.Hx,
2.Likely Ddx,
3.Mx.

Entered room. Greeted by examiner.


Greeted and introduced myself, asked about her child’s condition.
Since when, any medication given,
vomiting,
any change in nappies, any rash,
how was his appetite.
Any family members with similar symptoms (elder sister with URTI).
BINDS (missed 2 months immunisation, the child was having fever at that time and did not catch
up with immunisation).

DDx: Dear Cindy, there could be various causes for your child to have fever and not feeding well.
It could be infection to the urinary tract, covering of the brain, lungs etc. It can be a serious
condition called meningococcemia even though there is no rash however the rash could appear
later.

Mx: As your child is not feeding well and his condition, I would like to arrange an ambulance and
send him to the nearest ED. ( mother freaked out when I mentioned need to admit to hospital).
May I know why do you refuse to send your child to the hospital, was it due to some experience
you or your child had before? (It was my older daughter, she had gastra previously and was sent
to hospital, there was a lot of people coming to see my daughter and social worker… I do not
want my child to be taken away).
Alright, I understand your concern that your child might be take away. Could you please tell me
whether there is any financial issues at home, and is everything fine at home? (everything is fine,
no problem)
Let me reassure you that nobody has the right to take your child away from you. The reason why
we are sending your child to the hospital is to have a thorough assessment on his condition. As
you can see in the surgery there is limited equipment for me to treat him. Don’t worry, I will call
the doctor at the ED and will let him know your child’s condition. When you reach there, your
child will be seen by the doctor whom I talked to. There they will take some blood investigation
to find out the cause. If your child need to be admitted to the hospital, you will be able to stay
with your child, nobody is going to take away your child from you. We are doing the best for your
child. Are you with me so far? (yes doctor)

Bell rang.
Passed. Global score 4
Improvement: take FBE and blood culture in GP, commence IV fluids and antibiotics (cefotaxime
and gentamicin) until diagnosis confirmed.

Feedback 4-7-2018
2. suburban GP clinic-mum with 4/12 baby( doll)- Drowsy, pale
PE detail given-high temp, increased HR & RR CRFT>3, no rash
Task further hx & mx

Hx-fever-duration
detail infective foci-ENT, cough, rash, gastro sym, urine less feeding & nappy wets
BINDS-not immunised –when inquired –gave some reason-couldn’t remember now
No contact hx

Mx-explained life threatening septic condition-need urgent admission


Mum said not willing to go to hospital as had bad experience in the past
Need to reassure & stressed life threatening septic condition again & undesirable outcome if not
admitted

Will orgainse urgent transfer & inform paeds team Urgent bloods FBE. Inflamarker/culture Broad
spectrum ab-
Further septic screening-urine m/c/s, CXR, LP

Once baby recovered will have a followup review & discuss immunisation.

Recall 9-2-2018
A mum with severely ill 4 weeks old baby (feverish, blue & cold hands , no rash, oligourea ) and
when you advice for hospital admission, she will refuse because of some incidents happened
before with the social worker who stayed with her daughter and she was blamed that she is
not taking care from her daughter
Feedback 2018 (IMP)
Fever - PASS
Suburban Hospital*** (which I got totally wrong in trial exam :P); 4 month old baby brought into the ED
this morning by mom. A long list of symptoms and signs suggestive of septicaemia was given but no
rash, chest is clear. Fever – 39; no feeding well and looked lethargic. Vitals are stable.
Task: History (2/3 mins)
Explain significance of findings
Further management

2 min thinking: to find source of infection; to explore home situation; to arrange urgent hospitalization
(transferring to tertiary hospital as you are in suburban hospital)
History
Going into the room, I noticed mother looked dejected. So I knew that this was the case necessary to
arrange social workers for the children left at home.

Symptom analysis – when (last night), first time (apart from a few flu, yes), GH (good), BINDS
(immunization was up to date); poo and pee (the number of wet nappies were reduced)
Psychosocial history – been living with her sister; has 2 other girls aged around 2 and 3; divorce came
through when she was pregnant; financial problems (+); mood – struggling but trying her best (so, no
depression, I guess);

Explanation
The reason for his condition is due to the presence of bugs inside the blood stream. (didn’t say
septicaemia) In fact, this is a serious conditon which requires urgent admission and treatment by
antibiotics through veins. As of now, the baby is stable but he looks lethargic and urine output has
reduced. For now, couldn’t find out the source of infection, probably in the chest or urine or somewhere
else.

Management
So, to manage this, will arrange ambulance and refer to the hospital.
The patient refused admission – “I still have two daughters at home. My sister is quite busy, too. Can you
just give me some antibiotics and paracetamol?” – well, i can see your concern but right now the baby
condition is a serious condition that will only get better with hospital treatment. For the girls left at your
home, would like to arrange social workers to take care of them for the time being. “No, no, doctor. I don’t
want them to. I will just take care of my daughters myself” –any reason? Any bad experience with social
workers? “Yeah, previous time, when I asked help from them, they blamed me for poor parenting” –well,
that’s quite unfortunate. In that case, I would like to get involved this time to make sure that this wont
happen again. The patient agreed. Told her before transfer, I will do some tests FBE; blood culture; urine
test; x ray (with reasons) and give the first dose of antibiotic while waiting for ambulance. At hospital,
specialists will be there to treat your baby. Although this is a serious conditon, babies can definitely
recover very well without having long lasting effects -blah blah blah-

GS – 5
Key steps 1, 2, 3, 4 – all covered
Approach – 5
History – 5
Dx and DDx – 5
Mx plan – 5
Summary/ the stem which is the feedback one point to child with fever and poor feeding so you obviously
will know most likely emergency case
So first step would be to do hemodynamic stability and shift the patient to treatment room and take
history from there.
The history will be as he has fever so I will ask about duration, constant or intermittent, shivers, chills then
rash imp.
After that ask dehydration questions like lethargy and drowsy, feeding, and urine output.
then DDx like GIT (tummy, pain, vomiting, diarrhea) respiratory (cough, SOB, noisy breathing)
then BINDS but just brief birth like how was your pregnancy with your child any complications just 3 or 4
questions then immunisation, feeding already asked, development then few social questions along with
travel and contact.
Explanation and DDX will be like befor, I will talk first about septicemia then causes like UTI, chest
infection, meningitis, etc…
Management again is to take Ix and the imp is blood culture and start antibiotics and imp is allergy
then send to ED to be assessed by specialist then talk about other Ix like urine, chest x-ray and lumbar
puncture like before.
So the most important step is sending the child to hospital ED here. The mother will ask you a question
that she does not want her child to be sent because of previous experience so just warn her about the
emergency of the case and it is important to look for infections and no one will take her child from her and
like feedback 2018 talk. do not prolong the conversation like the first 2017 feedback just answer her
briefly.
Then once discharged and recover follow up the child and discuss immunisation
222-medication non-compliance (Sertraline)

young male, came to your GP 2 weeks ago, was diagnosed with depression, started on sertraline
50mg to be increased to 100mg after 1 week. Now the patient has come for regular follow up as
advised by you
Tasks
-Take relevant history
-Your management with reasons

History
1-Approach
-Confidentiality
-How do you feel now?
-do you think your condition is getting worse?
2-Medication questions
-do you take the medication regularly at the prescribed dose? (No, I do not take the medication)
-may I know the reason why you are not taking it? (I heard it cause side effects) and may I know
from where did you hear that (my sister told me and I googled it Sister told that I won’t be able
to sleep)
-so for How long did you take it? Have you ever had any SE?
-any problem with cost? (No)
3-good psychosocial history
-How is your mood ? Any time you feel high?
-have you lost interest in things you used to enjoy?
-have you ever thought of harming yourself or others? Any plan? any previous attempts? (has
suicidal ideation but no plan or attempt)
-how is your sleep?
-how is your appetite?
-have you lost weight recently?
-do you find it difficult to concentrate on things? Do you feel guilty?
-do you feel, hear, see things that others do not? what does it tell you?
-do you think someone is following you or spying on you?
-do you think someone is trying to harm you?
-do you think you need a professional help?
- let us suppose there is fire in this room, what would you do?
4-HEADS
-who do you live with? Any stress? Support? (live with sister and supportive)
-what do you do for living? Any stress?
-SAD (take 4 glasses wine / day so advise about safe limit and another consultation)
-social activities
5-General
-previous and family hx history of mental problems/ PMH/ PSH/ other medications

Counselling
-well john, let me assure you that any medication we start we do a risk benefit analysis and the
benefit of taking sertraline outweighs the risk.
-if you do not take the medication, your depression can worsen. You might have to struggle with
the activities of daily living. You can develop suicidal ideation. You can neglect your food and
drinks which again could be life threatening so it is always better to take medications at the
prescribed dose.
-Each medication has side effects but if you feel concerned about any, feel free to come and
contact me. We can reduce or change the medication accordingly.
-Usually, antidepressants takes 2-4 weeks to work.
-are you taking CBT?

Management
-NO ADMISSION (as just suicide idea and also has support)
-refer to psychologist for CBT (if no CBT) and also a good idea to refer u to psychiatrist to review
your medication and teach you some methods to overcome your stress
-life style modifications
-family meeting with consent
-support group.
-see you regularly and that you can always come back to see me if there are any issues with
taking the medications and we will help you.
Case (11/2/2017) (suicide ideation)
A man with depression, seen by your colleague a week back. Had anhedonia, feeling of
hopelessness, sleep problem, low mood. He was prescribed Sertraline 50 mg, to increase to 100
mg after a week and was planned to be seen after two weeks but he is back within a week.
Tasks:
1)to assess his improvement?clinical condition?
2)to address his concerns
3) to manage accordingly

For this case I appreciated him coming in. Asked how he was feeling. He volunteered info kind of
readily.
- Ensured confidentiality before that.
- Asked him how come he came back with him early? Is the medication helping? He said he did
not take.
- Ask non judgementally, why not taking, what are his concerns. He says sister took before n
had some issues ( dependance i think), did google search n had SE.
- Asked what did he find out.
- Also asked about the symptoms mentioned in the history one by one, any changes.
- Asked if he took anything else like alcohol. He said abt 4 glasses of wine. I acknowledged that
but I don’t remember what I said to him. I did not press on about the alcohol because I did not
want to derail from the main task.
- ask about suicide. He also mentioned that he has ideas of suicide but no plans or actual
intentions.

Management
- Appreciated him being honest with me.
- Told him that medication take like 2 weeks to work,
- assured that serious SE is rare. We start with low dose n increase or decrease as needed.
- Benefit of feeling better outweigh risk of taking this med.
- i didn’t mention admission.
- I spoke about seeing him regularly and that he can always come back to see me or his previous
doctor if there are any issues with taking the medications and we will help him.

Feedback: Mood change, Pass, Global score 6


Feedback 5-7-2018 Unknown status
young male, came to your GP 2 weeks ago, you diagnosed him with
depression, started on sertraline 50mg to be increased to 100mg after 1
week. Now the patient has come for regular follow up as advised by you
Task
Take relevant history
Your management with reasons
Patient looked depressed , looking down with low tone speech. And
scratching his fingers.
I devided my assessment into two parts, mental state and progress and
second part is medications and side effects.
Mental state has not changed still suicidal , talked about not being
approced for promotion. On medications assessment he divulged that he
did not take his medications. He went to pharmacist with the prescription
but didn’t buy the medication.
Asked about medication compliance dividing it into : doctor (everything
explained to you and why you need it? – yes) Patient (do you think you
need it – yes but I heard it has lots of side effects. From where did you
hear? My sister told me and I googled it too, it says that I wont be able to
sleep if I took it.) Is cost an issue? – no
You have support? Yes my sister Do you live alone ? yes SAD – no

said it’s good you have your sister around. There is no need to increase
dose as you haven’t taken it in first place. Antideppressants are not magic
pill. It needs around 4-6 weeks to work.. try to be active with good
lifestyle. Meet your sister and friends for good support.
However, I have to admit you to the hospital. as you are at risk of suicide
and I’m worried about you. He didn’t mind,. You will be looked after by
mental health team consisting of psychiatrist, psychologist (talk therapy),
mental health nurse)

Feedback 5-7-2018 pass


Pt with depression start on strapline 2 weeks ago come for review.
Task: 1- Hx 2- Mx
In the beginning the pt refused to talk, it took a lot of reassurance and confidentiality to agree to
talk, I discovered he is not taking his medication, at this point, in divided my hx into metal status
assessment (ASEPTIC, HEADSSS) then checking about the medication (cost, side effect). On Hx
the pt has suicidal ideation, yet no attempts, he lives with his sister. On Mx I told him, it’s a good
thing u have ur sister to look after u, yet u need to start the medication because it will help u to
overcome ur stress.
I understand u r afraid from the side effect, however we r using the medication in the low
effective dose, increasing it gradually to minimize the side effect and we r going to follow up ur
condition closely in case u developed and SE we can change the medication. u need to know that
I takes some time to kick in around 2 weeks, in order for u to feel better.
However right now u have this suicidal ideas, so it would be a good idea to refer u to psychist to
review ur medication and teach u some methods to overcome ur stress, I asked him if he is
happy to go with the plan and he didn’t mind
Passed: global score 5

223-Depression on Paroxetine
27 year old man visits your practice with anxiousness and restlessness. You have just completed
an examination and found no abnormality. 2 weeks back he was started on paroxetine by your
colleague as he was diagnosed with depression.
Tasks:
-Take psychosocial history
-Explain patient your diagnosis and other causes for his symptoms

History
1-Approach+ chief complaint question
-Confidentiality
-I can see that you have restlessness and anxiety. Can you tell me more?
-for how long?
2-Medication questions
-do you take the medication regularly at the prescribed dose? (Yes)
-Have you ever had any SE?
3-good psychosocial history
-How is your mood ? Any time you feel high? (Feel depressed) for how long? (3 months)
-have you lost interest in things you used to enjoy? (yes)
-have you ever thought of harming yourself or others? (I wouldn’t hurt himself, no particular
reason for that)
-how is your sleep? (disturbed)
-how is your appetite? (no appetite)
-have you lost weight recently? (yes)
-do you find it difficult to concentrate on things? Do you feel guilty?
-do you feel, hear, see things that others do not? what does it tell you? (No)
-do you think someone is following you or spying on you? (No)
-do you think someone is trying to harm you? (No)
-do you think you need a professional help?
- let us suppose there is fire in this room, what would you do?
4-HEADS
-who do you live with? Any stress? Support? (lives alone, divorced, no friends or support)
-what do you do for living? Any stress? (lost job)
-SAD (increasing alcohol recently)
-social activities (not socially active)
5-General
-previous and family history of mental problems (key questions) (father hanged himself a year
ago) I’m sorry for your loss.
-What effect did it have on you? (I have been feeling really low after that).
-Do such thoughts ever cross your mind or any current plan? (No plan but I have a sleeping
tablets under my pillow and has thought taking those to end it all) I am very sorry you feel this
way.
-PMH/ PSH/ other medications

Explain diagnosis and other causes


-from history and examination, you most likely have major depression which is a mental
condition where you can feel hopeless for a period of time with lost of interest in activities you
used to enjoy and LOA, LOW, disturbed sleep.
Cause is unknown, could be due to chemical substance in brain called serotonin.
Your father’s death, loss of job, breakup all may have contributed to your feelings.

-your symptoms could also be due to medication side effects which we need to review.

- As you are having some suicidal thought and plan. I am concerned about your current
condition. Would like to arrange an admission to the hospital for psychiatrist evaluation. (Most
of feedback the patient will accept straightaway)

**If not
Doctor, is this necessary, I am worried about the stigma from others
-I understand you are worried how others view about your condition, but let me reassure you
that depression is a kind of illness that can be treated.
-There are lots of awareness campaign in order for the public and community to understand this
condition.
Case (11/11/2017) Mood change
27 yr old man visits your practice with anxiousness and restlessness. You have just completed an
examination and found no abnormality. 2 weeks back he was started on paroxetine by your
colleague
Tasks:
Take psychosocial history
Explain pt your diagnosis and other causes for his symptoms

Introduction
Role player spoke in low tone and appeared flat
I completely forgot about restlessness and anxiousness
- After confidentiality statement
- He said that he has been feeling hopeless since 3 months,
- lost his job,
- feels alone, no friends, recent breakup, not social,
- doesn’t find life worth living,
- lost weight, no appetite, sleep disturbed.
- When asked about suicide- said he wouldn’t hurt himself, no particular reason for that.
- He has increased alcohol consumption recently, smokes cigarettes, no other drugs.
- No past history of mental illness.
- When asked about family history of mental illness- He said father hanged himself a year ago. I
said – I’m extremely sorry for your loss. What effect did it have on you? He said he has been
feeling really low after that. At this stage, I asked again- Do such thoughts ever cross your
mind? He said that he has a stash of sleeping tablets under his pillow and has thought taking
those to end it all.
I said- I’m very sorry you feel this way. I think you may be very unwell and medically this is called
Major depression. You are particularly serious because of your suicidal thoughts.
I further explained how his father’s death, loss of job, breakup all may have contributed to his
feelings. I asked him how he feels hearing this. He said he understands.

I told him we would need to admit for psychiatrist evaluation.


He said that he doesn’t want admission. I explained that I have legal obligation to do so as he is
very ill. He just nodded his head and looked down.
When I further spoke about treatment, the examiner stopped me and said to look at the stem
and stick to the task. I dint get the hint and completely forgot about paroxetine.
I think the other causes in the task was to explain briefly the side effects of antidepressants.
Key steps 1,2,3,4- Yes
Global score: 5 ( Pass)
Assessment domain
Approach to pt/relative: 5
History: 5
Diagnosis/ differential diagnosis: 5

Case (23/2/2017) (suicidal plan)


GP, young female, not feeling well ( forgot the phrase used ). Seen by previous GP who gave
Paroxitine.
Hx & Mx ( suicidal +, plan +, sleep problem, low energy & concentration, taking medication, no
side effects ).
passed, global score 5 (approach 6, Hx 5, DDx 4, all key steps Yes).
- I asked Hx of suicidal (yes), any plan (yes)
- mood (depressed), how long (3 months), anything happen during that time (lost her job), sleep
(bad, woke up early and could not sleep again), appetite (reduced), enegy level and concentration
(reduced),
- any partner (yes, but not in good terms, he has prob as well).
- whom does she live with,
- SADMA. ruled out PTSD, GAD, anxiety.
- Past psychiatry hx: asked about whether she is taking her medication (yes), does it help (get
worse).
Mx:
told her most likely she is having major depression. (forgot totally about the recent loss of job
that can aggravate her sx).
As she is having suicidal thought and plan, I am concerned about her current condition and would
like to arrange an admission to the hospital.
- (Doctor, is this necessary, I am worried about the stigma from others) I undestand you are
worried how others view about your condition, but let me reassure you that depression is a kind
of illness that can be treated and cured.
- There are lots of awareness campaign in order for the public and community to understand this
condition.

Feedback 10-5-2018
what was the case of pt with tremor or side effect? what eas the complain and task ? what was
your aprroach ?
So the paroxetine case question was: patient presenting with anxiety and restlessness. He was
started on paroxetine 2 weeks back as he was diagnosed with depression.
Task: take history
Explain diagnosis to patient
I started by asking a little more abt the presenting complains. Then asked if any other side
effects. I asked all symptoms of serotonin syndrome. Asked abt medication compliance. Then
took psych history.
Then I explained abt depression as his psych history was all positive for depression. No active
plan for suicide but he has sleeping pills under his bedding and sometimes he feels like taking
them. So in diagnosis medication side effect, depression and serotonin syndrome.

224-Schizophrenia relapse risk assessment


Middle age woman brought in by police to ED wandering in the street and got laceration in the
forehead with band aid. The wound is ok. She also claimed she needs to save the world. She has
a history of schizophrenia.
Task
-Assess her suicide risk
-Present this to the patient

* Inside the room was a lady with dishevelled hair and looked distracted.
1- I started by maintaining confidentiality
2- Psych hx
Mood:
-how’s your mood? How do you feel? (She told she felt wonderful as God was telling her to save
the world.)
Hallucination:
-Can you tell me about God, has God told you how to save the world? ( no )
-Any command of harming yourself or people? (Oh no never)
-do you hear other voices? (no)
-do you see God (no she admitted she felt special as God was inside her brain)
Delusion
-do you think someone is following you or spying on you?
-do you think someone is trying to harm you? (yes people are plotting against me)
-do you believe that you have a special power? (Yes)
-do you think thought are inserted in your mind? Withdrawn from your mind?
-do you think your thoughts are broadcasting through TV or radios?
-do you think other people’s action or behaviour are directed towards you? (Yes)
Rest of mood questions:
-have you ever thought of harming yourself or others? (No At this particular question she said I
want to save the world not to kill them).
-Any plan or attempt? (She had no plans to end her life nor anybody else's* She didn't have any
previous attempts at ending her life, nor did she have any intentions or plans at that moment.)
-Sleep, appetite: (She hadn't slept because she wanted to save the world.)
Cognition
-where are you right now?
-what time is it?
Insight: do you think you need a professional help? (No)
Judgement: let us suppose there is fire in this room, what would you do?
3-HEADS
-where do you live with? With whom? (She used to live in a hostel which she left because she
didn't like it in there and lived on road with God then)
- if ever had a partner? (Had a partner that had left long way before and didn't want to discuss
which I said was okay because I respected her wishes.)
- Any support? (She had no family support.)
-SAD (About drugs she said she didn't have any money to afford otherwise she would drink all
the alcohol in the world and smoke all the pot/ marijuana and heroin.)
-social activities
4-Schizophrenia hx
-since when? (She was diagnosed with schizophrenia 15 years back)
-do you take your medication? (No)
why you stopped taking the medication? ( because she thought she was fine and didn’t need it
and God will heal her)
5-General
-previous and family hx history of mental problems/ PMH/ PSH/ medications.

Explain to patient
-I am concerned because at the moment you are not in the right state of your mind. You are out
of touch with reality.
-Because of your disturbed thoughts, being alone as you do not have anybody to take care of you
and the thought to abusing drugs, and with a previous history of mental sickness, you are at a
high risk for harm to yourself and others or you may accidentally harm yourself or others
because of your disturbed mental state.
-So according to Mental Health Act, I need to admit you to the hospital so that you'll be assessed
by the team of psychiatrists who are specialists of mental health. Are you okay with this? She
said okay.

Another feedback:
-You have a history of schizophrenia which was controlled with medication. You stopped taking
the medication, you started to hear God again and can save the world and feel people plotting
against you.
-You feel you don’t have support and living in the street. You take marijuana.
-So I need to admit you to hospital to be seen by specialist.
Feedback 6-6-2018
relapse schizophrenia case – fail
Patient is middle age, brought by police to ED, wandering along the street and got lacerations to
the foot. Dressing done, wound is ok, he also claimed that he needs to save the world. He has
known history of schizophrenia.
Task: history

explain the condition/risk to the patient.

(Forgot to ask the patient about his wound and how he get it.) Patient denied he had
schizophrenia, not on any meds currently, god tells him to save the world, auditory hallucination
present, no visual hallucination, delusion present, lives alone.

Feedback 14-3-2018 pass


A female patient previously diagnosed with schizophrenia now brought by police. She was
homeless and wandering, living on streets.
Tasks,
Do Risk assessment
And tell the patient about her mental health.
2 min thinking.. Risk assessment questions SAD PERSONS
Ask questions for depression, psychosis, SADMA, HEADSSS (pardon me for so many mnemonics.
But passing psychiatry cases is all about this, so that you don’t miss on important points).
Inside the room was a lady with disheveled hair and looked distracted. I started by maintaining
confidentiality and asked how was she feeling and she told she felt wonderful as god was telling
her to save the world. On asking she admitted she felt special as God was inside her brain. She
didn’t see him or anything else nor did she hear any voices apart from that. She had ideas of
reference, but no thought insertion, broadcast or any guilt. She hadn’t slept because she wanted
to save the world. She wasn’t depressed rather euphoric. She had no plans to end her life nor
anybody else’s. (At this particular question she said I want to save the world not to kill them).
She didn’t have any previous attempts at ending her life, nor did she have any intentions or plans
at that moment. She used to live in a hostel which she left because she didn’t like it in there and
lived on road with god then.
About drugs she said she didn’t have any money to afford otherwise she would drink all the
alcohol in the world and smoke all the pot/ marijuana and heroine.
She had no family support. Had a partner that had left long way before and didn’t want to
discuss which I said was okay because I respected her wishes.
She was diagnosed with schizophrenia 15 years back but stopped taking medication because she
thought she was fine and didn’t need it.
I was running out of time and couldn’t ask about orientation and judgement .
The weird part was explaining it to the patient. So I told her that I’m concerned because at the
moment you are not in the right state of your mind.
You are out of touch with reality. Because of your disturbed thoughts, being alone as you don’t
have anybody to take care of you and the thought to abusing drugs, and with a previous history
of mental sickness, you are at a high risk for harm to yourself and others or you may accidentally
harm yourself or others because of your disturbed mental state. So according to Mental Health
Act, I need to admit you to the hospital so that you’ll be assessed by the team of psychiatrists
who are specialists of mental health. Are you okay with this? She said okay. Only for examiner I
said ‘that’s good otherwise we would have admitted you involuntarily. She said she was fine with
it. Bell rang!
Scenario … Behavioural Disturbance
Grade… Pass
Global score… 4
Key steps 1,2,3,5 … yes 4, No
Approach to patient… 4
History…. 4
Diag/ D/D…. 4

Case (29/3/2017) pass


Relapse of Schizo lack of compliance hearing god voices, middle aged woman
hearing God's voice,stopped her meds by himself 6 months back.
Tasks: Hx and risk assessment
2 mins: confidentiality
Confidentiality, consent to ask sensitive questions
What can I do for you- the police have brought me here. I am alright but.
(History)
- Organic dds-
any weather preference, loss of weight, bowel and bladder movements- no
head injury- no.
- When was she diagnosed with schizo- 10 years, back.
- she heard God talking to her,
- taking tablets but stopped them 6 months back on her own as she thinks that she does not
need them, and she now hears God again telling her to save the world, did not see the specialist
in a while.
- no side effects to the medicine
- Past h/o of mood disorders- no; but family h/o positive
- Home sitn: hostel, with no family members,
- Employment- not working, on disability pension, no financial crisis
- Activity- just hearing God talk to her; appetite- normal.
- Drugs- yes for a while; others – no
- Sex- not interested
- Sleep- normal
- Any thoughts of harming yourself or others- no
- social- no contact, no friends
- Emotions- jubilant.
(schizophrenia history)
1- Do you think you can see or hear things which other people cant- God talking to her; no visual
hallucinations
2- Do you think you have special powers- positive.
3- do you think that other people are plotting against you- positive.
4- Do you think tv programmes have special meaning,
5- are you guilty about something- no.
6- Do you think someone is controlling your thoughts- no
Do you think you need help- no
Do you know whom you are talking to now- doctor
What would you do if this room catches fire- run away.
Risk assessment
- you are hearing God’s voice again, previously diag with schizo and not taking med regularly.
- relapse of schizo most probably and are living alone, doing drugs and you are out of touch with
reality as you think you have special powers and people are plotting against you- hence I have to
admit you if needed even against your will and call the specialist.
Global score- 6

Case (13/10/2017) (it is not MSE so it is a bit of misleading do not know how he passed)
A young man with hx of chronic schizophrenia came to ED. Not taking his medication as he think
God is inside him.
Task:
take psych history tell the patient your psychiatric assessment as well as his risk assessment.
He was great actor! Hardly could I stop myself smiling when he was deepened in his role! I took
Hx.
No LOA NO LOW NO sleep problem Mood was elated and he said I am happy that I can save the
world.
Auditory Hallucination was positive as he could hear the God talking to him no one else.
No tactile hallo was there no visual.
He was taking Marijuana but could not afford buying alcohol and smoking.
Delusion of grandeur and persecurity as he was thinking that they want to prevent him from
helping God (who?? He did not know…)
no reference no broadcasting delusion.
No suicidal thought no plan no mean.
He did not medical help and if find fire in the room he will remove it to other place by God's
hand!...
Cognition was intact.
No weather preference no constipation no head trauma no CP no heart racing or SOB…

For MSE I was confused how to tell the patient all the jargons so I did it whatsoever ganna
happen! I had no time left.
I said well-dressed well-groomed! He was not as other said!:(…
behavior is cooperative and maintaining eye contact.
Speech tone and rate and rhythm is not affected.
Mood is elated congruent to the affect.
He has auditory hallo but no tactile or visual.
Thought form , there is a flight of idea but no tangentiality. Thought content involved persecurity
delusion as well as grandeur.
Rapport is OK.
Reliability is OK still I need collateral hx.
Insight is abnormal and cognition is intact and judgment is poor.
Risk is moderate to severe as the pt is hallucinated with no insight and doesn’t take his
medications. That is why I want to admit you in hospital and call mental health team to come
by and visit you! Bell rang!
Feedback: Behavioural disturbance, PASS(G.S:4)
Key steps:1 No2 ,3,4&5: Yes
Approach to patient:3
Hx:4
Dx/DDx:4

225-Major depression Fraud with NO suicide


Case (4/10/2017) Mood disorder- Pass
You are a GP. Patient is a 30-year-old lady, who has had an enquiry in workplace regarding a
fraud, was cleared already. Now complains of feelings of guilt, low mood, LOA, early morning
waking, etc. for 6weeks. (Enough lengthy details were given to definitively diagnose major
depressive episode.)
Tasks:
Take relevant further history
Explain your diagnosis to patient
Explain management to patient
Take relevant further history
Role player was much older looking lady, observed - normal appearance/ behaviour/speech.
Gives ready answers in detail.
- I asked few points to elaborate on her mood and symptoms such as sleep (wakes early
morning-cannot fall back into sleep)/ appetite/ anhedonia (present), not enjoying friends
company, do not go out/ still strongly feels guilt despite no case against her anymore/ problems
at workplace? Cannot remember all her replies.
- Most importantly - has she ever had suicidal ideas/ plans or past attempts? No/ Any passing
thoughts of same? (I sometimes wish I never wake up from sleep...but never thought of killing
myself.)
- Any family history of suicide? No
- No psychotic features...Not excessively anxious.
- SAD negative.
- Lives alone. No family support. No partner. Financially ok, but reluctant to work now.
- She generally gave me the idea that she has good insight into her condition. I appreciated that
and the fact that she came looking for help..
Explain your diagnosis to patient/ Explain management to patient
Explained most probable diagnosis as ‘major depressive episode’ possibly triggered by the
events at her workplace.
Explained her symptoms and how all these points to this diagnosis and explained about
pathophysiology a bit.
Very common condition and effective treatment available.
You are not going to feel like this all your life.
I do not remember if I gave any DDs here or whether it was in the tasks.
Treatment
involves SSRI and psychologist referral for CBT.
Stressed that SSRIs take at least 2weeks to show any change in condition, in the meantime her
symptoms may escalate.
She may need psychiatry referral later, but I will also follow her up frequently and regularly.
Told her that I am worried as she lives alone and no support available (faintly remember she said
she had a friend who lives near). Gave her a number that she can call 24/7 if she has any suicidal
ideas and need help/ she can go to nearest ED.
Alert her friend also. Stressed that psychotherapy is also very effective.
3 out of 4 key steps were covered.

Feedback 4-7-2018
Accountant(lady) found fraud then cleared with ix. Feels guilt about it Blood ix reports given on
the stem- nothing significantly abnormal (specifically checked for gamma GT-was normal)
Task
-hx/DDx/Mx

Typical psychy hx- Rp was yawning


Feels gulit/low mood/ no suicidal ideation-can’t remember other +ve findings ?social drinker

DDx-depression/delusion of guilt/acute stress disorder

Mx-CBT/support gp/helpline number

4R

Case (9/2/2017)
Stressed out 42 yo, consultant in company, lots happening, long hours Major Depression
• Hx
• Depression questions all positive • Insight was good • No suicide • Fraud at work- no
flashbacks
• DDx with reasons • Major depression • Adjustment disorder • PTSD • GAD

case (21/4/2017)
case 1/ young woman her colleague been accused and charged for stealing from the company
she feels bad about that can't sleep poor eating for 2.wks with feeling of guilt.

Dx depression

Fb mood disorder: pass


Case2/ young accountant with fraud in office . Low mood, depressed n difficulty in sleep Task; hx
, mx

Case (28/4/2017)
mood disorder Pass
Major depression case.typical symptoms in young lady.
told diagnosis,antidepressants,cbt.
also ruled out thyroid,anemia,drugs,psychosocial,suicide idea etc

226-Domestic Violence
Case 1
Pt came in to the suburban practice were you work with black eye. All xray and test have been
done and there is no fracture or any complications from the black eye.
Task:
-enquire about black eye.
-Tell her your initial management plan
1-Approach
-Hi Samantha My name is --- I am one of the doctors here. I understand from the notes that you
have a bruising around your eye.
-How are you feeling at the moment? Any pain? Any blurring of vision? Any headache or
nausea? (No)
- let me assure you that we have done some imaging; the results are normal and nothing is
serious.
-can you tell me how you got this bruise? How did you injure yourself? (I fell off from bed, doctor
if all the test reports where normal then I would like to go) (note/ if she told you she wants to go
home after explaining the results then say the following sentence and then ask her how she got
the bruise)
- Well. I won’t take so much time. It would be just a short consultation. I just need to ask you a
few questions in an attempt to help you. Would that be all right? (I needs to pick up my kids)
- How many kids do you have? what are their names.. How are they doing? (I think if you said
won’t take time she will not tell you she wants to pick up her kids However if she told say like
this and if no benefit tell her please bear with me just a few questions)
2-Revealing the real cause of her black eye + HEADS
-Confidentiality: Samantha. I understand that it is your personal matter but let me assure you
that everything we gonna talk about will be confidential unless there is a harm to yourself or
others.
-so you say you fell from the bed. But your bruise concerns me. Is there something else that you
would like to share? (No)
-ok. How is everything at home? (Good)
-who do you live with? (I live with my partner)
-do you have any kids? (yes two children.. I f you did not ask this before)
-how is your relationship with your partner? Are you arguing a lot? (yes)
-sometimes partner react strongly in arguments or may use physical force. Is this happening to
you by any chance? (Yes; she may reveal here that her husband did this to her )
-I am really sorry to hear that. Do not worry we are here to help you. How often does this
happen? (this is not the first time he did this to me whenever he gets drunk)
-did he do that in front of your kids? Has he ever hurt the children as well? (Never)
-have you or your partner taken recreational drugs?
-do you have enough support from family or friends?
-do you feel your home is safe for you and the kids? (Yes)
-any stress at work? Any financial problems?

3-mood questions, delusion, hallucination

Initial management plan


-I am really sorry to hear about what is happening between you and your husband. But I really
praise your effort. You are brave woman. This is known as domestic violence and it is not
acceptable. Many women are facing the same situation as you are. Let me assure you are not
alone. You do not have to go through this sort of violence and over time, these incidents can
tend to increase.

Note/ if she said (from youtube)


*he is so stressed and he only does this when he is drunk or angry. Tell her:
*I can understand that you might be making excuses for your relationship’s sake that he only
does it when he is drunk or angry. it is not ok for him to take out his stress on you. You do not
deserve to be treated this way.

-there are lots of things that we can do to help you:


1-there are a lot of organizations and support groups to help you get a safe place. They will also
help you financially through centerlink and offer you a shelter to live. Can give you the number
of domestic violence support groups.

2-I can arrange a social worker to help you and your children find a place to live if you have
nowhere to go.

3-You can also inform the police about your partner but that decision lies with you. (Not sure)

4-you can call a helpline which is available 24/7 and it is completely confidential. You can also
call 000.

5-if you decided to stay then it is always better to have a backup plan if there increase to a point
you are unable to handle. Hid some cash, clothes, documents in a go back so you can grab it and
leave.
6-I am also concerned about your kids. Would you mind if you could bring them over for a check
up.

7-our priority is your safety and your child’s safety. You are a strong confident woman and we
will be there to support you every step of the way.

Feedback 22-6-2018
Pt came in to the suburban practice were you work with black eye. All xray and test have been
done and there is no fracture or any complications from the black eye.
Task:
enquire about black eye,
tell her your initial management plan
Approach
I greeted the patient
Asked her about how she is feeling, any pain now, any blurring of vision or headache, nausea,
vomiting, fever.. She said no.. How exactly did she injure herself. She said she fell of from bed.
She said if all the test reports where normal then she would like to go
I told her I just need to ask her a few more questions.. she said she needs to pick up her kids
Said I understand.. then I asked her how many kids she has.. what are their names.. how are they
doing
She started relaxing and smiling while answering these questions.
Then asked her about home situation, whom she lives with, whether her partner is the kid’s
father she said no. I asked about work and financial stress. Drink, smoke or take recreational
drugs.
Mentioned I’ll ask some personal questions now and it would remain confidential unless it’s a
harm to u or anybody else.
Asked about whether they are a happy family.. Whether they argue a lot. She said yes. I asked
her in certain situations when we get angry we act impulsively and hurt ourselves or others does
that happen with your partner? She said yes.
Then I told her is this the first time? She said no. Then I asked her the details.. she said whenever
he drinks.. I asked her about whether he hurts the children she said never.
I said I understand what she is going through and it is not acceptable. It is called domestic
violence and a lot of women are facing the same situation as you are. You are not alone. I can
understand that you are making excuses for your relationahips sake that he only does it when he
is drunk or angry. But even in that situation he should not hurt you. Don’t worry we are here to
help you every step of the way if you allow us.
-I can arrange social worker to help you and your children.
-I need to call the child protective services. She looked worried I said don’t worry they will only
check your home situation and will not take you r kids away from you.
-There are shelters if you feel unsafe at home we can arrange for you and your kids to stay
there.
-You can also inform the police about your partner but that decision lies with you.
-I can arrange some support group so you can talk to them.
-Also I can arrange a family counseling with your consent if possible.
-Contact Centre link to arrange some financial support if required
-Don’t worry our priority is your safety and your child’s safety. You are a strong confident woman
and we will be there to support you every step of the way.

Karin case
Domestic Violence
Case: Your next patient in GP practice is a 24-year-old Fiona. She has visited you 4x for the last 6
months. The first she came to you with 5% burns on left hands. The second time she had some
injury on the right hand and the other 2 consultations, she complained of tiredness for which
she was thoroughly investigated and everything was normal. Today, she is visiting you with a
complaint of injury on her forehead.
Task
a. History (“I was running after the kid and hit my head on the door”, he is very short-tempered
and has lost his job because of it; and when he comes home he starts throwing things and pan
hit me especially when he drinks; at least 2x a week, beating started after birth of the 3rd child
when they had experienced financial problems; no support; have trouble with sleep).
b. Physical examination(distressed lady; 2x2 cm laceration on the forehead with erythema and
swelling; tender)
c. Diagnosis and management

History
- I understand that you’re getting injured quite often and now you have an injury on your
forehead. How did it happen?
- Confidentiality. I understand that it’s your personal matter but let me reassure you that …
Are you in a stable relationship? Do you have a partner? How is your relationship with your
partner? Sometimes, partners react strongly in arguments or may use physical force. Is this
happening to you by any chance? Have you ever been scared about your partner? Don’t worry
we’re here to help you. You need not be afraid.
- How often does this happen? When did it all start? do you have enough support from family or
friends? Do you have other injuries apart from the one in your head?
- How is your mood? Do you still find things pleasurable? Sleep? Appetite? Weight? Do you think
life is still worth living? What is your perspective towards life and towards what is happening to
you? Have you ever thought of harming or killing yourself?
SADMA? Do you have any strange experiences? Medication, illicit drugs, How’s your general
health?

Physical Examination
- General appearance
- Vital signs
- Are there any other scars or bruises. With the patient’s consent I would like to take some
pictures for record purposes
- Chest, heart, and CNS

Management
I would like to dress your wound and give you painkillers.
- From the history, I am really sorry to hear what’s happening between you and your husband
but I really praise your efforts. You are a very brave woman. This is known as domestic violence
and it is not acceptable. In my opinion, I believe you are not safe with the situation and in your
house. There are a lot of organizations and support groups to get a safe place. They will help you
financially as well and offer you shelter and a place to live. I would give you the numbers of the
domestic violence support groups.
- I will also organize a social worker. I would like you to see a counselor to talk about what
happened.
- I would recommend for you to stay in a separate room and don’t hesitate to call police if at any
time you feel it’s getting very serious.
- I would also like to see and examine your babies as contact the Child Protection Services to
assess your situation..
- If you think your partner agrees, I am happy to arrange a family meeting. If you want, I can
contact the social workers or police on your behalf. But I would like to recommend you to do
that because in my opinion you are not safe in your own home.

Failed feedback
35 yr old lady wit left eye bruise. Basal skull xray, ct-normal.
Hx, counsel
Asked few questions bot left eye
-Pain, bov? No
-Similar prob-no
-Hw it happend? Fell frm the bed
-Any bleeding problems? No -Fever- no -Headache-no -Bruise anywher else-
no
Confidentiality
-Pls tel me wat exactly happened? boyfrend problem ., hits whn he drinks..
- Mood , sleep, apet, wt-N, suicide thoughts-no
- Finance-N . Works. No stress at work
- Parents, siblings-not there
- Frends- ve a close frend
-Sad in her or partner- yes
-Children get along wit bf? Very wel
-Do u feel safe to go home? Yes. He s loving . Hits only whn he drinks
Dx: domestic violence. Not allowed in aus. U r not alone. Support
Crisis line
Dv support line
Womens service line, GP
Cal ur frend to visit u often.
Family meeting to address the alcohol issue if u wish.
Whn unsafe: call crisis line/Meals on wheels, shelter ll b arranged.
If u feel children r unsafe: child protection services.
Red flags Review FAILED
All key elements were covered. But scores were 3,3,3 for each task. So global
score was only 3. Failed! Din expect will fail this case

Passed feedback
It was the case of the lady who had a black eye and CT of head done was ok.
-So I started with introduction and commenting on the good CT results....
-asked her how the accident happened...she said that she fell off her bed...
-I asked her if it's the first time...
-was she dizzy or having a headache...
-what's her general health.
-how are things at home. work.
-who does she live with...then At that point I told her about the confidentiality thing, there was
good rapport at that point.
-then I told her that people react to anger and stress in different ways...some get violent...I told
her that am not pointing fingers but could there be the slightest chance that her partner did
that to her...she looked at me and said yes.
-Then I commended her for opening up and confiding in me...told her that she has already made
a big step and I will be with her all the way
-I firmly but kindly told her that any form of domestic violence is wrong both morally and
legally...and its not tolerated.
-Asked if the partner is the father of the children..she said no...am like ok...how is the
relationship with the kids..it's like just there...
-Do either of you take alcohol drugs...etc...she said the partner can get drunk then a bit violent...
-I asked if it was the first time he got violent and has he ever been violent to the children
-I asked her overall does she feel that her home is safe for her and her kids
-Then asked if she is financially stable..she said not
-Then I just spoke about counselling...social worker..Centrelink
-I told her that I am concerned about her kids and if she minded bringing them over for a check
up of which she agreed
-Then I told her that with her permission and if she is willing she can come with the partner and
we can organise for family therapy. She agreed
-I didn't mention about child protective services and all...but you can and should do if you get
time But most importantly just make sure there is rapport...look at the patient in the eye so that
you earn her trust and be very empathetic

Case (29/11/2017) (28/11/2017)


35 years Young lady has come in with a black eye. CT scan is normal and rest of examination
unremarkable and she wants to go home. Speak to her before she has to leave.
Tasks-
-Take History not more than 6 mins
-Tell about the management
( initially patient didn’t tell anything how that happened. She said she had to go urgently as she
needs to pick up the children from school. After pushing a lot she revealed that her husband is
alcoholic and when he gets drunk he beats her up. Never did it infront of children. Mentioned
many times he is very helpful financially, only does this when drunk. Some candidates said she
herself was using some drugs too)
She was ready to inform police as soon as she was given the option

Case 2
GP, 30 year old lady came to GP frequently for recurrent injury. Previously she refused to disclose
mechanism of injury. Later she opened up that she is physically abused by her husband. Today
she came to see you for further advice and management for her abuse. Investigation results
showed normal.
Task:
-Take the history
-Counsel and manage her.

History (may be it is better to read the stem well)


1-Approach
-appreciate her coming to you. (here she is scaring and feel unsafe)
-confidentiality
-any pain, headache, blurring of vision, nausea, dizziness, head injury
2- Abuse questions
-how long has this been happening to you?
-how often?
-can you tell me more about when and why this happens?
-any hospitalisation due to it?
3-HEADS
-do you have any kids? (No)
-do you have enough support from family or friends?
-have you or your partner taken recreational drugs?
-what is your work? any stress at work? Any financial problems?
-anhedonia
4-mood questions, delusion, hallucination
5-General questions
-PMH, PSH, SADMA

Management plan
-I am really sorry to hear about what is happening between you and your husband. But I really
praise your effort. You are brave woman. This is known as domestic violence and it is not
acceptable. Let me assure you are not alone.
-there are lots of things that we can do to help you:
1-we have a social worker to help you find a place to live.
2-there are a lot of organizations and support groups to help you get a safe place. They will also
help you financially through centerlink and offer you a shelter to live. Will give you the number
of domestic violence support groups.
3-You can also inform the police about your partner but that decision lies on you.
4-ther is a helpline, which is available 24/7 and it is completely confidential.
5-our priority is your safety. You are a strong confident woman and we will be there to support
you every step of the way.

Feedback 16-8-2018. Domestic violence


GP, 30 year old lady came to GP frequently for recurrent injury. Previously she refused to disclose
mechanism of injury. Later she opened up that she is physically abused by her husband. Today
she came to see you for further advice and management for her abuse. Investigation results
showed normal.
Task:
-Take the history
-Counsel and manage her.
I appreciate and give confidentiality. She is scared of her husband because of being abused. No
head injury. No history of hospitalization due to abuse. They got married for 8 years & started
abused for 6 years. No children (She said it is good one for her). Sometimes he abused her under
alcohol influence. No history of informed to police.
HEADSS: She worked as a Cleaner. No financial problem. No signs and symptoms of depression:
frustrated and fear (+), no appetite & sleep problem, no history of poor concentration, not think
life is worthless, no suicidal ideation. No smoking, alcohol & recreational drugs.
No past medical & surgical problem.

Mx: I appreciate again. Reassurance.


I ask about her attitude towards her husband: she said scaring of her husband. I ask want to
inform police: she said she accept everything. I said I will inform police if she agrees. As you are
scared of your husband, I can arrange shelter for you. Recurrence again. Refer to marriage
counsellor. Refer to stress counsellor. Learn emergency service contact number if being abused
again. Find exist route. Notify the neighbour. Support gp. Centre link.
Grade: Fail
Global score: 2
Key steps: 1: No 2: No 3: Yes 4. No
Approach to patient: 3 History: 3 Management plan: 3
227-Pyloric stenosis
Sample case/ You are a GP in metropolitan practice and a 3 week old baby boy, John presents
with vomiting since birth.
TASKS
-History
-Physical examination Findings from Examiner
-Diagnosis and further management

AMC Case 28-2-2018


6 weeks old Baby with vomiting after every feed for 1 week
Tasks
take history. pyloric stenosis.
PEFE
Explain to mother and advise management

Differential Diagnoses:
-Intestinal obstruction
-Pyloric stenosis
-GERD

History
1-Vomiting questions
-Hello. I am Dr. I will be your GP today. How can I address you?
-I understand from your notes that your child has been vomiting since birth. Can you tell me
more about this?
-Could you describe to me how the vomiting is? how much, what is the color, and What are its
contents? Is it forceful? How often? Any triggers?
2-Diarrhea
-Any changes in bowel motion? Is she passing gas?
3-tummy pain
-Does she draw up her legs while crying?
4-fever/ rash.
- Did she have any fever, and did it came with any rash?
5-Lumps
-Noticed any lumps in the body, in the groin area?
-Do you feel or observe any lumps or bumps in the tummy while your child is feeding?
6-Dehydration questions
-How is her feeding? Any differences in appetite? And could you say that your child seems to cry
in between feeds--hungry all the time?
-is there any change in the number of her wet nappies? Is it foul smelling and does she cry when
she pees?
-How is the activity of your child? Could you say that she became lethargic or still the same?

7-BINDS
-Is she your first child? How old were you when you were pregnant with her? Any complications
during the pregnancy? How about her delivery? Any problems at birth?
-Is she currently up to date with her immunizations?
-Is she thriving and growing normally?
-Do you have another child at home that may have been sick recently or anyone at home sick
recently?
8-Past history
-Any previous illness for your child? History of congenital diseases or in born diseases?

PEFE
Thank you for those information. I will just talk to my examiner and I will get back to you shortly.
1-GA
-Is the patient alert, drowsy, lethargic?
-What is the CRT, how is the skin turgor, is there sunken eees, and is the anterior fontanelle
bulging?
- Is there any rash in the body, or skin pigmentation?
-Any pallor or jaundice?
-Any dysmorphic features such as down slanting of the eyes, upturned nose, or simian crease?
-Are there any LN enlargements?

2- VITALS: TEMP, HR, RR, O2 SAT


[Im shifting the child to the resuscitation room. I will insert an IV line and take blood samples for
investigations such as a FBE and UEC.]
3-Growth chart
-What is the weight for height?

4-ENT and neck stiffness


-Any neck stiffness?
-Any tragal tenderness or ear discharge?

5-Abdominal examination
-visible distention or peristalsis? Palpable masses? *OLIVE SHAPED MASS. Tenderness?
-I would like to check the hernial orifice, genital exam to check for ambiguous genitalia, and per
anal inspection with the parent's consent

6-Office tests: urine dipstick, bsl

Investigations:
FBE, UEC, arterial blood gass (HYPOCHLOREMIC, HYPOKALEMIC METABOLIC ALKALOSIS),
Ultrasound of the abdomen

Explanation and management


Based on the history, examination, and investigations, it seems that most likely your child has
what we call Pyloric Stenosis. Sorry for using a medical term but let me explain this to you. But
before I continue, do you want me to call someone to be with you as I explain the diagnosis? Or
is it okay to continue?

*draw*

The milk goes to the mouth though a tube like structure called a food pipe, to the stomach. The
lower end of the stomach is called pylorus. From this stomach, the milk then goes to the small
gut.
Sometimes, due to some unknown reasons, in some infants, the lower end of the pylorus get
narrowed. It might be due to thickening or tightness of the muscle layer in that pylorus. That’s
the reason why the milk or whatever the child drinks, do not pass down to the small gut and the
child vomits it out.
Because the stomach is not full after vomiting, the child feels hungry after every feed. I can tell
that your child is suffering from this diagnosis because of the symptoms and signs that she
presented--projectile vomiting, hungry feed, not gaining weight, and movement of the gut at the
site of the obstruction

At this stage, it's an emergency condition and is related to life-threatening complications. I will
call the ambulance, arrange for the piper (pediatric emergency transfer system), PETS/NETS, and
arrange transfer of your child to the hospital so that she can be seen by a specialist. In general
the treatment involves stabilizing the current condition of the child first by correcting any
imbalances of his electrolytes--or the factors that maintain salt and water balance in the body,
and then assessment and management by a possible surgery by relieving the tightness of the
pylorus of the stomach. Do you understand so far? [ If she cries, pause for some time, ]
If we intervene early, further complications can be prevented.
As you have seen earlier, we already started correcting all the fluid losses of your child as I
already placed two IV lines to your child where we have already started giving fluids to correct
her dehydration. We also did further investigations earlier and found that there is an imbalance
in the electrolytes. Due to her repeated vomiting, hyponatremic, hypokalemic alkalosis. We
might also need to do an ultrasound to confirm the diagnosis.

For now I need you to stop feeding her by mouth temporarily as it may worsen her condition. I
will pass a small tube through the nose into the food pipe (sorry for using medical term), it's
called nasogastric tube, to remove the excess milk in the stomach and prevent further vomiting
(to decompress the stomach). Then I will call the pediatric surgical registrar in the hospital so
that everything for the surgery is arranged before the child is transferred, or during the time of
transferring. I will fax all the blood test results. The child might need surgery, most of the time it
can be a keyhole or an oval surgery. Usually the tightened part of the pylorus will be opened up
so that the tightness can be relieved.

In the meantime, if you have another child, I can arrange for a social worker to take care of her.
Do you need me to call anyone or your partner so that he can be with you right now?

When can I start breastfeeding? = it depends upon the specialist advise. Most of the time,
breastfeeding is started 24-48 hours after the surgery. Again it depends on the progress of the
child.

I will arrange a lactational nurse to teach you how to express your milk and how to store it for
now.

Though it can be a life threatening disease, we will do our best to manage your child the best
way we can. Do you have any further questions?

Case (29/4/2017)
GP, mother with 6 weeks old baby with vomiting after every feed for 1 week.

Tasks
- History
- Ask relevant PE from examiner
- Explain Dx and DDx with reasons to patient’s mother

Dx- pyloric stenosis

( the patient had mass in epigastrium and visible peristalsis only when asked specifically , some
candidates didn’t get those findings) and feeding test was also positive.

Note/ the tasks in the exam can be shorten sometimes will be no management and sometime
be.
Read handbook 127

228-Ureteric Calculi
Middle age male, abdominal pain worsen over last 3 hours, (you are in ED I think)
Task:
-history
-explain possible causes to the patient
-discuss with examiner the investigations you want to do
-initial mx.

TASK 1
History
1-Pain questions
-check hemodynamic stability
-pain severity+ painkillers
-since when? Sudden or gradual? Constant or come and go? If getting worse?
-exact site? Radiation? (Pain initially at right flank then move down to groin)
-type?
-if anything makes it better or worse?
-what were you doing when it started?
-has this happened before?

2-Associated symptoms questions


-do you feel nauseous?
-have you had any vomiting? (Yes twice) what does it contain? Any blood?
-how is you bowel motions? Any constipation or diarrhea or alternating constipation with
diarrhea? Any change in the colour of your motions? Any blood?
-how is your urine? Any change in colour or urine stream?
-Fever, rash, recent infections?
-LOW, LOA, lumps and bumps?
-Trauma
3- General questions.
-Eating, drinking and diet?
-Occupation
-medications?
-PMH
-PSH
-SAD
-family history

Physical examination findings from the examiner (if it was a task)


1-General appearance
pallor, jaundice, anxious, sweaty, in pain
2-Vital signs especially pulse (ask rate, rhythm)
3-Focus abdomen
inspection: movement with respiration, distension, visible masses, scars, dilated veins
palpation: tenderness including renal angle tenderness.
Auscultation: bowels sounds
-complete with: hernia orifice, genitalia, LN, per rectal
4-quick CVS and chest
6-Offiec tests
-urine dipstick (blood and protein)
-BSL
-ECG

Task 2
Explain possible causes to patient
-from history, there are few possibilities why you have abdominal pain.
-most likely, you have a condition we call a ureteric colic. Have you heard about it?
-draw a diagram. I will explain it to you using a diagram. If you have any confusion please let me
know and I will clarify it.
-so here is the kidney and there is a tube that connects the kidney to the bladder or the urine
bag called ureter. Ureteric colic means a pain, which is usually travelling from the kidney to the
bladder and the most likely cause for it is a stone.
-a stone is usually formed in the kidney then passes down the ureter and when it is moving, it
causes irritation of the inner lining of the ureter causing severe pain that goes to back and some
blood appears sometimes in urine.
-this condition can be caused by inappropriate diary habit, low fibers intake, low fluid, increase
in minerals and salts in body like calcium or troublesome with acid we call uric acid. It also might
run in families. Sometimes associated with metabolic of gland problems.

-other possibilities are:


*pyelonephritis (infection of kidney but less likely as no fever, nausea or vomiting)
*appendicitis (inflammation of worm shaped structure attached to large bowels but less likely
from history)
*cholecystitis (inflammation of gall bladder but less likely as its pain is in upper part)
*IBD (less likely)
*Trauma (less likely)

Task 3
discuss with examiner the investigations you want to do
-I am gonna order some Ix for this reason I need to ask the examiner all right?
-Urine dipstick (blood and protein)
-Urine MCS
-FBC, UCE
-X-ray
-CT scan without contrast (examiner will give you a photo of CT scan showing ureteric stone at
vesicoureteral junction)

Task 4
Management
-Ct confirmed the presence of stone in the ureter.
-will keep in you hospital to be seen by a registrar.
-give you strong painkillers
-management will depend on the size of the stone.
*if it is small one, it might pass spontaneously and the specialist might decide to discharge you
with some advice like:
ensuring high fluid intake, stay away from uric acid containing foods (red meat, red wine, beer,
organic meat) or oxalate foods (tea, coffee, cola, chocolate) will give lists of food to avoid.
Will advise you to pass urine in a container then strain and collect any stone for analysis
purpose.
*if the stone was a big one or if it was small but it did not pass or conditioned worsened then
specialist urologist might decide to take it out using instruments or US destruction or open
surgery.

Feedback 6-6-2018
Station 16 ureteric colic – pass
Middle age male , abdominal pain worsen over last 3 hours, (you are in ED I think)
Task: history, explain possible causes to the patient, discuss with examiner the investigations you
want to do , initial mx.
History: patient lying on the bed in 30’ position, hand on his flank. First, ask examiner for HD
stable, ask pt he wants pain killer or not.
Pain initially at right flank then move down to groin. All pain questions. No other symptoms no
urinary symptoms and no GI symptoms . no lump and bump. No fever. No symptoms of STD.
Possible causes : stone in urinary tract, less likely pyelonephritis, appendicitis, cholecystitis,
trauma, enteritis.
Investigation: FBE, urea and electrolyte, ultrasound, X rays ( I don’t remember the results
examiner gave me, I think he told me they were NAD) so I asked for CT abdomen, examiner gave
me the photo of CT scan showing ureteric stone at vesicoureteric junction.
Drew diagram and explained to the patient . initial mx.. IVF, pain killer, urologist review,
treatment depends on the size and site of the stone ,options: conservative, small stone will pass
by itself, or laser ,endoscopic removal and open sx.

Recall 10-4-2018
Right Sided Abdominal Pain – Pt lying in 45 degrees with hands on his back/Side (Renal Angle)
Slight nausea No vomiting Colic type pains He had taken pain medication Discuss Diff Dxx and
Definitive Dxx and what Investig you want Renal Calculi Investig Urine – 4 + No Leuc No Nitrates
Other Invest and Mxx of Renal Calculi work Counsel.

Case (29/11/2017) (1/3/2017) (24/6/2017)


Case (21/9/2017)
13-Rif pain in 52 yran in Ed
Hx,
Ddx,
Ix from examiner,
Mx
FB comment: Pricila Dria Pangesti 13 is Ureterolithiasis they give you CT when you ask them.

Handbook case 90
Acute right-sided pain and haematuria in a 25-year-old man

CANDIDATE INFORMATION AND TASKS

This 25-year-old man is being seen in the hospital Emergency Department with a first episode of
severe right-sided abdominal pain. The pain came on two hours earlier and was so severe that
the patient writhed in agony unable to relieve his symptoms. The pain started in the right side of
his back and radiated into his right groin and testicle.

He is now free of pain. He has had no pain like this previously and has been in good general
health.
Physical examination findings are normal, except that his urine is positive for blood on chemical
testing. There is no loin or other tenderness. You have just finished examining him.

YOUR TASKS ARE TO:

• Determine the most likely diagnosis and discuss initial investigations with the examiner.

• Explain the diagnosis to the patient.

• Outline your management plan, and any further investigations required, to the patient.

You will not need to take any additional history. There is no need for you to ask the examiner
about any other findings on clinical examination.

Task 3

Questions to ask unless already covered and appropriate responses from doctor/candidate
(Answers in parentheses after the question):

• ‘Do I need to be admitted to hospital?' (Not at this stage)

• 'Will the pain come back?' (Possibly, if the stone is still in the ureter).

• 'Why did I get the stone?' — Ask this if the candidate suggests a stone is the cause of the

pain.

• ‘How will I know if I have passed the stone?' (You will strain your urine).

• 'What happens if the stone does not pass?' (An instrument may have to be inserted to

retrieve it).

• 'When do I see you again?' (Follow up in a couple of days for investigation results).

Task 2

The candidate should explain that you have a small stone that is passing down your ureter. It
may take one to two days to pass the stone. You will be given strong painkillers for the pain in
case it recurs. Most stones pass spontaneously. You are unlikely to have future problems, but
tests on your urine and blood will be done to check this.

Task 1

The investigations required are likely to include:

• Culture of the urine to exclude infection.

• An ultrasound of the kidney may be performed looking for evidence of blockage of the ureter

caused by the stone.

• Imaging by computed tomography (CT) to define the size and site of the stone.

• Examination of the stone after you have passed it to determine the type of stone present.
• Other blood tests and specialist referral may be required in follow up.

Task 2+3
-You most likely have a condition called ureteric colic. Have you heard about it? No.

(draw a diagram) This is when a stone; which is most likely a small one in your case and this
stone is passing down your ureter; a tube between the kidney and bladder. Why did I get it? It
could be because…..

-It may take one to two days to pass the stone so that the pain may come back again and for this
reason; I will give you strong painkillers in case the pain recurs. (NSAID or pethidine.. no need for
pethidine now as the pain has gone)

Most stone pass spontaneously and you will know that you have passed it if you find that you
need to strain your urine. However if it did not pass then an instrument might have to be
inserted to retrieve it or open operation or ultrasound destruction might be needed.

You are unlikely to have future problems but tests on your urine and blood will be done to check
this and also you need to pass urine into a container and strain and save any stones found for
analysis.

You do not need to be admitted to hospital at this stage but I would like to see you again in a
couple of days to discuss the results.

You need to maintain high fluid intake. if stone is not passed in 48 hours or pain recurs and
worsens then you have to be referred to a specialist.

EXPECTATIONS OF CANDIDATE PERFORMANCE

• The candidate should indicate the pain is almost certainly due to renal colic, due to the fact a
small stone is being passed down the right ureter. No need for pethidine now as pain has gone. If
pain returns, give pethidine or Panadeine forte® or a nonsteroidal antiinflammatory drug
(NSAID).

• Give patient a brief description (draw diagram) of the anatomy of the kidneys, ureters, bladder
and urethra.

• Pass all urine into a container and strain (save any stones found for analysis).

• Imaging by computed tomography (CT) abdomen or plain X-ray, to identify the site of the stone
and use for subsequent assessment.

• Check urine for infection.


• Check serum uric acid, and serum calcium, urea and creatinine.

• Ultrasound to see if hydronephrosis is present is an acceptable and appropriate test, but

definitive imaging is by helical abdominal CT which will pick up very small stones.

• Advise high fluid intake. Refer to urologist if stone is not passed in 48 hours or pain recurs and
worsens. He may need an open operation, endoscopic removal, or ultrasound destruction if not

passed spontaneously (depends on size and site of stone).

• Followup to check progress.

KEY ISSUES

• Diagnosis of renal 'colic', probably due to a stone in right ureter.

• Explanation of the problem to the patient incorporating rapport with the patient and

communication skills.

• Providing an appropriate plan of management (including provision for further pain relief).

• Appropriate choice of investigations.

CRITICAL ERRORS

• Failure to make a diagnosis of renal (ureteric) colic.

• Failure to arrange appropriate investigations.

COMMENTARY

Investigations of presumed renal colic due to stone are by diagnostic imaging. Most urinary
calculi (80%) are radio-opaque so plain abdominal X-ray may be diagnostic (as illustrated),
however small stones may be missed and differentiation from pelvic phleboliths impossible. The
preferred investigation is a helical CT without intravenous contrast (as illustrated), which will pick
up any small stones and also identify urinary tract obstructions.

Radiolucent stones occur with hyperuricaemia and in some types of familial and metabolic
calculi. It is prudent in each patient with a urinary calculus to check for hypercalcaemiato pick up
cases of primary hyperparathyroidism presenting as renal calculi, a common mode of
presentation of parathyroid adenoma. Full investigation for a primary cause is mandatory in
patients with a history of recurrent calculi.

Karin case
Case: You are an HMO in ED and your next patient is a 55-year-old female complaining of
intermittent abdominal pain 2 hours ago.

Task

a. Relevant history (right flank travels down, blood in urine)

b. Perform abdominal examination (pale and sweaty, anxious, normal BMI, tachycardic, afebrile,
normotensive, non tender abdomen on palpation, renal tenderness positive, microscopic
hematuria on dipstick)

c. Investigation (IVP: 4mm stone within R ureter)

d. Management

Case: Mark aged 48 years presents to your surgery with pain in the right flank for the last 3-4
hours. He also feels nauseated and had vomited once. On further questioning he reveals of
having increased frequency of urine for the last 1-2 days. he denies similar problem in the past
and is otherwise having no significant medical problems. Mark lives independently and works as
business consultant. He is not on any regular medications and had no known allergies.

Task

a. Further focused history (started few hours ago, 10/10 in severity)

b. Examination and investigation findings from examiner (no fever; + pain flank, urine dipstick ++
rbc, +protein; do PR and genital examination )

c. Provisional diagnosis and management advise

Case3 (November 2012): Ben aged 35 years presents to your surgery on a busy Friday afternoon
and practice manager requests you to see this patient as his usual GP is sick today due to bad flu.
Ben was seen by his GP yesterday due to abdominal pain which is settled now and he ordered a
few tests including blood, urine and CT scan. Now he came back with results. Ben runs his own
business and lives independently. He had no significant past medical or surgical history and is not
on any medications.

- Results

o FBE: Hb 135, WCC 9, Plt 400

o UEC normal

o CRP 15, LFts normal, BSL 4.8 mmol/L

o Urine MCS shows blood and protein but culture is still pending

o CT abdomen: not reported yet

Task

a. Explain the test results and further management advice


Location Size Treatment

Renal Pelvis <2.5 cm ESWL

>2.5 cm PCNL

Ureter Upper half

<1 cm ESWL

>1cm ESWL/PCNL

Lower half

<1cm ESWL

>1cm ESWL/urethroscopy + basket

Bladder <3cm Transurethral lithotripsy

>3cm Cystostomy

History

- Please tell me more about the pain? How bad is the pain? I would like to ask the examiner to
offer morphine along with metoclopramide (5-10 mg IV or 15 mg IM + 10mg IV)

- SORTSARA? N/V? Have you noticed blood in the urine? Is this the first episode? What is the
nature of the pain? Is it colicky, constant, or intermittent? Fever? Past history of urinary
infections, diabetes, stones, or gynecological problems? When was your LMP? How’s your
general health?

Physical examination

- Lying supine on the bed

- Exposure: breast up to inguinal crease

- I am going to feel your tummy. If you develop a lot of pain, please tell me and I will stop.

- General appearance: pallor, jaundice, dehydration; Is the patient sweaty, restless or SOB? LOC?

- Vital signs

- Inspection: mass, peristalsis, distention, scars, pulsations, venous distention

- Palpation: start at left iliac fossa: in superficial palpation: mass, tenderness, deep: guarding,
rigidity, rebound tenderness, organomegaly, rovsing, psoas signs, obturator sign

- Auscultation for bowel sounds

- Cough and check inguinal orifices for any visible hernia

- Renal angle tenderness (patient sitting)


- DRE

Investigation:

- FBE, urine MCS, dipstick, serum Na+, K+, Ca++, Mg++, phosphate, uric acid

- Xray of abdomen, USD of abdomen, IVP/CT scan

Management

- You have a condition called ureteric colic where there is pain travelling from the kidneys to the
bladder. The most common cause of ureteric colic is stone (others: stricture, stenting, infection,
malignancy). Usually, a small stone starts traveling from the kidney towards the bladder. During
this movement, the stone irritates the lining of the ureter causing intermittent pain with/without
bleeding. The patient usually feels cold, shivery, clammy skin, along with nausea and vomiting.

- These stones are most commonly seen in males between the age of 30-50 years, the usual
causes for the formation of stones: previous urinary infections, inappropriate dietary habits
(tomatoes, citrus, cheese), presence of excessive calcium in the body and genetic tendency.
Usually, stones <5mm pass out in the urine spontaneously. In that case, we need to keep you in
the hospital a few hours in order to observe you. We do a test that is known as IVP which will
show us the exact location and size of the stone. I will call in the specialist urologist to take a look
at you. If the stone is bigger than 5mm, we have a few options which include lithotripsy where
we break the stone into smaller pieces with the help of sound or shock waves. These fragments
are then passed out with the urine. The next option is if the stone is higher up or within the
kidney, you will need to undergo surgery to remove the stone. Third option is if the stone is
found within or near the bladder, we will use a basket technique to retrieve the stone through
the urethra. The last option is chemolysis. This can be only be performed if the stone is a uric
acid stone that is small in size. We will give medications that will change the pH of urine. The
choice of treatment depends upon the decision of the urologist.

- At the moment, our aim is to relieve the pain completely and to followup all the investigations.
Once the pain settles down, you can go home and come back for followup at the surgical clinic. I
will write some painkillers for you when you go home. I will give you some written material
regarding the best dietary habits that you need to adopt because these stones can reoccur. The
risk is around 75% within the first 2 years from the first episode.
229-Papillary thyroid cancer BBN
27 yr old man with history neck swelling, USS solid mass at left lower lobe at (RT or left side, I
dont remember ) of thyroid gland , no LN enlargement both on examination and USS finding .
FNAC neuclear atypia , papillary thyroid CA ( alittle bit details was given in the stem about the
swelling)
Tasks
-Explain the investigation to patient.
-counsel the mx plan
-explain the risk of possible treatment you suggested.

1-Approach
-introduce yourself (I have come for the result)
-yes Brian, I have the result with me now. How are you doing these days (I am fine)
-any pain at biopsy site? (No)

2-Breaking bad news:


*Knowledge about the test
-Do you know why the biopsy has been done for you?
*Knowledge about the disease
-have you been told about your medical situation so far?
*How much information they want (optional)
How much do you want to know about the results?
*Breaking bad news
- now I am going to explain about the result, as you have swelling in your neck , we did the
examination and US to your neck , found out that it is originated from thyroid gland .
-The biopsy was done to that swelling. I am afraid to tell you the result comes back as cancer in
the thyroid gland.
*If there is reaction do like this:
-the patient will start crying so offer box of tissues and glass of water if available, wait for her to
settle down.
-would you like me to continue the consultation today?
*If there is no reaction say like this:
-how are you feeling on hearing this result? You can tell me what’s going through in your mind?
Are you worried? (Yes I am worried)
*offer a call for someone:
-would you like me to call anyone for you? (No)
*showing empathy
- I do understand that this comes as a shock to you. I can see that this is an upsetting news for
you and also very difficult situation to face.
*Showing support to reassure she is not alone
-But we can do a lot of things. You don’t have to go through this alone, if this is in early stage and
if we start treatment as early as possible you will have higher chance to be cure.
*ask if she is ready
Are you ready to discuss a treatment plan now, or should I arrange another appointment with
you?
3-Talk about the disease
-In our neck there is butterfly shaped gland, we called thyroid gland that produces thyroid
hormones which is very important for the metabolism of each and every cell in our body .
-Now the cancer is in the thyroid gland, (draw a diagram) , the type of cancer is papillary thyroid
cancer which is most common type of cancer in thyroid . The good thing is , it is slowly growing
Tumour, and it spread usually through the lymphatic channel, but I cannot see any gland or
lymph node enlargement in your neck, that’s the good point, most likely your cancer is in the
early stage

4-Management
MDT and Refer to the specialist for Ix to check for spread
-I will be referring you to some specialist and your care from here onwards. will be taken up by a
MDT including thyroid surgeon and cancer specialist, radiologist.
-they will do further assessment for staging , like blood tests and CT scan to your body to make
sure it’s not spread .
Treatment according to the disease
-the main part of treatment is surgery.
-before the surgery , they will need blood test to make sure you are fit for surgery, checking
thyroid hormone level in your blood as baseline.
-the surgeon may need to remove the whole thyroid gland to treat for it.
-So after the surgery, there will no longer thyroid hormone in your body , so you may need to
take hormone replacement throughout your life .
-the surgeon may consider giving you radioiodine therapy which is medicine that has radio-
iodine that you can take by mouth to make sure the possible remaining cancer cells are killed .
-But the specialist will decide for it for mx. I’m just mentioning the possible options.
Follow up (usually blood tests and imaging)
After treatment , regular frequent follow up is required with blood tests, regular neck US and
isotope scanning.
Complications (task 3)
every treatment is not without the complication , i want to tell you about them but dont be
scared , we can do many things to prevent it .
infection , injury , bleeding , anesthesia side effect , injury to nerve the supply the voice box ,
leading to HOV , low thyroid hormone level , sometimes injury to nearby gland what we called
parathyroid gland that maintains the normal calcium level in body can lead to low calcium level.
But the surgeon is well experienced so we will do many thing is to make cx as less as possible

5-Ending
Brian, we are here to help you . you are not alone . do you have family member , would you like
to bring them in next appointment , so that they can understand your condition and can give
support .
when you go home if you have any concern , you can contact me anytime . If you feel stressed
about this condition , i can refer you to psychologist as well . There are a lot of support gp
available for you.
Feedback 20-7-2018
Papillary thyroid cancer counselling
Neck lump Pass Global score 7
key step 1,2,3,4 Yes
approach to pt 7
interpretation of investigation 7
Management plan 7
patient counselling 6

27 yr old man with history neck swelling , USS solid mass at left lower lobe at (RT or left
side, i dont remember ) of thyroid gland ,
no LN enlargement both on examination and USS finding .
FNAC neuclear atypia , papillary thyroid CA ( alittle bit details was given in the stem about
the swelling)
Explain the investigation to patient ,
counsel the mx plan ,
explain the risk of possible treatment you suggested.

the patient said to me he has come for the result soon after my introduction , yes bryan , i
have the result with me now . how are you doing these days ... i am fine , any pain at biopsy
site ... no , do you come alone today ... yes.... , may i know what is your expectation about the
result .... i hope its fine.... bryan , now i am going to explain about the result , as you have
swelling in your neck , we did the examination and usg to your neck , found out that it is
originated from thyroid gland . the biopsy was done to that swelling . I am afraid to tell you
the result comes back as cancer in the thyroid gland .
i waited for his expression ....... seriously .... no expression .....
how are you feeling on hearing this result ? you can tell me whats going thr in your mind?
are you worried ...... yes i am worried, i can't see any worried face , even smiling from one
angle of his lip.( not a good actor)
I just continued , Bryan , i do utd that this comes as a shock to you . but we can do a lot of
things . you dont have to go thr this alone , if this is in early stage and if we start treatment
as early as possible you will have higher chance to be cure .
( i can see the examiner moved the position from my back to the place where i can see her
clearly)
Let me explain detail , is that okay with you , or do you want to make another appointment
in our neck there is butterfly shaped gland,we called thyroid gland that produces thyroid
hormones which is very important for the metabolism of each and every cell in our body .
Now the cancer is in the thyoid gland , i explained with pic , the type of cancer is papillary
thyroid cancer which is most common type of cancer in thyroid . the good thing is , it is slow
growing tumour . and it spread usu thr the lympatic channel , but i cannot see any gland or
lymph node enlargement in your neck , thats the good point , most likely your cancer is in
the early stage
we will help you with MDT , i will first refer you to ENT specialist and cancer specialist who
will do further assessemnt for staging , like CT scan to your body to make sure its not spread
.the main part of treatment is surgery , before the surgery , we will need blood test to make
sure you are fit for surgery , checking thyroid hormone level in your blood as baseline . the
surgeon may need to remove the whole thyroid gland to treat for it . so after the surgery ,
there will no longer thyroid hormone in your body , so you may need to take hormone
replacement throughout your life . the surgeon may consider giving you radioiodine therapy
which is medicine that has radio- iodine that you can take by mouth to make sure the
possible remaining cancer cells are killed . but the specialist will decide for it for mx . i'm just
mentioning the possible options . after treatment , regular frequent follow up is required with
blood tests .

Bryan, we are here to help you . you are not alone . do you have family member , would you
like to bring them in next appointment , so that they can utd your condition and can give
support . when you go home if you have any concern , you can contact me anytime . if you
feel stressed about this conditon , i can refer you to psychologist as well . there are alot of
support gp available for you.

my last task is to explain complication , well i actually dont want to tell too much risk of
treatment in BBN station but it was a task , so i have to
bryan , every treatment is not without the complication , i want to tell you about them but
dont be scared , we can do many things to prevent it .
infection , injury , bleeding , anesthesia side effect , injury to nerve the supply the voice box ,
leading to HOV , low thyroid hormone level , soemtimes injury to nearby gland what we
called parathyroid gland that maintains the normal calcium level in body can lead to low
calcium level
but the surgeon is well experienced so we will do many thing s to make cx as less as possible

Ps, even though the patient is not a good actor , be yourself a good actress/actor :D , showing
a lot of sympathy is key in BBN
Case (21/4/2017)
A young man who had a lump in front of thyroid recently underwent Ultrasound and FNAC
which showed a papillary thyroid cancer.
Task: tel him the result and discuss management
* young man with lump on neck,. Biopsy dine showed papillary ca of thyroid, LN not palpable
Breaking bad news. Reassure that it hasn't metastasize yet. Good prognosis. We will do ct, Surg,
Team work
Fb acute lump: pass

Karin case
Breaking Bad News (Papillary Thyroid Cancer)
Case: You are a GP and 30-year-old female comes to you 2 weeks ago. Your colleague has
arranged some investigations including an ultrasound and FNAC. The results are with you that
shows papillary carcinoma of thyroid.
Task
a. Explain diagnosis to patient
b. Discuss management options

Case 2: Sophie aged 52 years of age presents to your GP clinic complaining of a painless lump in
her neck which she noticed a few days ago. She denies any other associated symptoms. Sophie
had hypertension for the lat 10 years and is on coversyl 5mg daily. She had no other medical
problems and had no known allergies. She lives independently and works as an accountant in a
busy company
Task
a. Focused history (no weight loss, weather preference, smoker, no FHx)
b. Physical examination and investigation (looks anxious, BMI 29, VS normal, midline swelling, no
visible veins or scar marks, moves with deglutition, moves with deglutition but does not move
with tongue protrusion, USD).
c. Differential diagnosis and treatment advice (multinodular goiter 2.5cm and 1.5cm, TFTs
normal, FNAC papillary thyroid cancer)

History
- Since when? Any weather preference? Hoarseness or change in voice? SOB? Dyspnea?
Dysphagia? Odynophagia? Any weight loss or weight gain? Change in appetite? Night sweats?
Previous history of irradiation?
- FHx? SADMA?

Physical examination
- General appearance
- Vital signs
- Thyroid: visible veins, swelling, erythema; consistency, tenderness, attachment to underlying
structures, auscultate for bruit/thrill, pemberton sign; swallowing test
- LAD
- Chest and heart
Investigation
- TFTs (If high OR normal  proceed to FNAC biopsy)
- USD of thyroid
- FNAC
- Thyroid scan (Radioisotope scan)  do if TSH low

Diagnosis and Management


- I have the results of the tests we did on you. Unfortunately, I don’t have good news for you.
Would you like someone to be present during this consultation? Do you want me to call
someone? The results show that you have cancer of the thyroid gland. The type of cancer that
you have is called papillary carcinoma. It is the most common type of thyroid cancer that usually
affects young people. Fortunately, this cancer is a slow-growing nodule. It usually spreads by way
of the lymphatic system to the local lymph glands. The good news is that it carries a good
prognosis which means that it is quite responsive to treatment.

- PAUSE. Do you understand everything? Is there any part of the discussion you want me to
repeat? Your care from here onwards will be taken up by a team of specialists including the
thyroid surgeon, specialist oncologist, radiologist, as well as myself as your GP. First of all, we
need to find out at what stage the cancer is. We will do it by doing some blood tests as well as CT
scans. The most likely management includes surgical removal of the whole or part of the thyroid
gland.

This is called total thyroidectomy with removal of affected lymph glands. Usually, the whole
thyroid is taken out. However, lobectomy is performed if the size <1cm, unifocal, and if no lymph
nodes are proven to be involved. After the surgery we usually use RAIU to kill any cancer cells
that might have escaped. Most likely, we will need to replace thyroid hormones with the help of
medications for the rest of your life. The physician will explain to you about thyroxine treatment
later on. Usually with papillary carcinomas, chemotherapy and radiotherapy are not proven to
be beneficial. You will need to come for regular follow-ups after surgery where we will be doing
regular neck USD, isotope scanning, as well as measurement of serum thyroglobulin levels.

- The 5-year survival rate after treatment is 80-95% for papillary carcinomas. The poor prognostic
factors are being male, age is more, if larger than 1 cm, if tumor is poorly differentiated, if local
or distant spread has occurred. I am sending you to the thyroid surgeon and oncologist.
- If you feel you cannot cope with the stress, please contact me at any time or the hospital. If you
like, you can bring your family or partner for further discussions.
- Reading material

- 5-year-survival for Follicular CA: 92%

230-DVT leg
6th Sept 2018 recall
Patient had a long flight. No risk factors for dvt. Investigation show dvt in rt calf.
Task
Take history short.
Explain the condition.
Immediate management

Case (22/4/2017)
DVT case- results she came its dvt we have to explain her y she got it. She travelled to Europe 28
hrs flight.she was sleeping for 10 hrs continuously I asked her u went to gp she said yes.. and
asked about family hx she said Mom have clotting problem. Said sorry for that. No ocps use,No
medical illnesses..I said because of long travel when u stay in one place circulation won't be
proper and draw a picture for her and said clot has formed in the leg that's wat u have swelling
and she don't have any symptoms like shortness of breath ..
I said will refer her to specialist they will give u anticlotting medications heparin she was tensed I
said it's manageable we want to treat it otherwise it can travel to heart and lungs other parts of
the body and stopped. and said should do thrombophila screening, its like for all clotting factors
test if it's positive should use one more drug called warfarin life long..
But don't worry it's because of travelling u will be fine..she said u want me to tell anything I said I
anytime u feel difficulty breathing or swelling gets big u should come here thanked and came
out.. I should have told her next time before travelling walk in between..move ur legs and come
to me I didn't say.. and reading material..

DVT counselling
28 year old female comes to see you with right calf pain that developed a day after she flew back
to Melbourne from New York. A Doppler ultrasound shows multiple clots in the legs.
Tasks:
- Take a relevant history
- Explain the diagnosis and management

Karin case Deep Vein Thrombosis


Case: You are an HMO in ED and a 21-year-old female comes to you complaining of calf pain. She
had been seen by the specialist and an USD of the lower leg confirms DVT.
Task
a. Relevant history (just came back from a flight from New York; + FHx of DVTs; on OCP)
b. Explain diagnosis
c. Immediate and Long-term management

History
- Where is the pain? How bad is the pain? When did it start? Is it continuous? Is it getting worse?
Does it go anywhere else? Is there any associated shooting type of pain radiating towards the
foot? Did you hurt yourself in that area? Is there any associated fever, SOB, cough, chest pain?
- Did you have recent surgery? History of recent travel? Where did you go? How long was the
flight? Did you move around during the flight? Did you have any recent fractures or trauma to
the bones? Any history of clotting problems in yourself or family? Who had it? Did they have any
tests done for that? Have you ever suffered from any type of cancers?
Have you noticed any recent changes to your weight, appetite? Any lumps or bumps? How do
you consider your weight to be? When was your LMP? Are they regular? What is the cycle like?
Are you in a stable relationship? What method of contraception do you use?

Any chance you could be pregnant right now? What is your occupation? Any stress in your life?
SADMA (blood thinners)? Do you have any varicose veins? When was your last pap smear?

Management
- Let me explain to you what DVT means. Basically, there is a clot within the deep veins of your
legs. It could be because of a number of causes. Most likely, because you were immobile for a
long period of time during the flight, there is pooling of blood in the veins that leads to the
formation of clots. Sometimes the use of the pill can be a risk factor but I am wondering if the
family history may be relevant in this case.
- DVT carries certain risks including swelling and pain that gets worse with time or the clot might
travel toward the lung (PE) compromising its blood supply that can sometimes be life-
threatening. There is also a risk of defects developing in the coagulation system (consumptive
coagulopathy).
- We need to admit you to the hospital and I will call in the registrar. We will do some tests which
include FBE, coagulation profile including D-dimer, thrombophilia screening, tumor markers (if
suspecting malignancy), LFTs, and stool for occult blood. Later on, the specialist might decide to
do CTPA to check whether the clot has reached the lungs.

- What is important at this time is to start you on blood thinning medications known as heparin
and warfarin which prevent further clots from forming. Usually, we get injections of heparin
once or twice a day and at the same time, we will start you on warfarin. It is a tablet that you
need to take 2 or 3x a day. The aim of the treatment is to prevent further clotting in the lungs
and reduce the risk of post-thrombotic syndrome (coagulation defects which leads to DIC).

- We will check daily INR levels. Usually, a target is reached within 5-10 days and we will then
stop heparin and continue with warfarin. Depending upon the results of thrombophilia
screening, the warfarin might be continued lifelong. If no cause is detected, you can stop taking
the medication after 6 months after specialist consultation. I want you to be aware of certain
side effects to this medication. If you develop symptoms like excessive heavy periods, black
stools, nose bleeds, and unexpected bleeding after minor trauma, please contact the hospital
ASAP. If you need to take other medications, please inform your doctor beforehand because
there are certain drug interactions that might affect the metabolism of warfarin (antibiotics,
antihistamines, NSAIDs, antacids, ASA, antifungals).
- If you require any dental procedures or dental surgeries, you need to inform the surgeon
beforehand. I would advise for you to stop the pill as it is contraindicated in patients with clotting
problems. You can use another form of contraception like condoms.
- Before you travel again, please come and see the doctor as there are certain precautions that
you need to take like increasing fluid intake to maintain hydration, avoid prolonged sitting and
try to walk every hour or so. I can teach you some leg exercises that you can do while sitting.
Your doctor might decide to put you on heparin injections 2 days before traveling. Also you can
wear elastic stocking during your travel. I will give you some written material to read about these
precautions.
- Can I become pregnant? At the moment you cannot become pregnant because warfarin can
produce defects in the baby.
- Can I be treated at home? Not really.
- Other options: mechanical thrombectomy, stenting, vena cava filter, rTPA

231- Fundus less than dates (Handbook


materials)
Your next patient is a 28-year-old primigravida who works as a nurse in a renal transplant unit. You have
been looking after her pregnancy since the first trimester. You are seeing her in a general practice setting
in a shared care arrangement with the local obstetric hospital. All appeared to be normal up to and
including her last visit at 30 weeks of gestation, when the symphysis-fundal height was 28 cm. Today, four
weeks later, the symphysis-fundal height is 29cm and there appears to be a reduced amount of liquor
present.

TASKS:

 Take any further relevant history you require.


 Ask the examiner about relevant findings likely to be evident on general and obstetric
examination and available investigation results.
 Advise the patient of the diagnosis and subsequent management including any further
investigations you would arrange.

APPROACH

 History
o I read from the notes that you are currently in your 31st week of pregnancy now, and
your fundal height seems to be smaller than expected and there is reduced amount of liquor
in your tummy.
o How is the baby? Is the baby kicking well? Are the fetal movement same as it was
before?
o Possible causes of small fundus:
 Are your dates certain? Did you do your ultrasound at 18 weeks? What was the
result?
 Were you ever diagnosed with hypertension or renal disease? Ever been
diagnosed with lupus or arthritis or any thrombotic disorder in the past?
 Pre-eclampsia questions: any headaches, blurring of vision, edema or swelling,
proteins or glucose in the urine?
 Do you have pets at home? Did you have any contact with dogs or cats?
o Routine antenatal history: Any blood test and blood group done? Down syndrome
screening? Folic acid? Any complications in the position of the placenta? Sweet drink test at
28 weeks?
o Do you smoke, drink alcohol or take recreational drugs?
o Do you have enough support at home? Who do you live with?
 Physical Exam
o General appearance: pallor, edema, lymph node enlargement, BMI
o Vital signs
o Systemic examination
 CNS/CVS/Respiratory
 Abdomen: fundic height, FHR, lie, presentation, is the head engaged, uterine
tenderness
 Pelvic examination
 Inspection of the vulva and vagina
 Speculum: is the cervix closed or open? Is there discharge or bleeding?
o Office test: UDT for proteins
 Investigations
o CMV antibody testing
o Toxoplasma antibody testing
 Management
From history and physical examination, most likely your baby is having an intrauterine growth
restriction. This is when the baby is not able to grow properly inside the womb. This is why your
fundal height is less than expected and you have reduced amount of liquor. The reason for this, and
its severity need to be assessed by some investigations such as ultrasound examination to confirm
the size of the baby, to look for the amniotic fluid volume, and to see whether there is any obvious
congenital abnormality which might explain the IUGR. Ultrasound will probably need to be
repeated each 2-3 weeks. A Doppler study could also be done to assess the blood flow in the
umbilical cord which supplies the needs of the baby. A cardiotocographic evaluation will be done as
well twice a week from now until the time of delivery to assess and monitor the condition of the
baby.
There are many causes why IUGR could happen. It could be due to karyotypic abnormalities or
problems in the genetic makeup of the baby, kidney disease, pre-eclampsia or abnormal, abrupt
elevation of the patient's blood pressure with leakage of proteins into the urine,
or congenital infections due to CMV or toxoplasmosis or placental dysfunction. That is why, in
addition to the ultrasound, Doppler and CTG, we would also do a serum urea, uric acid, and
creatinine to look for evidence of renal compromise, lupus anticoagulant and anticardiolipin
antibodies, antibodies for toxoplasmosis, and amniocentesis to assess the karyotype of the baby
and to rule out toxoplasmosis in the specimen as well.
Your baby will most probably be delivered via caesarean section and will be done prior to your due
date. The timing of this will depend on the ultrasound evaluation, the CTG record and the amount
of growth which occurs subsequently.
But do not be too stressed, as long as the baby does not become deprived of oxygen and becomes
acidotic, and does not have a congenital malformation or a congenital infection, the long-term
prognosis for the baby is satisfactory.

KEY ISSUES

 Understanding the causes of oligohydramnios.


 Management of a growth-restricted fetus in late pregnancy.

CRITICAL ERRORS

 Failure to recognise that the clinical picture is of intrauterine growth restriction.


 Failure to arrange appropriate assessment (or referral to specialist). At the least ultrasound and
serial CTG must be done.

IMPORTANT POINTS FROM THE COMMENTARY

 This case assesses the candidate's ability to understand the causes of a fundus measuring less
than that expected for the gestational age. The most common of these is that the fetus is suffering
from intrauterine growth restriction (IUGR) with usually a degree of oligohydramnios.
 The important aspect of this case is to have an understanding of the management of a fetus
which is growth restricted in late pregnancy. The means of monitoring the fetus for the remainder
of the pregnancy must be known, and also the options for safe delivery. Referral to a specialist may
be advised, but the candidate will be expected to have an understanding of what the specialist will
advise the patient.
 Common problems likely with candidate performance are:
o Failure to consider the various causes of apparent oligohydramnios.
o Failure to advise that delivery prior to the estimated date of confinement will probably
be required and the chance of this needing to be by Caesarean section is certainly increased.

232-Hyperemesis gravidarum
Condition 144 Nausea and vomiting in the first trimester (Handbook materials)
Your next patient is a 38 year old woman who has come to the general practice because of severe nausea
and vomiting for the last two weeks in this, her first pregnancy. She claims that she has been unable to
keep foods or fluids down. Her last menstrual period was eight weeks previously, and pelvic examination
by your colleague in the general practice two weeks ago showed the uterine size was appropriate for
gestation and a pregnancy test was positive. She has had no previous operations or illnesses
TASKS
1. Take any further relevant history you require
2. Ask the examiner about relevant findings evident on general and obstetric examination which
would assist you in making a diagnosis
3. Advise the patient of the likely diagnosis
4. Advise the patient of the care you would advise for her, including any investigations you would
arrange

APPROACH
o HEMODYNAMIC STABILITY--VITAL SIGNS: What is her current BP and is there a postural drop,
pulse and rhythm, respiratory rate, oxygen saturation, and temperature?
o If UNSTABLE: please transfer the patient to the treatment room, secure IV lines, and take
blood for FBE, UEC, ESR, CRP, blood grouping and cross-matching. I would also like to do
urine dipstick to check for ketones. Please start IV fluids: normal saline, hartmann solution or
Haemaccel whichever is available.
o Please give her an oxygen mask and start high flow oxygen at 10/L (if rr / o2 sat
unstable)
o HISTORY
o Congratulations on your pregnancy. I have read from your notes that you are currently at
8 weeks of pregnancy, and you have been suffering from vomiting for about two weeks now.
I know this can be very distressing for you, but we'll do our best to manage you. Could you
tell me more about your vomiting?
 Is it getting worse? Does it usually come in the morning or a particular time
during the day? Could you describe to me the manner in how you vomit--is it
projectile, do you retch, etc? What does your vomit usually consist of? What's its
color? Does it have any blood? Did you eat anything out of the usual before you had
these symptoms?
 Any changes in your bowel motion? Do you still pass gas? (r/o bowel
obstruction or gastroenteritis)
 Assess dehydration
 How is your appetite? Are you still eating or drinking? Did you have
any fever, diarrhea, or dizziness? How is your waterworks? Any burning or
stinging sensation? Do you go to the toilet more/less than the usual? Any
change in the color of the urine? Any loin pain?
o QUESTIONS ABOUT CURRENT PREGNANCY
 Is this a planned pregnancy? How were you able to confirm your pregnancy?
 Are you in a stable relationship? Do you have support for this?
 R/O causes of vomiting
 Do you have any family history of twins?
 Is this a natural or an assisted pregnancy?
 Any bleeding or tummy cramps?
 Did you have your initial blood tests requested during your previous visits to
your GP? Were you advised regarding screening and confirmatory tests for diseases in
the baby?
o Any history of STIs?
o When was your last pap smear?
o Do you know your blood group?
o Do you smoke, drink alcohol, engage in recreational drugs?
o Do you take other medications? Any allergies?
o Do you have any history of any medical or surgical conditions?
o PHYSICAL EXAMINATION
o GA: dehydration--skin turgor, CRT, tongue & oral mucosa moist? Lymphadenopathies,
pallor?
o VS: BP with postural drop? Temperature? HR?
o CVS, CNS, Respi
o Abdomen: distended or any masses? Tenderness? Rigidity/guarding? Any bowel sounds?
o Pelvic exam not needed
o Office tests: Urine dipstick and BSL
o Check for ketones
o PE findings from the case:
o She looks unwell and drawn. Her tongue dry and firm. Tissue turgor of the skin is
diminished.
o Pulse: 110/min
o BP: 120/80
o Temperatue 36.8C
o Abdominal examination, uterus not palpable
o No loin tenderness
o Pelvic examination not repeated
o DIAGNOSIS AND MANAGEMENT
o From history and examination, it seems that most likely you have a condition we call as
Hyperemesis gravidarum. Have you heard about it? It is a condition common in early
pregnancy manifested by excessive nausea and vomiting. Its cause is multifactorial, however
it is usually implicated that the excessive vomiting and nausea is due to increasing hormone
levels especially the b-hcg hormone which supports your pregnancy.
o This condition usually goes away on its own, especially by 14 weeks of gestation as your
body becomes used to these new hormones, and the level of b-hcg goes down.
o However, it was seen from your examination that you are severely dehydrated,
supported by findings of ketones in your urine. Because of this, I will have to refer you to a
hospital so that you can be admitted for intravenous rehydration and monitoring. I will call an
ambulance to transfer you to the hospital.
o I will also refer you to a specialist who will see you and might do further investigations.
Although hyperemesis gravidarum can be a complication of a normal pregnancy, it also
occurs with increased frequency in association with other conditions such as a multiple
pregnancy, a urinary tract infection, or even a condition we call as a hydatidiform mole--
where there is an abnormal growth of placenta mimicking pregnancy. To rule out these
conditions, you will undergo blood tests such as an FBE, UEC, serum b hcg, liver function
tests, and also urine microscopy and culture, and a transvaginal ultrasound. You will be given
fluids through IV, and anti-vomiting/nausea meds (metoclopramide - maloxon/stemetil),
vitamin B supplementation (pyridoxine) to address your symptoms. You and your baby will
continually be monitored throughout your admission.
o Another thing that I would like to address is that you are currently 38 years old now, and
this puts you at a very high risk of having a baby with Down syndrome or other genetic
abnormalities.
 SHORTCUT: Because of this, I will arrange genetic counselling for you to
undergo screening tests for down syndrome which consists of a blood test done as
early as 9-13 weeks looking for factors in blood which denote possible presence of a
baby with down syndrome, combined with an ultrasound examination done at 11-13
weeks. We can also do a screening test in your second trimester which will also involve
a blood test looking for 4 factors in the blood (AFP, BHCG, ESTRIOL, INHIBIN) which will
screen for the condition. We can also do confirmatory tests: the chorionic villous
sampling test done during 11-14 weeks, which involves getting samples from your
placenta and to send it for genetic testing. We can also do amniocentesis, which can
be done at 15-18 weeks, which involves getting a sample of your bag of water and we
send it for genetic analysis for down syndrome. All of these confirmatory tests have
certain risks for miscarriage with 1:100 for CVS, and 1:200 for Amniocentesis.
 FULL EXPLANATION: We have two screening tests, we usually do blood tests
looking for factors namely the bhcg and pappa as early as 9-13 weeks, together with
an ultrasound of the baby's back of the neck at about 11--13 weeks of your pregnancy.
The bhcg is inc, and pappa is decreased in a baby suspected of having downs. The
detection rate of this combined test is 87%
Another test that we do is the Noninvasive Prenatal Test (NIPT) which is done at 10
weeks. We get a blood sample from you and we send it for genetic testing. The
detection rate is 99% but it is not covered by medicare and usually costs around 600-
700aud.
For the second trimester, 15-17wks. we can also do what we call a quadruple and a
triple screen. We test factors in your blood, namely the bhcg, inhibin, esriol and afp.
Both bhcg and inhibin are increased while the other two are decreased in a quad
screen.we test the bchg, estriol, and afp in the triple screen. Detection rates are 81%
and 71% respectively.
Once we get positive for downs in the screening tests, we do confirmatory testing for
downs
Chorionic Villous sampling that we do during 11-14 wks of pregnancy where
we insert a needle guided by ultrasound from down there and get a sample
from the placenta which we send for genetic testing. However miscarriage rates
are 1:100.
We can also do an amniocentesis, which is done at around 15-18 weeks. We
pass a needle guided by an ultrasound to your womb, to get a sample of your
bag of water and we send it for genetic analysis. Risk of miscarriage for this is
1:200.

KEY ISSUES
 Ability to investigate and treat a woman with hyperemesis gravidarum
 Recognition of the need for genetic counselling in the view of advanced maternal age

CRITICAL ERRORS
 Failure to recognise the need for hospitalization
 Failure to do ultrasound and urine examination to check pregnancy, diagnose twins, molar
pregnancy, urinary infection, and the presence of urinary ketones

IMPORTANT POINTS FROM THE COMMENTARY


 Management depends upon the degree of vomiting and therefore the potential diagnosis.
 Common problems
o Failure to advise that the vomiting will usually cease or be markedly reduced by around
14 weeks of gestation

Handbook 144
Condition 144 (R4)

Nausea and vomiting in the first trimester in a 38-year-old primigravida


CANDIDATE INFORMATION AND TASKS

Your next patient is a 38-year-old woman who has come to the general practice because of severe nausea
and vomiting for the last two weeks in this, her first pregnancy. She claims that she has been unable to
keep foods or fluids down. Her last menstrual period was eight weeks previously, and pelvic examination
by your colleague in the general practice two weeks ago showed the uterine size was appropriate for
gestation and a pregnancy test was positive. She has had no previous operations or illnesses.

YOUR TASKS ARE TO:

• Take any further relevant history you require.

• Ask the examiner about relevant findings evident on general and obstetric examination

which would assist you in making a diagnosis.

• Advise the patient of the likely diagnosis.

• Advise the patient of the care you would advise for her, including any investigations you would arrange.

AIMS OF STATION

To assess the candidate's knowledge about the diagnosis and management of hyperemesis gravidarum.

EXAMINER INSTRUCTIONS

You are 38 years old and have had severe nausea and vomiting for the last two weeks. You are now eight
weeks pregnant in this, your first pregnancy.

The examiner will have instructed the patient as follows: Appropriate answers to likely questions are:
Task 1

• No symptoms of urinary frequency, or pain with voiding. You need to get up once a night to empty your
bladder. (UTI- FREQUENCY, DYSURIA, NOCTURIA)
• No vaginal bleeding.(BLEEDING)
• Urine output satisfactory and apparently normal.(DEHYDRATION- URINE OUTPUT, FLUID INTAKE)

• You have kept some fluids down today but have had difficulty keeping any fluids or food down

for the last week or two.

• No family history of multiple pregnancy.(FAMILY HISTORY- TWINS)

• This is a spontaneous pregnancy and fertility drugs have never been used. You have been
trying to conceive for the last 12 months.

• You are aware of the increased risk of Down syndrome due to your age, and will want tests

done to ensure this baby does not have Down syndrome. If it did, you would wish the pregnancy
terminated.

Questions to ask unless already covered:

• 'Why am I sick?'

• 'How long is the sickness going to last?'

• 'Will my baby be OK?'


Task 2

Examination findings to be given to candidate by the examiner on request

She looks unwell and drawn. Her tongue is dry and firm. Tissue turgor of the skin is diminished.

Pulse 110/min.
Blood pressure 120/80 mmHg.
Temperature 36.8 °C
Abdominal examination, uterus not palpable No loin tenderness.

Pelvic examination not repeated.

EXPECTATIONS OF CANDIDATE PERFORMANCE


The candidate should convey the substance of what follows to the patient:
Task 3

• Hyperemesis gravidarum is a common condition in pregnancy, with significant nausea and vomiting in
up to 10% of patients, a small number of these requiring admission to hospital for

intravenous fluids and antiemetics.


Task 4

• In order to assess the severity of the process, a urine specimen needs to be tested for

ketones. If ketones are present, admission for intravenous rehydration is usually required. If the
candidate indicates the need for such ketone testing, advise that this was strongly positive. As the ketone
test was strongly positive, admission is required.

• Although hyperemesis gravidarum can be a complication of a normal pregnancy, it occurs with increased
frequency in association with multiple pregnancies, hydatidiform mole, and in association with urinary
tract infections. Ultrasound examination is therefore required to rule out the former diagnoses, and a
midstream urine specimen should be collected and subjected to culture to rule out a urinary infection.

• Treatment in hospital consists of rehydration with saline solutions and additional dextrose, usually
with vitamin B supplementation. Pyridoxine, or antiemetic therapy with Maxolon® or Stemetil® can also
be used in an attempt to resolve the vomiting.

• Investigations such as assessment of the serum electrolytes and urea, and liver function tests should be
performed to assess the degree of effect of the vomiting on the maternal bodily function.
• Usually the hyperemesis settles spontaneously, often having reached a maximum at about 70 days (ten
weeks) of pregnancy, and by the time of 100 days (14 weeks) most of the symptoms will have resolved.

• If nausea and vomiting do not settle satisfactorily, or following initial therapy, other causes such as small
bowel obstruction, cerebral tumour or Addison disease need to be excluded.

• As she is 38 years old, and is therefore at increased risk of a chromosomal abnormality of the

baby, genetic counselling should be arranged with advice given concerning the usefulness of the
quadruple maternal serum screening and nuchal fold thickness assessed by ultrasound examination
(screening procedures only) or the use of chorion villus biopsy (CVB) or amniocentesis to assess the fetal
karyotype and actually rule out Down Syndrome. As she has indicated this diagnosis needs to be ruled
out, the definitive tests of CVB or amniocentesis should be advised following advice to her concerning
the potential complication rates of each of these procedures (risk of abortion due to the procedure is
0.5% for amniocentesis at 15-16 weeks, and about 1.5% for CVB at 11-12 weeks).

KEY ISSUES

• Ability to investigate and treat a woman with hyperemesis gravidarum.

• Recognition of the need for genetic counselling in view of advanced maternal age.

CRITICAL ERRORS

• Failure to recognise the need for hospitalisation.

• Failure to do ultrasound and urine examination to check pregnancy, diagnose twins, molar

pregnancy, urinary infection, and the presence of urinary ketones.

Karin case
Case: 38 year-old woman who came in with a 2-week history of nausea and vomiting. She is 8 weeks
pregnant and her pregnancy is consistent with GA. She has no previous illness.
Task
a. History
b. Investigations (1 only) – MSU
c. Diagnosis and management
Differential diagnosis:
- Multifetal pregnancy - Hydatidiform mole (complete/incomplete) – UTI
- Infectious Gastroenteritis - Brain tumor/Addison disease/bowel obstruction
History:
- IS MY PATIENT HEMODYNAMICALLY STABLE?
- Congratulations on your pregnancy.
- How many episodes of vomiting did you have per day? - Is it getting worse? Is it in the morning or
throughout the day? What is the content?
- Do you have fever? Diarrhea?
- How is your appetite? Are you still drinking eating or drinking? - Did you eat outside?
- How is your waterworks? Any pain or burning sensation? Any increased frequency? Any change in
color of urine? Any loin pain?
- Pregnancy: is this a planned pregnancy? How did you confirm your pregnancy? Any family history of
twins? Is the pregnancy natural or assisted? Any abdominal cramps or vaginal bleeding?
- Periods? Pills? Partner? Pap smear? Blood group?
- SADMA?
Examination:
- General appearance: tired, signs of dehydration (tongue, skin turgor, CRT?
- VS: BP (check for orthostatic hypotension); PR (tachy), RR, T – normal
- Chest, heart, abdomen – normal
- No pelvic exam needed.
- Urine dipstick – nitrites, ketones (+), leukocytes
o MSU: (+) for ketones! – admit!!!
Investigations:
- MCU - Ultrasound examination - Test for electrolytes, urea, LFTs
Diagnosis and management:
- You have a condition called hyperemesis gravidarum. It means excessive nausea and vomiting in
pregnancy. These are common symptoms during initial pregnancy. However, 1 in 1000 women will have
excessive vomiting and require hospitalization.
- On examination, you are dehydrated and this was confirmed in urine analysis, so we need to admit you. I
will organize an ambulance.
In the hospital they will secure 2 IV cannulas, take the blood for FBE, U/E/, RFTs and LFTs because
dehydration can affect the liver and kidney. We need to do MCS to rule out UTI and USD to confirm
intrauterine pregnancy, rule out multiple pregnancy and molar pregnancy.
- They will also give medications to stop the vomiting (metoclopramide – mexalon, stemetil) and start IV
fluids and vitamin B6 (pyridoxine).
- We don’t know the exact mechanism behind it. However, it is usually due increased level of b-hCG which
is one of the pregnancy hormones. Once the body has become used to the new environment, the nausea
and vomiting settles and this usually happens by 14 weeks.
- You are a 38-year-old mother and that puts you at a very high risk of having a baby with Down syndrome.
So I would like to offer you screening for Down syndrome (during your 10th week – blood plus USD).

Case (29/11/2017)
Feedback (1st trimester complication)
You are a GP, a young female comes to you with 10 weeks of gestation with severe vomiting, not able to
keep anything down. Her antenatal course has been uneventful till now. ANC tests done.You are seeing her
for the 1st time. Further investigation, Tell her the diagnosis, management and explain the reason for this.
Greet, introduce
Took a bit of history- anc checkup done or not
Planned pg?
Folate intake, FH of chromosomal prob, multiple pregnancy, pv bleeding or discharge,
any ovulation inducing drug
With vomiting fever, tummy pain and other prob
All negative
Mx- common condition, hyperemesis
Vomiting can u due to urinary infection, multi pg, molar pg, chromosomal prob even along with other
health conditions but also a normal variant. Vomiting due to body’s adjustment to pg and pg hormones
.Usually goes away once the 1st trimester is over, but as u are not able to keep anything down. Will admit
u. try out some IV fluids and meds. Run some ix- urine test (ketone, protein, leukocyte didn’t mention
details), liver test, kidney test, few blood tests(b hcg, electrolyte) and usg. Is that ok with u?
Want me to call someone?
Support at home?

Feedback 7-9-2018
Station 9 – hyperemesis gravidarum – fail
STEM : ( stem is very long )
middle age lady – UPT – positive
LMP 8/52 ago
nil bleeding , nil abdominal pain
Nil family history
Other doctor did
nil FBE ( sure )
blood group – B positive
Antibody – negative
HIV , Syphilis , Hepatitis screen – NEG.
On examination :
- nil fever, pelvic examination – os – closed, others – NAD , uterine size 8/52 ,
- vomiting several times since 2 days ago ( forget exact dates )
TASK
1. Explain most likely cause and other possible causes to the patient
2. further investigations to the patient ( routine , other MMR serologies , serum Beta-hcg , U/S of
abdomen ) , refer to the specialist and explain about referral ( though mgmt is not necessary in the TASKS)

Feedback 30-5-2018
‘First trimester complication’:
8/40 c nausea & vomiting (hyperemesis gravidarum). Benign 1st ANC bloods given in stem.
Task: Hx, counsel what Ix to do
Feedback: Pass

 HOPC

o Basic rapport (you’re blds are reassuring, I’ll be going through your concerns step by
step, just need to ask some focussed Qs 1st)

o PregQs (basic ANC summary)

 LNMP (any issues)

 Planned/partner/support

 Blood group
 HPV

o When/how many episodes/keep anything down

o Eat anything funky/BOV/swelling/Pain/CCVO

o Any PV d/c /ccvo/grape-like tissue loss

o DdxQs

 Bowel/bladder func

 Fever

 FHx twins

 Ax & P

o Explained that it fits with n/v of early preg but needs confirmation

o 2/2 pregnancy hormone (bHcg)

o I’d like to examine you now and arrange…(as not given as task: I felt the need to mention
for sake of maintaining practical sequence of events the way they would occur in real
life)

o Basic ANC bloods (preg test)

o Send you to HRPC where you go to the same ANC but more frequent monitoring of you
& bub under MDT

o We need to assess the extent to which you’re producing the preg hormone & US to
characterize the pregnancy (as it depends on the bHcg to be at a certain level)

o Drew an arrow from 1 – 40/40 and explained 18/40(USS), 28/40(OGTT), 36/40(GBS)

 And more freq US & CTG & bHcg monitoring

o Other possibilities we need to r/o, not to scare you are

 Molar (GTD –with brief outline that it’s a growth not preg)

 Multiple pregnancy

 Gastroenteritis

o Summarized
Feedback 7-9-2018 hyperemesis gravidarum (important)
First trimester complication- PASS)
GP, A 22 year age old, young lady , c/o repeated vomiting for last 2 days. She has checked with her gp last
week- UPT-positive. 8 weeks. First pregnancy. Full blood invx- normal. HIV, Syphillis, Hepatitis A,B- Normal.
Blood group was given. No known medical condition. Her BP- 120/80, pulse 76/min- basically all vitals are
stable. A liitle bit hx and PE was given to indicate hyperemesis gravidarum.
- Explain dx, ddx
- Further invx you want to send
- Mx.
FEEDBACK-PASS Global Score: 5
Key Steps: 4/4 Approach to pt: 4 Choice of invx: 5 DDX: 5 MX: 4
2 mint thinking- PE details were not given, urine ketone bodies- present or not?( Dehydration??) vitals
stable- shall I refer to hospital?? I was confused to be honest about mx. Medication- metocloromide+ vit
B6.
I failed this case in my trial exam because I didn't admit the pt in hospital.
DDX:
1. Hyperemesis gravidarum 2. Multiple pregnancies 3. Molar pregnancy
4.UTI 5. Acute gastroenteritis
( I mentioned all of them, twice. I had 3/4 mints extra in this case!! No hx, just ddx, invx, mx- I sat down
quietly inside the room for few minutes!! :D)
As vitals were given outside, I didn't ask anything to examiner.
I directly started with the role player, an young lady, sitting comfortably.
I started like: Are you comfortable to talk right now? would it be alright if I start to explain why you are
having this? (yes)
I mentioned all the ddx with explanation and asked few questions in between as well.
Multiple pregnancy- have you taken help of assisted pregnancy clinic? Any family hx of twins?(no) . Okay,
sometimes, this symptoms are more common when multiple foetus insides. That's why I'm just asking you
this. Do you have family hx of twins? (No)
Molar pregnanacy- Have you noticed any bubbles like structure coming down below? (no) Okay. alright.
This condition we called molar pregnancy, where foetal parts do not grow well and forms bubbles like
structure inside. This actually resulted in higher level of pregnancy hormone into your body, that leads to
excessive vomiting. Seems like it's not in your case.
UTI- Any burning sensation while urinating? - no- Okay, good. It's also associated with urine infection.
Acute gastro- Any eating out recently? how's your bowel movements? - normal
Then, I mentioned, most probably, you are having a condition we called "hyperemesis graviderum' , It's
very common in first trimester and in first pregnancy. Our body reacts differently with this very new
change in your body because of hormonal disruption and pregnancy hormones start to rise. Don't worry,
this condition will settle down by 14 weeks. Make sense? (okay).
I will do further invx to rule out the other causes as well:
Blood tests: electrolytes. B-hcg level rising or not? high or normal?( I might forget to talk about LFT, RFT,
or it was mentioned in the stem)
Urine tests: Microscopy n culture, ketone bodies.
USG
I also mentioned, look we have already done these tests, so I'll not repeat those. I mentioned ddx again
while explaining invx to her to justify why I'll do it.
Mx: I asked; how are you feeling? As I can see, your vitals are stable now, can you manage yourself at
home? ( She said, yes) Then, I thought, might be it's a trap. I said then n there; good, but I would like to
refer you to hospital right now. It would be better for you as well, as they will do all the tests immediately
and specialist can come and check you up. They will give you medications, called metocloromide + vit B6,
to stop your vomiting and nausea feelings. Okay? (Yes). Shall I call an ambulance for you? (no) Is there
anyone with you atm?(yes, my husband)
233-Placenta Previa
Sample case/ You are an HMO at the ED of a tertiary hospital when 28 year old Jenny, who is in
her 34 weeks of gestation presents to you with vaginal bleeding.
TASKS
Further history
Examination findings from examiner
Diagnosis and management

Differential Diagnosis
1-Placenta Previa 2-Abruption placenta 3-Trauma 4-Bleeding disorders 5-Blood thinner meds

History
1-Is the patient hemodynamically stable?
2-Bleeding questions (duration, action, trauma, amount or severity, colour, odour, content,
dizziness, bleeding disorder and blood thinner)
- for how long have had vaginal bleeding? (Past 2 hours) is this the first episode of a bleed during
your pregnancy?
- What were you doing when you got the bleed?
- Any chance you had a hit to your tummy?
- How many pads have you used so far? Is it (are they) fully soaked or not?
- What is the color of the bleed?
- Is it smelly?
- Any clots?
-any tiredness, dizziness or funny racing of the heart?
-have you had a history of bleeding disorders? Do you take blood thinner medication?
3-Late pregnancy complications questions
- Any tummy pain? ( abruptio placenta, preterm labour)
- any fever, nausea and vomiting, abnormal offensive vaginal discharge?
- any headache, blurring of vision, leg swelling?
- Is the baby kicking well?
4-regular antenatal visits questions
-have you had regular antenatal checks,
- Have you done your pap smear? What was the result?
- Have you done your down syndrome screening?
- Ultrasound at 18 weeks? What is the position of the placenta? Any birth defects? Repeat
ultrasound at 32 weeks? What was the result? (not sure of the findings)
- sweet drink test at 28 weeks?
- are you aware of your blood group?
- Did you take your folic acid?

5-general questions
-Any past history of medical or surgical illness?
-smoking, alcohol or recreational drugs? (smoking cause placenta Previa)
-Do you have a good support?
Physical Examination findings from examiner
1-General appearance: (PODL) pallor, edema, dehydration, LAP
2-Vital signs: especially BP with postural drop.
3-quick chest and CVS examination
4-Abdomen:
-any uterine tenderness
- feel for any contractions
- fundal height (34 weeks)
- fetal heart rate (150 beat/min)
-fetal lie, fetal presentation (longitudinal and cephalic)
-Is the head of the baby higher than the lower uterine pole? (higher in previa)
5-Pelvic examination
Inspection of vulva and vagina:
-can I see any bleed? (+ve) Any clots?
-Any rash or vesicles?
Speculum exam:
-any bleeding from the cervix? (+ve)
-is the cervical OS open or closed? (closed)
**NEVER EVER DO A PER VAGINAL EXAM IN PREVIA (critical error)
6-Office tests:
UDT, BSL

Explanation (4C) if you do not know how to explain always remember 4C


Condition
-From the history and examination, most likely you have a condition called placenta Previa.
The placenta is the part that connects you to your baby, and which carries oxygen and food from
you to your baby. This placenta is usually situated on the upper part of your womb. But if it
attaches to the lower pole of your womb, it is called placenta previa. (draw)

Clinical feature
The bleed is mainly because of the head of the baby pressing against the placenta leading to
bleed from the placenta.

Cause
- the cause is unclear but it could be due to multiparty, advanced maternal age, chronic
hypertension, smoking, alcohol and drugs, previous C-section.

Complication
One of the major complications of this is bleeding, and you can go to a shock or coagulation or
you can go into preterm labor. The baby can have hypoxia or decreased oxygen supply, and
intrauterine growth retardation.
Management
- You need to be admitted and seen by the specialist.
- I am gonna put an IV line, take blood for investigation which include FBE, UEC, blood grouping,
cross-matching and hold. I will start you on IV fluid.
- Ultrasound will also be done to confirm diagnosis of placenta previa.

Ultrasound tell you what is happening (showed placenta Previa grade four)
Grade 1: low lying placenta previa, placental edge is not near the OS

Grade 2: marginal placenta previa, placental edge comes up to the level of the OS (Marginal PP)

Grade 3: partial placenta previa, partially covers the internal OS

Grade 4: total placental previa, completely covers the external OS]

- CTG will also be done to monitor the baby.


- bed rest at the moment.
-options
first: once you stop bleeding, wait in the hospital until 37 weeks when the specialist will do a
planned C-section. However, If the bleeding continues, and you become unstable, or if the baby
becomes unwell, an immediate C-section will be done.
Second: once you stop bleeding you can go home but if 3 criteria
*home near the hospital
*require constant companion
*need informed consent from the patient.
And the rest of your pregnancy should be in high risk pregnancy clinic
warning signs: bleeding, tummy pain, fluid discharge, baby not kicking well
I will give you reading materials about placenta previa for further insight.

Notes
***In minor grades (grade 1 and 2) of placenta previa, usually C-section is done in 38 weeks. If
major grades (grade 3 and 4), C-section is done at 37 weeks.

***any grade of placenta previa, any gestational age, if the patient has heavy bleeding, becomes
hemodynamically unstable, or if there is fetal distress, stabilize the patient and go in for an
immediate C-section.

***if the patient insists on vaginal delivery, tell that it is always a C-section that is preferred in
placenta previa (any grade). But a trial of vaginal delivery can be considered if the placental edge
is greater than 2cm from the OS and if the baby's head is below the placental edge.
Feedback
Case (11/2/2017)
you are a GP in a clinic 250km away from tertiary hosp. Ptn 34 weeks gestation comes for
antenatal visit. USG done recently and report says there is 4th degree placenta praevia, with
the position of placenta being mainly on posterior wall of uterus but covering the cervix.
Task:
Hx from ptn,
explain USG to ptn
management
Introduced myself, ask how is she doing.
A few questions on the pregnancy, if all is okay, baby kicking well etc. No positive hx there, no
bleeding or discharge from down below.
Then explained to her that the ultrasound shows something called placenta praevia. Drew the
uterus, and location of placenta. apologised that my drawing is horrible, i’m far away from being
an artist. As i was drawing, I told her the function of the placenta is for exchange of oxygen and
nutrients between mum and baby. Usually the placenta is located higher, here (i indicated on the
picture i was drawing) but for some reasons, its lower than it should be.
She asked me if it was dangerous, i said , yes it is a risky situation but not to worry we can
handle this. (I actually winged this case because I somehow did not read placenta previa during
my revision so if I’m wrong anywhere, ignore it).
I said that because of the location of the placenta, its too risky to have normal delivery so your
safest bet is a Caesarian section. What we need to do now is to do CTG. She didnt know what it
was so i explained that two (i cant remember what word i used, electrodes?) will be put on ur
belly and that will take the reading of the baby’s heart rate, movement and the uterine
contractions.
I would like to discuss with the specialist, and upon his discretion maybe you may need to be
sent to the tertiary hospital soon, but I will arrange all that accordingly. If not, you will need
regular monitoring wit possibly weekly USG and CTGs, also mentioned Blood grouping and cross
matching somewhere in my discussion.
Guys, I didn’t read this case and I really just talked on what I thought were the possibilities based
on my internship back at home 5 years ago.
Feedback: Late pregnancy complications, Pass, Global score: 4

Note/ according to the feedback, the patient presented to GP without bleeding or any
symptoms. She came for the results of ultrasound. So first we need to take history for 3 minutes,
no PE task then we need to discuss the result (Grade 4 placenta previa) I think the examiner will
give a card with the result or a picture and report. Just in case see images of grad 4. So explain
the results calmly, listen to the patient , draw a diagram. Then mx is to refer to specialist as her
home is far and you are in GP so refer to tertiary hospital to be seen by specialist, tell like option
1 in the lecture. Then the specialist will decide etc.. most likely CS planned 37 for such grade.

234-Bleeding in pregnancy
Feedback 4-7-2018

Feedback: Bleeding. Young female pt, with period was 6 week late, bleeding.
Task
-Hx,
-PEFE
-Investigations.

Hx:

How severe was the bleeding? Dizzy? Pain? (no)

6Ps? (Pt has nausea, breast and pelvic discomfort), but Pregnancy test 2 wks ago: Negative.

SADMA, …

PEFE: everything normal, pelvic examination: cervix closed, no pain, uterus height consistent
with 6 weeks pregnancy.

Investigations: Redo Pregnancy test right now and refer to USD, admit to hospital if pregnancy
test is positive.

Grade: PASS, Gs: 5

Key steps 1,2,3,4,5: Y (all covered)

Hx: 5, Choice of Examination:5, choice of investigations: 5

Note/ No dx in this case so just approach it like any previous lectures of vaginal bleeding with
general approach.

235-Solitary thyroid nodule


A man in his 30's come with complain of a lump in his neck for the past 3 years, now increased in
size which is in the anterior triangle, moves with swallowing
A) Explain the DD's
B) Further investiagtions

Case (30/5/2017) (31/8/2017)


Thyroid nodule: there was a role player was very long scenario in which by pe u found solitary
nodule explain ur result , likely dx and other ddx and wt inx u want to do plus further hx.
1-i said could be either cyst or solid
2-ddx hypo or hyper active thyroid.
3-could be colloid cyst( i was in trouble cause pt told me wt does this mean i told i will check by
lay term and explain to u).
4-could be LN
5-could due to nasty growth as well
6-other ddx i said thyroglossal cyst but unlikely cause need hx from birth .
I ordered US ,TFT i explained about FNA then radioactive study and other baseline inx.
I refer to specialist and in case she need sx she asked wt could happens if i do sx i told her u will
need thyroxine to replase the hormone as it help in our body metabolism i didnt have time to
say about recurrent laryngeal n. Injury.
Sorry was no hx here it was mx.
I just quick asked her about wt loss if any lumb bump else where amd explained why am asking
to rule out as could be features of nasty growth.
I didnt do well here and role player was so dull ( sorry but i felt that amd she was looking sleepy
already.

Karin case
Case: A 40 year-old female patient came in due to a lump on her neck. It moves with swallowing
and it was confirmed to be a thyroid nodule in the left lobe of the thyroid gland.
Task
a. Discuss differential diagnosis
b. Arrange appropriate investigations
c. Further management
Differential Diagnosis
- Colloid cyst
- Dominant nodule in a multinodular goiter
- True solitary nodule (adenoma)
- Thyroid Cancer
- Hashimoto thyroiditis
- Lymph Node
- Thyroglossal cyst *

Investigations
- TFTs and autoantibodies (TSI, TSH autoantibodies – Graves; antimicrosomal and
antithyroglobulin antibodies – hashimoto).
- USD (to check whether it is solid, cystic or mixed and locate site where to take biopsy)
- RAIU scan (hot, warm or cold nodule) o Cold: can be cancer; not taking up isotope scan o
Warm: takes up isotope; can be MNG, Graves, o Hot: taking up maximum isotope; usually a toxic
nodule (hyperthyroidism)
- FNAC (95%): benign, malignant, or indeterminate.

IF cancer: go for CXR, CT scan/MRI of the chest to see involvement of mediastinum, airway, and
lungs, ECG.

Management
- Colloid cyst: Aspiration and biopsy of wall of the cyst to look for malignancy; If recurrent,
surgeon may remove the cyst
- Dominant nodule of MNG: RAI ablation or thyroidectomy
- Adenoma: watchful waiting, regular reviews with TFTs and USG; if symptomatic or enlarged
surgery
- Carcinoma: total thyroidectomy
- Hashimoto thyroiditis (autoimmune): Thyroxine

Complications of Thyroid Surgery (followup with TFTs, USG and uptake scan)
- Bleeding - Infection
- Anesthetic complications
- Recurrent laryngeal nerve injury/palsy
- Thyroid crisis/storm
- Tension hematoma
- Hypocalcemia (hypeoparathyroid)
- Hypothyroidism (thyroxine)
- Recurrent Hyperthyroidism

Obstructive symptoms/Red flags - Stridor - Tracheal deviation - Dyspnea - Dysphagia - Neck vein
engorgement - Hoarseness

Thyroid scan - RAI will be injected into a vein of your arm while a special camera will be used to
produce an image of your thyroid in a computer screen. - It is a small amount of radiation that is
not harmful.

236-Temporal arteritis +polymyalgia rheumatica


Feedback 6-6-2018
Station 9 headache- Fail
57 yrs old lady, headache for 2 weeks.
Task, history, Dx ddx.
History: patient holding her left side of the head and temple, wants pain killer? No. headache
pain score , throbbing in nature, on and off, no special time, not related with food, no aura, no
photophobia, no visual pblm, no jaw pain on opening and closing the mouth, no cord like
structure at temple area, (other candidate gets the history of shoulder and hip pain and stiffness,
I forgot to ask about that), stress nil, fever nil, rash nil, neck pain nil, no head injury, early
morning vomiting nil, sinus pblm nil, limbs weakness nil, slurred speech nil, family history of ca
nil( forgot to ask FH of migraine), PMH, PSH nil, HRT nil, smoking alcohol nil
I am not sure of dx , I told her it could be migraine ( not a classic type though) and talked about
ddx. Other candidate gave dx of temporal artheritis, not sure she passes this case

Case (3/3/2017)
60yr old female with severe headache for past 2 weeks. Now worse.
Tasks
-History
-Explain dx and ddx
Left side temporal area pain. Throbbing. No visual disturbances no vomit no Low, no gait issues
or vomit. Has difficulty and pain chewing and opening mouth. Fmx of migraine

Case (7/3/2017)
72 old lady have headache for two weeks
Tasks
-take history for 5 mins
-tell diagnosis and ddx to pt.
Offered her a painkiller. Asked the history of headache everything she gave me the finding of lt
headache in temporal area and history of problem with chewing and pain in hips and shoulder
so temporal arteritis and told her the dd.

Case (7/3/2017)
HMO at ED, 72 yrs old lady complaint of headache.
Task
-history (5min)
-Diagnosis and DDx.
Finding-she showed her headache was around her right sided temple and severe, No radiation
but at that time I think it was right side of head like migraine, no vision problem or flashes, no
nausea and vomit, no FH of migraine or brain tumor, no travel, no medical problem. Everything
was fine. No positive finding apart from headache. I was panic again to give diagnosis only by
history. I should tell GCA first but I told her migraine. Others may be GCA, infection but u have
no fever, tension headache, head injury and less likely brain tumor. (Others told that TM joint
dysfunction in DDx)

Handbook condition 93
Temporal arteritis in a 58-year-old woman
You are working in a general practice. This 58-year-old woman has consulted you about the
recent onset (two weeks) of right-sided headache gradually becoming more and more severe,
and which is now constant. Over the last few days, the patient has also had a tight feeling in the
muscles on the right hand side of the face when chewing.

On physical examination you noted tenderness and tortuosity over the right temporal artery as
illustrated. Its pulsation cannot be felt as well as that of the temporal artery on the left. There
were no other abnormal physical findings. The patient is normotensive.
Based on this information you believe that the most likely cause of the patient's symptoms is
temporal arteritis ('cranial arteritis' or 'giant cell arteritis').

Brief Patient Profile

Married, works as an accountant. Nonsmoker. No significant past or family history except for
occasional migraine. Has been taking Panadol® (paracetamol 500 mg) for the headache.

YOUR TASKS ARE TO:

• Explain the diagnosis, and its implications, to the patient.

• Advise the patient about management — both immediate and longterm. This could include
any investigations you believe are necessary.

You do not need to take any further history. You have just concluded your physical examination
and are about to advise the patient of your diagnosis and management plans.

The Performance Guidelines for Condition 093 can be found on page 510

AIMS OF STATION

Toassess the candidate's knowledge of the treatment of temporal arteritis and its most
important possible sequel: visual impairment.

EXAMINER INSTRUCTIONS

The examiner will have advised the patient that she should ask about side effects of steroids and
whether some alternative medication is preferable, as she doesn't like the idea of steroids. She
should also ask about further tests such as X-ray and whether the headache might just be a
simple migraine.

Questions to ask unless already covered:

• 'Could it be a migraine?'

• 'Are there any (other) complications?'

• 'Are you sure that my eyesight will be all right?'

• 'Should I see an eye specialist?'

• 'Isn't "cortisone" dangerous?' (If 'cortisone' or 'steroids' are recommended)

• 'What are its side effects?'


• 'How long will the headache last once treatment lasts?'

• 'How long will I be on "cortisone"?'

• 'Can this trouble affect me in any other way?'

EXPECTATIONS OF CANDIDATE PERFORMANCE

This can become an emotive situation for the patient after being informed of the nature of the
condition and the possibility of severe visual impairment.

The doctor should not withhold this information, which should be given with empathy and
support. Politeness, respect and consideration rather than an authoritarian approach should be
demonstrated when discussing the threat of blindness and obtaining compliance with the use of
corticosteroids.

The doctor should listen carefully to the patient's queries and provide honest as well as accurate
answers. Generating trust and confidence and giving the correct level of reassurance are also
expected.

Initial management plan

An erythrocyte sedimentation rate (ESR) or C-reactive protein (CRP) should be arranged


immediately with request for a same day report (this requires liaison with the patient who
should contact the doctor later in the day).

The patient should be commenced on oral prednisolone in high dose at first (60-100 mg). A
stronger nonopioid analgesic than paracetamol should also be prescribed.

Referral to a surgeon with a view to temporal artery biopsy should be discussed and urgent
referral to an eye specialist should be advised.

Patient Education

The aetiology and prognosis of temporal arteritis are obscure. It is a manifestation of giant cell
arteritis.

Confirmation by ESR or CRP (usually markedly elevated) is essential, followed in most cases by
biopsy of the superficial temporal artery for confirmation because of the likely need for medium
to longterm corticosteroid therapy. Commencement of oral steroid therapy before completing
investigations is indicated to reduce the risk of visual impairment, especially in a case of this
duration (two weeks).

This patient should be seen again within 48 hours by which time significant resolution of
symptoms should have occurred. Once symptoms are controlled and ESR levels fall, the
prednisolone can be reduced to maintenance levels (5-10 mg three times daily).
The patient should be monitored closely by continuing review of symptoms and serial ESR levels.
Resolution may take up to 2-3 years. Concomitant use of H2 receptor antagonists should be
considered in patients with a history of dyspepsia or peptic ulcer.

KEY ISSUES

• Skill in conveying unpleasant news to patient in an honest and supportive manner with
guarded reassurance about the outcome.

• ESR or CRP must be ordered with urgent early report requested.

• Must commence prednisolone therapy immediately.

• Patient counselling and education is required regarding possible biopsy, referral to eye
specialist and longterm

nature of the condition.


CRITICAL ERRORS

• Failure to request ESR or CRP.

• Failure to commence prednisolone therapy.

COMMENTARY

Although this is not a common disorder, the high risk of preventable blindness and response to
early treatment makes it essential knowledge. A highly probable diagnosis is possible on clinical
grounds alone. Involvement of the ophthalmic artery or ciliary arteries may occur causing optic
atrophy and blindness. Vision is impaired in about 50% of patients at some stage. If blindness
occurs it is usually irreversible.

Temporal arteritis may follow polymyalgia rheumatica in about 20% of cases. The condition is
very responsive to corticosteroids which should be prescribed in high doses initially.

Maintenance steroid therapy in lower dose over 2-3 years may be required, which raises the
possibility of steroid-induced complications of osteopenia, hypertension, diabetes and changed
facies.
Handbook condition 68
Aches and pains in a 62-year-old man

You are working in a general practice Your next patient is retiree and aged 62 years. He ;s
consulting you about aches and pains, and you have just finished taking a history, which was as
follows:

Over the last six weeks, he has had a gradual onset of pain across the upper part of the back,
neck and shoulders which feel stiff. In the past week or so his hips are also feeling stiff and sore.

Since retirement two years ago he has been playing golf at least three times per week and
thought that he may have been overdoing it. At first the pain was just an aching feeling, but is
now more definite pain but hard to describe. It is continuous, worsened by movement and is
keeping him awake at night. The aching and stiffness is worse early in the morning and he finds it
difficult to get out of bed because of muscle weakness and pain which improves during the day.
Pain is not relieved by aspirin or Brufen®, nor worsened by coughing. There is no radiation to the
arms. Pain is felt in the muscles but not in the joints, although these feel stiff especially after
inactivity. He has not played golf for a week. He has noted a little difficulty in lifting himself up
from a chair. Muscles feel 'as if they are losing their strength'.

He has felt much more tired than usual over the last few weeks, especially after golf. His appetite
is not as good as usual. He thinks he may have lost a little weight and sometimes feels hot and
slightly sweaty at night in bed.

YOUR TASKS ARE TO:


• Specify to the examiner the essential features you would like to know from a focused physical
examination of this patient. The examiner will give you the results and ask you questions about
your provisional diagnosis and further investigations.

• Answer the questions put to you by the examiner.

• Advise the patient of your diagnostic and management plans.

You do not need to take any further history.

The Performance Guidelines for Condition 068 can be found on page 371
Aches and pains in a 62-year-old man AIMS OF STATION

To assess the candidate's knowledge of the clinical presentation of polymyalgia rheumatica and
the way in which this diagnosis is confirmed or excluded.

EXAMINER INSTRUCTIONS

The examiner will have instructed the patient as follows:

You are a retired office worker and will be advised by the candidate of the diagnostic and
management plans.

EXPECTATIONS OF CANDIDATE PERFORMANCE

Physical examination findings to be sought:

• Essential features of focused physical examination to be given to candidate on request by the


examiner.

~ Temperature

~ Pulse

~ Blood pressure

~ Muscle groups of neck, trunk, upper and lower limbs should be examined.

- Active movement of neck, shoulder and trunk muscles causes discomfort.

- Normal power and tone and coordination of movements.

- Examination of joints, particularly hands, shoulders, neck, sacroiliac joints and hips.

These show no abnormalities and a full range of movement.


~ Examination of lymph nodes, abdomen, and respiratory systems is expected to exclude any

medical conditions that could possibly give rise to this constellation of symptoms (e.g.
lymphoma, carcinoma) — normal findings.

~ Rectal examination to check prostate — normal.

After providing results of physicai examination, the examiner will ask the candidate

• 'What is your provisional diagnosis and differential diagnosis?'

• 'What further tests will you advise?'

• 'Please now give to the patient your diagnostic and management plans. ' Diagnosis/Differential
diagnosis

Polymyalgia rheumatica should be suspected from the history. The examination does not reveal
any specific diagnostic features but erythrocyte sedimentation rate (ESR) or C-reactive protein
(CRP) would be expected to confirm the diagnosis. Underlying malignancy should be a
consideration.

37 °C, normal 70/min regular

140/80 mmHg

Management

The candidate is expected to indicate to the patient that if the blood tests confirm the suspected
diagnosis of polymyalgia rheumatica, then the patient is likely to have a good response to a
limited course of prednisolone which may need to be given for up to two years, but that any
such treatment, how it is given and monitored, must await the results of the tests. The patient
should be advised to report any severe headaches, visual disturbance or pain in the jaw when
eating, since giant cell arteritis can occur together with polymyalgia rheumatica.

KEY ISSUES

• Focused physical examination which must include musculoskeletal system plus rectal
examination.

• Investigate with ESR and/or CRP.

• Polymyalgia rheumatica as the most likely diagnosis.

CRITICAL ERROR

• Failure to request ESR and/or C-reactive protein.

COMMENTARY
Polymyalgia rheumatica and giant cell arteritis are linked conditions of unknown aetiology. The
incidence varies with ethnicity and these conditions are more common in people of Northern
European descent.

Polymyalgia rheumatica commonly presents in middle aged or elderly patients with diffuse
symptoms of muscle pain particularly in the neck, shoulders and hip girdles. The myalgia is
symmetric and often begins in the shoulders. Muscle strength is normal but can appear
diminished because of pain. There is often a disparity between the severity of myalgia reported
and the physical findings. There are often constitutional symptoms including weight loss, malaise
and depression; spiking fevers are rare. The diagnosis in this instance would be confirmed by
investigations, specifically ESR and C-reactive protein, and full blood examination (FBE).

• Treatment of polymyalgia rheumatica is with oral prednisolone, initially in high dosage.

• Differential diagnosis to be considered would include:

~ Chronic fatigue syndrome: This condition is a 'medically unexplained condition'. It is usually


seen in younger patients, may follow a viral infection and the dominant feature is incapacitating
fatigue with other medical symptoms of subjective memory impairment, headaches, poor sleep,
generalised muscle pains, postexertional malaise lasting more than 24 hours, lymph node
tenderness. It is best viewed as a symptom complex resulting from interaction of physical and
psychosocial factors. The ESR, CRP and FBE tests are normal.

~ 'Fibromyalgia': Another of the 'medically unexplained' conditions, characterised by aching


pains across the shoulders and upper back, skin tenderness, poor sleep pattern and often
additional constitutional symptoms. ESR, CRP and FBE are normal.

Polymyositis: This is an uncommon inflammatory muscle disorder that may be associated with
an underlying neoplasm in older patients. The most frequently encountered mode of
presentation is the onset of painful muscles and proximal muscle weakness, often commencing
in the neck, shoulder girdle and proximal limb muscles, associated with some atrophy with
disproportionate weakness. ESR. CRP and FBE abnormalities may be indistinguishable from
polymyalgia rheumatica but elevated creatine kinase and abnormal autoantibodies are
characteristic. A positive muscle biopsy is diagnostic.

~ Underlying malignancy: prostate, breast in females, multiple myeloma, lung cancer.


237-Migraine
Recall 11-7-2018
Headache (migraine)-task was history taking and DD

Case (31/3/2017)
A middle aged woman comes with complains of headache for past 6 months. She has tried
ibuprofen but it has not improved. Concerned about brain tumor
Tasks
A) History
B) Dds and dx
C) management
typical migraine case and stress at work

Case (4/10/2017)
40-year-old male patient comes to your GP. Has had headaches since adolescence.
-Relevant history
-Explain most probable diagnosis and other diagnosis to the patient with reasons.

Patient gave a TYPICAL text book history of migraine since adolescence self-medicated with
ibuprofen up to now (it works most of the time), never sought medical help before.
No red flag symptoms present. (Not increased with coughing, sneezing/ not associated with eye
movement, blurred vision, stiff neck, rash, malaise/ no neurological signs/ no sudden severe)
Excluded other DDs but diagnosis of migraine was quite clear.
(When asked if anything about headache changed, to see why he came this time, said it’s getting
worse now.
Any other worries? His cousin was diagnosed with brain cancer recently. (Now started to show
anxiety. Do I have cancer too doctor??? Had to address this showing empathy and explaining his
symptoms are quite different from that of brain tumour.)
General health good. No significant past illness/medications. SAD -nothing significant. He is an
accountant had some worries at workplace. Can faintly remember that he had a family history of
migraine too (brother).
Explain most probable diagnosis and other diagnosis to the patient with reasons
Explained typical pattern of migraine pain. Tried to explain a bit of pathophysiology about vessel
dilatation etc. Explained other diagnosis like cluster headache/ tension headache/ brain tumour
for long standing headache, but said infectious causes (sinusitis/meningitis) also considered but
unlikely. Ran out of time to finish.
3/5 key steps covered.
Handbook 129
Migraine in a 30-year-old woman
CANDIDATE INFORMATION AND TASKS

This patient was seen a week ago. At that time, she had a severe headache, and the following
history was obtained. She is a married schoolteacher aged 30 years with two children. This was
the second episode of headache in the last month. The headache was felt over both
temporofrontal regions and was present on waking up, becoming throbbing as it progressed. It
lasted the whole day and prevented the patient from going to work. It was associated with
vomiting and photophobia and a visual aura. Panadol® (paracetamol) gave only limited relief. A
physical examination at that time was normal. A provisional diagnosis of migraine was made,
however a computed tomography (CT) of the brain was arranged because of the patient's
anxiety about the possibility of a cerebral tumour. This was normal. You are now seeing the
patient for followup in a general practice setting. The patient has indicated to you that no further
headaches have occurred since the last appointment one week ago.

YOUR TASKS ARE TO:

• Tell the patient the result of the brain CT scan.

• Discuss the future management of her migraine.

• The patient will ask you questions about the treatment which you advise. The examiner may
request clarification of this information

There is no need for you to take any additional history from the patient.

AIMS OF STATION

To assess the candidate's ability to provide clear information about the nature of migraine and
its expected course; to obtain the trust and confidence of a patient with recent onset migraine
with potential for recurrence; effectively communicate the necessary treatment measures,
including dual management of pain and nausea; and to counsel regarding principles of
management of migraine, including lifestyle.

EXAMINER INSTRUCTIONS

Candidates are expected to know the principles of treatment but should not be penalised for not
knowing the details of less commonly used medications.

The examiner will have instructed the patient as follows:

You are aged 30 years. You attended a week ago about a second attack of severe headache that
occurred over the last month. You were unable to go to work because of the attacks. The
headache was present when you woke, felt over the front and on both sides of your head, lasted
the whole day and was throbbing. You preferred to avoid light, felt nauseated, and vomited
several times. You had accompanying flashes and specks of light before your eyes. You took two
Panadol® (paracetamol) every four hours, without much effect.

The doctor who examined you suspected migraine, but decided to be cautious and arranged a
brain scan to exclude more serious conditions. This was in line with your wishes. You have now
come to find out the result, and receive further advice about the treatment of your headache.

You were concerned about the cause of your headache and the possibility that you could have a
brain tumour.

You understand you may have developed migraines, like your mother, but are puzzled about why
it has started at this stage in your life, and also what causes it. You remember that treatment was
not very effective for your mother. You lead a very busy life and find your work as a
schoolteacher increasingly demanding. Your two young children are involved in many activities,
which are demanding of your time. You have little opportunity to relax. There are no marital,
family or financial problems.

Patient Profile

• You are a happily married teacher with two children.

• Drug sensitivities — nil

• Known family history — mother used to suffer from migraine.

• Past medical history — no menstrual problems or known relationship of headache to


menstrual cycle. You are not on the contraceptive pill.

• Current medication — nil, except for Panadol® (paracetamol) for headache.

• Habits — nonsmoker, you have 2-3 standard alcoholic drinks occasionally.

• Your partner has had a vasectomy.

Questions to ask unless already covered:

• ‘What should I do when I have an attack?'

• ‘Can I prevent the attacks altogether?'

• ‘Do these drugs have side effects?' (Ask this if drug treatment is suggested)

• ‘Are there any natural remedies or other things I could do?'

• ‘Is the migraine going to occur more frequently?'

• 'Are you sure I don't have a brain tumour?'

• ‘What causes migraines?'

• ‘Can it lead to anything more serious?'

• ‘Why has it come on now?'


• 'Does anything in particular bring on an attack?'

• 'How long will I go on having migraine?'

• 'Are the children likely to get migraines?'

EXPECTATIONS OF CANDIDATE PERFORMANCE

First tell the patient the result of the brain scan (see tasks).

Then discuss the following with the patient:

• The mechanism and epidemiology of migraine, and knowledge of its treatment, trigger

factors and prevention — malignancy would not explain these symptoms and a tumour has

been excluded by the completely normal CT.

• Explanation of mechanism — functional disturbance, possibly episodic serotinergic/

vasomotor changes, affecting intracerebral/extracerebral arteries.

• Discussion of prevalence — affects approx 10% of population, more common in females,

onset in young adults (can be children), reduces in frequency over the age of 50 years.

• Trigger factors — fatigue, stress, menstrual cycle, foodstuffs especially alcohol, drugs
(especially vasodilators). Counsel against using codeine-containing analgesics regularly

(risk of dependence, constipation, rebound headaches).

• Satisfactory management requires followup of the patient by arranging review of progress.

Treatment of acute attack

Candidate MUST know the following principles of management:

• Rest in quiet, darkened room and avoid movement, reading, watching TV: apply cold packs

to head.

• Commence appropriate medication in adequate doses early in an attack.

• Appropriate drug therapy is selected according to severity of attack, previous efficacy, side

effects.

~ Mild — soluble aspirin 600-900 mg 4 hourly: or paracetamol 500 mg 2-3 tabs 4 hourly (4 g

daily): Maxolon® (metoclopramide) 10 mg; Stemetil® (prochlorperazine) 5mg; or Panadeine®


(paracetamol/codeine), or Mersyndol® (paracetamol/codeine/ doxylamine succinate).
~ Moderate — Ergodryl Mono® (ergotamine tartrate) 1 mg, repeat in 1 hour if necessary; or
Cafergot® (ergotamine with caffeine) two tablets initially or 2 mg/100 mg suppository; or
Naproxen (250 mg) 750 mg to maximum of 1250 mg in 24 hours.-

~ Severe — or ergotamine trial failed — Imigran® (sumatriptan) 50 mg tablet repeat in 1 hour, or


injection (sumatriptan contraindicated within 24 hours of ergotamine).

Preventive therapy

The following is a summary of current preventive therapy for migraine. The candidate should not
be penalised for lack of detailed knowledge of preventive measures, but should know of their
availability and potential side effects. Ongoing support is appropriate in all cases.

• Attention to lifestyle and other trigger factors.

• Preventive medication is not required in this patient with only two attacks; but is indicated for
three or more attacks a month, selecting from: beta blockers/pizotifen/ cyproheptadine/tricyclic
antidepressants/methysergide/calcium channel blockers. All these medications can have
significant side effects.

~ beta blockers — contraindicated in asthmatics; adverse effects of tiredness and bradycardia;


~pizotifen — adverse effects of drowsiness and weight gain;

~cyproheptadine — adverse effects of drowsiness and weight gain;

~ tricyclic antidepressants — impaired alertness, anticholinergic, autonomic side effects; ~


methysergide — six months use only; adverse effects include vasospastic

syndromes, retroperitoneal fibrosis.

~ calcium channel blockers — indicated in severe, life-disrupting cases only, because

of adverse effects; use is for a limited time only to break the cycle; can give improvement in
majority of cases.

KEY ISSUES

• Counselling and patient education.

• Appropriate drug therapy according to severity and including preventive measures,

CRITICAL ERRORS

• Failure to adequately address the treatment of acute attacks.

• Failure to consider trigger factors.


Karin case
Migraine Case: You are a GP and a 35-year-old female came in due to headaches.
Task
a. History (on and off x 6 months; attack started yesterday, on the back of head, pulsating)
b. Physical examination
c. Diagnosis
d. Management

History -
-Do you need some painkillers?
-Since when are you having this pain? How bad is it?
-What type of pain is it? Is it pulsating, throbbing or a dull ache?
-Is it one sided of all over your head?
-When the pain starts, where does it start first? How does it progress?
-is it aggravated by movement, noise or light?
-Do you get any symptoms before the headache starts for example visual problems, changes in
your sense of smell, nausea or vomiting?
-Is this the first time? How many episodes have you had previously? How long does it last?
-What relieves your pain?
-Is it worse in the morning?
-did you have fever recently? Any infection recently?
-Do you feel numbness or weakness in any part of your body? Did you hurt yourself in your
head?
-Do you think your headache is related to food especially red wine, cheese, chocolate, bananas,
Chinese food, coffee?
-How's your general health? Any history of HPN or DM?
-What is your occupation? Any stress at work or home? Any financial problems recently?
-How are your periods? When was your LMP? Do you think your headaches are related to your
periods?
-Any family history of migraines? SADMA?

Physical examination -
-General appearance: pallor, jaundice, dehydration
- Vitals
- ENT: signs of runny nose or watery eyes; check for PEARL; signs of meningism;
- auscultate chest and heart; LN; palpate tummy
- CNS: motor weakness, paresthesias or sensory disturbances

Differential diagnosis - Migraine - cluster headache - SAH - Meningitis - subdural hematoma -


temporal arteritis
Management -
You have a common condition called migraine. 10% of world's population suffers from migraine.
It is more common in females. Usual age of onset is at a young age. The frequency of headaches
reduces after the age of 50.
The exact cause is not known but there trigger factors including: fatigue, hunger, strong odors,
excessive noise, hormonal changes, and certain kinds of food.
- The mechanism is dilatation of the blood vessels outside the brain.
There are three types: a. Classic (presents with an aura), b. Common, c. Atypical (abdominal
migraine - commonly seen in kids).
- Treatment focuses on two aspects: 1. Treating the acute attack (rest in a quiet dark room, avoid
reading/tv, cold packs to head, and medications)
o mild migraine: soluble aspirin 600-900mg q4 or PCM 500mg q4 + an antiemetic o moderate:
ergotamine + antiemetic (metoclopramide).
o severe: sumatriptan + antiemetic (metoclopramide or chlorpromazine) o These medications
are also available in the form of combinations (mersyndol - paracetamol + codeine +
doxylaminesuccinate)
- If on OCP, review because it might aggravate migraine.
- For prevention: lifestyle modification, avoid trigger factors, and >3 attacks/mo may give
preventive medications such as beta-blockers, cyproheptadine, TCAs, calcium channel blockers x
6-12 months then taper and review.
238-Atrial Flutter+ Read Handbook 66
Feedback 13-12-2018 Palpitation and Dizziness: Pass
2 mins Outside: Again don’t remember the stem exactly. Middle aged man come to ED because of racing
of heart and dizziness. 3 rd attack.
Tasks: History, Explain ECG to examiner and Dx and DDs
Thinking outside: Was thinking about DDS only. Arrhythmias such as AF, flutter PSVT etc,
Pheochromocytoma, Hyperthyroidism, anxiety state such panic attack.
Inside: asked examiner about hemodynamic stability. Examiner said stable. Pt has no current symptoms.
So I started with history.
Me: How r u feeling at the moment? Do you have dizziness or racing of heart at the moment?
Pt: not at the moment. Feeling ok right now.
Me: I asked with open question such as please tell me more about your complaints.
Pt: He started with dizziness. 3 rd attack. First 2 when driving. He pulled over (I sympathise and asked
about anyone got hurt, he said no, I said thank god). This was third attack. All came without warning, by
itself and went away by itself. All came with palpitation.
Me: was palpitations were regular or irregular? Pt: regular
Me: Was the room spinning around you?
Pt: No room spinning (rule out vertigo).
Me: anything that brought attack?
Pt: No
Me: Any aggravating or reliving factors?
Pt: No
Me: any chest pain, tightness? Pt: no
Me: cough? SOB? Pt: No
Me: Any loss of consciousness? Pt: No
Me: Any aura? Flashing of lights? Headache? Visual blurring?
Pt: no
Me: any involuntary limb movements? Pt : no
Me: any vomiting, early morning headache? Pt: No
Me: any heart conditions such as HTN, known heart disease such as irregular heart rhythm? Pt: No.
Me: Known anxiety, panic attack? Pt: no
Me: Any weather preference? Loose motions? Racing of heart comes and goes? Pt: no
Me: any past heart conditions such as IHD or MI (asked in simple terms), diabetes?
Pt: No.
Me: any F/H. Pt: no (don’t remember about family heart disease).
Me: asked about medications and SADMA but I don’t remember the answers.
ECG: It was atrial flutter. I explained the examiner in systemic manner (according to RAZA’s ECG).
Started with rate, rhythm, P wave, QRS complex and then explained about Atrial flutter with 3:1 block.

DD: Explained to pt. Most probably u have a condition called atrial Flutter. (Drew heart and signal pathway
in simple diagram. Explained him about atria and ventricles). Pumping and signals in atria and ventricles
are not synchronised. It could be conditions we called atrial fibrillation, but its less likely because your HR
are regular or PSVT but generally it occurs in young person plus ECG does not show any changes.
It could be hyperactivity of thyroid gland or adrenal gland (the one sits on kidneys) but less likely because
u don’t have typical symptoms.

Global: 4 Key steps: 1 & 3 No, 2& 4 yes


Approach to pt: 4 History: 3 Interpretation of Investigation: 5 Diagnosis/DD: 4
239-Urinary tract diseases
Case 1 (UTI counselling)
A middle-aged man has come with complaints of burning urine. A complete history and
examination is over, with no significant findings except positive urine dipstick findings for infection
Tasks:
-dx/ ddx
-counsel, Ix
-management

Case 2 (BPH+ UTI)


1-Patient with problems during urination,
Task:
-Hx,
-DDs.
2- Guy with burning in pee.
Task:
-History
-dx and d/d

Case 3 (Hematuria)
Aged 50 -60 man comes in with blood in urine.
Task:
History
Physical examination findings from examiner
Dx and DDx with reasons to patient

Case 4 (urine retention+ BPH)


Lower abdomen pain with ACUTE urinary retention. Enlarged prostate.
Tasks
-Hx
-PEFE
-Mx

I think AMC Cases related to urinary tract are:


1-explanation of results of UTI to patient with Ddx
2-explain results of UTI and prostate cancer to patient with Mx
3-patient with burning urination (UTI) + BPH symptoms (UTI+ BPH)
4-patient with blood in urine + BPH symptoms (hematuria+ BPH).
5-patient with acute abdomen pain+ BPH symptoms (BPH+ urine retention)
History (Burning urination or any urinary problems)
1-Urine questions
-for how long?
-has this happened before? How often? (Recurrent UTI)
-do you have any pain on passing urine? (Dysuria; do not ask if it is chief complaint)
-do you need to go to toilet more frequently? (Frequency)
-any change in the colour of urine? (Colour)
-have you noticed any blood? (Hematuria) if +ve ask:
*since when?
*colour of blood?
*if it is present on all part of urine stream or just beginning or the end?
*if bleeding anywhere else?
-is it smelly? (Odour)
-do you need to get up at night to pass urine? (Nocturia)
-are you passing large or small amount of urine? (Polyuria, oliguria)
-BPH specific questions
*is you urine stream as good as it used to be? (Urine flow; BPH)
*do you think you need to go urgently when you need to void? (Urgency)
*any delay in initiating urine flow? (Delay)
*do you need to strain to push urine out? (Straining)
*any dribbling at the end? (Dribbling)
*do you think you need to pass urine after finishing urination? (Incomplete feeling)
2-Associated symptoms questions
-any pain or discomfort in your tummy?
-any nausea or vomiting? Any fever?
-any LOW, LOA, lumps or bumps?
-any discharge from penis?
-any scrotal pain, swelling or redness?
3-General questions
-medications+ allergy especially antihypertensive, blood thinner
-PMH (DM, kidney stones, bleeding disorder)
-PSH
-trauma
-sexual (sexually active, stable relationship, safe sex)
-family history urine problem (ca prostate)
-SADMA
-occupation
History (hematuria)
1-Hematuria questions
-since when
-if first time?
-colour of blood?
-present in all part of urine stream?
-bleeding anywhere else?
-dizziness, SOB?
2-urine questions
-dysuria, frequency, nocturia, polyuria
-BPH (urine output, delay initiating, dribbling, urgency, incomplete feeling)
3-Associated symptoms
-nausea, vomiting, fever, abdomen pain
-LOW, LOA, lumps and bumps
-discharge
-trauma
4-General
-medications (aspirin, antiHTN, OTC) + SADMA+ occupation
-PMH (DM, stone, bleeding disorder)
-PSH
-sexual
-Family history of cancer

Physical examination from examiner


1-GA (PODL+ BMI)
2-VS
3-Focus abdomen
-inspection (distension, mass)
-palpation (tenderness, mass, renal angle tenderness)
-percussion
-auscultation
-hernia orifices, inguinal LN
-Genitalia (ulcer, discharge, scrotal swelling, redness, tenderness)
-DRE (prostate enlargement, surface, consistency, tenderness, warmth, mobility, fluctuation,
median sulcus).
4-Office tests
-midstream urine dipstick (protein, leukocyte, nitrite, glucose, blood, ketone)
-BSL
Differential diagnosis
1-Hematuria + BPH symptoms
-BPH
-Stone
-UTI
-trauma
-bleeding disorders or blood thinners
-nasty growth
2-BPH symptoms+ dysuria
-BPH
-stone
-STI
-UTI
-nasty growth
3-Lower abdomen pain+ BPH symptoms (urine retention)
-BPH
-constipation
-kidney or bladder stone
-urethral stricture
-certain tumours or cancers
-medications
-nerve disease
-trauma
-fibroid, ovarian cyst, herpes (if female)

Explanation (UTI counselling)


1-Draw diagram (kidneys, Ureters, bladder, prostate, urethra)
2-define UTI= urethritis (infection of urethra or urine tube), cystitis (infection of bladder or urine
bag), prostatitis (infection of prostate) , pyelonephritis (infection of kidney)
3-Cause of UTI=bug or bacterial infection usually E coli.
4-Explain causes of UTI (lower and upper)
-urethra (stone, stricture or narrowing, cancer)
-prostate (stone, BPH or enlargement, cancer)
-bladder (stone, benign or polyps, cancer)
-ureter (stone, stricture)
-kidney (stone cancer)
5-symptoms indicates lower UTI (because of no fever, N&V or loin pain).
6-uncomon in men, while common in women with reasons (short urethra vs long urethra)
7-risk factors
-DM
chronic disease (HIV)
-low immunity, steroids
-PSH urinary tract
-previous UTI
Management (UTI counselling)
1-reassure treatable with medication
2-confirm with urine culture and sensitivity (critical error)
3-start antibiotic after taking sample
trimethoprim 33 mg/ once daily for 14 days
cephalexin 500 mg twice daily for 14 days
4-ample fluid
5-urine alkalanisation
6-repeat culture 7 days after complete antibiotic treatment
7-reading materials.
8-red flag (pain, fever, N&V, blood)
9-Further Ix??
-FBE, ESR/CRP, UCE
-PSA
-US or CT
10-refer to urologist??

Explanation (hematuria case)


1-draw diagram
2-most likely BPH and define it
3-other possibilities
-stone -UTI -trauma -nasty growth -bleeding d -blood thinners
4-Mx (if present)
-Ix (UMCS, PSA, US, Blood tests, imaging)
-refer to specialist

Explanation (BPH cases)


1-from history and examination most likely you have BPH. Have you heard of it?
2-Diagram
3-this is a small gland located at the base of the bladder called prostate. Its function is to form
some part of the semen.
4-sometimes it gets enlarged and blocked the urine tube resulting in these symptoms.
5-other possibilities are:
cystitis: infection of bladder (no suprapubic pain)
pyelonephritis=infection of kidney (no loin pain or fever)
STI (nor urethral discharge or unsafe sex)
Stone (no loin pain hematuria)
DM (no polyuria or history)
prostatitis (no fever, tenderness on ex)
cancer (no LOW, LOA, unlikely from prostate ex).
6-however, we need to do Ixs like …………… to rule out these possibilities
7-antibiotics+ urine MCS
8-refer to urologist
Case 1 (UTI counselling)
Feedback (29/3/2017)
A middle aged man has come with complaints of burning urine. A complete history and
examination is over, with no significant findings except positive urine dipstick findings for infection
Tasks: diag/ dds, counsel, Ix, management
2 mins: recollected all the dds and the Ix , treatment of male uti
Explained the urine dipstick results to the patient and with the help of diagrams explained all the
possible causes and the necessary investigations.
Urethra Stone, Urine MCS, visualisation
stricture, of the urethra
cancer
prostate Bph, Psa, ERUS +/-biopsy
prostatitis,
cancer
bladder Stone, cancer Urine MCS, cystoscopy,
KUB
ureter Stone, IVP, KUB
stricture,
ureteral
backflow,
posterior
urethral valve
kidney Pyelonephritis, Rft, sr electrolytes, blood
stone UR MCS, fbe, esr, crp
sti First pass urine/ swab
from genital but with
patient’s consent

Management- first obtain urine for microscopy and culture and start antibiotics for 14 days.
Refer to the relevant specialists for the necessary investigations as uti in males always needs to be
investigated. Review him again with the results, repeat urine culture a week after finishing the
antibiotics to check whether the organisms have been removed from the body. In the meanwhile if
his symptoms worsen, or any new symptoms appear, ask him to consult immediately.
Global score- 4

Handbook Condition 095


This 40-year-old postman is married with two children and has consulted you today in a general
practice setting complaining of the gradual onset of dysuria and frequency of micturition over
the last three days. There has been no urethral discharge and no history of extramarital sexual
contact. On examination the patient is afebrile and you found no abnormality on examination,
including rectal examination of the prostate.
A midstream urine specimen was collected and the following office laboratory tests were done
on the urine
• Dipstix — positive for protein, leucocytes and nitrites: negative for blood, glucose and ketones.
• Microscopy of uncentrifuged specimen — shows large numbers of leucocytes and bacilli.
The patient usually keeps in excellent health. He is aware that he is sensitive to penicillin but
otherwise his past history, family history, habits, and use of medication have no relevance to this
problem.
YOUR TASKS ARE TO:
• Advise the patient of your diagnosis.
• Advise the patient of your immediate management.
• Discuss the condition and answer any questions the patient may ask.
EXAMINER INSTRUCTIONS
The examiner will have instructed the patient as follows:
You consulted this doctor today because of the gradual onset of dysuria and frequency of
micturition over the last three days. The doctor has examined you (including rectal examination)
and asked you to provide a urine sample which was checked in the practice laboratory. You are
about to receive the doctor's advice about the problem.
Questions to ask unless already covered:
• 'What did you find in my urine?'
• 'Can this infection be treated easily?'
• 'Where do these bacteria come from?'
• ‘Is this the same as my wife gets?'
• 'Tell me exactly what tests I should have done?'
• 'Why do I need these tests?'
• 'Do you think I have something seriously wrong?'
• 'How long will I need to take the medication?'
• 'Could it occur again?'
• 'What will the urologist do?' (Ask only if referral is advised)
You are not overly concerned about your condition because your wife has suffered from
occasional urinary tract infections over the years which have always responded well to treatment
with antibiotics. She has had no other investigations other than urine laboratory tests. You
expect to recover quickly after receiving antibiotics.
If the doctor indicates that further special investigations are necessary followed by referral to a
urologist, be surprised and express some reluctance to undergo these procedures. If the doctor
handles your reaction satisfactorily agree to follow this advice.
EXPECTATIONS OF CANDIDATE PERFORMANCE
Approach to patient
• This may appear to be a straightforward clinical situation but it requires care to avoid alarming
the patient concerning the need for more investigations than just a urine culture
• Give a clear explanation of the nature of the condition.
• Obtain compliance in use of medication (clear instruction about frequency and duration)
• Emphasise importance of followup.
• Explain why further investigation is essential and obtain compliance for this.

Tell the patient what Is wrong A variety of terms may be used to describe a urinary tract
infection but the candidate should explain that it is most likely to be in the lower urinary tract
(bladder, prostate or urethra) rather than in the kidneys, because of the absence of fever and
loin pain.
Immediate management The candidate must advise that the midstream urine specimen
collected today will be sent for culture and antibiotic sensitivity assessment.
Choice of initial treatment — appropriate antibiotic — for example, trimethoprim 300 mg
orally, once daily; or cephalexin 500 mg orally, 12 hourly.
Urinary alkalisation may be used, for example, Ural® (sodium citrotartrate) 8 hourly.
Duration of therapy is 14 days.
Amoxycillin is often given but is inappropriate in this patient because of the penicillin
sensitivity. Advise patient to drink extra fluids.
The antibiotic therapy should be commenced today whilst awaiting the culture results.
Phone with results when through.
Early review if poor response to treatment.
Followup of this episode by repeat microscopy and culture after completion of antibiotic
therapy.

Discussion of condition and advice about investigation Significance of a urinary tract infection in
males. Usually associated with underlying pathology according to age group:
• Children — congenital abnormality especially vesicoureteric reflux.
• Younger adults — foreign body in bladder, sexually transmitted infection, including homosexual
activity.
• Older adults ~ calculus formation in kidney, ureter, or bladder; ~ prostatitis; ~ bladder polyps or
carcinoma; ~ benign prostatic hypertrophy; ~ carcinoma of prostate; ~ urethral stricture; or ~
genitourinary tuberculosis should not be forgotten.
• The clinical picture suggests a lower urinary tract infection. The main conditions to be excluded
are urinary neoplasm and calculus and prostatic pathology.

KEY ISSUES
• Approach to patient.
• Initial management plan.
• Choice of investigations.
• Patient education and counselling.

CRITICAL ERRORS
• Failure to arrange urine culture before commencing antibiotic therapy.
• Failure to advise the need for further investigations.
COMMENTARY
• Further investigation is essential to identify the underlying cause and to exclude malignancy.
• These would be undertaken in a staged manner.
• Urinary culture to define organism. Repeat culture after initial treatment.
• Ultrasound of kidneys, ureters, and bladder.
• Contrast enhanced CT of abdomen and pelvis.
• Prostatic specific antigen (PSA) level.
• Serum urea and electrolytes
• Referral to a urologist who may arrange: ~ cystoscopy, ~ voiding cystourethrogram.

Karin case
Urinary Tract Infection in Men
Case: A 40-year-old man came to see you in your GP clinic due to dysuria and increased
frequency. He has not history of STDs.
Task:
a. History
b. Physical examination
c. Investigation (dipstick: leukocytes and nitrites positive; MSU: large number of leukocytes)
d. Diagnosis and Management

Differential Diagnosis
- UTI
- Cystitis (loin/back pain and fever)
- Pyelonephritis (fever + flank pain + chills)
- Stones (loin to groin pain and hematuria)
- Prostatitis (fever, pain, dysuria, history of unprotected sex)
- Diabetes
- BPH (flow of urine)
- STDs
- Foreign Body
- Reiter syndrome (uveitis, arthritis, urethritis)

Significant:
- WBC >10c cells per ml (1x106/L): microscopy
- >105 CFU/ml or (1x108/L): culture
Management
- You most likely have a UTI. I would like to do urine microscopy and culture to confirm. We need
midstream urine.
- Most common organism is E. coli. Because you don’t have fever and loin pain, it is lower UTI.

- I will start you on antibiotics: trimethoprim 300mg OD x 14 days or cephalexin 500 mg BD x 2


weeks or Amoxiclav 625 mg x 14 days.
- Drink ample fluid (2-3 cups in AM, 1 cup every 30 mins)
- Empty bladder often
- Personal hygiene (wipe front to back)
- Panadol for pain
- Ural (sodium citrotartrate)  CI if prescribing nitrofurantoin.
- Unusual for male to have UTI so we need to investigate further for any other underlying disease
(bladder/prostate malignancy, calculus, prostatitis, urethral stricture, BPH) o For young adult:
higher chances of having STDs or foreign body
o Child: congenital abnormalities (VUR)
- Refer to urologist, Review and reading materials
Investigations
- Intravenous urethrogram
- PSA
- RFTs
- CT abdomen and pelvis
- USG
- Cystoscopy.

Case 2 (UTI+ BPH)


Case (10/2/2017)
1-BPH. Px with problems during urination, Task: Hx, DDs. Positive findings: pain passing urine,
difficulty in micturition, BPH Qs (initiating of urination, urinary dribbling, increased frequency,
slow stream, nocturia and urgency and straining) will be positive on Hx taking, father had also
problems with urination. Pain in lower part of abdomen. No blood DDs: BPH +/- UTI. Other d/ds
Cancer qs negative , STIs qs negative , Thnk GOD I passed

2- Guy with burning in pee.


Task: History, dx and d/d
on history all the findings for UTI was positive and features for BPH also for last one year. So
explained with diagram what is going on,.

Case (7/6/2017)
Middle age man has painful urination. Task - hx, tell dx to pt ( has frequency, difficulty in passing
last few drops, has mild discomfort and pain in lower tummy, no h/o Std/Dm/fever)

Feedback 27-10-2018 URINARY PROBLEM


60+ year old male patient with complaints of burning pain while passing urination.
Task
- history
- Explain Dx/DDx History
- duration (a few weeks)
- first time
- getting worse lately
- When I asked him to tell me more about his problem
- he told me that he has a difficulty passing urine and a weak flow as well.
- asked him
- frequency (+)
- urgency (+)
- nocturia (+)
- dribbling (-)

Other associating symptoms


- fever (-)
- dark color urine (+) Other DDx
- loss of weight (-), loss of appetite (-), back pain (-)
- tummy pain (-), loin pain (-), past history of stone (-)
- sexual history married, discharge from the penis (-), past history of STI (-)
- medication (-)

Dx/DDx
- BPH (most likely) drew and explained
- prostate cancer
- prostatitis
- UTI
- Stone
- STI, told him the rest are less likely

•then I finished too early, and realized that I didn’t go into detail about the urine color change
and started asking him again whether this dark color is seen at the start or middle or end of the
flow. then the bell went off.
Grade - pass
Global score - 4
Key steps
1,2,3,4 - No, Yes, Yes, Yes Approach to patient - 4

History - 3

Dx/DDx - 4

Karin case BPH/LUTS


Case: You are a GP and a 65-year-old male comes in complaining of urinary problems.
Task
a. History (4-5 weeks, urge to go to toilet but unable to pee properly, incomplete emptying of
bladder, no pain or burning while urinating, no blood in urine, fever, bowels are fine, backache
recently; diabetes previously but on diet, FHx negative, smoker for 10 cigarettes/day for several
years, alcohol socially, no medications; sexually active, no discharge, frequency, LUTS)

b. Physical examination (general appearance is normal, BMI normal, vitals normal, no abdominal
masses or tenderness, no pedal edema, , prostate enlarged , firm but smooth in consistency, no
nodule; urine dipstick and BSL normal)

c. Investigation (USD of KUB, PSA)


d. Management
Irritable symptoms
- Urgency
- Dysyuria
- Pain
- Hematuria
Obstructive symptoms
- Hesitancy
- Weak stream
- Dribbling
- Nocturia
- Feeling of inadequate emptying

History
- What exactly is the problem?
- Do you have frequency of urination? How many times during the day and how many times at
night do you have to go?
- Do you think you have to go urgently when you feel the need to void?
- Do you need to wait for some time before passing water?
- Any problems with the stream? Any dribbling afterwards?
- Any pain on passing water?
- Any change in color of urine? Any blood?
- Any burning sensation?
- Do you have fever?- Any loin pain?
- Do you think you have lost weight recently? Any change in your appetite? - Backache?
- Any other symptoms like cough, SOB, chest pain, palpitations, headaches?
- Have you noticed any lumps or bumps around the body?
- Any other previous medical or surgical conditions like diabetes or high blood pressure?
- Any family history of prostate cancer or other cancers? SADMA?

Physical examination
- General appearance
- Vital Signs and BMI
- Chest and heart
- Abdomen: tenderness, organomegaly, percuss bladder to check level after patient has voided
- DRE: size and consistency of prostate? Nodularity? Is median sulcus palpable? Attachment of
rectal mucosa to underlying gland?
- Urine dipstick and BSL
Investigation
- Urine for MCS, FBE, U&E, ESR/CRP, PSA, and USD of Abdomen, Transrectal prostate USD (best
approach)  PSA is normal

Diagnosis and Management


- From history and examination, you have a condition called BPH which is enlargement of the
prostate gland. This gland is a small gland located at the base of the bladder. Its function is to
form some parts of semen. Sometimes, the gland becomes enlarged. The water tube (urinary
tube/urethra) that passes through the prostate gland becomes blocked as a result of this
enlargement giving symptoms like hesitancy with passing water, a weak stream, frequency or
urination, and sometimes dribbling of urine.
- In majority of cases, this enlargement is quite harmless. However, some patients might present
in the same way but they are actually suffering from prostate cancer, so I would like to do a
transrectal ultrasound and a PSA to rule that out. However, in about 15-20% of prostate cancers,
the PSA might be normal in the early stages. Usually a PSA of more than 10 is suggestive of
cancer.
- At the moment, you don’t need to worry about anything because even if it turns out to be
prostate cancer, it is a very slow growing cancer where the rate of mortality is very low. I’m
sending you with these investigations to the specialist surgeon. He will review the results of the
USD and they might decide to do a transrectal punch biopsy of the prostate. It is usually done
within the surgeon’s clinic under local anesthesia. Later on, they might send you for bone scans.

Case 3 (Hematuria)
Feedback (5/10/2017) (important)
Blood in urine
Pass
Aged 50 -60 man comes in with blood in urine.
Task:
History
Physical examination findings from examiner
Dx and DDx with reasons to patient

Introduced myself and asked him how is he feeling?.


Then i told him i would like to check his vitals first cus he has blood in urine. Vitals were stable
from examiner.
Then started asking about the bleeding ( first time, don't remember the duration, blood is red
colour and present in all part of urine, not asso with pain, no stone passing, no abd pain, no
dysuria but frequency +, urgency, weakness of stream, difficult to start urination, no dribbling,
no cancer features, no fever and rash) .
He has HTN on one antihypertensive meds ( can't remember the name) and Aspirin.
No trauma and no bleeding disorder, SADMA history.
PEFE
- Mass + on suprapubic area but no tenderness, DRE showed BPH but no CA prostrate features
( examiners told me all the DRE findings, didnt need to ask him specifically).

Dx - BPH, explained with diagram, DDx- CA prostrate, Urinary Stones, UTI, Aspirin, Trauma,
Bleeding disorder

Covered all 4 key steps ( finally LOL )


Approach 6
History 6
Choice and Technique of exam 6
Dx and DDx 7
Global score 6

Case 4 (urine retention)


this case is start with abdomen pain questions then urine and etc
same PEFE
same BPH explanation without UTI

240-Non-specific abdomen pain


Gp setting, 7 years old girl brought in by mom complaining of abdominal pain for 4 weeks.
Tasks
-relevant focused history
-PEFE
-Dx and Ddx
-Mx accordingly

Chronic abdomen pain= pain more than 1 weeks

Differential diagnosis of chronic abdominal pain


1-constipation
2-UTI
3-lactose intolerance
4-abdominal migraine
5-IBS, IBD, Celiac
6-non specific abdominal pain

Organic Functional
-radiating pain -around umbilicus
-affects sleep -sleep unaffected
-vomiting common -no vomiting
-affects growth and development -normal G&D
-obvious cause -psychological factors

History
1-Abdomen pain questions (here no need for hemodynamic stability as pain is chronic)
-how severe is the pain from 1-10?
-is it constant or does it come and go? Is it getting worse? how long does each episode last?
-where exactly does she feel the pain? Does it go anywhere else?
-how does she describe the pain? Is it like colicky, sharp or dull ache?
-does anything make it worse? Is it related to any food? Does the pain worse after breakfast?
-does anything make it better? Has painkiller been required? Is passing motions makes it better?
-does it affect her sleep? Or daily activities?

2-Associated symptoms (vomiting, diarrhea, fever, rash, urine)


-is it associated with vomiting?
-have you noticed any change from usual bowels habit? Have you seen any blood or mucous in
the stools? Does she have diarrhoea, constipation or alternating? Has she ever had constipation?
Do you find fecal materials on underwear?
-is it associated with fever, rash?
-burning or stinging during uribation?

3-General questions
-PMH? Recent viral infections?
-medications and allergies?
-BINDS
*how is her birth?
*immunisation up to date?
*can you describe her typical daily diet for me? How is her feeding and appetite?
*is she thriving and growing normally?
*any siblings with similar symptoms?
*how is her school performance? Does the pain happen when the child goes to school? Does it
happen in the weekends?
*Psychosocial
I would like to ask if something bothering her mind so I might ask you some sensitive questions
any recent changes or stress at home? (grandmother diagnosed or died with cancer)
how is her relationship with her parents?
who takes care of the child most of the time?

PEFE
1-GA -DR PJL
2-VS
3-growth chart
4-abdomen
-distension
-tenderness, mass
-percussion for any dullness
-auscultation
-inspection of genitalia, anal area for fissure or rash, hernia orifices with consent of parents.
5-office tests -urine dipstick

Explanation
-from history and examination let me assure you that I could not find any physical issues. Most
likely, she have what we call a non-specific abdominal pain, which could be something like
mental or mind stress causing her symptoms. Mainly because of stress at home, as her
grandmother has cancer she is behaving like this.
-mind and body are connected together so whenever the mind is getting stressed the body can
give such symptoms.
-this is experienced by 10% of Australian kids.
-other possibilities are…… but less likely because….

Management
-I can arrange family meeting to discuss the issue together.
-Important to encourage her, try not to criticize her to avoid making the situation worse
-can take Panadol for severe pain.
-??would like to refer her to psychologist to confirm the diagnosis and they will support her.

Feedback 27-10-2018 RECURRENT ABDOMINAL PAIN


12 year old child with pain in his tummy Task
- History
- PEFE
- Dx/DDx
- Counsel the mother

History
- Duration (6 months)
- on and off
- pain around the umbilicus
- not sure about the type of the pain
- no radiation
- no relieving or aggravating factors
- severity (moderate)
- fever (-), vomiting (-), waterworks (normal), bowel habit (normal)
- pain during weekend (+)
- pain at night (-)

DDx
- family history of migraine (-)
- association with dairy products (-)
- constipation (-)
- family member on special diet (-)

HEADSS
- happy family
- family member getting on well with each other
- enjoy going to school
- good school performance and no complaints of abuse BINDSMA – normal
- then, I asked the mother “ is there any major changes in his life before 6 months such as
loss of a loved one or moving to new places?”
- Mother told me that his grandma was diagnosed with colon cancer 6 months back and
had a colostomy done since then.
- they are very close and her son is really upset about it.

PEFE
- everything was normal on examination

Dx
- told her that everything was normal on examination
- the pain is not associated with any food or any abnormalities in the tummy.
- it could be related to the stress he is going through.
- told her I’m sorry about your mom
- unconscious mind divert his stress to bodily symptoms
- that is why he is having pain in his tummy.
- this is not a serious condition. other possibilities

- abdominal migraine
- coeliac disease
- constipation
- lactose intolerance, but these are less likely in his case I would recommend that you
bring your son in next visit
so that i can talk to him and find out what is going on in his mind. Grade - pass

Global score - 5
Key steps 1,2,3 - Yes, Yes, Yes
History - 5
Choice and technique of examination - 6
organization and sequence
Dx/DDx - 6
Patient counselling/ education - 5

Case(10/2/2017)
child with Abdominal pain for almost 6months.
- Socrates is abdominal pain around the umbilicus no radiation , severity not v bad ,
- Alternate loose motion and constipation ,
- not related to food or mild ,
- no blood , no mucous ,
- no travel history.
- no wt loss.
- no vomiting no nausea no acute pain .
- She is over caring child regarding her school performance , very smart and good at school.No
bullying.
- Confidentiality no social issues
- FH : Mother is having some tolerance to dairy product but not the child .
- BINDARS .,
- PEFE all normal ,
- I told her this is most likely nonspecific abdominal pain ( as no red flags )Check caren notes it is
exactly as there . I passed this case
Case (24/6/2017) 7 yr old with non specific pain abdomen
Case (24/6/2017) (22/6/2017)
Recurrent abdominal pain in a young boy . Grandma just had op for colon cancer with
colostomy bag
Case (30/8/2017)
Chronic abdo pain in 8yo boy, investigated-all negative, grandmum has colon ca
Case (16/3/2017)
non organic abdominal pain ..(recurrent)
Task
History , physical examination , diagnosis and management
positive findings , periumblical pain , stressor: grandma colostomy tube Ex all normal .
Karin case Recurrent abdominal pain
Case: Your next patient is 10-year-old Brian who is suffering from abdominal pain for last 2 years.
The growth charts are normal. Palpation of abdomen: mild tenderness.
Task
a. Take history from anxious mother
b. Talk about further management plan

Case 2: Ronnie aged 6 years is brought by her mother Julie to your GP clinic. She tells you that
Ronnie had abdominal pain for the last few months and she is quite concerned as her usual GP
think that it’s not serious and seeks your opinion. On further questioning, Julie describes the
pain as intermittent and mainly around the umbilicus sometimes severe enough that Ronnie had
to miss school. Ronnie is otherwise well and had no significant medical or surgical problems.
Ronnie lives with his parents at home and had started school this year.
Task
a. Further history b. Physical examination
c. Differential diagnosis and management

Features
- 3 distinct episodes of abdominal pain over >3 months occur in 10% of school-aged children
- 5-10% organic cause can be found
o Pain is other than periumbilical
o Pain radiates rather than remains localized
o Pain wakes child from sleep
o Accompanied by N/V
o Not well between attacks
o Associated weight loss
o FTT

- Possible causes:
o Constipation
o Abdominal migraine
o Lactose intolerance
o Non-ulcer dyspepsia

- Investigations: urine MCS, FBE and ESR, Xray

- Non-organic
o Acute and frequent colicky abdominal pain
o Pain localized or just above umbilicus
o No radiation of pain
o Pain lasts <60 minutes
o Nausea frequent; vomiting rare
o Diurnal
o Minimal umbilical tenderness
o Anxious child
o Obsessive or perfectionist personality
- Management
o Give explanation, reassurance and support
o Avoid investigations
o Acknowledge that the child has pain
o Emphasize that disorder is COMMON and usually traverses childhood without ill effects
o Simple measures: local warmth, brief rest.
o Review: if episodes change in nature, pain persists for hours, or there are new symptoms
o Identify any life stresses and enquiry about family structures and school performance
o Discourage identification with sick role
o Refer for psychological assessment and counseling if necessary

Differential Diagnosis
- Migraine
- Travel History: Giardiasis
- Lactose Intolerance
History
- Pain:
Where: paraumbilical?
Since when?
How many episodes (3 episodes in 3 months)?
How long do they last (<60mins)?
In between the episodes the child is fine.
Severity: Acute/severe.
Character: (Colicky pain).
Radiation: It doesn’t radiation/
Anything increasing the pain (No)?
Anything decreasing the pain (pain killers)?
- Is the child constipated?
- Emotional factors: Family situation, personality of the child.
- Did he use any medications?
- Does he wake up at night (No)?
- Does the child have any fever? Infection? Intussusception? Volvulus? GORD? Gastroenteritis?
Any nausea and vomiting? Any bloating? Any heartburn? Regurgitation(if younger age)? Any
flatulence/distention (lactose intolerance)? Did you notice any mass in the tummy?
- Bowel motion: Is he toilet trained? Any previous history of constipation? Did you notice any
blood? Offensive? Hard to flush?
- Urine: Any urinary infection? Any dysuria? Any blood in the urine? Smelly urine?
- BINDS?
- Any medical condition? Surgery?
- Is the child pale/ pallor?
- Is the child complaining of headaches? Family history of migraine?
- Growth Chart: normal. Nutrition: Does the child drinking milk products? Is it associated with
milk products?
- Family history? Happy family? Any financial issue? Any relationship problems? How do you
cope?
- School performance? Any bullying? Does this pain happen when the child is going to school?
Does it happen in the weekend also?
- Can you describe his personality? Perfectionist? Anxious child?
- Intestinal worm: pain when sleeping? Any recent travel anywhere?

Physical Examination
- General Appearance: well/ playful, no pale, no dehydration, no jaundiced, no LN enlargement?
- Growth Chart: Normal. No FTT
- Abdominal: Any mass? Any scar marks? Palpation: is the abdomen soft? Yes, but mild
tenderness. No organomegaly? Fecal impaction? Hernia orifices? Bowel sound? Normal.
- Inspection: Any excoriation, skin tags, fissure, mucous discharge, soiling

Investigations: - FBE, ESR, Urine analysis, urine mic/cul, Plain X-ray if required (+ travel history or
if suspecting constipation).

Diagnosis and Management


- Most likely its non-organic recurrent abdominal pain, it’s a common disorder it doesn’t have
any ill effect on the child’s health.
- Reassurance: 8% of school aged children can have this problem. If there are any factors:
constipation/emotional factors need to be addressed.
- During the painful period:
1-take rest, Rest during painful periods
2-local warm (heat pack)..
3-Make a diary.
4-Involve the child in discussion.
5-Refer to the pediatric psychologist for the insight therapy.
6-Arrange family meeting to address family issue.
7-Contact teachers to find out causes in the school.

- Red Flags: If there’s any change in nature of the pain. Any new symptoms, N/V diarrhea, child
turn pale, FTT, if the pain wakes up the child at night, or poor school performance.

Case (30/5/2017)
10 yo female with 6 mo of intermittent abdominal pain
Hx, PE, Ddx,management
abdominal migraine in 10 yrs old hx dx and ddx (mother have migraine)
hx of abdominal pain for the last 6 months, on and off mainly central no radiation get better with
hot packs. Sometimes she is having diarrhea sometimes not, no on on special diet and she had
no allergy or any atopy, rule out bullying, home situation and any wt loss or lump bumps, mom
hx of migraine and having some diarrhea. When she eat dairy things i asked if she is having any
confirmed diary allergy she said no.
Fhx of migraine so i just give this as the first thing then other things as well and in pe i only asked
about GA VS and abdominal examination seems all normal so i didnt ask more like features of
malabsorption i dont know icoundnt think about it at that time myb is critical but in hx i rule out
many things
241-Tummy pain in children+ IBS cases
Case 1 (Celiac disease)
You are in GP, 16-year-old boy is presenting with tummy pain.
Task
- Take relevant history
- Tell the patient most likely Dx
- Discuss management

History
1-hemodynamic stability

2-Pain questions
-severity + pain killers and allergy
-duration, constant or come and go, getting worse (for 6 weeks or 2 months)
-site, radiation (around belly button, no radiation)
-character
-relieving and aggravating? If related to food? If pain relieving by passing stool? (Movement
worse)
-1st episode (yes)

3-Assoiciated symptoms questions


-Nausea and vomiting?
-urine problems?
-bowel motions? Diarrhea, constipation or alternating? (Diarrhea +ve)
*Duration?
*How often do you pass stool per day? (4 times a day)
*Are you passing large amount of stool?
*Is the stool hard, loose or watery? Greasy or bubbly? (sloppy)
*Any blood, pus or mucus?
*What’s the color? Is it pale?
*Is it smelly?
*Is it hard to flush?
*Do you have to race to the toilet to pass stool? (IBS)
*Does it wake you up at night?
-fever? (Giardiasis)
-rash, joint pain? (IBD)
-LOW, LOA, lumps and bumps

4-General
-medications, antibiotics
-PMH, PSH
-typical daily diet + family history of similar problem or special diet
-travel, contact (giardiasis)
-stress in life or perfectionist (IBS)
Explain diagnosis.
-most likely, you have coeliac disease have you heard about it?
-this is malabsorption disease when the bowels become sensitive to a specific substance present
in diet called gluten. So whenever the bowels exposed to gluten, it becomes inflamed causing
tummy pain. also the bowels cannot absorb the food causing frequent bowel motions.

Other possibilities are:


1-IBD
2-Giardiasis
3-IBS
4-UTI
5-medication

Management
-stool MCS
-FBC, LFT, UCE
-Celiac screen (antigliadin, antiendomysial, tissue transglutaminase)
-refer to GIT specialist for duodenal biopsy (say take a sample of tissue from the first part of the
gut)

Case (22/2/2017)
You are in GP, 16 year old boy is presenting with tummy pain. (The stem is too short)
Task
- Take relevant history
- Tell the patient most likely Dx
- Discuss management
2mins thinking
DDx depends on site of pain
Not to forgot HD stability, pain killer, I ll play accordingly
History taking
When I stepped into the room, a young guy in his twenties was sitting who speaks too fast with
Aussie accent. Sometimes I couldn’t catch him.
HD stability - Yes. Pain killer - Role player refused. Pain scale - 3-4
Pain Q - 1st time, for 6 weeks, not very severe, site - around the belly button (he showed me), no
radiation, aggravating factor - movement.
No fever, no N,V, water work – fine
When I asked about bowel work, he said he needs to pass motion for about 4 times a day and
stool are sloppy. No blood, no mucus, difficult to flush - no, foul smelling - no
(So far, I heard only floppy stool and asked him again if it is floppy. He told me, no, it is sloppy)
(I actually dun know what sloppy is. No problem it might be similar to floppy. LOL :p)

DDx - No injury to tummy. No LOW, LOA, lumps and bumps around body. Tummy pain is not
relieved by passing stool, no alternating constipation and diarrhea, no chest pain, no racing of
heart, no SOB
No travel HO, no contact HO
Are u on any special diet? (no) anyone in family on special diet (No)
Similar problem in family? (My uncle has some problem with his bowels but I dun know exactly)
General health, SADMA - all clear

Explain to roleplayer
John, according to HO and PE findings, there are a couple of conditions that can cause ur tummy
pain
(I am not sure about exact Dx though task is most likely diagnosis)
It could be coeliac ds, which is ur bowels are sensitive to a substance called gluten in diet. When
u eat diet containing gluten, it causes inflammation of ur bowels and cause tummy pain. It
cannot be digested and cause bulky stool and that’s why u need to pass motion for 4 times a day.
Other causes can be giardiasis which is infection of gut, it could be inflammation of ur bowel
called IBD, nasty growth in ur bowel.
Now, I ll run some blood test : check ur basic blood count, chemicals in ur blood, renal and liver
function, most importantly coeliac screening which is testing some proteins called Ab in ur
blood.
I ll do stool examination to find out if there is any infection.
Imaging of ur tummy called USG and Xray will be done if needed.
According to invx results, I ll treat u accordingly.
If it is coeliac ds, u need to avoid diet containing gluten which is barley, rye, oat and wheat. They
are available easily in every supermarket.
If it is infection, I ll give u antibiotics
If it is IBD, I ll get involved specialists
(Some say it is psychogenic pain, so pls elaborate more in psy HO which I didn’t ask
I didn’t talk about bowel biopsy to confirm coeliac)

AMC Feedback - Abdominal pain : Pass (Global Score - 4)


Approach to patient - 5
History - 4
Dx/ DDx - 4
Management - 3
Case 2 (Inflammatory bowel disease)
You are in GP, 16-year-old boy is presenting with tummy pain.
Task
- Take relevant history
- Tell the patient most likely Dx and ddx
- Discuss management plan

History
1-hemodynamic stability

2-Pain questions
-severity + pain killers and allergy
-duration, constant or come and go, getting worse (for 3 months, intermittent)
-site, radiation (central, no radiation)
-character
-relieving and aggravating? If related to food? If pain relieving by passing stool? (Relieved by
Panadol)
-1st episode (yes)

3-Assoiciated symptoms questions


-Nausea and vomiting?
-urine problems?
-bowel motions? Diarrhea, constipation or alternating? (Diarrhea +ve) DF ACC CSF US
*Duration?
*How often do you pass stool per day? (3-4 times a day)
*Are you passing large amount of stool?
*Is the stool hard, loose or watery? Greasy or bubbly?
*Any blood, pus or mucus?
*What’s the color? Is it pale?
*Is it smelly?
*Is it hard to flush?
*Do you have to race to the toilet to pass stool? (IBS)
*Does it wake you up at night?
-fever? (Giardiasis)
-rash, joint pain? (IBD) where is the rash and if painful or not? (Skin pale, rash and mouth ulcers)
-LOW, LOA, lumps and bumps (decreased appetite and weight loss with clothes loosening
noticed)

4-General
-medications, antibiotics
-PMH, PSH
-typical daily diet + family history of similar problem or special diet (+ve family history of bowels
problem)
-travel, contact (giardiasis)
-stress in life or perfectionist (IBS)
Explain Dx and Ddx
-from history there are several possibilities.
-most likely IBD or crohns which is inflammation of the bowels wall causing tummy pain on and
off. Cause is unknown but can be immune related. It can be complicated by bleeding,
perforation, obstruction or pus collections.

-other possibilities are Celiac, giardiasis, IBS , UTI, medications (explain why less likely)

Management
-stool MCS
-FBC, LFT, UCE
-refer to specialist for colonoscopy to have a look at the bowels from inside and if any lesion they
might take a tissue sample for be examined under microscope.
-treatment is usually by anti-inflammatory medications like steroid and other medication to
induce remission.
-MDT with regular follow-ups.

Feedback 19-7-2018 STATION 11 PASS(Key steps 234 yes, score 6, 6,5,6,6)


Abdominal pain
Just a single line scenario of a 16 year old boy comes to you with abdominal pain.
Task Hx ,Dx ,Dds ,Mx Plan
There was young guys sitting alone , I started with pain questions after offering painkiller, he
have central abdominal pain for 1-2 months with diarrhea, wt. loss ,skin lesions and joint pains
.Positive FHx( cousin have some problems),typical Hx of IBD. I ruled out travel ,IBS and all other
cause of chronic diarrhea including Ca.
Gave most likely IBD and could be IBS, celiac , giardiasis ,Ca.
In management I explained blood test, stool test and colonoscopy and biopsy, refral to
specialist ,on confirmation of diagnosis prolonged treatment with anti-inflammatory ,steroids
indications and SE, and regular FU in IBD clinics.

Case 29/11/2017
16 year old came with abdominal pain.
Task:
-Take history.
-explain diagnosis and what investigators would you do?
1. (perumbilical pain for last 3 months. Passing loose stool 4-5times per day. Pain/diarrhoea
wakes him up at night too. Loose floppy stool. No blood. Weight loss +ve. Friends say he has
become more pale. Mouth ulcers intermittently. No rash. No family hx of special diet.)
2. Same as above + Positive history of arthritis and rash
Recall 12-4-2018 ???
A 17- year old boy comes with abdominal pain. )IBD)
task:
* further hx not more than 5 min
* DDX and mx
no any P / E allowed.
during hx taking , started 2 m ago was 5-6/10, all abdomen no radiation, no R. with anything, no
special diet, has tiredness as well , has vomiting several times, no trauma, has wt loss, going
toilet 4-5 times and has increased, no burning sensation, no change in colour of ymurine or
stool, no bumps& lumps, no recent or recurrent infection, no rash or joint pain, not sexually
active, no smoke alcohol or illicit drug, no FH of any dis no PMH of any dis.
mood was ok intetest ok. no nedication, no allergy. no any investigation available.

Karin case
Case: You are a GP and your next patient is a 50-year-old chef who comes in due to recurrent
diarrhea for the last 2 years.
Task
a. History (5-6, bloody with mucus, + weight loss, went to Thailand but did not eat street food; +
FHx of GI problems - uncle)
b. Physical examination (+ pallor, soft, vague pain on all quadrants, no organomegaly, + mucus
and blood)
c. Investigation (if necessary)
d. Management

Recurrent Diarrhea
- IBD - IBS - Coeliac disease - Infectious diarrhea - Colorectal cancer

History
- I understand that you have come to see me because of recurrent diarrhea. Can you tell me
more about it? (diarrhea)
- How many motions per day do you have?
- Is it watery? Do you have blood or mucus in the stool?
- Is it foul-smelling?
- Is it greasy or hard to flush?
- Are you bleeding from anywhere else?
- Do you have any tummy pain? Does the pain get better after passing stool? Are you stressed
about something? (abdominal pain)
- Do you think you have fever? Do you feel tired? (fever/ weight/ appetite)
- How is your appetite? Have you lost weight?
- Have you traveled anywhere recently? Did you eat streetfoods?
- Do you have any rash, Joint pains, or trouble with your eyes? How is your waterwork?
(joints/urine)
- How is your health in general? Any past history of any medical or surgical condition?
- FHx of GIT disease, or colon cancers
- (risk factor for Crohn)SADMA
Diagnosis and management
- From history and examination, I suspect you have a condition called IBD, but it could be other
things like infections, malabsoprtion syndromes or cancer.
- I would like to do some investigations for you including FBE, iron studies, ESR/CRP, U&E, LFTs,
and stool microscopy and culture. I would also refer you to a gastroenterologist for colonoscopy.
It is a procedure done under anesthesia. A flexible optic tube is gently passed from the
backpassage and guided around the large bowel. During colonoscopy, biopsy will be performed
from suspicious areas. This means that the specialist will take a sample of tissue and send it for
examination under the microscope. After the procedure there might be some bleeding and
discomfort but it is usually safe.
- The type of treatment we offer will depend on the investigation results. IBD is a term used to
describe two diseases, namely Crohn disease and Ulcerative colitis which cause inflammation of
the bowel. Ulcerative colitis cause inflammation of the inner lining of the large bowel whereas
Crohn’s disease cause inflammation of the full thickness of the bowel and may involve any part
of the digestive tract up to the backpassage. The causes are unknown. Most likely, it is immune
related. The treatment will be decided by the gastroenterologist. I would advise you to adapt a
health lifestyle and diet.

- If smoker  arrange for smoking counseling


- Drugs such as steroids are used for acute flare-ups. Most people in remission phase are advised
to take a drug called sulfasalazine (or MTX or cyclosporin) to reduce the chance of relapse and
this is called maintenance therapy.
- The surgical treatment is reserved for complications.

- Will it affect my lifespan? People with IBD have useful and productive life even though they will
need to take continuous treatment.

- When the disease is inactive, they feel quite well and are usually free of symptoms, but I would
recommend for you to have regular followup.

- What are the complications? Anemia, small bowel obstruction, toxic megacolon/dilatation
(UC), abscess formation, perforation, bleeding, fistula formation and strictures

- Does it have a risk of getting cancer? You would be managed by experienced


gastroenterologists who are experts in this condition. IBD have increased chances of developing
cancer especially in long-standing cases of UC, but don’t worry we will do regular followup.

- Reading material. Review. Referral


Case 3 (???Irritable bowel syndrome)
10 years old brought by father concerned about tummy pain for 6 months
Tasks
-History
-Dx and ddx

Recall 10-4-2018
Father at the GP – Wants to know what could be the reason for Lower Abd pain just below the
Umbilicus.
Take a Hx.
Discuss the Diff Diagnosis with a motivation derived from Hx
Child had abdominal pains esp dring School ( weekdays) seems to be comfortable in the
weekends Nutrition on was normal – well balanced Irritable Bowel Dx Constipation, UTI
Abdomina Colic
Red Flags – Bowel Obstruction.

Case (10/11/2017) comment pass


10 years old tummy pain for 6 months , I told px father as this is already 6 months ,
is there any Ix has been done ? he said no ..
ok it will make pattern already , what did you notice from her ? she has tummy pain better when
se toilet sometime has constipation sometime has diarrhoea.
any hard to flush ? geasy ? blood ? watery ? offensive smell ? any rash ? He is not aware off
Did she show where exactly the pain any move everywhere, he not aware off.
No related with food, no fever, no weight loss.
normal activity at school , normal at the house no changing behaviour , no change in appetite ,
deny bully, still happy to go school no problem with sleep .
No family hx similar problem
BINDS all were normal
Ok , looking from the history that you give its most likely your daughter have IBS where her gut
bit sensitive compare other
we don’t know what exactly the cause that’s we need to make food diary and what symptoms
that related with the food that she taking.
could be sensitive to gluten we called it celiac disease then he started to remember , oh yeah
actually she done screening for that , colonoscopy as well and other Ix ( hmmmmmmmmmmm
now I confuse more I did asked at beginning any Ix has been done ) ok I keep going maybe have
lactose intolerance as well but it less likely
Feedback passed.
Approach /hx/technique /DD all were 5 ))))
IBS important questions in history
1-pain related to food
2-pain reliever by passing stool
3-history of alternating constipation and diarrhea
4-straing, urgency, feeling incomplete defecation
5-stress in life, perfectionist

IBS summary
-overactive movements of bowels leading to pain, bloating, diarrhea and even constipation
-body mind axis
-certain factors like lack of fibres, spicy food, dairy products
-common condition, not serious

Karin case Irritable Bowel Syndrome


Case: You are a GP and a 52-year-old female comes to you because of long-standing history of
constipation. During the last 3 months, she had undergone blood tests, gastroscopy and
colonoscopy which were found to be normal. At the moment, she complains of bloating and pain
in the left iliac fossa.
Task
a. Relevant history (pain, bloating, on-and-off constipation, better when bowels open, not
related to food, sometimes diarrhea and goes away on its own, anxious)
b. No further examination
c. Explain diagnosis and management

Case: You are a GP and a young female comes to you complaining of repeated episodes of
abdominal pain and diarrhea. All investigations are normal. The specialist has diagnosed her to
have IBS.
Task
a. Relevant psychosocial history
b. Elicit possible causes
c. Management

Features:
- Functional GIT disorder
- Chronic pain, dyspepsia, constipation, diarrhea
- Cause is unknown
- Genetic tendency
- Environmental factors: food, stress, type A personality, infections
- Rome Criteria: Recurrent abdominal pain or discomfort along with 2 of the following:
o Pain improving following stools
o Onset of pain associated with change in frequency of stools.
o Onset of pain associated with change in form of stools.

- Alarm symptoms: Blood in tissue or in stools, anemia, weight loss, prolonged fever, FHx of
bowel cancers, age >50 years, any change in symptoms
History
- How are you doing? Have you noticed any improvement since you last saw the doctor? Have
you taken any medications to help you? Do the medications help?
- May I ask, how is your mood these days? Do you feel low or guilty about anything?
- Any stress in your life, at home, or at work? Any financial stress? Whom do you live with at
home? How is your relationship with your partner? With kids? Do you have friends or family to
support you? Do you socialize much? Any exercise that you do? How many days a week? For
how long? What kind of work do you do? Any problems at work? How is your relationship with
your colleagues or workmates? Can you please describe your typical daily diet to me? Do you
prepare your own food or do you buy them? How much tea and coffee do you take in a day?
Have you noticed any relationship of your symptoms with any type of food (dairy products, spicy
food)? SADMA?

Counseling
- From the history and investigation findings, it seems like your gastrointestinal system is working
fine, except at certain times where you are under a lot of stress. As you know, the bowel is a
muscular tube that pushes food along the way with the help of waves or peristalsis. Sometimes,
these movements become overactive causing pain, bloating, diarrhea, and sometimes, even
constipation.
As you know, our mind and body are connected. Whenever the mind is stressed, the body
responds producing symptoms, something we call as brain-gut axis. Medically, we term it as
irritable bowel syndrome. Please understand that this is not cancer. Your gut is fine. What is
important is to identify the possible stressors in your life which could be related to work, home,
or financial situations. There are some other factors that might aggravate this condition like
dietary habits (lack of fiber, spicy foods and dairy products, smoking, medications that contain
codeine, overuse of laxatives, and depression).
- It is a very common condition especially with the lifestyle that you have.

-The management involves self-help mainly. You need to avoid the trigger factors, make some
changes to your diet, avoid smoking and intake of caffeine excessively, limit alcohol to safe levels,
have a daily exercise regime, and if possible, avoid stress in your life.
- I can refer you to a counselor for some talk therapy. It might be helpful to keep a food diary to
identify possible triggers. Try to eat a high fiber diet (Soluble
– oats and vegetables (pectins), barley, seed, husks, flaxseed, psyllium, dried beans lentils, peas,
soymilk and soy products; Avoid insoluble fibers like corn and bran).
- If after 12 months of trying lifestyle, dietary and counseling therapy, you are not relieved, we
will start you on anti-depressants.
- Written material. Referral to counselor. Dietitian.
- Give antispasmodics.
- Advice for Constipation:
Case 4 (Irritable bowel syndrome adult)
a 47- year old lady complaining of abdominal discomfort. She had constipation grom 4 years ago and all
investigations were nl ( iron, FBE, TFT, LFT, coeliac, endoscopy, colonoscopy....).
Fade up of this symptom.
tasks:
* further Hx
* DDX and MX
during Hx:
she has bloating, a little pain in LLQ, no any relashionship with any thing ,no wt loss, no loss of app, no
vomiting, no special diet, no FH of any cancer, no hospitalization, no surgery,...
has 2 kids. no any problem with 5P.

comment passed
I passed that case, it was IBS.In psychosocial history patient referred a stressful job, he had been recently
promoted and had more responsibilities. I explained all the other possible DDx with reasons, but as
investigations where normal and as per symptoms most likely IBS. Management: lifestyle modifications,
FODMAPs diet, relaxation techniques, CBT.

Patient was having chronic constipation, bloating, and abdominal pain relieved by opening bowel and
additionally he had notice some charges in the shape and consistency of stool.

I asked psychosocial hx in detail, like overall mood, type of personality, family life and job. He had a
perfectionist personality and he was a kind of workaholic. It was hard for him to balance job and family
life. He did not have time to relax or to do exercise.
Since all the investigations were normal I was almost certain that it was a case of IBS, I just asked a couple
of general qs about symptoms, but I was more focused on confirming my most likely diagnosis that is why I
asked specific questions regarding IBS. It couldn't be anything else otherwise any of the investigations
would be abnormal, but everything was fine.
I just said FODMAPs were some kind of sugars that aren’t absorbed properly in the gut, which can trigger
symptoms in people with IBS. FODMAPs are found naturally in many foods and food additives. It's a
temporary diet that needs to be done under supervision of a dietician, so I refer to dietician as well, and
only for a period of time while symptoms get better. That's all what I said regarding FODMAPs, no time to
explain anything else.

Feedback 11-12-2018
Patient has had recurrent abdominal pain that is increasing with time. You referred this patient to a
gastrospecialist who thoroughly investigated her for all possible GI dx including Celiac disease etc… - all
normal. He has diagnosed as case of mild irritable bowel syndrome.
Take a psychosocial history in relation to her GI symptoms.
Explain possible causes. Explain your diagnosis. And also give one other diagnosis.
*in this case ..i can only say that ..you have to play with communication and keep a constant check on
emotion. Patient is not co-operative , you have to be patient.
Hx- I ask IBS QUESTION, RULE OUT IBD, COELIC DISEASE, CANCER, pain anywhere else
Always ask about risk factors here-Mood and FOOD(DIET), EXERCISE, SMOKING, STRESS , HOW IT
AFFECTING PERSON LIFE?, HEADSSS.
AS I REMEMBER , Gastroenterologist started treatment , so I asked compliance with treatment. She is not
following specialist as she don’t like to eat type of food recommended by doc, so, you have to explain why
it is important. Explain ibs. Refer to psychologist for cbt. Empathy is key here ..so don’t miss that. I gave
medically unexplained symptoms as other diagnosis (so got 3 in that)
GLOBAL SCORE- 4, key steps covered
Feedback11-12-2018(important)
Station 12 (Abdominal pain) Global score 4 PASS (my last station)

A female patient comes to you complaining of abdominal pain. She had been seen by a gastro specialist
that made all possible investigations, including Celiac Disease tests, and found everything was normal. He
said it was a mild IBS.

Task: take a psychosocial history, explain causes and diagnosis.

I came in the room and greeted the examiner and the patient. The examiner was very nice.
I introduced myself. Patient was obviously very angry and agitated.
I assured confidentiality.
I started with an open question: “ Could you tell me what is happening to you?”
She answered: “ Doctor, I have this pain in my tummy and diarrhea. I have been to many doctors and they
all say I have no disease and that it is all in my head!”
Than I asked about her mood, suicidal ideation, thoughts of harming someone else and hallucinations. All
normal.
I asked if anything happened that started her with those symptoms. She denied.
SADMA unremarkable
I asked about her sleep and she said she had problem sleeping.
I asked about any stress in her job and she said: “ I hate my job. Everyone is awful there. I am trying to
change jobs but, you know doctor, no one wants to hire a person with this problems. I am fed up with this
job interviews.”
I asked about her home situation and she said: “ I think my husband is cheating on me.” I asked what
made her believe that and if she had any proof that and she said: “ No, doctor. But who wants someone
always sick like me.”
I asked if she usually anticipate problems and she said yes.
I started to explain her: “ Look, the fact that your problem is in your head doesn’t mean that the problem
is not real.” I noticed that I caught her attention and she sais: “ Doctor, I am tired of listening to this mind-
body axis explanation.” (I froze for a second because that was what I was about to say.) Than I said: “
Well, you know, people reacts in different ways when they are stressed. Some people have headache,
some people sweat too much, some people feel the heart racing… In your case you have tummy pain and
diarrhea and because you have diarrhea you get stressed. You are going round in this cycle and we have to
break it. “ (She smiled)
Than I told her that she also might have anxiety disorder.
Bell rang.
Both examiner and patient were smiling.

Key step 1:no. Approach to patient/relative: 5


Key step 2:yes. History: 4
Key step 3:yes. Patient Counseling/education. 4
Key step 4:no. Diagnosis/Differential diagnoses: 3
Feedback 5-12-2018 (IMP)
AMC Feedback: Abdominal Pain: IBS (Passed feedback: Muhit)
You are an intern at GP clinic and seeing a young lady complaining of abdominal pain for last 2 years for
which your fellow/specialist has diagnosed her as IBS. All inv like USG, Endoscopy, colonoscopy, CT
abdomen done but turn out to be normal, now she is here and quite anxious about her condition and
asking why she is having tummy pain then?
Task
1. Psych HX
2. Give another Dx with reason
3. Reason behind your dx
I entered the room, examine introduced me with a lady who was quite unhappy and before I could even
introduce myself she jumped into the Q why doc I’m having this pain? Then I said Im really sorry that you
are having this pain, hopefully I will get some details to get the bottom of this. Then I introduce myself and
asked intensity of pain( Said-5/6) and do you need any pain killer? (no), S(central pain)ORTSARA(all
negative) Ques 1st then I asked full psychosocial Hx coz my aim was to identify the trigger or stress.
So asked all Mood, Sleep( affected, it takes time to fall asleep, no flashback), Anhedonia, Concentration,
Appetite, Perception Q, Suicide Q. – all negative
Are you an anxious person (little bit)? Do u worry a lot about trivial things? (no)
Home situation – fight with husband
Work - a lot of stress, boss is complaining, not happy with her performance though she gives her 100%
Any other stress- no
Social- not that much though she has got friends
Family support- Less
SADMA( Smoking – sometimes, Drugs(-), Alcohol recently Increased, no medication but Panadol, no
known allergy.
[Throughout the Hx taking she was quite stressed and not cooperative]

So after Hx I gave DD: GAD, Somatoform disorder, IBS as your prev. doc diagnosed, Acute stress disorder,
depression unlikely, So all these are something that I can relate to your stress in your mind , drew a picture
but she was telling me how many time she saw that, I said look Merry at this moment your problem is
beyond your physical problem as we already rule it out by doing all investigation, we call it Body Mind axis(
She pretended not very much happy) . I said Merry your pain is real but sometime we cant find exact
cause and that can be linked to stress that you have in your workplace and home subconsciously affecting
your wellbeing. I am here to help you and your problem is hard to solve in one consultation, so I will
review you frequently, I want to refer you to councilor service as you are quite stressed, you will be taken
care of by MDT, Meanwhile continue to take Panadol, Hot pack, engage urself with social work/ Yoga/
Cycling/ swimming/ running whatever you like, limit ur Alcohol to safe limit. Bell Rang, Thanked her.

Key step- 1,2,3,4- No, yes ,yes, yes

Global- 5

242-Cholangitis (Post cholecystectomy syndrome)


Case 1
You are an HMO in the ED and a 65-year-old female comes to you complaining of pain in the
RUQ for the last few weeks.
Tasks
History
Physical examination from examiner
Diagnosis and Differential Diagnosis

Case 2
50yof c epigastric pain. RUQ/Chills/jaundice.
Task:
-History
-PEFE
-Dx, Ddx

History
1-is my patient hemodynamically stable (VS and O2 Sat)

2-Pain questions
-severity (painkillers + allergies) (5-6/10)
-onset? Sudden or gradual? Constant or come and go? Getting worse? (constant)
-exact site and radiation? (RUQ)
-character? (Colicky)
-does anything make it better like sitting or leaning forward? Medications?
-does anything make it worse like breathing, coughing, movement or fatty meal?
-if it is the first time? (he might reveal the history of cholecystitis if you asked this question)

3-associated symptoms
-nausea and vomiting? How often and what color and content?
-Bowels and bladder (especially change in colour) (pale stool and dark urine)
-fever, rash and recent infections (yes fever) so ask how high is it? Is it associated with chills,
night sweating? Constant or come and go?
-LOW, LOA, lumps and bumps
-yellowish Discolouration of skin (+ve) any itching
-chest pain, SOB

4-General questions
-Past medical history (gall stones, stomach ulcer, hyperlipidemia, DM, HTN, mumps)
-past surgical history (ERCP, cholecystectomy) (past history of cholecystectomy)
-medications (NSAIDS, steroids)
-Smoking, alcohol, drugs
-travel. Sexual history, blood transfusion
Physical examination findings from the examiner
1-General appearance
anxious, sweaty, in pain
pallor, jaundice, dehydration, LAP.
2-Vital signs
3-Focus abdomen
inspection: movement with respiration, distension, visible masses, scars, dilated veins
palpation: tenderness (RUQ+VE), rebound tenderness, guarding, rigidity, murphy sign (-ve).
auscultation: bowels sounds
-complete with: hernia orifice, genitalia, LN, per rectal
4-CVS
apex beat, thrill, heave, heart sounds and murmur
5-Chest
-dullness, air entry, breathing sounds, crepitation or wheeze,
6-Offiec tests
-urine dipstick (urobilinogen+++) -BSL -ECG

Explain diagnosis and differential diagnosis


-Draw diagram
-there are several possibilities
-most likely it is due to blockage of biliary trees here which could be due to:
*ascending cholangitis or inflammation of the common bile duct caused by a stone obstructing
the flow of bile and this lead to ascending infection from the gut causing inflammation. Which is
what I am suspecting the most likely one as you have the triad of this condition (fever and chills,
RUQ pain, jaundice).
*other possibilities are:
-pancreatitis: inflammation of the pancreas which is a gland that secretes both digestive
enzymes and important hormones that could be caused by gall stones or chronic heavy alcohol
intake, rarely trauma, surgery or medications.
-Stricture: narrowing of the duct after lap cholecystectomy.
- Choledocholithiasis (stone got stuck here)
-pneumonia, hepatitis or MI but less likely

Management
-admission
-Seen by surgeon, NPO
-Ix (FBC, UCE, LFT, lipase, hepatitis serology, ESR/CRP), US, ERCP, ECG, x-ray.
-IV fluid, analgesia+/- antibiotic
Feedback 30-5-2018
‘Jaundice’:
~50yof c epigastric pain. RUQ/Chills/jaundice. PHx Lap Chole 6/12 ago (upon asking).
Task: Hx, PEFE, Dx, Ddx
Feedback: Pass

 HOPC

o Vitals

o PainQs JaundiceQs FeverQs [same colicky pain as Phx cholecystitis>lap chole


6/12]

 EtOH + Gall stones + Diet/wtLoss/Apetite/early satiety Qs

 Shoulder tip pain

o DdxQs:

 MI

 Radiation/heaviness/Phx HTN or MI

 Liver

 Jaundice/meds/sick contacts/transfusion IVDU/travel/sexual hx

 GB

 Intermittent/worse w fatty food/acute cholepersistent


pain/mild fever/n/v/charcot’s triad

 Panc

 Boring to the back/better with leaning forward/

 Spleen

 Any trauma

 DU

 Burning

 LL Pnma

 Cough + fever

 Kidney

 F/U/B/D + hi Fever + chills/rigors n/v

o PMhx SHx FHx NKDA

 DM? HTN? Lipid level? FHx: GB stones/Dm/HTN


 O/E:

o GA; Vitals; sats; BMI; ROS

 Depth of jaundice

 Abdo: IPPA; ?murphy’s/courvoisier’s;

 Wards: uDIP; BG; ECG

 Ax & P:

o Drew abdo hexagon and DDx organs for the pain and explained broadly

o Said it’s most likely 2/2 a blockage at the biliary tract w correlates

o Could be 2/2 several things w brief description of pathophys

 Stricture (lap chole)

 Choledocholithiasis (stone got stuck here)

 Ascending cholangitis (most likely as triad +ve)

 Pancreatitis (inflammation 2/2 blockage or EtOH/GS)

o Very briefly touched on what needs to happen from here and summarized

Feedback 5-4-2018
Post-cholecystectomy Syndrome
You are an HMO in the ED and a 65-year-old female comes to you complaining of pain in the
RUQ for the last few weeks.
a. History (5-6/10 in intensity, continuous, no relation to eating or food, fever 39C, relieved by
panadol, decreased appetite, no SOB, pap smear last year, cholecystectomy done 2-3 years ago
also had hx of colon cancer laparotomy done (done remember time) �
b. Physical examination (ill, sweaty, 39C, tachycardia, jaundice, mildy dehydrated, tenderness on
RUQ and liver is palpable 2cm) �
c. Diagnosis and Differential Diagnosis
Feedback 11-12-2018 (Jaundice) Global score 5 PASS
67-year-old woman comes to GP complaining of abdominal pain and fever for two days.
Task: take history, PEFE, DDx, Mx.
I introduced myself and started with an open question: ‘Can you tell me more about your tummy pain?”
She said it started two days ago on RUQ.
(I was rushing with the questions because of time mx!)
Intensity? Moderate
First time? No
Anything makes it better or worse? No
Does it travel any where? No
Any relation with food? No (I was getting frustrated because I had cholecystitis in my mind)
Any nausea or vomiting? No
Fever? Yes Did you measure it? 38
Any change in skin color? “ I think I’m yellow”
Any change in urine color? Yes, it’s dark.
What about stools? It’s pale.
Any weight loss? No
Any recent travel? No Any blood transfusion? No
Alcohol? No Sorry, for asking but are you sexually active? No.
Have you ever used any kind of street drugs? No
Do you smoke? No
Are you using or have recently used any medication? No
Have you done any recent surgery? YES, doctor, I had my gallbladder removed 6 months ago because of
stones. (I was relieved!)
I excused and turned to the examiner for PEFE: VS (only fever) jaundice, tenderness on URQ, Murphy sign
negative, bowel sounds normal. Office test: urobilinogen +++
I thanked the examiner and went back to patient.
“ Look, according to your history and PE, you have stones forming inside your liver(I draw it!). As you
had your gallbladder removed, we call it Postcholecystectomy syndrome. It is a common condition. The
bile produced by your liver is not draining to your intestine. That’s why you are yellow, urine is dark and
stools are pale.
But I am concerned because you have pain, fever and jaundice. This might indicate inflammation in the
tubes inside your liver. That’s called cholangitis.
I will have to admit you. I will contact the hospital and talk to the gastroenterologist. I will take blood
samples for investigation like FBC, UEC, liver function9 AST, ALT, bilirubin, phosphatase, gama-GT)) ,
amylase, RFT, blood type, serology for hepatitis A,B and C, cardiac enzymes. ECG
Image investigation might be necessary like abdominal X-ray, US and CT scan.
A procedure called ERCP might be considered. A flexible camera is introduced through your mouth till
your stomach under anesthesia. The specialist can than introduce tools to remove the stones and drain
the bile. In some cases an open surgery is needed.
You will be started on IV fluids, antibiotics and painkillers. (I forgot antipyretics)
Don’t you worry I assure you will be on very safe hands.” “ Other causes for your condition could be
hepatitis, head of the pancreas tumor, cholangiocarcinoma and strictures.”
Key step 1:yes. Choice of investigation: 5
Key step 2:yes. History: 5
Key step 3:yes. Choice & Technique of exam, organization and sequence: 5

Diagnosis/Differential diagnoses: 5
Karin case Post-cholecystectomy Syndrome
Case: You are an HMO in the ED and a 65-year-old female comes to you complaining of pain in
the RUQ and fever for the last few weeks. The patient had cholecystecomy 3 years ago.
Task
a. History (5-6/10 in intensity, continuous, no relation to eating or food, fever 39C, relieved by
panadol, decreased appetite, no SOB, pap smear last year)
b. Physical examination (ill, sweaty, 39C, tachycardia, jaundice, mildy dehydrated, tenderness on
RUQ and liver is palpable 2cm)
c. Diagnosis and management.
Differential Diagnosis
- Hepatitis
- Stricture
- Cholangiocarcinoma
- Pancreatic cancer
- Cystic Duct Syndrome (except for jaundice)
- Residual stone causing Cholangitis
- Post-cholecystectomy syndrome
History
- Please tell me more about the pain. How bad is the pain in a scale of 1-10? What type of pain?
Where exactly with one finger? Does it get worse or better with anything like food, rest,
movement? Does it go anywhere? N/V/bloating/acidity/reflux?
- I can see from the notes that you have had fever. Since when? How high is the fever? Is it
associated with a lot of chills or rigors?
- Have you noticed any yellowish discoloration of your skin? Since when? Is it getting worse? Do
you have itchiness anywhere on your body?
- I understand you had your gallbladder removed 3 years ago; can you tell me more about that?
What symptoms did you have at the time? What tests were done? What was the result? What
diagnosis was made? Was it a keyhole surgery? Did you develop any complications afterwards?
Infection of the wound? Leaking from the area? Persistent swelling? Pain? Fever after surgery?
Persistent pain, fever or swelling after surgery? Did you get rid of your symptoms after surgery?
How have you been doing since then? Did you visit the doctor since then? Any tests afterwards?
- Do you feel like your tummy is distended or bloated? Any change in your bowel habits or
waterworks? Do you think you have lost weight recently? Any night sweats? Lumps?
- FHx of gallstones or cancers of the liver and biliary tree?
- At the moment, do you have any cough, SOB, chest pain?
- Recent travel? Any chance you came into contact with a person with similar symptoms?
- Did you receive any blood transfusion or have you had other surgical procedures?
- Have you ever had any tests done for hepatitis?
- SADMA?
- Did you get tattoos? Needle sharing?
- Are you on any medications or allergies?
- When was your LMP? Do you have kids? How many? When was the last delivery? Any
complications?
- Sexually active? STIs?
Physical Examination
- General appearance and BMI
- Vital signs
- Lymph nodes
- Chest and heart: breath sounds over lower lobes
- Abdomen: distention? Surgical scar? Tenderness? Enlarged organs especially liver? Is it tender?
Presence of pseudocyst? Epigastric tenderness? Bowel sounds? Hernial orifices?
- Urine dipstick and BSL
Post-cholecystectomy syndrome
- Continuation of biliary symptoms after removal of the gallbladder
- These symptoms can be new or they might continue from before the surgery
- Majority of symptoms are explained by the fact that the removal of gallbladder causes a deficiency
for the storage of bile that is produced.
- There is a higher flow into the GIT causing esophagitis, gastritis, irritant diarrhea, colicky abdominal
pain.
- Temporary diagnosis before a complete workup can be done
- Most common causes:
o Residual stone in the cystic duct or CBD
o Remnant of gallbladder
o Stricture of bile duct due to trauma during surgery
o Biliary dyskinesia (motility of biliary duct is irregular, sluggish and thicker leading to formation of
stone in the duct).
o Cystic Duct Syndrome (a pain reflex is developed arising from the stump of the cystic duct
o Cholangiocarcinomas
o Foreign body (surgical clip)
o Adhesions
o Extrabiliary causes: PUD, hiatal hernia, pancreatitis, gastric carcinomas
Management
- Most likely your condition is called post-cholecystectomy syndrome where symptoms of gallbladder
disease continue even after surgery. This phenomenon is seen in around 10-15% of patients who
have had their GB removed due to stones. It could be related to a number of different causes like a
stone developing within the draining duct or strictures within the duct. Sometimes, stomach
problems like gastritis and peptic ulcer can cause similar symptoms, but my most important cause of
concern is to rule out any chance of a nasty growth within the area, that we call as
cholangiocarcinoma.
- For this reason, I need to admit you and you will need to undergo a number of tests: FBE, ESR/CRP,
PT, LFTs, Hepatitis profile, cardiac enzymes, amylase/lipase tumor markers, ECG, abdominal xrays and
an USD (dilatation of CBD >12mm  diagnostic of distal obstruction).
- Later on, we might need to do some imaging to find out what exactly is causing these symptoms
and this includes ERCP with the use of contrast material, CT scan or HIDA scan. If nothing comes up in
all the tests, we might need to take up exploratory surgery.
- Meanwhile, we will start you on some symptomatic treatment which includes pain killers,
antipyretics, IV fluids and antibiotics until you are seen by the specialist gastroenterologist.
Depending on the results, each complication will be managed accordingly.sn
243-Trauma during pregnancy
(Materials)

38 year old Jane is brought to the ED of the hospital, where you are an HMO, with history of motor vehicle
accident. She's 32 weeks pregnant, and is complaining of abdominal pain.

TASKS
1. Focused history
2. PE from examiner
3. Diagnosis and Management

Positive points in the history: pain in upper part of tummy (rule out abruptio and preterm labor), it is a
dull pain, pain remains the same, bruise along the line of the seatbelt, baby is kicking well
Positive points in the PE: bruise along the line of the seatbelt in the abdomen, undal height: 32cm, FHR:
within normal limits, longitudinal lie with cephalic presentation

APPROACH
 History
o How are you feeling at the moment?
o Ask for consent. If it is okay with you, could you please tell me how that accident
happened?
o Were you driving the vehicle?
o Were you driving within speed limits?
o Were you wearing a seatbelt?
o When did this happen?
o Were you able to walk out the vehicle alone? (will tell you that she doesn't have any
major injuries)
o Did you hit your head or tummy anywhere?
o At any time, did you lose consciousness?
o Are you having any headache now? Any blurred vision? Any nausea or vomiting?
o Any neck pain? Any limitations in moving your neck?
o Any shortness of breath? Any chest pain? Any pain or limitation of movements of your
extremities?
o I have read from the case notes that you are having some pain. Where is the site of
pain? How severe is your pain? It it a continuous pain or an on and off pain? Is the pain
coming at regular intervals? What is the type of pain that you are having? Is the pain going
somewhere else or not? Is the pain worsening?
o Any vaginal bleed or watery discharge?
o Have you got any bruises over your tummy?
o Is the baby kicking well or not? (most important question)
o Did you have your regular antenatal checks? What is your blood group? Down syndrome
screening? Have you taken your folic acid? Ultrasound at 18 weeks? Sweet drink test at 26
weeks?
o Other medical or surgical conditions? Any medications that you are taking? Any known
allergies?
o Do you smoke, drink alcohol or take recreational drugs?
o Do you have a stable partner? Do you have a good support?
 Physical Exam
o General appearance: pallor, dehydration, edema, bruises
o Vital signs
o Head: hematoma, swelling, depressed fracture
o ENT: bleed, discharge
o Neck: midline tenderness, limitation of movements of the neck,
o Chest: accessory muscle breathing, position of the trachea, air entry, abnormal sounds;
S1 S2 normally heard, added sounds, murmurs
o Extremities: range of motion
o CNS: cranial nerves, neurological exam of the upper limb and lower limb
o Abdomen: fundal height: 32cm, FHR: within normal limits, longitudinal lie with cephalic
presentation
 Uterine tenderness, uterine contractions: none
o Pelvic examination:
 inspection of the vulva and vagina: bleed or discharge
 Speculum: bleed or discharge from the cervix, OS open or closed?
o Office test: UDT, BSL

 Diagnosis and Management


From the history and examination, there is no major issue that I can find, except for the pain that
you are having over your tummy. This pain may be due to the bruise over your abdomen. The baby
is doing well, as you can feel the baby kicking and I can also appreciate the fetal heart rate very well.
However, ss this is an accident during pregnancy, you need to be admitted, seen by the specialist, to
do an ultrasound to look for complications inside the uterus, and a CTG every 4 hours for the first 24
hours to check on the condition of the baby. Your vitals also needs to be monitored, and you need
to be observed in the ED for 24 hours as complications like premature labor, and abruptio placentae
can sometimes happen, within the first 24 hours. Once you are discharged, if your pain becomes
more severe or regular, if you break your waters, or if you have any bleeding from down below, or if
the baby is not kicking well, report immediately to the ED.
Do you have someone to be with you now?
244-Serotonin syndrome
32 year old patient on treatment for depression for 6 months with duloxetine with no
improvement, was seen by another doctor in the practise 2 days ago and was prescribed another
antidepressant mirtazapine. He has to come to see you because he is presenting with
restlessness, abdominal cramps, fever 40 and sweating.
Tasks
-History
-diagnosis with patient
-explain initial mx

History
1-hemodynamic stability
2-C/C presenting symptoms (abd cramps, restlessness, sweating, fever)
-how long have you been having these symptoms?
-did it all start suddenly or gradually?
-is it continuous?

3-when were you diagnosed with depression? How long have you been on medication?

4-Medication hx
-were you on any medications before that? (if only one medication given in stem)
-do you know why it was changed? (No improvement given in stem)
-do you know the current dose?
-how many days did you wait before starting the ne medication? Or are you taking both
medication together? (For washout)
-are you taking your medication regularly?
-are you taking any other medications like herbal or OTC?

5-Systematic symptoms+ psychosocial history (like previous lectures)

PEFE (not a task but focus on)


1-General appearance
2-Vital signs (+/- high temp, +/-high BP. +/- high PR)
3-eye (pupil size, shape)
4-CNS (gait, clonus, tremor, reflex, tone)
5-CVS
Explain Dx
-most likely you have a condition called serotonin syndrome when you have excess of serotonin
chemical in the brain leading to these symptoms (fever, tremor, sweating, etc….)
-usually here because of the use of 2 medication and both of them are antidepressant with same
mechanism of action (SSRI) and inadequate washout period leading to increase serotonin.
-Usually when we stop and start medication, we cannot start the new medication straight away
as the first med needs to be cleared from your system before starting the new med to avoid
excess level of serotonin that can lead to these symptoms.

Management
-Admit you to hospital here (if in ED)
-firstly we need to stop your antidepressant med at least for 24 hours.
-be put on monitor to observe your vitals, giving IV NS, antipyretics for fever (cooling blanket
+cold IV fluid).
-Call the psychiatric registrar to see you and depending on your situation, will either cut down
one of the med or reduce the dose.

*if in GP talk accordingly (like feedback 27/10/ 2018 refer to hospital for admission and etc…)

Q/ if you stopped med will my symptom come back again?


-MDT will monitor you and will reintroduce medication once symptom settled down.
Feedback 22-6-2018
32 yo pt on treatment for depression for 6 months with duloxetine with no improvement was seen by
another dr in the practice 2 days ago and was prescribed another antidepressant mirtazapine. Has come to
see you bc he is presenting with restlessness, abdominal cramps, fever 40c,sweating.
Task:
hx (6min),
dg with pt and
explain initial management

Approach
Greeted pt.
I said I am sorry that you are feeling would like some pain killer she said no
I said I’d look into ur vitals first and asked examiner said I will shift pt to treatment room start on IV saline,
Antipyretic and put her on cardiac monitors I think examiner said stick to the task ( can’t seem to
remember).
Then I took confidentiality
Started asking about HOPI- when, 1st time, continuous, associated symptom (racing in heart, dizziness,
nausea, vomiting, diarrhea, muscle stiffness, fever etc.)
Then asked about the medications is she following prescription. Any other medications she is taking. Any
recreational drugs that she is taking
Then Asked psychosocial hx just mood, suicidal risk, perception and orientation.
Details abo Home, education/employment, Activity (hobby), Support
SADMA
Past and family hx

Explained you are having a condition called Serotonin syndrome. Because of the 2 medications a chemical
called serotonin is high in your body and giving these symptoms.
I said first of all we need to stop your anti-depressant medication atleast for 24 hrs.
Will admit you to hospital
There they will put you on monitors to observe your vitals, give you IV Normal saline to keep you hydrates,
Antipyretics for fever and Cyproheptadine as an antidote to serotonin.

I will talk to the psychiatry registrar who will come and see you and depending upon your situation will
either cut down one of the antidepressants or reduce the doses. He will explain to you about your
situation in more detail.

I asked do you any questions for me? Did you understand everything I said. She said yes
I thanked the roleplayer and examiner and left.
The roleplayer was a very good actress she was sweating, holding her tummy and drinking water from
time to time.
Feedback 27-10-2018
30 - 40 year old patient came to GP with fever, sweating, tremor and diarrhea. She is also tachycardic and
has high BP. She’s been taking antidepressant medication (serotonergic) and another serotonergic
antidepressant was added recently by her specialist.
Task
- Take further relevant history
- Explain her condition
- Management plan

History
- Once I walked into the room, patient was siting in the chair looking restless and agitated.
- I told the examiner that as my patient’s vitals are unstable, I’d like to shift my patient to the
treatment area and start her on some medications.
- then examiner told me that you are in the GP clinic.
- I said “OK” and got back to my patient.
- told the patient that I understand that you are not feeling well.
- I will take a very quick history from you to find out the cause
- She said “alright”
- Duration (1-2 days)
- first time
- asked other symptoms of serotonin syndrome (+)
- symptom analysis of each of the symptoms Causes
- addition of new serotnergic meds (+) as given in the stem.
- taking other meds (-)
- alcohol and recreational drugs (-) For other causes of diarrhea
- travelling history and eating street food (-)
- contact history (-)
Forgot to ask about her current mood (hence only 3 in history)

Dx
- explained serotonin syndrome to her

Management
- referred her to the hospital
- where she’ll be seen by HMO and registrar
- told her that she will need to be admitted
- medications will be stopped.
- role player asked me what if her mood and depression symptoms come back again?
- told her that medications will be stopped temporarily until the serotonin level in the blood drops.
- then psych team will come and assess you to start you on medications again.
- they will also adjust the dosage to avoid the similar complications.
- then role player agreed.
- told her that I will arrange ambulance for her.

- I will give you some medications to bring down your temperature and observe your vitals while
awaiting for the ambulance.
Grade Global score 4 pass
Key steps 1,2,3,4 Yes, Yes, Yes, Yes
Approach 4 History 3 Counseling 4 Management plan 4

Handbook 88
Nausea, headache and feeling 'jittery' in a 30-year-old bank clerk

CANDIDATE INFORMATION AND TASKS

Your next patient is a 30-year-old bank clerk, who is consulting you in the Emergency
Department of a general hospital. She is complaining of severe nausea, headaches and the
'jitters'.

She attended here two days ago. Brief notes in the Emergency Department patient record state
that she was then complaining of back pain which was diagnosed as 'muscle pain'. The notes
also state that she had been taking the selective-serotonin-release-inhibitor (SSRI) Prozac®
(fluoxetine) 20 mg daily for depression for three weeks on the advice of her local doctor, without
much improvement. An alternative SSRI —Zoloft® (sertraline) 100 mg daily was prescribed when
she attended the Emergency Department.

YOUR TASKS ARE TO:

• Take a further focused history related to this situation.

• Ask the examiner for the appropriate examination findings you require to assist in diagnosis.

• Inform the examiner of your diagnosis.

• Counsel the patient about the likely cause of her symptoms, their treatment, and what you
recommend with regard to further management of her depression.

AIMS OF STATION

To assess the candidate's ability to recognise that the history strongly suggests a mild serotonin
syndrome. The candidate should ask for findings consistent with the serotonin syndrome,
namely: sweating, tremor, elevated blood pressure, increased pulse rate, increased muscle tone
and hyper-reflexia. If the candidate has not started to ask for these findings six minutes into the
examination, the examiner should advise to proceed with the tasks required, namely advising
the patient of the diagnosis and management of both the immediate problem and her
depressive illness.

EXAMINER INSTRUCTIONS

The examiner will have instructed the patient as follows

You are a 30-year-old bank clerk. You suffer from recurrent depression. Your medication has
recently been changed (see below). You have come back to the Emergency Department of a
nearby general hospital because of severe nausea, headache and the jitters.
Task 1
Your opening statement to the doctor should be: ‘I feel awful doctor — I'm nauseated, I've got a
headache and a feeling of the jitters. It all started yesterday'.

(could you tell me more) Without prompting — go on and tell the doctor: 'Three weeks ago my
local doctor started me on Prozac® (fluoxetine) 20 mg daily for a relapse of the depression I get.
It didn't seem to be helping much'.

Two days ago you felt some back pain and attended the hospital Emergency Department. You
saw a doctor and mental health nurse and were told it was due to muscle strain'. They were
more concerned with your depression and prescribed Zoloft® (sertraline) 100 mg daily. You told
them about being on Prozac® which you then stopped taking because you knew that both drugs
were antidepressants.

In answer to further questions which may be asked:


(depression questions)

• You feel anxious and 'aroused'. You are still sleeping poorly and waking about 4-5am. You
think your appetite was improving before the nausea started.

• You have never had suicidal ideas but you remain pessimistic about the future and find it hard
to concentrate at work.

• Your back pain has resolved. (back pain currently)


• You have had no other symptoms and your last period was two weeks ago.

You are wondering if the medication is the reason you feel so unwell. (Task 4) When the
candidate explains that this is the case, you are relieved. You are angry that the hospital doctors
did not warn you of this, but on the other hand you think there may well have been some
confusion over what was said to whom as they all seemed very busy and distracted and you felt
a bit sorry for them and wanted not to bother them too much. You are prepared to follow the
doctor's advice about further treatment, but do not want to have another reaction like this. You
could say something like 'Could this happen again?'

The candidate is not expected to conduct a physical examination, but will ask for appropriate
examination findings from the examiner.
Near the end of the exam, if the candidate has not told you to stop the treatment with Zoloft®,
you should ask 'Should I continue with the current dose of Zoloft®?'

Task 2
Examiner will provide details of physical examination on request as follows:

Pulse rate 90/min, regular.


Temperature 36.8 °C.
Blood pressure130/80 mmHg
She has a tremor, her palms feel sweaty, and tone and reflexes in limbs are brisk and mildly

hyperactive.

EXPECTATIONS OF CANDIDATE PERFORMANCE


(Task 4)
1-Serotonin syndrome should be diagnosed.

2-The appropriate advice regarding management is to stop Zoloft® (sertraline) and send patient
home and wait until the symptoms resolve, in about 24 hours.
3-As Prozac® (fluoxetine) has a long half life the candidate should recommend waiting at least
another week before reintroducing sertraline, at a lower dose, for example, 25-30 mg. A
reasonable alternative is to reintroduce fluoxetine, which has not yet had an adequate
therapeutic trial in this patient.
4-The candidate should continue to treat the depression and should arrange followup with the
Emergency Department or the patient's general practitioner the next day,
5-advice the patient to contact the after-hours service immediately if symptoms worsen.
6-Support will need to be provided for the patient during the 'washout' period as she is still
depressed.

KEY ISSUES

• Ability to diagnose the serotonin syndrome due to side effects of a Selective Serotonin
Reuptake Inhibitor (SSRI) drug.

CRITICAL ERROR

• Failure to recognise the need to stop the Zoloft® (sertraline) medication.

COMMENTARY

This scenario is a timely reminder about aspects of psychopharmacology. Side effects are
common with most psychotropics because they may be prescribed too enthusiastically and in
dosages that are inappropriately high, especially in management of 'depression' which is a
complex multifactorial complaint in our modern society. Not all patients with 'depression' or
depressive symptoms need antidepressants, but like antibiotics they are often prescribed reflexly
by doctors under time pressure as a ’quick fix – it can’t do any harm’ panacea for a patient in
distress or in tears. Often it is the doctor's helplessness that is being treated by the prescription
because there is never enough time to establish why this patient is depressed on this occasion.
Another common error is to start with too high a dose if the patient is 'really, really distressed
{more must be better and will work faster'). Antidepressants and antipsychotics take 3 4 weeks
to work. If the patient's symptoms improve within that time there may be other factors which
explain the improvement, such as reduction in anxiety or insomnia or the benefits of a
sensitive interview with the discussion of issues and problems, or relief that the problem has
been identified and that something is being done.
Often it is not symptoms per se that cause patients or relatives to seek treatment. Patients
present to doctors when they are worried or anxious about symptoms or behaviours, or
someone else is, who influences the patient to attend the consultation. Anxiety intensifies ALL
symptoms including 'depression' and is accompanied by typically exaggerated and catastrophic
cognitions about the conseguences and outcome of whatever is causing their distress. ‘Is it
fatal/terminal? Will I go mad/drop dead etc?'

An effective initial consultation with a patient who is 'depressed which attempts a biological-
psychological- sociocultural approach and allows sufficient time for the patient to be listened to,
to be understood and to be taken seriously, will in itself relieve a major part of the intensity of
the symptoms. This will only enhance the effectiveness of whatever is subseguently
recommended or prescribed.

Many people with 'depression' have mood fluctuations on a cyclical basis which are
subthreshold or relatively mild. These people are more likely to present at their peaks or
troughs when they are symptomatic in response to a life event or ongoing environmental
stress. Their symptoms may be naturally or temporally transient. If these people (as patients)
are then prescribed psychotropics, including antidepressants, when symptomatic (instead of
being managed expectantly), and they improve after a few days, they and their clinician may
mistakenly attribute their response to the medication. This may commit them to a future
psychological dependence on medication rather than learning to tolerate temporary oscillations
in mood and biological symptoms by using nonchemical coping strategies.

Some doctors and patients have become brainwashed by pharmaceutical companies into
believing that any degree of distress or suffering reguires a chemical solution that is quick and
effective (but freguently expensive and unnecessary). When a patient has been started on an
antidepressant and is appropriately reviewed a week later and reports no improvement, the
inexperienced or unaware clinician may recommend doubling the dose and seeing the patient
a week later. At two weeks, when there is still no major improvement or cure, the dose will be
increased or doubled again. By the third week when the patient reports some improvement at
last, this is wrongly attributed to the increase in dosage and not the latent response to the
initial dose.

SSRIs are potent drugs even in low dosage. Once the dosage increases then side effects and
toxicity will increase significantly. Most patients take such medications erratically or in fits and
starts (i.e. if they are having ' a good day' they will skip a dose: if it's a bad day', then they will
double the dose). Some patients are extremely somatically focused and will develop toxicity just
by reading the package inserts about product information.

This patient feels aggrieved that she has been mismanaged and ill-served by the doctors who
have unknowingly contributed to her serotonin syndrome. Patients deserve better and clinicians
must ensure that they are aware of both the risks and benefits of the drugs they prescribe. As
patients become better informed, they will not tolerate scenarios like this one lightly. Neither
will their legal advisers.
(Task 3)
The serotonin syndrome is caused by excess serotonin in the central nervous system, commonly
because of drug-drug interaction, in this case inadequate washout between a long half life agent
(fluoxetine) and a high starting dose of a second SSRI (sertraline).
The syndrome usually presents with changes in mental state (confusion, irritability, labile
mood), restlessness, myoclonus, hyper-reflexia. fever, sweating, shivering and tremor and
diarrhoea. Hypertension, convulsions, and death have been reported. Treatment is to cease
the medication and provide symptomatic care (e.g. cooling blankets).
Referral to an emergency specialist may be necessary in more severe cases.

245-Delirium hyponatremia
Feedback 16-8-2018 (FAIL)
Station 10. Hyponatremia induced confusion (Failed)
GP, Old age man living in nursing home. Nurse home staff asked his daughter/son to come to see
you because the father has confusion, irritability and drowsiness. He is taking
Hydrochlorothiazide (dose & freq given), Metoprolol (Dose & freq given), Atovastatin (dose &
freq given) and Aspirin (dose & freq given). Nursing home staff checked blood test and given
below. (Other non specific information also given- Long stem. I didn't finish reading)
Task: explain the test result to patient's daughter/son
Management
Investigation: *Na: 120
K: normal
HCO3: normal
*Cl: 72 (reduced)
*Anion gap: 17 (normal: 8-16 mEq/L)
*Urea: 6 (normal: 7 to 20 mg/dL)
*Creatinine: 0.5 (normal: 0.6 to 1.2 mg/dL)
eGFR: normal
*Cal osmolarity: ?270 (normal: 275–295 mosm/kg)  
I explained the blood test result. Blood chemical imbalance & fluid imbalance. She told me what
blood chemical abn. I explained it. She asked what is Anion gap. I explained it. She asked me
what is the significant of these finding. Due to this, patient suffered from symptoms. It is most
likely due to SE of medication. He is taking waterpills. She asked me what is waterpills. I
explained it. I told other possibilities: loose motion (she said her father has no loose motion),
head injury,.. I would like to admit the patient. She said her father is now in nursing home and
can treat in nursing home? I said Sorry, at this stage, I strongly suggest to admit. At hospital, S'list
will take care of him. Which S'list? I said Heart S'list (I don't know I may be wrong). Arrange some
more investigations: Bld test, urine test & imaging. Give fluid through vein. Closely monitor &
then recheck blood tests. Every thing fine, S'list let the patient discharge. In the mean time, I
stop the waterpills. S'list will monitor his BP condition. If high, add another medication. Recheck
understanding.

  Global score: 3
Key Steps: 1- Yes
2- Yes
3- Yes
  4- Yes
5- No
Approach to patient: 3
Interpretation of investigation: 3
Diagnosis/ Differential diagnosis: 2
Management plan: 2

Case 1 (21/9/2017)
Call from nurse about blood tests of 82 yr man in nursing home.
Tasks
-History
-Explain the cause
Details- SEs done hyponatremia ptn confused now
On hct, atorvastatin, aspirin. No fever, No trauma Previously well, No alcohol

Case 2 comment (Pass)


GP setting. An elderly man in nursing home, long history of HTN, DM, on
medication imipramine, atenolol and another medication. (not remember). (a bit
long stem).he has developed tired, confused and strange behaviour for the last
couple of days....investigations showed:
Na 120 (low).
Ka (normal)
Cl (normal)
HCO3 (normal)
Urea (normal)
Creatinin (normal)
Tasks:
• Explain the results to the patient's daughter
• Explain the possible cause of the condition of the patient
• Advice your management plan
Greetings the patient’s daughter
Checking the guardianship
Based on the symptoms of your father with confusion and disorientation, we consider
delirium or acute brain syndrome. There are many possible causes of that condition;
therefore we have to order several investigations. Among of those investigations are
electrolytes- minerals in our blood system, they play an important role, Low or high
value could cause delirium.
Now I’d like to explain to you your father’s electrolyte investigation results
As you can see here, I started with all normal values first. Only sodium is lower than
normal range, so that condition is called hyponatremia or low sodium in the blood.
As I mentioned earlier, delirium is possibly caused by many other underlying
conditions, such as infections, cerebral trauma- stroke and haemorrhage,
endocrinological disorders, chronic conditions etc
What we need to do at this stage is to arrange transferring your father to the hospital,
there are special units called Delirium unit that deal with such a condition.
Your father needs a full work up and medication review and they will change his
meds accordingly. A CTB will be required to role out if there is any abnormalities in
the brain, they will repeat his blood test and replace his electrolytes accordingly.
Also if possible presence of family in the hospital will help him to be reoriented and
more familiar with the environment. There will be a multidisciplinary team working
to look after you father, doctors , nurses, allied health services.
AMC feedback: Hyponatremia (PASSED)

Delirium Hyponatremia
2 cases
1-call from nurse  Hx + Mx (results available)
2-explain results to son/ daughter

History (focus on)


1-confusion/ behaviour changes
-duration, 1st episode or not
-sleep, hallucination, delusion, mood

2-Differential diagnoses questions


Hypoxia
-SOB, cough, chest pain, palpitations

Infection
-fever
-urine frequency, painful or burning urination, smelly urine (UTI)

Trauma/ head trauma


CNS.
-Blurring of vision, headache, weakness, numbness

Nausea, vomiting, diarrhea

3-General
Past medical history
-liver, kidney, heart, thyroid, DM, HTN

Medication, alcohol, illicit drugs

Family history

Explaining the results and possible cause


-from history, the most likely cause why your father is confused and behaving strangely is a
condition we call delirium or in another word “acute brain syndrome or acute confusional state”
is a common medical problem that is characterised by changes in mental function and occurs
more often among older people.
-There are several reasons behind this condition, that is why we have ordered some Ix and I got
the results with me, let us see it together.
Explain normal results first,
We ordered the levels of mineral body salts and it showed that all the mineral salts are normal
apart from Na level which appears to be low. We call this hyponatremia or low Na level in the
blood and I think this is the most likely cause behind your father’s delirium.
Other possibilities of delirium are: trauma, infection, UTI, low O2, medical illnesses, brain
diseases and medications.
From history, I can see your father is taking lots of medications which might be related to this
low Na level.
-sodium is an essential electrolyte that helps maintain the balance of water in and around your
cells, important for proper muscle and nerve function and stable BP levels.
-hyponatremia occurs when water and Na are out of balance in other word either too much
water like in heart, liver or kidney disease or too much water and salt lost from the body like in
water pills, medications, severe vomiting or diarrhea.
Management
-your father needs to be transferred to the hospital to be admitted in a special delirium unit
where he will be looked after by MDT (internal medicine specialists, nurse and psychiatrist..

-Delirium screen
we need to rule out other causes by doing further Ix like
Bloods: FBC, ESR, CRP, UCE, LFT, TFT, BSL, B12, Folate, blood culture if fever
ABG/VBG
urine: urine Na, osmolality, urine MCS, drug screen
imaging (CXR, CT brain, ECG)

-they will review his medications and change it accordingly

-they will repeat his blood test and replace his electrolyte accordingly.

-if possible, the presence of family in the hospital will help him to be reoriented.

Explaining anion gap in result


electrolyte are elements that occur naturally in the body and control important body functions.
the electrolytes have electrical charge; some are +ve and some are –ve they help to control the
balance of acids and bases in the body.
the anion gap value is the difference between the –ve and the +ve charge electrolytes. If high or
low it may be sign of an underlying disorder.

Karin case Hyponatremia (Delirium)


Case: Your next patient in GP practice is an elderly man with a long history of hypertension and
diabetes. He has developed tiredness, confusion, and he’s behaving strangely for the last 2 days.
Investigations show sodium is 120, potassium, chloride, bicarbonate, Urea and creatinine are all
normal.
Task
a. Explain results to daughter (indapamide, atenolol, imipramine, and meds for DM; good
control)
b. Explain possible causes and management

Basic Investigations:
- Hypoxia: pulse oximetry, oxygen saturation, ABG, CXR
- Sepsis: Blood culture, urine MCS, FBE, ESR/CRP, CXR
- Metabolic: ABG, BSL, urea and electrolytes
- ECG: cardiac enzymes and ECG
- Review Medications: diuretics, benzodiazepines, morphine, alcohol

Counseling
- Does the daughter have authority to talk in behalf of the father?
- From the blood reports, your father has a condition called hyponatremia. There are some
minerals in our body and one of it sodium and there is a low level of this in your father’s body.
Low sodium means low osmolarity causing leakage of water into the brain cells leading to edema
of the brain and this causes confusion, but there can be other reasons for confusion.

- Is your father on any medications? Does he take any alcohol? Any problem with his vision or
hearing? How is his diabetes? Is it well controlled? Is he on any medications? Have you seen him
gasping or having SOB or any sweet smell from the mouth? Any chance he could be lonely or
depressed? Any headaches or early morning vomiting? Any chance he had a fall? How are his
waterworks? Did he have any fever? Any problems with the heart or weakness or numbness of
his body? Has he vomited or had diarrhea?

- Most likely, the hyponatremia was caused by the indapamide. This is a medication which affects
the kidney’s diluting capacity and increases sodium excretion. (Once the depletion occurs, there
is non-osmotic release of ADH and causes water retention worsening hyponatremia).

- At this stage, I will call the ambulance because your father needs urgent treatment. In the
hospital he will be reviewed by a medical registrar. IV lines will be secured and blood taken for
further investigations. They will start him on IV fluids (PNSS or hypertonic saline 3% for rapid
correction). A cranial CT scan will also be organized.

246-DRE BPH examination


Your next patient in GP practice is a 76-year-old man who had one episode of urinary retention
3 days ago. His retention was resolved with catheterization. He has come to see you for further
assessment.
Tasks
-Perform abdominal examination on him.
-Perform DRE on a mannequin.
-Explain your examinations findings to the examiner

Abdominal examination (like in lecture 171)


1-Consent

2-General appearance end of bed (patient was in no distress).

3-inspection side of bed (there was a slight distension in the abdomen, which looked like been
due to fat tissue deposition, also there were many Cherry Angiomas that could be a sign of
chronic liver disease. But as there were no other signs of chronic liver disease such as
gynecomastia or skin rashes, it was probably an accidental and normal finding).
Note/ in the exam mostly will be normal but If you see anything sure comment on it.
4-palpation: superficial+ deep + organs (there was no tenderness or mass in abdomen. No
discomfort in hypogastrium, in deep palpation, there was no tenderness. The liver and spleen
were not enlarged)

5-percussion + auscultation

6-complete with hernia orifice, LN, genitalia and DRE

DRE
1-Introduction+ consent+ chaperone
- wash hands + gloves
(explain it might be uncomfortable but it should not be painful

*ask the patient to lower the trousers and underpants


*if you could lie on your left side, to bring your buttocks to the side of the couch and to bring
your knees up to your chest.

2-Inspection
-gently separate the buttocks and inspect the anus and surrounding skin.
*skin tags, excoriations, ulcers, fissures
*external hemorrhoids, prolapsed hemorrhoid, m.prolapse

3-palpation
-lubricate the index finger of your right hand
-position the finger over the anus as if pointing to the genitalia
-ask the patient to bear down to relax the anal sphincter
-gently insert the finger into the anus, through the anal canal and into the rectum.
-ask the patient to squeeze the finger to test anal tone
-rotate the finger so as to palpate the entire anal canal and rectum.
*look for masses, ulcers, induration, feces in the rectum
*prostate (size, shape, surface, consistency, middle groove)

-remove the finger and examine the glove. Look at the colour of any stool, mucous or blood.

4-remove and dispose the gloves


-clean off any lubricants or feces on the anus
-give the patient wipes to use if need to wipe
-give the patient time to put clothes back on
-ensure the patient is comfortable.

Differential diagnoses
1-BPH  size >3.5 cm
firm, rubbery
smooth surface
palpable midline groove

2-prostate cancer- enlarged+ asymmetrical


hard
irregular, nodular
midline groove no longer palpable

Findings from feedback


Inspection (there was no fissure or haemorrhoid pockets on anus, in the rectum wall there was
no irregularity or abnormal finding)
palpation (The prostate was enlarged, median sulcus was palpable, the surface was regular
without nodularity. I mentioned that to me the consistency of prostate seems slightly firm, but
as my hand is not experienced in the examination of prostate, I am not sure this much of being
firm is normal or abnormal, so in this aspect I would like to ask for a second opinion.)

Explanation from feedback


Finally, I explained about prostate anatomy and how it can cause difficulty with the flow of urine,
and explained about benign prostate hyperplasia and prostate cancer. I mentioned that none of
the findings in his examination were in favour of malignancy, however, I suggested that we could
do a blood test (PSA) to rule out malignancy. And the investigation results returned normal, the
treatment of his benign prostatic hyperplasia would be either medical or surgical according to
the severity of his symptoms and the opinion of his urologist.

Feedback 21-6-2018
Your next patient in GP practice is a 76-year-old man who had one episode of urinary retention 3
days ago. His retention was resolved with catheterization. He has come to see you for further
assessment. Perform abdominal examination on him. Perform DRE on a mannequin. Explain your
examinations findings to the examiner. I guess the last task was explaining the possible
differentials to the patient. Health Review or Pilot/ Pass with global Score 5 or 6.
I explained the examination to the patient and took consent, the washed my hands and started
with general appearance. In inspection the patient was in no distress, there was a slight
distension in the abdomen which looked like been due to fat tissue deposition, also there were
many Cherry Angiomas that could be a sign of chronic liver disease, but as there were no other
signs of chronic liver disease such as gynaecomastia or skin rashes, it was probably an accidental
and normal finding. Then started palpating the abdomen, there was no tenderness or mass in
abdomen. No discomfort in hypogastrium, in deep palpation, there was no tenderness. The liver
and spleen were not enlarged.

Then I proceeded to DRE examination on the dummy. Explained the examination to the patient
and explained what he can expect from the examination. Then after wearing gloves and using
lubricant, did the examination. mentioned that there was no fissure or haemorrhoid pockets on
anus, in the rectum wall there was no irregularity or abnormal finding. The prostate was
enlarged, median sulcus was palpable, the surface was regular without nodularity. I mentioned
that to me the consistency of prostate seems slightly firm, but as my hand is not experienced in
the examination of prostate, I am not sure this much of being firm is normal or abnormal, so in
this aspect I would like to ask for a second opinion.

Finally, I explained about prostate anatomy and how it can cause difficulty with the flow of urine,
and explained about benign prostate hyperplasia and prostate cancer. I mentioned that none of
the findings in his examination were in favour of malignancy, however, I suggested that we could
do a blood test (PSA) to rule out malignancy. And the investigation results returned normal, the
treatment of his benign prostatic hyperplasia would be either medical or surgical according to
the severity of his symptoms and the opinion of his urologist.

247-Benzodiazepine dependency
Handbook case Condition 100

Request for repeat benzodiazepine prescription from a 25-year-old man

You are working in a general practice. You saw this patient for the first time one week ago and provided a
prescription for his usual sleeping tablet, the benzodiazepine oxazepam (Serepax®) 30 mg daily, 25 tablets.
At that time, you were satisfied there were no comorbid problems such as depression. The patient has
returned today for another prescription. The patient's mental state is unchanged.

YOUR TASKS ARE TO:

• Evaluate the situation by taking a focused history.

• Outline to the patient the nature of the problem you have identified and proposed management.

• Answer any questions the examiner asks you.


AIMS OF STATION

To assess the candidate's ability to identify benzodiazepine dependency and counsel the patient
accordingly.

EXAMINER INSTRUCTIONS

The station assesses the candidate's awareness that longterm benzodiazepine use is a problem (abuse),
capacity to recognise overuse in this case by simple calculation (dependency), and preparedness to
intervene and address this with the patient. It also assesses ability to evaluate the problem further by
taking a focused history, communication skills in engaging the patient and ability to make an appropriate
management plan — including an awareness of potential problems following sudden cessation.

A brief survey of mood is all that is required in this case. The history does not suggest any other psychiatric
diagnosis except substance dependency.

At five minutes, if candidates have not already done so, interrupt and ask them to give their conclusions
and make their recommendation to the patient.

At six minutes, ask candidates whether they would immediately stop the prescription and why. You may
not need to ask this question if the candidate has already addressed this issue.
The examiner will have instructed the patient as follows:

You are a 25-year-old, divorced salesman. You were first prescribed this anxiolytic four years ago when
your marriage broke down. At the same time, your business failed and you were having problems going for
job interviews because you had lost so much confidence. You eventually found the anxiolytic was very
good at helping you get off to sleep quickly, and you have continued to use the anxiolytic since. Your work,
relationships and home life are generally okay now, but you expect this is at least partly because you can
get to sleep without fail and feel calm throughout the day.

Opening statement:

'This is just a quick one for you, Doc. I just need a refill of my sleeping tablets, thanks, and then I will be on
my way'.

If the doctor indicates in any way that you should not be taking the Serepax®, or that the doctor is going to
stop or substantially change the dose immediately, respond as follows:

‘I know I need to continue the Serepax® because on occasions I have taken only half a tablet for a few
days, or missed taking them for one to two days, and I ended up feeling edgy, jittery, shaky and was unable
to sleep or concentrate on my work. On one occasion, I thought I was going to have a heart attack because
my chest was pounding. As soon as I get back to my usual dose, I feel fine. I do not use, and have never
used, any other drugs.'
You are generally in good health. You do not smoke and rarely drink alcohol. You have not had any serious
illnesses, injuries or accidents, including car accidents. You have never had any fits, falls or faints or any
loss of consciousness. You did feel depressed when your marriage broke up and your business failed, but
that has been the only time.

Give the following responses and answers to further questions:

• When or if the candidate mentions addiction or dependency, say that you are aware of some
information about these tablets being addictive, but you don't think it applies to you (for no particular
reason), and also your symptoms on ceasing are evident to you that you have a 'real need'.

• When, or if, the candidate asks about the schedule of your use. respond by saying that you use them
only to get to sleep.

• When or if the candidate goes on to demonstrate to you that you have used more than one per day in
the past week, say that there has been extra pressure at work, and admit that you have taken a few extra.
You can expect the candidate to pursue the issue of quantities used. If the candidate approaches this
diplomatically, be cooperative in uncovering the overuse problem. If the candidate is critical or blaming, or
wishes to refer you immediately to a substance abuse unit or report you to the Health Department, take
offence and say you do not want to continue with the consultation: 'Just give me the script and I will go' or
'I'll sort this out myself.

• When the candidate explains a proposal of a period of monitoring your use and moods or stress,
followed by graded reduction, along with regular appointments, support and resource materials or
groups, agree with this management plan.

• If the candidate refuses to prescribe, take offence and respond as above: 'I'll sort this out myself.

• If the candidate agrees to provide a prescription with no suggested measures or comments such as 'we
will talk next time', accept that and say 'What days do you work. Doc. so that I can be sure to see you next
time?'

Be quite at ease and be pleasant. Provide your background history and the development of your habit in a

straightforward way. Respond to questions about your present use as outlined previously.
Questions to ask if not already covered:

• 'So what's the problem with taking these tablets, really?'

EXPECTATIONS OF CANDIDATE PERFORMANCE

The candidate should:

• Take a focused history of the patient's use of the benzodiazepine anxiolytic (Serepax®) and history of
other substance and alcohol use and patient mood.

• Advise the patient about the problems of benzodiazepine dependency and outline a plan of
management that includes a gradual reduction in use, along with regular appointments, support and
resource material and followup.
KEY ISSUES

• Identification and preparedness to address the issue of dependency and overdose.

• Appropriate language and attitudes in taking the history and discussing the problem.

• Knowledge of biological and psychosocial management of benzodiazepine dependency.

• Awareness of the risks of sudden cessation, such as acute withdrawal states, fits, agitation,
exacerbation of anxiety and treatment failure.
CRITICAL ERROR - none defined
COMMENTARY
The doctor is presented with the problem of being drawn into maintaining a longterm benzodiazepine use
habit, with clear evidence of over-use (approximately 75% greater consumption than the prescribed dose).
Benzodiazepines are recommended for short-term use only. While they have some place in the longterm
management of chronic severe anxiety, other treatments, including antidepressant medication and
psychological treatments (relaxation techniques, cognitive behavioural or interpersonal therapies) must be
applied first. Longterm prescription needs are to be closely supervised and monitored for over-use, such
as in this case. This case challenges candidates in a number of ways. Chiefly, it requires them to actively
and constructively intervene, not just to provide the prescription (with or without advice) or to just refuse
the prescription, thus provoking the patient to seek out another source, or risking precipitating a
withdrawal state. The satisfactory candidate, in addition to managing the immediate consultation needs,
will be aware of community support and self-help groups.
Thus, simply advising the patient that they will 'talk next time', or refusing to prescribe with no other
measures put in place, are both unsatisfactory. Similarly, referring the patient immediately to a substance
abuse unit would be unsatisfactory and counterproductive.

Karin case
Request for a Repeat Prescription of benzodiazepine (Drug Dependence)
Case: Michael aged 22 years comes into your practice with no appointment, seeking oxazepam to clam
him down and sleep. He looks a little unkempt, is agitated, fidgets and shakes his legs while seated. The
reception staff reports feeling uncomfortable around him and note that he was insistent to be seen today.
There was no overt aggression. When you ask about history of benzodiazepine use, Michael becomes
irritated and says he is asking for one script, but says he is been using them for years, started when he was
having difficulty coming off speed and the he had then become dependent. He says his current supply was
stolen with car this morning which has left him in anxious state with nothing to calm him down. He says he
usually gets them from a GP on the other side of the town but unable to get there without car. He says he
uses 5-6 tablets of 30mg oxazepam per day
Task
a. How will you address the request of the patient
- I am ready to help you. That is why I am here. Because you are not my regular patient, I need to ask a few
more questions before I can help you.
- Why are you taking this medication? How long have you been using this medication? How many times do
you take this in a day? Did you increase the doses by yourself?
- Drinking, smoking and illicit drug use?
- General health? serious illnesses, injuries or accidents (car accident)? Fits, falls, faints or loss of
consciousness? Did you ever feel depressed or diagnosed with depression or other psychiatric illness.
Management
- Explain: period of monitoring your use and moods or stress followed by graded reduction, along with
regular appointments, support and resource materials or groups
- Diazepam (valium): long-acting  I will change the medication to the same medication but will work
longer. Pick up daily supply.
Change to long acting diazepam and tell the patient to take the weekly dose from the pharmacy then we
will monitor the progress and assess his condition weekly and we can try reduce the dose and see how it
goes. Support group and community support and reading materials.

Case 25/ 8/2017


benzo dependence but patient wants to stop. Regular GP on leave. Patient stressed because son has
cancer-trouble sleeping taking diazepam 1 tab bd
-her son is good now/ has not tried to stop med
-no grief
-only stress as single mom on 3 financial problems
-gets help from family mom and sister, centerlink payment
-looking for jobs
Mx
1-psych review (refer to psychiatrist??
2-sleep hygiene, stress mx + CBT
3-taper off diazepam slowly
Patient response: if you give me a script I will be on my way?
I am sorry I can’t give you the script as it seems you are going dependent on this med.

Benzodiazepine Dependency
History (focus on)
1-Confidentiality+ calm the patient down
2-Medication Qs
-How long
-How often
-why
3-Tolerance
-have you tried to increase the dose to get the same effect?
4-Dependency
-have you missed doses? What happened?
5-Any SE like faints, falls or fits?
6-Psychosocial history (Mood, suicide, Delusion and hallucination)
(insight and judgement)
7-HEADS especially SAD
8-PMH, Past mental history, Family history

Counselling
-I understand that this medication has been very helpful for you, which is good and normally
used to help with sleep, stress and anxiety. However, this medication is only for a short term use
because of its problem with addiction and dependency.
(But I have been using it for along time?)
-I understand that and I know that you have been on it for a long time, but clearly now you need
to use more of it.
-so what we gonna do for you is we gonna change you from oxazepam to another medication
which is called diazepam which is of the same class of medication but it is longer acting and it
will help you not to get the effects of dependency and tolerance over long term. Then it will be
reduced gradually and slowly over time until you can be off the medication, to make sure that
you do not get any withdrawal symptoms or feel unwell.

-So I will give you the new medication for 2 weeks with weekly dispensing as you will need to
pick up the medication on a weekly basis from the pharmacy (to avoid the risk of overdose)

-I like to refer you to the psychologist as well for CBT so if you feel stressed you do not need to
use the medication alone but you can see psychologist and use the medication altogether.

-Sleep hygiene + relaxation techniques


-lifestyle advice (SNAP)
-social worker, support group, family meeting with consent
-review for any withdrawal symptom or SE

248-Liver metastasis
Feedback 9-5-2018
NOT SCORE Jaundice + TAC with a mass in the liver.
Patient on his fifties coming to the GP, with a history of jaundice, pale stool, and dark urine,
history of cholecystectomy I don’t remember how long ago.
A CT scan of the liver was taken and shows a mass in the liver (no more details about the mass
can´t remember if it said solid mass).
Task
-Take history for no more than 3 or 4 minutes.
-Give diagnosis and differentials with reasons to the patient.
-Further management.
I can´t really remember well this case sorry guys. Don’t remember for how long he has the
jaundice and the other symptoms, he has been losing weight, don´t remember if I asked about
pain but it wasn´t mention on the stem outside. When asked he has a history of colon cancer, 2
or 3 years ago, which was removed.
I managed this like breaking bad news at the moment of giving diagnosis because the mass in
the CT scan and medical history of colon cancer. I mentioned that it could be that the cancer has
spread to the liver as colon cancer spreads first to liver.
Patient couldn´t believe and he asked me if I was sure I told him I think is most likely based on
the history but I will refer him to specialist who will be taken a biopsy to confirm.
I also mentioned vaguely about the mass being blocking the duct that connects the liver and
bowel and that is why his symptoms. As differentials I mention about and abscess in the live
and does it didn´t have more time.
Feedback
50 plus male, given in the steam> had pale stools, tea coloured pees, epigastric pain, gall
bladder surgery, surgery for colon cancer and was OK so far, CT scan and found a lump in the
liver .
Tasks
-Further hx 5 mins,
-explain dx/ddx with reason
2 minutes outside: this case is really long stem outside, it seems complicated but believe me, if
any cases look complicated, it is an easy case. But you have to fullfill all the task and know what
is the main information. On the questions, they have 3 or 4 paragraphe and many investigations.
I scan quickly the idea and task. Also, this is the really long cases, i told myself so i have to solve it
carefully.
Inside the room: I talk with patient a little bit. As there is many investigations have been done
then i think one of the good point is explain to patient what investigations we did on him, what is
the positive and negative and ask him additional information only. For example, i asked him why
he did cholecystectomy, how long. When he was diagosis for colon cancer, what did he do and
any follow up. The more i ask , the more i think that the patient may have nasty growth or
recurrent colon cancer and spread to the liver. Only when i got that point, i did not ask any
more and jumb to the next task.(I finish history sooner as always) I told him about the strictere,
hepatis, stone in common ducts and nasty growth in liver or some where expecially in collon.
The patient keep silence and avoid my eye. I think he may shock because of the news so I did the
breaking bad new at that time. He ask me that: are you sure it is nasty growth. I said i know it is
difficult for him at the moment, i am not sure about that until the investigations come back,
again i listed all investigations i can do for further exporation and i will send him to specialish. I
told him he is not alone and i am here to help him. Bell rang.( This is the thing i studied from
other passed candida, including investigations and some managament in the DD cases).
Feedback
**50 plus male, given in the steam> had pale stools, tea coloured pees, epigastric pain, gall
bladder surgery, surgery for colon cancer and was OK so far, CT scan and found a lump in the
liver .
Task: Further hx (5 mins), explain dx/ddx with reason Karen med- 185
patient had pain for few weeks. . on history symptoms of obstructive jaundice.
He underwent cholicystectomy 18 months ago. CA colone was treated with surgery and was
not given radiotherapy and chemotherapy. last colonoscopy 1 year ago was clear.
CT scan was done. Found a mass in the liver.discription not given . explained possibilities to the
patient.
He didn't have any travel history. on HTN meds. weight loss + in the history.no alcohol use. ex
smoker.no substance abuse. cant remember the sexual history
Hx-
Stool- how long pale, any malabsorbtion problem, in any special diet, any
disrrhoes, blood in stool ever.
Urine- how long, eat any bitrout, paindul voiding, decrese voiding, any
injury in pelvic area.
Tummy pain- for how long, where is the pain, radiation, severaty,
aggreveting or reliving factors, associated with food.
General health- skin colour change any jaundice, itching all over the body,
tiredness, wt loss, loss of apetite, lump and bump, N/V, cough, night sweet,
chest pain, any heart, lung, kidney problem, DM, HTN.
Past medical Hx- GB surgery how long before, any problem after that. Ca
colon surgery how long before, r u on regular followup, after surgery any
other mx needed.
Fx hx of cancer
SADMA

Explain CT findings- We found a mass/growth inside ur tummy, looking like


attached to liver and size of liver gets bigger and all ur s/s going with liver
problem to me. Not sure yet what kind of tumour it is and from where it
came from. It can be liver mass or tumour, colangiocarcinoma- tumar in
the duct very close to liver or a spreading cancer from ur upper or lower
GIT- because u have past hx or ca colon. But is can come from pancrease,
lung, skin, prostate or breast.

Need to conducts some investigation- blood for CBC, tumour marker,


urinalysis, endoscopy and colonoscopy to check ur gut, LFT- LIVER biopsy
to see what kind of growth it is, ERCP chest Xray, full body Scan called
PET scan.
I will refar u to specialist for some invasive testa- biopsy, colonoscopy and
after result back can confirm dx
a trained team will asses ur condition and will plan ur mx as best possible
to give u better confortable life.
Nearly similar case:
Your next patient in an outpatient department is a 75 year old Mr. Jones who comes to find out
the results of his ‘check-up’ last week which he had been organised before a planned trip to
Greece in 3 weeks time. Amongst others you had arranged a CXR which incidentally showed a
liver lesion and the radiologist had suggested a CT of the liver. The CT is provided which shows a
very large liver almost occupying the whole picture with a large necrotic mass and multiple
lesions in both lobes of the liver.
Your tasks are to:
1. Explain the CT findings to the patient take a further history
2. Discuss further management options with the patient
3. Answer patient’s questions

CT INTERPRETATION:
The CT findings are most likely malignant, probably metastases from a primary tumour.
Whatever the primary tumour might be the prognosis is obviously very grim with such an
extensive involvement of the liver. This situation requires “breaking bad news” with a very
empathetic approach to the patient who’s main issue is actually to travel to Greece to the
wedding of a nephew. This question needs to be answered and weighed up with advantages vs.
disadvantages to further investigate the lesions and chasing the primary tumour.
The patient has been quite well and has not had any symptoms!

Management options:
1. Further diagnostic steps with needle biopsy, colonoscopy, gastroscopy etc.
A liver biopsy in itself has some risks and the patient might prefer not to have any investigations
performed.

2. Therapy seems to be pointless because the cancer would be very advanced and will follow its
own path (?6-12 months, refer to a specialist’s opinion)

3. The most important issue is probably a discussion about the travel plans, perhaps including
other family members etc.
249-Diverticulitis + ruptured diverticula
1-Acute diverticulitis
Case 20/4/2017
My first case was Station no 4
PE- 42yr old lady co wilt pain in the left lower abdomen.. I did as usual and give

Dx-diverticulitis

AMC's feedback- abdominal pain – pass

Case 24/6/2017

Diverticulitis - acute pain abdomen

Case (9/3/2017)
acute abdomen physical examination
pt having pain in left iliac fossa ..
Task
physical examination
diagnosis and d/ds
positive findings 1)tenderness right iliac fossa and rebound tenderness .temp=38

Karin case
Acute Diverticulitis

Case: Elizabeth aged 65 years presents to your GP clinic. She had abdominal pain for the last 1-2
days which is getting worse now. She also feels nauseous but no vomiting. She has not
experienced such pain in the past. She had hypertension and Type 2 DM and is on regular
Coversyl 5mg daily and metformin 1 gm BID. Elizabeth lives in an independent unit with her
elderly husband. Both her sons live interstate and visit them only on special occasions.

Task

a. Focused history (used Panadol and neurofen; dull LIF pain, does not go anywhere, feverish,
bowel motions every third day, no problems with waterworks; generally feels tired;

b. Physical examination (sick-looking, tired, mild dehydration, no jaundice, BMI is 23, PR: 96
regular, BP 100/70, T: 37.6, RR: 16, O2 98%, no distention, mild tenderness on the LIF, no
masses/rebound/guarding/rigidity, bowel sound is present; fecal matter is present on DRE; no
bleeding, fissures or hemorrhoids)
c. Diagnosis and management advise

Features:

- Over 40 years of age with long-standing, grumbling, left-sided abdominal pain and constipation,
but can have irregular bowel habit.

- Occurs in less than 10% of patients with diverticular disorder

- Symptoms: acute onset of pain in the left iliac fossa, pain increased with walking and change of
position, usually associated with constipation

- Triad: acute pain + left-sided radiation + fever

- Signs: tenderness, guarding and ridigity in LIF, fever, may be inflammatory mass in LIF

- Investigations: FBE, ESR/CRP, pus and blood in stools, abdominal USD/CT scan (detect, fistula,
abscess or perforation), erect CXR, erect and supine abdominal xray

- Complications: Bleeding, perforation, abscess, peritonitis, fistula, intestinal obstruction

- Treatment:

o Admission

o Rest GIT

o Analgesics

o Antibiotics

▪ Mild: Amoxicillin + Clavulanate TID x 7 days or Metronidazole + Cephalexin

▪ Severe: Ampicillin 2g IV q6 + gentamycine 5-7 mg/kg IV/day + metronidazole OR metronidazole


+ Ceftriaxone 1g IV/day

o Surgery for complications

o Screening colonoscopy after acute episode

- Refer husband to respite care/nursing home


2-Ruptured Diverticula
Case (20/4/2017)
Acute abdomen. Middle aged woman( forgot the age. One candidate remember she was old
enough) came with tummy pain. Task: examination. Possible causes . ( there was generalised
pain all over abdomen, maximum intensity to left side, epigastruc point, mc burney , Murphy all
were tender, though there was no distension or no rigidity girding. I asked per vaginal( though
not sure it was relevant / not, examiner told me bilateral Adnexal tenderness present. Cough
impulse tender but no herniation.

Case (4/5/2017)
Acute abdominal pain for 8 hrs. esp on lower tummy. Peritonitis features (+). Fever +.
TASK : history, Dx and differential D.Dx with reasons

Case (9/5/2017) Acute Abdomen - Dx and DDx (lower abd peritonitis, urine preg test not done)

Case (9/11/2017)
PE acute abdomen, pt not let touch the abdomen. Task- PE, and tell Dx, DDx to pt with
reassurance ( pain all over the abdomen going to the back, nothing else )

Karin case Acute Abdomen (Ruptured Diveticula)


Case: A 50-years-old man presented to you in ED because of sudden onset of acute abdominal
pain.
Task.
a. History (started 2 hours ago LIF, sharp pain, constant, nauseated, feels hot and sweaty,
movement and coughing make it worse.
BM every 2-3 days, no diarrhea, constipated x 10 years, no problems with waterworks.
no surgeries, no blood or change in mucus, first time to have abdominal pain, no history of PUD;
diverticular disease x 2 years, does not smoke, does not drink alcohol or drug use, no
medications)

b. Physical examination (unwell, very sick, vital signs: BP 110/70, PR 110, RR 23, 98, T:38;
tenderness on the LIF with marked guarding and rigidity, DRE: normal examination)

c. Investigation (FBE, blood culture, LFTs, XRay: air under diaphragm)

d. Management

- IV drug abuser – use tramadol


History
- I understand you have come to the ED because of abdominal pain. When did it start? Where is
the pain? Can you describe the pain? Is it constant or does it come and go? Does the pain travel
anywhere else? Can you recall any precipitating factors like trauma? How bad is the pain on a
scale of 0-10 (Give painkillers)? Does anything make it better or worse? Is it the first episode?
- Do you have a fever?
- How is your appetite? When was the last time you had something to eat?
- Do you have N/V?
- Did you have heartburns or hunger pains in the past?
- How’s your waterworks?
- Do you have regular bowel movements? Any diarrhea? When did you last pass stool?
- PMHx: Are you generally healthy? DM? Hernia? PUD? HTN?
- Any surgeries in the past?
- Medications? allergies? SAD?
- FHx of serious condition?

Physical examination
- General appearance
- Vital signs
- Lung and heart
- Abdomen:
o Inspection: move abdomen with respiration, distention, masses, visible peristalsis
o Palpation: superficial
o Percussion: tenderness
o Auscultation: bowel movements
- DRE
- Urine dipstick and BSL

Investigations
- FBE, ESR/CRP, LFTs, U&E, Blood cultures, ECG-12 leads, erect and supine abdomen and CXR
- CT scans

Diagnosis and Management


- According to the history and PE, I highly suspect rupture of diverticula. It explains your pain, PE,
and presence of air under the diaphragm on the plain xray. It is a serious condition and can lead
to peritonitis or generalized inflammation of the inner abdominal lining. That is why, you need to
be assessed by a surgical registrar ASAP and an urgent operation performed.

- From now on, you shouldn’t eat or drink. I will put IV cannula to give you fluids and broad-
spectrum antibiotics (Ampicillin + Gentamycin + Metronidazole or Timentin/tazocin).

- I will place an NGT to decompress the stomach. The surgeon will tell you more about the
operation. In general, the surgical team will most likely perform laparotomy and will resect part
of the bowel with the ruptured diverticula and then connect 2 end of the bowel. Most likely,
temporary proximal colostomy is needed. If stoma is needed, it will be closed electively in 2-3
months time.
Feedback 31-5-2018
acute abdomen - A lady with abdominal pain 2 days.? Vomiting,not passing stool.period ? 2
weeks ago.
Tasks-Examination ,dx and ddx.

Lady was lying on the bed. asked vitals BP 100/70 ,P 100, T ? Mild febrile.examiner also
mentioned no pain killer was relieved the pain.
General I checked for signs of dehydration -skin turgor -decreased.no pallor,no jaundice,mucus
membrane ? Examiner said dry. Inspection abdomen not move with respiration.
Light palpation. Tenderness all over, she was pretending pain by facial grimace and voluntary
guarding but the abdomen was so soft.
Maximum tenderness at SPA , so I was a bit confused but I did step by step. i did not do deep
palpation and percussion.
(I also did not mention the examiner why I skipped those) i asked the patient I want to do some
special test to confirm the diagnosis .i did mcburney test the patient expressed pain.
so I did cross rebound tenderness at LIF. i explained the patient about what I am going to do and
said I understand u r in pain but by doing the test I can get more detail about the problem.
I am sorry but please bare the pain. she said ok. Patient expressed severe pain at that time. Then
I realized the actual problem is from LIF, not from SPA and not appendicitis. BS reduced.
Told examiner conclude examination by do pelvic examination with the consent of patient and
chaperone and examiner gave all informations and those r normal. pregnancy test not available.
Explained-intestinal perforation could be d/t diverticulitis. could be obstruction, other possible
causes appendicitis, gynecological problem , CA less likely.
Emergency condition, need to be admitted and seen by specialist, arrange ambulance. Give fluid
to correct dehydration.
Keys 4/4 approach 6,technique organization and sequence 5,accuracy 5,dx and ddx 5
global 5

Feedback 21-4-2018
50 Y old Male Patient with acute onset of generalized abdominal pain . Smoker and Alcoholic.
You are in the ED.

Perform relevant examination – Offered analgesics , asked vitals and claims will move to resus ,
generalized abdominal tenderness with peritonism, Reduced bowel sounds asked for regularity
of pulses – regular , checked the inguinal regions and offered scrotal and DRE ( examiner said
normal )
Explain the Diagnosis and Differentials – Since signs of Peritonism present I said – Perforated
viscous likely peptic ulcer / Intestinal obstruction , Pancreatitis , Acute alcoholic hepatitis,
Mescenteric ischemia unlikely due to regular pulses , – All explained with a diagram

Global score - 4 ( Sequence of Ex 3, Examination 4 , DDs – 4)

My comment - If signs of Peritonism present its most likely Perforated viscous ( peptic ulcer ,
AAA, don’t forget about mescenteric ischemia in all elderly abdo pain )

Feedback 21-7-2018
Abdominal pain – PASS (G.S- 6)

Stem: middle aged man with acute LIF pain, recent onset diarrhea. No H/O past constipation
etc given.

Task: - Do PE

-Give your DDx

- Pt was in pain, offered pain relief. Asked for vitals (Was Febrile, rest normal) Did general
assessment, Performed all steps of exmn including insp, superficial and deep palpation, renal
angle tenderness, hepatomegaly splenomegaly, percussion, BS auscultation, , showing sympathy
and saying ” I’m sorry but I have to do this to come to a diagnosis” all the time – Generalised
tenderness, max tenderness in LIF. Asked for genitalia, H.O., DRE- all Normal.

- Gave DDx as Diverticulitis, diverticular abscess, Ca, viral gastro

Feedback 9-5-2018
Case: Abdominal Pain.
GP practice. Middle age male with pain on left hemi-abdomen, no other details on screen about
the pain or other symptoms related or I can´t remember sorry guys.
Tasks:
Take history.
PE from examiner.
Diagnosis and differentials with reasons and I think further management.
When taken history I think the abdominal pain was for the last 2 weeks or more. Pain 4/10 no
radiating, no fever no vomiting, no urinary symptoms; he has constipation so he was taking
laxatives so he was having some sluggish stool. He also mentioned hard stool with blood on the
toilet paper when cleaning. I forgot to ask about his diet. At that time I though constipation and
anal fissure as main diagnosis but then I asked about losing weight and he said he thinks that yes
because his clothes are loose and denied any changed on his diet or exercising. After that I got
lost in my thoughts probably because I had to add more differentials and I completely forgot
about the constipation and the anal fissure and focused on colon cancer. (this one was my 2 case
and I believe I was so nervous that I couldn’t think clear, maybe a good idea to take notes while
taking history) No any family history of cancer or something important. I think he has high blood
pressure and taking medication.
PE was completely normal I just asked vital signs and the examiner straight away said PE normal
no abnormal findings. I didn´t ask nothing else. But I think I should have asked about
proctoscopy.
I can´t remember exactly what did I say to the patient but for sure I mentioned about colon
cancer, urinary tract infection as differential and refer for a colonoscopy.
I am pretty sure I failed this case.
I passed this case. Global score 4

Feedback
50+ /M, Lif pain, for a month, hx, investigations with reasons and managements( pain qs,
tired, weight loss, blood pr, conspitated, nil bowel screening , / colonoscpy - colon cancer FBc -
tiredness , lft / rft - general health

Out side- LIF pain dd- diverticulitis, diverticulosis, UIT, trauma.This is a long cases
Inside- VS, pain questions, intestital questions- the patient had consipatient and bleeding in
stool. So i during history, i change my dd to cancer, IBS, divericulosis, diverticulitis, hemoroid.
After talking about dd, and investigations( colonoscopy, X ray, FOBT, CBC, iron, LFT)the
management is one of the task, i told patient that i will refer him to specialish, give for him pain
killer now, he has constipation for lifestyle modifications- bell rang-( I believe many candida will
miss the management as the questions wrote- talk with patient about dd and management), i
only saw it when i glance to the task in the lask minutes.
250-Rheumatoid arthritis counselling
Feedback 22-2-2017
Middle age lady with pain and stiffness in hand for a long time. You are in GP. U saw the patient
last week and gave diclofenac but it didn’t help her. Some blood tests were done and shows Rh
Factor positive, anti CCP positive, FBE – normal. Her mom has RA and now she is on medication.
Tasks
– Explain result and most likely diagnosis to patient
- Explain the course of the disease
- Discuss initial management plan with patient

2 mins thinking
- This is my favourite RA counseling case
- I ll talk Dx, risk factor, disease course, steroid and PRICE in acute Mx, MDT, centre link and
support group (One of my fris failed this case as she didn’t talk about support and centre link)
Counseling There was Indian male examiner and middle age white lady roleplayer.

Greeting – Good morning Lindsay, I am Dr Khine. Nice to meet u again. How is ur joint pain
today?
(RP : I still have it Dr)
How would u grade ur pain from a scale of 1 – 10? It s 4-5. Would u like to have pain killer first?
(No, Dr thank you)
We did some blood tests during our last appointment and it shows that u have a condition called
rheumatoid arthritis.
(I looked at role player’s face, she didn’t show any expression), I continued Have u ever heard of
it?
(Yep, My mom has it and she is now on medication, but I don’t know it too much)
Alright, I am gonna explain it. If u have any confusion during our conversation, please let me
know. I am more than happy to repeat it again. Ok?

RA is basically an autoimmune disease. Our body has immune system to fight against infection.
But in autoimmune disease, immune system attacks our own body tissues. In RA, it attacks
joints, esp small joints of hands and feet.

I drew a diagram of joint with capsule. And scribble on articular cartilage and joint capsule.
That’s why u have pain and stiffness in hands joints.

Why it happens, exact cause is unknown. But it runs in the family and u mom has similar
condition as well. Are u following me so far?
(Yep)
It is a life long disease and unfortunately, it is not curable. But it can be controlled very well by
medications. But if dun fix it properly, it can progress to deformity of hands which can affect
daily life severly.

This disease has a course of waxing and waning, it means that u may have acute joint pain for
about few days to weeks and then pain free for about some time, and then, pain comes again.

I will get involved bone and joint specialist rheumatologist and we ll manage u as a MDT which
involves me as a GP, nurses, social worker, occupational therapist and physiotherapist.
Our aim is to prevent the progress of disease.
Since now u are in acute pain, I will give u some suggestions,
P – pain killer, since diclofenac is not working, start steroid for short course. Since it is only for
short term, u dun need to worry for SE.
R – rest at home, I am happy to give u medical certificate to get off from work.
I – U can apply ice on areas of pain,
C – compression with ice and elevation. May I know ur occupation?
(RP : I am senior executive in office and I need to type a lot)
Ok, During acute attack, I want u to take a rest at home coz if u use ur joints excessively, it can
impose further damage of joint and progress to deformity rapidly. If u are happy, I will talk with
ur boss or supervisor to change u to another position where u dun need to type a lot.

If u have any financial concern, I can refer u to the centre link.


Then, u will see the specialist. He will consider doing some blood tests and xray of ur hands. He
may start u on DMARDs which are special meds to slow down the disease progress of RA. We ll
review u regularly and make sure u will have a good quality of life.
So far are u with me? (Yep Dr) U dun need to go through this alone.

I have a lot of patients with RA and I will give u contact no of RA support group where u can
share ur experiences each other.
Later on, when pain subsides, physiotherapist will teach u some exercises.
I will review u regularly, and give u reading materials so that u can understand it better. If u have
severe joint pain, high fever, feeling very unwell, go straight forward to ED asap.

AMC Feedback
- Painful, swollen hands : Pass (Global Score - 5)
Approach to patient - 4 Dx/ DDx - 4 Counseling - 5 Management plan – 5
Rheumatoid Arthritis
You are a GP, and 28/F Samantha is a professional pianist, who came to you last time with
swelling and pain in her hands. You ran some blood tests that show RF+, and mild anemia.
Diagnosis is early rheumatoid arthritis. She has been taking ibuprofen. Her mother has RA, and
takes corticosteroids.

TASKS
Explain the condition and management. No further history or examination is required

KEY POINTS
Progressive condition
Early diagnosis and treatment can slow the progression, but will not cure the disease
We can start DMARDS
Refer to rheumatologist

CRITICAL ERRORS

Not explaining side effects


Not referring to rheumatologist

COUNSELING FORMAT

What is the condition


Is it common/uncommon
Serious/not?
Cause
Symptoms
Diagnosis/Investigations
Management - Pharma/non-pharma/support
S/E or complications

APPROACH
COUNSELING FOR RHEUMATOID ARTHRITIS:
Good morning, I understand you are here to discuss the results of your test. The test has
confirmed what I was expecting. I'm sorry to inform you that what you have is rheumatoid
arthritis. Have you heard about it?

WHAT IS IT?

It is an autoimmune disease, that causes pain and swelling of the joints. The normal role of your
body's immune system is to fight off infection to keep you healthy. However, in an autoimmune
disease, the immune system starts attacking the body's own healthy tissues
In RA in particular, the immune system targets the lining of the joints, causing inflammation and
joint damage.

COMMMON/UNCOMMON
It is the most common form of autoimmune arthritis
SERIOUS/NOT
It can be serious when diagnosed late. But in your case, I am glad we caught it early. I am here to
help you. Let me explain more about it.
CAUSE
The exact cause is unknown. But it is more common in patients who have a family history
SYMPTOMS
It usually affects the small joints of the hands and feet. However, large joints like the hip and
knees might also be affected at a later stage.
The main symptoms are joint pain, swelling, and stiffness, especially in the morning
It is usually symmetrical--where both sides of your body are affected
What's good in your case is that you are at an early stage and with early diagnosis and the right
treatment, most people live a full and active life.
Course (key)
However, the course of RA varies, and no two cases are exactly the same.
It is a progressive degenerative condition that runs a chronic course
You may experience flares of acute attack when joints become inflamed and painful
This happens with no obvious cause
These flares are usually followed by months or even years of remission, where there is little
inflammation and symptoms if treated in a good way.
MANAGEMENT
Mx will involve a MDT
Myself as GP
Rheumatologist
Physiotherapist
Social worker
Occupational worker.
The good news is that currently we have a lot of recent and effective medications that can be
used which limits the impact on joints with less joint damage in the long term
These meds have improved dramatically over the past 20 years and are extremely helpful in all
patients of every stage.
Unfortunately there is NO CURE for RA.The role of these meds is to reduce or slow down the
degenerative process

I will organise a referral to a rheumatologist who will tailor your treatment to your symptoms
and he will decide which ones will work best for you
He may try different treatments which could be NSAIDs, especially meloxicam or celecoxib,
Corticosteroids are used during acute flare ups, and then tapered off and stopped to minimize
side effects

The cornerstone of treatment is known as DMARDS - disease modifying antirheumatoid drugs


like methotrexate and leflunomide
More recent DMARDS are biological agents like TNF and infliximab

SIDE EFFECTS
Each dmard does have side effects which we will be on the lookout for. Some cause nausea,
vomiting, or diarrhea. They may also make you prone to infections or cause hair fall
The advantages or using these meds will we assessed against the side effects and benefits should
outweigh the disadvantages before we use them in the long term

NONPHARMA MGT
I would advise you for short term, not to play the piano, as your joints need rest during acute
flare ups. I can give you a medical certificate. Usage can lead to further deformity and damage of
the joint. Splinting also helps to rest the joint

I would advise you to get a second opinion from the rheumatologist about the impact of the
condition on your career

In australia, there is a lot of help and support available. If you are unable to work because of
your condition, you can get an allowance from centrelink

You can also join the arthritis association australia, which is an online association with other
people going through the same condition, who share their experiences and give and gain
inspiration

Weight loss if the patient is overweight


Healthy balanced diet, fish oils, glucosamine

Physiotherapy

Stop smoking

Hydrotherapy in heated pools

Use joints to full range of motion during remission to prevent stiffness. The physiotherapist will
advise on this.

IMPORTANT POINT: METHOTREXATE IS PRESCRIBED WITH FOLIC ACID. REGULAR FF UP WITH


LFTs and FBE IS NECESSARY!

I will organize regular follow ups to recheck your inflammatory markers

I will give you these reading materials to give more insight to your problem

Do you have any questions for me?

Thank you.
Feedback 21-4-2018
Rheumatoid arthritis counselling. All symptoms of RA given outside. Pt saw you last time and
NSAIDs prescribed to control hand pain which was not effective; some blood tests ordered and
results given including anaemia, positive anti-CCP;
Tasks
-Explain condition and differentials.
-Explain the implications of blood tests
-the implications of the disease.
-Further management to the patient.

Middle age female Rheumatoid arthritis counselling. All symptoms of RA given outside. RF done
positive . Occupation – admin assistant
Tasks
Explain condition and differentials – explained the results first and told its Rheumatoid arthritis,
autoimmune disease, other differentials - Osteoarthritis , Septic arthritis of joints
Tell the implications of the disease – chronic infection will lead to joint deformities, explained
other manifestations of RA – eye ( conjunctivitis , uveitis ) , Cervical spondylosis

Further management to the patient – started saying u will have good and bad days ‘’as a GP my
target is to reduced the number of bad days and to increase the number of good days’’ , acute
mx ( a Rest, cold packs , medical certificate, analgesics, NSAIDS ), long term ( steroids – need to
monitor Side effects likey DM , osteopenia / NSADIS – told the side effects / DMARDs / Multi
disciplinary management - Rheumatologist , Occupational and physiotherapy / Social worker
with centre link in loosing job / Family conference with patients consent ) – patient seems to be
happy

Global score - 6 ( Approach 6, DDs – 6 ,Counselling 5, Management – 6)

My comment - Read Carren’s case in RA – don’t go in detail in drug mx since ur not the
Rheumatologist
251-Collection of pilot and other uncommon but important cases
Case 1 (ITP)
Feedback 15-8-2018
Scenario # unscored.
Stem: Young lady in 20s comes with rash on both legs ( pic was given) .
Tasks
~History
~pefe
~dx and ddx
~ Investigation
There was a young lady sitting on chair inside.
I greeted and explored more on history ( looking at the picture I was thinking about hsp more)
but all history related to hsp was negative. She gave positive history of recent fever and urti .( I
remember that’s the only thing positive on history)
Pefe
Examiner handed over a card. ( few petechie inside the mouth as well) .Rest of the examination
was unremarkable. I additionally asked for neck stiffness which was not in the card( examiner
said oh—no no not there). Additionally asked for UD (he said not done)

At the end I was more towards itp so gave itp as my dx and explained it.. Gve ddx of hsp,
meningococcemia, allergy..Other ddx least likely scarlet, Kawasaki etc. Role player was quiet and
expressionless
Investigation: Keeping itp in mind told about Fbe and smear ( looking for platelets)

Case 2 (hysterectomy request)


Feedback 4-7-2018
1.37yr old lady need request hysterectomy
Task- hx, PEFE, further Ix, address her concerns about hysterectomy
My 1st station-not satisfied with my performance.
Hx-5P-has heavy bleeding- stated that’s why she & her partner thought to have hysterectomy
3kids, LRT done -5yr ago
PAP uptodate
Asked about anaemia sym –no sym-as far as I remember, hypothyroid features
PEFE-in a card-no positive features
PAP normal , uterus normal size, anteverted-this only I can remember.
Further Ix- blood-FBC,Fe studies,TFT
Hysteroscopy-need gynae referral
Concerns about hysterectomy-
Talked about-other measures to control bleeding after Ix -hormonal/IUD
Didn’t have time to discuss about risks /benefits of hysterectomy

Case 3 (pulled elbow)


Feedback 21-2-2018
CASE 2: PULLED ELBOW- ASSESSED NOT SCORED[ PILOT]
4 years child with father in GP setting, with pain and not using right upper limb. Tasks: history,
PEFE, management

Asked for painkiller, greeted child and father first. Father noticed since 2 hours, at supermarket
the baby had temper tantrums he tried to pull the crying baby from floor , she had pain and not
moving after that. First time, no other swelling,BINDS normal, social and sibling history
normal[ ruing out abuse and stress], taken care by both mom and dad

Pefe: got dad consent for examination, assured I will be gentle, he thanked me and got the
findings from the examiner ;general exn: normal, no rash swelling upper limb exn: semipronated
with limited ROM

mentioned that it is pulled elbow, drew a picture and explained the dislocation, explained the
procedure what I will do now- the reduction techniqu acted like doing with the child and advised
about the recurrence chance, to be careful while lifting child and offered support if needed for
tantrum.

Case 4 (nose bleeding)


feedback 7-9-2018
Just tissue box and paper tape on the table. When I said I will wear the gloves first, Examnier
stop me and told me to continue.
Pt was sitting with a blood stain gauze at the lt nose and shouting anxiously.
I said to calm down, sit upright and bend neck and to press over nose ( tip of nasal bridge) I just
show the area with my hand cuz I forget the name of area �.
Then asked one side or both side. He said lt side, I told him to press only on lt side for few sec.
( actually I don’t know exactly what to do and did not know the procedure detail). May be to
press both side?
Then, pt said , I think the bleed become less and stop. He smile and I also smile �
Because I don’t know what to continue if the bleed didn’t stop. I haven’t use the tape .
Hx- allergic rhinitis for a long time and using steroid nasal spray, hypertension and taking pills for
that ( I don’t know the name of the pill that he said) . No nasal pickled.. other hx - normal
Explain- I said there can be many reason to hv nasal bleed, in ur case , may be due to allergic
rhinitis -> tissue inflammation are easily bleed, hypertension-> sudden increase BP can cause vs
break .
Other causes.......
Mx->
As he is in ED, we will observe for 4 hr and if it is stop, we will let u go home .
( I can’t remember whether I said about to inform ENT specialist, check ur nose , do
investigations to rule out other causes)
I forget to check vital sign as well � �
May be I’m wrong, please check with the guidelines

Case 5 (Heart failure)


Feedback 5-7-2018
55 yr old male, diagnosed with COPD , now complaining of b/l leg swelling.
Tasks
Relevant history
Explain d/d to the patient
Leg swelling for 6 months , no change in exercise tolerance, no increase in
SOB, could not elicit any liver/renal pathology. On request patient showed
me his script containing amlodipine.
So my d/d were cor pulmonale , amlodipine also contributory. But other
causes like liver and renal failure needs to be excluded

Case 6
8/3/2018
Right lower abd pain US gall stones no obstruction and ruptured graphian follicle

Case 7 (Endometritis physical examination) this case you can see endometritis PEFE and turn it
into examination
recall 15-3-2018
ED, 10 days post partum bleeding and fever
Task: perform PE, findings to examiner, explain pdx and ddx to patient (Endometritis dt rpoc, i
examined the tumny with palpation without asking for any painful area, damn. Good thing that i
pressed softly the tummy. Os is open. Pelvic and breast examination results given by examiner.
Uterus 2cm above the pelvic brim)

1)Endometritis PE
A young lady comes to the GP with heavy bleeding, she is 10 days post partum and had a normal
NVD. Task was to perform PE and give dx
I started with hemodynamically stability , asked few hx qs regarding bleeding and any discharge,
calf pain .I did GA , Vitals , quick chest examiner said normal and on PA examiner said uterus
palpable 2 cm above the symphysis pubis and tender on superficial palpation , Didnt do deep
palpation. I asked to do PV. He said that everything was normal except uterus was tender
Case 8 (medications induced dystonia)
recall 4-4-2018
27 yr male with runny nose with hx of hay fever for years. Had family hx of asthma and hay fever
in brothers and father.
Vitals stable. Chest clear, turbinates enlarged, watery nasal discharge.
Task
2 min hx about what treatment or medication he had taken
Acute management of his symptoms
Future mx.
Roleplayer was wearing glasses. And cleaning nose with tissues
Was moving his face like showing his all teeth it was seemed to be involuntary reponse. Had hay
fever for years but this year it was worse. Had yearly vaccination done.

Case 9 (abdomen examination unknown dx)


recall 5-4-2018)
Acute Abdomen
ED, 52 years old female c/o sudden onset of abdominal pain, n & v. Pain killer given.
Perform focus examination, Dx, Ddx
+ findings are bruises or scratch marks all over the tummy, guarding & rigidity present,
tenderness in all quadrants, RT +, Cannot do deep palpation & special tests, no obliteration of
liver dullness, BS absent, PR exam normal.

Case 10 (osteoporosis another scenario)


recall 5-4-2018
GP, 72 year old pt comes back for test result. He has moderately severe COPD, walking distance
reduced 200m, smokes too much ( dun remember the amount ). Family h/o
Fracture neck of femur+ve. test result shows DEXA scan (femur -2.35, spine normal) , Vit D level
reduced.
Task: Explain result to pt & significance of the findings ( not more than 4 min )
Management

Case 11
recall 10-4-2018
1-55 yr WHEAT FARMER (Occup) started feeling SOB – describes as feeling “puffy” out of breath
Pedal Oedema Exertional Dyspnoea Medic – Perindropril Cholesterol Meds Aspirin No Chest
pain Presently but slight early morning cough(Perindopril) Diff Dxx Explore Allergic Alveolitis as
one of the Diff Dxx.
2-Heart failure - 55 year old male with exertional dyspnea, mild chest tightness, no
syncope.orthpnoea+, sleeps on 3 pillows.MI 12months back’
Case 12
recall 11-4-2018
You are about to see patient 67 years old female. She is your regular patient she was diagnosed
with CHF before and dr has started her on atenolol, prindropril, aspirin.
She comes to see you with complaint of shortness of breath even on rest. Her vitals were
recorded this morning BP-150/100, pr 110, afebrile, O2 normal and rr 29.
Physical examination- bilateral basal crepitations were present and S1,S2 were normal no added
sounds.
Abdominal examination normal
Task is to
-Explain investigations to examiner
- explain condition and differential to the patients
- investigations and management to the patient.

Case 13
recall 12-4-2018
A 56- year old lady with neck pain comes to you.
Has pins and needles in her Rt. index finger from 2 w ago. has had a breast surgery 6 years ago
due to breast cancer and LN was cleared after that.
apart from that several other things in Hx that I can't remember because I did not finish the stem
completely.
task:
* counsel
* DDX

Case 14 (hernia preparation)


recall 12-7-2018
Pre operation preparation for hernia. (reducible hernia and not obstructed) and investigation
given for the man in 50s, K 3.4 Na upper normal , high Carbonate , glucose upper normal on
history he give me piece of paper(Hydrochlorothiazide 25mg) for HT otherwise nothing
significant Interpret the investigation to the patient, tell him the cause, not remember if there
was a Mx

Case 15
recall march 2017
Warfarin surgery counselling
Patient with DM, HT, aortic valve disease, bowel operation for sigmoid Colon cancer within three
weeks time. She is on Warfarin, metformin, atovarstatin, metoprolol...
Tasks
1)History
2) PEFE he will provide u a card
3)Discuss the management plan regarding the drugs she is on

Case 16 (Pelvic inflammatory disease)


Feedback 7-9-2018
Station – 14- Pelvic inflammatory disease ( not assessed ) – pilot
P.S – I didn’t do well this case and luckily – Pilot

17 y o lady comes in your GP clinic with abdominal pain for 2 days. Before No Gastro symptoms
With white discharge –ask
TASK
- focus history 5P – LMP ( 2/52 ago , regular period 28 days cycles )
-PEFE specifically.
ABD – vague abdominal pain over lower SPA area, nil masses or organomegaly
Pelvic examination – along with chaperon and patient’s consent
CMT –tenderness with white discharge , nil other NAD ( said take a swab during speculum - ask
what swab , bacterial swab and PCR swab ).
UPT – not available ( I shouldn’t ask because patient’ LMP 2/52 ago with regular period )
3.Explain dx and ddx to patient ( gave PID – main , mittelschicmerz pain ( mid-cycle ovulation
pain ) , to exclude possible Sexual transmitted infections )

Case 17 Thyroiditis
Feedback 9-5-2018
Thyroiditis.
GP center. 47 year old female, comes to see you because she has been feeling nervous and
anxious for the last month (I can´t remember well for how long)
Task
Take history
PE from examiner
Diagnosis and differentials with reasons to patient.

I offered confidentiality as I thought at the beginning based on the stem it was a psychi case.

Patient was feeling nervous and anxious for last month or so, she said she is been feeling bother
by her husband and kids but she said this was normal. When asked about shaking she said yes!
Feeling hotter, losing weight, I think she said yes for palpitations, I didn´t ask about constipation.

She had irregular periods, husband had vasectomy I think or she had tubal ligation.

I performed PE for Thyroid. BP was high can´t remember numbers maybe 150/100. No
exophthalmos, enlargement and tenderness of thyroid no lymph nodes. Proximal muscular
myopathy present. Rest of the systems were normal. Didn´t ask about lid lag.

Explained about thyroiditis as more likely diagnosis and why. I mentioned perimenopausal
symptoms as differential don´t remember if I mentioned anxiety as well.
Passed. Global score 5

Case 18 Neck lump painful then painless


Feedback 21-6-2018 Fail
A young to middle aged man with a lump in the neck. You have just examined him. In your examination
there was a firm round lump on the midline of the left lobe of the thyroid (I can't be sure about the exact
definition of the lesion). The lump moves upwards with swallowing. The lump appeared 6 months ago. It
was initially painful, but now the pain is resolved. He doesn't have any other symptoms. Explain the results
of your examination to him with explanation of the possible diagnoses. Neck lump:/Fail: Global score 3/
Approach to patient:5/ Dx/ D.Dx: 2 Choice of investigation:4 Management plan: 3
Before I enter the room, I was confused about the possible differentials for this case. When I entered the
room, I introduced the role player and greeted him, then he asked that “well, doctor, what is the problem
with this lump in my neck? Is it dangerous?” I said let me read the stem once more, because I am not sure
what to answer yet. Then as I could not waste more time, I started thinking loudly. So I said, I am not sure
exactly what this lesion is. I was thinking of several differentials. Firstly, I thought it could be a cyst (or a
bag of fluid), however, as the consistency of the lump is firm rather than fluctuant, it is very less likely.
Then I was thinking of thyroid nodule, but a nodule is much smaller than this (the diameter of the lump
was around several centimetres I guess), so it is less likely, then I can think of thyroiditis, such as
Hashimoto, but It usually does not happen unilaterally, (I used layman’s term), finally it could be an
enlarged lymph node. But as it moves with swallowing, it indicated that it is anatomically attached to the
thyroid gland. Finally, it could be something nasty, however, it is also less likely as he does not have any
symptoms, in favour of malignancy, such as constitutional symptoms or hoarseness, again I thought about
thyroiditis, as he had had pain 6 months ago, but he did not mention any symptoms in favour of hyper or
hypothyroidism, so I did not emphasis on this diagnosis. Finally, I said that I am not sure what this lesion is.
It can be anything from an unimportant benign growth like thyroiditis, or cyst, to something malignant like
cancer, however, he does not have any symptoms neither in favour of good things or bad things.
Therefore, I suggest we do some investigations. First of all, we need a blood investigation to check the
function of the thyroid gland. The we need a non-invasive imaging such as ultrasound to visualize the lump
and the surrounding tissue. And finally a radioactive iodine scan could help to understand whether this
lump, uptakes iodine or not (differentiating between hot and cold nodule) and after these investigations I
can tell him more about the lump. I also apologized him for not being able to give him any definitive
diagnosis. Now that I have received my feedback, I think that I have emphasized on malignancy, both in my
differentials and in my investigations, so probably I have failed due to wrongly worrying the patient, while
the stem guided towards thyroiditis.

Feedback 15-3-2018 unknown status


GP, complaining of lump in front of the neck which moves with deglutition. Having that for 6mths. Initially
tender but not now.
Tasks: explain the dx and ddx, mention invx u would like to do to ur patient and possible management
Approach :Midline Structures:thyroid, larynx and LN and then concentrate on thyroid
when I entered I told him that I know that you are bothered about this lump in your neck but is it ok if I
ask you a few questions. He said no problem. Asked a few questions about thyroid, larynx etc. but he
replied in negative for everything . I realised that he must be instructed to do so as history taking is not in
task. So I took pen and paper and started with explaining midline structures. thyroid , larynx and lymph
node but since it moves with swallowing most likely thyroid then explained a bit about thyroid. even in
thyroid there could be many causes like cyst or a growth which is overfunctioning or under functioning or
a growth that’s nasty . All this while I was drawing or scribbling something .For this we need to do some
test like basic bloods along with thyroid function tests. Explained abit about thyroid function test. Then
will do ultrasound and Guided FNAC. Explained FNAC . Specialised tests like thyroid scan and thyroid
antibodies may be required as well . Explained a bit about them .checked understanding.

Case 19 IgA Nephropathy


Recall 5-4-2018
You have a middle-aged clerical male coming to your GP practice with a 2-day and also 2 weeks
w similar episode of history of hematuria. He was fine until yesterday when he had an episode of
URTI. On examination, patient’s BP is 150/95, and urine dipstick blood and protein +++.
Task:
-Take detail hx for 6 min (ask if the urine was yellow dark or reddish he said is very dark almost
back)
-ask PEFE (it was given on a card)
-Dx and Ddx
Recall 5-4-2018
GP, 32 year old lady comes to you coz of urine color changes.
Task:
-history taking
-Examiner will give you PE card
-Dx & Ddx
On history, she noticed urine color changes starting from yesterday. Had sore throat a few days back. No tx
taken eg. AB. No itchiness, no jaundice.
On examination, T’ normal, BP 150/90, no jaundice, no organomegaly. I don’t remember whether UDS
given or not.

Case 20 ( abdominal ex volvulus)


Feedback 27-10-2018
80 year old lady with dementia living in aged care facility. She had recurrent constipation.

Task
- Perform abdominal examination on the dummy
- explain Dx/DDx to her nephew
- there was a dummy of abdomen lying on the couch.

- there is a handle at head end which can be used to deflate and inflate manually during
inspiration and expiration.
- bowel sound is increased on auscultation.
- DRE was normal
- Then, examiner handed me an X ray of sigmoid volvulus.
Case 21 (BOV ex)
Feedback 27-10-2018 BLURRING OF VISION (UNSCORED)
27 year old male patient with gradual blurring of vision. He has no history of hypertension or
diabetes mellitus.
Task
- perform examination
- explain possible reason to him

fundoscope, torch light, red pin are given. Inspection


- general inspection (normal)
- pupil are equal and reacting to light
- fundoscopic examination (normal)

Visual acuity (snellen’s chart was hanged on the wall)


- both right and left is reduced. (worse on the right)
- then examiner asked me his visual acuity in details. (couldn’t answer correctly) visual
fields (normal) no diplopia, no nystagmus, no ophthlamoplegia. asked for ishihara chart for color
vision. (not available) asked for pin-hole test at the end. (not available)

Dx: explained that it could be due to problem with your lens (drew and explained)

Case 22 (Sore throat child)


Feedback 4-7-2018 12 yr old boy with sorethroat –came with dad
Task-hx, PEFE, D/DDx & Mx
Hx-sorethroat x2d,High fever, No cough/breathing difficulty, no rash, not taking solids but drinks
fluid, no abd pain
Generally healthy well child-no recurrent sorethroat/tonsillitis.
No on any medication spe puffers
No allergic to ab.
( as there were 4 task & pt is 12yrs & only 2d of illness I didn’t go for more detail hx)
PEFE-only gave findings what you ask
Vitals ok-I guess tachycardia
T-40deg tender CLN, hydration –ok
Tender enlarged tonsil with white exudates, no rash, no organomegaly
D/DDx
More likely strepto. Tonsillitis-explained with positive features
DDx-EBV, candida-less likely
Mx-
Conservative-rest/fluid/
panadol/nurofen if fever
Gargle salty water/antiseptic mouth wash
Ab penicillin
Review & red flags
Feedback 11-12-2018
12 year old son who has developed sore throat the last 2 days.
Tasks-Take a hx from parent for 2 mins,
PEFE,
Diagnosis, ddx and management

History-When I went inside, father was sitting and looks worried..so reassure him . ask all question of sore
throat..always ask is it first time?(could be case of recurrent tonsillitis) Is it getting worse? Is he able to
attend school? Is able to swallow or drink anything, any change in voice, drooling of saliva, cough, fever(+),
night sweats, weight loss,(lymphoma),easily bruising(leukemia)..T-TRAVEL(TB), O- ANYONE SICK AT
SCHOOL,P- PETS(CMV),S- SMOKING (PARENTS), E- ANYONE ELSE AT HOME WITH SIMILAR COMPLAINTS.
BINDSMA—HE IS NOT ALLERGIC to any drugs (don’t miss in this case because if we have to give
pencillin,,,we should know allergy)

PEFE-GA-LOOK SICK
Vitals- fever was high
Local-(Ent) -mouth- enlarged tonsils, inflamed with exudate. No uvula deviation. Cervical lymph nodes
palpable(NON TENDER, nose –normal, ear- normal.
Systemic--), ABDOMINAL EXAMINATION- NORMAL, SKIN SURVEY- FOR RASH AND BRUISING
Urine dispstick- nad

DIAGNOSIS-BACTERIAL TONSILITIS..MOST LIKELY STREPTOCOOCAL


DDX- Its not viral tonisilitis as it don’t have white thing on tonsil and also there is most of time not high
fever
Cmv, toxoplasmosis- don’t think so,based on physical examination findings
Lymphoma, leukemia-(just use nasty things, as you don’t have time..becoz management is there..

MANAGEMENT- give prescription for pencillin, salty gargles, voice rest, and also ask him to bring back him
for review in 2 days..to see how he is doing.4 R’s.

Global score -4, key steps-2/4


Feedback11-12-2018
(Sore throat) Global score 4 Pass

The mother of a twelve-year old boy comes to your GP because her son developed sore throat for the last
2 days. She seems very worried.

Tasks; History(2min), PEFE, dx and ddx

This was a rather difficult case because of the many tasks. When outside I kept in mind the ddx but mostly
Quinsy because it is an emergency. Also, I knew I would have to establish if it was a recurrent infection
otherwise I would have to refer for tonsillectomy.
I came in the room, greeted the examiner and the mom.
I introduced myself. She was very stressed but cooperative.
I was fast on my questions: When did it start? 2 days ago. First time? Yes. Anyone else at home with same
symptoms? No. Fever? Yes 39ºC. Weight loss? No. Able to eat? Yes but very painful. Running nose? No.
Cough? No . Recent travel? No. Any difficulty breathing? Any noise when breathing? No. Able to sleep?
Yes.
Vaccination: up to date
Normal development
SADMA
Home situation? (first bell rang)
PEFE: general appearance: child seemed well.
Enlarged tonsils, inflamed with exudate. No uvula deviation. Palpable cervical lymph nodes. No
hepatosplenomegaly. No neck stiffness. Office test: normal
I started explaining the mother: “ According to the history and PE, Paul has an infection on his tonsils.
That’s what we call tonsillitis. The tonsils are organs in our throat responsible for protecting us from
infections. Different bugs can cause this infection. In your son case, most likely, it is a bacteria because of
the high fever and the white plaques. Don’t worry! It is a very common condition and it has a good
prognosis.
Other causes for this condition would be Quinsy, but he has no uvula deviation; dental abscess but his
teeth are in good condition; EBV but he has no hepatosplenomegaly; meningitis but he has no neck
stiffness. (I forgot scarlet fever!)
We will start him on antibiotics (Amoxicillin and clavulanic acid), antipyretic and lozenges. We’ll take blood
samples for investigation and oral swab for microscopy and sensitivity.
Red flags: difficult breathing and stridor. Must take immediately to ED.
On the weeks following the infection, must observe any change in urine quantity or color, facial edema,
swallowing of the legs, SOB and joint pain because it might affect the kidneys or develop rheumatic fever.
I want to review him when he finishes the antibiotics.
Reading materials.

The mother was still not happy.

Key step 1:yes. Approach to patient/relative: 5


Key step 2:yes. History: 4
Key step 3:no. Choice & Technique of exam, organization and sequence: 3
Key step 4:yes. Diagnosis/Differential diagnoses: 5
Management plan:
Case 23 (Post thyroid replacement)
Feedback 11-12-2018
Patient has come for review post thyroid replacement. Take a history, Ask specific investigation
from examiner and explain to patient, Explain diagnosis, dd.
Ix: thyroid profile normal, mild elevated Ca (9.2), PTH mildly elevated (ask for this). Patient didn’t
have any physical health issues. Was started on Ca and Ca diet 10 years ago when she was found
low bone density on DEXA. Afterwards was asked to stop a few years ago as Ca was on the
higher side
Unscored station—my diagnosis was parthyroid adenoma. As this was the brand new case on
that day but I follow this based on my theoretical knowledge-Bones, stones, groans and psychic
moans..investigations you have to ask from examiner..if you ask pth then they give you a card.

Feedback11-12-2018
56-year-old female patient comes to your GP for the results of her tests. She is on thyroid hormone
replacement therapy with Levothyroxine.

Task: take history ,ask Ix from examiner and explain to the patient, Dx and DDX.
Ix: TFT all normal. Ca=9,2. PTH was high.
I came in the room and greeted the examiner and the patient.
I introduced myself.
I started with an open question: How are you today? I am fine, doctor. Do you have any complaints? No.
Than I started making thyroid questions: any preferable weather? Any weight loss or gain? Any change on
bowel habits? Any tremor? Any funny racing of the heart? All negative.
Did you have any of this symptoms before starting on the Levothyroxine? She said no.
Why are you taking this medication? Because the doctor said my hormones were low.
I asked about water works, edema, kidney stones… normal.
Last period? Were they regular before? Yes. Any menopausal symptoms back than? Yes. Were you on HRT?
No. Do you take calcium for the bones? I used to doctor. I started 10 years ago when exam showed low
bone density. But I stopped a few years ago because calcium was on the high side.
SADMA
Family history
Home situation.
After history had not been positive for anything, I explained the patient what parathyroid are , I drew a
picture, explained the action of the hormones and told her that post menopausal women are at higher
risk. I could not come with many DDx , so I told her it could be an adenoma, hyperplasia and kidney
failure. Later I read at Murtagh that the use of Levothyroxine is a cause of Hyperparathyroidism.
Than I explained some complications and told her I would refer her to the Endocrinologist.
252-Buckle fracture
Recall 2-3-2018
1-3-year boy swelling of forearm. Fall in outstretched arm.
2-4 years old child with pain in rt forearm since yesterday
Tasks
-hx
-pefe
-ix from examiner
-diagnosis nd management

https://www.rch.org.au/clinicalguide/guideline_index/fractures/Distal_radius_and_or_ulna_met
aphyseal_fractures_Emergency_Department_setting/

comment
Father of 4-year boy complaint of falling from bike with outstretched arm. Tasks Hx, pefe, Ix dx
mx Hx had fallen yesterday and now c/o pain at wrist, ex swelling+, redness+ no deformity ix x
ray AP buckle fracture with clear intact periosteum, lateral- no displacement mx lot of
reassurance+ all other mx

comment
The boy was playing with his brother and fell on his hand with outstretched arm. On examination
only mild swelling on the distal part or right forearm and pain, no crepitus, deformity, normal
sensation and capillary refill time. Child moves his fingers but doesn't want to move the arm.
When you start explaining management and say that you are concerned about a fracture and
would like to do an Xray. The examiner shows you an Xray with Buckle fracture. Management
below the elbow backslab or cast for 3 weeks, PRICE . ( I forgot to say PRICE but I should have)

Feedback 20-9-2018 buckle fracture


Limb pain (PASS)
4 years old boy brought to your GP clinic or hospital ( not sure ) by mom because of pain in his
Right forearm since yesterday.
Tasks :
History taking
Ask PE from examiner.
Order Inx from examiner.
Explain Dx
Management plan

Approach :
I greeted both mom and the child. Offer pain killer to him. Asked her is there any concern?
History :
This is the first time. Then, I started with pain questions. Pain started last night but worsen this
morning.
The child does not cry a lot. On asking how did it happened, mom says he was playing with his
elder brother last night and may be he got injured back then. No h/o fall. No redness but
swelling +, no deformity. No features of complications like changes in colour or sensation of
fingers. No h/o pre-existing joint problem. On social h/o, she’s single mom with 2 kids. She can
cope well and she’s financially secure. She works part-time, but she send her kids to child care
while she’s going for work. (I intentionally left BINDS as there are so many tasks but later I regret
it as this could be the case of abuse.)
PE :
GA – in pain
VS – stable
GC - I forgot to ask
I said I’d like to examine the painful area as well as both UL comparing each other. On inspection,
there was a swelling in Rt forearm but no redness and no deformity, no skin color changes. On
palpation, temp normal, pulses normal, CR normal, there was tenderness around the swollen
area. Sensation normal. Movement of the joints normal. Other systemic examination normal. No
bruises all over the body.

Inx : I ordered Xray. There shows the buckle #.


Dx : Buckle # ( I explained it on the xray )
Mgt : I reassured her that it is not serious and that the sensation and blood flow of his hands are
intact. This can be managed effectively. I’ll give him pain killer. Then , will give a back slab to
stabilize the area and it should be there for like 2-3 weeks, meanwhile I’ll give him medical
certificate. I asked is there anyone who can take care of the baby apart from you? She says their
grandmother. She says I should’ve brought him last night, my bad. Then, I reassured her that I
know that she’s doing her best and it’s not late, baby’s doing fine, you’re such a good mom…and
so on. I told her that pls make sure he moves his fingers and told about the redflags, gave
reading material. (not sure)
DDx
- Post viral arthritis (reactive arthritis)
- Septic arthritis
- Osteomyelitis
- Trauma
- Non accidental injury
- Buckle injury
- Greenstick frx

Hx
- Can you tell me more?
- Since when?
- Is he in pain at the moment? If yes offer pain killers after asking
allergies
- Trauma? Did he hurt any other part of his body?
- SOCRATES1 (Site, Onset, Character, Radiation, Associated symptoms,
Timing, Exacerbating and relieving factors, severity, 1 st time?)
- Mechanism of injury (can you show me how he fell?)
- Any numbness? Any pins and needles?
- Can he move it?
- Was there any bleeding? Swelling? Bruising?
- Redness or change in color of skin? Skin warmth?
- Any deformity?
- DDx questions:
Any previous infection? Flu like symptoms?
Fever? Rash?
Social Hx in detail, and Hx of previous similar episodes
- Well baby questions
- BINDSMA
- PMHx, PSHx, Family Hx

PEFE
- General Appearance
- General Physical Examination (PICCLED, Rash)
- Vital Signs
- Growth Charts
- Full Systemic Examination, focusing on wrist joint (look, feel, move,
neurovascular assessment, compare both sides!!)Because its acute injury.

Ix
- Ask for wrist x-ray, 2 views
Dx
- Buckle Injury (or fracture), it is a small area of compressed bone,
 due to trauma and elasticity of children’s bones fracture type:
Buckle injury: Compression injury failure of bone resulting in the cortex bulging outwards
(unilateral or bilateral). Also known as a torus injury. Although there is a disruption to the

- Very stable injury, no need to worry, they heal quickly


- Tender, slightly swollen wrist, and pain on movement
- Not uncommon

Mx
- Backslab (below elbow) or removable wrist splint (which can be
removed when showering) for 3 weeks
- After removal of backslab or splint, wrist movement may be a little
stiff or sore at first
- Arm sling is optional for comfort
- Simple analgesia (paracetamol)
- No need for physiotherapy or frx clinic or follow up with repeat
x-ray
- Avoid skateboarding, contact sports, and rough play for 6 weeks
- Red flags: severe pain, swelling is increasing, constant pain or
swelling after removal of cast or splint, change in color
of fingers, refuses to move fingers or hand after
removal of cast or splint
- Reading material
- Review
253-Acute Cholecystitis
Sample case
Your next patient is a 45-year-old female complaining of colicky abdominal pain since last night
after eating fried chicken.
Your tasks are to:
-Take a relevant history
-Perform abdominal examination
-Appropriate investigations and results will be given to you
-Diagnosis and management

Cholecystitis
-Inflammation of gall bladder
-Most common cause : GALL STONE ( cholelithiasis )
-Causative organisms :
E. coli
Klebsiella sp.
-Risk factors:
Age :
Female > Male
Pregnancy
obesity
-5F
Fat
Fertile
Forty
Female
Precipitated by Fatty food

Clinical feature
History :
-Steady pain and tenderness
-Localised to right hypochondrium / epigastrium
-Can be referred to right scapula ( inferior tip )
-Nausea and vomiting
-Aggravated by deep inspiration
-History of ingestion of fatty food
-Common in the elderly

Examination
-Localised tenderness over GB ( Murphy’s sign )
-Guarding +
-Rebound tenderness
-Less common: Palpable gall bladder, Jaundice, Fever
Investigation
-Ultrasound
pericholecystic fluid,
>4 mm gallbladder wall thickening

Treatment
-Bed rest
-IV fluids
-NPO
-Analgesia
-Antibiotics in case of sepsis: Amoxicillin 1g + gentamycin 2-6mg/kg IV Change to oral when
afebrile
-Cholecystectomy

Biliary Colic
History
1-is my patient hemodynamically stable

2-pain questions
-severity, painkiller, allergy
-onset, constant or come and go, getting worse
-site and radiation
-character
-anything makes it better or worse
-1st episode or not

3-Associated symptoms
-nausea/ vomiting
-fever
-jaundice
any discolouration of your skin or eyes?
any change in colour of urine?
are your stools getting pale?

4-Differential diagnosis questions


-have you had heavy meal or fatty meal recently before the pain started? (biliary colic/
cholecystitis)
-do you have any itching? (cholangitis)
-have you travelled recently? (hepatitis)
-are you sexually active? Do you have a stable partner? Any history of unprotected sex?
(hepatitis)
-chest pain, SOB, palpitation? (MI, pneumonia)
-PMH (gall stone or peptic ulcer), medication, alcohol, NSAIDS (pancreatitis, peptic ulcer)
-family history of stone
PEFE
1-GA (pallor, jaundice, dehydration, BMI)

2-Vital signs

3-quick chest and heart

4-Abdomen
-Inspection (distension, mass, moving with respiration)
-palpation (superficial and deep tenderness, rebound tenderness, guarding, rigidity, murphy and
mcburney signs)
-auscultation (bowel sounds)
-hernia orifices, DRE

5-Leg

6-Office tests (urine dipstick, BSL, ECG)

Counselling
Explain differential diagnosis and investigations
From the history, there could be a number of reasons for your abdominal pain: gallstones,
hepatitis, stones in biliary tree, pancreatitis. Others but less likely stomach ulcer, heart or lung
diseases

I would like to run some investigations including USD of the abdomen, FBE, CRP, LFTs, urine MCS,
and hepatitis serology, lipase.

Explain about gall stones


From the history, what you have are gallstones. These are small hard stones, probably composed
of cholesterol, that develop in the gallbladder.
It is a common condition and these are the ones that form in the tummy.
Gallbladder normally collects bile which is secreted from the liver. Bile is important for digestion
of fats. Sometimes, bile gets precipitated in the gallbladder and leads to the formation of stones.

Explain about complications of gallstones


At this stage, your gallbladder is not inflamed, so we can wait and see. However, it is important
to get the gallbladder removed with the stones because it can cause complications.
It can lead to inflammation of the gallbladder wall called cholecystitis.
The stones can dislodge and cause obstruction causing jaundice or inflammation of the bile
ducts called cholangitis which is a potentially serious condition.
It can also lead to infection of the pancreas causing pancreatitis.

Surgery
The surgery can be laparoscopic or open surgery.
Most likely it will done laparoscopically which is a keyhole surgery. You need to stay in the
hospital for 1-2 days and full recover will occur in a weeks’ time.

It is usually a safe surgery, but it can lead to damage to nearby structures, bile leakage, infection
and abscess formation.
Case (13/10/2017)
The middle aged woman in ED has upper abdomen pain.
Tasks: Hx, PEFE, Ix , Dx ddx
The patient did not need painkiller.
Pain started 2 hours ago now is better. On RUQ it is going to back. Not relieved when leaning forward
or not related to bowel motion. It is started after eating fatty food.
No sob no cp no dp no palpitation.
She had these symptoms years ago as well relieved by itself.
No waterbrush or bitter taste at the back of the tongue, no heartburn was present. No N/V/D.
No PMH, No FH of cancer in digestive system.
PEFE:
GA: no pallor no jaundice no Dehydration. BMI NL.
Focusing on abd: inspection NL, Palpation only tenderness on RUQ no rebound or rigidity. No
organomegaly No morphie's sign positive. Inguinal orifice NL Auscultation NL. No percussion due to
pain.
DRE not available.
Lung and heart are NL.
Ix: U/s, FBE, U&E, CT, LFT ECG CXR, I don't ask for amylase lipase as at the moment I am not
suspecting pancreas dis
Most likely you have condition called gall bladder stone. Drew pic. Still it can be due to heart, lung,
pancreas, liver, stomach so that is why we do ix. But this condition is due to stone formation due to
fat deposition and accumulation and formation of stone there causing obstruction of neck of the gall
bladder as well as causing dilatation of gall bladder and due pain. Are you with me?...Bell rang.
Feedback: Abdominal pain, PASS(G.S:4)
Key steps: 1.2.3 & 4 :Yes Hx:4 Choice of Ix:4 Dx/DDx:4

Case (27/4/2017)
Acute Cholecystitis ( Murphy’s S/ + )
45 yr woman c/o rt upper abd pain for 2 days.
Task
H/o,
ask pe
explain the possible causes to pt.
Introduced myself and greet to the pt. Ask her whether she wants pain killer or not. She said its ok,
she had taken Panadol.
History
Ask routine pain Q. General health, past medical or surgical history-unremarkable.
SADMA-not significant. Then, I go directly to question like any recent heavy meal or drink.
- pain around the rt mid area+,
- travel around the back,
- took some panadol for pain,
- first time,
- last night she had malaysian meals cheesy+,
- no alcohol drinking,
- no jaundice or fever,
- poo and pee are nl.
- No chest pain or palpitation.
OE findings
- all normal, examiner give a card.)

I explained her about the most likely cause could be pancreatitis. I drew a diagram for pt and explain
the causes of pancreatitis. I also explained about other differential with reasons like cholecystitis, gall
stone, hepatitis, MI, pleuritis, injury, things like that.
There was still time left, so explained a bit of mx like investigation and medication for pt like pain
killer and reassure the pt.
Still time left, pt asked me what are you going to do, I explained about more mx like admission and
assessment.
Reassure the pt. I noticed that after explaining the differential diagnosis to pt, examiner just fold his
mark sheets and give less attention to the exam. Time up and thanks the pt.

FB-PASSED, OVERALL-4, HISTORY-5, DDX-3, CHOICE OF IVX-4.

Case (5/10/2017)
Station 16 Abdominal Pain
Pass
Middle aged woman comes with severe Abdominal Pain in upper part. T 38'C P 103 other vitals are
normal.

Task: Relevant physical examination

Examination findings to examiner

Dx and DDx to Patient

Introduced myself, asked pt whether she has pain or not now, she said she got pain meds and she is
in little bit of pain but it’s ok to proceed the examination. Did Abdominal examination and only
positive findings was tenderness in RHC region and Murphy’s sign positive. Then, informed the
examiner the findings and explained to patient Dx and DDX with diagram. Dx – Cholecystitis, DDx-
Cholangitis, Hepatitis due to viral ( B,C) or bacterial, Gastritis, Gastric ulcer, Pancreatitis. Explained Dx
and related her symptoms with Dx and DDx with reasons.
Covered 3 key steps out of 4
Choice and Technique of examination 6 Accuracy 6 Dx and DDx 7 Global score 7
Feedback 21-4-2018
Acute abdomen examination- long stem, long term moderate drinker, pain in the upper abdomen
and radiated to back.
Perform relevant examination. Diagnosis. Differentials.
Station 7 abdominal pain
I ask if pt needed pain killer, pt answered no
I ask examiner if haemodynamically stable, answered stable
I started with checking sign of dehydration, then went to abdo Inspection palpation and
auscultation, during palpation, severe tender epigastric area, Murphy sign positive, did not do
percussion, told examiner that I did not percuss because of pain. After auscultation, I reported BS
present not pitched, examiner did not say anything, some other candidates told examiner said BS
absent. I mentioned with pt consent, I will check scrotum and hernia orifices, examiner happy and
said all normal. When I started to tell PR exam, examiner stopped me and say normal or no need
I can’t remember.
Note: in this case, actually we were required to give findings/commentary to pt not examiner.
I forgot to check jaundice and pallor…..
Dx: I said pancreatitis, giving reasons
DDx: Acute cholecystitis, acute cholangitis, peptic ulcer, renal stone which are less likely
Choice and technique of exam, organisation and sequence: 3
Accuracy of examination: 4
Dx/DDx:4
Global Score: 4 pass
254-Ovarian Cyst Torsion
Case (13/10/2017)
A young woman has come to ED due to sever abd pain. PT was weakly positive and LMP was 8 wk
ago.
Task:
Px,
Most likely dx with ddx to the pt with reasons.

Inside, there were a clock and a sphygmomanometer on the table. Washed my hand introduced
myself. I started assessing V.S. Pulses as soon as I was checking examiner said it is NL. (Asian
examiner, he was checking the technique from close distance and was telling the result like he was
whispering!:)) Bp was NL as well I said no sunken eyes no dry mucosa no pallor.

Then I started abdominal exam by inspection( no dilated veins no scar no bruises no mass obviously) I
said first I check auscultation to prevent inadvertent changes(NL bowl sound). In palpation, started
from left side and went to right side just tenderness on RLQ no rebound no guarding. I said because
you have pain I won't do percussion and deep palpation to check organ enlargement.

Then I did Morphie sign(-) Obturator and psoas and Rovsing sign(all NL)Renal angel tenderness(-) I
said dipstick and bsl(not available)

Then I asked for DRE and PV with consent of the patient and presence of the chaperon.

Examiner gave me the card which was saying right adnexal mass which was tender to touch but no
CMT and size of uterine was enlarged.

To patient, I mentioned: Regarding the hx and px most likely you have condition called ovarian cyst
have you heard of it? (drew pic) it is fluid –filled sac formation in ovaries but it can be EP which fetus
and product of pregnancy can be formed in tube( fallopian tubes) instead of womb which is serious
condition.

It can be also a kidney stone or appendicitis which is a swelling at the end of your small bowl. Also it
can be mittelschmerz which is an inflammation in small bowel however it is unlikely due to your age
of pregnancy as it is happening in young girl in the middle of menstrual cycle.

We need to do U/S to confirm but most likely it is due to the ovarian cyst. Bell rang.

Feedback: Substance abuse, PASS(G.S:6)


Key steps:1,2,3 &4: Yes
Approach to patient:6
Choice and technique of examination and organization and sequence:6,
Accuracy of examination:5
Dx/DDx:5
comment
Task was Do physical examination
Diagnosis and management
UPT weak positive on stem mentioned
Severe pain at presentation
So pain killer offer
Then do general examination:
BP and Pulse to be measured on your own ..
complete abdominal examination
She had tender right iliac fossa
No rebound tenderness / no Mc burny point
No ascites
On pelvic examination : got a card ( adenexal tenderness)

Task : explain to patient about ectopic pregnancy .. patient crying as it was a planned pregnancy
Explained her urgent USG / about urgency of surgery and management with specialist
Time over
In afternoon group : patient had no pain .. she was okay .. rest similar case
Waiting for feedback to be sure

did u pass that pass? A detailed recall for 7 july with same scenario. Candidate diagnosed as ectopic
most probable diagnosis and gave other dds. Got only 2 score in diagnosis and failed that case

I failed that case too.. reason I didn't have time to mention DD and their invx and mx plan for Dd
So nutshell .. always mention DD to discuss along with the most likely situation
Show that you r safe and ready to take help from seniors

Case (9/3/2017)
female patient with pain in right iliac fossa for one week ,now got worse ,, her pregnancy test is
vaguely positive and 7 weeks amenorrhea.
Task
1)physical examination
2)tel most likely diagnosis
positive findings tenderness right iliac fossa uterus enlarged and retroverted , adnexa tender ..
(CERVICAL EXCITATION /CMT IS NEGATIVE)
Feedback 5-7-2018
Young lady with missed period, lower abdominal pain. PT weakly positive.no bleeding
Task:
1- do PE
2- tell DDx
I did general, vitals examiner stopped me at blood pressure, abdominal exam (inspection, palpation,
bowel sound) rovsing sign negative. Asked pelvic exam.
DDX.. ectopic pregnancy, ovarian cyst or rupture or tortion, appendicitis, ureteric colic.
Passed: global score 4

Feedback 5-7-2018
25 year old girl, has been having RIF pain for 6 days, UPT was weakly positive 1 week ago, no PV
bleeding
Tasks
Relevant history
Focused abdominal examination
Explain the d/d to the patient
offered pain killer to the patient and asked for allergy.
Pain for 6 days, no shifting, no PV bleeding. no urinary sx, bowel normal. No vomiting but felt
nauseous. Bowels, urine all normal .
Examination (if you wear hijab, then be prepared to use you stethoscope)
general (as you see) .. vitals (I have to check them) so I did pulse and then blood pressure (examiner
stopped me when I starting inflating the cuff). on abdominal examination , there was RIF tenderness,
no guarding, Rovsing or psoas sign negative. Bowel sounds normal
Asked Pelvic examination and DRE, she gave me paper. > os closed, pain on right adnexa. Uterine
size is slightly enlarged.
d.d like pregnancy with ovarian cyst, ovarian/cyst torsion .. twin pregnancy (one in womb and other
ectopic). Ectopic pregnancy. also pregnancy with appendicitis / muscle spasm / kidney problem.

Feedback 5-4-2018
abdominal examination
GP, 27 years old lady c/o pain in RIF, n & v present these days, home pregnancy test weakly positive.
Perform PE, Dx, Ddx
It took too much time for me to measure BP, only positive findings are tenderness in RIF, no RT, all
special tests negative, when it comes to VE, uterus slightly enlarged, no
Cervial excitation pain, but right adnexal tenderness +, no bulging
I told complicated ovarian cyst but didn’t have a chance to say Ddx again

It was my case: Acute abd Amenorrhe 6 weeks


Ut size was 6 weeks CMT -ve
Adenixial mass in rt side with pain Preg +ve
D.D $Preg with ovarian cyst $Ectpoic $Ap
Feedback 7-7-2017
20 year old presented with acute abdominal pain. bhcg was done and is weakly positive.
TASK
1. perform focused abdominal examination explaing to the patient what you are doing
2. Tell the patient your differential diagnosis

Hello examiner, hello Jane


I have been asked to examine your tummy that would involve looking at it, touching it..if you have
pain at anytime, let me know and i will stop...can you please lift up your shirt while I wash my hands
Im going to look at your tummy now
On inspection no distension, abdomen moving with respiartion, no scar marks
Im going to touch your tummy..tell me where does it hurt the most....she pointed to rif...im sorry for
my cold hands
did superficial palpation...

PATIENT KEPT ON SAYING YOUR HANDS ARE VERY COLD AND I KEPT ON APOLOGIZING
THE STEM SAID ACUTE ABDOMEN...PATIENT SEEMED AS IF NOTHING WAS WRONG WITH HER,
WHICH WAS A BIT CONFUSING
on deep tenderness in rif...no rebound, rovsings negative
checked renal angle tenderness...negative
checked liver and spleen.
wasnt sure if i should do percussion....because it was acute abdomen and the patient kept on saying
your hands are very cold
checked bowel spunds...present
Covered her up and TOLD THE PATIENT NOW I WANT TO EXAMINE YOUR PELVIC AREA.
examiner came and said no need to do that we have already done for you and gave me a card
cmt negative
right adnexa tender
Jane there are some reasons for your pain
Firstly ectopic pregnancy...pregnancy outside the womb in your tube
Secondly, twisting of your ovary
Thirdly, rupture of a cyst inside your ovary.
I have also ruled out appendicitis and any infection of your kidney
But to be sure we need to do an usg
Do you have any questions...no
Thank you examiner, thank you Jane
FAIL
keystep 1no 2yes 3yes 4yes
approach 4
choice of examination 4
accuracy of examination 4
diagnosis 2
255-Failure to thrive+ Fussy eater
You are in GP when 18 months old baby boy brought in by 19 years old single mum complaining
of child is not eating well.
Growth chart shows weight has dropped from 50th to 3rd
basic Ix are within normal except mild hypochromic microcystic anemia
Tasks
-further relevant Hx
-Give your Dx and Mx to mum.

History
1-could you describe what you mean by not eating well?

2-can you describe his typical daily diet for me? (not interested in food like cow milk and fruit
juice)
how much milk does he intake? what type?
have you introduced solids? Does he eat meat?

3-Has he had any vomiting? Diarrhea? Stool (mucous, blood, consistency


any recurrent infections since birth?
how about the number of nappies? Any reduced wet nappies?

4-BINDS
Birthplanned or not
Normal vaginal or CS
preterm or full term
how many days did you stay in hospital
did he have any screening tests imediately after birth
Immunisation
Nutrition and development: is he thriving and growing well
social How is his sleep
who do you live with? Any partner support? Family or friends support? (No support)
what do you do for living? What do you study? (No job)
do you have any financial support? Job?

5-SADMA

6-Family history of celiac disease or cystic fibrosis?

Workup
1-FBE, ESR, EUC, LFT
2-Iron studies
3-Urine MCS, stool MCS
4-stool for fatty lobules and fatty acid crystals
5-TFT
7-BSL
Diagnosis and management
-the good news is that John does not likely to have any organic medical problems.

-from the grwth chart we usualy expect the lines go parallel but in your child it is dropping down
which means poor growth. From the history it seems like the quantity and quality of food
provided are insufficent. (tell about positive findings of diet from history)

-Also from the history it seems like you have some problems with social life which we can
manage together.

-I can organise a social worker to help you with job and I will give numbers of support groups for
single mum to help you handling john’s problem.

-I can arrange a meeting with your family if you are happy.

- it is important to feed him a balanced healthy diet, Iron fortified cereals will help correct his
anemia which is likley because of dietart issues. I will give you a written materials about food.

-I will give you referral to a dietician who will help you making a proper dietary plan.

-Arrange Ix to rule out anemia.

Case (13/10/2017)
A 3 y/o boy was brought by mother Bredith because she was worry that her child is not eating
properly and there were a growth chart of the baby for the length and the weight separately.
The former was on 30% and the latter was on 10% after falling from 50% over 2 consecutive
months.
Tasks: take Hx, Explain the condition and tell the most likely diagnosis.
I was so stressed in this case and really was confused about the tasks so please ask other people
about it. Surprise I passed this case which I thought I will fail!!contrary to ear examination case!
I asked about the N/V/D was not there not hard to flush not sticking to pants. No fever no cough
SOB. Waterworks was fine. Mother said she divorced 6 months back and she is so alone and her
child is not eating well as he refuses to eat while she offer the foods. I asked whether she has
anybody to help her she said no. The role player was not happy at all.
I mentioned the most likely his condition is due to being fuzzy eater which is important this time
to be investigated as he is not gaining weight then I explained the growth chart but she was not
happy yet. I said I need to do some investigation to rule out other possibilities like fever diarrhea
celiac and so on. (I should have said this is FTT and ruled out child abuse)

Feedback: FTT, PASS(G.S:5)


Key steps:1,3 and 4 Yes…2:No
Approach to patient:4
Choice and technique of examination and organization and sequence:4
Dx/DDx:5
Feedback 21-6-2018
Your next patient is an 18 month old boy that his nurse has been concerned about the change in
his growth rate. ( the growth chart was provided. It had become a plateau. His height was not
affected.
Tasks
Take history
counsel the mother.

(FTT probably fussy eater , sad frustrated mum Single mum. hx, ddx, counsel) Failure to
Thrive:/Pass with global score 4
I entered the room and introduced myself and asked the mu how I could help her. Mum did not
make a good eye contact with me and seemed unconcerned or maybe slightly offended.
(obviously she was not happy and did not know exactly why the nurse had sent her)
I started asking about the child’s overall health condition, growth and development, nutrition,
eating habits, whether he had other siblings, the home situation, the child’s communications
social development. Any other concern in the mother about his health.
Overall, mum admitted that the child never seemed to be hungry and was not interested in
food and was a fussy eater.
Then I asked about home situation, (she was a single mum) I asked about any possible financial
difficulty, and how she coped with being a single mum, about depression in her (her mood and
sleep), asked her whether she worked or not and whether the child went to childcare or not, and
if there was any help available for her. Also I asked mum if she and to have any argument with
the child for having food, and asked how she usually reacts to the child’s behaviour about not
being interested in food.

Finally, I explained that as she looks frustrated I would like to involve a psychologist as they can
give good advice or suggestions about how new first- time mothers can deal with toddlers in
different situations, and also she can talk to the psychologist about any other problem or
concern she might have.
As well I suggested involving a social worker, despite she was on pension, maybe a social worker
could make a difference.
As well, I explained to her that his symptoms might be due to Iron deficiency anaemia, which
can be very common in his age group, considering that he is not taking any supplements. And as
Iron deficiency can cause loss of appetite, it can be simply due to iron deficiency anaemia. So I
would recommend taking a blood sample for investigating possible anaemia, and also
prescribing a supplement which may improve his appetite, and also asking help from a
psychologist to give some advice about how to deal with a fussy eater toddler.
And I would like to keep them in my follow up to see how they progress. I also said that being a
single mum can be a very demanding job and I am sure she is doing her best, and problems and
difficulties can always happen, and she can always count on us for any possible help she might
need.
Feedback 15-3-2018
GP, going to see a patient 1yr 6 mths brought in by mum. Mum no concern. Local nurse worried
abt weight gain. Wt chart and height chart given. Height is good. Wt grows well till 6 mths,
become stuck from 6 mths and now below 3rd centile.
Tasks: history, explain the condition

Approach: FTT cluster. A) decreased or faulty intake — fussy eater, too much cow milk are AMC
cases b) decreased absorption— cealiac disease is amc case c) increased utilisation- hyperthyroid
and DM — not AMC cases d) psychosocial issues— single mother this is a AMC case.

Yes this is a recall if I am not mistaken . This belongs to the FTT cluster so ask all the questions in
this cluster like intake issues diet cows milk+ve . Fruit juices , fussy eater, +ve decreased
absorption issues so go in detail about bowel habits, consistency , blood mucus in stools etc. take
BINDS for all peads cases ask are you a happy family and role player will tell you everything even
without asking any further.no significant past history or family history. Single mother no job no
support. When explaining I said generally when we plot the chart we expect that the line will
be parallel to the rest of the growth lines that you see but for your child it is dropping down.
Mentioned all the reasons Mx was not in task but even if it was I would not have reported to CPS
instead the mother required lots of support. Yes this is a recall if I am not mistaken . This belongs
to the FTT cluster so ask all the questions in this cluster like intake issues diet cows milk+ve .
Fruit juices , fussy eater, +ve decreased absorption issues so go in detail about bowel habits,
consistency , blood mucus in stools etc. take BINDS for all peads cases ask are you a happy family
and role player will tell you everything even without asking any further.no significant past history
or family history. Single mother no job no support. When explaining I said generally when we
plot the chart we expect that the line will be parallel to the rest of the growth lines that you see
but for your child it is dropping down. Mentioned all the reasons Mx was not in task but even if it
was I would not have reported to CPS instead the mother required lots of support. Mentioned
fussy eater , too much cow milk intake and psychosocial. Issues for FTT

256-Fussy Eater
Case (10-05-2018)
10 months old child fussy eater+ normal growth chart+ happy family

History
1-can you describe his typical daily diet for me? (yogurt & breast feeding)
how much milk? What type?
have you introduced solids? Type?
have you tried giving him a variety of food?

2-Does he eat at the table with the fa,ily?

3-Symptoms questions
Nausea, vomiting, diarrhea, tummy pain, fever, rash, lumps and bumps?
dehydration questions

4-BINDSMA
Birthplanned or not
Normal vaginal or CS
preterm or full term
complications during pregnancy
weight at birth
did he have any screening tests imediately after birth
Immunisation
Nutrition and development: is he thriving and growing well (not sure)
have you kept a record of his weight checks?
social contact hx, travel
who do you live with? Any partner support? Family or friends support?
what do you do for living? (happy family)
do you have any financial support? Job?
Medications and allergies

5-SAD

6-Family history of celiac disease or cystic fibrosis?

PEFE (All normal)

*when you ask about growth chart not sure in history or examination and according to feedback
the mother will give you the charts which are normal.
Diagnosis
-from history and examination your child is not likely having any organic problems. He looks
physically fine to me.
-Also his growth chart is within the normal ranges.
-It looks like he is fussy about his eating which means he does not like the shape, colour or
texture of particular foods. It is normal for children to like something one day but dislike it the
next day and to refuse new foods.

-This all happens because fussy eating is part of children’s development, its also because their
appetite goes up and down depending on how much they are growing and how active they are.
Children are likely to get less fussy as they get older.

-I can refer you to a dietician to help. I do also have some suggestions:


*try to offer a variety of food at different times throughout the day.
*please do not force him or threaten him or bribe him into eating.
*keep trying if they reject a new food initially. You may need to offer it 3-4 times in the
beginning.
*have a regular routine to eat meals together with the family at the table
*encourage the child to feed himself and to help you prepare his food.

-lastly as long as your child is putting on weight, try not to stress yourself about his health and
weight.

-Follow ups and reading materials

-Red flags

Feedback 10-5-2018
sorry for bothering again and again . how did you approach to the case of fuzzy eater?
Fuzzy eater question was mother coming with 10months baby as he is not eating anything. So
she is worried.
I approached the case as a mix of fuzzy eater and ruled out malnutrition. Asked abt all ddx like
the malnutrition case in Karen’s.
Growth chart was normal. Child was thriving well more than 75th percentile. I don’t remember
the exact. Baby was eating only yogurt and was breast feeding. When I asked abt any weight
loss she said no I don’t think he is losing any weight. Then i asked have u kept any record for it
and then the mother gave me the growth charts
Everything was normal... baby was breast feeding well, and was eating yogurt... mom was giving
different types of food but baby use to taste and leave it..
No I dont think so... don’t really remember exactly but I remember talking of giving a trial period
for few months and then a brief things to do like keep offering different foods. All children r
different so some take time to start eating But it will happen slowly but u have to keep trying.
And told tht I am not worried as baby is thriving well and there is no organic cause so just keep
trying. And gave red flags
257-Adenocarcinoma anal verge
37 years old lady comes to your GP clinic for biopsy report. Previously, she c/o abdominal pain
( don’t remember other symptoms ) I think she had family h/o bowel cancer. So Sigmoidoscopy
was done and took biopsy from the growth which is 10cm from anal verge showing
adenocarcinoma.
Tasks :
Explain Dx
Further investigation with reasons.
Management plan ( details not required )

Breaking bad news

I got the result of tissue sample or biopsy with me

-do you know why biopsy has been done for you?

-has anyone told you about you medical situation so far?

-how much do you want to know about the result?

-The news I have got for you unfortunately is not good. The sample of tissue that has been taken
from the lining of large bowl using sigmoidoscopy shown to be a nasty growth or cancer.

-crying tissue& water+ pause

-would you like to call someone for you?

-would you like to continue with this consultation?

-this is not what you wanted to hear and I also wish the news was much better.

-I can see it is upsetting news for you and also very difficult situation to face.

-we will be working as a team with you, supporting and monitoring you along with your family to
take you through this difficult situation

-are you ready to discuss the treatment plan? Or I can arrange another appointment with you
ASAP?
Counselling
-it is the most common cancer of GIT, mostly occurs after 50 years age. It is common in those
having family history of colon cancer.

-you will be managed by MDT including oncologist or cancer specialist, surgeon, radiologist,
specialist nurse, social worker, psychologist, support group and GP.

-I will refer you to oncologist.

-Further Investigation needed to be done to look if it has spread to any other areas which we are
not suspecting at the moment but we need to look for.
FBC, Blood group, EUC, LFT, coagulation profile
CXR chest
CT scan of chest + abd+ pelvis
Bone scan

-the treatment will be decided by the specialist depending on the imaging and blood tests
(staging)
they might do surgery +/- radio/ chemotherapy to destroy any remaining cancer cells and
improve survival rate and decrease the recurrence.

-you will also need after treatment to be follow up and monitored regularly by specialist and me
as your GP.
Follow up Ix:
Blood tests (CBC, EUC, LFT every 3 months)
CEA every 6 months (treatment response)
FOBT regularly
colonoscopy yearly-2 years

-you will be supported all the way, there are lots of support groups, financial supports and
emotional support, psychologist.
I can arrange family meeting with your consent to support you.

-feel free to call me at any time for further questions.

-we can arrange another consultation if you need to discuss about the treatment again.

-offer screening to family.


Feedback 20-9-2018 adenocarcinoma anal verge bad news Abdominal pain (PASS)
37 years old lady comes to your GP clinic for biopsy report. Previously, she c/o abdominal pain
( don’t remember other symptoms ) I think she had family h/o bowel cancer. So Sigmoidoscopy
was done and took biopsy from the growth which is 10cm from anal verge showing
adenocarcinoma.
Tasks :
Explain Dx
Further investigation with reasons.
Management plan ( details not required )
Approach :
I greeted patient. Asked her whether she comes here alone? How would she expect the result?
Dx :
I told her that the result came back and shows that you have bowel cancer. Then, asked her to
take her time, offered tissue and water. Patient was a good actor. She was teary and surprised
but she kept talking about how she feels right now instead of being silent. I reassured her.
After some time, I explained her about the condition with a drawing. Then told her that we need
to do further inx to look for any spread.
Inx :
Blood tests – tumor markers
CXR – because it can spread to lungs
CT chest, abdomen – because it can spread to lungs & organs inside tummy
Bone scan for similar reason
I was hesitated to do colonoscopy, then I didn’t tell about it.
( it was awkward to tell the reasons according to task because patient seemed stressed when I
tell about the spread, so needed to reassure her whenever possible. )
Management :
We’ll treat you with team approach like….
Main option is Surgery but other options like RT and Chemo are available which depends on
staging and
decision of specialist.
I explained about surgery with drawing, and temporary colostomy, and their purpose, good
points, bad
points, and stoma nurses.
Reassured her and asked her is there any Q? she said how can I think of Q in this situation.
Then, I kept reassuring and highlighted some important points in management.
Case (8/5/2017) (8/6/2017)
Breaking bad news, moderately differentiated adenocarcinoma in the large bowel 4 cm.

Case (31/8/2017)
U r in surgical unit,lady come back for bx results for colonoscopy, Bx-4cm ulcerating growth 15cm from
anal verge.(adenoca. Counsel)
Task
-explain results
- Tell her what investigations will u do next
- Explain further Mx (no need to tell in details) exact words wiritten in exam
258-COPD bad lung
There are 2 cases
both same stem of an old woman admitted to ED because of SOB and several Ix done, given mist then
discharged. On discharged one of the nurses told her she had bad lung. Nos she came to your GP to talk
about the results
so your tasks are:
-to take history (here you should ask about SOB, rule out all ddx and general questions are the critical one
which you need to ask).
-Then you will tell her about COPD and other ddx which you will say less likely.
-Then you need to tell what Ix you want to do for her and the examiner will give you paper to interpret the
result to patient (here the most important to ask about spirometry and chest x-ray) so in case 1 they will
give you x-ray to interpret, while spiro not available. in case 2 spirometry to interpret quickly while x-ray
not available.

Case 1 feedback (hx, dx and ddx, chest x-ray)


Recall 9-5-2018
64yr old recently discharged from hospital, for sob, nurse said.. U got "bad lungs"... Steam pointing
towards COPD,
tasks -hx
-Dx and ddx
-explain pt, investigations u would order n upon correct ordering.. U will be provided with one to explain
later.

Feedback 9-5-2018
Copd- 62 year old female patient recently admitted in the emergency dept due to sob and been
discharged. H/o sob on walking a flight of stairs and on exercise. Was prescribed ventolin in the ed. Past
h/o bronchitis. Smoker from 20 years of age. Father had heart and lung problems. Recent admission and
Dr told she had bad lungs.
Task. History . Probable diagnosis. Differentials. Investigations from the examiner. Examiner gave an X-ray
of chest which showed hyperinflated lungs.
Bad lung case, recent admission, Dr told pt she had bad lungs. After pt was discharged, she came to your
GP clinic
In Hx, I asked SIQRAA qx for SOB, and DDx qx for PE, Angina/heart failure, chest trauma, exercise
tolerance, PMHx, SADMA
Dx: COPD, explain the condition No PE required
Ix: Spirometry and CXR; CXR film given to me only when I asked examiner for it. No spirometry results.
I could not finish explaining CXR but still got a score 5
Approach to pt: 5 Hx: 4 Choice and technique of exam, organisation and sequence: 5
Dx/DDx:5 Interpretation of Ix: 5 Global Score: 5 pass

Recall 21-4-2018
62 year old female patient recently admitted in the emergency dept due to sob and been discharged. On
discharged nurse said ‘’ You have bad lungs’’. Echo and ECG normal.
H/o sob on walking a flight of stairs and on exercise. Was given mist in the ed. Past h/o bronchitis. Smoker
from 20 years of age. Father had heart and lung problems.
Task.
-History
-Probable diagnosis. Differentials.
-Investigations from the examiner. (Examiner gave an X-ray of chest which showed hyperinflated lungs.)
Feedback 21-4-2018
62 year old female patient recently admitted in the emergency dept due to SOB and been discharged. On
discharged nurse said ‘’ You have bad lungs’’. Echo and ECG normal.
Task
History - given some mist in the ED, no ICU admissions, previous bronchitis and a active smoker, chronic
non productive cough

Probable diagnosis. Differentials – COPD / BA unlikely with chronic smoking / Pneumoniae unlikely due to
no fever / Cardiac causes unlikely due to normal Echo and ECG / DVT
Ask Investigations from the examiner - Examiner gave an X-ray of chest which showed hyperinflated lungs-
no other changes in the CXR / Asked for Blood tests and Spirometry ( Not available ).
Global score – 4 ( Approach 4,History 3,DDx 5,Investigations 6, Interpret Ix 4 )
My Conclusion - Easy straight forward task – (no spirometry ) / Tell DDs as much as you can & interpret
CXR correctly with AP and Lateral views.

Feedback 26-10-2018 Station 9: SOB Passed (GS – 4)

65 yo female smoker with recurrent chest infections. Comes because she was told had bad lungs.

Task: Hx, DDx, Interpretation of X rays and explain the possible causes to patient.

After introducing myself, I asked short history.

HX:

 SOB details during admission and now


 Risk factors (smoking, underlying lung or heart disease, risk of inhaling pollutant/ occupation)
 DDX (features of heart failure, pneumonia, COPD, Asthma, anxiety disorders)

Positive findings: (SOB she’s experiencing is most likely COPD, no fever currently, no personal and family Hx
of asthma, no features of HF, denied anxiety features and stress.)

DDx: I explained my DDx as above.

Ask Xrays from examiner: give me one film with features of COPD.

Then I explained COPD to patient, together with CXR. .


Approach for case 1
62 year old female patient recently admitted in the emergency dept due to sob and been
discharged. On discharged nurse said ‘’ You have bad lungs’’. Echo and ECG normal.
Now she come to GP to talk about the results
Tasks
-History
-Probable diagnosis. Differentials.
-Investigations from the examiner. (Examiner will give you specific Ix)
-Explain Ix to the patient and reason behind it

History
1 Approaching.
-How are you doing? How are you feeling at the moment? Is it ok for you to ask some questions
first?

2-SOB
-onset, duration, progression
-I can see you were admitted to the hospital.. what happened and what did they do? Has this
happened before? (Was given mist continuously)
-how did it start?
-aggravating and relieving
-Exercise tolerance
-SOB on lying down or wake up from the bed? (PND) +/- present
-any time of the day it is worse?
-any relation to season?

3-Associated symptoms
-cough, sputum
-chest pain.
-racing of the heart, swelling of leg
-fainting, lightheadedness, dizziness
-fever, recent viral infection
-LOW, LOA, lumps and bumps

4-General
-PMH (DM, HTN, heart, lung disease)
-PSH, trauma
-Fhx of lung problems, cancers
-SADMA (smoking +ve 10/day for 20 years)
-work mining? Asbestos?

Diagnosis
-from history most likely you have a condition called COPD which means obstruction or
narrowing of the airways of your lungs. This narrowing can be because of a number of factors
like smoking or exposure to dust for a long time.
DDX (less likely)
-the breathing problem can also be due to asthma but no hx towards it.
-Can be due to a collection of fluid or air between the coverings of the lungs.
-can also be due to infection of the lungs or nasty growth, collapse of lung or any clot obstructing
the lung vessels
-can be due to heart problems

Investigation
-asked for spirometry and Chest x-ray
-explain to patient FEV1/FVC, FVC, FEV1

This is your chest x-ray having front and side views. You can see your ribs, heart, back bone etc..
Here. Explain your lungs are hyperinflated with air they seems more black and muscle below
lung called diaphragm seems to be flat as well due to hyperinflation.
Then draw respiratory tree and explain narrowing of airways.
Case 2 feedback (Hx, dx and ddx, spirometry)
Feedback 2018
64yr old recently discharged from hospital, for sob, Drs said.. U got "bad lungs"... Steam pointing
towards COPD, task hx, explain pt, investigations u would order n upon correct ordering.. U will
be provided with one to explain later.
- there is a list of investigations on the question, when they listed these investigations, it help
you to eliminate some diseases as well as explain to patient. There is the CBC results and ECG
results. DD for SOB is heart, lung, anemia, psy but the question showed that there is no problem
with anemia and heart.
Inside the room: I take VS, ask her about SOB and explore it. She only has symptoms when she
walking. I ask about heart symptoms like swelling, loss of consiouness, chest pain. Then go with
anemia, then psy. After locate this is the lung issue, i ask about lung symptoms and risk factors
like smoking, contact, occupation, travel.
I finish history quickly then i told the patient that i did CBC, ECG and it all normal so i thing the
issue come from the lungs.
I will run investigation for her lung like X ray, Ct, spirometer and i ask examiner for that.
Examiner give me the spirometer which is show COPD. I explain spirometer and COPD for
patient and ask her about BMI as it may have mix between obstruction and restrict lung
disease. Bell rang.

Feedback 28-11-2017 Shortness of breath - pass


Middle age lady (64-65yrs). Was admitted to hospital a week back for acute breathlessness.
Nurse said she had “bad lungs”. Her ECG and Echo was normal. Now she has come to GP to talk
about the results.
Tasks
-Take relevant history.
-Explain differentials
-Ask for investigations and examiner will only give you specific investigation
-explain investigation to the patient and reason behind it.
(the patient said the acute management that was given in the hospital was some mists that they
gave continuously. She woke up from sleep because of the SOB. This is the first time happening.
Before that intermittently “wheeze” (pt used this word) happened. No allergy history was
positive. Smoking cigerrates 10 sticks per day for last 5 years, before that 10 per day.
Spirometry was given where FEV1, FVC and ratio were given, pre and post bronchodilator value,
increase was very less)
Key steps: 4/5 Approach to the pt: 6
History: 6 Dx and DDx: 5 Choice of inv: 4 Interpretation of inv: 5 Global score: 5- pass
Approach:
History: How she was doing? How she was feleing atm? Is it ok to ask few questions? If she has
nay problems I will manage accordingly.
SOB: onset, duration, progression.
I can see you were admitted to the hospital….what happeden and did they do?
How did it start?
Aggravating and relieving?
Amny sob while lying down/ wake up from bed because of sob (rule out paroxysmal nocturnal
dyspnoea)
Any time of the day its more? Any relation o any season?
Occupation?
Where was she stayng (near any mining area/industriest nearby ))
As child ever diagnosed with asthma or any chronic lunch problem? Any couch? Any sputum>
any eposides of turinign blue any chest pain? Any racing of the heart> any swelling of the
ankles> any episides of fainting and dizziness? Any history of fever> any recent history of travel /
any hx of loss of wt, lumps and bumps? Recent hx of op (pulm embolism/atelectasis) any hx of
pain or swelling in legs( dvt) , hx of easy fatigability ? any trauma or injury to the chest?
Immobilization? (pulm embolism) Do you thing the sob is related to any pertucular events
happening in your life.?
SADMA (smoking was positive….smoker for a long time)
Past histry of dm, raised blood pressure, heart problem, lung problem.
Ever been hospitalized in the past.
Family history of lung problems/cancers.
Dd: from histry most like we call it copd.which means that the obstruction or narrowing of the
airways of your lung. This narrowing can be because of a number of factors like smoking and
exposure to dust for a long duration.
The breathing problem can also be because of asthma, but no histry towards asthma. Can be
also because of collection of fluids/air between your lung coverings, can also be due to any
infection of the lungs. Can be because of any nasty growth, collapse of lung, any clot obstructing
the lung vessels.
Can also be due to heart problem, but from the notes the heart condition seems normal.

Investigation:
Asked for spirometry from examiner and he gave me a paper showing table. It was a clear case
of copd.
Then I explained to the patient what spirometry was . showed her all the values. The normal
ones and what she got. Explained to her why it is copd from her chart.
Explained FEV1 and FVC.
COPD (materials)
Lisa aged 51 years presents to your GP clinic complaining of breathlessness which has gradually increased
over the last 6 years. She thought it was due to age, but now seeks your help because her activities have
become limited. She's able to walk slowly for approximately 50 to 100 meters before becoming breathless,
but becomes breathless much sooner if she hurries or walks upstairs. She has been well throughout except
for an episode of pneumonia 20 years ago.

TASKS

1. Further focused history


2. Ask for physical examination findings from examiner
3. Explain your most likely diagnosis and management to the patient

APPROACH

Doctor to examiner: What is the BP, PR, RR, and Sats of my patient?
E: 120/70, 75PR, 17 RR, 89% spO2
D: I would like to transfer my patient to the treatment room and started on 2L O2 via nasal prongs.
D to patient: Hi Lisa, I'm Dr. Lea your GP today, what might I help you with today?
L: I've been having this breathlessness, it's been there for a while but it's getting worse for the last 6
months to a point where I get breathless while I'm brushing my teeth, or especially when I have to
hurry.
D: Anything that aggravates or relieves it?
L: Well I don't feel breathless when I'm at rest, but feel breathless when I'm exerting or in a hurry.
D: Have your activities been affected?
L: Yes, I easily get short of breath during exertion, so I can't do as much activity now as I did before.
D: Have you been diagnosed with any known lung problems before? (COPD)
L: No.
D: Have you noticed any weight loss, loss of appetite, lumps and bumps around the head and neck?
L: No.
D: What is your occupation? (Occupational Lung Diseases)
L: I just stay at home now.
D: What is your usual diet? Have you noticed any bleeding or bleeding in your urine or bowel? How is
your bowel movement?
L: A good mix of meat and vegetables. No bleeding, and I move my bowels regularly.
D: Do you feel more short of breath when you lie down? Do you wake up multiple times in the middle
of the night short of breath?
L: No.
D: How many pillows do you use in sleep? Do you use pillows to relieve your shortness of breath?
L: Just 2 pillows, but it's my usual number.
D: Have you noticed any swelling of your feet or generalized swelling of your body?
L: No.
D: Are you generally an anxious person? Any stress at home or at work?
L: No.
D: Do you smoke, drink alcohol, take recreational drugs?
L: Yes I do smoke, but no alcohol or drugs.
D: How long have you been smoking, and how many sticks a day on the average?
L: I've been smoking since I was 17 years old, about 2 packs a day.
D: I would like to arrange another session with you to talk about your smoking habits, alright?
D: Do you have any known medical or surgical problems in the past? Have you been admitted to a
hospital in the past? Have you ever had this issue in the past?
L: No.
D: Do you have any family history of known cancers, especially lung conditions?
L: No.
D: Thank you for those information Lisa. I would just like to talk to my examiner and will get back to
you shortly.
D (to examiner):
D: Is there pallor, cyanosis, clubbing, lymph node enlargements, edema? Any signs of respiratory
distress?
E: None.
D: What is the skin turgor, capillary refill, JVP?
E: All normal.
D: Is there sunken eyes, dryness of the mucous membranes?
E: None.
D: What is the BMI?
E: 19.
D: What is the BP, pulse, RR, temp, and sats of the patient?
E: 120/70, 75PR, 17 RR, 89% spO2, afebrile
D: Is there equal chest movement ? Equal chest expansion? What is the tactile vocal fremitus? What is
the percussion note over the lung areas? Is there good air entry? Any adventitious breath sounds?
E: Expiratory wheeze on both lung bases, good air entry,
D: What are the heart sounds? Any murmurs?
E: Normal S1 and S2, no murmurs.
D: I would like to do a spirometry, ECG, urine dipstick and blood sugar level.
E: UDT is normal. BSL is normal. ECG is pending. Spirometry: FEV1/FEV is <80%. Post bronchodilator
improvement is 9%.
D: Thank you for those information examiner, I would like to go back to my patient.
Condition: Most likely you've got chronic obstructive pulmonary disease. I'm sorry I'm using medical
jargon. Do you have any idea about this?
The lung has small airways. Since you've been smoking for such a long time, these airways have been
damaged, leading to narrowing of these small airways.
Cause: Causes include smoking, occupational exposure to irritants, and family history (like alpha 1
antitrypsin)
Commonality: It's not very uncommon.
Clinical features: It gives you progressive shortness of breath over a long time especially on exertion
Management: Management of this condition is by national management plan called COPDX. C is
confirm diagnosis by doing spirometry, chest x-ray and sputum microscopy culture and sensitivity. We
can do some basic bloods, FBE, UEC. O is optimize lung function. I will refer you to the respiratory
physician who will give you some puffers which will help you to improve your lung function. I will also
refer you to a chest physiotherapist who will teach you some breathing and coughing exercises to
improve you lung function. P is prevent deterioration. The only treatment known to prevent
deterioration is complete cessation of smoking. Number 2 is lifestyle modification. Avoid going to
areas where there is too much pollution, or if unavoidable, wear a mask. Adopt a healthy lifestyle. D is
develop support. There is COPD support groups where you can meet other people with COPD who
may have more insight on how to manage the condition. Quit-line smoking which is a 24-hour service
if you are having troubles with quitting. X is exacerbation. Call an ambulance if you become severely
short of breath. 1 puff 6 breaths, total of 6 puffs, wait for 6 minutes, repeat it until the ambulance
arrives.
I will review you regularly and I will give you more reading materials so you can have better insight
into the problem. Do you have any questions at this point?
L: No more doctor, thank you.
D: Thank you Lisa, I will see you next time.
---------
1. GREETING.
2. ASK if hemodynamically stable. ASK FOR VS.
 If not, transfer to treatment room, manage. Give oxygen at 2LPM
 If stable, and comfortable, continue with the consult.
3. History of present complaint questions:
 *Open-ended: I can see from your notes that you have been becoming
breathless for the last 6 years. Could you tell me more about it?
 *Onset: When did you notice that it became more difficult to do your usual
activities?
 Character
 Radiation
4. *Associated Symptoms
 Cough? Do you have any phlegm? Color? Is it blood tinged?
 Chest pain? Where exactly is the pain? Did it occur before or after? How is the
pain like? Crushing? Cramping? Does the pain go anywhere else?
 Orthopnea: are you comfortable lying flat on the bed at night?
 PND: do you often wake up during your sleep at night because of the SOB?
 Edema: any swelling noted over your feet? Around your Eyes?
 Nausea/Vomiting
 Fever?
 Weight loss? Night sweats? (rule out malignancy)
 Anxious? Worried about stuff more than the usual?
5. *Timing: when does the SOB usually occur? At night? In the morning>
6. *E: what do you do to make it better? Anything that makes it worse?
7. *SADMAH W: Do you smoke? I’m quite worried about your smoking habit, but let me
arrange another review with you so that we can address your smoking habit.
 Alcohol, engage in recreational drugs
 Any medications (including prescribed/otc drugs) that you are currently taking?
8. Where do you work, are you usually exposed to fumes? How long have you been
working there? Have you noticed if the sob worsened after you worked there?
9. Any previous illnesses?
10. Any family history of other lung or heart conditions?

Physical Exam:
11. GA:
12. BMI, Pallor, Icterus, Cyanosis, Clubbing, LN enlargement, Edema, Dehydration
13. Other systems
14. CVS: Distinct heart sounds, murmurs
15. Pulmo: retractions, chest expansion, resonance, air entry, adventitious breath sounds
16. Edema?
17. Office tests: ECG*, Spirometry, BSL

Diagnosis and Management


18. CONDITION. DRAW. CAUSE. COMMONALITY. CLIN FEAT. RISK FACTORS.
From your history and physical examination, it seems that most likely you have a condition called
chronic obstructive pulmonary disease. I'm sorry I'm using medical jargon, but do you have any idea
what it is? The lungs got small airways. And since you have been smoking for a long time, these small
airways tend to get irritated and eventually become damaged, leading to scarring causing the
narrowing of these small airways, causing you to feel your current symptoms. Again, this is mainly
attributed from your smoking, and possibly from the fumes and chemicals that you inhale at work. It
is not very uncommon, and it usually presents with progressive shortness of breath especially upon
exertion. What we can do for you now is this:
The management of this condition is through a national management plan called COPDX
19. CONFIRM: I already confirmed your diagnosis by doing spirometry, but I still need to do
other tests such as a chest xray, sputum microscopy, culture, and sesnitivity, and basic blood tests
such as a full blood exam, urea, electrolytes, and liver function tests to rule out other possible causes
of your condition.
20. OPTIMIZE: secondly, we need to optimize your lung function. I will refer you to the
respiratory specialist who can give you puffers to improve your condition. I will also refer you to a
chest physiotherapist who will teach you breathing and coughing exercises to improve your lung
function.
21. PREVENT: third, we need to prevent the deterioration of your condition. And the only
proven prevention for COPD is complete smoking cessation. We can work together to form a plan for
you to help you quit the habit, and connect you to the Quit Line. Secondly, you need to improve your
lifestyle by eating healthy and exercising, and also avoid areas which are polluted, wear a mask, or
even ask your manager if you can be assigned to areas which are unexposed to these irritant smells.
22. DEVELOP SUPPORT: I will also give you information about support groups for patients
with COPD.
23. If you do become severely short of breath, call the ambulance, you can use the puffer
that I will prescribe to you, press 1 puff and do 6 breaths, and do this every 6 minutes until the
ambulance arrives.
CLOSING: 4R's:
24. Again, I will review you regularly, and will arrange a referral to the specialist and
physiotherapist for you. And if you do feel any severe sob, chest pain, please call the ambulance as I
told you earlier. Here are some information which you can read to provide you with more insight with
regard to your condition. And don't worry, COPD is manageable, and rest assured with our plan and
regular reviews, you will still be able to live a comfortable life.
Feedback 20-9-2018 COPD

Shortness of breath (PASS)

60 years old lady comes to your GP clinic for investigation results. Previously she c/o cough with sputum, &
SOB. She is a former courier driver. She used to be a heavy smoker but now she quitted smoking. Her BMI
is quite high. Her SPO2 is 93%. Pulmonary function test - Moderately severe COPD. CXR - Hyperinflated
lungs

Tasks :

Explain Dx

Tell her about your management plan.

Also tell her measures to improve pulmonary health.

Approach :

I greeted patient and asked her whether she’s comfortable or not.

Dx : I told her that her results came back and all pointing towards a condition we call COPD. Then
explained her with a drawing.

Management :

I followed the pneumonic COPD X. firstly, I asked her whether she’s taking any medication for her problem
and she said no. Then, I said I’ll prescribe you inhaler medication which u can use whenever you have
symptoms and explained her briefly about how to use it. I appreciated her for quitting smoking and told
her to avoid passive smoking as well as polluted areas because this condition is caused by inhalation of
dusts, smokes and fumes and is most likely due to your previous occupation and smoking. Now that you
don’t have exposure to these anymore so, it’s very good for you. Then told her to reduce weight (diet and
exercise), and advised about vaccination. For pulmonary health, I’ll liaise you with physiotherapist who’ll
teach you breathing exercise which you can also do at home. I forgot to tell her about O2 therapy. Also tell
her about exacerbation of COPD and redflags. Reassured her and told her about supports. Time left so,
just repeat & summarize the important points again.

COPD spirometry (Materials then feedback)


Spirometry - Asthma

65 year old male, truck driver, complained of shortness of breath and cough. You suspected COPD, and
organized spirometry.

Spirometry findings Normal range Baseline (pre-bronchodilator) Post bronchodilator

FEV1 (liters) >2.62 3.00 3.70

FVC (liters) >3.13 4.51 4.85

FEV1/FVC ratio (%) >71% 67% 76%

TASKS

1. Interpret the results to the patient

2. Explain most likely diagnosis to the patient

3. Explain inhaler technique to the patient


APPROACH

FEV1 is the maximal amount of air that you can exhale in 1 second after taking a deep breath in. It
indicates the compliance of the small and medium sized airways. It is a measure of obstruction.

FVC is the maximal amount of air that you can exhale after taking a deep breath in. It indicates TLC. Any
disease that will restrict the size of the lung will reduce the total lung volume. It is a measure of restriction.

If FEV1/FVC is less than lower limit of normal, if FEV1 only has gone down, it is an obstruction. If both
FEV1 and FVC have gone down, it is a mixed problem.

Reading spirometry is a 3-step process. 1st step is to look at the ratio. The ratio of less than normal gives
us two possibilities, either a pure obstruction or a mixed obstruction-restriction. 2nd step is to look at the
FVC. In our value, it is normal, so it means there is no restriction, means we have ruled out the possibility
of a mixed obstruction-restriction, and we know there is obstruction alone. 3rd step is to know what type
of obstruction it is, if it is reversible or irreversible. To do that we look at the measure of obstruction which
is FEV1. We look at the post bronchodilator value minus the prebronchodilator value all over the
prebronchodilator value x 100. If it is more than 12%, it is a reversible disease and if it is less than 12% it is
an irreversible disease. Since it is more than 12%, it means we have a reversible obstructive disease, or
most likely it's Asthma.

Asthma is a common chest condition in which there is temporary narrowing of the breathing tubes in the
lungs (airways) because they are hyperreactive (oversensitive). In asthma these tubes have inflammation
and swelling of their linings, increased mucus inside, tightening of the muscles in their walls and therefore
less flow of air in and out.

Mixed obstruction - send him to a specialist for further evaluation of lung volumes to determine what type
of restriction it is, whether it is intraparenchymal or extraparenchymal lung disease.

If FEV1/FVC is more than or equal to the lower limit of normal, there are two possibilities: either it is a
normal ventilatory function or FVC has gone down which means it is a restriction alone type of disease.
This person will need a referral to a specialist to determine if the problem is within the lungs or outside of
the lungs.

Spiromety findings Normal range Pre-B Post-B

FVC 3.5 - 5.3 4.4 4.6

FEV1 2.5 - 4.0 1.8 2.0

FEV1/FVC ratio 65 - 84 41 43
COPD case.

Inhaler technique

1. Sit upright

2. Check medication before use. Check expiry date

3. Shake puffer well. Take off the cap and mouthpiece

4. Breathe in and out normally

5. Seal the mouthpiece off the inhaler

6. Tilt your chin up

7. Press the puffer and simultaneously take a breath in through your mouth

8. Hold breath for 10 seconds

9. Breathe out normally through your nose

10. Use as instructed

Do you have any questions at this point?


Case/ Your patient, Joseph Thomas, is a 65 year old former courier driver, who was a heavy smoker until
two years ago (30 per day for 40 years). He stopped working two years ago. For three years, he has had a
cough productive of small amounts of clear sputum only, and associated with increasing exertional
dyspnea. Dyspnea is now evident walking 100 meters on the flat. When examined in the hospital primary
care clinic two days ago, he was overweight (BMI 35 kg/m2, waist circumference 115 cm) and had an
audible wheeze. The chest was hyperinflated, auscultation revealing reduced breath sounds and
occasional crackles as well as wheezes bilaterally. There were no signs of right heart failure. You arranged
pulmonary function tests.

Pre-bronchodilator Post-bronchodilator Normal range

Spirometry

Forced vital capacity (FVC; L) 4.4 4.6 3.5 - 5.3

Forced expiratory volume (FEV1; L) 1.8 2.0 2.5 - 4.0

FEV1/FVC (%) 41 43 65 - 84

Lung Volumes

Total Lung Capacity (TLC; L) 7.4 7.8 5.4 - 7.4

Residual volume (RV; L) 2.9 3.4 1.6 - 3.2

RV/TLC (%) 39 41 26 - 52

Diffusing Capacity

Diffusing capacity for carbon monoxide 14 15 17 - 33

(DLCO; mL/mmHg/min)

TASKS

1. Interpret the results of the lung function tests


2. Explain briefly to the patient your diagnosis and plan of management

3. After six minutes, you will be asked by the examiner to demonstrate methods of using the
pharmacological agents prescribed

APPROACH

 Explain results
Okay Thomas, here are your pulmonary function test results.
A spirometry test measures the amount of air you inhale, the amount of air you exhale and how
quickly you exhale. In a spirometry test, your lungs are tested to see how well they are working.
There are 3 important values that we look at in interpreting the spirometry results: FEV 1, FVC, and
FEV1/FVC ratio.
These values help us distinguish if the patient has a normal lung function, a restrictive pathology,
obstructive pathology or a mixed restrictive-obstructive pathology.

o First, we look at your FEV1/FVC ratio. This is the volume of expired air in the first second
(FEV₁) compared with the total volume expired (FVC). Clinically, this gives a useful index of
airflow limitation. Most healthy people with normal lung function can blow out
approximately 80% of their air out within the first second, that is, the FEV1/FVC ratio is 80%.

If this is decreased, most likely we are dealing with an obstructive pathology.

If it is not decreased, then we could be looking at a normal lung function or a restrictive


pathology. To distinguish between the two, we look at the another value, the FVC.

 In your case, your FEV1/FVC ratio is decreased, so most likely you have an obstructive
pathology. To rule out if it is a pure obstruction, or a mixed obstructive-restrictive, we
look at FVC.

o Next we look at the FVC. This is the total amount of air that you can forcefully exhale after a
full inhalation. It is the a measure of the total lung volume that can go in and out during
breathing.

If this is decreased, it means that something is restricting the size of the lungs, and most
likely we are dealing with a restrictive pathology.

 In your case, your FVC is within the normal range, so we can rule out a mixed
pathology, and most likely we are dealing with a pure obstructive case.

o If we are dealing with an obstructive pathology, we look at another value, the FEV1. This is the
amount of air that you can exhale in the first second of full exhalation after a full inhalation.
This reflects the compliance of the large and medium sized airways and gives a useful
measure of how quickly full lungs can be emptied. Normally, this is also 80% or more.

o We look at FEV1 for two reasons: first is to know if it is a reversible or irreversible type of
obstruction, and second is to know the severity of the obstruction.

 To know if it is reversible or irreversible, we look at the pre and post bronchodilator


values of the FEV1. If the difference between the post and pre bronchodilator values is
less than 12%, it is irreversible, if more than 12%, it is reversible.

 In your case, the difference is less than 12%, so most likely we are dealing with
an irreversible type of obstructive disease, or COPD.
Do you have an idea what COPD is?

Condition: Chronic obstructive pulmonary disease (COPD) is an umbrella term for a number of lung
diseases that prevent proper breathing. Three of the most common conditions are emphysema,
chronic bronchitis and chronic asthma that isn’t fully reversible. These conditions can occur
separately or together.

The lungs are spongy lobes inside the chest, protected by the ribcage. Inhaled air is directed down
the windpipe into two tubes called bronchi that each service a lung. The bronchi divide into smaller
tubes called bronchioles, and further still into tiny air sacs called alveoli.

Each alveolus has a fine mesh of capillaries where the exchange of oxygen and carbon dioxide takes
place.

A person with emphysema has damaged alveoli and bronchi. The weakened and ruptured air sacs
are unable to efficiently move oxygen from the air to the blood. As the disease progresses and
damages more air sacs, the person may eventually feel breathless even when they are resting.

Bronchitis means inflammation of the bronchi. The lungs normally produce a small amount of fluid
to keep healthy, but chronic bronchitis causes an overproduction of fluid. This leads to frequent and
productive coughing.

Cause: The most significant risk factor for COPD is cigarette smoking and around 20 to 25% of
smokers may develop COPD. Long-term exposure to lung irritants such as chemical vapors or dust,
and severe air pollution can also make COPD worse in smokers. It can also be associated with genes,
a genetic disorder known as alpha-1-antitrypsin deficiency can trigger emphysema, even if no other
risk factors are present.

Commonality: COPD used to be more common in men, but the disease is quite evenly spread
across the sexes now that women and men smoke in equal numbers.

Clinical features: The main symptoms of COPD are breathlessness, chronic cough and sputum
production.

Course of Management:

There is no cure for COPD, and the damaged airways don’t regenerate. However, there are things
you can do to slow progress of the disease, and improve your symptoms.

Management of COPD is by a plan called COPD-X.

o C is for confirming diagnosis of COPD. We have done this with your spirometry results
showing persistent airflow limitation or an FEV1/FVC of less than 70%. However, we can also
do further tests like FBE, UEC, ESR/CRP, LFT, chest x-ray and ECG to rule out other conditions
with similar presentation to COPD.

o O is for optimizing lung function.

 I advise you to completely stop smoking. I can arrange another appointment with you to
counsel about smoking cessation. I can also refer you to Quitline to help you to quit
smoking.

 I will also refer you for pulmonary rehabilitation which are programs that consist of
individual assessment followed by exercise training and education.
 I encourage you to also have a regular physical activity.

 I will refer you to the specialist who will prescribe you with medications depending on
the severity of your condition. This might include bronchodilator medication to open
the airways, corticosteroids to reduce inflammation and swelling of lung tissue,
expectorants to loosen the phlegm and make it easier to cough up, oxygen therapy,
and antibiotics to treat existing infections if present.

o P is for prevent deterioration.

 We can do this by again, smoking cessation, and preventing exacerbations by optimal


use of medications.

 I will advise you as well to have your pneumonia and flu vaccinations to reduce the risk
of infections in the future that might exacerbate your condition.

 Eat a healthy diet, keep adequately hydrated, avoid smoky or dusty environments, and
ensure plenty of rest.

o D is for develop support.

 We will develop a COPD Action Plan to aid you in self-management support. This plan
details your plan of action depending on how you are feeling at a particular time.

 I will also refer you to support groups who can provide education and psychological
support. Lung Foundation Australia operates an Australia-wide network of affiliated
patient support groups.

o X is for manage exacerbations.

I will give you reading materials regarding COPD and I will review you regularly. Just in case you develop
shortness of breath that is not improving with medication, call the ambulance immediately. While waiting
for the ambulance, you can use your puffer, press 1 puff and do 6 breaths, and do this every 6 minutes
until the ambulance arrives.

Feedback 6-6-2018 COPD –pass


60 years old patient, new pt to your GP, move to your town recently, history of short of breath
for ? 3 months, previous doctor requested some inv, came to your clinic today with results. FBE,
TFT,LFT normal. Spirometry result given (almost same as handbook)
task:
further hx,
PEFE card,
explain the result,
dx, ddx

history: any short of breath now? No, Vitals SPO2 checked with examiner, stable. 3 months SOB,
cough sometimes with clear phlegm, orthopnoea nil, chest pain nil, palpitation nil, leg swelling
nil, LOW/LOA nil, lump bump nil, fever nil, occupation courier driver stopped working since 3
mths back, travel x, TB contact nil smoker-heavy. Stopped 3 mths ago, appreciate that. system
review coz patient is new to your clinic and he wants to follow up at your clinic .
PEFE Card : nil distress, BMI 35, chest- audible wheezing present, cvs normal

Explain all the tests result, explain spirometry only FEV1, FEV1/FVC, before and after
bronchodilator,

Dx COPD, explain pathophysiology of Copd, causes: smoking( forget to mention about dust
exposure ,he was a driver and high BMI) .Ddx: anaemia, heart failure, time up did not get the
chance to say other respiratory differentials.
Feedback 20-4-2018
Case (22/4/2017)
Spirometry interpretation. (Handbook 132) Read about restrictive lung disease as well.
Tasks can be:
- Explain spirometry to patient, focused hx, Dx/DDx with reasons
- Explain spirometry to patient, Dx/DDx with reasons, Management.

Copd case spirometry findings given and explain it. Same book case. Examiner gave findings
again he gave to hand when I asked towards copd.
She was smoker and worked as a courier person. she quited smoking and retired now I
appreciated for it .
But initially she was coughing and like breathless she behaved I asked examiner vitals he asked
wat u want I said mainly o2 saturation 93% but she was behaving same so I said examiner I
would like to give o2 to my patient 1st I want my patient to be comfortable them he said ok.
I explained spirometry simply Fvc is the air wat u have inhaled and fev1 is amour of air u expelled
and TLC is increased a that means the amount of air ur taking is in lungs it's basically problem
with air exchange. air ways has got narrowed and that's y u have sob..
then examiner gave findings and said her wat has already has damaged we can't do anything and
from now we will take care of u I will refer u to pulmonary rehabilitation centre and there we
have specialist and they will teach u exercises and make u learn how to cope up with the
situation.. I should have gave her steroids and admit. And came out she was not so happy..
thanked both of them and came..please do like in handbook

comment
I know Spirometry can be a really hard case, so I collected information from past posts and put it
together, made some changes to help students who are about to take the exam soon. Hope it
helps :)
In exam, they have given the spirometry values outside the room that we are supposed to
interpret. Patient had history of shortness of breath. Was 72 years old. No history of allergies or
asthma. Dry cough for a long time ..

TAKE HISTORY AND EXPLAIN SPIROMETRY RESULTS TO THE PATIENT..


EXPLAIN DIAGNOSIS ONLY. NO NEED OF MANAGEMENT.

(In the history he had an occupation of dealing with plants and vegetables. But when inquired
further SPECIFICALLY about asbestos exposure ever in past . Then he opened up that YEA I
REMEMBER I WORKED IN THAT COMPANY TOO)
RECOMMENDED APPROACH, BUT AMEND AS IT FITS:
Good morning my name is Dr. Anala and I am your GP (if he is a new patient).
I understand you have come to see me to get the results of the spirometry which is the test you
done last week .
I have got your results please don't hesitate to stop me if you need anything to be clarified.
Actually (name of the patient ) the test you had called spirometry which we do it to assess the
lung function looking for the cause why you are short of breath .
The results include FVC which means the amount of air which can be forcibly exhaled from the
lungs after taking the deepest breath possible and FEV1 which means the amount of air you can
blow out or breath out in 1 second.
The readings showed the ability of your lungs to blow out the air is reduced.
What we do is to take the ratio of both (FEV1/FVC) which is also reduced.
That means you are suffering from obstructive lung disease.
There is two types of this disease one is asthma and the other type is COPD for which we gave
you the puffer to differentiate between them and we repeat the test again if there is
improvement in your lung function it means reversible (asthma). However in your case the
improvement was less than 12% which means you have COPD (tell the patient the name don't
use abbreviations in the exam) which is irreversible (at this point wait for 5 seconds and ask the
patient everything is clear so far ).

In COPD, the lung gets filled with air and this, we can see by increasing the total lung capacity
(TLC) however there is problem with air exchange inside the lung that is why the reading for
diffusing Carbon monoxide capacity is decreased (you can skip this part if you would like to).
Because of this the amount of oxygen going to your body decrease and that is why you feel short
of breath. (stop and ask your patient if he can understand you so far).

Unfortunately, this condition is irreversible as I mentioned before and it is also progressive


however we can control the symptoms by medication with excellent response.
First of all, you have to stop smoking it is the corner stone of treatment.
I will prescribe you puffer to open your airways so you can breathe easier.
You need to adopt healthy life style (keep ideal weight / eat healthy / exercise at least 30 min a
day 5 days a week / go for walk in fresh air) this condition increases the risk of chest infection
that is why it is recommended to take annual influenza vaccine and pneumococcal vaccine every
5 years .
Then give your patient some reading material and talk about antibiotics in case of chest infection
if required plus the red flags.

P.S: Both in obstructive and restrictive lung diseases FEV1 is reduced, but in restrictive FVC is also
reduced a lot, so the FEV1/FVC is higher than normal( because the denominator is low), where
as in obstructive FVC is constant so ratio depends only on FEV1 value, so its less than normal
259-pneumonia
Case 1
HMO in ED, 17 years old female had shortness of breath.
Tasks:
-Focused history from the patient
-Physical examination from the examiner
-Explain to the patient the probable diagnosis and differential diagnoses

Case 2
HMO in ED, 27 yrs old c/o SOB, fever and chest pain on rt side. No travel h/o, No other
symptoms.
Tasks
-Ask PE from examiner (no history)
-ask ivx from examiner and explain it to pt
-explain about ur possible diagnosis and ddx to pt.

Case 3
Chest pain in young woman after history of fever with chills. Shortness of breath present. Take
history, PEFE and advise management. Dx: Penumonia
same approach like chest pain but the diagnosis will be pneumonia.
CASE 1 FEEDBACK
Case (29/3/2017)
A teenage/ early twenty girl has come with sob
Tasks: h/o, specific findings from the examiner, diag, dds
2mins- vital signs, dds- pneumonia, pleural effusion, pul embolism, pneumothorax, copd,
asthma, heart failure/ pul edema, traumatic diaphragmatic hernia, anaemia, psychogenic
A young girl was lying down in obvious respiratory distress. Vital signs- rr increased and SpO2
decreased. I would like the patient to be hooked to the monitor to have her vitals continuously
monitored, give her O2 before taking h/o. The examiner asked me to proceed.
I reassured the patient that we have her vitals continuously monitored and I have arranged for
O2 and asked her if I could continue with h/o, she said yes.
- Sob duration- for 2 days, but worse since morning, gradual onset, only during exertion before
but continuous now, getting worse, no relieving factors
- Urti and sore throat- 2 days back,
- cough- positive, sputum- no, no coughing of blood
- feeling hot with chills,
- chest pain- yes, localised on the lower right side, not radiating, moderate, aggravated by
breathing.
- Positive contact h/o.
- No recent travel, no leg pain,
- no similar episodes in the past, no h/o asthma,
- no racing of heart, no swelling of legs, no fainting episodes, no trauma, no stress.
- Do you think you are pale- no
SADMA- no, prev hospitalisation/ surgeries- no
Flu shots and immunisation- ask (but I don’t remember whether I asked her).
PE:
gen appearance- ill looking, PICKLE- no
Vitals- same findings, but temp- raised as well
Cvs- normal
Rs-
Inspection- respiratory distress- accessory muscle involvement- yes
Chest movement- reduced on the right side No chest wall deformity, visible pulsations
Palpation- trachea- midline, no tug; no tactile fremitus Vocal fremitus- increased in the right
lower areas.
Percussion- woody dullness in the right lower fields
Auscultation- bronchial breath sounds, crepitations in the right lower areas, vocal resonance
increased.
Office tests- unavailable
Most probable diagnosis- pneumonia complicated by consolidation (lung infection due to some
bacteria due to positive contact history)
Dds- pneumonia, pleural effusion, pul embolism, pneumothorax, copd, asthma, heart failure/
pul edema, traumatic diaphragmatic hernia, anaemia, psychogenic (explain why you ruled them
out with reasons)
Global score- 6
Feedback 20-2-2018
HMO in ED, 17 years old female had shortness of breath.
Tasks:
-Focused history from the patient
-Physical examination from the examiner
-Explain to the patient the probable diagnosis and differential diagnoses

2 min thinking: DDx: spontaneous pneumothorax, pulmonary embolism, pneumonia, asthma,


foreign body, allergic reaction, psychiatric.

History:
Introduce myself. Patient is semi lying on the couch. Alert, conscious.
Asked how is she feeling right now? She could answer my question however on the safe side I
would like to assess her vital signs before proceed (I think the examiner said she is stable, can’t
recall properly).
Asked about chief complaint (SOB since when, getting worse/better, aggravating/ relieving
factors, associated symptoms like fever, cough, RN, sore throat, pain in the ears, difficulty in
breathing, pleuritic pain etc).
Recent URTI? (yes, few weeks ago, subsided, did not take any medication)
Recent long hour air travelling? (no)
Recent physical activity, injuries? (no)
PMHx: asthma, allergic, heart disorder (no)
FHx: can’t remember whether anyone with URTI/similar symptoms.
SADMA: unremarkable except allergic to penicillin
Stress home/school – none
Physical examination:
GA, VS – T?38, RR ?20, sPO2 ?, BP normal
ENT examination – unremarkable
Respiratory examination – inspection, palpation, percussion (dullness on right midzone I think),
auscultation (crepitation on right midzone as well),
Other systemic examinations unremarkable
No CXR or other office test available.
Explanation:
(With a diagram) Dear June, based on your history and physical examination, most likely you are
having infection of the lung tissues, and we call it pneumonia. Most likely it is due to some bugs
you contracted few weeks ago as you are having URTI. Don’t worry too much it could be treated,
most likely we need to admit you and give you antibiotics but definitely not penicillin (did not
proceed more as management is not my task). Otherwise it could be due to air trapped in
between lungs covering, clot blocking the blood vessels of the lungs, asthma, allergic reaction,
however these are less likely in your case. Assess patient’s understanding, reassurance.

AMC Feedback – Shortness of breath: PASS


Feedback 5-7-2018
Young lady present with fever and SOB emergency setting. Spo2 was low
Task: 1- Hx

2- PE from the examiner

3- DDX

Outside I thought about DDx of acute SOB (cardiac(pericarditis), pneumonia, pulmonary


embolism, pneumothorax, pleural effusion)

Inside the station a young lady lying on the bed, I introduced myself, tell the pt that I need to
speak to the examiner, I put my pt on o2 and continue my Hx in resuscitation cubicle and check
the vitals. In Hx I asked about my DDx and hx of recent URTI, SADMA and home situation, PMHx,
PSHx.

On examination there was reduced air entry on right side with crackeles.

I explained to the pt most likely Dx is pneumonia and explain other DDx my drawing on paper.

Passed: global score 5

Feedback 5-7-2018
21 year old female , coming to ED with SOB
Task
Relevant history
Examination finding from the examiner
Explain d/d to the patient
On entering to the room, patient crying on the bed. I introduced myself and told her it’s ok I’m
here to help you.
Offered painkiller and asked for allergy – she said allergic to penicillin ( saw examiner ticking
things off so don’t forget allergies!).
on hemodynamics , her sat was 90% , so I said it would take her to the resus area and put her on
o2 first and then started taking the history (doctor told me there is no management just
continue lol).
Asked about SOB, what were you doing ? she said walking. (I though spontaneous
pneumothorax first!!!! )
Were you ok before that? 2 days ago Fever and cough for few days (ok now that’s pneumonia),
sob+,pleuritic chest pain,
Runny nose/ sore throat – no travel ? – no leg pain ? – no (exclude DVT)
anyone sick home? – no
general appearance , vitals.
examination of lung percussion (dullness on axillary lower lobe of lung). Leg for DVT
Ddx: anything in that area even though it’s mostly lung problem (pneumonia / pleural effusion /
PE) Shingles/ kidney infection/ gallbladder… but not likely.
Feedback 21-7-2018
Shortness of breath – PASS (G.S- 6)

Stem: Young lady, with cough and shortness of breath

Task: -History

-PEFE (ask specific findings)

-Immediate investigations (ask specifically from examiner)

-Explain the investigation and DDx to patient

-Vitals- febrile, SpO2 90, RR high- arranged for oxygen.

-Young lady with recent flu, with cold, sorethroat, now developed productive cough since the
past couple of days, shortness of breath. Quite obviously pneumonia. Still….Negative DDx
questions-wt loss, lumps n bumps, travel, smoking

- PEFE- asked each and every step just like I’d do the PE..inspection, chest expansion, percussion
(dull note rt mid), auscultation (bronchial breathing rt mid lung fields)

-Investigations- Chest Xray, Sputum exam, Blood FBC, CRP, ESR, Examiner waited for me to list all
inv, saying ‘and…’ So said CT chest maybe. Then, slowly gave out the PA and Lateral CXR films-
quite clearly Rt mid lobe consolidation. Explained it to patient.

-Said its Pneumonia, likely bacterial, however other causes for SOB could be viral pneumonia,
COPD, Pulm embolism, cancer but highly unlikely. She said ‘isnt it only in old people?’ She was
fully immunized. Said there is no such rule, it can affect any age group, the resp mucosa is a
continuous lining so it has spread down from tour upper resp tract. We’ll treat you with Abx etc.
Feedback 11-12-2018
You are an ED doctor when a 22 year old girl was brought in complaining of SOB. Stats normal except O2
90%.
Task- A. HISTORY
B. PEFE
C. explain dx ddx

HISTORY- STARTED WITH hemodynamicaly stability then started oxygen and monitoing
Sob- ask onset-, progression, frequency, anything making it worse or better, timing of sob, sob at night?
(patient is feeling sob from yesterday and getting worse)
Associated symptoms- chest pain, cough (+) and dry, pain present when she takes deep breath
Ddx- pneumonia, (fever present , and positive urti last week)
Pulmonary embolism(no travel history, no calf tenderness)
Asthma(just ask any wheezingand cough at night)
Heart – rule out pericarditis by asking chest pain Socrates), angina and pneumonia
Panic attack (ask about anxious personality)
System review- genitourinary symptoms, git
Past history- NAD
Family history- any one with similar complaints at home
Sadma- nil
AT the end of history ..i m sure it is pneumonia..but you have to hit in all directions to pass
Pefe- GA,(PICCLED)
VITALS AGAIN- SAME
LUNGS- START WITH RESPIRATORY DISTRESS SIGNS
INSPECTION(NORMAL), PALPATION(NORMAL), PERCUSSION(DULLNESS IN RIGHT LOWER REGION),
AUSCULATATION(AIR ENTRY REDUCED IN LOWER LUNGS)
HEART- dual heart sounds present, no murmers, no rub
Legs- NAD
URINE DIPSTICK- NORMAL
ASK ECG, XRAY- NOT AVAILABLE
DX- PNEUMONIA(AS HISTORY POSTIVE FOR URTI AND EXAMINATION FINDINGS)
BUT NEED TO DO FURTHER MANAGEMENT to rule out other ddx(again mentioned above ddx)
Keysteps- all done except key step 1
Global score – 4

Feedback 11-12-2018
Station 2 (Shortness of Breath) Global Score 4 Pass
22-year-old woman comes to ED complaining of SOB. O2 Sat: 90%. Other vitals: normal.
Task: Take further history, explain Dx and DDx.
I came in the room, greeted the examiner and the patient. I confirmed the vital signs and said that I would
like to lay patient down in 45º, start on oxygen through catheter and oximeter. (Although patient was
already lying in bed on 45º and with a catheter on her nose).
I approached the patient, introduced myself and asked if she was comfortable at the moment and if she
would answer some questions. Than I told her if she’d feel any discomfort or SOB, I would stop. She said
she was fine to continue.
I started asking about the SOB:
First time? Yes. Sudden or gradual? Gradual. Any position that makes it better or worse? No. Any pain? Yes
when breathing in (I don’t remember in which side!) . Any fever? Yes. Any recent URTI? Yes. A Week ago.
Any phlegm? Yes. Greenish, no blood. Any weight loss? No. Any recent travel? No. Any pain or edema in
the legs? No. Any trauma? No Any funny racing of the heart? No.
Asked consent and asked about sexual life. OCP? No STDs? No. Regular periods? yes. Any heavy bleeding?
No. Any bleeding from some where else? No
SADMA HO- unremarkable
Diet: normal
Vaccination? Up to date.
Previous history? No asthma, no bronchitis, no cystic fibrosis, no anemia.
Occupation: nothing positive
Home situation: unremarkable.
Than I explained: “ According to your history, most likely you have an infection on your lung. We call it
pneumonia. When a bug comes into your lung, it causes an inflammation. That’s why you have fever,
sputum when you cough and SOB due to liquid in your lungs. It can be caused by several kinds of bugs like
virus, bacteria or fungus. It is a common condition with a very good prognosis. Don’t worry!
Are following me? (she said yes)
Other causes for your SOB could be:
Pulmonary embolism but you have no history of recent travel, use of OCP, pain in your legs or sudden
onset of SOB;
Pneumothorax but you had no trauma and the onset was not sudden;
Pleural effusion (I gave no explanation);
Anemia but your diet is normal, you had no bleeding and no family history;
Tuberculosis but you have no history of travelling to areas of risk and no weight loss;
Asthma but you have no previous or family history and usually it doesn’t come with fever and sputum; and
Cancer but you had no weight loss.”
I forgot to mention heart conditions like Heart failure, MI, angina.
I thanked both and left the room.
Key step 1:yes Approach to patient/relative: 4
Key step 2:yes. Choice & Technique of exam, organization and sequence: 4
Key step 3:no. Diagnosis/ Differential diagnoses: 4
Key step 4:yes. History: 4
Key step 5:yes

CASE 2
Case (27/4/2017)
Bilat Pneumonia PEFE, Invx,
HMO in ED, 27 yrs old c/o SOB, fever and chest pain on rt side.
No travel h/o, No other symptoms.
Ask PE from examiner, ask ivx and explain it to pt, explain about ur possible diagnosis and ddx to
pt.
When I entered the room, the patient was lying on the bed. I started introduction and greet to
the pt first. Ask her’ how are you feeling.’ She said she is feeling SOB and fever. I would like do
some examination, can you wait for awhile.

GA-as you can see


VS- T-39’C , BP-120\80, PR-90\min. RR-22, O2%-95 %
I told the examiner that I will give my patient O2 as pt got SOB. Examiner said OK.

Respiratory examination done.

*PF (chest wall m/m -reduced on rt middle, no trechea shift, palpation - reduced chest wall on rt
side , percussion-dullness on rt mid zone, vocal flamitus -incresed on rt side.) I am thinking about
consolidation at this stage. Then, I asked for ivx like FBE, ESR, CRP- not available, when I asked for
CXR, examiner gave me the film.

X ray - a bit more darker on upper mid zone but also like more prominents of vascular markings
and not sure of fluid collection..left side od the lung is also the same like rt side.

I am a bit confusing about the CXR. I just stay calm and explained it to pt.

This is your CXR or imaging we did. As you can see the two black shadow called lungs on both
sides.
You can see your rid bones here and the heart is here. I don’t know what to say about the x ray, I
just said that according to my assessment, you got fever, SOB and I also found some features of
consolidation on my examination. She asked me what is that, I explained her that it is a kind of
condensed mass. Do you understand my explaination, she seem not understand but I skip, I
suspect that you might have a condition called pneumonia, infection of the lungs. She told me
that pneumonia! its an old age thing. I said yes, it can also happen in young people. But, we also
need to considered other condition like viral infection of the lungs, lung abscess; pus collection
inside the lungs. Some infectious growth inside the lungs.

Sometimes, it can be due to a condition called pulmonary embolism which is blockage of vessels
supplying the lungs, PE; collection of air between the layers of the lung. It can be due to
underlying auto-immune condition like SLE, RA.

Check her understanding again and reassure….

FB-PASSED, OVERALL-6, APPROACH-5, PE-6, IVX ASK-5, INTERPRET IVX-4, DX,DDX-7.

Case (4/10/2017)
Shortness of breath – Pass.
Scenario.
A young girl in her 20s presented with respiratory symptoms- you are HMO -ask examination
findings and any available investigations from examiner and explain the condition to patient and
DDs.
PEFE
Asked GA/ VS with O2 saturation/ general inspection (don’t remember details - not much
positive except increased respiratory rate), then straight into respiratory system examination:
Chest expansion reduced R/S. Vocal fremitus and resonance increased. Bronchial breathing in R/
middle zone with few crepts. No rhonchi.

Asked for CXR examiner gave 2 x rays- lateral and AP with typical R/ middle lobe consolidation
(pneumonia).

Asked for some other Ix which were not yet available at the time.

Explained diagnosis to patient

Pneumonia R/lung cause is probably bacterial. “She said I thought it happens to old people!”.
Explained its more common in old age but can happen in your age specially after viral URTI then
secondary bacterial infection.
I think there was a hint on this in the scenario as well. Showed the patient the areas of
consolidation in x ray and corelated it with symptoms.
Explained DDs Saying there are other causes of shortness of breath and respiratory symptoms
that are less likely to be the cause here. Viral infection/ TB/ DVT (PE)
Please make sure you practice how you explain DDs to patient as this is the new trend in AMC
Clinical. I did not practise this well and it made me feel awkward in the exam. I did not know how
deeply I should go into this explanation... For an example when I said tuberculosis role player
asked what is that? Deep vein thrombosis- what is that? …Then you may run out of time if you
go on explaining one less likely diagnosis too much…

• 4/4 key steps covered

Recall 10-4-2018
ED Intern Young Woman – Had a simple viral infection Cought etc Runny Nose Now developed
Fever, Productive Cough Ask Examiner for Chest Examination Findings Ask Examiner what
Investigation – CXR Explain to Examiner Dx & Diff Dxx. No need to talk about Mxx.

Comment
It was a pneumonia case when u entered the lady was sitting on the bed and sniffling,u were
given all the classical finding of pneumonia on asking them and you had to tell her regarding the
investigation and the treatment along with X-ray findings.Hope that helps

260-Fibroadenoma counselling
27 year old girl , has a breast lump, 1cm , mobile , non tender, firm, had USS done last week.
Now coming to discuss the result
A pic of USS is given
Tasks
Explain d/d to the patient
Address her concern about breast cancer
Explain ultrasound
-This is your normal breast tissue and this is the lump that you are feeling. You can see it is round
in shape, it is border are very well defined and regular, not intruding into the surrounding
structures.
-these features are quite reassuring as it indicates the lesion is likely to be benign and not
cancerous.
Counselling
-most likely you have a condition called fibroadenoma which is
*a benign, non cancerous lump that arises from breast tissue, becomes fibrous and firm under
the influence of hormones.
*It is a very common condition. That usually seen in this age group.
*It is not a nasty growth and won’t undergo nasty changes.
*It can regress, increase in size or remain unchanged.
-other ddx could be
*Lipoma: collection of fat totally harmless
*cyst: fluid filled sac inside the breast tissue.
*abscess: collection of pus (less likely as no fever)
*cancer: very unlikely
- Although the features on US are quite promising, in any breast lump we routinely follow triple
test. To confirm fibroadenoma and to rule out any cancerous growth.
*the first part is clinical examination, which has been done.
*the 2nd part is imaging (US /MAMO )
*The 3rd part is cytology by using needle biopsy to be introduced into the lump and cells from it
will be taken out to be examined under microscope for any nasty growth. for this reason I need
to refer u to a surgeon to do something called FNA
-If it is confirmed to be fibroadenoma then it can be left behind as it is with regular monitoring
begin 6 monthly then annually by GP and monthly breast check after period.
-if there is discomfort or increase in size, you can be referred for surgery in order to relieve the
symptoms. However, it can recur even after removing it, in addition that the surgery can cause
complications like unwanted scarring on breast, dimpling as well as little bit duct injury.
-No convincing evidence exists that benign fibroadenoma are precancerous or turn into cancer
and much collateral evidence on screening program follow up suggests they are not.
-any family history of cancer? (No) Since you have no FHX of cancer It is advisable to do mamo
after 50 yo

Q/ can I continue the OCP?


-yes, if it is just a benign lump, you can continue with OCP, as it is protective against other benign
lumps as well. (in global score 7 feedback she said stop !!)
Q/ is FNA confirmatory or diagnostic of benign lumps
-the needle is directed at different points of the lump taking 5 or more samples. It would be
expected to give definitive and accurate dx with minimal likelihood of false – or + results.
Feedback 21-7-2018
Breast lump – PASS (G.S – 4)
Stem: USG pic given( It was somewhat similar to the HB pic). Young lady, had presented with a
breast lump last week, no family h/o breast cancer, she is on the pill. You had examined her n
found a solitary, freely mobile, non tender, firm mass measuring some 2 x 3cm in the upper
outer quadrant of rt breast, no Lymph nodes. She has come to discuss her result.

Tasks: -Discuss inv result with pt (no further history required)

-DDx

-Mx/ counsel her

- Asked first of all if she knew what was the inv done, she was well informed. So said, when I had
examined suspected it to be a fibroadenoma, a non cancerous growth, I had sent for an USG to
see the nature of the lump. It is also suggestive of the same (maybe it would have been safer to
say it maybe a cyst or fibroadenoma, but unlikely to be cancer. However, I’d like to wait for the
radiologist’s report- I got a low score for interpretation of inv).

-DDx Fibroadenoma/ breast cyst/ fibrocystic disease/ Ca

-To be absolutely sure I can order FNAC.

-Monthly self examination, gave red flags/-Yearly examination by GP/-2yearly mammography


from 50

-Can continue OCP, no surgery required at present. She agreed.

Feedback 5-7-2018
27 year old girl , has a breast lump, 1cm , mobile , non tender, firm, had
USS done last week. Now coming to discuss the result
A pic of USS is given
Tasks
Explain d/d to the patient
Address her concern about breast cancer
Ddx: fibroadenoma / fibrocystic disease/ cyst / abscess/ cancer.
I told her this is fibroadenoma .. common at your age .. the breast tissue
becomes fibrous and firm under the influence of hormones. There is no
risk for cancer. But we can be sure by doing something called triple test :
clinical examination / imagine (US)/ FNA or core cut biopsy)
Just to be sure.. so I’ll refer you for that.
She is on OCP so told her to consider changing it to barrier methods If
possible like condoms.
Told her management is either leaving it and checking it regularly or
removing it by surgery. It has high recurrence rate though, so even if you
remove it, it could recur because the hormones are still there

Feedback 5-7-2018 (good)


Young lady with breast lump, US was done, a pic was given
Task:
1- tell the pt DDx
2- address her concern.

When I entered a young nice lady was sitting, I introduce myself, I told her that I know u hear
today to discuss the US result, I explained that, this is ur normal breast tissue and this is the
lump that u r feeling, u can see its round in shape, its border r very nicely well-defined and
regular, not invading into surrounding structures, these features r quit reassuring as it indicates
the lesion is benign in other words it’s something we don’t worry about and its not a nasty
growth (she asked what is nasty I said it’s not a cancer). I said I’ll draw a diagram, I drew the
breast with fibrous and ducts I explained this lump could be a fibroadenoma (changes in breast
tissue due to hormonal effect), or lipoma (a collection of fat totally harmless), cyst (duct dilation
again due to hormonal effect), abscess collection (less likely as no fever, no lactation). Then I told
her although the features on US r quit promising, in any breast lump we can not be 100% sure
till we take a sample for examination under the microscope, for this reason I need to refer u to a
surgeon to do something called FNA, I explained the procedure, a told her if it is cyst there is a
very high chance it totally disappear after the aspiration, if it is lipoma or fibroadenoma MX well
be either leave it with monthly check after period, annual exam by GP or if its troublesome it can
be removed by surgery. In case of fibroadenoma I asked about OCP, I told we need to stop it. I
asked about family HX of breast Ca was negative, I said mammogram after 50.
Passed : global score 7

Case (20/4/2017)
Lady with breast lump/on microgynon 30
Stem provided Full breast examination findings + breast USG. Tasks-
1-explains ur diagnosis
2-Explain further investigations
3-Address concerns about Ca

Case (28/4/2017)
- breast lump.
Twenty years came with usg.explain usg to her and tell further management.
no comment on usg.looks like fibroadenoma.I explained regular pattern,no calcification,regular
outline.looks benign most likely fibroadenoma.
quickly asked wt loss,appetite,discharge nipple,distortion,skin change,family history etc.
reassured her,no further investigation,she was happy.
told monthly self examination after periods,if new change in this lump or any other lump come
back.red flags.
as no family history so start mamo at regular fifty years age.
sexually active so safe practice,pap
Pass

Handbook 60
Breast biopsy concerns in a 20-year-old woman with a family history of breast cancer

Your next patient for office consultation in a primary care community practice clinic is for review
of a 20-year-old single woman whom you saw four weeks ago with a complaint of cyclical
mastalgia for the last six months. Physical examination of the breasts was normal. She had no
previous history of breast problems.

Her 50-year-old mother, also a patient of the clinic, had a Stage 1 breast cancer treated by
mastectomy and axillary dissection five years ago and is well on follow up.

You ordered an ultrasound of the breasts in this young woman, which showed an impalpable,
focal well-circumscribed solid parenchymal lesion in the right breast 1 cm in diameter consistent
with a fibroadenoma.

You referred her to the female surgeon who treated her mother, who suggested an ultrasound-
guided percutaneous core biopsy to confirm the imaging diagnosis of benign fibroadenoma. The
patient was also reassured that if this showed, as expected, a benign lesion, surgery would not
be required, and she could be observed clinically with periodic ultrasound assessments.

The patient is unhappy with this advice and feels she would like the lump removed and has come
back to you to discuss this further. She is worried that the lump may be malignant or will become
so, and feels that just taking a piece of it will leave her still worried.

YOUR TASK IS TO:

• Discuss her concerns with her and advise her on the future management you would propose.

AIMS OF STATION

To assess the candidate's counselling and educational skills in a patient with concerns about
familial breast cancer risk.

EXAMINER INSTRUCTIONS
The examiner will have instructed the patient as follows:

You are very worried that this lump could be a cancer You feel it should be removed so you don't
have to worry about it anymore. If the doctor's reassurances are clear and convincing, you are
prepared to change your mind. If not, ask if you can have a second surgical opinion

Opening statement:

'I think this lump should be removed'.

Questions to ask if not already covered:

• 'How can you be sure it's not a cancer?'

• 'Isn't it likely to turn into a cancer'?

• 'Can’t I just have it out and then forget about it?

EXPECTATIONS OF CANDIDATE PERFORMANCE

• Reassurance about likely benign diagnosis

• Reassurance that impalpable fibroadenomas are very common in non-symptomatic

women on imaging and do not reguire excision and that they are not cancers and

do not become cancerous.

• Reassurance that the biopsy takes several representative pieces and can save

unnecessary surgery and avoid potentially unsightly scarring.

• Reassurance that with a homogeneous lesion such as this, the biopsy could be relied

upon to give a definitive diagnosis.

• Sympathy for concerns of patient about cancer, and about continuing clinical and

ultrasound monitoring; reassurance of noninvasive nature of ultrasound monitoring:

reassurance of noninvasive nature of ultrasound

• Assurance that if patient is still concerned, the surgeon would be likely to accede to

her wishes, and if not, she could be referred for a second opinion.

• Whatever the patient decides, periodic followup with clinical and imaging reviews will

be advisable because of her family history and her concerns.

KEY ISSUES
• Counselling and communication skills in dealing with an anxious patient.
• Knowledge of pathology and natural history of breast fibroadenomas.

CRITICAL ERROR - none defined

COMMENTARY

The scenario illustrates a common problem. Breast cancer is very common in Australian women and
around 1 in 14 women will develop breast cancer in their lifetime. The risk is increased in the
presence of family history of breast cancer in first degree relatives, as is the case here.

This young patient requires regular clinical screening and appropriate imaging. The ultrasound
ordered by the family doctor was appropriate as initial investigation. Note that her original problem
(cyclical mastalgia) has now been replaced by the more serious problem of 'I have a breast lump
which could be cancerous' Her natural reaction (which might be the correct solution to the problem)
is 'I want it out.'

Benign impalpable (or palpable) fibroadenomas and other benign parenchymal lesions are very
common in this age group. We know from mass screening that benign lesions occur throughout all
stages of life, and that the natural history of fibroadenomas may be to remain unchanged, to
increase in size or to regress. No convincing evidence exists that benign fibroadenomas are
premalignant, and much collateral evidence on screening programme followup suggests that they
are not.
This lesion is impalpable and both clinical findings (normal breasts) and imaging findings (typical
ultrasound appearance of a benign lesion) already favour a benign fibroadenoma. But these
findings alone are not enough, and pathological confirmation by biopsy is required additionally to
make our reassurance quite positive ('triple test check' — clinical, imaging, and pathology all
confirmed and negative for cancer).
Whether total excisional biopsy or partial core biopsy should be performed will depend on
circumstances, but an appropriate core biopsy would take five or more representative samples and
would be expected to give a definitively accurate diagnosis with minimal likelihood of either a
falsely negative or falsely positive result, and with minimal morbidity in experienced hands.
Pathology can be determined by fine needle aspiration cytology (FNAC) or by percutaneous image-
guided needle core biopsy. Choice will depend on circumstances and availability of expert cytology
and pathology services; but core biopsy will give a tissue diagnosis and has higher sensitivity and
specificity, so is generally preferred.

If the 'triple test' is negative, the lesion is virtually certain to be benign. The patient will require
continuing periodic clinical and imaging review from her family doctor and surgeon. The surgeon's
advice was therefore appropriate and concise, but she has not convinced the patient that it is the
right plan. The utility of any advice regarding management is only relevant and helpful if patient
acceptance is present. If this patient remains unconvinced and unhappy, despite repeated
reinforcement by the family doctor, clearly the best decision may be to agree to her own wishes
that the lesion is removed. This will require an image-guided needle localisation operation. It is
unlikely that the surgeon would not agree to this, even though she (the surgeon) correctly regards
core biopsy and observation as the best option.

261-Sorethroat requesting antibiotics


Middle age man 30 yrs. old with throat pain for 2 days want antibiotics.
Task
-hx
-pefe
-possible dx & mx

Hx.
1-Pain Qs
-How long ? 2days
-Why you want antibiotics ? ( have important presentation tomorrow at work )
-Etc….

2- Associated sx
-Flu like sx, fever, LOA, LOW, rash, joint pain, runny nose, stuffiness, ear pain.
-Difficulty swallowing (+) now liquid diet.
-Lock jaw, drooling, voice change
-Nausea, vomiting, diarrhea, water work
-SOB

3- General
-Pmhx (tonsillitis)
-Contact, travel, occupation ( engineer )
-SADMA ( smoker 5-6 cig /d )
-Sexual hx (-ve)

PEFE (+ve findings )


-Fever 99-100
-ENT throat mild congestion , no exudate ( no pic )

Explain
-Most likely viral tonsillitis caused by virus like seasonal flu and common cold, transmitted from 1
person to another by small tiny droplets when they cough or sneeze.
-Because its viral infection, the antibiotics are not effective for this condition bec abx only for
bacterial infection.
-The condition is self-limiting in nature, it will go away 5-6 days.
-I am going to take throat and nasal swabs an give you some painkillers.
-Take plenty of water and liquid diet like soap, but make sure it’s not too hot or too cold , just
room temp.
-Avoid crowded places and wear mask.
-Review in 3-4 days
-Red flag ( if get worse , cant swallow )
-If you agree, I can arrange another counseling session for smoking.
-Offer sick leave ( pt. doesn’t want )
-It’s advisable to take rest because of sore throat, your immune system can be low, easy to get
infections from others.

Recall 21-2-2018
Sore throat, asking for antibiotic, having important presentation tomorrow.

Feedback 31-5-2018
GP,middle age man(around 30) with throat pain for 2 days,wants antibiotic
Tasks-H/O for 4 mins,PEFE,Possible diagnosis and Management.
Greet the patient and ask for pain killer, he said it is ok, patient was acting impatience.
when I asked how long? he said 2 days,that is why he comes here to get antibiotic after that he
has to go to work and he can’t miss the work.
i said I understand your concern, but let me ask a few questions to find out the cause and we r
going to decide whether you need antibiotic or not.
Then I asked some pain questions, flu symptoms-feeling hot like mild fever, ear fullness, cough
dry or white sputum? (no running nose, stuffiness, jt pain, red eyes, eye itchiness, rash).
difficult to swallow and now taking liquid diet.
no drooling, no lock jaw.no hoarseness. no vomiting,no SOB.no lumps or bumps, no LOW and
LOA.bowel and water work ok,
no contact, smoke 5-6 cigarettes a day or drink (one of these,don’t remember), not on any
medication, no allergy, no recreational drugs, occupation-engineer, not sexually active, no travel.

PEFE-vital mild fever 99-100,ENT-throat mild congestion.no exudate.(no picture)the rest-normal.

Most likely viral tonsillitis, cause by virus like seasonal flu and common cold. transmit from one
person to another by small tiny droplets when they cough or sneeze.
because of virus infection, the antibiotic is not effective for this condition. because antibiotic is
only for bacterial infections.
The condition is self-limiting in nature, it will go away 5-6 days. I am going to take throat and
nasal swabs.
give you some painkillers and fever medication.
take plenty of water. take liquid diet like soup, but make sure not too hot or cold,just room
temperature. avoid crowded places and wear mask.(forgot to say gurgling with salt warm water)
review in 3-4 days.
If it gets worse or if u can’t swallow, please come back immediately. concern about smoking, if u
r agree I can arrange another counseling session for smoking,he said it is ok(like he does not
need).
Offer sick leave, he said he does not want. i told him need to take rest, because of sore throat
,your immune system can be low,easy to get infection from others. i recommend to take some
rest. he said it is ok.
Keys 4/4 approach,history,dx and ddx,management 4,5,5,5 global 5

Feedback 7-9-2018
Station 17 – sore throat – PASS
Sore thorat asking ABX (Viral tonsillopharyngitis )
Task –
1. history (4 mins ) – just introduce the patient and
Patient straight away abx to her sore throat as she needs to go back to work tomorrow
2. PEFE CARD – Vital – normal ( nil fever , others – normal , )
ENT – watery discharge from nasal turbinates , inflamed tonsils and pharynx , but NO EXUDATE
Nil lymphadenopathy , ABD – nil organomegaly
Others system- unremarkable
3. Explain the initial mgmt to the patient. ( asking ABX for 3 times consecutively , explained
about ABX is not necessary and explain about dangerous antibiotic resistance in later life and
patient said that oh really , I never known that before, previous doctors gave me abx and it
clearly very quickly . ) – explain general and specific measure to the patient about viral
tonsilopharyngitis ( viral URTI )

Feeback – Sore throat – PASS


- Global score – 6
Key step 1 to 4 – all YES
Approach – 5
History – 6
Dx and DDx – 5
Mgmt plan – 6
Feedback 7-9-2018 (SORE THROAT- FAIL)
22 year old female, complaining of sore throat for past few days. No known medical
condition.
-hx
-PEFE on card
- DX,DDX
-Mx.
FEEDBACK: FAIL
Global score: 3
key steps: 2/4 approach to pt:5 hx: 4 dx/ddx: 4 mx :3
I really don't know why I failed this station!! I was quite sure I would pass this one. As I can
see, mx was an issue. Maybe, I couldn't counsel the pt well. Or, I think , I forgot to talk about
invx to rule out other ddx.
I was happy to get PE on card as I mentioned I had issues with time mx. So, my plan was to
take short hx, enough to cover 4/5 ddx , key points and 4 marks. ( I was very keen to ask
more in hx but my study partners and my friends convinced me to change the plan, so that I
can finish all the tasks first.)
2 mints thinking:
Common cold? or Viral / bacterial tonsilitis? Smoker? Repeated infections? contact? travel?
sexual hx? allergies? fever? rash?
DDX:
1.Common Cold
2. Viral tonsilitis/pharyngitis
3. Bacterial tonsilitis
4. EBV/ Infectious mononucleosis
5. Pharyngitis due to gonorrhoea
6.HIV
7. Environmental/ seasonal allergies.
As soon as I introduced myself, she threw me this question: 'Doc, I came here to get some
antibiotics, please prescribe me some and I will be on my way' I said her politely as I would
ask her bunch of questions to make sure she really needed abx. She agreed.
chief complaint- ODIPARA
Onset- sudden
Duration- 2/3 days
Intensity- can't go to work but I have a coming important meeting, that's why I need to be
recovered soon.
Progression- fever, no cough, no discharge, no sputum, body ache.
Aggravating factors- nothing
Relieving factors- panadol, rest.
Associated- other ddx- travel? un-protected oral sex? contact? smoking? allergies? weight
loss? rash? (all negetive)
Past hx- previous sore throat hx, no other medical condition.
PE on card: all normal, except- inflamed tonsil.
MX: I didn't prescribe abx (she just said okay!) and ask her to do general mx. I even offered
her to give letter so that she could take few days off from work. She refused to take any leave
from work as she had an important meeting.( what! no abx! no medical certificate! I couldn't
figure out then, how to answer this!!). Then, I said general measurements like gurgling,
panadol for fever, flu vaccine, red flags. She was non-smoker , so I didn't get the chance to
talk about smoking. The role player was quite reluctant to whatever I mentioned.

262-Sorethroat requesting analgesics


Sore throat requesting analgesics
Recall 21/2/2018 , 22/4/2018

Male with sore throat came requesting analgesics for pain relief, smoker.
Task
-HX
-PEFE
-Dx/DDx

1- Pain Qs…
-severity + painkillers
-when, onset ( 2 days ago )
-site, radiation.
-aggravating , relieving factors
- 1st time

2- Associated sx
-dysphagia, odynophagia, neck pain ( can’t eat solid, just liquid )
-Difficulty opening the mouth
-voice change.
-SOB, cough.
-runny nose, ear pain. ( ear pain)
-nausea, vomiting, diarrhea, water work ( decreased appetite)
-Fever, recent infection, rash, LOW, LOA, lumps.

3- General
-Pmhx ( esp. tonsillitis )
-Pshx, trauma.
-SADMA (smoker)
-travel, contact , sexual hx.
-Fhx.
Pefe
1- G/A (PJDLR)
2- V/S (Fever 38.5)
3- ENT throat pic ( uvula congested& erythematous ,big tonsil in R. side, tender LAP ant
cervical)
4- Neck stiffness.
5- Systematic
6- Office test ( do not request swab )

DDX
-Quinsy. (refer to lecture 53)
-Tonsillitis, pharyngitis.
-EBV, CMV.

Recall 12-4-2018
young lady comes for prescription of pain killers for sore throat.
task:
* take further hx
* ask p/e from examiner
* explain condition and mx to pt
during hx, it started suddenly 2 days ago, app reduced, was not able to eat solid just fluid.no
vomiting. alittle ear pain as well. no bumps and lumps, no rash, no joint pain. no headache, no
blurred vision, no PMH of any dis, no close contact.vaccination uptodate, no allergy, no smoke,
no any medication, I forgot to ask about sexual hx and travel.
P/ E: febrile 38.5, ear exam NL, other vital sign nor, in throat they showed a pic. uvula was
congested and erythmatous, a big tonsil in the right side and erythmatous, a big and tender
lymph node in anterior cervical was,found, other exam was nor.

Recall 23-2-2018
Male with sore throat. Requesting analgesics for pain relief. Smoker. Hx/ PE and DDx with
reasons.
263-Gastro esophageal reflux disease
40 year old male with chest pain for two weeks. ECG done normal. Dad had recent heart attacks.
Tasks
-Hx.
-Dx.
-DDx.

Hemodynamic stability

1- Chest pain Qs.


- Are you in pain now ?
- Duration, onset ?
- Site, radiation ?
- Character ( stabbing , burning, contracting, tightening, heaviness, dull )
- Aggravated by activity, relieved by rest ?
- Is it related to food ?
- Is it aggravated by lying flat or leaning forward ?
- 1 st episode ?

2- Associated sx
-SOB, palpitation, edema, dizziness, sweating
-Cough, fever
-Nausea, vomiting, tummy pain.
-Have you troubled by indigestion , any heart burn (key)
-Bowl motion change + colon
-LOW, LOA, lumps & bumps.

3- General
-Pmhx ( HTN, DM, Lipid, heart, PVD, bowel disease)
-Pshx, trauma
-SADMA
-travel, diet
-Fhx + any stressor

Explain
From the hx there are several possibilities why you are having pain, first, it doesn’t seem
like you are having heart attack or heart problem. But you most likely have condition we
call gastro esophageal disease
Let me explain it to you using this diagram.

It’s a digestive disorder causing return of contents back up to the esophagus ( food pipe )
It’s usually caused when the lower esophageal sphincter, which is a ring of muscles
between the stomach and food pipe.
In normal digestion this sphincter open to allow food to pass into the stomach and close
to prevent food & acidic juices from flowing back into the food pipe.

Ger. occur when this sphincter is weak or relaxes inappropriately allowing the stomach
content to flow up into the esophagus. Causing indigestion, heartburn, chest pain
triggered by lying flat, leaning forwards, some diet, spicy food or increase weight, or
increase coffee intake. Smoking and heavy alcohol intake as well

Other possibilities,
- Pneumonia, pneumothorax, PE
- Esophagitis, esophageal spasm.
- Peptic ulcer.

Feedback 4-7-2018
40yr old male with 2/12 hx of episodic chest pain ECG done normal. Dad had recent
heart attack.
Task
-hx,
-explain D/Dx
Hx-started with pain Q
cardiac sym-no evectional chest pain/orthopnea/SOB/palpitation/dizziness
RS- no cough/breathing difficulty/SOB
GIT-reflux sym+ occ related to meal, no melena
PMHx/PSHx/SADMA-asked cardiac risk factors-including occupation/stress in life-
started business recently.
Alcohol 4-6 drinks/day-advised if he is happy will organized another consultation re
alcohol-
RP immediately looked at the examiner.
D/Dx-explained with diagram likely GORD
DDx-IHD/PANCREATITIS/lung condition

264-Post viral runny nose/ cough

Feedback 1-3-2018
nasal discharge PASS
3 yr old with runny nose
hx pefe diagnosis mx
took complete hx of chronic dischrge, no smoker at home no family hx of allergies or atopy or
asthma, child care hx positive . no ear prob or speech issue
no other prob all growth chart normal
in pefe mouth breathing was there
other ent was normal
i said i think its post viral dischrge and it gets better on its own children can get upto 5 6 viral inf
a year , she can get flu shots and try to keep child at home n not send to child care until hes
better , she got offended and said i work 2 jobs who will keep the child at home . so i smiled and
said i can only imagine how tough it must be for you , i can arrage a social worker to help you
out .
she said thanks
i said one thing that concerns me is mouth breathing so he may have enlarged adenoids so just
to be safe il refer him to ent to check it out .
bell rang
Feedback 7-9-2018 (post viral cough and runny nose
Cough- PASS
3 year old child, c/o cough and runny nose? for few days.
-HX
-PEFE
-DDX
FEEDBACK-PASS
Global score: 4
Key steps: 3/5
HX: 5
Choice & technique of examination, organisation and sequence : 4
DDX: 4
2 mints thinking:
Few days? - I thought it would be either common cold or post-viral cough? and thought
about ddx. fever, rash, contact hx, family hx of asthma/allergy/hay fever, carpet, pets.
smoking hx, childcare.
DDX:
1. Common cold
2. Post viral cough
3. Any Viral/bacterial infections- tracheitis/tracheo-bronchitis
4. Asthma
5. Pertusis
6. Foreign body
After I entered the room, I asked vitals. I asked specifically respiratory rate and SP02.
Examiner said RR 23. I said okay, normal. SP02 - UNKNOWN. I didn't ask further.( Would ask
details in PEFE)
Then, I asked about cough details.
- barking?
-cough, cough, cough and vomit?
- noisy breathing/wheezing? ---- yes! (okay! it's asthma!! )
-fever, rash, (no)previous hx, family hx of asthama, hay fever, eczema, pets, carpet , smoking
at home, big group of childcare( all positive)
BINDS
PEFE:
GA: use of accessory muscles?
Vitals: all details-normal
Growth chart: normal.
Respiratory details: expiratory wheezing.
Asked head to toe- eyes, nose, organomegaly?
urine dipstick, chest xray?
DDX:
Draw a picture of lungs with air pipes and explained ddx. I said; there are number of reasons
can cause cough; after a common cold, viral infection, allergies, inflammation around wind
pipe, lodged foreign body in windpipe all can cause cough. In your child's case, most
probably, it's asthma, because of strong family hx. Have you heard about this? (yes, my elder
daughter has asthma) Okay, good. Then, you know a lot of stuff about asthma.
I had enough time, I asked her to repeat what I said. She mentioned everything !!. I didn't
explain any mx part as it was not a task.
265-Recurrent fall stroke ( postural hypotension )
Pt. with hx. of stroke & years before that affected his right upper & lower limb, came with
frequent falls.
Task
Hx
Pefe
Dx, DDx

(+ve ) afraid from falls + living alone


(+ve) hemiparesis

Note summary (good detailed feedback available)


- Talk about stroke recovery (( he is doing regular physiotherapy, he is using walking stick)

1- Fall Qs.
- When ?
- Where did you fall ?
- How often? Any LOC ?
- What where you doing ?
- Has this happened before ? ( 2-3 times )

2- Before
- CVS
- CNS ( Stroke, Migraine, Meningitis, Epilepsy)
- Hypotension (+ve)
- Trauma
- Infection
- Vasovagal

3- During
4- After (get up by himself as he is living alone )
5- Stroke Qs.
6- General
Pmhx ( vision, hearing, Bppv, arthritis, DM, HTN, heart )
Medication, compliance, kidney problems ( diuretic, Lipitor, amitriptyline )
SAD.

PEFE

- G/A DRPJL + dyspnea


- V/S esp. postural hypotension (+ve), PR, temp
- ENT
- Neck Stiffness
- CVS carotid bruit, JVP, murmur / s1, s2
- CNS cranial n. / fundoscopy / UL ,LL ( hemiparesis ) neuro ex.
- Office test BSL, UDT, ECG
Explain
There are a lot of reasons, could be vision, neuropathy, mini stroke, medications.
Most likely postural hypotension

Management
1- Get up slowly + increase fluid + refer to the specialist to modify ( change )
medications.
2- Support ? offer family meeting .
Social worker + meals on wheels.
Continue physiotherapy + OT

Feedback 21-2-2018
RECURRENT FALL-PASS
Elderly male with past history of stroke and hemiparesis now with recurrent falls.
Tasks: history , pefe, and management
Recurrent falls few weeks mostly after getting up in the morning , prior to episode -no chest
pain, headache, giddiness, palpitations, recent weakness in limbs,
during episode- no twitching, incontinence, LOC,
after episode he helps himself to get up as living alone.
with minor bruises and abrasions; no head injury
Afraid of falls and not moving , not cooking, eating frozen foods. not alcoholic, sleep-good
On medications- showed a list of antihypertensive, Lipitor Had physio for stroke and improving
and the falls are not exactly after stroke. no joint problems, no visual problem, blood sugar
normal Floor not slippery , rails in bathroom
PEFE: gen -normal only positive findings are postural drop , hemiparesis findings, vision normal
with glass
Advised postural drop as the cause, might consider changing his medication list.
Offered meals on wheel , social support workers, OT, etc.
Feedback 22-6-2018
Pt. with hx of stroke 2 yrs. before, that affected his right upper and lower limb, came with
frequent falls.
Task:
hx.
PEFE
Explain dx and ddxs to pt.
Approach
Greeted patient asked to tell me a bit more about it.. So he said he fell down 3 to 4 times
recently and is worried. I asked about his stroke recovery and whether he is getting
physiotherapy. He said regular physiotherapy and is using walking stick.
Then started asking onset, course, duration, frequency.
Pre, during and post fall situations.
DD’s qs
Hypoglycemia—no diabetes, no insulin, no meal skip
Vaso vagal- No long standing or hot, humid
Postural hypotension- positive fall during posture change
Vertigo- weather feels like everything is spinning around him -ve
Arrhythmia- racing of the heart
Anemia- (on PE)
Stroke again—new onset of-- weakness, difficulty talking, walking, swallowing, tingling or
numbness
Head Trauma
Infection
Hypervolemia/ electrolyte imbalance- vomiting, diarrhea
Seizure
Medication ( positive hypotensive and antidepressant)
Recreational drugs
PEFE (examiner was old and a bit hard of hearing so asked in a loud and confident voice)
GA: Pallor, hydration status
Vitals- BP with postural drop ( positive)
Rest normal
BMI
Nervous system in details
ENT
Asked about ear
Neck stiffness
Nervous
Cranial nerve
Fundoscopy
UP and LL neurological examination
CVS
Office test: Blood sugar- normal, ECG, Urine dip stick
Explained DDs to the patient with reason and mentioned postural hypotension as most likely
diagnosis I had time asked whether he understood everything any question. He said no.
So I said I would arrange a consultation with specialist to change medication and
If he has enough support at home or not, also arrange social worker to visit home and make it
fall free for him. Continue physiotherapy.
Feedback 7-12-2018
70 yrs old male with recurrent falls with past history of stroke
TASK :
history , pefe, DD to patient
2min thinking: could be anything stroke weakness, postural, medication induced,
hypoglycemia, depression. need to evaluate support
entered in , 70 yrs old man , sitting with stick beside him,
history - recurrent falls, 4th episode, different scenario like while walking , at home etc no
SOB, chest pain, dizzy, pre-during-post fall not significant , not seizure, not witnessed,
recovering from stroke ,regular phsio,
past history - HTN +ve,
family history-not significant (just ask anything regarding your family i need to know- he said i
live single,as wife expired, i asked how are you coping - he said fine no issues).
SADMA (pt will give medication list on paper written thiazide and other two -forgot them).
PEFE -- GA, VITALS - bp -120/80 (need to ask at the starting that i would like to measure BP both
supine. and standing for 1 min), standing 94/56 , PiCKLE
chest and heart -normal
CNS- ITPRS- tone normal, power-3/5
UDT&BSL- normal.
explanation to patient -there could be many reasons for falls, either to due to residual weakness
due to stroke, but we found that while measuring you are not able maintain BP while st anding ,
which we call postural hypotension, could be due to medication, other causes- hypoglycemia,
electrolyte problems.
scenario- falls
Global -5(approach 6, history 4, choice of examination 5, d/ddx-5)
Feedback December 2018
Station 20: Fall - Pass

60 year old patient having recurrent falls in the past 6 mths. This morning, it happened once more.
Previously, he had stroke and has been left with some weakness on the left side of the body. Hearing
and vision have already been assessed and is found that he needs reading glasses only for reading. He
has been living alone.
Task: History
PEFE
Possible causes for fall

2 min thinking: Postural hypotension; Hypoglycaemia; Stroke; Peripheral neuropathy; A’mia; drugs;

History
-Fall – already 3 times in the last couple of months; this morning – happened when he woke up from
sleep; slightly felt dizzy before it; similar to previous episodes; all 3 episodes happened in the morning.
-No other ovious symptoms
-Past medical history – Hypertension (+); taking 4/5 medications for it (showed me a list of medications;
the list contains diuretics/ACEI/aspirin and other medications); taking ACEI od at night. No DM; no joint
problems;
-can manage himself well though living alone
-beer 1-2 cans/day;

PEFE
-GA
-Vitals – posturla hypotension (+); BP is well-controlled
-Neurological examination – the left side is not as strong as the right side but ok
ECG, RBS – both are normal

Possible causes
The reason you have been feeling dizzy and fell down is due to what we call postural hypotension.
Explained it. It is a very common side effect of the medication you have been taking for your BP control. As
you take the med at night, the incidents are happening in early morning (I was not very sure whether
there is a relation between taking ACEI at night and having orthostatic hypotension in early morning but
for some reasons, I just mentioned it.
Another factor that plays a part in your problem is the weakness from previous stroke though you have
been going to physiotherapy regularly and have been doing pretty well. Besides, the fact that you are
living alone doesn’t help the cause as well. So, if you wish, I can get some help from social workers. “Thank
you, doc”. And drinking 1-2 cans of beer shouldn’t be much problem for you but I want you to tone it
down, if you can. Bell rang.

GS – 5
Key steps 1 – No; 2, 3 and 4 – YES
Approach to patient – 4
History – 5
Choice and technique of examination – 5
Diagnosis and DDx – 3
266-Pancreatic cancer
Feedback 28-2-2018
A middle aged woman presets with complaint of “tea coloured” urine for few days and has central
abdominal pain and some nausea since few days.
Tasks
-Take appropriate history
-PE findings will be presented on a card by the examiner inside the room.
-advise further management.

On history taking patient was a female in her fifties who presented with history of central abdominal
pain that was +\-radiating to the back.
the urine colour changed recently and she had not been taking any significant medicines.
No signs of any urinary infection at the moment. No fever as well.
she had h/o of cholecystectomy 6 weeks ago, and the pain has occurred first time after that.
Mentioned some weight loss noticeable to her partner.
Has no significant medical history apart from this.

On P/E examiner gives a card that mentions pale stools and dark coloured urine. Rest all findings
seemed normal.

Gave all D/Dx of obstructive jaundice along with Ca pancreas head as a possibility.

In management the candidate mentioned sending blood samples for LFT and USG abdomen and
referral to the gastro specialists.

Feedback 5-4-2018
GP, 57 year old lady comes with vague abdominal pain.
Task:
-History taking
-PE card will be given by examiner.
-Explain about Dx & possible causes with reasons.

On history, pain is in epigastric area & it radiates to back. It’s been there off & on for 3 weeks.
my history was quite disorganized in this case, forgot to ask Jaundice & color of poo & pee, LOA
& LOW. h/o cholecystectomy present & another belly surgery ( her accent is difficult to catch ).
Smoking +, but no alcohol drinking.

On PE card, pt looks unwell, Temp 37.2’C, jaundice present, can see scars for cholecystectomy,
Liver is enlarged & tender, urine is dark in color, UDS shows bilirubin +ve. i know it’s OJ, & less
likely to be infection, but pt looks unwell & liver is tender so, I told cholangiohepatitis ( how
stupid ) then I told residual stones, infection from bowel & cancer from pancreas as causes.
Case
GP, 57 year old lady comes with vague abdominal pain.
Task:
-History taking
-PE card will be given by examiner.
-Explain about Dx & possible causes with reasons.

Positive findings
-pain in epigastric area radiated to the back, itching, weight loss
-on and off for 3 weeks
-hx of cholecystectomy
-smoking +
-PEFE card (looks unwell, temp 37.4, jaundice, scar for cholecystectomy, liver enlarged and
tender, UDT dark color urine and bilirubin +ve)

Differential diagnosis
-CBD stone (cholangitis)
-CBD stricture (lap chole)
-cholangiocarcinoma, pancreatic cancer
-sphincter of odi dysfunction
-hepatitis, pancreatitis, liver abscess
-Peptic ulcer disease

Investigation asked
-FBC, LFT, EUC, lipase
-hepatitis serology, tumor marker
-US, x-ray, ERCP

Hospital admission

History (refer to cholecystitis lecture for Organised approach)


1-stability
2-pain questions
3-associated symptoms (N/V, urinary and bowel symptoms, fever, rash, LOW, LOA, lumps and
bumps, jaundice, itching)
4-General
-PMH (stone, PUD)
-PSH
-SADMA, travel , contact, blood transfusion
-sexual history
-family history
Explain
-draw a diagram

-Explained no GB, but bile has to pass to bowel for metabolism , it seems there is an obstruction
in this pathway we call it obstructive jaundice.

-it might be due to inflammation of common bile duct; cholangitis or any stone or narrowing in
the bile duct that can cause ascending infection from the duct causing inflammation, but you
have no such pain or fever so less likely.

-it could be due to pressure from outside like gut, liver or pancreas (leaf like organ). As you have
weight loss and vague pain so I am thinking of nasty growth in any of these structures but we
still need to confirm it first.

Management
-admit to hospital to be seen by specialist (NPO, IV fluid , painkillers)
-Ix (like above)
267-Gout
Case 1
50-year-old man with complaint of foot pain since yesterday
Tasks
-history
-dx, ddx

Case 2
Middle-aged man comes with foot pain alcoholic, taking diuretics, high bp etc. All pointers
towards gout.
Tasks
~PEFE (no history)
~Ddx
~Investigation

Case 3
GP, 60 years old man come to see you for right leg pain. He lives alone and retired.
Task:
-history taking
-Ask PEFE.
-Dx , DDx
-Initial investigation to examiner

Case 4
middle aged taxi driver with leg pain, alcoholic, on thiazide,
tasks:
-history
-pefe
-management with patient.

Gout
Hx

1- Pain Qs
- Severity + painkillers/ allergy + what were you doing ? ( gardening )
- Duration, sudden/ gradual , constant / on off, getting worse
- Site, radiation?
- Character ?
- Aggravating , relieving factor ?
- Has this happen before ?
- Associated sx ( swelling, redness, hot )
(SOB, chest pain, fever, waterwork )

2- Precipitating factors
-trauma, travel, insect bite.
-Pmhx (DM, HTN, lipid, gout )
-Pshx +ve alcohol, red meat, HTN, medicine
-SADMA ( smoking, alcohol)
(medication esp. HCT, aspirin )
(diet )

PEFE
1- G/A in pain, BMI, edema (PODL)
2- V/S
3- chest, heart, abdominal examination
4- Focus foot examination
. Look – scar, skin, color, swelling, deformity, nail.
.Feel – temp., tenderness, pulse.
. Move
5- Office test ( UDT, BSL )

Explain Dx, DDX


Gout :
is a metabolic disturbance with an inherited tendency in which there is decreased
kidney clearance of uric acid, which is a waste product of the body. This causes
increase uric acid level in the blood and then deposition of uric acid crystal in soft
tissue (tophi), joints causing inflammation ( arthritis), and in urinary tract causing
kidney stones.

Cause : these crystals accumulate either due to reduce kidney clearance or excessive
synthesis & uric acid in the body.
Its common condition usually runs in families, more common in men and pt. who
are overweight, consume excess alcohol.
Some medications can also contribute to it .
Its frequently associated with HTN, increase lipid, DM type 2 .
C/F … most commonly begins with acute attack of pain, swelling, redness in the 1 st
toe, can occur in other joints.

Cx… although attack of gout are very painful and they do tend to recur with the
right treatment we can minimize recurrence and manage sx very well.
Frequent attacks can cause permanent damage to joints if left untreated.

- We need to run some Inx to confirm the dx. & rule out other DDx ( septic arthritis,
trauma, cellulitis , pseudo gout ((ca pyrophosphate crystals )), RA/OA )

Immediate mx
1- Stop HCT + Aspirin
2- Start Indomethacin / SE + ACE-I
3- Rest for 48 hrs.
4- Can use panadol as additional painkillers.
Inx…. S.uric acid if not done, full blood workup + x-ray +- aspiration.

Long Term Treatment


1- SNAP + Adequate water
. Avoid high purine rich diet.
. decrease weight & exercise.
. avoid smoking, decrease alcohol to safe limit.
2- Inx ( CV risk assessment ) + Review.
3- After 4 Weeks check uric acid.
4- After 8 Weeks might start allopurinol (100-300) to decrease further attacks.

Handbook case 99
Acute gout in a 48-year-old man
CANDIDATE INFORMATION AND TASKS
You are about to see a 48-year-old taxi driver who consulted you earlier today in a general
practice setting about continuous, severe, worsening, throbbing pain in the right first
metatarsophalangeal joint, which commenced two days ago. The joint was swollen and felt hot.
The overlying skin was red and shiny and the joint was exquisitely tender. There is a history of
previous attacks over the last two years. These have been diagnosed as gout. Each time
response to treatment was satisfactory. You took blood for serum urate estimation. The patient
has returned to find out the result (which was 0.74 mmol/L) and for treatment.

The normal serum urate range for males is 0.20-0.45 mmol/L. Urinalysis is normal.

The patient has always kept in good health apart from mild hypertension diagnosed two years
ago for which he takes hydrochlorothiazide. Over the past two days he has taken two or three
aspirin tablets for the pain.
YOUR TASKS ARE TO:

• Advise treatment of the acute attack.

• Discuss further management of his condition.

There is no need for you to take any additional history or perform any examination. The
AIM OF STATION
To assess the candidate's ability to manage an acute attack of gout and give advice about its
prevention.

EXAMINER INSTRUCTIONS

The examiner will have instructed the patient as follows:

You are suffering from severe pain in your right foot which began two days ago. You saw the
doctor earlier today who diagnosed gout (which you have had before), and arranged for a
confirmatory blood test. You have returned to find out the result and receive treatment.

You are a 48-year-old taxi driver and usually keep in good health. You do not smoke but drink
three or four stubbies of beer, after work, daily. No serious past medical problems but you are
taking tablets for mild blood pressure diagnosed two years ago. You are overweight. You have no
family or social problems.

You are anxious to get relief from the pain which is preventing you from driving your taxi. You
are somewhat irritated that you were asked to have a blood test, because this was not done
during previous attacks which responded well to treatment. You have little knowledge about the
cause of gout and are unaware that recurrent attacks are to be expected and can be prevented.
You have not suspected that the blood pressure tablets could have something to do with gout,
and are annoyed that you were not warned about this.
Questions to be asked if not covered

1- How long before I can resume work?'

2-What causes gout?'


3-Can it do any serious damage to my system?'

4-What is the best treatment?'

5-Should I have any other tests?'

6-What about having a beer after work?'

EXPECTATIONS OF CANDIDATE PERFORMANCE

• Approach to patient

It is essential to establish a satisfactory relationship with this patient because of the need for
compliance regarding his use of alcohol and control of his weight, and to defuse irritation about
having to have a blood test for this attack and finding out that the antihypertensive medication
has been a precipitating factor.

• Initial management plan — immediate (task 1)

~ Discontinue diuretic and aspirin.

~ Specific treatment:

- Nonsteroidal anti-inflammatory drug (NSAID) initially in high dose: e.g. indomethacin 25 mg


capsules — 50-75 mg immediately, 50 mg two hours later, 25 mg eight hourly for 48 hours, then
25 mg twice daily for one week, would be appropriate. Other NSAID such as naproxen or
ibuprofen are also effective.

-OR prednisolone 25 mg orally, daily in the morning, reducing to zero over 7 to 10 days.

-OR colchicine 0.5 mg tablets 2-3 immediately, then 1-2 every 4-6 hours or until diarrhoea
occurs. Maximum dose 6 mg/24 hours. As pain reduces the dose of colchicine can be reduced to
0.5 mg twice daily.

Note that allopurinol and probenecid are contraindicated for an acute attack.
Additional measures: (task 1)

-Increase fluid intake.

-Elevate and rest foot for 24-48 hours.

-Paracetamol (Panadol®) can be used for additional pain relief if needed.

-Warn regarding possible side effects of medication: indigestion and elevated blood pressure
from indomethacin or prednisolone: diarrhoea from colchicine.

-Suggest an alternative drug to reduce blood pressure (e.g. angiotensin converting enzyme
inhibitor [note that all thiazides and Beta blockers may exacerbate gout]).
-May return to work as soon as pain is relieved — should be within 48 hours. (Q1)
Initial Management — Preventive ( TASK 2)
-Ensure adequate intake of water.
-Do not take diuretics or salicylates.

-Reduce weight.
-Reduce intake of alcohol.
-Avoid foods rich in purine (offal, tinned fish, shell fish and game).

-Approximately eight weeks after this attack has subsided may commence allopurinol 50-100
mg daily, gradually increasing up to 300 mg daily (two strengths 100 and 300 mg tablets)(Q4)
-Check uric acid level after 4 weeks — aim to reduce below 0.4 mmol/L

-Colchicine (0.5 mg b.d.) can be used in conjunction with allopurinol if gout recurs during initial
therapy.

-Further assessment of patient should include review of blood pressure, serum lipids, fasting
blood sugar and urea and electrolytes. Other renal function tests are not indicated at this stage.
X-ray of the affected area is not required. (Q5)

• Patient education and counselling (TASK 1)

Gout is a metabolic disturbance with an inherited tendency in which there is decreased renal
clearance of urate causing hyperuricaemia with deposition of urate crystals in joints, soft
tissue (tophi) and urinary tract (urate stones).
It is frequently associated with hypertension, dyslipidaemia, and Type 2 diabetes. Thiazide
diuretics also predispose to diabetes. (Q2)
Gout:

- Particularly occurs in the great toe following minor trauma.

- Can follow any surgical operation.

- Can be precipitated by alcohol excess, and diuretics which inhibit sodium reabsorption.

- Is aggravated by diet high in purines.

- Is prone to recurrence.

- Exhibits a prompt response of the acute attack to appropriate treatment (24-48 hours).

KEY ISSUES

• Appropriate choice of drug therapy for initial management.


• Appropriate patient education and counselling regarding prevention of further attacks.

CRITICAL ERROR

• Failure to advise change of antihypertensive medication (thiazide diuretic).

COMMENTARY

Gout (uric acid arthropathy) may present as acute arthritis or be associated with a chronic
destructive arthropathy. Most cases of primary gout are due to excessive synthesis of uric acid
while one-third relate to reduced renal clearance of urate. Acute gout commonly affects the
great toe metatarsophalangeal joint, although other foot joints and the ankle are frequent sites.
It may affect any joint in the body or, unusually, it can present as a polyarticular arthritis
mimicking other systemic rheumatic conditions. The joint can be extremely painful, red and
tender and the patient may be intolerant of even a sheet touching the foot. If fluid can be
obtained from an affected joint, it will contain needle-shaped crystals that are negatively
biréfringent on phase-contrast microscopy. Plasma urate concentration may not be elevated in
the course of an acute attack, so hyperuricaemia is not a necessary diagnostic criterion.
It is important to treat hyperuricaemia in order to avoid chronic gouty arthritis, tophaceous
gout and renal complications (calculi, chronic renal failure due to interstitial nephritis) (Q3)
Hyperuricaemia is commonly exacerbated by excess alcohol intake and drugs. It is common in
clinical practice to come across an elderly patient who has been on long term diuretic therapy
with chronic tophaceous gout and renal impairment. This should be considered an iatrogenic
disease.

Hyperuricaemia is an independent risk factor for cardiovascular disease. Therefore, an attack


of gout provides an opportunity for the prescriber to review the cardiovascular risk factors (for
example, smoking, hyperlipidaemia, hypertension, obesity), and to recommend appropriate
management.

Feedback 21-2-2018
CASE 16: GOUT-PASS
middle aged taxi driver with leg pain, alcoholic, on thiazide,
tasks: history, pefe and management with patient.
Pain questions and painkiller, pain -acute onset , unilateral great toe, second episode, with
swelling, redness, no other joints involved, no trauma, no fever, except for Htn ,no significant
medical and family history.
Pefe: gen exn- normal; vitals stable, limb exn- examiner gave a picture- I explained the picture as
comparing with left side[ examiner told left side normal], swelling on great toe with erythema,
warmth, tender present , all other joints normal on exn, movement severe pain and restriction
right toe.
Explained dignosis- gout-uric acid crystals formed in body-excessive production or decreased
excretion results in deposition within the joints and tissues.
Confirmed by blood tests and aspiration ruling out septic arthritis. In your case due to alcohol
and thiazide, unfortunately recurrent but good treatment plan available.
Immediate management-RICE, medication, sick certificate as he is a cab driver. Long term-
further blood test, stop alcohol,change anti-Htn , specialist and allopurinol.regular follow-up

Feedback 16-8-2018
GP, 60 years old man come to see you for right leg pain. He lives alone and retired.
Task: history taking, Ask PEFE , Dx , DDx , Initial investigation to examiner
Offer painkiller (Y). Pain: 1st time, I ask where is the pain exactly. He said on right foot (wearing
sock on right foot & shoe with sock on left foot). When I try to point out big toe, he told loudly
YES Very Painful. It makes me shocked. I am sorry for your pain. No pain in other joint. When I
ask about injury, he was doing gardening this morning and he felt very painful to his right foot.
His neighbour helped him. He can't walk well due to pain. No fever, no pre-existing jt problem.
Normal poo & pee
He has high BP and is taking Micardis (I asked twice because I don't know this medication) (?
micardis) for many years. Drink alcohol every day for years. 3 days ago has party and drink
alcohol. Eat fish. I asked organ meat (he doesn't understand organ meat. I explain it and then he
said No). No smoking, no recreational drugs. I particularly ask Aspirin, he took only panadol for
his pain. No chronic medical p/b apart from high BP.
PEFE: All normal including BMI except gave picture of right big toe inflammation
Dx: gout. Draw picture and explain. Risk factors: alcohol, fish.
DDx: infection, injury, RA, OA, fracture

Invx: Bld test (FBE, CRP, RFT, LFT, EL, uric acid, blood sugar, blood lipid) Bell ring & examiner said
Time up.
Before I left, I talk X ray to joint , Joint aspiration to confirm diagnosis

Feedback 23-6-2018
Feedback: Foot pain: middle-aged male pt with foot pain, HT with diuretic, alcohol.
Ask PEFE, DDx, Counselling, Investigations
Approach: How are you doing? Any pains? Any pain killers?
PEFE: back pain, shooting pain going down, numbness, weakness, SLT? (NO), skin change, nail
change, hair, T, pulse? (Normal), Infection, ulceration? (NO), Windlass test, squeeze test?
(Normal). Where is the pain? gave me the picture of the swollen big toe.
DDX to the pt: GOUT, other DDx: infection, bunion, arthritis,…
Counseling: Explain GOUT to the pt: ….Ask the pt the risk factors: trauma, DM, aspirin, Diet,
diuretic Y, alcohol. Refer the pt to the cardiologist to fix the HT drugs, arrange another
appointment for alcohol, dietician for GOUT, podiastrist for foot wear.
Investigations: serum A. uric, GSL, Xray foot, aspiration.
Grade: PASS , GS: 5 Key steps 1,2,3,4,5: Y (all covered)
Approach:5, PEFE: 5, DDX: 5, Counselling:5, Investigations: 5

Feedback 8-2-2018

Feedback 15-8-2018
Scenario: Foot pain
Stem: Middle aged man comes with foot pain ( very long stem I have forgotten) alcoholic, taking
diuretics, high bp etc. All pointers towards gout ..
Tasks
~PEFE
~Ddx
~Investigation
I am sorry I have forgotten the details here but it was quite similar to book case please have a
look.
Inside was a middle aged man.greeted and asked pain scale, allergy and offered pain killers.
Excused and asked findings from examiner. He shown a typical pic of swollen red 1 st MTP joint.
Explained dx as gout and explained it,,,and gave ddx cellulitis, sepric arthritis, om, trauma, insect
bite, allergic reaction, dvt etc
Investigations. Told about fbe, inflammatory markers, lipids, bsl and uric acid levels. Once better
specialist might decide aspiration.
Grade: Pass
Please follow book case here.

Feedback 18-9-2018 Gout


gout
old man present with foot pain, have hypertension using ACEI and diuretics. Don’t smoke and
drink alcohol 4-5 glass a day.
BMI: 25. And something else in vital sign, it is normal I think
Task: PEFE
Identify risk factors for his condition
Order investigation
PEFE: ask to inspect both foots, picture given with typical gout; hot to touch, very tender
Did ankle examination, neurological examination LL are all normal
Risk factors: BMI, diuretics use, alcohol
Investigation: blood test: FBC, LFT, electrolyte, urea acid level, renal function test
Lipids, BSL,
After acute gout subside: joint aspiration
Possible: joint USD
Score: 4
Key step: yes/yes/yes/yes/yes
Approach: 5, PEFE: 4, Dx/D/D: 5, counselling: 4, Ix:4

Feedback 18-9-2018 Gout


Your next patient is a 42 year old male came to you with complaints of foot pain.
The pain started yesterday on his right foot while he was gardening at home. He
did not injure his foot recently by any activity. He was wearing gardening boots
while working and there is no history of cut injury. His general health is good, he is
non diabetic and has no history of asthma. He has hypertension and his condition is
well controlled by Hydrochlorthiazide 1 time at night daily. He drinks alcohol 2SD 4
days a week but has history of binge drinking on some weekends.He doesnot smoke.
He doesnot have any other joint pain or swelling in any part of his body. This is the
first time that he is having this problem.
On examination of this patient you found
pulse: 104/min
BP: 130/84 mmhg
Temp: normal
BMI: 26
RR: 18 breaths/min
Your task is to
1. Ask for physical examination findings from the examiner.( the examiner will only
give the findings you asked for)
2. Explain the possible condition to the patient.
3. Explain the possible causes behind this condition to the patient.
4. Give the immediate management to the patient.
5. Arrange necessary investigations that you would like to order.
( You are not allowed to take any further history from the patient at this station)
My Approach:
I went into the room ,the examiner checked ID and introduced to my patient Mark. He was
middle aged man sitting on a chair and resting his right leg on a stool with socks on both feet. He
was in pain and asked doctor whats happening with me, why am i having this much pain? So I
assured him that I am here to help and introduced myself and asked that if he needs any pain
killer? He said no he already got pain killer and he is ok to continue. then I told him I will talk to
my examiner regarding his pain and then come back to him again.
I asked the examiner GA : Ill looking and patient in pain
didn't ask the vital as its given already. Then I asked can I please remove the socks to see the foot
of my patient. Examiner gave me a picture of right foot. It was a typical picture of gout with
swollen red great toe. I know its gout but I got confused what else should I ask now as it was my
first station and I was nervous. So I asked hapazardly about local temp.: Raised.
Is it painful to touch: yes
Is there any joint movement restriction: no
Any skin change or discharge or signs of injury: You can see it from the picture
I would like to do an ECG, Urine deepstick and BSL: Not done but you can order.
Then I got back to my patient and told him that I am suspecting that it is Acute Gout, do you
know about that? No doc whats that? I told him that there are lots of elements in our blood and
uric acid is one of them. Due to some reasons when this element increases in blood it can travel
to the joint and settle down there and can cause sudden severe pain with redness of the skin
just similar to what you are experiencing.
There could be a number of reasons behind this, One is your medication Hydrochlorthiazide, we
need to review this one and alter it with other antihypertensives. Another is your binge
drinking , so I talked about SNAP here. Another is your diet, may be you are taking more red
meat and see foods and those foods contribute to increase uric acid level.
regarding management I forgot what medication I need to give him at first. I could not
remember the name anyway. So I told we will start you on NSAID which is one kind of
medication for pain relief.I forgot the dose as well due to nervousnees. I told we will start you on
25-50 mg at first and then icrease if needed and continue it in seperated dose for the next 2
days. the I told the name of the medicine is Indomethacin, sorry I forgot to tell you at the
begining. We might need to add another medicine too with it which is Colcichin depending on
your severity. If it occurs again and again then we will start you on Allopurinol. I did not mention
any doses coz I frogot.
Last task, Investigation. I told I would like to order FBE, UEC, Serum uric acid level, Urine
MCS. The tasks were so many and I was running out of time and it was so stressful.
AMC Feedback:
Station: 11
Scenario: Foot Pain
Grade: Pass Global Score: 4
Key step 1,2,3,4,5 yes,yes,no,yes,yes
Assessment domain:
Approach to pt/relatives: 5
Choice and technique of examination, organisation and sequence: 4
Dx/DDx: 4
Pt couselling /education: 4
Choice of Investigations: 3

268-STI screening

Case 1/
22 years old male with history of unprotected sex overseas in Bali. Come for sexual health screen
and coming back for results. Chlamydia +ve. Syphilis –ve, hep B, C –ve. Gonorrhea –ve.
Tasks
-explain results and implication of them
-tell mx plan

Case 2/
you are in GP when 27 years old lady wants to know how she can know whether she contracted
STI. She also wants to know about her partner. No sexual relationship with the new partner yet.
Both have been active in the past. So they want to know if either of them contracted STI. She
claimed that her partner is also ready to do any tests
task
-Hx (no stable partner, only male, vaginal and oral, clear discharge mainly on near menstruation
with no smell, no STI before, condom off and on use)
-PEFE
-Counsel

Case 3/
young woman starting a relationship wants to get herself screening foe STI. +ve hx of
unprotected sexual intercourse and IV drug abuse.
tasks
-Hx
-arrange any Ix

Case 4/
man come for checkup for STI. Condom rupture last day travelled to Philippine.
tasks
-Hx (oral and anal sex with men, Iv drug use heroin 1 yr ago)
-explain Ix needs to be done.
General approach for all of 4 cases

History
1- confidentiality
2-STI Symptoms
-female (tummy pain, vaginal discharge and bleeding, rash, ulcers, itching)
-male (tummy pain, penile discharge, rectum, rash, ulcers, itching)
-UTI Sx (pain on urination, frequency)
-fever, LOW, LOA , lumps or bumps
3-Sexual history
-how long have you been sexually active?
-are you in a stable relationship?
-have you had multiple partners in the past?
-do you practise sex with males or females or both?
-what sort of sexual practise do you prefer?
-do you practise safe sex? Use condom?
-have you or your partners ever been diagnosed with STI?
-do you have any pain or bleeding during or after sex? (female)
4-female (5Ps)
-period
-pregnancy
-pill
-pap/ Gardasil vaccine
5-general questions
-smoke, alcohol
-have you ever used any recreational drugs? which one?
-Have you shared needles?
-Any tattoos, blood transfusion?
-PMH, medications and allergies, PSH, FHx
-travel and occupation

PEFE
1-GA pallor , jaundice, LAP, rash

2-V/S

3-abdomen
4-genital and pelvic exam
inspection (rash, discharge, bleeding)
-speculum (OS, discharge, bleeding) + swab
-BME (uterus size and tenderness, adnexa mass and tenderness, CMT)

4-Office tests
UDT, BSL, UPT

Investigations
-from history and examination, you have not had any signs or symptoms of STI. However, some
of these STI can be present without symptoms. So in order to check for these infections, if you
want and with your consent, I can arrange for Ix to look for STI.
-I need to
*take urine sample (first pass urine for PCR) to look for chlamydia
*take swab from cervix (female) for chlamydia and gonnorhea
anorectal, throat swab, penile swab
*blood test serology (hep A, B, C) HIV
*basic blood tests
-if any of these tests turn to be +, we will treat the disease accordingly.
-with the consent of your partner, we can arrange for Ix for him
-advice safe sex, red flags and review

For Chlamydia case if +ve


-I understand that you are here for the results of STI
-do you know why it has been done for you?
-how much do you want to know?
-I have gone through your results and it appears to be + for chlamydia
-pause, tissue and water
-do you want to continue the consultation?
-do you want to call anyone for you?
-do you want to discuss the Mx now?

-Chlamydia is an infection of genital tract with a bug bacterial one called chlamydia trachomatis.
-it is usually transmitted by unprotected sexual intercourse
-in most of the case it is asymptomatic. However, it can be presented with discharge or painful
urination
-If left untreated, it can spread to body organs. The good thing is we picked it up early so we can
treated it and prevent any Cx.

Management
1-I will prescribe you antibiotic azithromycin to be taken orally 1 g stat (if asymptomatic)
doxycycline 100 mg BD for 7 days (if symptomatic)
SE of azithromycin (nausea/ vomiting, stomach upset)
2-one of the tests HIV needs to be repeated after 3 months
3-chlamydia like any STI is a notifiable disease.
-You need to inform contact if you are comfortable directly or you can you a website called (let
them know)
-My legal obligation is to inform the department of health service to break the chain of spread as
there is high possibility to spread by others. But let me assure you that your confidentiality will
be maintained.
4-it is important to avoid sexual activity during this time of infection, otherwise use safe sex for
which you need to use condoms.
5-safe sex
6-red flags

Feedback 22-6-2018
22 yo with hx of unprotected sex overseas in Bali came for sexual health screen and was
coming back for results. Chlamidya +, syphilis-, hiv-, hep b-, hep c-, gonorrhea -.
Task: explain results and implications of them,
tell management plan (he asked if he should know anything about the treatment)
Approach
Greeted patient
Asked about how he is feeling
Mentioned confidentiality
Asked specifically about UTI symptoms, any discharge from penis, any rash or lumps or
bumps in groin area and fever or not.
Then said I understand you are here for the results.
So I gave the good news first and mentioned the negatives
Then mentioned the positive about chlamydia
Asked do you know what this is?
He said no. So I explained.
Then I said I will give you antibiotic course of Azithromycin for this. I asked him if he is
following. He said any side effect of antibiotic I mentioned it is quite safe and simple side
effect like nausea, vomiting can occur. Rarely can cause racing of the heart and skin rash if
these happen come to ED immediately.
Then I mentioned that it is a ntifiable disease for statistical purposes only and will be
anonymous
Asked if he is sexually active now then will need to treat partner—he said no. Also said there
is a website called let them know through which trace your partners so they can be treated.
Mentioned not to indulge in sexual intercourse until the antibiotic course finishes. He was
making a bit of face so I said but if you do use barrier contraception but I would advise you
not to until your antibiotic course finishes.
Mentioned about always using barrier method.
Said I would refer him to STI clinic for further followup.
Said red flags—any rash, discharge, lumps to come see again
Then I asked did he follow me.. he said anything else you want to tell me doctor.
I mentioned that we need to repeatthe HIV test at 3 months as it is too early to detect
anything now and to be double sure.
Bell rang—I thanked the role player and examiner
( did not ask about sexual hx details as no task and did not ask about tattoos etc. as no hx)

Feedback 15-3-2018
STI screening. GP, 27 yrs old lady came to ur gp wants to know how she can know whether
she has contacted with STI. She also wants to know about her new partner. No sexual rship
with the new partner yet. Both have been active in the past. So they want to know either of
them has contracted any STI. She claimed that her new partner would also be ready to do
any tests.
Tasks: further focus history, ask PEFE not more than 5 min for these 2 tasks, counsel
according to her concern.
Approach: 5 Ps and extensive STD counseling

Feedback STI condom rupture


Entry: Patients attitude= shy, looking down
Good morng I am one of the doctors here, I went thru ur notes. Umm would you like to tell me
in ur own words what ur concerns are today ?
Still quiet:
Me: Softly—look james, I understand that it’s a sensitive issue, but let me reassure you that we
get many patients in our practice with similar issues, I hope you will be more open and clarify
what happerned or else I wont be able to give proper investigations and help you alright?? Our
conversation will be completely confidential and we will do whatever is in ur best interest today.
Patient : Dr. I went to phillipines and umm my condom ruptured in my last sex 2 days back there.
I think I have HIV doctor.
Me: Ok why do u think so? Patient : I donno
Me: Ok lemme ask u a few Q although its just been a few days, Any fever, lumps or bumps,
weightloss, discharge or rash in pvt area…(NONE answered by patient)
ME: ok sounds ok..now are u willing to talk a bit abt ur sexual history. Its just routine ans
necessary so that we know what samples to collect ok/?
Patient : yes I am ok doc
Me: Please tell me abt ur sexual orientation ? I mean male of female?----only male
Any female sexual partners at all ----no never.
How many sexual partners in last 6 months -----10 (6 in australlia and 4 in phillipines)
Ok any of them had any discharge or rash or u know if any of them had STDs---No doctor I don’t
know.
Are you aware with SAFE SEX PRACTICE using condoms? ---es doctor ofcorse I know, but just tat
this time the condom ruptured.
So ist the first time this happened?
What were the sexual routes—oral? Anal?—mainly anal…
Have u ever been diagnosed with any Sexual Transmited diseases.---No doctor
Ok after this incident where condom ruptured, have you had any sexual contacts till now---No
doctor..or given Blood.---No (coz I was being safe whether he spread it to his contacts).
Ok are u into use of illicit drugs at all—yes doctor---I use heroin…do u share needles –yes
sometimes..when was last time u shared needles –don’t remember doctor.
Xxxxx that’s it for historyxxxx
Then I went to 2nd task---Ok good.. we usually take a full consent from patient and then take
samples and then request you to come in person to collect reports in one weeks time..but I will
tell u that some test may not be positive immediately and we will repeat them in 3 months time.
Or if any test does come out pssotive, it will be notifiable but by maintaining confidentiality and
we need to trace contacts too..but lets not go into detailsss..lets tak abt sample..mainly blood,
swabs from anus and also urine….these three samples we will take…let me now explain what will
check for.
Blood---hiv syphilis, hep b and c( as u shared needles too)
First pas urine/ anal swab—chlamydia and gonorrhoea (these are other bugs which cause STD).
Yup so don’t worry I will give u all reading materials abt these tests and arrange to collect them if
u r ok to go ahead

Recall 9-3-2018
Asking for check of STD. Condom rupture last day and traveling to Philippine are mentioned in
stem.
Task : history taking and explain relevant investigation to patient.
Role player was reluctant to talk even with confidential provided twice. Maybe didn't ask in right
way. Every kind of unsafe sex and IV drug abuse present in history.

Recall 15-3-2018
STI screening. GP, 27 yrs old lady came to ur gp wants to know how she can know whether she
has contacted with STI. She also wants to know about her new partner. No sexual rship with the
new partner yet. Both have been active in the past. So they want to know either of them has
contracted any STI. She claimed that her new partner would also be ready to do any tests.
Tasks: further focus history, ask PEFE not more than 5 min for these 2 tasks, counsel according to
her concern. (No stable partner, only male, vaginal and oral, clear discharge present mainly on
near mens with no special smell, no sti before, condoms off and on used, generally healthy)

Recall 6-9-2018
Sti screening. Young women starting a relationship. Wants to get herself screening for sti.
Positive history of unprotected sex and iv drug abuse. History arrange for investigation
269-Down syndrome psych cases
Case 1 Down syndrome + change behaviour

Feedback 2018
A disability home support worker has come to you to talk about one of the patients who is a
diagnosed case of downs syndrome. There have been some changes in his behavior lately. You had
examined the whole group from that centre when they came to you for routine vaccinations 6
months ago and he appeared fine to you and his physical examinations were also normal.
TASKS:
-Talk to the support care worker and take history about the patients condition.
-Give your most likely diagnosis and differentials
Please note: The support care worker has the statutory declaration to talk about this patient. Please
do not mention any mx.
My response :
-Confidentiality
-I understand that you are concerned that you noticed changes in your patient’s behavior and (
DOCTOR HES VERY TIRED SO I HAD TO SKIP MY HX FRM CHG IN BEHAVIOR TO TIREDNESS BUT DIDNT
SPEND TOO MUCH TIME ON IT COZ I KNEW ITS A PSYcH CASE.
-When did it start? Is tiredness at end of day or whole day?
-is he sleeping well, eating well,
-any chronic health conditions he gets treatment for?
-any weight loss? any lumps of bumps? Do you think he has fever? Any chest pain or tummy pain?
Does he complain about anything?
-Any change in bowel motion, color of the stool, or waterworks?
-Any weather preference especially cold intolerance?
-Any change in color of the skin?
-Any history of getting heart problems?
-Has he traveled recently overseas or anywhere else?
then psych pattern of history -FULL ASEPTIC JC TYPE HISTORY WITH HEADS AND SADMA -
**mood? ITS LOW DOCTOR HES NOT SOCIALISING STAYNG ALONE IN HIS ROOM?
when did this start? 4-6 MOTHNS BACK?
anythng changed that time? YES DR..HIS ROOM MATE MOVED OUT TO INTERSTATE ?
Does he have the capability to express himself? YES HE CAN.
Has he ever thought of or done something to harm himself? -NO NEVER..
with this changed behavior has he become aggressve or hurt other room mates? NO-
do u think he can recognize ppl around him..is he aware of time and place?
any history of head inujuires..
Does he have family? Do they visit him? YES THEY DO BUT HIS FATHER DIED 2 YEARS AGON HE
WASNT VERY CLOSE THOUGH.
smoking no..ilicit drugs no..alochol ..no.
DD--- i said adjustment diorder possible but the stressor is in last 3 months and doesnt last longer
than 6 months..so other thng cud be hes going into depression as he has most of the features, other
cud be grief abt his dads death but its been 2 years ...then as he has downs cud be heart condition,
anemia, thyroid probs... but really need to review him and run some othe tests before confirming
what it is..

Another Feedback 2018


You are a GP and the next person is a healthcare worker who came to you to discuss about one
of your past patient Paul. Paul has down syndrome and you saw him last 6 months ago when he
came with other six patients from the same facility for vaccination. Last time when you saw him
he was very cooperative and happy. Now the worker is complaining that recently they have
noticed significant changes in Paul's behaviour. He is not talking with anyone, doesn't enjoy his
work anymore and he stays in his room all the time, He didn't even come to see you for his next
vaccine which was due 2 months ago. last time you did some investigations which are given
below There were a list of investigation and all of them were normal.
Your task is to
-Take history regarding Paul's condition from the health worker. ( this should take no more than 6
mins)
-explain the dx and dds
(The worker has legal authorization and consent to talk about your patient's condition)

My approach:
I went in and greeted the worker and then started taking history. when it started: 6 months ago.
I asked about HEADSSS , SADMA, head injury, recent infection, past medical hx, surgical
hx, Family hx and family issue, everything was negative.
Then I asked ok is there any recent change happened in his life like change of his job or change
of his room or roommate? then the worker said that his best friend recently moveout to
another facility and it was 6 months ago.
So the I told him that it is adjustment disorder and explained that in layman terms. Gave him dds
of depression, head injury(hematoma), Drug abuse, recent infection and offered home visit to
see this pt. also I asked about suicidal activity but he denied.

AMC Feedback:
Station 14: Behaviour change
Grade : Pass Global score: 4
Key steps: 1,2,3,4,5 yes, yes, yes, yes, yes
Assessment domain:
Approach to pt/relative: 4
History: 4
Dx/DDx: 4
Case 1 summary
A disability home support worker has come to you to talk about one of the patients who is a
diagnosed case of downs syndrome. There have been some changes in his behavior lately. You had
examined the whole group from that centre when they came to you for routine vaccinations 6
months ago and he appeared fine to you and his physical examinations were also normal.
TASKS:
-Talk to the support care worker and take history about the patients condition.
-Give your most likely diagnosis and differentials
Please note: The support care worker has the statutory declaration to talk about this patient. Please
do not mention any mx.

*Confidentiality
*I understand that you are concerned that you noticed changes in your patient’s behavior. (yes dr he
is very tired)

1-how long+ HEMIFAD quickly

2-psychosocial history (mood low, is not socializing, staying alone in the room)
-when did it start? (4-6 months)
-anything happened recently, changes? (roommate moved out to interstate)
-is he able to express himself (yes he can)
-rest of psychosocial history.

3-HEADS
-does he have family (yes)
-do they visit him (yes, they do but his father died 2 years ago, he was very close to him)
-Any job or work
-stresses at home and workplace
-SAD

4-General
-PMH , thyroid, head injury……

Differentials
There are possibilities behind this…
-possibly adjustment but stressor is in last 3 months and does not last onger than 6 months.
-could also be depression as he has most of the features.
-could be due to heart condition, anemia, thyroid px, infection but need to review him at home
and run some other tests before confirming what it is.

Adjustment abnormal or excessive reaction to an identifiable life stressor . the reaction is


more severe than would normally be expected. Sx must arise within 3 months of stressor and
last no longer than 6 months after the stressor has ended.

Karin case
Case: You are a GP and your next patient is a 26-year-old with Down Syndrome living in a support
home. There is change in behavior recently and he is very tired. He is afraid to coming to the GP
and that is the reason he hasn’t come today as well. Instead, there is a legal carer who has come
to see you.
Task
a. History from legal carer (he is not active, tired and doesn’t want to get out of bed recently
since last week when his close friend left the home and moved interstate; weight loss; no
medications; mood is sad; can’t express in words because is mentally changed)
b. Diagnosis and management.
History
- Confidentiality
- I understand that you are concerned that you noticed changes in your patient’s behavior and
that he is very tired.
- Could you please explain what you mean by change in behavior? When did it start? Did
anything happen around that time when it started? How is John’s mood? Does he have the
capability to express himself? What about his weight? Appetite? Sleep? Has he ever thought of
or done something to harm himself?
- Do you think he has fever? Any chest pain or tummy pain? Does he complain about anything?
Any change in bowel motion, color of the stool, or waterworks? Is there any smelly urine,
frequency or urgency? Any weather preference especially cold intolerance? Any change in color
of the skin?
- Any history of getting heart problems? Has he traveled recently overseas or anywhere else?
SADMA? Does he have family? Do they visit him? Does he have other friends in the support
home?
Management
- Most likely, he has a condition called adjustment disorder. Because he was emotionally
attached to his friend, he might have develop these symptoms when he left but it could be
depression resulting from hypothyroidism which is common in patient’s with Down syndrome. I
would like to run some basic investigations for him including FBE, TFTs, BSL, iron studies, urine
MCS, U&E, and LFTs.
- At this stage, I want to see him. If he doesn’t come, then I would like to arrange a home visit. If
he has a family, I can arrange a family meeting because he needs a lot of support right now. If
you are really concerned, I can refer him to the counselor to help him deal with stress.

- I would encourage you to engage him in his favorite activities.

- I would like to review him once the results are back.

- Reading material.

- Red flags: for severe depression

Case 2 Down syndrome + depression


Note/ this is the tricky case of 2018 because most of the candidate used to know that any case
of down will be adjustment but here they changed it. In this case the stem said that the patient
already diagnosed with depression and carer wants to talk to you about change in behaviour and
you need to counsel them about depression in down and management. No history in the tasks.
But the candidate who asked history before counselling passed and the one who did not failed
(there is no feedback on this case in facebook we got it only from skype first then the same
person commented on facebook later about her approach in brief)

Comments
- can you pls tell me what is the diagnosis of Down syndrome case, is it adjustment disorder or
depression case.
One candidate FAIL
- About Down’s syndrome depression case I failed the case so I cannot really talk abt it.
-Down syndrome patient lives in care facility, now diagnosed with major depression without
psychotic features
TASK
Counsel carer about depression(stem mentioned no need of history and investigations)
- Stem did not mention no history... it just said PE was done and it was normal. His basic blood
was normal and thyroid test was normal. And task was :
1) counsel carer about depression in downs.
2) talk abt management

- Another candidate pass


Guys this was my case and i passed it. The stem is similar to the one in the post. He has already
been dx with depression so the task was (Don't remember exact words) counsel the carer about
depression and relationship with downs and further managment. It was something like this. It
was different from the once we know, adjustment disorder in downs. So just explained to her
about depression and asked her if he was showing those symptoms, she said he is withdrawn
and stays in his room, asked he if anything happend recently, she said no. Just asked few
questions about his depends/disability she said he can do his selfcare good verbally and does
packing job...but not anymore. Asked about his family, they are old and don't visit much. (Asked
questions which might help me in the managment) So just explained to her the relation of
depression and how it affects downs people ( have not read anything about it to be honest so
just said what ever made sense) then told her that we will involve Psychiatrist they will fo CBT,
carer asked what it was...and also he will be given medication. She asked how will he take it, so
just told her he needs an assistance to make sure he takes his medication and just few things
about the medication. And told that he needs close observation and follow-up, and involving his
family might help as well. So i just made up my own approach and made an interactive
cancelling.

Case 2 summary
down syndrome patient lives in care facility, now diagnosed with major depression without
psychotic features carer present.
tasks
-counsel the carer about depression and relationship with down
-further Mx

*Confidentiality
1-Depression is a mental condition where you can feel extremely tired with low mood, LOW,
LOA, sleep disturbance and also not enjoying activities you used to enjoy before.
-is he showing any of these sx? For how long? (he is withdrawn and stays in the room)
-did anything happen recently ? (No)
-is he taking his meds regularly? (I do not know dr)

2-HEADS
-is he able to express himself and manage things by himself? (he can do his selfcare, good
verbally, does packing job but not more)
-how is the situation at care home? Any stress?
-does he work? (yes)
-does he have family? Do they visit him? (they are old and do not visit him much)
-SAD+ suicide

3-DDx
-thyroid, trauma, infection

Diagnosis
-Depression is much more than feeling sad or low for a while. It is ….. etc at least for 2 weeks.
-the exact cause is unknown but could be due to a reduction in a chemical substance in the
brain called serotonin, genetic and environmental factors can act as a triggers.
-It is at least as common in children and adults with down syndrome as it’s in the general
population and affects them in the same way.
-Depression is not part of the condition but it is one of the most frequently diagnosed mental
health issues for those with downs.

Management
1-will need to see him myself asses him
2-will need involvement of psychiatrist+ CBT Psychologist
3-will be given medication and need assistance to help him take it with close observation and
Follow ups
4-encourage exercise and getting involve in his favorite activities to help build self esteem.
5-family meeting
6-social support
7-reading materials
8- red flags

Feedback 30-5-2018
‘Relapse of Schizophrenia’:
Collateral Hx from care worker –behaviour change. Same recall with slightly new flavour.
Task: Hx, Ddx, Dx, Mx.
Feedback: Pass

• HOPC

o Same approach as recall (Known case Down Syndrome: carer consulting re behav)

o Ensure confidentiality/consent etc

o Ask re concerns –Sx analysis [+ves: florid psychosis]

o Then went into known case formula: Risk fx, Sx, Complications

 Compliance/contact w Dx Dr/Follow up/titration/side fx

• Revealed non-compliance

o So, probed further according to non-compliance Hx formula

 Dr factors

 Pt factors

 Medication factors

 Any possibility of exposure to illicits (I said I know you must have good security at the
care facility but do you think etc etc)

o Then investigated changes to environment

 Premorbid function

 Any major event


 Is the family in the picture

 Do you know much about his Dx –what Sx he suffered prior to Dx

• This Question revealed she didn’t know much beyond the prior 8/12, the time since
which the pt came to their care

• So delved further into what records they had from the other facility interstate—and
mentioned I’d chase up those details & not to worry

o Then quick risk assessment

 SIGECAPS PTICJ (most already covered in HOPC up till now)

o Very quick Organic Ddx Qs:

 Infection/Tumour/Trauma

 Thyroid/metabolic

o PMhx FHx(mental health) SHx SADMA

• Ax & P

o Same spiel as recalls

 There are several causes for people to behave this way

• Organic (but I found no features suggesting)

• Drugs (there is non-compliance)

• Mental health (exacerbation of pre-existing)

 I need to see him

 But given the current picture of his Sx (psychosis)

 He’s lost touch w reality and so in the interest of his safety and that of all the other
residents/staff at the facility

 It is important to enlist the services of CAT team to r/v him and they’ll decide on most
likely admission

• Where it would be the safest place for him right now

• There an MDT consisting of qualified specialists will work together to get to the bottom
of his current state

• There they’ll have all the necessary equipment & expertise to best delineate what the
cause of his current Sx are and Mx accordingly

o This may involve medication Mx (dose adjustment)

o Basic blood work


o Imaging

• I’ll get in touch with the family and his Dx Dr (DrPsych)

o Then summarised all the plan and made sure they understood and whether they had
any further concerns

270-Hypertension non-compliance
Case 1
Hypertension Non Compliance
History
1-Rapport + confidentiality

2-Memory questions (short term+ long term+ risk+ effect)


-can you tell me more about your memory problem?
-when did you notice it?
-has it started suddenly or gradually?
-short term: do you often forget where you placed keys or about your appointments?
-long term: do you forget about important events like anniversary or birthday dates?
-Risk: have you ever had an accident because of memory?
-how does it affect your life?

3-Hypertension questions+ non compliance.


-Diagnosis and medication
*when were you diagnosed with hypertension?
*what medications do you take? What is the dose?

-Doctor related (explanation and understanding)


*did you dr explain to you about HTN , its causes, risk and cx?
*did you understand the explanation and the benefit of taking the medications

-Drug related (side effects and cost)


*I can understand that you are not taking the med regularly so how many time?
*have you had any side effects?
*any problem with cost

-Patient related (belief and alternating treatment)


*what is your views? Do you believe the medication is helpful?
*are you trying any alternative treatment?
*regular check up

-do you take any other medications or OTC or recreational drugs?

4-SNAP
-smoking qs
-alcohol qs
-diet/ weight and appetite
-physical activity

5-Brief mood and HEADS


-How do you feel?
-anhydonia
-sleep
-work? Any stress?
-home? Any support or stress?
6-PMH
-DM, high fat in blood, heart, kidney, stroke

7-Current symptoms
-chest pain, SOB , palpitation (heart)
-urine output, colour (kidney)
-headache, BOV, weakness (stroke)
-weather preference, bowel motions (thyroid)
-confusion, loss of balance (alcohol)

8-Family history (heart, kidney, stroke, memory problem)

Counselling
Explain about hypertension
-high blood pressure is a very common condition, you are not alone with this and we are here to
help you.
-hypertension means your blood is pumping with more force than normal through your arteries.
-hypertension can cause heart problems like heart attack, kidney problem, stroke and memory
lproblem called vascular dementia that is why it is important to adhere to Life style and
medications.

Exaplain about memory problem


-your forgetfullness can be due to several reasons; can be sue to uncontrolled high blood
pressure causing vacular dementia, high alcohol intake causing acute brain damage, or other less
likley causes like decrease level of thyroid hormone or Vit B12 deficiency.

Counselling about non compliance


to help you with non compliance I would like to give you sme advice:
1-with your consent I can arrange family meeting to give you support.
2-you can put an alarm on your phone to remind you about the time of the meds. You can also
write down the time on a white board in your bedroom, kitchen.
3-there is also webster pack that you can take from the pharmacy which is helpful to organise
and comply with your meds.
4-send you social worker and district nurse to help you as well.

Life style advice


-limit alcohol to safe level not more than 2 SD a day.
-stop smoking
-healthy balanced diet
-regular exercise

refer to neurologist + Ix (FBE, LFT, EUC, BSL, lipid, urinalysis, TFT) (ECG, CXR)
Case 2
Feedback 30-5-2018 pass
‘Opportunistic health review’:
HTN non-compliance. Young lady recently from interstate (2/2 marital discord & stress from custody battle
–only if asked). Sutured recent accidental cut (no drug/psych relation). Known HTNsive but non-comp 2/2
stress & finances. [the majority of these +ves only if you dig them up].
Task: Hx & counsel
Feedback: Pass

 HOPC

o Started w wound hx/how’s it going/what happened/# of episodes of this

 Have you had similar injuries in the past

 By any chance was it intentional

 Tetanus etc

o So the practice nurse flagged your BP to me and it’s rather high…

o Known case formula: Risk fx/Sx/Cx

 Cx  head(vision/HA/weakness)/heart/kidneys [end organ damage Qs]

o Non-compliance proforma:

 Dr factors [was HTN explained to you/how much do you know]

 Pt factors [SEs]

 Med factors [finances are the main reason according to her]

o So I noticed you’ve recently come from interstate

 How’re you settling in

 Was this for work

 Do you feel supported/do you have family/friends here

 This revealed all the stress she’s in

 Marital discord

 Ugly custody battle

 Lost to f/u with health care

o This triggered a psychosocial hx

 SIGECAPS HEADS SS PTICJ

o PMHx FHx SHx SADMA

 Ax & P

o I’d like to take over your care if you like


 I’ll get in touch w your previous provider and get your records over

o You’re under a lot of pressure and this needs to be addressed

o Furthermore, this stress can double the impact of your poorly controlled HTN

 Then gave quick recap of HTN Cx

 & related back to them why I asked the HTN Qs –to screen for extent of effect
of HTN on her

o If you’re open to it, I think you would benefit from added supports to help manage the
stress you’re in –PSY r/v [clin psych for talking & strategies +/- DrPsych for meds]

o Once I’ve liaised w your Dx Dr –we’ll get your HTN under control

o Re: your financial issues –I’ll take necessary steps to organise support

o Will do some basic blood work & PE today to cover our bases

o Summarise etc
Hypertension Non Compliance
37 years old female reviewed by GP for suture removal. She was found to have BP> 160/100 on
two occasions. She had revealed that she defaulted her BP pills (perindopril) for the last 6
months.
Tasks
-History regarding med noncompliance.
-counsel the patient

Positive findings
-changed her residence thus missing out follow ups.
-lack of knowledge thought her BP was normal after initial course of treatment
-separation from husband, 2 kids living with him
-stress at work (ugly custody)
-living alone
-found previous med expensive

History
1-same as before but firstly address her wound
-how is it going? What happened?
-have you had similar injuries in the past
-by any chance was it intentional?
-tetanus vaccine?

2-the practise nurse flagged your BP to me and it is rather high


-risk factors/ Sx/ Cx
-non compliance qs (dr, med, pt)

3-I noticed you have recently come from interstate


-how are you settling in?
-Was it for work.
-do you feel supported? Do you have any family, friend?

4-HEADS + Psychosocial hx

5-SNAP+ PMH+ FHx

Management
I would like to take over your care if you like
*I will get in touch with your previous provider and get your records over.

*you are under lots of pressure and this needs to be addressed. This stress can double the
impact of your poorly controlled BP (define it)
-support group -centerlink (financial support)

*once I have liaised with your dr. we will get your BP under control.

*do blood Ix and physical examination today

*life style advice


Case 3 Feedback 8-11-2018

Note/ In the case of november 2018 : (in this case no wound)


-in this case when I entered the room I saw a middle ageed lady and when I explained that I can see that
you are here because of the results of the htn  she said yes dr I moved interstae so I was no longer
contacted with my GP and she voluntarily revealed part of the HEADS which is the divorce and travelling
interstate…
-after that I introduced myself and told her that I really appreciate that you are here coming to Visit me
making your health a priority and reassured here and ask her questions:
-Current sx -PMH, FHx, SNAP -HEADS and psychosocial hx
-let me assure you that I according to your GP all Kidney fuction normal etc….
-Then tell like Mx in case 2 and was poistive about the cost and happy with centrelink.

S-ar putea să vă placă și